Sunteți pe pagina 1din 1103

Objective

CHEMISTRY
for JEE MAIN
with BOARDS SCORE BOOSTER

­
So
lly
Fu ed
lv

F
F
F
F
F
JEE Main 2014-15 Solved Papers
Key Concepts with Illustrations
4 levels of Exercises
1200+Questions based on NCERT
1500+Past Competitive Exam MCQ's
F 3500+ Practice MCQ's for JEE Main
F Aligned as per Class 11 & 12 NCERT
• Head Office : B-32, Shivalik Main Road, Malviya Nagar, New Delhi-110017

• Sales Office : B-48, Shivalik Main Road, Malviya Nagar, New Delhi-110017
Tel. : 011-26691021 / 26691713

Typeset by Disha DTP Team

DISHA PUBLICATION
ALL RIGHTS RESERVED

© Copyright Author

For further information about the books from DISHA


Log on to www.dishapublication.com or email to info@dishapublication.com
PREFACE
With the Boards having a weightage (40%) for admission through JEE Main the need of a book that can help students
both in JEE Main and Boards becomes more prominent. The All New “Objective Chemistry for JEE Main with Boards
Score Booster” encapsulates the formula for cracking Boards & JEE Main simultaneously.
Salient features of Disha's “Objective Chemistry for JEE Main with Boards Score Booster” book are:
• Fully Solved 2015 JEE Main Question paper has been added.
• Exhaustive theory, with solved examples, explaining all fundamentals/ concepts to build a strong base.
• Illustrations to master applications of concepts & sharpen problem-solving skills.
• 4 level of graded exercises to ensure sufficient practice.
• 1200+ NCERT based Questions (Boards Score Booster) for Board exams covered in a separate exercise.
• 1500+ past Competitive Exam MCQ’s of JEE Main/ AIEEE and other entrance exams to provide a better exposure
covered in the exercise “Window to Competitive Exams.”
• 3500+ Practice MCQ’s for the JEE Main exam. Finally 2 Practice exercises at the end of each chapter – Basic
and Applied.
• The book covers all variety of questions as per the format of the previous year AIEEE/ JEE Main Papers.
• The book has a very strong focus on Practical Skills as per the syllabus of JEE Main exam and the questions that
have been asked in previous exams. The book covers Volumetric Analysis, Analytical Chemistry and other
practical skills as per the syllabus of JEE Main.
• Covers entire syllabus as per the latest NCERT books and JEE Main syllabus. The complete book has been
aligned as per the chapter flow of NCERT class 11 & 12 books. At places certain chapters have been divided into
sub-chapters. For example - Equilibrium is divided into 2 parts Part A – Chemical Equilibrium; Part B – Ionic
Equilibrium.
More than 25 out of the 30 JEE Main questions were directly or indirectly covered in our book and the readers must
have really benefitted from it.
We are highly thankful to the Disha Publication team who continuously encouraged us to update this book. We are
also thankful to the editorial team and the D.T.P. Team who worked day & night for bringing this work before you.
We would like to give a special thanks to Mr. Kulpreet Singh (Sr. SME - Chemistry) who has really spent a lot of time
in bringing this book to the present shape.
Last but not the least we would request all readers to come up with suggestions so that we can further improve the
book in the next edition.
With best of luck!!
AUTHORS
DR. DALJEET SINGH
Ph.D., Delhi University
Professor of Chemistry (Retd.)
MM PG College, Meerut University,
Modinagar (U.P.)
ER. PRAMIT SINGH
B.Tech. MS
Concordia University, Montreal, Canada
INDEX
JEE MAIN 2015 Solved Paper (with solutions) 2015-1 – 2015-4
JEE MAIN 2014 Solved Paper (with solutions) 2014-1 – 2014-7

1. Some Basic Concepts of Chemistry 1-26


Physical quantities in SI units & derived units; Standard prefixes; Conversion factors; Significant figures; Classification of matter; Physical
and chemical changes; Laws of chemical combinations; Atomic, molecular and equivalent mass; Solutions and representation of concentration.

2. Structure of Atom 27-58


Discovery and physical properties of subatomic particles; Rutherford’s nuclear model; Plank’s quantum theory; Electromagnetic spectrum;
Photoelectric effect; Bohr’s model of atom; Sommerfeld model of atom; De Broglie principle; Heisenberg’s uncertainity principle; Quantum
mechanical model of atom; Quantum numbers; Shape of orbitals; Pauli’s exclusion principle; Aufbau rule; Hund’s rule of multiplicity; Radial
and angular distribution curves.

3. Classification of Elements and Periodicity in Properties 59-74


Different laws regarding classification of elements; Modern periodic law; Structural features of long form of periodic table; Diagonal
relationship; Periodic properties; Valency; Atomic volume; Acid base behaviour of oxides and hydroxides.

4. Chemical Bonding and Molecular Structure 75-110


Reason for formation of chemical bond; Types of chemical bond; Factors affecting formation of the chemical bond; Born Haber cycle;
Properties of ionic compounds and covalent compounds; Valence bond theory; Molecular orbital theory; Energy level diagrams; Electronic
configurations; Hybridisation; VSEPR theory; Hydrogen bond and its applications; Dipole moment; Bond lengths, bond energy, bond angles.

5. States of Matter 111-144


States of matter and characteristics of common forms of matter; Measurable properties of gases; Gas laws Properties of gases; Kinetic
theory of gases and kinetic gas equation; Maxwell’s distribution of velocities; Deviation from ideal behaviuor of gas; van der Waal’s equation
& constants; Critical phenomenon and liquification of gases; Joule Thomson effect and Joule Thomson coefficient; Law of corresponding
states; Heat capacity of system.

6. Thermodynamics 145-178
Terminology used in thermodynamics; Internal energy and its transfer; Processes and their types; Thermodynamic equilibrium; First law of
thermodynamics; Factros affecting DH; Relation between DH and DE; Applications of first law; Hess’s law of constant heat summation;
Heat capacity of system; Entropy; Criteria for feasibility of a process; Zeroth law of thermodynamics; Third law of thermodynamics;
Clapeyron Clausius equation and its applications; Heat engine and its efficiency; Carnot cycle; Resonance energy; Calorific value; Types
of reactions and corresponding enthalpy changes; Laws of thermochemistry.

7A. Chemical Equilibrium 179-206


Reversible and irreversible reactions; Characteristics of chemical equilibrium and factors affecting it; Relation between K p and Kc; Different
types of chemical equilibrium; Van't Hoff isochore and isotherm; Henry’s law; Le Chatelier’s principle; Applications of law of mass action;
Degree of dissociation from density measurement.

7B. Ionic Equilibrium 207-242


Arrhenius theory of ionisation & factors affecting it; Ostwald’s dilution law; Acids and bases and their strength; Relation between K a and
Kb; Common ion effect; Solubility product; Dissociation constant of water; pH scale; Salt hydrolysis; Buffers; Neutralisation; Acid Base
indicators; Salts, solvents.
8. Redox Reactions 243-256
Oxidation, Reduction, Oxidising and reducing agents; Types of Redox reactions; Oxidation number and rules for determining the oxidation
number; Equivalent weight of oxidising and reducing agents; Balancing of chemical equation by different methods.

9. Hydrogen 257-270
Preparation and manufacturing of hydrogen and its properties; Different forms of hydrogen; Hydrides; Hydrogen peroxide; Water; Hydrates.

10A. The s-block Elements – Alkali Metals 271-286


General characteristics; Chemical properties; Abnormal behaviour of lithium; Metallurgy of sodium, potassium & their compounds.

10B. The s-block Elements – Alkaline Earth Metals 287-302


General characteristics; Chemical properties; Abnormal behavour of Be; Diagonal relationship between Be and Al; Metallurgy of magnesium,
calcium and their compounds.

11A. The p-Block Elements – Boron Family 303-318


General characteristics; Chemical properties; Abnormal behaviour of boron; Diagonal relationship between boron and silicon; Metallurgy of
boron, aluminium and their compounds; Alums.

11B. The p-Block Elements – Carbon Family 319-336


General characteristics; Chemical properties; Formation of carbon compounds; Types of silicates; Carbides; Different forms of carbon;
Carbon dioxide, carbon monoxide; Silicon and its compounds; Glass; Metallurgy of tin, lead and their compound.

12A. Purification, Qualitative and Quantitative Analysis of Organic Compounds 337-356


Methods of purification of solids; Methods of purification of liquids; Chromatographic methods; Chemical methods; Extraction; Drying of
organic substances; Criteria of purity; Detection of elements; Estimation of elements; Determination of molecular weights; Emperical and
molecular formula; Eudiometry.

12B. Classification and Nomenclature of Organic Compounds 357-386


Classification of organic compounds; Classification based on functional groups; Homologous series; IUPAC system of nomenclature;
Nomenclature of simple aliphatic compounds; Line-angle formula; Nomenclature of complex hydrocarbons; Nomanclature of cyclo alkanes;
Nomenclature of bicyclic compounds; Nomenclature of compounds conatining substituents (not regarded principal function groups);
Nomenclature of compounds containing more than one type of functional group; Nomenclature of aromatic compounds.

12C. Hybridisation & Shapes of Organic Molecules 387-398


Hybridisation of carbon; Hybridisation of nitrogen; Hybridisation of oxygen; Bond lengths of some important bonds; Bond angles in selected
molecules; Aromaticity and aromatic compounds; Huckel rule.

12D. General Organic (Basic Concepts) 399-426


Electronegativity; Inductive effect; Electromeric effect; Resonance; Mesomeric effect; Conjugation; Hyper conjugation; Cleavage of covalent
bond; Types of reagents; Reaction intemediates; Carbenes; Nitrenes; Types of reactions; Directive influence of atoms and groups.

12E. Isomerism 427-452


Types of isomerism; Structural isomerism; Stereo isomerism; Chirality; Lebel’s & Van't Hoff’s theory about optical isomerism; Condition for
chirality; Diastereoisomers; Assymetric synthesis; Racemisation; Walden inversion; Optical purity; Determination of configuration of geometrical
isomers; E,Z systems of nomenclature; R and S assignments.
13. Hydrocarbons 453-492
Nomenclature; Types of carbon atoms; General methods of preparation & their properties; Conformational analysis Bayer’s strain theory;
Unsaturated hydrocarbons, nomanclature, preparation; Aromatic hydrocarbons, preparation and properties; Stucture of benzene; Petroleum,
theories of origin of petroleum; Fractionation of petroleum; Important terms related to petroleum; Cracking; Synthetic petrol; No lead petrol.

14. Environmental Chemistry 493-504


Pollutant; Air pollution; Environment; Water pollution

15. The Solid State 505-528


Different types of solids & their properties; Unit cell & calculation of number of particles in unit cell; Packing fraction; Law of rationality of
indices or intercepts; Crystallography and X-ray diffraction; Symmetry in crystal systems and different crystal systems; Packing of
constituent particles in crystals and voids; Limiting radius ratio; Structure of simple ionic compounds; Imperfections in solids; Different types
of conductors and their properties; Silicates.

16. Solutions 529-558


Types of solutions; Expression of concentration of solution; Vapour pressure; Raoult’s Law; Ideal and non ideal solutions and their
graphical respresentation; Colligative properties; Determination of molecular weights by different methods; Abnormal molecular masses;
Van't Hoff's factor.

17. Electrochemistry 559-594


Faraday’s law of electrolysis; Amount of substances liberated or deposited at an electrode; Factors affecting electrolytic conduction;
Specific conductivity; Equivalent conductivity and molar conductivity; Kohlraush law and its applications; Daniel cell; Electrochemical series
& its applications; Nernst equation for cell potential; Some commercial cells and their types; Concentration cells and their types; Criteria of
formation of products in electrolysis; Fuel cells; Rusting of iron and its prevention.

18. Chemical Kinetics 595-628


Rate of reactions; Relation between average and instantanous rate; Rate constant and factors affecting it; Rate law equation; Molecularity
of reaction; Kinetic equations of different order; Determination of order of reaction; Collission theory; Energy of activation; Photochemical
reactions; Quantum efficiency, quantum yield and properties of radiations; Study of kinetics of some reactions.

19. Surface Chemistry 629-650


Different types of colloidal systems; Sols and their types; Characteristics of lyophilic and lyophobic sols; Classification of sols and their
preparation and properties; Coagulaion and precipitation; Hardy Schulze rule; Gold number; Emulsions and gels; Cleansing action of soaps;
Applications of colloids; Catalysis & different types of catalysts; Theory of catalysis Adsorption and their types; Factors affecting
adsorption; Freundlich isotherm; Langmuir isotherm; Applications of adsorption Distribution law, Henry’s law and their applications.

20. General Principles & Processes of Isolation of Elements 651-666


Minerals & ores; Concentration or dressing; Calcination, roasting; Refining; Metal source, main methods of extraction; Important terms used
in metallurgy.

21A. The p-Block Elements - Nitrogen Family 667-686


General characteristics; Chemical properties; Compounds of fifth group elements; Important oxides of nitrogen, phosphorous and their
oxyacids; Manufacturing of phosphorous and its compounds; Fertilizers.
21B. The p-Block Elements - Oxygen Family 687-704
General characteristics; Compounds of sixth elements; Ozone; Oxides; Oxygen; Compounds of sulphur.

21C. The p-Block Elements - Halogens 705-720


General characteristics; Preparation, propeties and uses of fluorine, chlorine, bromine & iodine and bleaching powder; Abnormal behaviour
of hydrofluoric acid.

21D. The p-Block Elements – Noble Gases 721-728


General characteristics; Chemical properties; Discovery of noble gases; Isolation of noble gases; Structure of compounds of noble gases;
Uses of noble gase; Clathrates.

22. The d & f-Block Elements 729-762


General discussion of transition metals and their physical properties; Metallurgy of iron; Compounds of iron; Metallurgy of copper and
compounds of copper; Important alloys of copper; Metallurgy of silver and its compounds; Metallurgy of gold and its compounds; Metallurgy
of zinc and its compounds; Metallurgy of mercury and its compounds; Preparation and properties of potassium permarganate; Preparation
and properties of potassium dichromate; Inner transition elements; General properties of lanthanides and actinides.

23. Coordination Compounds 763-788


Coordination compound; Important definitions; W erner’s theory; Nomenclature of coordination compounds; Isomerism in coordination
compounds; Valence bond theory; Crystal field theory; Properties of coordination compounds; Importance of coordination compounds;
Organometallic compounds, their synthesis and properties; Metal carbonyls.

24. Haloalkanes and Haloarenes 789-822


Preparation properties and uses SN1 and SN2 mechanism; Aryl and aryl substituted alkyl halides and their perparation & chemical properties;
Polyholagen derivatives, their preparation and properties; Chloroform, iodoform, carbon tertrachloride; Unsaturated halogen derivatives.

25. Alcohols, Phenols and Ethers 823-864


Classification; Nomenclature; Isomerism; General methods of preparation and properties; Methyl alcohol, manufacturing; Ethyl alcohol,
manufacturing; Important terms related to monohydroxyl alcohols; Polyhydroxyl alcohols; Ethylene glycol, preparation, propeties and uses;
Glycerol, preparation properties and uses; Aromatic hydroxyl compounds; Preparation, properties and uses of phenol, ethers - aliphatic
and aromatic; Thio- Ethers.

26A. Aldehydes & Ketones 865-898


Nature of carbonyl compounds; Nomenclature of aldehydes and ketones; Isomerism; General methods of perparation of aldehydes and
ketones and their properties; Aldol condensation; Important chemical reactions such as Cannizzaro’s reaction, Crossed Cannizzaro’s
reaction; Tischencho reaction, Reformatsky reaction, Beckmann’s rearrangement; Polymers of acetone, formadehyde and acetaldehyde;
Aromatic aldehydes and ketones and their properties.

26B. Carboxylic Acids and their Derivatives 899-938


Classification, nomenclature and isomerism; General methods of preparation and properties of monobasic acids; pKa values of some
important acids; Reducing character of formic acid; Acid derivatives, acid chlorides, anhydrides, esters and amides; Urea (carbamide);
Substituted acids; a, b, g amino acids; a, b, g hydroxy acids; a, b, g halogeno acids; Saturated dicarboxylic acids; Blanck’s rule; Tartaric
acid; Citric acid; Aromatic acids, benzoic acid; Acid strength of substituted benzoic acids; Salicylic acidnitrogen, phosphorous and their
oxyacids; Manufacturing of phosphorous and its compounds; Fertilizers.
27. Amines 939-976
Amines, classification, nomenclature and isomerism; Methods of preparation; Separation of amines (Hinsberg’s method and Hoffman’s
method); Methods giving primary amines only, secondary amines only, tertiary amines and their chemical properties; Aromatic amines;
Preparation properties and uses; Nitro compounds, aliphatic and aromatic; Methyl orange; Cyanides and isocyanides.

28. Biomolecules 977-996


Carbohydrates, their classification; Starch, cellulose, glucose, fructose; Mutarotation; Epimers and epimerisation; Amino acids and proteins,
classification; Essential and non essential amino acids; Nomenclature and structure; Stucture of proteins; Classification of proteins; Test for
proteins; Enzymes; Different types of enzymes and their characteristics; Application of enzymes; Nucleic acids; Nucleotides, nucleosides,
DNA, RNA and their sturctures; Function of nucleic acids; Lipids & their types; Difference between oils and fats; Waxes; Complex lipids;
Steroids and terpenes; Essential oils; Saponification value, iodine; value and Reichert -Meissel value (RM value); Vitamins; Hormones.

29. Polymers 997-1008


Polymerisation; Types of polymers; Classification of polymers, based on structure; and stereochemical structure; Classification based on
mode of synthesis; Chain growth polymers and step growth polymers; Classification on basis of physical properties and interparticle
forces; Vulcanisation of rubber; Antioxidants; Molecular mass of polymers; Poly dispersity index; Important addition polymers; Important
condensation polymers

30. Chemistry in Everyday Life 1009-1020


Dyes; Chromophores, auxochromes, bathochromes etc.; Classification based on structure application; Direct dyes, Vat dyes mordant
dyes, ingrain dyes, disperse; Chemotherapy, antiseptics, disinfectants, antipyretics; analgetics, tranqulizers, narcotics, antibiotics etc.;
Sulpha drugs; Rocket propellants; Solid, liquid and hybrid propellants; Chemicals in cosmetics; Chemicals in food; Insect attractants /
Repellents.

31A. Volumetric Analysis 1021-1036


Volumetric methods and their types; Types of indicators; Standard of solutions

31B. Principles related to Practical Chemistry 1037-1070


Detection of N,S, and halogens in organic compounds; Detection of functional groups; Chemistry involved in preparation of inorganic
compounds; Chemistry involve in preparation of organic compounds; Chemistry involved in the titrimetric exercises; Chemical principles
involved in the qualitative salt analysis; Detection of acid radicals; Analysis of basic radicals; Tests (chemical readings) involved in detection
of basic radicals; Action of heat on some inorganic compounds; Chemical principles involved in some experiments

APPENDIX I : REACTIONS & MECHANISM 1071-1084


JEE MAIN 2015 CHEMISTRY
(Held on 4th April-2015)

1. Which compound would give 5 - keto - 2 - methylhexanal 6. Which of the following compounds is not colored yellow ?
upon ozonolysis ? (a) (NH4)3 [As(Mo3O10)4 ] (b) BaCrO4
(c) Zn2[Fe(CN)6] (d) K3[Co(NO2)6]
CH3
7. Which of the following is the energy of a possible excited
CH3
state of hydrogen ?
H3C (a) –3.4 eV (b) +6.8 eV
(a) (b) (c) +13.6 eV (d) –6.8 eV
CH3 8. Which of the following compounds is not an antacid ?
(a) Phenelzine (b) Ranitidine
CH3 (c) Aluminium hydroxide (d) Cimetidine
CH3 9. The ionic radii (in Å) of N3–, O2– and F– are respectively :
(a) 1.71, 1.40 and 1.36 (b) 1.71, 1.36 and 1.40
(c) CH3 (d) (c) 1.36, 1.40 and 1.71 (d) 1.36, 1.71 and 1.40
10. In the context of the Hall - Heroult process for the extraction
CH3 of Al, which of the following statements is false ?
2. Which of the vitamins given below is water soluble ? (a) Al3+ is reduced at the cathode to form Al
(a) Vitamin E (b) Vitamin K (b) Na3AlF6 serves as the electrolyte
(c) Vitamin C (d) Vitamin D (c) CO and CO2 are produced in this process
3. Which one of the following alkaline earth metal sulphates (d) Al2O3 is mixed with CaF2 which lowers the melting point
has its hydration enthalpy greater than its lattice enthalpy ? of the mixture and brings conductivity
(a) BaSO4 (b) SrSO4 11. In the following sequence of reactions :
(c) CaSO4 (d) BeSO4 KMnO SOCl H / Pd
Toluene ¾¾¾¾
4
® A ¾¾¾
2
® B ¾¾¾®
2
C
4. In the reaction BaSO 4

the product C is :
NH2
(a) C6H5CH2OH (b) C6H5CHO
(c) C6H5COOH (d) C6H5CH3
¾¾¾¾¾
NaNO /HCl
2
® D ¾¾¾¾¾
CuCN/KCN
® E + N2 12. Higher order (>3) reactions are rare due to :
0–5° C D
(a) shifting of equilibrium towards reactants due to elastic
collisions
CH3 (b) loss of active species on collision
(c) low probability of simultaneous collision of all the
the product E is : reacting species
CH3 (d) increase in entropy and activation energy as more
CN
molecules are involved
13. Which of the following compounds will exhibit geometrical
(a) (b) isomerism ?
(a) 2 - Phenyl -1 - butene
(b) 1, 1 - Diphenyl - 1 - propene
CH3
(c) 1 - Phenyl - 2 - butene
COOH (d) 3 - Phenyl -1 - butene
14. Match the catalysts to the correct processes :
Catalyst Process
(c) (d) H3C CH3 (A) TiCl4 (i) Wacker process
(B) PdCl2 (ii) Ziegler - Natta
polymerization
CH3
(C) CuCl2 (iii) Contact process
5. Sodium metal crystallizes in a body centred cubic lattice with (D) V2O5 (iv) Deacon's process
a unit cell edge of 4.29Å. The radius of sodium atom is (a) (A) - (ii), (B) - (iii), (C) - (iv), (D) - (i)
approximately : (b) (A) - (iii), (B) - (i), (C) - (ii), (D) - (iv)
(a) 5.72Å (b) 0.93Å (c) (A) - (iii), (B) - (ii), (C) - (iv), (D) - (i)
(c) 1.86Å (d) 3.22Å (d) (A) - (ii), (B) - (i), (C) - (iv), (D) - (iii)
2 CHEMISTRY
15. The intermolecular interaction that is dependent on the 24. Which among the following is the most reactive ?
inverse cube of distance between the molecules is : (a) I2 (b) IC1
(a) London force (b) hydrogen bond (c) Cl2 (d) Br2
(c) ion - ion interaction (d) ion - dipole interaction 25. The standard Gibbs energy change at 300 K for the reaction
16. The molecular formula of a commercial resin used for
ˆˆ† B + C is 2494.2 J. At a given time, the composition
2A ‡ˆˆ
exchanging ions in water softening is C8H7SO3– Na+ (Mol.
wt. 206. What would be the maximum uptake of Ca2 + ions by
the resin when expressed in mole per gram resin ? 1 1
of the reaction mixture is [A] = , [B] = 2 and [C] = . The
2 2
2 1 reaction proceeds in the : [R = 8.314 J/K/mol, e = 2.718]
(a) (b)
309 412 (a) forward direction because Q < Kc
1 1 (b) reverse direction because Q < Kc
(c) (d) (c) forward direction because Q > Kc
103 206
17. Two Faraday of electricity is passed through a solution of (d) reverse direction because Q > Kc
CuSO4. The mass of copper deposited at the cathode is 26. Assertion: Nitrogen and oxygen are the main components in
(at. mass of Cu = 63.5 amu) the atmosphere but these do not react to form oxides of
nitrogen.
(a) 2g (b) 127 g
Reason: The reaction between nitrogen and oxygen requires
(c) 0 g (d) 63.5 g
high temperature.
18. The number of geometric isomers that can exist for square
(a) The assertion is incorrect, but the reason is correct
planar complex [Pt (Cl) (py) (NH3 ) (NH2 OH)] + is
(py = pyridine) : (b) Both the assertion and reason are incorrect
(a) 4 (b) 6 (c) Both assertion and reason are correct, and the reason is
the correct explanation for the assertion
(c) 2 (d) 3
(d) Both assertion and reason are correct, but the reason is
19. In Carius method of estimation of halogens, 250 mg of an
not the correct explanation for the assertion
organic compound gave 141 mg of AgBr. The percentage of
bromine in the compound is : 27. Which one has the highest boiling point ?
(at. mass Ag =108; Br = 80) (a) Kr (b) Xe
(a) 48 (b) 60 (c) He (d) Ne
(c) 24 (d) 36 28. Which polymer is used in the manufacture of paints and
lacquers ?
20. The color of KMnO4 is due to :
(a) Polypropene (b) Polyvinyl chloride
(a) L ® M charge transfer transition
(c) Bakelite (d) Glyptal
(b) s - s* transition
29. The following reaction is performed at 298 K.
(c) M ® L charge transfer transition
(d) d – d transition ˆˆ† 2NO (g)
2NO(g) + O2(g) ‡ˆˆ 2
21. The synthesis of alkyl fluorides is best accomplished by :
The standard free energy of formation of NO(g) is 86.6 kj/mol
(a) Finkelstein reaction (b) Swarts reaction
at 298 K. What is the standard free energy of formation of
(c) Free radical fluorination (d) Sandmeyer's reaction
NO2(g) at 298 K? (Kp = 1.6 × 1012)
22. 3 g of activated charcoal was added to 50 mL of acetic acid
solution (0.06N) in a flask. After an hour it was filtered and ln (1.6 ´ 1012 )
the strength of the filtrate was found to be 0.042 N. The (a) 86600 –
R (298)
amount of acetic acid adsorbed (per gram of charcoal) is :
(b) 0.5[2 × 86,600 – R(298) ln(1.6 × 1012)]
(a) 42 mg (b) 54 mg
(c) R(298) ln(1.6 × 1012) – 86600
(c) 18 mg (d) 36 mg
(d) 86600 + R(298) ln(1.6 × l012)
23. The vapour pressure of acetone at 20°C is 185 torr. When 1.2
g of a non-volatile substance was dissolved in 100 g of 30. From the following statements regarding H2O2, choose the
acetone at 20°C, its vapour pressure was 183 torr. The molar incorrect statement :
mass (g mol–1) of the substance is : (a) It has to be stored in plastic or wax lined glass bottles in
(a) 128 (b) 488 dark
(c) 32 (d) 64 (b) It has to be kept away from dust
(c) It can act only as an oxidizing agent
(d) It decomposes on exposure to light
JEE MAIN 2015 Solved Paper 3

SOLUTIONS
1. (d) When 1, 3-dimethylcyclopentene is heated with ozone 10. (b) In the metallurgy of aluminium, purified Al2O3 is mixed
and then with zinc and acetic acid, oxidative cleavage with Na3AlF6 or CaF2 which lowers the melting point
leads to keto - aldehyde. of the mix and brings conductivity.
11. (b)
CH3 CH3
O CH3 COOH COCl CHO
1O - 78° C
¾ ¾ ¾3¾ ¾ ¾ ¾ ® O C–H
2 - Zn- CH 3 COOH KMnO4 SOCl2 H2/Pd
BaSO 4
CH3 CH3
(A) (B) (C)
O O
|| || Rosenmund’s
CH3 — C— CH 2 — CH 2 — CH— C— H reaction
|
CH3 12. (c) Reactions of higher order (>3) are very rare due to very
less chances of many molecules to undergo effective
5- keto – 2 – methylhexanal collisions.
2. (c) Water-soluble vitamins dissolve in water and are not
stored by the body. The water soluble vitamins include H
|
the vitamin B-complex group and vitamin C.
13. (c) H3C — C = CH — CH 2
3. (d) In alkaline earth metals, ionic size increases down the |
group. The lattice energy remains constant because Ph
sulphate ion is so large, so that small change in cationic
size does not make any difference. On moving down 1- Phenyl-2-butene the two groups around each of the
the group the degree of hydration of metal ions doubly bonded carbon
decreases very much leading to decrease in solubility. Because, all are different. This compound can show
cis-and trans-isomerism.
\ BeSO 4 > MgSO 4 > CaSO 4 > SrSO 4 > BaSO 4
14. (d) (A) - (ii), (B) - (i), (C) - (iv), (D) - (iii)
4. (a) 15. (b) Hydrogen bond is a type of strong electrostatic dipole-
+ – dipole interaction and dependent on the inverse cube
NH2 N = NCl CN of distance between the molecular ion-dipole
1
¾ NaNO
¾¾¾ 2 /HCl
¾¾ ® ¾ CuCN/KCN
¾ ¾ ¾ ¾® interaction µ .
0 - 5° C D r3
16. (b) 2 mole of water softner require 1 mole of Ca 2+ ion
CH3 CH3 CH3
1
5. (c) In bcc the atoms touch along body diagonal So, 1 mole of water softner require mole of Ca2+ ion
2
\ 2r + 2r = 3a
1 1
Thus, = mol / g will be maximum uptake
3a 3 ´ 4.29 2 ´ 206 412
\ r= = = 1.857Å
4 4 17. (d) Cu 2+ + 2e – ¾¾ ® Cu
6. (c)
2F i.e. 2 × 96500 C deposit Cu = 1 mol = 63.5 g
–13.6Z2 18. (d) Square planar complexes of type M[ABCD] form three
7. (a) Total energy = eV isomers. Their position may be obtained by fixing the
n2 position of one ligand and placing at the trans position
where n = 2, 3, 4 .... any one of the remaining three ligands one by one.
Putting n = 2
HOH2N Cl HOH2N NH3
-13.6
ET = = -3.4eV
4
8. (a) Phenelzine is an antidepressant, while others are Pt Pt
antacids.
9. (a) For isoelectronic species, size of anion increases as
negative charge increases. Thus the correct order is py NH3 py Cl
trans cis
N 3- > O 2 - > F -
(1.71) (1.40) (1.36)
4 CHEMISTRY
HOH2N where w1, M1 = mass in g and mol. mass of solvent
NH3 w2, M2 = mass in g and mol. mass of solute
Let M2 = x
Pt p° = 185 torr
ps = 183 torr

Cl py 185 - 183 1.2 ´ 58


cis = (Mol. mass of acetone = 58)
183 100 x
19. (c) Mass of substance = 250 mg = 0.250 g x = 64
Mass of AgBr = 141 mg = 0.141 g \ Molar mass of substance = 64
1 mole of AgBr = 1 g atom of Br 24. (b) ICl
188 g of AgBr = 80 g of Br Order of reactivity of halogens
\ 188 g of AgBr contain bromine = 80 g
Cl2 > Br2 > I2
80 But, the interhalogen compounds are generally more
0.141 g of AgBr contain bromine = ´ 0.141
188 reactive than halogens (except F2 ), since the bond
This much amount of bromine present in 0.250 g of between two dissimilar electronegative elements is
organic compound weaker than the bond between two similar atoms i.e, X – X
80 0.414 25. (d) DG° = 2494.2J
\ % of bromine = ´ ´100 = 24%
188 0.250 ˆˆ† B + C.
2A ‡ˆˆ
20. (a) L ® M charge transfer spectra. KMnO4 is colored
because it absorbs light in the visible range of R = 8.314 J/K/mol.
electromagnetic radiation. The permanganate ion is the e = 2.718
source of color, as a ligand to metal, (L ® M) charge 1 1
transfer takes place between oxygen's p orbitals and [A] = , [B] = 2, [C] =
2 2
the empty d-orbitals on the metal. This charge transfer
takes place when a photon of light is absorbed, which [B] [C] 2 ´ 1 2
Q= = 2 =4
leads to the purple color of the compound. [A]2 æ 1ö
21. (b) Alkyl fluorides are more conveniently prepared by çè ÷ø
2
heating suitable chloro – or bromo-alkanes with organic
fluorides such as AsF3, SbF3, CoF2, AgF, Hg2F2 etc. DG° = – 2.303 RT log KC.
This reaction is called Swarts reaction. 2494.2 J = – 2.303 × (8.314 J/K/mol) × (300K) logKC

CH 3Br + AgF ¾¾
® CH 3 F + AgBr 2494.2 J
Þ log KC = –
2.303 ´ 8.314 J / K / mol ´ 300 K
2CH3CH 2 Cl + Hg 2 F2 ¾¾ ® 2CH 3CH 2 F + Hg 2Cl 2 Þ log KC = – 0.4341
22. (c) Let the weight of acetic acid initially be w1 in 50 ml of KC = 0.37
0.060 N solution. Q > KC.
w 1 ´ 1000 26. (c) Nitrogen and oxgen in air do not react to form oxides of
Let the N = (Normality = 0.06 N) nitrogen in atmosphere because the reaction between
M.wt . ´ 50 nitrogen and oxygen requires high temperature.
w1 ´ 1000 27. (b) Xe. As we move down the group, the melting and boiling
0.06 = points show a regular increase due to corresponding
60 ´ 50
increase in the magnitude of their van der waal forces of
0.06 ´ 60 ´ 50 attraction as the size of the atom increases.
Þ w1 = = 0.18 g = 180 mg.
1000 28. (d) Glyptal is used in the manufacture of paints and
After an hour, the strength of acetic acid = 0.042 N lacquers.
so, let the weight of acetic acid be w2 29. (b) DG°NO(g) = 86.6k J/mol = 86600 J/mol
w ´1000 G° NO2 (g) = x J/mol
N= 2
60 ´ 50 T = 298, KP = 1.6 × 1012
w ´ 1000 DG° = – RT ln KP
0.042 = 2 Given equation,
3000
Þ w2 = 0.126 g = 126 mg ˆˆ† 2NO (g)
2NO(g) + O2 (g) ‡ˆˆ 2
So amount of acetic acid adsorbed per 3g
= 180 – 126 mg = 54 mg \ 2DG°NO – 2DG°NO = – R (298) ln (1.6 × 1012)
2
Amount of acetic acid adsorbed per g 2DG°NO – 2 × 86600 = – R (298) ln (1.6 × 1012)
2
2DG°NO = 2 × 86600 – R (298) ln (1.6 × 1012)
54 2
= = 18 mg 1
3 DG°NO = [2 × 86600 – R(298) ln (1.6 × 1012]
23. (d) Using relation, 22
= 0.5 [2 × 86600 – R (298) ln (1.6 × 1012)]
p° - ps w 2 M1
= 30. (c) H 2O2 has oxidizing and reducing properties both.
ps w1M 2
JEE MAIN 2014 CHEMISTRY
(Held on 6th April-2014)
1. The correct set of four quantum numbers for the valence 7. The equivalent conductance of NaCl at concentration C and
electrons of rubidium atom (Z = 37) is: at infinite dilution are lC and l¥ , respectively. The correct
1 1
(a) 5, 0, 0, + (b) 5,1, 0, + relationship between lC and l¥ is given as:
2 2
(Where the constant B is positive)
1 1
(c) 5,1,1, + (d) 5, 0,1, + (a) lC = l ¥ + ( B ) C (b) lC = l ¥ - ( B ) C
2 2
2. If Z is a compressibility factor, van der Waals equation at low
pressure can be written as: (c) lC = l¥ - ( B ) C (d) lC = l¥ + ( B ) C

RT a 8. Consider separate solutions of 0.500 M C 2 H5 OH(aq),


(a) Z = 1 + (b) Z = 1- 0.100 M Mg3 (PO4)2 (aq), 0.250 M KBr(aq) and 0.125 M
Pb VRT
Na3PO4(aq) at 25°C. Which statement is true about these
Pb Pb
(c) Z = 1 - (d) Z = 1 + solutions, assuming all salts to be strong electrolytes?
RT RT
(a) They all have the same osmotic pressure.
3. CsCl crystallises in body centred cubic lattice. If ‘a’ is its
edge length then which of the following expressions is (b) 0.100 M Mg3 (PO4 )2 (aq) has the highest osmotic
correct? pressure.
(c) 0.125 M Na3PO4(aq) has the highest osmotic pressure.
3a
(a) rCs + + rCl - = 3a (b) r
Cs +
+r
Cl -
= (d) 0.500 M C2H5OH(aq) has the highest osmotic pressure.
2
1
9. ˆˆ† SO3 g , if
For the reaction SO 2( g ) + O 2( g ) ‡ˆˆ
3 ( )
(c) r + + r - = a (d) rCs + + rCl - = 3a 2
Cs Cl 2
x
4. For the estimation of nitrogen, 1.4 g of an organic compound K P = KC ( RT ) where the symbols have usual meaning then
was digested by Kjeldahl method and the evolved ammonia the value of x is (assuming ideality):
M 1
was absorbed in 60 mL of sulphuric acid. The unreacted (a) –1 (b) -
10 2
M 1
acid required 20 mL of sodium hydroxide for complete (c) (d) 1
10 2
neutralization. The percentage of nitrogen in the compound 10. For the non - stoichimetre reaction 2A + B ® C + D, the
is: following kinetic data were obtained in three separate
(a) 6% (b) 10% experiments, all at 298 K.
(c) 3% (d) 5%
5. Resistance of 0.2 M solution of an electrolyte is 50 W.
The specific conductance of the solution is 1.4 S m–1 . Initial Initial Initial rate of
The resistance of 0.5 M solution of the same electrolyte is 280 Concentration Concentration formation of C
W. The molar conductivity of 0.5 M solution of the electrolyte (A ) (B ) –1 –1
(mol L s )
in S m2 mol–1 is: –3
(a) 5 × 10–4 (b) 5 × 10–3 0.1 M 0.1 M 1.2 × 10
(c) 5 × 10 3 (d) 5 × 102 0.1 M 0.2 M –3
1.2 × 10
6. For complete combustion of ethanol, 0.2 M 0.1 M 2.4 × 10
–3

C2 H5OH ( l ) + 3O2 ( g ) ¾¾
® 2CO2 ( g ) + 3H 2 O ( l ) ,
the amount of heat produced as measured in bomb calorimeter, The rate law for the formation of C is:
is 1364.47 kJ mol–1 at 25ºC. Assuming ideality the enthalpy dc dc
= k [ A][ B ] = k [ A] [ B ]
2

of combustion, DcH, for the reaction will be: (a) (b)


dt dt
(R = 8.314 kJ mol–1)
dc dc
= k [ A][ B ] = k [ A]
2
(a) -1366.95 kJ mol-1 (b) -1361.95 kJ mol-1 (c) (d)
dt dt
(c) -1460.95 kJ mol-1 (d) -1350.50 kJ mol-1
2014-2 CHEMISTRY
11. Among the following oxoacids, the correct decreasing order 19. Which series of reactions correctly represents chemical
of acid strength is: reactions related to iron and its compound?
(a) HOCl > HClO2 > HClO3 > HClO4 dil. H 2SO 4
(a) Fe ¾¾¾¾¾ H 2SO 4 , O 2
® FeSO 4 ¾¾¾¾¾ ®
(b) HClO4 > HOCl > HClO 2 > HClO3
Fe 2 ( SO4 )3 ¾¾¾
heat
® Fe
(c) HClO4 > HClO3 > HClO2 > HOCl
O2 , heat dil. H2SO4 heat
(d) HClO2 > HClO 4 > HClO3 > HOCl (b) Fe ¾¾¾¾ ® FeO ¾¾¾¾¾ ® FeSO 4 ¾¾¾ ® Fe
12. The metal that cannot be obtained by electrolysis of an Cl2 , heat heat , air Zn
aqueous solution of its salts is: (c) Fe ¾¾¾¾® FeCl3 ¾¾¾¾® FeCl 2 ¾¾® Fe
(a) Ag (b) Ca O 2 , heat CO , 600°C
(c) Cu (d) Cr (d) Fe ¾¾¾¾ ® Fe3O 4 ¾¾¾¾¾ ®
13. The octahedral complex of a metal ion M3+ with four CO , 700°C
FeO ¾¾¾¾¾
® Fe
monodentate ligands L1, L2, L3 and L4 absorb wavelengths
in the region of red, green, yellow and blue, respectively. The 20. The equation which is balanced and represents the correct
increasing order of ligand strength of the four ligands is: product(s) is:
(a) L4 < L3 < L2 < L1 (b) L1 < L3 < L2 < L4 (a) Li 2O + 2KCl ® 2LiCl + K 2 O

(c) L3 < L2 < L4 < L1 (d) L1 < L2 < L4 < L3 +


(b) éë CoCl ( NH3 )5 ùû + 5H + ® Co2 + + 5NH +4 + Cl-
14. Which one of the following properties is not shown by NO?
(a) It is diamagnetic in gaseous state 2+ 4-
(c) éë Mg ( H 2 O )6 ùû + ( EDTA )
excess NaOH
(b) It is neutral oxide ¾¾¾¾¾¾
®
(c) It combines with oxygen to form nitrogen dioxide
2+
(d) It’s bond order is 2.5 éë Mg ( EDTA ) ùû + 6H 2 O
15. In which of the following reactions H2O2 acts as a reducing
agent? (d) CuSO 4 + 4KCN ® K 2 éëCu ( CN ) 4 ùû + K 2SO 4
(a) H 2 O 2 + 2H + + 2e - ® 2H 2 O 21. In SN2 reactions, the correct order of reactivity for the
following compounds:
(b) H 2 O 2 + 2e - ® O2 + 2H +
CH3Cl , CH3CH 2 Cl , (CH3 )2 CHCl and ( CH3 )3 CCl is:
(c) H 2 O 2 + 2e - ® 2OH -
(a) CH3 Cl > ( CH3 )2 CHCl > CH3 CH 2Cl > ( CH3 )3 CCl
(d) H 2 O 2 + 2OH - - 2e - ® O2 + 2H 2 O
16. The correct statement for the molecule, CsI3 is: (b) CH3Cl > CH 3CH 2 Cl > ( CH3 ) 2 CHCl > ( CH 3 )3 CCl
(a) It is a covalent molecule.
(c) CH3CH 2 Cl > CH3Cl > ( CH3 ) 2 CHCl > ( CH 3 )3 CCl
(b) It contains Cs+ and I3- ions.
(c) It contains Cs3+ and I– ions. (d) ( CH3 )2 CHCl > CH3CH 2Cl > CH3 Cl > ( CH3 )3 CCl
(d) It contains Cs+, I– and lattice I2 molecule. 22. On heating an aliphatic primary amine with chloroform and
17. The ratio of masses of oxygen and nitrogen in a particular ethanolic potassium hydroxide, the organic compound formed
gaseous mixture is 1 : 4. The ratio of number of their molecule is:
is: (a) an alkanol (b) an alkanediol
(a) 1 : 4 (b) 7 : 32 (c) an alkyl cyanide (d) an alkyl isocyanide
(c) 1 : 8 (d) 3 : 16 23. The most suitable reagent for the conversion of
18. Given below are the half-cell reactions:
R - CH 2 - OH ® R - CHO is:
Mn 2 + + 2e - ® Mn ; E 0 = -1.18 V (a) KMnO4

( )
2 Mn3+ + e - ® Mn 2+ ; E 0 = +1.51V
(b) K2Cr2O7
(c) CrO3
The E0 for 3Mn 2 + ® Mn + 2Mn 3+ will be: (d) PCC (Pyridinium Chlorochromate)
24. The major organic compound formed by the raction of
(a) –2.69 V; the reaction will not occur
1, 1, 1-trichloroethane with silver powder is:
(b) –2.69 V; the reaction will occur
(a) Acetylene (b) Ethene
(c) –0.33 V; the reaction will not occur
(d) –0.33 V; the reaction will occur (c) 2 - Butyne (d) 2 - Butene
JEE MAIN 2014 SOLVED PAPER 2014-3

25. Sodium phenoxide when heated with CO2 under pressure at 27. For which of the following molecule significant m ¹ 0?
125ºC yields a product which on acetylation produces C.
CI CN
ONa 125° H+
+ CO 2 ¾¾¾¾ ®B ¾¾¾® C
5 Atm Ac2 O
(i) (ii)
The major product C would be
OCOCH3 OH CI CN

COOH COCH3
(a) (b) OH SH

COCH3 (iii) (iv)

OH OCOCH3
OH SH
COOCH3 (a) Only (i) (b) (i) and (ii)
(c) (d)
COOH (c) Only (iii) (d) (iii) and (iv)
28. Which one is classified as a condensation polymer?
26. Considering the basic strength of amines in aqueous solution,
which one has the smallest pKb value? (a) Dacron (b) Neoprene
(a) (CH3)2NH (b) CH3NH2 (c) Teflon (d) Acrylonitrile
(c) (CH3)3N (d) C6H5NH2 29. Which one of the following bases is not present in DNA?
(a) Quinoline (b) Adenine
(c) Cytosine (d) Thymine
30. In the reaction,
LiAH PCl Alc.KOH
CH 3 COOH ¾¾¾¾
4 ® A ¾¾¾
5 ® B ¾¾¾¾® C,

the product C is:


(a) Acetaldehyde (b) Acetylene
(c) Ethylene (d) Acetyl chloride
2014-4 CHEMISTRY

SOLUTIONS
1. (a) The electronic configuration of Rubidium (Rb = 37) is
1.4 ´ meq. of acid
1s 2 2s 2 2 p 6 3s 2 3 p 6 3d 10 4s 2 4 p 6 5s1 4. (b) % of N =
mass of organic compound
Since last electron enters in 5s orbital
1 M
Hence n = 5, l = 0, m = 0, s = ± meq. of H2SO4 = 60´ ´ 2 = 12
2 10

PV M
2. (b) Compressibility factor ( Z ) = meq. of NaOH = 20´ = 2
RT 10
(For one mole of real gas)
van der Waals equation \ meq. of acid consumed = 12 – 2 = 10

a 1.4 ´ 10
(P + )(V - b) = RT \ % of N = = 10%
2 1.4
V
At low pressure, volume is very large and hence 5. (a) Given for 0.2 M solution
correction term b can be neglected in comparison to
R = 50 W
very large volume of V.
i.e. V - b » V k = 1.4 S m–1 = 1.4 × 10–2 S cm–1

æ a ö l 1 l
Now, R = r = ´
çè P + 2 ÷ø V = RT a k a
V
a l
PV + = RT Þ = R ´ k = 50 ´1.4 ´ 10-2
V a
a For 0.5 M solution
PV = RT -
V R = 280 W
PV a k =?
= 1-
RT VRT
l
= 50 ´ 1.4 ´ 10-2
a a
Hence, Z = 1 -
VRT
l 1 l
Þ R=r = ´
3. (c) a k a
Cl – Cl –
1
Cl – Cl Þ k= ´ 50 ´ 1.4 ´ 10 -2
280

Cs+ =
1
´ 70 ´ 10-2 = 2.5 × 10–3 S cm–1
280

Cl – Cl k ´ 1000 2.5 ´ 10-3 ´ 1000


Cl – Now, L m = =
Cl – M 0.5
= 5 S cm2 mol–1 = 5 × 10–4 S m2 mol–1
Relation between radius of cation, anion and edge
length of the cube 6. (a) C 2 H5 OH(l) + 3O2 ( g ) ® 2CO ( g ) + 3H O( l)
2 2

2r + 2r = 3a Bomb calorimeter gives DU of the reaction


Cs+ Cl-
Given, DU = –1364.47 kJ mol–1
3a
r +r = Dng = – 1
Cs+ Cl- 2
JEE MAIN 2014 SOLVED PAPER 2014-5

DH = DU + DngRT 11. (c) Acidic strength increases as the oxidation number of


central atom increases.
1 ´ 8.314 ´ 298
= -1364.47 - = – 1366.93 kJ mol–1 Hence acidic strength order is
1000
(+7) (+5) (+3) (+1)
7. (c) According to Debye Huckle onsager equation,
HClO4 > HClO3 > HClO2 > HClO
lC = l¥ - B C
12. (b) On electrolysis of aqueous solution of s-block elements
8. (a) p = i CRT H2 gas discharge at cathode.
p
C2 H5 OH = 1´ 0.500 ´ R ´ T = 0.5 RT 1
At cathode: H 2O + e- ® H 2 + OH -
2
p
Mg3 (PO 4 )2 = 5 ´ 0.100 ´ R ´ T = 0.5 RT

pKBr = 2 ´ 0.250 ´ R ´ T = 0.5 RT 13. (b) B


V G
p Na PO
3 4
= 4 ´ 0.125 ´ RT = 0.5 RT

Since the osmotic pressure of all the given solutions is


equal. Hence all are isotonic solution. R Y

1 O
9. (b) ˆˆ† SO3 (g)
SO 2 (g) + O 2 (g) ‡ˆˆ
2
For a given metal ion, weak field ligands create a complex
x with smaller D, which will absorbs light of longer l and
K P = K C ( RT )
thus lower frequency. Conservely, stronger field ligands
create a larger D, absorb light of shorter l and thus
where x = Dng = number of gaseous moles in product
higher v i.e. higher energy.
– number of gaseous moles in reactant
Red < Yellow < Green < Blue
l = 650 nm 570 nm 490 nm 450 nm
æ 1ö 3 1
= 1 - ç1 + ÷ = 1 - = - So order of ligand strength is
è 2 ø 2 2
L1 < L3 < L2 < L4
d [C ] 14. (a) Nitric oxide is paramagnetic in the gaseous state
10. (d) Let rate of reaction = = k[A]x [B]y because of the presence of one unpaired electron in its
t
outermost shell.
Now from the given data
The electronic configuration of NO is
1.2 × 10 – 3 = k [0.1]x[0.1]y .....(i)
1.2 × 10 – 3 = k [0.1]x[0.2]y .....(ii) s12s s1*2s s 22 s s*2 2 2 2 *1
2 s s 2 p p 2 p x = p2 p y p 2 p x
z
2.4 × 10 – 3 = k [0.2]x[0.1]y .....(iii) 15. (d) The reducing agent loses electron during redox
Dividing equation (i) by (ii) reaction i.e. oxidises itself.
-1 -2
1.2 ´10-3 k[0.1]x [0.1] y (a) H 2O 2 + 2H + + 2e - ¾¾
® 2H 2 O (Re d.)
Þ =
1.2 ´10-3 k [0.1]x [0.2] y
-1 0
We find, y = 0
(b) ® O 2 + 2H + + 2e- (Ox.)
H 2O 2 ¾¾
Now dividing equation (i) by (iii)
-1 -2
1.2 ´10-3 k[0.1]x [0.1] y (c) H 2O 2 + 2e - ¾¾
® 2 OH - (Re d.)
Þ =
2.4 ´10 -3 k [0.2] x [0.1] y

We find, x = 1 0
(d) H 2O-2 1 + 2OH - ¾¾
® O 2 + H 2 O + 2e - (Ox.)
d [C ]
Hence = k[ A]1 [ B ]0
dt
2014-6 CHEMISTRY
16. (b) CsI3 dissociates as
CsI3 ® Cs+ + I3– Cl
|
w 24. (c) 2Cl - C - CH3 + 6Ag
17. (b) Number of moles of O 2 = |
32 Cl
4w w
Number of moles of N 2 = = 1, 1, 1-trichloroethane
28 7
¾¾
® CH 3C º CCH 3 + 6AgCl
\ Ratio = w : w = 7 : 32
32 7 2-butyne

18. (a) (a) Mn 2+ + 2e- ® Mn; E 0 = -1.18V ; ... (i)


ONa OH
3+ 2+ 0 + CO2 ¾¾
®
(b) Mn + e ® Mn ; E = -1.51V ; ... (ii) 25. (a)

Now multiplying equation (ii) by two and subtracting COONa


Sodium
from equation (i) Phenoxide

3Mn 2+ ® Mn + + 2Mn 3+ ;
OH
H SO
0
E = EOx. + ERed. = – 1.18 + (– 1.51) = – 2.69 V ¾¾¾¾
2 4®

COOH
(–ve value of EMF (i.e. DG = +ve) shows that the
Salicylic acid
reaction is non-spontaneous)
19. (c) In equation (i) Fe2(SO4)3 and in equation (ii) Fe2(SO4)3
on decomposing will form oxide instead of Fe. (CH 3 CO) 2 O
The correct sequence of reactions is
O ,heat Co,600° C
Fe ¾¾¾¾
2 ® Fe3O 4 ¾¾¾¾¾ ®
O
D ||
Fe 2 (SO 4 )3 ¾¾® Fe O - C - CH3
+ CH3COOH
20. (b) The complex [CoCl(NH3 )5 ]+ decomposes under
COOH
acidic medium, so Aspirin
(Acetyl Salicylate)
[CoCl(NH3 )5 ]+ + 5H + ¾¾
® Co 2+ + 5NH 4 + + Cl -

21. (b) Steric congestion around the carbon atom undergoing 26. (a) Arylamines are less basic than alkyl amines and even
the inversion process will slow down the SN2 reaction, ammonia. This is due to resonance. In aryl amines the
hence less congestion faster will the reaction. So, the lone pair of electrons on N is partly shared with the
order is ring and is thus less available for sharing with a proton.
CH3Cl > (CH3)CH2 – Cl > (CH3)2CH – Cl > (CH3)3CCl In alkylamines, the electron releasing alkyl group
22. (d) R – CH2 – NH2 + CHCl3 + 3KOH (alc) ¾¾
® increases the electron density on nitrogen atom and
Carbyl amine reaction thus also increases the ability of amine for protonation.
R – CH2 – NC + 3KCl + 3H2O Hence more the no. of alkyl groups higher should be
Alkyl isocynide
the basicity of amine. But a slight discrepancy occurs
in case of trimethyl amines due to steric effect. Hence
23. (d) An excellent reagent for oxidation of 1° alcohols to
the correct order is
aldehydes is PCC.
PCC
(CH3 )2 NH > CH3 NH 2 > (CH3 )3 N > C 6 H5 NH 2
R - CH 2 - OH ¾¾¾
® R - CHO
JEE MAIN 2014 SOLVED PAPER 2014-7

27. (d)
O O
H
[CO CH 2 - CH 2 - O - ] n
H Dacron (Polyester)

29. (a) DNA contains ATGC bases


H
So quinoline is not present in DNA.
S S
H
30. (c) CH3 COOH ¾¾¾¾
LiA1H
4 ® CH CH OH
In both the molecules the bond moments are not 3 2
(A)
canceling with each other and hence the molecules
PCl5
has a resultant dipole and hence the molecule is polar.
28. (a) Except Dacron all are additive polymers. Terephthalic CH3CH2Cl
acid condenses with ethylene glycol to give Dacron. (B)

Alc. KOH
HOOC COOH +
CH2 = CH2
(C)
Terephthalic acid

HO – CH2 – CH2 – OH ¾¾
® Hence the product (C) is ethylene.
Ethylene glycol
1
Some Basic Concepts
of Chemistry
PHYSICAL QUANTITIES AND SI UNITS : Physical Definition SI Unit
The 11th general conference of weights and measures in 1960 Quantity
recommended the use of international system of units. volume length cube m3
Abbreviated as SI Units (after the French expression La System area length square m2
International de units). The SI system has seven basic units of speed distance travelled ms– 1
physical quantities as follows : per unit time

Physical quantity Abbreviation Name of unit Symbol acceleration speed changed ms– 2
per unit time
time t second s
density mass per unit volume kg m–3
mass m kilogram kg
pressure force per unit area kgm–1s–2 or Nm–2
length l metre m (pressure = Pa)
temperature T kelvin K force mass times acceleration kgms–2
of object (Newton N)
electric current I ampere A
energy force times distance kgm2s–2
light intensity Iv candela Cd
travelled (Joule J)
amount of substance n mole mol frequency cycles per second s–1 (hertz = Hz)

DERIVED UNITS : power energy per second kgm2s–3 or Js–1


(Watt = W)
The units obtained by combination of basic units are
electric charge ampere times second As (coloumb = C)
known as derived units e.g. velocity is expressed as distance/
time. Hence unit is m/s or ms–1. Some common derived units are electric potential energy per unit JA –1 s –1 or kgm 2 s – 3
difference charge A–1 (volt = V)

SOME NON SI UNITS IN COMMON USE :

Quantity Unit Symbol SI definition SI Name

Length angstrom Å 10–10 m 0.1 nanometers (nm)

Volume litre L 10–3 m3 1 decimeter (dm3)

Energy calorie cal kg m2s–2 4.184 Joule (J)


2 Chemistry

Mass atomic mass unit amu 1.66057 × 10–27 kg

pound lb 0.453502 kg

Force dyne dyn 10–5 N

pound force lbf 4.44822 N

Pressure bar bar 105 Nm–2 (or Pa)

atmosphere atm 101325 Nm–2 (or Pa)

Torr torr 101325 / 760 Nm–2 (or Pa)

mm of Hg mmHg 13.5951 × 980.665 × 10–2 Nm–2

STANDARD PREFIXES FOR EXPRESSING THE DECIMAL FRACTIONS OR MULTIPLES OF FUNDAMENTAL


UNITS :
Fraction Prefix Symbol Multiple Prefix Symbol

10 –1 deci d 10 1 Deka da
10 –2 centi c 10 2 Hecta h
10 –3 milli m 10 3 kilo k
10 –6 micro m 10 6 Mega M
10 –9 nano n 10 9 Giga G
10 –12 pico p 10 12 Tera T
10 –15 femto f 10 15 Peta P

10 –18 atto a 10 18 Exa E

SOME CONVERSION FACTORS : 6. Force : 1 N = 105 dyne = 0.22481 lbf


1. Mass : 1 kg = 10 3g = 2.2046 lb; 1 mg (milligram) = 10 –3g; 7. Power : 1 Watt = 107 ergs–1 = 1.341 × 10–3 hP
1 mg (micro gram) = 10–6g 1 W = 1 J sec–1 = 1 VA
1 1 hP = 745.7 Watt
1 amu = mass of isotope of C12 8. Temperature :
12
K = °C + 273.15
1 amu = 1.6605 × 10–24 g = 1.6605 × 10–27 kg
2. Length : 1Å = 10–8 cm = 10–10 m; 5
°C = ´ (°F - 32)
1 pm = 10–10 cm = 10–12 m= 10–2Å 9
1 nm = 10–7 cm = 10–9 m = 10 Å
9
1 nm = 103 pm °F = ´ °C + 32
5
3. Volume : 1 litre = 1000 cm 3 = 1000 ml = 1 dm 3 = 10 –3 m 3
= 0.22 gal Dimensional analysis factor label method or unit factor method.
1 m3 = 106 cm3 = 35.3146 ft3 It is conversion of units from one system to other.
4. Pressure : 1 atm = 760 torr = 760 mm Hg = 76 cm Hg Example : Conversion of L (litre) into m3 (metre). Find unit factor
= 1.013 × 105 Pa relating L and m3
= 1.01325 bars = 101325 Nm–2 = 14.7 lb m–2 (psi) 1L = 1000 cm3
= 76.0 × 13.595 × 980.66 = 1.01325 × 106 dyne cm–2
5. Work & Energy : 1L
=1 ...(i)
1J = 107 erg = 0.7373 ft lbf 1000 cm3
1 cal = 4.184 J = 4.184 × 107 erg 1m = 100 cm or 1m3 = (100 cm)3
1 eV = 1.602 × 10–12 erg = 1.602 × 10–19 J
1 eV/molecule or atom = 96.4853 kJmol–1 1 m3
= 23.06 kcal mol–1
=1 ...(ii)
106 cm3
1 J = 1 kg m2 sec–2 = 1 V × 1C = 1VA sec
Some Basic Concepts of Chemistry 3
From (i) and (ii) which are unit factors (i) Significance of zero : If zero is used to locate the decimal
point it is not considered as significant figure. Thus in 0.0072
1L 1 m3 there are only two significant figures whereas in 70.40, there
3 =
1000 cm 106 cm3 are four significant figures since zero is after 4. Again in
0.0070 there are two significant figures, since zero after 7 is
\ 1L = 10–3m3 and
significant for it has a meaning when written in exponentials.
1m3 = 103 L
If we compare 7.0 × 10–3 and 7 × 10–3 , the first term has
PRECISION AND ACCURACY uncertainty of one in seventy and second has uncertainty
Precision : It is the closeness of various measurements for the of one in seven. The exponential term does not add to
same quantity. number of significant figures.
Accuracy : It is the agreement of a particular value to the true (ii) Addition and substraction of quantities : In this case the
value.
uncertainty in the result is equal to the sum of the
Example : Let the true weight of a substance be 3.00g. The
uncertainties of the individual quantities.
measurement reported by three students are as follows
(iii) Multiplication and division : In this case the uncertainty in
Student Measurements/g Average/g the result is equal to the sum of the percentage of individual
1 2 uncertainties.
A 2.95 2.93 2.94
Examples :
B 3.01 2.99 3
C 2.94 3.05 2.99 (a) How many significant figures are there in following digits.
(i) 0.0426 (ii) 526
Case of student : It is precision but no accuracy since measurements (iii) 4.50 × 103 (iv) 0.00269 (v) 2.725 × 104
one close but not accurate.
Sol. (i) 0.0426 g has 3 significant figures.
Case of student : Measurements are close (precision) and accurate
(Accuracy) (ii) 526 ml has 3 significant figures.
(iii) 4.50 × 103 kg has 3 significant figures.
Case of C student : Measurement are not close (no precision) and
not accurate (no accuracy) (iv) 0.00269 has 3 significant figures.
Stoichiometry : It is calculation of masses or volumes of reactants (v) 2.725 × 104 has 4 significant figures.
and products involved in a chemically balanced reaction. Consider (b) Write the following in exponential notation.
the formation of ammonia. (i) 2365
N2 (g) + 3H2 (g) ƒ 2NH3(g) (ii) 60000 g (three significant figures)
All are gases indicated by letter (g) and coefficients 3 for H2 and (iii) 72400 g (four significant figures)
2 for NH3 are called stoichiometric coefficients. The formation of (iv) 0.0030 g
ammonia can be interpreted in many ways. Sol. (i) 2.365 × 103 or 23.65 × 102 or 0.2365 × 104
• One mole of N2(g) reacts with three moles of H2(g) to give
(ii) 6.00 × 104
two moles of NH3(g).
• 28g of N2(g) reacts with 6g at H2(g) to give 34g of NH3(g). (iii) 7.240 × 104
• 22.4L of N2(g) reacts with 67.2L of H2(g) to give 44.8L of (iv) 3.0 × 10–3
NH3(g) (iv) Rounding off : The following rules are observed.
Scientific Notation : The representation of large numbers in the (a) If the digit after the last digit to be retained is
form of N × 10n is known as scientific notation or exponential less than 5, the last digit is retained as such
notation. N can have value 1 to 10 and n is exponent having e.g. 1.752 = 1.75 (2 is less than 5).
positive or negative value.
(b) If the digit after the last digit to be retained is more than
Example (i) 0.000000028 can be written an 2.8 × 10–8
5, the digit to be retained is increased by 1
(ii) 4200000000 can be written as 4.2 × 109
e.g. 1.756 = 1.76 (6 is more than 5).
SIGNIFICANT FIGURES : (c) If the digit after the last digit to be retained is equal to
The weight 7.52 gm of a substance indicates that it is reliable to the 5, the last digit is retained as such if it is even and
nearest hundredth of a gram and may be expressed as 7.52 ± 0.01. It increased by 1 if odd.
means slightest variation may occur at the second place of decimal or * *
we can say that uncertainty is ± 0.01 g. e.g. 1.7 5 5 = 1.76 (* odd) & 1.7 6 5 = 1.76 (* even)
Now consider the weight 6.4234 g. It may correctly be expressed (v) Calculations involving addition and substraction : In case
as 6.4234 ± 0.001 g. of addition and substraction the final result should be reported
In the first case the weight contains three significant figures and to the same number of decimal places as the number with the
in the second case weight contains five significant figures. minimum number of decimal places e.g.
4 Chemistry
(a) 34.72 (has two decimal places) SEPARATION OF MIXTURES :
8.1 (has one decimal place) Mixtures can be separated into constituents by following methods:
------------ (i) Filtration can separate those mixtures whose one component
42.82 is soluble in a particular solvent and other is not.
------------ (ii) Distillation can be used to separate constituents of mixtures
Ans. = 42.82 but it should have only one decimal place so having different boiling points.
answer is 42.8. (iii) Extraction dissolves one out of several components of
(b) 59.273 mixture.
– 24.92 (iv) Crystallisation is a process of separating solids having
------------ different solubilities in a particular solvent.
34.353 (v) Sublimation separates volatile solids which sublime on
------------ heating from non-volatile solids.
Ans. = 34.35 (upto two decimal places) (vi) Chromatography is the technique of separating constituents
(vi) Calculations involving multiplication and division : In this of a mixture which utilises the property of difference of
case the final result should be reported having same number adsorption on a particular adsorbent.
of significant digits as that of the number having least (vii) Gravity separation separates constituents having different
significant digits. densities.
Example : 9.24 × 3.6 = 33.264 Rounded off to 33. (viii) Magnetic separation can separate magnetic components
3.6 is the number with least significant figures (two), hence from non magnetic ones.
final result is limited to two significant digits. Result is 33.
PHYSICAL AND CHEMICAL CHANGES :
5.235 A change which does not affect chemical composition and
In case of division = 0.3996 . Rounded off to 0.400.
13.1 molecular structure is a physical change and the one that involves
13.1 is the number with least (three) significant figures hence alteration of chemical composition and molecular structure is a
the result will be 0.400. chemical change.
MATTER : (i) Chemical Combination is reaction between two or more
Anything which occupies space, possesses mass and can be felt elements or compounds to form a single substance.
is called matter. H2 + I2 2HI
CLASSIFICATION OF MATTER :
(ii) Displacement means replacement of one element of compound
MATTER by another.
Physical Chemical
(iii) Decomposition involves splitting of a compound to form
Classification Classification
two or more substances.

SOLIDS LIQUIDS GASES CaCO 3 ¾


¾® CaO + CO 2
(iv) Combustion is a complete and fast oxidation of a substance.
PURE SUBSTANCES MIXTURES
C 2 H 4 + 3O 2 ¾
¾® 2CO 2 + 2H 2 O
ELEMENTS COMPOUNDS
(v) Neutralisation is the reaction between acid and base to form
HOMOGENEOUS HETEROGENEOUS a salt.
MIXTURES MIXTURES

ELEMENT : HCl + NaOH ¾


¾® NaCl + H 2 O
Pure substance consisting of one type of particles in the form of (vi) Polymerisation is the combination of molecules of same or
atoms eg. Cu, Na, Fe or molecules eg. H2, O2 etc. different substances to form a single molecule called polymer.
COMPOUND :
nCH 2 = CH 2 ¾
¾® -( CH 2 - CH 2 -) n
Pure substance consisting of molecules formed by the combination
of atoms of different elements eg. CO2, H2O etc. (vii) Photochemical changes occur in presence of visible or
MIXTURES : ultraviolet light.
Mixtures are substances made of two or more elements or (viii) Double decomposition or metathesis is the exchange of
compounds in any proportion. They may be homogeneous or oppositely charged ion on mixing two salt solutions.
heterogeneous. (ix) Hydrolysis involves reaction of salts with water to form acidic
or basic solutions.
Some Basic Concepts of Chemistry 5
LAWS OF CHEMICAL COMBINATIONS : (i) Deducing atomicity of elementary gases
(i) Law of conservation of mass : This law was given by French (ii) Deriving relationship between molecular mass and vapour
chemist A. Lavoisier (1774) which states that "during any density
physical or chemical change, the total mass of products is (iii) Deriving formula of substances
equal to the total mass of reactants". It is also called law of (iv) Determining molecular wt. of a gas
indistinctibility. It does not hold good for nuclear reaction. (v) Deducing the gram molecular volume.
(ii) Law of definite proportions : This law was given by Proust
(1799) and states that "a chemical compound always ATOM :
contains some elements combined together in same Atom is the smallest particle of element which might not be able to
proportion by mass". For example different samples of pure exist independently.
CO2 always have carbon and oxygen in 3 : 8 ratio by mass. MOLECULE :
(iii) Law of multiple proportions : This law was given by John Molecule is the smallest particle of the substance which can
Dalton (1803) and states that "when two elements combine exist independently. It can be subdivided as
to form two or more compounds, the different mass of one of (i) Homoatomic molecules are molecules of same element and
the elements and the fixed mass of the one with which it
can be further divided as monoatomic, diatomic and
combines always form a whole number ratio". This law
polyatomic molecules depending upon number of atoms. eg:
explains the concept of formation of more than one compound
He, O2, P4 etc.
by two elements.
(ii) Heteroatomic molecules are molecules of compound. They
(iv) Law of reciprocal proportions : This was given by Richter
can be diatomic and polyatomic. eg: H2O, PCl5, H2SO4, NO
(1792) and states that "when two elements combine
etc.
separately with a fixed mass of third, the ratio of masses in
which they do so is same or whole number multiple of the ATOMIC MASS UNIT (A.M.U.) :
ratio in which they combine with each other." This law is
1
also called law of equivalent proportions and is helpful in It is the unit of representing atomic masses. 1 a.m.u. = th the
12
determining equivalent weights.
(v) Gay Lussac's law of combining volumes : This law states mass of C-12.
that when gases react with each other, their volumes bear a MOLE :
simple whole no. ratio to one another and to volume of It is a unit which represents 6.023 × 1023 particles. The number
products (if gases) and similar conditions of pressure and
6.023 × 1023 is called Avogadro's number and is represented by N0
temperature.
or NA. Avogadro's number of gas molecules occupy a volume of
(vi) Dalton's atomic theory :
22400 cm3 at N.T.P. Number of molecules in 1 cm3 of gas at NTP is
Proposed by John Dalton in 1808. Main points are :
Loschmidt N0. With value 2.688 × 1019.
(a) Matter is made up, by indivisible particles called atoms
(b) Atoms of same elements are identical in physical and ATOMIC MASS :
chemical properties. "It is the number of times the atom of the element is heavier than
(c) Atoms of different substances are different in every H atom" was the first proposed definition. Later on oxygen was
respect preferred as standard. In 1961 C-12 was chosen as standard and
(d) Atoms always combine in whole numbers to form thus "the number of times the atom of an element is heavier than
compounds 12th part of C-12 is called atomic mass of the element.
(e) Atoms of resultant compounds possess similar
properties Mass of an atom of the element
Atomic mass =
DRAWBACKS OF DALTON'S THEORY : 1
´ mass of C - 12 atom
12
(i) Does not explain structure of atom.
(ii) Fails to explain binding forces between atoms in compounds. AVERAGE ATOMIC MASS :
(iii) Does not explain Gay Lussac's law. It is the mass of each isotope determined separately and then
(iv) Does not differentiate between atom and molecule. combined in ratio of their occurrence. Suppose a and b are two
AVOGADRO'S LAW : isotopes of an element with their occurence ratio p : q then
It states that "equal volumes of all gases, under similar conditions
p´ a + q ´b
of temperature and pressure contain equal number of molecules". Average atomic mass =
p+q
Applications are
6 Chemistry
DETERMINATION OF ATOMIC MASS : (ii) Victor meyer method : This method can determine the
(i) Dulong and petit's rule : It is based on experimental facts. molecular mass as
"At ordinary temperature, product of atomic mass and W
specific heat for solid elements is approximately 6.4 and this Molecular mass = × 22400
V
product is known as atomic heat of the element".
where W is the mass of liquid in gm. occupying a volume V
Atomic mass × specific heat = 6.4 ml at STP.
The law is valid for solid elements except Be, B, Si (iii) Vapour density method : Vapour density is the ratio of
and C. volume of a gas to the mass of same volume of hydrogen
Correct At. mass = Eq. mass × valency under identical conditions.
Cp Mass of V litre of gas
(ii) Specific heat method : This method is for gases. = , V.D. =
Cv Mass of V litre of hydrogen
where C p = specific heat at constant pressure and
Cv = specific heat at constant volume. the ratio g is a constant 1 Weight of volatile substance
or V.D. = ´ 22400
= 1.66 for monoatomic, 1.40 for diatomic, 1.33 for triatomic 2 Volume at STP
gas
Thus molecular mass = 2 × V.D.
and atomic mass of gaseous element
(iv) Colligative properties method : This method can be helpful
Mol. mass Mol. mass in determining molecular mass as
= = .
atomicity g
Kb ´ w ´1000
(iii) Chloride formation method : This method converts the elevation in boiling point ( DTb ) =
W ´ mol. mass
element (whose mass is to be determined) into volatile
chloride whose vapour density is found by Victor Mayer Where DTb is elevation in b.p., Kb is molal elevation constant
method. w is wt. of solute W is wt. of solvent
Molecular mass = 2 × V.D.
Kf ´ w ´1000
(iv) Vapour density method is suitable for elements having volatile Depression in freezing point ( DTf ) =
chlorides. W ´ mol. mass
Atomic mass = Eq. mass of metal × valency. GRAM MOLECULAR MASS OR MOLAR MASS :
(v) Mitscherlich's law of isomorphism : It states that That amount of substance whose mass in grams is equal to its
isomorphous substances have similar chemical constitution. molecular mass or the equivalently molecular mass of a substance
Isomorphous substances form crystals of same shape and expressed in grams is called gram molecular mass. Gram molecular
valencies of elements forming isomorphous salts are also
mass is also called one gram molecule. thus
same. eg: ZnSO4. 7H2O, MgSO4.7H2O and FeSO4.7H2O are
isomorphous. Wt. of substance (gms.)
No. of gm molecules =
GRAM ATOMIC MASS (GAM) GMM of substance
Is the mass of an atom expressed in gms. EQUIVALENT MASS :
Mass of element in grams It is the number of parts by weight of the substance that combines
No. of Gm-atoms of element =
GAM of element or displaces, directly or indirectly, 1.008 parts by mass of hydrogen
or 8 parts by mass of oxygen or 35.5 parts by mass of chlorine.
MOLECULAR MASS :
It can be calculated as
It is the average relative mass of the molecule as compared with
mass of C-12 atom. Atomic mass
(i) Equivalent mass for elements = Valency
Molecular mass = Average relative mass of one molecule
1
´ Mass of C - 12 atom Molecular mass
12 (ii) Equivalent mass for acids =
Basicity of acids
CALCULATION OF MOLECULAR MASS :
(i) Graham's law of diffusion : It states that rate of diffusion of Molecular mass
(iii) Equivalent mass for bases =
two gases is inversely proportional to the square root of Acidity of base
ratio of their molecular weights.
(iv) Equivalent mass for salts
r1 M2
= Formula mass
r2 M1 =
(Valency of cation ) ( No. of cations)
Some Basic Concepts of Chemistry 7
(v) Equivalent mass for oxidising agents (iv) Electrolytic method : It states that the quantity of substance
Formula mass that reacts at electrode when 1 Faraday of electricity is passed
= is equal to its GEM.
No. of electrons gained per molecule
GEM = Electrochemical equivalent × 96500
(vi) Equivalent mass for reducing agents
and ratio of weights deposited by equal amount of electricity
Formula mass is in ratio of their equivalent masses.
No. of electrons lost per molecule (v) Oxide method :
(vii) Equivalent weight of radicals
Weight of metal
Equivalent mass of metal = ´ 8.0
Formula mass of radical Weight of oxygen
= No. of units of charge
Weight of metal in gram
FORMULA MASS : = ´ 5.6 litre
It is obtained by adding atomic masses of various atoms present Vol. of O2 in litre
in the formula and this term replaces molecular mass in ionic (vi) Double decomposition :
compounds.
ACIDITY : AB + CD ¾
¾® AD ¯ + CB
It is the number of OH– ions that can be displaced from one
molecule of a substance. Weight of compound AB
BASICITY : Weight of AD
It is the number of H+ ions that can be displaced from one molecule
of a substance. Eqv. mass of A + Eqv. mass of B
=
GRAM EQUIVALENT MASS (GEM) : Eqv. mass of A + Eqv. mass of D
It is the mass of a substance expressed in grams or equivalently
the quantity of substance whose mass in grams is equal to its Wt. of salt Eqv. mass of salt
Þ =
equivalent mass is called one gram equivalent or gram equivalent Wt. of ppt. Eqv. mass of salt in ppt.
mass.
(vii) Neutralisation method for acids and bases :
Mass in gm
No. of gm equivalents = . Equivalent mass of acid (base)
GEM
METHODS OF DETERMINING EQUIVALENT Wt. of acid (base)
=
MASSES : Vol. of acid (base) in one litre required for neutralization
(i) Hydrogen displacement method : It is for metals which can ´ Normality of acid (base)
displace H2 from acids.
Equivalent mass of metal (viii) Silver salt is method commonly used for organic acids.
Weight of metal 108 ´ Mass of silver salt
= ´ 1.008 Eqv. mass of acid = - 107
Weight of displaced hydrogen Mass of Ag metal
Weight of metal in gram Mol. mass of acid = Eqv. mass of acid × Basicity
= ´11.2 litre
Vol. of H 2 in litre (ix) Platinichloride method for bases :
(ii) Metal displacement method : It utilises the fact that one Eqv. mass of base
GEM of a more electropositive metal displaces one GEM of a
195 ´ Mass of platinum salt
W1 E1 = - 205
less electropositive metal from its salt. = Mass of pt. metal ´ 2
W2 E 2
Mol. mass of base = Eqv. mass of base × Acidity
(iii) Conversion method : When one compound of a metal is
converted to another compound of similar metal then (x) Chloride method :

Weight of first compound Weight of metal


Eqv. mass of metal = ´ 35.5
Weight of second compound Weight of chlorine
E + Eqv. mass of first radical
= Weight of metal in gram
E + Eqv. mass of second radical = ´11.2 litre
Vol. of Cl 2 in litre
where E is the eqv. mass of the metal.
8 Chemistry
(xi) Volatile chloride method PARTS PER MILLION (ppm) :
Valency of metal It is the mass of solute present in one million parts by mass of
solution.
2 ´ VD of chloride 2 ´ VD
= = Mass of solute
Eq. mass of metal chloride E + 35.5 ppm = ´ 10 6
Mass of solution
2 ´ VD of chloride NORMALITY :
ÞE= - 35.5 It is the number of gram equivalents of a solute present in one litre
Valency
of solution.
CHEMICAL EQUATION : Gram equivalent s of solute
Normality =
It is the equation representing chemical change in terms of formula Volume of solution in litre
of reactants and products
(i) An equation which has not been equalised in terms of number Mass of solute in gms.
=
of atoms of reactants and products is called a skeleton GEM of solute ´ volume of solution in litre
equation. Normality depends on temperature. Also if strength is given in
(ii) An equation having equal number of atoms of various kinds normalities, N1 of A & N2 of B
on both sides is a balanced equation. Then N1V1 = N2V2.
MOLARITY :
EMPIRICAL FORMULA : It is the number of moles of solute present in one litre of solution.
It is the simplest formula of a compound giving simplest whole
number ratio of atoms present in one molecule. e.g. CH is empirical Moles of solute
M=
formula of benzene. Volume of solution (L)
MOLECULAR FORMULA : Weight of solute
=
It is the actual formula of a compound showing the total number GMM of solute ´ Volume of solution (L)
of atoms of constituent elements e.g. C6H6 is molecular formula of
benzene. Þ Moles = M ´ V(in litre) and millimoles = M × V(in ml).
Molecular formula = n × empirical formula, where n is simple whole Molarity and mass percentage have the relation M
number.
mass percentage ´10 ´ d
= , where d = density
SOLUTION : GMM of solute
It is a homogenous mixture of two or more substances. The If a solution of molarity M1 and volume V1 adds up with a solvent
component of solution having larger proportion is solvent and to a final volume V2, then molarity M2 is given by
others are solute.
M1V1
MOLE FRACTION : M2 =
V2
It is the ratio of moles of a constituent to the total number of moles
in a solution. If two different solutions (M1, V1) and (M2, V2) are mixed then
Let A be solute & B is solvent then mole fraction of solute (xA) molarity of resulting solution is
M1V1 + M 2 V2
nA M=
= , where n is the number of moles. V1 + V2
nA + nB
Also, Molarity × GMM of solute = Normality × GEM of solute
Mole fraction of solution x A + x B = 1 MOLALITY :
It is the number of moles of solute in 1 kg of solvent.
MASS PERCENTAGE :
It is the number of parts by mass of solute per hundred parts by Moles of solute
Molality (m) =
mass of solution. If WA is mass of solute and WB the mass of Weight of solvent (Kg)
solvent, then
Weight of solute ´ 1000
=
WA Mol. wt. of solute ´ Wt. of solvent
Mass percentage of A = ´100 .
WA + WB Molality is independent of temperature.
FORMALITY (F) :
VOLUME PERCENTAGE : It is the number of gram formula mass of ionic solute dissolved in
It is the number of parts by volume of solute per hundred parts by 1 litre of solution.
volume of solution. If VA is volume of solute and VB is the volume
of solvent then Mass of solute (gm)
Formality =
Volume of solution (litre)´ GFM of solute
VA
Volume percentage of A = ´100 LIMITING REAGENT :
VA + VB It is the reactant which is completely consumed during the reaction.
Some Basic Concepts of Chemistry 9

1. Calculate the weight of Iron which will be converted into its Sol. Since terpentine oil absorbs O3, the volume of O3 absorbed
oxide by the action of 18 g of steam. by terpentine oil is 10 ml.
Sol. The required equation is 3Fe + 4H2O ¾¾ ® Fe3O4 + 4H2 Volume of O2 = 100 – 10 = 90 ml
From gas equation (m)
3 × 56 g 4 × 18 g
3´ 56 WRT
g 18 g PV =
4 M
\ Weight of Fe converted into oxide = 42 g. Mol. wt of ozonised oxygen
2. The vapour density of a mixture containing NO2 and N2O4 is WRT 1.5 ´ 0.0821´ 273
38.3 at 27°C. Calculate the moles of NO2 in 100 g mixture. = = = 33.62
PV 1´1
Sol. Molecular weight of mixture = 2 × 38.3 = 76.6
Let weight of NO2 present in mixture be x g Mol. ratio of O2 and O3 90 : 10.
M. wt. of ozonised oxygen
x 100 - x 100
then + = \ x = 20.10 g
46 92 76.6 90 ´ 32 + 10 ´ a
= = 33.62 Þ a = 48.2 .
100
20.10
Hence moles of NO2 in mixture = = 0.43 Mol. wt. of ozone = 48.2.
46
5. How many millilitre of 0.5M H2SO4 are needed to dissolve
3. A mixture of 20 ml of CO, CH4 and N2 was burnt in excess of
0.5 g of copper (II) carbonate.
O2, resulting in reduction of 13 ml of volume. The residual
gas was then treated with KOH solution to show a Sol. Millimoles of H2SO4 = Millimoles of CuCO3
contraction of 14 ml in Volume. Calculate volume of CO, CH4 0.5 ´1000
and N2 in mixture. All measurements are made at constant T and P. 0.5 ´ V = or V = 8.097 ml.
123.5
Sol. Let a, b and c be the volume of CO, CH4 and N2 in mixture.

1 6. The formula weight of an acid is 82.0. 100 cm3 of a solution of


(I) CO+ O 2 ¾
¾® CO 2 this acid containing 39.0 g of the acid per litre were completely
2
a a a neutralised by 95.0 cm3 of aqueous NaOH containing 40.0 g
2 of NaOH per litre. What is the basicity of the acid?

(II) CH 4 + 2O 2 ¾
¾® CO 2 + 2 H 2 O 40 ´ 1000
Sol. Normality of NaOH = =1
b 2b b 40 ´ 1000
(III) N 2 + O 2 ¾
¾® No Reaction 39 ´ 1000 39n
Normality of acid = =
Since reduction in volume is 13. This is volume of O2 used 82
then we have ´ 1000 82
n
a n = Basicity of acid
+ 2b = 13 (I)
2 N1V1 = N2V2
Volume of CO2 formed is volume absorbed by KOH
a + b = 14 (II) 39n ´ 100
= 1 ´ 95 n = 1.99 = 2 Basicity is 2 .
\ c = 20 – 14 = 6 ml 82
Solving I & II we get a = 10 ml, b = 4 ml.
\ CO = 10 ml, CH4 = 4 ml and N2 = 6 ml 7. For the reaction N2O5 (g) 2NO2 (g) + 0.5 O2 (g).
Calculate the mole fraction of N2O5 (g) decomposed at a
4. The weight of one litre sample of ozonised oxygen at NTP
was found to be 1.5 g. When 100 ml of this mixture at NTP constant volume and temperature if the initial pressure is 600
were treated with terpentine oil the volume was reduced to mm Hg and the pressure at any time is 960 mm Hg. Assume
90 ml. Hence calculate the molecular weight of Ozone. ideal gas behaviour.
10 Chemistry

1 10. A mixture in which the mole ratio of H2 and O2 is 2 : 1 is used


Sol. N2O5 2NO2 + O2 to prepare water by the reaction ;
2
Initial pressure 600 0 0 2 H 2 ( g ) + O 2 (g ) ¾
¾® 2 H 2 O (g )
p The total pressure in the container is 0.8 atm at 20°C before
Final pressure 600 – p 2p
2 the reaction. Determine the final pressure at 120°C after
p µ n reaction assuming 80% yield of water.
At constant T and V. Sol. 2H 2 + O 2 ¾
¾® 2 H 2 O
p Initial mole 2a a 0 Total : = 3a
= Total moles = 600 – p + 2p + = 960 ; \ p = 240
2
2a ´ 80
Moles of water formed = = 1.6a
240 100
\ mole fraction of N2O5 decomposed = = 0.4 .
600 Final moles= 0.4 a 2a 1.6a Total = 2.2 a
8. 1.20 g sample of Na2CO3 and K2CO3 was dissolved in water Now PV = nRT
to form 100 ml of a solution. 20 ml of this solution required 40 0.8 ´ V 3a ´ 0.0821´ 293
ml of 0.1 N HCl for complete neutralisation. Calculate the = or P = 0.787 atm.
P´V 2.2a ´ 0.821 ´ 393
weight of Na2CO3 in mixture. If another 20 ml of this solution
is treated with excess of BaCl2. What will be the weight of 11. 1.00g of copper was dissolved in nitric acid and on ignition
precipitate? gave 1.25g of cupric oxide. 1.00g of cuprous oxide when
Sol. Let weight of Na2CO3 in the sample be a g then weight of
ignited in a current of hydrogen gave 0.888g of copper. Show
K2CO3 in the sample (1.20 – a)g.
that these results illustrate the law of multiple proportion.
gev of Na2CO3 + gev of K2CO3 = gev of HCl
Sol. Weight of cupric oxide = 1.25g
a (1.20 - a ) 40 ´ 0.1 ´ 5 Weight of copper = 1.00g
\ + =
53 69 1000 Weight of oxygen reacting with 1g of copper
On solving a = 0.59629 = (1.25 - 1.00) = 0.25g
\ weight of Na2CO3 = 0.59629 g Weight of Cuprous oxide = 1.00g
Addition of BaCl2 will form precipitate of BaCO3 Weight of Copper = 0.888g
Meq of BaCO3 = (Meq of Na2CO3 + Meq. of K2CO3) in 20 ml
Weight of oxygen reacting with 0.888g of Copper
= Meq of HCl in 20 ml = 40 × 0.1 = 4
= (1.00 – 0.888)
W Weight of oxygen reacting with 1.00g of Copper
\ ´ 1000 = 4 \ Wt . of BaCO 3 = 0.394 g
98.5 = 0.112g
9. A mixture of ethane (C2H6) and ethene C2H4 occupies 40 litre 1.112
= 0.127g
at 1.00 atm and at 400 K. The mixture reacts completely with 0.888
130 g of O2 to produce CO2 and H2O. Assuming ideal gas
Ratio of oxygen reacting with fixed amount of copper in
behaviour, calculate the mole fraction of C2H4 and C2H6 in
two oxides 0.25 : .0127 = 2:1
the mixture.
Sol. Moles of C2H6 and C2H4 in the mixture It shows the law of multiple proportion.
PV = nRT 12. Show that the following figures illustrate the law of
1 × 40 = n × 0.0821 × 400 Reciprocal proportion :
n = 1.2195 Methane Carbon dioxide Water
Let a be the moles of ethane, then moles of ethene
C = 75% C = 42.86% H = 11.11%
(1.2195 – a)
H = 25% O = 57.14% O = 88.89%
7
Again C 2 H 6 + O 2 ¾ ¾® 2CO 2 + 3H 2 O Sol. In methane 75g carbon combines with 25g of H2
2
57.14
C 2 H 4 + 3O 2 ¾¾® 2CO 2 + 2H 2 O In CO2 75g of carbon combines with ´ 75g = 100g
moles of O 2 required for complete combustion 42.86
of O2.
7 130
a + (1.2195 - a ) ´ 3 = Ratio of H2 and O2 reacting with fixed amount of carbon=
2 32
1:4
On solving a = 0.08.
In water the ratio of hydrogen and oxygen
0.808 = 11.11:88.89
= Mole fraction of ethane = = 0.66
1.2195 = 1:8 = 1:2×4
Mole fraction of C2H4 = 0.34. It show the law of Reciprocal Proportion.
Some Basic Concepts of Chemistry 11
13. Three oxides of a metal contain respectively 92.85%, 90.63% 16. 0.50g of Ag in nitric acid gave on addition of hydrochloric
and 86.51% of the metal. Examine if these figure are in acid, 0.665g of silver chloride. Find the equivalent weight
agreement with the law of multiple proportion. of silver.
Sol. Sol. Equivalent Wt. of Silver
I II III 0.50
wt of silver
Metal 92.85 90.63 86.51 = ´ 35.46 = ´ 35.46 = 107.99
wt of chlorine 0.165
Oxygen 7.15 9.37 13.49
(wt of chlorine = 0.665 – 0.50 = 0.165)
Ratio of oxygen reacting with 1 part of metal
7.15 9.37 13.49 17. 1g of KCl solution gives 1.925g of AgCl when heated
: :
92.85 90.63 86.51 with excess of AgNO3 solution. Taking equivalent wt of
0.077 : 0.103 : 0.156 Ag as 108 and that of Cl as 35.5 calculate the equivalent
2 :3 :4 wt of potassium
It shows the law of multiple proportion. wt of KCl eq. wt . of K + eq. wt . of Cl
Sol. =
wt of AgCl eq. wt. of Ag + eq. wt . of Cl
14. A chloride of phosphorous contains 22.57% of
phosphorous, Phosphine contains 8.82% hydrogen and 1 Eq. wt of K + 35.5
=
hydrogen chloride gas contains 97.23% chlorine. Prove 1.925 108 + 35.5
that these data illustrate the law of equivalent proportion \ Eq. wt. of K = 39.05
Sol. (i) In ph osphorous chloride, phosphorous is
= 22.57% 18. 0.622g of a nitrate of heavy metal on heating to constant
weight gave 0.466g of its oxide. Calculate the equivalent
So chlorine is (100 – 22.57) = 77.43%
weight of the metal from the above data
Ratio between masses of phosphrous and chlorine
22.57:77.43 = 1:3.43 wt of metal nitrate Eq. wt. of metal nitrate
Sol. =
(ii) In phosphine, hydrogen is = 8.82% wt of metal oxide Eq. wt. of metal oxide
So phosphorous is (100 – 8.82) = 91.18%
0.662 E + 62
Ratio between masses of phosphorous and hydrogen =
0.466 E+8
91.18 : 8.82 = 1 : 0.096 \ E = 120.4
In compounds (i) and (ii) the ratio between masses of
hydrogen and chlorine = 0.096:3.43=1.35.7 19. 0.4426g of a metallic chloride was dissolved in water and
solution made up to 100 ml., 50 ml of this solution required
(iii) In hydrogen chloride gas, chlorine is = 97.23%
1.02g of AgNO3 to precipitate the chloride completely. The
So hydrogen is (100 – 97.23) = 2.77%
specific heat of the metal was 0.057. Calculate the
therefore H : Cl = 2.77 : 97.23= 1 : 35.5
equivalent weight, atomic weight and valency of metal.
The ratio is the same, hence it illustrates the law of
Sol. Let the equivalent wt of metal be E, then
equivalent proportion.
weight of metal chloride Eq. wt. of metal chloride
15. Hydrogen sulphide contains 5.88% of hydrogen Water =
weight of silver nitrate Eq. wt. of silver nitrate
contains 11.11% of hydrogen and sulphur dioxide contains
50% of sulphur. Show that these figures illustrate the law
0.4426 E + 35.5
of reciprocal proportion =
1.02 170
Sol. Find the same amount of hydrogen which combines with
\ E = 38.26
sulphur in H2S and oxygen in H2O
According to Dulong Petit’s rule : Atomic wt × Sp. heat =
5.88g hydrogen combines with 94.12g sulphur in H2S
6.4
88.89
5.88g of hydrogen combines with ´ 5.88g oxygen 6. 4
11.11 \ Atomic wt = = 112.3
in H2O. 0.057
The ratio of S and O combining with fixed amount of H is Atomic wt. 112.3
valency = = =3
2:1. In SO2 the S:O is 1:1 Eq. wt. 38.26
Hence exact atomic wt. = 38.26×3 = 114.78
It illustrates the law of Reciprocal proportion.
12 Chemistry
20. One g of the chloride of an element was found to contain 22. 0.2988g of the silver salt of a dibasic acid on ignition gave
0.835 g of chlorine. It’s vapour density is 85. Find the 0.1944g of Silver. Find the equivalent weight of acid.
molecular formula of chloride. Sol. Equivalent weight of the acid
Sol. Weight of element = (1 – 0.835) = 0.165g æ wt. of silver salt ´ 108 ö
=ç - 107 ÷
è wt. of silver ø
Eq. wt. of element = 0.165 ´ 35.5 = 7.014
0.835
æ 0.2988 ´ 108 ö
=ç - 107 ÷ = 59
Let x be the valency of element è 0 .1944 ø
Atomic wt. of element = 7.014 x 23. 0.298g platinichloride salt of the mono acid base yielded
The Molecular formula of chloride MCl x 0.0975g of platinum. Find the equivalent weight of mono
\ Molecular weight of chloride acid base
= 7.014.x + x.35.3 = V.D ´ 2 Sol. Equivalent weight of the base
or, (42.541)x = 2 × 85 1 æ wt of Pt.salt´195 ö
= ç - 410÷
\ x= 4 2è wt of Pt. ø
Hence Molecular formula of chloride = MCl4
1 æ 0.298 ´ 195 ö
21. Copper sulphide and Metal sulphide are isomorphous. In = ç - 410 ÷ = 93
2 è 0.0975 ø
the first compound sulphur is 20.14% and in the second
compound 12.94%. If the atomic weight of Copper is 63.57. 24. 1.0g of metal on treatment with dilute mineral acid gave
Find the atomic weight of Metal 1242 ml of hydrogen at NTP. Calculate the equivalent
weight, atomic weight and valency of the metal, if its
Sol. Find the fixed (same) amount of sulphur reacting with
specific heat is 0.238
copper and Metal separately.
In the first-compound 1g sulphur combines with wt. of metal
Sol. Equivalent weight of metal = wt. of hydrogen ´ 1.008
(100 - 20.14)
= 3.96 g Copper 1 ml of hydrogen at NTP = 0.00009g
20.14
1
In the second compound 1g sulphur combines with \ Eq. wt of metal = ´ 1.008 = 9 g
0.00009 ´ 1242
By Dulong Petit’s law
(100 - 12.94)
= 6.72 g Metal Atomic mass×sp. heat = 6.4
12.94
6.4
Atomic wt. of Metal wt of Metal Atomic mass = = 26.86
= 0.238
Atomic wt. of Cu wt of Copper 26.86
valency = =3
6.72 9
Atomic wt. of Metal = ´ 63.57 = 107.87
3.96 \ Exact atomic mass = 9×3 = 27
Some Basic Concepts of Chemistry 13

Very Short/ Short Answer Questions Long Answer Questions


1. A colourless liquid used in rocket engines, whose empirical 12. (i) Concentrated aqueous sulphuric acid is
formula is NO2, has a molar mass of 92. What is its molecular 98% H 2 SO 4 by mass and has a density of
formula? 1.84 g mL–1. What volume of the concentrated acid is
2. Reaction, 2Br – (aq) + Cl2 (aq) 2Cl – (aq) + Br2(aq), required to make 5.0L of 0.50 M H2SO4 solution?
is used for commercial preparation of bromine from its salts. (Mol. weight of sulphuric acid = 98)
Suppose we have 50.0 ml of a 0.060M solution of NaBr. (ii) You are given a solution of 14.8M NH3. How many
What volume of a 0.050 M solution of Cl2 is needed to react milliliters of this solution do you require to give 100 ml
completely with Br – ? of 1MNH3? How much of water will you add?
3. 45.4L of dinitrogen reacted with 22.7 L of dioxygen and 45.4 13. (i) Copper oxide was prepared by the following methods:
L of nitrous oxide was formed. The reaction is given below: (a) In one case, 1.75 g of the metal were dissolved in
2N2(g) + O2(g) 2N2O(g) nitric acid and igniting the residual copper nitrate
Which law is being obeyed in this experiment? Write the yielded 2.19 g of copper oxide.
statement of the law. (b) In the second case, 1.14 g of metal dissolved in
4. Describe the difference between the mass of a mole of nitric acid were precipitated as copper hydroxide
oxygen atoms (O) and the mass of a mole of oxygen by adding caustic alkali solution. The precipitated
molecules (O2). copper hydroxide after washing, drying and
5. Alkaline solution of KMnO4 reacts as follows: heating yielded 1.43g of copper oxide.
2KMnO4 + 2KOH 2K2MnO4 + H2O + [O] (c) In the third case, 1.45 g of copper when strongly
Calculate the equivalent weight of KMnO4 in basic medium. heated in a current of air yielded 1.83 g of copper
6. Dinitrogen combines with dihydrogen to form ammonia oxide.
according to the following reaction. Show that the given data illustrate the law of
ˆˆ† 2NH 3 (g)
N 2 (g) + 3H 2 (g) ‡ˆˆ constant composition.
What is the ratio of their volumes under similar conditions (ii) Elements A and B form two different compounds. In
of temperature and pressure? first case 0.52 grams of A combines with 0.72 grams of
7. Zinc and hydrochloric acid react according to the equation B and in second case 0.15 grams of A combines with
Zn(s) + 2HCl (aq) ZnCl2(aq) + H2(g) 0.65 grams of B. Show that these data illustrate the Law
If 0.30 mole of Zn are added to hydrochloric acid containing of multiple proportion.
0.52 mole of HCl. Which of the two reactant is limiting reagent 14. (i) Calculate the volume at STP occupied by (a) 14 g of
and how many moles of H2 are produced? nitrogen, (b) 1.5 moles of carbon dioxide and (c) 1021
8. A glass of juice contains 9 gm of glucose (C6H12O6). How molecules of oxygen.
many atoms of each element (C, H and O) are there in the juice. (ii) Ammonia contains 82.35% of nitrogen and 17.65% of
9. Two bulbs B1 and B2 of equal capacity contain 10g oxygen hydrogen. Water contains 88.90% of oxygen and
(O2) and ozone (O3) respectively. Which bulb will have 11.10% of hydrogen. Nitrogen trioxide contains 63.15%
greater number of O-atoms and which will have greater of oxygen and 36.85% of nitrogen. Show that these
number of molecules? data illustrate the law of reciprocal proportions.
10. 4 g of copper chloride on analysis was founded to contain Multiple Choice Questions
1.890 g of copper (Cu) and 2.110 g of chlorine (Cl). What is
15. Two students performed the same experiment separately
the empirical formula of copper chloride? [Atomic mass of
and each one of them recorded two readings of mass which
Cu = 63.5 u, Cl = 35.5 u]
are given below. Correct reading of mass is 3.0 g. On the
11. Calculate the moles of NaOH required to neutralize the basis of given data, mark the correct option out of the
solution produced by dissolving 1.1 g P4O6 in water. Use following statements.
the following reactions: Students Readings
P4O6 + 6H2O 4H3PO3 (i) (ii)
2NaOH + H3PO3 Na2HPO3 + 2H2O A 3.01 2.99
(Atomic mass/g mol–1 ; P = 31, O = 16) B 3.05 2.95
14 Chemistry
(a) Results of both the students are neither accurate nor 20. One mole of a gas occupies a volume of 22.4 L. This is
precise. derived from
(b) Results of student A are both precise and accurate. (a) Berzelius’ hypothesis
(c) Results of student B are neither precise nor accurate. (b) Gay-Lussac’s law
(d) Results of student B are both precise and accurate. (c) Avogadro’s law
(d) Dalton’s law
16. A measured temperature on Fahrenheit scale
is 200 °F. What will this reading be on Celsius scale ? 21. Two containers P and Q of equal volume (1 litre each) contain
6 g of O2 and SO2 respectively at 300 K and 1 atmosphere,
(a) 40° C (b) 94° C
then
(c) 93.3 °C (d) 30° C (a) number of molecules in P is less than that in Q
17. Which of the following is an example of a heterogeneous (b) number of molecules in P and Q is same
substance? (c) number of molecules in Q is less than that in P
(a) Bottled water (b) Table salt (d) Either (a) or (b)
(c) Pieces of copper (d) Candle 22. In a chemical reaction
18. Which of the following statements about a compound is K 2 Cr 2 O 7 + xH 2 SO 4 + ySO 2
incorrect ?
¾ ¾® K 2SO 4 + zCr2 (SO 4 ) 3 + H 2 O
(a) A molecule of a compound has atoms of different elements.
(b) A compound cannot be separated into its constituent the values of x, y, z are
(a) 4, 1, 4 (b) 1, 3, 1
elements by physical methods of separation.
(c) 3, 2, 3 (d) 2, 1, 2
(c) A compound retains the physical properties of its
constituent elements. 23. 25.4 g of I 2 and 14.2 g of Cl 2 are made to react completely
(d) The ratio of atoms of different elements in a compound to yield a mixture of ICl and ICl3 . Calculate moles of ICl
is fixed.
and ICl3 formed
19. In compound A, 1.00g of nitrogen unites with 0.57g of
(a) 0.1, 0.1 (b) 0.2, 0.2
oxygen. In compound B, 2.00g of nitrogen combines with
(c) 0.1, 0.2 (d) 0.2, 0.1
2.24g of oxygen. In compound C, 3.00g of nitrogen combines
24. The Statue of Liberty is made of 2.0 × 105 lbs of copper
with 5.11g of oxygen. These results obey the following law sheets bolted to a framework. (1lb = 454 g). How many atoms
(a) law of constant proportion of copper are on the statue?
(b) law of multiple proportion (Atomic weight: Cu = 63.5).
(c) law of reciprocal proportion (a) 2.1 × 1027 (b) 8.6 × 1029
(c) 4.3 × 10 26 (d) 8.6 × 1026
(d) Dalton’s law of partial pressure

1. In the final answer of the expression How would the weight of the sample be reported?
(a) 3.93 g (b) 3g
(29.2 - 20.2)(1.79 ´ 10 )
(c) 3.9 g (d) 3.929 g
1.37 5. Dimension of pressure are same as that of
the number of significant figures is (a) Energy (b) Force
(a) 1 (b) 3 (c) 2 (d) 4 (c) Force per unit volume (d) Energy per unit volume
2. Given the numbers : 161 cm, 0.161 cm, 0.0161 cm. The number 6. The prefix 1018 is
of significant figures for the three numbers are (a) giga (b) kilo
(a) 3, 4 and 5 respectively (b) 3, 3 and 4 respectively (c) exa (d) nano
(c) 3, 3 and 3 respectively (d) 3, 4 and 4 respectively 7. Which of the following halogen can be purified by
3. One fermi is sublimation?
(a) 10–15 cm (b) 10–13 cm (a) I2 (b) Cl2
–10
(c) 10 cm (d) 10–12 cm (c) Br2 (d) F2
4. A sample was weighted using two different balances. The 8. A mixture of sand and iodine can be separated by
results were (a) crystallisation (b) distillation
(i) 3.929 g (ii) 4.0 g (c) sublimation (d) fractionation
Some Basic Concepts of Chemistry 15
9. Among the following pairs of compounds, the one that (b) 0.44 g of CO2
illustrates the law of multiple proportions is (c) 22.4 L of CO2 at STP
(a) NH3 and NCl3 (b) H2S and SO2 (d) None of these
(c) CS2 and FeSO4 (d) CuO and Cu2O 22. Number of g of oxygen in 32.2 g Na2SO4.10 H2O is
10. Irrespective of the source, pure sample, of water always yields (a) 20.8 (b) 2.24 (c) 22.4 (d) 2.08
88.89% mass of oxygen and 11.11% mass of hydrogen. This is 23. The specific heat of a metal is 0.16, its approximate atomic
explained by the law of weight would be
(a) conservation of mass (b) multiple proportions (a) 32 (b) 16 (c) 64 (d) 40
(c) constant composition (d) constant volume 24. The weight of a molecule of the compound C60H122 is
11. If NA is Avogadro's number then number of valence electrons (a) 1.09 × 10–21 g (b) 1.4 × 10–21 g
(c) 5.025 × 10 g 23 (d) 16.023 × 1023 g
in 4.2 g of nitride ions (N3–) is
(a) 4.2 NA (b) 2.4 NA 25. The number of water molecules present in a drop of water
(c) 1.6 NA (d) 3.2 NA (volume 0.0018 ml) density = 18 ml–1at room temperature is
12. Two containers P and Q of equal volume (1 litre each) contain (a) 1.084 × 1018 (b) 6.023 × 1019
(c) 4.84 × 10 17 (d) 6.023 × 1023
6 g of O2 and SO2 respectively at 300 K and 1 atmosphere.
then 26. The percentage of Se in peroxidase anhydrous enzyme is
(a) Number of molecules in P is less than that in Q 0.5% by weight (atomic weight = 78.4). Then minimum
(b) Number of molecules in P and Q is same molecular weight of peroxidase anhydrous enzyme is
(c) Number of molecules in Q is less than that in P (a) 1.568 × 103 (b) 1.568 × 104
(c) 15.68 (d) 3.136 × 104
(d) Either (a) or (b)
27. Equivalent weight of crystalline oxalic acid is
13. The number of moles of oxygen in one litre of air containing
21% oxygen by volume, under standard conditions are (a) 90 (b) 53 (c) 63 (d) 45
28. 3 g of an oxide of a metal is converted to chloride completely
(a) 0.0093 mole (b) 0.21 mole
and it yielded 5 g of chloride. The equivalent weight of the
(c) 2.10 mole (d) 0.186 mole metal is
14. The vapour density of a gas is 11.2. The volume occupied (a) 3.325 (b) 33.25 (c) 12 (d) 20
by 11.2 g of the gas at NTP will be
29. The simplest formula of a compound containing 50% of
(a) 22.4 L (b) 11.2 L element X (atomic mass 10) and 50% of element Y (atomic
(c) 1 L (d) 44.8 L mass 20) is
15. The amount of zinc required to produce 224 ml of H2 at STP on (a) XY (b) XY3 (c) X2Y (d) X2Y3
treatment with dil. H2SO4 will be 30. The hydrogen phosphate of certain metal has formula
(a) 6.5 g (b) 0.65 g MHPO4. The formula of metal chloride would be
(c) 65 g (d) 0.065 g (a) MCl (b) M2Cl2 (c) MCl2 (d) MCl3
16. The volume occupied by 4.4 g of CO2 at STP is 31. A compound contains atoms of three elements as A, B and
(a) 22.4 L (b) 0.224 L C. If the oxidation number of A is +2, B is +5 and that of C is
(c) 2.24 L (d) 0.1 L –2, the possible formula of the compound is
17. Assuming fully decomposed, the volume of CO2 released at (a) A3(B4C)2 (b) A3(BC4)2
STP on heating 9.85 g of BaCO3 (Atomic mass, Ba = 137) will (d) ABC2 (d) A2(BC3)2
be 32. If 0.44 g of a colourless oxide of nitrogen occupies 224 ml at
(a) 1.12 L (b) 2.24 L 1520 mm Hg and 273°C, then the compound is
(c) 4.06 L (d) 0.84 L (a) NO2 (b) N2O
18. 3 3
10 dm of N2 gas and 10 dm of gas X at the same temperature (c) NO4 (d) N2O2
contain the same number of molecules, the gas X is 33. The mass of a molecule of water is
(a) CO2 (b) CO (c) H2 (d) NO (a) 3 × 10–25 kg (b) 3 × 10–26 kg
19. 7.5 grams of a gas occupy 5.6 litres of volume at STP. The (c) 1.5 × 10–26 kg (d) 2.5 × 10–26 kg
gas is 34. Number of moles of KMnO4 required to oxidize one mole of
(a) N2O (b) NO (c) CO (d) CO2 Fe(C2O4) in acidic medium is
(a) 0.167 (b) 0.6 (c) 0.2 (d) 0.4
20. 1 amu is equal to
35. 100 cm3 of 0.1 N HCl is mixed with 100 cm3 of 0.2 N NaOH
1 1 solution. The resulting solution is
(a) of O-16 (b) of C-12
14 12 (a) 0.1 N and the solution is basic
(c) 1 g of H2 (d) 1.66 × 10–23 kg (b) 0.1 N and the solution is acidic
21. Which of the following contains maximum number of atoms? (c) 0.05 N and the solution is basic
(a) 6.023 × 1021 molecules of CO2 (d) 0.05 N and the solution is acidic
16 Chemistry
36. For preparing 0.1 N solution of a compound from its impure 46. The volume of water to be added to 100 cm3 of 0.5 N H2SO4 to
sample of which the percentage purity is known, the weight get deci normal concentration is
of the substance required will be (a) 400 cm3 (b) 450 cm3
(a) less than the theoretical weight (c) 500 cm 3 (d) 100 cm3
(b) more than the theoretical weight 47. 250 ml of a sodium carbonate solution contains 2.65 grams
(c) same as the theoretical weight of Na2CO3. If 10 ml of this solution is diluted to one litre,
what is the concentration of the resultant solution?
(d) none of these
(mol. wt. of Na2CO3 = 106)
37. 10 g CaCO3 gives on strong heating CO2. It gives quicklime
(a) 0.1 M (b) 0.01 M
(in grams)
(c) 0.001 M (d) 10–4 M
(a) 5g (b) 4.4 g (c) 5.6 g (d) 4 g
48. 3.92 g of ferrous ammonium sulphate crystals are dissolved in
38. Haemoglobin contains 0.33% of iron by weight. The 100 ml of water. 20 ml of this solution requires 18 ml of potassium
molecular weight of haemoglobin is approximately 67200. permaganate during titration for complete oxidation. The weight
The number of iron atoms (at. wt. of Fe = 56) present in one of KMnO4 present in one litre of the solution of
molecule of haemoglobin is (a) 3.476 g (b) 12.38 g
(a) 6 (b) 1 (c) 2 (d) 4 (c) 1.238 g (d) 34.76 g
39. The number of molecules in 16 g of methane is 49. 30 g of Magnesium and 30 g of oxygen are reacted, then the
residual mixture contains
16
(a) 3.0 × 1023 (b) ´ 10 23 (a) 50 g of Magnesium oxide and 10 g of oxygen
6.02
(b) 40 g of Magnesium oxide and 20 g of oxygen
16 (c) 45 g of Magnesium oxide and 15 g of oxygen
(c) 6.02 × 1023 (d) ´ 10 23 (d) 60 g of Magnesium oxide only
3.0
50. In order to prepare one litre normal solution of KMnO4, how
40. 50 ml 10 N H2SO4, 25 ml 12 N HCl and 40 ml 5 N HNO3 were
many grams of KMnO4 are required if the solution is to be
mixed together and the volume of the mixture was made 1000 ml
used in acid medium for oxidation?
by adding water. The normality of the resultant solution will be
(a) 158 g (b) 62.0 g
(a) 2 N (b) 1 N (c) 3 N (d) 4 N
(c) 31.6 g (d) 790 g
41. A molal solution is one that contains 1 mole of a solute in 51. In a chemical reaction
(a) one litre of the solvent K 2 Cr 2 O 7 + xH 2 SO 4 + ySO 2
(b) 1000 g of the solvent
(c) one litre of the solution ¾
¾® K 2SO 4 + zCr2 (SO 4 ) 3 + H 2 O
(d) 22.4 litres of the solution the values of x, y, z are
42. A 100 ml solution of 0.1 N HCl was titrated with 0.2 N NaOH (a) 4, 1, 4 (b) 1, 3, 1
solution. The titration was discontinued after adding 30 ml (c) 3, 2, 3 (d) 2, 1, 2
of NaOH solution. The remaining titration was completed 52. The maximum amount of BaSO4 precipitated on mixing equal
by adding 0.25 N KOH solution. The volume of KOH required volumes of BaCl2 (0.5 M) with H2SO4 (1 M) will correspond to
for completing the titration is (a) 1.0 M (b) 0.5 M
(a) 16 ml (b) 32 ml (c) 1.5 M (d) 2.0 M
(c) 35 ml (d) 70 ml 53. In the reaction
43. An aqueous solution of 6.3 g of oxalic acid dihydrate is ® 4NO(g) + 6H 2 O(l ) ,when 1 mole
4NH 3 (g) + 5O 2 (g) ¾¾
made up to 250 ml. The volume of 0.1 N NaOH required to of ammonia and 1 mole of O2 are made to react to completion
completely neutralise 10 ml of this solution is (a) 1.0 mole of H2O is produced
(a) 20 ml (b) 40 ml (b) 1.0 mole of NO will be produced
(c) 10 ml (d) 4 ml (c) all the ammonia will be consumed
44. The percentage of nitrogen in urea is about (d) all the oxygen will be consumed
(a) 85 (b) 46 (c) 18 (d) 28 54. The set of numerical coefficients that balances the equation
45. How much of NaOH is required to neutralise 1500 cm3 of 0.1 ¾® K 2 Cr2 O 7 + KCl + H 2 O is
K 2 CrO 4 + HCl ¾
N HCl? (Na = 23) (a) 2, 2, 1, 2, 1 (b) 2, 2, 1, 1, 1
(a) 60 g (b) 4 g (c) 6 g (d) 40 g (c) 2, 1, 1, 2, 1 (d) 1, 1, 2, 2, 1
Some Basic Concepts of Chemistry 17

1. The mass of carbon anode consumed (giving only 12. With increase of temperature, which of these changes?
carbondioxide) in the production of 270 kg of aluminium (a) molality [AIEEE 2002]
metal from bauxite by the Hall process is (Atomic mass: (b) weight fraction of solute
Al = 27) [CBSE-PMT 2005] (c) fraction of solute present in water
(a) 270 kg (b) 540 kg (c) 90 kg (d) 180 kg (d) mole fraction.
2. The number of moles of KMnO4 reduced by one mole of 13. Number of atoms in 558.5 gram Fe (at. wt. of Fe = 55.85
KI in alkaline medium is: [CBSE-PMT 2005] g mol–1) is [AIEEE 2002]
(a) one (b) two (c) five (d) one fifth (a) twice that in 60 g carbon (b) 6.023 ´ 1022
3. Volume occupied by one molecule of water (c) half that in 8 g He (d) 558.5 ´ 6.023 ´ 1023
(density = 1 g cm–3) is : [CBSE-PMT 2008] 14. One mole of magnesium nitride on the reaction with an
(a) 9.0 × 10–23 cm3 (b) 6.023 × 10– 23 cm3 excess of water gives : [AIEEE 2004]
(a) two moles of ammonia (b) one mole of nitric acid
(c) 3.0 × 10–23 cm3 (d) 5.5 × 10– 23 cm3
- (c) one mole of ammonia (d) two moles of nitric acid
4. Number of moles of MnO4 required to oxidize one mole of
15. If we consider that 1/6, in place of 1/12, mass of carbon atom
ferrous oxalate completely in acidic medium will be : is taken to be the relative atomic mass unit, the mass of one
[CBSE-PMT 2008] mole of the substance will [AIEEE 2005]
(a) 0.6 moles (b) 0.4 moles (a) be a function of the molecular mass of the substance
(c) 7.5 moles (d) 0.2 moles (b) remain unchanged
5. An organic compound contains carbon, hydrogen and (c) increase two fold
oxygen. Its elemental analysis gave C, 38.71% and H, 9.67%. (d) decrease twice
The empirical formula of the compound would be : 16. How many moles of magnesium phosphate, Mg3(PO4)2 will
[CBSE-PMT 2008] contain 0.25 mole of oxygen atoms? [AIEEE 2006]
(a) 1.25 × 10 –2 (b) 2.5 × 10 –2
(a) CH3O (b) CH2O (c) CHO (d) CH4O
6. How many moles of lead (II) chloride will be formed from a (c) 0.02 (d) 3.125 × 10–2
reaction between 6.5 g of PbO and 3.2 g of HCl ? –1
17. The density (in g mL ) of a 3.60 M sulphuric acid solution
[CBSE-PMT 2008] that is 29% H2SO4 (molar mass = 98 g mol–1) by mass will
(a) 0.044 (b) 0.333 (c) 0.011 (d) 0.029 be [AIEEE 2007]
7. 10 g of hydrogen and 64 g of oxygen were filled in a steel (a) 1.45 (b) 1.64
vessel and exploded. Amount of water produced in this (c) 1.88 (d) 1.22
reaction will be: [CBSE-PMT 2009] 18. In the reaction, [AIEEE 2007]
(a) 3 mol (b) 4 mol (c) 1 mol (d) 2 mol 2Al( s ) 6HCl( aq ) 2Al3 (aq) + 6Cl (aq) + 3H2 ( g)
8. The number of atoms in 0.1 mol of a triatomic gas is :(NA = (a) 11.2 L H2(g) at STP is produced for every mole HCl(aq)
6.02 ×1023 mol–1) [CBSE-PMT 2010] consumed
(a) 6.026 × 1022 (b) 1.806 × 1023 (b) 6 L HCl(aq) is consumed for every 3 L H2(g) produced
(c) 3.600 × 1023 (d) 1.800 × 1022 (c) 33.6 L H2(g) is produced regardless of temperature and
9. Which has the maximum number of molecules among the pressure for every mole Al that reacts
following ? [CBSE-PMT 2011 M] (d) 67.2 H2(g) at STP is produced for every mole Al that
reacts.
(a) 44 g CO2 (b) 48 g O3
19. The molality of a urea solution in which 0.0100 g of urea,
(c) 8 g H2 (d) 64 g SO2
[(NH2)2CO] is added to 0.3000 dm3 of water at STP is :
10. How many grams of concentrated nitric acid solution should
[AIEEE 2011 RS]
be used to prepare 250 mL of 2.0M HNO3 ? The concentrated
acid is 70% HNO3 [NEET 2013] (a) 5.55 10 4 m (b) 33.3 m
(a) 90.0 g conc. HNO3 (b) 70.0 g conc. HNO3 (c) 3.33 × 10–2 m (d) 0.555 m
(c) 54.0 g conc. HNO3 (d) 45.0 g conc. HNO3 20. A gaseous hydrocarbon gives upon combustion 0.72 g of
20
11. 6.02 × 10 molecules of urea are present in 100 mL of its water and 3.08 g. of CO2. The empirical formula of the
solution. The concentration of solution is : [NEET 2013] hydrocarbon is : [JEE Main 2013]
(a) C2H4 (b) C3H4 (c) C6H5 (d) C7H8
(a) 0.01 M (b) 0.001 M (c) 0.1 M (d) 0.02 M
18 Chemistry
21. Experimentally it was found that a metal oxide has formula 22. Consider a titration of potassium dichromate solution with
M0.98O. Metal M, present as M2+ and M3+ in its oxide. acidified Mohr's salt solution using diphenylamine as
Fraction of the metal which exists as M3+ would be : indicator. The number of moles of Mohr's salt required per
[JEE Main 2013] mole of dichromate is [IIT-JEE 2007]
(a) 7.01% (b) 4.08% (c) 6.05% (d) 5.08% (a) 3 (b) 4 (c) 5 (d) 6

1. The correctly reported answer of addition of 29.4406, 3.2 12. A compound made up of two elements A and B is found to
and 2.25 will have significant figures contain 25% A (atomic mass = 12.5) and 75% B (atomic mass
(a) 3 (b) 4 (c) 2 (d) 5 = 37.5). The simplest formula of the compound is
2. On dividing 0.25 by 22.1176 the actual answer is 0.011303. (a) AB (b) AB2 (c) AB3 (d) A3B
The correctly reported answer will be 13. On analysis a certain compound was found to contain iodine
and oxygen in the ratio of 254 gm of iodine (atomic mass
(a) 0.011 (b) 0.01 (c) 0.0113 (d) 0.013
127) and 80 gm oxygen (at mass = 16). What is the formula of
3. In which of the following number all zeros are significant? the compound.
(a) 0.0005 (b) 0.0500 (c) 50.000 (d) 0.0050 (a) IO (b) I2O (c) I5O3 (d) I2O5
4. If law of conservation of mass was to hold true, then 20.8 14. Two oxides of a metal contain 50% and 40% metal (M)
gm of BaCl2 on reaction with 9.8 gm of H2SO4 will produce respectively. If the formula of first oxide is MO2 the formula
7.3 gm of HCl and BaSO4 equal to : of second oxide will be
(a) 11.65 gm (b) 23.3 gm (a) MO2 (b) MO3 (c) M2O (d) M2O5
(c) 25.5 gm (d) 30.6 gm 15. The ratio of the molar amounts of H2S needed to precipitate
5. One of the following combination which illustrates the law the metal ions form 20 ml each of 1 M Ca(NO3)2 and 0.5M
of reciprocal proportions ? CuSO4 is
(a) N2O3, N2O4, N2O5 (b) NaCl, NaBr, NaI (a) 1 : 1 (b) 2 : 1 (c) 1 : 2 (d) indefinite
(c) CS2, CO2, SO2 (d) PH3, P2O3, P2O5 16. 12 gm of Mg (atomic mass 24) will react completely with
hydrochloric acid to give
6. If isotopic distribution of C-12 and C-14 is 98% and 2%
(a) One mol of H2
respectively then the no. of C-14 atoms in 12gm of carbon is
(b) 1/2 mol of H2
(a) 1.032 × 1022 (b) 3.0 × 1022
(c) 2/3 mol of O2
(c) 5.88 ×1023 (d) 6.02 × 1023
(d) both 1/2 mol of H2 and 1/2 mol of O2
7. Which of the following contains maximum number of atom
17. The total number of protons in 10 gm of calcium carbonate
(a) 2.0 mole of S8 (b) 6.0 mole of S is (No = 6.023 × 1023)
(c) 5.5 mole of SO2 (d) 44.8 litre of CO2 of S.T.P. (a) 1.5057 × 1024 (b) 2.0478 × 1024
8. A sample of AlF3 contains 3.0 × 1024 F– ions. The number of (c) 3.0115 × 1024 (d) 14.0956 × 1024
formula unit of this sample are 18. 2.76 gm of silver carbonate (at mass of Ag 108) on being
(a) 9 × 1024 (b) 3 × 1024 heated strongly yield a residue weighing
(c) 0.75 × 10 24 (d) 1.0 × 1024 (a) 2.16 gm (b) 2.48 gm
9. What mass of calcium chloride in grams would be enough (c) 2.32 gm (d) 2.64 gm
to produce 14.35 gm of AgCl ? 19. If 0.5 mol of BaCl2 is mixed with 0.2 mol of Na3PO4 then maximum
(a) 5.55 gm (b) 8.295 gm (c) 16.5 gm (d) 11.19 gm number of moles of Ba 3 (PO 4 ) 2 that can be formed is
10. If potassium chlorate is 80% pure, then 48 gm of oxygen (a) 0.7 (b) 0.5 (c) 0.3 (d) 0.1
would be produced from (atomic mass of K =39) 20. 1.12 ml of a gas is produced at S.T.P. by the action of 4.12 mg
(a) 153.12 gm of KClO3 (b) 122.5 gm of KClO3 of alcohol ROH with methyl magnesium Iodide. The
(c) 245 gm of KClO3 (d) 98 gm of KClO3 molecular mass of alcohol is
11. If 224 ml of a triatomic gas has a mass of 1 gm at 273K and 1 (a) 16.0 (b) 41.2 (c) 82.4 (d) 156.0
atmospheric pressure then the mass of one atom is 21. An unsaturated hydrocarbon weighing 1.68 gm has volume
(a) 8.30 × 10–23 gm (b) 2.08 × 10–23 gm of 488 ml at S.T.P. If it contains 14% of hydrogen, then the
(c) 5.53 × 10–23 gm (d) 6.24 × 10–23 gm family to which the hydrocarbon belongs is
(a) alkane (b) alkene (c) alkyl (d) benzene
Some Basic Concepts of Chemistry 19
22. The number of molecules in 8.96 litre of a gas at 0ºC and 1 32. A gas mixture of 3 litres of propane (C3H 8 ) and butane
atm. pressure is approximately
(C 4 H10 ) on complete combustion at 25° C produced 10
(a) 6.023 × 1023 (b) 12.04 × 1023
(c) 18.06 × 1023 (d) 24.08 × 1022 litre CO 2 . Find out the composition of gas mixture (Propane
20
23. If 3.01 × 10 molecules are removed from 98 mg of H2SO4, : Butane)
then the number of moles of H2SO4 left are (a) 2 : 1 (b) 1 : 2 (c) 1.5 : 1.5 (d) 0.5 : 2.5
(a) 0.1 × 10–3 (b) 0.5 × 10–3 33. On subjecting 10 ml mixture of N 2 and CO to repeated electro
(c) 1.66 × 10–3 (d) 9.95 × 10–2 spark, 7 ml of O 2 was required for combustion. What was
24. 25.4 g of I 2 and 14.2 g of Cl 2 are made to react completely the mole percent of CO in the mixture? (All volumes were
measured under identical conditions)
to yield a mixture of ICl and ICl3 . Calculate moles of ICl (a) 60 (b) 40 (c) 6 (d) 4
and ICl3 formed 34. How many of 0.1N HCl are required to react completely with
1 g mixture of Na 2CO 3 and NaHCO 3 containing equimolar
(a) 0.1, 0.1 (b) 0.2, 0.2 (c) 0.1, 0.2 (d)0.2, 0.1
amounts of two ?
25. 2 g of a mixture of CO and CO 2 on reaction with excess (a) 157.7 ml (b) 15.77 ml
I 2 O 5 produced 2.54 g of I 2 . What be the mass % of CO 2 (c) 147.7 ml (d) 14.77 ml
in the original mixture ? 35. 1 mole of mixture of CO and CO 2 requires exactly 28 g KOH
(a) 35 (b) 70 (c) 30 (d) 60 in solution for complete conversion of all the CO 2 into
26. The hydrated salt Na 2 CO 3. xH 2 O undergoes 63% loss in K 2 CO 3 . How much amount more of KOH will be required
mass on heating and becomes anhydrous. The value of x is for conversion into K 2 CO 3 . If one mole of mixture is
(a) 10 (b) 7 (c) 5 (d) 3
completely oxidized to CO 2
27. Gastric juice contains 3.0 g of HCl per litre. If a person
(a) 112 g (b) 84 g (c) 56 g (d) 28 g
produces 2.5 litre of gastric juice per day. How many antacid
36. 10 g CaCO3 were dissolved in 250 ml of M HCl or the
tablets each containing 400 mg of Al(OH )3 are needed to
solution was boiled. What volume of 2M KOH would be
neutralize all the HCl produced in one day ? required to equivalence point after boiling ? Assume no
(a) 18 (b) 14 (c) 20 (d) 17 change in volume during boiling.
28. Sulfuryl chloride (SO 2Cl 2 ) reacts with water to give a (a) 50 ml (b) 25 ml (c) 75 ml (d) 60 ml
37. 2.24 ml of a gas ‘X’is produced at STP by the action of 4.6
mixture of H 2SO 4 and HCl. How many moles of baryta mg of alcohol (ROH) with methyl magnesium iodide the
would be required to neutralize the solution formed by adding molecular mass of alcohol and the gas ‘X’ are respectively.

4 mol of SO 2Cl 2 to excess of water ? (a) 0.46,CH 4 (b) 4.6,C2 H6

(a) 1 (b) 2 (c) 3 (d) 4 (c) 46, CH 4 (d) 46, C 2 H 4


29. A chloride of a metal (M) contain 65.5% of chlorine. 100 ml 38. 1.575 g of oxalic acid (COOH) 2 . xH 2O are dissolved in
of vapour of the chloride of metal at STP weights 0.72 g. The water and the volume made upto 250 ml. On titration 16.68
molecular formula of the metal chloride is ml of this solution requires 25 ml of N/15 NaOH solution for
(a) MCl 4 (b) MCl3 (c) MCl 2 (d) MCl complete neutralization calculate x.
(a) 3 (b) 2 (c) 4 (d) 5
30. 7.36 g of a mixture of KCl and KI was dissolved in H 2 O to 39. Suppose two elements X and Y combine to form two
prepare 1 litre solution. 25 ml of this required 8.45 ml of 0.2 N compounds XY2 and X 3Y2 when 0.1 mol of former weighs
AgNO 3 , what are % of KI in mixture ? 10 g while 0.05 mol of the latter weights 9 g. The atomic
masses of X and Y are respectively
(a) 57.28 (b) 47.28 (c) 5.72 (d) 49.12
(a) 60 & 40 (b) 30 & 40
31. When burnt in air, 14.0 g mixture of carbon and sulphur
(c) 40 & 30 (d) 40 & 60
gives a mixture of CO 2 and SO 2 in the volume ratio of 2 :
40. The vapour density of a mixture containing NO 2 and
1, volume being measured at the same conditions of
N 2 O 4 is 27.6. Mole fraction of NO 2 in the mixture is
temperature and pressure moles of carbon in the mixture is
(a) 0.8 (b) 0.6
(a) 0.75 (b) 0.5 (c) 0.40 (d) 0.25
(c) 0.4 (d) 0.2
20 Chemistry

EXERCISE 1 7. (a) Iodine undergoes sublimation (volatile solid). Others do


1. The molecular formula is N2O4 not sublime.
8. (c) By sublimation since I2 sublimes.
2. V1 (Volume of chlorine solution)
9. (d) In CuO and Cu2O the O : Cu is 1 : 1 and 1 : 2. This is law
= 3.00 × 10 –2L
of multiple proportion.
5. Equivalent weight = 158 g 10. (c) The H : O ratio in water is fixed, irrespective of its source.
6. The ratio of their volumes is 1 : 3 : 2. Hence it is law of constant composition.
7. HCl is the limiting reagent. Moles of H2 produced = 0.26 11. (b) Moles of nitride ion
mol. 4.2
= = 0.3 mol = 0.3 N A nitride ions.
8. 36.12×1022 ‘H’ atoms and number of 'O' atoms will be similar 14
to 'C' atoms as per the molecular formula of glucose. Valence electrons = 8 × 0.3 N A = 2.4 N A
9. Both contain the same number of atoms but bulb B1 contains (5 + 3 due to charge). One N3– ion contains 8 valence
more number of molecules. electrons.
10. Empirical Formula = CuCl2.
6
11. 0.04 moles of NaOH. 12. (c) Mol. of O2 = & mol. of SO2 = 6 . The number of
32 64
0.50 5 molecules of SO2 will be less than that of O2.
12. (i) V1 = = 0.136L 13. (a) 21% of 1 litre is 0.21 litre.
18.4
22.4 litres = 1 mole at STP
(ii) V1 = 6.76 ml,
0.21
Volume of water added = 93.24ml 0.21 litre = = 0.0093 mol
22.4
14. (i) (a) 14 g of N2 occupy = 11.2 litres at STP
14. (b) V.D. = 11.2 M. Wt = 22.4 g
(b) 1.5 moles of CO2 occupy = 33.6 litres at STP It corresponds to 22.4 litres at STP
(c) 1021 molecules of O2 occupy 11.2 g = 11.2 L
= 37.2 cm3 at STP 15. (b) Zn + H 2SO 4 ZnSO 4 + H 2
15. (b) 16. (c) 17. (d) 18. (c) 19. (b)
65 g Zn gives 1 mole of H2 = 22400 ml of H2
20. (c) 21. (c) 22. (b) 23. (a) 24. (b) 224 ml of H2 will be obtained from 0.65 g Zn.
EXERCISE 2 16. (c) 1 Mol CO2 = 44 g = 22.4 litre at N.T.P.
1. (c) Solution gives 1.1 × 106, hence significant figures are 2. 4.4 g CO2 = 2.24 L at NTP.
The exponential term does not add to significant figures. 17. (a) BaCO 3 BaO + CO 2
2. (c) Each has three significant figures. When zero is used to
locate the decimal point it is not considered as singnificant 192 g of BaCO3 1 gives mol of CO2 = 22.4 L
figure. 9.85 g of BaCO3 will give 0.05 mol of CO2 which is equal
3. (b) One fermi is 10–13 cm. to 1.12 litre.
4. (a) Out of two 3.929 g is more accurate and will be reported 18. (b) The number of molecules of N2 and X are same. Hence
as 3.93 after rounding off. they must have the same molecular weights.
Energy X is CO.
5. (d) which can be shown
volume 7.5
Pressure 19. (b) PV = nRT 5.6 × 1 = 0.0821 273
M. Wt.

= Force =
Work (energy/di stance)
=
Energy M. Wt = 30.12 Hence gas is NO.
area Area Volume
6. (c) Exa = 1018 (see text.) 1
20. (b) 1 amu = of the mass of C-12.
12
Some Basic Concepts of Chemistry 21
21. (c) 22.4 L of CO2 at STP = 1 mole = 6.023 × 1023 molecules. 34. (b) The required equation is
Hence number of atoms 3 × 6.023 × 1023.
22. (c) M. Wt of Na2SO4.10 H2O is 322 g which contains 224 g 2KMnO 4 + 3H 2SO 4 ¾
¾®
oxygen. \ 32.2 g will contain 22.4 g oxygen. K 2SO 4 + 2MnSO 4 + 3H 2O + 5 [O]
6.4 nascent oxygen
23. (d) = Apx. Atomic mass
Specific heat
2Fe(C2 O 4 ) + 3H 2SO 4 + 3 [ O ] ¾¾
®
6.4 Fe 2 (SO 4 )3 + 2CO 2 + 3H 2O
\ Apx. At.wt = = 40
0.16 O required for 1 mol. of Fe(C2O4) is 1.5, 5O are obtained
24. (b) M. Wt of C60H122 = 842g from 2 moles of KMnO4
842 2
\ Wt of one molecule = 23
= 1.4 ´10 - 21 g \ 1.5 [O] will be obtained from = ´ 1.5 = 0.6 moles of
6.023 ´10 5
KMnO4.
25. (b) 0.0018 ml = 0.0018 g = 0.0001 mole of water = 10-4 mole
35. (b) Normality = N1V1 - N 2 V2 = 0.2 ´ 100 - 0.1´100
\ Number of water molecules = 6.023 ´ 10 23 ´ 10 -4 V1 + V2 100 + 100
= 6.023 × 1019
26. (b) 0.5% by weight means. If Mol. wt. is 100 then mass of Si 10
= = 0.05 N NaOH
is 0.5. If at least one atom of Se is present in the molecule 200
then 36. (b) More than theoretical weight since impurity will not
100 contribute.
M. Wt = ´ 78.4 = 1.568 ´ 10 4
0.5 37. (c) CaCO3 CaO + CO2
27. (c) Eq. Wt of crystalline oxalic acid (COOH)2. 2H2O 100 g 56 g
10 g CaCo3 will give 5.6 gCaO
M.Wt . 126
= = = 63
2 2 0.33
38. (d) Weight of Iron in 67200 = ´ 67200 = 221.76
100

Wt. of metal oxide 221.76


28. (b) Number of atoms of Iron = = 3.96 º 4
Wt. of metal chloride 56
39. (c) 16 g CH4 is 1 mol. Hence number of molecules
Eq. wt of metal + Eq. wt of oxygen
= = Avogadro number = 6.023 × 1023.
Eq. wt of metal + Eq. wt of chlorine
N1V1 + N 2 V2 + N 3 V3
40. (b) N =
3 E+8 Total volume
= \ E = 33.25
5 E + 35.5
50 ´10 + 25 ´ 12 + 40 ´ 5
29. (c) 50% x atomic mass 10, 50% y atomic mass 20. Relative = = 1N
1000
50 41. (b) Molal solution contains 1 mole of solute in 1000 g solvent.
number of atoms of X = 50 = 5 and that Y = = 2. 5
10 20 42. (a) N1V1 = N2V2 + N3V3
Simple Ratio 2 : 1. Formula X2Y 0.1 × 100 = 0.20 × 30 + 0.25 V3 \ V3 = 16 ml.
30. (c) Formula of metal phosphate is M++H+PO4– – – . 6.3 ´ 1000
Valency of metal + 2. Hence metal chloride is MCl2. 43. (b) Normality of oxalic acid = = 0. 4 N
63 ´ 250
31. (b) Sum of oxidation states must be equal to zero which is
N1V1 = N2V2 10 × 0.4 = V × 0.1 = 40 ml.
given by A3(BC4)2 = A3B2C8 (+ 6 + 10 – 16 = 0)
44. (b) NH2CONH2 M. Wt of urea=60.
32. (b) PV = nRT
224 ´ 1520 0.44 % N in urea = 28 ´100 = 46
= ´ 0.0821´ 546 60
1000 ´ 760 M
45. (c) For neutralisation gev of acid = gev of base
\ M = 44 approx. Hence gas = N2O
0.1´ 1500
33. (b) Mass of one molecule of Water gev. of HCl = = 0.15 º gev. of NaOH
1000
18
= 23
= 3 ´10 - 23 g = 3 ´10 - 26 Kg 1 gev. of NaOH = 40 g
6.023 ´10 \ 0.15 gev. of NaOH = 40 × 0.15 = 6 g.
22 Chemistry
46. (a) N1V1 = N2V2
270 ´ 10 3
\ 100 × 0.5 = 0.1 × V2 No. of gram equivalent of Al = = 30 × 103
\ V2 = 500 water to be is added 400 ml. 100 ml is already 9
present. Hence,
No. of gram equivalent of C = 30 × 103
Wt ´ 1000 2.65 ´ 1000 Again,
47. (c) Molarity = = = 0.1 M
MW ´ V 106 ´ 250 No. of gram equivalent of C
M1V1 = M2V2 mass in gram mass
\ 10 × 0.1 = 1000 × M2 = 0.001 M = Þ 30 × 103 =
gram equivalent weight 3
48. (a) Normality of ferrous Amm. sulphate
Þ mass = 90 × 10 g = 90 kg
3

= 3.92 ´1000 = 0.1 (Eq. wt of FAS is 392) +7 - +6


392 ´ 100 2. (a) KMnO4 ¾OH
¾
¾® K 2 MnO 4
N1V1 = N2V2 Change in oxidation number of Mn in basic medium is 1.
20 × 0.1 = 18 × N2 N2 = 0.111 Hence mole of KI is equal to mole of KMnO4.
1 gev. of KMnO4 = 31.6g Mass
0.111 gev. of KMnO4 = 31.6 × 0.111 = 3.5 g. 3. (c) Density =
Volume
49. (a) 2Mg + O 2 ¾¾
® 2MgO 1gram
1 gram cm–3 =
2 × 24 2×16 2 × 40 cm 3
48 g 32 g 80 g Mass 1gram
given30 g 30 g Volume = = = 1cm 3
Density 1gram cm -3
Actually
\ Volume occupied by 1 gram water = 1 cm3
Reacting 30 g 20 g 50g (formed)
or Volume occupied by
O2 left (30 – 20) = 10 g MgO formed 50 g.
50. (c) Eq. wt of KMnO4 in acid medium is 31.6 g. Hence this 6.023 ´ 1023
molecules of water = 1 cm3
much amount must be dissolved in 1 litre to prepare 18
normal solution. 1
[\ 1g water = moles of water]
18
51. (b) K 2Cr2 O 7 + H 2SO 4 + 3SO 2 ¾
¾® Thus volume occupied by 1 molecule of water
1 1 3
1 ´ 18
= cm 3 = 3.0×10–23 cm3.
K 2SO 4 + Cr2 (SO 4 )3 + H 2 O 6.023 ´ 10 23
i.e. the correct answer is option (c).
52. (b) BaCl 2 + H 2SO 4 ¾
¾® BaSO 4 + 2HCl 4. (a)
1 Mole 1 Mole 1 Mole 2KMnO4 + 3H2SO4 K2SO4 + 2MnSO4 + 3H2O + 5(O)] × 3
COO
0.5 Mole 0.5 Mole 0.5 Mole 2 Fe + 3(O) Fe2O3 + 2CO2 ]×5
COO
53. (d) 4 NH 3 (g ) + 5O 2 (g ) ¾
¾® 4 NO(g ) + 6H 2 O (l) 6KMnO4 + 9H2SO4 +10 COO
Fe 3K2SO4 + 6MnSO4 + 9H2O
(6 moles) COO
4 moles 5 moles 4 moles 6 moles + 5Fe2O3 + 10CO2
(10 moles)
given 1 Mole 1 Mole
From above equation 6 moles MnO4– required to oxidise
Reacting 0.8 1 ® 0.8 1.2 10 moles of oxalate.
(formed) Thus number of moles of MnO4– required to oxidise one
All, O2 consumed being limiting. 6
mole of oxalate = = 0.6 moles
10
54. (a) 2K 2 CrO 4 + 2HCl ¾¾® K 2 Cr2 O 7 + 2KCl + H 2 O
2 2 1 2 1
5. (a) Element Percentage Atomic Atomic Simple
weight ratio ratio
EXERCISE 3 C 38.71 12 38.71 3.23
= 3.23 =1
1. (c) 2Al 2O3 + 3C ¾¾ ® Al + 3CO 2 12 3.23
Gram equivalent of Al2O3 º gm equivalent of C H 9.67 1 9.67
= 9.67
9.67
=3
1 3.23
27
Now equivalent weight of Al = =9 O 100 - 16 51.62 3.23
3 (38.71 + 9.67) = 3.23 =1
16 3.23
= 51.62
12 0 +4
Equivalent weight of C = = 3 (C CO 2 ) Thus empirical formula is CH3O.
4
Some Basic Concepts of Chemistry 23
6. (d) Writing the equation for the reaction, we get
14. (a) Mg 3 N 2 + 6H 2 O 3Mg(OH ) 2 + 2 NH 3
PbO + 2HCl ¾¾ ® PbCl2 + H2O
207 + 16 2 × 36.5 207 + 71 15. (d) Relative atomic mass
= 223 g = 73g = 278g
6.5 Mass of one atom of the element
No. of moles of PbO = = 0.029 = th
223 1/12 part of the mass of one atom of Carbon - 12
3.2
No. of moles of HCl = = 0.0877 Mass of one atom of the element
36.5 ´ 12
or
Thus PbO is the limiting reactant 1 mole of PbO produce mass of one atom of the C - 12
1 mole PbCl2.
Now if we use 1/6 in place of 1/12 the formula becomes
0.029 mole PbO produces 0.029 mole PbCl2.
1 Mass of one atom of the element
7. (b) H2 + O H2 O ´6
2 2 Mass of one atom of C - 12
10g 64g
16. (d) 1 Mole of Mg3(PO4)2 contains 8 mole of oxygen atoms
æ 10 ö æ 64 ö
ç =5 mol ÷ ç = 2 mol ÷ \ 8 mole of oxygen atoms º 1 mole of Mg3(PO4)2
è2 ø è 32 ø
In this reaction oxygen is the limiting agent. Hence 1 -2
amount of H2O produced depends on the amount of O2 0.25 mole of oxygen atom º ´ 0.25 = 3.125 ´ 10
8
taken
Q 0.5 mole of O2 gives H2O = 1 mol 17. (d) Since molarity of solution is 3.60 M. It means 3.6 moles
\ 2 mole of O2 gives H2O = 4 mol of H2SO4 is present in its 1 litre solution.
8. (b) The number of atoms in 0.1 mole of a triatomic gas Mass of 3.6 moles of H2SO4
= 0.1 × 3 × 6.023 × 1023. = Moles × Molecular mass = 3.6 × 98 g = 352.8 g
= 1.806 × 1023 \ 1000 ml solution has 352.8 g of H2SO4
9. (c) No. of molecules 29% H2SO4 by mass means 29 g of H2SO4 is present in
44 100 g of solution
Moles of CO2 = =1 NA \ 352.8 g of H2SO4 is present in
44
100
48 = ´ 352.8 g of solution = 1216 g of solution
Moles of O3 = =1 NA 29
48
Mass 1216
8 Density = = = 1.216 g/ml = 1.22 g/ml
Moles of H2 = = 4 4NA Volume 1000
2
18. (a) 2Al(s) + 6HCl(aq) ® 2Al3+(aq) + 6Cl– (aq) + 3H2(g)
64
Moles of SO2 = =1 NA Q 6 moles of HCl produces = 3 moles of H2
64 = 3 × 22.4 L of H2 at S.T.P
wt 1000 3 ´ 22.4
10. (d) Molarity (M) = \ 1 mole of HCl produces = L of H2 at S.T.P
.P
mol. wt. vol (ml) 6
= 11.2 L of H2 at STP
wt. 1000 19. (a) Molality = Moles of solute / Mass of solvent in kg
2= ×
63 250 0.01/ 60 0.01
Molality = = ;
63 0.3 60 0.3
wt. = gm
2 d = 1 g/ml
100
wt. of 70% acid = ´ 31.5 = 45 gm = 5.55 10 4 m
70 20. (d) Q 18 gm, H2O contains = 2 gm H
6.02 ´ 10 20 ´ 1000 6.02 ´ 1021
11. (a) M = = 2
100 ´ 6.02 ´ 10 23 6.02 ´ 1023 \ 0.72 gm H2O contains = 0.72 gm = 0.08 gm H
18
= 0.01 M Q 44 gm CO2 contains = 12 gm C
12. (c) Volume increases with rise in temperature. Thus, some
water molecule may be evaporated at high temp. \ 3.08 gm CO2 contains = 12 3.08 = 0.84 gm C
44
558.5
13. (a) Fe (no. of moles) = = 10 moles 0.84 0.08
55.85 \C: H= :
12 1
C (no. of moles) = 60/12 = 5 moles. = 0.07 : 0.08 = 7 : 8
\ Empirical formula = C7H8
24 Chemistry
21. (b) For one mole of the oxide 9. (a) The desired equation is
Moles of M = 0.98
CaCl 2 + 2AgNO3 ® 2AgCl + Ca(NO3 ) 2
Moles of O2– = 1 111g 143.5´2g
Let moles of M3+ = x
\ Moles of M2+ = 0.98– x \ 14.35 g of AgCl will be obtained from
on balancing charge 111 ´ 14.35
= 5.55gm
(0.98 – x) × 2 + 3x – 2 = 0 143.5 ´ 2
x = 0.04
heat
10. (a) 2KClO3 ¾¾¾
0.04 ® 2KCl + 3O 2
\ % of M3+ = ´100 = 4.08% 2´122.5g 3´32g
0.98 245 96
22. (d) The following reaction occurs:
48 gm of oxygen will be produced from 122.5 g of KClO3
6Fe 2 + Cr2 O 27 + 14H + 6Fe3+ + 2Cr 3+ + 7H2O
\ Amount of 80% KClO3 needed
From the above equation, we find that Mohr's salt
(FeSO4.(NH4)2SO4.6H2O) and dichromate reacts in 6 : 1 100
= ´ 122.5 = 153.12gm
molar ratio. 80
EXERCISE 4 11. (c) The conditions given are standard conditions
1. (a) Sum of the figures 29.4406, 3.2 and 2.25 is 34.8906. The 224 ml has mass = 1g ;
sum should be reported to the first place of decimal as 3.2 22400 ml will have mass = 100g. This is mol. wt of gas
has only one decimal place. After rounding off the sum is 6.023 × 1023 molecules have 3 × 6.023 × 1023 atoms since
34.9. Hence number of significant figures is three. gas is triatomic
2. (a) 0.25/22.1176 = 0.011303. The least precise term has two \ weight of one atom
significnat figures, hence the result is 0.011.
100
3. (c) If zero is used to locate the decimal point it is considered = = 5.5 ´ 10 - 23 g
a significant figure. In 50.000 all zero are significant. 3 ´ 6.023 ´ 10 23
4. (b) The desired equation is 12. (a) Proceed as follows :
Element % At. wt. RNA Simplest ratio
BaCl 2 + H 2SO 4 ® BaSO 4 + 2HCl
208g 98g 233g 73g
7.3g 25
20.8g 9.8g 23.3g A 25 12.5 =2 1
12.5
5. (c) In law of reciprocal proportions, the two elements
combining with the third element, must combine with each 75
B 75 37.5 =2 1
other in the same ratio or multiple of that Ratio of S and O 37.5
when combine with C is 2 : 1. Ratio of S and O is SO2 is 1 : 1
\ The simplest formula of compound is AB
2 ´ 12
6. (a) 2% of 12 gm of carbon is = = 0.24g which is 254
100 13. (d) Moles of Iodine present = =2
127
C-14
\ The number of atoms in it 80
Moles of oxygen = =5
16
0.24 ´ 6.02 ´10 23
= = 1.03 ´10 22 atoms \ The molecular formula is = I2O5
14
7. (c) Number of atoms in 2.0 mole of S8 14. (b) Let the atomic weight of metal be M then
M = 50% 50/M = 1
= 2 ´ N A ´ 8 = 16 N A
O = 50% 50/16 = 2
number of atoms in 6.0 mole of S = 6 × NA = 16.5 NA
The formula is = MO2
Number of atoms in in 5.5 mole of SO2
= 5.5 × NA × 3 = 16.5 NA 16 1
Number of atoms in 44.8 litre at STP of \ = . Thus atomic weight of metal = 32
M 2
CO2 = 2 × NA × 3 = 6 NA
8. (d) An, AlF3 the number of F is 3 for one AlF3 molecule For second oxide
3F– º 1 formula unit of AlF3 M = 40% 40/32 = 1.25 1
O = 60% 60/16 = 3.75 3
1
3.0 × 1024 F– º ´ 3.0 ´ 10 24 AlF3 units \ Formula is MO3
3
Some Basic Concepts of Chemistry 25
23. (b) Moles of H2SO4 in 98 mg of H2SO4
20 ´ 1
15. (b) Moles of Ca++ to be precipitated = = 0.02 1
1000 = ´ 0.098 = 0.001
98
20 ´ 0.5 Moles of H2SO4 removes
Moles of Cu++ to be precipitated = = 0.01
1000
3.01 ´ 10 20
Hence molar amount of H2S will be in the ratio 2 : 1 = = 0.5 ´ 10 -3 = 0.0005
6.02 ´ 10 23
æ Molarity ´ volume in ml ö Moles of H2SO4 left = 0.001 – 0.0005 = 0.5 × 10–3
ç Remember Moles = ÷
è 1000 ø
24. (a) I 2 + 2Cl 2 ¾
¾® ICl + ICl3
16. (b) Mg + 2 HCl ® MgCl 2 + H 2 ­ 25.4 14.2
1 mole 1 mole 0 0 moles in the beginning
1 1 1 254 71
mole mole (12g of Mg = mol )
2 2 2 0.1 0.2 0 0
0 0 0.1 0.1 moles after reaction
10 25. (c) 5CO + I 2 O 5 ® 5CO 2 + I 2
17. (c) 10g of CaCO3 is = 0.1 mole
100
25.4
Number of protons in 1 mole of CaCO3 = (At. No. of Ca + Moles of I 2 O 5 =
254
At. No. of C + 3 At. No. of O) = 20 + 6 + 24 = 50 moles.
= 0.01 º 0.05 moles of CO
Protons in 0.1 mole of CaCO3 = 0.1 × 50 × 6.02 × 1023
Weight of CO = 0.05 × 28 = 1.4 g;
= 3.01 × 1024
18. (a) Decomposition of Ag2CO3 gives silver Weight of CO 2 = 2 – 1.4 = 0.6 g
0.6
2Ag 2CO3 ® 4Ag + 2CO2 + O 2 Hence % of CO 2 = × 100 = 30%
2´276g 4´108g 2
2.76g 2.16g
26. (a) Na 2 CO 3. xH 2 O . Its weight = 106 + 18 x .
19. (a) The chemical equation is
Weight of anhydrous Na 2CO 3 = 106
3BaCl 2 + 2 Na 3 PO 4 ® Ba 3 (PO 4 ) 2 + 6NaCl 18 x ´ 100
3Mole 2 mol 1mol % loss in weight = = 63
0.3mol 0.2 mol 0.1mol 106 + 18x
Hence Ba3(PO4)2 obtained is 0.1 mole \ x = 10
20. (c) Let the alcohol be ROH and x its molecular weight 3
27. (b) geq of HCl = ´ 2.5 = 0.20548 = geq of Al(OH )3
ROH + CH 3 MgI ® CH 4 + ROMgI 36.5
xg 16 g 0.20548 ´ 78
Weight of Al(OH)3 = = 5.342 g = 5342 mg
3
4.12 16 4.12
g of alcohol will produce ´ g of methane
1000 x 1000 5342
\ No of tablets = = 13.35 = 14
400
16 ´ 1.12
Methane actually obtained is = g 28. (b) SO 2Cl 2 + 2H 2O ® H 2SO 4 + 2HCl
22400
H 2SO 4 + Ba (OH) 2 ® BaSO 4 + 2H 2 O
16 ´ 4.12 16 ´1.12
equal = = \ x = 82.4 2HCl + Ba (OH) 2 ® BaCl 2 + 2H 2O
x ´ 1000 22400
Total moles of Ba (OH) 2 required = 2
21. (b) Element % At. wt. RNA Simplest Ratio
29. (b) Molecular weight of metal chloride
C 86 12 86/12=7.16 1
H 14 1 14/1 = 142 0.72 ´ 22400
= = 161.28 g
\ Empirical formula µ CH2 corresponds to general 100
formula CnH2n of alkene Weight of chlorine in metal chloride
22. (d) At S.T.P. 22.4 litre of gas contains 6.023 × 1023 molecules 65.5 ´ 161.28
\ molecules in 8.96 litre of gas = = 105.64 g
100
6.023 ´10 23 ´ 8.96 105.64
= = 24.08 ´10 22 Mole atoms of chlorine = =3
22.4 35.5
Hence metal chloride is MCl3
26 Chemistry

30. (a) K I + KCl; (geq of mixture = geq of AgNO 3 ) 1


35. (b) CO + O 2 ® CO 2 ;
xg (7.36 – x) g 2

æ x 7.36 - x ö 8.45 ´ 0.2 ´ 40 CO 2 + 2KOH ® K 2 CO 3 + H 2O


ç + ÷ =
è 166 74.5 ø 1000 28
Moles of KOH = = 0.50
\ on solving x = 4.216x 56
\ % of KI = 57.28% It corresponds to 0.25 mol of CO 2
31. (b) Let weight of C be xg, then S will be (14 – x) g
Hence mol of CO = 1 – 0.25 = 0.75 º mole of CO 2 formed
x / 12 2 Mol of KOH requred = 2 × 0.75 = 1.5 = 1.5 × 56 = 84 g
=
(14 - x) / 32 1
10
36. (b) Meq. of CaCO3 = × 1000 = 200
6 50
\ x = 6 g ; Moles of C = = 0.5 Meq. of HCl = 250 × 1 = 250
12
Meq. of HCl left in the solution = 250 – 200 = 50
32. (a) C3H 8 + 5O 2 ® 3CO 2 + 4H 2O \ Meq. of KOH requred V × 2 = 50
a 3a \ V = 25 ml
13 37. (c) ROH + CH 3 MgI ® CH 4 + ROMgI
C 4 H10 + O 2 ® 4CO 2 + 5H 2 O
2 1 mol of ROH º 1 mol of CH 4 º 22400 at STP
(3 – a) 4 (3 – a) 0.0046
But 3a + 4 (3 – a) = 10 Hence molecular wt of ROH = × 22400
2.224
\ a = 2 (Propane ) and 3 – 2 = 1 (Butane )
= 46 g
33. (a) N 2 + O 2 ® 2 NO
1
a a 2a 38. (b) Meq of oxalic acid in 16.68 ml = Meq of NaOH = 25 ×
15
1
CO + O2 ® CO 2 1 250
2 Meq of oxalic acid in 250 ml = 25 × × = 24.98
15 16.68
(10 - a )
(10 – a) (10 – a) 1.575
2 × 1000 = 24.98
(90 + 18x ) / 2
(10 - a ) \ x= 2
a+ =7
2
10
\ a= 4 39. (c) Molecular weight of XY2 = = 100
0 .1
volume of CO = 6 ml
9
6 ´ 100 molecular weight of X 3Y2 = = 180
Mole % of CO = = 60 0.05
10
Let a and b be the atomic masses of X and Y respectively.
34. (a) Na 2CO 3 NaHCO 3 Then a + 2b = 100
x (1 – x) 3a + 2b = 180
\ a = 40 and b = 30
x 1- x
= given (moles are equal ) 40. (a) V.D max = X NO2 (V.D) NO2 + X N 2O 4 (V.D) N 2O 4
106 84
27.6 = X × 23 + (1 – X) × 46
x = 0.557
\ X NO2 = 0.8
0.557 0.443 V ´ 0.1
+ =
53 84 1000
V = 157.7 ml
2
Structure of Atom
ATOM : (VIII)Atomic mass unit : It is 0.0005486 amu.
John Dalton proposed in 1808 that atom is the smallest indivisible (IX) Mass of one mole of electron : It is 0.55 mg.
particle of matter. CATHODE RAYS :
ATOMIC RADII : Originate from cathode. Electrons were discovered by cathode
Atomic radii are of the order of 10–8 cm (1Å) and radii of nuclei are ray experiment.

1 SOME PROPERTIES OF CATHODE RAYS :


nearly 10–13 cm. Radius of the nucleus is thus th of the radius (I) They cast shadow of the object in their path
10 5
(II) Rotate a mica wheel
of atom.
(III) Deflected by electric and magnetic fields in a direction showing
–10 negative charge.
radius of atom 10 m
–15 PROTON (H+) :
radius of nucleus 10 m
Discovered by Goldstein (1886) through perforated cathode rays
experiment which showed the presence of anode or canal rays.
Radius of atom = 105 ´ Radius of nucleus (I) Mass : It was found to be 1.672 × 10–24 g or 1.672 × 10–27 kg
or 1.00728 amu. It is about 1837 times heavier than an electron.
ELECTRON :
(II) Charge : It carries unit positive charge 1.602 × 10–19
(I) It was discovered through the study of Cathode rays coulombs or 4.803 × 10–10 esu.
(discovered by Zulius Plucker) and the name was proposed (III) Specific charge : It is 9.58 × 104 coulomb/gm. It varies with
by Stoney. nature of gas and is maximum if H2 is used.
(II) Charge : It was determined by Mullikan by oil drop method (IV) Charge on 1 mole of proton is 96500 coulomb or 1 Faraday.
as –1.602 × 10–19 coulombs or 4.803 × 10–10 e.s.u.
(III) Mass : It was found by J. J. Thomson as æ4 ö
(V) Volume : The volume ç pr 3 ÷ for proton is approximately
9.11 × 10–28 g. è 3 ø
(IV) Specific charge : e/m ratio is called specific charge and was 1.5 × 10–38 cm3.
determined by Thomson as 1.76 × 108 coulombs/gm.
NEUTRON (0n1) :
(V) Radius : It is of the order 10–15 cm.
Discovered by Chadwick by bombarding Be or B atoms (sheet)
(VI) Density : 2.17 × 1017 g/cc.
with high speed a-particles
m rest 9
(VII) Mass of electron at speed v is m = 4 Be + 2 He 4 ¾
¾® 6 C12 + 0 n1
2
ævö (I) Mass : Its mass is 1.675 × 10–24 gm or 1.675 × 10 –27 kg or
1- ç ÷
ècø 1.00866 amu.
28 Chemistry
(II) It is heavier than proton by 0.18%. (II) Radius of nucleus : It is of the order 1.5 × 10–13 cm to
(III) Density : Its density is 1.5 × 1014 g/cm3. 6.5 × 10–13 cm (1.5 – 6.5 Fermi). In general r n = r0 ´ A1 / 3 cm
(IV) Specific Charge : It is zero. Where r0 is a proportionality constant with value 1.4 × 10–13
cm. and A is mass number.
(V) Stability : It is least stable of all elementary particles present
(III) Volume of nucleus : It is about 10–39 cm3. and that of atom is
in an atom.
10–24 cm3
(VI) Disintegration : Isolated neutron is unstable and (IV) Density of nucleus : It is about 10 14 g/cm 3 .
distintegrates into electron, proton and neutrino.
Mass of nucleus Mass number
(VII) Among all elementary particles neutron is the heaviest and Density = =
Volume of nucleus 6.023 ´ 10 23 ´ 4 pr 3
least stable. 3
OTHER SUBATOMIC PARTICLES : (V) Diameter : It is about 10–15 m or 1 fm (1 fm = 10–15 m)
(I) Positron (Positive electron +1e0). Discovered by Dirac (1930) (VI) Nucleus contains neutrons and protons, collectively known
and C. D. Anderson (1932). They are highly unstable and as nucleons.
produce g-rays on combining with electrons. ATOMIC NUMBER/MOSELEY'S POSTULATES :
The number of protons present in an atom is called the atomic
(II) Neutrino and Antineutrino are particles of small mass and number, denoted by Z. Moseley postulated that the frequency of
no charge as stated by Fermi (1934). Anti-neutrino spin X-rays produced when beam of strong electrons fall on metal
clockwise and neutrino spin anticlockwise. target, called anti-cathode is related to the charge present on the
(III) Meson : They are unstable particles and include pions (p+, nucleus of an atom of the element used as anti cathode.
Mathematically
p– or p0) Kaons (K+, K–, K0, K–0) and eta meson ( h0 ).
n = a(Z - b) ; where n = frequency of X-rays, Z = nuclear charge,
Mass : They have mass intermediate of electron and proton.
a and b = constants
Discovery : By Yukawa (1935) and Kemmer. MASS NUMBER :
(IV) Anti proton (–1p1) : Negative proton produced by Segre and It is sum of number of protons and neutrons present in the nucleus
Weigland (1955) by proton-proton and proton-neutron (nucleons as a whole) and denoted by A. It is always a whole
collisions. number.
(V) v-particles : They may be positive, negative or neutral.
AVERAGE ATOMIC MASS :
It is the average mass of all existing isotopes and not necessarily
Discovered by G. D. Rochester and C C. Butler v– and v0
a whole number.
are 2200 times heavier than electron. Heavier disintegrate
ISOTOPES :
into pions and lighter into mesons.
Isotopes are atoms of the same element having same atomic number
THOMSON'S ATOMIC MODEL : but different mass number. e.g. 8O16, 8O17 and 8O18. They were
Atom is a sphere of positive electricity with a number of electrons discovered by Soddy (1911).
distributed within the sphere. ISOBARS :
Atoms of different elements having same mass number but different
RUTHERFORD'S NUCLEAR MODEL : atomic numbers e.g. 19K40, 20Ca40.
It is based upon a particles scattering experiment. Only a few (one ISOTONES :
in 10,000) a-particles were returned back from the Au-foil (10–4 Atoms of different elements with different atomic and mass
mm thick). numbers but same number of neutrons e.g. 14Si30, 15P31, 16S32.
ISODIAPHERS :
1 Atoms having same Isotopic number.
Scattering of a-particles µ
æqö ISOELECTRONIC SPECIES :
sin 4 ç ÷ Species having same number of electrons e.g. CO and CN– (both
è2ø
contain 14 electrons each) Na+ and Ne (both contain 10 electrons
Conclusion : Atom consists of two parts - (a) Nucleus each).
(b) Extra nuclear part. The ionic size decreases with increasing effective nuclear charge
of iso-electronic species.
Drawbacks - This model fails to explain the stability of the atoms
and line spectrum of hydrogen. P
Ionic size of isoelectronic species depend on (effective nuclear
e
THE NUCLEUS :
charge).
(I) Nucleus - It is small heavy and positively charged material
located in the centre of atom and electrons are distributed in 1
Ionic size µ
extra nuclear part of atom and revolve around the nucleus. Effective nuclear charge
Structure of Atom 29

Species C4– N3– O2–


WAVELENGTH :
F
It is the distance between two neighbouring crests or troughs of
Nuclear Charge 6 7 8 9
wave.
Total electrons 10 10 10 10
FREQUENCY :
P It is the number of waves which pass through a particular point in
.6 .7 .8 .9
e one second. Unit is Hertz (Hz) or cycles per second. 1 Cps = 1 Hz.
Ionic Radius (Å) 2.60 1.70 1.40 1.36 VELOCITY :
Species Na+ Mg2+ Al3+ Si4+ It is the distance travelled by wave in one second. Unit is
Nuclear Charge 11 12 13 14 m sec–1 and denoted by c.
Total electrons 10 10 10 10 c = nl.

P WAVE NUMBER :
(effective 1.1 1.2 1.3 1.4 It is the number of wavelengths per cm. It is equal to the inverse of
e
Nuclear charge) wavelength. Unit is cm–1 and is denoted by n .
Ionic radius (Å) 0.95 0.65 0.50 0.41
1
FAILURE OF RUTHERFORD'S MODEL : n=
l
According to classical theory of electromagnetism whenever a
charge is subjected to an acceleration around an opposite charge, AMPLITUDE :
it emits radiations continuously. Therefore the electron while It is the height of crest or trough. Square of amplitude determines
moving around nucleus in circular path must loose energy, go the amount of energy carried by the wave.
into spiral motion and ultimately fall into the nucleus. Practically it ELECTROMAGNETIC SPECTRUM :
does not happen. Arrangement of all electro-magnetic radiations in the increasing
PLANCK QUANTUM THEORY : order of their wavelengths or decreasing order of frequencies is
According to Max Planck (1901) radiant energy is emitted or called electromagnetic spectrum.
absorbed only in discrete units in form of bundle or packets of
energy called photon (quantum). Photon is not a material body. It Rays Wavelength Frequency
is massless bundle of energy (l in Å) n in Hz
Energy associated with each photon (quantum) Cosmic Rays 3×1021 to ¥
c g Rays 0.01 3×1019 to 3×1020
E = hn = h
l X Rays 1.0 2×1016 to 3×1019
h = Planck's constant = 6.626 × 10 –34 Js in S.I. units
(or 6.6726 × 10–27 ergs in c.g.s. units). n = frequency of radiation UV Rays 150 7.9×1014 to 2×1016
(each photon). Visible Light 3800 3.9×1014 to 7.9×1014
c = velocity of light, l = wavelength of radiation. Infra Red 7600 1×1011 to 3.95×1014
Thus a body can radiate energy in multiples of quantum hn, 2hn, Micro Waves 6×106 1×109 to 5×1011
3hn .... nhn where n is an integer.
Radio Waves 3×109 1×105 to 1×109
INTENSITY OF LIGHT :
ATOMIC SPECTRUM :
It is defined as number of photons falling per unit area per sec.
and depends upon wavelength of photons. Atoms of different elements emit electromagnetic radiations of
definite frequencies when excited by heating, passing current or
or it is defined as amount of energy falling per unit area per sec
electric discharge. Arrangement of these radiations in decreasing
and depends upon wavelength of photons.
order of frequencies is called atomic spectrum.
ELECTROMAGNETIC RADIATION :
Electromagnetic radiation by James maxwell (1870). An electrically DISPERSION :
changed particles moving under acceleration produces alternating Phenomenon of splitting of beam of light into radiations of different
electrical and magnetic fields mutually perpendicular to each other. frequencies after passing through a prism is called dispersion.
These fields are transmitted in the form of waves having same CONTINUOUS SPECTRUM :
wavelengths, frequency, speed and amplitude and are called It is obtained by passing sunlight (white light) through a prism.
electromagnetic waves or electromagnetic radiations. In vaccum The light is dispersed or resolved into continuous spectra of
all types of electromagnetic radiations travel at the same speed colours from Violet to Red. It contains radiations of all the
(3.0 × 108 ms–1) regardless of wavelengths. frequencies.
30 Chemistry
LINE SPECTRUM : (III) Number of Lines in a Transition : Mathematical formula for
It is an atomic spectrum of an element which consists of a number number of lines is follows as
of bright lines separated by dark bands. Atomic Spectra of most
(n 2 - n1 )(n 2 - n1 + 1)
elements is line spectrum. No. of lines =
2
ABSORPTION SPECTRUM :
BLACK BODY RADIATION :
It is obtained by passing white light through solutions or vapours
of chemical substance and then is analysed by spectroscope. It The radiation emitted by a body when heated is called black body
has few dark lines in otherwise continuous spectrum. radiation. The frequency of radiation increases with temperature.
At a given temperature the intensity of radiation emitted increases
EMISSION SPECTRUM :
with decrease of wavelength, reaches a maximum value and then
It is obtained by passing radiations from the atoms through prism. starts decreasing with further decrease of wavelength. A black
It has few bright lines against a dark back ground. body can emit and absorb all frequencies.
HYDROGEN SPECTRUM : PHOTOELECTRIC EFFECT :
It is obtained by passing light being emitted from discharge tube Phenomenon of ejection of electrons from the surface of a metal
containing hydrogen at low pressure through spectrograph. when light of suitable frequency strikes on it is called photoelectric
Hydrogen Spectrum has five Series effect.
Spectral Line Region n1 n2 (I) Threshold frequency (v0) : The minimum frequency of
Lyman Series U.V. 1 2, 3, 4..... incident radiation to cause the photoelectric effect is called
Balmer Series Visible 2 3, 4, 5..... threshold frequency.
Paschen Series I.R. 3 4, 5, 6..... (II) Work function : A part of the photons energy that is absorbed
Brackett Series I.R. 4 5, 6, 7.... by the metal surface to release the electron is known as work
Pfund Series I.R. 5 6, 7, 8.... function of the surface denoted by f. The remaining part of
the energy of photons goes into the Kinetic energy of the
éIn Balmer Series of hydrogen spectrum ù electron emitted.
ê the first line (3 ® 2) is known as L line ú
ê the second line (4 ® 2) is L line. a ú If n0 is the threshold frequency and n the frequency of incident
ë b û
light then f = hn 0 and E = hn.
Wavelength of line in spectrum is given by the expression
K.E. = E - f = hn - hn 0 = h (n - n 0 )
1 1 1 Note: • K.E. is independent of the intensity of light.
= = R H Z2 ÷
n12 n 22 ÷ • Number of photoelectrons µ Intensity of light
• K.E. is directly proportional to frequency of incident
RH = Rydberg Constant, Z = charge on nucleus, light.
n1, n2 = electronic levels involved in transition, = Wave number • E = K.E. + f is known as Einstein's photoelectric
2 equation.
2 me 4
RH = = 109677.76 cm -1 also for hydrogen • Energy required to stop the ejection of electrons is given
ch 3
by eV0 where e is the electric charge and V0 is stopping
æ 1 potential.
1 ö÷ -1
n = 3.29 ´ 1015 ç - sec where is frequency..
çn 2
è 1 n2 ø
2÷ BOHR’S MODEL OF ATOM :
Proposed by Niel Bohr to overcome the drawbacks of Rutherford’s
(I) For calculation of longest wavelength line use n 2 nearest
model.
and for shortest wavelength line use n 2 infinity e.g. value of
(i) Electrons revolve around nucleus only in certain selected
longest wave length in Balmer Series of hydrogen spectrum
circular orbits. These orbits are associated with definite
use n1 = 2 and n2 = 3.
energies and are called energy shells or levels.
(II) Last line of spectrum is called Series limit. Last line is the line
of shortest wavelength and high energy when n 2 = ¥ we (ii) Electrons can move only in those circular orbits where
get last wavelength angular momentum is a whole number and multiple of h/2p.
nh
1 R H2 n12 i.e. mvr = . or simply an integral number of wavelengths
= l (series limit) = , 2
n12 R H2
should fit in given electron orbit of radius r i.e. n =2 r.
Structure of Atom 31
(iii) Electrons energy in a particular orbit is constant. (xii) Number of revolutions per second in n th orbit,
(iv) Lowest energy state is called ground state and when electron vn velocity of electron in n th orbit
absorbs energy and jumps to higher state, it is called excited =
2prn circumference of the n th orbit
state v Velocity
(v) Electronic energy is negative because at infinite distance (xiii)Angular velocity = =
r radius
there is no interaction between electron and nucleus thus (xiv) Angular momentum = mvr
energy is zero. While when close to nucleus, attraction takes (xv) Number of spectral lines when electron jumps from the n th
place, energy is released and it becomes negative as it was
n (n - 1)
already zero. The energy of electron increases with the value to ground level = =
of n, but the difference of energy between two successive 2
orbits decreases. Thus (xvi) The electrons energy is generally expressed in kcal or kJ
E2 – E1 > E3 – E2 > E4 – E3 .......... etc. mol–1 or in electron volts eV.
(vi) Energy of electron in nth orbit 1 ergmol–1 = 1.44×1013 kcalmol–1 = 6.022×1013 kJmol–1
1eV = 1.602×10–19J
k2 2 2
me 4 z 2 -13.6Z2 (xvii)Some important values :
En = - = eV atom -1
n 2h 2 n2 In c.g.s. system,
m = 9.109×10–28g
-2.178 ´10-18 Z2 e = 4.803×10–10 esu,
= J atom -1
2
n h = 6.626×10–27 ergs,
where m = Mass of the electron, k= 1
e = Charge on the electron,
2 p 2 me 4
h = Planck's constant RH = cm - 1
ch 3
n = Principal quantum number,
k = A universal constant = 9.0 × 109 J.m/C2 In S I system,
The constant k is inverse of permitivity factor 4p 0 of the m = 9.109×10–31kg
e = 1.602×10–19C,
1
medium k = . The numerical value of permitivity factor h = 6.626×10–34 J.s,
4p 0
is 4p 0 = 1.11264 × 10–10 C2N–1m–2. In C.G.S. system k = 1. k = 9.0 × 109 Jm/C2
(vii) The radius of nth orbit
2 p 2 me 4
RH = = m -1
n2h 2 a0n 2
0.528Ån 2 2
(4 p o ) ch 3
rn = = = Å
2 2 Z Z
4 me Zk
(viii)The velocity of electron in nth orbit, e
In S I system the charge e is replaced by 4p o
nh 2 Ze 2 k 2.19 ´ 108 Z
vn = = = cm/sec. LIMITATIONS OF BOHR’S MODEL ARE :
2 mr nh n
1 (i) Explains the spectrum of elements having only one electron
The velocity of electron in first orbit of hydrogen is of (ii) Does not explain splitting of spectral lines under magnetic
137
the velocity of light. field (Zeeman effect) and electric field (stark effect)
(iii) Does not explain quantisation of angular momentum.
(ix) Kinetic energy of electron in nth orbit, (iv) It goes against the Heisenberg’s uncertainity principle.
2m p 2 e 4 Z 2 k 2 13.6 ´ Z 2 SOMMERFIELD MODEL :
Ek = = eV
n2h2 n2 (i) Motion of electrons is in closed elliptical paths of definite
(x) Potential energy of electron in nth orbit, energy levels having nucleus on either of the focii.
(ii) Angular momentum is quantized
- e2 - 4m p 2 e 4 Z 2 k 2 - 27.2 Z 2
Ep = = = eV n major axis length
r n2h2 n2 (iii) = where k = 1, 2 ---------n.
k minor axis length
(xi) Total energy of electron in nth orbit,
(iv) It does not explain distribution of electrons in extranuclear
part of atom and also does not explain for de Brogalie
k 2 2 2me4 Z2 13.6Z2
En = Ek + Ep = = eV concept.
n 2h 2 n2
32 Chemistry
QUANTUM MECHANICS : (v) has no physical significance but 2 gives intensity of
It was developed independently by Warner Heisenberg and Erwin electrons and thus gives probability of electron in a particular
Schrodinges and takes into account the dual behaviour (particle region.
and wave nature) of matter proposed by de Broglie. ORBITALS :
Planck’s Quantum theory successfully explains.
Orbitals are the regions in space around nucleus where probability
(i) Photoelectric effect
of finding the electron is maximum.
(ii) Black-body radiation
(iii) Line spectra of H-atom (i) Probability does not become zero even at infinity and is
(iv) Variation of heat capacity of solids with temperature. given by 2.
(ii) Electron orbitals in atoms arecalled atomic orbitals while
de- BROGLIE PRINCIPLE (1924) :
those in molecules are called molecular orbitals.
(i) Proposes that just as radiations have particle nature, the
(iii) Orbitals have definite energy and momentum and are
material particles are also associated with wave nature.
quantized. i.e, En = –E1/n2 thus Bohr’s concept of well
h h defined orbits is ruled out.
(ii) de Broglie wavelength is l = = h = Planck’s constant
mv p QUANTUM NUMBERS :
m = mass of object ; v= velocity and this equation is called
the de Broglie equation. (i) Four quantum numbers (n, l, m, s) help in providing complete
information about an electron in an atom.
DAVISSON AND GERMER’S EXPERIMENT :
(ii) Principal quantum number (n) determines the energy and
Confirms the wave nature of electrons.
average distance of electron. It has whole number values also
SCINTILLATION METHOD AND PHOTOELECTRIC denoted as K, L, M, N. etc. As n increases, distance of electron
EFFECT : from nucleus increases and energy increases.
Confirm the particle nature. (iii) Azimuthal quantum number (l) determines angular momentum
HEISENBERG’S UNCERTAINITY PRINCIPLE : of the electron. It also determines the shape of orbitals and it
may have all possible whole number values from 0 to n–1 for
“It is not possible to determine simultaneously the position and
each principal energy level. The sublevels are:
momentum of small moving sub-atomic e.g.,
Value of l 0 1 2 3
+
1H , He + , 1B, , n particle, such as electron with entire Sub-shell s p d f
certainty”. Magnitude of angular momentum of an electron in orbital,
h
(i) Mathematically ± Dx ´ Dp ³ h mvr = l (l + 1)
4p 2
where, D x = uncertainity in position Angular momentum of an electron in any orbit,
D p = uncertainity in momentum and h = Planck’s constant
(ii) As the mass of particle increases, the uncertainity decreases nh
mvr =
QUANTUM MECHANICAL MODEL OF ATOM : 2p
(i) Based on de Broglie's and Heisenberg’s principle. (iv) Magnetic quantum number (m) defines the orientation of
(ii) Put forward by Schrodinger (1920). Behaviour of electron electrons cloud in a particular sub shell. Values of m are the
was described in terms of equation known as Schrodinger number of orbitals associated with a particular sub shell in
wave equation main shell. Values of m lie from 0 to ± l. Total values of ‘m’
2 2 2 2
for a given n is n 2. Total values of ‘m’ for a given l is 2l +1.
m The table shows a clear relation between quantum numbers.
+ + +
(E - V) = 0 where is
2 2 2 2
x y z h Shell (n) Sub -shells (l) Orbitals (m)
amplitude of electron wave and is also called wave function.
n=1 Þ K shell l= 0 Þ 1s m= 0
x, y, z, are space coordinates, m is mass of electron, h is
Planck’s constant, E is total energy and V is potential energy n=2 Þ L shell l= 0 Þ 2s m= 0
of the electron. l= 1 Þ 2p m = –1,0, +1
(iii) Many solutions for this equation are possible for hydrogen
but only certain solutions are permissible and are called ( 2p x , 2p y , 2pz )
eigen values
n=3 Þ M shell l= 0 Þ 3s m= 0
(iv) The solution must be single valued, should satisfy the
l= 1 Þ 3p m = –1, 0, +1
l= 2 Þ 3d m = –2, –1, 0,+1, +2
relation dr = 0 and must be finite and continuous.
æ 3d , 3d , 3d , 3d 3d ö÷
0 ç xy yz zx x2 - y2 , z2 ø
è
Structure of Atom 33
(v) Spin quantum number (s) tells the spin of the electron. It Shape of d-orbitals :
+1 -1 y y z
can have two value (clockwise) and (anticlockwise).
2 2
x z x
h
Mathematically S = s(s +1) where s is amplitude of spin
2 dyz
dxy dxz
quantum angular momentum.
SHAPE OF ORBITALS : m = 0 for d
z2
(i) s orbitals are spherically symmetrical.
(ii) p orbitals are dumbell shaped. m = ± 1 for d and d xy
x 2 - y2
(iii) d orbitals have five different orientation. Three of them dxy,
dyz, dxz are identical in shape but have different orientation. m = ± 2 for d and d
x2 y2
(iv) The plane passing through nucleus where probability of
finding the electron is zero is called a nodal plane. Number z
y
of nodal planes in an orbital = l. Number of nodal planes
increases with increasing value of n. e.g. 1s has no nodal
plane. 2s has one nodal plane. For e.g. : s orbitals (l=0) have x x
no nodal plane, p orbital (l=1) have one nodal plane, d
orbitals (l=2) have two nodal planes.
d
Nodal plane = n - l - 1 x 2 - y2 d
z2
(v) Orbitals of a sub shell having same energy are called
degenerated orbitals. PAULI’S EXCLUSION PRINCIPLE :
(vi) Spherical surface within an orbital where probability of No two electrons in an atom can have same values for all the four
finding an electron is zero is called spherical or radial node. quantum numbers.
Number of spherical nodes = (n–l–1). Angular or non (i) It is not possible to accomodate more than two electrons in
spherical nodes = (l). Thus total nodes = (n – 1). an orbital. In other words. s sub shell can have maximum of
Shape of s-orbital : 2 electrons p sub shell can have maximum of 6 electrons.
z Thus max. no. of electrons in a shell can be 2n2.
(ii) Maximum number of electrons in a sub shell can be 2, 6, 10,
14 in s, p, d, f respectively and max. electrons in an atomic
x
orbital can be 2.
y Maximum number of electrons in a sub shell is equal to
4l + 2
s orbital
Shape of p-orbital : where l = 0, 1, 2, 3

for p z m = 0 and for p x or p y , m = ± 1 l = 0 (s) l = 1 (p) l = 2 (d) l = 3 (d)


Electrons Electrons Electrons Electrons
z = 4 ´ 0 + 2 = 4 ´1 + 2 = 4´ 2 + 2 = 4´3+ 2
=2 =6 = 10 = 14
x
y Note : Maximum numbers of electrons in an orbital = 2

px AUFBAU’S RULE :
Electrons are added to orbitals in increasing order of energies.
z The order of energies for orbitals is 1s, 2s, 2p, 3s, 3p, 4s, 3d, 4p,
z 5s, 4d, 5p, 6s, 4f, 5d, 6p, 7s.
(i) The order of energies can be calculated by (n + l) rule. i.e.
x x orbitals are filled in order of increasing (n+l) values the one
y with lower n value is filled first.
y
py (ii) The energy of atomic orbitals for H-atom depends on the
pz value of n only.
1s < 2s = 2p < 3s = 3p = 3d < 4s = 4p = 4d = 4f
34 Chemistry
HUND’S RULE OF MAXIMUM MULTIPLICITY : In case of s orbitals the number of peaks is equal to n,
The pairing of electrons in orbitals of a subshell does not take In case of p orbitals the number of peaks is equal to (n–1),
place until all orbitals of sub shell are singly occupied. In case of d, orbitals the number of peaks is equal to (n–2)
(i) This arrangement leads to lower energy level. The point at which the probability of finding the electrons is zero
(ii) Singly occupied orbitals should have same spins giving rise iscalled nodal point.
to lower energies. The distance of maximum probability increases with increase in
the value of n. hence 2s, 2p electrons are greater distance than 1s.
RADIAL PROBABILITY DISTRIBUTION CURVES :
and have greater energy also.
2
The electron density is directly proportional to . The larger ANGULAR PROBABILITY DISTRIBUTION CURVES :
the value of
2
more is the probability of finding the electrons. The total angular ( ). (f) depends only on the direction and
Schrodinger wave equation may be separated into a product of remain independent of the distance electrons from the nucleus
three functions dependent on r, q, f P
P
Q
z
R(r) ( ) ( ) O
( r, q, f ) = { 14243 O
¯ ¯ px
Radial Part Angular part

R (r) = Radial wave function, it may be 0, or ± P


R2 = Radial density in per unit volume of spherical shell. P
It is always positive. O
O
4 r 2 .dr.R 2 = Radial probability. It is defined as maximum density
of electrons in the volume of spherical shell having small thickness
dr. py pz

Note : ® 4 r 2 .dr is the volume of spherical shell having small Angular probability distribution curves for s and p orbitals. The
length of the line OP is proportional to the probability of finding
thickness dr.
the electrons. The length of the line OP is the same in all directions
n=1 for s orbital Hence there are equal chances for finding electrons
n=2
R .4pr .dr

R .4pr .dr

in all directions from the nucleus.


2

1s 2s The length of line decreases with increasing in the value of angle


q. Hence there are more chances for finding the electrons along
2

®r ®r the axes for p orbitals.


RITZ. COMBINATION PRINCIPLE :
n=3
R .4pr .dr

3s It states that the wave number n (reciprocal of wavelength) of


2

any line in hydrogen spectrum of a particular series can be


represented as a difference of two terms, one of which is constant
2

®r and other varies throughout the series. Mathematically,


n =2 æ 1
1 n 1 ö÷
n=3 n= = = RHç - .
R .4pr .dr
R .4pr .dr

l c çx 2
y 2 ÷ø
è
2

2p
2

3p
RH = Rydberg constant
2
2

®r ®r COMPTON EFFECT :
The decrease in energy (or increase in wavelength) of X-rays
after the scattering from the surface of carbon or light element is
R .4pr .dr

n=3
2

3d known as Compton effect.


2

®r
Structure of Atom 35

1. An electron beam can undergo diffraction by crystals. 4. A bulb emits light of = 4500Å . The bulb is rated as 150
Through what potential a beam of electrons be accelerated
so that its wavelength becomes equal to 1.54Å. (IIT 1997) watt and 8% of the energy is emitted as light. How many
photons are emitted by the bulb per second?
1 h Sol. Energy of one photon
Sol. mv 2 = eV and l =
2 mv
hc 6.6 ´ 10 -34 ´ 3 ´ 10 8
2 = E = hu = = = 4.42 ´ 10 -19 J
1 h l -10
\V = . 4500 ´ 10
2 ml2 e
150 ´ 8
Energy emitted by bulb = J
(6.62 ´10 -34 ) 2 100
=
2 ´ 9.108 ´10 -31 (1.54 ´ 10 -10 ) 2 ´1.6202 ´10 -19
19 150 8
= 63.3 volt. n .42 10 =
100
2. Wavelength of photon having energy 2 eV.
\ n = 27.2 ´1018 photons
-19 -12
Sol. 1 eV = 1.6 ´ 10 J or 1.6 ´ 10 ergs
5. If the critical wavelength for producing photoelectric effect
-34 8 is 2000Å. Then what wavelength of light will be required to
hc 6.62 ´10 ´ 3 ´ 10
l= = = 6.20 ´103 Å produce photoelectrons with double the K.E. of those
E -19
2 ´1.6 ´ 10
produced by light of wavelength of 1500Å.
3. The velocity of electron in a certain Bohr's orbit of
H-atom bears the ratio 1: 275 to the velocity of light. æ hc hc ö
Sol. K.E. with 1500Å = ç - ÷ (I)
(i) What is quantum number n of orbit. è 1500 2000 ø
(ii) Wave number of radiations emitted when electron jumps
from (n + 1) to ground state. æ hc hc ö
KE with xÅ = ç - ÷ (II)
è x 2000 ø
1
Sol. (i) Velocity of electron = ´ Velocity of light
275 According to question 2I = II

æ hc hc ö æ hc hc ö
2.19 ´ 108 ´ Z 2ç - ÷=ç - ÷
Again Vn = cm / sec è 1500 2000 ø è x 2000 ø
n

2.19 ´ 108 ´ 1 1 \ x = 1200Å


\ = ´ 3 ´1010
n 275 6. Calculate the wavelength of radiations emitted producing a
line in Lyman Series when an electron falls from fourth
2.19 ´ 10 8 ´ 275 stationary state in hydrogen atom (RH = 1.1 × 107 m–1)
\n = =2
3 ´ 1010
1 é 1 1 ù
Sol. = RH ê - ú
1 æ 1 1 ö l 2 2
(ii) Wave number = = R H Z2 ç - ÷ ëê n1 n 2 ûú
ç n2 n2 ÷
è 1 2ø
1 é1 1 ù
= 1.1 ´ 10 7 ê - ú
æ1 1 ö l ë12
42 û
= 109678çç - ÷÷ = 9.75 ´ 10 4 cm -1
è 1 32 ø
\ l = 0.9696 ´10 -7 meter
36 Chemistry
7. Calculate the momentum of a particle with de-Broglie
Sol. Magnetic moment = n ( n + 2)
wavelength 1Å.
Where n is the number of unpaired electrons.
Sol. 1Å = 10 -10 m
1.73 = n ( n + 2) \n =1
-34
h 6.62 ´ 10 \ Vanadium atom must have one unpaired electron hence its
p= = = 6.62 ´ 10 -24 kgms -1
-10
10 configuration is 23 V 4 + = 1s 2 , 2s 2 p 6 , 3s 2 p 6 3d1
8. The uncertainity in the position of a bullet weighing 20 g is 12. Find the ratio of the radius of the first Bohr orbit for the
± 10–4 m. Calculate the uncertainity in its velocity. electron orbiting the hydrogen nucleus to that of the electron
Sol. 20 g = 0.02 Kg orbiting the deuterium nucleus (mass nearly twice that of the
hydrogen nucleus).
h
± Δx.Δp = a0n 2
4 Sol. rn = ;
Z
h
or Dx.mDv = a 0 ´1 a 0 ´1
4p r1 = for hydrogen; r 2 = for deuterium;
1 1
6.62 ´ 10-34
\± Δv = = 2.63 ´ 10-29 ms-1 r1 1
4 ´ 3.14 ´ 0.02 ´10 -4 = ; 1 :1.
r2 1
9. What transition in the hydrogen spectrum would have the 13. In an atom 2 electrons move round the nucleus in circular
same wavelength as th e Balmer transition n = 4. orbits of radii R and 4R. The ratio of the time taken by them to
(n = 2 of He+ Spectrum). complete one revolution is.
1 æ 1 1 ö Sol. T 2 µ R 3
Sol. For He+ = R H Z 2 çç - ÷÷
l è2 2
42 ø T12 = K(R )3 ; T22 = K(4R ) 3

1 æ 1 1 ö÷ T12 R3 T1 1
For H = RH ç - \ = Þ =
l ç 2 2 ÷
T22 3 T2 8
è n1 n 2 ø 64R
14. Calculate the speed of electron in third orbit of hydrogen
1 ù é 12é 1 ù
1 atom. Also calculate the number of revolutions per second
Since l is the same Z ê 2 - 2 ú = ê 2 - 2 ú that this electron makes around the nucleus. Given
ë2 4 û êë n 1 n 2 úû
4p 0 = 1.112 ´10 -10 C 2 N -1m -2 .
é1 1 ù é 1 1 ù
Since Z = 2 ê 2 - 2 ú = ê 2 - 2 ú 2p(Ze2 / 4p 0 )
ë1 2 û êë n1 n 2 úû Sol. Speed v =
nh
n1 = 1 and n2 = 2
2 ´ 3.14(1.6 ´ 10 -19 C) 2 / 1.112 ´ 10 -10
= = 7.27 ´ 10 5 ms -1
10. Find out the number of waves by a Bohr electron in one 3 ´ 6.626 ´ 10 -34
complete revolution in its 3rd orbit. [IIT 1994]
Sol. Number of waves in an orbit v
Number of revolutions made by the electron =
2 pr
Circumference of orbit 2 r
= =
Wavelength n2 h2
r= 2
4 m( Ze2 / 4 0)
æ nh ö
2 ç ÷ 34
2 r 2 (mvr) 2 9 (6.626 10 )
= = = è ø =n =
h / mv h h 2 31 19 2 10
4(3.14) (9.1 10 kg)(1.6 10 ) / 1.112 10
(Since angular momentum mvr = nh/ 2p) Ans. 3.
= 4.782 ´ 10 -10 m
11. A compound of Vanadium has a magnetic moment of 1.73
BM. Work out the electronic configuration of the vanadium 7.27 ´ 10 5
v= = 2.42 ´ 10 4 s -1
ion in the compound. [IIT 1997] 2 ´ 3.14 ´ 4.782 ´ 10 -10
Structure of Atom 37
15. Calculate the frequency, energy and wavelength of the - 12
radiation corresponding to the spectral line of lowest D E = E ¥ - E 2 = 5.425 × 10 ergs
frequency in Lyman series in the spectra of H atom. Also
hc 6.626 ´ 10 - 27 ´ 3 ´ 1010
calculate the energy for the corresponding line in the spectra E= = 5.425 × 10 - 12 =
of Li2+ l l

(RH = 1.09678 × 10 7 m -1 , c = 3 × 108 ms -1 , 6.626 ´ 10 - 27 ´ 3 ´ 1010


l= cm = 3.66 × 10 - 5 cm
5.425 ´ 10 -12
h = 6.625 × 10 - 34 Js )
18. Iodine molecule dissociates into atoms after absorbing light
é 1 of 4500 Å. If one quantum of radiation is absorbed by each
1 1 ù
Sol. = RH ê - ú molecule, calculate the kinetic energy of iodine atoms (Bond
l 2 2
êë n1 n 2 úû
energy of I 2 = 240 KJ mol -1 )
é1 1 ù 3 hc
= RH ê 2 - 2 ú = RH Sol. Energy given to I 2 molecule =
ë1 2 û 4 l

4 4 1 6.626 ´ 10 - 34 ´ 3 ´ 108
l = 3R = × = 1.216 × 10 - 7 m = = 4.417 × 10 - 19 J
H 3 1.09678 ´ 107 m -1 4500 ´ 10 -10

c 3 ´ 10 8 ms - 1 Energy used for breaking up of I 2 molecules


v = = -7
1.216 ´ 10 m
240 ´ 10 3
= = 3.984 × 10 - 19 J
= 2.47 × 1015 s -1 6.023 ´ 10 23
\ Energy used in imparting K.E. to two I atoms
E = hu = 6.625 × 10 - 34 Js × 2.47 × 1015 s -1 = 16.36 × 10 - 19 J
2 = (4.417 × 10 - 19 – 3.984 × 10 - 19 ) J = 0.433 × 10 - 19 J
E 2+ = Z 2 × E H = 3 × 16.36 × 10 - 19 J
Li
K.E. per atom = 0.216 × 10 - 19 J
- 19
= 147.24 × 10 J
19. The energy required to remove an electron from the surface
16. Calculate the wavelength and energy of radiation emitted of Na metal is 2.3 eV. What is the largest wavelength of
for the electronic transition from infinity ( ¥ ) to stationary radiation which can show the photoelectric effect
state of H-atom.
hc
Sol. E = hu =
1 é1 1ù é1 1 ù l
Sol. = RH ê - ú = 1.09678 × 10 7 m -1 ê 2 - 2 ú
l 2 2
êë n1 n2 úû ë1 ¥ û
6.62 ´10 - 34 ´ 3 ´ 108
2.3 ×1.6 × 10 -19 J=
l
= 1.09678 × 10 7 m - 1

l = 9.11 × 10 - 6 m 6.62 ´ 10 - 34 ´ 3 ´ 108


l= = 5.38 × 10 - 7 m
2.3 ´ 1.6 ´ 10 -19
hc 6.625 ´ 10 - 34 Js ´ 3.0 ´ 108 ms -1
E= = 20. A certain photochemical reaction is found to acquire
l 9.11 ´ 10 - 6 m
8.1 × 10 -19 J energy per water molecule. Calculate the
= 2.18 × 10 - 20 J
number of photons per water molecule of light with
17. Calculate the energy required to remove an electron
wavelength of 2.21 × 10 -6 m that is required to initiate the
completely from n = 2 orbit. What is the longest wavelength
(in cm) of light that can be used to cause this transition. reaction.
Sol. Electron energy in H - atom E 8.1 ´ 10 -19 ´ 2.21 ´ 10 - 6
Sol. N = = = 9 photons
2.17 ´ 10 -11 hc 6.62 ´ 10 - 34 ´ 3 ´ 108
E= ergs
n2
21. An electron has a velocity of 300 ms -1 , that is accurate to
-11 0.01%. With what accuracy we can locate the position of
2.17 ´10
E2 = ergs = 0.5425 × 10 -11 ergs this electron.
4
38 Chemistry
23. Find out the electronic state from which an electron drops to
0.01 ´ 300
Sol. Accurate velocity = = 0.03 ms -1 emit radiation with a wavelength of 926Å and lying in
100 ultraviolet region (Lyman series) in the spectrum of hydrogen
atom.
6.62 ´ 10 - 34
Dx = = 0.0019 m R H = 1.097 × 10 7 m -1
4 ´ 3.14 ´ 0.03 ´ 9.1 ´ 10 - 31
-8
22. A photon of wavelength 4000 Å strikes a metal surface, the Sol. l = 926 Å = 926 × 10 cm = 926 × 10 - 10 m
work function of the metal being 2.13 eV. Calculate
1 æ 1 1 ö÷
(i) the energy of photon in eV = = RH ç -
l ç n2 n2 ÷
(ii) the kinetic energy of the emitted photoelectron and è 1 2ø
(iii) the velocity of the photoelectron Since the radiation is in the U.V region and belongs to Lyman
Sol. (i) Energy of the photon series therefore n1 = 1
hc
E = hu = 1 æ 1 1 ö÷
l \ = - 10
= 1.097 × 10 - 7 m -1 ç -
9.26 ´10 m ç 12 n 2 ÷
è 2ø
6.626 ´ 10 - 34 Js ´ 3 ´ 108 ms -1
E= = 4.97 × 10 - 19 J 1 1
4000 ´ 10 -10 m 1= =
n 22 -10
9.26 ´10 ´ 1.097 ´10 7
- 19
eV = 1.602 × 10 J
On solving , n 2 = 8
- 19
4.97 ´ 10 J 24. The normal human eye responds to visible light of
E= -19 = 3.10 eV
1.602 ´ 10 J wavelength raging from about 390 to 710 nm. Determine the
frequency range of the human eye
(ii) Kinetic energy of emitted photo electron
Sol. Wavelength l and frequency u are related to the velocity
Work function f = 2.13 eV
c
Kinetic energy = hu – f = 3.10 eV – 2.13 V = 0.97 eV by u=
l
Velocity of photoelectron
3.00 ´ 108 ms -1
1 2
K.E. = mv = 0.97 eV = 0.97 × 1.602 × 10 - 19 J = = 7.7 × 10 14 s - 1
2 390 ´ 10 - 9 m

3.00 ´ 108 ms -1
= 1.55 ´10-19 J and u = = 4.2 × 10 14 s - 1
710 ´ 10 - 9 m
1/ 2
é 2 ´ 0.97 ´ 1.602 ´ 10 -19 ù The frequency range of the human eye is 4.2 × 10 14 to
(iii) v = ê ú = 5.85 × 10 5 ms -1
êë 9.109 ´ 10- 31 úû 14
7.7 × 10 Hz.
Structure of Atom 39

Very Short/Short Answer Questions Long Answer Questions


1. From the following nuclei, select the isotopes and isobars: 14. (i) Calculate the wavelength of photon which will be
238 234 234 234 emitted when the electron of hydrogen atom jumps
92 U, 90 Th, 92 U, 91 Pa
from the fourth shell to the first shell. The ionization
2. Which energy level do not have p-orbital? energy of hydrogen atom is 1.312 × 103 kJ mol–1.
3. Which orbital is non-directional in nature? (ii) Which orbital in each of the following pairs is lower in
4. Name the experiments evidence to support the wave nature energy in a multi–electron system?
of light.
(a) 2s,2p (b) 3p, 3d (c) 3s, 4s (d) 4d, 5f
5. What is the probability of finding a 4d electron right at the
nucleus? 15. (i) Calculate the shortest and the longest
wavelengths of the Lyman series. Given, Rydberg
6. When moving with the same velocity which one of the
constant = 10967700 m–1.
following particles has the largest de Broglie wavelength
and why? (a) Electron (b) Proton (c) a-particle (ii) Calculate the frequency, energy and wavelength of
7. What would you predict for the atomic number of the noble the radiation corresponding to the spectral line of
gas beyond Rn, if such an element had sufficient stability lowest frequency in Lyman series in the spectrum of
to be prepared or observed? Assume that 'g' orbitals are still hydrogen atom.
not occupied in the ground states of the preceding elements. 16. What were the weaknesses or limitations of Bohr’s model
8. Calculate the kinetic energy of the ejected electron when of atoms ? Briefly describe the quantum mechanical model
ultra–violet radiation of frequency 1.6 × 1015 s–1 strikes the of atom ?
surface of potassium metal. Threshold frequency of 17. (i) What is an emission spectrum ?
potassium is 5 ×1014 s–1. (h = 6.63 × 10–34 Js)
(ii) Explain the hydrogen spectrum.
9. Calculate the wavelength of an electron that has been
accelerated in a particle accelerator through a potential 18. (i) Write outer electronic configuration of Cr atom. Why
difference of 100 million volts. are half filled orbitals more stable ?
(1 eV = 1.6 × 10–19 J, me = 9.1 × 10–31 kg, (ii) State Heisenberg’s uncertainty principle. An electron
h = 6.6 × 10–34 Js, c = 3.0 × 108 ms–1) has a velocity of 50 ms–1 accurate upto 99.99%.
Calculate the uncertainty in locating its position.
B
10. Energy in a Bohr orbit is given to be equal to , where (Mass of electron = 9.1 × 10–31 kg,
n2
B = 2.179 × 10–18 J. Calculate the wavelength of the emitted h = 6.6 × 10–34 Js)
radiation when electron jumps from the third orbit to the Multiple Choice Questions
second.
19. Which of the following statements about the electron is
11. (i) An atomic orbital has n = 3. What are the possible
incorrect?
values of l and m ?
(a) It is negatively charged particle
(ii) List the quantum numbers (m and l) of electron for 3d
orbital. (b) The mass of electron is equal to the mass of neutron.
(iii) Which of the following orbitals are not possible ? (c) It is a basic constituent of all atoms.
1s, 2p, 1p, 3f (d) It is a constituent of cathode rays.
12. Why does the charge to mass ratio of positive rays depends 20. Which of the following properties of atom could be explained
on the gas taken in the discharge tube whereas charge to correctly by Thomson Model of atom?
mass ratio of cathode rays is same for all gases? (a) Overall neutrality of atom.
13. When light of frequency v is thrown on a metal surface with
(b) Spectra of hydrogen atom.
threshold frequency v0, photo – electrons are emitted with
maximum kinetic energy = 1.3 × 10–18J.If the ratio, v : v0 (c) Position of electrons, protons and neutrons in atom.
= 3 : 1, calculate the threshold frequency v0. (d) Stability of atom.
40 Chemistry
21. Which of the following pairs have identical values of e/m? 26. In a Bohr model of an atom, when an electron jumps from n
(a) A proton and a neutron = 3 to n = 1, how much energy will be emitted?
(b) A proton and deuterium (a) 2.15 × 10–11 ergs (b) 2.389 × 10–12 ergs
(c) Deuterium and an a-particle (c) 0.239 × 10–10 ergs (d) 0.1936 × 10–10 ergs
(d) An electron and g-rays
27. The uncertainty in the momentum of an electron is 1.0 × 10–
22. Chlorine exists in two isotopic forms, Cl-37 and Cl-35 but 5 kg ms–1. The uncertainty in its position will be (h = 6.62 ×
its atomic mass is 35.5. This indicates the ratio of Cl-37 and
Cl-35 is approximately 10–34 kg m2s–1)
(a) 1 : 2 (b) 1 : 1 (a) 1.05 × 10–26 m (b) 1.05 × 10–28 m
(c) 1 : 3 (d) 3 : 1 (c) 5.27 × 10–30 m (d) 5.25 × 10–28 m
23. If E1, E2, and E3 represent respectively the kinetic energies 28. Which one of the following set of quantum numbers is not
of an electron and an alpha particle and a proton each having possible for 4p electron?
same de-broglie wavelength then
(a) E1 > E3 > E2 (b) E2 > E3 > E1 1
(a) n = 4, l = 1, m = –1, ms = +
(c) E1 > E2 > E3 (d) E1 = E2 = E3 2
24. If travelling at same speeds, which of the following matter
waves have the shortest wavelength? 1
(b) n = 4, l = 1, m = 0, ms = +
(a) Electron (b) Alpha particle (He2+) 2
(c) Neutron (d) Proton
25. The third line of the Balmer series, in the emission spectrum 1
(c) n = 4, l = 1, m = 2, ms = +
of the hydrogen atom, is due to the transition from the 2
(a) fourth Bohr orbit to the first Bohr orbit
(b) fifth Bohr orbit to the second Bohr orbit 1
(d) n = 4, l = 1, m = –1, ms = –
(c) sixth Bohr orbit to the third Bohr orbit 2
(d) seventh Bohr orbit to the third Bohr orbit

1. A neutral atom (Atomic No > 1) has 4. Which of the following statements is incorrect?
(a) electron and proton (a) Cathode rays carry negative charge
(b) neutron and electron (b) Frequency of X-rays is higher than that of microwaves
(c) neutron, electron and proton (c) Unit for wave number is m–1
(d) neutron and proton. (d) Threshold frequency is the same for all metals.
2. Which of the following set of quantum numbers belong to 5. Which is not true with respect to cathode rays?
highest energy? (a) A stream of electrons
(b) Charged particles
1
(a) n = 4, l = 0, m = 0, s = + (c) Move with speed same as that of light
2 (d) Can be deflected by magnetic fields
1 6. Rutherford’s a-particle dispersion experiment concludes
(b) n = 3, l = 0, m = 0, s = + (a) all positive ions are deposited at small part
2
(b) all negative ions are deposited at small part
1 (c) proton moves around the electron
(c) n = 3, l = 1, m = 1, s = + (d) neutrons are charged particles.
2
7. Ratio of mass of proton and electron is
1 (a) infinite (b) 1.8 × 103
(d) n = 3, l = 2, m = 1, s = +
2 (c) 1.8 (d) None of these.
1-
8. Which of the following is the main cause of late discovery of
3. The number of electrons in é 19
40 ù
K is neutron?
ë û (a) Neutron is chargeless particle
(a) 20 (b) 40 (b) Neutron is highly unstable particle
(c) 18 (d) 19 (c) Neutron in the nucleus moves very fast
(d) All of these.
Structure of Atom 41
9. Which of the following does not contain number of neutrons (a) 3 : 1 (b) 2 : 1
40 (c) 1 : 2 (d) 1 : 3
equal to that of 18 Ar ? 20. The energy of a photon is given as DE/atom
(a) 41 (b) 43
19 K 21 Sc 3.03 ´10-19 J atom-1 . Then the wavelength (l) of the
(c) 40 (d) 42 photon is
21 Sc 20 Ca
(a) 65.6 nm (b) 656 nm
10. Rutherford’s experiment which established the nuclear model
of the atom used a beam of (c) 0.656 nm (d) 6.56 nm
(a) b-particles which impinged on a metal foil and got 21. If wavelength of photon is 2.2×10–11 m, h = 6.6 ×10–34 Js,
absorbed then momentum of photon is
(b) g-rays which impinged on a metal foil and ejected (a) 3 × 10–23 kg/s (b) 3.33 × 1022 kg/s
–44
electrons (c) 1.452 × 10 kg/s (d) 6.89 × 1043 kg/s
(c) helium atoms, which impinged on a metal foil and got 22. Which orbital of carbon can absorb photon but not emit it?
scattered (a) 1 s (b) 2 s
(d) helium nuclei, which impinged on a metal foil and got (c) 3 p (d) 2 p.
scattered 23. Brackett series are produced when the electrons from the
11. Number of protons, neutrons and electrons in the element outer orbits jump to
231 (a) 2nd orbit (b) 3rd orbit
89 X is (c) 4th orbit (d) 5th orbit.
(a) 89, 89, 242 (b) 89, 142, 89 24. The first emission line in the atomic spectrum of hydrogen in
(c) 89, 71, 89 (d) 89, 231, 89 the Balmer series appears at
12. An element has atomic number 11 and mass number 24. What
9R 7R
does the nucleus contain? (a) cm -1 (b) cm -1
(a) 11 protons, 13 neutrons 400 144
(b) 11 protons, 13 neutrons, 13 electrons
3R 5R
(c) 13 protons, 11 neutrons (c) cm -1 (d) cm -1
(d) 13 protons, 11 electrons 4 36
13. The number of electrons and neutrons of an element is 18 25. In hydrogen spectrum which of the following lies in the
and 20 respectively. Its mass number is wavelength range 350–700 nm?
(a) 2 (b) 17 (a) Balmer series (b) Lyman series
(c) 37 (d) 38 (c) Brackett series (d) paschen series.
14. The hydride ion is isoelectronic with 26. In Balmer series of hydrogen atom spectrum which electronic
(a) H+ (b) He+ transition causes third line?
(c) He (d) Be. (a) Fifth Bohr orbit to second one
15. Which of the following isoelectronic species has the smallest (b) Fifth Bohr orbit to first one.
atomic radius? (c) Fourth Bohr orbit to second one
(a) N 3- (b) O 2- (d) Fourth Bohr orbit to first one.
27. The spectrum of He is expected to be similar to that of
(c) F - (d) Ne. (a) H (b) Na
16. Among the following groupings which represents the (c) He +
(d) Li+
collection of isoelectronic species? 28. In hydrogen atom, energy of first excited state is –3.4eV.
(a) NO + , C 22 - , O -2 , CO (b) N 2 , C 22 - , CO, NO Then find out the KE of the same orbit of hydrogen atom
(a) + 3.4 eV (b) + 6.8 eV
(c)CO, NO + , CN - , C 22 - (d) NO, CN - , N 2 , O -2 (c) – 13.6 eV (d) + 13.6 eV
17. Which of the following is not iso electronic 29. In the Bohr’s orbit, what is the ratio of total kinetic energy
and total energy of the electron?
+
(a) Na (b) Mg 2+ (a) –1 (b) –2
(c) 1 (d) +2
2-
(c) O (d) Cl - 30. The kinetic energy of an electron accelerated from rest
18. Chloride ion and potassium ion are isoelectronic. Then through a potential difference of 5 V will be
(a) their sizes are same (a) 5 J (b) 5 erg
(b) chloride ion is bigger than potassium ion (c) 5 eV (d) 8 × 10–19eV
(c) potassium ion is relatively bigger 31. The angular momentum of an electron revolving around the
(d) depends upon the other cation or anion nucleus is integral multiple of
19. Ratio of energy of photon of wavelength 3000 Å and 6000Å (a) 2n (b) h
is (c) h / 2p (d) 2pn
42 Chemistry
32. The orbital angular momentum of an electron in 2 s orbital is 44. The radius of hydrogen atom in the ground state is 0.53Å.
The radius of Li2+ ion (atomic number = 3) in a similar state is
1 h
(a) + . (b) zero (a) 0.17 Å (b) 1.06 Å
2 2p (c) 0.53 Å (d) 0.265 Å
h h 45. The Bohr orbit radius for the H-atom (Z=1) is approximately
(c) (d) 2. 0.53 Å. The radius for the first excited state orbit is
2p 2p
(a) 0.13 Å (b) 1.06Å
33. The energy of an electron in the first Bohr orbit of H atom is
(c) 4.77 Å (d) 2.12 Å
– 13.6 eV. The possible energy value(s) of the excited state
46. The ratio of the radius of the first Bohr orbit for the electron
(s) for electrons in Bohr orbits of hydrogen is (are)
orbiting the hydrogen nucleus to that of the electron orbiting
(a) – 3.4eV (b) – 4.2eV
the deuterium nucleus (mass nearly twice that of the
(c) – 6.8 eV (d) + 6.8eV. hydrogen nucleus) is approximately
34. If the radius of first Bohr orbit be a0, then the radius of the (a) 2 : 1 (b) 1 : 1
third orbit would be (c) 1 : 2 (d) 4 : 1
(a) 3 × a0 (b) 6 × a0 47. The energy of the first electron in helium will be
(c) 9 × a0 (d) 1/9 × a0 (a) –13.6 eV (b) –54.4 eV
35. According to Bohr’s theory of hydrogen atom, which of the (c) –5.44 eV (d) zero
following is quantised for an electron? 48. Who modified Bohr’s theory by introducing eiliptical orbits
(a) Angular momentum (b) Angular acceleration for electron path?
(c) Acceleration (d) Velocity. (a) Hund (b) Thomson
36. According to Bohr’s theory the energy required for the (c) Rutherford (d) Sommerfeld.
transition of H-atom from n = 6 to n = 8 state is 49. Wavelength associated with electron motion
(a) equal to the energy required for the transition from n = (a) increases with increase in speed of electron
5 to n = 6 state (b) remains same irrespective of speed of electron
(b) larger than in (a) (c) decreases with increase of speed of e– (electron)
(c) less than in (a) (d) is zero.
(d) equal to the energy required for the transition from n = 50. If the Planck’s constant h = 6.6×10–34 Js, the de Broglie
7 to n = 9 state. wavelength of a particle having momentum of 3.3 × 10–24
37. The wave number of the light emitted by a certain source is kg ms –1 will be
2 × 106 m . The wavelength of this light is (a) 0.002 Å (b) 0.5Å
(a) 500 m (b) 200 nm (c) 2Å (d) 500Å
(c) 500 nm (d) 5 × 107 m 51. What is the wavelength associated with an electron moving
38. For which of the following species, Bohr’s theory is not with a velocity of 106 m/s?
applicable? (given h = 6.63 × 10–34 Js and m = 9.11 × 10–31 kg)
(a) Be3+ (b) Li2+ (a) 72.7 nm (b) 0.727 nm
2+
(c) He (d) H. (c) 7.27 nm (c) None of these.
39. As the nuclear charge increases from Neon to Calcium, the 52. The wavelength associated with a golf balf weighing 200 g
orbital energies and moving at a speed of 5 m/h is of the order
(a) increase (b) increase very rapidly (a) 10–10 m (b) 10–20m
–30
(c) increase very slowly (d) fall (c) 10 m (d) 10–40m
40. The ratio of ionization energy of H and Be+3 is 53. The uncertainty in the position of an electron
(a) 1 : 1 (b) 1 : 3 (mass = 9.1 × 10–28 g) moving with a velocity of 3.0 × 104
(c) 1 : 9 (d) 1 : 16 cm s–1 accurate upto 0.011% will be
41. In a Bohr model of on atom, when an electron jumps from n = (a) 1.92 cm (b) 7.68 cm
3 to n = 1, how much energy will be emitted? (c) 0.175 cm (d) 3.84 cm.
(a) 2.15 × 10–11 ergs (b) 2.389 × 10 –12 ergs 54. The uncertainty in the momentum of an electron is
(c) 0.239 × 10 ergs–10
(d) 0.1936 × 10 –10 ergs 1.0 × 10–5 kg ms –1. The uncertainty in its position will be (h =
42. In H-atom electron jumps from 3rd to 2nd energy level, the 6.62 × 10–34 kg m2 s–1)
energy released is (a) 1.05 × 10–26 m (b) 1.05 × 10–28 m
–30
(a) 3.03 × 10–19 J/atom (b) 1.03 × 10–19 J/atom (c) 5.27 × 10 m (d) 5.25 × 10–28 m
(c) 3.03 × 10–12 J/atom (d) 6.06 × 10–19 J/atom 55. Azimuthal quantum number determines the
43. The value of the energy for the first excited state of hydrogen (a) size
atom will be (b) spin
(a) –13.6 eV (b) –3.40 eV (c) orientation
(c) –1.51 eV (d) –0.85eV (d) angular momentum of orbitals
Structure of Atom 43
56. When the value of azimuthal quantum number, l = 2, value of 65. The electrons, identified by quantum numbers n and l (i) n =
‘n’ will be 4, l = 1 (ii) n = 4, l = 0 (iii) n = 3, l = 2 (iv) n = 3,
(a) 3 (b) 4 l = 1 can be placed in order of increasing energy, from the
(c) 5 (d) any one of these. lowest to highest, as
57. The values of quantum numbers n, l and m for the 5th electron (a) (iv) < (ii) < (iii) < (i) (b) (ii) < (iv) < (i) < (iii)
of Boron will be (c) (i) < (iii) < (ii) < (iv) (d) (iii) < (i) < (iv) < (ii)
(a) n = 1, l = 0, m = –1 (b) n = 2, l = 1, m = –1 66. Bohr's model of atom is in conflict with
(c) n = 2, l = 2, m = –1 (d) n = 1, l = 2, m = –1 (a) Pauli's exclusion principle
(b) Heiseinberg's uncertainity principle
58. The total number of orbitals in a shell with principal quantum
(c) Max Planck's quantum theory
number ‘n’ is
(d) All the above three
(a) 2 n (b) 2 n2
2 67. What is the maximum number of electrons that can be
(c) n (d) n.
accomodated in an atom in which the highest principal
59. Which of the following combinations of quantum numbers quantum number value is 4?
is allowed? (a) 10 (b) 18
n l m ms (c) 36 (d) 54.
(a) 3 2 1 0 68. The total number of orbitals possible for principal quantum
1 number n is
(b) 2 0 0 - (a) n (b) n 2
2
(c) 2n (d) 2n2
1 69. Which of the following is not possible ?
(c) 3 –3 –2 + (a) n = 3, l = 0, m = 0
2
(b) n = 3, l = 1, m = –1
1 (c) n = 2, l = 0, m = –1
(d) 1 0 1 +
2 (d) n = 2, l = 1, m = 0.
60. Which one of the following set of quantum numbers is not 70. For how many orbitals, the quantum numbers
possible for 4p electron? n = 3, l =2, m = +2 are possible?
(a) 1 (b) 2
1
(a) n = 4, l = 1, m = –1, mS= + (c) 3 (d) 4
2 71. The correct set of quantum numbers for a 4d electron is
1 1
(b) n = 4, l = 1, m = 0, mS = + (a) 4, 3, 2, + (b) 4, 2, 1, 0
2 2
1 1 1
(c) n = 4, l = 1, m = 2, mS = + (c) 4, 3, - 2, + (d) 4, 2,1, -
2 2 2
1 72. For a, f-orbital, the values of m are
(d) n = 4, l = 1, m = –1, mS = - (a) –2, –1, 0, +1, +2
2
(b) –3, –2, –1, 0, +1, +2, 3
61. The maximum number of electrons in subshell with
(c) –1, 0, +1
l = 2 and n = 3 is
(d) 0, +1, +2, +3
(a) 2 (b) 6
73. The values of Planck's constant is 6.63 × 10–34 Js. The velocity
(c) 12 (d) 10 of light is 3.0 × 108 m s–1. Which value is closest to the
62. An electron has principal quantum number 3. The number of wavelength in nanometres of a quantum of light with
its (i) sub-shells and (ii) orbitals would be respectively frequency of 8 × 1015 s–1?
(a) 3 and 5 (b) 3 and 7 (a) 5 × 10–18 (b) 4 × 101
7
(c) 3 and 9 (d) 2 and 5 (c) 3 × 10 (d) 2 × 10–25
63. An e– has magnetic quantum number as –3, what is its 74. The number of nodal planes ‘d’ orbital has
principal quantum number? (a) 1 (b) 2
(a) 1 (b) 2 (c) 3 (d) 0
(c) 3 (d) 4 75. What do you mean by degenerate orbitals?
64. The four quantum numbers of the valence electron of (a) Orbitals having equal energy
potassium are (b) Oribitals having equal wave function
(a) 4, 1, 1, 1/2 (b) 4, 0, 0, 1/2 (c) Oribitals having equal energy but different wave
(c) 4, 5, 0, 1/2 (d) 4, 4, 0, 1/2 function
(d) Orbitals having equal energy and equal wave function.
44 Chemistry
76. The number of nodal planes in a px orbital is 90. The number of unpaired electrons in a Nickel atom (ground
(a) one (b) two state) are (Atomic No. of Ni = 28)
(c) three (d) zero. (a) 2 (b) 5 (c) 3 (d) 7
77. A 5f orbital has 91. The element with its electronic configuration of its
(a) one node (b) two nodes atom 1 s2 2 s2 2 p6 3 s2 3 p6 3 d10 4 s1 is
(c) three nodes (d) four nodes. (a) Fe (b) Co (c) Ni (d) Cu.
78. “ No two electrons in an atom can have same set of all the 92. In Cu. (At. No. 29)
four quantum numbers” is known as (a) 13 electrons have spin in one direction and 16 electrons
(a) Hund’s rule in other direction
(b) Aufbau principle (b) 14 electrons have spin one direction and 15 electrons
(c) Uncertainty principle in other direction
(d) Pauli’s exclusion principle. (c) one electron can have spin only in the clockwise
79. Which of the following has maximum number of unpaired direction
electrons ? (d) None of the above is correct.
3+ 93. Which of the following atom has no neutron in its nucleus?
(a) Mg 2+ (b) Ti 3+ (c) V (d) Fe 2 +
(a) Helium (b) Lithium
80. An element M has an atomic mass 19 and atomic number 9, (c) Protium (d) Tritium
its ion is represented by 94. In the ground state, an element has 13 electrons in its M-
(a) M + (b) M 2 + (c) M - (d) M 2 - shell. The element is
2 2 5 1
81. The configuration 1s 2 s 2p 3 s shows (a) zinc (b) chromium
(a) ground state of fluorine (c) nickel (d) iron
(b) excited state of fluorine 95. Which one of the following pairs of ions has the same
(c) excited state of neon atom electronic configuration?

(d) excited state of O -2 ion. (a) Cr 3+ , Fe 3+ (b) Fe3+ , Mn 2+


82. Heisenberg's Uncertainity principle is applicable to (c) Fe 3+ , Co 3+ (d) Sc 3+ , Cr 3+
(a) atoms only (b) electron only
96. The correct order of increasing energy of atomic orbitals is
(c) nucleus only (d) any moving object
(a) 5 p < 4 f < 6 s < 5 d (b) 5 p < 6 s < 4 f < 5 d
83. Electronic configuration of four elements are given below.
(c) 5 p < 5 d < 4 f < 6 s (d) none of these
Which of the corresponding element would be most
97. The number of d-electrons retained in Fe2+ (At. no. of
paramagetic?
Fe = 26) ion is
(a) 1s2 2s2 2p6 (b) 1s2 2s2 2p1
2 2 5 (a) 3 (b) 4
(c) 1s 2s 2p (d) 1s2 2s2 2p4
(c) 5 (d) 6
84. Which of the following is the electronic configuration of
98. The correct electronic configuration of Cu (29) is
Cu2+ (Z = 29)?
(a) 1 s2 2 s2 2p6 3 s2 3 p6 3 d 10 4 s1
(a) [Ar] 4s1 3d8 (b) [Ar] 4s2 3d10 4 p1
1 10 (b) 1 s2 2 s2 2p6 3 s2 3 p6 3 d 6
(c) [Ar] 4s 3d (d) [Ar] 3d9
(c) 1 s2 2 s2 2p6 3 s1 3 p3 3 d 10
85. Which of the following explains the sequence of filling
(d) 1 s2 2 s2 2p6 3 s2 3 p6 3 d 4 4 s2
electrons in different shells ?
99. The paramagnetic character follows the order
(a) Hund’s rule (b) Octet rule
(a) Mn > Cr > Zn (b) Fe > Zn > Co
(c) Aufbau principle (d) All of these.
(c) Cr > Fe > Zn (d) Hg > Mn > Fe.
86. Which of the following has more unpaired d-electrons?
100. If the nitrogen atom had electronic configuration 1s7 it would
(a) Zn+ (b) Fe2+ (c) Ni3+ (d) Cu+
have energy lower than that of the normal ground state
87. Which of the following configurations is correct for iron?
configuration 1s2 2s2 2p3 because the electrons would be
(a) 1s2 2s2 2p6 3s2 3p6 3d5
closer to the nucleus. Yet 1s7 is not observed. It violates
(b) 1s2 2s2 2p6 3s2 3p6 4s2 3 d5
(a) Heisenberg’s uncertainty principle
(c) 1s2 2s2 2p6 3s2 3p6 4s2 3 d7
(b) Hund’s rule
(d) 1s2 2s2 2p6 3s2 3p6 4 s2 3d6
(c) Pauli exclusion principle
88. The maximum number of electrons that can be accomodated
(d) Bohr postulate of stationary orbits
in d-sub-shell is
101. Which of the following elements outermost orbit’s last
(a) 10 (b) 8 (c) 6 (d) 4
electron has magnetic quantum number m = 0?
89. The number of unpaired electrons in an atom of atomic
(a) Na (b) O
number 24 (in ground state) is
(c) Cl (d) N
(a) 3 (b) 4 (c) 5 (d) 6
Structure of Atom 45

1. If uncertainty in position and momentum are equal, then 11. The value of Planck’s constant is 6.63 × 10–34 Js. The speed
uncertainty in velocity is : [CBSE-PMT 2008] of light is 3 × 1017 nm s–1.. Which value is closest to the
wavelength in nanometer of a quantum of light with frequency
1 h h 1 h h of 6 × 1015 s–1? [NEET 2013]
(a) (b) (c) (d)
2m p 2p m p p (a) 25 (b) 50 (c) 75 (d) 10
2. The measurement of the electron position is associated with 12. What is the maximum numbers of electrons that can be
an uncertainty in momentum, which is equal to 1×10–18 g cm s–1. associated with the following set of quantum numbers?
The uncertainty in electron velocity is, [CBSE-PMT 2008] n = 3, l = 1 and m = –1 [NEET 2013]
(mass of an electron is 9 × 10– 28 g) (a) 6 (b) 4 (c) 2 (d) 10
(a) 1 × 109 cm s–1 (b) 1 × 106 cm s–1 æ Z2 ö
5 –1 (d) 1 × 1011 cm s–1
(c) 1 × 10 cm s 13. Based on equation E = – 2.178 × J çç 2 ÷÷ , certain
10-18
3. The energy absorbed by each molecule (A2) of a substance èn ø
is 4.4 × 10–19 J and bond energy per molecule is 4.0 × 10–19 J. conclusions are written. Which of them is not correct ?
The kinetic energy of the molecule per atom will be: [NEET 2013]
[CBSE-PMT 2009] (a) Larger the value of n, the larger is the orbit radius.
(a) 2.2 × 10–19 J (b) 2.0 × 10–19 J (b) Equation can be used to calculate the change in energy
(c) 4.0 × 10–20 J (d) 2.0 × 10–20 J when the electron changes orbit.
4. Maximum number of electrons in a subshell of an atom is (c) For n = 1, the electron has a more negative energy than it
determined by the following: [CBSE-PMT 2009] does for n = 6 which mean that the electron is more loosely
(a) 2 l + 1 (b) 4 l – 2 (c) 2 n2 (d) 4 l + 2 bound in the smallest allowed orbit.
5. Which of the following is not permissible arrangement of (d) The negative sign in equation simply means that the
electrons in an atom? [CBSE-PMT 2009] energy or electron bound to the nucleus is lower than it
(a) n = 5, l = 3, m = 0, s = + 1/2 would be if the electrons were at the infinite distance
(b) n = 3, l = 2, m = – 3, s = – 1/2 from the nucleus.
(c) n = 3, l = 2, m = – 2, s = – 1/2 14. In a hydrogen atom, if energy of an electron in ground state is
(d) n = 4, l = 0, m = 0, s = – 1/2 13.6. eV, then that in the 2nd excited state is [AIEEE 2002]
6. A 0.66 kg ball is moving with a speed of 100 m/s. The associated (a) 1.51 eV (b) 3.4 eV (c) 6.04 eV (d) 13.6 eV.
wavelength will be ( h = 6.6 ´ 10 -34 Js) : [CBSE-PMT 2010] 15. Uncertainty in position of a minute particle of mass 25 g in
(a) 1.0 ´ 10–32m (b) 6.6 ´ 10–32m space is 10–5 m. What is the uncertainty in its velocity (in
(c) 6.6 ´ 10 m–34 (d) 1.0 ´ 10–35m ms–1)? (h = 6.6 ´ 10–34 Js) [AIEEE 2002]
7. The total number of atomic orbitals in fourth energy level of (a) 2.1 ´ 10 –34 (b) 0.5 ´ 10–34

an atom is : [CBSE-PMT 2011] (c) 2.1 ´ 10–28 (d) 0.5 ´ 10–23.


(a) 8 (b) 16 (c) 32 (d) 4 16. The number of d-electrons retained in Fe2+ [AIEEE 2003]
8. The energies E1 and E2 of two radiations are 25 eV and 50 eV, (At. no. of Fe = 26) ion is
respectively. The relation between their wavelengths i.e., l1
(a) 4 (b) 5 (c) 6 (d) 3
and l2 will be : [CBSE-PMT 2011]
(a) l1 = l2 (b) l1 = 2l2 17. The orbital angular momentum for an electron revolving in an
1 h
(c) l1 = 4l2 (d) 1 2 orbit is given by l (l + 1) . . This momentum for an s-
2 2p
9. If n = 6, the correct sequence for filling of electrons will be :
electron will be given by [AIEEE 2003]
(a) ns ® (n – 2) f ® (n – 1) d ® np [CBSE-PMT 2011]
(b) ns ® (n – 1) d ® (n – 2) f ® np h
(a) zero (b) h (d) + 1 . h
(c)
(c) ns ® (n – 2) f ® np ® (n – 1) d 2p 2. 2 2p
2p
(d) ns ® np ® (n – 1) d ® (n – 2) f
18. A reduction in atomic size with increase in atomic number is a
10. According to the Bohr Theory, which of the following
characteristic of elements of [AIEEE 2003]
transitions in the hydrogen atom will give rise to the least
energetic photon ? [CBSE-PMT 2011 M] (a) d-block (b) f-block
(a) n = 6 to n = 1 (b) n = 5 to n = 4 (c) radioactive series (d) high atomic masses
(c) n = 6 to n = 5 (d) n = 5 to n = 3
46 Chemistry
19. Which one of the following groupings represents a collection of 28. Of the following sets which one does NOT contain
isoelectronic species ? [AIEEE 2003] isoelectronic species? [AIEEE 2005]
(At. nos. : Cs : 55, Br : 35)
(a) BO 33 - , CO 32 - , NO 3- (b) SO 32 - , CO 32 - , NO 3-
(a) N3–, F–, Na+ (b) Be, Al3+, Cl–
2+ +
(c) Ca , Cs , Br (d) Na+, Ca2+, Mg2+ (c) CN - , N 2 , C 22 - (d) PO 34 - , SO 24 - , ClO -4
20. In Bohr series of lines of hydrogen spectrum, the third line
29. According to Bohr's theory, the angular momentum of an
from the red end corresponds to which one of the following
electron in 5th orbit is [AIEEE 2006]
inter-orbit jumps of the electron for Bohr orbits in an atom of
hydrogen [AIEEE 2003] (a) 10 h / p (b) 2.5 h / p
(a) 5 ® 2 (b) 4 ® 1 (c) 2 ® 5 (d) 3 ® 2 (c) 25 h / p (d) 1.0 h / p
21. The de Broglie wavelength of a tennis ball of mass 60 g moving 30. Uncertainty in the position of an electron (mass = 9.1 ×
with a velocity of 10 metres per second is approximately 10–31 kg) moving with a velocity 300 ms–1, accurate upto
Planck’s constant, h = 6.63 × 10–34 Js [AIEEE 2003] 0.001% will be (h = 6.63 × 10–34 Js) [AIEEE 2006]
–31
(a) 10 metres –16
(b) 10 metres (a) 1.92 × 10–2 m (b) 3.84 × 10–2 m
(c) 10–25 metres (d) 10–33 metres (c) 19.2 × 10–2 m (d) 5.76 × 10–2 m
22. Which of the following sets of quantum numbers is correct 31. Which one of the following sets of ions represents a collection
for an electron in 4f orbital ? [AIEEE 2004] of isoelectronic species? [AIEEE 2006]
(a) n = 4, l = 3, m = + 1, s = + ½
(a) N3–, O2–, F–, S2– (b) Li+, Na+, Mg2+, Ca2+
(b) n = 4, l = 4, m = – 4, s = – ½
(c) n = 4, l = 3, m = + 4, s = + ½ (c) K+, Cl–, Ca2+, Sc3+ (d) Ba2+, Sr2+, K+, Ca2+
(c) n = 3, l = 2, m = – 2, s = + ½ 32. Which of the following sets of quantum numbers represents
23. Consider the ground state of Cr atom (X = 24). The number of the highest energy of an atom? [AIEEE 2007]
electrons with the azimuthal quantum numbers, l = 1 and 2 (a) n = 3, l = 0, m = 0, s = +1/2
are, respectively [AIEEE 2004] (b) n = 3, l = 1, m = 1, s = +1/2
(a) 16 and 4 (b) 12 and 5 (c) 12 and 4 (d) 16 and 5
(c) n = 3, l = 2, m = 1, s = +1/2
24. The wavelength of the radiation emitted, when in a hydrogen
atom electron falls from infinity to stationary state 1, would be (d) n = 4, l = 0, m = 0, s = +1/2.
(Rydberg constant = 1.097×107 m–1) [AIEEE 2004] 33. The ionization enthalpy of hydrogen atom is
(a) 406 nm (b) 192 nm 1.312 × 106 J mol–1. The energy required to excite the electron
(c) 91 nm (d) 9.1×10–8 nm in the atom from n = 1 to n = 2 is [AIEEE 2008]
25. Which one of the following sets of ions represents the (a) 9.84 × 105 J mol–1 (b) 6.56 × 105 J mol–1
collection of isoelectronic species? [AIEEE 2004] (c) 7.56 × 105 J mol–1 (d) 8.51 × 105 J mol–1
(a) K+, Cl–, Mg2+, Sc3+ (b) Na+, Ca2+, Sc3+, F– 34. Which one of the following constitutes a group of the
(c) K+, Ca2+, Sc3+, Cl– (d) Na+, Mg2+, Al3+, Cl– isoelectronic species? [AIEEE 2008]
(Atomic nos. : F = 9, Cl = 17, Na = 11, Mg = 12, Al = 13, K = 19, (a) C2– –
2 , O2 , CO, NO (b) CN – , N 2 ,O 2– 2–
2 , C2
Ca = 20, Sc = 21)
(c) NO+ , C2– –
2 , CN , N 2 (d) N 2 , O 2– , NO + , CO
26. Of the following outer electronic configurations of atoms, the
highest oxidation state is achieved by which one of them ? 35. Calculate the wavelength (in nanometer) associated with a
[AIEEE 2004] proton moving at 1.0 × 103 ms –1. [AIEEE 2009]
3
(a) (n – 1)d ns 2 (b) (n – 1)d ns1
5 (Mass of proton = 1.67 × 10–27 kg and h = 6.63 × 10–34 Js)
8
(c) (n – 1)d ns 2 (d) (n – 1)d5 ns2 (a) 0.40 nm (b) 2.5 nm (c) 14.0 nm (d) 0.32 nm
27. In a multi-electron atom, which of the following orbitals 36. In an atom, an electron is moving with a speed of 600 m/s with
described by the three quantum members will have the same an accuracy of 0.005%. Certainity with which the position of
energy in the absence of magnetic and electr ic the electron can be located is ( h = 6.6 × 10–34 kg m2s–1, mass
fields ? [AIEEE 2005] of electron, em = 9.1 × 10–31 kg) [AIEEE 2009]
(a) 5.10 × 10 m–3 (b) 1.92 × 10 –3 m
(A) n = 1, l = 0, m = 0 (B) n = 2, l = 0, m = 0
(c) 3.84 × 10 m –3 (d) 1.52 × 10 –4 m
(C) n = 2, l = 1, m = 1 (D) n = 3, l = 2, m = 1
(E) n = 3, l = 2, m = 0 37. The energy required to break one mole of Cl – Cl bonds in Cl 2
Options is 242 kJ mol–1. The longest wavelength of light capable of
breaking a single Cl – Cl bond is [AIEEE 2010]
(a) (D) and (E) (b) (C) and (D)
(c = 3 × 108 ms–1 and NA = 6.02 × 1023 mol–1).
(c) (B) and (C) (d) (A) and (B)
(a) 594 nm (b) 640 nm (c) 700 nm (d) 494 nm
Structure of Atom 47
38. Ionisation energy of He+ is 19.6 × 10–18 J atom–1. The energy 41. The increasing order of the ionic radii of the given
of the first stationary state (n = 1) of Li2+ is [AIEEE 2010] isoelectronic species is : [AIEEE 2012]
(a) Cl–, Ca2+ , K+, S2– (b) S2–, Cl–, Ca2+ , K+
(a) 4.41 × 10–16 J atom–1 (b) –4.41 × 10–17 J atom–1
(c) Ca2+ , K+, Cl–, S2– (d) K+, S2–, Ca2+, Cl–
(c) –2.2 × 10–15 J atom–1 (d) 8.82 × 10–17 J atom–1
18 Z2
39. The frequency of light emitted for the transition n = 4 to n = 2 42. Energy of an electron is given by E = – 2.178 × 10 J
÷.
n2
of the He+ is equal to the transition in H atom corresponding Wavelength of light required to excite an electron in an
to which of the following ? [AIEEE 2011 RS] hydrogen atom from level n = 1 to n = 2 will be :
(a) n = 2 to n = 1 (b) n = 3 to n = 2 (h = 6.62 × 10 –34 Js and c = 3.0 × 108 ms–1)
[JEE Main 2013]
(c) n = 4 to n = 3 (d) n = 3 to n = 1 (a) 1.214 × 10–7 m (b) 2.816 × 10.–7 m
40. The electrons identified by quantum numbers n and l : (c) 6.500 × 10–7 m (d) 8.500 × 10–7 m
43. The kinetic energy of an electron in the second Bohr orbit of
(A) n = 4, l = 1 (B) n = 4, l = 0 a hydrogen atom is [a0 is Bohr radius] : [IIT-JEE 2009 S]
(C) n = 3, l = 2 (D) n = 3, l = 1 h2 h2 h2 h2
can be placed in order of increasing energy as :[AIEEE 2012] (a) (b) (c) (d)
4 2 ma02 16 2 ma02 32 2 ma02 64 2 ma02
(a) (C) < (D) < (B) < (A) (b) (D) < (B) < (C) < (A) 44. Given that the abundances of isotopes 54Fe, 56Fe and 57 Fe
(c) (B) < (D) < (A) < (C) (d) (A) < (C) < (B) < (D) are 5%, 90% and 5%, respectively, the atomic mass of Fe is
[IIT-JEE 2012]
(a) 55.85 (b) 55.95 (c) 55.75 (d) 56.05

1. Which statement is not correct ? (c) electrons come out of metal with a constant velocity
which depends on frequency and intesity of incident
æ 4 3ö -38 light.
(a) Volume of proton is approx. ç pr ÷1.5 ´ 10 cm3.
è3 ø (d) electrons come out of metal with different velocities
not greater than a certain value which depends upon
(b) Radius of e– is 42.8 × 10–13 cm3
frequency of incident light and not on intensity.
(c) Density of nucleus is 1014 g/cm3. 5. If E1, E2 and E3 represent respectively the kinetic energies of
(d) All are correct an electron and an alpha particle and a proton each having
2. The potential energy of electron present in ground state of same de-broglie wavelength then
Li2+ ion is represented by : (a) E1 > E3 > E2 (b) E2 > E3 > E1
+ 3e 2 -3e (c) E1 > E2 > E3 (d) E1 = E2 = E3
(a)
4p 0 r
(b) 4p 0 r 6. The angular speed of the electron in n th orbit of Bohr
hydrogen atom is
- 3e 2 - 3e 2 (a) directly proportional to n
(c) (d)
4p 0 r 2 4p 0 r (b) inversely proportional of n
3. Which principle/rule limits the maximum no. of electrons in (c) inversely proportional to n 2
an orbital to two ? (d) inversely proportional to n 3
(a) Aufbau principle 7. The Bohr's energy equation for H atom reveals that the energy
(b) Pauli's exclusion principle level of a shell is given by E = –13.58/n2eV. The smallest
(c) Hund's rule of max. multiplicity amount that an H atom will absorb if in ground state is
(d) Heisenberg's uncertainty principle (a) 1.0 eV (b) 3.39 eV
4. Photoelectric effect is the phenomenon in which (c) 6.79 eV (d) 10.19 eV
(a) photons come out of metal when hit by a beam of 8. If the Planck's constant h = 6.6 × 10–34 Js, the de-Broglie's
wavelength of a particle having momentum of 3.3 × 10–24
electrons
m/s will be
(b) photons come out of the nucleus of an atom under the (a) 0.02Å (b) 0.5Å
action of an electric field (c) 2Å (d) 500Å
48 Chemistry
9. Ionization potential of hydrogen atom is 13.6eV. Hydrogen 19. The angular distribution functions of all orbitals have
atom in ground state are excited by monochromatic light of (a) l nodal surfaces (b) l – 1 nodal surfaces
energy 12.1 eV. The spectral lines emitted by hydrogen (c) n + 1 nodal surfaces (d) n – l –1 nodal surfaces
according to Bohr's theory will be 20. Which of the following radial distribution graphs correspond
(a) one (b) two to l = 2 for the H atom ?
(c) three (d) four
10. The wavelength of radiations emitted when electrons
falls from 4 th Bohr's orbit to 2 nd in H atom is : (RH =
(a)
1.09678 × 10–7 m –1).
(a) 972 nm (b) 486 nm
(c) 243 nm (d) 182 nm
11. The energy of e– in first orbit of He+ is –871.6 × 10–20 J. The
energy of e– in first orbit of H is:
(a) – 871.6 × 10–20 (b) –435.8 × 10–20 J
–20 (b)
(c) – 217.9 × 10 J (d) –108.9 × 10–20
12. The wave no. of radiation of wavelength 500 nm is :
(a) 5 × 10–7 /m (b) 2 × 107/m
6
(c) 2 × 10 /m (d) 500 × 10–9/m
+
13. In ground state of Cu . The no. of shells occupied, subshells,
filled orbitals, and unpaired electrons respectively are :
(a) 4, 8, 15, 0 (b) 3, 6, 15, 1 (c)
(c) 3, 6, 14, 0 (d) 4, 7, 14, 2
14. The quantum numbers +1/2 and –1/2 for the electron spin
represent
(a) rotation of the electron in clockwise and anticlockwise
direction respectively
(b) rotation of the electron in anticlockwise and clockwise
direction respectively (d)
(c) magnetic moment of the electron pointing up and down
respectively
(d) two quantum mechanical spin states which have no
classical analogue
15. Let mp be the mass of a proton, mn that of a neutron, M1 that
of a 20 nucleus and M2 that of a 40 nucleus. Then 21. Which of the following graphs correspond to
10 Ne 20 Ca one node?
(a) M2 = 2M1 (b) M1<10(mp + mn)
(c) M2 > 2M1 (d) M1 = M2
16. Which of the following pairs have identical values of e/m?
(a) A proton and a neutron (a) (b)
(b) A proton and deuterium
(c) Deuterium and an a- particle
(d) An electron and g-rays
ao
17. The wavelength of the third line of the Balmer series for a
hydrogen atom is

21 100
(a) (b) (c) (d)
100R ¥ 21R ¥

21R ¥ 100R ¥ ao
(c) (d) ao
100 21
22. A body of mass 10 mg is moving with a velocity of
18. Which of the following electronic configurations have zero
-1
spin multiplicity ? 100 ms . The wavelength of de-Broglie's wave associated
with it would be
(a) (b)
(a) 6.63 × 10 -35 m (b) 6.63 × 10 -31 m
(c) (d) (c) 6.63 × 10 -37 m (d) 6.63 × 10 -34 m
Structure of Atom 49

23. If N 0 represents the Avogadro number, then which of the 31. The ratio of magnetic moments of Fe(III) and Co(II) is
(a) 7 : 3 (b) 3 : 7
following represent correct value of one atomic mass unit.
(c) 7 : 3 (d) 3 : 7
(a) N 0 ´ 10 - 3 kg (b) N0 g
32. The average life of an excited state of hydrogen atom is of
-1 1
(c) N 0 g (d)
16
Mass of 0-16 atom the order 10 -8 s. The number of revolutions made by an
electron when it is in state n = 2 and before it suffers a
24. If the shortest wavelength of the spectral line of H-atom in
transition to state n = 9 are
the Lyman series is X, then the longest wavelength of the
(a) 8.23 × 10 6 (b) 2.82 × 10 6
line in Balmer series of Li 2+ is
x (c) 22.8 × 10 6 (d) 2.28 × 10 6
(a) 9x (b) 33. The number of concentric spherical surfaces for 3s orbital at
9
which the probability of finding electrons is zero, are
5x 4x (a) 3 (b) 2
(c) (d)
4 5 (c) 1 (d) 0
25. If the subsidiary quantum number of a sub-energy level is 4, 34. If the second ionization potential of helium is 54.4 eV, then
the maximum and minimum values of the spin multiplicities
possible energy states of He + ion is/are
are
(a) 10, 2 (b) 4, – 4 (a) 13.6 eV (b) – 13.6 eV
(c) 10, 1 (d) 9, 1 (c) + 3.4 eV (d) None of these
35. If the uncertainties in position and momentum are equal, the
26. Li 3+ and a proton are accelerated by the same potential, uncertainty in the velocity is
then de-Broglie wavelengths l Li and l p have the ratio
1 h h
(a) (b)
(assume m Li = 9 mp ) 2m p 2p

(a) 1: 3 3 (b) 1 : 1 h
(c) 1 : 2 (d) 1 : 4 (c) (d) None of these
p
27. One Bohr magnet on is equal to
36. The de-Broglie wavelength of an electron accelerated by an
c h electric field of V volts is given by
(a) (b)
4phm e 4pem e
1.23 1.23
(a) l= nm (b) l= nm
eh hc V h
(c) (d)
4 me 4 me
1.23 1.23
(c) l= nm (d) l= nm
-1 m
28. The dissociation energy of H 2 is 430.53 KJmol . If V
hydrogen is dissociated by illumination with radiation of 37. The photoelectric current decreases if
wavelength 253.7 nm the fraction of the radiant energy which (a) the intensity of the source of light is decreased
will be converted into kinetic energy is given by (b) the frequency of incident radiation decreases below
(a) 100% (b) 8.76% threshold frequency
(c) 2.22% (d) 1.22%
(c) the exposure time decreases
29. The magnetic moment of M x + (atomic number M = 25) is (d) None of these
38. If a proton and a -particle are accelerated through the same
15 BM. The number of unpaired elections and the value
of x respectively are potential difference, the ratio of de-Broglie wavelengths l p
(a) 5, 2 (b) 3, 2 and l a is
(c) 3, 4 (d) 4, 3
30. The threshold frequency of a metal is 1 × 1015 s -1 . The ratio (a) 3 (b) 2 2
of the maximum kinetic energies of the photoelectrons when (c) 1 (d) 2
the metal is irradiated with radiations of frequencies 1.5 × 39. Which of the following pairs of nucleides are isodiaphers ?
1015 s -1 and 2.0 × 1015 s -1 respectively would be (a) 13
6 C and 16
8 O
(b) 1
1H and 12 H
(a) 4 : 3 (b) 1 : 2
(c) 3 and 42 He (d) 55 and 65
(c) 2 : 1 (d) 3 : 4 1H 25 Mn 30 Zn
50 Chemistry
40. A system irradiated for 10 min. is found to absorb 39
42. The species 19 K and 19
9 F are called
18
3 × 10 quanta per sec. If the amount of substance (a) isosters (b) isobars
decomposed is 3 × 10 -3 mol ( N A = 6 × 10 23 ). The quantum (c) isotones (d) isodiaphers
43. A particle of mass 1 micro gram is moving with a speed of
efficiency of the reaction is
1 Kms–1. The de-Broglie wavelength of the particle is
(a) 2.5 (b) 2
(c) 1.5 (d) 1 (a) 7.04 × 10 -25 m (b) 6.626 × 10 -28 m
41. If the radius of first orbit of H-atom is a 0 , then de-Broglie (c) 6.626 × 10 -15 m (d) 6.626 × 10 -31 m
wavelength of electron in 4th orbit is
a0
(a) 8pa 0 (b)
4
(c) 16a 0 (d) 2pa 0
Structure of Atom 51


EXERCISE 1 14. (c) Hydride ion H contains two electrons and He contains
238 234
two electrons.
1. Isotopes = U, U
92 92
15. (c)
Isobars = 234
90 Th, 234
91 Pa, 234
92 U 16. (c) The species CO, NO+, CN– and C22–. Contain 14 electrons
each.
2. First energy level.
3. s-orbital 17. (d) Na+, Mg++, O2– contain 10 electrons each Cl– has 18
electrons.
4. Interference and diffraction
18. (b) Nuclear charge of K+ (19) is more than nuclear charge of
5. There is no probability of finding a d electron right at the
Cl– (17).
nucleus.
6. Electron, because of its least mass. hc E1 2 6000
19. (b) E = ; = = = 2 :1
8. K.E. = 7.29 × 10–19J E2 1 3000

9. l = 1.24 × 10–14 m
hc
10. l = 6572 Å 20. (b) DE = hn = ;
l
13. v0= 9.81×1014 Hz.
19. (b) 20. (a) 21. (c) 22. (c) hc 6.63 ´ 10 -34 (3 ´ 108 )
\l = = = 656 nm
23. (a) 24. (b) 25. (b) 26. (d) DE 3.03 ´ 10 -19
27. (c) 28. (c)
h 6.63 ´10-34
EXERCISE 2 21. (a) p = = = 3 ´10-23 kg / s
l 2.2 ´10 -11
1. (c) A neutral atom with Z > 1 can have neutron, proton and
electron. 22. (a) It is in the lowest energy level, it can absorb and not emit
2. (d) Atomic orbitals are 4s, 3s, 3p and 3d. (n + l) values being energy.
4, 3, 4 and 5. Hence 3d has highest energy. 23. (c) See text for Brackett n 1 = 4, n2 = 5, 6, 7 etc.
3. (a) 19 + 1e– = 20 electrons.
æ 1 1 ö 5R
4. (d) Threshold frequency is not the same for all metals. 24. (d) For Balmer n1 = 2 and n2 = 3; n = R çç - ÷÷ = cm -1
2 2 36
è2 3 ø
5. (c) incorrect. (see text).
6. (a) All positive ions are deposited at small part. (nucleus of 25. (a) 350-700 nm lies in visible region hence Balmer Series.
atom). 26. (a) For Balmer n 1 = 2 and n2 = 3, 4, 5. For third line
7. (b) Ratio of mass of proton and electron is approx. n1 = 2 & n2 = 5.
1837 = 1.8 × 103. 27. (d) Li+ and He both have two electrons each.
8. (a) Neutron being chargless particle could not effect certain 13.6 ´ 1
13.6 Z 2 = +3.4 eV .
properties. 28. (a) KE = eV when n = 2. KE =
22
2
40
n
9. (c) 18Ar contains 22 neutrons and 21Sc40 contains 19
neutrons. The number of neutrons = (A – Z)
10. (d) Rutherford used doubly charged helium particle. 13.6Z2 13.6Z 2
29. (a) KE = eV and Total energy = - eV .
(a - particle) n2 n2
11. (b) Number of p = number of e – = 89 and neutrons Hence ratio = –1.
231 – 89 = 142. 30. (c)
12. (a) Z = 11, A = 24. Hence protons = 11 the neutrons
(24 – 11) = 13. nh
31. (c) Angular momentum mvr =
13. (d) For neutral atom . No. of p = No. of e– = 18 and 2p
A = Z + No. of neutrons = 18 + 20 = 38.
52 Chemistry

32. (b) Orbital angular momentum = l(l + 1)


h
. For s the value - 13.6 Z 2
47. (b) E n = ; For He = –13.6 × (2)2 = –54.4 eV..
2p n2
of l = 0. Hence orbital angular momentum = 0. 48. (d) Sommerfield.

49. (c) l = h ; 1 hence answer (c).


2
33. (a) Total energy = – 13.6 Z eV. Put n = 2, 3, 4 and get the \l µ
n 2 mv v
value for subsequent orbits.
h 6.6 ´ 10 -34
50. (c) l = = = 2 ´ 10 -10 m = 2 Å
a0 p 3.3 ´ 10-24
34. (c) Radius of nth orbit rn = ´ n 2 ; \ r3 = 9a 0
Z
h 6.63 ´ 10 -34
35. (a) Angular momentum is quantised. 51. (b) l = = = 0.727 ´ 10-9 m = 0.727 nm
mv 9.11´ 10-31 ´ 106
36. (c) The difference between the energy of adjacent energy
levels decreases on moving away from nucleus. ´ -34
52. (c) l = h = 6.63 10 = 2.4 ´ 10 -30 m
1 mv 5
37. (c) Wave number u = = 0.5 ´ 10 - 6 m = 500nm ( wave 0.200 ´
6 3600
2 ´ 10
h h
1 53. (c) Dx . Dp = or Dx . mDv = ;
number = ) 4p 4p
0.011
38. (c) He2+ does not contain any electron. Bohr's theory is Dv = ´ 3 ´ 10 4 = 3.3 cms -1
100
applicalbe to species containing one electron.
6.6 ´ 10 - 27
39. (b) En µ Z2 increases rapidly.. Dx =
4 ´ 3.14 ´ 9.1 ´ 10 - 28 ´ 3.3
= 0.175 cm

40. (d) Ionisation energy = E ¥ - E1 h 6.63 ´ 10 -34


54. (c) Dx = = = 5.27 ´ 10 -30 m
For H = 0 – (–13.6). For Be3+ = 0 – (–13.6 × 42). 4p ´ Dp 4 ´ 3.14 ´10 -5
Therefore Ratio is 1 : 16.
55. (d) Azimuthal Quantum number represent quantised values
-13.6
41. (d) E1 = ; E3 - E1 = -1.5 - (-13.6); = 12.1 eV of angular momentum L = l (l + 1) .
h
.
1 2p
= 12.1´ 1.6 ´10-12 ergs 56. (d) For l = 2 (d subshell) n ³ 3 .
57. (b) 5B = 1s2, 2s2, 2px1py0pz0; n = 2, l = 1, m = –1
= 0.1936 ´10 -10 ergs (1 ev = 1.6 × 10–12 ergs). 58. (c) Number of orbitals with principal quantum number
n = n2.
13.6 -13.6
42. (a) E3 = - eV = – 1.5 eV; E2 = eV = -3.4 eV 59. (b) Follow text.
9 4
60. (c) For 4p electron n = 4, l = 1, m = –1, 0 + 1 and s = +½ or –½
E 3 - E 2 = [ - 1.5 - ( - 3.4)] = 1.9 eV = 3.04 ´ 10 - 19 J / atom 61. (d) l = 2 for d subshell hence 10 electrons.
62. (c) When n = 3 we have s,p and d sub-shells.
-13.6 `Hence subshells = 3 and obitals 1 + 3 + 5 = 9.
43. (b) E 2 = = -3.40 eV
(2)2 63. (d) When m = – 3, l = 3, \ n = 4 .
64. (b) Valence electron of K is 4s1. \ n = 4, l = 0, m = 0, s = ½
a 0 ´ n 2 0.53Å ´ n 2 65. (a) (n + l) rule the higher the value of (n + l), the higher is the
44. (a) rn = = ;
Z Z energy. When (n + l) value is the same see
0.53Å ´ (1) 2
value of n.
For Li++ = = 0.17 Å
3 I II III IV
(n + l) (4 + 1) (4 + 0) (3 + 2) (3 + 1)
0.53 ´ ( 2) 2
45. (d) First excited state means n = 2. \ r2 = = 2.12Å 5 4 5 4
1
\ IV < II < III < I
2
2 a 0 (1) 66. (d) According to Bohr's model electrons follow definite path.
46. (b) rn = a 0 n , r1 = for hydrogen
Z 1 67. (c) Energy of subshells follow the order 1s2, 2s2, 2p6, 3s2,
2 3p6, 4s2, 3d10, 4p6, 5s, 4d, 5p, 6s..... we can fill them by
a (1) r1 1
electrons upto 4p hence 36 electrons.
r2 = 0 for deuterium \ = =1:1
1 r2 1
Structure of Atom 53
68. (b) Total number iof orbitals for principal quantum number n 96. (b)
is n2 5p 4f 6s 5d
69. (c) For l = 0, m cannot be –1. (n + l) 5+1 4+3 6+0 5+2
70. (a) m = +2 signifies only one orbital. 6 7 6 7
71. (d) For 4d electron we have n = 4, l = 2, m = –2, –1, 0, Hence the order is 5p < 6s < 4f < 5d
+1, +2 & s = ½ or –½.
97. (d) Fe2+ 1s2, 2s2p6, 3s2p6d6 hence 6 electrons.
72. (b) For f orbital, the values of m are –3, –2, –1, 0, +1,
98. (a) Pauli's exclusion principle.
+2, +3.
99. (c) Write electronic configuration and find unpaired electrons.
ch c Cr (6), Fe (4), Mn (5), Cu (3), Zn (0), Hg (0).
73. (b) E = hu = ;and u =
l l
The more the number of unpaired electrons the more is
the paramagnetic character.
3.0 ´108
8 ´1015 = 100.(c) Not more than two electrons can be present in same atomic
l orbital. This is Paulis exclusion principle.
101.(a) 11Na = 1s2, 2s2p6, 3s1 for 3s1, l = 0. Hence m = 0.
3.0 ´108
\ l= = 0.37 ´ 10-7 = 37.5 ´ 10 -9 m = 4 ´ 101 nm
8 ´ 1015
EXERCISE 3

74. (b)
Number of nodal planes in d orbitals is 2. h
1. (a) We know Dp.Dx ³
4p
75. (a)
Degenerate orbitals have equal energy.
since Dp = Dx (given)
76. (a)
One nodal plane in the YZ plane.
h
77. (a)
Angular nodes = l, spherical nodes (n – l – 1); \ Dp.Dp =
4p
Total (n – 1). Hence spherical nodes for 5f orbits.
= ( 5 –3 –1) = 1 h
or mDv m v. = [\ Dp= mDv]
78. (d) (follow text.) 4
2 h
or ( Dv ) =
79. (d) Mg 2 + = 1s 2 , 2s 2 p 6 , Ti 3+ = 1s 2 2s 2 p 6 3s 2 p 6 d 1 , 4 pm 2
V 3+ = 1s 2 , 2s 2 p 6 , 3s 2 p 6d 2 , Fe2+ = 1s2, 2s 2p6, 3s 2p6d 6 h 1 h
or Dv = =
(4 unpaired electrons in d). 4pm 2m p
2

80. (c) Atomic number 9 is for F and ion is F–. Thus option (a) is the correct option.
81. (c) 1s2, 2s22p5, 3s1. Total electrons (10) excited state of Ne. 2. (a) Dp = mDv
Substituting the given values of Dx and m, we get
82. (d) Heisenberg's uncertainty Principle is applicable to any 1×10–18 g cm s–1 = 9×10–28 g × Dv
moving object.
1´ 10 -18
83. (d) Configuration contains 2 unpaired electrons hence most or Dv =
paramagnetic. 9 ´ 10-28
= 1.1 × 109 cm s–1 ; 1×109 cm s–1
84. (d) For Cu2+ electronic configuration is [Ar]3d9.
i.e. option (a) is correct.
85. (c) Aufbau Principle. 3. (d) K.E per atom

(4.4 ×10 ) – (4.0 ×10 )


86. (b) Zn+ (1 unpaired electron); Fe++ (4); Ni3+ (3); Cu+ (0). –19 –19
87. (d) is correct. =
88. (a) d subshell can accomodate 10 electrons. 2
89. (d) Atomic number 24 is for Cr 1s2, 2s2p6, 3s2p6d5, 4s1. Ans 0.4×10 –19
(d). = 2.0 10 –20
2
90. (a) Ni (28) = 1s 2 , 2s 2 p 6 , 3s 2 p 6d 8 , 4s 2 . Unpaired 4. (d) The number of sub shell is (2 l + 1). The maximum number
electrons 2. of electrons in the sub shell is 2 (2 l + 1) = (4 l + 2)
91. (d) Cu. Ans (d). 5. (b) m = – l to +l, through zero thus for l = 2, values of m will
92. (b) Write electronic configuration of Cu. be – 2, –1, 0, + 1, + 2.
Therefore for l = 2, m cannot have the value –3.
93. (c) In protium 1H1. No neutron.
94. (b) M-shell means 3 principle energy level the element is Cr. h 6.6 ´10-34
6. (d) l = = = 1´ 10-35 m
95. (b) Fe3+ and Mn2+ write and check the ans (b). mv 0.66 ´100
54 Chemistry
7. (b) Total no. of atomic orbitals in a shell = n2. 23. (b) Electronic configuration of Cr atom (z = 24)
Given n = 4; Hence number of atomic orbitals in 4th shell
will be 16. = 1s 2 ,2s 2 p 6 , 3s 2 p 6 d 5 ,4s1
8. (b) Given E1 = 25eV E2 = 50 eV
when l = 1, p - subshell,
hc hc E1 2
E1 = E2 = = Numbers of electrons = 12
E2
1 2 1 when l = 2, d - subshell,
2 = 25 = 1 Numbers of electrons = 5
1 2 2
1 50 2
9. (a) ns ® (n - 2) f ® (n - 1)d ® np [n = 6] 1 æ 1 1 ö÷
ç
10. (c) Energy of photon obtained from the transition n = 6 to 24. (c) l = R ç 2 - 2 ÷
è n1 n 2 ø
n = 5 will have least energy.
æ 1 1 ö 1 æ1 1 ö
DE = 13.6Z 2 - = 1.097 ´ 10 7 ç - ÷ = 1.097 ´ 10 7
çè n 2 n 2 ÷ø l è1 ¥ ø
1 2
11. (c) c = nl
l = 91.15 ´ 10 -9 m » 91nm
17
c 3 ´10
+
l= = = 50 nm 25. (c) 19 K, 20 Ca 2 + , 21 Sc 3+ ,17 Cl - each contains 18
6 ´ 1015
electrons
12. (c) n = 3 for 3rd shell
26. (d) (n–1)d5ns2 attains the maximum O.S. of + 7
l = 1 for p sub shell.
27. (a) The energy of an orbital is given by (n + l) in (d) and (c).
m = – 1 is possible for two electrons present in an (n + l) value is (3 + 2) = 5 hence they will have same
orbital. energy.
13. (c) Energy of an electron at infinite distance from the
nucleus is zero. As an electron approaches the nucleus, 28. (b) 1. BO33 5 8 3 + 3 = 32
the electron attraction increases and hence the energy 2
CO3 6 8 3 + 2 = 32 iso electronic
of electron decreases and thus becomes negative. Thus
as the value of n decreases, i.e. lower the orbit is, more NO3 7 8 3 + 1 = 32
negative is the energy of the electron in it.
14. (a) 2nd excited state will be the 3rd energy level. 2. SO32 16 8 3 + 2 = 42
13.6 13.6 2
En = eV or E= eV = 1.51 eV. CO3 32 not iso electronic
2 9
n
NO3 32
h
15. (c) Dx. Dp = ;
4p
3. CN 6 + 7 + 1 = 14
-34
6.62 ´ 10
\ Δv = = 2.1´ 10 -28 ms -1 N2 7 2 = 14 iso electronic
-5
4 ´ 3.14 ´ 0.025 ´ 10
2
16. (c) Fe+2 = 3d6, 4s0 C2 6 2 + 2 = 14
17. (a) For s-electron, l = 0 \ angular momentum = zero
18. (b) f-block elements show a regular decrease in atomic size
4. PO34 15 8 4 + 3 = 50
due to lanthanide/actinide contraction.
19. (a) N3–, F– and Na+ contain 10 electrons each. 2
SO 4 16 8 4 + 2 = 50 iso electronic
20. (a) The lines falling in the visible region comprise Balmer
series. Hence the third line would be n1 =2, n 2 = 5 i.e. ClO 4 17 8 4 + 1 = 50
5 ® 2.
29. (b) Accroding to Bohr's theory angular momentum of an
-34
h 6.6 ´ 10
21. (d) l = = = 10 -33 m electron i in nth orbital is given by mvr =
nh
mv 60 ´ 10 -3 ´10 2p
22. (a) The quantum numbers for 4f electron when n = 5
1 5h 2.5h
n = 4, l = 3, m = – 3, –2 –1, 0, 1 , 2 , 3 and S = ± Angular momentum of electron = =
2 2p p
Structure of Atom 55
30. (a) Given m = 9.1 × 10–31kg 36. (b) According to Heisenberg uncertainty principle.
h = 6.6 × 10–34Js h h
Dx.mDv = Dx =
4p 4pmDv
300 ´ .001
DV = = 0.003ms–1 600 ´ 0.005
100 Here Dv = = 0.03
100
From Heisenberg's uncertainity priciple 6.6 ´ 10 -34
So, Dx =
6.62 ´ 10 -34 4 ´ 3.14 ´ 9.1´ 10-31 ´ 0.03
Dx = = 1.92 ´ 10 - 2 m = 1.92 × 10–3 meter
4 ´ 3.14 ´ 0.003 ´ 9.1 ´ 10 -31
37. (d) Energy required to break one mole of Cl – Cl bonds in
31. (a) N3– = 7 + 3 = 10e–, O–– ––® + 2 = 10e Cl2
P– = 9 + 1 = 10e–, S – – ––® 1 + 2 = 1 e–
242 ´ 103 hc
(not iso electronic) = 23
=
6.023 ´ 10 l
(b) Li+ = 3+1= 4e–, Na+ = 11–1 = 10e–,
Mg++ = 12–2=10e– 6.626 10 34 3 108
=
Ca++ = 20 – 2 = 18e– (not iso electronic)
(c) K+ = 19 – 1= 18e–, Cl– =17 + 1 = 18e–, 6.626 ´ 10 -34 ´ 3 ´108 ´ 6.023 ´10 23
Ca++ = 20 – 2 =18e–, Sc3+ = 21–3 = 18e– (iso electronic) \l=
242 ´ 108
(d) Ba++ 56 – 2 = 54e–, Sr++ 38–2 = 36e– = 0.4947 × 10–6 m = 494.7 nm
K+= 19–1 = 18e–, Ca++= 20–2 = 18e– (not iso electronic)
Z2
32. (c) (a) n = 3, l = 0 means 3s-orbital and n + l = 3 38. (b) I. E = 13.6eV ...(i)
n2
(b) n = 3, l = 1 means 3p-orbital n + l = 4
I1 Z12 n22
(c) n = 3, l = 2 means 3d-orbital n + l = 5 or I ...(ii)
2 n12 Z 22
(d) n = 4, l = 0 means 4s-orbital n + l = 4 Given I1 = – 19.6 × 10–18 , Z1 = 2,
Increasing order of energy among these orbitals is n1 = 1 , Z2 = 3 and n2 = 1
3s < 3p < 4s < 3d Substituting these values in equation (ii).
\ 3d has highest energy.. 19.6 ´10 -18 4 1
33. (a) (DE), The energy required to excite an electron in an – = ´
I2 1 9
atom of hydrogen from n = 1 to n = 2 is given by the
18 9
following relation (difference in energy E2 and E1) or I 2 19.6 10
4
-1.312 ´106 ´ (1)2 = – 4.41 × 10–17 J/atom
E2 = = –3.28 × 105 J mol–1 39. (a) For He+
(2)2
1 æ 1 1 ö
E1 = – 1.312 × 106 J mol–1 v = = RH Z 2 ç 2 - 2 ÷
l è2 4 ø
\ DE = E2 – E1
2 1 1 1 1
= [–3.28 × 105] – [–1.312 × 106 ] J mol–1 RH (2) 2 2÷
RH ÷
= (–3.28 × 105 + 1.312 × 106) J mol–1 2 4 (1)2 (2)2
For H
= 9.84 × 105 J mol–1
Thus the correct answer is (a) 1 1 1÷
34. (c) Species having same number of electrons are v= = RH
n12 n22 ÷
isoelectronic calculating the number of electrons in
each species given here, we get. For same frequency,
CN– (6 + 7 + 1 = 14); N2 (7 + 7 = 14);
O22–(8 + 8 +2 = 18) ; C22– (6 + 6 + 2 = 14); 1 1 1 1
RH 2 ÷ = RH ÷
O2– (8 + 8 + 1 = 17) ; NO+ (7 + 8 – 1 = 14) (1) (2 )2 n12 n22 ÷
CO (6 + 8 = 14) ; NO (7 + 8 = 15)
From the above calculation we find that all the species 1 1 1 1
\
listed in choice (c) have 14 electrons each so it is the n12 1 n22
22 2

correct answer. \ n1 = 1 & n2 = 2


40. (b) (A) 4 p (B) 4 s
h 6.63 ´ 10-34 (C) 3 d (D) 3 p
35. (a) l= =
mv 1.67 ´ 10-27 ´ 1 ´103 According to Bohr Bury's (n + l) rule, increasing order
of energy will be (D) < (B) < (C) < (A).
= 3.97 × 10–10 meter = 0.397 nanometer » 0.40 nm Note : If the two orbitals have same value of (n + l)
then the orbital with lower value of n will be filled first.
56 Chemistry
41. (c) Among isoelectronic species ionic radii increases as 4. (d) In photoelectric effect K.E., hence velocity is directly
the charge increases. proportional to frequency of the incident light and
Order of ionic radii Ca2+ < K+ < Cl– < S2– independent of the intensity of light.
The number of electrons remains the same but nuclear
charge increases with increase in the atomic number 1 2 h
5. (a) Since K.E. = m and = .
causing decrease in size. 2 m
18 1 1 hc
42. (a) E 2.178 10 2 2÷
= 1 h2 h2
1 2 K.E. = m. = . As is the same.
2 m2 2 2m 2

18 3 hc
2.178 10 = 1
4 K.E. µ
m
34
6.62 10 3 108
= V 1
6. (d) Angular speed is . Vn µ and rn µ n 2 .
r n
34
6.62 10 3 108 4 Angular speed is inversely proportional to n
=
2.178 10 18 3 7. (d) The smallest value of energy of an electron in H atom in
= 1.214 × 10–7m ground state can absorb is = E2 – E1.
43. (c) As per Bohr’s postulate,
13.58 13.58
nh = ÷ = 10.19
mvr = 4 1
2
nh h 6 .6 1 0 3 4
So, v = 8. (c) de-Broglie wavelength = = = 2Å
2 mr m 3 .3 1 0 2 4
1 2 9. (c) After absorbing 12.1 eV the electron in H atom is excited
KE = mv
2 to 3 shell.
2
1 nh 13.6
So, KE = m ÷ En E1 = ( 13.6) = 12.1
2 2 mr n2
a n2 13.6
Since, r = + 13.6 = 12.1
z n2
So, for 2nd Bohr orbit n=3
a 22 The possible transitions are (3 1) = 3
r= = 4a
1
1 1 1
1 22 h 2 10. (b) = RH = 4.86 10 7 m = 486 nm
KE = m 2 2
÷
2 2
4 2
2 4 m (4a )2 ÷

h2 11. (c) E1 + E1H Z2


KE = He
32 2 ma 2
44. (b) Average atomic mass of Fe 871.6 10 20 E1H 4
(54 5) (56 90) (57 5)
= = 55.95 E1H 217.9 10 20 J
100
EXERCISE 4 1 1
12. (c) = =
9
= 2 106 m 1

1. (d) All the statements are correct. 500 10


13. (c) The ground state electronic configuration, of Cu+ ion is
Ze 2 1s2, 2s2, 2p6, 3s2, 3p6, 3d10
2. (d) In S.I. units the P.E. = . For Li 2+ , Z = 3.
4 0r n = 3 (number of shells) ; Number of subshells occupied
= 6, Number of filled orbitals = 14 ; There is no unpaired
3e 2 electron.
P.E. = . 14. (d) Magnetic moment for an electron exists even if the orbital
4 0r
angular momentum is zero. This is explained by the spin
3. (b) According to Pauli's exclusion principle the maximum magnetic because it is the spin which produces magnetic
number of electrons with opposite spin present in an moment.
atomic orbital is two.
Structure of Atom 57
20 25. (a) When l = 4, the number of degenrate orbitals
15. (a) 10 Ne contains10 protons and 10 neutrons
= 2l + 1 = 9,
\ M1 = 10 mp + 10mn
1
40 contains 20 protons and 20 neutrons the maximum total spins = 9 × and
20 Ca 2
\ M2 = 20 mp + 20 mn
\ M2 =2M1 1
minimum total spin =
16. (c) Deuterium and an a-particle have identical values of e/m. 2
æ 1 9
1 ç 1 ö÷ Maximum multiplicity = 2s + 1 = 2 × + 1 = 10
17. (b) l = R ¥ ç 2 - 2 ÷ for hydrogen atom. 2
è n1 n 2 ø
1
For the Balmer series n 1 = 2 and n2 = 5 for third line Minimum multiplicity = 2 × +1=2
2
1 æ 1 1 ö 21 100 26. (a) From de-Broglie equation,
\ = R ¥ çç - ÷÷ = R ¥ thus l =
l è2 2 2
5 ø 100 21 R ¥ h
18. (c) Spin multiplicity is given by A = (2 S + 1). ­ represent ½ l= ;
2Vem
and ¯ repr esent – ½ spin values. For
h
l= (for proton)
2Vemp
S=–½
\ A = (2 ´ -½ + 1) = 0 h
l= (for Li 3+ )
19. (a) 2V3emLi

r2 2
lLi 3+ 2Vemp 1
20. (c) l = 2 represent d orbital for which \ = =
lp 6Vemp ´ 9 3 3
r
21. (b) The point at which the radial function acquirs zero value he
is called a node. Examine the graph the answer is (b). (c) 27. (c) One Bohr magneton =
4 mc
graph contains in nodes (a) and (d) none. 28. (b) Energy of 1 mole of photons,
h 6.63 ´ 10 -34 E = N0 × h u
22. (b) l = = = 6.63 × 10 -31 m
mv 10 ´ 10 -6 ´ 100
N0 ´ h ´ c
1 1 =
23. (c) 1 amu = g = N 0 -1 or × mass of C-12 atom l
N0 12
6.023 ´ 10 23 ´ 6.63 ´ 10 -34 ´ 3 ´ 10 8
1 æ ö =
24. (d) = R H Z2 ç 1 - 1 ÷ 253.7 ´ 10 -9
l çn 2 n 2 ÷ = 472.2 kJ
è 1 2 ø
Energy converted into KE = (472.2 – 430.53) kJ
To calculate shortest wavelength take n 2 = ¥ and
(472.2 - 430.53)
longest wavelength take nearest value of n 2 . % of energy converted into KE =
472.2
For H-atom,
= 8.76 %
1 29. (c) Magnetic moment
n 2 = ¥ , Z = 1, n1 = 1
l shortest
n ( n + 2) = 15
1 \ n = 3 number of unpaired electrons
\ = RH (Lyman series)
x 5 2
25 M = 3d , 4s
1
For l longest for Li 2+ , Z = 3, n1 = 2, n 2 = 3 (Balmer M x = 3d3
\ x = 4 for three unpaired electrons
series)
(KE )1 1- 0
1 1 æ 1 1 ö 5 30. (c) =
= × 3 2 çç 2 - 2 ÷÷ = (KE ) 2 2- 0
l longest x è2 3 ø 4x
(1.5 ´1015 - 1 ´1015 ) 1
4x = =
\ l longest = (2.0 ´ 10 15
- 1 ´10 ) 15 2
5
58 Chemistry

31. (c) Fe (III) = [Ar] 3d 5 unpaired electrons = 5; h 1.23


36. (a) l = nm=
Magnetic moment = 5(5 + 2) ; 2eVm V
37. (a) Statement (a) is correct.
Ratio = 7: 3
h
Co(II) = [Ar] 3d 7 unpaired electrons = 3; 38. (b) l p = ;
2eVmp
Magnetic moment = 3(3 + 2)
h h
l = =
He 2 + 2 ´ 2eV ´ 4m p
Ratio = 7 : 3 2 ´ 2eVm
He2+
32. (a) Velocity of electron
lp
Z \
v n = 2.19 × 106 ms-1 l 2+
= 2 2
n He

The distance travelled by electron in 10 -8


s in Second 39. (d) Isodiaphers have same difference of number of neutrons
Bohr’s Orbit and protons or (A – 2Z) must be same.
40. (d) Quantum efficiency
2.19 ´10 6 ´1´10 -8 Number of moles of substance reacted
= m =
2 Number of Quanta absorbed
-2
= 1.095 × 10 m
3 ´ 10 - 3 ´ 6 ´ 10 23
The circumference of second orbit = 2pr2 = =1
3 ´ 1018 ´ 60 ´ 10
= 2p ´ 0.529 ´ 10 -10 × 2 2 2
41. (a) rn = a 0 ´ n
- 10
= 13.3 × 10 m
2
r4 = a 0 ´ (4) = 16a 0
1.095 ´ 10 -2
\ Number of revolutions = = 8.23 × 10 6
13.3 ´ 10 -10 nh 4h
mvr = ; mv = ;
33. (b) The number of nodal surfaces is given by 2p 2p ´16a 0
n–l–1=3–0–1=2
h h
34. (b) E1 (He + ) = -IE 2 (He ) = – 54.4 eV l= = = 8pa 0
mv h / 8pa 0
-E1 42. (d) Isodiaphers since (A–2Z) value is the same for both.
The other energy states can be obtained from where
n2 43. (d) Apply de Broglie equation to solve.
n = 1, 2, 3.
h 2 h
35. (a) Dx. Dp = or Dp = or D (mv) 2
4p 4 p

1 h
\ Dv =
2m p
3
Classification of Elements
and Periodicity in Properties
THE PERIODIC TABLE OF THE ELEMENTS

PERIODIC TABLE : NEWLAND’S LAW OF OCTAVES :


It is a table of elements in which the elements with similar properties This was an arrangement of elements in order of increasing atomic
are placed together. weights in which it was observed that every eighth element had
DOBEREINER’S LAW OF TRIADS : properties similar to those of the first just like the eight node of an
This was the classification of elements into groups of three octave of music.
elements each with similar properties such that the atomic weight
of the middle element was the arithmetic mean of the other two Li Be B C N O F
e.g. Ca, Sr, Ba; Cl, Br, I etc. Na Mg Al Si P S Cl
The difference between atomic weights of any two successive Prouts Hypothesis : Atomic weight of an element is simple multiple
elements is nearly constant. of atomic weight of hydrogen.
60 Chemistry
MENDELEEV’S PERIODIC TABLE : (b) Periods - (i) The 7 horizontal columns (or rows) are called
This is based upon Mendeleev’s periodic law which states that periods. The seven periods of periodic table are
the physical and chemical properties of the elements are a periodic Shortest period - 1st period (1H ® 2He) contains 2 elements.
function of their atomic weights. It is the shortest period.
Mendeleev named Gallium and Germanium elements as eka- Short periods - 2nd period (3Li ® 10Ne) and 3rd period
aluminium and eka-silion respectively. (11Na ® 18Ar) contains 8 elements each. These are short
STRUCTURAL FEATURES OF THE MENDELEEV’S periods.
PERIODIC TABLE: Long periods - 4th period (19K ® 36Kr) and 5th period
(37Rb ® 54Xe) contain 18 elements each and are called long
(i) Mendeleev’s original periodic table consists eight vertical
periods.
columns are called groups I-VIII & seven horizontal rows are
Longest period - 6th period (55Cs – 86Rn) contains 32
called periods 1-7. But modified Mendeleev’s periodic table
elements and is the longest period.
contains nine groups, i.e., I-VIII and zero (of noble gases).
Incomplete period - 7th period (87Fr –) is, however, incomplete
(ii) All groups except VII and zero have been further divided
and contains at present only 26 elements.
into two sub-groups called A and B. A groups of left hand
(c) Blocks - The periodic table is divided into four main blocks
side consist of normal elements while groups B of right
(s, p, d and f) depending upon the subshell to which the
hand side consist of transition elements.
valence electron enters into.
(iii) Elements of group IA are called alkali metals while those of
(i) s-Block - Elements of group 1 and 2 constitute s-block.
group IB (i.e, Cu, Ag, Au) are called coinage metals.
(ii) p-Block - Elements of group 13 to 18 constitute
MODERN PERIODIC LAW : p-block.
Moseley formed the basis of the modern periodic law. He (iii) d-Block - Elements of group 3 to 12 constitute
discovered that the square root of the frequency of the more d-block.
prominent X-rays emitted by a metal was proportional to the atomic (iv) There are three complete series and one incomplete series
number and not the atomic weight of the atom of the metal. Hence of d-block elements. These are:
atomic number should be the basis of classification of the
1st or 3d-transition series which contains ten elements
elements.
with atomic number 21-30 (21Sc–30Zn).
Modern periodic law states, “that the physical and chemical
2nd or 4d-transition series which contains
properties of the elements are a periodic function of their atomic ten elements with atomic number 39–48
number.” (39Y–48Cd).
It was observed that the elements with similar properties reoccurred 3rd or 5d-transition series which also contains ten
at regular intervals of 2, 8, 8, 18 or 32. These numbers (2, 8, 8, 18 elements with atomic number 57 and 72-80 (57La, 72Hf–
and 32) are called magic numbers, and cause of periodicity in
80Hg).
properties.
4th or 6d-transition series which contains only ten
Structural features of the long form of the periodic table elements.
(a) Groups - (v) The f-block elements comprise two horizontal rows
(i) The 18 vertical columns, of the periodic table, are called placed at the bottom of the periodic table to avoid
groups its un-necessary expansion and make the symmetrial
(ii) Elements of groups 1, 2, 13-17 are called normal or nature of periodic table. The two series of f-block
representative elements. elements containing 14 elements each. Lanthanides -
(iii) Elements of groups 3-12 are called transition elements The 14 elements from 58Ce–71Lu in which 4f-subshell is
(iv) The elements belonging to a particular group is known being progressively filled up are called lanthanides or
as a family and is usually named after the first element. rare earth elements. Actinides - Similarly, the 14 elements
For example, Boron family (group 13). In addition to from 90Th – 103Lr in which 5f-subshell is being
this, some groups have typical names. For example, progressively filled up are called actinides.
Elements of group 1 are called alkali metals (vi) Elements of s and p-blocks are called normal or
representative elements, those of d-block are called
Elements of group 2 are called alkaline earth metals
transition elements while the f-block elements are called
Elements of group 3 are called pnicogens inner transition elements.
Elements of group 16 are called chalcogens
(vii)The 11 elements with Z = 93–103 (93Np – 103Lr) which
Elements of group 17 are called halogens occur in the periodic table after uranium and have been
Elements of group 18 are called noble gases prepared by artificial means are called transuranics.
These are all radioactive elements.
Classification of Elements and Periodicity in Properties 61
NOMENCLATURE OF ELEMENTS WITH ATOMIC (a) Covalent radius : It is half of the distance between
NUMBER > 100 the nuclei of two like atoms bonded together by a
single covalent bond, hence it is also known as single
According to IUPAC the nomenclature can be derived using
bond covalent radius (SBCR). Thus, covalent radius
numerical roots for 0 and numbers 1-9 for atomic numbers of
1
elements. (rcov . ) = d.
2
The roots are put together in order of digits which make the atomic where d = internuclear distance between two
number and ‘ium’ is added at the end. Use the following table covalently bonded like atoms.
(b) Van-der Waal’s radius : It is one-half of the distance
Digit Name Abbrevation
between the nuclei of two adjacent atoms belonging
0 nil n to two neighbouring molecules of an element in the
1 un u solid state.
2 bi b The covalent radius is always smaller than the
3 tri t corresponding van der waal’s radius.
(c) Metallic radius : It is half of the distance between
4 quad q
two successive nuclei of two adjacent metal atoms
5 pent p in the metallic closed packed crystal lattice. Metallic
6 hex h radius of an element is always greater than its
7 sept s covalent radius.
8 oct o (d) Ionic radius : It is the effective distance from the
nucleus of the ion upto the electrons in the outer
9 enn e
shell to which it can influence the ionic bond.
Example : Name the IUPAC name of the element of atomic An atom can be changed into a cation (by loss of
number 108 : Name will be Unniloctium and symbol - UnO electron) which is always much smaller than the
(2) Example – Name the element with atomic number 115. corresponding atom, or to an anion (by gaining of
Name will be - Ununpentium and symbol UuP electrons) which is always larger than the
corresponding atom.
DIAGONAL RELATIONSHIP:
(iv) Factors influencing covalent radius :
Certain elements of 2nd period i.e., Li, Be, B. show similarity with (a) Multiplicity of bond : Covalent radii depends on
their diagonal elements in the 3rd period i.e., Mg, Al, Si, as shown
the multiplicity of bonds. e.g.,
below:
Bond length Radius of C-atom
Group 1 Group 2 Group 13 Group 14
1.54
2nd period Li Be B C H 3C - CH 3 1.54Å Å = 0.77 Å
2
3rd period Na Mg Al Si
This is called diagonal relationship and is due to the reason that 1.34
H 2C = CH 2 1.34Å Å = 0.67Å
these pairs of elements have almost identical ionic radii and 2
polarizing power. (i.e. charge/size ratio). (b) Percentage of ionic character: Covalent radius of
PERIODIC PROPERTIES : H in HCl, HBr and HI are different.
Properties which show a regular gradation when we move from (c) Effective nuclear charge : Greater the effective
left to right in a period or from top to bottom in a group are called nuclear charge, more tightly is the hold with nucleus
periodic properties. These properties are atomic size, ionisation and hence smaller the radius.
energy electron affinity etc. (v) Periodic variation of atomic radii :
(A) Atomic size: It refers to the distance between the centre of (a) On moving down the group the valence shells
nucleus of atom to its outermost shell of electrons. The become far away from the nucleus and thus the
absolute value of atomic radius cannot be determined because atomic radius increases.
(i) It is not possible to locate the exact position of electrons (b) On moving along the period, the effective nuclear
in an atom as an orbital has no sharp boundaries. charge increases and thus the electron cloud is
(ii) It is not possible to isolate an individual atom.
attracted more strongly towards the nucleus
(iii) In a group of atoms, the probability distribution of
resulting in the contraction of atomic radius.
electrons is influenced by the presence of neighbouring
atoms. (vi) Isoelectronic ions or species : These are ions of the
Since absolute value of atomic size cannot be determined, different elements which have the same number of
it is expressed in terms of the operational definitions electrons but different magnitude of the nuclear charge.
such as covalent radius, vander waal’s radius, ionic radius The size of isoelectronic ions decreases with the increase
and metallic radius. in the nuclear charge.
62 Chemistry
(B) Ionisation energy (I.E.) : The amount of energy (work) (i) EA values are expressed in eV/atom or kcal/mol
required to remove an electron from the last orbit of an isolated or kJ/mol.
(free) atom in gaseous state is known as ionisation potential (ii) The energy change accompanying the addition of first,
or energy or better first ionisation potential of the element, second, third etc. electrons to neutral isolated gaseous
i.e., ¾® M + (g ) + e -
M ( g ) + I.E ¾ atoms are called successive electron affinities and are
designated as EA1, EA2, EA3 etc.
(i) The amount of energies required to remove the
subsequent electrons (2nd, 3rd, ...) from the monovalent (iii) The first EA is always taken as positive. However, the
gaseous cation of the element one after the other are addition of second electron to the negatively charged
collectively called successive ionisation energies. These ion is opposed by coulombic repulsion and hence
are designated as I.E1, I.E2, I.E3, I.E4 and so on. It may be required (absorbed) energy for the addition of second
noted that. I.E4 > I.E3 > I.E2> I.E1 (for a particular element) electron. Thus, second electron affinity (EA2) of an
IE is expressed in eV/atom or kcal mol–1 or kJ mol–1 element is taken as negative. For example,
Note that eV atom–1 = 23.06 kcal mol–1 = 96.3 kJ mol–1
O (g ) + e - ¾
¾® O - (g ); EA1 = +141 KJ mol -1
(ii) In general, the first I.E. increases along the period from
left to right. However there are some exceptions to the (Exothermic)
general trend -
(a) I.E. decreases from elements of group 2 ® 3. O - (g ) + e - ¾
¾® O 2- (g ); EA 2 = -780 KJ mol -1
(b) I.E. decreases from elements of group 15 ® 16. (Endothermic)
(iii) In a group of the periodic table, the ionisation energy (iv) Electron affinity increases in moving along the period
decreases from top to bottom. from left to right due to increase in charge. But the values
(iv) The factors which affect the ionisation energy are are unexpectedly low in elements of group 2, 15 and 18
(a) Atomic size or radius : I.E. decreases as the atomic due to stable electronic configurations of exactly half-
size increases so the attractive force decreases. filled and completely filled orbitals.
(b) Number of electrons in the inner shell (screening (v) Within a group of the periodic table the electron affinities
effect) : On moving down a group, the number of decreases from top to bottom.
inner shells increases which increase the screening
(vi) In general, electron affinity follows the following trend:
effect and hence the ionisation potential tends to
decrease. Halogens > Oxygen family > Nitrogen family > Metals
(c) Nuclear charge : On moving along the period, of groups 1 and 13 and Non-metals of group 14 > Metals
effective nuclear charge increases due to addition of group 2.
of electrons in same valence shell and hence (vii)The electron affinities are indirectly measured with the
ionisation energy increases. help of Born-Haber Cycle, i.e.,
(d) Stable cofiguration : Half filled or completely filled
1
subshells possess extra stable nature and thus it is DH f = S + D + IE + EA + U
more difficult to remove electron and hence more is 2
I.E. Where, S, D, IE, EA and U are the heat of sublimation,
(e) Penetration effect : More penetrating bond dissociation energy, ionization energy, electron
(i.e. more closes) are subshells of a shell to the affinity and lattice energy respectively.
nucleus, more tightly the electrons are held towards (viii)Electron affinity depends upon-
the nucleus and more is I.E. (a) Effective nuclear charge - Greater the effective
I.E. : s > p > d > f for a given shell
nuclear charge, more is the attraction of nucleus
Penetration power µ Surface area of a subshell
towards the electron and hence higher will be the
(v) In second period elements, the correct increasing order
value of E.A.
of ionisation energies is
IE1 : Li < B < Be < C < O < N < F < Ne (b) Atomic size - Greater the atomic radius of the atom,
IE2 : Be < C < B < N < F < O < Ne < Li less will be the attraction of the nucleus to the
(vi) In third period elements, the correct increasing order of electron to be added and hence lower will be the
ionisation is value of E.A.
IE1 : Na < Al < Mg < Si < S < P < Cl < Ar (c) Penetrating power - Due to penetrating power, E.A.
IE2 : Mg < Si < Al < P < S < Cl < Ar < Na for addition of electron show the order
(C) Electron affinity (EA): It is the amount of energy released s>p>d>f
when a gaseous atom accepts the electron to form gaseous (d) Electronic configuration : Half filled and fully filled
anion . subshell are extra stable and thus oppose the
X (g ) + e - ¾
¾® X - (g ) + EA addition of electron which leads to lower, E.A. value
e.g. EA, of C > EA, of N.
Classification of Elements and Periodicity in Properties 63
(D) Electronegativity (EN) : It is the tendency of an atom in a VALENCY :
molecule to attract the bonded shared pair of electrons,
Valency of an element is the number of electrons gained or lost or
towards itself
shared with other atoms in the formation of compounds.
(i) There are several electronegativity scales-
(a) Mulliken scale: On the Mulliken scale, Valency of group 1 and 2 elements is equal to the number of
electronegativity X is taken as average of IE and electrons in the outermost shell, while that for groups 13 to 14 is
EA, i.e., group number -minus 10 and that for group 15–18 is 8 -minus the
number of electrons in the outermost shell.
IE + EA
X= where IE and EA are expressed in ATOMIC VOLUME :
2
electron volts It may be defined as the volume occupied by one mole atoms of
the element at its melting point in solid state.
IE + EA
or X= where IE and EA are (i) It is obtained by dividing the gram atomic mass of the element
540
by its density.
expressed in kJ mol–1
(ii) It decreases along the period, reaches a minimum in the middle
IE + EA and then starts increasing, because of different packing
or X= where IE and EA are
130 arrangement of atoms in different elements in the solid state,
expressed in kcal mol–1 i.e., P4, S8 etc.
(b) Pauling scale: This is the most widely used scale (iii) In moving down the group atomic volume goes on increasing
and is based upon bond energy data. According to gradually
Pauling, the difference in electronegativity of two
atoms A and B is given by the relationship as ACID-BAS E BEHAVIOUR OF O XIDES AND
HYDROXIDES :
XA XB 0.208 E 0.088 E (in SI units)
The oxides or hydroxide of an element may act either as base or an
where XA and XB are electronegativities of the atoms acid depending upon its ionization energy.
A and B respectively while.
(i) If the IE is low, it acts as a base and if the IE is high, it acts as
DE = E A - B - ( E A -A ´ E B-B an acid.
(ii) The IE of alkali metals is the lowest, therefore, their oxides
where E A–B , E A–A and E B–B represent bond
and hydroxides are the strongest bases. The basic character
dissociation energies of the bonds A-B, A-A and B-
of their hydroxides increases in the order:
B respectively. The Pauling and the Mulliken scales
are related to each other by the relation, CsOH > RbOH > KOH > NaOH > LiOH
(iii) The IE of halogens is quite high, therefore, their oxides are
X Mulliken
X Pauling = the strongest acids. The acidic character of their oxides and
2.8
hydroxides decreases in the order:
(ii) In a period, EN increases from left to right due to decrease
HClO4 > HBrO4 > HIO4
in size and increase in nuclear charge of atoms.
(iv) Within a period, the ionization energies of the elements
(iii) In a group, EN decreases from top to bottom due to
usually increase and hence their oxides and hydroxides show
increase in atomic size.
a gradual variation from strongly basic through amphoteric
(iv) Electronegativity depends on: to strongly acidic character. For example:
(a) Atomic size
Na2O MgO Al2O3 SiO2 P4O10 SO2 Cl2O7
(b) Nuclear charge
(c) Shielding effect Strongly Basic Amphoteric Weakly Strongly Very strongly
(d) Oxidation state - EN increases as the positive basic acidic acidic acidic
oxidation state increases. (v) The non-metallic character, oxidising character and acidic
(e) Hybridization - Greater is the s-character in a hybrid nature of oxides of the elements increases from left to right
orbital more is electronegativity. in a period and decrease from top to bottom in a group.
(v) If electronegativity difference is greater than 1.7 the bond The stability of the metal increases and activity decreases
is ionic otherwise covalent. from left to right in a period whereas the stability decreases
(vi) In general, greater is difference of EN between two atoms and activity increases down the group.
smaller is the bond length.
(vii) Smaller the electronegativity, larger is the atomic size.
64 Chemistry

Very Short/Short Answer Questions (ii) Arrange the above five elements is order of increasing
electron gain enthalpy.
1. Name a species which is iso electronic with Al3+.
13. Predict the formula of a stable binary compound that would
2. Helium has a electronic configuration of 1s2 but it is placed
be formed by the following pairs of elements
in p-block in group 18. Explain.
(a) aluminum and phosphorous
3. Name the elements with the lowest and the highest ionisation (b) lithium and nitrogen
energy. (c) calcium and sulphur
4. Predict the position of the element in the Periodic Table
satisfying the electronic configuration (n – 1) d1 ns2 Long Answer Questions
for n = 4.
14. Name the four blocks of elements in the periodic table and
5. Write equations to demonstrate the difference between list the difference in their electronic configurations. Give
electron affinity and the reverse of ionization potential. examples of each type of elements. Write at least four
6. All the lanthanoid elements form stable characteristics of each block.
compounds containing the +3 cation. Of the few other ionic 15. Find the best choice in the following list and give brief reason
forms known, Ce forms the stable + 4 series of ionic com- for your answer
pounds and Eu the stable + 2 series. Account for these (i) highest IE1; Se, S,Te
unusual ionic forms in terms of their electronic configura- (ii) smallest radius; Cl–, Br–, I–
tions. (iii) most parmagnetic; Fe, Co, Ni
7. Nitrogen has positive electron gain enthalpy whereas oxygen (iv) smallest atom; Sn, I, Bi
has negative. However, oxygen has lower ionisation
(v) lowest IE2 ; Ar, K, Ca
enthalpy than nitrogen. Explain.
16. (i) Explain the terms (a) screening effect,
8. Explain the following : (b) penetration effect, (c) metallic character.
(a) Electronegativity of elements increase on moving from (ii) Why are electron affinities of noble gases zero ?
left to right in the periodic table. Arrange halogens in increasing order of electron
(b) Ionisation enthalpy decrease in a group from top to affinity.
bottom. 17. The second ionization enthalpy (IE2) of the elements of the
9. Give the order in which the melting points of halides of second period are given below :
sodium decrease and why ? Element : IE2 (kJ/mol) :
10. The first (IE1) and second (IE2) ionisation enthalpies (kJ Li 7294
mol–1) of three elements I, II and III are given below : Be 1756
I II III B 2430
C 2354
1E1 403 549 1142
N 2856
1E2 2640 1060 2080 O 3396
Identify the element which is likely to be (a) non-metal (b) F 3377
an alkali metal (c) an alkaline earth metal. Ne 3966
11. Show by a chemical reaction with water that Na2O is a basic (i) Explain
oxide and Cl2O7 is an acidic oxide. (a) Why is IE2 of Li so much higher than for all other
12. Consider the ground state electronic configuration of the elements of this period ?
five elements I to V given below : (b) What is the general trend from Be to Ne of
Elements Electronic Configuration increasing IE2?
I 1s2 2s2 2p5 (c) Why IE 2 of F is just about the same
or very slightly less than that of oxygen ?
II 1s2 2s2 2p4
(ii) Would you predict the IE2 of Na in kJ/mol is
III 1s2 2s2 2p6 3s2 (a) greater than 7294
IV 1s2 2s2 2p6 3s1 (b) between 3966 and 7294 or
V 1s2 2s2 2p6 (c) less than 3966 ? Explain.
(i) Which of the above configuration is associated with
the highest and which is associated with the lowest
ionisation enthalpy ?
Classification of Elements and Periodicity in Properties 65

Multiple Choice Questions 23. The period number in the long form of the periodic table is
equal to
18. Element A belongs to Group VII in p-block and element B
(a) magnetic quantum number of any element of the period.
belongs to Group I in s-block of the periodic table. Out of
(b) atomic number of any element of the period.
the following assumptions, the correct one is :
(c) maximum Principal quantum number of any element of
(a) A and B are metals
the period.
(b) A and B are non-metals
(d) maximum Azimuthal quantum number of any element
(c) A is a metal and B is a non-metal
of the period.
(d) A is a non-metal and B is a metal 24. The lightest liquid metal is
19. Examine the following elements : (a) Hg (b) Ga
N, O, F, Ne (c) Cs (d) Fr
P, S, Cl, Ar 25. Which of the following elements A, B, C, D and E with atomic
Br, Kr number 3, 11, 15, 18 and 19 respectively belong to the same
I, Xe group?
In modern periodic table, on which side these elements are (a) A, B, C (c) B, C, D
placed (c) A, D, E (d) A, B, E
(a) Top left side (b) Bottom left side 26. Which of the following statement(s) about the Modern
(c) Top right side (d) Middle side Periodic Table is/are incorrect ?
(i) The elements in the Modern Periodic Table are arranged
20. The screening effect of ‘d’ electrons is
on the basis of their decreasing atomic number
(a) Much less than s-electrons (ii) The elements in the Modern Periodic Table are arranged
(b) Much more than s-electrons on the basis of their increasing atomic masses
(c) Equal to s-electrons (iii) Isotopes are placed in adjoining group(s) in the Periodic
(d) Equal to p-electrons Table
21. Which of the following is not an actinoid ? (iv) The elements in the Modern Periodic Table are arranged
(a) Curium (Z = 96) on the basis of their increasing atomic number
(a) (i) only (b) (i), (ii) and (iii)
(b) Californium (Z = 98) (c) (i), (ii) and (iv) (d) (iv) only
(c) Uranium (Z = 92) 27. An element having electronic configuration
(d) Terbium (Z = 65)
22. Electronic configurations of four elements A, B, C and D are 1s 2 2s 2 2p 6 3s 2 3p 6 4s1 forms
given below : (a) Acidic oxide (b) Basic oxide
(i) 1s2 2s2 2p6 (ii) 1s2 2s2 2p4 (c) Amphoteric oxide (d) Neutral oxide
2 2 6
(iii) 1s 2s 2p 3s 1 (iv) 1s2 2s2 2p5 28. An element which is an essential constituent of all organic
compounds belongs to
Which of the following is the correct order of increasing
(a) group 1 (b) group 14
tendency to gain electron ?
(c) group 15 (d) group 16
(a) A < C < B < D (b) A < B < C < D
(c) D < B < C < A (d) D < A < B < C

1. As per the modern periodic law, the physical and chemical 3. The most significant contribution towards the development
properties of elements are periodic functions of their of periodic table was made by
(a) atomic volume (a) Mendeleev (b) Avogadro
(b) electronic configuration (c) Dalton (d) Cavendish
(c) atomic weight 4. Mendeleev’s periodic law is based on
(d) atomic size (a) atomic number (b) atomic weight
2. Eka-aluminium and Eka-silicon are known as (c) equivalent weight (d) valency
(a) Gallium and Germanium 5. Which of these does not reflect the periodicity of the
(b) Aluminium and Silicon elements?
(c) Iron and Sulphur (a) Bonding behaviour (b) Electronegativity
(d) Manganese and Magnesium (c) Ionization potential (d) Neutron/proton ratio
66 Chemistry
6. The statement that is not true for the long form of the periodic 18. Electronic configuration of most electronegative element is
table is (a) 1s2 2s2 2p4 3s1 (b) 1s2 2s2 2p6 3s2 3p5
(a) It reflects the sequence of filling electrons in the order of (c) 1s2 2s2 2p5 (d) 1s2 2s2 2p6 3s2 3p6
sub-energy levels s, p, d and f. 19. Going from fluorine to chlorine, bromine and iodine, the
(b) It helps to predict the stable valence states of the electronegativity
elements (a) increases
(c) It reflects trends in physical and chemical properties of (b) decreases
the elements (c) first decrease then increases
(d) It helps to predict the relative ionicity of the bond (d) changes randomly
between any two elements. 20. Which of the following is most electronegative?
7. Which of the following pairs of atomic numbers represents (a) Lead (b) Silicon
elements belonging to the same group? (c) Carbon (d) Tin
(a) 11 and 20 (b) 12 and 30 21. On going from right to left in a period in the periodic table,
(c) 13 and 31 (d) 14 and 33 the electronegativity of the elements
8. Which of the following sets of atomic number belong to that (a) increases
of alkali metals? (b) decreases
(a) 1, 12, 30, 4, 62 (b) 37, 19, 3, 55 (c) remains unchanged
(c) 9, 17, 35, 53 (d) 12, 20, 56, 88 (d) decreases first then increases
9. What is the atomic number of the next halogen if discovered? 22. The electronegativity follows the order
(a) 85 (b) 117 (a) F > O > Cl > Br (b) F > Cl > Br > O
(c) 167 (d) 104 (c) O > F > Cl > Br (d) Cl > F > O > Br
10. All the elements of a group in the periodic table have the 23. Electronegativity values for the elements help in predicting
same (a) polarity of bonds
(a) atomic weight (b) dipole moments
(b) number of protons (c) valency of elements
(c) mass number (d) position in the electrochemical series
(d) number of electrons for bonding 24. Which of the following is correctly matched?
11. The elements in which 4f orbitals are progressively filled up (a) C—C bond length — 0.077 nm
are called as (b) Ionic radius of Na+ — 0.136 nm
(a) actinides (b) transition elements (c) C—Cl bond length — 0.176 nm
(c) lanthanides (d) halogens (d) Ionic radius of F– — 0.095 nm
12. Which of the following is not a transition metal? 25. The atomic radius increases as we move down in a group
(a) Zr (b) Tc because
(c) Re (d) Np (a) effective nuclear charge increases
13. Which of the following have same number of electrons in (b) atomic mass increases
the outermost orbit? (c) additive electrons are accommodated in new electron
(a) Pb, Sb (b) N, O level
(c) As, Bi (d) P, Ca (d) atomic number increases
14. Representative elements belong to 26. The correct order of radii is
(a) s- and p-blocks (b) p- and d-blocks (a) N < Be < B (b) F - < O 2 - < N 3-
(c) f-block only (d) d- and f-blocks
15. Element with atomic number 56 belongs to which block? (c) N < Li < K (d) Fe 3+ < Fe 2 + < Fe 4+
(a) s (b) p 27. Which of the following is the smallest cation?
(c) d (d) f (a) Na+ (b) Mg2+
(c) Ca 2+ (d) Al3+
16. Which of the following remains unchanged on descending
in a group in the periodic table? 28. The ionic radii of N , O , F and Na+ are in the order
3– 2– –

(a) Valence electrons (b) Atomic size (a) N 3- > O 2 - > F - > Na +
(c) Density (d) Metallic character
17. Which is true about the electronegativity order of the (b) N 3- > Na + > O 2- > F -
following elements? (c) Na + > O 2 - > N 3- > F -
(a) P > Si (b) C > N
(c) Br > Cl (d) Sr > Ca (d) O 2- > F - > Na + > N 3-
Classification of Elements and Periodicity in Properties 67
29. Ionic radii of 41. Highest electron affinity is shown by
(a) Ti 4+ < Mn 2+ (b) 35 -
Cl < 37
Cl - (a) O– (b) F–
(c) Cl2 (d) F2
(c) K + > Cl -1 (d) P 3+ > P 5+ 42. Which of the following has least electron afinity?
30. The radii of F, F–, O and O2– are in the order (a) Oxygen (b) Argon
(a) O 2 - > F - > F > O (c) Nitrogen (d) Boron
43. Which of the following has highest electron affinity?
(b) F - > O 2 - > F > O
(a) Li+ (b) Na+
(c) O 2 - > O > F - > F (c) F– (d) Cl–
(d) O 2 - > F - > O > F 44. The element having very high ionizaiton energy but zero
electron affinity is
31. Which stable nucleus has radius half than that of 54Xe
(a) H (b) F
(a) 40Ca (b) 23Na
(c) He (d) Be
(c) 32S (d) 7Li
45. The lower electron affinity of fluorine than that of chlorine is
32. For which of the following ionic species radius would be
due to
maximum?
(a) smaller size
(a) C4– (b) N3–
2– (b) smaller nuclear charge
(c) O (d) Mg2+
(c) difference in their electronic configurations
33. The decreasing order of the size of the following ions is
(a) Li+ > H+ > H– (b) H+ > H– > Li+ (d) its highest reactivity
(c) H– > Li+ > H+ (d) H– > H+ > Li+ 46. Which of the following orders is correct for the first ionization
potential of B, C and N?
34. Chloride ion and potassium ion are isoelectronic. Then
(b) B > C > N (b) N > C > B
(a) their sizes are same
(c) N > C < B (d) N < C < B
(b) Cl– ion is bigger than K+ ion
47. Consider the following changes
(c) K+ ion is relatively bigger
(d) their sizes depend on either cation or anion A ® A + + e - : E1 and A + ® A 2 + + e - : E 2
35. Which has the smallest size? The energy required to pull out the two electrons are E1 and
(a) Na+ (b) Mg2+ E 2 respectively. The correct relationship between two
(c) Al3+ (d) P5+ energies would be
36. Which of the following is isoelectronic with carbon atom? (a) E1 < E2 (b) E1 = E2
(a) Na+ (b) Al3+ (c) E1 > E2 (d) E1 ³ E2
(c) O2– (d) N+ 48. Which of the following isoelectronic ions has the lowest
37. + ++ 3+ 4+
Na , Mg , Al and Si are isoelectronic. The order of their ionization energy?
ionic size is (a) K+ (b) Ca2+
(a) Na+ > Mg++ < Al3+ < Si4+ (c) Cl– (d) S2–
(b) Na+ < Mg++ > Al3+ > Si4+ 49. The decreasing order of the ionization potential of the
(c) Na+ > Mg++ > Al3+ > Si4+ following elements is
(d) Na+ < Mg++ > Al3+ < Si4+ (a) Ne > Cl > P > S > Al > Mg
38. Electron affinity depends on (b) Ne > Cl > P > S > Mg > Al
(a) atomic size (c) Ne > Cl > S > P > Mg > Al
(b) nuclear charge (d) Ne > Cl > S > P > Al > Mg
(c) atomic number 50. The electronic configuration of elements A, B and C are [He]
(d) atomic size and nuclear charge both 2s1, [Ne] 3s1 and [Ar] 4s1 respectively. Which one of the
39. The element with highest electron affinity among halogens following order is correct for IE1 (in kJ mol–1) of A, B and C?
is (a) A > B > C (b) C > B > A
(a) F (b) Cl (c) B > C > A (d) C > A > B
(c) Br (d) I 51. The set representing the correct order of first ionization
40. The correct order of electron affinity among the following is potential is
(a) F > Cl > Br (b) Br > Cl > F (a) K > Na > Li (b) Be > Mg > Ca
(c) Cl > F > Br (d) F > Br > Cl (c) B > C > N (d) Ge > Si > C
68 Chemistry
52. The correct order of ionization energies is 62. Consider the following statements
(a) Zn < Cd < Hg (b) Hg < Cd < Zn I. The radius of an anion is larger than that of the parent
(c) Ar > Ne > He (d) Cs < Rb < Na atom.
53. Which of the order for ionization energy is correct? II. The ionization energy generally increases with increasing
(a) Be < B < C < N < O (b) B < Be < C < O < N atomic number in a period.
(c) Be > B > C > N > O (d) B < Be < N < C < O III. The electronegativity of an element is the tendency of
an isolated atom to attract an electron.
54. First ionization potential will be maximum for
Which of the above statements is/are correct?
(a) Uranium (b) Hydrogen
(c) Lithium (d) Iron (a) I alone (b) II alone
55. The incorrect statement among the following is (c) I and II (d) II and III
(a) the first ionization potential of Al is less than the first 63. Which of the following order is wrong?
ionization potential of Mg (a) NH 3 < PH 3 < AsH 3 — Acidic
(b) the second ionization potential of Mg is greater than the
second ionization potential of Na (b) Li < Be < B < C — IE1
(c) the first ionization potential of Na is less than the first (c) Al 2 O3 < MgO < Na 2 O < K 2 O — Basic
ionization potential of Mg
(d) the third ionization potential of Mg is greater than the (d) Li + < Na + < K + < Cs + — Ionic radius
third ionization potential of Al. 64. Gradual addition of electronic shells in the noble gases causes
56. The second ionization potential of an element M is the energy a decrease in their
required to (a) ionization energy (b) atomic radius
(a) remove one mole of electrons from one mole of gaseous (c) boiling point (d) density
cations of the element
65. Which of the following elements represents highly
(b) remove one mole of electrons from one mole of gaseous electropositive as well as highly electronegative character in
anions its period?
(c) remove one mole of electrons from one mole of (a) Hydrogen (b) Nitrogen
monovalent gaseous cations of the element
(c) Fluorine (d) None of these
(d) remove 2 moles of electrons from one mole of gaseous
atoms 66. The cause of diagonal relationship is
57. Highest ionization potential is shown by (a) similar electronegativities
(a) Alkali metals (b) Transition elements (b) similar ionic or atomic radii
(c) Halogens (d) Inert gases (c) similar polarizing power of ions
58. With reference to ionization potential which one of the (d) All the three
following sets is correct? 67. Variable valency is generally exhibited by
(a) Li > K < Cs (b) B > Li > K
(a) representative elements
(c) Cs > Li > B (d) Cs < Li < K
(b) transition elements
59. Which of the following species has lowest ionization
potential? (c) non-metallic elements
(a) O (b) O2 (d) metallic elements
(c) O2 + (d) O2– 68. Which of the following elements does not belong to first
60. The factor not affecting the ionization energy is transition series?
(a) size of atom (a) Fe (b) V
(b) charge in the nucleus (c) Ag (d) Cu
(c) type of bonding in the crystalline lattice 69. In the periodic table, with the increase in atomic number, the
(d) type of electron involved metallic character of an element
61. The screening effect of inner electrons of the nucleus causes (a) decreases in a period and increases in a group
(a) decrease in the ionization energy (b) increases in a period and decreases in a group
(b) increase in the ionization energy
(c) increases both in a period and the group
(c) no effect on the ionization energy
(d) decreases both in a period and the group
(d) increases the attraction of the nucleus for the electrons
Classification of Elements and Periodicity in Properties 69

1. Identify the correct order of the size of the following: 11. Which one of the following ions has the highest value of
(a) Ca2+ < K+ < Ar < Cl– < S2– [CBSE-PMT 2007] ionic radius ? [AIEEE 2004]
2+ + – 2– (a) O2– (b) B3+
(b) Ar < Ca < K < Cl < S
(c) Li+ (d) F–
(c) Ca2+ < Ar < K+ < Cl– < S2– 12. Among Al2O3, SiO2, P2O3 and SO2 the correct order of acid
(d) Ca2+ < K+ < Ar < S2– < Cl– strength is [AIEEE 2004]
2. In which of the following electronic configuration an atom (a) Al2O3 < SiO2< SO2 < P2O3
has the lowest ionisation enthalpy? [CBSE-PMT 2007] (b) SiO2< SO2 < Al2O3 < P2O3
2 2
(a) 1s 2s 2p 3 (b) 1s 2s 2p5 3s1
2 2 (c) SO2< P2O3 < SiO2 < Al2O3
2 2 6 (d) Al2O3 < SiO2< P2O3 < SO2
(c) 1s 2s 2p (d) 1s2 2s2 2p5 13. Beyllium and aluminium exhibit many properties which are
3. Which one of the following ionic species has the greatest similar. But, the two elements differ in [AIEEE 2004]
proton affinity to form stable compound? [CBSE-PMT 2007] (a) forming covalent halides
(a) NH -2 (b) F – (b) forming polymeric hydrides
(c) exhibiting maximum covalency in compounds
(c) I– (d) HS–
(d) exhibiting amphoteric nature in their oxides
4. The stability of + 1 oxidation state increases in the sequence: 14. Which of the following oxides is amphoteric in character?
[CBSE-PMT 2009] (a) SnO 2 (b) SiO 2 [AIEEE 2005]
(a) Tl < In < Ga < Al (b) In < Tl < Ga < Al
(c) CO 2 (d) CaO
(c) Ga < In < Al < Tl (d) Al < Ga < In < Tl
15. In which of the following arrangements, the order is NOT
5. Amongst the elements with following electronic
according to the property indicated against it?
configurations, which one of them may have the highest (a) Li < Na < K < Rb : [AIEEE 2005]
ionization energy? [CBSE-PMT 2009] Increasing metallic radius
2
(a) Ne [3s 3p ] 2 (b) Ar [3d 4s24p3 ]
10
(b) I < Br < F < Cl :
2
(c) Ne [3s 3p ] 1 (d) Ne [3s23p3] Increasing electron gain enthalpy
6. Which of the following represents the correct order of (with negative sign)
increasing electron gain enthalpy with negative sign for the (c) B < C < N < O
Increasing first ionization enthalpy
elements O, S, F and Cl ? [CBSE-PMT 2010]
(a) Cl < F < O < S (b) O < S < F < Cl (d) Al3+ < Mg 2+ < Na + < F -
(c) F < S < O < Cl (d) S < O < Cl < F Increasing ionic size
7. Among the elements Ca, Mg, P and Cl, the order of increasing 16. The increasing order of the first ionization
enthalpies of the elements B, P, S and F (Lowest first)
atomic radii is : [CBSE-PMT 2010]
is [AIEEE 2006]
(a) Ca < Mg < P < Cl (b) Mg < Ca < Cl < P (a) B < P < S < F (b) B < S < P < F
(c) Cl < P < Mg < Ca (d) P < Cl < Ca < Mg (c) F < S < P < B (d) P < S < B < F
8. What is the value of electron gain enthalpy of Na+ if IE1 of 17. Following statements regarding the periodic trends of
Na = 5.1 eV ? [CBSE-PMT 2011 M] chemical reactivity of the alkali metals and the halogens are
(a) –5.1 eV (b) –10.2 eV given. Which of these statements gives the correct picture?
[AIEEE 2006]
(c) +2.55 eV (d) +10.2 eV
(a) Chemical reactivity increases with increase in atomic
9. According to the Periodic Law of elements, the variation in number down the group in both the alkali metals and
properties of elements is related to their [AIEEE 2003] halogens
(a) nuclear masses (b) In alkali metals the reactivity increases but in the halogens
(b) atomic numbers it decreases with increase in atomic number down the
(c) nuclear neutron-proton number ratios group
(d) atomic masses (c) The reactivity decreases in the alkali metals but increases
10. Which is the correct order of ionic sizes (At. No. : Ce = 58, in the halogens with increase in atomic number down the
group
Sn = 50, Yb = 70 and Lu = 71) ? [AIEEE 2003]
(d) In both the alkali metals and the halogens the chemical
(a) Ce > Sn > Yb > Lu (b) Sn > Ce > Yb > Lu reactivity decreases with increase in atomic number down
(c) Lu > Yb > Sn > Ce (d) Sn > Yb > Ce > Lu the group
70 Chemistry
18. The charge/size ratio of a cation determines its (a) Al3+ > Mg 2+ > Na + > F- > O 2 -
polarizing power. Which one of the following sequences (b) Na + > Mg 2+ > Al3+ > O2- > F-
represents the increasing order of the polarizing power of (c) Na + > F - > Mg 2 + > O 2 - > Al3+
the cationic species, K+, Ca2+, Mg2+, Be2+? [AIEEE 2007]
(d) O2- > F- > Na + > Mg 2+ > Al3+
(a) Ca2+ < Mg2+ < Be+ < K+ 21. The correct order of electron gain enthalpy with negative
(b) Mg2+ < Be2+ < K+ < Ca2+ sign of F, Cl, Br and I, having atomic number 9, 17, 35 and 53
(c) Be2+ < K+ < Ca2+ < Mg2+ respectively, is : [AIEEE 2011 RS]
(d) K+ < Ca2+ < Mg2+ < Be2+. (a) F > Cl > Br > I (b) Cl > F > Br > I
19. In which of the following arrangements, the sequence is not (c) Br > Cl > I > F (d) I > Br > Cl > F
strictly according to the property written against it? 22. The increasing order of the ionic radii of the given
[AIEEE 2008] isoelectronic species is : [AIEEE 2012]
(a) HF < HCl < HBr , HI : increasing acid strength (a) Cl–, Ca2+ , K+, S2– (b) S2–, Cl–, Ca2+ , K+
(b) NH3 < PH3 < AsH3 <SbH3 : increasing basic strength (c) Ca2+ , K+, Cl–, S2– (d) K+, S2–, Ca2+, Cl–
(c) B < C < O < N : increasing first ionization enthalpy 23. Which of the following represents the correct order of
(d) CO2 < SiO2 < SnO2 < PbO2 : increasing oxidising power increasing first ionization enthalpy for Ca, Ba, S, Se and Ar ?
20. The correct sequence which shows decreasing order of the [JEE M 2013]
ionic radii of the elements is [AIEEE 2010] (a) Ca < S < Ba < Se < Ar (b) S < Se < Ca < Ba < Ar
(c) Ba < Ca < Se < S < Ar (d) Ca < Ba < S < Se < Ar

1. An element X occurs in short period having configuration 8. The valence shell of element A contains 3 electrons while the
ns2 np1. The formula and nature of its oxide is valence shell of element B contains 6 electrons . If
(a) XO3, basic (b) XO3 acidic A combines with B, the probable formula of the compound
(c) X2O3, amphoteric (d) X2O3 basic formed will be
2. The law of triads is applicable to a group of (a) AB2 (b) A2B
(a) Cl, Br, I (b) C, N, O
(c) A2B3 (d) A3B2
(c) Na, K, Rb (d) H, O, N
9. Pauling scale of electronegativity of elements helps to
3. Elements of IA group give flame colour due to
determine
(a) low IP
(a) covalent nature of an element
(b) low m.pt.
(c) softness (b) position of an element in EMF series
(d) one electron in outermost shell. (c) dipole moment of molecules
4. The ionic radii of N3–, O2– and F– are respectively given by (d) polarity of bond.
(a) 1.36, 1.40, 1.71 (b) 1.36, 1.71, 1.40 10. The statement that is not correct for the periodic classification
(c) 1.71, 1.40, 1.36 (d) 1.71, 1.36, 1.40 of elements is
5. The first ionisation potential of aluminium is smaller than (a) The properties of elements are the periodic functions of
that of magnesium because their atomic number
(a) Atomic size of Al > Atomic size of Mg. (b) Non-metallic elements are lesser in number than metallic
(b) Atomic size of Al < Atomic size of Mg. elements
(c) Al has one electron in p - orbital (c) The first ionization energies of elements along a period
(d) None of these do not vary in a regular manner with increase in atomic
6. Which of the following halides is not oxidized by MnO2 number.
(a) F– (b) Cl– (d) For transition elements the d-sub shells are filled with
electrons monotonically with increase in atomic number
(c) Br– (d) I–
11. Which is coloured ion
7. Which species has the maximum ionic radius
(a) Na+ (b) O2– (a) [Cu ( NH 3 ) 4 ]+ (b) [Cu( NH 3 ) 4 ]2 +
(c) F– (d) Mg2+
(c) [ Zn(H 2 O) 6 ]2 + (d) [Ca (H 2 O) 6 ]2 +
Classification of Elements and Periodicity in Properties 71
12. Considering the elements from left to right in the second 17. Which transition involves maximum amount of energy
period of the periodic table, the gram atomic volume of the
elements. (a) M - (g ) ¾
¾® M (g ) + e
(a) first increases then decreases
(b) M - ( g ) ¾
¾® M + (g ) + 2e
(b) decreases
(c) increases at constant rate (c) M + (g ) ¾
¾® M 2+ (g ) + e
(d) remains unchanged
(d) M 2 + (g ) ¾
¾® M 3+ (g ) + e
13. Atomic radii of fluorine and neon in angstrom units are
respectively given by 18. A sudden jump between the values of second and third
(a) 0.72 ; 1.60 (b) 1.60 ; 1.60 ionization energies of an element would be associated with
(c) 0.72 ; 0.72 (d) None of these the electronic configuration
14. The screening effect of ‘d’ electrons is (a) 1s 2 2s 2 2p 6 3s1 (b) 1s 2 2s 2 2p 6 3s 2 3p1
(a) much less than s- electrons
(c) 1s 2 2s 2 2p 6 3s 2 3p 2 (d) 1s 2 2s 2 2p 6 3s 2
(b) much more than s- electrons
(c) equal to s- electrons 19. The element with outer electronic configuration 3d 6 4s 2 is a
(d) equal to p- electrons (a) metalloid (b) non-metal
15. An element having electronic configur ation (c) transition metal (d) noble gas
1s 2 2s 2 2p 6 3s 2 3p 6 4s1 forms 20. In the process, Cl(g) e – H
Cl (g); H is
(a) acidic oxide (b) basic oxide (a) positive (b) negative
(c) amphoteric oxide (d) neutral oxide (c) zero (d) unpredictable
16. Which pair of elements belongs to same group 21. Among the following the lowest degree of paramagnetism
per mole of the compound at 298 K will be shown by
(a) Elements with atomic no. 17 and 38
(b) Elements with atomic no. 20 and 40 (a) MnSO 4 .4H 2O (b) CuSO 4 .5H 2O
(c) Elements with atomic no. 17 and 53 (c) FeSO 4 .6H 2 O (d) NiSO 4 .6H 2 O
(d) Elements with atomic no. 11 and 33
72 Chemistry

EXERCISE 1 23. (a) Higher the value of electronegativity difference, more


1. Mg2+ polar is the bond.
3. Cs, He. 24. (c) 25. (c) 26. (b) 27. (d)
4. It lies in the 4th period and III B (or 13th) group. 28. (a)
5. Electron affinity : A + e–® A– 29. (d) P5+ has more effective nuclear charge and smaller size than
Reverse of ionization potential; A+ + e–® A P3+.
8. (a) Decrease in size of atom and increase in nuclear charge.
30. (d) 31. (c)
(b) Increase in atomic size.
9. NaF > NaCl > NaBr > NaI high melting point is due to high 32. (a) Among the isoelectronic species, the anion having more
lattice energy. negative charge would be the largest.
10. (a) III is non-metal. 33. (c) 34. (b) 35. (d) 36 (d)
(b) I is an alkali metal. 37. (c) Among the isoelectronic species, the cation having more
(c) II is alkaline earth metal. positive charge would be the smaller in size.
11. Na 2O H2O 2NaOH 38 (d) 39. (b) 40. (c)
Sod. oxide Sod. hydroxide
41. (a) Due to inert gas configuration molecules F2, Cl2 and anion
Cl2O7 + H2O ¾¾
® 2HClO4 F– have almost zero electron affinity.
perchloric acid
42. (b)
12. (i) Highest ionisation enthalpy element V Lowest
43. (a) Addition of an electron to F– and Cl– ions will involve
ionisation enthalpy element IV
repulsions whereas in case of Na + ion, hence EA of Li+ is
(ii) The correct order of increasing electron gain enthalpy
highest among these.
is
V < III < IV < II < I. 44. (c) He has zero EA because of its completely filled subshells
13. (a) AlP (aluminum phosphide) (1s2 2s2).
(b) Li3N (lithium nitride) 45. (a) Since F has small size and its relatively compact
(c) CaS (calcium sulphide) 2 p-subshell’s electron-electron repulsion, do not allow
18. (d) 19. (c) 20. (a) 21. (d) 22. (a) the addition of an extra electron whereas Cl has bigger
23. (c) 24. (c) 25. (d) 26. (b) 27. (b) size than F, allows the addition of an extra electron more
28. (b) easily.
EXERCISE 2 46. (b) Due to stable electronic configuration and effective
nuclear charge. IE1 of N > C> B
1. (b) 2. (a) 3. (a) 4. (b)
5. (d) 6. (b) 7. (c) 8. (b) 47. (a) IE1 is always less than IE2.
9. (b) The atomic number 85 lies in the sixth period and since 48. (d) Amongst the isoelectronic species, the size of S2– is
the next halogen, if discovered, will lie in the 7th period. largest hence its IE will be the lowest.
Hence, its atomic number must be 85+32=117. 49. (b) Closed shell (Ne), half filled (P) and completely filled
10. (d) All the elements in a group have the same valence shell configuration (Mg) are the cause of higher value of I.E.
electrons. 50. (a) 51. (b) 52. (d) 53. (b)
11. (c) 54. (b) Due to presence of most penetrating one s-electron,
12. (d) N P is an-inner-transition element. hydrogen (1s) shows maximum IP out of list.
13. (c) 14. (a) 15. (a) 16. (a) 55. (b) IE2 of Mg is lower than that of Na because in case of
17. (a) 18. (c) 19. (b) 20. (c) Mg+, 3s-electron has to be removed where as in case of
21. (b) Na+, an electron is removed from the stable inert gas
22. (a) F and O belong to 2nd period whereas Cl and Br belong configuration which is difficult.
to 3rd and 4th periods respectively. Hence the sequence 56. (c)
of the E.N. is F > O > Cl > Br
Classification of Elements and Periodicity in Properties 73
57. (d) Inert gases due to completely filled and stable. s 2p6 11. (a) The ionic radii follows the order
configuration show highest I.E.
O2- > F- > Li + > B3+
58. (b)
59. (d) The additional p -molecular electron in O2– is added to 12. (d) SO 2 > P2 O3 > SiO 2 > Al 2 O 3
Acidic Weak acidic Amphoteric
the less stable antibonding orbital. Hence, it is easy to
remove that electron from O2– ion. Thus I.E. (I.P.) of O2– 13. (c) The valency of beryllium is +2 while that of aluminium is
is lowest. +3
60. (c) 14. (a) CaO is basic as it form strong base Ca(OH) 2 on reaction
61. (a) Higher the screening effect, lower is the I.E. with water.
62. (c) CaO + H2O –––––® Ca(OH)2
63. (b) The right sequence of I.E1 of Li < B < Be < C. CO2 is acidic as it dissolve in water forming unstable
64. (a) carbonic acid.
65. (a) First period has H and He only out of which He is inert, H2O + CO2 –––––® H2CO3
hence, H behaves as a highly electropositive as well as Silica (SiO2) is insoluble in water and acts as a very
electronegative. weak acid.
66. (c) 67. (b) 68. (c) 69. (a) SnO2 is amphoteric as it reacts with both acid and
EXERCISE 3 base.
1. (a) For isoelectronic species, size of anion increases as SnO2 + 2H2SO4–––––® Sn(SO4)2 + 2H2O
negative charge increases whereas size of cation SnO2 + 2KOH–––––® K2SnO3 + H2O
decreases with increase in positive charge. Further ionic 15. (c) In a period the value of ionisation potential increases
radii of anions is more than that of cations. Thus the from left to right with breaks where the atoms have some
correct order is what stable configuration. In this case N has half filled
Ca++ < K+ < Ar < Cl– < S– – stable orbital. Hence has highest ionisation energy. Thus
the correct order is
2. (b)
B < C < O < N not B < C < N < O
3. (a) Proton affinity decreases in moving across the period
16. (b) On moving along a period ionization enthalpy
from left to right due to increase in charge, within a group
increases from left to right and decreases from top to
the proton affinities decreases from top to bottom.
bottom in a group. But this trend breaks in case of atoms
Nitrogen family > Oxygen family > Halogens having fully or half filled stable orbitals. In this case P
4. (d) The stability of +1 oxidation state increases from has a stable orbitals filled electronic configuration hence
aluminium to thallium i.e. its ionisation enthalpy is greater in comparision to S.
Al < Ga < In < Tl Hence the correct order is B < S < P<F.
5. (d) The smaller the atomic size, larger is the value of 17. (b) The alkali metals are highly reactive because their first
ionisation potential. Further the atoms having half filled ionisation potential is very low and hence they have
or fully filled orbitals are comparitively more stable, hence great tendency to loses electron to form unipositive ion.
more energy is required to remove the electron from such On moving down group- I from Li to Cs ionisation
atoms. enthalpy decreases hence the reactivity increases. The
6. (b) O < S < F < Cl halogens are most reactive elements due to their low
bond dissociation energy, high electron affinity and high
Electron gain enthalpy – 141, –200, – 333, – 349 kJ mol–1
enthalpy of hydration of halide ion. However their
7. (c) 12 Mg 15 P 17Cl 20Ca reactivity decreases with increase in atomic number
160p 110 99 197 (pm) 18. (d) Smaller the size and higher the charge more will be
Cl < P < Mg < Ca polarising power of cation. Since the order of the size of
\ Correct choice : (c)
cation is K + > Ca ++ > Mg ++ > Be++ . So the correct
8 (a) IE1 of Na = – Electron gain enthalpy of Na +
order of polarising power is
= – 5.1 eV.
K+ < Ca2+ < Mg2+ < Be2+
9. (b) The properties of elements change with a change in atomic 19. (b) In hydrides of 15th group elements, basic character
number. decreases on descending the group i.e.
10. (b) Correct order of ionic size is Sn > Ce > Yb > Lu.
NH3 > PH3 > AsH3 > SbH3.
74 Chemistry
20. (d) All the given species contains 10 e – each i.e. 6. (a) F2 is strongest oxidising agent. F- is not oxidised by
isoelectronic.
MnO2
For isoelectronic species anion having high negative
charge is largest in size and the cation having high 7. (b) O 2 - has minimum effective nuclear charge, hence has
positive charge is smallest. maximum ionic radius
21. (b) As we move down in a group electron gain enthalpy
8. (c) The element A is ns 2 p1 and B is ns2 p 4 . They can form
becomes less negative because the size of the atom
increases and the distance of added electron from the compound of the type A 2 B3
nucleus increases. Negative electron gain enthalpy of F 9. (d) Pauling scale of electronegativity helps to determine
is less than Cl. This is due to the fact that when an polarity of bond
electron is added to F, the added electron goes to the 10. (d)
smaller n = 2 energy level and experiences significant 11. (b) Due to presence of one unpaired electron
repulsion from the other electrons present in this level.
In Cl, the electron goes to the larger n = 3 energy level (Cu 2+ = 3p 6 d 9 ) the complex [Cu ( NH 3 ) 4 ]2 + is
and consequently occupies a larger region of space coloured
leading to much less electron–electron repulsion. So the 12. (a)
correct order is Cl > F > Br > I. 13. (a) In inert gases we have van der waal’s radii which is always
22. (c) higher than atomic radii
23. (c) On moving along a period from left to right I.E. increases 14. (a) The screening effect follows the order s > p > d > f
and on moving down a group I.E. decreases.
15. (b) It is electronic configuration of alkali metal. Hence it will
hence correct order is : Ba < Ca < Se < S < Ar
form basic oxide
EXERCISE 4 16. (c) Atomic no. 17 (Cl) and 53 (I) are present in the same
group
1. (c) ns2 np1 is the electronic configuration of III A period.
17. (d) The energy involved is ionisation energy (I.E.). Further
Al2O3 is amphoteric oxide
the 3rd ionisation energy will be greater than the 2nd and
2. (a) According to the law of triads the atomic wt of the middle 1st.
element is arithmatic mean of I and III. At wt of Br 18. (d) There is a sudden jump in the value of IP when there is
At.wt of Cl + At wt of I
= change of principal energy level
2
3. (a) Due to low value of IP, valence electrons are. migrated to 19. (c) In 3d 6 4s 2 the differentiating electron enters d sub shell,
higher energy levels Hence it represents transition metal, Fe.
20. (b) The process represents the first electron affinity which
4. (c) The ionic radii must follow the order N 3- > O 2 - > F -
is always exothermic. Hence DH is negative
5. (c) Al ( 3s2 p1) and Mg (3s2). Lower energy is required to 21. (b) Ion = Mn 2+ Cu2+ Fe2+ Ni2+
EC = 3d 5 3d 9 3d6 3d8
remove 3p1 electron than 3s1 electron (penetrating
Number of unpaired
effect is s > p > d > f ) . Secondly Mg has stable electrons = 5 1 4 2
electronic configuration than Al Hence lowest paramagnetism is shown by CuSO4.5H2O
4
Chemical Bonding
and Molecular Structure
CHEMICAL BOND : Only valence electrons are involved in the combilation of
A chemical bond is a sort of attraction which keeps the two atoms two atoms. The representation of valence electrons on an
together. Thus depending upon the variety of force, there may be atom called Lewis Symbol eg.
variety of chemical bonds. (Second Period
Li Be B C N O F Ne
REASONS FOR THE FORMATION OF A CHEMICAL elements)
BOND : TYPES OF CHEMICAL BOND : (By Kossel and Lewis)
(i) Energy concept : When two atoms approach each other, the Depending upon the mode of acquiring the stable electronic
attractive and repulsive forces operate between them. The configuration, the chemical bonds may be
distance at which the attractive forces overweigh the i) Ionic or electrovalent bond
repulsive forces is known as the bond distance, the potential
energy of the system is minimum and the bond is said to be ii) Covalent bond
formed. iii) Coordinate or dative bond
iv) Metallic bond
v) Hydrogen bond
vi) Weak van der Waal's forces of attraction
IONIC BOND (By Kossel and Lewis)
Energy kJ/mol

Ionic bond is formed by the complete transference of one or more


0 valence electrons of one atom to the valence shell of the other
atom. Both atoms are converted into ions and have the electronic
configuration of nearest noble gases. The electrostatic attraction
between these oppositely charged ions, which always tends to
decrease the potential energy of the system is known as the ionic
Internuclear distance bond. Consider the formation of KCl. The electronic configuration
Bond length of K, Cl and their ions (K+ and Cl–) are given below
K (19) 1s2, 2s2, 2p6, 3s2, 3p6, 4s1
(ii) Lewis and Langmuir (octet rule) : Concept of stable K+(18) 1s2, 2s2, 2p6, 3s2, 3p6 Inert gas (Ar) configuration
electronic configuration.
Cl (17) 1s2, 2s2, 2p6, 3s2, 3p5
Atoms enter into chemical bonding to acquire the stable Cl– (18) 1s2, 2s2, 2p6, 3s2, 3p6 Inert gas (Ar) configuration
inert gas electronic configuration. They can do so by losing,
K+ +Cl– KCl
gaining or shairing of electrons.
The number of electrons lost or gained by an atom represent the
Lewis symbols : The electrons present in the outermost
energy level of an atoms and known as valence electrons. electrovalency of the atom.
76 Chemistry
FACTORS AFFECTING THE FORMATION OF IONIC Where q1 and q2 are the charges of ions in coulombs, 4 0 is the
BOND : permitivity factor and r is the distance between ions. Hence lattice
There are three main factors energy depends upon
(i) Ionisation energy (I) : The lower the value of the ionisation
i) Charge on ions : The higher the charge on ions, the greater
energy of an atom greater will be the ease of formation of
will be the force of attraction between them and greater will
cation from it.
be the lattice energy. The order of lattice energy for different
Note: the size of the cation is always smaller than the atom
solids is as follows
from which it is derived
(ii) Electron affinity (E) : The higher the electron affinity of an bi - bivalent uni - bivalent uni - univalent
> >
atom the greater will be the ease of formation of anion from it. solids solids solids
The size of the anion is always larger than the atom from MgO > CaCl2 > NaCl
which it is derived.
ii) Size of ions : The smaller the size of the ions, the lesser will
(iii) Lattice structure : The electrostatic field of cations and
be distance between them, the greater will be the force of
anions extends in all directions, their union is not limited to
form a single molecule, rather a cluster of ions, having three attraction and so greater will be the lattice energy.
dimensional orderly arrangement, known as the lattice FAJAN’S RULE :
structure is formed. This rule is for covalent character of an ionic bond. Covalent
(iv) Lattice energy (U) : The amount of heat evolved when one character of an ionic bond is favoured by
mole of ionic compound is formed from positive and negative
i) Small positive ion ii) Large negative ion
ions in the crystalline form is known as the lattice energy.
iii) Large charge on ions
BORN HABER CYCLE :
Thus for a fixed cation, the larger the size of anion, the more the
The formation of ionic compounds in terms of energy can be seen
magnitude of the charge, the more is covalent character eg.
by Born Haber cycle eg. formation of NaCl is as follows
covalent character of sodium halides follows the order
sublimation ionisation
Na(s) Na(g) Na (g) + e
+
NaI > NaBr > NaCl > NaF
+ S + I
For fixed anion, the smaller the size of cation, the more the
1 Dissociation Addition of e magnitude of the charge, the more is the covalent character eg.
Cl2 (g) Cl(g) Cl (g)
2 1 E BeCl2 > MgCl2 > CaCl2 > SrCl2 > BaCl2
+ D
2 It has been observed that a cation having 18 (s2p6d10) electrons
crystal formation in outermost shell (pseudo noble gas configuration) can polarise
Na ( g ) + Cl (g )
+
NaCl (s) the anion more than cation having 8 (s2p6) noble gas electronic
U
where S = Heat of sublimation of Na metal configuration. Hence CuCl is more covalent than NaCl. Similarly
I = Ionisation energy of Na AgCl is more covalent than KCl.
D = Heat of dissociation of molecular chlorine POLARISING POWER OF CATION :
E = Electron affinity of chlorine atom It is generally represented by and also known as ionic potential
U = Lattice energy of NaCl
charge density. It can be represented as
The amount of heat liberated (Q) in the overall reaction is the heat
of formation of NaCl. charge on cation
Thus Q = S + I + 1/2 D – E – U =
radius of cation
(The negative sign indicates heat is evolved or liberated)
The important energy terms are I, E and U. The more the negative value POLARISATION :
of Q, the greater will be the stability of ionic compound. Hence formation When cation and anion come closer to each other, the electron
of ionic compound is favoured by cloud of anion is attracted towards the cation, some partial sharing
(i) Low ionisation energy (I) of the metal of electrons take place, the anion is distorted and the effect is
(ii) High electron affinity (E) of the non-metal. known as polarisation.
(iii) Higher lattice energy (U) of the compound
In general the alkali metals have low values of ionisation
energy and halogens have high values of electron affinity. +
Hence they form the stable ionic compounds.
More the effect of polarisation, more is the sharing of electrons and
FACTORS AFFECTING LATTICE ENERGY (U) : the more is covalent character of ionic bond.
The force of attraction between two oppositely charged ions is PROPERTIES OF IONIC COMPOUNDS :
q 1q 2 The important properties are as follows
given by well known Coulombs law as, F = 2
(4 0 )r i) Crystal structure : The crystalline ionic compounds have
well defined crystal structure or crystal lattice eg in NaCl
Chemical B onding and Molecular Structure 77
each Na+ is surrounded by six Cl- ions and vice versa COVALENT BOND : (By lewis and Langley)
× According to G. N. Lewis atoms may also combine by sharing of
× × electrons present in their outermost shells and attain noble gas
electronic configuration. One shared pair of electrons constitute
× a single bond, two electron pairs constitute a double bond and so
× × × Na
+ on. The bonds thus formed ore known as covalent bonds.
× COVALENCY :
× × –
Cl
× It is the number of electron pairs shared by one atom of the element
in combination with other atoms in a molecule.
× × Variable Covalency :
× Generally the covalency of an element is equal to the total number
Cubical structure of sodium chloride crystal of unpaired electrons in s– and p– orbitals of the valence shell.
ii) Melting and boiling points : ionic compounds have high For example
m.pt and b.pt due to powerful electrostatic force between
6C = 1s 2 , 2s1, 2 p1x , 2 p1y , 2 p1z covalency of carbon = 4
ions.
iii) Solubility : They are generally soluble in polar solvents
(having high value of dielectric constant). The solubility of 8O = 1s 2 , 2s 2 , 2 p 2 x , 2 p1 y , 2 p1z covalency of oxygen = 2
ionic compounds decreases with increase in covalent The variable covalency is shown by elements having vacant d-
character of ionic compounds. It is also governed by orbitals in their valence shell. The unpairing of the s- and p-
a) Lattice energy : More the lattice energy, lesser is the electrons is possible by promoting them to d- orbitals. For example
solubility, eg. sulphates and phosphates of Ba and Sr
Ground state Covalency of sulphur = 2
are insoluble in water due to high lattice energy.
b) Heat of hydration : More the heat of hydration, more is 1s 2 , 2s 2 , 2 p6 , 3s 2 , 3 p 2 x , 3 p1y , 3 p1z
16 S =
the solubility. eg AlCl3 though covalent in nature is
soluble in water due to high value of heat of hydration. First excited state Covalency of sulphur = 4
iv) Electrical conductivity : In solid state they do not conduct
= 3 s 2 , 3 p1 x , 3 p 1 y , 3 p1 z , 3 d 1 xy
electricity since there is no free movement of electrons but in
molten state and in solution they conduct electricity. Second excited state Covalency of sulphur = 6
v) Isomorphism - NaF and MgO are isomorphous due to similar
electronic structure = 3s1, 3 p1x , 3 p1 y , 3 p1z , 3d 1xy , 3d1 yz

Na +
F Mg + +
O Similarly, other than fluorine, the halogens have high covalencies
2,8 2,8 2,8 2,8 equal to 3, 5 and 7. Phosphorous shows covalencies equal to 3
and 5.
Similarly K2S and CaCl2 are isomorphous
K+ S2- K+ Cl- Ca++ Cl- The elements having no d-orbitals do not exhibit variable
2,8,8 2,8,8 2,8,8 2,8,8 2,8,8 2,8,8 covalency
VARIABLE ELECTROVALENCY : EXAMPLES OF COVALENT COMPOUNDS :
It is due to unstable atomic core and inert pair effect. When valence (i) Chlorine (Cl2)
electrons from a metal atom are removed it leaves behind the
atomic core or atomic kernel. The latter with 2 or 8 electrons are
Cl + Cl Cl Cl
stable. In case of transition elements the atomic cores are not very
stable. They may lose one or more electrons thus exhibiting 8e

8e

variable electrovalency eg. iron and copper etc. (ii) Carbon dioxide (CO2)
Fe (2, 8, 14, 2) 1s2, 2s2p6, 3s2p6 d6, 4s2
++
Fe (2, 8, 14) 1s2, 2s2p6, 3s2p6 d6
+++
Fe (2, 8, 13) 1s2, 2s2p6, 3s2p6 d5 O C O O = C =O
The energy difference between 4s and 3d subshell is not very – –
large. 8e 8e 8e
(iii) Nitrogen (N2)
INERT PAIR EFFECT :
The reluctance of s electron pair of some heavy elements
(p - block elements present at the bottom of group) to take part in N N N N
bonding is known as inert pair effect. Thus Tl (6s2, 6p1) show +1
– –
electrovalence. Similarly Sn(5s2p2) and Pb (6s2p2) show + 2 8e 8e
oxidation state commonly. In such elements the lower oxidation
state is more stable than higher oxidation state.
78 Chemistry
NATURE OF COVALENT BOND :
+ + Addition
The nature of covalent bond is explained by (a) + 1s bonding
+
(i) Heitler - london theory Valence bond theory 1s + 1s
(ii) Pauling - slater theory
(iii) Hund - mulliken theory } Molecular orbital theory + • Subtraction
(antibonding)

+ + – 1s
VALENCE BOND THEORY :
1s – 1s
A covalent bond is formed by the overlapping between two half
filled atomic orbitals having electrons with opposite spin. (b) p x p x or p x p x and p y p y or p y p y overlap to form

i) Sigma bond ( bond) - The following overlappings result in bond.


the formation of sigma bond px + px
a) s - s overlapping
b) s - p overlapping
c) p - p head on overlapping
ii) pi - bond ( - bond) - It is formed by the sidewise or lateral Addition
overlapping between unhybridised p - atomic orbitals
p - p side wise or lateral overlapping. px
• bond is a weaker bond than - bond. bonding
• The strength of bond depends upon the extent of
overlapping between atomic orbitals. Greater the
Subtraction
overlapping stronger the - bond is. It follows the –
following order
s–s>s–p>p–p
px
• Single bond is always bond. antibonding
• In multiple bonds only one is bond, others are -
bonds
• - bond is not formed by hybrid orbitals. (c) pz + pz form bond and their combination is according to
inter nuclear distance.
LIMITATIONS OF VALENCE BOND THEORY :
i) It fails to explain the magnetic properties of some molecules
Addition
ii) Bonding in electron deficient compounds.
MO LECULAR ORBI TAL THEO RY O R HUND - pz pz
MULLIKEN THEORY :
According to this theory the atomic orbitals combine to form the
pz
molecular orbitals. The number of molecular orbitals formed are
pz pz
equal to the number of atomic orbitals involved and they belong Subtraction
to the molecule. –
i) The molecular orbitals are formed by LCAO method (linear
combination of atomic orbitals) i.e. by addition or subtraction
of wave functions of individual orbitals thus
MO = A ± B ENERGY LEVEL DIAGRAM :
b = A + B When two atoms A and B are brought nearer to one another, the
a = A – B atomic orbitals present in atom A undergo overlapping with atomic
ii) Molecular orbital of lower energy is known as bonding orbitals present in atom B, having the same energy and molecular
molecular orbital and of higher energy is known as orbitals are formed. There are two types of energy level diagrams
antibonding molecular orbital. i) for molecules upto N2 (here the energy difference between
iii) Molecular orbitals are characterised by a set of quantum 2s and 2p orbitals is small)
numbers. ii) for molecules after N2 (here the energy difference between
2s and 2p orbitals is large and they cannot interact)
iv) Aufbau rule, Pauli's exclusion principle and Hund's rule are
applicable to molecular orbitals.
v) Their shape is governed by the shape of atomic orbitals eg
by s - s and p - p overlapping we have
Chemical B onding and Molecular Structure 79
ELECTRONIC CONFIGURATION / BOND ORDER
* (2Pz) OF SIMPLE DIATOMIC MOLECULES :
2p 2p The electronic configuration and the bond order in case of simple
p*(2Px) p*(2Py) diatomic molecules can be obtained by filling the molecular orbitals
p(2Px) p (2Py) by applying Aufbau principle and Hund's rule
1
Bond order = [Number of bonding electrons - Number of
2
antibonding electrons]
(2Pz)
The magnetic properties of molecules can also be ascertained
2s
2s BONDING IN SOME DIATOMIC MOLECULES AND
*(2s)
IONS :
(a) Hydrogen molecule (H 2 ) - Total number of
electrons = 2, filling in molecular orbitals we have
(2s) 1s 2
1s 1s
*(1s) ( N b - Na )
2-0
=
=
2
=1
Bond order =
2 2 2
(1s)
Hence there is a single bond between two hydrogen
atoms (H - H). Since there is no unpaired electron, H2 is
diamagnetic
Atomic Molecular Atomic
(b) Helium molecule (He2) - The total number of electrons = 4
orbitals orbitals orbitals
and filling in molecular orbitals we have
Molecular orbital energy level diagram for *
molecules after N2 1s 2 < 1s 2
(Nb - Na ) 2 - 2
Bond order = = =0
2 2
*(2Pz) Hence He2 molecule cannot exist
2p 2p (c) Nitrogen molecule (N2) - The total number of electrons = 14
p*(2Px) p*(2Py) and filling in molecular orbitals we have
(2pz) * *
1s 2 < 1s 2 < 2s 2 < 2s 2 < 2p2x = 2p 2y < 2p2z

(Nb - Na ) 10 - 4 6
Bond order = = = = 3 \N º N
2 2 2
p (2Px) p (2Py)
(d) Oxygen molecule (O2) - Total number of electrons = 16
2s and electronic configuration is
2s
*(2s) 1s2 < 1s 2 < 2s 2 < 2s 2 < 2p2z
(2s) 2 * 1 * 1
< 2p 2x = 2py < 2px = 2py
N b - N a 10 - 6 4
Bond order = = = =2
1s 1s 2 2 2
*(1s) As shown by electronic configuration the O2 molecule
contains two unpaired electrons, hence it is paramagnetic
in nature

(1s) (e) O +2 ion - Total number of electrons (16 – 1) = 15.


AOs MOs AOs
Electronic configuration
1s2 < 1s 2 < 2s 2 < 2s 2 < 2p2z
Molecular orbital energy level diagram for
molecules upto N2 2 * 1
Simply we can write < 2p 2x = 2p y < 2px

s1s < s*1s < s2s < s* 2s < s2p z < [p2p x = p2p y ] N b - Na 10 - 5 5 1
Bond order = = =2
2 2 2 2
< [p* 2p x = p* 2p y ] < s* 2p z It is paramagnetic
80 Chemistry
(h) Carbon monoxide (CO) - Total number of electrons = (6 + 8)
(f) O2– (Super oxide ion) Total number of electrons (16 + 1) = 17 .
= 14
Electronic configuration
The electronic configuration is
1s2 < 1s 2 < 2s 2 < 2s 2 < 2p2z
1s2 < 1s2 < 2s2 < 2s2 < 2p2z < 2p2x = 2p2y
< 2p2x = 2p2y < 2p2x 2p1y
Bond order Nb Na 10 4 6
Bond order = = = = 3
2 2 2
(Nb Na ) 10 7 3 1
= 1 It is paramagnetic
=
2
=
2
=
2 2 It is diamagnetic
(i) Nitric Oxide (NO) - Total number of electrons = (7 + 8) = 15
(g) Peroxide ion O 22 - Total number of electrons
1s2 < 1s2 < 2s2 < 2s2 < 2p2z <
(16 + 2) = 18 . The electronic configuration is

1s2 < 1s 2 < 2s 2 < 2s 2 < 2p2z < 2p 2x 2p2x = 2p2y < 2p1x

= 2p 2y < 2p 2x = 2p2y Nb Na 10 5 5 1
Bond order = = = = 2
2 2 2 2
Nb Na 10 8 2
Bond order = = = 1
2 2 2 It is paramagnetic.
It is diamagnetic

TABLE : M.O. CONFIGURATION OF SOME MOLECULES


Molecule total no. MO configuration Bond Magnetic
or ion of electrons order behaviour
He2 4 ( 1s 2 ), ( *1s) 2 0 Diamagnetic
2 2 2 2 2
N2 14 KK ( 2s) ( * 2s) ( 2 p x ) ( 2 p y ) ( 2p z ) 3 Diamagnetic

O2 16 KK ( 2s) 2 ( * 2s) 2 ( 2p z ) 2 ( 2 p x ) 2 ( 2 p y ) 2 2 Paramagnetic

( * 2p x )1 ( * 2p y )1

O2+ 15 Remove one electron from * 2 p y from O2 2.5 Paramagnetic

O2- 17 KK ( 2s) 2 ( * 2s) 2 ( 2p z ) 2 ( 2 p x ) 2 ( 2 p y ) 2 1.5 Paramagnetic

( * 2 p x ) 2 ( * 2 p y )1

O22- 18 KK ( 2s) 2 ( * 2s) 2 ( 2p z ) 2 ( 2 p x ) 2 ( 2 p y ) 2 1 Diamagnetic


2 2
( * 2p x ) ( * 2p y )
F2 18 Same as above 1 Diamagnetic

Ne2 20 18 as above and ( * 2p z ) 2 0 Diamagnetic

CO 14 Same as in N2 3 Diamagnetic
NO 15 Same as in O2+ 2.5 Paramagnetic
NO+ 14 Same as in N2 3 Diamagnetic

NO2+ 13 KK ( 2s ) 2 ( * 2s ) 2 ( 2 p z ) 2 ( 2 p x ) 2 ( 2 p y )1 2.5 Paramagnetic

NO- 16 Same as in O2 2 Paramagnetic


CN 13 Same as in NO2+ 2.5 Paramagnetic
CN- 14 Same as in N2 3 Diamagnetic
Chemical B onding and Molecular Structure 81
IONIC CHARACTER OF A COVALENT BOND : Solution - For 100% ionic character the dipole moment should
i) By electronegativity difference - It is determined by the be
electronegativity difference of the participating atoms. The = e × d = 4.8 × 10-10 × 1.275 × 10-8 = 6.12 D
higher the electronegativity difference between the two The observed dipole moment is = 1.03 D
atoms, more is the percentage ionic character of a covalent
bond. The mathematical equations for calculating, the % ionic 1.03
% ionic character = 100 = 16.83%
character are 6.12
a) Pauling equation % ionic character CHARACTERISTICS OF COVALENT COMPOUNDS :
1
(XA X B ) i) Physical state - They may be gases, liquids or solids
100 1 exp 4 ii) Crystal structure - They may exist as simple molecules held
b) Hannay and smith equation together by weak forces, giant molecules united by covalent
= 16 (xA – xB) + 3.5 (xA –xB)2 link eg diamond, silicon carbide, alumina, aluminium nitride
where xA and xB are the electronegativities of atoms etc and can have layer structure. eg graphite.
Percentage ionic character and the electronegativity difference iii) Solubility - Generally soluble in non polar solvents.
iv) Melting/ boiling points - Low as compared to ionic
xA - xB value % ionic xA - xB value %ionic compounds.
character character v) Electrical conductivity - Being non electrolytes they do not
0.1 0.5 1.7 51.0 conduct electricity, graphite does due to presence of free
0.4 4.0 2.0 63.0 electrons.
0.7 12.0 2.3 74.0 vi) Stereo isomerism - They exhibit structural and stereo -
1.0 22.0 2.6 82.0 isomerism both
1.4 39.0 2.9 88.0
ii) By dipole moment - By dipole moment the % ionic character
HYBRIDISATION :
of a covalent bond can be obtained from the following Hybridisation is the redistribution of energy levels at the time of
equation formation of molecules. It may also be defined as intermixing of
% ionic character atomic orbitals of nearly the same energy and resulting in the
formation of new atomic orbitals same in number and identical in
observed dipole moment of bond all respects (shape, energy and size). The new atomic orbitals are
= 100
calculated dipole moment of bond known as hybrid atomic orbitals.
The completely filled or half filled atomic orbials can take part in
Example : Calculate the percentage ionic character of HCl
hybridisation and hybrid atomic orbitals form stronger bonds.
molecule. The bond length is 1.275 Aº and observed dipole
moment is 1.03 D

Shape of molecules / ions having only bond pair of electrons


No. of bp of Hybridisation Bond Shape Examples
electrons at the angle of molecule
central atom
2 sp 180º Linear BeH2, BeX2 (X = F, Cl, Br, I), CO2, HgCl2, ZnCl2,
HCN, MgCl 2 , C 2 H2 (alkynes), [Ag(NH3 ) 2 ] +
[Ag(CN)2]–
3 sp 2 120º Trigonal [BeF3]–, BF3, BCl3, BBr3, BH3, AlCl3, SO3,
planar C2H4 (alkenes), NO , CO , C6H6
3
2

4. sp 3 109.5º Tetrahedral SiX 4 , SnCl 4 , CH4 (alkanes), CX 4 , [BeF 4 ] 2– ,


SO 2

4
, ClO 4 , NH , BF , PO
+

4 4
3

4. dsp2 90º Square planar [Ni(CN)4]2–, [PtCl4]2–, [Cu(NH3)4]2+


5. sp3 d 90º, 120º Trigonal PCl5, PF5, SbCl5
bipyramidal
6. sp3d 2 90º Octahedral SF 6 , SeF 6 , TeF 6 , PF , SnCl
6
[CrF 6 ] 3– ,
2

[Co(NH3)6]3+, [SiF6]2–, [Co(CN)6]3–,W(CO)6


7. sp3d 3 90º, 72º Pentagonal IF7
bipyramidal
82 Chemistry
Geometry of molecules/ions having bond pair as well as lone pair(s) of electrons
Type of Bond No. of Hybridi- Bond Expected Actual Examples
molecule pairs lone pairs sation angle geometry geometry
of electrons
AX 3 2 1 sp 2 120º Trigonal V-shape, SO2, SnCl2, NO 2

planar Bent, Angular

AX 4 2 2 sp 3 109º 28' Tetrahedral V-shape H2O, H2S, SCl2,


PbCl2, OF2, NH , 2

ClO 2

AX 4 3 1 sp 3 109º 28' Tetrahedral Pyramidal NH3, NF3, PCl3,


PH 3, AsH 3,
ClO , H3O+
3

AX 5 3 2 sp 3 d 90º Trigonal T- shape ICl3, IF3, ClF3


bipyramidal

AX 5 2 3 sp 3 d 180º Trigonal Linear XeF2, I3–, ICl2–


bipyramidal

AX 6 5 1 sp 3 d 2 90º Octahedral Square ICl5, BrF5, IF5


Pyramidal

AX 6 4 2 sp 3 d 2 - Octahedral Square planar XeF4, ICl4–

AX 7 6 1 sp 3 d 3 - Pentagonal Distorted XeF6


bipyramidal octahedral

METHOD FOR FINDING THE TYPE OF VSEPR THEORY VALENCE SHELL ELECTRON PAIR
HYBRIDISATION : REPULSION THEORY :
Apply the following formula to find the hybridisation of central The following types of repulsions between different pair of
atom. electrons change the regular geometry of the molecule.
Lone pair - lone pair repulsion > Lone pair - bond pair repulsion >
Number of valence electrons of central atom Bond pair - bond pair repulsion or lp - lp > lp - bp > bp - bp eg:
1 ÷
(i) Shape of H2O molecule - The hybridisation of oxygen is sp3
+ number of monovalent atoms attached to it ÷ = z
2 ÷ and angle should be 109º 28’. But actually the HOH angle is
+ negative charge if any – positive charge if any
104.5º. The VSEPR theory explains this fact
Value of z 2 3 4 5 6 7
O V-shape
Hybridisation sp sp 2 sp 3 sp3 d sp3d 2 sp3d 3
H H
Examples : All types of repulsions (lp–lp, lp–bp, bp–bp) are present in
1 H2O.
i) Hybridisation of NH3 = [5 + 3 + 0 - 0] = 4; sp3 (ii) Shape of NH3 molecule - The hybridisation of nitrogen is
2
1 sp3. The H–N–H angle is 107.5º instead of 109º28’. Due to
ii) Hybridisation of H2O = [6 + 2 + 0 - 0] = 4; sp3 lp–bp and bp–bp repulsions the value of angle change
2
1 N
iii) Hybridisation of SO3 = [6 + 0 + 0 - 0] = 3; sp2 H H Pyramidal
2 H
1 (iii) Structure of PCl5 : The phosphorous is in sp3d hybridised
iv) Hybridisation of SO42– = [6 + 0 + 2 - 0] = 4; sp3
2 form. The five hybrid orbitals are not equivalent. The bonds
1 pointing towards the three corners of equilateral triangle are
v) Hybridisation of CO32– = [4 + 0 + 2 - 0] = 3; sp2 known as equatorial bonds making an angle of 120° between
2
them. The remaining two bonds are at right angle to the
1
vi) Hybridisation of PCl 5 = [5 +5 + 0 - 0] = 5; sp3d plane of three make an angle of 90° and are called axial bonds.
2 The axial bonds are slightly longer and weaker then equatorial
1 bonds. Since they suffer more repulsive interaction from the
vii) Hybridisation of SF6 = [6 + 6 + 0 - 0] = 6; sp3d2
2 later. Consequently PCl5 is very reactive molecule.
Note : Species having same hybridisation are isostructral in nature.
Chemical B onding and Molecular Structure 83
Cl H–F ............ H–F ............ H–F
Cl The chains possess a Zig-Zag structure
P – –
F F
Cl Cl
+ + + +
Cl H H H H
CO-ORDINATE COVALENT BOND OR DATIVE – –
BOND : F F
When both the elctrons for sharing between two atoms are Hydrogen bond is purely electrostatic in nature. It is a weak bond,
contributed by one atom only the bond formed is known as the strength of the strongest being about 5 - 10 kcal per mole. The
coordinate bond or dative bond. more the electronegativity of atom involved in H - bonding, the
In terms of orbital theory the co-ordinate covalent bond is formed more is the bond strength eg.
by overlapping between empty and completely filled atomic H F > H O > H N
orbitals.
10 kcal/mole > 7 kcal/mole > 2.0 kcal/mole
The atom donating the pair of electrons is called donor and the
Types of hydrogen bonds - Hydrogen bond is of two types
atom which accepts the pair of electrons is called acceptor. The
compounds containing coordinate bonds are known as co- (i) Intermolecular H-bonding (Association).
ordination compounds. The bond is represented by an arrow ( ) H–bonding involving two or more molecules.
pointing head towards the acceptor. (ii) Intramolecular H-bonding (chelation). H–bonding taking
Once the coordinate bond is formed it is indistinguishable from a place within single molecule.
covalent bond. Examples (i) Applications of intermolecular H-bonding.
(i) Formation of SO2 (a) Water - Water has the lowest molecular weight among
the hydrides of group 16 elements yet it has the highest
O S O or melting and boiling points
O=S O
H2O H2S H2Se H2Te
ii) Formation of SO3
Melting point 0ºC -85.5ºC -66ºC -51.2ºC
O O Boiling point 100ºC -60.4ºC -41.5ºC +2ºC
O S O or It is due to intermolecular H - bonding through which
O=S O water molecules associate
iii) Formation of Hydroxonium ion + + + +
H H H H
H – + – + – + – +
H H O –H O– H O– H O– H
O H +
or O H or O+ H (b) Ice has less density than water - In crystal structure of
ice every water molecule is associated with four other
H H
H water molecules by H-bonding in a tetrahedral fashion
iv) NH3 and BF3 form addition product by Co-ordinate covalent
bond –
O
H F + +
| | H H
– –
H N: B F or H 3 N.BF3 O O
| | + + +
H F H H H H

CHARACTERITICS OF CO-ORDINATE COMPOUNDS O
Co-ordinate compounds have volatile character in between ionic + +
H H
and covalent compounds. Other properties like solubility, electrical
conductivity and stereo isomerism are similar to covalent
compounds. It gives rise to cage likes tetrahedral structure of ice with
HYDROGEN BOND : large empty spaces. On melting the ice H-bonds are
broken and space between water molecules decreases
It may be defined as the force of attraction existing between
and density of water increases upto 4ºC. Above 4ºC more
hydrogen atom covalently bonded to highly electronegative atom
H - bonds are broken, the water molecules move apart
(N, O or F) and the electronegative atom belonging to another
molecule of the same or different substance. It is represented by from each other and the density again decreases. Thus
dotted lines water has maximum density at 4ºC.
84 Chemistry
(c) Alcohols - The marked difference between the melting (c) Ethyl aceto acetate - It exists in two forms
and boiling points of alcohols and corresponding O
mercaptans is also due to association. ||
CH 3OH C2H 5OH C3H 7OHC4H 9OH CH 3 C CH 2 . COOC 2 H 5
Keto form
Boiling point 64.4ºC 78ºC 97ºC 117ºC
CH3SH C2H5SH C3H7SH C4H9SH OH
|
Boiling point 5.8ºC 37ºC 67ºC 97ºC
CH3 C = CH.COOC2 H 5
R R R R Enol form
– + – + – + – + Enolic form is more volatile due to chelation
O –H O–H O–H O–H
(d) Amides - Amides associate and have higher melting and O H
+

O
boiling points | ||
CH 3 C = CH C OC 2 H 5
R H R H R
| + | | + | | (d) Maleic acid is stronger acid than fumaric acid
H2N C = O H N C= O H N C= O
O
(e) Amines - Primary and secondary amines are associated || – +
and have higher boiling points than isomeric tertiary H C C O H
H C COOH || |
amines. ||
H C COOH H C C O
(f) Fatty acids - In organic solvents, fatty acids form ||
a dimer O
Maleic acid Chelation in maleate ion
– + – (e) Acid character of nitrophenols. It follows the following
O H O
order
R C – C R p - nitrophenol > o - nitrophenol > m - nitrophenol
+ –
O H O Acid character of o - nitrophenol is suppressed by
chelation
It is confirmed by electron diffraction method.
(ii) Applications of Intramolecular H - bonding. O +
H
When the H - bonding takes place within a single molecule –
this is known as intramolecular H - bonding. It also effects O
N
the physical and chemical properties of compound.
(a) Volatile character of nitrophenols - o-nitrophenol is O
more volatile (b.pt 214ºC) as compared to meta (b.pt
(f) Other compounds showing intramolecular
290ºC) and para (b.pt 279ºC). It is due to chelation H - bonding
– O O +
O + + H
H H
O –
O – Cl
– C
N
H
O
o-Chlorophenol o-hydroxy benzaldehyde
In meta and para isomer chelation is not possible due to O +
the formation of desired size of ring. They therefore H
O –
associate to some extent and have higher B.P C
(b) Salicylic acid is stronger acid than o - methoxy benzoic
acid OH
o-hydroxy benzoic acid
O O
METALLIC BOND :
C C
O – O It has been found that metals have generally low ionisation
H
+ CH3 energies indicating that valence electrons are weakly bound to
O O the atomic kernel (or core).
H-bonding in salicylate ion No-H-bonding in
Metals contain few valence electrons and valence orbitals which
are empty. Thus valence electrons are completely delocalised and
o-methoxy benzoate ion
are frequently exchanged between atoms. The atoms thus acquire
Chemical B onding and Molecular Structure 85
a positive charge and are arranged in a regular fashion (lattice • The greater the polarizability of a molecule, the stronger are
structure). the London forces.
• The polarizability increases with number of electrons and
e e e
their distance from the nucleus.
e e e • These forces are operative over a very short range.
• The inert gases have weak van der Waals forces of attraction
e e e
• The striaght chain hydrocarbons boil at higher temperature
The valence electrons form an “electron gas” or "sea of than isomeric branched chain hydrocarbons due to greater
electrons". The attractive interaction between the mobile magnitude of van der Waal's forces of attraction.
electrons and a number of ions constitutes a weak bond
known as metallic bond. Dipole - dipole attraction
It is the attraction between the positive end of the one molecule
CONSEQUENCES OF METALLIC BOND :
and negative end of the another molecule
1. Electrical conductivity - It is due to presence of mobile
valence electrons. + +
or
+ +

H Cl H Cl Cl H Cl H
2. Thermal conductivity - On heating one part of metal, the
K.E. of electrons is increased and they conduct heat to the This type of interaction is called dipole - dipole interaction. The
other parts of the metal. force of dipole - dipole attraction is inversely proportional to fourth
3. Metallic lustre- The mobile elctrons are promoted to excited 1
states by absorption of light and on coming back from the power of their separation of distance r. F = . The dipole dipole
excited state light of all wavelengths in the visible region is r4
emitted. The surface, therefore emits metallic lustre. attractions in gaseous molecules are particularly small since the
4 Malleability - Metals can be made thin sheets gas molecules are in continuous motion. This attraction increases
5. Ductility - Metals can be drawn into wires since metal kernels the m.pt. and b.pt. of substances and makes their liquification
can easily be shifted. easy.
6. Electrical conductivity decreases with temperature - It is Dipole-induced dipole attraction
due to random motion of mobile electrons which increases The attractive forces operate between polar molecules having
with increase of temperature. permanent dipole and non-polar molecules. The polarity in the
van der Waal's Forces : non-polar molecules is induced by the polar molecule
Attractive forces between uncharged molecules are known as
van der Waals forces. These forces may be divided into three + – + –

groups A B A–A
(i) Dispersion or London forces. These are due to transient
polarisation. Polar molecules Non-polar molecules

(ii) Dipole - dipole attraction. These are due to permanent


polarisation. DIPOLE MOMENT :
(iii) Dipole-Induced fipole forces If a covalent bond is formed between two disimilar atoms, eg. A
and B, one of the atoms (A or B) must be more electronegative
London forces - The negative electrons in a neutral molecule are
balanced by the positive charges on the nuclues. Since the than the other. If A is more electronegative than the shared pair of
electrons are in motion the centre of density of the electrons does electrons is drawn near A leaving a positive charge on B and
not coincide continuously with the centre of density of positively hence making the molecule dipolar (A–B+). The percentage of
charged nuclei, the molecules acquire an electric dipole. Polarised polar character is given in terms of dipole moment ( ).
molecules exert an attraction for other molecules having a dipole.
Consider the case of helium. The dipole moment is defined as the product of electric charge q
and the distance r between the two atoms of a polar molecule.
– ( = e × d) Dipole moment is a vector quantity with direction same as
–+– +
that of the line joining positive and negative centres.
Non polar Polarised Thus molecules having dipole moment ( = 0) are called non
atom atom
polar molecules and molecules with > 0 are polar. Greater is ,
greater is the polarity.
+ +


+

+

For polyatomic molecules with two or more bonds, net dipole


Attraction between polarised atom moment is the resultant of vector addition of individual moments.
86 Chemistry
For e.g. in the figure shown, the resultant dipole moment, H Cl H Cl
= +
C C
1 2

C C
H Cl Cl H
O
Cis ( 0 ) trans ( = 0 )
O
1
(5) Dipole moment is greatest for ortho isomer, zero for para
O 2
isomer and less than that of ortho for meta isomer.
While expressing dipole moments, generally charge is given in o > m > p. e.g.
electrostatic units (esu) and distance in angstrom units (1Aº = Cl
10–10 m). Thus dipole moment of an electron separated from unit Cl Cl
positive charge by a distance 1Aº would be (4.80 × 10–10 esu) × Cl
(10-8 cm) = 4.8 × 10–18 esu cm = 4.8 Debye.
Some compounds and their dipole moments are
Substance Moment Substance Moment Cl
Cl
(Debye) (Debye) ortho meta para
CO2 0.00 = 2.53 Debye = 1.48 Debye = 0

HBr 0.78 (6) Ionic character can be determined by using dipole moment
For e.g. experimental dipole moment for HCl is 1.03 and
CH3Cl 1.85 CH2F2 1.96
= r e
NH3 1.48 O3 0.52 as r = 1.26 and, e = 4.8 × 10-10 esu,
H2O 1.85 CO 1.11 = 6.05 Debye.
CH3I 1.35 CH3OH 1.68
1.03 1
Thus the ionic character 100%
C6H5NO2 4.08 C2H5OH 1.65 6.05 6
CH4 0.00 N2O 0.17 Thus we can say % ionic character
HF 1.91 H2S 0.92 experimental dipole moment
= 100
SO2 1.61 H2O2 1.84 theoretical dipole moment
NF3 0.24 PH3 0.59 (7) Hybridisation can be determined by dipole moment for eg.
C6H5OH 1.70 HCN 2.93 (i) If a molecule AB2 has =0, the orbitals used by
A (z < 21) must be sp hybridised e.g. BeF2
HCl 1.03 CCl4 0.0
(ii) If a molecule AB3 has =0, the orbitals used by
HI 0.38 CS2 0.0
A (z < 21) must be sp2 hybridised e.g. BF3
H2 0.95 PCl5 0.0
(iii) If a molecule AB4 has =0, the orbitals used by
Applications of dipole moment A (z < 21) must be sp3 hybridised e.g. CCl4
(1) Dipole moment is helpful in predicting the geometry of the
molecule. BOND LENGTH :
(2) Dipole moment helps in determining the polarity. It is the distance between the nuclei of two atoms between which
the bond is formed. Other names for bond length are interatomic
(3) Dipole moment can distinguish between symmetrical and
non symmetrical molecules. eg. CO2 has 0 dipole moment as distance or bond distance. It is usually expressed in angstrom
it is symmetrical whereas H2O has a dipole moment of 1.85D. unit (1Å=10-10m). Pauling’s rule states that for covalently bonded
atoms, bond length is sum of covalent radii. Thus for a covalent
O compound AB, the bond length is rA + rB. where rA and rB are
O C O 105º covalent radii of A and B
= 0 Bond length depends on:-
H
H
µ = 1 .8 5
(4) Cis and trans isomers can be distinguished by dipole i) Bond order - Bond length decreases with bond order. For
moments, usually cis isomers have higher dipole moment e.g. C - C length in CH3 - CH3 is 1.54 Å, C = C length in
and hence higher polarity e.g. CH2 = CH2 is 1.34 Å and C C length in CH CH is 1.20 Å.
Larger the s character, smaller is bond length.
Chemical B onding and Molecular Structure 87
ii) Electronegativity - If one of th e atoms is mor e 4) Hybridisation - Due to directional character of hybridised
electronegative than other, the bond length is found to be orbitals, extent of overlapping increases and hence stronger
smaller than expected value. The relation that holds true for bonds are formed.
most compounds is 5) Type or extent of overlapping - Coaxial overlappings
rAB = rA + rB - 0.09 (xA - xB) (s-s, s-p, p-p) have higher extents and hence are stronger
than collateral overlappings. Thus sigma bond (coaxial
where xA and xB are electronegativities of A and B
overlapping) is stronger than pi bond (collateral
iii) Hybridisation - Larger is s character, shorter is orbital and overlapping).As p orbitals are more directional, p - p coaxial
hence shorter is the length of bond formed by it. overlapping gives stronger bond in comparison to s - s
iv) Resonance and delocalisation - C-C bond length is 1.54Å overlapping.
and C = C bond length is 1.34 Å but in benzene, due to 6) Repulsion between lone pair - In a bonding atom having lone
pair the bond formed is weaker due to non localisation of lone
pair. Mutual repulsion takes place between lone pair clouds of
two atoms, decrease the strength and hence the bond energy.
For e.g. as lone pair at each C - C is 0, N - N is one, O - O is two,
so values of their bond energies are in order C - C > N - N >
resonance, the carbon-carbon bond is neither single nor O - O.
double but intermediate between single and double and same Determination of bond energy
holds for bond length and is equal to 1.39 Å. Bond energy is determined by thermochemical methods by
Average Bond Distance of Covalent Bonds determining either the heat of atomisation i.e. heat required to
break the molecule into its atoms or by the heat of formation of
Bond Bond Bond Bond Bond Bond
compound.
length (Å) length (Å) length (Å)
Some average Bond Energies (kcal/mole)
C-C 1.54 C-H 1.12 H- H 0.74
Bond Energy Bond Energy Bond Energy
C= C 1.34 C-F 1.42 F-F 1.42
H–H 103.4 C–N 69.3 C–F 107.0
C C 1.20 C - Cl 1.77 CI - CI 1.99
C–H 98.4 C N 135.0 C – Cl 78.0
C- N 1.47 C - Br 1.91 Br - Br 2.28
C– C 81.6 O– O 34.9 C – Br 65.0
C= N 1.28 C–I 2.13 I–I 2.67
C=C 146.1 O=O 119.0 C–I 57.0
C N 1.15 O-H 0.97 H- F 0.92
C C 192.1 N– N 39.0 H–F 132.4
C- O 1.43 N-H 1.03 H - Cl 1.27
C–O 81.5 N=N 97.6 H – Cl 102.7
C= O 1.20 S-H 1.35 H - Br 1.41
C= O 173.0 N N 225.0 H – Br 87.3
O=O 1.21 N=N 1.09 H- I 1.61
C O 256.0 N–H 92.9 H–I 71.4
BOND ENERGY :
C=S 114.0 S–H 87.5 F–F 37.0
Bond strength or bond energy is the amount of energy required
to break a bond in one gram mole of a substance in gaseous state. N=O 146.0 O–H 110.2 Cl – Cl 58.0
Bond energy is also called bond dissociation energy. Its units are
BOND ANGLE :
kJmol–1. Bond dissociation energy and (average) bond energy
are same in diatomic molecules as they have only one bond. But Bond angle is the angle between two adjacent bonds in a molecule.
in molecules having many bonds, bond dissociation energy of O
the bonds are different and bond energy is the average value of 180°
bond dissociation energy O C O H 105° H
Factors affecting the bond energy Thus bond angles in CO2 and H2O are 180º and 105º.
1) Atomic size - Strength of shorter bond is more e.g. atomic Factors affecting bond angle
size of Cl, Br, I has the order Cl < Br < I and their bond 1) Electronegativity - Cl - O- Cl bond angle in Cl2O is more
energies, Cl - Cl (243 kJ mol -1 ) > Br - Br (192 kJ mol –1 ) than F - O - F bond angle in OF2 because in Cl2O, O atom is
> I - I(151 kJmol–1) more electronegative compared to Cl atom due to which
2) Bond order - Higher bond order implies higher energy e.g. shared electrons are attracted towards O. This makes bonded
electrons of two Cl-O bonds to come close and hence more
bond energies for C C, C C and C C are 347.2, 610.0
repulsion resulting in wider bond angle. Whereas in OF2, F
and 835.1 kJ mol-1 respectively. is more electronegative than O thus bonded electrons of F-O
3) Electronegativity-Larger the difference in electronegativities bond are closer to F atom. This makes bonded electrons of
of bonded atoms, more is the attraction, shorter is bond two O - F bonds to go apart, feeling less repulsion and hence
length and hence higher is bond energy. a lower value for bond angle.
88 Chemistry
2) Hybridisation - sp3 hybridisation at central atom implies a (2) Expansion of octet : Consider the following molecules
bond angle of 109º 28 each e.g. CH4, CH3CH3 etc. sp2 .. ..
hybridisation leads to 120º bond angle as in BF3 . sp .. Cl .. F ..

..
..
..
..
hybridisation makes angle 180º e.g. BeCl2. Cl
.. F F

..
.. ..

..

..
..
.. P Cl .. .... .. .. S ..

..
3) Lone pair repulsions - Presence of lone pair at central atom Cl
..
.. ..F– Cl – F.. F
.. F

..
..
..

..
..

..
results in repulsion of shared pair of electrons. Thus bonds
Cl
.. F
..

..
..

..
..F

..
at central atom are displaced inside making the value of bond

..
..
angle smaller e.g. bond angle in ammonia is 107º even though (10 electrons around P) (10 electrons around Cl) (12 electrons around S)
hybridisation at nitrogen is sp3. It is due to presence of lone
Explanation for the failure of Octet rule : The concepts given
pair of electrons at nitrogen atom which causes more
repulsion between N and H and hence decreasing the bond below explain the failure of the octet rule :
angle. Bond angle can be measured by X-ray techniques or (a) Sidwick's rule of maximum covalency : The maximum
by I.R. spectroscopy. covalency of an element depends on the period to which
Baeyer’s strain theory states that any shift in normal bond it belongs and octet can be exceeded for n = 1 it is 2, for n
angle causes strain over the molecule making it less stable = 2 it is 4, for n = 3 it is 6 and for n > 4 it is 8.
and more reactive. Thus CH4, C2H6 are quite stable while (b) Sugden's view of singlet linkage : According to Sugden
ethylene is less stable (bond angle 120º). the octet rule is never violated. There can be one electron
OCTET RULE : bond known as Singlet linkage, Singlet or half bond is
formed by sharing of one electron between two atoms. It is
During formation of a covalent bond, the atoms attain an intert represented by half arrow ( ). Thus SF6 and PCl5 have
gas electronic configuration (ns2p6 configuration). This is known the structures :
as Octet rule. There are exceptions of octet rule.
.. ..
(1) Incomplete octet : Consider the following molecules .. F .. .. Cl
..
..

..
..
.. .. F F Cl
..

..
.. ..
..

..
..
.. .. ..F– B – F.. .. .. Cl
..

S P
..

..

..
Cl
.. – Be – Cl
.. F F Cl
..
..

..

..
..

F ..
..
..

..
..
..
..

(4 electrons around Be) F


(6 electrons around B) .. Cl
..
..

..
..
..

..
.N ..
O
..
(7 electrons around N)
Chemical B onding and Molecular Structure 89

Very Short/Short Answer Questions Long Answer Questions


1. Out of bonding and antibonding molecular orbitals which 13. Give difference between Bonding Molecular orbital and
one has lower energy and which one has higher stability? Anti–bonding molecular orbital.
2. The H—S—H bond angle in H2S is 92.2° whereas the 14. What are the characteristics of resonance?
H—O—H bond angle in H2O is 104.5°, why?
15. Determine the formal charges on the atoms in the two
3. Out of NaCl and MgO, which has higher value of lattice structures shown below for the molecule POCl3. Which of
energy?
these two structures is the better structure (i.e., which is
4. Which of the following structures contributes least towards preferred)?
resonance hybrid?
– + + 2– + ..Cl.. ..Cl..

..

..
.. _
N = N = O :N N—O:
..
:N—N
..
O+ :
..Cl O.. ..Cl O..
I II III

.. ..

..

.. ..

.. ..
P P
5. Which of the following has higher dipole moment
H3C – CH2 – CH = CH2( 1–butene) or .. Cl.. ..Cl..

..

..
H3C – CH2 – C º CH( 1–butyne)? I II
6. Predict the dipole moment of 16. Draw all possible resonance structures for the sulfate ion,
(i) A molecule AX4 with tetrahedral geometry SO42– on the basis of formal charge
(ii) A molecule AX2 with linear geometry 17. State the hybridization and draw the molecular structure of
(iii) A molecule AX3 with distorted tetrahedral geometry. the following
7. Three elements have the following Lewis symbols: (a) PF5 (b) I3– (c) NH3 (d) XeO3 (e) CH4
A B C 18. Give reasons for following :
(a) Assign groups of the periodic table to these elements. (i) What is the decreasing order of lattice energy of
various ions?
(b) Which elements are expected to form ions and what is
the expected charge over these ions? (ii) PbSO4, BaSO4, AgCl, AgBr, AgI are insoluble in water,
(c) Write the formulae and Lewis structures of the covalent why?
compounds formed between (iii) Which is more ionic and why? FeCl 2 or FeCl3?
(i) A and B (iv) What is the decreasing order of strengths of bonds
(ii) A and C between sp3, sp2 and sp hybrid orbitals?
8. Why are lone pair-lone pair repulsions stronger than lone (v) What is the order of dipole moment of ortho, meta and
pair-bond pair? para disubstituted derivatives?
9. Account for the following : Multiple Choice Questions
(i) N2 has a higher bond dissociation energy than NO. 19. Sodium chloride is an ionic compound whereas hydrogen
(ii) N2 and CO both have the same bond order but CO is chloride is mainly covalent because
more reactive than N2. (a) sodium is less reactive
10. Account for the following : (b) hydrogen is non-metal
(i) He2 is not found to exist in nature (c) hydrogen chloride is a gas
(ii) N2+ is not a diamagnetic substance (d) electronegativity difference in the case of hydrogen
and chlorine is less than 2.1.
(iii) The dissociation energy of H+2 is almost the same as
20. Which one of the following molecules will have unequal
that of He2+.
M – F bond lengths ?
11. Which is more covalent and why : BeCl2 or MgCl2 ?
(a) NF3 (b) BF3
12. Why does Pauli’s exclusion principle permit bonding only if
the electrons are of different spins? (c) PF5 (d) SF4
90 Chemistry
21. Match List I (Molecules) with List II (Bond order) and select 25. Number of bonds and bonds in the following structure
the correct answer using the codes is H H
List I List II H H
I. Li2 A. 3
II N2 B. 1.5 H H
III Be2 C. 1.0 H H
IV O2 D. 0 (a) 6, 19 (b) 4, 20
E. 2 (c) 5, 19 (d) 5, 20
Codes 26. The states of hybridisation of boron and oxygen atoms in
(a) I - B, II - C, III - A, IV - E boric acid (H3BO3) are respectively
(b) I - C, II - A, III - D, IV - E (a) sp3 and sp2 (b) sp2 and sp 3
(c) I - D, II - A, III - E, IV - C 2
(c) sp and sp 2 (d) sp3 and sp3
(d) I - C, II - B, III - E, IV - A 27. Which of the following statement is not correct from the
22. Polarity in a molecule and hence the dipole moment depends view point of molecular orbital theory?
primarily on electronegativity of the constituent atoms and (a) Be2 is not a stable molecule.
shape of a molecule. Which of the following has the highest
dipole moment? (b) He2 is not stable but He 2 is expected to exist.
+

(a) CO2 (b) HI (c) Bond strength of N 2 is maximum amongst the


(c) H2O (d) SO2 homonuclear diatomic molecules belonging to the
23. Which does not show resonance ? second period.
(a) Benzene (b) Aniline (d) The order of energies of molecular orbitals in N2
(c) Ethylamine (d) Toluene molecule is
24. Which of the following overlap is correct ? 2s < * 2s < 2pz < ( 2px = 2py)
< ( *2px = *2py) < *2pz
+ + +
(a) + 28. The correct representation of H-bond in solid HF
– – –
(a) H – F H–F H–F
(b) + – + .+. F F F
(b) H H H H H
F F
(c) + – + + – – + –
(c) H H H H
(d) None of the above F F F
H H
(d) F F F F
H
Chemical B onding and Molecular Structure 91

1. Chemical bond implies 11. Ground state electronic configuration of valence shell
(a) repulsion electrons in nitrogen molecule (N 2) is written as
(b) attraction
( 2s ) 2 ( *2s ) 2 ( 2p ) 4 ( 2p ) 2 . Hence the bond order of
(c) attraction and repulsion balanced at a particular distance
(d) attraction and repulsion nitrogen molecule is
2. The bonding is electrovalent in (a) 2 (b) 3
(a) NaCl (b) Br2 (c) 0 (d) 1
(c) PF5 (d) XeF4 12. The bond order of NO molecule is
3. Bonding in ferric chloride is (a) 1.5 (b) 2.0
(a) covalent (b) ionic (c) 2.5 (d) 3.0
(c) co-ordinate (d) None of the above 13. Which of the following cannot be formed ?
4. Which of the following species has lowest ionization (a) He2+ (b) He+
potential? (c) He (d) He2
(a) O (b) O2
14. N 2 and O 2 are converted into monoanions, N 2– and O 2
(c) O +
(d) O–
2 2
respectively. Which of the following statements is wrong ?
5. Polarization power of a cation increases when
(a) charge on the cation increases (a) In N – , N – N bond weakens
2

(b) size of the cation increases


(b) In O – , O - O bond order increases
(c) charge on the cation decreases 2

(d) has no relation to its size or charge (c) In O – , O - O bond order decreases
2
6. The correct order of decreasing polarisability of following
ions is (d) N – becomes paramagnetic
2


(a) Cl , Br , I , F – – – (b) – – – –
F , I , Br , Cl
15. N2 and O2 are converted into monocations, N +

2
and
– –
(c) F , CI , Br , l – – – –
(d) I , Br , Cl , F – –
O respectively. Which of the following statements is
+

7. Which of the following statements regarding covalent bond


2

is not true ? wrong?


(a) The electrons are shared between atoms (a) In N , N - N bond weakens
+

(b) The bond is non - directional


(c) The strength of the bond depends upon the extent of (b) In O , the O - O bond order increases
+

overlapping
(d) The bond formed may or may not be polar (c) In O , paramagnetism decreases
+

8. Among the following, electron deficient compound is


(d) N becomes diamagnetic
+

(a) CCl4 (b) PCl5 2

(c) BeCl2 (d) BCl3 16. Which of the following molecule is paramagnetic?
9. Consider the following statements: (a) CO2 (b) SO2
(i) A sigma ( ) bond is formed when two s - orbitals overlap (c) NO (d) H2O
(ii) A pi ( ) bond is formed when two 17. In PO 34– , the formal charge on each oxygen atom and the
p- orbitals axially overlap
P - O bond order respectively are
(iii) A -bond is weaker than -bond.
Which of the above statements is/are correct ? (a) –0.75, 0.6 (b) –0.75, 1.0
(a) i and ii (b) ii and iii (c) –0.75, 1.25 (d) –3, 1.25
(c) i alone (d) ii alone 18. Which of the following ion has not bond order of 2.5 ?
10. Number of bonds in naphthalene are (a) O – (b) O +

2 2

(a) 6 (b) 3
(c) 4 (d) 5 (c) N +

2
(d) N– 2
92 Chemistry
19. Oxygen molecule is 32. The values of electronegativity of atoms A and B are 1.20
(a) diamagnetic with no unpaired electrons and 4.0 respectively. The percentage of ionic character of
(b) diamagnetic with two unpaired electrons A - B bond is
(c) paramagnetic with two unpaired electrons (a) 50% (b) 72.24%
(d) paramagnetic with no unpaired electrons (c) 55.3% (d) 43%
20. The number of electrons that are paired in oxygen molecule 33. The electronegativities of F, Cl, Br and I are 4.0, 3.0, 2.8, 2.5
is
respectively. The hydrogen halide with a high percentage of
(a) 16 (b) 12 ionic character is
(c) 7 (d) 8
(a) HF (b) HCl
21. Which of the following pairs have identical bond
order ? (c) HBr (d) HI
34. Covalent compounds have low melting point because
(a) N , O 2 +
(b) N ,O– 1

2 2 2 2
(a) covalent molecules have definite shape
(c) N – , O2 2
(d) O , N 2 +

2
(b) covalent bond is weaker than ionic bond
22. Which one of the following should be most stable? (c) covalent bond is less exothermic
(a) H +
(b) H+ (d) covalent molecules are held by weak van der Waal’s
2
forces of attraction
(c) H (d) H– 35. The number of sigma ( ) bonds in 1-butene is
23. The number of antibonding electron pairs in O – molecular
2

2
(a) 8 (b) 10
ion on the basis of molecular orbital theory are (at. no. O (c) 11 (d) 12
= 8) 36. The structure of PF5 molecule is
(a) 2 (b) 3 (a) tetrahedral
(c) 4 (d) 5 (b) square planar
24. Which of the following is not paramagnetic ?
(c) trigonal bipyramidal
(a) N +

2
(b) CO (d) pentagonal bipyramidal
(c) O – (d) NO 37. Which of the following has sp 2-hybridization?
2
(a) C2H6 (b) C2H4
25. Which of the following molecular species has unpaired
electron (s) ? (c) BeCl2 (d) C2H2
(a) N2 (b) F2 38. The structure and hybridization of Si(CH3)4 is
(a) bent, sp (b) trigonal, sp2
(c) O – (d) O –2

2 2
3
(c) octahedral, sp d (d) tetrahedral, sp3
26. Which of the following has least covalent P–H bond?
(a) PH3 (b) P2H6 39. Which of the following molecule is not linear in shape
(c) P2H5 (d) PH6+ (a) HgCl2 (b) Hg2Cl2
27. Which contains both polar and non - polar bonds? (c) CO2 (d) SnCl2
(a) NH4Cl (b) HCN 40. Among the following compounds the one that is polar and
(c) H2O2 (d) CH4 has the central atom with sp2 hybridisation is
28. Which of the following has least polarity in bond? (a) H2CO3 (b) SiF4
(a) H – F (b) H – Cl (c) BF3 (d) HClO2
(c) H – O (d) H – S 41. Which one of the following compounds has sp2 hybridisation?
29. Which of the following substances has the least ionic (a) CO2 (b) SO2
character ?
(c) N2O (d) CO
(a) FeCl2 (b) ZnCl2
(c) CdCl2 (d) MgCl2 42. The AsF5 molecule is trigonal bipyramidal. The hybrid
orbitals used by the As atoms for bonding are :
30. The correct order of increasing ionic character is
(a) BeCl2 < MgCl2 < CaCl2 < BaCl2 2
(a) d x 2 y2 , d z , s, p x , p y (b) d xy , s, p x , p y , p z
(b) BeCl2 < MgCl2 < BaCl2 < CaCl2
(c) BeCl2 < BaCl2 < MgCl2 < CaCl2
(c) s, p x , p y , p z, d 2z (d) d , s, p x , p y
(d) BaCl2 < CaCl2 < MgCl2 < BeCl2 x 2 y2
31. Which of the following contains both covalent and ionic bond? 43. The hybridization of S atom in SO42– is
(a) NH4Cl (b) H2O (a) sp (b) sp 2
(c) CCl4 (d) CaCl2 (c) sp 3 (d) sp 3 d
Chemical B onding and Molecular Structure 93

55. The geometry of ClO– ion according to Valence Shell


3

44. In piperidine , the hybrid state assumed by N is Electron Pair Repulsion (VSEPR) theory will be
N (a) planar triangular (b) pyramidal
H (c) tetrahedral (d) square planar
(a) sp (b) sp 2 56. Among the following ions, the p – d overlap could be
(c) sp 3 (d) dsp 2 present in
45. Which of the following will be octahedral ? (a) NO – (b) PO 3 –
3 4

(a) SF6 (b) BF4–



– (c) CO 2
(d) NO –
(c) PCl5 (d) BO 3

3
3 2


46. The hybridization of atomic orbitals of nitrogen in 57. In NO ion, number of bond pairs and lone pairs of electrons
3

on nitrogen atom are


NO 2 , NO 2 and NH 4 are
+ +

(a) 2, 2 (b) 3, 1
(a) sp2, sp3 and sp2 respectively (c) 1, 3 (d) 4, 0
(b) sp, sp2 and sp3 respectively 58. In OF2, number of bond pairs and lone pairs of electrons are
(c) sp2, sp and sp3 respectively respectively
(d) sp2, sp3 and sp respectivley (a) 2, 6 (b) 2, 8
47. The compound MX4 is tetrahedral. The number of XMX (c) 2, 10 (d) 2, 9
formed in the compound are 59. The compound containing co-ordinate bond is
(a) three (b) four (a) H2SO4 (b) O3
(c) five (d) six (c) SO3 (d) All of these
48. The shape of gaseous SnCl2 is 60. Sulphuric acid provides a simple example of
(a) tetrahedral (b) linear (a) co-ordinate bonds
(c) angular (d) t-shape (b) non-covalent compound
49. Which of the following molecule is linear ? (c) covalent ion
(d) non - covalent ion
(a) SO2 (b) NO +

2 61. Which of the following does not contain coordinate bond ?


(c) NO – 2
(d) SCl2 (a) BH – 4
(b) NH 4 +

50. Each carbon in carbon suboxide (C3O2) is (c) CO32 (d) H 3O +

(a) sp2 - hybridized


62. The maximum number of water molecules that one water
(b) sp3 - hybridized
molecule can hold through hydrogen bonding is
(c) sp - hybridized
(a) 2 (b) 4
(d) sp2 - hybridized but linked with one co-ordinate bond
(c) 6 (d) 8
51. In XeF2, XeF4 and XeF6, the number of lone pairs on Xe are
63. NH3 has abnormally high boiling point because it has
respectively
(a) alkaline nature (b) distorted shape
(a) 2, 3, 1 (b) 1, 2, 3
(c) sp3 - Hybridization (d) hydrogen bonding
(c) 4, 1, 2 (d) 3, 2, 1
64. In which of the following pairs hydrogen-bonding is not
52. In which of the following species is the underlined carbon
possible ?
having sp3 - hybridisation ?
(a) NH3, NH3 (b) NH3,CH4
(a) CH – COOH
3
(b) CH CH OH 3 2 (c) H2O, CH3OCH3 (d) CH3OH,CH3OCH3
(c) CH COCH (d) CH = CH CH 65. The high density of water compared to ice is due to
3 3 2 3
(a) H - bonding interactions
53. A square planar complex is formed by hybridisation of which
(b) dipole - dipole interactions
atomic orbitals ?
(c) dipole-induced dipole interactions
(a) s, p x , p y , d yz (b) s, px , p y , d x2 y2 (d) induced dipole-induced dipole interactions
66. Two ice cubes are pressed over each other until they unite to
(c) s, p x , p y , d z 2 (d) s, p x , p z , d xy form one block. Which one of the following forces dominate
54. According to VSEPR theory, the most probable shape of the for holding them together?
molecule having 4 electron pairs in the outershell of the central (a) Dipole-dipole interaction
atom is (b) Van der waals’ forces
(a) linear (b) tetrahedral (c) Hydrogen bond formation
(c) hexahedral (d) octahedral (d) Covalent attraction
94 Chemistry
67. Which of the following molecules will form a linear polymeric 81. The sulphate of a metal has the formula M2(SO4)3. The
structure due to hydrogen bonding ? formula of its phosphate will be
(a) HCl (b) HF (a) M(HPO4)2 (b) M3(PO4)2
(c) H2O (d) NH3 (c) M2(PO4)3 (d) MPO4
68. Among H2O, H2S, H2Se and H2Te, the one with the highest 82. KF combines with HF to form KHF2. The compound contains
boiling point is the species
(a) H2O because of hydrogen bonding (a) K+, F– and H+ (b) K+, F– and HF
(b) H2Te because of higher molecular weight +
(c) K , and [HF2] – (b) [KHF]+and F2
(c) H2S because of hydrogen bonding
83. Which of the following molecular orbitals has two nodal
(d) H2Se because of lower molecular weight
planes ?
69. Which of the following is soluble in water ?
(a) 2s (b) 2p y
(a) CS2 (b) C2H5OH
(c) *2py (d) *2px
(c) CCl4 (d) CHCl3
70. A metallic bond is 84. The electronic configuration of metal M is 1s2 2s2 2p6 3s1.
(a) ionic (b) polar covalent The formula of its oxide will be
(c) non-polar covalent (d) electrostatic in nature (a) MO (b) M2O
71. Type of forces between molecules of C6H6 are (c) SO3 (d) All of these
(a) dipole-dipole interaction (b) dispersive forces 85. The number of possible resonance structure for CO 2 –
is
(c) H-bonding (d) van der Waals’ forces
3

72. Which of the following has the highest dipole moment? (a) 2 (b) 3
(a) AsH3 (b) SbH3 (c) 6 (d) 9
(c) PH3 (d) NH3 86. Match List I (Molecules) with List II (Bond order) and select
73. The molecule having non-zero dipole moment is the correct answer using the codes
(a) H2O2 (b) CH4 List I List II
I. Li2 A. 3
(c) C2H6 BF –
(d) 4
II N2 B. 1.5
74. Which of the following hydrocarbons has the lowest dipole III Be2 C. 1.0
moment ? IV O2 D. 0
H3C CH 3 E. 2
(a) C (b) CH C CCH Codes
3 3

H H (a) I - B, II - C, III - A, IV - E
(b) I - C, II - A, III - D, IV - E
(c) CH CH C CH (d) CH 2 = CH – C CH
3 2
(c) I - D, II - A, III - E, IV - C
75. Which of the following has zero dipole moment?
(d) I - C, II - B, III - E, IV - A
(a) ClF (b) PCl3
87. An ether is more volatile than an alcohol having the same
(c) SiF4 (d) CFCl3
molecular formula. This is due to
76. Dipole moment is shown by
(a) 1, 4-dichlorobenzene (a) dipolar character of ethers
(b) cis 1,2-dichlorobenzene (b) alcohols having resonance structures
(c) trans 1, 3-dichlorobenzene (c) inter-molecular hydrogen bonding in ethers
(d) trans 2, 3-dichloro -2- butene (d) inter-molecular hydrogen bonding in alcohols
77. Which one of the following molecules will have unequal 88. The reason for double helical structure of DNA is operation
M – F bond lengths ? of
(a) NF3 (b) BF3 (a) van der Waal’s forces (b) dipole-dipole interaction
(c) PF5 (d) SF4 (c) hydrogen bonding (d) electrostatic attraction
78. The bond length in LiF will be 89. Among the following, the molecule with the highest dipole
(a) less than that of NaF (b) equal to that of KF moment is
(c) more than that of KF (d) equal to that of NaF (a) CH3Cl (b) CH2Cl2
79. Which has the least bond angle ? (c) CHCl3 (d) CCl4
(a) NH3 (b) BeF2 90. Which of the following represents the given mode of
(c) H2O (d) CH4 hybridisation sp2 –sp2 – sp - sp from left to right ?
80. The correct sequence of decrease in the bond angles of the
following hydrides is (a) H C = CH C
2
N (b) HC C–C CH
(a) NH3 > PH3 > AsH3 > SbH3 CH2
(b) NH3 > AsH3 > PH3 > SbH3 (c) H C = C = C = CH (d) H C
2 2
2
(c) SbH3 > AsH3 > PH3 > NH3
(d) PH3 > NH3 > AsH3 > SbH3
Chemical B onding and Molecular Structure 95

1. In which of the following pairs, the two species are iso- 9. In which of the following molecules the central atom does
structure? [CBSE-PMT 2007] not have sp3 hybridization? [CBSE-PMT 2010]
(a) SO32– and NO3– (b) BF3 an NF3 (a) NH +4 (b) CH4 (c) SF4 (d) BF4–
(c) BrO3– and XeO3 (d) SF4 and XeF4
2. The correct order of increasing bond angles in the following 10. Which one of the following species does not exist under
triatomic species is : [CBSE-PMT 2008] normal conditions? [CBSE-PMT 2010]
(a) NO 2 < NO +2 < NO 2 (b) NO2 < NO2 < NO2 +
(a) Be 2 +
(b) Be2 (c) B2 (d) Li 2
(c) NO2 < NO 2 < NO2 (d) NO2 < NO2 < NO2 11. Considering the state of hybridization of carbon atoms, find
3. Four diatomic species are listed below in different sequences. out the molecule among the following which is linear ?
Which of these presents the correct order of their increasing (a) CH3– CH = CH–CH3 [CBSE-PMT 2011]
bond order ? [CBSE-PMT 2008] (b) CH3 – C C – CH3
(a) O2 < NO < C22 < He2 (b) NO < C22
+
< O2 < He 2+
(c) CH2 = CH – CH2 – C CH
(c) C 22 < He 2 < NO < O 2 (d) He2 < O2 < NO < C 22 (d) CH3 – CH2 – CH2 – CH3
4. What is the dominant intermolecular force or bond that must 12. Which of the two ions from the list given below have the
be overcome in converting liquid CH3OH to a gas? geometry that is explained by the same hybridization of
(a) Dipole-dipole interactions [CBSE-PMT 2009] orbitals, NO2–, NO3–, NH2–, NH4+, SCN– ?
(b) Covalent bonds [CBSE-PMT 2011]
(c) London dispersion forces (a) NO2– and NO3– (b) NH2– and NO3–
(d) Hydrogen bonding (c) SCN– and NH2– (d) NO2– and NH2–
5. In which of the following molecules / ions BF3, NO 2 , NH 2 13. Which of the following has the minimum bond length ?
and H2O , the central atom is sp2 hybridized ? [CBSE-PMT 2011]
[CBSE-PMT 2009] (a) O2 + (b) O2 –
(a) NH 2 and H2O (b) NO 2 and H2O (c) O2 2– (d) O2
(c) BF3 and NO2 (d) NO2 and NH 2 14. The pairs of species of oxygen and their magnetic
6. According to MO theory which of the following lists ranks behaviours are noted below. Which of the following presents
the nitrogen species in terms of increasing bond order? the correct description ? [CBSE-PMT 2011 M]
[CBSE-PMT 2009] (a) O -2 , O 22 - – Both diamagnetic
(a) N 2–
2 < N 2– < N2 (b) N 2 < N 22– < N 2– (b) O +2 ,O22 - – Both paramagnetic
(c) N 2– < N 2–
2 < N2 (d) N 2– < N2 < N 2–
2 (c) O+2 ,O2 – Both paramagnetic
7. In which one of the following species the central atom has (d) None of these
the type of hybridization which is not the same as that
15. Which one of the following pairs is isostructural (i.e., having
present in the other three? [CBSE-PMT 2010]
the same shape and hybridization) ? [CBSE-PMT 2012]
(a) SF4 (b) I3– (c) SbCl52– (d) PCl5
8. Some of the properties of the two species, NO3 and H3O+ (a) BCl3 and BrCl3 (b) NH3 and NO3
are described below. Which one of them is correct?
[CBSE-PMT 2010] (c) [ NF3 and BF3 ] (d) BF4 and NH 4+
(a) Similar in hybridization for the central atom with different 16. Bond order of 1.5 is shown by : [CBSE-PMT 2012]
structures.
(a) O +2 (b) O 2 (c) O 22 (d) O2
(b) Dissimilar in hybridization for the central atom with
different structures. 17. Which of the following species contains three bond pairs
(c) Isostructural with same hybridization for the central and one lone pair around the central atom ?
atom. [CBSE-PMT 2012]
(d) Isostructural with different hybridization for the central
atom. (a) H2O (b) BF3 (c) NH -2 (d) PCl3
96 Chemistry
18. The pair of species with the same bond order is : 29. The states of hybridization of boron and oxygen atoms in
[CBSE-PMT 2012] boric acid (H3BO3) are respectively [AIEEE 2004]
(a) O 2– (b) O +2 , NO+ (a) sp3 and sp2 (b) sp2 and sp 3
2 , B2
(c) sp2 and sp2 (d) sp3 and sp3
(c) NO, CO (d) N2, O2
30. Which one of the following has the regular tetrahedral
19. Which of the following is electron - deficient ?
structure ? [AIEEE 2004]
(a) (SiH3)2 (b) (BH3)2 [NEET 2013]
(c) PH3 (d) (CH3)2 (a) BF4 (b) SF4
20. Which one of the following molecules contains no bond?
(a) H2O (b) SO2 [NEET 2013] (c) XeF4 (d) [ Ni(CN) 4 ]2
(c) NO2 (d) CO2 (Atomic nos. : B = 5, S = 16, Ni =28, Xe = 54)
21. Which of the following is paramagnetic ? [NEET 2013] 31. The maximum number of 90º angles between bond pair-bond
pair of electrons is observed in [AIEEE 2004]
(a) O2 (b) CN–
(a) dsp2 hybridization (b) sp3d hybridization
(c) NO+ (d) CO (c) dsp3 hybridization (d) sp3d2 hybridization
22. Dipole-induced dipole interactions are present in which of 32. Which of the following species is diamagnetic in
the following pairs : [NEET 2013] nature? [AIEEE 2005]
(a) Cl2 and CCl4
(a) H 2 (b) H 2+

(b) HCl and He atoms


(c) SiF4 and He atoms (c) H 2 (d) He 2 +

(d) H2O and alcohol 33. Lattice energy of an ionic compound depends upon
23. In which of the following species the interatomic bond angle (a) Charge on the ion and size of the ion [AIEEE 2005]
is 109° 28’? [AIEEE 2002] (b) Packing of ions only
(a) NH3, (BF4)1– (b) (NH4)+, BF3 (c) Size of the ion only
(c) NH3, BF4 (d) (NH2)1–, BF3. (d) Charge on the ion only
24. Which one of the following pairs of molecules will have 34. The molecular shapes of SF4 , CF4 and XeF4 are
permanent dipole moments for both members ? [AIEEE 2005]
[AIEEE 2003] (a) different with 1, 0 and 2 lone pairs of electrons on the
(a) NO2 and CO2 (b) NO2 and O3 central atom, respectively
(c) SiF4 and CO2 (d) SiF4 and NO2 (b) different with 0, 1 and 2 lone pairs of electrons on the
25. Which one of the following compounds has the smallest central atom, respectively
bond angle in its molecule ? [AIEEE 2003] (c) the same with 1, 1 and 1 lone pair of electrons on the
(a) OH2 (b) SH2 central atoms, respectively
(c) NH3 (d) SO2 (d) the same with 2, 0 and 1 lone pairs of electrons on the
central atom, respectively
26. The pair of species having identical shapes for molecules of
35. Which of the following molecules/ions does not contain
both species is [AIEEE 2003]
unpaired electrons? [AIEEE 2006]
(a) XeF2, CO2 (b) BF3, PCl3
(c) O 22
+
(c) PF5, IF5 (d) CF4, SF4 (a) N2 (b) O2 (d) B2
27. The correct order of bond angles (smallest first) in H2S, NH3, 36. Among the following mixtures, dipole-dipole as the major
BF3 and SiH4 is [AIEEE 2004] interaction, is present in [AIEEE 2006]
(a) H2S < NH3 < SiH4 < BF3 (a) KCl and water
(b) NH3 < H2S < SiH4 < BF3 (b) benzene and carbon tetrachloride
(c) H2S < SiH4 < NH3 < BF3 (c) benzene and ethanol
(d) H2S < NH3 < BF3 < SiH4 (d) acetonitrile and acetone
28. The bond order in NO is 2.5 while that in NO+ is 3. Which of 37. In which of the following molecules/ions are all the bonds
the following statements is true for these two species ? not equal? [AIEEE 2006]
[AIEEE 2004] (a) XeF4 (b) BF4 –
(a) Bond length in NO+ is equal to that in NO
(c) SF4 (d) SiF4
(b) Bond length in NO is greater than in NO+
38. Which of the following species exhibits the diamagnetic
(c) Bond length in NO+ is greater than in NO behaviour ? [AIEEE 2007]
(d) Bond length is unpredictable (a) NO (b) O2 2– (c) O2+ (d) O2
Chemical B onding and Molecular Structure 97
39. The charge/size ratio of a cation determines its polarizing 48. Which of the following is the wrong statement ?
power. Which one of the following sequences represents [JEE Main 2013]
the increasing order of the polarizing power of the cationic –
(a) ONCl and ONO are not isoelectronic.
species, K+, Ca2+, Mg2+, Be2+? [AIEEE 2007]
(b) O3 molecule is bent
(a) Ca2+ < Mg2+ < Be2+ < K+
(b) Mg2+ < Be2+ < K+ < Ca2+ (c) Ozone is violet-black in solid state
(c) Be2+ < K+ < Ca2+ < Mg2+ (d) Ozone is diamagnetic gas.
(d) K+ < Ca2+ < Mg2+ < Be2+ 49. In which of the following pairs of molecules/ions, both the
40. In which of the following ionization processes, the bond species are not likely to exist ? [JEE Main 2013]
order has increased and the magnetic behaviour has
changed? [AIEEE 2007] (a) H 2 , He 22 (b) H 2 , He 22
(a) N 2 N2 +
(b) C2 C2 +

(c) H 22 + , He2 (d) H 2 , He 22 +


(c) NO NO +
(d) O 2 O2 +

41. Which of the following hydrogen bonds is the strongest? +


50. Stability of the species Li2, Li 2 and Li 2 increases in the
[AIEEE 2007]
order of : [JEE Main 2013]
(a) O – H - - - F (b) O – H - - - H
(c) F – H - - - F (d) O – H - - - O (a) Li 2 < Li 2 < Li 2 (b) Li 2 < Li +2 < Li 2
42. Which one of the following pairs of species have the same
bond order? [AIEEE 2008] (c) Li 2 < Li 2 < Li +2 (d) Li 2 < Li 2 < Li 2+
(a) CN– and NO+ (b) CN– and CN+
51. Among the following, the paramagnetic compound is
(c) O2 and CN– (d) NO+ and CN+
[IIT-JEE 2007]
43. The bond dissociation energy of B – F in BF3 is 646 kJ mol–1
whereas that of C – F in CF4 is 515 kJ mol–1. The correct (a) Na2O2 (b) O3
reason for higher B – F bond dissociation energy as compared (c) N2O (d) KO2
to that of C – F is [AIEEE 2008] 52. The species having bond order different from that in CO is
(a) stronger bond between B and F in BF3 as compared (a) NO– (b) NO+ [IIT-JEE 2007]
to that between C and F in CF4. (c) CN – (d) N2
(b) significant p – p interaction between B and F in BF3
53. Assuming that Hund’s rule is violated, the bond order
whereas there is no possibility of such interaction
and magnetic nature of the diatomic molecule B2 is :
between C and F in CF4.
(c) lower degree of p – p interaction between B and F in [IIT-JEE 2010]
BF3 than that between C and F in CF4. (a) 1 and diamagnetic (b) 0 and dimagnetic
(d) smaller size of B– atom as compared to that of C– atom. (c) 1 and paramagnetic (d) 0 and paramagnetic
44. The number of types of bonds between two carbon atoms 54. The species having pyramidal shape is : [IIT-JEE 2010]
in calcium carbide is [AIEEE 2011 RS]
(a) SO3 (b) BrF3
(a) One sigma, One pi
(b) Two sigma, one pi (c) SiO32– (d) OSF2
(c) Two sigma, two pi
(d) One sigma, two pi 55. Geometrical shapes of the complexes formed by the reaction
45. The molecule having smallest bond angle is : of Ni2+ with Cl– , CN– and H2O, respectively, are
(a) NCl3 (b) AsCl3 [AIEEE 2012] [IIT-JEE 2011]
(c) SbCl3 (d) PCl3 (a) octahedral, tetrahedral and square planar
46. In which of the following pairs the two species are not (b) tetrahedral, square planar and octahedral
isostructural ? [AIEEE 2012] (c) square planar, tetrahedral and octahedral
(a) CO32 - and NO3 (d) octahedral, square planar and octahedral
(b) PCl+4 and SiCl4 56. In allene (C3H4), the type(s) of hybridisation of the carbon
(c) PF5 and BrF5 atoms is (are) : [IIT-JEE 2012]
(d) AlF63 and SF6 (a) sp and sp3 (b) sp and sp2
47. Which one of the following molecules is expected to exhibit (c) only sp3 (d) sp2 and sp3
diamagnetic behaviour ? [JEE Main 2013]
(a) C2 (b) N2 (c) O2 (d) S2
98 Chemistry

1. Sodium chloride is an ionic compound whereas hydrogen 10. Which one of the following arrangements of molecules is
chloride is mainly covalent because correct on the basis of their dipole moments?
(a) Sodium is less reactive (a) BF3 > NF3 > NH3 (b) NF3 > BF3 > NH3
(b) Hydrogen is non-metal
(c) NH3 > BF3 > NF3 (d) NH3 > NF3 > BF3
(c) Hydrogen chloride is a gas
11. Which of the following is the correct electron dot structure
(d) Electronegativity difference in the case of Hydrogen and
of N2O molecule?
chlorine is less than 2.1. .. + ..
2. For two ionic solids, CaO and Kl. Identify the wrong (a) : N = N = O : (b) : N = N = O
.. :
statement among the following .. .. .. ..
(a) Lattice energy of CaO is much larger than that of Kl (c) N = N = O : (d) : N = N = O ..
:
..
(b) KI is soluble in benzene 12. Consider the following statements. The common features of
(c) CaO has higher melting point the molecules BF3, CO & NO are that
(d) KI has lower melting point I. all are Lewis acids
3. The correct order of increasing C - O bond length of CO, II. all are gaseous in nature
CO 2 –
, CO2 is III. all contain unpaired electrons
IV. all do not conform to the octet rule
3


(a) CO 2

3
< CO 2
< CO (b) CO 2
< CO 2

3
< CO Which of the above statements are correct ?
(a) I and II (b) III and IV
(c) CO < CO – < CO (d) CO < CO < CO –
2

3 2 2
2

3 (c) I and III (d) II and IV


4. Anti - bonding molecular orbital is formed by 13. The nodal plane in the -bond of ethene is located in
(a) Addition of wave functions of atomic orbitals (a) the molecular plane
(b) Subtraction of wave functions of atomic orbitals (b) a plane parallel to the molecular plane
(c) Multiplication of wave functions of atomic orbitals (c) A plane perpendicular to the molecular plane which
(d) None of these bisects the carbon-carbon sigma bond at right angle
5. Among the following species, identify the isostructural pairs (d) a plane perpendicular to the molecular plane which contains
NF3 , NO3– , BF3 , H3O , HN3 + the carbon - carbon -bond
14. Which of the following statements is true ?
(a) [NF3 , NO3– ]and[BF3 , H3O ] +
(a) HF is less polar than HBr
(b) Absolutely pure water does not contain any ions
(b) [ NF , HN ] and[ NO – , BF ]
3 3 3 3
(c) Chemical bond formation takes place when forces of
attraction overcome the forces of repulsion
(c) [NF3 , H3O +
]and[NO3– , BF3 ]
(d) In covalencey, transference of electrons takes place
(d) [NF3 , H3O + ]and[HN3 , BF3 ] 15. In the anion HCOO– the two carbon - oxygen bonds are
6. Which of the following is planar ? found to be of equal length. What is the reason for it ?
(a) XeO4 (b) XeO3F (a) Electronic orbitals of carbon atom are hybridised
(c) XeO2F2 (d) XeF4 (b) The C = O bond is weaker than the C –O bond
7. The correct order of hybridization of the central atom in the (c) The anion HCOO– has two resonating structures
following species NH3, [PtCl4]2–, PCl5 and BCl3 is (d) The anion is obtained by removal of a proton from the
(a) dsp2, dsp3, sp2 and sp3 (b) sp3, dsp2, sp3d, sp2 acid molecule
(c) dsp2, sp2, sp3, dsp3 (d) dsp2, sp3, sp2, dsp3 16. The pair of species having identical shapes for molecules of
8. The common features among the species CN–, CO and NO+ both species is
are (a) CF4, SF4 (b) XeF2, CO2
(a) bond order three and isoelectronic (c) BF3, PCl3 (d) PF5, IF5
(b) bond order three and weak field ligands 17. The solubilities of carbonates decrease down the magnesium
(c) bond order two and - acceptors group due to a decrease in
(d) isoelectronic and weak field ligands (a) lattice energies of solids
9. Number of sigma bonds in P4O10 is (b) hydration energies of cations
(a) 6 (b) 7 (c) inter-ionic attraction
(c) 17 (d) 16 (d) entropy of solution formation
Chemical B onding and Molecular Structure 99
18. In compounds of type ECl3, where E = B, P, As or Bi, the 26. Anhydrous AlCl3 is covalent but hydrated AlCl3.6H 2 O
angles Cl - E- Cl for different E are in the order.
is ionic because
(a) B > P = As = Bi (b) B > P > As > Bi
(c) B < P = As = Bi (d) B < P < As < Bi (a) AlCl3 dissolves in CS 2
19. Specify the coordination geometry around and hybridisation (b) AlCl3 has planar structure
of N and B atoms in a 1 : 1 complex of BF3 and NH3
(c) IE of Al is low
(a) N : tetrahedral, sp3; B : tetrahedral, sp3
(d) Hydration energy of Al compensates the IE
(b) N : pyramidal, sp3; B : pyramidal, sp3
(c) N : pyramidal, sp3; B : planar, sp2 27. The molecule XY2 contains two and two bonds and
(d) N : pyramidal, sp3; B : tetrahedral, sp3 one lone pair of electrons in valence shell of X. The
20. Which of the following are isoelectronic and isostructural? arrangement of lone pair and bond pairs is
NO3–, CO32–, ClO3–, SO3 (a) linear
(a) NO3–, CO32– (b) SO3 ,NO3– (b) trigonal planar
(c) ClO3–, CO32– (d) CO32–, SO3 (c) square pyramidal
(d) unpredictable
21. Consider the chemical species NO 3 , NO 2 and NO 2 and
+

28. A bonded molecule MX 3 is T-shaped. The number of


point out the correct statement given below
non bonded pair of electrons is
(a) The hybrid state of N in NO 2 is sp 2
+
(a) 0
(b) The hybrid state of N in all the species is the same (b) 2
(c) 1
(c) The shape of both NO 2 and NO 2 is bent while NO 3
+
(d) can be predicted only if atomic number is known
is planar 29. Which of the following overlap is correct ?
(d) The hybrid state of N in NO 3 and NO 2 is the same + + +
(a) +
– – –
22. What is the true about – NF5
(a) The molecule has trigonal bipyramidal geometry (b) + – + .+.
(b) The oxidation state of N is 5
(c) N – F bonds are covalent Co-ordinate bonds (c) + – + + – – –
+
(d) The molecule does not exist
(d) None of the above
23. The molecules BF3 an d NF3 are both covalent
OH
compounds, but BF3 is non polar whereas NF3 is polar.. 30. The vapour pressure of is higher than
The reason for this is
NO 2
(a) atomic size of Boron is larger than nitrogen
(b) Boron is metal while nitrogen is gas OH
(c) B – F bonds are non-polar while N – F bonds are due to
polar
O 2N
(d) BF3 is planar but NF3 is pyramidal
(a) Dipole moment
24. In a metallic crystal (b) Dipole- dipole interaction
(a) Both kernels as well electrons move rapidly (c) H – bonding
(b) The valence electrons remain within the field of (d) Lattice structure
influence of their kernels 31. Of the molecules XeF4, SF4, SiF4 one having tetrahedral
(c) The valance electrons constitute sea of mobile structure
electrons
(a) Only SF4 and XeF4 (b) Only SiF4
(d) The valance electrons are localised between two (c) Both SF4 and SiF4 (d) All the three
kernels
32. Considering greater polarisation in LiCl than in NaCl which
25. Which of the following accounts for the bond angle of
of the following is wrong
104° in water molecule
(a) Fused LiCl would be less conducting than fused NaCl
(a) sp 3 hybrid state of oxygen (b) LiCl has lower melting point than NaCl
(b) high electro negativity of H-atoms (c) LiCl dissolves more in organic solvents whereas NaCl
(c) low ionisation energy of H-atoms does not
(d) LiCl would ionise in water more than NaCl
(d) lone pair-bond pair repulsions
100 Chemistry
33. In which of the following set of molecules the order of (a) 2.86 D (b) 2.25 D
boiling point is incorrect. (c) 1.5 D (d) 0 D
42. The experimental value of the dipole moment of HCl is 1.03
(a) C3H 8 > C 2H6 > CH 4 (b) Xe > Ar > He
D. The length of H – Cl bond is 1.275 Å. The percentage of
(c) HCl > HF > HBr (d) H 2 O > H 2Se > H 2S ionic character on HCl
34. The stability of ionic crystals depends mainly on (a) 7 (b) 17
(a) low heat of sublimation of cation formed (c) 43 (d) 21
(b) high electron affinity of anion formed 43. CaO and NaCl have the same crystal structure and
(c) low I.E. of cation forming species. approximately the same ionic radii. If U is the lattice energy
(d) lattice energy of crystal of NaCl, the approximate lattice energy of CaO is
35. Silicon carbide is a (a) U/2 (b) U
(a) covalent solid (b) ionic solid (c) 4 C (d) 2 U
(c) molecular solid (d) None of the above 44. Which of the following compounds has the highest lattice
36. If x-axis is the molecular axis of a diatomic molecule, then energy?
molecular orbital are formed by overlap of (a) MgO (b) LiCl
(a) s-atomic orbitals (c) NaCl (d) LiF
(b) s and px atomic orbitals 45. A diatomic molecule has a dipole moment of 1.2 D, if the
(c) py and py-orbitals bond distance is 1 Å, what percentage of electronic charge
(d) px and pz-orbitals exists on each atom.
(a) 25 % of e (b) 29 % of e
37. The molecular size of ICl and Br2 is approximately same, (c) 19 % of e (d) 12 % of e
but boiling point of ICl is about 40°C higher than b.pt. of 46. Which of the following statements is/are true
Br2 , because of 1. PH 5 and BiCl5 donot exist
(a) I has larger size than Br 2. p –d bond is present in SO 2
(b) I-Cl bond is stronger than Br-Br bond
(c) I E of lodine < I E of Br 3. I3 has bent geometry
+

(d) ICl is polar while Br2 is non polar 4. SeF4 and CH 4 have same shape
38. An element forms compounds of the formula ACl3 , A 2O 5 (a) 1, 2, 3 (b) 1, 3
and Mg 3A 2 but does not form ACl5 , the A could be (c) 1, 3, 4 (d) 1, 2, 4
47. Trimethylamine is a pyramidal molecule
(a) Al (b) P
(c) B (d) N N
CH 3 and formamide is a planar molecule
39. The magnitude of the lattice energy of a solid increases if H 3C CH 3
(a) charges on both the ions are small
(b) the ions are large O
(c) the ions are small ||
(d) the ions are of equal size C H
, The hybridisation of Nitrogen in both is
40. The correct representation of H-bond in solid HF H N
(a) H – F H–F H–F |
F F F
H
(b) H H H H H 3 2
F F (a) sp 2 , sp 2 (b) sp , sp
H H H H
(c) F F (c) sp 3 , sp 3 (d) sp 2 , sp
F
H H 48. If climbing of water droplets is made to occur on a coated
(d) F F F F microscope slide, the slide would have to be coated in
H which of the following way
Cl

41. The dipole moment of chlorobenzene is 1.5 D.


A B C D
Cl (a) A (b) B
(c) C (d) D
Cl
The dipole moment of is
Cl Cl
Chemical B onding and Molecular Structure 101

EXERCISE 1 3 3
O O
1. Bonding molecular orbital. | |
2. Because b.p. –b.p. repulsion is less in H2S than in H2O. 17. (c) O P O O P= O
3. MgO || |
O O
4. III, because like charges reside on adjacent atoms.
5. 1 – butyne has more dipole moment. 3 3
O O
6. (i) Non-polar due to symmetric structure. || |
(ii) Non-polar due to linear molecular structure. O P O O= P O
(iii) Polar due to non-symmetric molecular structure. | |
O O
7. (a) A – group 1, B – group 14, C – group 17
(b) A, C are expected to form ions A+ and C– Bond order
A Number of bonds 5
.. = = = 1.25
(c) (i) A : B : A (ii) A : C : or A+Cl +
Number of Resonating structures 4
..
A Three unit negative charge is being shared by four O atoms.
11. BeCl2 Since Be2+ ion is small in size. Formal charge = –3/4
19. (b) 20. (c) 21. (b)
18. (a) O – has Bond order 1.5 (see text)
22. (c) 23. (c) 24. (a) 2

25. (c) 26. (b) 27. (d) 19. (c)


28. (b) 2
*12p x
2p x
2 2 2 *2 2 *
20. (c) *1s
EXERCISE 2 1s 2s 2s 2pz 2
*12p y
2pz
2p y
1. (c) At bond distance the attractive forces overweigh the
There are 7 pair of electrons
repulsive forces.
2. (a) In NaCl the bonding is ionic (electrovalent) 21. (a) Bond order in N2 and O 2 +

2
is 3 (calculate by energy level
3. (b) Covalent ( Fajan's rule see text)
diagram)
4. (d) *2px2 *2py1 antibonding molecular orbitals contain
22. (b) H+
three electrons in O – . Their energy being more and
2

ionisation potential less.


2
* 2

23. (c)
2
* 2 2
* 2 2
*
5. (a) Polarizing power = charge/ radius. Thus µ charge.
2
* 2

6. (d) The larger the size of anion the more is its polarizability 4 pairs
7. (b) Covalent bonds are directional in nature 24. (b) CO is diamagnetic while others are paramagnetic
8. (d) BCl3 is lewis acid and electron deficient. The octet around
25. (c) O – (see text)
B is not complete 2

9. (c) Statement 1 is correct (see text) 26. (d) In PH6 , P – H bond will have ionic character due to
+

10. (d) number of bonds is 5 electron attracting tendency of P carrying +ve charge
and least covalent.
Nb8 2 Na 27. (c)
11. (b) Bond order = = 3 =
2 2 28. (d) The electronegativity of S is least among others hence H
12. (c) Bond order in NO is 2.5 (see text) -S bond is least polar in nature.
13. (d) Bond order in He2 is zero (see text) 29. (b) Polarizing power of Zn2+ is more than others, hence ZnCl2
is least ionic in nature.
14. (c) Species N2 O2 N–2
O–2 30. (a) Electropositive character and size increases down the
Bond order 3 2 2.5 1.5 group, the ionic character increases.
The O-O bond is O – decreases 31. (a) NH 4Cl contains both covalent and ionic bonds
2

+
2 + H
15. (d) b *a2 2
b( *2
2
a
1 2
b ) b (N 2 = 13electrons)
b |
it contains one unpaired electron hence paramagnetic H – N– H Cl –
|
16. (c) NO is paramagnetic H
102 Chemistry
32. (b) Electronegativity difference is 4.0 - 1.20 = 2.8 percentage (–)
ionic character is 72.24% when the electronegativity O
difference is 1.7, the % ionic character is approx 51%. O–N
57. (d) it has 4 bond pairs and none lone pair..
33. (a) Ionic character follows the order HF > HCl > HBr > HI O
34. (d)
35. (c) H3C – CH2 – CH = CH2. It has 11 bonds 58. (b) F – O – F 2 bond pairs and 8 lone pairs
36. (c) Hybridisation in PF5 is = ½ [5 + 5 + 0 –0] = 5 sp3d, hence
..
structure is trigonal bipyramidal. :O:
H .. ­ ..
H
59. (d) All contain coordinate bond H – O
..
– S –O
..
–H ,
37. (b) C = C Hybridisation of C is sp2 ¯
:O
. .:
H H
38. (d) Hybridisation in Si(CH3)4 ( ½ [4 + 4 + 0 – 0] = 4) is sp3 ..
which is tetrahedral. :O:
..
39. (d) Hybridisation in SnCl2 (½ [4 + 2 + 0 - 0] =3) sp3. It has V- O
shape. S
,
. O :O:

..
.O

..
O

..
..

..
40. (a) H2CO3 and BF3 both have triangular planar structure

..
F H–O 60. (a) Coordinate bond
B–F C = O.
F H–O 61. (c) CO 2 – Its structure is
BF3 is symmetrical
\ m = 0 hence non polar, H2CO3 is not symmetrical
hence polar in nature. 62. (b) It is 4 See H - bonding.
41. (b) SO2 has sp2 hybridisation. 63. (d) NH3 forms intermolecular H - bonding.
64. (b) In H - bonding atoms involved are F, O and N.
42. (c) AsF5 is trigonal bipyramidal and sp3d hybridised.
65. (a) H -bonding interactions.
43. (c)
66. (c) It is due to H - bonding.
44. (c) Hybridisation of N = ½ [5 + 3 + 0 – 0] = 4 hence sp 3 67. (d) In HCl No–H–bonding, H – F form zig zag; (cage like by
45. (a) SF6 is octahedral (see text) H2O, only NH3 forms linear polymeric structures)
46. (b) NO +2 = ½ [5 + O + O –1] = 2 sp; NO 2– = ½ [5 + 0 + 1 –0] = 3 68. (a) H2O (see H - bonding)
69. (b) C2H5OH soluble in water due to H-Bonding
sp2; NH + = ½ [5 + 4 + 0 - 1] = 4 sp3 70. (d) Metallic bond is electrostatic in nature.
71. (b) Non polar molecules have dispersive forces.
X 72. (d) The electronegativity difference is maximum between N &
47. (d) M three angle below M and three above M H hence NH3 will have highest magnetic moment.
X X X
H
hence = 6 O
48. (c)
73. (d) H2O2 it is nonplanar O
49. (b) NO2+ has sp hybridisation hence linear. H
50. (c) O = C = C = C = O is sp hybridised
51. (d) In XeF2 Total number of valence electrons of Xe = 8, two The bonds are not in the plane of paper.
electrons shared with 2F atoms, 6 electrons left hence 3
74. (b) H C . C º C . CH symmetrical and linear. Hence D.M.
lone pairs, in XeF4 4 shared with 4 F atoms 4 left hence 2
lone pairs; in XeF6 6 shared with 6 F atoms 2 left hence 1 ( m ) = 0.
lone pair. 75. (c) In SiF4 DM = zero due to symmetrical structure.
52. (b) In CH3 CH2OH underlined C is forming 4 bonds, hence
Cl
sp3 hybridisation. In others it is sp2 hybridised (due to 3 Cl
bonds). 76. (b) Cis1, 2 - dichloro benzene will have some D.M.
53. (b) dsp2 hybridisation involves d x 2 - y2 . F
F
54. (b) Tetrahedral 77. (c) In PF5 the hybridisation is sp3d P–F axial. P – F bonds
55. (b) Hybridisation is sp3 and shape pyramidal
F F
56. (b) Hybridisation in PO – = ½ [5 + 0 + 3 –0] = 4 sp3. In are longer than equatorial bonds.
bonding only d orbital of P, p orbital of O can be 78. (a) It is due to small size of Li compared to size of Na and K.
involved. Since hybrid atomic orbitals do not 79. (c) Bond angles NH3 (107.5º); BeF2 (180º) H2O (104.5º) and
form bond. CH4 (109º28’)
Chemical B onding and Molecular Structure 103

80. (a) Bond angles NH PH AsH SbH 3. (d) Calculating the bond order of various species.
10 . º 2 1 0 O-2 : KK s 2s 2 , s* 2 s 2 , s2 pz2 ,
81. (d) M +2 (SO ) – Here metal atom is trivalent. Hence formula
p2 px2 = p2 p 2y , p* 2 px2 = p* 2 p1y
for phosphate will be MPO4
82. (c) Since F form H-bond [HF2]– exists. Therefore KHF2 gives 1
K+ + HF2– B.O. = (Nb - N a )
2

8-5
83. (c) It has two nodal planes. It is p*2py = or 1.5
2
NO =
84. (b) Electronic configuration reveals it is monovalent (in fact K K 2s 2 , 2s 2 ,62 p 2z , 2 p x2 = p2 p 2y , p* 2 p1x
Na) hence its oxide will be M2O.
– – Nb - N a 8 - 3
– – B.O. = = or 2.5
O O O 2 2
85. (b) C–O C=O C–O
2- 2 * 2 2
O O O = C2 : KK s2s , s 2s , p2 px = p2 p 2y , s 2 p z2
– –

Nb - N a 8 - 2
4–2 B.O. = = or 3
86. (b) B.O. = ½ [Nb – Na] Li 2 = = 1; N2 = 3; Be = 0, O2=2. 2 2
2
87. (d) In alcohol intermolecular H-bonding is possible whereas He 22+ = s1s 2 s*1s1
in ether it is not possible.
88. (c) Double helical structure of DNA is stabilised by Nb - N a 2 - 1
B. O. = = or 0.5
H - bonding. 2 2
89. (a) Dipole moment follows the order CH3Cl > CH2Cl2 > CHCl3 From these values we conclude that the correct order
> CCl4 of increasing bond order is
90. (a) CH 2 = C H - C º N He22+ < O 2- < NO < C22-
1 2 3 4
4. (d) Due to intermolecular hydrogen bonding in methanol,
3 bonds (sp 2 hybridisation);2 bonds (sp-
it exist as assosiated molecule.
hybridisation)
5. (c) On determining hybridisation from H = 1/2 (V + M – C + A).
C1 = 3 bonds, C2 = 3 bonds,
The hybridisation of BF3, NO2–, NH2– and H2O are
C3 = 2 bonds
sp2, sp2, sp3 and sp3 respectively.
EXERCISE 3 6. (a) Molecular orbital configuration of
1. (c) Both BrO3 and XeO3 have sp3 hybridization but N 2– 2 2 2 2
2 = s1s s *1s s2 s s * 2 s -
due to presence of one lp of electrons they have 2 1
ïp 2 px ï p * 2 px
trigonal pyramidal geometry. s2 p2z
2 1
2. (b) From the structure of three species we can determine ïp 2 p y ïp * 2 p y
the number of lone pair electron(s) on central atom (i.e. 10 – 6
Bond order = =2
N atom) and thus the bond angle. 2
p2 px2 1
. .N. +
N 2– = s1s 2 s *1s 2 s2 s 2s * 2 s 2 ï s2 pz2
ï p * 2 px
.. ....O.. ..O ......
N N 2 0
..O.. .... O ..O ïp2 p y ï p *2 p y
|
||
|
|

||
||

..

O
..

- + 10 5
(NO )2 NO 2 NO 2 Bond order = = 2.5
2
We know that higher the number of lone pair electron(s) 2 px2
on central atom, greater is the lp – lp repulsion. Thus N 2 = s1s 2 s *1s 2 s 2 s 2 s * 2 s 2 , 2 pz2
2 p 2y
smaller is bond angle.
The correct order of bond angle is 10 – 4
Bond order = =3
+
2
NO -2 < NO 2 < NO 2 i.e., option (b) is correct. \ The correct order is = N 2– –
2 < N2 < N2
104 Chemistry

5+5+2 14. (c) MOT configurations of O2 and O2+ :


7. (c) For SbCl52– , H = = 6 means sp3d 2 O2+ : (s1s)2 (s*1s)2 (s2s)2 (s*2s)2 (s2pz)2
2
hybridization (p2p2x = p2p2y) (p*2px1 = p*2p0y)
Number of unpaired electrons = 1, so paramagnetic.
I3– , SF4 , and PCl5 ; all have sp3 d hybridization.
O2 : (s1s)2 (s*1s)2 (s2s)2 (s*2s)2 (s2pz)2
8. (b) In NO3 , nitrogen have sp2 hybridisation, thus planar (p2p2x = p2p2y) (p*2px1 = p*2p1y)
Number of unpaired electrons = 2, so paramagnetic.
in shape. In H3 O+ , oxygen is in sp3 hybridisation,
15. (d) BF4 hybridisation sp3, tetrahedral structure.
thus tetrahedral geometry is expected but due to
presence of one lp of electrons on central oxygen atom NH +4 hybridisation sp3, tetrahedral structure.
it is pyramidal in shape.
16. (b) (O2) = 1s 2 , 1s 2 , 2s 2 , 2s 2 , 2 p z2 ,
9. (c) NH +4 : sp3 hybridisation
2 p x2 = 2 p 2y , 2 p1x = * 2 Py1
CH4: sp3 hybridisation
SF4: sp3d hybridisation N b - N a 10 - 6 4
Bond order = = = =2
BF4– : sp3 hybridisation 2 2 2
10. (b) Bond order of Be2 = 0, hence Be2 cannot exist. ( +
O 2 ion = 1)
s 2
, 1s 2
, 2 s 2
, 2 s 2
, 2 p z2 ,
2
sp 3 sp sp sp 3 2 px2 = 2 p y , 2 p1x
11. (b) H3C — C C — CH3
Nb - N a 10 - 5 5 1
Linear due to sp hybridized C atom. Bond order = = =2
2 2 2 2
1
12. (a) Hybridisation =
2
[No. of valence electrons of central ( O -2 ) = 1s 2 , 1s 2 , 2s 2 , 2s 2 , 2 Pz2 ,
atom + No. of monovalent atoms attached to it + 2 px2 = 2 p y2 , 2 px2 = 2 p1y
Negative charge if any – Positive charge if any]
1
NO2– H = [5 + 0 + 1 - 0] = 3 = sp 2 Bond order = ( N b - N a ) = 10 - 7 = 3 = 1 1
2 2 2 2 2
1
NO3– H = [5 + 0 + 1 - 0] = 3 = sp 2
2
( )
2 2 2 2
O2 = 1s , 1s , 2s , 2s , 2 p z , 2 2

1 2 px2 = 2 p y2 , 2 px2 = 2 p 2y
NH2– H = [5 + 2 + 1 + 0] = 4 = sp3
2
Nb - Na 10 - 8 2
1 Bond order = = =1
NH4+, H = [5 + 4 + 0 - 1] = 4 = sp 3 2 2 2
2
SCN– = sp 17. (d) PCl3
i.e., NO2– and NO3– have same hybridisation.
P
13. (a) O2 (16) 1s 2 , *1s 2 , 2s 2 , * 2s 2 , 2 p 2z , Cl
Cl
Cl
2 px2 2 p 2y , * 2 p1x * 2 p1y
18. (a) Both O2–
2 and B2 has bond order equal to 1.
B.O. = ½ (Nb – Na) = ½ (10 – 6) = 2
O2 + (15) 1s 2 , *1s 2 , 2s 2 , * 2s 2 , 2 p z2 , B2 (10) = 1s 2 *
1s 2 2s 2 * 2s 2
2 p1x = 2 p y1
2 px2 2 p 2y , * 2 p1x * 2 p0y
Nb - N a
B.O. = ½ (Nb – Na) = ½ (10 – 5) = 2.5 Bond order =
2
O2 (17) 1s 2 , *1s 2 , 2s 2 , * 2s 2 , 2 p z2 , 6-4 2
= =1
=
2 px2 2 p 2y , * 2 px2 * 2 p1y 2 2
B2 is known in the gas phase
B.O. = ½ (Nb – Na) = ½ (10 – 7) = 1.5
O22 - = (s1s )2 (s *1s)2 ( s2 s ) 2 (s * 2 s ) 2 ( s2 p z )
2
O2 2 (18) 1s 2 , *1s 2 , 2 s 2 , * 2 s 2 , 2 p z2 ,
( )( ) =( )
2 2 2
2 px2 2 p 2y , * 2 px2 * 2 p 2y ( 2 px ) =
2
2 py *
2 px *
2 py

B.O. = ½ (Nb – Na) = ½ (10 – 8) = 1 1


Since, the bond length decreases as the bond order
Bond order (10 8) = 1
2
increases, hence, O2+ have least bond length.
Chemical B onding and Molecular Structure 105
19. (b) (BH3)2 or (B2H6) Number of 90° angle Number of 90° angle
H H between bonds = 4 between bonds = 6
H
B B
H H H
It contains two 3 centre - 2 electron bonds and present
above and below the plane of molecules compounds
which do not have sufficient number of electrons to
form normal covalent bonds are called electron defi-
cient molecules.
sp3d2 hybridisation
20. (a) O O ¬ S=O O ¬ N =O O = C = O Number of 90° angle
H H between bonds = 12
32. (c) H2 is diamagnetic as it contains all paired electrons
21. (a) Molecular orbital configuration of O-2 is
H 2 = sb2 , H 2+ = s1b , H 2- = sb2 , s*1 +
a ; He 2
O-2 (17) = s1s2, s*1s2, s2s2, s*2s2, (diamagnetic) (paramagnetic) (paramagnetic) (paramagnetic
s2pz2, p2px2 = p2py2, p*2px2 = p*2py1
+
22. (b) This type of attractive force operates between the s*1
a ; He 2 = sb2 , s*1
a
polar molecules having permanent dipole and the (paramagnetic) (paramagnetic)
molecules lacking permanent dipole.
33. (a) The value of lattice energy depends on the charges
HCl is polar (m ¹ 0) and He is non polar
present on the two ions and the distance between them.
(m = 0), thus gives dipole-induced dipole interaction.
34. (i) SF4 – Configuration of excited S atom :
23. (a) In NH3 and BF4- the hybridisation is sp3 and the bond 3s2 3p
3 1
3d
angle is almost 109º 28' sp d
3

24. (b) Both NO2 and O3 have angular shape and hence will shape-square pyramidal, one lone pair
have net dipole moment.
CF4 – Configuration of excited C-atom :
25. (b) In H2S, due to low electronegativity of sulphur the l.p. -
3
l. p. repulsion is more than bp. - bp. repulsion and hence 2s1 2p
3
the bond angle is 92º. sp
26. (a) Both XeF2 and CO2 have a linear structure. shape-tetrahedral; no lone pair
27. (a) The order of bond angles XeF6 – configuration of excited Xe atom :
BF3 > SiH 4 > NH 3 > H 2S
120º 109º 28¢ 107 º 92.5º 4 2
5s2 5p 5d
3 2
28. (b) The lower the bond order the greater the bond length sp d hybridization
and vice-versa. shape-square planar, 2 lone pairs
29. (b) In H 3BO 3 hybridisation of B is sp2 and O is sp3. 35. (c) The distribution of electrons in MOs is as follows:

30. (a) XeF4 (sp3d 2 square planar), p2 s1 p* s*


N2+(electrons 13) s2 s*2 s2 s*2
p2 p*
[ Ni(CN) 4 ]2 - (dsp2 square planar ), 2 *1
O2 (electrons 16) s2 s*2 s2 s*2 s2 p 2 p*1 s*
p p
BF4- (sp3 tetrahedral), SF4 (sp3d see saw shaped)
2 *
O22– (electrons 18) s2 s*2 s2 s*2 s2 p 2 p* s*
p p
1
M B2 (electrons 10) s2 s*2 s2 s*2 p1 s p* s*
31. (d) p
Only O22– does not contain any unpaired electron.

dsp2 hybridisation sp3d or dsp3 hybristion


106 Chemistry
42. (a) For any species to have same bond order we can expect
d+ d-
36. (d) Acetonitrile ( CH 3 - C º N ) and acetone them to have same number of electrons. Calculating
the number of electrons in various species.
d+ O2– (8 + 8 + 1 = 17) ; CN - (6 + 7 + 1 = 14)
( CH3) d– NO+(7 + 8 – 1= 14); CN+ (6 + 7 –1 = 12)
d– C = O) both are polar molecules, hence We find CN– and NO+ both have 14 electrons and bond
( CH3) order 3.
dipole-dipole interaction exist between them. Between 43. (b) The delocalised p p bonding between filled p-
KCl and water ion-dipole interaction is found and in orbital of F and vacant p-orbital of B leads to shortening
benzene-ethanol and benzene-carbon tetra chloride of B–F bond length which results in higher bond
disperion force is present dissociation energy of the B–F bond.
37. (d) In SF4 the hybridisation is sp3d and the shape of
molecule is
F
F
F B F
S F

F
F Vacant Filled
2p-orbital 2p -orbital
38. (b) Diamagnetic species have no unpaired electrons
O 2 2- Þ s1s2, s*1s2, s2s2, s*2s2, 2 p z ,
2
F F
+
p2px2 = p2py2, p*2px2 = p*2py2 B=F B–F
+
Whereas paramagnetic species has one or more F F
unpaired electrons as in +1/3
+
F F
O2 2
1s , 2
1s , 2 s , 2
2s ,2
2 p 2z ,{ 2 p 2x 2 p 2y , +1/3
B–F B F
+1/3
{ 2 p1x = 2 p 1y = 2 unpaired electrons F F
44. (d) Calcium carbide exists as Ca2+ and C22–. According to
O2 1s 2 , 1s 2 , 2 s 2 , 2 s 2 , 2 pz2 , the molecular orbital model, C 2 2– should have
2 molecular orbital configuration :
2 px2 = 2 p y 2 p1x * 2 p0y = 1 unpaired
unpaired electron
electron
s1 s 2 , s *1 s 2 , s 2 s 2 , s *2 s 2 ,
NO 1s 2 , 1s 2 , 2 s 2 , 2s 2 , 2 p 2z ,
{p 2 px2 = p 2 p 2y }, s 2 pz2
* *
p2 px2 = p2 p2y , p 2 p1y =p 2 pz0 = 1 unpaired electron Thus M.O. configuration suggests that it contains one
39. (d) Smaller the size and higher the charge more will be the & 2p bonds.
polarising power of cation. Since the order of the size 45. (c) All the members form volatile halides of the type
AX3. All halides are pyramidal in shape. The bond
of cation is K + > Ca ++ > Mg ++ > Be++ . So the
angle decreases on moving down the group due to
correct order of polarising power is decrease in bond pair-bond pair repulsion.
K+ < Ca2+ < Mg2+ < Be2+ 46. (c) PF5 trigonal bipyramidal
40. (c) (i) N2 : bond order = 3, diamagnetic
N2– : bond order = 2.5, paramagnetic F
F
(ii) C2 : bond order = 2, diamagnetic
F P
C2+ : bond order = 1.5, paramagnetic F
(iii) NO : bond order = 2.5, paramagnetic
F
NO+ : bond order = 3, diamagnetic
(iv) O2 : bond order = 2, paramagnetic BrF5 square pyramidal (distorted due to presence of
one lp of electrons on central atom)
O2+ : bond order = 2.5, paramagnetic
41. (c) Greater the difference between electro-negativity of F
bonded atoms, stronger will be the bond. Since F is F F
most electronegative hence F – H ...... F is the strongest Br
bond. F F
Chemical B onding and Molecular Structure 107
47. (a) & (b) The molecular orbital structures of C2 and N2 are (iii) No. of valence electrons of all atoms in N2O
N2 1s 2
*1s 2
2s 2
* 2s 2
2 px2 2 py2 2 pz2 = 2 × 5 + 6 = 16. Hence, here also all electrons are
paired. So it is diamagnetic.
C2 1s 2 *1s 2 2s 2 * 2s 2 2 py 2 2 Pz2
(iv) In KO2, we have O2 . No. of valence electrons of
Both N2 and C2 have paired electrons, hence they are
diamagnetic. all atoms in O2 = 2 × 8 + 1 = 17,
48. All options are correct,
1s 2 , *1s 2 , 2s 2 , * 2s 2 , 2 pz2
(a) ONCl = 8 + 7 + 17 = 32e not isoelectronic
ONO = 8 + 7 + 8 + 1 = 24e 2 px2 2 p 2y , * 2 px2 * 2 p1y
..
7 8Å O 1.278A° 2 Thus it has one unpaired electron, hence it is
(b) 1. 2 The central atom is sp paramagnetic.
O 116.8° O hybridized with one lone pair.
52. (a) Molecular electronic configuration of
(c) It is a pale blue gas. At – 249.7°, it forms violet black
crystals. CO: 1s 2 , *1s 2 , 2 s 2 , * 2 s 2 ,{ 2 px 2 2 p y 2 , 2 pz 2
(d) It is diamagnetic in nature due to absence of unpaired
electrons. Nb Na 10 4
Therefore, bond order = = =3
49. (c) H2+
2 = s1s s*1s
0 0 2 2
1 NO+ : 1s2 , *1s2 , 2 s 2 , * 2 s 2 , 2 pz 2 ,{ 2 px 2 2 py2
Bond order for H2+
2 =
(0 0) = 0
2
He2 = s1s2s*1s2 10 4
Bond order = =3
2
1
Bond order for He2 = (2 2) = 0
2 CN 1s 2 , *1s 2 , 2 s 2 , * 2s 2 ,
so both H22+ and He2 does not exist.
50. (b) Li2 = s1s2 s*1s2 s2s2 { 2 px 2 2 p y 2 , 2 pz 2
1
\ Bond order = (4 - 2) = 1 10 4
2 Bond order = =3
2
Li2+ = s1s2 s*1s2 s2s1
1 N2 : 1s 2 , *1s 2 , 2s2 , *2s 2 ,{ 2 p2x 2 p2y , 2 pz2
B.O. = (3 - 2) = 0.5
2 10 4
Li2– = s1s2 s*1s2s2s2s*2s1 Bond order = =3
2
1
B.O. = (4 - 3) = 0.5 NO – : 1s 2 , *1s 2 , 2 s 2 , * 2 s 2 , 2 p z2 ,
2
The bond order of Li2+ and Li2– is same but Li2+ is more { 2 p x2 2 p 2y , { * 2 p1x * 2 p1y
stable than Li2– because Li2+ is smaller in size and has 2
electrons in antibonding orbitals whereas Li2– has 3 10 6
Bond order = =2
electrons in antibonding orbitals. Hence Li2+ is more 2
stable than Li2–. \ NO– has different bond order from that in CO.
51. (d) (i) In Na2O2, we have O 22 ion. Number of valence 53. (a) Molecular orbital configuration of B2(10) as per the
electrons of the two oxygen in O 22 ion = 8 × 2 + 2 condition given in question will be
=18 which are present as follows s1s2, s* 1s2, s2s2, s* 2s2, 2 px2
6 4
2
s1s2, s*1s2, s2s2, s*2s2, 2 p z2 , { 2 px2 = 2 p y , Bond order of B2 = =1.
2
{ * 2 p x2 = * 2 p 2y B2 will be diamagnetic.
6+2
\ Number of unpaired electrons = 0, hence, O 22 is 54. (d) OSF2 : H = =4.
2
diamagnetic.
It has 1 lone pair.
(ii) No. of valence electrons of all atoms in
O3 = 6 × 3 = 18.
:

Thus, it also, does not have any unpaired electron,


hence it is diamagnetic. S (Shape is trigonal pyramidal)
O F
F
108 Chemistry

The shapes of SO3, BrF3 and SiO32- are triangular planar 4s 4p 4d


3d
respectively. [Ni(H2O)6]2+
2
55. (b) Ni 4Cl [NiCl4 ]2 3 2
sp d hybridisation
sp 3

[NiCl4]2–. = 3d8 configuration with nickel in + 2 H2O 2+


oxidation state, Cl– being weak field ligand does not H2O OH2
compel for pairing of electrons.
So, Ni

[NiCl4 ]2 - H2O OH2


H2O
4s 4p
3d
sp 2 sp sp 2
56. (b) Allene (C3H4) is H 2 C = C = CH 2
3
sp hybridisation
EXERCISE 4
1. (b) Hydrogen is non metal and non metal atoms form covalent
Hence, complex has tetrahedral geometry bond
2. (b) KI is insoluble in benzene being ionic compound (Like
Cl 2–
dissolves like)

Ni 3. (d) The Bond order of C- O in CO, CO2 and CO 2 – is 3, 2 and


Cl Cl 1.33.
Cl Hence bond length follows the order CO < CO2 < CO 2 –
4. (b)
2
Ni 4CN [Ni(CN) 4 ]2 5. (c) Hybridisation in NF3 and H3O+ is sp3 and they have
[Ni(CN)4]2– = 3d 8 configuration with nickel in + 2 pyramidal shape. Hybridisation in NO – and BF3 is sp2
oxidation state, CN– being strong field ligand compels
and they have triangular planar shape.
for pairing of electrons.
6. (d) In XeO4 hybridisation is ½[8 + 0 + 0 –] = 4 sp3
So, tetrahedral; XeO3F = ½[8 + 1 + 0 –0] = Q + R = 5
4s 4p sp3d trigonal bipyramidal; XeO2F2 = ½ [8 + 2 + 0–0] = 5
3d sp3d trigonal bipyramidal one equatorial position
[NiCN4]–2 occupied by lone pair of electrons; XeF4 = ½ [8 + 4 + 0
– 0]=6 sp3d2 octatradral two axial position occupied by
2 lone pair of electrons.
dsp hybridisation
F F
Hence, complex has square planar geometry. Xe
2– F F
NC CN 7. (b) Hybridisation in NH3 = sp , [PtCl4]2– dsp2 (inner complex);
3

PCl5 = sp3d and BCl3 is sp2.


Ni 8. (a) Bond order in CN – , CO and NO+ is three and they are
isoelectronic.
NC CN
O
P
Ni +2 + 6H 2 O ¾¾
®[Ni(H 2 O) 6 ]2 + O O
O
[Ni(H2O)6]2+ = 3d 8 configuration with nickel in + 2 9. (d) Structure of P4O10 is O P P O Num ber
oxidation state. As with 3d 8 configuration two O
O P
d-orbitals are not available for d 2sp3 hybridisation. O
So, hybridisation of Ni (II) is sp3d 2 and Ni (II) with six O
co-ordination will have octahedral geometry. of bonds in P4O10 is 16
Chemical B onding and Molecular Structure 109
10. (d) In NF3 D.M. due to lone pair and F act in opposite direction
20. (a) NO 3- and CO 32 - both have same number of electrons
and cancel each other while in NH3 it acts in one direction
only Hence D.M. of NH3 is more than NF3. (equal to 32) and central atom in each being sp2 hybridised,
are isostructural too.
..
11. (b)
N N 21. (d) The hybrid state of N in NO 3- and NO -2 is the same and
F F F H H H
it is sp 2 while in NO +2 it is sp
N –– N– O octet of each atom is complete. 22. (d) Since the N does not contain d orbitals, the formation of
12. (d) All gaseous and do not conform to octet rule NF5 is not possible
13. (a) The molecular plane does not contain electron density..
d-
Hence nodal plane in the bond is in the molecular F d + d-
plane. 23. (d) The shape of BF3 is trigonal planar B – F and
d- F
14. (c)
m = 0 hence it is non polar. The shape of NF3 is pyramidal

O O +

:
15. (c) H – C or Resonance Nd
H–C
d- F F d - and m ¹ 0 hence it is polar..
O F d-
O
– 24. (c) In metallic crystals the Kernels constitute the lattice and
electrons are mobile and form sea of electrons.
25. (d) Bond angles are affected by repulsions which are
O
lp – lp > lp – bp > bp – bp.
hybrid H – C due to resonance C – O bond length 26. (d) The hydrated Aluminium chloride trivalent complex ion
O
[Al.(H 2 O)6 ]3+ , formation of which is exothermic process.
is the same.
The energy released is sufficient to cause the ionisation
F :O: of Al.
| ||
16. (c) XeF2 and CO2 have linear structure Xe ; C 27. (b) Since XY2 forms 2 s , 2 p bonds and has 1 lone pair of
| ||
F :O:
electrons. It must have the structure Y = X = Y. Hence Y
1 is divalent. The hybridisation of X is
17. (b) Solubility µ . Lattice energy opposes
lattice energy æ1 ö
sp 2 ç (6 + 0 + 0 - 0)= 3 ÷ . So XY2 is trigonal planar
solubility and Solubility µ heat of hydration . Heat of è 2 ø
hydration favours solubility. For second groups ions heat .
..
of hydration is Be++ > Mg++ > Ca++ > Sr++ > Ba++ hence || X || (like SO 2 ).
solubility decreases down the group. Y Y
Cl 28. (b) For T-shape geometry the molecule must have 3 bonded
18. (b) BiCl3: Cl Bi ; sp2 - Hybridisation (Trigonal pair and 2 lone pair of electrons.
Cl 29. (a) See text
geometry); Bond angle = 120º 30. (c) Ortho-nitrophenol has intramolecular H-bonding
.
. . OH
sp3
P . O and para-nitr ophenol has
In PCl3 (Pyramidal
||

Cl Cl Cl geometry)
. N
Bond angle = O
sp3 below 109o 28’ intermolecular H-bonding.
As and decreases from
In AsCl3– (Pyramidal PCl3 to BiCl3 NO 2 NO 2 NO 2
Cl Cl Cl geometry)
sp3
In BiCl3– Bi (Pyramidal ,
Cl Cl Cl geometry) | | |
O– H O– H O– H
In these, order of bond angle : BCl3 > PCl3 > AsCl3 > BiCl3 Hence former is more volatile than latter.
19. (a) H 3 N ® BF3 where both N, B are attaining tetrahedral
31. (b) SF4 (sp 3 d) has see saw shape, XeF4 (sp 3d 2 ) square
geomerty.
planar and SiF4 (sp 3 ) is tetrahedral.
110 Chemistry

32. (d) LiCl is covalent in nature due to small size of Li + ion, -10 -8
42. (b) m cal = e × l = (4.802 ´10 esu ) (1.275 ´10 cm)
hence LiCl will ionise less than NaCl.
33. (c) Due H -bonding in H – F its boiling point is more than m obs 1.03
Percentage ionic character = ´100 = =100=17%
HCl. m cal 6.12
34. (d) Although all the factors are correct but out of these lattice
energy is the most important one. (See Born-Haber cycle). Product of charges
43. (c) Lattice energy =
35. (a) Silicon carbide is covalent solid. interionic distance
36. (c) A p bond is formed by orbitals having same symmetry
In NaCl the product of charges = 1 × 1; In CaO product of
about the internuclear axis.
charges = 2 × 2 = 4 while the inter ionic distance is almost
37. (d) The more the polar nature of molecule the more is the
same in both. Thus lattice energy of CaO is almost four
boiling point.
times the lattice energy of NaCl.
38. (d) Nitrogen can form NCl3, N 2 O5 and Mg 3 N 2 but not
44. (a) MgO has highest lattice energy, since 2 × 2 = 4 is maximum
NCl5 due to non availability of d atomic orbitals. in MgO.
39. (c) The smaller the size, the more the charge, the more is the 45. (a) m = e × d
lattice energy.
40. (b) m 1.2 D 1.2 ´ 10-18 esu cm
\ e= = =
d 1. 0 A 1.0 ´ 10-8 cm
Cl 120
1
6 Cl = 1.2 × 10 -10 esu
41. (c) 2 Dipole moments of 2Cl and 5Cl
3 Percentage of electronic charge
Cl 5 4 Cl
1.2 ´ 10 -10 esu
are vectorically cancelled. = ´ 100 = 25 %
4.8 ´ 10 -10 esu
2 2
It is due 1 Cl and 3 Cl m 2 = m1 + m 2 + 2m1 m 21 cos q 46. (a) All the statements are correct (see text).

= (1.5) 2 + (1.5) 2 + 2 ´1.5 ´1.5 cos 120 47. (b) In amine the nitrogen is sp 3 hybridised and in amide the
\ m = 1.5 D
nitrogen is sp 2 hybridised.
48. (c) Since water is polar in nature and like dissolves like, the
coating must be nonpolar to polar manner.
5
States of Matter
STATES OF MATTER : MEASURABLE PROPERTIES OF GASES :
The three important states of the matter are (i) Solid state (ii) Four measurable properties are.
(i) Mass : It is expressed in grams or kg. 1 kg=103 g moles of
Liquid state (iii) Gaseous state, which can exist together at a
particular temperature and pressure e.g. water has three states Mass in grams m
gas = =
in equilibrium at 4.58 mm and 0.0098ºC. Molar mass M
PLASMA STATE : (ii) Volume : It is equal to the volume of the container and is
expressed in terms of litres (L), millilitres (ml), cubic
It is the gaseous mixture of electrons and positive ions existing at
extremely high temperatures (in the interior of stars) or internal centimeters ( cm 3 ) , cubic meters (m3) or cubic decimeters
electrical fields in discharge tubes. (dm3).
SINGLE SUPER ATOM STATE : 1 l = 1000 ml = 1000 cm 3 = 1 dm 3
At extermely low temperature, all atoms lose their identity and get
1 m 3 = 10 3 dm 3 = 10 6 cm 3 = 10 6 ml = 10 3 l
condensed into a single entity behaving like a single super atom.
(iii) Pressure : It is equal to force per unit area and expressed in
SOME CHARACTERISTICS OF THE THREE COMMON the units such as atmosphere, millimetres (mm), centimetres
FORMS OF MATTER ARE : (cm), torr, bar etc. SI unit of pressure is pascal (Pa) or
Property Gaseous state Liquid State Solid state
kilopascal (kPa)
1 General It has definite mass It has definite It has definite
but no definite shape mass and volume mass, volume 1atm = 76cm of Hg = 760 mmof Hg = 760 torr
and volume but no definite shape and shape.
2. Forces Almost negligible Weaker than those Strongest
1atm = 101.325 kPa = 101325Pa = 101.325 Nm -2
in solids
= 1.01325 bar = 14.7lb m -2 (psi)
3. Density Low Lower than solids High
4. Motion Molecules have large Low values of No translatory 1bar = 105Pa. Pressure is measured with manometer
rotatory, vibratory motions or rotatory
and translatory motion. Possess (iv) Temperature : It is measured in celcius scale (°C)or in Kelvin
motions vibratory motion scale (K). SI unit of temperature is Kelvin (K) or absolute
5. Packing No proper packing Less closely Molecules
packed closely packed degree T (K) = t °C + 273
6. Energy Least Higher than solids Molecules possess GAS LAWS :
maximum energy
7. Thermal High Higher than solid Least
Boyle’s Law - The volume of a given mass of a gas is inversely
Expansion proportional to its pressure at constant temperature.
8. Compression High Slightly higher than Least compressibility
1
solid V or VP = k , a constant
9. Intermixing Spontaneous Spontaneous but slow Least-intermixing P
10. Pressure Exert pressure on Negligible Negligible Value of k depends on mass, temperature and nature of gas.
the walls of container
When mass and temperature are the same we have P1V1 = P2 V2
112 Chemistry
ISOCHORES :
ISOTHERMS :
A graph of P vs T at constant volume is known as Isochore
Graphs of V vs P or PV vs P at constant temperature are known as
Isotherms. GRAPHICAL REPRESENTATION OF GAY LUSSAC’S
GRAPHICAL REPERSENTATION OF BOYLES LAW : LAW :

T2 V1 V2 > V1
P P PV T1 P V2

V 1/V P T

CHARLE’S LAW : AVOGADRO’S LAW :


The volume of the given mass of a gas increases or decreases by The volume of a gas is directly proportional to number of molecules
or moles at constant temperature and pressure.
1
of its volume at 0°C for each degree rise or fall of V n at constant T and P
273.15
temperature at constant pressure. V V1 V2
or = K or = at constant T and P
n n1 n 2
æ t ö
V t = V 0 ç1+ at constant.P and n
è 273.15 ÷ø GRAPHICAL REPRESENTATION OF AVOGADRO’S
or
LAW :
The volume of a given mass of a gas is directly proportional to the
absolute temperature at constant pressure.
V at constant P & T
V V V
V T at constant P and n or = const. or 1 = 2
T T1 T2
n
ABSOLUTE ZERO : IDEAL GAS EQUATION :
If in the above equation we put the value of t as –273.15 ºC the
volume of the gas will be zero. It means gas will not exist, which is 1
V , T and n constant (Boyle 's law )
not possible. In fact all gases get liquified before this temperature P
is reached. This hypothetical or imaginary temperature at which V T , P and n constant (Charle 's law )
the gases occupy zero volume is called absolute zero.
V n , P and T constant ( Avogadro's law )
ISOBAR :
A graph of V vs T at constant pressure is known as Isobar. nT
GRAPHICAL REPRESENTATION OF CHARLE’S LAW: V or PV nT or PV = nRT.
P
This is known as ideal gas equation. R is known as universal gas
P1 P2 > P1 constant.
V P2
IDEAL GAS :
The gas which obeys the equation PV = nRT at every
T Isobars temperature and pressure range strictly.
GAY LUSSAC’S LAW : (AMANTON’S LAW) : REAL OR NON-IDEAL GASES :
The pressure of a given mass, of gas increases or decreases by
Since none of the gases present in universe strictly obey the
1 equation PV = nRT , hence they are known as real or non ideal
of its pressure at 0°C for each degree rise or fall of
273.15 gases. Real gases behave ideally at low P and high T.
temperature at constant volume. DENSITY OF A GAS :
æ t ö w
P t = P 0 ç1 + ÷ at constant V and n We have PV = nRT , PV = RT ;
è 273.15 ø M
or
The pressure of a given mass of a gas at constant volume is æ weight (w) ö
ç Moles(n) = ÷
directly proportional to absolute temperature è Molecular weight (M) ø
P
P T or P = kT or = k at constnat V and n
T Further, d = w = PM
V RT
P1 P2 The above equation shows that density of a gas depends on P
or T1 = T2 and T.
States of Matter 113
UNIT OF DENSITY OF GAS : NUMERICAL VALUE OF R :
It is usually expressed in gm/ litre As the work can be expressed in different systems of units, R will
VAPOUR DENSITY (V.D.) : have different values.
It is the ratio under similar conditions of P and T (i) In litre - atmosphere :
At NTP, P = 1atm, V=22.4 litre and T= 273.15K
Density of gas
V.D. = 1 ´ 22.4
Density of H 2 which is 0.00009 R= = 0.0821litre atm K -1 mol-1
273.15
Mass of 1litre of gas Mass of V litre of gas
= = (ii) In cgs system :
Mass of 1litre of H 2 Mass of V litre of H 2
P = 1 atm = 1×76×13.6×980 dyne cm-2; V = 22400 cm3;
Mass of N molecules of gas T = 273.15 K
=
Mass of N molecules of H 2
1´ 76 ´13.6 ´ 980 ´ 22400
Mass of 1 molecule of gas R=
= 273.15
Mass of 1 molecule of H 2
= 8.314 ´ 107 ergs K -1mol-1
Mass of 1 molecule of gas
=
2 ´ mass of 1 atom of H =1.987 or 2.0 Cal K -1mol -1
1
= molecular mass of gas Since 1 Cal = 4.184 ´ 10 7 ergs
2
(iii) In mks or SI units :
UNIT OF VAPOUR DENSITY :
Unit of R is JouleK–1 mol -1
It is a ratio and has no unit.
NATURE OF GAS CONSTANT, (R) : 107 ergs = 1 Joule or R = 8.314 Joule K–1 mol -1
NTP OR STP AND SATP
PV (Force / Area) ´ Volume
R= = Normal or standard temperature & pressure means 0°C or 273.15
nT Moles ´ Degree / K
K and 1 atm pressure.
[Force /(Length) 2 ] Normal boiling : At 1 atm pressure the boiling temperature is
= ´ (length)3
Moles ´ Degree / K called Normal boiling point.
Standard boiling point : At 1 bar pressure the boiling temperature
Force ×length work is called standard boiling point.
= =
Moles × degree Moles×degree
Normal boiling point of water is 100 °C (373.13 K)
= Work done per degree per mole Standard boiling point of water is 99.6 °C (372.6K)
Deviation from ideal behaviour in terms of compressibility factor Z Standard temperature and pressure (STP)
N2 273.15 K(0 °C) temperature
1.8
H2 1 bar (105 pascal) pressure
1.6 O2 CH
4
1.4 CO2 Molar volume at STP = 22.71098 L mol–1
Z = PV/nRT

1.2 Normal temperature and pressure


ideal gas
1.0 273.15 K(0 °C) temperature
0.8 1 atom (1.01325 pascal) pressure
0.6 Molar volume at NTP = 22.413996 L mol–1
0.4
Standard ambient temperature and pressure (SATP)
200 400 600 800 1000 298.15 K (25 °C) temperature
P/bar 1 bar (105 pascal) pressure
Deviation from ideal behaviour in terms of PV-P curve Molar volume at SATP = 22.784 L mol–1
CO BOLTZMANN CONSTANT (k) :
CH4
R
H2 It is the gas constant per molecule thus k = ; Where R=gas
N0
He
ideal gas constant., N0= Avogadro number.
PV
-1 -1 23 -1
Value of k = 8.314 JK mol /6.02 ´ 10 mol
P
= 1.38 ´10-23 J K -1
114 Chemistry
DALTON’S LAW OF PARTIAL PRESSURE :
At constant temperature the total pressure, exerted by a mixture r1 V /t d2 M2
= 1 1 = =
of non reacting gases, is the sum of partial pressure of each gas r 2 V2 / t 2 d1 M1
P = p1 + p 2 + p 3 + .....
Partial pressure p of a gas = mole fraction of the gas × total r1 T2 d2 M2
If volume diffused is same = = =
pressure. r2 T1 d1 M1
For a gas A, pA = xA × P
If n 1 , n 2 and n 3 are moles of non reacting gases filled in a r1 V1 d2 M2
vessel of volume V at temperature T. If time of diffusion is same = = =
r 2 V2 d1 M1
The total pressure P is given by PV = (n 1 + n 2 + n 3 )RT
This is the equation of state of a gaseous mixture. Effect of pressure on the rate of diffusion.
RELATIONSHIP BETWEEN TOTAL PRESSURE AND
r1 p1 M2
INDIVIDUAL PRESSURE : Rate of diffusion pressure, therefore, r = p × M
On mixing of gases at constant temperature. If a gas A at pressure 2 2 1
P1 and volume V1 is mixed with gas B at pressure p2 and volume
APPLICATIONS OF GRAHAM’S LAW OF
V2 at same temperature, then
DIFFUSION:
p1V1 +p 2V 2
Total pressure, P = (i) Determination of densities and molecular masses of unknown
V1 +V 2 gases.
AQUEOUS TENSION : (ii) Separating the gases having different densities.
It is the pressure exerted by water vapour at a particular (iii) Separation of Isotopes
temperature. It depends upon temperature.
PRESSURE OF A DRY GAS : ATMOLYSIS :
When a gas is collected over water, its observed pressure is The phenomenon of separation of a mixture of gases due to
equal to the sum of the pressure of dry gas and the pressure of difference in their rates of diffusion is called atmolysis.
water vapour (aqueous tension), then KINETIC THEORY OF GASES :
Pressure of dry gas = pressure of moist gas
Postulates of kinetic theory of gases.
– aqueous tension.
DIFFUSION : (i) A gas consists of large number of tiny particles called
The tendency of every gaseous substance to distribute itself molecules.
uniformly throughout the available space is known as diffusion. (ii) Volume occupied by gas molecules, is negligible as compared
It also takes place through a porous vessel. to the total volume of gas.
EFFUSION : (iii) There is continuous rapid random motion of gas molecules.
The movement of a gas through a small hole when it is subjected The molecules collide with each other and against walls of
to pressure is known as effusion. container.
GRAHAM’S LAW OF DIFFUSION (OR EFFUSION) : (iv) The molecules are perfect elastic bodies and there is no loss
At constant temperature and pressure, the rate of diffusion or of kinetic energy during collisions.
effusion of a gas is inversly proportional to the square root of its
(v) There are no attractive forces between the molecules of gas.
density.
(vi) The pressure exerted by a gas is due to bombardment of gas
1 molecules against the walls of the container.
Rate of diffusion
d
(vii) The different molecules possess different velocities and hence
If r1 and r2 are rates of diffusion of two gases and d1 and d 2 their different energies. The average K.E. is directly proportional
respective densities then, to absolute temperature.
KINETIC GAS EQUATION :
r1 d2 V.D 2 M2
= = = Based upon the postulates of Kinetic theory of gases, the kinetic
r2 d1 V.D1 M1
gas equation is

Volume diffused or effused 1


Rate of diffusion = PV = mNU 2
time taken 3
Moles diffused or effused where m = mass of a gas molecule, N= number of molecules,
=
time taken U= Root mean square velocity.
States of Matter 115
KINETIC ENERGY OF GAS : (iv) Calulation of velocity when pressure and temperature are
It can be obtained from kinetic gas equation given but are not the same as NTP

KE of one molecule = 1 mv 2 U=
3PV ; In such case molecular volume (22400ml) is
2 M
converted into the volume under given conditions of T and
1 2 1 2
PV = mNU 2 = . mNU 2 = KE = RT
3 3 2 3 P using relation, P1V1 = P 2 V 2
T1 T2
( N = n and m ´ n = M)
MAXWELL’S DISTRIBUTION OF VELOCITIES :
3 The molecules present in a given sample of gas move with different
\ KE = RT for 1 mole of a gas
2 velocites in all possible directions. Velocities and directions of
(i) KE of n moles of gas = 3/2 nRT molecules keep on changing due to intermolecular collisions.
(ii) At absolute zero, KE is zero Hence it is impossible to find out the individual velocity of each
molecule. It is however possible to predict fraction ( DN / N ) of
MAXWELL’S GENERALISATION :
the total number of molecules having specific velocities at a
Kinetic Energy of translation of ideal gas is directly proportional
particular temperature. As shown by the curve,
to absolute temperature of gas or its pressure and is independent
of the nature of gas. The gases show ideal behavior at low presence/large volume.
Since the volume of molecules can be neglected and at high
THERMAL MOTION : temperature since intermolecular forces decrease.
KE U 2 or U 2 T U T
Most probable velocity
The molecular velocity of a gas is proportional to square root of
Fraction (percentage) of

the absolute temperature. The molecular motion is called thermal Average velocity
motion of molecules. RMS velocity
molecules

CALCULATION OF MOLECULAR VELOCITIES OF T1


T2
GASES :
(i) Calculation of velocity when temperature alone is given :
From Kinetic gas equation, T3
1 3PV 3RT
PV = MU 2 , U 2 = = Velocity
3 M M

3RT ; Fraction of molecules having very small or very high velocities is


\ U= R = 8.314 ´ 10 7 ergs K -1 mol -1
M very low. No molecule has zero velocity. Fraction of molecules
possessing a particular velocity at a particular temperature is
constant. The different velocities possessed by gas molecules
3 ´ 8.314 ´107 ´ T T
U= = 1.58 ´104 cm / sec are:
M M
(i) Most probable velocity ( a ) : It is the velocity possessed by
(ii) Calculation of velocity at NTP maximum fraction of gas molecules at a particular temperature
3PV 2RT
It is given by the formula, U= Mathematically, a =
M M

3 ´ 76 ´ 13.6 ´ 981´ 22400 (ii) Average velocity ( v ) : This is the average of the different
U=
M velocities of all the molecules.

AT NTP, P= 76 ´13.6 ´ 981 dynes cm -2, V = 22400 ml n 1 c1 + n 2 c 2 + n 3 c 3 + .....


v=
(molar volume). n 1 + n 2 + n 3 + ......
(iii) Calculation of velocity when pressure and density are given. where c1,c2,c3 etc. are individual velocities of n1,n2,n 3
It is given by the following formula, molecules.
3PV 3P M 8RT
U= = ( sin ce, D = ) Mathematically, v =
M D V pM
116 Chemistry
(iii) Root mean square velocity (U) : It is the square root of the (ii) Pressure : The deviation is more at high pressure. CO2 and
mean of the square of the different velocites of the molecules N2 show negative deviation at low pressure and positive
2 2 2
deviation at high pressure.
n 1 c1 + n 2 c 2 + n 3 c 3 + ... (iii) Temperature : The deviation is more at low temperature. H2
U= ;
n 1 + n 2 + n 3 + ....... and He always show positive deviations at 0°C
BOYLE’S TEMPERATURE:
3RT
Mathematically, U = Temperature at which a real gas exhibits ideal behaviour for
M
considerable range of pressure is called Boyle’s temperature.
RELATIONSHIP BETWEEN DIFFERENT TYPES OF Mathematically,
VELOCITIES :
a
Tb = , where a and b are Vander Waal’s constants
2 RT 8RT 3RT 8 bR
a:v:U º : : = 2: : 3
M pM M (i) Boyle temperature is different for different gases
= 1.414 : 1.595 : 1.732 = 1 : 1.128 : 1.224 (ii) Below Boyle’s temperature a gas shows negative deviation.
Calculation of root mean square velocity (Urms) : As already (iii) Above Boyle’s temperature a gas shows positive deviation
discussed, the Urms can be calculated by any of the following
CAUS E OF D EVIATION FRO M THE ID EAL
relations:
BEHAVIOUR :
3RT 3PV 3P It is due to two faulty assumptions of Kinetic theory of gases
Urms = = =
M M D particularly not valid at high pressure and low temperature.
RATIO OF URMS FOR TWO DIFFERENT GASES AT (i) Volume occupied by the gas molecules is negligible as
THE SAME TEMPERATURE compared to the total volume of gas.
(ii) There are no attractive forces between the gas molecules.
U1 M2
= VAN DER WAAL'S EQUATION :
U2 M1
After volume & pressure correction, van derWaals obtained the
RATIO OF URMS FOR THE SAME GAS AT DIFFERENT following equation for n moles of a gas
TEMPERATURE
æ 2 ö
ç P + n a ÷(V - nb) = nRT æ
çç P +
a ö
÷÷ (V - b) = RT
ç , For one mole
U1 T1 è V 2 ÷ø è V2 ø
=
U2 T2
Excluded volume or co-volume, (b) :
EFFECT OF TEMPERATURE ON DISTRIBUTION OF The constant b in van der Waal's equation is known as excluded
MOLECULAR VELOCITIES: volume or co-volume. It is nearly four times the actual volume
Most proabable velocity increases with the rise in temperature occupied by the gas molecules.
but fraction of molecules possessing such velocity decreases. Excluded or co-volume (b) = 4 × actual volume of gas molecules.
Deviation from ideal behaviour of gas :
MAGNITUDE OF ATTRACTIVE FORCES BETWEEN
At high pressure and low temperature the gases deviate
GAS MOLECULES (a) :
considerably from the ideal behaviour. Deviation can be expressed
in terms of compressibility factor (Z), expressed as It is given by van der Waals constant ‘a’. Different gases have
different value for ‘a’.
PV
Z= (i) The greater the value of ‘a’, the greater the strength of van
nRT der Waals forces.
In case of ideal gas, PV = nRT, Z = 1 (ii) The greater the value of ‘a’, the greater is the ease with
In case of real gas, PV ¹ nRT, Z ¹ 1 which a gas can be liquefied.

NEGATIVE DEIVATION: UNITS FOR VAN DER WAAL'S CONSTANT:


In such case, Z < 1, gas is more compressible. Pressure correction,
POSITIVE DEVIATION : n 2a pV 2
In such case, Z >1, gas is less compressible. p= 2
\ a= 2
= atm litre 2 mole -2
V n
FACTORS AFFECTING THE DEVIATION :
The factors affecting the deviation are: V
Volume correction, V = nb \ b= = litre mole -1
(i) Nature of the gas : In general, the most easily liquefiable n
and highly soluble gases show larger deviation.
States of Matter 117
vAN dER WAAL’S C ONSTANTS FOR SOME EXCEPTIONAL BEHAVIOUR OF HYDROGEN AND
COMMON GASES : HELIUM :
Due to their small size, the attractive forces between the molecules
Gas a (lit 2atmmol-2 ) b.(lit mol-1)
a
NH3 4.17 0.0371 are too small, is negligible, then
V2
CO2 3.59 0.0427
PV Pb
CO 1.49 0.0399 =1+ ;Z > 1
RT RT
Cl2 6.49 0.0562 Hence they always show positive deviation.
H2 0.024 0.0266 LIMITATION OF VAN DER WAAL’S EQUATION :
There is specific range of temperature and pressure, to apply the
HCl 3.67 0.0408
equation. It deviates at too high pressure and too low temperature.
NO 1.34 0.0279 OTHER EQUATIONS OF STATES :
O2 1.36 0.0318 -a
SO2 6.71 0.0564 (i) Dieterici equation : P(V - b) = RT e RTV
He 0.034 0.0237 a
(ii) Berthelot’s equation : (P + )(V - b) = RT
Water 5.46 0.0305 TV 2
DISCUSSION OF VAN DER WAAL’S EQUATION : a
(iii) Clausius equation : (P + )(V - b) = RT
van der Waal’s equation for one mole of gas T ( V + c) 2
a Where c is a new constant
(P + 2
)(V - b) = RT (iv) Kammerling onne's equation (virial equation):
V
(i) At low pressure and ordinary temperature : V is very large PV = A + BP + CP 2 + DP 3
and b can be neglected then (a) The coefficients A,B,C,D, etc are known as first, second,
third, fourth virial coefficients
a (b) At low pressure only A (which is equal to RT) is
(P + )V = RT
V2 important, the others cancel out.
(c) A is always positive and increases with rise of
a PV a temperature.
or PV = RT - or = (1 - );
V RT RTV (d) At Boyle temperature B=0
Therefore Z is less than 1. (e) Relation between B and van der Waals constant is
Hence at low pressure the gases show negative deviation. a
B= b-
(ii) At high pressure and ordinary temperature : At high RT
a CRITICAL PHENOMENON AND LIQUEFACTION OF
pressure, volume V will be quite small. The quantity 2 GASES :
V
become negligible but ‘b’ cannot be ignored. Increase of pressure and decrease of temperature tend to cause
liquefaction of gases. The effect of temperature is, however more
P(V - b) = RT important.
PV Pb CRITICAL TEMPERATURE (Tc) :
or = 1+ ; Therefore Z is more than 1.
RT RT It may be defined as the temperature above which no gas can be
At high pressure the gases show positive deviation liquefied howsoever high the pressure may be, critical temperature
of CO2 is 31.1°C.
Note : At some intermediate range of pressure and ordinary
temperature the gas shows the ideal behaviour. Critical temperature (Tc) of some gases,

a He 5.2 CO2 304.1 CH4 190.2


(iii) At low pressure and high temperature: The effect of H2 33.2 N2O 309.6 HCl 324.5
V2
and b is negligible. We have N2 126.0 NH3 405.5 H2S 373.5

PV CO 134.4 Cl2 417.1 Ar 150.7


PV = RT or =1 ;
RT O2 154.3 SO2 430.3
Therefore Z = 1. Hence gas shows ideal behaviour.
118 Chemistry
CRITICAL PRESSURE (Pc) : JOULE-THOMSON COEFFICIENT ( m J.T. ) :
At critical temperature the pressure needed to liquefy a gas is The temperature change produced per atmosphere drop in
known as critical pressure. pressure under constant enthalpy conditions on passing a gas
CRITICAL VOLUME (Vc) : through the porous plug is called Joule-Thomson coefficient
The volume occupied by one mole of a gas at critical temperature
The quantity æç

and critical pressure is known as critical volume. ÷ is called Joule -Thomson coefficient.
è P øH
RELATIONSHIP BETWEEN CRITICAL CONSTANTS
AND VAN DER WAAL’S CONSTANT: æ Tö 1 æ 2a ö
ç ÷ = ç - b÷
è P øH C p è RT ø
8a a
V c = 3b, T c = , Pc=
27Rb 27b 2 æ Tö
(i) Joule Thomson Coefficient ç ÷ is positive for real gases,
Relationship between boiling point and critical temperature: è P øH
2 (cooling)
Boiling point = critical temperature.
3 2a
Or Joule Thomson Coefficient will be positive when >b
Compressibility for 1 mole of gas at critical state: RT

Pc Vc 3 æ Tö
z= = = 0.375 , almost constant. (ii) When Joule Thomson Coefficient ç ÷ is negative,
RTc 8 è P øH
(heating)
JOULE THOMSON EFFECT :
2a
When a gas under high pressure is allowed to expand adiabatically Or Joule Thomson Coefficient will be negative when <b
into a region of extremly low pressure, it suffers change of temperature. RT
The phenomenon is kown as Joule -Thomson effect. æ Tö
(iii) When Joule Thomson Coefficient ç ÷ is zero, (no heating
CONDITION FOR JOULE-THOMSON EFFECT è P øH
(INVERSION TEMPERATURE): or cooling)
In Joule -Thomson effect, cooling is observed only if the gas is
2a
present below certain temperature known as inversion temperature, Or Joule Thomson Coefficient will be zero when =b
Ti. It is characteristic of each gas and related to van der Waal’s RT
Since a, b and R are constants, the sign of Joule-Thomson
2a Coefficient will depend only upon the temperature at which
constant a and b as, Ti =
Rb the gas is being allowed to expand. The temperature at which
At exact inversion temperature there is no Joule-Thomson effect. the Joule-Thomson Cofficient changes sign is known as the
Above inversion temperature, there is heating during Joule- inversion temperature.
Thomson effect. Below inversion temperature there is cooling 2a 2a
during Joule-Thomson effect. = b \ Ti =
RTi Rb
Inversion temperature for hydrogen = –80°C and for He = –240°C.
Joule-Thomson effect is zero in an ideal gas and enthalpy remains LAW OF CORRESPONDING STATES :
constant. When an ideal gas expands in vacuum, it does no work When the values of pressure, volume and temperature are
i.e. W=0 ; DE = 0 (Adiabatic condition). Hence internal energy expressed as fractions of the corresponding critical values we
of a given quantity of an ideal gas at constant temperature is P V T
have. = p, = f, =q
independent of its volume. Pc Vc Tc

æ Eö where p , f and q are termed, the reduced pressure, the reduced


ç ÷ = 0 (for an ideal gas) ; This quantity is called internal
è V øT volume and the reduced temperature respectively. If we replace
P,V and T by p Pc, f Vc and q Tc in van der Waal’s equation and
pressure and is positive for real gases.
Thermodynamically an ideal gas may be defined by following the put the values of Pc, Vc and Tc terms of a, b and R we get,
equations. 3
(p + )(3f - 1) = 8q
(i) PV = constant, at constant temperature. f2

æ Eö If the two substances have the same reduced temperature and the
(ii) ç ÷ =0 same reduced pressure, they will have the same reduced volume.
è V øT The statement is known as the law of corresponding states.
States of Matter 119
HEAT CAPACITY OF SYSTEM : (ii) Collision frequency (Z) : The number of collisions
For gases it can be at constant volume or at constant pressure. experienced by molecules per cc of a gas per second is known
(i) Heat capacity at constant volume (CV) is defined as the 1 2
increase in internal energy of a gas per degree rise of as collision frequency of gas, Z = vN 2
2
temperature.
v = average velocity
æ Eö 3 N = number of molecules per cm3
CV = ç ÷ = R
è T øV 2 s = collision diameter
(ii) Heat capacity at constant pressure (Cp) is defined as increase 2
in enthalpy of a gas per degree rise of temperature. At constant pressure, Z T3

æ Hö 5 At constant temperature, Z p2
Cp = ç ÷ = R
è T øP 2 (iii) Collision number (Z1) : Number of collisions undergone by
For one mole of gas, the heat capacities at constant volume a molecule with other molecules per second present in 1cm3.
and constant pressure are denoted by Cv and Cp and are Z1 = Collision number = 2 ps 2 vN
termed as molar heat capacities.
s = diameter of molecules, v = Average velocity,,
RELATION BETWEEN Cp AND Cv : N = Number of molecules per unit volume of the gas
Cp– Cv = R (iv) Mean free path (l) : The average distance travelled by the
R = 1.987 cal or 8.314 Joule molecule between two successive collisions
R= PDV = Work done by one mole of an ideal gas in expansion at 1
constant pressure when heated through 1°C. l=
2
2 N
MOLAR HEAT CAPACITIES FOR POLYATOMIC
GASES: At constant pressure, l T

3 5 1
The values C V = At constant temperature, l
R and C p = R are for monoatomic gases P
2 2
VOLUME COEFFICIENT :
like He, Ar etc. where the energy supplied increases translational
It is defined as the ratio of the increase in volume of the gas at
kinetic energy only. In polyatomic gases heat supplied is utilised
constant pressure per degree rise of temperature to its volume at
to increase vibrational and rotational energy also. Thus we have
0°C
3 5 v t -v 0
CV = R+x and C p = R+x
2 2 aV =
v0 ´ t
5 V0 =Volume of a given mass of a gas at 0°C
Cp R
5
For monoatomic gas, x = 0, g = = 2 = = 1.66 Vt= Volume of a given mass of a gas at t°C
CV 3 3
R V t = V0 (1 +a V t )
2
The value of a v is found be 1/273 for all gases (charle's law)
5
Cp R+R t
7
for diatomic gases, x = R, g = = 2 = = 1.46 Therefore, V t = V0 (1 +
273
)
CV 3 5
R+R
2 PRESSURE COEFFICIENT ( P) :
5 3 It is defined as the ratio of the increase in pressure of the gas at
C R+ R constant volume per degree rise of temperature to its pressure at
3 p
= 2 2 = 8 = 1.33
for triatomic gases, x = R , g = 0°C
2 CV 3 3 6
R+ R
2 2 P t -P 0
ap =
P 0 ´t
COLLISION PROPERTIES : P0=Pressure of given mass of gas at 0°C
(i) Molecular diameter or collision diameter : The distance Pt=Pressure of given mass of gas at t°C
between the centers of the molecules at the point of their
closest approach. P t =P 0 (1 +a p t )
The value found by Ragnault & Gay Lussac was in the vicinity of
1 æ ö t
273 for all gases and hence P t = P0 çè1 + 273 ÷ø
120 Chemistry
AMAGAT LAW OF PARTIAL VOLUME : Liquefaction is further based on the following principles
The total volume of a mixture of non reacting gases at constant (a) Cooling by freezing mixture : eg NaCl & ice (-22°C), CaCl2
temperature and pressure is equal to the sum of the individual & ice(-55°C), KOH & ice (-65°C)
partial volumes of the constituents (b) Cooling by adiabatic expansion (Claude’s Method) : The gas
in this process suffers a loss in temperature.
V(total)= V1+V2+V3+.....+Vn = Vi
DE = q + w if q = 0 then DE = w or - DE = - w
LOSCHMIDT NUMBER :
Work is done by the gas at the cost of internal energy and
It is the number of molecules present in 1cc of a gas or vapour at temeprature is lowered.
STP. Its value is 2.617×1019 per cc.
(c) Cooling by Joule-Thomson effect ( Linde’s method) :
AVERAGE MOLECULAR WEIGHT OF A GASEOUS Expansion of a gas through a small jet under adiabatic
MIXTURE : conditions results in cooling and liquefaction of gas

niMi LIQUID STATE :


Mmix= l Liquid state of the matter is the intermediate state between
ni
the gaseous and the solid state.
ni= number of moles l In liquids the molecules of the matter are held together by
Mi= molecular weight of each component strong intermolecular forces in comparison to those in gases.
BAROMETIC FORMULA : l On the basis of kinetic molecular model, the liquid state is
The decrease in the atmospheric pressure with increase in altitude described as follow:
given by the following expression is called barometic formula. (i) A liquid is composed of small molecules.
(ii) The molecules of the liquid are held closer by some kind
æ P ö Mgh of intermolecular forces.
ln çç ÷=
÷ RT
è P0 ø (iii) The intermolecular forces are not very strong and thus
P0=Pressure at the sea level the molecules are always in constant random motion.
P = Pressure at height h (iv) The average kinetic energy of molecules of a
liquid is directly proportional to their absolute
M = Average molecular mass of air, 29 ´ 10 -3 kgmol -1 temperature.
g = acceleration due to gravity, 9.8 ms-2 l Properties of the liquid :
R = gas constant ( 8.314 JK-1 mol-1) Most of the physical properties of liquids are controlled by
T= Temperature in kelvin the strength of intermolecular attractive forces existing
ESCAPE VELOCITY : between molecules of a liquid. These intermolecular forces
Velocity required by an object to escape from the gravitational arranged in order of their increasing strength are
London forces/induced diplole<dipole attraction< Hydrogen
field of a body. It is given by V e = 2gr
bonding.
AVERAGE KINETIC ENERGY : (1) Shape: Liquids have no definite shape of thier own. It
acquires the shape of the container in which
3 3 R
It is KE of a single molecule, and KE = kT = . T , where k= it is put.
2 2 No
(2) Volume: Liquids have a definite volume.
æ R Gas Constant ö (3) Density: The densities of liquids are nearly 1000 times
Boltzmann constant ç k = = ÷ more than the densities of the gases under identical
è No Avogadro Number ø
conditions.
(4) Compressibility: Liquids are very less compressible
Total KE =N (average KE), N ( KE ) = 3 2 nRT
compared to gases.
LIQUEFACTION OF GASES : LIQUEFACTION OF (5) Diffusion: Liquids diffuse like gases but the process of
GAS CAN BE ACHIEVED BY diffusion (also known as intermixing) is much slower.
(6) Evaporation: The process of conversion of a liquid into
1. Increasing pressure : It increases attraction among
molecules. its vapours (i.e.) gaseous state) at room temperature is
2. Decreasing temperature : It decrease Kinetic energy of known as evaporation.
molecules. The temperature of gas must be lower than its VAPOUR PRESSURE :
critical temperature, TC. At equilibrium state the pressure exerted by vapour phase is called
vapour pressure at a specific temperature.
States of Matter 121
l The magnitude of vapour pressure depends upon the SURFACE TENSION :
following two factors. l The surface tension is defined as the force per cm acting
(i) The liquids having weak intermolecular attraction have perpendicular to the tangential line on the surface of the
greater tendency of escaping from the liquid surface in liquid which tend to compress the surface area.
comparison to liquids having stronger forces of The units of surface tension are force per unit length i.e.
intermolecular attraction. For example, the vapour dynes cm–1 or Nm–1 (in SI units)
pressure of ether and acetone is more than that of water l Stronger the forces of intermolecular attraction (cohesive
or acetic acid at any specific temperature. forces) greater is the surface tension.
l The surface tension decreases with rise in temperature or
(ii) The vapour pressure of a liquid increases with the
surface tension in inversely proportional to temperature.
increase in temperature.
Surface tension µ 1/Temperature
l The vapour pressures of a given liquid at two different Measurement of surface tension of a liquid by the drop
temperatures may be compared using Clausius Clapeyron number method is the most convenient method.
equation
VISCOSITY:
P2 DH é T2 - T1 ù l Viscosity may be defined as the force of friction between
log = ê ú two layers of a liquid moving past one another with different
P1 2.303R ë T 1T2 û
velocities.
where P1and P2 are the vapour pressures at temperature T 1 l The viscosities of liquids are compared in terms of coefficient
and T2 D H is the heat of vapourisation and R is molar gas of viscosity which is defined as the force per unit area needed
constant. to maintain a unit difference in velocities between two
consecutive parallel layers which are one cm. apart.
l The temperature at which the vapour pressure of liquid
l The units of viscosity are poise (P) where 1P = g cm–1 sec–1.
becomes equal to atmospheric pressure (or the external
In SI units of viscosity 1P = 0.1 N sec m –2.
pressure) is termed as the boiling point of liquid. For example
l The liquid having stronger forces of attraction has a higher
at 1 atmospheric pressure acetone boils at 56°C. benzene at
viscosity.
80°C, ethyl alcohol at 78.4°C and water at 100°C.
l With the rise in temperature, viscosity of a liquid decreases
l The temperature at which the solid state and the liquid because the intermolecular attractive force between
form of a substance are in equilibrium at one atmospheric consecutive layers decrease as temperature increases.
pressure is known as freezing point

1. The density of a gas at 30°C and 1.3 atmosphere pressure is 2. Density of ammonia is 0.77 g/l. Calculate its vapour density.
0.027 g/cc. Calculate the molecular weight of the gas.
Densityof gas 0.77
Sol. V.D. = = =8.55
RTd Densityof H 2 0.09g / l 0.09
Sol. M = ; R = 0.0821 atm lit K -1 mol-1;
P
3. A gas cylinder containing cooking gas can withstand a
d = .027 g / cc = 27 g / l; pressure of 14.9 atmosphere. The pressure gauge of cylinder
indicates 12 atmosphere at 27°C. Due to sudden fire in the
T = 273 + 30 = 303K; P =1.3 atm
building the temperature starts rising. At what temeprature,
27 ´ .0821 ´ 303 cylinder will explode.
M= = 516 .66 g / mol
1. 3
Sol. P1 = P2 Þ 14.9 12
= Þ T1 = 372.5 K
T1 T2 T1 300
122 Chemistry
4. The total pressure exerted by a mixture of gases containing 12
0.4g H2, 2.2g of CO2, 1.4 g N2 and 3.2g SO2 is 2.5 atmosphere Second equation 1.1 ´ V = ´ .0821(t + 273 + 10)
120
. What are the partial pressures of each gas under the same
conditions? 1 t + 273
Dividing = or t = - 173°C
Sol. Partial pressure of a gas = mole fraction × total pressure 1.1 t + 283
= 273 - 173° C = 100 K
Total moles = 0.4 + 2.2 + 1.4 + 3.2 = 0.35
2 44 28 64 12
V= ´ 0.0821(100) = 0.821 litre
120
0.2
pH 2 = ´ 2.5 = 1.428 atm; 8. Calculate the molecular mass of a gas if its specific heat at
0.35
constant pressure is 0.125 and at constant volume is 0.075
0.05 Cp
pCO2 = ´ 2.5 = 0.357 atm 0.125 5
0.35 Sol. = = = 3 = 1.66 hence gas is monoatomic
Cv 0.075
0.05
pN 2 = ´ 2.5 = 0.357 atm;
0.35 Molar heat at constant volume C v = 3 2 R = 3 2 2 = 3 cal.

0.05 Also, specific heat molar mass = C v


pSO2 = ´ 2.5 = 0.357 atm
0.35
3
5. Calculate the total pressure in a 10 litre cylinder which contains 0.075 M 3 M= = 40 g / mol
0.075
0.4 g He, 1.6g O2 and 1.4 g of N2 at 27°C. Also calculate the
partial pressure of the He gas in the cylinder. Assume ideal
behaviour of gases, R = 0.0821 atm lit K-1 mol-1. 9. An open vessel at 27°C is heated until 3 5 th of the air in it
Sol. PV = (n1 + n 2 + n 3 ) RT has been expelled. Assuming that the volume of the vessel
remains constant find,
æ 0.4 1.6 1.4 ö (a) Temperature at which vessel is heated
P ´10 = ç + + ÷ ´ 0.821 ´ 300 ; P= 0.4926 atm
è 4 32 28 ø (b) The air escaped out of it if vessel is heated to 900 K
Total moles = 0.2 (c) The temperature at which the half of air escapes out
0.1 Sol. An open vessel has constant V and P.
pHe 0.4926 = 0.2463atm
0.2 \ n1T1 = n 2 T2
6. For the reaction N2O5 (g) 2 NO2 + 0.5 O2 .Calculate
(a) 1 ´ 300 = 2 T2
the mole fraction of N2O5(g) decomposed at constant volume 5
and temperature, if initial pressure is 600 mm Hg and pressure
at any time is 960 mm Hg. Assume ideal gas behaviour. 5 (
T2 = 750K = 477 ° C 3 expelled, 2 left
5 )
Sol. At constant V and T, Pressure is directly proportional to
number of moles. (b) 1 ´ 300 = n 2 ´ 900 n2 = 1 (1/ 3left, 2 expelled)
3 3
N 2 O 5 (g ) 2 NO 2 + 0.5O 2
Initial Moles 1 0 0 (c) 1´ 300 = 1 ´ T
At time t moles 1– x 2x 0.5x 2
Total moles = 1+1.5x
æ1 ö
Pressure of (1 + 1.5x) moles = 960 mm = 600K = 327° C ç left, 1 2 expelled ÷
è2 ø
960 10. A balloon of diameter 20m weights 100 kg. Calculate its pay
Pr essure of 1mole = =600
(1 + 1.5x) load if it is filled with helium at 1.0 atm and 27°C. Density of
Mole fraction decomposed, x = 0.4 air = 1.2 kg/m3, R= 0.0821 dm3 atm K–1 mol–1.
7. The pressure exerted by 12 g of an ideal gas at temperature
3 4 22
tºC in a vessel of volume V litres is one atmosphere. When Sol. Volume of balloon 4
3 r (10)3 = 4190.47m 3
the temperature is increased by 10 degrees at the same 3 7
volume, the pressure increases by 10%. Calculate t and V
(molecular weight of gas = 120). Mass of the air displaced 4190.47 1.2 = 5028.56 kg
Sol. PV = nRT
1 4190.47 103
12 Moles of helium in the balloon = = 170344
First equation 1´ V = ´ .0821(t + 273) 0.082 300
120
States of Matter 123

Mass of helium 4 170344 g = 681.376 kg


Sol. r1 P1 M2
or
n1 t2 P1 M2
Mass of filled balloon = 681.376 + 100 = 781.376 kg r2
=
P2 M1 n2 t1
=
P2 M1
Pay load Mass of air displaced Mass of filled balloon 1 57 0.8 M 2
=

5028.56 781.376 = 4247.184 kg 38 1 1.6 28


M 2 = 252
11. An evacuated glass vessel weighs 50.0g when empty, 148.0g
when filled with a liquid of density 0.98g/ml and 50.5g when The compound is XeF6 = 131 + 19 6 = 245
filled with an ideal gas at 760 mm Hg and 300K. Determine the Where atomic weight of Xe is 131 and atomic weight of F is
molecular weight of gas. 19. If two Xe atoms were there, weight would have exceeded
Sol. Weight of liquid 148 50 = 98g 252 which is not possible.
15. A 4:1 molar mixture of He and CH4 is contained in a vessel at
98 20 bar pressure. Due to hole in the vessel, the gas mixture
Volume of liquid = = 100 ml = volumeof vessel
0.98 leaks out what is the composition of the mixture effusing
out initially? (IIT 1994).
weight of gas 50.5 50.0 = 0.5g
Sol. Molar ratio of He and CH4 is 4:1
w 760 100 Partial pressure ratio of He and CH 4 = 16 : 4
Gas equation PV RT or
M 760 1000
(QTotal Pressure = 20bar)
0.5
.0821 300 r1 p1 M2 r 16 16
M = ; or He = = 8 :1
r2 p2 M1 rCH4 4 4
or M = 123
12. 20 dm3 SO2 diffuse through a porous partition in 60 seconds Composition of the mixture effusing initially for He and CH4
What volume of O2 will diffuse under similar conditions in 30 is 8:1.
seconds. 16. A container of capacity 2 litre contains 1024 gas molecules
22
V / 30 64 each weighing 10 g. The root mean square velocity is
Sol. = or V = 14.1 dm 3
20 / 60 32
1.5 105 cms 1 Calculate (i) Total pressure (ii) Total kinetic
13. At 27°C hydrogen is leaked through a tiny hole into a vessel
energy (iii) Temperature of the gas
for 20 minutes. Another unknown gas at the same temperature
and pressure as that of hydrogen leaked through the same 1
Sol. (i) Kinetic gas equation is PV = mNU 2
hole for 20 minutes. After the effusion of the gases the mixture 3
exerts a pressure of 6 atm. The hydrogen content of the
22
mixture is 0.7 mole. If the volume of the container is 3 litre, 1 10 1024 (1.5 105 ) 2
P 3.75 108 dynes / cm2
what is the molecular mass of the unknown gas? 3 2 103
Sol. The partial pressure of hydrogen and unknown gas (ug) can
be obtained as 1
(ii) Total Kinetic energy = N mU 2 ÷
2
0.7
p H2 0.0821 300
3 1024 10 22
= (1.5 105 ) 2 = 1.125 1012 ergs
n 2
pug 0.0821 300 n = number of moles
3
1 10 24
(iii) The number of moles = = 1.66 moles
1
pH 2 + pug = 6 = 0.0821 300(0.7 + n) 6.02 10 23
3 3 nRT
KE =
or n = 0.0308 mole 2

0.7 / 20 m 2KE 2 1.125 1012


Applying law of diffusion , = , T=
= = 5434. 3K
0.308 / 20 2 3nR 3 1.66 8.314 10 7
or m = 10.33 17. Calculate translational KE of 2 moles of a gas at 27°C
14. One mole of nitrogen gas at 0.8 atm takes 38 seconds to 3 3
diffuse through a pinhole, whereas one mole of an unknown Sol. KE= nRT = 2 8.314 10 7 300 ergs
2 2
compound of xenon with fluorine at 1.6 atm. takes 57 seconds
to diffuse through the same hole. Calculate the molecular 7482.5 107 ergs = 7482.5 Joules
formula of the compound.
124 Chemistry
18. Calculate the average and total kinetic energy of 0.5 mole of
PV 7.57 ´ 103 ´ 10-3
an ideal gas at 0°C \ T= = ´ 6.023 ´ 10 23
nR 2 ´ 10 21 ´ 8.314
3 R T = 274.2 K
Sol. Average KE per molecule of the gas = . T
2 N0
3RT 3 ´ 8.314 ´ 274.2
8.314 ´ 107 U rms = = (M.W.in Kg)
=3 ´ ´ 273 =5.65 ´ 10-14 ergs M 28 ´ 10-3
2 6.023 ´ 1023
Total Kinetic energy of 0.5moles of a gas = 494.22 ms -1
-21
= 5.65 ´ 10 Joules = 3 nRT = 3 ´ 0.5 ´ 8.314 ´ 10 ´ 373
7
2 2 U mp
= 0.82 ; U mp = 494.22 ´ 0.82 = 405.26 ms -1
=1.702 ´ 1010 ergs =1.702kJ U rms
19. The composition of the equilbrium mixture, Cl2 2Cl, 22. A gaseous mixture of helium and oxygen is found to a density
which is attained at 200º C, is determined by measuring the of 0.518 g dm-3 at 25°C and 720 torr. What is the percent by
rate of effusion through a pin hole. It is observed that at 1.80 mass of helium in the mixture?
mm Hg pressure, the mixture effuses 1.16 times as fast as
krypton effuses under the same condition. Calculate the Sol. M av = dRT = 0.518 ´ 0.0821´ 298 = 13.36 g mol -1
fraction of the chlorine molecules dissociated into P 720 / 760
atmosphere. (Atomic wt. of krypton 84) Let x be the mole fraction of helium in the mixture,
Sol. r mix = M kr 84 then M av = xM He + (1 - x ) M O 2
or 1.16 = Þ M mix = 62.425
r Kr M mix M
13.36 = x ´ 4 + (1 - x )32 \ x = 0.666
Dissociation: Cl2 2Cl
1-a 2a 0.666 ´ 4
Total number of moles = 1+a Mass percent of helium = ´ 100
(0.666 ´ 4 + 0.334 ´ 32)
Normal M.wt
= 1+ a ;
71
=1+ a
= 19.95
Experimental M. wt 62.425
23. Using Van der Waal's equation calculate the constant ‘a’
a = 0.137 = 13.7%
when the moles of a gas confined in a 4 l flask exerts a pressure
20. The average speed at T1 K and the most probable speed at of 11.0 atm at a temperature of 30 K. The value of ‘b’ is 0.05 lit
T2 K of CO2 gas is 9×104 cm sec-1. Calculate the value of
mol-1.
T1and T2.
æ n 2a ö
8RT1 Sol. çç P + 2 ÷÷ (V - nb) = nRT
Sol. Average speed at T1K = è V ø
pM
æ 2 ö
Most probable speed at T2 K =
2RT 2 ç11 + 2 a ÷ ( 4 - 2 ´ 0.05) = 2 ´ 0.0821´ 300
ç 4 2 ÷ø
M è
According to Question ,
8RT1
=
2RT2
= 9 ´10 4 a = 6.46 atm lit 2 mol - 2
pM M
24. (a) Calculate the pressure exerted by 5 mol of CO2 in one
8 ´ 8.314 ´10 7 ´ T1 2 ´ 8.314 ´10 7 ´ T2 litre vessel at 47°C using van der Waal's equation. Also
\ = = 9 ´ 10 4
3.14 ´ 44 44 report the pressure of gas if it behaves ideally in nature.
On calculating, we get T2 = 2143.37K and T1 = 1684.0 K Given that a = 3.592 atm ltr2 mol-2 . b = 0.042 litre mol-1
(b) If volume occupied by CO2 molecules is negligible, then
21. A glass bulb of 1 litre capacity contains 2 ´ 1021 molecules calculate the pressure exerted by one mol of CO2 gas at
of N2 exerting pressure 7.57×103 Nm–2. Calculate the RMS 273K .
speed and temperature of gas molecules. If the ratio of Ump
to Urms is 0.82 calculate the Vmp for these molecules at this æ 2 ö
Sol. (a) ç P + n a ÷ ( V - nb) = nRT
temperature. ç
è V 2 ÷ø
Sol. Given P = 7.57 ´103 Nm-2 æ 2 ö
ç P + 5 ´ 3.592 ÷ (1 - 5 ´ 0.0427) = 5 ´ .0821´ 320
V = 1litre = 10 -3 m 3 , R =8.3145 J K -1 mol-1 ç 1 ÷
è ø
P = 77.218 atm
2 ´ 10 21
n= when the gas behaves ideally then PV = nRT
6.023 ´ 10 23 mol
P ´1 = 5 ´ .0821´ 320 . Þ P = 131.36 atm
States of Matter 125

28. It is found that at 1470 K the mixture ( Cl 2 2Cl) diffuses


é a ù
(b) For 1 mole êP + ú [V - b] = RT 1.16 times as fast as Krypton (83.8) diffuses under identical
ë V2 û
conditions. Find the degree of dissociation at equilibrium.
RT a .0821´ 273 3.592
If b is negligible P = - = -
V V2 22.4 (22.4) 2 Sol. Cl 2 2Cl
P =0.9922 atm 1–x 2x
Where x is degree of dissociation.
25. The compression factor's (compressibility factor) for 1 mol The molecular weight of mixture of Cl 2 and Cl at equilibrium
of a van der Waal's gas at 0°C and 100 atmospheric pressure is given by
is found to be 0.5. Assuming that the volume of gas molecules
(1 - x ) ´ 71 + 2x ´ 35.5 71
is negligible, calculate the van der Waals constant ‘a’. M mix = =
(1 - x) + 2 x 1 +x

PV 100 ´ V R mix M Kr 83.5


Sol. Z = , 0.5 = V = 0.112litre. = = = 1.16
nRT 1 ´ .0821 ´ 273 R Kr M mix 71 /(1 + x )
van der Waal 's equation when ' b ' is negligible
\ x = 0.14
é a ù é a ù
ê P + 2 ú V = RT or ê100 + 2ú
= 0.0821 ´ 273 29. What is the molar volume of N 2 at 500 K and 600 atm
ë V û êë (0.112) ûú
according to (a) ideal gas law (b) virial equation. The virial
a = 1.253atm litre2 mol-2
coefficient. B of N 2 (g) at 500 K = 0.0169 lit mol -1 . How do
you interpret the result.
26. Using van der Waal’s equation calculate the constant ‘a’
Sol. For 1 mole of ideal gas PV = 1RT
when two moles of a gas is confined in a four litre flask exerts
a pressure of 11.0 atm at a temperature of 300 K. The value of 0.0821´ 500
V= = 6.84 × 10 - 2 lit mol -1
600
‘b’ is 0.5 litre mol -1 . (b) using virial equation, neglecting high powers
Sol. van der Waals equation for n moles
PV B BP
Z= =1+ =1+
é n aù2 RT V RT
êP + 2 ú (V – nb) = nRT
T
ëê v ûú 0.0169 ´ 600
Z= 1 + = 1.247
0.082 ´ 500
Given T = 300 K, V = 4 litre, P = 11.0 atm, n = 2 For 1 mol of a real gas
We have
PV ZRT 1.247 ´ 0.0821´ 500
Z= ; V= =
é 22 a ù RT P 600
ê11 + 2 ú [4.2 × 0.05] = 2 × 0.0821 × 300
êë 4 úû = 8.53 × 10 - 2 lit mol -1
The molar volume of the real gas is greater because of finite
a = 6.46 atm litre 2 mol - 2 volume of the gas molecules.
27. At room temperature ammonia gas at 1 atm pressure and 30. An evacuated glass vessel weighs 50 g when empty 148 g
hydrogen chloride at p atmospheric pressure are allowed to when filled with a liquid of density 0.98 g/ml and 50.5 g when
filled with ideal gas at 760 mm Hg at 300 K. Determine the
diffuse through identical pin holes from opposite ends of a
molecular wt of the gas.
glass tube of 1 meter length and uniform cross section.
Sol. Weight of liquid = 148 – 50 = 98 g
Ammonium is first formed at a distance of 60 cm from the
through which HCl gas is sent in what is the value of p? w 98
Volume of liquid = = = 100 ml
d 0.98
rNH3 M HCl PNH3 40
Sol. = × = wt of gas = 50.5 – 50 = 0.5g
rHCl M NH3 PHCl 60
W
PV = RT
T
M
36.5 1 40
or × =
17 PHCl 60 760 ´ 100 0.5
or = × 0.821 × 300
760 ´1000 m
PHCl = 2.198 atm or m = 123.15
126 Chemistry

Very Short/Short Answer Questions 12. A bacterial culture isolated from sewage produced 41.3 ml of
1. When do real gases behave as ideal gas? methane, CH4 at 31ºC and 753 mm Hg. What is the volume of
2. The size of weather balloon becomes larger and larger as it this methane at STP?
ascends up into higher altitudes. Why? Long Answer Questions
3. What do you mean by Boyle temperature ? Give its expression 13. (i) A quantity of hydrogen is confined in a chamber of
and its relation with inversion temperature.
constant volume. When the chamber is immersed in a
4. Liquid is transferred from a large beaker to a small beaker,
bath of melting ice, the pressure of the gas is 1000atm.
what will be the effect on its vapour pressure ?
5. Explain, why the bubbles of a gas in a boiling liquid generally (a) What is the Celsius temperature when the pressure
increase in volume as they approach the upper surface ? manometer indicates an absolute pressure of 400
6. What is the effect of temperature on the vapour pressure of atm?
a liquid ? (b) What pressure will be indicated when the chamber
7. Two different gases ‘A’ and ‘B’ are filled in separate is brought to 100 ºC?
containers of equal capacity under the same conditions of (ii) A steel tank containing air at 15 atm pressure at 15° C is
temperature and pressure. On increasing the pressure
provided with a safety valve that will yield at a pres-
slightly the gas ‘A’ liquefies but gas B does not liquify even
on applying high pressure until it is cooled. Explain this sure of 30 atm. To what minimum temperature must the
phenomenon. air be heated to blow the safety valve?
8. A gas occupying a volume of 100 litres is at 20°C under a 14. (i) One mole of the CO2 occupies 1.5 L at 25ºC. Calculate
pressure of 2 bar. What temperature will it have when it is the pressure exerted by the gas using:
placed in an evacuated chamber of volume 175 litres ? The
(a) an ideal gas equation
pressure of the gas in the chamber is one-third of its initial
pressure. (b) van der Waal’s equation if:
9. The values of the van der Waal’s constants for a gas are a = a = 3.012 atm mol–2 and b = 0.04 L mol–1.
4.10 dm6 bar mol–2 and b = 0.035 dm3 mol–1. Calculate the (ii) The critical temperature and pressure for NO gas are
values of the critical temperature and critical pressure for
177 K and 64.5 atm. respectively. Calculate the van der
the gas.
Waal’s constants ‘a’ and ‘b’.
pV 15. (i) A spherical balloon of 21 cm diameter is to be filled with
10. Compressibility factor, Z, of a gas is given as Z =
nRT hydrogen at STP from a cylinder containing the gas at
(i) What is the value of Z for an ideal gas ? 20 atm at 27ºC. If the cylinder can hold 2.82 L of water,
(ii) For real gas what will be the effect on value of Z above calculate the number of balloons that can be filled up.
Boyle’s temperature ?
(ii) Two van der Waals gases A and B have volumes 0.112
11. For real gases the relation between p, V and T is given by
and 0.111 L mol –1 r espectively. Calculate the
van der Waals equation :
compressibility factors for one mole of each at 273 K
an2 and 200 atm and hence state which gas is more
p ÷ (V nb) = nRT compressible.
V2
Multiple Choice Questions
where ‘a’ and ‘b’ are van der Waals constants, ‘nb’ is
approximately equal to the total volume of the molecules of 16. Dipole-dipole forces act between the molecules possessing
a gas. permanent dipole. Ends of dipoles possess ‘partial charges’.
‘a’ is the measure of magnitude of intermolecular attraction. The partial charge is
(i) Arrange the following gases in the increasing order of (a) more than unit electronic charge
‘b’. Give reason. O2, CO2, H2, He
(b) equal to unit electronic charge
(ii) Arrange the following gases in the decreasing order of
magnitude of ‘a’. Give reason. (c) less than unit electronic charge
CH4, O2, H2 (d) double the unit electronic charge
States of Matter 127
17. A plot of volume (V) versus temperature (T) for a gas at From the above data what would be the order of liquefaction
constant pressure is a straight line passing through the of these gases ?
origin. The plots at different values of pressure are shown Start writing the order from the gas liquefying first
in figure. Which of the following order pressure is correct (a) H2, He, O2, N2 (b) He, O2, H2, N2
for this gas ? (c) N2, O2, He, H2 (d) O2, N2, H2, He
21. Atmospheric pressures recorded in different cities are as
(a) p1 > p2 > p3 > p4 p1 follows :
(b) p1 = p2 = p3 = p4 Volume (mL) p2 Cities p in N/m2
(c) p1 < p2 < p3 < p4 p3 Shimla 1.01 × 105
(d) p1 < p2 = p3 < p4 Bangalore 1.2 × 105
p4
Delhi 1.02 × 105
Mumbai 1.21 × 105
Consider the above data and mark the place at which liquid
Temperature (K) will boil first.
18. The pressure of a 1 : 4 mixture of dihydrogen and dioxygen (a) Shimla (b) Bangalore
enclosed in a vessel is one atmosphere. What would be the (c) Delhi (d) Mumbai
partial pressure of dioxygen ? 22. Which curve in figure represents the curve of ideal gas ?
(a) 0.8 × 105 atm (b) 0.008 Nm–2 (a) B only FE
4
(c) 8 × 10 Nm –2 (d) 0.25 atm (b) C and D only
19. The ratio of Boyle’s temperature and critical temperature for (c) E and F only D
a gas is : (d) A and B only
C
8 27 A
1 2

pV
(a) (b) (c) (d) B
27 8 2 1
20. Gases possess characteristic critical temperature which
depends upon the magnitude of intermolecular forces
between the particles. Following are the critical temperature 0 p
of some gases. 23. Increase in kinetic energy can overcome intermolecular
Gases H2 He O2 N2 forces of attraction. How will the viscosity of liquid be
Critical temperature 33.2 5.3 154.3 126 affected by the increase in temperature ?
in Kelvin (a) Increase (b) No effect (c) Decrease
(d) No regular pattern will be followed

1. Which one of the following statements is wrong for gases? 4. Densities of two gases are in the ratio 1:2 and their
(a) Gases do not have a definite shape and volume temperatures are in the ratio 2:1 then the ratio of their
(b) Volume of the gas is equal to the volume of the container respective pressures is
confining the gas (a) 1:1 (b) 1:2
(c) Confined gas exerts uniform pressure on the walls of its (c) 2:1 (d) 4:1
container in all directions 5. Gas equation PV = nRT is obeyed by
(d) Mass of the gas cannot be determined by weighting a (a) only isothermal process
container in which it is enclosed. (b) only adiabatic process
2. Non reacting gases have a tendency to mix with each other. (c) both (a) and (b)
This property is known as
(d) None of these
(a) diffusion (b) fusion
6. The following graph illustrates
(c) mixing (d) None of these V
(a) Dalton’s law
3. Which of the following mixtures of gases does not obey
Dalton’s law of partial pressure ? (b) Charle’s law
(a) O2 and CO2 (b) N2 and O2 (c) Boyle’s law
(c) Cl2 and O2 (d) NH3 and HCl (d) Gay-Lussac’s law
Temp. (ºC)
128 Chemistry
7. 4.4 g of a gas at STP occupies a volume of 2.24 L, the gas can 15. For an ideal gas, correct relation is-
be
(a) O2 (b) CO é dE ù é dE ù
(a) ê dV ú = 0 (b) ê dT ú = 0
(b) NO2 (d) CO2 ë ûT ë ûP
8. Which of the following volume (V) - temperature (T) plots
represents the behaviour of one mole of an ideal gas at one é dE ù
(c) ê dT ú = 0 (d) All of these
atmospheric pressure ? ë ûV

V(L) V(L) 16. The value of R in SI units is :


(36.8 L (26.8 L
373 K)
(a) 8.315 ´10 7 erg K -1 mol -1
(22.4 L 373 K) (22.4 L
273K) 273K)
(b) 8.315 JK -1 mol-1

(c) 0.0815 litre atm K -1 mol -1


T(K) T(K)
(a) (b)
(d) 2 cal K -1 mol-1
17. The compressibility factor for H2 and He is usually :
V(L) V(L)
(30.6 L
(a) >1 (b) =1
(22.4 L 373 K) (22.4 L (c) <1 (d) Either of these
273K) 273K) 18. The densities of two gasses are in the ratio of 1: 16. The ratio
(14.2 L of their rates of diffusion is
373 K)
(a) 16:1 (b) 4:1
(c)
T(K)
(d)
T(K) (c) 1:4 (d) 1:16
19. At what temperature, the rate of effusion of N2 would be
o
9. If the four tubes of a car are filled to the same pressure with 1.625 times than that of SO2 at 50 C ?
N2, O2, H2 and Ne separately, then which one will be filled (a) 110 K (b) 173 K
first ? (c) 373 K (d) 273 K
(a) N 2 (b) O2 20. The rate of diffusion of methane at a given temperature is
twice that of X. The molecular weight of X is
(c) H 2 (d) Ne
(a) 64.0 (b) 32.0
10. When the product of pressure and volume is plotted against (c) 40.0 (d) 80.0
pressure for a given amount of the gas, the line obtained is
21. X ml of H2 gas effuse through a hole in a container in 5
(a) parallel to X-axis
seconds. The time taken for the effusion of the same volme
(b) parallel to Y-axis of the gas specified below under identical conditions is
(c) linear with positive slope
(a) 10 seconds : He (b) 20 seconds : O2
(d) linear with negative slope
11. Air at sea level is dense. This is a practical application of (c) 25 seconds : CO (d) 55 seconds : CO 2
(a) Boyle’s law (b) Charle’s law 22. The rate of diffusion of a gas having molecular weight just
(c) Kelvin’s law (d) Brown’s law double of nitrogen gas is 56 ml s–1 . The rate of diffusion of
12. Use of hot air balloons in sports and meteorological nitrogen will be
observations is an application of (a) 79.19 ml s–1 (b) 112.0 ml s–1
(a) Boyle’s law (b) Charle’s law (c) 56.0 ml s –1 (d) 90.0 ml s–1
(b) Kelvin’s law (d) Gay-Lussac’s law 23. Which pair of the gaseous species diffuse through a small
13. “Equal volumes of all gases at the same temperature and jet with the same rate of diffusion at same P and T
pressure contain equal number of particles.” This statement (a) NO, CO (b) NO, CO2
is a direct consequece of (c) NH3, PH3 (d) NO, C2H6
(a) Perfect gas law (b) Avogadro’s law 24. The rate of diffusion of SO2, CO2 , PCl3 and SO3 are in the
(c) Charle’s law (d) Boyle’s law following order
14. If 300 ml of a gas at 27 C is cooled to 70 C at constant
0
(a) PCl3 > SO3 > SO2 >CO2
pressure, its final volume will be (b) CO2 > SO2 > PCl3 > SO3
(a) 135 ml (b) 540 ml (c) SO2 > SO3 > PCl3 > CO2
(c) 350 ml (d) 280 ml
(d) CO2 > SO2 > SO3 > PCl3
States of Matter 129
25. Which of the following expression correctly represents the 34. At what temperature the RMS velocity of SO2 be same as
relationship between the average molar kinetic energy KE that of O2 at 303 K ?
of CO and N2 molecules at the same temperature (a) 273 K (b) 606 K
(c) 303 K (d) 403 K
(a) KE CO = KE N 2
35. The temperature of an ideal gas is reduced from 9270C to
270C. the r.m.s. velocity of the molecules becomes.
(b) KE CO > KE N 2
(a) double the inital value
(c) KE CO < KE N 2 (b) half of the initial value
(c) four times the initial value
(d) Can not be predicted unless the volumes of the gases
(d) ten times the initial value
are not given
36. If the average velocity of N2 molecues is 0.3 m/s at 27ºC,
26. Kinetic theory of gases presumes that the collisions between
then the velocity of 0.6 m/s will take place at
the molecules to be perfectly elastic because
(a) 273 K (b) 927 K
(a) the gas molecules are tiny particles and not rigid in nature
(c) 1000 K (d) 1200 K
(b) the temperature remains constant irrespective of collision
37. Gas deviates from ideal gas nature because molecules
(c) collision will not split the molecules
(a) are colouress
(d) the molecules are large particleand rigid in nature
(b) attaract each other
27. The average kinetic energy of an ideal gas per molecule in SI
unit at 25° C will be (c) contain covalent bond
(a) 6.17 × 10-21 kJ (b) 6.17 × 10–21 J (d) show Brownian movement.
(c) 6.17 × 10–20 J (d) 7.16 × 10–20 J 38. The compressibility factor for an ideal gas is
28. Boyle’s law, according to kinetic equation can be expressed (a) 1.5 (b) 1.0
as (c) 2.0 (d) ¥
(a) PV = KT (b) PV = RT 39. A gas will approach ideal behaviour at
(a) low temperature and low pressure
3 2 (b) low temperature and high pressure
(c) PV = kT (d) PV = kT
2 3 (c) high temperature and low pressure
29. The ratio between the root mean square velocity of H2 at 50 (d) high temperature and high pressure
K and that of O2 at 800 K is 40. The compressibility of a gas is less than unity at STP.
(a) 4 (b) 2 Therefore
(c) 1 (d) 1/4 (a) Vm > 22.4 litres (b) Vm < 22.4 litres
30. The r.m.s velocity of hydrogen is 7 times the r.m.s velocity (c) Vm = 22.4 litres (d) Vm = 44.8 litres
of nitrogen. If T is the temperature of the gas , than 41. An ideal gas will have maximum density when
(a) T(H 2 ) = T ( N 2 ) (b) T (H 2 ) > T ( N 2 ) (a) P = 0.5 atm, T = 600 K (b) P = 2 atm, T = 150 K
(c) P = 1 atm, T = 300 K (d) P = 1 atm, T = 500 K
(c) T(H 2 ) < T ( N 2 ) (d) T (H 2 ) = 7 T ( N 2 )
42. 120 g of an ideal gas of molecular weight 40 g mol -1 are
31. The r.m.s velocity of CO2 at temperature T (in kelvin) is
confined to a volume of 20 L at 400 K. Using R=0.0821 L atm
x cms -1 . At what temperature (in kelvin) the r.m.s. velocity -1
K-1 mol , the pressure of the gas is
of nitrous oxide would be 4x cms–1 ? (a) 4.90 atm (b) 4.92 atm
(a) 16 T (b) 2 T (c) 5.02 atm (d) 4.96 atm
(c) 4 T (d) 32 T 43. The volume of 0.0168 mol of O2 obtained by decomposition
32. Which of the following has maximum root mean square of KClO3 and collected by displacement of water is 428 ml at
velocity at the same temperature? a pressure of 754 mm Hg at 250 C. The pressure of water
(a) SO2 (b) CO2 vapour at 250 C is
(c) O2 (d) H2 (a) 18 mm Hg (b) 20 mm Hg
33. Density ratio of O2 and H2 is 16:1. The ratio of their r.m.s. (c) 22 mm Hg (d) 24 mm Hg.
44. Pressure of a mixture of 4 g of O2 and 2 g of H2 confined in
velocities will be
a bulb of 1 litre at 00 C is
(a) 4 : 1 (b) 1 : 16
(a) 25.215 atm (b) 31.205 atm
(c) 1 : 4 (d) 16 : 1
(c) 45.215 atm (d) 15.210 atm
130 Chemistry
45. Same mass of CH4 and H2 is taken in container, The partial 54. The value of van der waals constant ‘a’ for gases
pressure caused by H2 is O2, N2 , NH3 and CH4 are 1.360, 1.390, 4.170 and 2.253 liter 2
atm mol–2 respectively. The gas which can most easily be
8 1 liquefied is :
(a) (b)
9 9 (a) O2 (b) N2
1 (c) NH3 (d) CH4
(c) (d) 1
2
æ dT ö
22 55. Joule- Thomson coefficient ç ÷ for an ideal gas is
46. There are 6.02 ´ 10 molecules each of N2, O2 and H2 è dP ø H
which are mixed together at 760 mm and 273 K. The mass of (a) zero (b) positive
the mixture in grams is
(c) negative (d) None of these
(a) 6.02 (b) 4.12
56. The inversion temperature (T i) for a gas is given by
(c) 3.09 (d) 6.2
(a) a / Rb (b) 2a / Rb
47. The density of air is 0.00130 g/ml. The vapour density of air
will be Rb 2Rb
(a) 0.00065 (b) 0.65 (c) (d)
a a
(c) 14.4816 (c) 14.56 57. The Joule-Thomson coefficient for a gas is zero at
48. An ideal gas is one which obeys the gas laws under (a) Inversion temperature
(a) a few selected experimental conditions
(b) Critical temperature
(b) all experimental conditions
(c) Absolute temperature
(c) low pressure alone
(d) Below 0ºC
(d) high temperature alone
58. Longest mean free path under similar conditions of P and T
49. The temperature at which real gases obey the ideal gas laws
stands for.
over a wide range of pressure is called
(a) N2 (b) O2
(a) Critical temperature
(b) Boyle’s Temperature (c) H2 (d) Cl2
(c) Inversion temperature 59. The mean free path ( l ) of a gas sample is given by
(d) Reduced temperature
1
50. Which of the following exhibits the weakest inter- molecular (a) l = 2ps 2 N (b) l=
forces 2.ps2 N
(a) NH3 (b) HCl
(c) He (d) H2O (c) l = 2pus 2 N (d) None of these
51. The van der Waal’s equation reduces itself to the ideal gas 60. Which of the following statements is false
equation at (a) Avogadro Number = 6.02 × 1021
(a) high pressure and low temperature
(b) The relationship between average velocity (n ) and root
(b) low pressure and low temperature
(c) low pressure and high temperature mean square velocity (u) is (n ) =.9213u
(d) high pressure alone (c) The mean kinetic energy of an ideal gas is independent
52. An ideal gas obeying kinetic theory of gases can be liquefied of the pressure of the gas
if : (d) The root mean square velocity of the gas can be
(a) its temperature is more than critical temperature TC
calculated by the formula (3RT / M)½
(b) its pressure is more than critical pressure PC
(c) its pressure is more than PC at a temperature less than TC 61. At higher altitude the boiling point of water lowers because
(d) it cannot be liquefied at any value of P and T (a) atmospheric pressure is low
53. The ratio of Boyle’s temperature and critical temperature for (b) temperature is low
a gas is : (c) atmospheric pressure is high
8 27 (d) None of these
(a) (b) 62. Which of the following liquid will exhibit highest vapour
27 8
pressure?
1 2 (a) C2H5OH(l) (b) NH3 (l)
(c) (d)
2 1 (c) HF (l) (d) H2O(l)
States of Matter 131
63. A manifestation of surface tension is : 66. Water drops stick to a glass surface due to:
(a) rise of liquid in a capillary tube (a) cohesion (b) adhesion
(b) spherical shape of liquid drops (c) flocculation (d) None of these
(c) upward movement of water in soils 67. The internal resistance to flow in liquid is called
(d) All the above (a) Fluidity (b) Specific resistance
64. Generally , liquid drops assume spherical shape because: (c) Viscosity (d) Surface tension.
(a) a sphere has maximum surface area 68. With rise in temperature, viscosity of a liquid
(b) a sphere has minimum surface area (a) increase
(c) sphere is symmetrical in shape (b) decrease
(d) none of these (c) remains constant
65. On heating a liquid, its surface tension (d) may increase or decrease.
(a) increases (b) decreases 69. With the increasing molecular weight of a liquid, the viscosity
(c) remains same (d) is reduced to zero (a) decreases (b) increases
(c) remain constant (d) All are wrong

1. A weather ballon filled with hydrogen at 1 atm and 27°C has 7. A bubble of air is underwater at temperature 15°C and the
volume equal to 12000 litres. On ascending it reaches a place pressure 1.5 bar. If the bubble rises to the surface where the
where the temperature is –23°C and pressure is 0.5 atm. The temperature is 25°C and the pressure is 1.0 bar, what will
volume of the balloon is [CBSE-PMT 1991] happen to the volume of the bubble ?
(a) 24000 litres (b) 20000 litres [CBSE-PMT 2011M]
(c) 10000 litres (d) 12000 litres (a) Volume will become greater by a factor of 1.6.
2. If a gas expands at constant temperature, it indicates that : (b) Volume will become greater by a factor of 1.1.
[CBSE-PMT 2008] (c ) Volume will become smaller by a factor of 0.70.
(a) kinetic energy of molecules decreases (d) Volume will become greater by a factor of 2.5.
(b) pressure of the gas increases 8. 50 mL of each gas A and of gas B takes 150 and 200 seconds
(c) kinetic energy of molecules remains the same respectively for effusing through a pin hole under the similar
(d) number of the molecules of gas increases condition. If molecular mass of gas B is 36, the molecular
3. The pressure exerted by 6.0g of methane gas in a 0.03 m3 mass of gas A will be : [CBSE-PMT 2012]
vessel at 129°C is (Atomic masses : C = 12.01, H = 1.01 and (a) 96 (b) 128 (c) 20.25 (d) 64
R = 8.314 kpa dm3K–1 mol –1) [CBSE-PMT 2010] 9. A certain gas takes three times as long to effuse out as
(a) 31684 Pa (b) 215216 Pa helium. Its molecular mass will be : [CBSE-PMT 2012]
(c) 13409 Pa (d) 41777 Pa (a) 27 u (b) 36 u (c) 64 u (d) 9 u
4. By what factor does the average velocity of a gaseous 10. Maximum deviation from ideal gas is expected from :
molecule increase when the temperature (in Kelvin) is (a) N2(g) (b) CH4(g) [NEET 2013]
doubled ? [CBSE-PMT 2011] (c) NH3 (g) (d) H2(g)
(a) 2.0 (b) 2.8 (c) 4.0 (d) 1.4 11. For an ideal gas, number of moles per litre in terms of its
5. Two gases A and B having the same volume diffuse through presure P, gas constant R and temperature T is
a porous partition in 20 and 10 seconds respectively. The [AIEEE 2002]
molecular mass of A is 49 u. Molecular mass of B will be : (a) PT/R (b) PRT
[CBSE-PMT 2011] (c) P/RT (d) RT/P
(a) 50.00 u (b) 12.25 u (c) 6.50 u (d) 25.00 u 12. Value of gas constant R is [AIEEE 2002]
(a) 0.082 litre atm (b) 0.987 cal mol–1 K–1
6. A gaseous mixture was prepared by taking equal mole of CO
and N2. If the total pressure of the mixture was found 1 (c) 8.3 J mol–1 K–1 (d) 83 erg mol–1 K–1.
atmosphere, the partial pressure of the nitrogen (N2) in the 13. Kinetic theory of gases proves [AIEEE 2002]
mixture is : [CBSE-PMT 2011] (a) only Boyle’s law
(b) only Charles’ law
(a) 0.5 atm (b) 0.8 atm (c) 0.9 atm (d) 1 atm
(c) only Avogadro’s law
(d) All of these.
132 Chemistry
14. The heat required to raise the temperature of body by 1 K is 22. If 10–4 dm3 of water is introduced into a 1.0 dm3 flask at
called [AIEEE 2002] 300 K, how many moles of water are in the vapour phase
(a) specific heat (b) thermal capacity when equilibrium is established ? [AIEEE 2010]
(c) water equivalent (d) none of these. (Given : Vapour pressure of H2O at 300 K is 3170 Pa;
15. What volume of hydrogen gas, at 273 K and 1 atm pressure R = 8.314 J K–1 mol–1)
will be consumed in obtaining 21.6 g of elemental boron (atomic
(a) 5.56× 10–3 mol (b) 1.53 × 10–2 mol
mass = 10.8) from the reduction of boron trichloride by –2
hydrogen ? [AIEEE 2003] (c) 4.46 × 10 mol (d) 1.27 × 10–3 mol
(a) 67.2 L (b) 44.8 L 23. When r, P and M represent rate of diffusion, pressure and
(c) 22.4 L (d) 89.6 L molecular mass, respectively, then the ratio of the rates of
16. According to the kinetic theory of gases, in an ideal gas, diffusion (rA / rB ) of two gases A and B, is given as :
between two successive collisions a gas molecule travels [AIEEE 2011 RS]
(a) in a wavy path [AIEEE 2003]
(b) in a straight line path (a) ( PA / PB ) ( M B / M A )1/ 2 (b) ( PA / PB )1/ 2 ( M B / M A )
(c) with an accelerated velocity
(c) ( PA / PB ) ( M A / M B )1/ 2 (d) ( PA / PB )1/ 2 ( M A / M B )
(d) in a circular path
17. As the temperature is raised from 20ºC to 40ºC, the average 24. The molecular velocity of any gas is [AIEEE 2011 RS]
kinetic energy of neon atoms changes by a factor of which (a) inversely proportional to absolute temperature.
of the following ? [AIEEE 2004] (b) directly proportional to square of temperature.
(a) 313 293 (b) (313 / 293) (c) directly proportional to square root of temperature.
(d) inver sely proportion al to the squar e root of
(c) 1 2 (d) 2 temperature.
25. The compressibility factor for a real gas at high pressure is :
18. In van der Waals equation of state of the gas law, the constant
[AIEEE 2012]
‘b’ is a measure of [AIEEE 2004]
(a) volume occupied by the molecules RT pb pb
(b) intermolecular attraction (a) 1 + (b) 1 (c) 1 + (d) 1 –
pb RT RT
(c) intermolecular repulsions
(d) intermolecular collisions per unit volume 26. For gaseous state, if most probable speed is denoted by C*,
average speed by C and mean square speed by C, then for
19. Which one of the following statements is NOT true about
a large number of molecules the ratios of these speeds are :
the effect of an increase in temperature on the distribution
of molecular speeds in a gas? [AIEEE 2005] (a) C* : C : C = 1.225 : 1.128 : 1 [JEE Main 2013]
(a) The area under the distribution curve remains the same (b) C* : C : C = 1.128 : 1.225 : 1
as under the lower temperature (c) C* : C : C = 1 : 1.128 : 1.225
(b) The distribution becomes broader (d) C* : C : C = 1 : 1.225 : 1.128
(c) The fraction of the molecules with the most probable 27. At constant volume and temperature conditions, the rate of
speed increases diffusion DA and DB of gases A and B having densities rA
(d) The most probable speed increases and rB are related by the expression. [IIT-JEE 1993]
20. The volume of a colloidal particle, VC as compared to the
12 12
volume of a solute particle in a true solution VS , could be (a) DA = DB A (b) DA = DB B
[AIEEE 2005] B A
VC ~ 3 VC ~ - 3 12 12
(a) - 10 (b) - 10 B
VS VS (c) A (d) DA = D B
DA =DB ÷ ÷
B A
VC ~ 23 VC ~
(c) - 10 (d) -1 28. The compression factor (compressibility factor) for 1 mole
VS VS of a van der Waal’s gas at 0°C and 100 atm pressure if found
21. Equal masses of methane and oxygen are mixed in an empty to be 0.5. Assuming that the volume of gas molecules is
container at 25°C. The fraction of the total pressure exerted negligible, calculate the van der Waal’s constant 'a'.
by oxygen is [AIEEE 2007]
(a) 1/2 (b) 2/3 [IIT-JEE 2001]
(a) 0.253 L2 mol–2 atm (b) 0.53 L2 mol–2 atm
1 273
(c) ´ (d) 1/3. (c) 1.853 L2 mol–2 atm (d) 1.253 L2 mol–2 atm
3 298
States of Matter 133
29. The given graph represents the variation of Z [IIT-JEE 2009]
PV 2 2
(compressibility factor = ) versus P, for three real gases an an
nRT (a) nb (b) 2 (c)
(d) – nb –
A, B and C. Identify the only incorrect statement V V2
[IIT-JEE 2006] 31. For one mole of a van der Waals gas when b = 0 and
T = 300 K, the PV vs, 1/V plot is shown below. The value of
C the van der Waals constant a (atm. liter2 mol–2) is :
A [IIT-JEE 2012]

1
24.6
23.1
Z

PV(liter atm mol )


–1
21.6
0
P (atm) 20.1
(a) For the gas A, a = 0 and its dependence on P is linear at
all pressure.
(b) For the gas B, b = 0 and its dependence on P is linear at
all pressure
(c) For the gas C, which is typical real gas for which neither
a nor b = 0. By knowing the minima and the point of 0 2.0 3.0
intersection, with Z = 1, a and b can be calculated –1
1/V(mol liter )
(d) At high pressure, the slope is positive for all real gases
30. The term that corrects for the attractive forces present in a (a) 1.0 (b) 4.5 (c) 1.5 (d) 3.0
real gas in the van der Waals equation is

1. N2O4 is 20 % dissociated at 27°C and 760 torr. The density of 5. The partial pressure of hydrogen in a flask containing 2.016
the equilibrium mixture is g of H2 and 96.0 g of O2 is
(a) 3.1 g/l (b) 6.2 g/l (a) 1/8 of the total pressure (b) 1/6 of the total pressure
(c) 12.4g/l (d) 18.6 g/l (c) 1/4 of the total pressure (d) 2/3 of the total pressure
2. Helium atom is two times heavier than a hydrogen molecule 6. At 27°C a gas was compressed to half of its volume. To what
at. 298K. The average KE of helium is temperature it must be now heated so that it occupies just
(a) 2 times of H2 molecule its original volume. The pressure remains constant
(b) same as that of H2 molecule (a) 54°C (b) 327°C
(c) 4 times that of hydrogen molecule (c) 600°C (d) 327 °C.
1 7. Equal volumes of the gases which do not react together are
(d) that of H2 molecule
2 confined in separate vessels. The pressure is
3. The units of ‘a’ in van der Waals equation of state is 200 mm and 400 mm of Hg respectively. If the two gases are
mixed together what will be the pressure of the resulting
(a) atm. litre mol–1 (b) atm. litre2 mol–2
2 2 mixture (temperature remaining constant)
(c) atm litre mol (d) atm. litre mol–2
4. A container contains certain gas of mass ‘m’ of high pressure. (a) 400 mm (b) 400 mm
Some of the gas has been allowed to escape from the container
(c) 300 mm (d) 200 mm
and after some time the pressure of the gas becomes half and
its absolute temperature 2/3 rd. The amount of the gas 8. A flask containing air (open to the atmosphere) is heated
from 300 K to 500 K. The percentage of the air escaped into
escaped is
the atmosphere is
(a) 2/3 m (b) 1/2 m
(a) 16.6 (b) 40
(c) 1/4 m (d) 1/6 m
(c) 60 (d) 20
134 Chemistry
9. A sample of O2 gas is collected over water at 23°C at a 18. A bubble of the gas released at the bottom of a lake increases
barometer pressure of water at 751 mm Hg (Vapour pressure to eight times the original volume when it reaches at the
of water at 23°C is 21 mm Hg). The partial pressure of O2 gas surface. Assuming that the atmospheric pressure is equivalent
in the sample collected is to pressure exerted by a column of water 10 m high, what is
(a) 21 mm Hg (b) 751 mm Hg the depth of the lake
(c) 0.96 atm. (d) 1.02 atm. (a) 80 m (b) 90 m
10. Which of the following is a false statement ? (c) 10 m (d) 70 m
(a) Gases having same molecular masses diffuse at the
19. SO 2 and He are kept in a container at partial pressure P1
same rate
(b) 0.5 litre of nitrogen and 1 litre of helium will have the and P2 . A thin perforation is made in the wall of the container
same number of the atoms at the same temperature and and it is observed that gases effuse at the same rate. The
pressure
ratio of P1 and P2 will be
(c) The value of molar gas constant does not vary with the
nature of the gas (a) 4 : 1 (b) 1 : 4
(d) None is false (c) 1 : 16 (d) 16 : 1
11. The density of SO2 at STP is 2.06 kg m-3. Its density at 819°C 20. Helium has the van der Waals constant b = 24 ml mol -1 . The
and 2 atmosphere is
molecular diameter of helium will be
(a) 2.86 kg m-3 (b) 1.43 kg m-3
-3
(a) 267 pm
(c) 0.715 kg m (d) 4.2686 kg m-3
(b) 133.5 pm
12. When helium is allowed to expand into vacuum, heating effect
(c) 26.7 pm
is observed. Its reason is that
(d) Data not sufficient for calculation the diameter.
(a) helium is an ideal gas
21. At 100°C and 1 atm, if the density of liquid water is
(b) helium is an inert gas
1.0 g cm–3 and that of water vapour is 0.0006 g cm–3, then the
(c) inversion temperature of helium is low volume occupied by water molecules in 1 litre of steam at
(d) the boiling point of helium is the lowest amongst the that temperature is
element. (a) 6 cm3 (b) 60 cm3
13. What would be the vapour density of a gas 260 cm3 of which (c) 0.6 cm 3 (d) 0.06 cm3
at 290 K and 100.40 KPa pressure weights 0.160g 22. Which of the following graphs is not a straight line for an
(a) 17.9 gL-1 (b) 14.2 gL-1 ideal gas?
(a) n ® T (b) T ® p
(c) 7.4 g cm -3 (d) None of these
1 1
14. Reducing the pressure from 1.0 atm to 0.5 atm would change (c) n ® (d) n®
T p
the number of molecules in one mole of ammonia to
23. A volume V of a gas at temperature T1 and a pressure p is
(a) 25% of its initial value
enclosed in a sphere. It is connected to another sphere of
(b) 50% of its initial value
volume V/2 by a tube and stopcock. The second sphere is
(c) 75% of its initial value initially evacuated and the stopock is closed. If the stopcock
(d) None of the above is opened the temperature of the gas in the second sphere
15. The ratio of diffusion of nitrogen at 25°C would be ......... becomes T2. The first sphere is maintained at a temperature
times that of carbon dioxide at 75°C T1. What is the final pressure p1 within the apparatus ?
(a) 0.90 (b) 1.16 2pT2 2pT2
(c) 1.41 (d) 1.76 (a) (b)
2T2 + T1 T2 + 2T1
16. The average kinetic energy of 28 g CO at, 300 K is E kcal.
The average kinetic energy of 2 g H2 at the same temperature pT2 2pT2
would be..... kcal. (c) (d)
2T2 + T1 T1 + T2
(a) E (b) 14 E 24. The molecular velocities of two gases at the same temperature
(c) 1/14 E (c) 28 E are u1 and u2 and their masses are m1 and m2 respectively.
17. The critical temperature of water is higher than that of O2 Which of the following expressions are correct ?
because the H2O molecule has m1 m2
(a) fewer electrons than O2 (a) = (b) m1u1 = m 2 u 2
u12 u 22
(b) two covalent bonds
(c) V-shape m1 m 2
(c) = (d) m1u12 = m 2 u 22
(d) dipole moment. u1 u2
States of Matter 135
25. At low pressure, the van der Waal's equation is reduced to 27°C. The cylinder can hold 2.82 L of water at NTP. The number
of balloons that can be filled up is
pVm aP pVm b
(a) Z = = 1- (b) Z= = 1+ p (a) 15 (b) 10
RT RT RT RT (c) 20 (d) 25
pVm a 32. The mass of N 2 in a 15 L gaseous mixture at 20°C and 740
(c) pVm = RT (d) Z= = 1-
RT RT mm pressure of the composition of the mixture by volume
26. The van der Waal's equation for n = 1 mol may be expressed is H 2 = 10%, O 2 = 20% and N 2 = 70%
æ RT ö 2 aV ab (a) 11.91 g (b) 16.2 g
as V 3 - çç b + ÷V + - =0 (c) 21.91 g (d) 28.00 g
è p ÷ø p p
33. What will be the mole fraction of N 2 in a mixture of N 2
Where V is the molar volume of the gas. Which of the
and O 2 if partial pressure of O 2 is 63 cm and total
following is correct?
(a) For a temperature less than Tc, V has three real roots pressure of the mixture is 90 cm
(b) For a temperature more than Tc, V has one real and two (a) 3.0 (b) 0.3
imaginary roots (c) 0.7 (d) 0.5
(c) For a temperature equal to Tc all three roots of V are real 34. The bottles of NH 3 and HBr gases are connected through
and identical a tube of 1 metre length. The distance of white solid formed
(d) All of these in the tube from the end of NH 3
27. According to kinetic theory of gases, for a diatomic
(a) 68.56 cm from NH 3 bottle
molecule
(b) 68.56 cm from HBr bottle
(a) the pressure exerted by the gas is proportional to the
(c) At the centre of the tube
mean velocity of the molecule.
(d) None is correct.
(b) the root mean square velocity of the molecule is 35. At a temperature TK the pressure of 4 g of argon in a bulb
inversely proportional to the temperature. is p. The bulb is put in a bath having temperature higher
(c) the pressure exerted by the gas is proportional to the by 50 K than the first one. 0.8 g of argon has to be removed
rms velocity of the molecule. to maintain original pressure. The temperature T is equal
(d) the mean translational kinetic energy of the molecule to
is proportional to the absolute temperature. (a) 510 K (b) 200 K
(c) 73 K (d) 100 K
28. A perfect gas is found to obey the relation PV 3 / 2 = 36. Let the most probable velocity of hydrogen molecules at a
constant. If the gas is compressed to half of its volume at temperature of t° C be V0 . When the temperature is raised
temperature T adiabatically, the final temperature of the
to (2t + 273)°C the new rms velocity is (suppose all the
gas will be molecules dissociate into atoms at latter temperature)
(a) 2T 2 (b) 4T (a) (b) 6V0
2 3V0
(c) T 2 (d) 2T
æ 273 ö 2
29. A vessel containing gas at pressure 60 cm of Hg was (c) 3ç 2 + ÷V0 (d) V0
è t ø 3
connected to an arm A of open end manometer. The
atmospheric pressure was recorded as 74 cm of Hg. If 37. A vessel is filled with a mixture of O 2 and N 2 . At what
mercury in arm A stands at 84.5 cm height, the mercury in ratio of partial pressures will be the mass of gases be
arm B will stand at identical
(a) 24.5 cm (b) 70.5 cm (a) p( O ) = 8.75 p( N )
2 2
(c) 88 cm (d) 74 cm
30. Three gases A, B, C have volumes V1 , V2 , V3 at pressure (b) p( O ) = 0.78 p( N )
2 2
P and temperature T are mixed keeping the temperature (c) p( O ) = 0.875 p ( N )
2 2
and pressure constant. The final volume of the gaseous
mixture will be (d) p( O 2 ) = 11.4 p( N 2 )
V1 + V2 + V3 38. A compound exists in the gaseous state both as monomer A
(a) V1 + V2 + V3 (b)
3 and dimer A 2 . The M. wt. of monomer is 48. In an experiment
96 g of the compound was confined in vessel of 33.6 L and
V1 + V2 + V3 heated to 273° C. Calculate the pressure developed, if the
(c) (d) V1 ´ V2 ´ V3
P compound exists as a dimer to the extent of 50% by weight
31. A spherical balloon of 21 cm diameter is to be filled with under the conditions
(a) 0.9 atm (b) 4.0 atm
H 2 at NTP from a cylinder containing the gas at 20 atm at (c) 2.0 atm (d) 1.0 atm
136 Chemistry
39. A mixture of 1 mol of H 2 and 1 mole of Cl 2 with little 41. A closed vessel contains He and Ozone at pressure of P
charcoal in a 10 L evacuated flask was irriadiated with light atm. The ratio of He and oxygen atoms is 1 : 1. If He is
until the reaction was completed. Subsequently 5L of water removed from the vessel, the pressure of the system will
was introduced into the flask and the flask was cooled to reduce to
27°C. The pressure exerted by the system is approximately (a) 0.25 P (b) 0.5 P
equal to (c) 0.75 P (d) 0.33 P
42. The factor that has the largest effect on vapour pressure of
2 ´ 0.0821´ 300
(a) atm a liquid is
5 (a) liquid surface area
(b) » 26 mm Hg (b) molecular dipole moment
2 ´ 0.0821 (c) presence of H-bonding
(c) atm (d) molecular mass of liquid
10
43. The molecular mass of a compound does not effected by
4 ´ 0.0821´ 300 (a) vapour pressure of a liquid
(d) atm
5 (b) vapour density
40. The relative humidity of air is 80% at 27°C. If the aqueous (c) vapour pressure of solid
tension at the same temperature is 27 mm Hg. The partial (d) molar volume of vapour
pressure of water vapour in the air will be 44. Which of the following liquids has the highest viscosity?
(a) 21.60 mm Hg (b) 27 mm Hg (a) Benzene (b) Carbon disulphide
(c) 25 mm Hg (d) 23 mm Hg (c) Acetone (d) Ethanol
States of Matter 137

EXERCISE 1 5. (c) PV = nRT is for and ideal gas following isothermal or


adiabatic process.
1. At high temperature and low pressure.
6. (b) Charle’s law V µ T at constant P..
a 7. (d) 44g at STP occupies volume 22.4 litre which is molecular
3. Tb = , Ti = 2Tb
bR mass of CO2. Molecular mass occupies 22.4 litre at STP.
4. No effect, because vapour pressure is an intensive property.
5. The pressure on them decreases and hence volume 8. (c) V1 = V2 at const. pressure
T1 T2
increases.
6. Vapour pressure is directly proportional to the temperature. 22.4 V2
Þ = , V2 = 30.6 litre
7. Gas ‘A’ is at or below its critical temperature and gas ‘B’ is at 273 373
a temperature higher than critical temperature.
9. (c) Lesser the density more is the volume and H2 has lowest
8. T2 = –102.1°C
9. Tc = 422.76 K MP
density. ( d = at constant P, R and T, d µ M )
Pc = 123.96 bar RT
10. (i) Z = 1 for ideal gas 10. (a) Plot of PV vs P at constant T is parallel to X axis.
(ii) For a real gas Z > 1 above Boyle’s temperature.
12. V2 = 36.7 ml. æ MP ö
11. (a) dµ p, Boyle’s law, ç d = ÷ . At sea level pressure is
13. (i) (a) T2 = – 164° C è RT ø
(b) P2 = 1.37 × 103 atm more, hence density of air is more.
(ii) T2 = 303°C 12. (b) Hot air is lighter due to less density (Charle’s law)
14. (i)
æ MP ö
(a) P = 16.29 atm. çd = ÷
è RT ø
(b) P = 15.137 atm.
(ii) a = 1.34 L2 atm. mol–2 13. (b) When P, V and T are same no. of particles will also be
b = 0.02813 L mol–1 same (Avogadro law)
15. (i) Number of balloons filled = 10. 14. (d) At constant pressure
(ii) \ ZA = 1.00062
V1 V2 300 V2
and ZB = 0.9917 = Þ = or V2 = 280 ml
T1 T2 300 280
ZA > ZB, therefore B is more compressible.
16. (c) 17. (c) 18. (c) 19. (b)
æ Eö
20. (d) 21. (a) 22. (a) 23. (c) 15. (a) ç ÷ = 0 For ideal gas and positive for real gas.
è V øT
EXERCISE 2
1. (d) Mass of the gas can be determined by weighing the 16. (b) In SI unit value of R= 8.314 J K-1 mol-1 (see text)
(container + gas) – container alone.
PV
2. (a) Intermixing of gases in known as diffusion. 17. (a) Since for H 2 and He, PV > nRT and Z = . Hence
nRT
3. (d) Dalton's law of partial pressure is applicable to non
reacting gases. Here NH3 and HCl react to form NH4Cl. Z is more than 1

P1 d T 1 2 r1 d2 16
4. (a) P d and T, = 1 1 = ´ Þ P1 : P2 = 1 : 1 18. (b) = = = 4 :1
P2 d 2 T2 2 1 r2 d1 1
138 Chemistry

3RT U CO 2 TCO 2 ´ M N 2 O
19. (c) r U and U = =
M 31. (a)
U N 2O M CO 2 ´ TN 2 O

r1 TM rN T1 ´ 64
= 1 2 or 2
= = 1.625 or
r2 T2 M1 rSO 2 323 ´ 28 x TCO2 44
= ´ TN 2O = 16TCO 2
4x 44 TN 2O
T2 = 373K

2 Mx 1
20. (a) = M x = 64 32. (d) U hence hydrogen (molecular weight being the
1 16 M
lowest) has the maximum root mean square velocity.
21. (b) For effusion of same volume,

t1 M1 t1 t2 U1 d2 1
= = 33. (c) = = =1: 4
t2 M2 U2 d1 16
M1 M2

This is clearly seen from the options that the ratio of U1 T1M 2
34. (b) Apply =
t æ U2 T2 M1
5 20 5 ö
is same for H2 and O2. ç = = ÷÷
M ç
è 2 32 2ø
3RT U1 T1 1200 1
35. (b) U = = = = 2; U2 = U1
M N2 M U2 T2 300 2
rx 28 56 1
22. (a) = = = or rN 2 = 79.19 mls -1
rN 2 Mx 56 rN 2 2 36. (d)
23. (d) NO and C2H6 both have same molecular weight. Hence 37. (b) Due to intermolecular interactions appreciable at high P
their rate of diffusion will be same. and low T, the ideal gas deviates from ideal behaviour.

1 PV
38. (b) Ideal gas strictly follows PV= nRT, Z= =1
24. (d) Rate . The smaller the value of M the more is the nRT
M
rate of diffusion 39. (c) Ideal behaviour at low P and high temperature. The larger
25. (a) At the same temperature KE is the same, as KE T.. the volume the lesser are the interactions, higher the
temperature the more is KE the lesser are interactions
26. (a) The gas molecules are tiny particles and not rigid in nature
between the molecules.
rather they are perfect elastic bodies
PV
3 3 8.313 40. (b) Z = <1 nRT > PV
27. (b) KE = kT = ´ ´ 298 = 6.17 ´10-21 J . nRT
2 2 6.023 ´1023
or 1´ 0.0821´ 273 > 1´ V or V < 22.4
(Average Kinetic energy KE = 3 kT = 3 R T )
2 2 N æ MP ö
41. (b) Higher P lower T greater the density. ç d = ÷
è RT ø
1 2 1 2 2 2
28. (d) PV = mnU = ´ mU = KE = KT .
3 3 3 3
42. (b) PV = RT , PV = w RT , 20P = 120 ´ .0821 ´ 400
M 40
The product PV will have constant value at constant
temperature. This is Boyle’s law or P = 4.92 atm

U1 TM 50 ´ 32 43. (d) Volume of 0.0168 moles at STP


29. (c) = 1 2= =1
U2 T2 M1 800 ´ 2 = 0.0168 × 22400 = 376.3ml.

P1V1 P2 V2 760 ´ 376.3 P2 ´ 428


= or = or P2 = 730mm
U H2 TH2 ´ 28 TH2 1 T1 T2 273 298
30. (c) = = 7 = TN2 = 2TH 2
U N2 TN2 ´ 2 TN2 2
Pressure of water = 754 – 730=24 mm

TN 2 > TH 2
States of Matter 139
44. (a) Total moles
60. (a) Avogadro number is 6.023 ´ 10 23
= 4 + 2 = 1.125 ; PV = nRT Þ P = 1.125 ´ .0821´ 273 61. (a) At higher attitudes, boiling point is low because
32 2
atmospheric pressure is low.
P = 25.215 atm 62. (b) The strength of H-bonding is in the order N....H< O....H <
x x F....H
45. (a) Let the mass be x, then moles of CH4 and H2 are & ;
16 2 63. (d) All the given phenomenon occurs due to surface tension
64. (b) Liquid drops assume spherical shape because a sphere
9x has minimum surface area.
Total moles =
16 65. (b) Surface tension decreases with increase in temperature
66. (b) Water sticks to a glass surface due to force of adhesion.
x/2 8
pH 2 = mole Fraction ´ P = .P = P 67. (c) The internal resistance of a liquid to flow is called
9 x / 16 9
viscosity.
46. (d) 6.02 ´ 10 23 molecules : 28 g N 2 = 32 g O 2 = 2 g H 2 68. (b) Visocity µ 1/temperature
69. (b) Visocity µ molecular weight
6.02 ´ 10 22 molecules : 2.8 g N 2 = 3.2 g O 2 = 0.2 g H 2
EXERCISE 3
Total mass = 2.8 + 3.2 + 0.2 = 6.2
47. (d) VD of air = mass of 11200cc of air = 11200 × density of air P T 1 250
1. (b) V2 = 1 2 .V1 = ´ ´12000 lit. = 20000 lit.
= 11200 × .00130 = 14.56 P2 T1 0.5 300
48. (b) An ideal gas obeys the gas laws under all experimental
2. (c) At any constant temperature the K.E. of gaseous
conditions.
molecules remains same (K.E. µ T). Thus option (c) is
49. (b) Boyle’s temperature by definition. At this temperature
correct answer.
the real gases obey ideal gas laws over wide range of P.
nRT 6 ´ 8.314´ 402
50. (c) He possess van der waals forces of attraction, which are 3. (d) P = = 41777 Pa
V 16 0.03´103
weak in nature.
51. (c) At low pressure and high temperature terms a and b are 8RT
negligible hence PV= nRT 4. (d) Average velocity =
pM
52. (d) Ideal gas cannot be liquefied at any value of P and T
since there are no intermolecular interactions between i.e., v µ T
molecules
v2 2T
53. (b) Boyle’s temperature, \ = = 1.41
v1 T
a 8a T 27
Tb = and critical temperature, Tc = \ b =
Rb 27Rb Tc 8 rA MB
5. (b) r = M
B A
54. (c) The higher the value of ‘a’, more the value of T c , easy is
the liquefaction
V
55. (a) Ideal gas does not exhibit Joule Thomson effect. Hence MB
20 = M B Þ 1 =
æ dT ö V 49 2 49
ç ÷ , the Joule-Thomson coefficient is zero 10
è dP ø H
1
2a MB = ´ 49 = 12.25
56. (b) Ti = Where Ti is inversion temperature. 4
Rb
57. (a) At exact inversion temperature there is neither heating 6. (a) Given nCO = n N2
nor cooling or no Joule Thomson effect
PCO + PN2 = 1 atm
1 Partial pressure of a gas
58. (c) Mean free path µ
(Radius)2 = mole fraction of gas × total pressure

59. (b) Formula for mean free path. is l = 1 n N2 n N2


1
2 .ps 2 N \ PN2 = ´ 1 = 2n ´ 1 = = 0.5 atm.
n CO + n N2 N2 2
140 Chemistry
7. (a) Given 16. (b) According to kinetic theory the gas molecules travel in a
P1 = 1.5 bar T1 = 273 + 15 = 288 K V1 = V straight line path but show zig-zag motion due to
P2 = 1.0 bar T1 = 273 + 25 = 298K V2 = ? collisions.

PV
1 1 = P2V2 3
K ´ 313
T1 T2 K.E of neon at 40 °C 313
17. (a) = 2 =
K.E of neon at 20°C 3 293
1.5 ´V 1´V2 K ´ 293
= 2
288 298
V2 = 1.55 V i.e., volume of bubble will be almost 1.6 time 18. (a) In van der waal's equation ‘b’ is for volume correction
to initial volume of bubble. 19. (c) As temperature rises the most probable speed increases
and the fraction of molecules possessing most probable
VA VB MB speed decreases.
8. (c) =
tA tB MA
20. (a) Particle size of colloidal particle = 1m m to 100 m m
(suppose 10 ml)
200 36 4 36
Þ = Þ =
150 MA 3 MA 4 3
Vc = pr
3
16 36 81
Þ = Þ MA = = 20.25
9 MA 4 4 3
= Vc = p(10)
9. (b) t1 - Time taken by unknown gas 3
t2 - Time taken by helium
Particle size of true solution particle = 1m m
M1 - Molar mass of unknown gas
M2 - Molar mass of helium 4 3
Vs = p (1)
t1 = 3t2 3

t1 M1 3t M1 Vc
= Þ 2 = 3
t2 M2 t2 4 hence now = 10
Vs

M1 21. (d) Let the mass of methane and oxygen = m gm.


9=
4 Mole fraction of O2
M1 = 36
Moles of O2
10. (c) Higher the critical temperature more easily will be the =
Moles of O 2 + Moles of CH 4
gas liquify. Now since most easily liquifiable gas
show larger deviation, NH3 will show maximum
m / 32 m / 32 1
deviation from ideal behaviour. = = =
11. (c) PV = nRT (number of moles = n/V) m / 32 + m /16 3m / 32 3

\ n/V = P/RT Partial pressure of O2 = Total pressure × mole fraction of


12. (c) 8.314 JK–1 mol–1 1 1
13. (d) O2, PO2 = P × = P
3 3
14. (b) The heat rquired to raise the temperature of body by 1K
22. (d) From the ideal gas equation :
is called thermal capacity or heat capacity.
PV = nRT
21.6
15. (a) No. of moles of boron = = 2 for BCl3 PV 3170 ´ 10-3
10.8 or n = = = 1.27 × 10–3
RT 8.314 ´ 300
\ 1mole of Boron = 3 mole of Cl
\ 2 mole of Boron = 6 mole of Cl P
23. (a) rµ
m
H 2 + Cl 2 ® 2HCl
Þ 3 moles of Hydrogen is required rA PA MB
=
= 3 × 22.4 = 67.2 Litre rB PB MA
States of Matter 141
24. (c) The different type of molecular velocities possessed 29. (b)
by gas molecules are 30. (b) Correction factor for attractive force for n moles of real
2RT gas is given by the term mentioned in (b).
(i) Most probable velocity (a) =
M 31. (c)
24.6
8RT
(ii) Average velocity v =
pM
(iii) Root mean square velocity in all three cases PV 21.6
3RT 20.1
v=
M
In all the above cases
3.0
Velocity µ T 2.0 1/V
0
æ a ö a
25. (c) çè P + 2 ÷ø (V - b) = RT at high pressure 2 can be æ a ö
V V ç P + 2 ÷ (V ) = RT
è V ø
neglected PV + a/V = RT
PV – Pb = RT and PV = RT + Pb a
PV = RT -
PV Pb V
= 1+
RT RT y = RT – a(x)
Pb 21.6 - 20.1
Z = 1+ ; Z > 1 at high pressure So, slope = – a = = – a = – 1.5
RT 2-3
a = 1.5
2RT EXERCISE 4
26. (c) Most probable speed (C*) =
M 1. (a) 2NO2
1-0.2 0.4
8RT Total moles after dissociation = 1 –0.2+0.4 = 1.2
Average Speed (C) =
pM
0.8 ´ 92 0.4 ´ 46
Molecular weight of mixture, M = + = 74
3RT 1.2 1.2
Root mean square velocity (C) =
M 75 ´ 1
Density = PM = = 3.1 g / l
RT 0.0821´ 300
2RT 8RT 3RT
C*: C : C = : : 3 n
M pM M 2. (b) KE = RT . Average KE is the same for all
2 No
4 3 gaseous molecules
= 1: : = 1:1.128 :1.225
p 2 3. (b)
12 12 m
DA r ér ù ær ö 4. (c) Gas equation is PV = RT (i)
27. (d) = B =ê Bú ; \ DA = DB ç B ÷ M
DB rA ër A û è rA ø
P m 2
28. (d) We know that Again V = 1 R . T (ii) Divide (i) by (ii)
2 M 3
PV 100 ´ V
Z= Þ 0.5 = Þ V = 0.112 lit. m 3 3 1
nRT 1´ 0.0821´ 273 2 = ´ \ m1 = m . Gas escaped is then = m
m1 2 4 4
According to van der Waals equation

æ 2.06 96.0
an2 ö 5. (c) Moles of H 2 = ; Moles of O 2 = = 6 Total
ç P + 2 ÷ (V - nb) = nRT 2 16
ç V ÷ø
è moles = 8
æ a ö 2 1
PH = P = (partial pressure = Mole fraction × Total
çç 100 + 2 ÷÷
(0.112 - 0) = 0.0821´ 273 2 8 4
è (0.112) ø
pressure )
a = 1.253 L2 mol–2 atm
142 Chemistry
17. (d) Critical temperature of water is higher than O2 because
V1 V2
6. (b) Let the original volume be V then = at constant H2O molecule has dipole moment which is due to its
T1 T2
V-shape.
V V 18. (d) Let V be the original volume of bubble. The final volume
P, = \ T2 = 600K = 327°C
300 ´ 2 T2 will be 8 V. Let p be the atmospheric pressure and p1 the
7. (c) When vessels are joined the volume is doubled and pressure at the bottom
pressure is reduced to half \ Pressure of mixture
\ p × 8 V = p1 × V;;
200 400
= + = 300mm 8p = p1
2 2
8. (b) P,V and R are constant p1 = atmospheric pressure + pressure due to water lake
\ n l T1 = n 2T2 ,100 ´ 300 = n 2 ´ 500, : n 2 = 60 = p + 7p
Air escaped is 40%. The p = 10 m high, the 7p will be = 70 m high
9. (c) Pressure of gas = (Total pressure of gas - aqueous So the depth = 70 m
tension ) = 751-21 = 730 mm = 0.96 atm.
1 rSO2 P1 M He
10. (d) Moles of 0.5 litre of N 2 = ´ 0.5 = 0.02, 19. (a) =
22.4 rHe P2 M SO 2

moles of atoms = 0.04 (N2 is diatomic),


P1 4 P P1
1 = = 1 \ =4
Moles or atoms of He = = 0.04 P2 64 4P2 P2
22.4
11. (b) Apply 20. (a) Since b = 4 × volume occupied by molecules of 1 mol of a
gas
P1 P 1 2
= 2 ; = ;d = 1.43 kg m -3
d1T1 d2 T2 2.86 ´ 273 1092 ´ d æ4 3ö
b = 4 N A ç pr ÷
12. (c) When a gas is present above the inversion temperature, è3 ø
heating is observed.
13. (c) Find the value of volume at STP, 1 1
æ 3b ö3 é 3 ´ 24cm 3 mol -1 ù3
260 V ´101.325 ´ 103 \ r = çç ÷÷ =ê ú
´ 100.4 ´103 = , \ = 242.5cm3 23 -1
290 273 è 16 N A p ø ëê16(6.023 ´10 mol ´ 3.14) ûú
wt of 11200 cm3 is equal to vapour density
= 1.335 ´ 10 - 8 cm = 133.5 pm
0.160
\ V.D = ´ 11200 = 7.4 g Molecular diameter, d = 2r = 267 pm.
242.5
14. (d) One mole of a substance contains the number of m
21. (c) Mass of 1L water vapour = 0.6g, V = = 0.6cm 3
molecules which is independent of pressure. d
28p
15. (b) Density of N 2 at 25°C, d = and density of 1
R ´ 298 22. (d) The equation of state is PV = nRT \ n ® will not give
P
44p straight line.
O 2 at, 75°C, d =
R ´ 348 23 (a) If n is the total number of moles of gas and n1 moles are in
the larger shpere and n 2 moles in the smaller shpere.
rN 2 44p / R 348
= = 1.16 Then n = n1 + n2 and pV = nRT
rCO 2 28p / R 298
pV p¢ V p¢ V
2 = +
28 RT1 RT1 2RT2
16. (a) 28g of CO = = 1 mole; 2g H2 = = 1 mole
28 2
Since the temperature and moles are the same, the average 2pT2
kinetic energy will also be the same. p¢ =
2T2 + T1
States of Matter 143

1 1 V1
24 (d) U1 and U 2 But V2 = (given)
m1 m2 2
1
U 12
m
\ = 2 \ m1 U12 = m 2 U 22
2
2
U2 m1 æ ö
ç ÷
V
\ T2 = T1 ç 1 ÷ = T1 2
æ a ö ç V1 ÷
25. (a) van der Waals eqation is ç p + ÷(Vm - b ) = RT ç ÷
ç Vm2 ÷ è 2 ø
è ø
29. (b) Let the difference in two columns be h. Since the
V
Where Vm = = molar volume. At low pressure Vm is atmospheric pressure is more.
n
\ 60 = 74 – h
high and b can be ignored
or, h = 74 – 60 = 14 cm
æ a ö
\ç p + ÷Vm = RT or pVm + a = RT Thus Hg will stand in column B = 84.5 – 14 = 70.5 cm
ç Vm2 ÷ Vm
è ø 30. (a) The final volume will be V1 + V2 + V3 (Amagat’s law).

a pVm a 4 3
\ pVm = RT - or =1- 31. (b) The volume of the balloon = pr
Vm RT RTVm 3
a aP æ 1ö 3
or Z = 1 - = 1- çè since V µ ÷ø 4 22 æ 21 ö
RTVm RT P = ´ ´ ç ÷ = 4851 ml
3 7 è2ø
26. (d) The given equation is cubic equation in the variable V
and, therefore, for a single value of P and T, there should Volume of the cylinder = 2820 ml
be three values of V, all of which may be real or one real
and two imaginary. 20 ´ 2820 ´ 273
Volume of H 2 at NTP = ml
At Tc there values of V become identical. 300 ´ 1
1 = 51324 ml
27.(d) PV = NmU 2rms
3 After filling the cylinder will have H 2 equal to its volume
1 = 2820 ml
æ 3KT ö 2
\ U rms =ç ÷ \ Volume of H 2 for filling balloons = 51324 – 2820
è m ø
= 48504 ml
1 2 3
KE = mv = KT 51324
2 2 Hence no. of balloon to be filled = = 10
3
2820
28. (c) PV 2 = constant.
15 ´ 273 ´ 740
32. (a) 15 L (20°C, 740 mm) = = 13.61 L (NTP)
nRT 293 ´ 760
Again P =
V
13.61
3 70% N 2 = ´ 70 = 9.527 L (NTP)
nRT 100
\ ´V2 = constant (K)
V
28´ 9.527
1 9.527 L (NTP) of N 2 = = 11.91 g
K 22.4
or, TV 2 = = K' (constant)
nR
33. (b) p N 2 = r - rO2 = 90 - 63 = 27 cm
1 1
2 2
For two states, T1V1 = T2 V2 p N2 = mole fraction of N 2 ´ P

1 \ 27 = X N 2 × 90
2
æV ö
T2 = T1çç 1 ÷÷ \ X N 2 = 0.3
è V2 ø
144 Chemistry
34. (a) Let the distance be x metre.
48
Moles of dimer = = 0.5
R NH3 x M HBr 81 96
\ = = =
R HBr 1- x M NH3 17
1.5 ´ 0.0821 ´ (273 + 273)
\ P= = 2.00 atm
\ x = 0.6856 = 68.56 cm 33.6

4TR 3.2 39. (b) H 2 (g ) + Cl 2 ( g ) ¾¾® 2 HCl ( g )


35. (b) PV = and PV = (T + 50) R
M M Since the HCl is highly soluble in water and no gas is left,
the pressure in flask will be only due to water vapour at
4TR 3.2
\ = (T + 50) 27°C.
M M
40. (a) % Relative humidity
\ T = 200 K
Partial pressure of water in air ´100
2RT =
36. (b) V0 = = R (273 + t ) Aqueous tension
M
27 ´ 80
3(2t + 273 + 273)R \ pH 2 O in air = = 21.60 mm Hg
100
U rms =
1
41. (a) Since the ratio of He and O 3 atoms is 1 : 1. The ratio of
= 6( t + 273) R = 6 . V0
1
moles will be 1 : .
x x 60 x 3
37. (c) + = ,
32 28 32 ´ 28
1 4
Total moles = 1 + = . Total pressure P..
PV = nRT; 3 3
x x
P1V = RT ; P2 V = RT 1
32 28
Hence pO3 = 3 . P = 0.25 P
\ 32PO 2 = 28PN 2 4
3
or, PO2 = 0.875 N 2
42. (c)
38. (c) Wt. of compound is 96 g. The compound exists 43. (d) Molar volume of a gas or vapour (say at STP) is indepen-
as 50% monomer and 50% dimer. dent of molecular mass of the gas.
48 44. (d) Intermolecular H-bonding exists in ethanol unlike other
Moles of monomer = = 1, liquids, hence it has strongest intermolecular attractions
48
and highest viscosity.
6
Thermodynamics

It is the branch of chemistry which deals with the energy changes EXTENSIVE PROPERTIES :
taking place during physical and chemical changes. The properties which depend upon the quantity of the substance
Terminology used in thermodynamics :- or substances present in the system e.g. volume, enthalpy, free
energy, entropy, heat capacity.
SYSTEM :
A system is that part of universe which is under investigation . INTENSIVE PROPERTIES :
The properties which are independent of the quantity of the
SURROUNDINGS :
substance present in the system e.g. density, molar volume,
The part of the universe other than the system is known as temperature, melting point, boiling point, freezing point etc.
surroundings. Note – Any extensive property if expressed as per mole or per gm
Thus universe = system + surroundings becomes intensive property e.g. mass and volume are extensive
In simple case surroundings implies air or water both. properties but density is an intensive property.
TYPES OF SYSTEM : STATE VARIABLES AND STATE FUNCTIONS :
(i) Open system - A system which can exchange matter as well These are the macroscopic properties of the system which change
as energy with surroundings. with the change in the state of system. They depend upon the
(ii) Closed system – A system which can exchange energy and initial and final state of system. They are temperature, pressure,
not matter with surroundings. volume, chemical composition, energy, entropy, free energy.
(iii) Isolated system - A system which can exchange neither INTERNAL ENERGY, INHERENT ENERGY OR
matter nor energy with surroundings. HIDDEN ENERGY (IE) :
(iv) Homogeneous system – A system consisting of one phase It is the sum of all types of energies (as given below) associated
only e.g. pure solid, a liquid or a mixture of gases. with a system or substance.
(v) Heterogeneous system – It may consists of two or more (i) Translational energy of the molecules, Et
phases e.g. a solid in contact with liquid etc. (ii) Rotational energy, Er
STATE OF SYSTEM : (iii) Vibrational energy, Ev
(iv) Electronic energy, Ee
The condition of existence of a system when its macroscopic
(v) Nuclear energy, En
properties have definite values is known as state of system e.g.
(vi) Interaction energy of molecules, Ei
at 1 atm pressure H2O is (a) Solid below 0°C, (b) liquid between
E = Et + Er + Ev + Ee + En + Ei
0°C - 100°C and (c) gas above 100°C.
It is a state function, depends upon chemical nature of the
MACROSCOPIC PROPERTIES : substance, amount, temperature, pressure etc. Its absolute
The properties which arise out of collective behaviour of large value cannot be determined but change during a chemical
number of chemical entities e.g. pressure, volume, temperature, process can be determined.
composition, colour, refractive index etc. DE = Eproducts - E reactants = EP - ER
146 Chemistry
If E P > E R DE = +ve change is endothermic (iv) Isochoric process ( DV = 0) : Volume remains constant but
If E P < E R DE = - ve change is exothermic pressure changes.

Internal energy depends on temperature, pressure, volume O 2 + 2H 2 ® 2H 2 O pressure decreases


and quantity of matter. In most stable form the internal N 2O 4 ® 2 NO 2 pressure increases
energy of an element is zero.
(v) Cyclic process ( DE = 0) : After undergoing a number of
INTERNAL ENERGY CHANGE ( D E) :
changes, the system returns to its original state. In cyclic
It is the amount of heat evolved or absorbed when a chemical
process DE = 0; DH = 0; DS = 0
reaction is carried out at constant volume and constant
temperature. It is measured in bomb calorimeter. THERMODYNAMIC EQUILIBRIUM :
C ´ DT ´ M When there is no change in macroscopic properties with time.
DE = The types are :
W
(i) Thermal equilibrium - The temperature of the system
Where, C= Heat capacity of calorimeter remains constant throughout and no heat flows from one
DT = Rise in temperature part to another part of system.
M = Molecular mass of the substance (ii) Mechanical equilibrium - The pressure remains constant
W = Mass of the substance taken and no mechanical work is done by one part of system on
MODES OF THE TRANSFER OF ENERGY : another part.
Exchange of energy between system and surroundings can take (iii) Chemical equilibrium - The composition of various phases
place by remains the same.
(a) Heat (b) Work FIRST LAW OF THERMODYNAMICS :
(c) Electrical Energy (d) Radiations (i) Law of equivalence of different forms of energies
EXPRESSION FOR PV WORK: (ii) Energy can neither be created nor destroyed but can be
transformed from one form to another.
The mathematical expression for pressure-volume work
(iii) Total energy of universe is conserved (fixed, constant)
is WPV = - Pext. D V
MATHEMATICAL EXPRESSION OF FIRST LAW :
SIGN CONVENTIONS : If q is the amount of heat absorbed by the system and w is the
(i) Heat absorbed by the system is positive +ve work done on the system, the change in internal energy,
(ii) Heat evolved by the system is negative –ve ΔE = q + w
(iii) Work done on the system is positive +ve
ΔE = q + PΔV (work is pressure volume work)
(iv) Work done by the system is negative –ve
PROCESSES AND THEIR TYPES: If volume remains constant, no work is done then DE =q v
(i) Process – The operation by which a system changes from Note :- q and w are not state functions but the sum of the two (q
one state to another state is called process + w) which represents DE is a state function
(ii) Reversible process – If the driving force is only DISCUSSION OF EQUATION :
infinitesimally greater than the opposing force and process
can be reversed at any instant, it is known as reversible (i) when w = 0, DE =q v
process. If heat is absorbed by system, internal energy increases.
(iii) Irreversible process - It does not take place infinitesimally If heat is lost by system, internal energy decreases.
slow. (ii) when q = 0, DE = W
THERMODYNAMIC PROCESSES : If work is done on the system, internal energy increases.
(i) Isothermal process (DT = 0) : During each stage of process, If work is done by the system, internal energy decreases.
the temperature remains constant. If the process is In adiabatic process work is done by the system at the
exothermic heat is given out to surroundings, If the process expense of internal energy.
is endothermic heat is absorbed from surroundings.
(iii) when DE = 0 q = -w
(ii) Adiabatic process (dq=0) : No heat leaves or enter the
system, temperature of a system rises in exothermic process, Heat absorbed by the system is equal to work done by the
temperature falls in endothermic process. system
If we write w = -q
(iii) Isobaric process ( DP = 0) : Pressure remains constant,
only volume changes. If work is done on the system (when DE = 0 ) heat flows

e.g. O 2 + 2H 2 ® 2H 2O volume decreases from system to surroundings.


(iv) Cyclic process, DE = 0, q = –w
N 2O 4 ® 2 NO 2 volume increases Work done by the system is equal to heat absorbed.
Thermodynamics 147
SIGNIFICANCE OF DE : DE AND DH FOR MONOATOMIC GAS :
It represents the heat change taking place during the process For such a gas
occuring at constant volume and constant temperature.
3
q v = DE Internal Energy = KE = RT per mole and
2
ENTHALPY (H) AND ENTHALPY CHANGE (DH) :
Enthalpy is the heat content of the system and related to internal 3
Enthalpy = RT + PV
energy as follows. 2
H = E + PV
3 5
We can measure change in enthalpy and not absolute value of = RT + RT = RT per mole (Q PV = RT )
enthalpy. It is a state function. 2 2

DH = Hproducts – H Reactants= HP – HR APPLICATIONS OF FIRST LAW OF


THERMODYNAMICS :
If HP > HR, the change is endothermic. If HR > HP, the change is
exothermic. Hess’s law of constant heat summation - Law is based upon the
SIGNIFICANCE OF DH : first law of thermodynamics and states that if a chemical change
can be made to take place in two or more ways involving one or
The enthalpy H is given by
more steps, the net amount of heat change in the complete process
H = E + PV
is the same regardless of the method employed.
the change in enthalpy is given by when pressure is kept constant
DP = 0, DH = DE + D(PV ) = DE + PDV + VDP Q

DH = DE + PDV ; DH = q ( p) (From first law of thermodynamics) A D Q = q1 + q 2 + q 3


q1 B C q3
The enthalpy change represents the heat change taking place q2
during the process occuring at constant pressure and constant
temperature. Applications of Hess’s law
FACTORS AFFECTING THE DH (i) Calculation of heats of reactions
(i) Physical state of reactants and products (ii) Determination of heat changes of slow reactions
(ii) Quantities of reactants and products (iii) Calculation of enthalpies of formation
(iii) Allotropic forms (iv) Calculation of bond energies
(iv) Temperature WORK DONE IN ISOTHERMAL REVERSIBLE
(v) Conditions of constant P and Constant V at a particular EXPANSION (MAXIMUM WORK) :
temperature. It is given by the expression
KIRCHOFF’S EQUATION :
V2 P
If reaction takes place at constant pressure, the variation of DH Wmax = - 2.303nRT log = - 2.303nRT log 1
with temperature is given by this equation V1 P2

DH 2 - DH1 WORK DONE IN ADIABATIC REVERSIBLE


= DCP EXPANSION :
T2 - T1
It is given by the expression
DCp = Difference in the heat capacities of products and the
reactants at constant pressure. nR (T2 - T1 )
If the reaction takes place at constant volume. W= where is Poisson 's ratio which is
-1
DE 2 - DE1
= DC v Cp
T2 - T1 equal to and Cp – Cv = R
Cv
DCv = Difference in the heat capacities of products and the
If we put the value of we get the expression for workdone
reactants at constant volume.
STANDARD ENTHALPY : W = nCv (T2 - T1 )
The enthalpy change at the standard conditions i.e. at 1 atm Relation between temperature, volume and pressure in reversible
pressure and 298K temperature is called standard enthalpy of the adiabatic expansion
reaction and is denoted by DH° æ 1- ö
RELATION BETWEEN DH AND DE: -1 çè ÷ø
(I) TV = Const. (II) PV = Const. (III) TP = Const.
It is given by H = E + nRT , where
Dn = (Number of moles of products - number of moles of reactants) or T P1– = const.
gaseous only.
148 Chemistry
Work done in adiabatic irreversible expansion MOLAR HEAT CAPACITY FOR POLYATOMIC GAS AT
w irr = -Pext DV = -Pext (V2 - V1 ) CONSTANT VOLUME :
3
é RT2 RT1 ù R [ P1T2 - P2 T1 ] Cv = R+X
= - Pext ê - ú = - Pext 2
ë P2 P1 û P1 P2
MOLAR HEAT CAPACITY FOR POLYATOMIC GAS AT
During expansion in vacuum
CONSTANT PRESSURE :
Pext = 0, work done = 0, DE = w = 0 also DT = 0 and DH = 0
5
If V2 > V1 then Wirr = - ve i.e. expansion work or work done by Cp = R+X
2
system. For monoatomic gas X=0, for diatomic gas X=R and for triatomic
If V2 < V1 then Wirr is +ve i.e. compression work or work done
3
on the system. gas X = R
2
HEAT CAPACITY OF A SYSTEM :
LIMITATION OF FIRST LAW OF
It is the amount of heat required to raise the temperature of the
THERMODYNAMICS:
system by 1 degree (K or °C)
It fails to predict the feasibility and the direction of the change
q
C= . SPONTANEOUS PROCESS :
m(T2 - T1 )
A process which has an urge or a natural tendency to occur
In case of single substance, when m=1 g, C is the specific heat of either of its own or after proper initiation under a given set of
system; when m = Mg or 1 mole, it is molar heat capacity. Since conditions. It is irreversible process and may only be reversed
heat capacity varies with temperature its true value is given by by some external agents.
dq (i) Spontaneous processes that need no initiation
C= (differenti al equation ) ; (a) Evaporation of water H2O (l) ® H2O (g)
dt
(b) Dissolution of sugar
dE æ E ö C12H 22O11 (s) ® C12 H 22O11 (aq.)
Heat capacity at constant volume C v = =ç ÷
dt è T ø v
(c) Intermixing of gases
Heat capacity at constant pressure (ii) Spontaneous processes that need initiation
dq dH æ H ö Electric, discharge
(a) 2 H 2( g ) + O 2(g ) ¾¾ ¾ ¾ ¾ ¾¾® 2H 2 O (l )
Cp = = =ç ÷
dt dt è T ø p
(b) Electrolysis of water 2H 2O(l ) ® 2H 2 (g) + O2 (g)
Also C p = c p ´ M and C v = c v ´ M It requires continuous supply of energy.
where c P &c v are specific heats at constant pressure and SECOND LAW OF THERMODYNAMICS :
constant volume respectively. The entropy of the universe increases in every spontaneous
(natural) change. The entropy of the universe is continuously
C P & C v are molar heat capacities at constant pressure and
increasing.
constant volume respectively. Other statements of second law of thermodynamics
Dulong and Petit’s rule : Atomic mass × specific heat = 6.4
(i) Clausius : Heat cannot itself pass from a colder body to
The rule is valid for solid elements except Be, B, Si and C. a hotter body.
Units : In SI units the specific heat or molar heat is expressed in (ii) Kelvin : It is impossible to get a continuous supply of work
terms of Joules ( 1 Cal = 4.184 J)
from a body or engine which can transfer heat from a single
MOLAR HEAT CAPACITY OF A MONOATOMIC GAS heat reservoir.
AT CONSTANT : (iii) Planck : It is impossible to construct a device which will
3 work in a single complete cycle and convert heat into
volume C v = R work without producing any change in the surroundings.
2
(iv) All natural and spontaneous processes take place in one
MOLAR HEAT CAPACITY OF A MONOATOMIC GAS direction and thus are irreversible in nature.
AT CONSTANT PRESSURE : (v) In a reversible process the entropy of the universe is
5 constant. In an irreversible process the entropy of the
Cp = R universe increases.
2
Thermodynamics 149
ENTROPY (S) : CRITERIA FOR FEASIBILITY OR SPONTANEITY OF
It is a measure of degree of disorder or randomness in a system. A PROCESS :
More the disorder or randomness, the more is entropy.Heat added Enthalpy or entropy alone cannot predict the spontaneity of a
to a system increases randomness and hence the entropy. Heat change. Gibb’s free energy is very useful factor for this
added to a system at lower temperature causes greater randomness G = H – TS
than the system at higher temperature. At higher temperature the DG = DH – TDS (at constant T)
system already has higher randomness. Thus entropy change is DSTotal = DSsys + DSsurr > 0
inversly proportional to temperature. DH surr DH sys
DS surr = =- since
qrev T T
DS = Heat lost by system = Heat gained by surroundings and
T
vice versa.
For a spontaneous process
DSTotal = DSsystem + DSsurr > 0 æ DH sys ö
DSTotal = DSsys + çç - ÷
or DSuniverse >0
è T ÷ø
TDSTotal = TDSsys – DHsys
UNITS OF ENTROPY : = –(DHsys – TDSsys)
Jmol -1K -1 ; It is a state function. The change in entropy is = –(DG)
Hence,
represented by (i) If DG is negative, the process is spontaneous
DS = (Sfinal – Sinitial) = qrev / T (ii) If DG is zero, the process is in equilibrium
qrev is the heat absorbed by the system in a reversible manner (iii) If DG is Positive the process does not occur
and isothermally.
STANDARD FREE ENERGY CHANGE (DG°) AND
ENTROPY CHANGE FOR AN IDEAL GAS UNDER EQUILIBRIUM CONSTANT (K) :
DIFFERENT CONDITIONS: WITH CHANGE IN P, V They are related as follows DGº = –2.303 RT log K
AND T :
FREE ENERGY CHANGE AND CELL POTENTIAL :
T2 V DGº = - nFEº
DS = C V ln + R ln 2 (when T and V are two variables)
T1 V1 VARIATION OF FREE ENERGY WITH
TEMPERATURE AND PRESSURE IN A REVERSIBLE
T2 P
DS = C P ln + R ln 1 (when T and P are two variables) PROCESS :
T1 P2
dG = VdP - SdT
V2 P The equation is called the total differential equation.
For isothermal process (DT = 0), DS = R ln = R ln 1 (a) If temperature is kept constant dT=0 we have
V1 P2
æ Gö
T
(dG )T = (VdP )T or ç ÷ =V
For isobaric process (DP = 0), DS = C P ln 2 è P øT
T1 (b) If pressure is kept constant dP = 0 we have
æ Gö
For isochoric process (DV = 0), DS = CV ln
T2 (dG )P = - (SdT )P or ç ÷ = -S
T1 è T øP

CHANGE IN FREE ENERGY FOR ISOTHERMAL


GIBB’S FREE ENERGY :
PROCESS :
(G) - It is the maximum amount of energy which can be converted
P2 V
into the useful work (other than PV work). It is related to enthalpy (DG )T = nRT ln = nRT ln 1
(H) and entropy (S) as follows. P1 V2

G = H - TS and change is given by CRITERIA OF FEASIBILITY OR SPONTANEITY IN


TERMS OF :
DG = DH - TDS
-DG = Wmax - PDV = Useful work (i) Entropy : DS system + DS surrounding > 0 or DS total > 0

HELMHOLTZ FREE ENERGY (WORK FUNCTION) DS total = 0 , the process is reversible and system is in
(A): The maximum work obtainable from a system is given equilibrium when internal energy and volume are kept
by this energy. constant
( DS ) E, V > 0 the process is irreversible and if ( DS) E ,V = 0 ,
A = E - TS ; E= Internal energy, S = Entropy, T= Temperature
the process is reversible.
150 Chemistry
(ii) Internal energy : When entropy and volume kept constant (i) For water (liquid ) = water (vapour) we have
If (DE) S, V < 0 , the process is irreversible and if dP DH V
=
( DE) S, V = 0 , the process is reversible. dT T(Vg - Vl ) ;
(iii) Enthalpy (H) : When entropy and pressure are kept constant DH V = molar heat of vaporisation ,
( DH) S,P < 0 , the process is irreversible and if (ii) For water (Solid, Ice) = water (liquid)
( DH) S,P = 0 , the process is reversible. dP DH f
= ;
(iv) Work function : (A) when temperature and volume are kept dT T (Vl - Vs )
constant
DH f = molar heat of fusion of ice
( D ) T ,V < 0 , the process is irreversible and if (iii) For solid = vapour equilibrium
( DA) T ,V = 0 , the process is reversible. dP DH s
=
(v) Free energy (G) : When pressure and temperature are kept dT T(Vg - Vs ) ; DH s = molar heat of sublimation
constant
INTEGRATED FORM OF CLAPEYRON - CLAUSIUS
( DG ) P ,T < 0 , the process is irreversible and if
EQUATION :
( DG ) P,T = 0 , the process is reversible. For Liquid = gas system in Equilibrium
NOTE : Criteria of feasibility or spontaneity in terms of free P2 DH v é T2 - T1 ù
energy is most important since most of the processes are carried log = ê ú
P1 2.303R ë T1T2 û
out at constant temperature and pressure.
ZEROTH LAW OF THERMODYNAMICS : APP LICATIONS OF CLAPE YRON C LAUSIUS
If two bodies have separately equality of temperature with a
EQUATION :
third body, they also have equality of temperature with each (i) Calculation of molar heat of vaporisation
other. (ii) Effect of temperature on vapour pressure of a liquid
(iii) Effect of pressure on boiling point
THIRD LAW OF THERMODYNAMICS :
(iv) Calculation of molal elevation constant (kb) and molal
At absolute zero temperature, the entropy of a perfectly crystalline depression constant (kf ) of a solvent.
substance is taken as zero. This law was formulated by Nernst in
1906. 0.002T 2
kb =
Calculation of absolute value of entropy: lv
Let S0 be the entropy of substance at 0 K and S be its entropy at lv= latent heat of vaporisation per gram of solvent
T K. T= boiling point of pure solvent
T
Cp 0.002T2
DS = S0 - S = dT kf =
T lf
0
T= freezing point of solvent
Where Cp is the heat capacity of the substance at constant
lf = latent heat of fusion per gram of solvent
pressure. According to the third law
HEAT ENGINE :
T
Cp Machine which converts heat into work is called heat engine.
S0 = 0, thus S = dT .
T EFFICIENCY OF HEAT ENGINE :
0
The fraction of the heat absorbed, converted into work is called
The value of the integral can be obtained from a plot of the efficiency of machine.
CP / T Vs T. The area under the curve between 0 and T K gives the
W Work done
value of the integral and hence of S at temperature T. h= =
Q Heat absorbed
CLAPEYRON - CLAUSIUS EQUATION :
CARNOT CYCLE :
It gives the change in pressure dP accompanying change in
temperature dT or vice versa for a system containing two phases It is a process where a system after undergoing a number of
of a pure substance in equilibrium, q is heat exchanged reversibly successive changes returns to its original state. It consists of
per mole of the substance during the phase transformation at four different operations.
temperature T. (i) Isothermal expansion
(ii) Adiabatic expansion
dP q (iii) Isothermal compression
=
dT T (VB - VA ) (iv) Adiabatic compression
Thermodynamics 151
EFFICIENCY OF CARNOT CYCLE OR ENGINE : TYPES OF REACTIONS AND CORRESPONDING
ENTHALPY CHANGES :
W T -T Q - Q1
h= = 2 1= 2 (i) Heat of reaction at constant volume and certain temperature
Q T2 Q2
It is defined as the change in internal energy ( DE ) of the
system when requisite number of molecules of reactants
T2 - T1
Since is always less than unity, hence efficiency
T2 react to form the products. ( DE ) = E p - E r = q v = heat of
reaction at constant volume
is always less than unity
(ii) Heat of reaction at constant pressure and certain
CARNOT THEOREM : temperature
Efficiency of reversible heat engine is independent of the nature
It is defined as the difference in enthalpies ( DH ) of products
of working substance and depends upon the temperature of
source and sink. and reactants DH = H p - H r = q p = heat of reaction at
Or constant pressure
All machines working reversibly between the same temperature (iii) Relation between heat of reaction at constant volume ( q v )
of source and sink have same efficiency.
RESONANCE ENERGY : and at constant pressure ( q p )

(Expected value of DH°f ) – (Observed value of DH°f ) DH = DE + PDV


(iv) Sign of DH and DE : Negative sign of DH or DE shows
Example - Calculate the resonance energy of N2O from the
that heat is evolved and reaction is exothermic while a
following data
positive sign of DH and DE shows that heat is absorbed
DH°f of N2O = 82 kJ mol–1, Bond energies of N º N, N=N, O=O and reaction is endothermic.
and N=O bands are 946, 418, 498 and 607 kJ mol–1 respectively (v) Factors affecting the heat of reaction
(a) Physical state of the reactants and products
Sol. Calculated value of D H fo of N 2 O
(b) Amount of the reactants
1 (c) Temperature
N 2 (g) + O2 (g) ® N 2O(g) (d) Pressure
2
(vi) Thermochemical equation
.-. + .. Stoichiometrically balanced, mentioning the physical state
1
N º N(g) + ( O = O ) (g) ® N. . = N = O. . (g) of the reactants and products and showing the values of
2
DE and DH equation is called thermochemical equation
DH f0 = [Energy required for breaking of bonds ] (vii) Heat of formation ( DH f ) : It is defined as the enthalpy
- [Energy released for forming of bonds ] change DH accompanying the formation of one mole of the
compound from its elements.
1 2C + H 2 ® C 2 H 2 ; DH = 53.140 kcal
= (DH( N º N ) + DH ( O = O ) - (DH N = N + DH N = O )
2
(viii) Standard heat of formation DH of : It is the enthalpy change
1
= (946 + ´ 498) - (418 + 607) = 170 kJ mol -1 accompanying the formation of one mole of a compound
2
from its elements. All substances being in their standard
Resonance energy = 170 - 82 = 88 kJ mol-1 states. The condition of standard state is 25ºC and 1 atm
pressure.
CALORIFIC VALUE : (ix) Heat of combustion : It is the change in enthalpy of the
The heat liberated on burning unit mass of a fuel is called calorific system, when 1 gram mole of the substance is completely
value. oxidized

Heat of combustion CH 4 (g) + 2O 2 (g) ® CO 2 (g) + 2H 2O ; DH = -890.3kJ


Calorific Value =
Molar mass of fuel The combustion is always exothermic process.
(x) Application of heat of combustion
The more the calorific value the more the efficient is the fuel.
(a) Calorific value of fuel
Oxygen atoms present in a molecule reduce the calorific value.
(b) Calculation of heat of formation
The more the number of hydrogen atoms per carbon atom, the
(c) Constitution of organic compounds
more is the calorific value.
152 Chemistry
(xi) Heat of neutralization : It is defined as the enthalpy change In polyatomic molecules the bond dissociation energy is
( DH ) accompanying the neutralization of one gram not the same for successive bonds though the bonds are of
equivalent of the acid by a base in dilute solution at that the same type.
temperature.
H3C - H = +425 kJ mol-1 ;
HCl (aq) + NaOH(aq) ® NaCl(s) + H 2O(aq);
H 2 C - H = +470 kJ mol-1
DH = -57.32kJ
Heat of neutralization of every strong acid by a strong base HC - H = +416 kJ mol-1 ;
is identical. When acid or alkali is weak the heat of
neutralization is different because the reaction involves the C - H = +335 kJ mol-1
dissociation of the weak acid or the weak alkali.
Heat of neutralization of weak acids with NaOH at 25°C 425 + 470 + 416 + 335
Hence C - H Bond energy =
Acid Heat of Heat of 4
neutralisation dissociation = 411.5kJ mol-1
(kJ per g eq.) (kJ per g eq.)
Acetic acid –55.23 +2.09 (xvii) Use of bond enthalpy :
Formic acid –56.06 +1.26 (1) Calculating standard enthalpy of reactions
Hydrocyanic acid –12.13 +45.19
DH =[sum of bond energies of reactants]
Hydrogen sulphide –15.90 +41.42
– [sum of bond energies of products]
(xii) Heat of solution : The enthalpy change per mole of solute
= [ Total energy required for breaking the bonds]
when it is dissolved in large excess of solvent (generally
– [Energy given out in forming the bonds]
H2O) so that further dilution of the solution produces no
(2) Calculation of bond energy of some specific bond in the
heat exchange.
molecule
KCl + aq ® KCl(aq) ; DH = 4400 cal (18.58kJ)
THE LAWS OF THERMOCHEMISTRY :
MgSO 4 + aq ® MgSO 4 (aq) ; DH = -20280 cal (1) Lavosier and Laplace law : The heat evolved or absorbed
(xiii) Heat of dilution : The enthalpy change when a solution in the course of a chemical reaction is equal to the heat
containing one mole of a solute is diluted from one absorbed or evolved when the reaction is reversed.
concentration to another.
1 1
(xiv) Heat of hydration : The enthalpy change when one mole of H 2 (g) + Cl 2 (g) ® HCl(g) ; DH = - 92.0 kJ
2 2
a substance combines with the required number of moles of
1 1
water to form a specified hydrate. HCl(g) ® H 2 (g) + Cl 2 (g) ; DH = + 92.0 kJ
Heat of hydration = Heat of solution of the hydrated 2 2
compound – Heat of solution of the unhydrated compound (2) Hess’s law of constant heat summation
(xv) Heat of precipitation : The enthalpy change when one mole TROUTON’S RULE :
of a sparingly soluble substance precipitates on mixing dilute
Fredrick Trouton (1884) found that for many liquids molar heat of
solutions of stable electrolytes is called heat of precipitation.
vaporization in calories per mole divided by the normal boiling
BaCl 2 (aq.) + Na 2SO 4 (aq.) ¾
¾® BaSO4 (s ) + 2 NaCl(aq.) ; point in degree kelvin is a constant.
DH = -19.50 kJ DH v
= 21 cal / kmol = 88 J / K mol
(xvi) Bond enthalpy : It is defined as the average amount of energy Tb
required to break all the bonds of a particular type in one
DH v
mole of the substance. It is also known as bond energy and The quantity is also called entropy of vaporization DSv.
expressed in kJ mol–1. Tb
In a diatomic molecule the bond energy and bond ENDOTHERMIC COMPOUNDS :
dissociation energy are the same. The compounds having positive values of formation are called
endothermic compounds. Ozone is an example.
Thermodynamics 153

1. Six moles of an ideal gas expand isothermally and reversibly Sol. Work done is irreversible as at constant pressure,
from a volume of 1litre to a volume of 10 litres at 27º C. What
DV = 5 – 3 = 2 dm3 = 2 lit.
is the maximum work done? Express the results in various
units. W = – PDV = – 3 × 2 = – 6 atm. lit. = – 6 × 101.3J
Sol. Work done = 607.8J
This work is used for heating water W = m × s × DT
V2 V
W = nRT ln = 2.303nRT log 2 607.8 = 10 × 18 × 4.184 × DT
V1 V1
Therefore DT = 0.807
10 Hence final temperature = 290 + 0.807 = 290.807 K
= 2.303 ´ 6 ´ 0.0821 ´ 300log = 340.3 lit. atm. 5. 10 g of argon gas is compressed isothermally and reversibly
1
at a temperature of 27º C from 10 litres to
If we put the value of
5 litres. Calculate q, w, DE and DH for this process. R=2.0
R= 8.314 J K–1 mol–1 W = 34464.8J cal K–1 mol–1 log 2 = 0.3010
Atomic weight of Argon = 40
R = 8.314 ´10 7 ergs K -1mol -1 W = 3.44 ´1011 ergs
R = 1.987 cal K -1 mol -1 W = 8236.91 cal V2 10 5
Sol. W =- 2.303nRT log =- 2.303 ´ ´ 2 ´ 300 log
V1 40 10
2. A system performs 101.3J of work on its surroundings and
absorbs 15 kJ of heat from surroundings. What is the change W = 103.635 cal
in internal energy of the system. For isothermal proccess DE = 0, q = –W = – 103.635 cal
Sol. Work done by the system is taken as negative DH = 0
W = –101.3J 6. Show that in an isothermal expansion of an ideal gas
Heat absorbed by the system is taken as positive (a) DE = 0 and (b) DH = 0
q = 15kJ = 15000 J Sol. (a) For one mole of a gas Cv= [ E / T]v
DE = q+w ( First law of thermodynamics) For finite change DE = Cv DT
=15000J-101.3J= 14898.7J For isothermal process DT = 0 therefore DE = 0
3. One mole of liquid water at its boiling point vapourises (b) We know that DH = DE + D( PV) = DE + D( RT)
against a constant external pressure of 1 atm. at the same
(PV = RT)
temperature. Assuming ideal behaviour and initial volume
of water vapours as zero, calculate the work done by the Therefore DH = DE + RDT. Since DT = 0, DE = 0, therefore
system. DH = 0.
7. Calculate the change in free energy at 25º C for the reaction
Sol. For ideal behaviour of water vapour PV = nRT can be applied
to find the volume. 1
CO(g) + O2 (g) ® CO2 (g) DH = -67.37 kcal
1 × V = 1 × 0.0821 × 373 2
V = 30.62 lit. and the change in entropy accompanying the process is –
DV = V2 - V1 = 30.62 lit. – 0 lit. = 30.62 lit. 20.7 cal deg–1 mol–1
Sol. DG = DH – TDS
work done = – PDV = –1 × 30.62 = – 30.62 lit atm.
= – 67.37 – 298 (–20.7 × 10–3)
= – 30.62 × 101.3J = -3101.8 J
= -61.2014 kcal mol -1
4. A gas expands from 3 dm3 to 5 dm3 against a constant
pressure of 3 atm. The work done during expansion is used 8. Calculate DH, DS, DG and DE when 1 mole of water is
to heat 10 mole of water of temperature 290 K. Calculate the vapourised at 100º C and 1 atm pressure. The latent heat of
final temperature of water. Specific heat of water is vapourisation of water is 540 cal g–1
4.184 Jg–1 K–1
154 Chemistry
Sol. (i) 1 mole of water is 18 g therefore enthalpy change (i) Latent heat of fusion for 900 g ice
DH = 18 × 540 cal = 9720 cal
900 ´ 5.99
(ii) Entropy change DS = DH/ T = 9720/373 DH fus = kJ = 299.5kJ
18
= 26.06 cal K–1
(iii) Free energy change DG = DH – TDS DH fus 299.5 ´1000
DS = = = 1097 JK -1
DG = 9720 – 373 × 26.06 = 0 T 273
(iv) DH = DE + PDV DH fus
(ii) DS for freezing of liquid water = -
therefore DE = DH – PDV = DH – P(V2 – V1) T
V1 = Volume of 1 mole of liquid water = 18 cm3 when liquid water freezes heat is evolved
V2 = Volume of 1 mole of steam at 373 K 5.99 ´ 1000
DS = - = - 21.94 Jmol -1K -1
Applying ideal gas equation, 273
11. Calculate the entropy change in isothermal reversible
P1V1T2 1´ 22400 ´ 373 expansion of 5 moles of an ideal gas from a volume of 10
V2 = =
P2 T1 1´ 273 litres to a volume of 100 litres at 300 K.
Sol. For isothermal process T1 = T2 hence the equation
V2 = 30600 cm3 T2 V
DS = C v ln + nR ln 2 is reduced to
DV= V2 – V1 = (30600 – 18) cm3 = 30600 cm3 T1 V1
(18 negligible) V2 V
3 DS = nR ln = 2.303nR log 2
PDV =1atm ´ 30600 cm = 1 ´ 30.6lit. = 30.6 lit atm. V1 V1
Given n = 5,
30.6 × 24.2 cal = 740.5 cal (1atm lit = 24.2 cal)
V1 =10 litres, V2 = 100 litres, R = 8.314JK -1mol -1
Hence DE = DH – PDV = 9720 – 740.5 = 8979.5 cal
100
9. A certain volume of dry air at NTP is expanded reversibly to DS = 2.303´ 5 ´ 8.314 log = 95.75JK -1
three times its volume (a) isothermally (b) adiabatically. 10
Calculate the final pressure and temperature in each case, 12. Calculate the temperature at which the reaction
assuming ideal behaviour. (Cp/Cv for air is 1.4) 1
Ag 2 O(s) ® 2Ag(s) + O 2 (g) at 1 atmospheric pressure
Sol. Let V1 be the initial volume of dry air at NTP 2
(a) Isothermal expansion. Since will be in equilibrium. The value of DH and DS for the
reaction are 30.58 kJ and 66.11JK–1 respectively and these
DT = 0 hence P1V1 = P2 V2 value do not change much with temperature.
Sol. DG = DH - T DS for equilibrium DG = 0
P1V1 1´ V1
P2 = = = 0.333 atm
V2 3V1 DH DH 30580
DS = T= = = 462.6 K
T DS 66.1
(b) Adiabatic expansion we have 13. Calculate the standard entropy change for the reaction
-1 1.4 -1 A B if the value of DHº = 28.40kJ mol -1 and
T1 é V2 ù 273 é 3V1 ù
=ê ú or =ê ú equilibrium constant is 1.8 × 10-7 at 298K
T2 ë V1 û T2 ë V1 û
Sol. DGº = - 2.303RT log K
c

T2 = 176K = - 97o C = - 2.303´ 8.31JK -1 mol-1 ´ 298 log1.8 ´10 -7


Final pressure under adiabatic conditions = 38466J mol -1
DHº -DGº 28400 - 38466
P1 é V2 ù 1 é 3V1 ù
1.4 DSº= = = - 33.78 JK -1 mol-1
=ê ú =ê T 298
ú P = 0.217 atm
P2 ë V1 û P2 ë V1 û 14. Calculate the equilibrium constant for the following reaction
at 400 K
10. The latent heat of fusion of ice is 5.99kJ/mol at its melting
point. Calculate. 2NOCl(g) ® 2NO(g) + Cl 2 (g)
(i) DS for fusion of 900g ice D Hº = 77.2 kJmol -1 ; D Sº = 122 JK -1mol -1 at 400K
(ii) DS for freezing of liquid water Sol. DGº = DHº -TDSº

273K = 77.2 ´ 103 - 400 ´ 122


Sol. H 2 O(s) H 2 O(l )
= 28.4 ´ 103 J mol -1
Thermodynamics 155
DGº = - 2.303RT log K c 18. The heats of combustion of C2 H 4 ( g ) , C2 H 6 ( g )
3
DGº -28.4 ´10
log k c = - = = -3.708 and H 2 ( g ) are –1409.5, –1558.3 and –285.6 kJ respectively..
2.303 ´ RT 2.303 ´ 8.314 ´ 400
taking antilog, we get Calculate the heat of hydrogenation of ethylene.
-4 Sol. The desired equation is
K c = 1.995 ´10
1 C 2 H 4 (g) + H 2 (g) ® C 2 H 6 (g)
15. Show that the reaction CO(g) + O 2 (g) ® CO 2 (g) at 300
2 DH = Heat of combustion of reactants
K is spontaneous and exothermic, when the standard entropy – Heat of combustion of products
change is – 0.094 kJ mol–1 K–1. The standard Gibb’s free
= – 1409.5 + (–285.6)–(–1558.3) = –136.8 KJ
energies of formation of CO2 and CO are – 394.4 and –137.2
KJ mol–1 respectively. 19. At 300 K the standard enthalpies of formation of
Sol. The given reaction is C6 H5COOH (s), CO2 (g), and H 2 O (l ) are r e s p e c t i v e l y
1 -1
CO (g) + O 2 (g) ® CO 2 (g) - 408, - 393 and - 286 kJ mol . Calculate the heat of
2
DG ° (for reaction) = G° products – DG ° reactants combustion of benzoic acid at (i) Constant pressure (ii)
Constant volume
= G°CO2 - (G°CO + G°O2 )
(R = 8.314 J mol-1K -1 )
= - 394.4 - (-137.2 + 0) = - 257.2
DGº = DHº - TDSº Sol. DH = Enthalpies of formation of products
- Enthalpies of formation of reactants.
-257.2 = DHº + 298 (0.094) or DHº = -285.2KJ
Since DGº is negative the process is spontaneous and DHº Desired equation is
is also negative the process is exothermic. 15
16. The heat of combustion of gaseous methane CH4 at constant C6H5 COOH(s) + O2 (g) ® 7CO2 ( g) + 3H2O (l )
2
volume measured in bomb calorimeter at 298.2 K is found to
be – 885389 J/mol. Find the value of enthalpy change. DH = 7 ( -393) + 3(-286) - ( -408 - 0) = - 3201kJ
Sol. CH 4 (g) + 2O 2 (g) ® CO 2 (g) + 2H 2 O (l)
Also DH = DE + DnRT
Given DE = - 885389 J mol -1
or –3201 kJ = DE + (-0.5) ´ 8.314 ´10 -3 ´ 300
Dn =1 - (1 + 2) =- 2 T = 298.2 K; R =8.314 JK -1mol-1
D E = - 3199.75 kJ
DH = DE + DnRT
20. Given that the bond enthalpies for C– H, Cl –Cl, C – Cl and
= -885389 + (-2) 8.314 ´ 298.2 H – Cl are respectively 99, 58, 78 and 103 k cal / mol, determine
DH for the reaction
= -890.34 kJ
For solving the heat of reaction follow the following rules CH 4 (g) + Cl2 (g) ® CH3 Cl ( g ) + HCl(g)
(i) When standard enthalpy of formation of compounds
are given, apply the equation Sol. DH (reaction) = Bond energy of (bond broken in reactants
– bond formed in products).
DH = DHº f (products) – DH º f (reaction).
(ii) When heat of combustion of compounds are given apply = Bondenergyof [ (C - H + Cl - Cl) - (C - Cl + H - Cl) ]
the equation =[ (99 + 58) - (78 + 103)] =- 24kcal
DH = DH comb of reactants – DH comb of products.
21. Estimate the S – F bond energy in SF6. The standard heat of
17. The molar heats of combustion of C2 H 2 (g) , C(graphite) formaton values of SF6(g), S(g) and F(g) are –1100, 275 and
and H2 (g) are – 310.62, – 94.05 and – 68.32 kcal respectively. 80 kJ mol–1 respectively.
Calculate the heat of formation of C2H2.
Sol. The desired equation is Sol. SF (g) ® S (g) + 6F (g)
6
DH = (DH S + 6DH F ) - DH SF
2 C (graphite) + H 2 (g) ® C 2 H 2 (g) 6
= 275 + 6 ´ 80 - ( -1100) =1855
DH = Heats of combustion of reactants
- Heats of combustion of products 1855
Average bond energy = = 309.17 kJ
6
= 2 (-94.05) + (-68.32) - (-310.62) = 54.2 kcal
156 Chemistry
22. The heat of reaction (DH) for the formation of water at 25°C
Sol. W = q(T2 - T1 ) = 1897.86 ´ 373 - 283 = 457.92 kJ
is – 68.4 kcal. Find the value at 90°C. The moler heat capacities T2 373
of H2(g), H2O (l ) are 6.62, 6.76 and 18 cal respectively..
28. Calculate DH at 358 K for the reaction
Sol. DH2 - DH1 = DCp (T2 - T1 )
Fe 2O 3 (s) + 3 H 2 (g) ¾
¾® 2Fe(s) + 3H 2 O(l)
DCp = molar heat capacities of products
Given that DH 298 = – 33.29 kJ mol-1 and Cp for Fe 2O 3 (s),
- molar heat capacities of reactants
Fe(s), H 2 O (l) and H 2 (g) are 103.8, 25.1, 75.3 and 28.85 J/K
1
The formation of water is H 2 + O 2 ® H 2 O mol.
2
Sol. DCp = (2 × 25.1 + 3 × 75.3) – (103.8 + 3 × 28.8)
1
DC p =18 - (6.62 + (6.67) = 8 cal = 8 ´10 -3 k cal = 85.9 J/Kmol
2
DH 2 =- 68.4 + 8 ´10-3 (363 - 298) = - 67.88k cal DH 2 - DH1
= DCp ( Kirchoff ’s equation)
23. The heat of solution of MgSO4 and MgSO4.7H2O are T2 - T1
– 20.3 and 3.8 kcal respectively. Find heat of hydration of
DH 358 - (-33290)
MgSO4. = 85.9
358 - 298
Sol. Heat of hydration = Heat of solution of anhydrous salt
DH 358 = – 28136 J/mol
- Heat of solution of hydrated salt.
= – 28.136 kJ/mol
= -20.3 - (3.8) = - 24.1k cal.
29. Ka for CH 3COOH at 25°C is 1.754 × 10 - 5 at 50°C. Kais
24. Integral heat of solution (DH) for 1 mole of KCl dissolved
in 20 moles of water is + 15.90 kJ. When 1 mole is dissolved 1.633 × 10 - 5 . What are DH° and DS° for the ionisation of
in 200 moles of water, DH is 18.58 kJ. Calculate heat of CH 3COOH ?
dilution.
Sol. (DG°) 298 = – 2.303 RTlog Ka
Sol. KCl (s) + 20H 2 O ® KCl (20 H 2 O) DH1 = 15.90 kJ
= – 2.303 × 8.314 × 298 × log 1.754 × 10 - 5
KCl (s) + 200H 2 O ® KCl (200H 2 O) DH 2 = 18.58 kJ
= – 27194 J
Heat of dilution = DH 2 - DH1 = 18.58 - 15.90 = 2.68 kJ -5
25. At what temperature will water boil when the atmospheric (DG°)323 = – 2.303 × 8.314 × 323×log 1.623 × 10
pressure is 528 mm Hg. Latent heat of vaporisation of water = –29605 J
is 545.5 cal g–1 DG° = DH° – T DS°
Sol. The integrated form of Clapeyron equation is 27194 = DH° – 298 DS°
P2 DH v é 1 1ù 29605 = DH° – 323 DS°
log = ê - ú
P1 2.303R ë T1 T2 û DH° = – 1.55 KJ/mol
DS° = – 96.44 J/mol
Here P1 = 528 mm Hg; P2 =1atm = 760mm Hg; T1 = ?; 30. Calculate the heat of neutralisation from the following data
T2 = 100°C = 373K 200 ml of 1 M HCl is mixed with 400 ml of 0.5 M NaOH. The
temperature rise in the calorimeter was found to be 4.4°C.
DH v = 545.5 cal g -1 = 545.5 × 18 = 9819 cal mol–1 Water equivalent of calorimeter is 12 g and specific heat is 1
cal/ml/degree for solution
760 9819 é1 1 ù
log = ê - ú Sol. When HCl and NaOH are mixed (neutralised) the heat
528 2.303 ´ 1.987 ë T1 373 û or T1 = 363 K
produced is taken up by calorimeter and solution in it
26. Calculate the maximum efficiency of a steam engine working \ DH1 = m1s1DT + m 2s 2 DT
between 110°C and 25°C. = 12 × 1×4.4 + 600 × 1 × 4.4
Sol. Efficiency of a heat engine is given by (Total sol. = (200 + 400) ml
T - T1 383 - 298 = 2692.8 cal
h= 2 or h = = 0.222 or 22.2%
= – 2692.8 cal
T2 383
This is neutralisation of 200 Meq
27. Calculate the useful work done by the heat engine which Neutralisation of 1000 Meq. or one equivalent
works between 10°C and 100°C. Heat supplied to engine is = – 2692.8 × 5 = – 13464 cal
1897.86 kJ = – 13.464 kcal
Thermodynamics 157

Very Short/ Short Answer Questions (b) The entropy of a substance increases in going from
the liquid to the vapour state at any temperature.
1. Predict DH > DU or DH < DU or DH = DU.
(c) Reaction with DG° < 0 always have an equilibrium
(a) C (graphite) + O2 (g) ® CO2(g) constant greater than 1.
(b) PCl5 (g) ® PCl3(g) + Cl2 (g). 11. Give reasons for the following :
2. ˆˆ
In the equation N2(g) + 3H2(g) ‡ ˆ†
ˆ 2NH3(g), what whould (a) Why the heat produced be different if same mass of
be the sign of work done? diamond and graphite are burnt in oxygen?
3. Give an example of following energy conversions: (b) The dissolution of ammonium chloride in water is en-
(i) Radiation energy into chemical energy. dothermic still it dissolves in water
(ii) Radiation energy into electrical energy. (c) A real crystal has more entropy than an ideal crystal.
4. A system is changed from an initial state to a final state by 12. Calculate the value of log Kp for the reaction N2(g) + 3H2(g)
a manner such that DH = q. If the change from the initial ˆˆ† 2NH3 (g) at 25°C. The standard enthalpy of
‡ˆˆ
state to a final state were made by a different path, would formation of NH3(g) is – 46 kJ mol–1 and standard entropies
DH be the same as that for the first path? would q too be of N2(g) , H2(g) and NH3(g) are 191, 130, 192 JK–1 mol–1
the same? respectively.
5. What property of enthalpy provides the basis of Hess’s
Long Answer Questions
law?
6. A chemist while studying the properties of gaseous 13. Define the following terms :
C2Cl 2F2, a chlorofluorocarbon refrigerant, cooled 1.25 g (a) System
sample at constant atmospheric pressure of 1.0 atm from (b) Isothermal processes
320 K to 293 K. During cooling, the sample volume
(c) Adiabatic processes
decreased from 274 to 248 ml. Calculate DH and DU for the
chlorofluorocarbon for this process. (d) State variables/state functions
The value of molar heat capacity is 80.7 J mol –1 K–1 . (e) Work
7. Gobar gas obtained by bacterial fermentation of animal 14. (i) Calculate the standard molar entropy change for the
refuse contains mainly methane. The heat of combustion following reactions at 298 K.
of methane to CO2 and water as gas is given by (a) 4 Fe (s) + 3 O2 (g) ¾¾ ® 2 Fe2O3(s).
CH4(g) + 2O2(g) ® CO2(g) + 2H2O(g) + 809 kJ (b) Ca (s) + 2 H2O (l) ¾¾ ® Ca(OH)2 (aq) + H2 (g)
How much gobar gas would have to be produced per day Given:
for a small village community of 100 families, if we assume S° Fe(s) = 27.28, S°O2(g) = 205.14, S° Fe2O3 (s) = 87.4
that each family has to be supplied 20000 kJ of energy per
S°Ca(s) = 41.42, S°H2O(l) = 69.9, S°Ca(OH)2(aq) = 74.5,
day to meet all its needs and that the methane content in
the gobar gas is 80 percent by weight. S°H2(g) = 130.68,
8. Why is it more convenient to predict the direction of (ii) Calculate the standard molar Gibbs energy of formation
reaction in terms of DGsys instead of DStotal? Under what of CS2, given that its standard enthalpy of formation is
conditions can DGsys be used to predict the spontaneity of 89.7 kJ mol–1 and the standard molar entropies of
a reaction? graphite, S and CS2 are 5.7, 31.8 and 151.3JK–1mol–1
respectively.
9. Which of the following processes are accompanied by
increase of entropy ? 15. For the reaction
(a) Stretching of rubber band, ˆˆ† 2H 2 (g) + O2 (g)
2H 2O(l ) ‡ˆˆ
(b) Boiling of an egg at 25°C, the equilibrium constant is 7.0 × 10–84.
(c) A deodrant is sprayed. (i) Calculate the standard Gibbs energy formations of
10. Comment on the following statements : water at 25°C.
(a) An exothermic reaction is always thermodynamically (ii) Dr H°H O = – 280 kJ mol–1. Calculate the entropy
2
spontaneous. change for the reaction.
158 Chemistry
Multiple Choice Questions The work can also be calculated from the pV–plot by using
the area under the curve within the specified limits. When
16. Thermodynamics is not concerned about____. an ideal gas is compressed (a) reversibly or (b) irreversibly
(a) energy changes involved in a chemical reaction. from volume Vi to Vf. Choose the correct option.
(b) the extent to which a chemical reaction proceeds. (a) w (reversible) = w (irreversible)
(c) the rate at which a reaction proceeds (b) w (reversible) < w (irreversible)
(d) the feasibility of a chemical reaction. (c) w (reversible) > w (irreversible)
17. Which of the following statements is correct? (d) w (reversible) = w (irreversible) + pex. DV
(a) The presence of reacting species in a covered beaker 21. During complete combustion of one mole of butane, 2658
is an example of open system. kJ of heat is released. The thermochemical reaction for above
(b) There is an exchange of energy as well as matter change is
between the system and the surroundings in a closed (a) 2C4H10(g) + 13O2(g) ® 8CO2(g) + 10H2O(l)
system. DcH = –2658.0 kJ mol–1
(c) The presence of reactants in a closed vessel made up
of copper is an example of a closed system. 13
(b) C4H10(g) + O (g) ® 4CO2(g) + 5H2O (g)
(d) The presence of reactants in a thermos flask or any 2 2
other closed insulated vessel is an example of a closed DcH = –1329.0 kJ mol–1
system.
13
18. Which one of the following statements is false? (c) C4H10(g) + O (g) ® 4CO2(g) + 5H2O (l)
2 2
(a) Work is a state function
DcH = –2658.0 kJ mol–1
(b) Temperature is a state function
(c) Change in the state is completely defined when the 13
(d) C4H10(g) + O (g) ® 4CO2(g) +5H2O (l)
initial and final states are specified 2 2
(d) None of the above DcH = + 2658.0 kJ mol–1
19. A heat engine absorbs heat Q1 at temperature T1 and heat 22. On the basis of thermochemical equations (1), (2) and (3),
Q2 at temperature T2. Work done by the engine is J (Q1 + find out which of the algebric relationships given in options
Q2). This data (a) to (d) is correct.
(a) violates 1st law of thermodynamics (1) C (graphite) + O2(g) ® CO2(g); DrH = x kJ mol–1
(b) violates 1st law of thermodynamics if Q1 is –ve
1
(c) violates 1st law of thermodynamics of Q2 is –ve (2) C(graphite) + O2(g) ® CO (g); DrH = y kJ mol–1
2
(d) does not violate 1st law of thermodynamics.
20. The pressure-volume work for an ideal gas can be calculated 1
(3) CO (g) + O (g) ® CO2 (g); DrH = z kJ mol–1
Vf
2 2
by using the expression w = - pex dV . (a) z = x + y (b) x = y–z
Vi (c) x = y + z (d) y = 2z – x
Thermodynamics 159

1. Which of the following is closed system ? 11. Enthalpy of a reaction DH is expressed as


(a) Jet engine
(a) DH= DHP– DHR (b) DH= D HP+ D HR
(b) Tea placed in a steel kettle
DH P HP
(c) Pressure cooker (c) DH = (d) DH =
(d) Rocket engine during propulsion DH R D HR
2. An isolated system is that system in which: 12. The enthalpy change of a reaction does not depend on
(a) There is no exchange of energy with the surroundings (a) The state of reactants and products
(b) There is exchange of mass and energy with the (b) Nature of reactants and products
surroundings (c) Different intermediate reactions
(c) There is no exchange of mass or energy with the (d) Initial and final enthalpy change of a reaction.
surroundings
13. Enthalpy change ( D ) of a system depends upon its
(d) There is exchange of mass with the surroundings
(a) Initial state
3. The intensive property among these quantities is
(a) mass (b) volume (b) Final state
(c) enthalpy (d) mass/volume (c) Both on initial and final state
4. Which is an extensive property of the system ? (d) None of these
(a) Volume (b) Viscosity 14. The relationship between enthalpy change and internal
(c) Temperature (d) Refractive index energy change is
5. Internal energy and pressure of a gas of unit volume are (a) DH = DE + PDV (b) DH = (DE + VDP)
related as : (c) DH = DE – PDV (d) DH = PDV – DE
2 1 15. If a reaction involves only solids and liquids which of the
(a) P= E (b) P= E
3 2 following is true ?
(a) DH < DE (b) DH = DE
3 (c) DH > DE (d) DH = DE + RTDn
(c) P= E (d) P = 2E
2 16. C (diamond ) ® C (graphite), DH = -ve . This shows that
6. The latent heat of vapourization of a liquid at 500 K and 1 (a ) Graphite is more stable
atm pressure is 10.0 kcal/mol. What will be the change in
(b) Graphite has more energy than diamond
internal energy (DU) of 3 moles of liquid at the same
temperature (c) Both are equally stable
(a) 13.0 kcal/mol (b) –13.0 kcal/mol (d) Stability cannot be predicted
(c) 27.0 kcal mol (d) –7.0 kcal/mol 17. One mole of a non-ideal gas undergoes a change of state
7. Mechanical work is specially important in systems that (2.0 atm, 3.0 L, 95 K) ® (4.0 atm, 5.0 L, 245 K) with a change
contain in internal energy, DU = 30.0 L atm. The change in enthalpy
(a) solid –liquid (b) liquid –liquid DH of the process in L atm is.
(c) solid –solid (d) gases (a) 40.0
8. In a closed insulated container a liquid is stirred with a paddle (b) 42.3
to increase the temperature. Which of the following is true? (c) 44.0
(a) (b) D E = W, q = 0 (d) Not defined because pressure is not constant
DE = W 0
18. The enthalpy of vapourisation of water from the following
(c) DE=0, W=q 0 (d) W=0, DW=q 0
two equations is.
9. An adiabatic expansion of an ideal gas always has
(a) decrease in temperature (b) q = 0 1
H2 (g) + O2 (g) ¾¾
® H2O (l), DH =- 286kJ
(c) w = 0 (d) DH = 0 2
10. For the reaction C + O ® CO 1
2 2 H 2 (g) + O 2 (g) , ® H 2 O (g) , DH = - 245.5 kJ
2
(a) DH > DE (b) DH < DE
(a) 6.02 kJ (b) 40.5 kJ
(c) DH = DE (d) None of these
(c) 62.3 kJ (d) 1.25 kJ
160 Chemistry

DH 27. The heat required to raise the temperature of a body by 1K


19. In the process Cl (g) + e- ¾¾¾ ® Cl - (g) , DH is is called
(a) positive (b) negative (a) specific heat (b) thermal capacity
(c) zero (d) unpredictable (c ) water equivalent (d) none of these
20. The variation of heat of reaction with temperature is given 28. For a reaction to occur spontaneously
by : (a) DS must be negative
(a) Van’t Hoff equation (b) (DH - TDS ) must be negative
(b) Clausius- Clapeyron equation
(c) (DH + TDS ) must be negative
(c) Nernst equation
(d) DH must be negative
(d) Kirchoff’s equation
29. The enthalpy and entropy change for a chemical reaction
21. Kirchoff’s equation is :
are – 2.5 ´ 103 cal and 7.4 cal deg-1 respectively. The
- E a / RT
(a) k = Ae reaction at 298K will be
(a) spontaneous (b) reversible
k2 Ea æ T2 - T1 ö (c) irreversible (d) non – spontaneous.
(b) log = ç ÷
k1 2.303R è T1 T2 ø 30. Heat exchanged in a chemical reaction at constant
temperature and constant pressure is called
2.303 RT C (a) internal energy (b) enthalpy
(c) E cell = log 2
nF C1 (c) entropy (d) free energy
31. For the process
DH 2 - DH1 Dry ice ® CO2 (g)
(d) = D Cp
T2 - T1
(a) DH is positive and DS is negative
22. Hess’s law deals with : (b) Both DH and DS are negative
(a) changes in heats of reaction
(c) Both DH and DS are positive
(b) rates of reactions
(d) DH is negative whereas DS is positive
(c) equilibrium constants 32. The favourable conditions for a spontaneous reaction are
(d) influence of pressure on the volume of a gas
23. If a chemical change is brought about by one or more methods (a ) TDS > DH, DH = + ve, DS = + ve
in one or more steps, then the amount of heat absorbed or (b) TDS > DH, DH = + ve, DS = - ve
evolved during the complete change is the same whichever (c) TDS = DH, DH = -ve, DS = - ve
method was followed. This rule is known as : (d) TDS = DH, DH = + ve, DS = + ve
(a) Le Chatelier principle (b) Hess’s law
33. Identify the correct statement regarding entropy.
(c) Joule Thomson effect (d) None of these (a) At absolute zero temperature, entropy of a perfectly
24. One mole of an ideal gas at 300 K is expanded isothermally crystalline substance is taken to be zero
from an initial volume of 1 litre to 10 litres. The DE for this (b) At absolute zero temperature, the entropy of a
process is (R = 2 cal mol–1 K–1) perfectly crystalline substance is positive
(a) 163.7 cal (b) zero (c) Absolute entropy of a substance cannot be
(c) 1381.1 cal (d) 9 lit atm determined
25. One mole of an ideal gas is allowed to expand reversibly and (d) At 0°C, the entropy of a perfectly crystalline substance
is taken to be zero
adiabatically from a temperature of 27ºC. If the work done
during the process is 3 kJ, the final temperature will be equal 34. For which of the following process, DS is negative?
to (Cv = 20 JK–1) (a) H 2 ( g ) ® 2 H (g )
(a) 150 K (b) 100 K (c) 26.85 K (d) 295 K
(b) N 2 (g)(1atm) ® N 2 (g)(8atm)
26. 2 moles of an ideal gas at 27ºC temperature is expanded
reversibly from 2 lit to 20 lit. Find entropy change (c) 2SO 3 (g) ® 2SO 2 (g) + O 2 (g)
(R = 2 cal/mol K) C (diamond) ® C (graphite)
(d)
(a) 92.1 (b) 0 (c) 4 (d) 9.2
Thermodynamics 161
35. According to second law of thermodynamics a process 44. According to third law of thermodynamics, which one of
(reaction )is spontaneous if during the process the following quantities for a perfectly crystalline solid is
(a) DSuniverse > 0 (b) DSuniverse = 0 zero at absolute zero.
(a) Entropy (b) Free energy
(c) DH system > 0 (d) DS universe = DSsystem (c) Internal energy (d) Enthalpy.
36. The law formulated by Nernst is 45. The calorific value of fat is
(a ) first law of thermodynamics (a) less than that of carbohydrate and protein
(b) second law of thermodynamics (b) less than that of protein but more than carbohydrate
(c) third law of thermodynamics (c) less than that of carbohydrate but more than that of
(d) Both (a) and (b) protein
37. If 900 J/g of heat is exchanged at boiling point of water, then (d) more than that of carbohydrate and protein.
what is increase in entropy? 46. Heat of neutralisation of strong acid against strong base is
(a) 43.4 J/mole (b) 87.2 J/mole constant and is equal to
(c) 900 J/mole (d) Zero (a) 13.7 kcal (b) 57 kJ
(c) 5.7 × 10 J4 (d) All of the above
38. Given the following entropy values (in JK-1 mol -1 ) at
298 K and 1 atm : 47. The heat of formation of the compound in the following
H2(g) : 130.6, Cl2(g) : 223.0 and HCl (g) : 186.7. The entropy reaction is
change (in JK-1 mol -1) for the reaction H 2 ( g ) + Cl2 ( g ) ® 2HCl ( g ) + 44kcal
H 2(g) + Cl2 (g) ® 2HCl(g)is
(a) -44 kcal mol-1 (b) - 22 kcal mol -1
(a) +540.3 (b) 727.0
(c) –166.9 (d) 19.8 (c) 11 kcal mol -1 (d) - 88 kcal mol -1
39. Entropy change involved in the conversion of 1 mole of
liquid water at 373 K to vapour at the same temperature will
48. Given that C + O2 ® CO 2 DH o = - xkJ
be (DH vap. = 2.257kJ / g)
2CO + O2 ® 2CO 2 DH o = - ykJ
(a) 0.119 kJ (b) 0.109 kJ
the enthalpy of formation of carbon monoxide will be
(c) 0.129 kJ (d) 0.12 kJ
40. Which relation is correct ? 2x - y y - 2x
(a) (b)
(a) DG = DH - TDS (b) DG = DH + T DS 2 2
(c) DG = TDS - DH (d) D G = D H - SdT (d) 2x - y (d) y = 2x.
41. The value of free energy change at equilibrium is 49. The enthalpy of neutralization of acetic acid and sodium
(a) positive (b) negative hydroxide is –55.4 kJ. What is the enthalpy of ionization of
(c) zero (d) not definite. acetic acid ?
42. The standard Gibb’s free energy change, DG 0 is related to (a) –1.9 kJ (b) +1.9 kJ (c) +5.54 kJ (d) –5.54 kJ
50. The neutralisation of a strong acid by a strong base
equilibrium constant, K P as
liberates an amount of energy per mole of H +
DG o (a) depends upon which acid and base are involved
é e ù
(a) K P =- RT ln DG o
(b) KP =ê ú (b) depends upon the temperature at which the reaction
ë RT û
takes place
DG (c) depends upon which catalyst is used
K P =-
o
(c) (d) K P = e -DG / RT
(d) is always the same
RT
51. Equal volumes of methanoic acid and sodium hydroxide are
43. What is the free energy change. ' DG' when 1.0 mole of water
mixed. If x is the heat of formation of water, then heat
at 100º C and 1 atm pressure is converted into steam at 100°C evolved on neutralization is
and 1 atm. pressure
(a) more than x (b) equal to x
(a) 540 cal (b) –9800 cal
(c) less than x (d) twice x.
(c) 9800 cal (d) 0 cal
162 Chemistry
60. The heat of combustion of CH4(g), C (graphite), H2(g) are –
3
52. S + O 2 ® SO 3 + 2x kcal 20 kcal, –40 kcal –10 kcal respectively. The heat of formation
2 of methane is
1 (a) –40 kcal (b) + 40 kcal
S O 2 + O 2 ® SO 3 + y kcal (c) –80.0 kcal (c) + 80 kcal
2
Find out the heat of formation of SO2 61. H 2 + Cl2 ® 2HCl, DH = - 194 kJ. In this reaction, heat
(a) (y -2x) (b) (2x + y) of formation of HCl in kJ is
(c) (x + y) (d) 2x/y (a) + 194 kJ (b) + 97 kJ
53. The enthalpy of neutralisation of a weak acid by a strong (c) –97 kJ (d) – 194 kJ
base is 62. Enthalpy of neutralisation of HCl with NaOH is x. The heat
(a) – 57.32 kJ evolved when 500 ml of 2 N HCl are mixed with 250 ml of 4N
(b) +57.32 kJ NaOH will be.
(c) equal to -57.32kJ + enthalpy of ionization of week acid (a) 500 x (b) 100x
(d) more than –57.32 kJ (c) x (d) 10 x
54. The enthalpy of formation for C2H4 (g), CO2 (g) and H2O (l) 63. When 1 M H2SO4 is completely neutralised by sodium
at 25º C and 1 atm pressure are 52, –394 and – 286 kJ mol–1 hydroxide, the heat liberated is 114.64 kJ.
respectively. The enthalpy of combustion of C 2H4 (g) will What is the enthalpy of neutralisation ?
be
(a) +114.64 kJ (b) –114.64 kJ
(a) +1412 kJ mol-1 (b) –1412 kJ mol-1
(c) –57.32 kJ (d) + 57.32 kJ
(c) 141.2 kJ mol -1 (d) –141.2 kJ mol -1 64. The amount of energy released when 20 ml of 0.5 M NaOH
55. When water is added to quicklime, the reaction is are mixed with 100 ml of 0.1 M HCl is x kJ. The heat of
(a) explosive (b) endothermic neutralisation (in kJ mol-1) is
(c) exothermic (d) photochemical. (a) –100x (b) –50x
56. Mark the correct statement (c) + 100 x (d) +50x
(a) For a chemical reaction to be feasible, DG should be 0 65. The DH f 0 for CO 2 (g), CO(g) and H 2 O (g)
(b) Entropy is a measure of order in a system are–393.5, –110.5 and –241.8 kJ mol-1 respectively. The
standard enthalpy change (in kJ) for the reaction
(c) For a chemical reaction to be feasible, DG should be
positive CO2 (g) + H 2 (g) ® CO (g) + H 2O (g) is
(d) The total energy of an isolated system is constant. (a) 524.1 (b) 41.2
57. One mole of an ideal gas is allowed to expand freely and (c) –262.5 (d) -41.2
adiabatically into vaccum until its volume has doubled. The 66. H 2 (g) + Cl 2 (g) ® 2 HCl(g) DH = - 44 kcal
expression which is not true concerning statement is
(a) DH = 0 (b) DS = 0 2 Na (s) + 2 HCl(g) ® 2 NaCl (s) + H 2 (g)

(c) DE = 0 (d) W = 0. DH = -152kcal


58. Which of the following statements is true ? For the reaction
(a) DG may be lesser or greater or equal to DH 1
Na (s) + Cl 2 (g) ¾
¾® NaCl(s) DH = ?
(b) DG is always proportional to DH 2
(c) DG is always greater than DH (a) –180 kcal (b) –196 kcal
(c) –98 kcal (d) 54 kcal
(d) DG is always less than DH
67. The heat required to decompose a compound into its
59. Compounds with high heat of formation are less stable
elements is equal to the heat evolved when the compound
because is formed from its elements. This is in accordance with :
(a) it is difficult to synthesize them (a) Lavosier and laplace law
(b) energy rich state leads to instability (b) Second law of thermodynamics
(c) high temperature is required to synthesize them (c) Joule- Thomson law
(d) molecules of such compunds are distorted. (d) Kirchoff’s law
Thermodynamics 163

1. Consider the following reactions: [CBSE-PMT 2007] 7 From the following bond energies: [CBSE-PMT 2009]
+ –
(i) H (aq) + OH (aq) ¾¾ ® H2O(l), H – H bond energy: 431.37 kJ mol–1
DH = – X1 kJ mol–1 C = C bond energy: 606.10 kJ mol–1
C – C bond energy: 336.49 kJ mol–1
1 C – H bond energy: 410.50 kJ mol–1
(ii) H2(g) + O (g) ¾¾
® H2O(l),
2 2 Enthalpy for the reaction,
DH = – X2 kJ mol–1
H H H H
(iii) CO2(g) + H2(g) ¾¾ ® CO(g) + H2O, | | | |
DH = – X3 kJ mol–1 C = C + H - H ¾¾
® H - C- C - H
| | | |
5 H H H H
(iv) C2H 2 ( g ) + O2 ( g ) ¾¾
® 2CO2(g) + H2O(l),
2
will be:
DH = + 4X4 kJ mol–1 (a) – 243.6 kJ mol–1 (b) –120.0 kJ mol–1
Enthalpy of formation of H2O (l) is (c) 553.0 kJ mol–1 (d) 1523.6 kJ mol–1
(a) + X3 kJ mol– 1 (b) – X4 kJ mol– 1 8. Standard entropies of X2 , Y2 and XY3 are 60, 40 and
(c) + X1 kJ mol– 1 (d) – X2 kJ mol– 1 50 JK–1mol–1 respectively. For the reaction
2. Given that bond energies of H – H and Cl – Cl are [CBSE-PMT 2010]
430 kJ mol– 1 and 240 kJ mol–1 respectively and DfH for HCl
1 3
is – 90 kJ mol– 1, bond enthalpy of HCl is [CBSE-PMT 2007] X 2 + Y2 ƒ XY3 , DH = – 30 kJ
2 2
(a) 380 kJ mol–1 (b) 425 kJ mol–1 to be at equilibrium, the temperature should be:
(c) 245 kJ mol–1 (d) 290 kJ mol–1 (a) 750 K (b) 1000 K
3. Which of the following are not state functions ? (c) 1250 K (d) 500 K
[CBSE-PMT 2008] 9. For vaporization of water at 1 atmospheric pressure, the values
(I) q + w (II) q of DH and DS are 40.63 kJmol–1 and 108.8 JK–1 mol–1,
(III) w (IV) H - TS respectively. The temperature when Gibbs energy change
(a) (I) and (IV) (b) (II), (III) and (IV)
(c) (I), (II) and (III) (d) (II) and (III)
( DG ) for this transformation will be zero, is:
4. For the gas phase reaction, [CBSE-PMT 2008] [CBSE-PMT 2010]
ˆˆ† PCl3(g) + Cl2(g) (a) 293.4 K (b) 273.4 K
PCl5(g) ‡ˆˆ
(c) 393.4 K (d) 373.4 K.
which of the following conditions are correct ? 10. Match List -I (Equations) with List-II (Type of processes)
(a) DH = 0 and DS < 0 and select the correct option. [CBSE-PMT 2010]
(b) DH > 0 and DS > 0 List I List II
(c) DH < 0 and DS < 0 Equations Type of processes
(d) DH > 0 and DS < 0 (1) Kp > Q (i) Non spontaneous
5. Bond dissociation enthalpy of H2, Cl2 and HCl are 434 , 242 and (2) DG ° < RT lnQ (ii) Equilibrium
431 kJ mol–1 respectively. Enthalpy of formation of HCl is : (3) Kp = Q (iii) Spontaneous and
[CBSE-PMT 2008] endothermic
(a) 93 kJ mol –1 (b) – 245 kJmol–1 DH
(c) – 93 kJmol –1 (d) 245 kJmol–1 (4) T > (iv) Spontaneous
DS
6. The values of DH and DS for the reaction, Options:
C(graphite) + CO2 (g) ® 2CO(g) are 170 kJ and 170 JK–1, (1) (2) (3) (4)
respectively. This reaction will be spontaneous at (a) (ii) (i) (iv) (iii)
[CBSE-PMT 2009] (b) (i) (ii) (iii) (iv)
(a) 910 K (b) 1110 K (c) (iii) (iv) (ii) (i)
(c) 510 K (d) 710 K (d) (iv) (i) (ii) (iii)
164 Chemistry
11. The following two reactions are known : 18. Standard enthalpy of vapourisation Dvap H° for water at
[CBSE-PMT 2010] 100°C is 40.66 kJ mol–1. The internal energy of vaporisation
of water at 100°C (in kJ mol–1) is : [CBSE-PMT 2012 S]
Fe2O3(s) + 3CO (g) ¾¾
® 2Fe(s) + 3CO2(g); DH = –26.8 kJ (a) + 37.56 (b) – 43.76
(c) + 43.76 (d) + 40.66
FeO(s) + CO(g) ¾¾
® Fe(s) + CO2(g); DH = –16.5 kJ
(Assume water vapour to behave like an ideal gas).
The value of D H for the following reaction 19. Equal volumes of two monoatomic gases, A and B, at same
Fe2O3(s) + CO(g) ¾¾ ® 2FeO(s) + CO2(g) is; temperature and pressure are mixed. The ratio of specific
(a) + 6.2 kJ (b) + 10.3 kJ heats (Cp/Cv) of the mixture will be : [CBSE-PMT 2012]
(c) – 43.3 kJ (d) – 10.3 kJ (a) 0.83 (b) 1.50
12. Three moles of an ideal gas expanded spontaneously into (c) 3.3 (d) 1.67
vacuum. The work done will be : [CBSE-PMT 2010] 20. A reaction having equal energies of activation for forward
(a) Zero (b) Infinite (c) 3 Joules (d) 9 Joules and reverse reaction has : [NEET 2013]
13. If the enthalpy change for the transition of liquid water to (a) DG = 0 (b) DH = 0
steam is 30 kJ mol–1 at 27ºC, the entropy change for the (c) DH = DG = DS = 0 (d) DS = 0
process would be : [CBSE-PMT 2011]
21. If an endothermic reaction is non-spontaneous at freezing
(a) 10 J mol –1 K–1 (b) 1.0 J mol–1 K–1
point of water and becomes feasible at its boiling point,
(c) 0.1 J mol–1 K–1 (d) 100 J mol–1 K–1
then [AIEEE 2002]
14. Enthalpy change for the reaction, [CBSE-PMT 2011]
(a) DH is –ve, DS is +ve
4H( g ) ¾¾
® 2H 2 ( g ) is – 869.6 kJ.
(b) DH and DS both are +ve
The dissociation energy of H–H bond is :
(c) DH and DS both are –ve
(a) – 434.8 kJ (b) – 869.6 kJ
(d) DH is +ve, DS is -ve
(c) + 434.8 kJ (d) + 217.4 kJ
22. A heat engine abosrbs heat Q1 at temperature T1 and heat
15. Consider the following processes :
Q2 at temperature T2. Work done by the engine is J (Q1 +
DH (kJ/mol)
Q2). This data [AIEEE 2002]
1/2 A® B +150 st
(a) violates 1 law of thermodynamics
3B ® 2C + D –125 (b) violates 1st law of themodynamics if Q1 is –ve
E + A ® 2D +350 (c) violates 1st law of thermodynamics of Q2 is –ve
For B + D ® E + 2C, DH will be : [CBSE-PMT 2011 M] (d) does not violate 1st law of themodynamics.
(a) 525 kJ/mol (b) – 175 kJ/mol 23. For the reactions, [AIEEE 2002]
(c) – 325 kJ/mol (d) 325 kJ/mol C + O2 ® CO2 ; DH = -393 J
16. In which of the following reactions, standard entropy
2Zn + O2 ® 2 ZnO ; DH = -412 J
change (DS°) is positive and standard Gibb’s energy change
(a) carbon can oxidise Zn
(DG°) decreases sharply with increase in temperature ?
(b) oxidation of carbon is not feasible
[CBSE-PMT 2012 S]
(c) oxidation of Zn is not feasible
1 (d) Zn can oxidise carbon.
(a) C (graphite) + O (g) ® CO(g)
2 2 24. The internal energy change when a system goes from state
1 A to B is 40 kJ/mole. If the system goes from A to B by a
(b) CO(g) + O (g) ® CO2(g) reversible path and returns to state A by an irreversible path
2 2
what would be the net change in internal energy ?
1 [AIEEE 2003]
(c) Mg(s) + O (g) ® MgO(s)
2 2 (a) > 40 kJ (b) < 40 kJ
1 1 1 (c) Zero (d) 40 kJ
(d) C (graphite) + O2(g) ® CO2(g) 25. If at 298 K the bond energies of C — H, C — C, C = C and H
2 2 2
— H bonds are respectively 414, 347, 615 and 435 kJ mol–1,
17. The enthalpy of fusion of water is 1.435 kcal/mol. the value of enthalpy change for the reaction
The molar entropy change for the melting of ice at 0°C is :
H 2 C = CH 2 (g) + H 2 (g) ® H 3C — CH 3 (g)
[CBSE-PMT 2012 S]
(a) 10.52 cal / (mol K) (b) 21.04 cal / (mol K) at 298 K will be [AIEEE 2003]
(c) 5.260 cal / (mol K) (d) 0.526 cal / (mol K) (a) – 250 kJ (b) + 125 kJ
(c) – 125 kJ (d) + 250 kJ
Thermodynamics 165
26. In an irreversible process taking place at constant T and P and 34. An ideal gas is allowed to expand both reversibly and
in which only pressure-volume work is being done, the change irreversibly in an isolated system. If T i is the initial
in Gibbs free energy (dG) and change in entropy (dS), satisfy temperature and Tf is the final temperature, which of the
the criteria [AIEEE 2003] following statements is correct? [AIEEE 2006]
(a) ( D S)V, E > 0, ( D G)T, P < 0 (b) ( D S)V, E = 0, ( D G)T, P = 0 (a) (Tf)rev = (Tf)irrev
(b) Tf = Ti for both reversible and irreversible processes
(c) ( D S)V, E = 0, ( D G)T, P > 0 (d) ( D S)V, E < 0, ( D G)T, P < 0
(c) (Tf)irrev > (Tf)rev
27. The correct relationship between free energy change in a
(d) Tf > Ti for reversible process but Tf = Ti for irreversible
reaction and the corresponding equilibrium constant Kc is
process
[AIEEE 2003]
35. The standard enthalpy of formation (DfHº) at 298 K for
(a) – DG = RT ln Kc (b) DGº = RT ln Kc
methane, CH4 (g) is –74.8 kJ mol –1 . The additional
(c) – DGº = RT ln Kc (d) DG = RT ln Kc information required to determine the average energy for C
28. The enthalpy change for a reaction does not depend upon – H bond formation would be [AIEEE 2006]
[AIEEE 2003] (a) the first four ionization energies of carbon and electron
(a) use of different reactants for the same product gain enthalpy of hydrogen
(b) the nature of intermediate reaction steps (b) the dissociation energy of hydrogen molecule, H2
(c) the differences in initial or final temperatures of involved (c) the dissociation energy of H2 and enthalpy of
substances sublimation of carbon
(d) the physical states of reactants and products (d) latent heat of vapourization of methane
29. An ideal gas expands in volume from 1×10 –3 to 36. The enthalpy changes for the following processes are listed
1 × 10 –2 m 3 at 300 K against a constant pressure of below : [AIEEE 2006]
1×10 5 Nm –2 . The work done is [AIEEE 2004] Cl2(g) = 2Cl(g), 242.3 kJ mol –1

(a) 270 kJ (b) – 900 kJ I2(g) = 2I(g), 151.0 kJ mol–1


(c) – 900 kJ (d) 900 kJ ICl(g) = I(g) + Cl(g), 211.3 kJ mol–1
30. The enthalpies of combustion of carbon and carbon I2(s) = I2(g), 62.76 kJ mol–1
monoxide are – 393.5 and – 283 kJ mol–1 respectively. The Given that the standard states for iodine and chlorine are
enthalpy of formation of carbon monoxide I2(s) and Cl2(g), the standard enthalpy of formation for ICl(g)
per mole is [AIEEE 2004] is : [AIEEE 2006]
(a) – 676.5 kJ (b) 676.5 kJ (a) +16.8 kJ mol–1 (b) +244.8 kJ mol–1
(c) 110.5 kJ (d) – 110.5 kJ (c) –14.6 kJ mol–1 (d) –16.8 kJ mol–1
31. For a spontaneous reaction the D G, equilibrium constant 37. (DH – DU) for the formation of carbon monoxide (CO) from
its elements at 298 K is [AIEEE 2006]
(K) and E oCell will be respectively [AIEEE 2005] (R = 8.314 J K–1 mol–1)
(a) –ve, >1, –ve (b) –ve, <1, –ve (a) –2477.57 J mol–1 (b) 2477.57 J mol–1
(c) +ve, >1, –ve (d) –ve, >1, +ve
(c) –1238.78 J mol–1 (d) 1238.78 J mol–1
32. Consider the reaction : N 2 + 3H 2 ® 2 NH 3 carried out at 38. In conversion of lime-stone to lime, [AIEEE 2007]
constant temperature and pressure. If DH and DU are the CaCO3 ( s ) ® CaO( s ) + CO 2 ( g ) the values of DH° and
enthalpy and internal energy changes for the reaction, which
DS° are + 179.1 kJ mol-1 and 160.2 J/K respectively at 298 K
of the following expressions is true ? [AIEEE 2005]
and 1 bar. Assuming that DH° and DS° do not change
(a) DH > DU (b) DH < DU with temperature, temperature above which conversion of
(c) DH = DU (d) DH = 0 limestone to lime will be spontaneous is
(a) 1118 K (b) 1008 K
33. If the bond dissociation energies of XY, X 2 and Y2 (all
(c) 1200 K (d) 845 K.
diatomic molecules) are in the ratio of 1 : 1 : 0.5 and DHf for 39. Assuming that water vapour is an ideal gas, the internal
the formation of XY is – 200 kJ mole -1 . The bond energy change (DU) when 1 mol of water is vapourised at
dissociation energy of X 2 will be [AIEEE 2005] 1 bar pressure and 100°C, (given : molar enthalpy of
vapourisation of water at 1 bar and 373 K = 41 kJ mol–1 and
-1
(a) 400 kJ mol (b) 300 kJ mol -1 R = 8.3 J mol–1 K–1) will be [AIEEE 2007]
-1 -1 (a) 41.00 kJ mol–1 (b) 4.100 kJ mol–1
(c) 200 kJ mol (d) 100 kJ mol
(c) 3.7904 kJ mol–1 (d) 37.904 kJ mol–1
166 Chemistry

40. Identify the correct statement regarding a spontaneous 46. Consider the reaction : [AIEEE 2011 RS]
process: [AIEEE 2007] 4NO2 ( g ) + O 2 ( g ) ® 2N 2O5 ( g ),
(a) Lowering of energy in the process is the only criterion DrH = – 111 kJ.
for spontaneity.
If N2O5(s) is formed instead of N2O5(g) in the above reaction,
(b) For a spontaneous process in an isolated system, the
the DrH value will be :
change in entropy is positive.
(given, DH of sublimation for N2O5 is –54 kJ mol–1)
(c) Endothermic processes are never spontaneous.
(a) + 54 kJ (b) + 219 kJ
(d) Exothermic processes are always spontaneous.
41. Oxidising power of chlorine in aqueous solution can be (c) – 219 J (d) – 165 kJ
determined by the parameters indicated below: 47. A piston filled with 0.04 mol of an ideal gas expands
[AIEEE 2008] reversibly from 50.0 mL to 375 mL at a constant temperature
of 37.0ºC. As it does so, it absorbs 208 J of heat. The values
1
1 D H D eg H of q and w for the process will be: [JEE M 2013]
2 diss
Cl2 ( g ) ¾¾¾¾¾ ® Cl( g ) ¾¾¾¾ ® Cl– ( g )
2 (R = 8.314 J/mol K) (ln 7.5 = 2.01)
(a) q = + 208 J, w = – 208 J
D H
Hyd
¾¾¾¾ ® Cl – (aq) (b) q = – 208 J, w = – 208 J
(using the data, (c) q = – 208 J, w = + 208 J
–1 –1 (d) q = + 208 J, w = + 208 J
D diss H Cl = 240 kJ mol D eg H Cl = –349 kJ mol
2
48. ˆˆ† B is
The value of log10 K for a reaction A ‡ˆˆ
–1
D hyd H Cl – = –381 kJ mol ), will be (Given : D r H °298K = -54.07 kJ mol- 1 ,
(a) + 152 kJ mol–1 (b) – 610 kJ mol–1
(c) – 850 kJ mol–1 (d) + 120 kJ mol–1 D r S°298K = 10 JK–1 mol–1 and R = 8.314 JK–1 mol–1;
42. On the basis of the following thermochemical data : 2.303 × 8.314 × 298 = 5705) [IIT-JEE 2007]
+
[Δ f H°(H (aq) = 0)] [AIEEE 2009] (a) 5 (b) 10
(c) 95 (d) 100
H 2O(l ) ® H + ( aq ) + OH – ( aq ); DH = 57.32kJ
49. For the process H2O(l) (1 bar, 373 K) ® H2O(g) (1 bar, 373
1 K), the correct set of thermodynamic parameters is
H2 (g )+ O2 (g ) ¾¾ ® H2O(l); ΔH= –286.20kJ
2 [IIT-JEE 2007]
The value of enthalpy of formation of OH– ion at 25° C is: (a) DG = 0, DS = +ve (b) DG = 0, DS = –ve
(a) –228.88 kJ (b) +228.88 kJ
(c) DG = +ve, DS = 0 (d) DG = –ve, DS = +ve
(c) –343.52 kJ (d) –22.88 kJ
50. The species which by definition has ZERO standard molar
43. The standard enthalpy of formation of NH 3
enthalpy of formation at 298 K is [IIT-JEE 2010]
is – 46.0 kJ mol–1. If the enthalpy of formation of H2 from its
atoms is – 436 kJ mol–1 and that of N2 is – 712 kJ mol–1, the (a) Br2 (g) (b) Cl2 (g)
average bond enthalpy of N – H bond in NH3 is (c) H2O (g) (d) CH4 (g)
[AIEEE 2010] 51. Using the data provided, calculate the multiple bond energy
(a) – 964 kJ mol–1 (b) + 352 kJ mol–1 (kJ mol–1) of a C º C bond in C2H2. That energy is (take the
(c) + 1056 kJ mol–1 (d) – 1102 kJ mol–1 bond energy of a C – H bond as 350 kJ mol–1)
44. For a particular reversible reaction at temperature T, DH 2C(s) + H2(g) ¾¾ ® HC º CH(g); DH = 225 kJ mol–1
and DS were found to be both +ve. If Te is the temperature 2C(s) ¾¾ ® 2C(g) ; DH = 1410 kJ mol–1
at equilibrium, the reaction would be spontaneous when H2(g) ¾¾ ® 2H(g) ; DH = 330 kJ mol–1 [IIT-JEE 2012]
(a) Te > T (b) T > Te [AIEEE 2010] (a) 1165 (b) 837
(c) Te is 5 times T (d) T = Te
(c) 865 (d) 815
45. The value of enthalpy change (DH) for the reaction
52. The standard enthalpies of formation of CO2(g), H2O(l)
C2 H5 OH(l) + 3O2 ( g ) ® 2CO2 ( g ) + 3H 2O(l) and glucose(s) at 25°C are –400 kJ/mol, –300 kJ/mol and –
at 27° C is – 1366.5 kJ mol–1. The value of internal energy 1300 kJ/mol, respectively. The standard enthalpy of
change for the above reaction at this temperature will be : combustion per gram of glucose at 25°C is
[AIEEE 2011 RS] [JEE Advanced 2013]
(a) – 1369.0 kJ (b) – 1364.0 kJ (a) +2900 kJ (b) –2900 kJ
(c) – 1361.5 kJ (d) – 1371.5 kJ (c) –16.11 kJ (d) +16.11 kJ
Thermodynamics 167

1. For determining the spontaneity of a process which of the 9. What is the free energy change for the conversion of
following is considered ? 1 mole of water into steam at 373.2 K . The heat
(a) DS system of vaporization (DHv) of water of 373.2 K is 9.1 kcal. mol –1.
(b) DS surroundings The entropy change is 25.5 cal/mol deg.
(c) DS system + DS surroundings (a) –401.6 cal/mol (b) –416.6 cal/mol
(d) DS system – DS surroundings (c) 516.5 cal/mol (d) –516.5 cal/mol
2. Which of the following is always true for a spontaneous 10. The ‘thermite reaction’ involving the reaction between ferric
change at all temperatures ? oxide and metallic aluminium produces molten iron. Given
(a) DH > 0 ; DS < 0 (b) DH < 0 ; DS < 0 that
(c) DH < 0 ; DS > 0 (d) DH > 0 ; DS > 0 2Al + 3 / 2 O 2 ® Al 2 O 3 ; DH1 = -400 kcal/ mol
3. What is the change in entropy when ice melts at 0°C, enthalpy
of fusion of one mole of ice is 6.02 kJ ? 2Fe + 3 / 2 O 2 ® Fe 2O 3 ; DH 2 = -200 kcal/mol.
(a) 6.02 kJ K–1 mol–1 (b) 22.1 J K–1 mol–1 What is DH for the formation of 1 mole of iron ?
(c) 1 J K–1 mol–1 (d) 22.0J K–1 mol–1 (a) –100 kcal (b) –200 kcal
4. In which of the following processes, the entropy decreases (c) +100 kcal (d) +200 kcal
? 11. Given the reaction at 927°C and 1 atm.
(a) CaCO3 (s) ® CaO(s) + CO2 (g) ˆˆ† CaO(s) + CO2 (g) DH= 176 kJ/mol; then
CaCO3 (s) ‡ˆˆ
(b) 2NH 3 (g) ® N 2 (g) + 3H 2 (g) DE equals
(c) 2H 2 (g) + O 2 (g) ® 2H 2 O(l) (a) 100 kJ (b) 166.03 kJ
(c) 180.0 kJ (d) 186.4 kJ
(d) O 3 (g ) ® O 2 (g ) + O(g) 12. What is the enthalpy change for the reaction
5. For which one of the processes represented by the following H 2 (g ) + C 2 H 4 (g ) ® C 2 H 6 (g )
equations the enthalpy (heat) change is likely to be negative
The bond energies are given below:
(a) Cl - (g) + aq ® Cl - (aq) (b) Cl(g) ® Cl + (g) + e -
( H - H ) ® 103; ( C - H ) ® 99, ( C - C ) ® 80 and
(c) 1 / 2Cl 2 (g ) ® Cl(g ) (d) Cl 2 (l) ® Cl 2 (g)
( C = C ) ® 145 kcal/mol respectively
6. The enthalpy change of formation of CO 2 (g)
is – 393 kJmol–1 and that of H2O (l) is – 286 kJmol–1. The (a) –10 kcal mol–1 (b) +10 kcal mol–1
enthalpy of combustion of one mole of ethanol (C2H5OH) is (c) –30 kcal mol–1 (d) +30 kcal mol–1
–1360. The enthalpy change for the formation of one mole of 13. For the process H2O(l) H2O(g) DH = 45.0 kJ mol–1 and
ethanol form its constituent elements is DS = 1.20 × 102 J K–1 mol–1. At what temperature the above
(a) –681 kJ (b) –284 kJ process is at equilibrium?
(c) +965 kJ (d) 1360 kJ (a) 273 K (b) 373 K
7. What is Dngas for the combustion of 1 mole of benzene, when (c) 370 K (d) 375 K
both the reactants and the products are at 298 K ?
14. Third law of thermodynamics is used to determine the
(a) 0 (b) 1/2
(a) spontaneity of a process
(c) 3/2 (d) –3/2
(b) enthalpy of substance
8. Which of the following statements is false ?
(c) absolute values of entropy of a substance
(a) For 1 mole of an ideal gas, Cp – Cv = R
(d) free energy of a substance
æ Eö 15. In thermodynamics, a process is called reversible when
(b) ç ÷ = 0 for an ideal gas
è T øT (a) surroundings and system change into each other
(b) there is no boundary between system and surroundings
(c) Dq = Dw + pDV (c) the surroundings are always in equilibrium with the
(d) For reversible isothermal expansion of 1 mole of an ideal system
gas from volume V1 to V2, work done is equal to RT ln (d) the system changes in to the surroundings
(V2/V1) spontaneously
168 Chemistry
16. Which one of the following statements is false? 23. For the reaction
(a) Work is a state function
(b) Temperature is a state function 2C6 H5CO2 H(s) + 15O2 (g) ¾¾
®14CO 2 (g) + 6H 2O(g)
(c) Change in the state is completely defined when the initial DUº = –772.7 kJ mol–1 at 298 K. Calculate DHº
and final states are specified (a) +760.3 kJ mol–1 (b) –760.3 kJ mol–1
(d) Work appears at the boundary of the system (c) +670.3 kJ mol –1 (d) –790.3 kJ mol–1
17. The standard enthalpies of formation of H + and 24. 1 gram equivalent of H2SO4 is treated with 112 g of KOH for
OH– ions in water are zero and –229.6 kJ mol–1 respectively. complete neutralization. Which of the following statements
The standard enthalpy of formation of liquid water is –285.6kJ is correct?
mol–1. Then the enthalpy of neutralization of HCl(aq) by (a) 13.7 kcal of heat is evolved with the formation of
KOH(aq) is 87 g of K2SO4, leaving no KOH.
(a) 229.6 kJ mol–1 (b) –173.4 kJ mol–1
(b) 27.4 kcal of heat is evolved with the formation of
(c) 56.0 kJ mol –1 (d) –56.0 kJ mol–1 87 g of K2SO4, leaving 4 gram equivalent of KOH.
18. Which of the following salts should cause maximum cooling (c) 15.7 kcal of heat is evolved with the formation of 1 gram
when 1 mole of it is dissolved in the same volume of water ? equivalent of K2SO4, leaving 56 g of KOH.
(a) NaCl; DHº = 5.35 kJ mol–1 (d) 13.7 kcal of heat is evolved with the formation of 87g of
(b) KNO3; DHº = 53.5 kJ mol–1 K2SO4, leaving 1 gram equivalent of KOH.
(c) KOH; DHº = –56.0 kJ mol–1 25. From the following data DH of the following reactions
(d) HBr; DHº = –83.3 kJ mol–1
19. The molar enthalpies of combustion of isobutane and n-butane 1
C(s) + O 2 (g ) ¾¾® CO(g); DH = – 110 kJ and
are –2870 kJ mol–1 & –2875 kJ mol–1 respectively at 298 K and 2
1 atm. Calculate DHº for the conversion of 1 mole of n-butane
to 1 mole of isobutane C(s) + H 2 O(g) ¾¾® CO(g) + H 2 (g); DH = 132 kJ
(a) –8 kJ mol–1 (b) +8 kJ mol–1 Calculate the mole composition of the mixture of steam and
(c) –5748 kJ mol –1 (d) +5748 kJ mol–1 oxygen on being passes over coke at 1273 K, keeping the
temperature constant.
20. For the reaction
(a) 1 : 0.6 (b) 0.6 : 1
1 (c) 2 : 3 (d) 3 : 2
CO(g) + O 2 (g) ¾ ¾® CO 2 (g ); DHº = –67650 cal at
2
o -1
25 ºC. Calculate DHº at 100°C, given that the required molar 26. DHf of water is – 285.8 kJmol . If enthalpy of
heat capacities are as follows. neutralisation of monoacidic strong base is
CP(CO,g) = 6.97 cal ºC–1
–57.3 kJmol - 1 . DHf of OH - ion will be
o
CP(CO2,g) = 8.97 cal ºC–1
CP(O2,g) = 7.00 cal ºC–1 (a) – 114.25 kJmol -1 (b) 114.25 kJmol
-1

(a) –54.6 cal (b) –67650.4 cal


-1
(c) –67684.4 cal (d) –67762.5 cal (c) 228.5 kJmol (d) – 228.5 kJmol -1
21. The heat of atomization of PH3(g) is 228 kcal mol–1 and that 27. Temperature of 5 moles of a gas is decreased by 2K at constant
of P2H4(g) is 335 kcal mol–1. The energy of the P–P bond is pressure. Indicate the correct statement
(a) 102 kcal mol–1 (b) 51 kcal mol–1 (a) Work done by gas is = 5 R
(c) 26 kcal mol–1 (d) 204 kcal mol–1 (b) Work done over the gas is = 10 R
22. The enthalpy of solution of a substance is given by (c) Work done by the gas = 10 R
º º
(d) Work done = 0
(a) DH so ln = U (lattice energy) + DH hydration 28. An ideal gas occuping a volume of 2dm3 and a pressure of 5
º º
bar undergoes isothermal and irreversible expansion against
(b) DH so ln . = U ( lattice energy) - DH hydration external pressure of 1 bar. The final volume of the system
and the work involved in the process is
º º
(c) DH so ln . = - U ( lattice energy) + DH hydration
(a) 10 dm 3 , 1000 J (b) 8 dm 3 , – 800 J
º º
(d) DH so ln . = - U (lattice energy) - DH hydration (c) 10 dm 3 , – 800 J (d) 10 dm 3 , – 1000 J
Thermodynamics 169
29. At 25°C and 1 bar which of the following has a non-zero 35. The DH at 358 K for the reaction
DHof Fe 2O 3 (s) + 3 H 2 (g) ¾
¾® 2Fe(s) + 3 H 2 O (l)
(a) Br2 (l) (b) C (graphite) -1
given that DH 298 = – 33.29 kJ mol and Cp for Fe 2O 3 (s),
(c) O 3 (g) (d) I 2 (s) Fe(s), H 2 O (l) and H 2 (g) and 103.8, 25.1, 75.3 and 28.8 J/K
mol
30. Bond enthalpies of H 2 , X 2 and HX are in the ratio 2 : 1 : 2.
-1 -1
(a) – 28.136 k J mol (b) – 38.3 k J mol
-1
If enthalpy of formation of HX is – 50 kJmol , the bond
-1 -1
(c) 42.5 k J mol (d) 56.2 k J mol
enthalpy of X 2 is
-1
-1 -1
36. DH°f of NF3 is – 113 k J mol and N–F bond energy is
(a) 100 kJmol (b) 300 kJmol
-1
-1 273.0 k J mol . If N º N and F–F bond energies are in the
(c) 200 kJmol - 1 (d) 400 kJmol
rates 6 : 1, their magnitudes will be
31. When 1 mole of oxalic acid is treated with excess of NaOH in
-1 -1
dilute aqueous solution, 106 kJ of heat is liberated. Predict (a) 780.0 k J mol , 130.0 k J mol
the enthalpy of ionisation of the acid -1 -1
(b) 840 k J mol , 140 k J mol
-1
(a) 4.3 kJmol -1 (b) – 4.3 kJmol -1 -1
(c) 950.0 k J mol , 158.3 k J mol
(c) 8.6 kJmol -1 (d) – 8.6 kJmol -1 -1 -1
(d) 941.3 k J mol , 156.9 k J mol
32. An athlete is given 100 g of glucose (C 6 H12O 6 ) of energy
37. The enthalpy of neutralisation of NH 4 OH and
equivalent to 1560 kJ. He utilises 50% of this gained energy
in the event. In order to avoid storage of energy in the body. CH 3COOH is – 10.5 kcalmol -1 and enthalpy of
Calculate the weight of water he would need to perspire. neutralisation of CH 3COOH with strong base is – 12.5
-1
The enthalpy of evaporation of water is 44 kJmol . kcalmol-1 . The enthalpy of ionisation of NH 4 OH will be
(a) 319.1 g (b) 119.7 g
(a) 3.2 kcalmol -1 (b) 2.0 kcalmol -1
(c) 227.5 g (d) 250.2 g
33. The enthalpy change for hydration (c) 3.0 kcalmol -1 (d) 4.0 kcalmol -1

¾® CuSO 4 .5H 2O (s)


CuSO 4 (s) + 5H 2 O (g ) ¾ 38. Standard enthalpy of combustion of CH 4 is – 890 kJ mol-1
and standard enthalpy of vaporisation of water is
-1
is – 71.50 kcalmol . If enthalpy of vaporisation of water is -1
40.5 kJ mol . The enthalpy change of the reaction
-1
10.5 kcalmol at 25°C, what would be the enthalpy change
CH 4 (g) + 2 O 2 (g) ¾
¾® CO 2 (g) + 2 H 2 O (g)
for the hydration
(a) – 809.5 kJ mol-1 (b) – 890 kJ mol-1
CuSO 4 (s) + 5H 2 O( l) ¾
¾® CuSO 4 .5H 2O (s)
-1
(c) 809 kJ mol-1 (d) – 971 kJ mol
-1 -1
(a) – 19.0 kcalmol (b) 19.0 kcalmol 39. The enthalpy of combustion of 2 moles of benzene at 27°C
differs from the value determined in bomb calorimeter by
-1
(c) 61.0 kcal mol (d) – 61.0 kcalmol -1 (a) – 2.494 kJ (b) 2.494 kJ
(c) – 7.483 kJ (d) 7.483 kJ
-1
34. The lattice energy of solid NaCl is 180 kcal mol and 40. The heats of neutralisation of CH 3COOH , HCOOH, HCN
-1
enthalpy of solution is 1 kcal mol . If the hydration energies and H 2S are – 13.2, – 13.4, – 2.9 and – 3.8 kcal per equivalent
respectively. Arrange the acids in increasing order of
of Na + and Cl - ions are in the ratio 3 : 2, what is the strength
enthalpy of hydration of sodium ion ?
(a) HCOOH > CH 3COOH > H 2S > HCN
(a) – 107.4 kcal mol-1 (b) 107.4 kcal mol-1 (b) CH 3COOH > HCOOH > H 2 S > HCN

(c) 71.6 kcalmol -1 (d) – 71.6 kcalmol -1 (c) H 2S > HCOOH > CH 3COOH > HCN
(d) HCOOH > H 2S > CH 3COOH > HCN
170 Chemistry
41. When 1.8 g of steam at the normal boiling point of water is 43. The enthalpy of atomisation of CH 4 and C 2 H 6 are 360
converted into water at the same temperature, enthalpy and
entropy changes respectively will be ( DHVaporisation for and 620 kcal mol -1 respectively. The C – C bond energy is
expected to be
H 2 O = 40.8 kJ mol-1 )
-1 -1
(a) 210 k cal mol (b) 80 k cal mol
-1 -1
(a) 4.08 kJ, 10.93 J K (b) – 4.08 kJ, – 10.93 J K
-1
(c) 130 k cal mol -1 (d) 180 k cal mol
-1 -1
(c) – 4.08 kJ, – 40.8 J K (d) 4.08 kJ, 40.8 J K
44. A certain reaction is non spontaneous at 298K. The entropy
42. The enthalpy of neutralisation of a weak acid in 1 M solution
change during the reaction is 121 J K -1 . Is the reaction is
with a strong base is – 56.1 kJ mol-1 . If the enthalpy of
endothermic or exothermic ? The minimum value of DH for
ionisation of acid is 1.5 kJ mol-1 an d enthalpy of the reaction is
neutralisation of the strong acid with a strong base is –57.3
(a) endothermic, DH = 36.06 kJ
kJ eq-1 . What is the % ionisation of the weak acid in molar (b) exothermic, DH = – 36.06 kJ
solution (assume the acid is monobasic)
(c) endothermic, DH = 60.12 kJ
(a) 25 (b) 20
(c) 15 (d) 10 (d) exothermic, DH = – 60.12 kJ
Thermodynamics 171

EXERCISE 1 10. (c) DH = DE + DnRT ; Dn = 0 DH = DE . In the given


1. (a) DH = DU because Dn = 0 reaction C is solid.
(b) DH > DU because Dn = 1 11. (a) Enthalpy of reaction, D H = DH products - DH Reactants
2. The sign of work done will be +ve
3. (i) Photosynthesis 12. (c) In accordance with Hess’s law
(ii) Photoelectric effect. 13. (c)
4. DH is the same but q will be different. 14 (a) DH = D E + PDV
5. (a) Enthalpy is a state function and (b) law of conservation
15. (b) DH = DE + PDV, for solid and liquid, DV = 0
of energy are the two basis of Hess’s law.
6. DU = – 20.48 J or DH = DE + DnRT, for solids and liquids Dn = 0
DH = – 23.11 J 16. (a) As heat is evolved, heat content of diamond > heat
7. Amount of gobar gas produced per day = 49.44 kg content of graphite. Hence graphite is more stable.
9. (b) and (c). DH = - 245.5 kJ - ( -286 kJ )
11. (a) because they have different crystal structure.
(b) DG will be –ve. Hence, the process is spontaneous, = 40.5 kJ
(c) A real crystal has some disorder due to presence of 17. (c) DH = H 2 - H1 = (E 2 + P2 V2 ) - (E1 + P1V1 )
defects
12. log Kp = 5.83 × 10–3 = (E 2 - E1 ) + (P2 V2 - P1V1)
14. (i) (a) DS° = –549.74 JK–1 mo1–1 = 30 + 4 ´ 5 - 2 ´ 3 = 44 L atm
(b)DS° = 23.96 JK–1 mol–1
(ii) 65.27 kJ mol–1. 18 . (b) Desired equation is H 2 O (l) ® H 2O(g), DH = ?
15. (i) DfG°H O = – 237.22 kJ mol–1 (Equation II - Equation I)
2 DH = –245.5 kJ – (–286 kJ) = 40.5 kJ
(ii) DrS°= 327 J mol–1 K–1 19. (b) This is electron affinity and exothermic process.
16. (c) 17. (c) 18. (a) 19. (d)
DH 2 - DH1
20. (b) 21. (c) 22. (c) 20. (d) Kirchoff’s equation, = DC P
T2 - T1
EXERCISE 2
21. (d)
1. (c) Closed system can exchange energy and not matter with 22. (a) Hess's law deals with changes in heat of reaction.
surroundings. Pressure cooker provides closed system. 23. (b) Hess’s law
2. (c) Isolated system cannot exchange mass or energy. 24. (b) For a finite change DE = C v DT . For an isothermal
3. (d) Mass/volume = density, hence it is intensive property.
process DT = 0. Hence DE = 0
4. (a) Volume depends upon mass. Hence it is extensive
property. 25. (a) Work done during adiabatic expansion =C V (T2 - T1 )
5. (a) Kinetic gas equation, or -3000 = 20(T2 - 300) T2 = 150K
1 2 2
PV = m nU 2 = KE. For unit volume , P = E 26. (d) Entropy change at constant temperature
3 3 3
V2 20
6. (c) 3H 2O (l) ® 3H 2O (g) ; = 2.303nR log = 2.303´ 2 ´ 2 log = 9.2 cal K -1mol-1
V1 2
2 27. (b) Concept of thermal capacity is, the heat required to raise
Dn = 3, DE = DH - DnRT = 30 - 3 ´ ´ 500 = 27 kcal
1000 the temperature of a body.
7. (d) Gases can undergo expansion hence perform mechanical 28. (b) DG = DH - TDS , DG must be negative hence
work . (PDV work)
DH - TDS must be negative
8. (a) In insulated system no heat or matter is exchanged,
q = 0 DE = W . The work done on the system 29. (a) DG = D H - TDS = - 2.5 ´103 - 298 ´ 7.4 = - ve , hence
increases internal energy. spontaneous.
9. (b) For adiabatic process q = 0 30. (b) q p = DH (Enthalpy)
172 Chemistry
31. (c) Endothermic process, increase in entr opy, 60. (a) CH 4 + 2O2 ® CO2 + 2H 2 O; DH = 20 kcal........(1)
DH = + ve, DS = + ve
C + O 2 ® CO 2 ; DH = -40 kcal.....(2)
32. (a) DG = DH - T DS, DH + ve, DS is + ve; TDS > DH for
spontaneous process. It will make DG, –ve 2H 2 + O 2 ® 2H 2 O ; DH = -20 kcal........(3)
33. (a) Third law of Thermodynamics. (2) + (3) - (1) we have, C + 2H 2 ® CH 4
34. (b) High pressure reduces volume, decreases entropy, hence
or DH = -20 + (-40) - (-20) = - 40 kcal
DS negative.
35. (a) For spontaneous process, DS total is +ve. -194
61. (c) DH for two moles = -194kJ.For one mole = = -97 kJ
36. (c) Third law of thermodynamics is due to Nernst. 2
DH 900 ´18 62. (c) Enthalpy of neutralisation of HCl with NaOH is x. In
37. (a) DS = = = 43.4 J K -1 mol-1 question gev of HCl
T 373
æ 500 ´ 2 ö æ 250 ´ 4 ö
38. (d) DSº = 2Sº HCl - (Sº H 2 +Sº Cl2 ) ç = 1gev ÷ and NaOH ç = 1 gev ÷ hence the
è 1000 ø è 1000 ø
= 2 ´ 186.7 - (130.6 + 223.0) value x.
-1 -1 63. (c) Enthalpy of neutralisation is enthalpy change when 1
= 19.8JK mol
gev of acid is neutralised by 1 gev of base. In question,
2.257 ´18 2 gev of H 2SO 4 is being neutralised. Hence enthalpy of
39. (b) DS = = 0.109 kJ K -1mol -1
373
-114.64
40. (a) DG = DH - TDS neutralisation is = -57.32 kJ
2
41. (c) At equilibrium, DG = 0 64. (a) In question 0.01 gev of NaOH is being neutralised by
42. (d) DG = -RT ln Kp or Kp = e–DG/RTT 0.01 gev of HCl and value is x kJ, for 1 gev the value is -
100x (exothermic process).
43. (d) Condition of equilibrium, hence DG = 0
65. (b) DH = Hº f (products) - H º f (reactants)
44. (a) Statement of third law.
45. (d) Calorific value of fat > carbohydrate and protein; Butter = –110.5 + (–241.8) – (–393.5+0) = 41.2 kJ
34, Sugar 16, Egg 14. 66. (c) Add equation (I) and (II) divide by 2 the equation
46. (d) All values are same . - 196
47. (b) For one mole the value is - 22 kcal mol-1 obtained – 44 + (–152) = = - 98kcal
2
48. (b) C + O 2 ® CO 2 ; DHº = -x kJ.......(1) 67. (a) Lavoisier and Laplace law.
2CO + O 2 ® 2CO2 ; DHº = - y kJ......(2) EXERCISE 3
multiply equation (1) by 2 and substract equation (2) 1. (d) This reaction shows the formation of H2O, and the – X2
and divide final equation by 2 represents the enthalpy of formation of H2O because
49. (b) 57.3 – 55.4 = 1.9 kJ (57.3 kJ is heat of neutralisation of as the definition suggests that the enthalpy of formation
strong acid and strong base) is the heat evolved or absorbed when one mole of
50. (d) It is infact the formation of one molecule of H2O always substance is formed from its constituent elements.
(H + + OH - ¾¾ ® H 2 O) 2. (b)
1 1
H 2 + Cl2 ¾¾ ® HCl
51. (c) Neutralisation of weak acid with strong base hence < x. 2 2
Extra heat is utilised to effect the ionisation of weak acid. D f H HCl = B.E. of reactants
52. (a) Equation (1) – Equation (2) gives
S + O2 ® SO2 , DH = (y - 2x) - B.E. of products
53. (c) It is always = – 57.32 + ionisation of weak acid
1 1
54. (b) Desired eqn; C2 H 4 + 3O 2 ® 2CO 2 + 2H 2 O -90 = ´ 430 + ´ 240 - B.E. of HCl
2 2
DH = 2(-394) + 2(-286) - (52) = -1412kJ B.E. of HCl = 215 + 120 + 90
55. (c) CaO + H 2O ® Ca (OH) 2 ; Exothermic process. = 425 kJ mol–1
3. (d) We know that heat (q) and work (w) are not state
56. (d) Energy of isolated system is constant.
functions but (q + w) is a state function. H – TS (i.e. G)
57. (d)
is also a state function. Thus II and III are not state
58. (a) Depending upon the value of TDS , DG can be lesser,, functions so the correct answer is option (d).
equal or greater to DH 4. (b) For the reaction
59. (b) The more the energy, the lesser is the stability. PCl5 ( g ) ƒ PCl3 ( g) + Cl2 ( g)
Thermodynamics 173
The reaction given is an example of decomposition 11. (a) Fe2O3(s) + 3CO(g) ¾¾ ® 2Fe(s) + 3CO2(g) ;
reaction and we know that decomposition reactions DH = –26.8 KJ ...(i)
are endothermic in nature, i.e., DH > 0. FeO(s) + CO(g) ¾¾ ® Fe(s) + CO2(g) ;
Further DH = –16.5 KJ ...(ii)
Dn = (1+1) – 1= +1 eq. (i) – 2 × eq. (ii), we get
Hence more number of gaseous molecules are present
Fe2O3(s) + CO(g) ¾¾ ® 2FeO(s)+ CO2(g)
in products which shows more randomness i.e., DS >
0 (DS is positive) D H = –26.8 + 33.0 = + 6.2 kJ
5. (c) The reaction for formation of HCl can be written as 12. (a) Ideal gas during spontaneous expansion into
H2 + Cl2 ® 2HCI vacuum does not do any external work.
H – H + Cl – Cl ® 2 (H – Cl) 13. (d) Given DH = 30 kJ mol–1 T = 273 + 27 = 300 K
Substituting the given values, we get enthalpy of DH T 3 ´ 10 4
formation of DST = = J mol–1
T 300
2HCl = ( 676 – 862) = –186 kJ.
= 100 J mol–1 K–1
-186 14. (c) Given
Enthalpy of formation of HCl = kJ = –93 kJ.
2 4H( g ) ¾¾® 2H 2 ( g ); DH = -869.6 kJ
6. (b) DG = DH – T D S
or 2H 2 ( g ) ¾¾
® 4H( g ); DH = 869.6 kJ
At equilibrium, DG = 0
0 = (170 × 103 J) – T (170 JK– 1) H 2 ( g ) ¾¾
® 2H( g ); DH =
869.6
= 434.8 kJ
T = 1000 K 2
For spontaneity, DG is – ve, which is possible only if 15. (b) Given DH
T > 1000 K. 1
7. (b) Enthalpy of reaction A ¾¾®B + 150 ...(1)
2
= B.E(Reactant)– B.E(Product)
3B ¾¾
® 2C + D –125 ...(2)
= éë B.E (C= C) + 4 B.E.(C–H) + B.E.(H - H) ùû E + A ¾¾ ® 2D +350 ...(3)
To calculate DH operate
- éë B.E.(C - C) + 6 B.E.(C - H) ùû
2 × eq. (1) + eq. (2) – eq. (3)
= [606.1 + (4 × 410.5) + 431.37)] – [336.49 + (6 × 410.5)] DH = 300 – 125 – 350 = – 175
= –120.0 kJ mol–1 16. (a) Since, in the first reaction value of Dn is positive.
1 3 1 1
8. (a) ΔS for the reaction X 2 + Y2 ƒ XY3 Dn = 1 - = hence entropy will increase i.e.,
2 2 2 2
Ds = +ve.
ΔS = 50 – (30 + 60) = – 40 J
For equilibrium DG = 0 = DH – T DS C (gr.) + 1 O2(g) ® CO(g); DS° = + ve
DH -30000 2
T= = = 750 K Since, DG° = DH° – TDS hence the value of DG
DS -40 decreases on increasing temperature.
1atm
9. (d) H 2O ( l) ‡ˆˆˆ
ˆˆˆˆ†
ˆ H2O(g) DH 1.435 ´ 103
17. (c) DS = =
DH = 40630 J mol –1 T 273
DS = 108.8 JK–1 mol –1 = 5.260 cal / molK
DG = DH - T DS When DG = 0,
18. (d) H 2O (l) H 2O (g) DH = 40.66 KJmol–1
DH - T DS = 0 T = 373 K
DE = DH – DnRT
DH 40630 J mol-1
T = DS = 108.8 Jk -1 mol-1 = 373.4 K. = 40.66×103 Jmol–1–1× 8.314JK–1mol–1× 373 K
D E = 37558 J / mol
Correct choice : (d) D E = 37.56 kJ mol–1
10. (d)
(1) Kp > 0 (iv) Spontaneous 5
CP 2 R 5
(2) D G < R T ln Q (i) Non 19. (d) = = = 1.67
spontaneous CV 3 3
R
(3) Kp = 0 (ii) Equilibrium 2
DH 20. (b) DH = Ea - Ea = 0
(4) T> (iii) Spontaneous f b
DS
and endothermic
174 Chemistry
21. (b) At low temperature the DS is – ve which makes DG 34. (c) In a reversible process the work done is greater than in
positive (DG = DH – TDS). But at higher temperature DS irreversible process. Hence the heat absorbed in
is +ve which makes the DG negative (condition for reversible process would be greater than in the latter
spontaneity) case. So
22. (d) It violate 2nd law of thermodynamics, not 1st. Tf (rev.) < Tf (irr.)
23. (d) DH negative shows that the reaction is spontaneous. 35. (c) The standard enthalpy of formation of CH4 is given by
Higher value for Zn shows that the reaction is more the equation :
feasible. C (s ) + 2 H 2 (g ) ¾
¾® CH 4 (g )
24. (c) For a cyclic process the net change in the internal energy Hence, dissociation energy of hydrogen and enthalpy
is zero because the change in internal energy does not of sublimation of carbon is required.
depend on the path. 36. (a) I 2 (s) + Cl 2 (g) ¾¾
® 2ICl(g)
25. (c) CH2 = CH2 (g) + H2 (g) ® CH3 - CH3 (g) DH = [DHI (s) –––®I (g) + DHI–I + DHCl–Cl ]
D H = 1(C = C) + 4 (C - H) + 1 (H - H) - 1 (C - C) - 6 (C - H) – [DHlI – Cl]
2 2
= 1 (C = C) + 1 ( H - H) - 1 (C - C) - 2 (C - H) = 151.0 + 242.3 + 62.76 –2 × 211.3 = 33.46
= 615 + 435 - 347 - 2 × 414 = 1050 - 1175 = – 125 kJ.
26. (a) For spontaneous reaction, dS > 0 and DG and DG should 33.46
DH of (ICl) = = 16.73 kJ / mol
be negative i.e. < 0. 2
27. (c) DGº = – RT lnKc or – DGº = RT lnKc 1
28. (b) Enthalpy change for a reaction does not depend upon 37. (d) C (s ) + O 2 (g ) ¾
¾® CO (g)
the nature of intermediate reaction steps. 2
DH – DU = DnRT
29. (c) W = -PDV = -10 -5 (1 ´ 10 -2 - 1 ´ 10-3 ) = -900J
1
30. (b) = ´ 8.314 ´ 298 = 1238.78 J mol -1
2
(i) C + O2 CO2 ; DH = -393.5 kJmol-1 38. (a) DG° = DH° – TDS°
For a spontaneous reaction DG° < 0
1
(ii) CO + O 2 CO 2 ; DH - -283.0 kJmol -1 DH °
2 or DH° – TDS° < 0 T>
After, DS°
(i) - (ii),
179.1 ´ 103
We have T> > 1117.9K 1118K
160.2
1 39. (d) Given DH = 41 kJ mol–1 = 41000 J mol–1
C+ O 2 ® CO DH = -110.5 kJmol -1
2 T = 100°C = 273 + 100 = 373 K
31. (d) For spontaneous reaction DG should be negative. n=1
Equilibrium constant should be more than one DU = DH – DnRT = 41000 – (1 × 8.314 × 373)
(DG = – 2.303 RT log Kc, If Kc = 1 then DG = 0; = 37898.88 J mol–1 37.9 kJmol–1
If Kc < 1 then DG = +ve). Again DG = -nFE ocell . Eocell 40. (b) Spontaneity of reaction depends on tendency to
acquire minimum energy state and maximum
must be +ve to have DG –ve. randomness. For a spontaneous process in an isolated
32. (b) DH = DU + DnRT for N 2 + 3H 2 ¾ system the change in entropy is positive.
¾® 2 NH 3
41. (b) The energy involved in the conversion of
Dng = 2 – 4 = – 2
1
D H = D U - 2 RT or DU = D H+ 2R T D U > D H Cl2 ( g ) to Cl–1 (aq) is given by
2
33. (N) X 2 + Y2 ¾¾® 2 XY , DH = 2(–200). 1
Let x be the bond dissociation energy of X2. DH = D diss H Cl2 + D eg H Cl + D hyd H Cl
2
Then Substituting various values from given data, we get
DH = -400 = + -2 = x + 0.5x – 2 x = -0.5x
DH = çæ ´ 240 ÷ö + (–349) + (–381) kJmol –1
x-x y- y x-y 1
400 è2 ø
or x = = 800 kJ mol -1 = (120 – 349 – 381) kJ mol–1 = – 610 kJ mol–1
0.5
(In the question paper, this option was not mentioned. So the
i.e., the correct answer is (b)
answer has been marked ‘N’) 42. (a) Given, for reaction
® H + (aq.) + OH – ( aq.);
(i)H2O (l) ¾¾
CH3 CH3 CH3
DH r = 57.32 kJ
H Cl Cl H H Cl
1
Cl H H Cl H Cl ® H 2 O(l);
(ii) H 2 ( g ) + O 2 ( g ) ¾¾
2
CH3 CH3 CH3 DH r = –286.20 kJ
Thermodynamics 175
For reaction (i)
46. (d) 4NO 2 (g) + O 2 (g) ® 2N 2 O5 (g), D r H = – 111 kJ
+ –
DH r = DH°f (H .aq)+DH°f (OH .aq)–
– 54 kJ
DH°f (H2 O, l) DH'
2N 2 O5 (s)
57.32 = 0 + DH°f (OH – , aq) – DH°f (H 2 O, l) …(iii)
For reaction (ii) – 111 – 54 = D H'
DH r = DH°f (H 2 O, l) – D H' = – 165 kJ
47. (a) Process is isothermal reversible expansion, hence
1 DU = 0, therefore q = – W.
DH°f (H 2 , g) – DH°f (O 2 , g )
2 Since q = + 208 J, W = – 208 J
ˆˆ† B
48. (b) A ‡ˆˆ
–286.20 = DH°f (H 2 O, l)
DG° = D H° – TDS°
On replacing this value in eq. (iii) we have DG° = – 2.303 RTlog10 K
57.32 = DH°f (OH – , aq) – (–286.20) – 2.303 RT log10 K = DH° – TDS°
2.303 RT log10 K = TDS° – DH°
DH°f (OH - , aq) = –286.20 + 57.32 T DS ° - DH ° 298×10 +54.07×1000
log10 K = =
= –228.88 kJ 2.303RT 2.303×8.314×298
43. (b) ® 2NH 3 DH = 2 ´ -46.0 kJ mol–1
N 2 + 3H 2 ¾¾ = 9.998 » 10
49. (a) Since, liquid is passing into gaseous phase so
Let x be the bond enthalpy of N – H bond then
entropy will increase and at 373 K during the phase
[Note : Enthalpy of formation or bond formation
transformation it remains at equilibrium. So, DG = 0.
enthalpy is given which is negative but the given
50. (b) The species in its elemental form has zero standard
reaction involves bond breaking hence values should molar enthalpy of formation at 298 K. At 298K, Cl2 is
be taken as positive.] gas while Br2 is liquid.
DH = Bond energies of reactants – Bond energies
of products 51. (d) (i) 2C(s) + H2(g) ¾¾
® H – C º C – H(g)
2 × – 46 = 712 + 3 × (436) – 6x
– 92 = 2020 – 6x DH = 225 kJ mol–1
6x = 2020 + 92
(ii) 2C(s) ¾¾® 2C(g)
6x = 2112
DH = 1410 kJ mol–1
x = + 352 kJ/mol
1410
44. (b) At equilibrium DG = 0 C( s) ¾¾
® C( g ) DH = = 705kJmol -1
Hence, DG = DH – TeDS = 0 2

DH (iii) H2(g) ¾¾
\ DH = TeDS or Te = ® 2H(g)
DS DH = 330 kJ mol–1
For a spontaneous reaction From equation (i) :
DG must be negative which is possible only if DH – TDS < 0
\ DH < TDS 225 = éë 2 ´ DHC(s) ¾¾
® C( g )
+ 1 ´ BE H – H ù
û
DH - [2 ´ BE C – H + 1 ´ BECº C ]
or T > ; Te < T
DS 225 = [1410 + 1 × 330] – [2 × 350 + 1 × BEC º C]
225 = [1410 + 330] – [700 + BEC º C]
45. (b) C2H5 OH(l) + 3O 2 ( g ) ® 2CO 2 ( g ) + 3H 2O(l) 225 = 1740 – 700 – BEC º C
BEC º C = 815 kJ mol–1
Dng = 2 - 3 = -1 52. (c) The standard enthalpy of the combustion of glucose
can be calculated by the eqn.
DU = DH - Dng RT C6H12O6(s) + 6O2(g) ® 6CO2(g) + 6H2O(l)
DHC = 6 × DHf(CO2) + 6 × DHf (H2O) – DHf [C6H12O6]
8.314 DH° = 6 (–400) + 6(–300) – (–1300)
= -1366.5 - ( -1) ´ ´ 300
103 DH° = –2900 kJ/mol
For one gram of glucose, enthalpy of combustion
8.314
= -1366.5 + (1) ´ ´ 300 2900
103 DH° = – = - 16.11kJ / gm
= – 1366.5 + 0.8314 × 3 = – 1364 .006 kJ 180
176 Chemistry
EXERCISE 4 14. (c) The third law of thermodynamics is about entropy.
15. (c) 16. (a)
1. (c) The criteria for the spontaneity, DS system + DS 17. (d) It is neutralisation of strong acid with strong base.
surroundings, must be + ve 18. (b) Dissolution of KNO3 is endothermic, hence heat is
2. (c) DG = DH – TDS. For spontaneous process DG < 0. To absorbed and cooling is observed.
get DG < 0. we must have DH < 0 ; DS > 0
19. (a) Isobutane + oxygen ® CO 2 + H 2 O
DH fusion 6.02 ´103
3. (d) DS = ; DS = = 22.0 JK -1 mol -1 DH = -2870 kJ mol -1 ......(i)
T 273
4. (c) Examine the chemical equations . If there is decrease in n-butane + oxygen ® CO2 + H 2O
number of moles of gaseous products, the entropy will
decrease and vice versa. DH = -2878 kJ mol -1 ....(ii)
5. (a) Gaseous ions, when dissolved in water, get hydrated (ii) - (i); n-butane – Isobutane,
and heat is evolved (heat of hydration). DH = (–2878 + 2870)
= – 8 kJ mol–1.
Cl - (g) + aq ® Cl- (aq) is such reaction .
DH 2 - DH1
6. (b) Write the proper chemical equations. 20. (d) Apply Kirchoff’s equation = C p - C R = DC p
T2 - T1
C(graphite) + O 2 (g) ® CO2 (g)
21. (b) Bond dissociation energy of PH3(g) = 228 kcal mol–1
DH = -393kJ ........(i)
228
P – H bond energy = = 76 kcal mol-1
1 3
H 2 (g) + O 2 (g) ® H 2 O(l)
2 H H
DH = -286kJ .........(ii)
P P
C2 H 5OH (l) + 3O2 (g) ® 2CO 2 (g) + 3H 2 O(l) H H
....(iii) Bond energy of 4 (P – H) + (P –P) = 355 kcal mol–1
DH = -1360kJ
or 4 × 76 + (P – P) = 355 kcal mol–1
From ( 2 ´ I + 3 ´ II ) – (III) we get P–P bond energy = 51 kcal mol–1
22. (a) DHº solution =U (lattice energy) + DHº hydration
1
2C + 3H 2 + O ® C 2 H 5 OH 23. (b) DH º = DU º + DnRT
2 2
5 ´ 8.314 ´ 298
2(-393kJ) + 3(-286kJ) - (-1360kJ) = -284kJ DHº = -772.7 + = -760.3 kJ mol -1
1000
15
7. (d) C6 H 6 (l) + O2 (g) ® 6CO 2 (g) + 3H 2O (l) 24. (d) H 2SO 4 + 2KOH ® K 2 SO 4 + 2H 2 O
2 98 112 174 2 mole
56
Dn = 6 - 15/2 = –3/2 49 87 1 mole
8. (c) C is incorrect ; The correct is DE = Dq + Dw 13.7 kcal is the heat evolved when 1 gev of strong acid is
9. (b) DG =DH – TDS neutralised by 1 gev of strong base.
DG = 9100 - 373.2 ´ 25.5 = -416.6 cal mol–1 25. (a) The first reaction is exothermic and the second reaction
10. (a) I – II, gives, 2Al + Fe2O3 ® 2Fe + Al2O3 is endothermic. If on passing the mixture of O 2 and
we have -400 - (-200) = -200; For one mole of iron H 2 O (steam) over coke while keeping the temperature
the value is –100 cal constant DH of both the reactions must be same.
11. (c) Moles of O 2 needed to evolve 132 kJ
12. (b) Enthalpy change = ( Energy required for breaking of
bonds – Energy evolved by formation of bonds) 0.5 ´ 132
= = 0.6
H H H H 110
H–H+ C=C H C–C H Hence steam : O 2 ratio must be 1 : 0.6
H H H H 1

( )- ( )
26. (d) H 2 (g) + O 2 (g ) ¾
¾® H 2 O (l);
H -H 103 80 C -C 2
C = C 145 2´99 C -H = -30 kcal mol–1
DH = – 285.8 kJ .... (i)
13. (d) DG = DH – TDS ; At equilibrium DG = 0 ; D =TDS ;
H + (aq) + OH (aq) ¾
¾® H 2 O (l);

DH
T= DH = – 57.3 kJ ... (ii)
DS
1
45000 ¾® H + (aq) + e - ; DH = 0
H 2 (g) + aq ¾
T= = 375K 2
1.2 ´ 10 2 (by convention) .... (iii)
Thermodynamics 177
(i) – (ii) – (iii) gives,
33. (a) CuSO 4 (s) + 5 H 2 O (g) ¾
¾® CuSO 4 .5 H 2 O (s);
1 1
H 2 (g ) + O 2 (g ) + e - + aq ¾
¾® OH - (aq) DH = – 71.5 kcal ........ (i)
2 2
5 H 2 O (l) ® 5 H 2 O (g) ;
DH = – 285.8 + 57.3 = – 228.5 kJ
27. (b) For 5 moles of gas at temperature T, DH = 5 × 10.5 k cal = 52.5 kcal ........ (ii)
(i) + (ii),
PV1 = 5RTT
For 5 moles of gas at temperature T – 2, CuSO 4 (s) + 5 H 2 O (l) ¾
¾® CuSO 4 .5 H 2 O (s)

PV2 = 5R(T – 2) DH = – 71.5 + 52.5 = – 19.0 kcalmol -1


P (V2 - V1 ) = 5R(T – 2 – T); 34. (a) DH hyd. = DH sol. – DH lattice
PDV = – 10R, = 1 – 180 = – 179 kcal mol -1
– PDV = 10R
When DV is negative, W is + ve. Then DH hyd. ( Na + ) + DH hyd. (Cl - ) = – 179
28. (d) P1V1 = P2 V2 2
or DH hyd. ( Na + ) + hyd.( Na + ) = – 179
3
5bar ´ 2dm 3
V= = 10 dm 3
1bar or DH hyd. ( Na + ) = – 107.4 kcalmol -1
Work, W = – Pext ( (Vfinal - Vinitial ) 35. (a) D C p = 2 × 25.1 + 3 × 75.3 – [103.8 + 3 × 28.8]
= – 1 bar ( 10 – 2) = 85.9 J K -1mol -1
5 -3 3
= – 1 × 10 Pa × 8 × 10 m
DH 2 - DH1
= – 800 J = DC p ;
T2 - T1
29. (c) Ozone is allotropic form of oxygen and is of higher energy
(by 68 kcal mol -1 ) than O 2 . Hence it cannot be taken DH 2 - ( -33290)
= 85.9
as the reference a standard state. 358 - 298

1 1 DH = – 28136 J mol -1
30. (a) H2 + X2 ¾ ¾® HX
2 2 = – 28.136 kJ mol -1
Let the bond enthalpy of X – X bond be x.
1 3
1 1 36. (d) N 2 (g) + F2 (g) ¾
¾® NF3 ; DH = – 113 kJ
DH f (HX ) = – 50 = DH H– H + DH X - X – DH H - X 2 2
2 2
3
1 1 -x or DH N º N + DH F- F – 3 DH N - F = – 113 kJ
= 2x + x – 2x = 2
2 2 2
Let x kJ mol-1 be the bond energy of F – F bond then
-1
x = 50 × 2 = 100 kJ mol bond energy of N º N bond = 6x

31. (c) H + (aq) + OH - (aq) H 2 O (l); DH = – 57.3 kJ ... (i) 1 3


× 6x + × x – 3 × 273 = – 113 kJ
2 2
H2C2O4+2OH– C 2 O -4 - +2H2O; DH =–106 kJ On solving,
...... (ii) x = 156.9 kJ mol-1 and
For the second reaction the value should have been
2 × (– 57.3) = – 114.6 kJ N º N bond energy = 6 × 156.9 = 941.4 kJ mol-1

The difference (114.6 – 106) = 8.6 kJ mol-1 is used to 37. (b) DH N for strong base and strong acid
-1
effect of the ionisation of oxalic acid. = – 13.7 kcal eq
32. (a) After the event the energy remained in the body of the
DH ion (CH 3COOH )
1560
athlete =
2
= 780 kJ = – 12.5 – (– 13.7) = 1.2 kcal mol-1
Weight of water to be evaporated by 780 kJ of energy DH ion ( NH 4 OH)
18 = – 10.5 – (– 13.7) – D H ion (CH 3COOH )
= × 780 = 319.1 g
44 = 13.7 – 10.5 – 1.2
= 2 kcalmol -1
178 Chemistry

38. (a) CH 4 (g) + 2 O 2 (g) ¾


¾® CO 2 (g) + 2 H 2 O (l) DH (ionisation ) = 1.5 kJ mol-1
DH = – 890 kJ .... (i) Hence % ionisation in 1 M solution
2 H 2 O (l) ¾
¾® 2 H 2 O (g);
(1.5 - 1.2)
DH = 2 × 40.5 = 81 kJ .... (ii) = × 100 = 20
1.5
From (i) + (ii), we get
43. (b) Atomisation of methane
CH 4 (g) + 2 O 2 (g) ¾
¾® CO 2 (g) + 2 H 2 O (g)
DH = – 890 + 81 = – 809 kJ CH 4 (g) ¾
¾® C(g) + 4H(g); DH = 360 kcal
39. (c) By bomb calorimeter we get DE .
360
C – H bond energy = = 90 kcal mol -1
¾® 12 CO2 ( g ) + 6 H 2 O (l)
2C 6 H 6 (l) + 15 O 2 (g) ¾ 4
DH – DE = DnRT
= (12 – 15) × 8.314 × 300 = – 7.483 kJ ¾® 2C(g) + 6H(g); DH = 620 kcal
C 2 H 6 (g) ¾
40. (a) The greater the (negative value) of heat of neutralisation,
or H C -C + 6 H C -H = 620
the more is the strength of the acid. Hence,
HCOOH > CH 3COOH > H 2S > HCN H C- C = 620 – 6 H C -H
= 620 – 6 × 90 = 80 kcal
41. (b) H 2 O (l) ¾
¾® H 2 O (g) DH = 40.8 kJ mol-1
44. (a) For non spontaneous reaction
Hence for
-1
DG = + ve
¾® H 2 O (l) DH = – 40.8 kJ mol
H 2 O (g) ¾
DG = DH – T DS and
1 .8
DH = – 40.8 × = – 4.08 kJ DS = 121 J K -1
18

DH 4.08 ´103 For DG = + ve


DS = =– = – 10.93 J K -1 DH has to be positive. Hence the reaction is endothermic.
T 373.15
42. (b) The enthalpy of ionisation of weak acid is given by The minimum value of DH can be obtained by putting
DH ion ( HA) DG = 0
= DH N( weak acid / strong base) – DH N(strong acid / strong base) DH = TDS = 298 × 121 J
= 36.06 kJ
= – 56.1 – (– 57.3) = 1.2 kJ mol-1
7A
Chemical Equilibrium

REVERSIBLE REACTIONS : (v) It can be achieved in a closed container.


Reactions which do not always proceed to completion and may (vi) The observable properties of the process become constant
be made to proceed in the opposite direction under suitable and remain unchanged.
conditions are called reversible reactions e.g. EQUILIBRIUM STATE AND FREE ENERGY CHANGE:
3Fe + 4H2O Fe3O4 + 4H2 At equilibrium DG is equal to zero and we have
DG = DH – TDS \ DH = TDS
H2 + I2 2HI LAW OF MASS ACTION :
N2O4 2NO2 It was put forward by Guldberg and Waage. The law states that
the rate at which a substance reacts is directly proportional to its
N2 + 3H2 2NH3
active mass and the rate of a chemical reaction is directly
IRREVERSIBLE REACTIONS : proportional to the product of the active masses of the reacting
Reactions which always proceed to completion in one direction substances. For a general reaction
only are called irreversible reactions. aA + bB cC + dD
AgNO 3 + HCl ¾¾ ® AgCl + HNO 3 üï Rate of forward reaction µ [A]a [B]b = K f [A]a [B]b
ý Precipitation
BaCl 2 + Na 2 SO 4 ¾¾
® BaSO 4 + 2NaCl ïþ
Rate of backward reaction µ [C]c [D]d = K b [C]c [D]d
NaOH + HCl ¾¾
® NaCl + H2 O Neutralisation where Kf and Kb are velocity constants for forward and backward
reactions respectively. At equilibrium point,
MnO Rate of forward reaction = Rate of backward reaction
2KClO 3 ¾¾ ¾
¾2 ® 2 KCl + 3O
2 Decomposit ion

CHEMICAL EQUILIBRIUM : K f [ A ]a [ B ]b = K b [C ]c [ D ]d
When a reversible reaction is carried out in a closed vessel a
Kf [C ]c [ D ]d
stage reached when the speed of the forward reaction equals the = Kc =
speed of the backward reaction and chemical equilibrium is said Kb [A]a [B]b
to be established. Kc is called the equilibrium constant.
CHARACTERISTICS OF CHEMICAL EQUILIBRIUM : FACTORS INFLUENCING EQUILIBRIUM
(i) Equilibrium can be attained from either side. CONSTANT:
(ii) Equilibrium is dynamic in nature i.e. at equilibrium, reaction (i) The equilibrium constant is not influenced by :
does not stop. (1) Concentration of reactants and products.
(iii) At equilibrium there is no change in the concentration of (2) Presence of a catalyst.
various species. (3) Pressure.
(iv) The equilibrium state remains uneffected by the presence of (4) Presence of inert materials.
catalyst. Catalyst helps to attain the equilibrium state rapidly. (5) The direction from which the equilibrium state is reached.
180 Chemistry
(ii) The equilibrium constant is influenced by : RELATION BETWEEN Kc AND Kp FOR DIFFERENT
(A) Temperature : The variation of equilibrium constant is given TYPES OF REACTIONS :
by Van't Hoff equation.
(i) When Dn = 0, Kp = Kc e.g. for reaction A B.
(K p ) 2 DH æ T 2 - T1 ö [H2(g) + I2(g) 2HI(g)]
log = ç ÷
(K p ) 1 2.303R çè T1T 2 ÷ø
(ii) When Dn = +ve, Kp > Kc e.g. for reaction A 2B.
where DH = Enthalpy change , (K p ) 1 and (K p ) 2 = [PCl5(g) PCl3(g) +Cl2(g)]
Equilibrium Constant at temperature T1 & T2 (iii) When Dn = –ve, Kp < Kc e.g. for reaction 2A B.
R = Universal gas Constant
[N2(g) +3H2(g) 2NH3(g)]
For exothermic reaction : Kp decreases with increase of temperature
æ Kf ö THE UNITS OF Kp AND Kc :
since Kf decreases. çç K p = K ÷÷ (i) Unit of Kp = (atm)Dn
è bø
For endothermic reaction. Kp increases with increase of (ii) Unit of Kc = (mol lit–1)Dn
temperature since Kf increases. CHARACTERISTICS OF EQUILIBRIUM CONSTANT :
(B) The mode of representing the reaction :
1. It has definite value for every chemical reaction at a particular
The reaction A + B C + D may be written as temperature
C+D A+ B 2. The more is the value of Kc or Kp, the more is the completion
of reaction or the more is the concentration of products.
K f [C][D] 3. When the reaction can be expressed as sum of two other
Kc = =
K b [A][B] reactions, the Kc of overall reaction is equal to the product
of equilibrium constants of individual reactions.
K ' f [A][B] 1 Illustration :
K 'c = = \ Kc =
K 'b [C][D] K 'c If K1, K2 and K3 are the quilibrium constants for the reaction
(1), (2) and (3) respectively, then prove that K1×K2=K3.
(C) Stoichiometric representation of equation :
H2S HS– + H+ ...(1)
(I) N2 + 3H2 2NH3
HS– S– – + H+ ...(2)
[ NH 3 ]2
Kc = H2S S– – + 2H+ ...(3)
[ N 2 ][H 2 ]3
[HS- ][H + ] [S-- ][H + ]
1 3 Sol. K1 = K2 =
N2 + H2 NH3 [H 2S] [HS- ]
2 2
[ NH 3 ] [S - - ][H + ]2
K' c = On multiplying K1 ´ K 2 = = K3
1 3 [H 2S]
[N 2 ] 2 [H 2
2]
4. The equilibrium constant is independent of initial
\ K c = K 'c concentrations of reacting species.
5. Independent of presence of catalyst.
USE OF PARTIAL PRES SURE INS TEAD OF 6. Kc for backward reaction is inverse of Kc for forward
CONCENTRATIONS : reaction.
For gaseous reactin g substances partial pressures are 7. Kc changes with stoichiometric representation of
conveniently used since at any fixed temperature partial pressure reaction.
is directly proportional to concentration. For a general reaction Illustration :
aA + bB ƒ cC + dD For the reaction 2SO2 + O2 2SO3 the equilibrium
constant is 49. Find the equilibrium constant for
éPC ù ´ [ PD ]
c d
K p = ë ûa 1
O2
[ PA ] ´ [ PB ]b SO2 +
2
SO3.

RELATION BETWEEN Kc AND Kp : [SO 3 ]2


Sol. 49 = (I)
K p = K c [RT ]Dn [SO 2 ]2 [O 2 ]

where [SO 3 ]
Dn = [moles of products – moles of reactants] gaseous only. Kc = (II)
1
[SO 2 ][O 2 ] 2
Chemical Equilibrium 181
taking under root of (I) PCl5 PCl3 + Cl2
[SO 3 ] a
7= = Kc Initial molar Conc. 0 0
1
[SO 2 ][O 2 ] 2 V

\ Kc for second reaction = 7 a-x x x


Eqb. molar Conc.
HOMOGENEOUS EQUILIBRIUM : V V V
In homogeneous equilibrium the reactants and products are present Applying the law of chemical equilibrium
in the same phase (gaseous or liquid).
x x
´
2SO2 (g) + O2 (g) 2SO3 (g) [PCl 3 ][Cl 2 ] V V x2
Kc = = = .......(i)
HETEROGENEOUS EQUILIBRIUM : [PCl5 ] a-x V(a - x )
In heterogeneous equilibrium the reactants and products are V
present in two or more phases. The eq.(i) contains the V term in denominator . If volume increases,
3Fe(s) + 4H2O(g) Fe3O4 (s) + 4H2 (g) the dissociation of PCl5 must also increase to keep Kc constant.
The decrease of pressure will cause increase in volume and so the
CHEMICAL EQUILIBRIUM APPLIED TO dissociation.
HOMOGENEOUS SYSTEM :
(a) Gaseous system : They are of two types x2
If the value of x is small then K c = V ; x µ V µ 1/ p
(i) Gaseous reactions in which the number of moles of
products remain the same as that of reactants. (b) Liquid system : Examples are :
H2(g) + I2(g) 2HI(g) (i) Esterification of acetic acid
N2(g) + O2(g) 2NO(g) CH3 COOH + C2 H5OH CH3COOC2 H5 + H2O
H2(g) + CO2(g) CO(g) + H2O(g) At equilibrium 2/3rd of acetic acid is converted into ester.
Hydrogen - iodine equilibrium : Suppose, a moles of H2 and b Hence alcohol consumed will also be 2/3rd.
moles of I2 are present in a container of V litres. At equilibrium x (ii) Reaction between amylene and tricholoroacetic acid
moles of each have combined to form HI. Cl3C.COOH + C5H10 Cl3CCOOC5H11
H2 + I2 2HI CHEMICAL EQUILIBRIUM APPLIED TO
a b HETEROGENEOUS SYSTEM :
Initial molar conc. 0 (i) Dissociation of calcium carbonate
V V
CaCO3(s) CaO(s) + CO2(g)
a - x b - x 2x Applying the law of chemical equilibrium
Eqb. molar conc.
V V V
p CaO ´ p CO 2
Applying the law of chemical equilibrium Kp =
p CaCO3
2
æ 2x ö The active mass of a solid reactant and product is assumed
ç ÷
[HI ]2 V 4x 2
Kc = = è ø = ..........(i)
to have a constant value and is taken as unity. The equilibrium
[H 2 ][I 2 ] a - x b - x (a - x)(b - x ) constant is determined by gaseous substances only
´
V V
\ K p = pCO2
The equilibrium constant written as Kc indicates that active masses
(ii) Reaction of steam on heated iron
are expressed in terms of molar concentrations.
The eq. (i) does not contain the volume term. Thus equilibrium is 3Fe(s) + 4H2O(g) Fe3O4(s) + 4H2(g)
independent of volume and therefore of pressure. 4
é pFe O ù ´ é pH ù 4
(ii) Gaseous reactions in which the number of moles of products Kp = ë
3 4 û ë 2 û = pH2
4 4
and reactants are different. 3 pH
ëépFe ûù ´ ëé pH2 O ûù 2O

PCl5 PCl3 + Cl2


Partial pressures of solid is taken unity.
N2O4 2NO2 (iii) Water gas reaction :
N2 + 3H2 2NH3 C(s) + H2O(g) CO(g) + H2(g)
2SO2 + O2 2SO3
p CO ´ p H 2
Dissociation of PCl5 : Suppose 'a' moles of PCl5 are present in a Kp =
container of V litres. At equilibrium x moles have dissociated. p C ´ p H 2O
182 Chemistry
Since partial pressure of carbon (solid) is taken as unity, the
Concentrat ion terms of products
equilibrium constant is given by viz.
Concentrat ion terms of reactants
p CO ´ p H 2 Jeq means the partial pressure of the products and the reactants at
Kp =
p H 2O the equilibrium. Hence Jeq can be replaced by Kp.

VAN'T HOFF ISOCHORE : \ DG° = –RT ln K p


A relationship between the equilibrium constant K p , at HENRY'S LAW :
any temperature T and constant pressure P, and heat of reaction The mass of a gas dissolved per unit volume of solvent is
DH°. proportional to the pressure of the gas in equilibrium with the
d log e K p DH° solution at constant temperature.
= • The volume of the gas dissolved remains the same inspite of
RT 2
dT
increase in pressure.
The enthalpy change DH does not vary appreciably with change
• The dissolution of a gas in a liquid is spontaneous process
in partial pressures of reactants and products. Therefore DHº can
(DG = 0), accompanied by decrease in entropy (DS = –ve).
be taken as DH whatever may be the partial pressures of reactants
and products Since DG = DH – TDS, DG can only be negative if DH is –ve.
Therefore dissolution of a gas in a liquid is always exothermic
d log e K p DH in nature.
=
RT 2dT FACTORS ALTERING THE STATE OF EQUILIBRIUM
The integrated form of the equation is - LE CHATELIER'S PRINCIPLE :
Kp There are three main factors which alter the state of equilibrium.
DH é T2 - T1 ù
log 2
= ê ú They are (I) Concentration, (II) Temperature, and (III) Pressure.
K p1 2.303R ë T1T2 û
Le Chatelier's principle states that if a system at equilibrium is
Three important conditions may arise subjected to a change of concentration, pressure or temperature,
(i) when DH = 0 no heat is evolved or absorbed the equilibrium shifts in the direction that tends to undo the effect
Kp Kp
of the change.
log 2
= 0 i.e. 2
= 1 or K p 2 = K p1 (1) Effect of change of concentration :
K p1 K p1
If at equilibrium the concentration of one of the reactants is
Equilibrium constant does not change when no change in increased, the equilibrium will shift in the forward direction
temperature. and vice versa. Consider the following equilibrium
(ii) when DH = +ve i.e. heat is absorbed Fe3+ (aq) + SCN– (aq) [Fe(SCN)]2+ (aq)
Kp Pale yellow Colourless Dark brown
log 2
>1 K p 2 > K p1
K p1 If ferric salt is added the colour of the solution darkens
immediately i.e. Fe 3+ ions are consumed and more
Equilibrium constant increases with increase of temperature. [Fe(SCN)]2+ are formed. If some sulphocyanide salt is added
(iii) when DH = –ve i.e. heat is evolved the colour also darkens. If Potassium ferrisulphocyanide
Kp capable of giving complex ion [Fe(SCN)]2+ is added the
log 2
<1 K p2 < K p1 ; Equilibrium constant colour lightens to pale yellow.
K p1
(2) Effect of change in pressure :
decreases with increase of temperature (a) No effect of pressure on equilibria having same moles of
(iv) when Dn = 0 i.e. there is no change in volume during a reaction reactants and products e.g.
KP = Kc. The variation of equilibrium constant with
(I) N2 + O2 2NO
d log e K c DE
temperature is given by = (II) H2 + I2 2HI
dT RT 2 (b) When there is change in the number of moles the
DE heat of reaction at constant volume. equilibrium will shift in the direction having smaller
VAN'T HOFF REACTION ISOTHERM : number of moles when the pressure is increased and
It gives the free energy change of a reaction at any given vice versa e.g.
temperature, pressure and composition of the reacting system. N2 + 3H2 2NH3
DG = DG° + RT ln J More pressure more ammonia
At equilibrium DG = 0 then DG° = –RT ln Jeq
J stands for reaction quotient of partial pressure of products and PCl5 PCl3 + Cl2
reactants. The more the pressure, the lesser the dissociation
of PCl5.
Chemical Equilibrium 183
(3) Effect of temperature : (2) Synthesis of NO (nitric acid birkland eyde process)
(I) When process is exothermic - Low temperature favours N2 (g) + O2 (g) 2NO(g)– 43.2 kcal
the formation of products.
(I) High temperature
(II) When process is endothermic - High temperature favours
(II) Excess of N2 and O2
the formation of products
(III) No effect of pressure
e.g. N2 + 3H2 2NH3 + 24.0 kcal.
(3) Formation of SO3 (sulphuric acid contact process)
Since the production of NH 3 is exothermic low
2SO2 (g) + O2 (g) 2SO3 + 42.0 kcal
temperature favours its formation.
(4) Effect of addition of inert gas : (I) Low temperature
(I) Addition of Inert gas at constant volume : The total (II) High pressure
pressure of the system is increased, but the partial (III) Excess of SO2 and O2
pressure of each reactant and product remains the same. (4) Formation of nitrogen dioxide
Hence no effect on the state of equilibrium. 2NO + O2 2NO2 + 27.8 kcal
(II) Addition of Inert gas at constant pressure : The total
(I) Low temperature
volume is increased, the number of moles per unit volume
of each reactant and product is decreased. Hence (II) High pressure
equilibrium will shift to the side where number of moles (III) Excess of NO and O2
are increased e.g. (5) Dissociation of nitrogen tetraoxide
PCl5 (g) PCl3 (g) + Cl2 (g) N2O4 2NO2 – 14 kcal
Introduction of inert gas at constant pressure will shift (I) High temperature
the equilibrium to right hand side. (II) Low pressure
(5) Effect of catalyst : The presence of catalyst does not change (III) Excess of N2O4
the position of equilibrium. It simply fastens the attainment (6) Oxidation of CO by steam (Bosch process)
of equilibrium.
CO + H2O CO2 + H2 + x kcal
LE CHATELIER'S PRINCIPLE APPLICABLE TO
PHYSICAL EQUILIBRIUM : (I) Low temperature
(II) Excess of steam and CO
(i) Effect of pressure on solubility : The increased pressure,
will increase the solubility of a gas and vice versa. (III) No effect of pressure
(ii) Effect of temperature on solubility : The substances which (7) Dissociation of PCl5
dissolve with the absorption of heat, their solubility will PCl5 PCl3 + Cl2 – 15 kcal
increase with increase of temperature and vice versa e.g. (I) High temperature
dissolution of NH4Cl, KCl, KNO3 is endothermic which
(II) Low pressure
increases with increase of temperature. The dissolution of
calcium acetate and Calcium hydroxide is exothermic, their (III) Excess of PCl5
solubility is lowered at higher temperature. TRIPLE POINT :
(iii) Effect of pressure on the melting point of ice : The temperature and pressure at which the three states of a
substance can exist in equilibrium is known as triple point e.g.
Ice liquid water
Ice (s) water (l) vapour (g) can exist at 0.0098°C
The ice occupy the more volume than liquid water, so
increased pressure will result in melting of ice according to and 4.58 mm.
Le Chatelier's principle. DEGREE OF DISSOCIATION FROM DENSITY
FAVOURABLE CONDITIONS FOR SOME MEASUREMENT :
IMPORTANT REACTIONS : The density of one mole of gas is given by
(1) Synthesis of ammonia (Haber's process) :
MP
D= where M = Mol. wt of gas; P = Total pressure.
N2 (g) + 3H2 (g) 2NH3 (g) + 22.4 kcal. RT
(I) Low temperature (500°C) The volume of the gas increases on dissociation in proportion to
(II) High pressure (200 – 1000 atm.) increase in the total number of moles, but total weight remains
(III) Excess of N2 and H2 constant. Hence density decreases in the same proportion.
184 Chemistry
Consider dissociation of PCl5. Let x be degree of dissociation
1
PCl5 PCl3 + Cl2 D= Molecular mass
2
1– x x x Total moles (1 + x) d is observed vapour density at temperature tºC.
D 1+ x D-d æ 1 ö If nx moles of products are formed, then total number of moles
\ = \x = çQ D µ ÷ after dissociation
d 1 d è moles ø
1 – x + nx = 1 + x (n – 1)
where D is the theoretical vapour density and
D 1 + x ( n - 1) D-d
\ = \x =
d 1 d ( n - 1)

1. A sample of air consisting of N2 and O2 was heated to 2500K


[I 2 ] = 2 - 0.684 = 1.316 mol lit -1
until the equilibrium
N2 (g) + O2 (g) 2NO (g) [HI ] = 3 + 2(0.684) = 4.368 mol lit -1
was established with an equilibrium constant 3. At 700 K hydrogen and bromine react to form hydrogen
Kc = 2.1 × 10–3. At equilibrium, the mole % of NO was 1.8. bromide. The value of equilibrium constant for this reaction
Estimate the initial composition of air in mole fraction of N2 is 5 × 108. Calculate the amount of H2, Br2 and HBr at
and O2. equilibrium if a mixture of 0.6 mole of H2 and 0.2 mole of Br2 is
Sol. N2 + O2 2NO heated at 700K.
Initial a (100 – a) 0 Sol.
Final (a – x) (100 – a – x) 2x H 2 + Br2 ƒ 2HBr
Given 2x = 1.8 \ x = 0.9. Initial moles 0.6 0.2 0
2 2 At equilibrium 0.6–x 0.2–x 2x
[NO] (2x)
Kc = =
[N 2 ][O2 ] (a - x)(100 - a - x)
\ Kc =
( 2x )2
On substituting the value of x in the given eqn. ( 0.6 - x ) ( 0.2 - x )
( 2 ´ 0.9 )2 4x 2
Kc = = 2.1´ 10-3 or 5 ´108 =
( a - 0.9 ) (100 - a - 0.9 ) ( 0.6 - x ) ( 0.2 - x )
On solving the value of a, we get
a = 79%; 100 – a = 21% (
or 4x 2 = 5 ´108 x 2 - 0.8x + 0.12 )
2. One mole of H2, two moles of I2 and three moles of HI are
On solving the equation we get
injected in one litre flask. What will be the concentration of
H2, I2 and HI at equilibrium at 500°C. Kc for reaction. x = 0.6 or 0.2
H2 + I2 2HI is 45.9. But x can't be exceed than 0.2 because initial moles of Br 2 is
0.2
Sol. H2 + I2 2HI
\ x = 0.2
Initial mole 1 2 3
Thus, at equilibrium
Mole at equilibrium (1 – x) (2 – x) (3 + 2x)
[ H2 ] = 0.6 - 0.2 = 0.4 mole
[HI]2 (3 + 2x ) 2
Kc = = = 49.5 On solving x = 0.684
[H 2 ][I 2 ] (1 - x )(2 - x ) [ Br2 ] = 0.2 - 0.2 = 0.0 mole
\ [H 2 ] = 1 - 0.684 = 0.316 mol lit -1 [ HBr ] = 2 ´ 0.2 = 0.4 mole
Chemical Equilibrium 185
4. At 540 K 0.10 mole of PCl5 are heated in a 8 litre flask. The
pressure of the equilibrium mixture is found to be 1.0 atm. 0.158 0.158
p PCl 5 ´ p Cl2 ´1´ ´1
Calculate Kp and Kc for the reaction. Kp = = 1.158 1.158 = 0.0256
p PCl 5 0.847
Sol. PCl5 PCl3 + Cl2 1.158
Moles before dissociation 0.1 0 0 7. The degree of dissociation of PCl5 at 50 atm is 0.186. The
Moles after dissociation 0.1 – x x x degree of dissociation at 200 atm will be?
Given volume of container = 8 litre
1
Sol. Degree of dissociation x µ vµ
x x p
.
[PCl3 ][Cl 2 ] 8 8 = x2
Now Kc = = (I)
[PCl5 ] 0.1 - x 8(0.1 - x ) x1 p2 0.186 200
8 = \ =
x2 p1 x2 50
Also PV = nRT for the equilibrium mixture at 540 K
1 × 8 = (0.1 + x) × 0.082 × 540 (II) 0.186
\ x2 = = 0.093
On solving the equation we get 2
8. 2.7 ml H2 and 3.1 ml I2 vapour react-at 444°C to form 4.5 ml HI.
x = 0.08 Calculate the Kc at this temperature
Now substitute the value of x in eqn (i)
Sol. H2 + I2 2HI
0.08 ´ 0.08 Initial moles 2.7 3.1 0.0
Kc = = 4 ´ 10- 2 mol lit -1 ; Kp = Kc (RT)Dn Moles at equil. 2.7– x 3.1– x 2x
8(0.1 – 0.08)
But 2x = 4.5 (given) \ x = 2.25
[H2] = 2.7 – 2.25 = 0.45, [I2] = 3.1 – 2.25 = 0.85
= 4 × 10–2 (0.082 × 540) = 1.77 atm [ where Dn = 2 - 1 = 1]
5. The equilibrium constant Kp for the reaction [HI]2 (4.5) 2
Kc = = = 52.94
N2 + 3H2 2NH3 is 1.64 × 10–4 atm at 400°C. What [H 2 ][I 2 ] 0.45 ´ 0.85
will be the equilibrium constant at 500°C. If the heat of reaction 9. A mixture of SO2, SO3, and O2 gases present in a 10 litre
in this temperature ranges –105185.8 J. flask at certain temperature at which Kc for the reaction
Sol. Van't Hoff equation is 2SO2 + O2 2SO3 ; is 100
(i) If the No. of moles of SO2 and SO3 in the flask are
K p '' DH æ T2 - T1 ö equal, how many moles of O2 are present.
log =
Kp ' 2.303R çè T1T2 ÷ø (ii) If the No. of moles of SO3 in the flask is twice the No.
of moles of SO2, how many moles of O2 are present.
DH æ T2 - T1 ö Sol. (i) 2SO2 + O2 2SO3
log K p '' = log K 'p +
2.303R çè T1T2 ÷ø 2
é1ù
[SO3 ]2 ê10 ú
-4 - 105185.8 æ 773 - 673 ö ë û
= log 1.64 ´ 10 + ç ÷ Kc = = = 100 ;
2.303 ´ 8.314 è 773 ´ 673 ø [SO2 ] [O2 ] é 1 ù 2 é O 2 ù
2

ê 10 ú ê 10 ú
On solving we get K''p = 0.144 × 10–4 atm. ë û ë û
6. The vapour density of PCl5 when in equilibrium with its \ [O2] = 0.1 mole
dissociation products was found to be 90. Calculate its degree 2
of dissociation. Also calculate the dissociation constant of é2ù
ê10 ú
PCl5 at this temperature if the pressure is in atmosphere. ë û = 100 ;
(ii) 2
é 1 ù é O2 ù
31 + 177.5 ê 10 ú ê 10 ú
Sol. Theoretical density D = = 104.25 ë û ë û
2
Observed density d = 90 [O2] = 0.4 mole
10. 1 mole of N2 is mixed with 3 moles of H2 in a 4 litre container.
D-d 104.25 - 90 If 0.25% N2 is converted into NH3 according to the equilibrium
Degree of dissociation x = = = 0.158
d(n - 1) 90 N2 + 3 H2 2 NH3. Calculate Kc. What will be the value of
Kp can be calculated as PCl5 PCl3 + Cl2 K 'c for the given reaction.
1 – 0.158 0.158 0.158
Total moles = 1 – 0.158 + 0.158 + 0.158 = 1.158 1 3
N2 + H2 NH3
2 2
186 Chemistry

Sol. N2 + 3H2 2 NH3 Sol. K p = K c (RT ) Dn = 0.50(0.0821´ 673) -2 = 1.64 ´ 10 -4


1 3 0 Initial moles
1 – 0.0025 3 – 0.0075 0.005 Moles at- 14. Prove that the pressure necessary to obtain 50% dissociation
equilibrium of PCl5 at 250°C is numerically three times of Kp
(0.9975) (2.9975) (0.005) Sol. PCl5 PCl3 + Cl2
Molar conc. at eqb.
4 4 4 1–0.5 0.5 0.5 Moles at equilibrium
[ NH 3 ]2 [0.00125]2 Total moles at equilibrium = 1– 0.5 + 0.5 + 0.5 = 1.5
Kc = = = 1.49 ´ 10 -5
[ N 2 ][H 2 ]3 [0.2494][0.748]3 0.5 0.5
p PCl ´ p Cl p´ p
1 3 Kp = 3 2
= 1.5 1.5
For N2 + H2 NH3 p PCl 0.5 ( where p is total pressure)
2 2 5 p
1.5
[ NH 3 ]
K 'c = = K c = 1.49 ´10 -5 = 3.84 ´10 -3
½
[ N 2 ] [H 2 ] 3/ 2 1
Kp = p or p = 3K p
3
11. At certain temperature the Kc is 16 for the reaction
15. An equilibrium mixture at 300 K contains N2O4 and NO2 at
SO2 + NO2 SO3 + NO 0.28 and 1.1 atm pressure respectively. If the volume of the
If we take one mole each of the four gases in one litre container is doubled, calculate the new equilibrium pressure
container, what would be equilibrium concentrations of NO of the two gases
and NO2
Sol. N2O4 2NO2
Sol. SO2 + NO2 SO3 + NO 0.28 atm 1.1 atm at equilibrium
1 1 1 1 Initial moles
1– x 1–x 1+ x 1 + x Molar conc. (1.1) 2
\ Kp = = 4.32
at equilibrium 0.28

[SO 3 ][ NO] (1 + x )(1 + x ) when the volume of the container is doubled, the pressure of
Kc = = 16 ; = 16 N2O4 and NO2 shall be reduced to half
[SO 2 ][ NO 2 ] (1 - x )(1 - x)
N2O4 2NO2
3 0.14 0.55
x = = 0.6
5 (0.14 –x ) atm (0.55 + 2x) atm At new equilibrium
\ Concentration of NO = 1 + x = 1 + 0.6 = 1.6 mole
(0.55 + 2x ) 2
Concentration of NO2 = 1– x = 1– 0.6 = 0.4 mole. Kp = = 4.32
0.14 - x
12. Kc for PCl5 PCl3 + Cl2 is 0.04 at 25°C. How many moles
of PCl5 must be added to a 3 litre flask to obtain a Cl 2 of \ x = 0.045
concentration 0.15 M. Since temperature remains the same Kp will not change
Sol. At equil. No. of mole of Cl2 in 3L = 0.15×3 = 0.45 \. p = 0.14 - 0.045 = 0.095 atm
N 2O 4
PCl5 PCl3 + Cl2
x 0 0 Initial moles
and p NO2 = 0.55 + 2 ´ 0.045 = 0.64 atm
x – 0.45 0.45 0.45 Moles at eqb.
( x - 0.45) (0.45) (0.45) 16. The value of Kp at 298 K for the reaction
Molar conc. at eqb.
3 3 3 1/ 2 N 2 + 3/ 2 H 2 NH3
[ PCl 3 ][Cl 2 ] (0.15)(0.15) is found to be 826.0, partial pressures being measured in
Kc = =
[ PCl 5 ] ( x - 0.45) atmospheric units. Calculate DG ° at 298K
3
Sol. D G ° = -2.303 RTlogK p
\ x = 2.1 mole
13. The value of Kc is 0.50 for the reaction = -2.303 ´ 1.98 ´ 298 log 826 = -3980 calories
N2 + 3H2 2NH3 17. If 50% of CO2 converts to CO at the following equilibrium
at 400°C. Calculate the value of Kp for the given reaction at C (s) + CO2 (g) 2CO (g)
the same temperature (R = 0.0821 atm litre/ deg. mol) and the equilibrium pressure is 12 atm calculate Kp .
Chemical Equilibrium 187

Sol. C(s) + CO 2 (g) ƒ 2 CO(g) 1.84 ´ 0.0821´ 348


1=
Initial mole 1 0 92
mole at eqb. 1–0.5 (1 + x )
2 ´ 0.5 total mole = 1.5
\ x = 0.75
0.5 1
p p (2x ) 2 4 ´ (0.75) 2
1.5 1.5 Kc = = = 9.0
1- x 1 - 0.75
0.5 1 19. Calculate Kp for the following reaction at 25°
partial pressure = ´ 12 atm = ´ 12 atm
1.5 1.5
2N2O (g) 2N2 (g) + O2 (g)
= 4 atm = 8 atm
DG°f ( N 2 O) = 104.2 kJ / mole, DG °f ( N 2 ) = 0 and
2
p2
\ K p = CO =
( 8) = 16 atm DG °f (O 2 ) = 0 .
p CO2 4

18. The density of an equilibrium mixture of N2O4 and NO2 at 1 Sol. DG° = 2 ´ DG °f (N 2 ) + DG °f (O 2 ) - 2 ´ DG °f (N 2O)
atm and 348K is 1.84 g/c. Calculate Kc for the equilibrium
N2O4 (g) 2 NO2 (g) = (2 ´ 0 + 0) - 2 ´104.2 = -2.084 ´105 J / mol
Sol. N2O4 2 NO2 For gas phase reaction DG° is related to Kp as follows
1–x 2x
DG° = -2.303 RT log K p
92(1 - x) + 46 ´ 2x 92
Molecular wt of mixture = =
1+ x 1+ x
- 2.084 ´105 = -2.303 ´ 8.314 ´ 298 log K p
dRT
Further P =
M \ K p = 3.3 ´1036
188 Chemistry

Very Short/Short Answer Questions If a mixture of 0.482 mol N2 and 0.933 mol of O2 is
placed in a reaction vessel of volume 10L and allowed
1. In a reaction a + b ƒ c + d the change in enthalpy is –10 to form N2O at a temperature for which Kc = 2.0 × 10–37.
kcal. If heat is added to the system in which direction the Determine the composition of the reaction mixture.
reaction would proceed? (ii) Bromine monochloride, BrCl, decomposes into bromine
2. What is meant by the statement ‘Equilibrium is dynamic in and chlorine and reaches the equilibrium
nature’? ˆˆ† Br2(g) + Cl2(g)
2BrCl(g) ‡ˆˆ
3. A + B ƒ AB; K = 1 ´ 102 for which Kc = 32 at 500 K. If initially pure BrCl is present
at a concentration of 3.30 × 10–3 mol L–1; what is its
E + F ƒ EF; K = 1 ´ 10 -3 molar concentration in the mixture at equilibrium?
Out of AB and EF, which one is more stable AB or EF ? 13. (i) How much of 0.3 M ammonium hydroxide should be
4. Which will have CO2 to more extent, hot cold drink bottle or mixed with 30 mL of 0.2 M solution of ammonium
chilled cold drink bottle, why ? chloride to give buffer solution of pH 8.65 and 10?
5. Write the expression for equilibrium constant Kp, for the [pKb = 4.75]
reaction. (ii) How much volume of 0.1 M HAc should be added to
ˆˆ† Fe3O4(s) + 4H2(g)
3Fe(s) + 4H2O(g) ‡ˆˆ 50 mL of 0.2 M NaAc solution if we want to prepare a
buffer solution of pH 4.91. (pKa = 4.76)
6. In a reaction a + b ƒ c + d will addition of c to the system
change the value of K? Multiple Choice Questions
7. The equilibrium constant for a gas phase reaction is 14. Which of the following is not a general characteristic of
4 equilibria involving physical processes ?
[CS2 ][H 2 ]
Kc = (a) Equilibrium is possible only in a closed system at a
[CH 4 ][H 2S]2 given temperature.
Write the balanced chemical equation corres- ponding to (b) All measurable properties of the system remain
this expression. constant.
8. In a gaseous reaction a + b ‡ˆˆˆˆ† c + 2d, 3 mol of inert gas (c) All the physical processes stop at equilibrium.
is introduced into the vessel at constant pressure. What (d) The opposing processes occur at the same rate and
will be its effect on equilibrium? there is dynamic but stable condition.
9. For the reaction 2a ƒ c + d , the equilibrium constant is 15. A reaction is A + B C + D. Initially we start with
1.0 × 10–3 . At equilibrium, the concentrations observed were equal concentrations of A and B. At equilibrium we find that
[c] = 1.2 × 10–3 M, [d] = 3.8 × 10–6 M. the moles of C is two times of A. What is the equilibrium
What was [a] at equilibrium? constant of the reaction?
10. On the reaction 2NH3 (g) ‡ˆˆ ˆˆ† N (g) + 3H (g) 1 1
2 2
DH = + 93.6 kJ. Tell us what would be effect of (a) (b)
4 2
(a) temperature (c) 4 (d) 2
(b) pressure 16. In the two gaseous reactions (I) and (II), at 25°C
11. The equilibrium constant for the following reaction is 1.6 ×
105 at 1024 K (I) NO (g) + 1 2 O 2 (g ) NO2 (g), K1
ˆˆ† 2HBr(g)
H2(g) + Br2(g) ‡ˆˆ
(II) 2NO2 (g) 2NO (g) + O2 (g), K2
Find the equilibrium pressure of all gases if 10.0 bar of HBr
is introduced into a sealed container at 1024 K. the equilibrium constants K1 and K2 are related as
1 K1
Long Answer Questions (a) K 2 = (b) K2 =
K1 2
12. (i) Reaction between nitrogen and oxygen takes place as
following: 1
(c) K 2 = (d) K 2 = K12
ˆˆ† 2N2O(g)
2N2(g) + O2(g) ‡ˆˆ K12
Chemical Equilibrium 189
17. For the reversible reaction, 21. On increasing the pressure, in which direction will the gas
N2(g) + 3H2(g) 2NH3(g) at 500°C, the value of Kp is phase reaction proceed to re-establish equilibrium, is
predicted by applying the Le Chatelier’s principle. Consider
1.44 ´ 10 -5 when partial pressure is measured in the reaction.
atmospheres. The corresponding value of K C, with
concentration in mole litre–1, is N 2 (g) + 3H 2 (g) ƒ 2NH3 (g)
Which of the following is correct, if the total pressure at
1.44 ´10 -5 1.44 ´10 -5
(a) (b) which the equilibrium is established, is increased without
(0.082 ´ 500)- 2 (8.314 ´ 773)- 2 changing the temperature?
(a) K will remain same
1.44 ´10 -5 1.44 ´10 -5 (b) K will decrease
(c) (d)
(0.082 ´ 773)2 (0.082 ´ 773)-2 (c) K will increase
(d) K will increase initially and decrease when pressure is
ˆˆ† 2CO(g) , the partial
18. For the reaction C(s) + CO2 (g) ‡ˆˆ very high
22. In which of the following reactions, the equilibrium remains
pressures of CO2 and CO are 2.0 and 4.0 atm respectively at
unaffected on addition of small amount of argon at constant
equilibrium. The Kp for the reaction is.
volume?
(a) 0.5 (b) 4.0
(a) H2 (g) + I2 (g) ƒ 2HI (g)
(c) 8.0 (d) 32.0
19. The reaction quotient (Q) for the reaction (b) PCl5 (g) ƒ PCl3 (g) + Cl2 (g)
N2 (g) + 3H2 (g) 2NH3 (g) (c) N2 (g) + 3H2 (g) ƒ 2NH3 (g)
(d) The equilibrium will remain unaffected in all the three
[NH 3 ]2 cases.
is given by Q = . The reaction will proceed from 23. Which of the following statements is incorrect?
[N 2 ][H 2 ]3 (a) In equilibrium mixture of ice and water kept in perfectly
right to left if insulated flask mass of ice and water does not change
(a) Q < Kc (b) Q > Kc with time.
(c) Q = 0 (d) Q = Kc (b) The intensity of red colour increases when oxalic acid
where Kc is the equilibrium constant. is added to a solution containing iron (III) nitrate and
potassium thiocyanate.
20. DG° for the reaction X + Y Z is –ve 4.606 kcal. The
(c) On addition of catalyst the equilibrium constant value
equilibrium constant for the reaction at 227°C is is not affected.
(a) 100 (b) 10 (d) Equilibrium constant for a reaction with negative DH
(c) 2 (d) 0.01 value decreases as the temperature increases.

1. Which is a reversible reaction? 3. A reaction attains equilibrium when the free energy change
(a) H 2 + I 2 ¾
¾® 2HI accompanying the reaction is
(a) positive and large
(b) H 2SO 4 + Ba (OH ) 2 ¾¾® BaSO 4 ¯ +2H 2 O
(b) zero
(c) NaCl + AgNO 3 ¾
¾® NaNO 3 + AgCl ¯ (c) negative and large
(d) negative and small
(d) 2KClO 3 ¾ ¾® 2 KCl + 3O 2 ­ 4. The equilibrium constant for the reversible reaction
2. The standard state Gibb's free energy change for the
N2 + 3H2 2NH3 is K and for the reaction
isomerisation reaction
cis-2-pentene trans-2-pentene is 1 N2 + 3 H 2 NH3, the equilibrium constant is K'.
–1
–3.67 kJ mol at 400 K. If more trans-2-pentene is added to 2 2
the reaction vessel K and K' will be related as
(a) more cis-2-pentene is formed
(a) K = K' (b) K' = K
(b) equilibrium shifts in the forward direction
(c) equilibrium remains unaltered (c) K = (d) K × K' = 1
K'
(d) more trans-2-pentene is produced
190 Chemistry
5. If K1 and K2 are respective equilibrium constants for the two 12. For a chemical reaction 2A + B C, the thermodynamic
reactions equilibrium constant Kp is
XeF6 (g) + H2O (g) XeOF4 (g) + 2HF (g) (a) in atm–2 (b) in atm–3
XeO4 (g) + XeF6 (g) (c) in atm –1 (d) dimensionless
XeOF4 (g) + XeO3F2 (g) 13. When two reactants, A and B are mixed to give products C
the equilibrium constant for the reaction and D, the reaction quotient Q, at the initial stage of the
reaction
XeO4 (g) + 2HF (g) XeO3F2 (g) + H2O (g)
(a) is zero (b) decreases with time
will be
(c) is independent of time (d) increases with time
K1
(a) (b) K1.K 2 14. In the equilibrium reaction involving the dissociation of
K 22 CaCO3
K1 K2 CaCO3 (s) CaO (s) + CO2 (g)
(c) (d)
K2 K1
the equilibrium constant is given by
6. A cylinder fitted with a movable piston contains liquid water
in equilibrium with water vapour at 25°C. Which operation p CaO ´ p CO 2 p CO 2
(a) (b) C CaO ´
result in a decrease in the equilibrium vapour pressure p CaCO3 C CaCO3
(a) Moving the piston downward a short distance
(b) Removing a small amount of vapour p CaO
(c) (d) p CO2
(c) Removing a small amount of the liquid water pCaCO3
(d) Dissolving salt in the water 15. Steam reacts with iron at high temperature to give hydrogen
7. The volume of the reaction vessel containing an equilibrium gas and Fe3O4 (s). The correct expression for the equilibrium
mixture in the reaction constant is
SO2Cl2 (g) SO2 (g) + Cl2 (g)
is increased when the equilibrium is re-established PH2 (PH 2 ) 4
2
(a) (b)
(a) The amount of SO2 (g) will decrease PH2 O
2
(PH 2O ) 4
(b) The amount of SO2Cl2 (g) will increase
(c) The amount of Cl2 (g) will increase
(d) The amount of Cl2 (g) will remain unchanged (PH 2 ) 4 [Fe 3O 4 ] [Fe3O 4 ]
(c) (d)
8. In gaseous equilibrium the correct relation between Kc and 4
(PH 2O ) [Fe] [Fe]
Kp is
16. In lime kiln, the reversible reaction
(a) K c = K p ( RT) Dn (b) K p = K c ( RT) Dn
CaCO3 (s) CaO (s) + CO2 (g)
Kc Kp proceeds to completion because
(c) = ( K p ) Dn (d) = ( K c ) Dn
RT RT (a) of high temperature (b) CO2 escaped out
9. In which of the following reaction Kp > Kc (c) CaO is removed (d) of low temperature
(a) N2 + 3H2 2NH3 (b) H2 + I2 2HI 17. If 1.0 mole of I2 is introduced into 1.0 litre flask at
(c) PCl3 + Cl2 PCl5 (d) 2SO3 O2 + 2SO2 1000 K, at quilibrium (Kc = 10–6), which one is correct
(a) [I2 (g)] > [I– (g)] (b) [I2 (g)] < [I– (g)]
10. For reaction PCl3 (g) + Cl2 (g) PCl5 (g), the value of
Kc at 250°C is 26 mol–1 litre1. The value of Kp at this 1 -
(c) [I2 (g)] = [I– (g)] (d) [I2 (g)] = [I (g )]
temperature will be 2
(a) 0.61 atm–1 (b) 0.57 atm–1 18. At equilibrium, if Kp = 1, then
(c) 0.83 atm –1 (d) 0.46 atm–1 (a) DG° = 0 (b) DG ° > 1
11. The equilibrium constant for the reaction,
(c) DG ° < 1 (d) None of these
N2 (g) + O2 (g) 2NO (g)
–4
19. Van't Hoff's equation giving the effect of temperature on
is 4 × 10 at 2000 K.
chemical equilibrium is represented as
In presence of a catalyst, equilibrium is attained ten times
faster. Therefore, the equilibrium constant, in presence of the d ln F DH d ln K p DHT 2
(a) = (b) =
catalyst, at 2000 K is dT RT 2 dT R
(a) 40 × 10–4
(b) 4 × 10–4 d ln K p DH d ln K p RT 2
(c) 4 × 10–3 (c) =
2 (d) =
dT RT dT DH
(d) difficult to compute without more data
Chemical Equilibrium 191
20. Solubility of a substance which dissolves with a decrease in 28. If the equilibrium constant for the reaction
volume and absorption of heat will be favoured by 2AB A2 + B2
(a) high P and high T (b) low P and low T
(c) high P and low T (d) low P and high T is 49, what is the value of equilibrium constant for
21. In what manner will increase of pressure affect the following 1 1
equation? AB A 2 + B2 ?
2 2
C (s) + H2O (g) CO (g) + H2 (g) (a) 49 (b) 2401
(a) Shift in the forward direction (c) 7 (d) 0.02
(b) Shift in the reverse direction
(c) Increase in the yield of hydrogen 29. The reaction, SO 2 + Cl 2 ¾
¾® SO 2 Cl 2 is exothermic and
(d) No effect reversible. A mixture of SO2 (g), Cl2 (g) and SO2Cl2 (l) is at
22. In a reaction, A + 2B 2C, 2.0 mole of 'A', 3.0 mole of 'B' equilibrium in a closed container. Now a certain quantity of
and 2.0 mole of 'C' are placed in a 2.0 L flask and the equilibrium extra SO2 is introduced into the container, the volume
concentration of 'C' is 0.5 mole/L. The equilibrium constant remaining the same. Which of the following is/are true?
(K) for the reaction is (a) The pressure inside the container will not change
(a) 0.073 (b) 0.147 (b) The temperature will not change
(c) 0.05 (d) 0.026 (c) The temperature will increase
23. 4.5 moles each of hydrogen and iodine were heated in a (d) The temperature will decrease
sealed ten litre vessel. At equilibrium 3 moles of HI were
30. The rate of forward reaction is two times that of the reverse
found. The equilibrium constant of
reaction at a given temperature and identical concentration.
H2 (g) + I2 (g) 2HI (g) is Kequilibrium is
(a) 1 (b) 10 (a) 0.5 (b) 1.5
(c) 5 (d) 0.33 (c) 2.5 (d) 2.0
24. A reaction is A + B C + D. Initially we start with 31. 1 mole of hydrogen and 2 moles of iodine are taken initially in
equal concentrations of A and B. At equilibrium we find that a 2 litre vessel. The number of moles of hydrogen at
the moles of C is two times of A. What is the equilibrium equilibrium is 0.2. Then the number of moles of iodine and
constant of the reaction? hydrogen iodide at equilibrium are
(a) 1.2, 1.6 (b) 1.8, 1.0
1 1
(a) (b) (c) 0.4, 2.4 (d) 0.8, 2.0
4 2
(c) 4 (d) 2 32. For the reaction PCl5 (g) PCl3 (g) + Cl2 (g) the forward
25. In which of the following, the forward reaction is favoured reaction at constant temperature is favoured by
by use of high pressure? (a) introducing an inert gas at constant volume
(a) H2 + I2 2HI (b) introducing PCl3 (g) at constant volume
(b) N2 + O2 2NO (c) introducing PCl5 (g) at constant volume
(d) introducing Cl2 (g) at constant volume
(c) 2NH3 N2 + 3H2
33. For the chemical reaction :
(d) 2SO2 + O2 2SO3 3X (g) + Y (g) X3Y (g)
26. In a reaction A + B C + D, the initial concentrations, the amount of X3Y at equilibrium is affected by
of A and B were 0.9 mol. dm–3 each. At equilibrium the (a) temperature and pressure
concentration of D was found to be 0.6 mol dm –3. What is (b) pressure only
the value of equilibrium constant for the reaction (c) temperature only
(a) 8 (b) 4 (c) 9 (d) 3
(d) temperature, pressure and catalyst
27. On the basis of Le-Chatelier's principle, predict which of the
34. When 3 mole of reactant A and one mole of the reactant B are
following conditions would be unfavourable for the formation
of SO3? Given that mixed in a vessel of volume 1 litre, the following reaction
takes place
2SO2 + O2 2SO3; DH = –42 kcal
A (g) + B (g) 2C (g)
(a) Low pressure and low temperature
(b) High pressure and low temperature If 1.5 mole of C is formed at equilibrium, the equilibrium
(c) High temperature and low pressure constant (Kc) of the reaction is
(d) High concentration of SO2 (a) 0.12 (b) 0.50 (c)0.25 (d) 4.00
192 Chemistry
35. Kc for the reaction 42. 8 mole of a gas AB3 are introduced into a 1.0 dm3 vessel. It
N2 (g) + O2 (g) 2NO (g) dissociates as, 2AB3 (g) A2 (g) + 3B2 (g). At
–6
at 300 K is 4.0 × 10 . Kp for the above reaction will be equilibrium, 2 mole of A2 are found to be present. The
(R = 2 cal mol–1 K–1) equilibrium constant of reaction is .... in mol2L–2
(a) 2.4 × 10–3 (b) 4 × 10–6 (a) 2 (b) 3
–6
(c) 4 × 10 (RT) 2 (d) 16 × 10–12
(c) 27 (d) 36
36. Which of the following equilibria will shift to right side on
increasing the temperature? 43. DG° for the reaction X + Y Z is –ve 4.606 kcal. The
(a) CO (g) + H2O (g) CO2 (g) + H2 (g) equilibrium constant for the reaction at 227°C is
(b) 2SO2(g) + O2 (g) 2SO3 (g) (a) 100 (b) 10
(c) 2 (d) 0.01
(c) H2O (g) H2 (g) + 1 2 O 2 (g ) 44. The partial pressure of CH3OH (g), CO (g) and H2 (g) in
(d) 4HCl (g) + O2 (g) 2H2O (g) + 2Cl2 (g) equilibrium mixture for the reaction,
37. 1 mole of N2 and 2 moles of H2 are allowed to react in a 1 dm3 CO (g) + 2H2 (g) CH3OH (g) are 2.0, 1.0 and 0.1 atm
vessel. At equilibrium 0.8 mole of NH3 is formed. The respectively at 427°C. The value of Kp for the decomposition
concentration of H2 in the vessel is of CH3OH to CO and H2 is
(a) 0.6 mole (b) 0.8 mole
(a) 102 atm (b) 2 × 102 atm–1
(c) 0.2 mole (d) 0.4 mole 2
(c) 50 atm (d) 5 × 10–3 atm2
38. The equilibrium
45. Which of the following is not affected by pressure change?
SO2Cl2 (g) SO2 (g) + Cl2 (g)
is attained at 25°C in a closed container and an inert gas, (a) 2NO2 (g) N2O4 (g)
helium is introduced. Which of the following statement is (b) CO2 (g) + H2O (l) CO2 solution
correct?
(c) H2 (g) + I2 (g) 2HI (g)
(a) More chlorine is formed
(b) Concentration of SO2 is reduced (d) 2O3 (g) 3O2 (g)
(c) More SO2Cl2 is formed 46. The degree of dissociation of dinitrogen tetroxide
(d) Concentration of SO2Cl2, SO2 and Cl2 do not change
39. For which reaction high pressure and high temperature is ¾® 2 NO 2 (g ) at temperature T and total
N 2 O 4 (g ) ¾
helpful in obtaining a high equilibrium yield pressure P is a. Which one of the following is the correct
(a) 2NF3(g) N2(g) + 3F2(g) – 54.40 kcal expression for the equilibrium constant (Kp) at this temperature?
(b) N2(g) + 3H2(g) 2NH3(g) + 22.08 kcal 2a a 2P
(a) (b)
(c) Cl2(g) + 2O2(g) 2ClO2(g) – 49.40 kcal (1 - a 2 ) 1- a
(d) 2Cl2O7 (g) 2Cl2 (g) + 7O2(g) + 126.8 kcal
40. For the reversible reaction 4a 2 4a 2 P
(c) (d)
N2 (g) + 3H2 (g) 2NH3 (g) (1 - a 2 ) (1 - a 2 )
at 500°C, the value of Kp is 1.44 × 10–5 when partial pressure
47. Calculate the degree of dissociation of PCl5, the density at
is measured in atmospheres. The corresponding value of Kc
230°C is 70
with concentration in mole litre–1, is
(a) 97.8% (b) 48.9%
(a) 1.44 × 10–5 / (0.082 × 500)–2
(b) 1.44 × 10–5 / (8.314 × 773)–2 (c) 4.89% (d) 24.45%
(c) 1.44 × 10–5 / (0.082 × 773)2 48. When heated to 100°C the V.D. of N 2 O 4 becomes
(d) 1.44 × 10–5 / (0.082 × 773)–2 24.5. The degree of dissociation of N2O4 at this temp. will be
41. A 1 M solution of glucose reaches dissociation equilibrium (a) 80% (b) 87.74%
according to the equation given below (c) 40.34% (d) 60%
6HCHO C6H12O6 49. The correct relationship between free energy change in a
What is the concentration of HCHO at equilibrium if reaction and the corresponding equilibrium constant Kc is
equilibrium constant is 6 × 1022 (a) DG = RT ln K c (b) - DG = RT ln K c
(a) 1.6 × 10–8 M (b) 3.2 × 10 –6 M
–4
(c) DG° = RT ln K c (d) - DG° = RT ln K c
(c) 3.2 × 10 M (d) 1.6 × 10–4 M
Chemical Equilibrium 193
50. The reaction quotient (Q) for the reaction 51. In Haber process 30 litres of dihydrogen and 30 litres of
N2 (g) + 3H2 (g) 2NH3 (g) dinitrogen were taken for reaction which yielded only 50%
of the expected product. What will be the composition of
[ NH 3 ]2 gaseous mixture under the aforesaid conditions in the end?
is given by Q = . The reaction will proceed from
[ N 2 ][H 2 ]3 (a) 10 litres of ammonia, 25 litres nitrogen, 15 litres hydrogen
(b) 20 litres ammonia, 10 litres nitrogen, 30 litres hydrogen
right to left if
(a) Q < Kc (b) Q > Kc (c) 20 litres ammonia, 25 litres nitrogen, 15 litres hydrogen
(c) Q = 0 (d) Q = Kc (d) 20 litres ammonia, 20 litres nitrogen, 20 litres hydrogen
where Kc is the equilibrium constant.

1. The following equilibrium constants are given: 4. The value of equilibrium constant of the reaction
1 1
ˆˆ† 2NH3 ; K1
N 2 + 3H 2 ‡ˆˆ [CBSE-PMT 2003, 2007] HI ( g ) ƒ H 2 ( g ) + I 2 is 8.0 [CBSE-PMT 2008]
2 2
The equilibrium constant of the reaction
ˆˆ† 2NO; K 2
N 2 + O2 ‡ˆˆ
H 2 ( g ) + I2 ( g ) ƒ 2HI( g ) will be:
1
ˆˆ† H 2 O; K 2
H 2 + O 2 ‡ˆˆ 1 1 1
2 (a) (b) (c) 16 (d)
16 64 8
The equilibrium constant for the oxidation of NH3 by oxygen 5. In which of the following equilibrium Kc and Kp are not equal?
to give NO is
(a) 2 NO(g ) ƒ N 2 (g ) + O 2 (g ) [CBSE-PMT 2010]
K 2 K32 K 22 K3 (b) SO 2 (g )+NO 2 (g ) ƒ SO3 (g ) + NO(g )
(a) (b)
K1 K1
(c) H 2 (g )+I2 (g ) ƒ 2 HI(g )

K1 K 2 K 2 K33 (d) 2C(s )+O 2 (g ) ƒ 2 CO 2 (g )


(b) (d)
K3 K1
6. The reaction 2A( g ) + B( g ) ƒ 3C( g ) + D( g ) is began with
2. The dissociation equilibrium of a gas AB2 can be represented the concentrations of A and B both at an initial value of 1.00
as : [CBSE-PMT 2008] M. When equilibrium is reached, the concentration of D is
ˆˆ† 2AB( g ) + B2 ( g ) measured and found to be 0.25 M. The value for the equilibrium
2AB2 ( g ) ‡ˆˆ
constant for this reaction is given by the expression
The degree of dissociation is ‘x’ and is small compared to 1. [CBSE-PMT 2010]
The expression relating the degree of dissociation (x) with 3 2
(a) [(0.75) (0.25)] [(0.75) (0.25)]
equilibrium constant Kp and total pressure P is : (b) [(0.75)3 (0.25)] [(1.00)2 (1.00)]
(a) (2Kp/P) (b) (2Kp/P) 1/3 (c) [(0.75)3 (0.25)] [(0.50)2 (0.75)]
(c) (2Kp/P)1/2 (d) (Kp/P) (d) [(0.75)3 (0.25)] [(0.50)2 (0.25)]
3. The values of Kp1 and Kp2 for the reactions 7. Given that the equilibrium constant for the reaction
ˆˆ† Y + Z
X ‡ˆˆ ...(1) [CBSE-PMT 2008] 2SO2 (g) + O2(g) ‡ˆˆˆˆ† 2SO3(g) has a value of 278 at a
particular temperature. What is the value of the equilibrium
ˆˆ† 2B
and A ‡ˆˆ ...(2)
constant for the following reaction at the same temperature ?
are in the ratio of 9 : 1. If degree of dissociation of X and A be [CBSE-PMT 2012M]
equal, then total pressure at equilibrium (1) and (2) are in the
ratio : ˆˆ† SO2 ( g ) + 1 O2 ( g )
SO3 ( g ) ‡ˆˆ
2
(a) 3 : 1 (b) 1 : 9 (c) 36 : 1 (d) 1 : 1
(a) 1.8 × 10–3 (b) 3.6 × 10–3 (c) 6.0 × 10–2 (d) 1.3 × 10–5
194 Chemistry
8. Given the reaction between 2 gases represented by A2 and 16. In a cell that utilises the reaction
B2 to give the compound AB(g). [CBSE-PMT 2012M]
Zn (s) + 2H + (aq) ® Zn 2+ (aq) + H 2(g ) addition of H2SO4 to
A2(g) + B2(g) ‡ˆˆˆˆ† 2 AB(g).
cathode compartment, will [AIEEE 2004]
At equilibrium, the concentration
(a) increase the E and shift equilibrium to the right
of A2 = 3.0 × 10–3 M
(b) lower the E and shift equilibrium to the right
of B2= 4.2 × 10–3 M
(c) lower the E and shift equlibrium to the left
of AB = 2.8 × 10–3 M
(d) increase the E and shift equilibrium to the left
lf the reaction takes place in a sealed vessel at 527°C, then
the value of Kc will be : 17. The exothermic formation of CIF3 is represented by the
(a) 2.0 (b) 1.9 (c) 0.62 (d) 4.5 equation :
9. Change in volume of the system does not alter CI 2(g ) + 3F2(g ) 2ClF3 (g ) ; Δ H = – 329 kJ
the number of moles in which of the following
equilibria? [AIEEE 2002] Which of the following will increase the quantity of CIF3 in
(a) N2 (g) + O2 (g) 2NO (g) an equilibrium mixture of CI 2 , F2 and CIF3 ? [AIEEE 2005]
(b) PCl5 (g) PCl3 (g) + Cl2 (g)
(c) N2 (g) + 3H2 (g) 2NH3 (g) (a) Adding F2
(d) SO2Cl2 (g) SO2 (g) + Cl2 (g). (b) Increasing the volume of the container
10. In which of the following reactions, increase in the volume at (c) Removing Cl2
constant temperature does not affect the number of moles at
(d) Increasing the temperature
equilibrium. [AIEEE 2002]
(a) 2NH3(g) ® N2(g) + 3H2(g) 18. For the reaction [AIEEE 2005]
(b) C (s)+ (1/2)O2 (g) ® CO(g) 2 NO 2(g) 2NO (g ) + O 2(g ) ,
(c) H2 (g) + O2 (g) ® H2O2 (g)
(d) None of these. (K c = 1.8 ´ 10- 6 at 184°C) (R = 0.0831 kJ/ (mol. K))
11. For the reaction [AIEEE 2002]
CO (g) + (1/2) O2 (g) CO2 (g), Kp / Kc is When K p and K c are compared at 184°C, it is found that
(a) RT (b) (RT)–1
(c) (RT) –1/2 (d) (RT)1/2 (a) Whether K p is greater than, less than or equal to K c
12. For the reaction equilibrium depends upon the total gas pressure
N2O4 (g) 2 NO2 (g)
the concentrations of N2O4 and NO2 at equilibrium are 4.8 × (b) K p = K c
10–2 and 1.2 × 10–2 mol L–1 respectively. The value of Kc for
the reaction is [AIEEE 2003] (c) K p is less than K c
(a) 3 × 10–1 mol L–1 (b) 3 × 10–3 mol L–1
(d) K p is greater than K c
(c) 3 × 103 mol L–1 (d) 3.3 × 102 mol L–1
13. Consider the reaction equilibrium
19. An amount of solid NH 4 HS is placed in a flask already
2 SO2 (g) + O2(g) 2 SO3 (g) ; DHº = –198 kJ
containing ammonia gas at a certain temperature and 0.50
On the basis of Le Chatelier’s principle, the condition
atm pressure. Ammonium hydrogen sulphide decomposes
favourable for the forward reaction is [AIEEE 2003]
(a) increasing temperature as well as pressure to yield NH 3 and H 2S gases in the flask. When the
(b) lowering the temperature and increasing the pressure decomposition reaction reaches equilibrium, the total
(c) any value of temperature and pressure pressure in the flask rises to 0.84 atm? The equilibrium
(d) lowering of temperature as well as pressure
constant for NH 4 HS decomposition at this temperature is
14. What is the equilibrium expression for the reaction
P4(s) + 5O 2(g ) (a) 0.11 (b) 0.17 [AIEEE 2005]
P4O10(s) ? [AIEEE 2004]
(c) 0.18 (d) 0.30
(a) K c = [O 2 ]5 (b) K c = [ P4 O10 ] / 5[ P4 ][O 2 ] 20. The equilibrium constant for the reaction

(c) K c = [P4 O10 ] /[P4 ][O 2 ]5 (d) K c = 1 /[O 2 ]5 1


SO3(g) SO 2 (g ) +O 2 (g)
2
Kp
15. For the reaction, CO (g ) + Cl 2 COCl 2 (g ) the Kc is Kc = 4.9 × 10–2. The value of Kc for the reaction
(g )
is equal to [AIEEE 2004] 2SO2(g) + O2(g) 2SO3(g)
(a) RT (b) RT will be [AIEEE 2006]
(a) 9.8 × 10–2 (b) 4.9 × 10–2
(c) 1 RT (d) 1.0 (c) 416 (c) 2.40 × 10–3
Chemical Equilibrium 195

21. The equilibrium constants K p and K p2 for the reactions (a) K1 K 2 = K3 (b) K 2 K3 = K1
1

X ƒ2Y and Z ƒ P + Q, respectively are in the ratio of 1 : (c) K3 = K1 K2 (d) K3 .K 23 = K12


9. If the degree of dissociation of X and Z be equal then the 23. A vessel at 1000 K contains CO2 with a pressure of 0.5 atm.
ratio of total pressures at these equilibria is [AIEEE 2008] Some of the CO2 is converted into CO on the addition of
(a) 1 : 1 (b) 1: 36 graphite. If the total pressure at equilibrium is 0.8 atm, the
(c) 1 : 3 (d) 1 : 9 value of K is : [AIEEE 2011]
22. For the following three reactions a, b and c, equilibrium (a) 1.8 atm (b) 3 atm (c) 0.3 atm (d) 0.18 atm
constants are given: [AIEEE 2008]
24. The equilibrium constant (K c ) for the reaction
(i) CO( g ) + H 2O( g ) ƒ CO2 ( g ) + H 2 ( g ); K1 N2(g) + O2(g) ® 2NO(g) at temperature T is 4 × 10–4. The
(ii) CH 4 ( g ) + H 2O( g ) ƒ CO( g ) + 3H 2 ( g );K 2 value of Kc for the reaction [AIEEE 2012]
1 1
(iii) CH 4 ( g ) + 2H 2O( g ) ƒ CO2 ( g ) + 4H 2 ( g );K 3 NO( g ) ® N 2 ( g ) + O 2 ( g ) at the same temperature is:
2 2
(a) 0.02 (b) 2.5 × 102 (c) 4 × 10–4 (d) 50.0

1. The decomposition of N2O4 to NO2 is carried out at 280 K in 6. For the reaction H2(g)+I2(g) 2HI(g) at 721 K, the value
chloroform. When equilibrium has been established, 0.2 mol of equilibrium constant is 50, when equilibrium concentration
of N2O4 and 2 ´ 10 -3 mol of NO2 are present in a 2l solution. of both is 5M. Value of Kp under the same conditions will be
(a) 0.02 (b) 0.2
The equilibrium constant for the reaction, N2O4 2NO2 is
(c) 50 (d) 50 RT
(a) 1 × 10–2 (b) 2 × 10–3 7. For which one of the following systems at equilibrium, at
(c) 1 × 10–5 (d) 2 × 10–5 constant temperature will the doubling of the volume cause
2. In an equilibrium reaction for which DG° = 0 , the equilibrium a shift to the right?
constant, K , should be equal to (a) H2(g)+Cl2(g) 2HCl(g)
(b) 2CO(g)+O2 (g) 2CO2(g)
(a) 0 (b) 1
(c) N2(g)+3H2(g) 2NH3(g)
(c) 2 (d) 10
3. The pressure change due to which equilibrium is not (d) PCl5 ( g ) ƒ PCl3 ( g ) + Cl 2 ( g )
affected is 8. For the reaction C(s)+CO2(g) 2CO(g) , the partial
(a) N 2 ( g ) + O2 ( g ) ƒ 2NO ( g ) pressures of CO2 and CO are 2.0 and 4.0 atm respectively at
equilibrium. The Kp for the reaction is.
(b) 2SO2(g)+O 2(g) 2SO3(g) (a) 0.5 (b) 4.0
(c) 8.0 (d) 32.0
(c) 2O3(g) 3O2(g) 9. In a chemical reaction calculate rate constant of backward
reaction when the rate constant of forward reaction is 20 and
(d) 2NO2(g) N2O4(g)
the equilibrium constant is 50
4. A reversible chemical reaction having two reactants in (a) (0.2) (b) (0.1)
equilibrium. If the concentration of the reactants are doubled (c) (0.4) (d) None of these
then the equilibrium constant will 10. The rate constant is given by the equation K=PZe–E/RT.
(a) be doubled (b) be halved Which factor should register a decrease for the reaction to
proceed more rapidly?
(c) become one-fourth (d) remain same (a) T (b) Z
5. 3.2 moles of hydrogen iodide were heated in a sealed bulb at (c) E (d) P
444°C till the equilibrium state was reached. Its degree of 11. For the reaction CO(g) + H2O(g) CO2(g) + H2(g) at a
dissociation at this temperature was found to be 22% The given temperature the equilibrium amount of CO2(g) can be
number of moles of hydrogen iodide present at equilibrium increased by
are (a) adding a suitable catalyst
(a) 2.496 (b) 1.87 (b) adding an inert gas
(c) decreasing the volume of the container
(c) 2 (d) 4 (d) increasing the amount of CO(g).
196 Chemistry
12. At constant temperature, the equilibrium constant (Kp) for 18. By which of the following reactions, the equilibrium constant
the decomposition reaction N2O4 2NO2 is expressed is related to tempearture?
2 2
by Kp = (4x P)/(1-x ), where P = pressure, x = extent of DH º T2 æ1ö
decomposition. Which one of the following statements is (a) ln K 2 - ln K1 = dç ÷
R T1 èTø
true?
(a) Kp increases with increase of P DHº 1/ T 2 æ 1 ö
(b) ln K 2 - ln K 1 = - d
(b) Kp increases with increase of x R 1/ T1 çè T 2 ÷ø
(c) Kp increases with decrease of x
DH º T2 æ 1 ö
(d) Kp remains constant with change in P and x (c) ln K 2 - ln K1 = - dç ÷
13. Consider the following equilibrium in a closed container R T1 è T ø
N2O4(g) 2NO2(g) At a fixed temperature, the volume DHº 1/ T1 æ 1 ö
(d) ln K 2 - ln K 1 = - d
of the reaction container is halved. For this change, which of R 1/ T 2 çè T ÷ø
the following statements holds true regarding the equilibrium 19. Which of the following expressions is incorrect?
constant (Kp) and degree of dissociation ( a )?
(a) Kp does not changes, but a changes æ ln K p ö æ ln K c ö
(b) Kp changes, but a does not change (a) çç p ÷ø
÷ =0 (b) çç p
÷÷ = 0
è T è øT
(c) both Kp and a change
(d) neither Kp nor a changes æ ln K x ö - Dn
14. Which of the following is true at chemical equilibrium? (c) çç p
÷÷ =
( p / pº ) (d) All of these
è øT
(a) (DG)T, P is minimum and (DS)U,V is also minimum
20. K c for PCl5 (g) PCl3 (g) + Cl 2 (g) is 0.04 at 250°C.
(b) (DG)T,V is minimum and (DS)U,V is maximum
(c) (DG)T,V is maximum and (DS)U,V is zero How many moles of PCl 5 must be added to a 3 L flask to
(d) (DG)T,P is zero and (DS)U,V is also zero obtain a Cl 2 concentration of 0.15 M
15. For the reaction A(g) B(g)+C(g), (a) 4.2 moles (b) 2.1 moles
(a) Kp = a p3 (b) Kp = a2(Kp+p+1) (c) 5.5 moles (d) 6.3 moles
æ Kp + p ö 21. If CuSO 4 .5H 2O (s) CuSO 4 .3H2O(s) + 2H 2O(g)
(c) Kp = a2(Kp+p) (d) K p = a 2 çç ÷
÷
è p ø K p = 1 .086 ´ 10 - 4 atm 2 at 25° C. The eflorecent nature of
16. In the van’t Hoff equation CuSO 4 . 5H 2O can be noticed when the vapour pressure of
d1nK DHº H 2 O in atmosphere is
=
dT RT 2 (a) > 9.72 mm (b) < 7.92 mm
(c) < 7.92 mm (d) < 11.92 mm
(a) when dln K < 0 , the reaction is exothermic 22. In a closed system, A(s) 2B(g) + 3C(g), if partial pressure
dT of C is doubled, then partial pressure of B will be
(a) 2 2 times the original value
(b) when dln K < 0 , the reaction is endothermic
dT 1
(b) times the original value
(c) the slope of the graph is positive throughout 2
(d) the slope of graph increases and then decreases (c) 2 times the original value
17. Consider the expression DG = –RTlnKp+RTlnQp and indicate 1
the correct statement at equilibrium (d) times the original value
2 2
(a) DG = 0, Qp> Kp the equilibrium reaction will shift
from left to right 23. For the reactions
(b) DG = 0, Qp=Kp the equilibrium reaction will shift A B Kc =2
from left to right B C Kc =4
(c) DG = ¥,Qp<Kp the equilibrium reaction will shift
C D Kc =6
from right to left
(d) DG<0, Qp>Kp the equilibrium reaction will shift K c for the reaction A D is
from right to left
2´ 4
where Qp and Kp term refer to reaction quotient and (a) 2 × 4 × 6 (b)
6
equilibrium constant at constant pressure respectively.
4´ 6
(c) 2 + 4 + 6 (d)
2
Chemical Equilibrium 197
29. For the equilibrium reaction
24. 1 mol of N 2 and 3 mol of H 2 are placed in a closed container
2NOBr(g) 2NO(g) + Br2 (g)
at a pressure of 4 atm. The pressure falls to 3 atm at the same
temperature when the following equilibrium is attained P
If PBr2 = , where P is total pressure at equilibrium. The
N 2 (g) + 3H 2 (g ) 2 NH 3 (g ) . 9
Kp
The K p for the dissociation of NH 3 is ratio is equal to
P

3´3 1 1
(a) atm - 2 (b) 0.5 ´ (1.5) 3 atm 2 (a) (b)
0.5 ´ (1.5) 3 3 81
1 1
(c) (d)
0.5 ´ (1.5) 3 (1.5) 3 9 27
(c) atm 2 (d) atm - 2
3´3 0.5 30. Equilibrium constant for the reaction
25. The reactions NH 4 OH (aq) + H + (aq) NH +4 (aq) + H 2 O ( l )
PCl5 (g) PCl3 (g) + Cl 2 (g) and is 1.8 × 10 9 . Hence equilibrium constant for ionisation
COCl 2 (g ) CO(g) + Cl 2 (g) NH 3 + H 2 O NH+4 (aq) + OH- (aq) is
are simultaneously in equilibrium in a vessel at constant (a) 1.80 ´ 10 - 9 (b) 1.80 ´ 10 5
volume. If some CO is introduced into the vessel then at the
new equilibrium (c) 1.80 ´ 10 - 5 (d) 5.55 ´ 10 -10
31. The degree of dissociation of PCl5 (a) obeying the
(a) Cl 2 is greater
equilibrium PCl 5 PCl 3 + Cl 2 is related to the
(b) PCl 5 is less equilibrium pressure by
(c) PCl 3 remain unchanged 1 1
(a) a µ (b) a µ
(d) PCl 5 is greater P4 P

26. For the complexation reaction 1


(c) a µ 2 (d) a µ P
P
Ag + (aq) + 2 NH 3 (aq) [Ag( NH 3 ) 2 ]+ (aq) 32. In the following hypothetical reaction
The rates of forward and reverse reactions are given by A + 3B 2C + D
initial moles of A are twice that of B. If at equilibrium moles of
rate (f) = 1.0 ´ 10 6 L2 mol - 2S -1 é Ag + ù [ NH3 ]2 B and C are equal. Percentage of B reacted is
ë û
(a) 60% (b) 40%
rate (r) = 2.0 ´ 10 - 2 L2 mol-2S-1 [Ag( NH 3 ) 2 ]+ (c) 10% (d) 20%

The instability constant of the complex is 33. I - ions react with iodine in aqueous solution to form I3- ion
as
(a) 2.0 ´ 10 - 8 (b) 5.0 ´ 10 9
I - (aq) + I 2 (aq) I3- (aq).
(c) 2.0 ´ 10 - 4 (d) 2.0 ´ 10 4 When L of solution containing 1 mol of KI and 0.25 mol of
I 2 was reacted with excess of AgNO 3 , 0.80 mol of yellow
27. The equilibrium constant for a reaction is 1´ 10 20 at 300 K.
The standard free energy change for the reaction is ppt. was obtained. The stability constant of I3- ion is
(a) + 115 KJ (b) + 166 KJ (a) 0.20 (b) 5.0
(c) – 115 KJ (d) – 166 KJ (c) 20.0 (d) 0.05
34. A reaction A + 2B 2 C + D was studied using the initial
28. At temperature T K, PCl 5 is 50% dissociated at an concentrations of B which was 1.5 times that of A. At the
equilibrium pressure of 4 atm. At what pressure it would quilibrium, concentration of A and C were found to be equal.
dissociate to 80% at the same temperature Then K c for the equilibrium is
(a) 0.75 atm (b) 0.50 atm (a) 8 (b) 4
(c) 0.60 atm (d) 2.50 atm (c) 6 (d) 0.32
198 Chemistry
35. 1.0 mol of AB5 (g) is placed in a closed container under one 41. When the reactants A and B are mixed to give products C
and D, the reaction quotient Q at the initial stage of reaction
atmosphere and at 300K. It is heated to 600K, when 20% by
is
mass of it dissociates as
(a) zero (b) independent of time
AB5 (g) ¾ ¾® AB(g) + 2 B2 (g). (c) increases with time (d) decreases with time
The resultant pressure is 42. The eqilibrium constant for the reaction
(a) 1.2 atm (b) 2.4 atm CO(g) + 2H 2 (g) CH 3OH(g)
(c) 1.4 atm (d) 2.8 atm is 4.3 at 250° C and 1.8 at 275° C. The reaction is
36. The pressure necessary to obtain 50% dissociation of PCl 5 (a) exothermic (b) endothermic
(c) Both (a) and (b) (d) Neither (a) nor (b)
at 250°C is numerically equal to
43. The element Bi melts at 271°C and has density of 9.73 g/ml
(a) three times of K p (b) 6 times of K p as a solid and 10.05 g/ml as a liquid at this temperature. For
(c) 2.5 times of K p (d) 8 times of K p the equilibrium Bi(s) Bi( l ), the melting point is
favoured in this endothermic reaction either by
37. The activation energies for the forward and reverse reactions (a) increasing temperature, decreasing pressure
in the system A B are 10.303 and 8.000 kcal respectively (b) decreasing temperature, decreasing pressure
at 500 K. Assuming that pre exponential factor to be same for (c) increasing temperature, increasing pressure
both the forward and the reverse steps and R = 2 cal K–1 mol– (d) there is no effect of pressure on melting point
1. Calculate the K of the reaction 44. Consider the gaseous equilibrium of
c
(a) 1.00 (b) 10.0 H 2 (g) + I 2 (g) 2HI (g)
(c) 0.1 (d) 100 In the following graphs the three gases are at equilibrium in
38. 28g N 2 and 6.0 g of H 2 are heated over catalyst in a closed a container. At some time t, extra I 2 is added. Which of the
one litre flask of 450°C. The entire equilibrium mixture following sets will respond to this situation ?
required 500 ml of 1.0 M H 2SO 4 for neutralisation. The value
HI HI
of K c for the reaction
N 2 (g) + 3H 2 (g ) 2 NH 3 (g) is P I2 P I2
H2 H2
(a) 0.06 mol -2 L2 (b) 0.59 mol -2 L2
t t
(c) 1.69 mol 2 L-2 (d) 0.03 mol 2 L-2
A B
39. 2 mol of SO 2 and 1 mol of O 2 are heated in a closed vessel
to reach the equilibrium
HI HI
2 SO 2 (g) + O 2 (g) 2 SO 3
The equilibrium mixture exerted a presure of 5 atm and required P I2 P I2
1/3 mol of K 2Cr2 O 7 in acidic medium. K p for the reaction H2 H2
is t
(a) 1.0 (b) 2.0 t
(c) 0.5 (d) 4.0 C D
40. A gaseous compound of molecular mass 82.1 dissociates on (a) A (b) B
heating to 400 K as (c) C (d) D
X 2 Y4 (g) X 2 (g) + 2 Y2 (g)
The density of the equilibrium mixture at a pressure of 1 atm
and temperature of 400K is 2.0gL-1 . The percentage
dissociation of the compound is
(a) 12.5% (b) 48.5%
(c) 90.1% (d) 25.0%
Chemical Equilibrium 199

EXERCISE 1 11. (b) Kc is not influenced by presence of a catalyst.


1. In the backward direction. 12. (a) Unit of Kp = (Atm)Dn = (Atm)–2
(Dn = moles of products – moles of reactants)
3. AB is more stable. Higher the value of K more will be the
13. (d) Q increases with the formation of products
stability of product formed.
4. Solubility of gases in liquid increases with decrease in [Conc. of Products]
temperature. Q=
[Conc. of Reactants]

( pH ) [Fe and Fe O are solids] 14. (d) Kp = pCO2 others are solids. The concentration terms for
4
2
5. Kp = solids and liquids are taken as unity.
( pH O )
4 3 4
2 15. (b) 3Fe(s) + 4H2O (steam) Fe3O4 (s) + 4 H2 (g)
6. No, K remains constant. (p H 2 ) 4
7. CH4(g) + 2H2S(g) ® 4H2(g) + CS2(g) Kp = (p 4 only gaseous products and reactants.
H 2O )
9. 4.56 × 10–6 mol2/l2
10. (a) Increase in temperature would favour the forward 16. (b) Forward reaction is favoured by removal of products.
reaction while decrease in temperature would favour 17. (a) I2 2I–
the backward reaction. 1–x 2x
(b) Increase in pressure favours the reaction towards less
number of moles of gaseous species. (2x ) 2
Kc = = 10 -6
11. pH = pBr = 2.5 × 10–2 bar (1 - x)
2 2
pHBr = 10.0 – 5.0 × 10–2
Soln. shows that (1 – x) > 2x \ éë I2 ( g ) ùû > éë I ( g ) ùû
-
» 10.0 bar
14. (c) 15. (c) 16. (c) 17. (d) 18. (c)
18. (a) DG° = –2.303 RT log Kpwhen Kp = 1, DG° = 0 since log 1 = 0
19. (b) 20. (a) 21. (a) 22. (d) 23. (b)
d ln K p DH
EXERCISE 2 19. (c) Van't Hoff's Isochore = . Relation between
dt RT 2
1. (a) The product is separated as solid or escapes as gas in an
T and Kc.
irreversible reaction.
20. (a) See Le Chatelier's principle.
2. (a) Le Chatelier's principle, equilibrium shifted in the
21. (b) Reverse reaction, Le Chatelier's principle Dn=2 – 1 = 1.
backward direction.
3. (b) DG = 0 at equilibrium. 22. (c) A + 2B ƒ 2C
Initial moles in 2 litres 2 3 2
4. (b) K ' = K
At Equilibrium moles in 2 litres 2.5 4 1
5. (d) Reaction (II) and reverse of reaction (I) gives the desired Molar conc. moles in 2 litres 1.25 2 0.5
1 K (0.5) 2
reaction hence K = K 2 ´ = 2 . Kc = = 0.05
K1 K1
(1.25)(2) 2
6. (d) Dissolution of salt lowers the V.P. It is also effected by
temperature. 23. (a) H2 + I2 2HI
7. (c) It will decrease the concentration. The equilibrium will Initial moles 4.5 4.5 0
shift in the direction where more moles are formed to Moles at eqm. 3 3 3
keep Kc constant.
(3) 2
8. (b) Relation is Kp = Kc (RT)Dn K= = 1 . Note : when there is no change in the
3´ 3
9. (d) Dn = 3 – 2 = 1; Kp = Kc (RT) hence Kp > Kc
number of moles, Dn = 0 the volume or capacity of vessel
10. (a) Q Dn = –1 \ Kp = 26 × (0.0821 × 523)–1 = 0.61 atm–1
can be ignored.
200 Chemistry
24. (c) A+ B C +D 41. (d) C6H12O6 6HCHO
At eqb. x x 2x 2x Initial 1M
At eqb. 1–x 6x
2x.2x
Kc = =4
x.x [6x]6 1- x
Kc = for reverse reaction = 6 ´ 10 22 ;
1- x 6
(6 x )
25. (d) See Le Chatelier's principle.
On solving 6x = 1.6 × 10–4 M.
26. (b) A+ B C + D
42. (c) 2AB3 A2 (g) + 3B2 (g)
Initial 0.9 0.9 0 0
Initial moles 8 0 0
At eqm. 0.3 0.3 0.6 0.6
Moles at eqb. 8–x x 3x
2 2
0.6 ´ 0.6
Kc = =4
0.3 ´ 0.3 [B2 ]3[A 2 ] (6)3 ´ 2
given x 2 = 2 \ x = 4 Kc= = = 27
27. (c) Le Chatelier's principle since reaction is exothermic hence [AB3 ]2 (4)2
low temperature will favour forward reaction also volume
is decreased by applying high pressure. 43. (a) DGo = – 2.303 RT log K
– 4.606 × 103 = – 2.303 × 2 × 500 log Kc
28. (c) Reaction (II) is 1 2 of (I). \ K = K = 49 = 7 . \ Kc = 100.

29. (c) By addition of SO2 equilibrium will shift to RHS which is p CH 3OH 2
44. (d) K p = = = 200 ;
exothermic. Hence temperature will increase. p CO ´ p H 2 1 ´ (0.1) 2
Kf 2
30. (d) Keqb = = =2 1 1
Kb 1 For reverse reaction = = 5 ´10 -3 atm 2
K p 200
31. (a) H2 + I2 2HI
45. (c) No effect of pressure when Dn = 0.
Initial 1 2 0
46. (d) N2O4 2NO2
At eqb. 0.2 2 – 0.8 = 1.2 2 × 0.8 = 1.6
1–a 2a
32. (c) Le Chatelier's principle.
Total moles at eqb=1 – a + 2a = 1 + a
33. (a) Temperature and pressure (Le Chatelier's principle)
2
Dn ¹ 0 . æ 2a ö
p 2NO ç .P ÷
1+ a ø 4a 2 P
34. (d) A + B 2C Kc = 2
=è =
p N 2O 4 1- a 1- a2
.P
Initial moles 3 1 0 1+ a
At eqb. 3 – 0.75 1 – 0.75 1.5 47. (b) Degree of dissociation
(1.5) 2 D-d 104.25 - 70 M
Kc = =4 a= = = 48.9% (D = )
(2.25)(0.25) d(n - 1) 70(2 – 1) 2

35. (b) Kp = Kc (RT)Dn since Dn = 0, Kp = Kc. 46 - 24.5


36. (c) Reaction (c) is endothermic. Electrolysis or decomposition 48. (b) a = = 87.74%
24.5(2 - 1)
of H2O is endothermic in nature.
49. (d) Relation DGo = –RT ln Kc or –DGo = RT ln Kc
37. (b) N2 + 3H2 2NH3 50. (b) Q > Kc will make the reaction to proceed from right to left.
Initial mole 1 2 0 51. (a) N2 + 3H2 2NH3
Moles at eqb.1 – 0.4 = 0.6 2 – 3 × 0.4 = 0.8 0.8 30 30 taken
38. (d) No change in concentration at constant volume only 10 30 20 If yield were 100%
pressure is increased when some inert gas is introduced. 5 15 10 Since yield is 50%
39. (c) Follow Le Chatelier's principle. 25 15 10 On completion of reaction
reactants, the left and
1.44 ´ 10 -5
40. (d) Kp = Kc (RT)Dn; Kc = products formed.
(0.082 ´ 773) - 2
Chemical Equilibrium 201
EXERCISE 3 x
1. (d) Given, PB = ´P
2 2
ˆˆ† 2NH 3 ; K1
N 2 + 3H 2 ‡ˆˆ ....(i)
ˆˆ† 2NO; K 2
N 2 + O2 ‡ˆˆ ....(ii) (PAB )2 ( PB2 )
Now, K P =
(P 2 )
2
1
ˆˆ† H 2O; K3
H 2 + O2 ‡ˆˆ ....(iii) AB
2
We have to calculate x
( x )2 ´ P 2 .P ´
4NH 3 + 5O2 ¾¾ ® 4NO + 6H 2O; K = ? = 2
5 (1 - x )2 ´ P 2
or 2NH 3 + O 2 ¾¾ ® 2NO + 3H 2 O
2
x 3 .P3
2
[NO] [H 2O] 3 = [\ 1 – x ; 1 ]
For this equation, K = 2 ´ 1 ´ P2
[NH3 ]2 [O 2 ]5 / 2
1
[NH3 ]2 [NO]2 x3 .P 2.K p æ 2K p ö 3
but K1 = , K2 = = or x3 = or x = ç
[N 2 ] [H2 ]3 [N 2 ] [O2 ] 2 P è P ÷ø
[H 2O] [H 2O]3 3. (c) Given reactions are
& K3 = or K33 =
X ƒ Y+Z
½ 3 3/ 2
[H 2 ] [O 2 ] [H 2 ] [O2 ] ..... (i)
and A ƒ 2B ......(ii)
K 2 . K33
Now operate, Let the total pressure for reaction (i) and (ii) be P1 and P2
K1 respectively, then
[NO]2 [H 2 O]3 [N ] [H2 ]3 KP
= ´ . 2 1 9
[N 2 ] [O2 ] [H 2 ]3 [O2 ]3 / 2 [NH3 ]2 = (given)
KP2 1
[NO]2 [H 2 O]3 After dissociation,
= =K
2
[NH3 ] [O2 ] 5/ 2 X ƒ Y+Z
At equilibrium (1– a) a a
K 2 . K 33 [Let 1 mole of X dissociate with a as degree of
\ K=
K1 dissociation ]
Total number of moles = 1– a + a + a
2. (b) For the reaction = (1+ a)
2AB2 (g) ƒ 2AB(g) + B2 (g) æ1- ö æ ö
at equi Thus PX = ç .P ;P = P;
è 1 + ÷ø 1 Y çè 1 + ÷ø 1
2(1- x ) 2x x

Kc =
[ AB]2 [ B2 ] or K = (2 x)2 ´ x æ ö
PZ = ç .P
è 1 + ÷ø 1
c
[AB2 ]2 {2(1 - x )}2

= x3 [(1– x) can be neglected in denominator (1– x) ; 1] æ ö


\ K P1 = ç .P ´ .
The partial pressure at equilibrium are calculated on the è 1 + ÷ø 1 (1 + )
basis of total number of moles at equilibrium.
Total number of moles æ1 - ö
P1 / ç .P ....... (i)
= 2 (1–x) + 2x + x = (2 + x) è 1 + ÷ø 1
2(1 - x) Similarly for A ƒ 2B
\ PAB = ´ P , where P is the total pressure.
2 (2 + x) At equilibrium (1– a) 2a
We have,
2x x
PAB = ´ P , PB = ´P æ 2 P2 ö æ 1 - ö
2
(2 + x ) 2 (2 + x ) K P2 = ç / P ........(ii)
è 1 + ÷ø çè 1 + ÷ø 2
Since x is very small so can be neglected in denominator
Thus, we get Dividing (i) by (ii), we get
PAB = (1 – x) × P PAB = x × P KP 2
.P1 KP
2 1 = 1 = 1 . P1
2 or
KP 4 .P2 KP 4 P2
2 2
202 Chemistry
9. (a) In this reaction the ratio of number of moles of reactants
1 P é KP ù
ê 1 = 9ú to products is same i.e. 2 : 2, hence change in volume will
or 9 = . 1 not alter the number of moles.
4 P2 ê KP 1ú
ë 2 û 10. (d) In (a), (b) and (c) no. of mole of products is not equal to
P1 36 no. of mole of reactants.
or = or P1 : P2 = 36 : 1
P2 1 æ 1ö 3 1
11. (d) K p = K c ( RT ) Dn ; Dn = 1 - ç1 + ÷ = 1 - =- .
i.e. option (c) is correct answer. è 2ø 2 2
4. (b) Given : Equilibrium constant (K1) for the reaction Kp -1 / 2
1 1 \ K = (RT )
ˆˆˆ†
HI( g ) ‡ˆˆ
K1
ˆ H 2 ( g ) + I2 ( g ); K1 = 8; .........(i) c
2 2
To find equilibrium constant for the following reaction [ NO 2 ]2 [1.2 ´10-2 ]2
12. (b) K C = = = 3 ´10 -3 mol/L
H 2 ( g ) + I2 ( g ) ƒ 2HI( g ); K2 = ? .....(ii) [ N2O4 ] -2
[4.8 ´10 ]
For this multiply (i) by 2, we get 13. (b) Due to exothermicity of reaction low or optimum
2HI( g ) ƒ H 2 ( g ) + I 2 ( g ); temperature will be required. Since 3 moles are changing
to 2 moles.
K1 = 82 = 64. .....(iii)
\ High pressure will be required.
[Note: When the equation for an equilibrium is multiplied
by a factor, the equilibrium constant must be raised to 14. (d) For P4 (s) + 5O 2 (g) P4O10 (s)
the power equal to the factor] 1
Now reverse equation (iii), we get Kc = . The solids have concentration unity
(O 2 ) 5
1
H 2 ( g ) + I2 ( g ) ƒ 2HI( g ); K = .....(iv)
64 Dn
Kp 1
[Note: For a reversible reaction, the equilibrium constant 15. (c) K p = K c (RT ) ; Dn = 1 - 2 = 1 \ =
Kc RT
of the backward reaction is inverse of the equilibrium
constant for the forward reaction.] 16. (a) Zn(s) + 2H + (aq) Zn 2 + (aq) + H 2 (g )
Equation (iv) is the same as the required equation (ii),
0.059 [Zn 2 + ][H 2 ]
1 E cell = E °cell - log
thus K2 for equation (ii) is i.e. option (b) is correct. 2 [H + ]2
64
Addition of H2SO4 will increase [H+]and Ecell will also
5. (d) 2C(s )+O2 (g ) ƒ 2 CO 2 (g ) increase and the equilibrium will shift towards RHS
Dn = 2 – 1 = + 1 17. (a) The reaction given is an exothermic reaction thus
\ Kc and Kp are not equal. according to Le chatalier ’s principle lowering of
temperature, addition of F2 and Cl2 favour the forward
6. (c) 2A(g) + B(g) ƒ 3C(g) + D(g)
direction and hence the production of ClF3.
Mole ratio 2 1 3 1
Molar concentration 1 1 0 0 at t = 0 18. ˆˆ†
(d) For the reaction:- 2NO2 ( g ) ‡ˆˆ 2NO(g) + O2 (g)
Equilibrium molar 1– 0.5 1– 0.25 0.75 0.25
Given Kc = 1.8 × 10–6 at 184 ºC
concentration = 0.50 = 0.75
R = 0.00831 kJ/mol. K
7. (c) ˆˆ† 2SO3 K = 278 (given)
2SO2 + O2 ‡ˆˆ Dn g
K p = K c ( RT )
æ 1ö
ˆˆ† SO 2 + 1 O2
SO3 ‡ˆˆ K' = ç ÷
Dn g = 3 - 2 = 1
2 è Kø
Kp= 1.8 × 10–6 × 0.00831 × 457
= 6.836 × 10–6
1 –2
= = 35.97 ´ 10-4 = 6 × 10 Hence it is clear that Kp > Kc
278
19. (a) NH 4 HS(s ) NH 3 (g ) + H2S (g )
[AB]2 start 0.5 atm 0 atm
8. ˆˆ† 2AB
(c) A2 + B2 ‡ˆˆ Kc = At equib . 0.5 + x atm x atm.
[A 2 ][B2 ]
Then 0.5 + x + x = 2x + 0.5 = 0.84 (given)
(2.8 ´ 10-3 )2 Þ x = 0.17 atm.
Kc =
3 ´ 10 -3 ´ 4.2 ´ 10 -3 p NH3 = 0.5 + 0.17 = 0.67 atm ; p H 2S = 0.17 atm
(2.8)2
= = 0.62 K= p NH3 ´ p H 2S = 0.67 ´ 0.17 atm 2 = 0.1139 = 0.11
3 ´ 4.2
Chemical Equilibrium 203

PT1 1 é K P1 1 ù
20. (c) SO3(g) SO 2 (g ) +
1
O 2 (g) or = êQ = given ú
2 PT2 9 ëê K P2 9 úû
[SO 2 ] [O 2 ]½
KC = = 4.9 ´10 -2 ; PT1 1
[SO 3 ] or = or 1 : 36
PT2 36
On taking the square of the above reaction
[SO 2 ]2 [O 2 ] i.e., (b) is the correct answer.
= 24.01´10 - 4 22. (c) Reaction (c) can be obtained by adding reactions (a)
2
[SO 3 ] and (b) therefore K3 = K1. K2
Now K'C for 2SO2(g) + O2(g) 2SO3 Hence (c) is the correct answer.
[SO 3 ]2 1 23. (a) ˆˆ† 2CO
CO2 + C(graphite) ‡ˆˆ
= = = 416
[SO 2 ]2 [O 2 ] 24.01´10 - 4 Pinitial 0.5atm 0
21. (b) Let the initial moles of X be ‘a’ and that of Z be ‘b’ then Pfinal (0.5 – x)atm 2 x atm
for the given reactions, we have Total P at equilibrium = 0.5 – x + 2x = 0.5 + x atm
0.8 = 0.5 + x
X ˆˆ†
‡ˆˆ 2Y \ x = 0.8 – 0.5 = 0.3 atm
Initial a moles 0 Now Kp = (PCO ) /PCO
2
2
At equi a(1 – a) 2aa
2
(moles) (2 ´ 0.3) (0.6) 2
= = = 1.8 atm
Total no. of moles = a (1 – a) + 2aa (0.5 – 0.3) (0.2)
= a – aa + 2aa 24. (d) For the reaction
= a (1 + a) N2 + O2 ¾¾ ® 2NO K = 4 × 10–4
Hence for the reaction
Dn
(n y ) 2 æ PT ö 1 1 1 1
KP = ´ç 1 ÷ NO ¾¾
® N2 + O2 K'= = = 50
Now, ç n÷ 2 2 K 4×10 – 4
1 nx
è ø
EXERCISE 4
(2aa )2 .PT1
or, K P1 = 1. (c) N2O4 2NO2
[a(1 - a)][a(1 + a)]
0.2/2 2×10–3 /2 Concentration at equilibrium

Z ˆˆ† P + Q
‡ˆˆ [ NO 2 ]2 = 10 -3 ´ 10 -3
Kc = = 1 ´ 10 - 5
Initial b moles 0 0 [ N 2O 4 ] 0.1
At equi b(1 – a) ba ba 2. (b) D G ° = - R T ln K c for DG° = 0 , Kc = 1; since ln 1 = 0
(moles) 3. (a) When moles of gaseous reactants and the products are the
Total no . of moles = b(1 – a) + ba + ba same on both sides, their is no effect of pressure
= b – ba + ba + ba 4. (d) The equilibrium constant does not vary with any factor
other than temperature
= b(1 + a)
3.20 ´ 22
Dn
5. (a) 2HI H2+I2 .It is 22% decomposed , \ = 0.704
nQ ´ nP é PT ù 100
Now K P2 = ´ê 2 ú (3.2–0.704) is equal to HI present at equilibrium which is
n2 ëê n ûú = 2.496
6. (c) H2(g)+I2(g) 2HI(g)
(ba)(ba).PT2
or K P2 = Kp = Kc (RT) Dn ; Dn = 2 - 2 = 0; \ K p = K c
[b(1 - a)][b(1 + a)] 7. (d) When volume is increased the conc. decreases & the
equilibrium shifts in the direction where more moles are
formed.
K P1 4a 2 .PT1 (1 - a )2 4PT1
or = ´ = 2
PCO 4´4
K P2 (1 - a 2 ) PT2 .a 2 PT2 8. (c) Kp = ; Kp = = 8; C(s) = 1; The concentration
PCO2 2
of solids and liquids are taken as unity
204 Chemistry

f K 20 If H 2 O pressure at 25°C is less than 7.92 mm.


9. (c) Kc =K ;Kb =50 ;Kb =0.4
b The reaction
10. (c) The lower the energy of activation, the more is the rate
of reaction CuSO 4 .5H 2O (s) ¾ ¾® CuSO 4 .3H 2 O (s) + H 2 O (g)
11. (d) will not proceed in RHS.
12. (d) With change of pressure, x will change in such a way that
2 3
Kp remains constant. 22. (d) K p = p B 1 ´ p C 1
13. (a) For the equilibria : N2O4(g) 2NO2(g)
Again, K p = p B2 2 ´ (2p C1 )3
K P = K C ´ (RT )Dn Since temperature is constant so KC
or KP will remain constant. Further since volume is halved, 2 3 2 3
\ p B1 ´ p C1 = PB 2 ´ 8 p C1
the pressure will be doubled so a will decrease so as to
maintain the constancy of KC or KP. p 2B 1 p B1
14. (d) The condition for the equilibrium is \ = p 2
B2 or,, = p B2
8 2 2
( DG) T,P = 0 and ( DS) U,V = 0

a2 p [B] [C] éDù


15. (c) K p = 23. (a) = 2, = 4 and ê ú = 6
[ A] [B] ëCû
(1 - a 2 )
Multiply the three equations,
or K p (1 - a 2 ) = a 2 p
[D]
2×4× 6= = Kc
\ K p = a 2 (K p + p) [ A]

d ln K DH º 24. (b) N 2 (g) + 3H 2 (g) 2 NH 3(g)


16. (a) In Van’t Hoff equation = when d ln K < 0
dT RT 2 dT 1–x 3 – 3x 2x at equilibrium
we will get negative value for DHº, the reaction will be Total moles,
exothermic in nature. 1 – x + 3 – 3x + 2x = 4 – 2x = 3 (given)
17. (b) Van’t Hoff reaction isotherm is DG = DGº+RT ln Qp (Since 4 moles = 4 atm given)
When the reaction is in a state of equilibrium \ x = 0.5
DG = 0
p N 2 ´ p 3H 2
Then DGº = – RT ln Qp = – RT ln Kp K p for dissociation of NH 3 =
p 2N H 3
ln K p DHº
18. (c) Van’t Hoff isochore is =
dT RT 2 3
æ 1 - 0 .5 ö é æ 3 - 3 ´ 0 .5 ö ù
T2 ç ´ 3 ÷ ´ êç ÷ ´ 3ú
DH º è 3 ø ëè 3 ø û
integrating ln K 2 - ln K1 = - d (1 / T) = 2
R é 2 ´ 0 .5 ´ 3 ù
T1
ê 3 ú
ë û
19. (d) Kp and KC are independent of pressure. Kx is related to
pressure.
= 0.5 × (1.5) 3 atm 2
20. (b) At equilibrium the moles of Cl 2 must be = 0.15 × 3 = 0.45
25. (b) Introduction of CO will suppress the dissociation of
PCl 5 PCl 3 + Cl 2 COCl 2 and concentration of Cl 2 will decrease. This
x - 0.45 0.45 0.45 will favour the dissociation of PCl 5 .
Eqb. Conc.
3 3 3 26. (a) Instability constant
éPCl3 ûù [ Cl 2 ] 2.0 ´ 10 -2
Kc = ë =
1
=
K ( reverse )
=
éëPCl5 ùû formation constant K (forward ) 1.0 ´ 10 6
0.15´ 0.15 = 2.0 ´10-8
\ 0.04 = \ x = 2.1 moles
(x - 0.45) / 3
27. (c) DG ° = – 2.303 RT log K p
21. (b) The efflorescent salts loss water to atmosphere
K p = p 2 = 1.086 ´ 10 -4 ; = – 2.303 × 8.314 × 300 log 10 20
H 2O
DG ° » – 115 KJ
p H 2O = 1.042 ´ 10 -2 atm = 7.92 mm
Chemical Equilibrium 205

28. (a) PCl 5 PCl3 + Cl 2 a – 3x = 2x (condition given)


1–x x x at pressure P a
\x =
5
1 – x1 x1 x1 at pressure P1
The % of B reacted is
x x 3x 3 a
.P .P ´ 100 = . ´ 100 = 60%
x 2p x12 P1
Kp = 1 + x 1 + x = = a a 5
1- x (1 - x 2 ) (1 - x12 )
.P
1+ x 33. (b) I - (aq) + Ag + (aq) ¾ ¾® AgI(s)
0.8 mol at equilibrium.
Put x = 0.5, P = 4 atm, x1 = 0.8, P1 = ?
Thus mole of I - reacted = 1.0 – 0.8 = 0.2
P1 will come out be 0.75 atm
29. (b) 2NOBr (g) 2NO(g) + Br2 (g) I- + I2 I 3 - (aq)
1 mol 0.25 mol 0 mol
6P 2P P 1 – 0.2 0.25 – 0.20 0.2
9 9 9 = 0.8 = 0.05
Since total pressure is P.
I 3- 0.2
6 P 2P P Stability constant K = = =5
So, + + =P -
[I ] [I 2 ] 0.8 ´ 0.05
9 9 9
2 34. (d) A + 2B 2C + D
æ 2P ö P 2 3 0 0 initial conc.
p 2NO ´ p Br2 ç ÷ ´
9 9
\ Kp = = è ø 2 2 – x 3 – 2x 2x x equilibrium conc.
p 2N O B r æ 6P ö Again 2 – x = 2x
ç ÷ \ x = 2/3
è 9 ø
[2x ]2 [ x ]
Kp1 Kc =
\ = [ 2 – x ][3 – 2 x ] 2
P 81
NH +4 (aq) + H 2O(l) Put x = 2/3 \ K c = 0.32
30. (c) NH 4OH(aq) + H + (aq)
+ 35. (d) AB5 (g) AB(g) + 2 B2 (g)
[N H 4 ] 1– a a 2a
K1 = = 1.8 × 10 9 ;
[ NH 4 OH ][H + ] Total moles at equilibrium = 1 + 2 a
20 ´ 2
NH 4 OH (aq) NH +4 (aq) + OH - (aq) = 1+ = 1. 4
100
[ NH 4+ ] [OH - ] p1 nT 1 1´ 300
K2 = = 1 1; =
[ NH 4 OH ] p 2 n 2 T2 p 2 1.4 ´ 600
Multiply this by [H + ] and divide also \ p 2 = 2.8 atm

[NH 4+ ] [OH – ] [H + ] 36. (a) PCl 5 PCl 3 + Cl 2


K2 = = K1 ´ K w 1 – 0.5 0.5 0.5 Moles at equilibrium
[NH 4 OH] [H + ]
Total moles = 1.5
= 1.8 ´ 10 9 ´ 1.0 ´ 10 -14 = 1.80 ´ 10 -5 0.5 0.5
P ´ P P
31. (b) PCl 5 PCl 3 + Cl 2 1.5 1.5
Kp = = \ P = 3 Kp
1– a a a 0.5 3
P
1.5
P´ P a 2P
\ Kp = 1 + 1+ - E / RT
= 37. (c) K f = Ae - E f / RT and K r = Ae r
1- 1- a 2
P
1+ Kf
\ = K c = e E r -Ef / RT
Kr
Kp
or, K p = a 2 P \a = when 1 - a 2 = 1 Er - Ef (8.000 – 10.3010) ´ 1000
P or, log K c = =
RT ´ 2.3010 2.3010 ´ 2 ´ 500
32. (a) A + 3B 2C + D
2a a 0 0 \ K c = 0.1
2a – x a – 3x 2x x
206 Chemistry

28 6 PM 1 ´ 82.1
38. (b) Moles of N 2 = = 1, Moles of H 2 = =3 40. (a) D = = = 2.5 g L-1
28 2 RT 0.0821´ 400
500 ´ 1
= 0.5 d = 2.0 g L-1 (given)
Moles of H 2SO 4 required =
1000 D-d 2.5 - 2
Moles of NH 3 neutralised by H 2SO 4 = 1.0 a= = = 0.125 = 12.5%
d(n - 1) 2(3 –1)

( 2NH 3 + H 2SO 4 ¾¾
® (NH 4 ) 2 SO 4 ) 41. (a) The reaction quotient Q is
pC ´ pD
. In the initial stage
pA ´ pB
Hence 1 mol of NH 3 by the reaction between N 2 and
C and D are not present. So p C and p D are zero. Then
H2 .
With the passage of the time p C and p D increase and
N 2 + 3H 2 2 NH 3
1 3 0 initial p A and p B decrease.
1 – 0.5 3 – 0.5 × 3 1 at equilibrium 1
42. (a) For exothermic reaction, K c µ
1´ 1 Temperature
Kc = = 0.592
0.5 ´ (1.5) 3 43. (c) Since the melting is endothermic increasing temperature
will favour melting. Since density of solid is less
39. (a) Eq. wt. of SO 2 = Mol. wt./2 (SO 2 ¾
¾® SO 3 )
æ Mö
2- 3+ çD = ÷ , the high pressure will reduce V and increase
Eq. wt. of K 2Cr2 O 7 = Mol. wt./6 (Cr2 O 7 ® 2Cr ) è Vø
1 6 D, hence more melting.
mol of K 2 Cr2 O 7 =
3 3 p 2HI
44. (b) K p =
or, 2 geq of K 2Cr2 O 7 = 2 geq of SO 2 = 1 mol of SO 2 p H 2 ´ p I2
It means only 1 mole of SO 2 reacted with O 2 . Addition of I 2 will increase p I 2 , hence to keep K p
2 SO 2 + O 2 2SO 3
1 mol 1 mol 0 mol initial constant, either p H 2 should decrease or p HI should
1 mol (1 – 0.5) mol 1 mol at equilibrium increase. Hence the graph (B).
\ Total moles at equilibrium = 1 + 0.5 + 1 = 2.5
2
æ 1 ö
2 ç ´ 5÷
p SO 3 è 2. 5 ø
Kp = = = 1.0
2
p 2SO 2 ´ p O 2 æ 1 ö æ 0.5 ö
ç ´ 5÷ ´ ç ´ 5÷
è 2 .5 ø è 2.5 ø
7B
Ionic Equilibrium
ARRHENIUS THEORY OF IONISATION : 5. Solvation : The more the solvation, the more is the
On the basis of colligative properties of solutions of salts, acids ionisation.
and bases, Arrhenius proposed the theory of ionisation i.e. 6. Presence of the ions in the solution : Ionisation decreases
spliting of these substances into ions in solution. It is reversible in presence of common ions.
process, effects electrical conductivity, colligative properties like ELECTROLYTE :
depression in freezing point, elevation in boiling point, lowering A substance which splits into ions in solution is called electrolyte.
of vapour pressure, osmotic pressure. It can be an acid, base or salt.
EVIDENCES IN FAVOUR OF IONISATION : (i) Strong electrolyte : Which dissociates almost completely
into ions even in concentrated solution eg. NaOH, KOH,
1. X-ray diffraction studies
HCl, H2SO4, NaCl, CaCl2.
2. Ionic reactions
(ii) Weak electrolyte : Which dissociates to a small extent into
3. Heat of neutralisation ions in solution eg CH3COOH, NH4OH etc.
4. Colour of compounds and their solutions Note : Salts are always strong electrolytes.
5. Colligative properties
OSTWALD'S DILUTION LAW :
6. Conductance of electrolytes in solution
The degree of ionisation or dissociation (a) of weak electrolytes
DEGREE OF IONISATION OR DISSOCIATION (a) : increases with dilution and law of mass action can be applied to
The fraction of the total number of molecules which is ionised at them.
the equilibrium state is known as degree of ionisation or AB A+ + B–
dissociation.
C 0 0 initial conc.
Number of moles ionised C(1-a) Ca Ca equilibrium conc.
a= Ionisation constant K = Ca2
Total number of moles
K 1
FACTORS AFFECTING IONISATION OR \a = aµ
DISSOCIATION : C C
1. Nature of electrolytes : The stronger the electrolyte, the Concentration of A+ or B– = Ca
more is the ionisation and vice versa. a approaches unity with dilution.
2. Nature of solvent : The more the dielectric constant of ACIDS AND BASES :
solvent, the more is the ionisation. 1. Arrhenius concept : An acid is a substance that dissociates
3. Concentration : The lesser the concentration, the more is to give hydrogen ions when dissolved in water eg. HCl,
the ionisation. CH3COOH, H3PO4.
4. Temperature : The higher the temperature, the more is the A base is a substance that dissociates to give hydroxyl ions
ionisation. when dissolved in water eg. NaOH, Ca(OH)2.
208 Chemistry
2. Lowry and bronsted concept : An acid is a substance which Soft bases : Species having donor atom of lower
has a tendency to donate a proton (H+) to any other electronegativity and higher polarisability e.g. P, As, S, Se
substance. etc. Examples R3P, R2S, I–.
A base is a substance which has a tendency to accept a 7. Lux-flood concept of acids and bases : An oxide ion donor is
proton (H+) from any other substance. a base and an oxide ion acceptor is an acid.
Acid H+ + Base BaO + SiO 2 ¾
¾® BaSiO 3
Acid and base differing by a proton are known as conjugate base acid
pair. The weaker the acid, the stronger the base in conjugate MgO + SO 3 ¾
¾® MgSO 4
pair and vice versa. base acid
Acid1 + Base2 Acid2 + Base1 8. Ingold concept : All electrophiles are acids and nucleophiles
+ – are bases.
HCl + H2O H3O + Cl
+ –
H2SO4 + H2O H3O + HSO4 STRENGTH OF ACIDS AND BASES :
+ –
HCl + NH3 NH4 + Cl The greater the value of Ka or Kb the stronger is the acid or base.
+
Conjugate acid Conjugate base + H the smaller the value of pKa the sronger is the acid.
3. Lewis concept : An acid is a substance which can accept a pKa values of some acids (Acid strength)
pair of electrons from any other substance e.g. BF3, AlCl3 Acid Base Approximate pKa
(incomplete octet), SnCl4, SF4 (central atom has vacant d- 1. HClO4 ClO4– –10
orbital) or cations Fe3+, Cu2+ etc. 2. HI I– –10
A base is a substance capable to donating a pair of electrons –
3. H2SO4 HSO4 –9.5
to any other substance eg. anions X – , OH – ,
4. HBr Br – –9
CN– or neutral molecules having lone pair(s) of electrons on
.. .. 5. HCl Cl – –7
..
one or more atom R O
..H,
NH 3 , H 2 O
. . etc. 6. Ar. SO3H ArSO3– –6.5
Lewis acid may be any of the following types of substances 7. CH(CN)3 C–(CN)3 –5
1. Molecules having an atom with incomplete octet
2. Simple cations 8. H- C-H H- C - H –4
|| ||
3. Molecules with central atom having empty Å OH O
d-orbitals
4. Molecules with a multiple bond between atoms of Å
9. R - O- R R–O–R –3.5
different electronegativities |
Strength of some Lewis acids H
BX3 > AlCl3 > FeX3 > GaX3 > SbX5 > InX3 > SnX4 > 10. H3O+ H2O –1.74
AsX5 > ZnX2 > HgX2 11. HNO3 NO3– –1.4
4. Extended Lewis concept : When the central atom is bonded
12. HSO4– SO4 –– 1.99
to atoms of different electronegativities by multiple bonds,
the substance is known as extended Lewis acid e.g. CO2, 13. HF F– 3.17
CS2 etc. 14. HNO2 NO2– 3.29
Extended Lewis base e.g. CO and unsaturated hydrocarbons +
like alkenes, alkynes etc. are also known as border line Lewis 15. Ar NH 3 ArNH2 3.5
bases. 16. RCOOH RCOO– 4.5
5. Hard acids : Cations of lighter elements, smaller size, higher
17. H2CO3 HCO3– 6.35
charge not easily polarisable e.g. light alkali and alkaline
metal ions of B, Al, Si, Ti4+, Cr3+, Co2+, Fe3+ (lighter transition 18. H2S HS– 7.00
elements). 19. NH4+ NH3 9.24
Soft acids : Cations of heavier elements, larger size, lower 20. ArOH ArO– 8-11
charge and easily polarisable e.g. heavy transition metal 21. HCO3– CO3 –– 10.33
ions (second and third row) e.g. Hg2+, Pd2+, Cd2+, Cu+, 22. H2O OH– 15.74
Ag+, Hg+ etc.
6. Hard bases : Species having donor atoms of higher 23. RCONH2 RCONH 17
electronegativity and low polarisability e.g. N, O, F, Cl etc. 24. RCH2OH RCH2O– 18
Examples H2O, NH3, ROH.
Ionic Equilibrium 209
25. HC º CH HC º C– 25 Ionisation of polybasic acids : Polybasic acids ionise in various
26. NH3 NH2– 34 steps e.g. Orthophosphoric acid H3PO4.
K1
27. Ar. CH3 ArCH2– 35 H3PO4 H+ + H2PO4–
28. CH2=CH2 CH2=CH– 36.5
K2
29. PhH Ph– 37 H2PO4– H+ + HPO4– –
30. CH4 CH3– 40 K3
HPO4– – H+ + PO4– –
31. C2H6 C2H5– 42
32. Cyclohexane (C6H12) C6H11– 45 K1 > K2 > K3 and overall dissociation const. K = K1 × K2 × K3
(i) Relative strengths of acids: For weak acid Ka = Ca2. For AMPHOTERIC OR AMPHIPROTIC SUBSTANCE OR
AMPHOLYTES :
two acids with dissociation constants Ka1 and K a 2 at the
A substance acting as an acid as well as a base, eg. water acts as
same concentration C, an acid with ammonia and as base with acetic acid. A substance
acting as proton donor and proton acceptor.
a1 K a1 Strength of acid HA1
= = COMMON ION EFFECT :
a2 Ka2 Strength of acid HA 2
The degree of ionisation of an electrolyte is suppressed by the
(ii) Relative strengths of bases : For weak base addition of another electrolyte having a common ion. This is
known as common ion effect e.g. ionisation of CH3COOH is
Kb = Ca2. For two bases with dissociation constants
suppressed by the addition of HCl or CH3COONa.
K b1 and K b2 at the same concentration C. (i) It helps in controlling the concentration of ions furnished
by weak electrolytes.
a1 K b1 Strength of base B1OH (ii) It effects the solubility of salts.
= = Mixture of weak acid and its salt with a strong base:
a2 K b 2 Strength of base B 2 OH
The hydrogen ion (H+) concentration of a mixture of a weak acid
LEVELLING EFFECT : HA and its highly dissociated salt say NaA is given by
All the strong acids in aqueous, solution appear almost equally (I) HA H+ + A–
strong since water acts as strong base. For example HClO4, HBr, (II) NaA Na+ + A–
H2SO4, HCl and HNO3 appear equally strong.
HA + H2O H3O+ + A– [ H + ][ A - ]
Ka =
Hence relative strengths in aqueous solution cannot be compared. [ HA ]
This phenomenon is known as levelling effect.
K a [ HA ] K a [ ACID ]
EFFECT OF SOLVENT ON ACID STRENGTH : [H+] = =
- [SALT ]
[A ]
(i) In acetic acid :
HA + CH3COOH CH3COOH2+ + A– HA being weak acid and secondly due to common ion (A– )
remains almost unionized. Salts are almost 100% ionised.
As acetic acid has a little tendency to accept proton, even
Mixture of weak base and its salt with a strong acid
strong acids are feebly ionised in acetic acid. For
The hydroxyl ion (OH–) concentration of mixture of weak base
example HClO 4 > HBr > H 2 SO 4 > HCl > HNO 3 BOH and its highly dissociated salt say BCl is given by
.. (I) ionisation of BOH B+ + OH– (negligible)
(ii) In liquid NH3 : HA + N H 3 NH4 + + A–
(II) ionisation of BCl B+ + Cl– (100%)
As ammonia has a great tendency to accept proton, even
weak acids appear strong in liquid ammonia. For example [B+ ][OH - ] K [BOH]
HCl, HNO3 and CH3COOH appear equally strong in liquid Kb = ; [OH - ] = b +
[BOH] [B ]
ammonia.
(iii) In HF : Since HF is a strong acid, the other acids act as a
K b [BASE]
base when dissolved in HF eg. [OH - ] =
[SALT]
+
HNO3 + HF H 2N O3 + F- Ex.: Calculate the [H+] concentration of solution which is 0.1 M in
HA and 0.5M in NaA .Ka of HA is 1.8 × 10–6.
RELATION BETWEEN Ka AND Kb :
Ka × Kb = Kw or pKa + pKb = pKw = 14 at 25°C. + K a [ACID] 1.8 ´ 10 -6 ´ 0.1
Sol. : [H ] = = = 3.6 ´ 10 -7 M
[SALT] 0.5
210 Chemistry
Ex. :What will be the H+ ion concentration of a solution obtained --
by mixing 500 ml of 0.20 M CH3COOH and 500 ml of 0.30 M Ag2 S Ksp = [Ag+]2 [S ]
CH3COONa (Ka of CH3COOH= 1.8 × 10–5) --
Sb2S3 Ksp = [Sb3+]2 [S ] 3
Sol. :Concentration of CH3COOH in the mixture
Ex.: Ksp of AgCl is 2.8 × 10–10 at 25°C. Calculate the solubility of
0.20 ´ 500 AgCl in (I) Pure Water (II) 0.1 M AgNO3 solution (III) 0.1 M
= = 0.10 mol / l
1000 NaCl.
Concentration of CH3COONa in the mixture Sol. (I) Let S mole/l be the solubility of AgCl
Ksp = [Ag+][Cl–] = S × S = S2;
0.30 ´ 500
= = 0.15 mol / l
1000 \ S = K sp = 2.8 ´ 10 -10 = 1.67 ´10 -5 mol / l

1.8 ´ 10 -5 ´ 0.10 (II) AgCl Ag+ + Cl–


[H + ] = = 1.2 ´ 10 -5 mol / l
0.15 Ksp = [Ag+] [Cl–]
SOLUBILITY PRODUCT (Ksp) : Solubility of AgCl will be equal to
At constant temperature and pressure the saturated solution of a K sp 2.8 ´ 10 -10
sparingly soluble salt has an equilibrium between the excess of [Cl - ] = +
= = 2.8 ´ 10 -9 mol / l
[Ag ] 0.1
the solute and the ions furnished by it. e.g.
AgCl AgCl Ag+ + Cl– (III) Solubility of AgCl will be equal to
Solid dissolved but ions in sol. K sp 2.8 ´10 -10
undissolved not ionised [Ag + ] = = = 2.8 ´ 10-9 mol / l
[Cl - ] 0.1
[Ag + ][Cl - ] Ex. :Solubility product of SrF2 in water is 8 × 10–10. Calculate the
Applying law of mass action, K =
[AgCl] solubility in 0.1 M NaF aqueous solution.
or K[AgCl] = [Ag+] [Cl–], Ksp = [Ag+] [Cl–] Sol. : SrF2 Sr++ + 2F–
The constant Ksp is known as solubility product. It is equal to the Solubility of SrF2 will be equal to
product of the concentration of ions in saturated solution.
K sp 8 ´ 10 -10
When Ksp > [Ag+] [Cl–] Solution is not saturated [Sr + + ] = - 2
= 2
= 8 ´ 10 -8 mol / lit
+ – [F ] (0.1)
When Ksp < [Ag ] [Cl ] Solution is supersaturated
and precipitation takes place DISSOCIATION CONSTANT OF WATER / IONIC
+
When Ksp = [Ag ] [Cl ] – Solution is saturated PRODUCT OF WATER (KW) :
For general electrolyte AxBy. Water being weak electrolyte is slightly ionised as follows :
Ksp = [A+y]x [B–x]y
2H 2 O H 3O + + OH -
(I) Ksp is independent of the source of ions.
(II) Helps to know the solubility of electrolytes. [H 3O + ][OH - ]
(III) Predicting ionic reactions. K= or K[H 2 O]2 = [H 3O + ][OH - ]
[ H 2 O] 2
(IV) Qualitative analysis.
(V) Purification of common salt, salting out of soap and Solvay or K w = [H 3 O + ][OH - ]
ammonia soda process.
Kw is known as ionic product of water or dissociation constant
Relation between solubility product (Ksp) and solubility (S).
of water. It is equal to the product of concentration of [H3O+] and
1. For binary electrolyte e.g. AgCl, BaSO4
[OH–] ions in water. At constant temperature of 25°C, the value of Kw
S= K sp is 1.0 × 10–14.
In pure water [H3O+] = [OH–] = 1.0 × 10–7 at 298 K
2. For ternary electrolyte e.g. CaF2, PbI2 Molar concentration of water is 55.55 mol/lit
1 K[H2O] = [H+] [OH–]
æ K sp ö3 K × 55.5 = Kw.
S = çç ÷
÷ Hence ionic product of water is 55.5 times greater than K. Kw
è 4 ø
increases with temperature.
Representation of Ksp for various electrolytes. The addition of salt, acid or base does not change value of KW.
Mg(OH)2 Ksp = [Mg++] [OH–]2 Its value changes with temperature only.
Ionic Equilibrium 211
Hydrogen and hydroxyl ion concentration in aqueous solution of SALT HYDROLYSIS :
Acids and Bases. Salts are strong electrolytes and on dissolution in water split into
When an acid is added to water H+ (aq.) ion combine with OH– ions which react with H+ or OH– ions furnished by water yielding
(aq.) ions so that Kw remains constant. Thus addition of an acid acidic or basic solution. The process is known as salt hydrolysis.
decreases the conc. of OH– (aq.) ions and addition of base 1. Hydrolysis of salt of a strong acid and strong base : e.g.
decreases the conc. of H+ ions. In both cases the self ionisation KCl, NaCl, Na2SO4 etc.
of water is suppressed due to extra supply of H+ or OH– ions.
KCl + H 2O KOH + HCl
EXPRESSING HYDROGEN ION CONCENTRATION
: pH SCALE Representing strong electrolytes as ions
Any aqueous solution of some electrolyte or non electrolyte K + + Cl - + H 2 O K+ + OH– + H+ + Cl–
having equal concentrations of H+ and OH– ions, is neutral. It
has been observed that H+ ion concentrations can usually vary we have overall reaction H2O H+ + OH–
from 0 to 1 ´ 10 -14 moles/l. Sorensen represented the acidic or The solution is neutral since [H+] = [OH–], pH = 7, thus salts
basic character of an aqueous solution in terms of pH. The pH of of strong acids and strong bases are not hydrolysed.
a solution is the numerical value of the negative power to which 2. Salts of Weak Acids and Strong Bases : e.g. CH3COONa,
ten must be raised to express the H+ ion concentration. KCN
[H+] = 10–pH CH3COONa + H2O CH3COOH + NaOH
In pure water [H+] = 10–7 = 10–pH at 25ºC or 298 K
Hence pH of pure water is 7. or CH3COO– + Na+ + H2O CH3COOH+ Na+ + OH–
log [H+] = – pH log 10 CH3COO– + H2O CH3COOH + OH–
1 (i) Due to OH– ions, the solution becomes basic, hence
\ pH = – log (H+) = log salt is hydrolysed.
[H + ]
Thus pH of a solution is the negative logarithm of hydrogen ion (ii) Hydrolysis is known as anionic hydrolysis because
concentration. Similarly, negative logarithm of hydroxyl ion CH3COO– is an anion.
concentration is known as pOH. Kw
pOH = –log[OH–] (iii) Salt hydrolysis constant Kh =
Ka
An aqueous solution having pH value less than 7, is acidic and
more than 7 at 298 K is basic. (iv) pH of solution more than 7
Strongly Moderate Weakly Weakly Moderate Strongly
Kw
(v) Degree of hydrolysis a =
acidic acidic acidic basic basic
Ka ´ C
0 1 2 3 4 5 6 7 8 9 10 11 12 13 14 (pH values)
pH + pOH = 14 + Kw ´Ka
The pH changes with temperature. It decreases with rise in (vi) [H ] =
C
temperature.
pH range of some important substances: 1 1
(vii) pH = 7 + pK a + log C ,
Subs. pH range Subs. pH range 2 2
Tears 7.4 Vinegar 2.4 - 3.4 where C is conc. of anion i.e. CH3COO–
Blood plasma 7.3 - 7.42 Soft drinks 2.0 - 4.0 3. Hydrolysis of salts of weak bases and strong acids : e.g.
Human saliva 6.5 - 7.5 Gastric Juice 1.0 - 3.0 NH4Cl
Human Urine 4.8 - 8.4 Boiling water 6.56
Ex. :What will be the resultant pH when 200 ml of an aqueous NH 4Cl + H 2 O NH 4 OH + HCl
solution of HCl (pH = 2) is mixed with 300 ml of an aqueous
or NH 4+ + Cl - + H 2 O NH 4 OH + H + + Cl -
solution of NaOH (pH = 12).
Sol. :HCl and NaOH are strong electrolytes. NH +4 + H 2 O NH 4 OH + H +
pH of HCl = 2 [HCl] = 10–2 M
(i) Due to H+ ions, the solution becomes acidic, hence
pH of NaOH = 12 [NaOH] = 10–2 M
salt is hydrolysed
Mev. of HCl = 200 × 10–2 = 2
(ii) Hydrolysis is known as cationic hydrolysis.
Mev. of NaOH = 300 × 10–2 = 3
On mixing mev. of NaOH left = (3–2) = 1, because NH +4 is a cation
gev of NaOH = 0.001
[OH–] = 2 × 0.001 = 0.002 Kw
(iii) Salt hydrolysis const. Kh =
– log [OH–] = pOH. Kb
\ pOH = 2.6989; pH = 14 - 2.6989 = 11.3011 (iv) pH of solution less than 7
212 Chemistry
Ex. :Calculate the pH of an aqueous soln. of 1.0 M ammonium
Kw
(v) Degree of hydrolysis µ = formate assuming complete dissociation (pKa of formic
Kb ´ C acid = 3.8 and ammonia = 4.8) (IIT 1995)

K w .C 1 1
[H + ] = Sol. pH = (pK w + pK a - pK b ) = (14 + 3.8 - 4.8) = 6.5
(vi) Kb 2 2
BUFFERS :
1 1
(vii) pH = 7 - pK b - log C Solutions which resist the change in the value of pH when small
2 2 amount of acid or base is added to them are known as buffers.
4. Hydrolysis of salts of weak bases and weak acids : e.g. Types
CH3COONH4 (I) Simple buffers : Solution of salt of weak acid and weak base
CH3COONH4 + H2O CH3COOH + NH4OH CH3COONH4, NH4CN.
or CH3COO– + NH4+ + H2O CH3COOH + NH4OH (II) Mixed buffers :
(a) Acidic buffers : Solution of equimolar mixture of a weak
(i) The solution may be neutral, acidic or basic depending
upon the relative strength of weak acid CH3COOH acid and its salt with a strong base e.g. (CH3COOH +
and weak base NH4OH formed. The salt is said to be CH3COONa); (H2CO3 + NaHCO3); (Boric acid + borax);
hydrolysed. (H3PO4 + NaH2PO4)
(ii) Hydrolysis is known as cationic as well as anionic (b) Basic buffers : Solution of equimolar mixture of a weak
hydrolysis. base and its salt with a strong acid e.g. (NH4OH + NH4Cl)
Buffer action of simple buffer : CH3COONH4 exist as ions
Kw
(iii) Salt hydrolysis Const., Kh = CH3COO– and N + H 4 in solution. Added acid (H+) combine with
Ka ´ Kb
(iv) pH of the solution may be 7, > 7 or <7 CH3COO– to give feebly ionised CH3COOH and added base
(OH–) combine with NH4+ to give feebly ionised NH4OH. Thus
Kw pH remains unchanged
(v) Degree of hydrolysis, a =
Ka ´ K b Buffer action of mixed buffers :
(a) Acidic buffer :
K w ´ Ka
(vi) H+ = +
Kb
CH3COONa CH 3 COO - + Na CH3COOH
H+
1 1 1
(vii) pH = pK w + pK a - pK b
2 2 2 When small amount of an acid is added to the buffer the
Ex. :Calculate the degree of hydrolysis and hydrolysis constant added H+ ions combine with CH3COO– to form feebly
of decinormal solution of NaCN. The dissociation constant ionised CH3COOH and when small amount of a base is added
of HCN is 7.2 × 10–10 at 25°C. to the buffer the added OH– ions combine with H+ to form
feebly ionised H2O. In the latter case more CH3COOH ionises
Kw 1´ 10-14 to keep K a of acid constant and hence constant
Sol. a = = = 1.178 ´10 - 2
C´ Ka 0.1´ 7.2 ´10 -10 concentration of H+ ions. Thus whether we add acid or a
base, the H+ concentration remain constant and pH of
Kw 1´10 -14 solution does not change
Kh = = = 1.388 ´10 -5
K a 7.2 ´10 -10 (b) Basic buffer :
Ex. :How many grams of sodium acetate must be added NH4Cl NH 4+ + Cl - NH4OH
to 500 ml of water to give a solution of pH = 8.52 OH -
(Ka = 1.8 × 10–5)
When small amount of an acid is added to the buffer the
Sol. Sodium acetate is salt of weak acid and strong base
added H+ ions combine with OH– to form feebly ionised
1 H2O, equilibrium is disturbed. More NH4OH ionises to keep
pH = 7 + pK a + log C
2 [OH–] fixed.
1 When small amount of a base is added to the buffer the
8.52 = (14 + 4.74 + log C) added OH– ions combine with NH4+ to form feebly ionised
2
NH4OH to keep [OH–] fixed. Hence there is no change of pH
(- log K a = pK a \ - log 1.8 ´ 10-5 = 4.74) in both cases.
C = 0.02 mol/litre
Hence wt. of CH3COONa in 500 ml = 0.01 × 82 = 0.82 g
Ionic Equilibrium 213
Buffer capacity = + -
+ - + -
H Cl + Na OH Na Cl + H 2 O
Moles of acid or a base added to 1 litre of buffer
Change in pH Net reaction H+ (aq) + OH– (aq) H2O,

Calculation of pH value of buffers : Acidic buffer contains DH = –13.7 kcal/mole


weak acid and its salt with strong base. Neutralisation is complete. pH of resulting solution is 7.
The H+ ion concentration is given by (II) Neutralisation of a weak acid with a strong base : While
sodium hydroxide is completely ionised in aqueous solution,
K a [ACID] the acid is only weakly ionised.
[H + ] =
[SALT] CH3COOH + NaOH CH3COONa + H2O
Net reaction CH3COOH + OH– CH3COO– + H2O,
[SALT] DH = –13.20 kcal/mole.
- log[H + ] = - log K a + log
[ACID] Solution remains basic (pH > 7) due to presence of OH–
ions. Heat of neutralisation is less by 0.5 kcal/mol which is
[SALT] utilised for the dissociation of acetic acid.
pH = pKa + log
[ACID] (III) Neutralisation of a strong acid and weak base : The strong
This is known as Henderson's equation. acid is completely ionised in aqueous solution and weak
Basic buffer contains a weak base and its salt with strong base is feebly ionised.
acid. The OH– ion concentration is given by NH4OH + HCl NH4Cl + H2O
K b [BASE ] Net reaction NH4OH + H+ NH4+ + H2O,
[OH - ] =
[SALT] DH = –12.3 kcal/mole.
The solution remains acidic due to presence of H+ ions
[SALT]
- log[OH - ] = - log K b + log (pH < 7) 1.4 kcal/mole is utilised for dissociation of NH4OH.
[BASE ] (IV) Neutralisation of a weak acid and a weak base : The acid
and base are weakly ionised in aqeous solution.
[SALT]
pOH = pKb + log +
[BASE] (1) CH3COOH + Water CH 3COO - + H
Note : pH + pOH = 14 = pKw
(2) NH 4 OH + water NH +4 + OH –
A buffer has maximum buffer capacity when
Net reaction H+ + OH– H2O,
[SALT] [SALT ]
or =1 DH = –11.8 kcal/mol
[ACID] [BASE ]
The combination of H+ and OH– take place to a limited extent
In such case pH = pKa for acid buffer and pOH = pKb for
only. This combination is suffiecient to disturb., the
basic buffer. For a buffer the ratio of concentration of salt to
equilibrium in (1) and (2) and dissociation is complete.
acid or base must lie between 0.1 and 10. Thus pH range for
When neutralisation is completed, pH is almost 7. Heat
acid buffer is from pKa – 1 to pKa + 1. For basic buffer the
evolved during neutralisation is even less, due to effecting
pOH range is pKb – 1 to pKb + 1.
the ionisation of weak acid and weak base.
NEUTRALISATION :
HYDROGEN ION INDICATORS OR ACID BASE
The combin ation of H +(aq) and OH – (aq) furnished by INDICATORS :
an acid and a base in aqueous solution to produce undissociated
Such indicators change their colour with the change of the pH of
water to maintain the value of KW at
solution. They are either weakly acidic or basic in nature e.g.
1 × 10–14, is known as neutralisation.
phenolphthalein which is colourless below pH 8 and pink at 9.8.
HA + Water H+(aq) + A–(aq) Theory of Acid-Base indicators :
BOH + Water OH– (aq) + B+(aq) (1) Ostwald's theory : The unionised molecule of the indicator
has one colour while the ionised form has another colour.
H+(aq) + OH–(aq) H2O
HIn H+ + In–
(I) Neutralisation of a strong acid with strong base : The
strong acids and strong bases are almost completely ionised Colour A Colour B
in aqueous solutions. When their solutions are mixed, the
[H + ][In - ]
only effective change is the formation of unionised water. K In =
[HIn]
214 Chemistry
Consider the ionisation of phenolphthalein which is weakly Methyl Orange :
acidic in nature

HPh H+ + Ph– SO3 N=N N(CH3)2
Colourless Pink
Addition of a strong base will disturb the equilibrium (OH– Benzenoid form (Yellow)
+ H+ H2O) and more phenolphthalein will ionise
giving pink solution. Addition of a strong acid will suppress +

N=N N(CH ) SO3 NH – N N(CH3)2
the ionisation of phenolphthalein by common ion effect,
solution will be colourless. The indicator is not suitable for
titrating weak base like NH4OH against strong acid.
Benzenoid The
form (Yellow) Quinonoid form (Red)
OH– ions furnished by weak base are insufficient to shift
the equilibrium and pink colour does not appear just at the Types of indicators
end point. Excess of weak base is required. 1. Internal : Which can be added to reacting substances further
Action of methyl orange : Methyl orange is a weak base they can be acid base indicator (phenolphthalein, methyl
and is ionised as follows : orange etc.) redox indicators, used for redox titrations (N-
phenyl anthranilic acid) adsorption indicators (starch).
MeOH Me+ + OH–
2. External : Which can not be added to reacting substances
Yellow Red e.g. potassium ferricyanide for titration of K2Cr2O7 vs.
Addition of a strong acid will disturb the equilibrium (H+ + FeSO4(NH4)2SO4.6H2O.
OH– H2O) and more methyl orange will ionise giving 3. Radioactive : 8O18 and D2 for studying reaction mechanism.
red solution. Addition of a strong base will suppress the Choice of indicator
ionisation of methyl orange by common ion effect hence 1. For strong acid and strong base - methyl orange,
solution will be yellow in colour. The indicator is not suitable phenolphthalein and Litmus.
for titrating weak acid like CH3COOH against strong base.
2. For weak acid and strong base - phenolphthalein.
The H+ ions furnished by weak acid are not sufficient to
shift the equilibrium and red colour does not appear just at 3. For strong acid and weak base - methyl orange.
the end point. Excess of weak acid is required. 4. For weak acid and weak base - Phenol red.
(2) Quinonoid theory : Some important Acid Base Indicators
In the unionised form the indicator is generally in the Indicator pH range Colour in Colour in
benzenoid form which is less intense in colour and in the
acidic sol. alkaline sol.
ionised form it is in the Quinonoid form which is more intense
in colour. Cresol red (acid) 1.2 - 1.8 Red Yellow
Phenolphthalein : Thymol blue (acid) 1.2 - 2.8 Red Yellow
Methyl Orange 3.1 - 4.5 Red Yellow
H–O OH Methyl red 4.2 - 6.3 Red Yellow
Phenol red 6.4 - 8.2 Yellow Red
C
Cresol red (base) 7.0 - 8.1 Yellow Red
Thymol blue (base) 8.1 - 9.6 Yellow Red
O Phenolphthalein 8.1 - 9.8 Colourless Pink
C Alizarin yellow 10.1 - 12.1 Yellow Lilac (violet)
:O: SALTS :
Benzenoid form (colourless) The compounds containing positive and negative ions are known
as salts. Their solutions may be acidic, basic or neutral. They are
– classified as
:O O
(i) Simple salts : Formed by neutralisation of an acid and a
base. They may be further classified as follows.
(a) Normal salts : Salts not containing replaceable
C +H+ hydrogen or a hydroxyl group e.g. Na2SO4, KNO3,
Al2 (PO4)3, CaCl2.
O
(b) Acidic salts : Salts containing replaceable hydrogen

C– O: atoms NaHSO4, Na2HPO4, NaHCO3.
(c) Basic salts : Salts containing replaceable hydroxyl
:O: O
groups Zn(OH)Cl, Mg(OH)Cl, Fe(OH)2Cl.
oid form (colourless) Quinonoid form (Pink)
Ionic Equilibrium 215
(ii) Double Salts : Formed by the combination of two simple SOLVENTS :
salts The substances which provide medium to carry out chemical
FeSO4.(NH4)2SO4.6H2O, K2SO4. Al2(SO4)3. 24H2O. reactions. They are classified as
(iii) Complex Salts : Salts containing complex ion e.g. (I) Protophilic : Such solvents have a tendency to accept
K4[Fe(CN)6], [Cu(NH3)4]SO4. proton e.g. H2O, liquour ammonia and alcohol etc.
(iv) Mixed Salts : Salts containing more than one type of cation (II) Protogenic : Such solvents have a tendency to donate
OCl proton e.g. HCl (l), glacial CH3COOH and H2O etc.
or anion e.g. Ca ; NaKSO4. (III) Amphiprotic : Solvents capable of donating and accepting
Cl a proton e.g. H2O, alcohol etc.
All salts are strong electrolytes. (IV) Aprotic : Solvents which neither donate nor accept a proton
e.g. benzene, CS2, CCl4 etc.

1. Nicotinic acid (Ka = 1.4 × 10–5) has formula HNiC. Calculate 4. A solution contains 0.1 M H2S and 0.3 M HCl. Calculate the
its percent dissociation in a solution which contains 0.10 concentration of S– – and SH– ions in solution. Given K a1 and
mole of nicotinic acid per 2.0 litre of solution.
Ka 2 for H2S are 10–7 and 1.3 × 10–13 respectively..
Sol. HNiC + H2O H3O+ + NiC–
Ka
Molar concentration of nicotinic acid = 0.05 mole/lit. 1
Sol. (I) H2S H++ HS–
% Degree of dissociation
Ka
2
-5 (II) HS– H+ + S– –
Ka 1.4 ´ 10
a= = ´100 = 1.67.
C 0.05 [H + ][HS - ] [H + ][S -- ]
K a1 = Ka2 =
2. Enthalpy of neutralisation of CH3 COOH by NaOH [H 2S] [SH - ]
is –55.9 kJ/mol. DH for the ionisation of CH3COOH is
Sol. 57.1 – 55.9 = 1.2 KJ/mol [H + ]2 [S- - ]
K a1 ´ K a 2 =
3. Calculate the amount of heat released when [H 2S]
(I) 100 ml of 0.2 M HCl soution is mixed with 200 ml of H2S is weak electrolyte in presence of HCl its ionisation is
0.2 M KOH suppressed due to common ion effect.
(II) 200 ml of 0.2 M H2SO4 is mixed with equal volume of 0.2 K a1 ´ K a 2 [H 2S]
M KOH [S - - ] =
[H + ]2
100 ´ 0.2
Sol. (I) Moles of H+ ions = = 0.02 1 .0 ´ 10 - 7 ´ 1 .3 ´ 10 -13 ´ 0 .1
1000 = = 1 .44 ´ 10 - 20 M
( 0 . 3) 2
200 ´ 0.2 Putting [S– –] in equation (II)
Moles of OH– ion = = 0.04
1000
[H + ][S - - ] 0.3 ´ 1.44 ´ 10 -20
H2O formed is = 0.02 Moles [SH–] = = = 3.3 ´ 10 -8 M
Ka2 1.3 ´ 10 -13
Hence heat released = 57.1 × 0.02 = 1.142 kJ
5. The pH of 0.1 M hydrocyanic acid solution is 5.2. What is
200 ´ 0.2 the value of Ka for hydrocyanic acid.
(II) Moles of H+ ions = = 0.04
1000 1
Sol. log = pH = 5.2
200 ´ 0.2 [H + ]
and Moles of OH– = = 0.04
1000 [H+] = 6.2 × 10–6 M, [H+] = ca = C ´ Ka
Moles of H2O formed is = 0.04
(6.3 ´10 -6 ) 2
Heat released = 57.1 × 0.04 = 2.18 kJ =K a ; \ K a = 3.96 ´10 -10 M
0.1
216 Chemistry
6. Calculate the pH of Sol. The minimum concentration of S– – ion required to precipitate
(I) 1.0 × 10–8 M HCl (II) 1.0 × 10–8 M NaOH MS is
Sol. K sp 6.0 ´10 -21
(I) [H+] conc. due to water and HCl = [1 × 10–7 + 1 × 10–8] (S- - ) = = = 1.2 ´ 10-19 M
[M + + ] 0.05
[H+] = 10–8 (10 + 1) = 10–8 × 11
–log [H+] = 8 log 10 – log 11 ionisation of H2S; H2S H+ + HS–
pH = 8 – 1.04 = 6.96. [H + ][SH - ]
(II) Total [OH–] = [1 × 10–7 + 1 × 10–8] K1 = 10–7, K1 =
[H 2S]
Solving as in case (I) pOH = 6.96. \ pH = 7.04.
7. What is the hydrogen ion concentration of 0.1 N CH3COOH HS– H+ + S– –; K2 = 1.3 × 10–13
solution. The ionisation constant of CH3COOH is 1.8 × 10–5.
What is the pH of solution. [H + ][S -- ]
K2 =
[SH - ]
+
Sol. [ H ] = C ´K a = 0.1´ 1.8 ´ 10 -5 -3
= 1.34 ´ 10 M On multiplying,
pH = – log [H+] = – log 1.34 × 10–3 = 2.8729. [H + ]2 [S- - ]
8. What volume of 0.10 M sodium formate solution should be K1 × K2 =
[H 2S]
added to 50 ml of 0.05 M formic acid to produce a buffer
solution of pH 4.0, pKa of formic acid is 3.80. [H+ ]2 ´1.2 ´10-19
10-7 ´1.3 ´10-13 =
Salt 0.1
Sol. pH = pKa + log +
Solving for [H ] = 1.04 × 10 –1
Acid
pH = – log [H+] = – log (1.04 × 10–1) = 0.98.
Let the volume added be V ml. then 11. The pH of 0.05 M aqueous solution of diethylamine is 12.
V ´ 0.10 Calculate its Kb.
4 = 3.8 + log Sol. pH + pOH = 14
0.05 ´ 50
pOH = 14 – 12 = 2; [OH–] = 10–2
Solving for V we get V = 39.5 ml. (C2H5)2 NH + H2O (C2H5)2 NH2+ + OH–
9. Freshly precipitated aluminium and magnesium hydroxides 0.05 – 0.01 0.01 0.01
are stirred vigrously in a buffer solution containing 0.05 mol
0.01´ 0.01
L–1 of NH4OH and 0.25 mol L–1 of NH4Cl. Calculate the Kb = = 2.5 ´ 10 -3
conc. of aluminium and magnesium ions in solution. Ksp of 0.04
Al(OH)3 = 6 × 10–32, Ksp of Mg(OH)2 = 6 × 10–10, Kb of NH4OH 12. Ka for butyric acid is 2 × 10–5. Calculate the pH and hydroxyl
= 1.8 × 10–5. ion concentration of 0.2 M aqs. solution of sodium butyrate.
Sol. Sodium butyrate is a salt of weak acid and
K b ´ [BASE] strong base
Sol. The [OH - ] =
[SALT]
Kw ´Ka 1´10 -14 ´ 2 ´10 -5
[H + ] = = = 10 -9
1 .8 ´ 10 -5 ´ 0.05 C 0.2
= = 0. 36 ´ 10 - 5 mol L-1
0. 25 Hence pH = 9
10–14 = [H+] [OH–]
K sp of Al(OH) 3 10–14 = 10–9[OH–]
[Al 3+ ] = [OH–] = 10–5
[OH - ]3
13. Calculate the pH of an aqueous solution of 1.0 M ammonium
formate assuming complete dissociation pK a for formic acid
6 ´ 10-32
= = 1.29 ´10 -10 mol L-1
(0.36 ´10 -5 )3 = 3.8 and pK b of ammonia = 4.8.
Sol. Ammonium formate is a salt of weak acid and weak base.
2+ K sp of Mg (OH) 2 6 ´10 -10 -1
[Mg ]= = = 46.3 mol L 1 1
[OH - ]2 (0.36 ´ 10 -5 ) 2 Hence pH = 7 + pKa – pKb
2 2
10. An aqueous solution of metal bromide MBr2 (0.05 M) is
1 1
saturated with H2S what is the minimum pH at which MS =7+ × 3.8 – × 4.8 = 6.5
will precipitate? Ksp for MS = 6.0 × 10–21. Conc. of saturated 2 2
H2S = 0.1 M. K1 = 10–7 and K2 = 1.3 × 10–13 for H2S.
Ionic Equilibrium 217
14. A certain weak acid has Ka = 1.0 × 10–4. Calculate the
1 1
equilibrium constant for its reaction with strong base. =7+ ( – log Ka) + log C
2 2
ˆˆ† H + + A - K a = 1 ´ 10-4
Sol. HA ‡ˆˆ
1 1
HA + BOH BA + H2O =7+ ( – log 1.8 × 10 - 5 ) + log 0.1 = 8.87
Weak Strong 2 2

HA + OH– A– + H2O -4
18. The Ka for formic acid and acetic acid are 2.1 × 10 and 1.1

[A - ] [A - ][H + ] K 10 -4 × 10 - 5 respectively. Calculate the relative strength of acids.


K= = = a = = 1010
[HA][OH] [HA][OH - ][H + ] K w 10 -14
K a1 C1
15. Calculate for 0.01 N solution of sodium acetate. Sol. Relative strengths of weak acids =
Ka 2 C2
(a) Hydrolysis constant (b) Degree of Hydrolysis
(c) pH. Given Ka = 1.9 × 10–5. Assuming C1 = C 2
Sol. Sodium acetate is salt of weak acid and strong base
K a1 2.1´10 - 4
K 10-14 Relative strength = = = 4.37 : 1
(a) K h = w = = 5.26 ´ 10 -10 Ka2 1.1´ 10 - 5
K a 1.9 ´10 -5
Formic acid is 4.37 times more acidic than acetic acid
(b) Degree of hydrolysis 19. Calcium lactate is a salt of weak organic acid and is
represented by Ca(Lac)2. A saturated solution of Ca(Lac)2
Kw 10 -14 contains 0.13 mole of this salt in 0.50 litre solution. The pOH
= = = 2.29 ´10 - 4
C ´ Ka 0.1´ 1.9 ´10 -5 of this solution is 5.60. Assuming the complete dissociation
of the salt. Calculate Ka of lactic acid.
Sol. For salt of weak acid and strong base
1 1 1 1
(c) pH = 7 + p K a + log C = 7 - log K a + log C
1
2 2 2 2 pH = (pKw + pKa + log a)
2
1 1
= 7- log 1.9 ´ 10 - 5 + log 0.01 = 8.36 where a is concentration of lactate ion i.e.
2 2
2 × 0.26) mol L-1
16. Ka for ascorbic acid (HAsc) is 5 × 10–5. Calculate the
hydrogen ion concentration and percentage of hydrolysis 1
(14 – 5.6) = (14 – logKa + log 0.52)
in aqueous solution in which the concentration of Asc - 2
ions is 0.02 M. log Ka = – 3.084 = 4 .916
Sol. Degree of hydrolysis
on taking antilog Ka = 8.24 × 10 - 4
20. Calculate the hydrolysis constant for NH4Cl, pH value and
Kw 1´10 -14
a= = = 10 -4 = 0.01% [OH–] in 0.1 M NH 4Cl solution.
C ´ Ka 0.02 ´ 5 ´ 10 -5
K NH4OH = 1.75 ´10 -5
1 1 1 1
pH = 7 - log K a + log C = 7 - log 5 ´ 10 -5 + log 0.02 = 8.3
2 2 2 2 Kw = 1 × 10 - 14
[H+] = 5 × 10–9. Sol. Salt hydrolysis constant
17. When 0.2 M acetic acid is neutralised with 0.2 M NaOH in Kw
0.5 litre of water the resulting solution is slightly alkaline. Kh = = 5.7 × 10 - 10
Kb
Calculate the pH of the resulting solution.
Hydrolysis of NH 4 Cl is represented as
Ka for acetic acid is = 1.8 × 10 - 5
NH 4 + + H 2 O NH 4 OH + H+
Sol. CH 3COOH + NaOH CH 3 COONa + H 2 O
-1
1
The concentration of [ CH 3 COONa] will be 0.1 mol L . It pH = (pKw – pKb – log C)
2
is salt of weak acid and strong base
1
= (14 + log Kb – log C)
1 1 2
pH = 7 + pK a + log C
2 2
218 Chemistry

1 0.02 0.1 0.1 Mole initially


= (14 + log 1.75 × 10 - 5 – log 0.1) = 5.12 0 0.1 – 0.02 0.1+0.02 mole after reaction
2
0 0.08 0.12
log [ H + ] = – pH; Hence
[ H + ] = 7.55 × 10 - 6 M 0.08 0.12
-14
[ NH 4 OH] = ; [ NH 4 Cl] =
Kw 1´ 10 1 1
[ OH – ] = +
= = 1.32 × 10 - 9 M
[H ] 7.55 ´10 - 6 0.12
pOH = – log 1.8 × 10 - 5 + log = 4.9208
21. Calculate the amount of ammonium chloride required to 0.08
dissolve in 500 ml water to have pH = 4.5 pH = 14 – 4.9208 = 9.6792
(Kb for NH 4 OH = 1.8 × 10 -5
) Change in pH = 9.2553 – 9.0792
= 0.1761
Sol. pH = 4.5;
(b) Addition of 0.02 moles of NaOH
log [H + ] = – 4.5; NaOH + NH 4 Cl ¾¾® NaCl + NH 4 OH
+ -5 0.02 0.1 0 0.1
[H ] = 3.162 × 10 M
(Before reaction)
Let the concentration of NH 4 Cl be C 0 0.1 – 0.02 0.02 0.1 + 0.02
(After reaction)
NH 4 + + H 2 O NH 4 OH + H + 0.08 0.12
C (1 – a ) Ca Ca [0.08]
pOH = –log 1.8 × 10–5 + log
[0.12]
Kw 10 -14
Kh = Ca 2 = = -5
= 5.5 × 10 - 10 pOH = 4.5686;
Kb 1.8 ´ 10 pH = 14 – 4.5686 = 9.4314
Change in pH = 9.4314 – 9.2553
Kh Kh 5.5 ´ 10 -10 = 0.1761
a= = = = 1.739 × 10 - 5
Ca [H + ] 3.162 ´ 10 - 5 23. The pH of bloodstream is maintained by a proper balance of
H 2CO 3 and NaHCO 3 concentrations. What volume of 5
[H + ] 3.162 ´ 10- 5
C= = -5
= 1.8 mol L- 1 M NaHCO 3 solution should be mixed with 10 ml of sample
a 1.739 ´ 10
of blood which is 2 M in H 2CO 3 in order to maintain a pH
500 ml of water contains = 0.9 mole
of 7.4? Ka for H 2CO 3 in blood is 7.8 × 10 - 7
The weight of NH 4 Cl = 0.9 × 53.5 = 48.15 g
Sol. [ H 2CO 3 ] in blood = 2.0 M
22. Calculate the change in pH of one litre buffer solution
containing 0.1 mole each of NH 3 and NH 4 Cl upon addition [ NaHCO 3 ] = V ml
of Total volume on mixing (10 + V) ml
(a) 0.02 mole of dissolved HCl gas [Salt]
pH = – log Ka + log
(b) 0.02 mole of dissolved NaOH [Acid]
Assume no change in volume K NH3 = 1.8 × 10 - 5 5V /(V + 10)
= – log 7.8 × 10 - 7 + log
Sol. Use Henderson’s equation 20 /(V + 10)

[Salt] \ V = 78.36 ml
pOH = – log Kb + log 24. Zn salt is mixed with (NH4)2S of molarity = 0.021 M, what
[Base]
amount of Zn 2+ will remain unprecipitated in 12ml of the
solution . Ksp of ZnS = 4.51 × 10–24
0 .1
= – log 1.8 × 10 - 5 + log Sol. (NH4)2S = 0.021 M ; [S– –] = 0.021 M
0 .1
--
pOH = 4.7447 At equilibrium [ Zn + + ] [S ] = Ksp of ZnS
pH = 14 – 4.7447 = 9.2553
4.51´10 - 24
(a) Addition of 0.02 mole of HCl [ Zn + + ] = = 2.15 × 10 - 22 M
0.021
HCl + NH 4 OH ¾
¾® NH 4 Cl+ H 2 O
Ionic Equilibrium 219

[ Zn 2+ ] left in solution = 2.15 × 10 - 22 × 65 g/litre 26. Calculate pH of a saturated solution of Mg (OH ) 2 . Ksp for

2.15 ´ 10 - 22 ´ 65 ´ 12 Mg (OH ) 2 is 8.9 × 10 - 12


= g/12ml -
1000 Sol. Mg (OH ) 2 Mg 2 + + 2 OH
= 1.677 × 10 - 22 g/12 ml S 2S

25. Will a precipitate of Mg (OH ) 2 be formed in a 0.001 M Let the solubility of Mg (OH ) 2 be S mole lit -1 ;

solution of Mg ( NO 3 ) 2 if the pH of the solution is adjusted Ksp = 4S3


- 12
to 9? Ksp of Mg (OH ) 2 = 8.9 × 10 - 12 = 4S3 = 8.9 × 10 ;

Sol. pH = 9; \ [H + ] = 10 - 9 M S = 1.305 × 10 - 4 M
-4
or [OH - ] = 10 - 5 M [OH - ] = 2S = 2 × 1.305 × 10
pOH = 3.5832
Product of ionic conc. = [ Mg 2 + ] [OH - ]2 pH = 10.4168
= [0.001] [10- 5 ]2 = 10 - 13

Which is less, than Ksp of Mg (OH ) 2 i.e. 8.9 × 10 - 12 . Hence


Mg (OH ) 2 will not precipitate.
220 Chemistry

Very Short/Short Answer Questions 15. (i) How much of 0.3 M ammonium hydroxide should be
1. The equilibrium constants for HCN and HAc are 4.0 ×10–10 mixed with 30 mL of 0.2 M solution of ammonium
and 1.9 ×10–5 mole/L respectively. Which acid is stronger? chloride to give buffer solution of pH 8.65 and 10?
[pKb = 4.75]
2. A certain metal sulphide MS, has a solubility product (ii) How much volume of 0.1 M HAc should be added to
1.3 ×10–18 mole2/l2 . What must be [S – 2] be in a 0.10M 50 mL of 0.2 M NaAc solution if we want to prepare a
buffer solution of pH 4.91. (pKa = 4.76)
solution of M2+ when MS just starts to precipitate.
3. pKa values of acids A, B, C, D are 1.5, 3.5, 2.0 and 5.0. Which
Multiple Choice Questions
of them is strongest acid ? 16. Which of the following can act both as Bronsted acid and
4. Which of the following is strongest Lewis acid ? CCl4, AlCl3, Bronsted base?
NCl3, OCl2. (a) Na2CO3 (b) OH–
5. Which of the following is strongest conjugate base? CH3– (c) HCO3– (d) NH3
, NH2–, OH–, F–. 17. Which equilibrium can be described as an
6. What happens to ionic product of water if temperature is acid-base reaction using the Lewis acid-base definition but
increased? not using the Bronsted-Lowry definition?
7. All Bronsted acids are not Lewis acids. Explain. (a) 2NH3 + H2SO4 2NH4+ + SO42–
8. Calculate the molar concentration of an acetic acid solution (b) NH3 + CH3COOH
which is 2% ionized? ( Ka = 1.8 × 10–5)
NH4+ + CH3COO–
9. Which has the greater molarity in water, AgCl or Mg(OH)2
(c) H2O + CH3COOH
(Ksp (AgCl) = 1.8 ´ 10 -10 ,
(KspMg(OH)2 = 1.2 × 10–11) H3O+ + CH3COO–
10. Calculate the pH of the solution produced when an aqueous (d) [Cu(H2O)4]2– + 4 NH3
solution of pH = 5 is mixed with equal volume of an aqueous [Cu(NH3)4]2+ + 4H2O
solution of pH = 3.
18. The ionisation constant of an acid, Ka, is the measure of
11. A sparingly soluble salt gets precipitated only when the strength of an acid. The K a values of acetic acid,
product of concentration of its ions in the solution (Qsp)
hypochlorous acid and formic acid are 1.74 × 10–5, 3.0 × 10–8
becomes greater than its solubility product. If the solubility
and 1.8 × 10–4 respectively. Which of the following orders
of BaSO4 in water is 8 × 10–4 mol dm–3. Calculate its
of pH of 0.1 mol dm–3 solutions of these acids is correct?
solubility in 0.01 mol dm–3 of H2SO4.
(a) acetic acid > hypochlorous acid > formic acid
Long Answer Questions (b) hypochlorous acid > acetic acid > formic acid
12. (i) A sample of orange juice was found to have a pH of 10– (c) formic acid > hypochlorous acid > acetic acid
3. 80. What were the H+ and OH– concentrations in the (d) formic acid > acetic acid > hypochlorous acid
juice?
19. K a1 , Ka2 and K a3 are the respective ionisation constants
(ii) What is the pH of a 5.0×10–4 M solution of NaOH at
25°C? for the following reactions.
13. (i) What is the pH of a solution that contains 0. 10 M HCl
H 2S ƒ H + + HS-
and 0.10 M CH3COOH? For acetic acid, Ka = 1.8×10–5.
(ii) What are the concentrations of all the species present HS- ƒ H + + S2 -
in a 0. 50 M solution of acetic acid. For CH3COOH Ka =
1.8 × 10–5. H 2S ƒ 2H + + S2 -
14. Carbonic acid, H2CO3, is a weak diprotic acid formed by the
The correct relationship between K a1 , Ka2 and Ka3 is
reaction of carbon dioxide with water, For this acid,
Ka1 = 4.3 × 10–7and K a = 5.6 × 10–11. What are the (a) K a3 = K a1 ´ K a2 (b) K a3 = K a1 + K a2
2
equilibrium concentrations of each species in a 0. 10 M K a3 = K a1 - K a2 (d) K a3 = K a1 / K a2
(c)
solution of carbonic acid?
Ionic Equilibrium 221
20. The pKa of a weak acid, HA, is 4.80. The pKb of a weak base, 23. Which of the following will produce a buffer solution when
BOH, is 4.78. The pH of an aqueous solution of the mixed in equal volumes?
corresponding salt, BA, will be (a) 0.1 mol dm–3 NH4OH and 0.1 mol dm–3 HCl
(a) 9.58 (b) 4.79 (b) 0.05 mol dm–3 NH4OH and 0.1 mol dm–3 HCl
(c) 9.22 (d) 7.01
(c) 0.1 mol dm–3 NH4OH and 0.05 mol dm–3 HCl
21. What will be the value of pH of 0.01 mol dm –3 CH3COOH
(d) 0.1 mol dm–3 CH4COONa and 0.1 mol dm–3 NaOH
(K )
= 1.74 ´ 10-5 ?
a 24. A solution which is 10–3 M each in Mn 2+, Fe2+, Zn 2+ and
(a) 3. 4 (b) 3. 6 Hg2+ is treated with 10–6 M sulphide ion. If Ksp of MnS,
(c) 3. 9 (d) 3. 0 FeS, ZnS and HgS are 10 –15, 10 –23, 10 –20 and 10 –54
22. The pH of a solution which is 0.1 M in HA and 0.5 M in NaA. respectively which one will precipitate first
Ka for HA is 1.8 × 10–6 (a) FeS (b) MgS
(a) 5.44 (b) 6.44
(c) HgS (d) ZnS
(c) 6.0 (d) 4.73

1. Which will not affect the degree of ionization? 9. Which of the following is not a Lewis base?
(a) Temperature (b) Concentration (a) CH4 (b) C2H5OH
(c) Type of solvent (d) Current (c) Acetone (d) Sec amine
2. At infinite dilution, the percentage ionisation for both strong 10. Which of the following is not a Lewis acid?
and weak electrolytes is (a) BF3 (b) AlCl3
(a) 1% (b) 20% (c) FeCl3 (d) PH3
(c) 50% (d) 100% 11. In the given anions, the strongest Bronsted base is
3. A 0.2 molar solution of formic acid is 3.2% ionized. Its (a) ClO– (b) ClO2–
ionisation constant is (c) ClO3– (d) ClO4–
(a) 9.6 × 10–3 (b) 2.1 × 10–4 12. Which of the following can act as both Bronsted acid and
(c) 1.25 × 10–6 (d) 4.8 × 10–5 Bronsted base?
4. The hydrogen ion concentration of 0.2 N CH3COOH which (a) Na2CO3 (b) OH–
(c) HCO3 – (d) NH3
is 40% dissociated is
(a) 0.08 N (b) 0.12 N 13. Which of the following is the strongest Lewis acid?
(c) 0.80 N (d) 1.2 N (a) BI3 (b) BBr3
5. Degree of dissociation of 0.1 N CH3COOH is (Dissociation (c) BCl3 (d) BF3
constant = 1 × 10–5) 14. The strongest conjugate base is
(a) 10–5 (b) 10–4 (a) NO3– (b) Cl–
(c) SO4 2– (d) CH3COO–
(c) 10–3 (d) 10–2
15. A base, as defined by Bronsted theory, is a substance which
6. A monoprotic acid in a 0.1 M solution ionizes to 0.001%. Its
can
ionisation constant is
(a) lose a pair of electrons
(a) 1.0 × 10–3 (b) 1.0 × 10–6
(b) donate protons
(c) 1.0 × 10–8 (d) 1.0 × 10–11
(c) gain a pair of electrons
7. The concentration of water molecules in one litre of water at
(d) accept protons
298 K is
16. Which of the following is strongest Lewis base?
(a) 10–7 M (b) 55.5 M
(a) CH3– (b) NH2–
(c) 5.55 M (d) 7.26 M –
(c) OH (d) F–
8. Which of the following will occur if 0.1 M solution of a weak 17. Aluminium chloride is
acid is diluted to 0.01 M at constant temperature
(a) Bronsted Lowry acid (b) Arrhenius acid
(a) [H+] will decrease to 0.01 M (c) Lewis acid (d) Lewis base
(b) pH will decrese 18. BF3 is an acid according to
(c) Percentage ionization will increase (a) Arrhenius concept (b) Bronsted-Lowry concept
(d) Ka will increase (c) Lewis Concept (d) Both (b) and (c)
222 Chemistry
19. Which of the following is not a Lewis acid? 29. A compound having the formula NH2CH2COOH may behave
(a) CO (b) SiCl4 (a) only as an acid
(c) SO3 (d) Zn 2+ (b) only as a base
20. Ammonium ion is (c) both as an acid and base
(a) a conjugate acid (d) Neither acid nor base
(b) a conjugate base 30. Water is a
(c) both an acid and a base (a) protophobic solvent (b) protophilic solvent
(d) neither an acid nor a base (c) amphiprotic solvent (d) aprotic solvent
21. Among the following, the weakest base is 31. To Ag2CrO4 solution over its own precipitate, CrO42– ions
(a) H– (b) CH3– are added. This results in
(c) CH3O – (d) Cl– (a) increase in Ag+ concentration
In the equation I 2 + I - ¾ (b) decrease in Ag+ concentration
22. ¾® I 3- , which is the Lewis base?
(c) increase in solubility product
(a) I2 (b) I–

(d) shifting of Ag+ ions from the precipitate into the solution
(c) I3 (d) None of these
32. To suppress the dissociation of acetic acid, the compound
23. Which one of the following compounds is not a protonic
to be added to it is
acid
(a) sodium oxalate (b) sodium acetate
(a) PO(OH)3 (b) SO(OH)2
(c) sodium carbonate (d) sodium nitrate
(c) SO2(OH)2 (d) B(OH)3
24. Which equilibrium can be described as an acid-base reaction 33. Addition of which chemical will decrease the hydrogen ion
using the Lewis acid-base definition but not using the concentration of an acetic acid solution
Bronsted-Lowry definition? (a) NH4Cl (b) Al2(SO4)3
(c) AgNO3 (d) NaCN
(a) 2NH3 + H2SO4 2NH4+ + SO42–
34. Why only As+3 gets precipitated as As2S3 and not Zn+2 as
(b) NH3 + CH3COOH ZnS when H2S is passed throgh an acidic solution containing
NH4+ + CH3COO– As+3 and Zn+2?
(c) H2O + CH3COOH (a) Solubility product of As2S3 is less than that of ZnS
H3O+ + CH3COO– (b) Enough As+3 are present in acidic medium
(c) Zinc salt does not ionise in acidic medium
(d) [Cu(H2O)4]2– + 4 NH3
(d) Solubility product changes in presence of an acid
[Cu(NH3)4]2+ + 4H2O 35. The precipitation occurs if ionic concentration is
25. Which one of the following is the strongest acid? (a) less than solubility product
(a) ClO3OH (b) ClO2 (OH) (b) more than solubility product
(c) SO(OH)2 (d) SO2(OH)2 (c) equal to solubility product
26. Why are strong acids generally used as standard solutions
(d) None of these
in acid-base titrations?
36. The solubility of AgCl will be minimum in
(a) The pH at the equivalent point will always be 7
(a) 0.001 M AgNO3 (b) pure water
(b) They can be used to titrate both strong and weak bases
(c) Strong acids form more stable solutions than weak acids (c) 0.01 M CaCl2 (d) 0.01 M NaCl
(d) The salts of strong acid do not hydrolyse 37. The solubility of PbCl2 is
27. At a certain temperature the dissociation constants (a) K sp (b) (K sp )1 / 3
of formic acid and acetic acid are 1.8 × 10 –4 and (c) (K sp / 4)1 / 3 (d) (8K sp )1/ 2
1.8 × 10–6 respectively. The concentration of acetic acid
solution in which the hydrogen ion has the same 38. The solubility of calcium phosphate in water is
concentration as in 0.001 M formic acid solution is equal to x mol L–1 at 25°C. Its solubility product is equal to
(a) 0.001 M (b) 0.01 M (a) 108 x2 (b) 36 x3
(c) 36 x5 (d) 108 x5
(c) 0.1 M (d) 0.0001 M
28. What is the decreasing order of basic strengths of 39. Which of the following sulphides has the lowest solubility
OH–, NH2–, H–C º C– and CH3 – CH2– product?
(a) FeS (b) MnS
(a) CH 3 - CH -2 > NH -2 > H - C º C - > OH -
(c) PbS (d) ZnS
(b) H - C º C - > CH 3 CH -2 > NH -2 > OH - 40. The K sp of CuS, Ag 2 S and HgS are 10 –31 ,
10–44 and 10 –54 respectively. The solubility of these
(c) OH - > NH -2 > H - C º C - > CH 3 - CH -2 sulphides are in the order
(a) Ag2S > CuS > HgS (b) AgS > HgS > CuS
(d) NH -2 > H - C º C - > OH - > CH 3 - CH -2
(c) HgS > Ag2S > CuS (d) CuS > Ag2S > HgS
Ionic Equilibrium 223
41. Which of the following on reaction with H2S does not 53. What is the minimum concentration of SO42– required to
produce metallic sulphide? precipitate BaSO 4 in a solution containing
(a) CdCl2 (b) ZnCl2 1.0 × 10–4 mole of Ba2+ ? Ksp for BaSO4 = 4 × 10–10
(c) COCl2 (d) CuCl2 (a) 4 × 10–10 M (b) 2 × 10–7 M
42. What is the correct representation for the solubility product (c) 4 × 10–6 M (d) 2 × 10–3 M
of SnS2? 54. Solubility of an MX2 type electrolyte is 0.5 × 10–4 mole/lit,
(a) [Sn 2+] [S2–]2 (b) [Sn4+] [S2–]2 then Ksp of the electrolyte is
2+
(c) [Sn ] [2S ] 2– (d) [Sn4+] [2 S2–]2 (a) 5 × 10–12 (b) 25 × 10–10
43. How do we differentiate between Fe3+ and Cr3+ in group III? (c) 1 × 10–13 (d) 5 × 10–13
(a) By taking excess of NH4OH 55. The pH of blood does not appreciably change by a small
(b) By increasing NH4+ ion concentration addition of acid or a base because blood
(c) By decreasing OH– ion concentration (a) contains serum protein which acts as buffer
(d) Both (b) and (c) (b) contains iron as a part of the molecule
44. The solubility product of barium sulphate is 1.5 × 10–9 at (c) can be easily coagulated
18°C. Its solubility in water at 18°C is (d) is body fluid
(a) 1.5 × 10–9 mol L–1 (b) 1.5 × 10–5 mol L–1 56. The pH of solutions A, B, C, D are respectively 9.5, 2.5, 3.5,
5.5. The most acidic solution is
(c) 3.9 × 10–9 mol L–1 (d) 3.9 × 10–5 mol L–1
(a) A (b) B
45. The solubility product of AgCl is 4.0 × 10–10 at 298 K. The
solubility of AgCl in 0.04 M CaCl2 will be (c) C (d) D
57. pH of 10 M solution of HCl is
(a) 2.0 × 10–5 M (b) 1.0 × 10–4 M
–9 (a) 1 (b) 0
(c) 5.0 × 10 M (d) 2.2 × 10–4 M
(c) 2 (d) less than 0
46. The solubility of AgCl at 20°C is 1.435 × 10–3 gm/lit. The
58. It is found that 0.1 M solution of four sodium salts NaA,
solubility product of AgCl is
NaB, NaC and NaD have the following pH values
(a) 1.0 × 10–10 (b) 2 × 10–10
–5
7.0, 9.0, 10.0 and 11.0 respectively.
(c) 1.035 × 10 (d) 108 × 10–3
(a) NaD (b) NaC
47. The solubility product of Ag2CrO4 is 32 ´ 10 -12 . What is (c) NaB (d) NaA
Which one of the corresponding acids is strongest ?
the concentration of CrO 4 -2 ions in that solution (in g 59. A white salt is readily soluble in water and gives a colourless
ions L–1) solution with a pH of about 9. The salt would be
(a) 2 × 10–4 (b) 8 × 10–4 (a) NH4NO3 (b) CH3COONa
(c) 8 × 10–8 (d) 16 × 10–4 (c) CH3COONH4 (d) CaCO3
48. The solubility product of silver chloride is 1.8 × 10–10 at 298 60. The value of ionic product of water at 393 K is
K. The solubility of AgCl in 0.01 M HCl solution in mol/dm3 (a) less than 1 × 10–14 (b) greater than 1 × 10–14
is (c) equal to 1 × 10–14 (d) equal to 1 × 10–7
(a) 2.4 × 10–9 (b) 3.6 × 10–8 61. Which has the highest pH?
(c) 0.9 × 10 –10 (d) 1.8 × 10–8 (a) CH3COOK (b) Na2CO3
49. –5
Ksp for HgSO4 is 6.4 × 10 , then solubility of the salt is (c) NH4Cl (d) NaNO3
(a) 8 × 10–6 (b) 8 × 10–3 62. A solution of MgCl2 in water has pH
(c) 4.6 × 10 –5 (d) None of these (a) < 7 (b) > 7
50. The maximum amount of BaSO4 precipitated on mixing BaCl2 (c) 7 (d) 14.2
(0.5 M) with H2SO4 (1M) will correspond to 63. A solution of an acid has pH = 4.70. Find out the number of
(a) 0.5 M (b) 1.0 M OH– ions (pKw = 14)
(c) 1.5 M (d) 2.0 M (a) 5 × 10–10 (b) 4 × 10–10
(c) 2 × 10 –5 (d) 9 × 10–4
51. At 20°C, the Ag+ ion concentration in a saturated solution
of Ag2CrO4 is 1.5 × 10–4 mole/litre. At 20°C, the solubility 64. The pH of a solution whose [H+] is 3.0 × 10–4 M is
product of Ag2CrO4 will be (a) 4.45 (b) 3.75
(a) 3.3750 × 10–12 (b) 1.6875 × 10–10 (c) 4.36 (d) 3.523
(c) 1.6875 × 10–12 (d) 1.6875 × 10–11 65. pH of 4.0 gm/litre NaOH solution is
52. The solubility of BaSO4 in water is 2.33 × 10–3 g L–1. (a) 13 (b) 11
Its solubility product will be (molecular weight of (c) 13.5 (d) 12
BaSO4 = 233) 66. Highest pH (14) is given by
(a) 1 × 10–5 (b) 1 × 10–10 (a) 0.1 M H2SO4 (b) 0.1 M NaOH
(c) 1 × 10 –15 (d) 1 × 10–20 (c) 1 N NaOH (d) 1 N HCl
224 Chemistry
67. The pH of 0.0001 M NaOH is 80. Which of the following solution cannot act as a buffer?
(a) 4 (b) 10 (a) NaH2PO4 + H3PO4
(c) 12 (d) 11 (b) CH3COOH + CH3COONa
68. The pOH value of a solution whose hydroxide ion (c) HCl + NH4Cl
concentration is 6.2 × 10–9 mol/litre is (d) H3PO4 + NaH2PO4
(a) 8.21 (b) 6.21 81. A buffer solution of pH 9 can be prepared by mixing
(c) 7.75 (d) 7.21 (a) CH3COONa and CH3COOH
69. The pH of a solution obtained by mixing 50 ml of 0.4 N HCl (b) NaCl and NaOH
and 50 ml of 0.2 N NaOH is
(c) NH4Cl and NH4OH
(a) – log 2 (b) – log 0.2
(d) KH2PO4 and K2HPO4
(c) 1.0 (d) 2.0
70. The pH of 1.0 M aqueous solution of a weak acid HA is 6.0. 82. Which of the following is the buffer solution of strong acidic
Its dissociation constant is nature?
(a) 1.0 × 10–12 (b) 1.0 × 10–6 (a) HCOOH + HCOO–
(c) 1.0 (d) 6.0 (b) CH 3COOH + CH 3COO -
71. The pH of a solution is increased from 3 to 6; its H+ ion (c) H2C2O4 + C2O42–
concentration will be (d) H3BO3 + BO33–
(a) reduced to half (b) doubled 83. One litre of a buffer solution containing 0.01 M NH4Cl and
(c) reduced by 1000 times (d) increased by 1000 times 0.1 M NH4OH having pKb of 5 has pH of
72. When CO2 dissolves in water, the following equilibrium is (a) 9 (b) 10
established CO2 + 2H2O H3O+ + HCO3– (c) 4 (d) 6
for which the equilibrium constant is 3.8 × 10–7 and pH = 6.0. 84. A certain buffer solution contains equal concentration of
The ratio of [HCO3–] to [CO2] would be X– and HX. The Ka for HX is 10–8. The pH of the buffer is
(a) 3.8 × 10–13 (b) 3.8 × 10–1 (a) 3 (b) 8
(c) 6.0 (d) 13.4 (c) 11 (d) 14
73. An example of a salt that will not hydrolyse is 85. In a mixture of a weak acid and its salt, the ratio of the
(a) NH4Cl (b) KCl concentration of acid to salt is increased tenfold. The pH of
(c) CH3COONH4 (d) CH3COOK the solution
74. pH of 2 M NaCl will be (a) decreases by one (b) decreases by one tenth
(a) 3 (b) 6.5
(c) increases by one (d) increases by ten-fold
(c) 7 (d) 10
86. How much sodium acetate should be added to 0.1 M solution
75. Dissociation constant of NH4OH is 1.8 × 10–5. The
of CH3COOH to give a solution of pH 5.5
hydrolysis constant of NH4Cl would be
(pKa of CH3COOH = 4.5)
(a) 1.80 × 10–19 (b) 5.55 × 10–10
(c) 5.55 × 10 –5 (d) 1.80 × 10–5 (a) 0.1 M (b) 0.2 M
76. What is the percentage hydrolysis of NaCN in N/80 solution (c) 1.0 M (d) 10.0 M
when the dissociation constant for HCN is 1.3 × 10–9 and 87. For preparing a buffer solution of pH 6 by mixing sodium
Kw = 1.0 × 10–14 acetate and acetic acid, the ratio of the concentration of salt
(a) 2.48 (b) 5.26 and acid should be (Ka = 10–5)
(c) 8.2 (d) 9.6 (a) 1 : 10 (b) 10 : 1
77. The pH of a 1 M CH3COONa solution in water will be nearly (c) 100 : 1 (d) 1 : 100
(a) 2.4 (b) 5.4 88. What is [H+] of a solution that is 0.1 M HCN and 0.2 M
(c) 7.4 (d) 9.4 NaCN? (Ka for HCN = 6.2 × 10–10)
78. A physician wishes to prepare a buffer solution at (a) 3.1 × 1010 (b) 6.2 × 105
pH = 3.58 that efficiently resists a change in pH yet contains (c) 6.2 × 10 –10 (d) 3.1 × 10–10
only small conc. of the buffering agents. Which one of the 89. The pH of a buffer containing equal molar concentrations
following weak acid together with its sodium salt would be of a weak base and its chloride
best to use?
(Kb for weak base = 2 × 10–5, log 2 = 0.3) is
(a) m-chloro benzoic acid (pKa = 3.98)
(a) 5 (b) 9
(b) p-chloro cinnamic acid (pKa = 4.41)
(c) 4.7 (d) 9.3
(c) 2,5-dihydroxy benzoic acid (pKa = 2.97)
(d) Acetoacetic acid (pKa = 3.58) 90. How many ml of 1 M H2SO4 is required to neutralise 10 ml of
79. Which one is Buffer solution? 1 M NaOH solution?
(a) [PO4–3] [HPO42–] (b) [PO43–] [H2PO4–] (a) 2.5 (b) 5.0
(c) [HPO42–] [H2PO4–] (d) All of these (c) 10.0 (d) 20.0
Ionic Equilibrium 225
91. Molar heat of neutralization of NaOH with HCl in comparison (a) 1.0 × 10–12 (b) 1.0 × 10–10
to that of KOH with HNO3 is (c) 1.0 × 10 –8 (d) 1.0 × 10–16
(a) less (b) more 97. When rain is accompanied by a thunderstorm, the collected
(c) equal (d) depends on pressure rain water will have a pH value
92. The mutual heat of neutralization of 40 g NaOH and 60 g
(a) slightly lower than that of rain water without
CH3COOH will be
thunderstorm
(a) 57.4 kJ (b) Less than 57.4 kJ
(b) slightly higher than that when the thunderstorm is not
(c) More than 57.4 kJ (d) 13.7 kJ
there
93. Heat of neutralization of NH4OH and HCl is
(c) uninfluenced by occurrence of thunderstorm
(a) equal to 13.7 kcal (b) more than 13.7 kcal
(d) depends on the amount of dust in air
(c) less than 13.7 kcal (d) more than one is correct
98. H3BO3 is
94. The pH indicators are
(a) salts of strong acids and strong bases (a) monobasic and weak Lewis acid
(b) salts of weak acids and weak bases (b) monobasic and weak Bronsted acid
(c) either weak acids or weak bases (c) monobasic and strong Lewis acid
(d) either strong acids or strong bases (d) tribasic and weak Bronsted acid
95. Identify the indicator used to titrate Na2CO3 solution with 99. A solution which is 10–3 M each in Mn 2+, Fe2+, Zn 2+ and
HCl Hg2+ is treated with 10–6 M sulphide ion. If K sp of MnS,
(a) Phenolphthalein (b) dil. H2SO4 FeS, ZnS and HgS are 10–15, 10 –23, 10 –20 and 10–54
(c) Methyl orange (d) None of these respectively which one will precipitate first
96. The solubility product of AgI at 25°C is 1.0 × 10–16 mol2 L–2. (a) FeS (b) MgS
The solubility of AgI in 10–4N solution of KI at 25°C is (c) HgS (d) ZnS
approximately (in mol L–1)

1. A weak acid, HA, has a Ka of 1.0 × 10–5. If 0.1 mole of this 6. The dissociation constants for acetic acid and HCN at 25°C
acid dissolved in one litre of water, the percentage of acid are 1.5 × 10–5 and 4.5 × 10–10 respectively. The equilibrium
dissociated at equilbrium is closest to [CBSE-PMT 2007] constant for the equilibrium [CBSE-PMT 2009]
(a) 1.0% (b) 99.9% (c) 0.1% (d) 99.0%
CN– + CH3COOH ƒ HCN + CH3COO– would be:
2. Calculate the pOH of a solution at 25°C that contains
1× 10– 10 M of hydronium ions, i.e. H3O+. [CBSE-PMT 2007] (a) 3.0 × 10– 5 (b) 3.0 × 10– 4
(a) 4.0 (b) 9.0 (c) 1.0 (d) 7.0 (c) 3.0 × 104 (d) 3.0 × 105
3. Equal volumes of three acid solutions of pH 3, 4 and 5 are 7. Which of the following molecules acts as a Lewis acid ?
mixed in a vessel. What will be the H+ ion concentration in [CBSE-PMT 2009]
the mixture ? [CBSE-PMT 2008]
–4 (a) (CH3)2 O (b) (CH3)3 P (c) (CH3)3 N (d) (CH3)3 B
(a) 1.11 × 10 M (b) 3.7 × 10–4 M

(c) 3.7 × 10 M 3 (d) 1.11× 10–3 M 8. The ionization constant of ammonium hydroxide is 1.77 × 10–5
4. If the concentration of OH– ions in the reaction at 298 K. Hydrolysis constant of ammonium chloride is:

ˆˆ† Fe3+ (aq) + 3OH– (aq) is [CBSE-PMT 2009]


Fe(OH)3(s) ‡ˆˆ
(a) 6.50 × 10– 12 (b) 5.65 × 10–13
1
decreased by times, then equilibrium concentration of Fe3+ (c) 5.65 × 10–12 (d) 5.65 × 10–10
4
will increase by : [CBSE-PMT 2008]
9. If pH of a saturated solution of Ba (OH)2 is 12, the value of
(a) 8 times (b) 16 times (c) 64 times (d) 4 times
5. Equimolar solutions of the following were prepared in water its K(sp) is : [CBSE-PMT 2010]
separately. Which one of the solutions will record the highest
pH ? [CBSE-PMT 2008] (a) 4.00 × 10 -6 M 3 (b) 4.00 × 10–7 M3
(a) SrCl2 (b) BaCl2
(c) MgCl2 (d) CaCl2 (c) 5.00 × 10 -6 M 3 (d) 5.00 × 10 -7 M 3
226 Chemistry
10. What is [H + ] in mol/L of a solution that is 0.20 M in 20. 1 M NaCl and 1 M HCl are present in an aqueous solution.
CH3COONa and 0.10 M in CH3COOH ? Ka for CH3COOH The solution is [AIEEE 2002]
(a) not a buffer solution with pH < 7
= 1.8 × 10-5 . [CBSE-PMT 2010]
(b) not a buffer solution with pH > 7
(a) 3.5 × 10 -4 (b) 1.1 × 10 -5 (c) a buffer solution with pH < 7
(d) a buffer solution with pH > 7.
(c) 1.8 × 10 -5 (d) 9.0 × 10 -6
21. Species acting as both Bronsted acid and base is
11. In a buffer solution containing equal concentration of B–
(a) (HSO4)–1 (b) Na2CO3
and HB, the Kb for B– is 10–10. The pH of buffer solution is :
(c) NH3 (d) OH–1.
[CBSE-PMT 2010]
22. Let the solubility of an aqueous solution of Mg(OH)2
(a) 10 (b) 7 (c) 6 (d) 4 be x then its Ksp is [AIEEE 2002]
12. Which one of the following molecular hydrides acts as a Lewis (a) 4x3 (b) 108x5
acid? [CBSE-PMT 2010] (c) 27x4 (d) 9x.
(a) NH3 (b) H2O (c) B2H6 (d) CH4 23. The solubility of Mg(OH) 2 is S moles/litre.
13. A buffer solution is prepared in which the concentration of The solubility product under the same condition
NH3 is 0.30M and the concentration of NH4+ is 0.20 M. If the is [AIEEE 2002]
equilibrium constant, Kb for NH3 equals 1.8 × 10–5, what is (a) 4S 3 (b) 3S 4
the pH of this solution ? (log 2.7 = 0.433). [CBSE-PMT 2011] (c) 4S2 (d) S 3
(a) 9.08 (b) 9.43 (c) 11.72 (d) 8.73 24. Which one of the following statements is not
14. Which of the following is least likely to behave as Lewis true? [AIEEE 2003]
base ? [CBSE-PMT 2011] (a) pH + pOH = 14 for all aqueous solutions
(a) H2O (b) NH3 (c) BF3 (d) OH– (b) The pH of 1 × 10–8 M HCl is 8
15. In qualitative analysis, the metals of Group I can be separated (c) 96,500 coulombs of electricity when passed through a
from other ions by precipitating them as chloride salts. A CuSO4 solution deposits 1 gram equivalent of copper at
solution initially contains Ag+ and Pb2+ at a concentration of the cathode
0.10 M. Aqueous HCl is added to this solution until the Cl–
concentration is 0.10 M. What will the concentrations of Ag+ (d) The conjugate base of H 2 PO -4 is HPO 24-
and Pb2+ be at equilibrium? [CBSE-PMT 2011M] 25. The solubility in water of a sparingly soluble salt AB2 is 1.0
(Ksp for AgCl = 1.8 × 10 , Ksp for PbCl2 = 1.7 × 10–5)
–10 × 10 –5 mol L –1 . Its solubility product number
(a) [Ag+] = 1.8 × 10–7 M ; [Pb2+] = 1.7 × 10–6 M will be [AIEEE 2003]
(a) 4 × 10 –10 (b) 1 × 10 –15
(b) [Ag+] = 1.8 × 10–11 M ; [Pb2+] = 8.5 × 10–5 M
(c) [Ag+] = 1.8 × 10–9 M ; [Pb2+] = 1.7 × 10–3 M (c) 1 × 10–10 (d) 4 × 10–15
(d) [Ag+] = 1.8 × 10–11 M ; [Pb2+] = 8.5 × 10–4 M 26. The conjugate base of H 2 PO -4 is [AIEEE 2004]
16. pH of a saturated solution of Ba(OH)2 is 12. The value of (a) H3PO4 (b) P2O5
solubility product (Ksp)of Ba(OH)2 is : [CBSE-PMT 2012 S]
(a) 3.3 × 10– 7 (b) 5.0 × 10–7 (c) 4.0 × 10–6 (d) 5.0 × 10–6 (c) PO 34- (d) HPO 24-
17. Buffer solutions have constant acidity and alkalinity because 27. The molar solubility (in mol L –1 ) of a sparingly
[CBSE-PMT 2012S] soluble salt MX4 is ‘S’. The corresponding solubility product
(a) these give unionised acid or base on reaction with added is Ksp. ‘S’ is given in term of Ksp by the relation :
acid or alkali. [AIEEE 2004]
(b) acids and alkalies in these solutions are shielded from (a) S = (256Ksp )
1/ 5
(b) S = (128Ksp )
1/ 4
attack by other ions.
(c) they have large excess of H+ or OH– ions (c) S = (Ksp /128)
1/ 4
(d) S = (Ksp / 256)
1/ 5

(d) they have fixed value of pH


18. Identify the correct order of solubility in aqueous medium: 28. The solubility product of a salt having general formula MX 2 ,
[NEET 2013]
in water is : 4 × 10 -12 . The concentration of M 2+ ions in the
(a) ZnS > Na2S > CuS (b) Na2S > CuS > ZnS aqueous solution of the salt is [AIEEE 2005]
(c) Na2S > ZnS > CuS (d) CuS > ZnS > Na2S
19. Which of these is least likely to act as Lewis base? (a) 4.0 ´ 10 -10 M (b) 1.6 ´ 10 -4 M
(a) F– (b) BF3 [NEET 2013] (c) 1.0 ´ 10 -4 M (d) 2.0 ´ 10- 6 M
(c) PF3 (d) CO
Ionic Equilibrium 227
29. Hydrogen ion concentration in mol/L in a solution of 37. Four species are listed below: [AIEEE 2008]
pH = 5.4 will be : [ AIEEE 2005] +
i. HCO3– ii. H3 O iii. HSO4 – iv. HSO3F
-6 -6
(a) 3.98 ´ 10 (b) 3.68 ´ 10 Which one of the following is the correct sequence of their
acid strength?
(c) 3.88 ´ 106 (d) 3.98 ´ 108 (a) iv < ii < iii < i (b) ii < iii < i < iv
(c) i < iii < ii < iv (d) iii < i < iv < ii
30. What is the conjugate base of OH - ? [AIEEE 2005] 38. The pKa of a weak acid, HA, is 4.80. The pKb of a weak base,
BOH, is 4.78. The pH of an aqueous solution of the
(a) O 2 - (b) O -
correspondng salt, BA, will be [AIEEE 2008]
(c) H 2 O (d) O 2 (a) 9.58 (b) 4.79 (c) 7.01 (d) 9.22
39. Solid Ba(NO3)2 is gradually dissolved in a 1.0 × 10– 4 M
31. Which of the following statements is true? [AIEEE 2006] Na2CO3 solution. At what concentration of Ba2+ will a
(a) HClO4 is a weaker acid than HClO3 precipitate begin to form? (Ksp for BaCO3 = 5.1 × 10–9)
(b) HNO3 is a stronger acid than HNO2 [AIEEE 2009]
(a) 5.1 × 10 –5 M (b) 8.1 × 10 –8 M
(c) H3PO3 is a stronger acid than H2SO3 (c) 8.1 × 10 –7 M (d) 4.1 × 10 –5 M
(d) In aqueous medium HF is a stronger acid than HCl 40. Three reactions involving H2PO4– are given below :
32. Given the data at 25ºC [AIEEE 2006] (i) H3PO4 + H2O® H3O+ + H2PO4–
- -
(ii) H2PO4– + H2O ® HPO42– + H3O+
Ag + I ¾
¾® AgI + e E º = 0.152 V (iii) H2PO4– + OH– ® H3PO4 + O2–
¾® Ag + + e -
Ag ¾ E º = -0.800 V In which of the above does H 2 PO4- act as an acid ?
What is the value of log Ksp for AgI? (2.303 RT/ F = 0.059 V) [AIEEE 2010]
(a) (ii) only (b) (i) and (ii) (c) (iii) only (d) (i) only
(a) –37.83 (b) –16.13
41. In aqueous solution the ionization constants for carbonic
(c) –8.12 (d) +8.612 acid are
33. The first and second dissociation constants of an acid H2 A K1 = 4.2 × 10–7 and K2 = 4.8 × 10–11.
are 1.0 × 10–5 and 5.0 × 10–10 respectively. The overall Select the correct statement for a saturated 0.034 M solution
dissociation constant of the acid will be [AIEEE 2007] of the carbonic acid. [AIEEE 2010]
(a) 0.2 × 105 (b) 5.0 × 10–5 2 -
(a) The concentration of CO3 is 0.034 M.
(c) 5.0 × 1015 (d) 5.0 × 10–15. (b) The concentration of CO32- is greater than that of
34. The pKa of a weak acid (HA) is 4.5. The pOH of an aqueous HCO3- .
buffer solution of HA in which 50% of the acid is ionized is
(c) The concentrations of H+ and HCO3- are approximately
[AIEEE 2007]
(a) 7.0 (b) 4.5 (c) 2.5 (d) 9.5 equal.
35. In a saturated solution of the sparingly soluble strong (d) The concentration of H+ is double that of CO32 - .
electrolyte AgIO3 (molecular mass = 283) the equilibrium 42. Solubility product of silver bromide is 5.0 × 10–13. The
which sets is AgIO3(s) ‡ˆˆˆˆ† Ag + ( aq ) + IO3- ( aq ) . If the quantity of potassium bromide (molar mass taken as 120 g
solubility product constant K sp of AgIO 3 at a given mol–1) to be added to 1 litre of 0.05 M solution of silver nitrate
temperature is 1.0 × 10–8, what is the mass of AgIO3 contained to start the precipitation of AgBr is [AIEEE 2010]
in 100 ml of its saturated solution? [AIEEE 2007] (a) 1.2 × 10–10 g (b) 1.2 × 10–9 g

(a) 1.0 × 10 g 4 (b) 28.3 × 10 g–2 (c) 6.2 × 10–5 g (d) 5.0 × 10–8 g
(c) 2.83 × 10 g–3 (d) 1.0 × 10–7 g. 43. At 25°C, the solubility product of Mg(OH)2 is 1.0 × 10–11. At
which pH, will Mg2+ ions start precipitating in the form of
36. For the following three reactions a, b and c, equilibrium
Mg(OH)2 from a solution of 0.001 M Mg2+ ions?
constants are given: [AIEEE 2008]
[AIEEE 2010]
(i) CO( g ) + H 2O( g ) ƒ CO2 ( g ) + H 2 ( g ); K1 (a) 9 (b) 10 (c) 11 (d) 8
44. An acid HA ionises as
(ii) CH 4 ( g ) + H 2O( g ) ƒ CO( g ) + 3H 2 ( g );K 2
ˆˆ† H + + A -
HA ‡ˆˆ
(iii) CH 4 ( g ) + 2H 2O( g ) ƒ CO2 ( g ) + 4H 2 ( g );K 3 The pH of 1.0 M solution is 5. Its dissociation constant would
be : [AIEEE 2011RS]
(a) K1 K 2 = K3 (b) K 2 K3 = K1
(a) 5 (b) 5 ´ 10 -8
(c) K3 = K1 K2 (d) K3 .K 23 = K12
(c) 1 ´ 10-5 (d) 1 ´ 10-10
228 Chemistry
45. The Ksp for Cr(OH)3 is 1.6 × 10–30. The solubility of this (a) 3 × 10–1 (b) 1 × 10–3 (c) 1 × 10–5 (d) 1 × 10–7
compound in water is : [AIEEE 2011RS] 49. How many litres of water must be added to 1 litre an aque-
(a) 4 -30 (b) 4
1.6 ´ 10 -30 ous solution of HCl with a pH of 1 to create an aqueous
1.6 ´ 10 / 27
solution with pH of 2 ? [JEE Main 2013]
(c) 1.6 ´ 10-30 / 27 (d) 1.6 ´ 10 -30 (a) 0.1 L (b) 0.9 L
46. A vessel at 1000 K contains CO2 with a pressure of 0.5 atm. (c) 2.0 L (d) 9.0 L
Some of the CO2 is converted into CO on the addition of
50. 2.5 ml of (2/5) M weak monoacidic base (Kb = 1 × 10–12 at 25°)
graphite. If the total pressure at equilibrium is 0.8 atm, the
is titrated with (2/15) M HCl in water at 25°C. The
value of K is : [AIEEE 2011]
concentration of H+ at equivalence point is (Kw = 1 × 10–14 at
(a) 1.8 atm (b) 3 atm (c) 0.3 atm (d) 0.18 atm
25°C) (IIT-JEE 2008S)
47. The equilibrium constant (K c ) for the reaction
N2(g) + O2(g) ® 2NO(g) at temperature T is 4 × 10–4. The (a) 3.7 × 10–14 M (b) 3.2 × 10–7 M
value of Kc for the reaction [AIEEE 2012] (c) 3.2 × 10–2 M (d) 2.7 × 10–2 M

1 1 51. Solubility product constant (Ksp) of salts of types MX, MX2


NO( g ) ® N 2 ( g ) + O 2 ( g ) at the same temperature is: and M3X at temperature T are 4.0 × 10–8, 3.2 × 10–14 and
2 2
2.7 × 10–15, respectively. Solubilities (mol dm–3) of the salts
(a) 0.02 (b) 2.5 × 102 (c) 4 × 10–4 (d) 50.0
at temperature 'T' are in the order – (IIT-JEE 2008S)
48. The pH of a 0.1 molar solution of the acid HQ is 3. The value
(a) MX > MX2 > M3X (b) M3X > MX2 > MX
of the ionization constant, Ka of the acid is :
[AIEEE 2012] (c) MX2 > M3X > MX (d) MX > M3X > MX2

1. How many gms of CaC 2O 4 will dissolve in litre of saturated 6. The solubility of CaF2 in 0.01 M solution of NaF. The Ksp of
solution. Ksp of CaF2 is 3.4 × 10–11
CaC2 O 4 is 2.5 ´10 -9 mol 2 lit -2 (a) 3.4 ´ 10-7 mol / l
(a) 0.0064 g (b) 0.0128 g
(b) 3.4 ´ 10-5 mol / l
(c) 0.0032 g (d) None of these
(c) 3.4 Mol / l
2. For NH3, Kb = 1.8 × 10–5 and Ka for NH +4 would be
(d) None of these
(a) 1.8 ´ 10 -5 (b) 5.56 ´ 10 5 7. If the reaction between CO2 and H2O is
+
(c) 1.8 ´ 1010 (d) 5.56 ´ 10 -10 CO 2+H 2O H 2CO 3 H+HCO 3.
3. The pH of a solution which is 0.1 M in HA and 0.5 M in NaA. If CO2 escapes from the system
-6 (a) pH will decrease
Ka for HA is 1.8 ´ 10
(b) H+ concentration will decrease
(a) 5.44 (b) 6.44
(c) H2CO3 concentration will be altered
(c) 6.0 (d) 4.73
4. The pH of a solution obtained by mixing of 100.0 ml of 0.1 M (d) The forward reaction is promoted
8. A solution contains 0.1 M H2 S and 0.3 M HCl, the
HCl and 100 ml of 0.2 M NH3, Kb for NH3 is 1.8 ´ 10 -5
- -7
(a) 4.74 (b) 9.26 concentration of SH ions is ( K a = 1. ´10 )
(c) Less than 7 (d) None of these
(a) 3.3 ´ 10 -8 M (b) 33 ´ 10 -8 M
5. The percentage hydrolysis of 0.15 M solution of ammonium
acetate, K a for CH 3COOH is 1.8 ´ 10 -5 and K b for (c) 3.3 ´ 10 -7 M (d) 10 -7 M
9. 20 ml of 0.2 M. NaOH are added to 50 ml of 0.2
NH 3 is 1.8 ´10 -5
M CH3COOH (K a = 1.8 ´10 -5 ) the pH of the solution is
(a) 0.556 (b) 4.72
(c) 9.38 (d) 5.56 (a) 4.56 (b) 4.73
(c) 9.45 (d) 6.78
Ionic Equilibrium 229
10. The pH at which the Mg(OH)2 begins to precipitate from a 18. An acid-base indicator has Ka = 3.0 ´ 10 -5 . The acid form of
solution containing 0.10 M Mg++ ions. The Ksp of Mg(OH)2 the indicator is red and the basic form is blue. The amount of
is 1 ´ 10 -11 change in [H+] required to change indicator from 75% red
(a) 9 (b) 5 the 75% blue is
-5
(c) 11 (d) 4 (a) 8 ´ 10 -5 M (b) 9 ´ 10 M
11. The 0.001M Solution of Mg (NO3)2 is adjusted to pH 9, Ksp -4
(c) 1 ´ 10 -5 M (d) 3 ´ 10 M
of Mg(OH)2 is 8.9 ´ 10 -12 . At this pH 19. Which of the following statements(s) is (are) correct?
(a) Mg(OH)2 will be precipitated (a) The pH of 1.0 × 10–8 M solution of HCl is 8
(b) Mg(OH)2 is not precipitated (b) The conjugate base of H2 PO -4 is HPO 24-
(c) Mg(OH)3 will be precipitated (c) Autoprotolysis constant of water decreases with
(d) Mg(OH)3 is not precipitated temperature
12. Calculate the pH of 0.5 M aqueous solution of NaCN, the (d) When a solution of a weak monoprotic acid is titrated
pKb of CN - is 4.70 against a strong base, at half-neutralisation point pH =
(1/2)pKa.
(a) 4.70 (b) 11.5
20. The pH of 0.1 M solution of the following salts increases in
(c) 7 (d) 6.5
the order.
13. The pH of aqueous solution of 1M HCOONH4, pKa of
(a) NaCl < NH4Cl < NaCN < HCl
HCOOH is 3.8 and pKb of NH3 is 4.8
(b) HCl < NH4Cl < NaCl < NaCN
(a) 6.5 (b) 4.8
(c) NaCN < NH4Cl < NaCl < HCl
(c) 3.8 (d) 8.6
(d) HCl < NaCl < NaCN < NH4Cl
14. The hydrolysis of Na2CO3 involves the reaction between
21. What will be the H+ ion concentration in a solution prepared
+ by mixing 50 mL of 0.20 M NaCl, 25 mL of 0.10 M NaOH and
(a) Na ions and water
25 mL of 0.30 N HCl?
(b) Na + and OH - ions (a) 0.5 M (b) 0.05 M
2- (c) 0.02 M (d) 0.10 M
(c) CO3 and H 2O
22. Determine the pH of the solution that results from the addition
(d) CO32 - and H + of 20.00 mL of 0.01 M Ca(OH)2 to 30.00 mL of 0.01 M HCl.
(a) 11.30 (b) 10.53
15. The solubility of BaF2 in a solution of Ba(NO3)2 will be (c) 2.70 (d) 8.35
represented by concentration term 23. Two weak solutions are isohydric when their
- (a) hydrogen-ion concentrations are the same before mixing
(a) [ Ba 2+ ] (b) éë F ùû
(b) hydrogen-ion concentrations are same before and after
_ mixing
1 é -ù
(c) F (d) 2[ NO 3 ] (c) degree of dissociation are the same
2ë û
(d) chemical properties are the same
16. The enthalpy of neutralization of HCl and HCN by NaOH are
24. The dissociation constant of monobasic acids A, B and C
–55.9 and –12.1 kJ mol–1 respectively, the enthalpy of are 10–4, 10–6 and 10–10 respectively. The concentration of
ionisation of HCN is each is 0.1 M. Which of the following has been arranged in
(a) - 43.8 kJ mol -1 order of increasing pH?
(a) A < B < C (b) C < A < B
(b) - 68.0 kJ mol -1 (c) B < C < A (d) B < A » C
25. Calculate the molar solubility of Fe(OH)2 at a pH of 8.00 [Ksp
(c) 43.8 kJ mol -1 of Fe (OH)2=1.6×10–14]
(a) 0.06 (b) 0.016
(d) 68.0 kJ mol -1
(c) 0.010 (d) 0.16
17. Ka for HCN is 5 ´ 10 -10 at 25°C. For maintaining at constant 26. K1 and K2 for oxalic acid are 6.5×10–2 and 6.1×10–5
pH of 9, the volume of 5M KCN solution required to be respectively. What will be [HO–] in a 0.01 M solution of
added to 10ml of 2M HCN solution is sodium oxalate ?
(a) 4 ml (b) 7.95 ml (a) 9.6×10–6 (b) 1.4×10–1
(c) 2 ml (d) 9.3 ml (c) 1.3×10–6 (d) 1.3×10–8
230 Chemistry

27. Calculate the pH of a solution containing 0.1 M HCO 3- and (a) 3.52 × 10 -3 (b) 6.75 × 10 -4

0.2 M CO 32- [K1(H2CO3) = 4.2×10–7×10 and K2( HCO 3- ) = (c) 5.38 × 10 -2 (d) 1.74 × 10 - 5
4.8×10–11]. 36. The degree of dissociation of 0.1M weak acid HA is 0.5%. If
2 ml of 1.0 M HA solution is diluted to 32 ml the degree of
(a) 3.18 (b) 10.62
+
(c) 6.62 (d) 9.31 dissociation of acid and H 3O ion concentration in the
28. At what pH will a 1×10–4 M solution of an indicator will (Kb resulting solution will be respectively
indicator) = 1×10–11 change colour? -4
(a) 7.0 (b) 3.0 (a) 0.02 and 3.125 × 10 (b) 1.25 × 10 - 3 and 0.02
(c) 5.5 (d) 11.0 (c) 6.02 and 1.25 × 10 - 3 (d) 0.02 and 8.0 × 10
- 12
29. Calculate the pH of a 0.01 M NaHCO3 solution [K1(H2CO3) = 37. The dissociation constants of a weak acid HA and weak
4×10–7, K2( HCO 3- ) = 4.8×10–11]. base BOH are 2 × 10 - 5 and 5 × 10 - 6 respectively. The
(a) 9.38 (b) 6.38 equilibrium constant for the neutralisation reaction of the
(c) 8.38 (d) 7.38 two is
30. Blood plasma is maintained at a pH of 7.4 largely by the
(a) 1.0 × 10 4
(b) 1.0 × 10 - 4
(a) HCO 3- / H 2 CO 3 buffer - 10
(c) 1.0 × 10 (d) 2.5 × 10 - 1
(b) HPO 24 - / PO 34- buffer 38. The ionisation constant of an acid-base indicator
(a weak acid) is 1.0 × 10 - 6 . The ionised form of the indicator
(c) HCO 3- / H 2 CO 3 is red whereas the unionised form is blue the pH change
required to alter the colour of the indicator from 80% blue to
and HPO 24 - / H 2 PO -4 buffer 80% red is
(d) HPO 24- / H 3 PO 4 and haemoglobin buffers (a) 1.40 (b) 1.20
(c) 0.80 (d) 2.00
31. In the titration of a weak diprotic acid (H2A) with a strong 39. The pH at the equivalence point in the titration of 25 ml of
base (NaOH), [H+] is given by 0.10 M formic acid with a 0.1 M NaOH solution (given that
(a) K a1 C a (b) Ka Ca1 pKa of formic acid = 3.74)
(a) 8.74 (b) 8.37
(c) K a Ca1 (d) K a1 (c) 4.74 (d) 6.06
32. The pH of a solution prepared by mixing 2.0 ml of a strong 40. In H S , I , RNH 2 , NH 3 the order of proton accepting
acid (HCl) solution of pH 3.0 and 3.0 ml of a strong base tendency will be
(NaOH) of pH 10.0
(a) 4.5 (b) 3.4 (a) H S > RNH 2 > NH 3 > I
(c) 2.5 (d) 6.5
(b) I > NH 3 > RNH 2 > H S
33. For a sparingly soluble salt ApBq, the relationship of its
solubility product Ls ® Ksp with its solubility (S) is (c) NH 3 > RNH 2 > H S > I

pq P+q (d) RNH 2 > NH 3 > H S > I


(a) Ls ® Ksp = S (pq)
41. The pH of a weak monoacid base at 80% neutralisation with
(b) Ls = Sp + q .p p q q a strong acid in a dilute solution is 7.40. The ionisation
p+q q p constant of the base is
(c) Ls ® Ksp = S .p q
pq p q
(a) 1.6 × 10 - 7 (b) 1.0 × 10 - 5
(d) Ls ® Ksp = S p q
-6
34. The correct order of increasing solubility of AgCl in (A) water, (c) 1.0 × 10 (d) 2.0 × 10 - 7
(B) 0.1 M NaCl, (C) 0.1 M, BaCl 2 , (D) 0.1 M NH 3 is 42. What will be the volume of 1 M NH 3 and 1 M HCl required
(a) D > A > B > C (b) D > C > B > A to prepare 300 ml of a buffer of pH = 9.26 (pKa = 9.26 for
(c) B > A > D > C (d) A > D > B > C
NH +4 )
35. If pKb for fluoride ion at 25°C is 10.83, the ionisation constant
of hydrofluoric acid in water at this temperature is (a) 225 ml, 75 ml (b) 200 ml, 100 ml
(c) 100 ml, 200 ml (d) 150 ml, 150 ml
Ionic Equilibrium 231
49. pH of a solution obtained on mixing 50 ml of 0.1M NaCN and
43. Ksp of M(OH) 2 is 3.2 × 10 -11 . The pH of saturated solution
50 ml of 0.2 M HCl will be (pKa for HCN = 9.40)
in water is
(a) 1.00 (b) 9.40
(a) 3.40 (b) 10.30
(c) 9.10 (d) 1.30
(c) 10.60 (d) 3.70
44. The concentration of hydroxyl ion in a solution left after 50. On addition of increasing amount of Ag NO 3 to 0.1 M each
mixing 100 ml of 0.1 M MgCl 2 and 100 ml of 0.2 M NaOH of NaCl and NaBr in a solution, what % of

-11 Br - ion get precipitated when Cl - ion starts precipitating. Ksp


(Ksp of Mg (OH ) 2 = 1.2 ´10 ) is
-4 -3
(AgCl) = 1 .0 ´ 10 - 10 , Ksp (AgBr) = 1 ´ 10 -13
(a) 2.8 ´ 10 (b) 2.8 × 10
(a) 0.11 (b) 99.9
(c) 2.8 ´ 10 -2 ( d) 2.8 ´ 10 -5 (c) 0.01 (d) 9.99
45. In the titration of monoacid base (weak) with a strong acid 51. A solution of NH 4 Cl and NH 3 has pH = 8.0. Which of the
the pH at half of the equivalence point.
following hydroxides may be precipitated when this solution
(a) 14 – pKb (b) = pKb is mixed with equal volume of 0.2 M of metal ion.
(c) 7 – pKb (d) 7 + pKb
46. The pKavalue for the A ® B, B ® C and C ® D (a) Ba(OH)2 (K sp = 1.1 ´ 10-4 )
dissociations are 2.09, 3.86 and 9.82 respectively. Since only
B has an equal number of positive and negative charges, the (b) Mg(OH)2 (K sp = 3.5 ´ 10-4 )
value of the isoelectric point is
(a) 5.26 (b) 2.98. (c) Fe(OH) 2 (K sp = 8.1´ 10-16 )
(c) 3.86 (d) 15.77
47. Mass loss of 1.0000 g of the AgCl (K sp = 1.0 ´10-10 ) on (d) Ca (OH) 2 (K sp = 2.1 ´ 10 -5 ).
repeated washing with 10 L of water is (Ag = 108, 52. In order to prepare a buffer of pH 8.26, the amount of
Cl = 35.5)
(NH4 )2 SO4 required to be mixed with 1L of 0.1 M
-2 -3
(a) 1.43 × 10 g (b) 1.43 ´10 g NH 3 (pK b = 4.74) is
(c) 1.0 ´ 10 -4 g (d) 1.34 × 10 -3 g (a) 5 mol (b) 0.5 mol
(c) 10.0 mol (d) 1.0 mol
48. 0.1 M acetic acid solution is titrated against 0.1 M NaOH
solution what would be the difference in pH between 1/4 53. Assuming that the buffer in the blood is CO2 - HCO3– .
and 3/4 stages of neutralisation of acid Calculate the ratio of conjugate base to acid necessary to
(a) 2 log 1/4 (b) 2 log 3/4 maintain blood at its proper pH of 7.4.
(c) log 1/3 (d) 2 log 3
K1 (H 2 CO 3 ) = 4.5 × 10 -7
(a) 11 (b) 8
(c) 6 (d) 14
232 Chemistry

EXERCISE 1 16. (a)


17. (c) AlCl3 electron deficient hence Lewis acid.
1. HAc is stronger
18. (c) Lewis concept.
2. [S–2] = 1.3× 10–17 M
19. (a) CO cannot accept electrons. Hence it is not Lewis acid.
3. Acid ‘A’ with pKa = 1.5 is strongest acid, lower the value of
pKa stronger will be the acid. 20. (a) NH 3 + H + NH +4 (Acid)
4. AlCl 3 is strongest Lewis acid because its octet is not 21. (d) HCl is strongest hence Cl– weakest.
complete. 22. (b) I– is electron donor. Hence it is Lewis base
5. CH 3- is strongest conjugate base. 23. (d) The acids are H3PO4 (orthophosphoric acid), H2SO3
6. It increases with increase in temperature because (Sulphurous acid) and H 2SO4 (Sulphuric acid) are
dissociation of water is endothermic process. protonic acid whereas Orthoboric acid is not protonic
8. 0.045 M acid.
9. 1.4 × 10–4 M, Mg(OH)2 has greater. 24. (d) [Cu(H2O)4]2+ + 4NH3 [Cu(NH3)4]2+ + 4H2O
10. pH = 3.301 involves lose and gain of electrons. H2O is coordinated
11. S = 6 × 10–4 mol dm–3 to Cu by donating electrons (LHS). It is then removed by
16. (c) 17. (d) 18. (d) 19. (a) 20. (d) withdrawing electrons.
21. (a) 22. (b) 23. (c) 24. (c) 25. (a) HClO4 (perchloric acid) is strongest.
EXERCISE 2 26. (b) Strong acid can ionise the weak base also.
1. (d) Current does not effect the degree of ionisation.
27. (b) [H+] = C ´ K a = 0.001 ´ 1.8 ´ 10 -4 for formic acid
2. (d) Ionisation is 100% at infinite dilution.
3. (b) K = Ca2 = 0.2 (0.032)2 = 2.1 × 10–4. [H+] = C2 ´ 1.8 ´10-5 for acetic acid Equating and
40 solving for C2 = 0.01 M.
4. (a) [H+] = Ca = 0.2 × 0.40 = 0.08 M. ( a = = 0.40 )
100 28. (a) Follow Lowry Bronsted concept for conjugate pair. Acid
5. (d) strength follows the order H2O > C2H2> NH3> C2H6
conjugate base will follow the order
6. (d) K a = Ca 2 = 0.1´ (1´ 10-5 )2 = 1´10-11
C 2 H 5- > NH 2 > C 2 H > OH
7. (b) 1 litre of H2O = 1000 ml = 1000 g;
..
(Q density of water 1 g/ml) 29. (c) H 2N.CH 2 COOH can donate and accept H+ ions.
1000 30. (c) H2O can donate and accept protons.
\ = 55.5 moles 31. (b) To keep Ksp constant addition of CrO42– will decrease
18
[Ag+].
8. (c) Ostwald's dilution law, dilution increases ionisation.
32. (b) Common ion effect. CH3COONa will suppress ionisation
9. (a) Methane cannot donate electrons. While others can of CH3COOH.
donate electrons. Lewis bases are electron doners.
33. (d) NaCN + H2O NaOH + HCN (hydrolysis)
10. (d) PH3 is Lewis base and not Lewis acid.
11. (a) The weaker the conjugate acid, the stronger is the base Strong NaOH will give OH– ions and react with H+ to form
and vice versa. HClO is a weak acid, hence ClO– is a H2O hence decrease the concentration of H+ ions.
strong base. 34. (a) Ksp of As2S3 is less than ZnS. In acid medium ionisation
of H2S is suppresed (common ion effect) and Ksp of ZnS
12. (c) H2CO 3 H+ + HCO 3– H+ + CO 3–. does not exceed.
HCO 3- can donate and accept H+. 35. (b) For precipitation the product of concentration of ions >
Ksp.
13. (a) Back bond formation decreases with increase in size of 36. (c) Due to common ion effect.
halogen atom. Hence B in BI3 is more electron deficient.
37. (c) PbCl2 Pb++ + 2Cl– , Ksp = (S) (2S)2
14. (d) CH3COOH is a weak acid, hence CH3 COO - is a 1
strong base. æ Ksp ö 3
15. (d) Base accepts protons and acid donates protons. \S = ç ÷ S = solubility
è 4 ø
Ionic Equilibrium 233
38. (d) Ca3(PO4)2 3Ca++ + 2PO4– – 53. (c) [Ba++] [SO4– –]
3S 2S Ksp=(1.0 × 10–4) [SO4– –] = 4 × 10–10
Ksp= (3S)3 (2S)2 = 108 S5 \ [SO 4- - ] = 4 ´10 -6
39. (c) Sulphides of IIA and IIB have low value of Ksp.
54. (d) MX2 is ternary salt. K sp = 4S3 = 4 (0.5 × 10 –4) 3
40. (a) For CuS solubility is (10–31)1/2; = 5 × 10–13
1 1 55. (a) Blood contains serum protein which acts as buffer.
æ K sp ö 3 æ 10-44 ö 3
( )
1 56. (b) Smaller the pH, the more the acidic character.
For Ag2S = ç ÷ =ç ÷ and for HgS = 10 -54 2
è 4 ø è 4 ø 57. (d) – log [H+] = pH; – log 10 = pH. Hence pH = –1 less than
0.
41. (c) COCl 2 is phosgene hence will not give any metal
58. (d) The pH values indicate that NaD, NaC and NaB are salts
sulphide.
of strong base and weak acid. pH of NaA = 7 it is salt of
42. (b) SnS2 Sn4+ + 2S– –
Ksp = [Sn 4+] [S– –]2 strong acid and strong base.
43. (d) When NH4Cl is added for III group NH4+ concentration 59. (b) Salt must be of strong base and weak acid as pH = 9,
increases and OH– concentration decreases (by common hence the salt CH3COONa.
ion effect). 60. (b) Kw increases with temperature.
44.(d) Solubility of 61. (b) The higher the pH more, the basic character, pH of Na2CO3
-9 1/ 2 -5 > CH3COOK.
BaSO4= K sp = (1.5 ´ 10 ) = 3.87 ´10
62. (a) MgCl2 gives acidic solution hence pH < 7.
45. (c) Solubility of AgCl
63. (a) – log [H+] = 4.70. \[H + ] = 2 ´10 -5 ,
K sp -10
4 ´ 10
[Ag+] = -
= = 5.0 ´ 10-9 M 10 -14
[Cl ] 0.08 [OH - ] = = 5 ´ 10 -10
-5
0.08 M is concentration of [Cl–] from CaCl2. 2 ´ 10
64. (d) – log [H+] = pH; –log 3 × 10–4 = pH; \ pH = 3.523.
1.435 ´ 10 -3
46. (a) Solubility of AgCl = = 10 -5 moles L-1 4
143.5 65. (a) Molar concentration of NaOH = 0. 1 M .
40
Ksp = s2 = (10–5)2 = 10–10
47. (a) For ternary electrolyte solubility \[OH- ] = 10-1; [H + ] = 10-13; pH = 13.
1 1 66. (c) 1 N NaOH = 1 M NaOH.
æ K sp ö 3 æ 32 ´ 10-12 ö 3 -4
=ç ÷ =ç ÷ = 2 ´ 10 \[OH - ] = 100 ; [H + ] = 10-14 ; pH = 14.
è 4 ø è 4 ø
67. (b) 0.0001 M NaOH.
48. (d) Solubility in AgCl is 0.01 M HCl
\[OH- ] = 10-4 ; [H + ] = 10-10 ; pH = 10.
K sp -10
1.8 ´ 10 68. (a) –log (OH) = pOH; – log 6.2 × 10–9 = pOH;
= -
= = 1.8 ´ 10-8
[Cl ] 0.01 \ pOH = 8.21
69. (c) mev. of HCl = 50 × 0.4 = 20, mev. of NaOH = 50 × 0.2 = 10
49. (b) Solubility = K sp = 6.4 ´ 10 -5 = 8 ´ 10 -3 M mev. of acid left = 10 or gev. of acid = 0.01, volume 100 ml
0.01´ 1000 + -1
50. (a) BaCl 2 + H 2SO 4 ¾
¾® BaSO 4 + 2HCl [HCl ] = = 0.1 ; \ [H ] = 10 ; pH = 1
0.5 0.5 0.5 100
(BaCl2 is limiting reagent) 70. (a) For weak acid [H+] = Ca.

¾® 2Ag + + CrO 24-


51. (c) Ag 2 CrO 4 ¾ 10 -6
\a = ; (pH = 6, \[H + ] = 10- 6 ) ;
1
1 1.5 ´10 -4
[CrO24- ] = [Ag + ] = = 0.75 ´10 - 4 Ka = Ca2 = 1 × (10–6)2 = 1 × 10–12
2 2 71. (c) pH = 3. \ [H+] = 10–3; pH = 6 \ [H+] = 10–6.
K sp = (1.5 ´ 10 -4 ) 2 (0.75 ´ 10 -4 ) = 1.68 ´10 -12 Hence [H+] reduced by 10–3 times.
72. (b) CO2 + 2H2O H3O+ + HCO3– ;
-3
2.33 ´10
52. (b) Solubility of BaSO4 = = 10 -5 ML-1 - -7
233 [H3 O + ][HCO3- ] [HCO 3 ] = 3.8 ´ 10 = 3.8 ´10 -1
Kc = ;
[CO 2 ] [CO 2 ] 10 -6
\ K sp = (10 -5 ) 2 = 10 -10
234 Chemistry
73. (b) Salt of strong acid and strong base give neutral solution
[SALT]
(pH = 7). Hence such salts are not hydrolysed. pH = 7 89. (d) pOH = –log Kb + log
[BASE]
74. (c) NaCl solution (salt of strong base and strong acid) hence
pH = 7.
æ SALT ö
= - log K b ç since = 1÷
Kw 10-14 è BASE ø
75. (b) Kh = = = 5.5 ´10-10 .
K b 1.8 ´10-5 = –log 2 × 10 = 4.7. \ pH = 9.3
–5

90. (b) N1V1 = N2V2 (1 MH2 SO4 = 2N H2SO4)


Kw 10-14 2 × V1 = 10 × 1 V1 = 5 ml
76. (a) a = = = 2.48%. 91. (c) (NaOH + HCl) (KOH + HNO3). Both constitute the pair of
Ka ´ c 1
1.3 ´10-9 ´ strong base and strong acid.
80
92. (b) 57.4 kJ mol–1 is heat of neutralisation for strong acid and
1 strong base. Since acetic acid is weak, the value will be
77. (d) pH = (14 + 4.75 - 0) = 9.4 Salt of weak acid and strong less than 57.4 kJ mol–1.
2
base. Remember for CH3COOH, pKa = 4.75 93. (c) Less than 13.7 kcal. NH4OH is a weak base. It requires
less energy for ionisation.
[SALT] 94. (c) Weak acids or weak bases e.g. phenolphthalein (weak
78. (d) pH = pk a + log . acid) or methyl orange. (weak base)
[ACID]
95. (c) Methyl orange is a weak base hence must be used with
[SALT] strong acid.
For maximum buffer capacity = 1.
[ACID]
K sp of AgI 1.0 ´10-16
79. (b) PO43– and H2PO4– is combination of acid and salt. 96. (a) Solubilityof AgI in KI soln. = =
[I- ] 10-4
80. (c) HCl + NH4Cl. HCl is strong acid hence not used in buffer.
81. (c) The pH of basic buffer is more than 7. NH4Cl and NH4OH = 1.0 ´10 -12 mol -1
combination provides basic buffer.
97. (a) Rain water is acidic due to dissolved oxides of nitrogen
82. (a) Formic acid is stronger than other acids given.
[SALT] (N 2 + O 2 ¾
¾® 2 NO and NO + O 2 ¾
¾® 2 NO 2 )
0.01
83. (b) pOH = pkb + log = 5 + log =4. formed by thunderstorm.
[BASE] 0.1
Hence pH less than 7.
\ pH = 10 (Q pH + pOH = 14) 98. (a) H3BO3 is not proton donor but behaves as a Lewis acid
by accepting a lone pair of electrons from OH– ions
[SALT]
84. (b) pH = –log Ka + log = – log 10–8 + 0 = 8. -
[ACID] OH é OH ù
| ê | ú
-
[ACID] 10[SALT] HO - B + OH ¾¾® ê HO - B ¬ OH ú
85. (a) pH = pKa – log = pKa – log | ê | ú
[SALT] [SALT] OH ë OH û
Q [Acid] = 10 [Salt] 99.(c) Since Ksp of HgS is minimum among others, HgS will
Hence pH will decrease by 1. precipitate first.
[SALT] EXERCISE 3
86. (c) pH = pKa + log
[ACID] 1. (a) Given Ka = 1.00×10–5, c = 0.100 mol
for a weak electrolyte,
[SALT ]
5.5 = 4.5 + log [SALT] = 1 M. degree of dissociation
0. 1
Ka 1 ´ 10 –5
-5 SALT SALT (a) = = = 10 -2 = 1%
87. (b) 6 = - log 10 + log = 5 + log c 0.100
ACID ACID
2. (a) Given [H3O+] = 1 × 10–10 M
SALT SALT 10 at 25º [H3O+] [OH–] = 10–14
log must be 1. \ = or 10 : 1 .
ACID ACID 1
10 -14
\ [OH - ] = = 10-4
+ [ACID] 0.1 10 -10
88. (d) [H ] = K a ´ = 6.2 ´ 10 -10 ´ = 3.1 ´10 -10
[SALT] 0. 2 pOH = – log [OH–] = – log [10 –4] = 4
\ pOH = 4
Ionic Equilibrium 235
3. (b) [H3O]+ for a solution having pH = 3 is given by 2+
9. ® Ba (aq) + 2 OH - (aq)
(d) Ba (OH) 2 (s) ¾ ¾
[H3O]+ = 1×10–3 moles/litre
[\ [H3O]+ = 10–pH] pH = 12 or pOH = 2
Similarly for solution having pH = 4, [OH - ] = 10 -2 M
[H3O]+ = 1 × 10–4 moles/ litre and for pH=5
® Ba 2 +
Ba(OH) 2 ¾¾ + 2 OH–
[H3O+] = 1×10–5 moles/ litre
Let the volume of each solution in mixture be IL, then 0.5× 10-2 10 -2
total volume of mixture solution L = (1 + 1 + 1) L =3L
Total [H 3 O] + ion present in mixture solution [\ Concentration of Ba 2 + is half of OH - ]
= (10–3 + 10–4 + 10–5) moles
K sp = [Ba 2+ ] [OH - ]2
Then [H3O]+ ion concentration of mixture solution
10 -3 + 10 -4 + 10-5 0.00111 = [0.5 × 10-2 ] [1 × 10-2 ]2
= M= M
3 3
= 0.00037 M = 3.7 ×10–4 M. = 0.5 × 10-6 = 5 × 10-7 M 3
4. (c) For this reaction Keq. is given by é Salt ù
10. (d) pH = p Ka+ log ê
éFe3+ ù éOH - ù3 ë Acid úû
ë ûë û
K=
[ Fe(OH)3 ] log é H + ù = log Ka – log é Salt ù
ë û êë Acid úû
= [Fe3+] [OH–]3 [\ [solid]=1].

+ é Acid ù
If [OH– ] is decreased by
1
times then for reaction log éë H ùû = log Ka + log ê
4 ë Salt úû
equilibrium constant to remain constant, we have to
increase the concentration of [Fe3+] by a factor of 43 i.e é H + ù = K é Acid ù
ë û aê
4× 4 × 4 = 64. Thus option (c) is correct answer. ë Salt úû
5. (b) The highest pH will be recorded by the most basic 0.1
solution. The basic nature of hydroxides of alkaline earth = 1.8 × 10 -5 ´ = 9 × 10-6 M
0.2
metals increase as we move from Mg to Ba and thus the
solution of BaCl2 in water will be most basic and so it 11. (d) Kb = 10-10 ; Ka = 10-4 or pKa = 4
will have highest pH.
For the buffer solution containing equal concentration
6. (c) Given, CH3COOH ƒ CH3COO– + H+ ; of B– and HB
Ka1 , = 1.5 × 10– 5 ....(i) pH = pKa + log 1

HCN ƒ H++ CN–; K a 2 = 4.5 × 10–10 pH = pKa = 4


12. (c) Boron in B2H6 is electron deficient
or H+ + CN– ƒ HCN; 13. (b) Given [NH3] = 0.3 M, [NH4+] = 0.2 M, Kb = 1.8 × 10–5 .
1 1
K'a 2 = = ...(ii) [salt]
Ka 2 4.5 ´ 10–10 pOH = pK b + log [pKb = –log Kb;
[base]
\ From (i) and (ii), we find that the equilibrium constant pKb = –log 1.8 × 10–5]
(Ka) for the reaction , \ pKb = 4.74
CN– + CH3COOH ƒ CH3 COO– + HCN, is
0.2
–5
= 4.74 + log = 4.74 + 0.3010 – 0.4771 = 4.56
1.5×10 1 0.3
K a = K a1 ´ K a' 2 = = ´ 105 = 3.33 ´ 10 4
4.5×10 –10 3 pH = 14 – 4.56 = 9.436
14. (c) BF3 behaves as lewis acid.
7. (d) (CH3)3 B - is an electron deficient, thus behave as a
15. (c) Ksp = [Ag+] [Cl–]
lewis acid.
1.8 × 10–10 = [Ag+] [0.1]
8. (d) Ammonium chloride is a salt of weak base and strong
acid. In this case hydrolysis constant K h can be [Ag+] = 1.8 × 10–9 M
calculated as Ksp = [Pb+2] [Cl–]2
1.7 × 10–5 = [Pb+2] [0.1]2
Kw 1 ´ 10-14
Kh = = = 5.65 ´ 1010 [Pb+2] = 1.7 × 10–3 M
K b 1.77 ´ 10 -5
236 Chemistry
16. (b) Given pH = 12 ++
or [H+] = 10–12 28. (c) MX 2 ƒ M + 2X -
1S 2S
Since, [H+] [OH–] = 10–14 Where S is the solubility of MX2
10 -14 then Ksp = 4S3; S× (2S)2 = 4×10–12 = 4S3; S = 1 × 10–4
\ [OH–] = = 10–2 \ S = 1[M++] = 1 × 10– 4
10-12
1
Ba(OH)2 Ba 2+ + 2OH 29. (a) pH = –log [H+] = log
s 2s [H + ]
[OH–] = 10–2
1
10-2 5.4 = log
2s = 10–2 or s = [H + ]
2
On solving [H+] = 3.98 × 10–6
æ 10 -2 ö 3 30. (a) Conjugate acid-base pair differ by only one proton.
Ksp = 4s3 = 4 ´ ç ÷ = 5 × 10–7 OH - ¾
¾® H + + O 2 - Conjugate base of OH– is O2–
è 2 ø
17. (a) Lets take an example of an acidic buffer CH3COOH and +5 +3
31. (b) The HNO3 is stronger than HNO 2 . The more the
CH3COONa.
CH3COOH CH3COO – + H + ; oxidation state of N, the more is the acid character.
CH3COONa CH3COO– + Na+
when few drops of HCl are added to this buffer, the H+ 32. (b) Given
of HCl immediatly combine with CH3COO– ions to (i) ¾® Ag + + e -
Ag ¾ E º = - 0.800 V
form undissociated acetic acid molecules. Thus there + -
® AgI + e - Eº = 0.152 V
(ii) Ag + I ¾¾
will be no appreciable change in its pH value. Like wise
if few drops of NaOH are added, the OH – ions will From the eqn (i) and (ii) we have,
combine with H+ ions to form unionised water molecule. ¾® Ag + + I - E º = -0.952 V
AgI ¾
Thus pH of solution will remain constant.
18. (c) Solubility of alkali metal is maximum among the 0.059
Eocell = log K
following. Among ZnS (1.7 × 10–5) & CuS (8 × 10–37) n
ZnS has higher value of Ksp.
19. (b) BF3 is Lewis acid(e– pair acceptor) 0.059
–0.952 = log [Ag+] [I–]
20. (a) A buffer is a solution of weak acid and its salt with 1
strong base and vice versa. HCl is strong acid and
0.952
NaCl is its salt with strong base. pH is less than 7 due - = log K sp
to HCl 0.059
21. (a) (HSO4)– can accept and donate a proton –16.13 = log Ksp
(HSO4)– + H+ ® H2SO4 (conjugate acid) 33. (d) ˆˆ† H + HA
H 2A ‡ˆˆ
(HSO4)– – H+ ® SO42–. (conjugate base)
22. (a) Mg(OH)2 ® [Mg2+] + 2[OH–] \ K = 1.0 × 10 –5 = [H + ][HA - ]
1 (Given)
x 2x [H 2 A]
ksp = [Mg] [OH] = [x][2x] = x.4x2 = 4x3.
2 2

HA- ¾¾
® H+ + A--
23. (a) Mg(OH)2 Mg ++ + 2OH- ;
S 2S
[H + ][A -- ] (Given)
Ksp = (s) (2s)2 = 4s3 \ K 2 = 5.0 ´ 10 -10 =
24. (b) An acidic solution cannot have a pH > 7. [HA - ]
25. (d) [A] = 1.0 × 10–5, [B] = [1.0 × 10–5] ,
ksp = [2.B]2 [A] = [2 × 10–5]2 [1.0 × 10–5] [H + ]2 [A 2 - ]
= 4 × 10–15 K= = K1 ´ K2
[H 2 A]
+
–H = (1.0 × 10–5) × (5 × 10–10) = 5 × 10–15
26. (d) H 2 PO–4 HPO2–
4
Acid con ugate base
conjugate é salt ù
34. (d) For acidic buffer pH = pKa + log ê ú
4+ - ë acid û
27. (d) MX4 M + 4X
S 4S
1/ 5
éA- ù
æ K sp ö ë û
or pH = pK a + log
ksp = [S] [4S]4 = 256 S5 \ S = çç ÷
÷ [ HA ]
è 256 ø
Ionic Equilibrium 237
Given pKa = 4.5 and acid is 50% ionised.
[HA] = [A–] (when acid is 50% ionised) 41. (c) H 2CO3 (aq)+ H 2O(l ) ƒ HCO3– (aq) + H3O + (aq)
0.034 x x x
\ pH = pKa + log 1
\ pH = pKa = 4.5 [HCO3- ][H 3O + ] x´ x
K1 = =
pOH = 14 – pH = 14 – 4.5 = 9.5 [H 2 CO 3 ] 0.034 - x
35. (c) Let s = solubility
7 x2
ˆˆ† Ag + IO3
AgIO3 ‡ˆˆ 4.2 10 x 1.195 10 4
s s
0.034
Ksp = [Ag+] [IO3–] = s × s = s2 As H2CO3 is a weak acid so the concentration of
H2CO3 will remain 0.034 as 0.034 >> x.
Given Ksp = 1 × 10–8
x = [H+] = [ HCO3- ] = 1.195 × 10–4
\ s = K sp = -8
1´10
Now, HCO3– (aq) + H 2 O(l ) ƒ CO32– ( aq ) + H3O+ ( aq )
= 1.0 × 10–4 mol/lit = 1.0 × 10–4 × 283 g/lit x y y y
As HCO3- is again a weak acid (weaker than H2CO3)
(Q Molecular mass of Ag IO3 = 283)
with x >> y.
1.0 ´10-4 ´ 283 ´100
= gm /100ml [CO32– ][H3O+ ] y ´ (x + y)
1000 K2 = =
[HCO3– ] ( x - y)
= 2.83 × 10–3 gm/ 100 ml
36. (c) Reaction (c) can be obtained by adding reactions (a) Note : [H3O+] = H+ from first step (x) and from second
and (b) therefore K3 = K1. K2 step (y) = (x + y)
Hence (c) is the correct answer. [As x > > y so x + y ; x and x – y ; x]
37. (c) The correct order of acidic strength of the given species y´x
So, K 2 ; =y
x
HSO3 F > H 3 O + > HSO 4 - > HCO3-
(iv) (ii) (iii) (i) Þ K 2 = 4.8 ´ 10-11 = y = [CO32 - ]
or (i) < (iii) < (ii) < (iv)
So the con centration of [H + ] ; [ HCO3– ] =
It corresponds to choice (c) which is correct answer.
38. (c) In aqueous solution BA(salt) hydrolyses to give concentrations obtained from the first step. As the
dissociation will be very low in second step so there
BA + H2O ‡ˆˆˆˆ† BOH + HA will be no change in these concentrations.
Base acid Thus the final concentrations are
Now pH is given by
[H+] = [ HCO3- ] = 1.195 × 10–4 & [CO23 - ] = 4.8 ´ 10-11
1 1 1
pH = pK w + pKa - pK b
2 2 2 42. (b) ˆˆ† Ag + Br
AgBr ‡ˆˆ
substituting given values, we get Ksp = [Ag+] [Br–]
1 For precipitation to occur
pH = (14 + 4.80 - 4.78) = 7.01 Ionic product > Solubility product
2
K sp 5´10-13
39. (a) Na 2 CO3 ¾¾
® 2Na + + CO32- [Br- ] = = = 10-11
1 ´10-4 M 1´10-4 M 1´10-4 M [Ag+ ] 0.05
i.e., precipitation just starts when 10–11 moles of KBr
Ksp(BaCO3 ) = [Ba 2+ ][CO32– ] is added to 1l AgNO3 solution
\ Number of moles of Br– needed from KBr = 10–11
5.1 ´ 10 -9
[Ba 2+ ] = -4
= 5.1 ´ 10 -5 M \ Mass of KBr = 10–11 × 120 = 1.2 × 10–9 g
1 ´ 10
43. (b) ˆˆ† Mg ++ + 2 OH -
Mg(OH) 2 ‡ˆˆ
40. (a) (i) ® H 3O + H 2 PO 4-
H3PO 4 + H 2 O ¾¾ +
Ksp = [Mg++][OH–]2
acid1 base2 acid 2 base1 1.0 × 10–11 = 10–3 × [OH–]2
-
® HPO 4-- + H 3O +
(ii) H 2 PO 4 + H 2 O ¾¾ 10-11
acid1 base 2 base1 acid 2 [OH- ] = = 10-4
10-3
- -
+ ® H3 PO 4 + O --
¾¾
(iii) H 2 PO 4 OHacid base
\ pOH = 4
base1 2 acid1 2
\ pH + pOH = 14
Hence only in (ii) reaction H2PO4– is acting as an acid. \ pH = 10
238 Chemistry
44. (d) pH = 5 M1V1 = M2V2
\ [H+] = 10–5 0.1 × 1 = 0.01 × V2
ˆˆ† H + + A -
HA ‡ˆˆ V2 = 10 litres
\ Volume of water added = 10 – 1 = 9 litres
t=0 c 0 0
50. (d) Let the weak monoacidic base be BOH, then the
teq c (1 – a) ca ca reaction that occurs during titration is
[H + ][A - ] (c ) 2 [H + ]2 BOH + HCl ® BCl + H2O
Ka = = =
[HA] c (1 – ) c - [H + ] ˆˆ† BOH + H +
Equilibrium : B+ + H 2 O ‡ˆˆ
c (1- h) c.h c.h
(As [H+]) = [A–] = ca)
But, [H+] << c Using the normality equation, N1V1 = N 2 V2
(acid) (base)
\ Ka = (10–5)2 = 10–10
45. (b) ˆˆ† Cr 3+ (aq.) + 3OH - (aq.)
Cr(OH)3 (s) ‡ˆˆ Substituting various given values, we get
s 3s 2 2
´ V1 = 2.5´
(s) (3s)3 = Ksp 15 5
27 S 4 = K sp 2 15
or V1 = 2.5´ ´ = 2.5 × 3 = 7.5 ml
1/ 4 -30 ö 1/ 4
5 2
æ K sp ö æ 1.6 ´ 10
s=ç =ç ÷ Then the concentration of BCl in resulting solution is
è 27 ÷ø è 27 ø given by
46. (a) ˆˆ† 2CO
CO2 + C(graphite) ‡ˆˆ
2
Pinitial 0.5atm 0 2.5 1
[BCl] = 5 = or 0.1 M
10 10
Pfinal (0.5 – x)atm 2 x atm
Total P at equilibrium = 0.5 – x + 2x = 0.5 + x atm [Total volume = 2.5 + 7.5 = 10 ml]
0.8 = 0.5 + x Kw
\ x = 0.8 – 0.5 = 0.3 atm Since K h =
Kb
Now Kp = (PCO ) /PCO
2

1´10-14
2
(2 ´ 0.3) 2
(0.6) 2 \ Kh = = 10–2
= = = 1.8 atm 1´10-12
(0.5 – 0.3) (0.2)
47. (d) For the reaction 0.1h 2 0.1h 2
Thus K h = or 10-2 =
N2 + O2 ¾¾ ® 2NO K = 4 × 10–4 (1- h) (1- h)
Hence for the reaction
or 10–2 – 10–2 h = 0.1 h2
1 1 1 1
NO ¾¾
® N2 + O2 K'= = = 50 or 0.1 h2 + 10–2 h – 10–2 = 0
2 2 K 4×10 – 4
(Solving this quadratic equation for h, we get)
48. ˆˆ† H + Q
(c) HQ ‡ˆˆ
é -b ± b 2 - 4ac ù
c(1–a) ca ca êx = ú
Using ê 2a ú
[H + ] ë û
[H + ] = c a ; a =
c
-10-2 ± (10-2 )2 + 4 ´10-1 ´ 10 -2
10 –3 h=
or a = = 10-2 2 ´ 0.1
0.1

K a = c a 2 = 0.1´ 10-2 ´10 -2 = 10–5 -10-2 ± 10-4 + 4´10-3


=
49. (d) Q pH = 1 ; H+ = 10–1 = 0.1 M 2´ 0.1
pH = 2 ; H+ = 10–2 = 0.01 M
\ M1 = 0.1 V1 = 1 -0.01 ± .0001 + 0.004
=
M2 = 0.01 V2 = ? 0.2
From
-0.01 ± 0.0041
=
0.2
Ionic Equilibrium 239

0.01 ± 0.064 2. (d) Ka × Kb = Kw


=
0.2 \1.8 ´ 10-5 ´ K a = 1´ 10-14 \ k a = 5.56 ´ 10-10
0.54 K a [ACID] 1.8 ´ 10 -6 ´ 0.1
= [Neglecting the negative term] +
3. (b) [H ] = ;[H + ] =
0.2 [SALT] 0.5
= 0.27
\ [H+] = c.h = 0.1 × 0.27 = 2.7 × 10–2 M = 0.36 ´ 10-6 ; – log[H + ] = - log[0.36 ´ 10-6 ]
Thus the correct answer is [d]. \ pH = 6.44
51. (d) ˆˆ† M + + X -
MX ‡ˆˆ (Where s is the solubility) 4. (b) NH4OH + HCl NH4Cl + H2O
s s
100×0.2 100×0.1 0 0 Initial millimoles
10 10 10 After neutrilisation
Then Ksp = s 2 or s = K sp
0.01
® M 2+ + 2X -
Similarly for MX 2 ¾¾ pOH= –log 1.8× 10–5 + log
0.01
,\ pOH = 4.73; pH = 9.26

s 2s
1 Kw 1 ´ 10-14
5. (a) a = = = 0.55
é K sp ù 3 Ka ´ K b 1.8 ´ 10-5 ´ 1.8 ´ 10-5
K sp = s ´ (2 s ) 2 = 4 s3 or s=ê ú
ë 4 û
Ksp 3.4 ´10 -11
ˆˆ† 3M + X
and for M 3 X ‡ˆˆ + -3 6. (a) [Ca ++ ] = ; Solubility = = 3.4 ´ 10 -7
3s s [F- ]2 [0.01] 2

1
+ –
7. (b) CO2+H2O H2CO3 H +HCO3 .
é Ksp ù 4
Ksp = (3s )3 ´ s = 27 s 4 or s = ê ú If CO2 escapes, the equilibrium will shift to LHS and [H+]
ë 27 û concentration will decrease
From the given values of Ksp for MX, MX2 and – +
8. (a) H S SH + H ;
2
M3 X, we can find the solubilities of those salts at
temperature, T. [SH][H + ] 10-7 ´ 0.1
Ka = ;[SH] = = 3.3 ´ 10 -8
Solubility of MX = 4´10-8 = 2´10-4 [H 2S] 0.3

1
9. (a) NaOH + CH 3COOH CH 3COONa + H 2O
1
é 3.2 ´10-14 ù 3 é 32 -15 ù 3
20×0.2 50×0.2 0 0
Solubility of MX 2 = êê ú
ú or ê ´10 ú
êë 4 úû êë 4 úû 4 10 0 0
0 6 4 4
1 Millimoles 6 4
Conc. = ;[CH3COOH] = ;[CH3COONa] =
= éê8´10-15 ùú 3 or 2 10 5 Total Volume 70 70
ë û
1 pH = – log 1.8 ´10 -5 + log 4 / 70 , pH = 4.56
é 2.7 ´10-15 ù 4 6 / 70
Solubility of M3 X = êê ú
ú
êë 27 úû K sp 1´ 10-11
1 10. (a) [OH - ]2 = ++
= = 10-10 ;
é10-16 ù 4 –4 [Mg ] 0.1
= ê úû or 10
ë
Thus the solubilities are in the order MX > M3 X> MX2 [OH- ] = 10-5 ;[H+ ] = 10-9 ; pH = 9
i.e the correct answer is (d).
11. (b) pH = 9 ; [H+ ] = 10-9 ;[OH- ] = 10-5 ;
EXERCISE 4
2+ 2- [Mg ++ ] = 1´10 -3 ;[Mg++ ][OH - ]2 = 1´10-13
1. (a) CaC O ƒ Ca + C O ; Ksp = S2
2 4 2 4
given Ksp of Mg(OH ) 2 = 8.9 ´10 -12 which is more than
-9 -5 mol/litre
\ S = K sp = 2.5 ´ 10 = 5 ´ 10
1 ´ 10 -13 . Hence Mg(OH)2 will not precipitate
solubility = 5 ´10 -5 ´128 = 0.0064 g
240 Chemistry
12. (b) NaCN is a salt of strong base and weak acid ; pH
NH +4 + H 2 O NH 4 OH + H +
1 1
= 7 + pK a + log C
2 2 NaCN Slightly alkaline due to the reaction
pKa for HCN = 14–4.70 = 9.30 CN - + H 2 O HCN + OH -
1 1 HCl highly acidic
\ pH = 7 + ´ 9.30 + log 0.5; pH = 11.5
2 2 The pH of the solution will follow the order highly acidic
13. (a) HCOONH 4 is a salt of weak acid and weak base ; < slightly acidic < neutral < slightly alkaline
i.e. HCl < NH4Cl < NaCl < NaCN
pH = ½ pKw + ½ pKa – ½ pKb
+ (25 ´ 0.3 - 25 ´ 0.1)
1 1 1 21. (b) [H ] = meq = 0.05M
\ pH = ´ 14 + ´ 3.8 - ´ 4.8 ; pH = 6.5 100
2 2 2
- (20 ´ 0.02) - (30 ´ 0.01)
14. (c) Na2CO3 +2H2O 2NaOH+H2CO3 22. (a) [OH ] = = 0.002M
50
CO3-- + 2H 2 O ƒ 2OH - + H 2 CO3 Anionic hydrolysis – log [OH–] = pOH = – log .002 = 2.7
\ pH = 14 – 2.7 = 11.3
15. (c) BaF2 Ba ++ + 2F- , Ba(NO3 )2 23. (a) Isohydric solutions have the same [H+] concentration
24. (a) A B C
-
F- are obtained from BaF2 . Then ½ [F ] = [BaF2] Ka 10–4 10–6 10–10
16. (c) HCN is a weak acid and energy is required to affect its The higher the value of Ka, the stronger the acid and the
ionisation. Hence the enthalpy of neutralisation of HCN lower is pH. Hence order of pH A < B < C.
is less than the enthalpy of neutralisation of strong acid. 25. (b) Fe(OH) 2 Fe + + + 2 OH
The difference of enthalpy gives the enthalpy of
ionisation. 55.9 kJ-12.1 kJ = 43.8 kJ pH = 8, [H+] = 10–8, [OH–] = 10–6
17. (c) Let the volume of KCN to be added is Vml ; Conc.of KCN K sp 1.6 ´ 10-4
= S×V and Fe++ = = = 0.016
[OH - ]2 [10-6 ]2
Conc. of HCN = 10 ×2 ; pH = 9 , \ H+ = 10–9
26. (c) The hydrolysis of C 2 O 24 - is as follows
K [ACID] -9 5 ´ 10 -10 ´ 10 ´ 2
+
H = a ;10 = ;V = 2ml
[SALT] 5´ V C 2 O 24 - + H 2 O HC2 O4- + OH –
18. (c) An acid base indicator is InH. Its ionisation is as follows
K w 1.0 ´10-14 x2
InH In – + H+ Kh = = -5
=
K 2 6.0 ´10 0.01
acid form (Red) basic form (blue)
\ x = 1.3 × 10–6
+
[In][H ] K [InH] 3 ´ 10 -5 ´ 25
Ka = ;[H + ] = a ;[H + ] = = 1 ´ 10 -5 27. (b) HCO 3- ® H + + CO 32-
[InH] [In] 75
19. (b) pH of 1 ´ 10-8 M HCl is below 7. [H + ][CO32- ]
H2PO4– + H2O HPO42– + H3O+ K2 = = 4.8 ´10-11
[HCO3- ]
conjugate base of H2PO–4 is H3PO4
4.8 ´10 -11[HCO 3- ]
H 2 PO4- + H + ƒ H3PO4 [H + ] = = 4.8 ´10 -11 (0.1 / 0.2)
[CO 32- ]
K (Autoprotolysis constant of water i.e. ionic product of
water) increases with temperature. For half neutralisation pH = – log [H+] = – log (4.8 × 10–11 × 0.5) =10.62
of a weak acid by a weak base, 28. (b) The basic indicator is InOH
InOH In+ + OH–
[Salt]
pH = pKa + log
[Acid] Kb =
[In + ][OH]
[InOH]
[Salt] = [Acid], \ pH = pKa
20. (b) The characteristics of the given solutions are:
[ In + ]
NaCl neutral solution Indicator will change colour when =1
[ InOH ]
NH4Cl slightly acidic due to the reaction
\ Kb = [OH–] = 1×10–11 and [H+] = 10–3
\ pH = 3
Ionic Equilibrium 241
29. (c) 37. (a) HA + BOH BA + H 2 O BA is Salt
30. (c) The pH of blood plasma (7.4) is maintained by
Kw
HCO 3- / H 2 CO 3 and HPO 24 - / H 2 PO -4 buffers \ KH = .
Ka ´ K b
31. (a) H2A H+ + HA–
Ka ´ Kb
[H + ][HA - ] [H + ]2 The inverse of K H is Kc =
Ka = = Kw
[H 2 A] Ca
2 ´ 10 -5 ´ 5 ´ 10 -6
\[H+ ] = Ka .Ca = -14
= 1.0 × 10 4
1 ´ 10
+ -3
32. (b) pH = 3, [H ] = 10 (2.0 ml) ; pH = 10, 38. (b) HIn H + + In

[H + ] = 10 -10 (3.0 ml). [In ]


pH = pKIn + log
On mixing the volume becomes 5 ml [HIn ]

+ 2 ´ 10 - 3 3 10 -10 20
\ [H ] = + ´ 10 -10 = (2 ´ 10 7 + 3) pH1 = pK In + log = pK In - 2log 2
5 5 5 80

10 -10 2 -3 pH 2 = pK In + log
80
= pK In + 2log 2
or [H + ] = ´ 2 ´ 107 = ´ 10
5 5 20
pH 2 - pH1 = pK In + 2log 2 - (pK In - 2log 2)
– log [H + ] = – log 2 + 3 log 10 + log 5
= 4 log 2 = 1.20
\ pH = – 0.3010 + 3 + 7 = 3.4
39. (b) HCOOH + NaOH ƒ HCOONa + H 2 O,
33. (b) ApBq pA + + qB
Sp Sq
HCOONa is salt of weak acid and strong base which is
0.10 M.
Let the solubility be s mol/liter
Thus, 1
pH = (K w + pK a + log C)
2
+ p – q p q p. q p+q
Ksp = [A ] [B ] = [sp] [sq] = p q (s) .
1
34. (a) The solubility decreases by common ion effect. = (14 + 3.74 + log 0.1) = 8.37
2
A[Cl - ] = 0, B[Cl - ] = 0.1 40. (d) The stronger the base, the more is proton accepting
C [Cl - ] = 0.2 D [Cl–] = 0 tendency RNH 2 is stronger than NH 3 and HI is strong

[Soluble complex is formed [Ag(NH 3 ) 2 ]Cl] -


acid, hence I is weak base
35. (b) K w = K a ´ K b
é [salt] ù
-14
41. (c) pH = 14 – pOH = 14 – ê pK b + log ú
Kw 10 -4 ë [base] û
\ Ka = = = 6.75 ´ 10
K b 1.48 ´ 10-11
80
= 14 + log Kb + log
36. (c) a1 = 0.005 = K a ´ C1 20

Molarity of diluted solution 2 × 1 = 32 × M, Kb = 1.0 ´ 10 - 6


\
M = 116 (C 2 ) 42. (b) Let volume of HCl required be V ml then volume of NH 3
= 300 – Vml.
Ka
a2 = = 0.005 16 = 0.02 [salt]
C2 pH = pK a + log ;
[base ]
+ 0.02
H3 O = C 2 a 2 = = 1.25 ´ 10 -3 M V
16 9.26 = 9.26 – log ; V = 100 ml.
(300 – V) - V,
242 Chemistry
50. (b) To precipitate the AgCl
43. (c) Ksp =4 S3 = 3.2 ´ 16 -11 \ S = 2 × 10 - 4 M
[Ag + ] required
[ OH ] = 2 S = 2 × 2 × 10 -4 M
K sp (AgCl) 1.0 ´10-10
\ pH = 14 – pOH = 14 + log 4 ´ 10 -4 = 10.60 = = = 1.0 ´10-9 M
[Cl - ] 0.1
44. (a) MgCl 2 +2NaOH ® Mg(OH) 2 +2NaCl
100 × 0.1 100 × 0.2 0 0 Initial moles K sp (AgBr) 1.0 ´ 10 –13
[Br - ] left at this stage = =
0 0 10 20 After mixing [Ag + ] 1.0 ´ 10-9
2 = 1.0 × 10–4 M
é 10 ù é 20 ù -4
\ Ksp = ê úê ú = 5 ´10 .
ë 200 û ë 200 û 1.0 ´ 10 -4
% of remaining [ Br - ] = ´100 = 0.1
0.1
But actually Ksp of Mg (OH ) 2 = 1.2 ´ 10 - 11
% of Br - to be precipitated = 100 – 0.1 = 99.9
\ 4S3 = 1.2 ´ 10 -11
- -4 .
51. (c) pH = 8, pOH = 6; [OH - ] = 10 -6 M;
Find S then [OH ] = 2S which is 2.8 ´ 10
-6 2
45. (a) At the half equivalence point we have equal Ionic product of Fe(OH)2 = 0.2 × (1´10 )
concentrations of base left and the salt formed.
= 2 ´ 10 -13 > K sp ( = 8.1 ´ 10 -16 )
[salt]
\ pOH = pKb + log ; pOH = pKb 52. (b) pOH = 14 – pH = 14 – 8.26 = 5.74
[base]
and pH = 14 – pKb [ NH 4 + ]
= pKa + log
46. (b) The pH at which the dipolar ion does not migrate in the [ NH 3 ]
electric field is called isoelectric point. It is given by
pK a1 + pK a 2 [ NH 4 + ]
2.09 + 3.86 = 4.74 + log
= = 2.98. 0.1
2 2
+
47. (a) The solubility S of AgCl, S2 = Ksp; S2 = 1.0 × 10 - 10 ; [ NH 4 ] =1 molL -1
S = 1´10 -5 mol L-1 . 1
\ ( NH 4 ) 2 SO 4 = of [ NH 4 + ] = 0.5 mol L-1 )
The mass dissolved in 10 L 2
= 10 ´ 1.0 ´ 10 -5 ´ 143.5 = 1.435 ´ 10 -2 g 53. (a) CO 2 with H 2 O forms H 2 CO 3

48. (d) CH 3COOH + NaOH ¾


¾® CH 3COONa + H 2 O CO 2 + H 2 O H + + HCO 3-

0. 3 0 .1 [H + ][HCO3- ]
at. 1/4 neutralisation K1 = = 4.5 ´10-7
4 4 [CO 2 ]
0 .1 0. 3
4 4
at 3/4 neutralisation Again pH = - log[H + ] = 7.4

1 3 \ [H + ] = 4.0 ´10 -8
pH1 = pKa + log ; pH 2 = pKa + log
3 1
[HCO3- ] 4.5 ´10-7
\ = = 11
1 [CO 2 ] 4 ´ 10-8
pH1 – pH 2 = log – log 3 = 2 log 3
3

49. (d) NaCN(aq) + HCl(aq) ¾ ¾® HCN(aq) + NaCl (aq).


Millimoles of NaCN = 50 × 0.1 = 5;
Millimoles of HCl = 50 × 0.2 = 10
+
Excess millimoles of HCl = 5 in 100 ml : [H 3 O] = 0. 05
pH = – log 0.05 = 1.30
8
Redox Reactions
OXIDATION : REDOX REACTIONS :
Oxidation may be defined in any of the following terms Reactions comprising of simultaneous oxidation and reduction
(a) Addition of oxygen. and called oxidation - reduction or redox reactions.
2 Mg + O2 ® 2MgO SnCl2 + 2HgCl2 ® SnCl4 + Hg2Cl2
(b) Removal of hydrogen
TYPES OF REDOX REACTIONS :
3O2 + 4NH3 ® 2N2 + 6H2O
(c) Addition of electronegative portion (i) Intermolecular redox reactions - In this case one substance
Cu + Cl2 ® CuCl2 is oxidised and another is reduced.
(d) Removal or decrease in the electropositive portion 4 HCl + MnO2 ® MnCl2 + Cl2 + 2H2O
H2S + Cl2 ® 2 HCl + S Here HCl is oxidised and MnO2 is reduced.
(ii) Disproportionation - In this case the same substance is
¾® M n + + ne-
(e) De - electronation M ¾
oxidised and reduced eg.
REDUCTION : +5 +7
Reduction may be defined in any of the following terms 4 KClO3 ® 3KClO 4 + KCl –1
(a) Addition of hydrogen (iii) Intramolecular redox reactions - In this case one element
N2 + 3H2 ® 2NH3
of the compound is reduced while another element of the
(b) Addition of electropositive portion
same compound is oxidised
CuCl2 + Cu ® Cu2Cl2
(c) Removal of oxygen ( NH 4 ) 2 Cr2 O 7 ® N 2 + Cr2 O 3 + 4H 2 O
CuO + H2 ® Cu + H2O Cr is reduced and N is oxidised
(d) Removal or decrease in the electronegative portion
OXIDATION NUMBER :
2HgCl2 + SnCl2 ® Hg2Cl2 + SnCl4
(e) Electronation M + ne– ® Mn– It is the number of electrons lost or gained by an
element during its change from free state in a particular compound.
OXIDANT OR OXIDISING AGENT :
Or
As stated above the oxidising agent may be defined as a substance
It is defined as the formal charge present on an atom in a particular
supplying oxygen or electronegative element, removing hydrogen
compound determined by certain arbitrary rules.
or electropositive element and can accept electrons. They show
decrease in oxidation number Examples. RULES FOR DETERMINING OXIDATION NUMBER:
K2Cr2O7, KMnO4, H2O2, Cl2, Br2, KClO3, FeCl3 etc. (i) O.N. of elements in free state is zero eg Cl 2 , N 2 , Mg, Ca
REDUCTANT OR REDUCING AGENT : (ii) O.N. of hydrogen is always +1 except in ionic metal hydrides
A substance supplying hydrogen or electropositive element, where it is – 1.
removing oxygen or electro negative element and can donate
(iii) O.N. of oxygen is –2 except in OF2 where it is + 2 and in
electrons. They show increase in oxidation number.
peroxides where it is – 1.
Examples SnCl2, H2, H2S, Mg, FeSO4, H2C2O4, H2SO3.
244 Chemistry
(iv) O.N. of metals is always +ve. For IA group elements it is +1 CALCULATION/ DETERMINATION OF OXIDATION
and for IIA group elements it is +2. NUMBER OF UNDERLINED ELEMENT IN SOME
(v) O.N. of halogens is –1 in metal halides. COMPOUNDS:
(vi) O.N. of ion or radical is the number of electrons it must gain
(a) K 2 Cr 2 O 7 -
or lose to acquire neutrality i.e. it is equal to the electric
Let the O.N. of Cr be x then
charge for SO4– - O.N. is –2. 2 × (+1) + 2 × (x) + 7 × (–2) = 0
(vii) O.N. of an atom within compound can be +ve, –ve integer, 2 + 2x –14 = 0 \ x = +6
zero or fraction. (b) KMnO4 -
(viii) The algebraic sum of all the O.N. of elements is equal to zero. Let the O.N. of Mn be x then
(ix) The algebraic sum of all the O.N. of elements in an ion is 1 × (+1) + 1 × (x) + 4 × (–2) = 0
equal to net charge on the ion. 1 + 1x - 8 = 0 \ x = +7
(c) H2SO4 -
(x) Maximum O.N. of an element is equal to number of valence
Let the O.N. of S be x then
electrons i.e. group number.
2 × (+1) + 1 × (x) + 4 × (–2) = 0
(xi) Minimum O.N. of an element (except metals)
2 + x –8 = 0 \ x = +6
= (8 – group number).
(xii) In metal corbonyl, and amalgams, O.N. of metals is zero. (d) NH4NO3 - Split into two ions NH +4 and NO 3–
COVALENCY AND OXIDATION STATE Let O.N. of N be x in NH +4 ion then
(i) Covalency : It is the number of hydrogen atoms which can 1 × (x) + 4 × (+1) = +1
combine with a given atom x + 4 = +1 \ x = –3
or Let the O.N. of N be x in NO 3– ion then
It is the number of single bonds which an atom can form.
1 × (x) + 3 × (–2) = –1
or
x – 6 = –1 \ x = +5
It is the number of electrons an atom can share. Valency is
always a whole number. (e) PO 3–
4 -
(ii) Oxidation state : It is defined as the O.N. per atom. Let the O. N. of P be x then
STOCK NOTATION : 1 × (x) + 4 × (–2) = –3
Representation of oxidation state of element by Roman numerals x – 8 = –3 \ x = +5
within parenthesis is known as stock notation eg FeCl3 is written (f) HNO3 -
as Iron(III) chloride and FeSO4 as Iron (II) sulphate. Let the O.N. of N be x then
CHEMICAL BONDING METHOD FOR 1 × (+1) + 1 × (x) + 3 × (–2) = 0
DETERMINATION OF OXIDATION NUMBER 1+ x–6=0 \x = 5
Sometimes wrong results are obtained when the O.N. is determined (g) KI3 -
by applying the above mentioned simple rules. In such cases Let the O.N. of I be x then
applying the chemical bonding method is very useful. The rules 1 × (+1) + 3 × (x) = 0
are \ 1 + 3x = 0 \ x = –1/3
(i) For one covalent bond assign one unit negative charge to (h) NaO2 - It is super oxide.
electronegative atom and one unit positive charge to less Let O.N. of O be x then
electronegative atom eg 1 × (+1) + 2 × (x) = 0\ 1 + 2x = 0 \ x = –1/2
(i) Fe3O4 - It is mixed oxide FeO.Fe2O3 and Fe has O.N. +2
A –1 – B +1 (electronegativity A> B).
(ii) No charge when the covalent bond is between like atoms. and +3 respectively.
(iii) In case of coordinate bond assign two unit negative charge (j) Fe0.96 O
to acceptor atom and two unit positive charge to donor atom
Let O.N. of Fe be x then 0.96 x + ( -2 ) = 0
e.g. A 2 – ¬ B 2+ (electronegavity A > B).
(iv) No charge when donor in coordinate bond is more \ 0.96 x - 2 = 0
electronegative than acceptor eg
+2
A ® B (electronegavity A > B). \x =
0.96
(v) When coordinate bond is between similar atoms
assign two unit negative charge to acceptor and (k) N3H (hydrazoic acid)
two unit positive charge to donor e.g., Let O.N. of N be x then
3 × (x) + 1 × (+1) = 0
A 2 – ¬ A 2 + (electronegativity same).
3x + 1 = 0 \ x = –1/3
Redox Reactions 245
(l) K4[Fe(CN)6] - 2 × (+1) + 1 × (–1) + 1 × (x) + 3 × (–2) = 0
Let the O.N. of Fe be x then for Na for S for S* for O
4 × (+1) + 1 × (x) + 6 × (–1) = 0 2 –1 + x –6 =0 \ x = +5
(sum of O.N. of CN– = –1) \ O.N. of S* = +5 and another S = –1
4 + x – 6 = 0\ x = 2 (r) Na2S4O6 - Sodium tetra thionate - its structure is as
Determination of oxidation number by chemical bonding follows
method
(m) CrO5 - Let the O.N. of Cr be x then O O
1 × (x) + 5 × (–2) = 0
Na + - O – S* - S - S - S - O - Na +
x – 10 = 0 \ x = 10 (wrong)
Apply chemical bond method O O

O O Let the O.N. of S* be x then


Cr 2 × (+1) + 6 × (–2) + 2 × (0) + 2 × (x) = 0
O O
for Na for O for middle S
O
\ 2 – 12 + 0 + 2x = 0 \x=5
x + 1 × (–2) + 4 × (–1) = 0
–2 +1
(for Cr) (one=O) (four O – O ) O – Cl
+2
\ x = +6 (s) CaOCl2 - Its structure is Ca –1
(n) HCN - Its structure is H – C º N Cl
for H – C bond H = +1, C = –1 \ O.N. of Cl is –1 and +1.
for C º N bond C = +3,N = –3 +1
O –1
\ sum of O.N. of H = +1, C = +2 and N = –3 (t) O3 - The structure of O3 is O O
(o) H – N = C for H – N bond H = +1, N = –1 \ O.N. of O in ozone is + 1 and –1
for N = C, N = –2 and C = +2 (for two covalent bonds) OXIDATION NUMBER CONCEPT OF OXIDANT
No contr ibution of N ® C bond sin ce N more (OXIDISING AGENT) AND REDUCTANT
electronegative than C) (REDUCING AGENT) :
\ O.N. of different atoms H = +1, C = +2, N = –3 (i) Oxidising agent : A substance can act as oxidising agent if
(p) H2SO5 (Caro acid) - Write structure and apply chemical the oxidation number of one of its element is maximum eg
bond method HNO3 (O.N. of N = 5 which is maximum value)
-2
The more the electronegativity of element and the more is
O O.N., the more is the oxidising power eg KClO4, KBrO4.
+1 – 2 ­+ 6 -1 -1 +1 KMnO4, K2Cr2O7, HClO4, HNO3 etc. Oxyanions are stronger
H – O - S - O- O- H
¯ oxidising agents in acidic solution than in basic or neutral
O solution.
-2
2 × (+1) + x + 3 × (–2) + 2 (–1) = 0 (ii) Reducing agent : A substance can act as reducing agent if
for H for S for O for O – O \ x = +6 the oxidation number of one of its element is minimum eg
(q) Na2S2O3 SnCl2 (O.N. of Sn = 2 which is minimum value), FeSO4,
Na2S2O3, H2S, H2C2O4 Electronegative elements I-, Br–, N3–
S are powerfully reducing in nature.
(iii) Reducing as well as oxidising agent : A substance that can
(Structure I) Na + O – –– S* -O – Na +
act as both, reducing as well as oxidising agent if O.N. of one
O of its element is in between the maximum and the minimum
value eg HNO2 (O.N. of N = +3 which is intermediate of +5
2 × (+1) + x + 1 × (–2) + 3 × (–2) = 0
and 0).
for Na for S* for S for O
\ O.N. of S* = +6 and another S = –2 OXIDATION NUMBER AND ACID STRENGTH :
The greater the O.N. of the element in oxyacids, the greater is the
O acid strength.
­
(Structure II) Na + S – S* - ONa + +7 +5 +3 +1
¯ HClO 4 > HClO 3 > HClO 2 > HClO
O
246 Chemistry
EQUIVALENT WEIGHT OF AN OXIDISING AGENT EQUIVALENT WEIGHT OF REDUCING AGENT :
It can be obtained by dividing the molecular weight by the number It can be obtained by dividing the molecular weight by the number
of electrons gained represented in a chemical balanced equation
of electrons lost as represented by a chemical balanced equation
+7 +2
eg. 2K MnO + 3H SO ® K SO + 2MnSO + 3H O + 5O +2 +6 -8 +4 -4
4 2 4 2 4 4 2
H2 C2 O4 + O ¾
¾® 2 C O 2 + H 2 O
Equivalent wt. of KMnO4 in acid medium
Mol.wt of KMnO 4 The change in O.N. of two atoms of carbon is +2. Hence
=
5 Mol.wt of Oxalic acid
Equivalent weight of oxalic acid =
Cr2 O2– + 3+ 2
7 + 14H + 6e ® 2Cr + 7H 2 O

Mol.wt of K 2Cr2 O
Eq. wt. of K2Cr2O7 =

SOME OXIDISING AND REDUCING AGENTS


Oxidising agents
Substance Radical O.N. of Reduction New Difference Gain of
effective product O.N. in O.N. electrons
element
K2Cr2O7 Cr2O 2 – +6 Cr3+ +3 3 3

KMnO4 (acidic) MnO – +7 Mn 2+ +2 5 5


KMnO4 (alkaline) MnO – +7 MnO 24 – +6 1 1

KMnO4 (neutral) MnO – +7 MnO2 +4 3 3


H2O2 O2 0 O2– –2 2 2
Cl2 Cl 0 Cl– –1 1 1
KClO3 ClO– +5 Cl– –1 6 6
FeCl3 Fe3+ +3 Fe2+ +2 1 1

HNO3 NO +5 NO2 +4 1 1
Reducing agents
Substance Radical O.N. of Oxidation New Difference Loss of
effective product O.N. in O.N. electrons
element
SnCl2 Sn2+ +2 Sn4+ +4 2 2
FeSO4 Fe2+ +2 Fe3+ +3 1 1
H2 H 0 H+ +1 1 1
H2S S2– –2 S0 0 2 2
Mg Mg 0 Mg2+ +2 2 2
H2SO3 SO 2 – +4 SO 2 – +6 2 2

Na 3AsO3 AsO33 – +3 AsO 34 – +5 2 2


Na Na 0 Na+ +1 1 1
H2C2O4 C 2 O 24 - +3 CO2 +4 1 1

BALANCING OF CHEMICAL EQUATIONS : (b) Balance the total number of atoms undergoing change
For balancing a chemical equation the two important methods are in oxidation state.
(1) Oxidation number method - The certain rules are as follows (c) Balance the number of electrons gained and lost.
(a) Assign oxidation number to the atoms showing a change (d) Balance [O] on both sides by adding H2O.
in oxidation state. (e) Balance H atoms by adding H+ ions
Redox Reactions 247
(f) If the reaction proceeds in basic solution add sufficient
Cr2 O 27 - + H + ® 2Cr 3+ + H 2 O
number of OH– ions on both sides.
Example 1 : Balance O and H atoms on both sides by adding H2O and H+
Balance the equation involving oxidation of ammonia ions.
by copper oxide to give Cu, N2 and H2O Cr2 O 72 - + H + ® 2Cr 3+ + H 2 O
+2 –3 +13H + + 6 H 2O
0
Solution - CuO + 2 NH 3 ® Cu + N 02 + H2O Balance charge on both sides by adding electrons
­ ¯
3´ 2 = 6e 6e Cr2 O72- + 14H + + 6e - ® 2Cr 3+ + 7H 2O
\ 3CuO + 2NH3 ® 3Cu + N2 + H2O (ii) Oxidation half-reaction
Balance O by adding H2O to RHS
3 CuO + 2NH3 ® 3Cu + N2 + 3H2O Fe 2 + – e- ® Fe3+
Example 2 : Balance electrons of two half reactions
The reduction of permanganate ion by ferrous ion in
6Fe 2+ – 6e - ® 6Fe3+
presence of a dilute acid
Adding two half reaction (electrons are cancelled)
MnO4– + Fe 2 + + H + ® Mn 2 + + Fe 3+ + H 2 O
Cr2 O72 - + 6Fe2+ + 14H + ® 2Cr 3+ + 6Fe3+ + 7H 2O
Solution =
Example 2 :
+7 Balance the equation
MnO 4– + Fe 2+ + H + ® Mn 2+ + Fe 3+ + H 2 O
­ ¯ H 2 C 2 O + H 2 O 2 ® CO 2 + H 2 O
5e 5´1e Solution (i) Oxidation half - reaction
H 2 C 2 O ® CO 2
MnO 4– + 5Fe 2 + + H + ® Mn 2 + + 5Fe 3+ + H 2 O
Balance C, O, H atoms and charge
Balance O and H by adding H2O and H+; 3H2O on RHS
and 7H+ on LHS H 2C 2O4 ® 2CO2 + 2H + + 2e - .......... (i)
MnO 4– + 5Fe 2 + + 8H + ® Mn 2 + + 5Fe 3+ + 4H 2 O (ii) Reduction half reactions
Example 3 : H 2 O 2 ® H 2O
The reduction of dichromate ion by an iodide in Balance O, H atoms and charge
presence of a dilute acid H 2 O2 + 2H + + 2e- ® 2H 2O .......... (ii)
Cr2 O 27 - + 3+
+ I + H ® Cr + I 2 + H 2 O Add two balanced half reactions (i) and (ii)
Solution - Balancing atoms H 2 C 2 O 4 + H 2 O 2 ® 2CO 2 + 2H 2 O
+12 Example 3 :
Cr2 O 72 - + 2I + H + ® 2Cr 3+ + I 2 + H 2 O Write the complete balanced equation for the change
­ ¯
6e 3 ´ 2e CrO24 – + SO 32 – ® Cr(OH) 4– + SO 24 – taking place in basic
solution
\ Cr2 O27 - + 6I- + H + ® 2Cr 3+ + 3I 2 + H 2O Solution - (i) The equation is
Balance O by adding 6H2O on RHS and balance H+ by
adding 13H+ on LHS CrO24 – + SO 32 – ® Cr(OH) 4– + SO 24 – reduction half
reaction.
\ Cr2 O27 - + 6I - + 14H + ® 2Cr 3+ + 3I 2 + 7H 2O
(2) Ion electron method - The rules are as follows: CrO24 – ® Cr (OH ) 4–
(a) Split up the reaction into two half reactions showing Balancing O, H atoms and charge on both sides
oxidation and reduction separately.
CrO24 – + 4H + + 3 e ® Cr(OH) 4– (i)
(b) Balance number of atoms undergoing the change of
oxidation state. (ii) Oxidation half reaction
(c) Balance O on both sides by adding H2O. SO 32 – ® SO 24 –
(d) Balance H atoms by adding H+ ions.
Balancing O, H, charge and atoms on both sides
(e) Balance charge by adding required number of electrons
(f) Make the number of electrons equal in two half reactions SO 32 – + H 2 O – 2e ® SO 24 – + 2H + (ii)
by multiplying with suitable coefficient. Multiply (i) by 2 and (ii) by 3 and add
(g) Add the two half reactions
Example 1 : 2CrO24 – + 8H + + 3SO 32 – + 3H 2 O ® 2
Oxidation of ferrous salt by potassium dichromate in acid
solution. ® 2Cr (OH) 4– + 3SO 24 – + 6H +
or
Cr2 O 72 - + Fe 2+ + H + ® Cr 3+ + Fe3+ + H 2 O
Solution - (i) Reduction half reactions 2CrO24 – + 2H + + 3SO 32 – + 3H 2 O ®
® 2Cr(OH) 4– + 3SO 24 –

Again adding 2OH on both sides.
Cr2 O 27 - + H + ® Cr 3+ + H 2 O
Equalize Cr atoms CrO242–
22CrO –
SO 3232–
4 ++33SO

++55H O®
H 22O ® 2Cr(OH) 4– + 3SO42– + 2OH -
248 Chemistry

Very Short/Short Answer Questions (ii) Balance the following reaction by oxidation number
method
1. Why is anode called oxidation electrode whereas cathode is As2S5 + H+ NO3–(conc.) ¾¾ ®
called reduction electrode? H3AsO4 + 5H2SO4 + H2O + NO2
2. 2Cu2S + 3O2 ¾¾ ® 2Cu2O + 2SO2. In this reaction which 14. (i) Balance the following equations using the half-reaction
substance is getting oxidised and which substance is getting
method in the acidic medium.
reduced. Name reducing agent and oxidising agent.
3. Calculate oxidation number of Cr in K2Cr2O7 and S in S2O32– (a) Zn + NO3- ¾¾
® Zn 2+ + NH +4
4. Fe decomposes steam while Cu does not, why?
5. Arrange the following in order of increasing oxidation number (b) MnO 4- + H 2 C 2 O 4 ¾¾
® Mn 2 + + CO 2
of iodine. I2, HI, ICl.
6. In the reaction given below which species is called a spectator (c) Cr2 O 27 - + Cl - ¾¾
® Cr 3+ + Cl 2
ion and why? (ii) Balance the following redox reaction in basic medium
Zn( s) + Cu 2+ (aq) + SO 24- (aq ) ¾¾
® using the half-reaction method:
Zn 2+ (aq) + Cu(s ) + SO24 - (aq ) (a) Mn 2 + + ClO3- ® MnO2 + ClO2
7. Show that oxidation and reduction go side by side by taking
a suitable example. (b) Cl2 ® Cl - + ClO3-
8. MnO 24 - undergoes disproportionation reaction in acidic 15. (i) What are the highest oxidation numbers of N, S and
Cl?
medium but MnO4- does not. Give reason.
(ii) ˆˆ† 2CrO24-+ 2H+ a redox reaction?
Is Cr2O27- + H 2O ‡ˆˆ
9. Starting with the correct half-reaction, write the correct overall
net ionic reaction in the following changes. (iii) Calculate oxidation number of :
(i) Chromite ion (CrO3–) is oxidised by H2O2 in strongly (a) Cr in CrO5
basic solution. (b) S in H2SO5
(ii) Iodate ion (IO3– ) oxidises the iodide I– to I2 in acidic (c) Fe in Fe3O4.
solution.
Multiple Choice Questions
(iii) Permanganate ion MnO–4 oxidises the oxalate ion (C2O24–)
to CO2 in acidic solution. 16. Which of the following is not an example of redox reaction?
10. Is it possible to store: (a) CuO + H2 ® Cu + H2O
(i) copper sulphate solution in a zinc vessel? (b) Fe2O3 + 3CO ® 2Fe + 3CO2
(ii) copper sulphate solution in a nickel vessel? (c) 2K + F2 ® 2KF
(iii) copper sulphate solution in a silver vessel? (d) BaCl2 + H2SO4 ® BaSO4 + 2HCl
(iv) copper sulphate solution in a gold vessel? 17. The oxidation number of an element in a compound is
(Take help from electrochemical series) evaluated on the basis of certian rules. Which of the following
11. The standard electrode potential corresponding to the rules is not correct in this respect?
reaction (a) The oxidation number of hydrogen is always + 1.
Au3+(aq) + 3e– ® Au(s) is + 1.42 V. (b) The algebraic sum of all the oxidation numbers in a
Predict if gold can be dissolved in 1 M HCl solution and on compound is zero.
passing H2 gas through gold solution, metallic gold will be (c) An element in the free or the uncombined state bears
precipitated or not? oxidation number zero.
12. Arrange A, B, C, D, E and H in order of increasing electrode (d) In all its compounds, the oxidation number of fluorine is
potential in the electrochemical series: – 1.
A + H2SO4 ® ASO4 + H2 18. Which of the following arrangements represent increasing
ACl2 + C ® CCl2 + A oxidation number of the central atom?
ECl2 + C ® No reaction
2BCl + D ® DCl2 + 2B (a) CrO-2 , ClO3- , CrO24 - , MnO -4
H2SO4 + D ® No reaction
(b) ClO3- , CrO42 - , MnO -4 , CrO -2
Long Answer Questions
(c) CrO-2 , ClO3- , MnO4- , CrO42 -
13. (i) Balance the following reaction by ion–electron method
K+ MnO4 + H+ Cl– ¾¾ ® (d) CrO24 - , MnO4- , CrO2- ,ClO3-
K+Cl– + Mn2+(Cl–)2 + H2O + Cl2
Redox Reactions 249
19. Nitric oxide acts as a reducing agent in the reaction –
E Values :

Fe3+/Fe2+ = + 0.77; I2(s)/I = + 0.54;
(a) 4 NH 3 + 5 O 2 ® 4 NO + 6 H 2O Cu2+/Cu = + 0.34; Ag+ /Ag = + 0.80V
(b) 2 NO + 3I 2 + 4H 2O ® 2NO3– + 6 I – + 8H + (a) Fe3+ (b) I2(s) (c) Cu2+ (d) Ag+
23. Which of the following elements does not show
(c) 2 NO + H 2SO 3 ® N 2 O + H 2SO 4 disproportionation tendency?
(d) 2 NO + H 2S ® N 2 O + S + H 2 O (a) Cl (b) Br (c) F (d) I
20. What products are expected from the disproportionation 24. For the redox reaction
reaction of hypochlorous acid?
(a) HCl and Cl2O (b) HCl and HClO3 MnO – + C 2O 2 – + H + ® Mn 2+ + CO 2 + H 2 O
(c) HClO3 and Cl2O (d) HClO2 and HClO4 the correct coefficients of the reactants for the balanced
21. Thiosulphate reacts differently with iodine and bromine in reaction are:
the reactions given below:
2S2 O32 - + I2 ® S4O62- + 2I - MnO – C 2O 2 – H+
S2 O32 - + Br2 + 5H 2 O ® 2SO 42- + 2Br - + 10H + (a) 2 5 16
Which of the following statements justifies the above dual (b) 16 5 2
behaviour of thiosulphate? (c) 5 16 2
(a) Bromine is a stronger oxidant than iodine.
(d) 2 16 5
(b) Bromine is a weaker oxidant than iodine.
(c) Thiosulphate undergoes oxidation by bromine and 25. ( x) MnO – + ( y)H 2 O 2 ®
reduction by iodine in these reactions.
(d) Bromine undergoes oxidation and iodine undergoes 2Mn +2 + 5H 2 O + 13/ 2O 2 + (z)e -
reduction in these reactions.

22. The more positive the value of E , the greater is the tendency In this reaction, value of (x), (y) and (z) are
of the species to get reduced. Using the standard electrode (a) 2, 5, 6 (b) 5, 2, 9 (c) 3, 5, 5 (d) 2, 6, 6
potential of redox couples given below find out which of the
following is the strongest oxidising agent.

1. In the following reaction, which is the species being oxidised ? 4. Which substance serves as reducing agent in the following
reaction?
2Fe3+ (aq) + 2I- (aq) ® I 2 (aq) + 2Fe2+ (aq)
(a) Fe + (b) I– 14H + + Cr2 O 2– 3+
7 + 3Ni ® 2Cr + 7H 2O + 3Ni
2+

(c) I2 (d) Fe 2 + (a) H 2 O (b) Ni


2. Which of the following reactions depicts the oxidising
property of SO2 ? (c) H+ (d) Cr2O 2-
(a) SO 2 + H 2 O ® H 2 SO 5. In the reaction
(b) 2H 2S + SO 2 ® 3S + 2H 2O 3Br2 + 6CO32- + 3H 2O ® 5Br - + BrO3– + 6HCO 3-
(c) Cl 2 + SO 2 ® SO 2 Cl 2 (a) bromine is oxidised and carbonate is reduced.
(b) bromine is reduced and water is oxidised
(d) 2MnO 4– + 5SO 2 + 2H 2 O ® SO 2– + 2Mn 2+ + H +
(c) bromine is neither reduced nor oxidised
3. Which of the following statements is not correct ?
(d) bromine is both reduced and oxidised
(a) Potassium permanganate is a powerful oxidising
substance. 6. Which of the following is a redox reaction ?
(b) Potassium permanganate is a weaker oxidising agent than (a) H2SO4 with NaOH
potassium dichromate (b) In atomosphere, O3 from O2 by lightning
(c) Potassium permanganate is a stronger oxidising agent (c) Nitrogen oxides from nitrogen and oxygen by lightning
than potassium dichromate (d) Evaporation of H2O
(d) Potassium dichromate oxidises a secondary alcohol into
a ketone.
250 Chemistry
7. For the redox reaction 17. In which of the following compounds, the oxidation number
– 2– + 2+ of iodine is fractional ?
MnO + C 2O + H ® Mn + CO 2 + H 2 O
the correct coefficients of the reactants for the balanced (a) IF7 (b) I -
reaction are: (c) IF5 (d) IF3
MnO –
C 2O 2–
H+ 18. In which of the following compounds, nitrogen has an
oxidation state of –1 ?
(a) 2 5 16
(b) 16 5 2 (a) N2O (b) NO -2
(c) 5 16 2 (c) NH2OH (d) N2H4
(d) 2 16 5 19. Which of the following involves transfer of five
8. KMnO4 oxidises oxalic acid in acidic medium. The number of electrons ?
CO2 molecules produced as per the balanced equation is
(a) 10 (b) 8 (a) MnO - ® Mn 2+ (b) CrO 2- ® Cr +

(c) 6 (d) 3
(c) MnO2- ®MnO2 (d) Cr2O2- ® 2Cr +
9. Of the following reactions, only one is a redox reaction.
Identify it 20. Oxidation number of nitrogen in (NH4)2SO4 is
(a) Ca (OH) 2 + 2HCl ® CaCl 2 + 2H 2 O (a) –1/3 (b) –1
(c) +1 (d) –3
(b) BaCl 2 + MgSO ® BaSO + MgCl2 21. Oxidation number of carbon in CH2Cl2 is
(a) –4 (b) +4
(c) 2S 2O 2 – + 2H 2 O ® SO 2– + H +
(c) 0 (d) –2
(d) Cu 2S + 2FeO ® 2Cu + 2Fe + SO 2 22. Which of the following elements has least oxidation number ?
10. The reaction of KMnO4 and HCl results in (a) Ni(CN)4 (b) Ni(CO)4
(a) oxidation of Mn in KMnO4 and production of Cl2 (c) Fe2O3 (d) SF6
(b) reduction of Mn in KMnO4 and production of H2 23. The oxidation number of cobalt in K3[Co(NO2)6] is
(c) oxidation of Mn in KMnO4 and production of H2 (a) 0 (b) +4
(d) reduction of Mn in KMnO4 and production of Cl2 (c) +3 (d) +6
11. The reaction, 24. The brown ring complex is formulated as
[Fe(H2O)5 NO]SO4. The oxidation number of iron is
2H 2 O(l) ® 4H + (aq) + O 2 (g) + 4e - is (a) 1 (b) 2
(a) an oxidation reaction (c) 3 (d) 0
(b) a reduction reaction 25. In which of the following complexes, oxidation state of metal
(c) a redox reaction is zero ?
(d) a hydrolysis reaction (a) [Pt(NH3)2Cl2] (b) [Cr(CO)6]
12. The chemical that undergoes self oxidation and self reduction (c) [Cr(NH3)3Cl3] (d) [Cr(en)2Cl2]
in the same reaction is 26. The oxidation state of chromium in potassium dichromate
(a) benzyl alcohol (b) acetone (K2Cr2O7) is
(c) formaldehyde (d) acetic acid (a) –5 (b) +6
13. In which of the following pairs, there is greatest difference in (c) +2 (d) –2
the oxidation number of the underlined elements ?
27. The oxidation state of osmium (Os) in OsO4 is
(a) NO 2 and N 2 O (b) P 2 O and P O10 (a) +7 (b) +6
(c) +4 (d) +8
(c) N 2 O and N O (d) SO2 and SO3
28. In H2O2, the oxidation state of oxygen is
14. Phosphorus has the oxidation state of +3 in (a) –2 (b) –1
(a) phosphorous acid (c) 0 (d) –4
(b) orthophosphoric acid 29. A, B and C are three elements forming a part of compound in
(c) hypophosphorous acid
oxidation states of +2, +5 and –2 respectively. What could
(d) metaphosphoric acid
be the compound ?
15. In which of the compounds does 'manganese' exhibit highest
(a) A2(BC)2 (b) A2(BC4)3
oxidation number ?
(c) A3(BC4)2 (d) ABC
(a) MnO2 (b) Mn 3O4
30. Among the following, identify the species with an atom in
(c) K2MnO4 (d) MnSO4
+6 oxidation state
16. One of the following has both positive and negative oxidation
states (a) MnO – (b) Cr(CN) –

(a) F (b) Cl
(c) He (d) Na (c) NiF2 – (d) CrO 2 Cl 2
Redox Reactions 251
31. On reduction of KMnO4 by oxalic acid in acidic medium, the 44. The reaction in which hydrogen peroxide acts as a reducing
oxidation number of Mn changes. What is the magnitude of agent is
this change? (a) PbS + H 2 O 2 ® PbSO + H 2 O
(a) From 7 to 2 (b) From 6 to 2
(c) From 5 to 2 (d) From 7 to 4 (b) 2KI + H 2O2 ® 2KOH + I 2
32. The oxidation number of iron in Fe3O4 is (c) 2 FeSO 4 + H 2SO 4 + H 2 O 2 ®
(a) +2 (b) +3
(c) 8/3 (d) 2/3 Fe 2 (SO 4 ) 3 + 2H 2 O
33. The oxidation number of S in H2S2O8 is (d) Ag 2 O + H 2 O 2 ® 2Ag + H 2 O + O 2
(a) +2 (b) +4
45. HNO3 acts as
(c) +6 (d) +7
(a) acid (b) oxidising agent
34. When KMnO4 acts as an oxidising agent and ultimately forms
(c) reducing agent (d) Both (a) and (b)
MnO –2 , MnO2, Mn 2O3 and Mn+2, then the number of 46. When KMnO4 reacts with acidified FeSO4
electrons transferred in each case respectively is (a) FeSO4 is oxidised and KMnO4 is reduced
(a) 4, 3, 1, 5 (b) 1, 5, 3, 7 (b) only KMnO4 is oxidised
(c) 1, 3, 4, 5 (d) 3, 5, 7, 1 (c) only FeSO4 is oxidised
35. The oxidation number of sulphur in Na2S4O6 is (d) None of these
(a) 1.5 (b) 2.5 47. Formula weight divided by the change in oxidation number
(c) 3 (d) 2 gives
36. In which of the following reactions, there is no change in (a) equivalent weight of an oxidant
valency ? (b) equivalent weight of the reductant
(a) 4KClO3 ® 3KClO4 + KCl (c) the number of electrons gained in the reaction
(b) SO2 + 2 H2S ® 2H2O + 3 S (d) the equivalent weight of the oxidant or reductant
(c) BaO2 + H2SO4 ® BaSO4 + H2O2 48. The equivalent weight of Mohr’s salt,
(d) 2 BaO + O2 ® 2 BaO2 FeSO ( NH ) 2 SO . H 2 O is equal to
37. What is the coefficient of oxalate ion in the following (a) its molecular weight
reaction ? (b) its atomic weight
MnO 4– + C 2 O 24 – + H + ® Mn 2+ + CO2 + H 2O (c) half-its molecular weight
(d) one-third its molecular weight
(a) 4 (b) 2
49. In the reaction between SO2 and O3 the equivalent weight of
(c) 3 (d) 5
ozone is
38. The compound that can work both as an oxidising as well as
(a) the same as its molecular weight
a reducing agent is
(b) half of the molecular weight
(a) KMnO4 (b) H2SO4
(c) one-third of the molecular weight
(c) BaO2 (d) H2O2
(d) one-fourth of the molecular weight
39. When SO2 is passed through the solution of potassium
50. Equivalent weight of iron in Fe2O3 would be
iodate, the oxidation state of iodine changes from
(a) 28 (b) 56
(a) + 5 to 0 (b) + 5 to – 1
(c) 18.6 (d) 112
(c) – 5 to 0 (d) – 7 to – 1
51. Atomic number of an element is 22. The highest O.S. exhibited
40. Which of the following behaves as both oxidising and
by it in its compounds is
reducing agents ?
(a) 1 (b) 2
(a) H2SO4 (b) SO2
(c) 3 (d) 4
(c) H2O (d) HNO3
52. The equivalent weight of potassium permaganate in acid
41. In the reaction
solution is
2FeCl 3 + H 2S ® 2 FeCl 2 + 2 HCl + S (a) 158 (b) 31.6
(a) FeCl3 acts as an oxidising agent (c) 52.16 (d) 79
(b) Both H2S are FeCl3 are oxidised 53. The equivalent weight of phosphoric acid (H3PO4) in the
(c) FeCl3 is oxidised while H2S is reduced reaction:
(d) H2S acts as an oxidising agent. NaOH + H PO ® NaH 2 PO + H 2 O
42. Which of the following is not a reducing agent?
(a) 59 (b) 49
(a) SO2 (b) H2O2
(c) 25 (d) 98
(c) CO2 (d) NO2
54. Equivalent mass of oxidising agent in the reaction,
43. Number of moles of K2Cr2O7 reduced by one mole of Sn2+
ions is SO 2 + 2H 2S ® S + 2H 2 O is
(a) 1/3 (b) 3 (a) 32 (b) 64
(c) 1/6 (d) 6 (c) 16 (d) 8
252 Chemistry
55. In the chemical reaction, 57. The set of numerical coefficients that balances the equation
K 2 Cr2 O + X H 2 SO + YSO 2 ® K 2 CrO + HCl ® K 2 Cr2 O + KCl + H 2 O
K 2SO4 + Cr2 (SO4 )3 + Z H 2O (a) 1, 1, 2, 2, 1 (b) 2, 2, 1, 1, 1
X, Y and Z are (c) 2, 1, 1, 2, 1 (d) 2, 2, 1, 2, 1
(a) 1, 3, 1 (b) 4, 1, 4
58. (X) MnO 4– + (Y)H 2O2 ®
(c) 3, 2, 3 (d) 2, 1, 2
56. Consider the following reaction
2Mn +2 + 5H 2 O + 9O 2 + (Z)e-
– –
5H 2 O2 + X ClO2 + 2OH ® X Cl + Y O2 + 6H 2O
In this reaction, value of (X), (Y) and (Z) are
The reaction is balanced if (a) 2, 5, 6 (b) 5, 2, 9
(a) X = 5, Y = 2 (b) X = 2, Y = 5 (c) 3, 5, 5 (d) 2, 6, 6
(c) X = 4, Y = 10 (d) X = 5, Y = 5

1. The oxidation number of phosphorus in pyrophosphoric 7. Which of the following chemical reactions depict the
acid is [CBSE-PMT 1999] oxidising beahviour of H2SO4? [AIEEE 2006]
(a) +3 (b) +1 (a) NaCl + H 2 SO 4 ¾¾® NaHSO 4 + HCl
(c) +4 (d) +5 (b) 2PCl 5 + H 2SO 4 ¾
¾® 2 POCl 3 + 2HCl + SO 2 Cl 2
2. The oxidation states of sulphur in the anions SO32–, S2O42–
and S2O62– follow the order [CBSE-PMT 2003] (c) 2 HI + H 2 SO 4 ¾
¾® I 2 + SO 2 + 2 H 2 O
(d) Ca (OH ) 2 + H 2SO 4 ¾
¾® CaSO 4 + 2 H 2 O
(a) S 2 O 6 2 - < S 2 O 4 2- < SO 3 2-
(b) S2 O 4 2 - < SO 3 2- < S2 O 6 2- 8. What products are expected from the disproportionation
reaction of hypochlorous acid? [AIEEE 2006]
(c) SO32- < S2 O4 2- < S2O 6 2- (a) HCl and Cl2O (b) HCl and HClO3
(d) S2 O 4 2 - < S 2 O 6 2- < SO 3 2- (c) HClO3 and Cl2O (d) HClO2 and HClO4
9. Consider the following reaction :
3. Oxidation numbers of P in PO3– 2–
4 , of S in SO 4 and that of
z
Cr in Cr2 O 72– are respectively [CBSE-PMT 2009] xMNO4 - + yC2O42- + zH + ® xMn 2+ + 2yCO2 + H 2O
2
(a) + 3, + 6 and + 5 (b) + 5, + 3 and + 6
The value’s of x, y and z in the reaction are, respectively :
(c) – 3, + 6 and + 6 (d) + 5, + 6 and + 6
[JEE M 2013]
4. When Cl2 gas reacts with hot and concentrated sodium
hydroxide solution, the oxidation number of chlorine (a) 5, 2 and 16
changes from : [CBSE-PMT 2012 S] (b) 2, 5 and 8
(a) zero to +1 and zero to –5 (c) 2, 5 and 16
(b) zero to –1 and zero to +5 (d) 5, 2 and 8
(c) zero to –1 and zero to +3 10. The compound YBa 2 Cu 3 O 7 which shows
(d) zero to +1 and zero to –3 superconductivity has copper in oxidation state ..........
5. Which of the following is a redox reaction? [AIEEE 2002] Assume that the rare earth element Yttrium is in its usual +3
(a) NaCl + KNO3 ® NaNO3 + KCl oxidation state
(a) 3/7 (b) 7/3 [IIT 1994]
(b) CaC2O4 + 2HCl ® CaCl2 + H2C2O4
(c) 3 (d) 7
(c) Mg(OH)2 + 2NH4Cl ® MgCl2 + 2NH4OH
11. For H3PO3 and H3PO4 the correct choice is
(d) Zn + 2AgCN ® 2Ag + Zn(CN)2
[IIT SCREENING 2003]
6. The oxidation state of chromium in the final product formed
(a) H3PO3 is dibasic and reducing
by the reaction between KI and acidified potassium
dichromate solution is: [AIEEE 2005] (b) H3PO3 is dibasic and non-reducing
(a) + 3 (b) + 2 (c) H3PO4 is tribasic and reducing
(c) + 6 (d) + 4 (d) H3PO3 is tribasic and non-reducing
Redox Reactions 253

1. The oxidation number of sulphur in S 8, S 2F 2 , H2 S 8. In the following reaction


respectively, are
(a) 0, +1 and –2 4P + 3KOH + 3H 2 O ¾
¾® 3KH 2 PO 2 + PH 3
(b +2, +1 and –2 (a) only phosphorus is oxidised and reduced.
(c) 0, +1 and +2
(b) only phosphorus is reduced.
(d) –2, +1 and –2
(c) phosphorus is both oxidised and reduced.
2. When SO2 is passed through acidified solution of potassium
dichromate, then chromium sulphate is formed. The change (d) phosphorus is neither oxidised nor reduced.
in valency of chromium is 9. The following acts as a reducing as well as oxidising agent
(a) +4 to +2 (b) +5 to +3
(a) O3 (b) ClO-4
(c) +6 to +3 (d) +7 to +2
3. One mole of N2H4 loses 10 moles of electrons to form a new
compound, y. Assuming that all nitrogen appear in the new (c) F2 (d) MnO-4
compound, what is the oxidation state of nitrogen in y (There 10. Oxidation state for nitrogen is incorrectly given for
is no change in the oxidation state of hydrogen ) compound oxidation state
(a) –1 (b) –3
(c) +3 (d) +5 (a) [Co( NH 3 )5 Cl]Cl 2 0

4. In the reaction : C + 4HNO 3 ® CO 2 + 2H 2 O + 4NO 2 (b) NH2OH –1


HNO3 act as (c) (N2H5)2SO4 –2
(a) an oxidizing agent (d) Mg3N2 –3
(b) an acid 11. The most powerful oxidizing agent from the following is
(c) an acid as well as oxidizing agent
(a) H3BO3 (b) HPO3
(d) a reducing agent.
5. Which of the following reaction involves neither oxidation (c) H3PO4 (d) H2SO4
nor reduction 12. The species that undergoes disproportionation in an alkaline
medium are
(a) CrO24- ® Cr2O72- (b) Cr ® CrCl3
(a) Cl2 (b) MnO24 -
(c) Na ® Na + (d) 2S 2 O 32 - ® S4 O 26 -
6. A compound of Xe and F is found to have 53.5% of Xe. (c) NO2 (d) All of these
What is oxidation number of Xe in this compound ? 13. Which of the following reactions will not take place
(a) –4 (b) 0
(a) Fe + H 2SO 4 ® FeSO 4 + H 2
(c) +4 (d) +6
7. In the standardization of Na 2S 2O3 using K2Cr 2O7 by (b) Cu + 2AgNO3 ® Cu(NO3)2 + 2Ag
iodometry, the equivalent weight of K2Cr2O7 is
(a) (molecular weight)/2 (c) 2KBr + I 2 ® 2KI + Br2
(b) (molecular weight)/6
(d) CuO + H 2 ® Cu + H 2O
(c) (molecular weight)/3
(d) same as molecular weight
254 Chemistry

EXERCISE 1 8. (a) 10 moles of CO2 are produced.


9. (d) In redox reaction oxidation and reduction take place side
1. At anode, oxidation takes place whereas at cathode,
reduction takes place. by side. Cu 2S + 2FeO ® 2Cu + 2Fe + SO 2 .
2. Cu2S oxidised whereas O2 reduced. O.N. of Cu changes from +1 to 0 (reduction) and O.N. of
3. Cr = + 6, S = + 2 S changes from –2 to +4 (oxidation).
4. Fe has lower reduction potential and Cu has higher reduction +7 +2
potential than H2. 10. (d) 2KMnO 4 + 16HCl ® 2KCl + 2MnCl 2 + 8H 2O + 5Cl 2
5. HI, I2, ICl is order of increasing oxidation number. Mn (O.N. changes from +7 to +2 gain of electrons) reduced
6. SO 24 - ion as it does not participate in the reaction. and Cl2 formed.
11. (d) Since there is loss of electrons, hence it is oxidation
8. In MnO4- , Mn is in the highest oxidation state i.e. +7. reaction.
12. (c) In Cannizaro’s reaction
9. (i) 2CrO3- + H 2 O 2 ¾¾
® 2CrO 4-2 + H 2 O
(2HCHO + KOH ® CH 3OH + HCOOK )
(ii) IO3- + 5I- + 6H+ ¾¾ ® 3I2 + 3H2O
formaldehyde is reduced as well as oxidised.
(iii) 5C2 O 4-2 + 2MnO 4- + 16H + ¾¾ ® 10CO 2 13. (d) O.N. of N in NO2 and N2O4 is +4 \ difference is
+2Mn +2 + 8H 2 O zero.
10. (i) No, because E o < Eo O.N. of P in P2O5 and P4O10 is +5 \ difference is
Zn 2 + / Zn Cu 2 + / Cu zero
(ii) No, O.N. of N in N2O is +1 and in NO is +2. The difference is 1
(iii) Yes, as Eo < Eo O.N. of S in SO2 is +4 and in SO3 is +6. The difference is +2.
Cu 2+ /Cu Ag+ / Ag
14. (a) H3PO3 phosphorous acid O.N. of P is +3. orthophosphoric
(iv) Yes,
12. E < C < A < H < D < B. +5 +1
acid H 3 PO 4 , Hypophosphorous acid H3PO 2
16. (d) 17. (a) 18. (a) 19. (b) 20. (b)
21. (a) 22. (d) 23. (c) 24. (a) 25. (a) +5
Metaphosphoric acid HPO3
EXERCISE 2
15. (c) O.N. of Mn in K2MnO4 is +6 (find O.N. of Mn in others)
1. (b) O.N. of I- is –1 and in I2 O.N. is zero (loss of electrons).
16. (b) Cl has O. S. as –1, +1 +3 +5 and +7
Hence I- oxidised. 17. (b) O.N. of iodine in I3- is –1/3
2. (b) SO2 oxidises H2S to S, since the O.N. of S change from
–2 to 0
H +1
3. (b) MnO4- +
+ 8H + 5e ® Mn 2+
+ 4H 2 O \ –2
18. (c) N – O – 2 – H +1
+1
Cr2 O 27 – + 14H + + 6e - ® 2Cr 3+ + 7H 2 O . As the gain /
H +1
of electrons per mole is more in case of Cr2O 2 – then O.N. of N is NH2OH is –1
MnO 4 hence it is stronger oxidising than MnO – 19. (a) O.N. of Mn in MnO – is +7 and in Mn 2+ it is +2. The
4. (b) (Ni ® Ni2+) Ni loses electrons hence act as reducing difference is of 5 electrons.
agent. 20. (d) (NH4)2 SO4 is split into ions NH +4 . Let O.N. of N be x
5. (d) 3Br2 + 6CO32 – + 3H 2 O ® 5Br – + BrO3– + 6HCO 3– then, 1 × (x) + 4x (+1) = 1 \ x = –3
O.N. of Br2 changes from 0 to –1and +5 hence it is reduced
H +1
as well as oxidised. |
+1
6. (c) N2 + O2 ® 2NO 21. (c) Find O.N. by chemical bond method H – C 22-+ – Cl -1
O.N. of N changes from 0 to +2 (oxidation) and O.N. of O |
changes from 0 to –2 (reduction). Cl - 1
7. (a) The balanced equation is
O.N. of C is zero
2MnO 4– + 5C 2 O 24 – + 16H + ® 2Mn 2 + + 10CO 2 + 8H 2 O
Redox Reactions 255
22. (b) In metal carbonyls metal always has O.N. zero.
23. (c) K3[Co(NO2)6] Let O.N. of Co be x then 43. (a) (Cr O
2
2–
7 + 14H + + 6e ¾
¾® 2Cr 3+ + 7H 2 O )13
3x(+1) + 1×(x) + 6x (–1) = 0 \ x = +3
24. (a) [Fe(H2O)5NO] SO4 Let O.N. of Fe be x then, Sn 2+ - 2e - ¾
¾® Sn 4+
1× (x) + 5 × (0) + 1× (+1) + 1× (–2) = 0 \ x = +1
1 14 +
25. (b) Cr(CO)6 is metal carbonyl, hence O.N. of chromium is Hence Cr2O72–+ H + Sn 2+ ¾¾
®
zero. 3 3
26. (b) K2Cr2O7 O.N. of Cr is +6. 2 3+ 7
Cr + H 2 O + Sn 4 +
27. (d) OsO4 Let O.N. of Os be x then 1×(x) + 4(–2) = 0 3 3
\x = 8 44. (d) In Ag2O (O.N. of Ag +1) in Ag the O.N. is 0. There is gain
28. (b) H – O – O – H.O.N. of oxygen is –1. of electrons, hence H2O2 is reducing.
29. (c) Put values of O.S. of A, B and C in all the given 45. (d) In HNO3 O.N. of N is +5 which is maximum value hence it
compounds. The compound giving sum of O.S. as zero can act as oxidising agent and as an acid only.
will be the real compound. For A 3 (BC 4 ) 2 +7 +2
= 3 × (+2) + 2 × (+5) + 8 × (–2) = 0 46. (a) 2K Mn O 4 + 3H 2SO 4 ® K 2SO 4 + 2 Mn SO 4 + 3H 2 O + 5O
+2 +3
30. (d) MnO – (O.S. of Mn +7); Cr(CN)3– 2 Fe SO 4 + H 2SO 4 + O ® Fe 2 (SO 4 ) 3 + H 2O
6 (O.S. of Cr +3),
O.N. of Mn changes from +7 to +2 (Reduction)
NiF62 – (O.S. of Ni +4) and CrO2Cl2 (O.S. of Cr +6) O.N. of Fe changes from +2 to +3 (Oxidation)

31. (a) In acid medium MnO changes to Mn2+ , hence O.N. Molecular weight
47. (d) Eq.wt =
changes from +7 to +2. Change in O.N.
32. (c) In fact Fe3O4 is a mixed oxide of FeO and Fe2O3. O.N. of 48. (a) FeSO4 is oxidised to Fe2(SO4)3, change in O.N. of Fe is
Fe in FeO is +2 and O.N. of Fe in Fe2O3 is +3. The average by 1. Hence equivalent weight of Mohr's salt is M/1 = M.
O.N. of Fe will be 3× (x) + 4 × (–2) = 0 49. (b) SO 2 + O ® SO
\ x = 8/3 O.N. of S changes from +4 to +6. Two electron change
33. (c) O.N. of S in H2S2O8 is + 6. See text and find O.N. by \ Eq. Wt = M/2.
chemical bond method.
50. (c) Fe 2 O ® 2Fe
34. (c) In KMnO4 the O.N. of Mn is +7, in MnO –2 +6, in MnO2 The change in O.N. of Fe is from +3 to 0. Hence equivalent
+4 in Mn2O3 +3 and in Mn2+, it is +2. The difference
being 1, 3, 4 and 5 respectively. weight of iron = 1 ´ 56 = 18.6
3
35. (b) Na2S4O6 Let O.N. of S be x then
51. (d) The element is Ti (At. no. 22). Electronic configuration is
2 × (+1) + 4 × (x) + 6 × (–2) = 0 1s2, 2s2p6, 3s2p6d2, 4s2. the energy level of 3d and 4s is
\ x = 2.5. By chemical bonding method the two very close. It can have Ti4+ O.S.
S, atoms have O.N. + 5 and two S atoms have O.N. zero. 52. (b) (158/5 = 31.6),
36. (c) In BaO 2 + H 2SO 4 ® Ba SO 4 + H 2 O 2 all atoms are 53. (d) NaOH + H 3 PO 4 ® NaH 2 PO 4 + H 2 O
present in the same O.S. in reactants and products. Here 1 gev. of NaOH is reacting with 1 gev of H3PO4
37. (d) The balanced equation is hence Eq. wt of H3PO4 is 98.
54. (c) SO 2 + 2H 2S ® S + 2 H 2 O . SO2 is oxidising and change
2MnO 4– + 5C 2 O 24 – + 16H + ® 2Mn 2 + + 10CO 2 + 8H 2 O
in O.N. of S is from +4 to zero.
Hence coefficient of oxalate ion is 5
38. (d) In H2O2 the O.N. of O is –1 which can be increased to 0 or 64
\ Eq. Wt. of SO2 = = 16
decresed to –2 hence H2O2 can work as reducting and 4
oxidising agent. 55. (a) K 2 Cr2 O7 + H 2 SO 4 + 3SO 2 ®
X =1 Y =3
39. (b) O.S. of I, 3KIO3 + 3SO 2 ® 3KI+ 3SO 3
+5 -1 K 2 SO 4 + Cr2 (SO4 )3 + H 2O
40. (b) In SO2 the O.N. of S can increase and decrease. Hence Z =1
can behave as reducing and oxidising agent. Oxidation 56. (b) 5H O + 2ClO + 2OH – ® 2Cl – + 5O + 6H O
2 2 2 2 2
state of S varies from –2 to 6. X =2 Y =5
41. (a) In 2FeCl 3 + H 2S ® 2FeCl 2 + 2HCl + S 57. (d) 2K 2 CrO4 + 2HCl ® K 2Cr2O 7 + 2KCl + H 2 O
O.N. of S changes from –2 to 0 (hence oxidised) O.N. of Coefficients are 2, 2, 1, 2, 1
Fe changes from +3 to +2 hence reduced. 58. (a) 2MnO 4– + 5H 2 O2 ® 2Mn 2 + + 5H 2O + 9O2 + 6e -
42. (c) In CO2, O.N. of C is +4 which is maximum. So it can act as
oxidising agent and not as reducing agent. X = 2, Y = 5 and Z = 6
256 Chemistry

EXERCISE 3 H
|
1. (d) Pyrophosphoric acid H4P2O7 11. (a) H - O - P - OH , hence it is dibasic. It acts as reducing
Let oxidation state of phosphorus is x
(4 × 1 + (– 2) × 7 + 2 x) = 0 O
\ 2x = 10 or x = +5 agent also.
2. (c) SO 32 - ® S is in + 4 oxidation state EXERCISE 4
S2 O 24- ® S is in + 3 oxidation state 1. (a) ON of S in S8 = 0
ON of S in S2F2 = + 1
S2 O 62- ® S is in + 5 oxidation state ON of S in H2S = -2
2. (c) K2Cr2O7 + 3SO2 + 4H2SO4 ®
3. (d) PO3–
4 = x + 4 (– 2) = – 3; x – 8 = – 3; x = + 5
K 2SO 4 + Cr2 (SO 4 )3 + 3SO 3 + 4H 2 O
SO 2– = x + 4 (– 2) = – 2; x – 8 = – 2; x = + 6 O.N.of chromium changes from +6 to +3
4
loss of 10 e
3. (c) N 2-4 H 4+4 ¾¾¾¾¾ ® N 2+6 (y);
Cr2 O72– = 2x + 7 (– 2) = – 2; 2x – 14 = – 2; N
2x =12; x = + 6 O.N.of N changes from -2 to +3
4. (b) On reaction with hot and concentrated alkali a mixture 4. (a) O.N. of C changes form 0 to + 4 by oxidation. Hence
of chloride and chlorate is formed HNO3 is oxidising agent.
Hot
3Cl2 + 3 NaOH(excess) ¾¾¾ 5. (a) CrO24 - ® Cr2 O72– O.N of Cr does not change . It remains
®
+6 on both sides . Hence there is no oxidation or reduction.
-1 +5
5NaCl + NaClO3 + 3H 2O 6. (d) Xe = 53.5 % \ F = 46.5%
Relative number of atoms Xe
5. (d) The oxidation states show a change only in
reaction (d). 53.5 46.5
= = 0.4 and F = = 2.4
131.2 19
0 +1 0 +2
-2e - Simple ratio Xe = 1 and F = 6 ; Molecular formula is XeF6
Zn + 2A gCN ¾¾¾® 2Ag + Zn(CN) 2
+2e - O.N.of Xe is +6
7. (b) In iodometry, K2Cr2O7 liberates I2 from iodides (NaI or
2- - +
6. (a) Cr2O7 + 6I + 14H ¾¾ ® 2Cr3+ +3I2 +7H 2O KI). Which is titrated with Na2S2O3 solution.
+ 3 oxidation state of Cr. K 2Cr2O7 + I – + H + ¾¾
® Cr3+ + I2
+6 +4 Here, one mole of K2Cr2O7 accepts 6 mole of electrons.
7. (c) 2HI -1 + H 2SO 4 ¾ ¾® I 02 + SO 2 + 2H 2 O molecular weight
In this reaction O.N. of S is decreasing from + 6 to +4 \Equivalent weight =
6
hence undergoing reduction and O.N. of I is increases
from –1 to 0 hence undergoing oxidation therefore H2SO4 8. (c) 4P + 3KOH + 3H 2 O ® KH 2 PO 2 + PH 3
is acting as oxidising agent. O.N of P = 0, In KH2PO2 it is + 1, In PH3 it is –3. Hence P
8. (b) During disproportionation same compound undergo is oxidised and reduced
simultaneously oxidation and reduction. 9. (a) Ozone is reducing as well as oxidising is nature
+1
Oxidation 10. (a) [Co( NH 3 ) 5 Cl.]Cl 2 , H –1 –2 +1
N — O — H,
–3 +1H
- + -1 +5
3HO Cl ¾
¾® 2HCl+ HClO 3 reduction H .. H
H N-N , Mg3N2
Reduction H H
– 1, – 2, –3
9. (c) On balancing the given equations, we get
2MnO 4- + 5C 2 O 4 2- + 16H + ¾¾
® 2Mn ++ 11. (d) In H2SO4 , sulphur is in highest oxidation state (+6),
Hence H2SO4 will be strongest by oxidising agent.
+10CO 2 + 8H 2O -1 +1
So, x = 2, y = 5 & z = 16 12. (d) Cl0 + 2NaOH ® NaCl+ NaClO+ H O
2 2
10. (b) YBa2Cu3O7
+6 +7 +4
3 + 2 × 2 + 3x – (2 × 7) = 0 3M nO-- - –
4 + 2H 2O ® 2 Mn O 4 + M nO 2 + 4OH
3 + 4 + 3x – 14 = 0
3x = 7 +4 +5 +3
2 N O 2 + H 2 O ® HN O 3 + H NO 2
7
x= . All undergo disproportionation
3 13. (c) I2 is less electronegative than Br2
9
Hydrogen
GENERAL INTRODUCTION : MANUFACTURE OF HYDROGEN :
Symbol of hydrogen is H. Electronic Configuration 1s1. Hydrogen i) From water gas (Bosch process) : By passing water gas
is the lightest and most abundant element in the universe. It was mixed with steam over heated catalyst Fe3O4 and Cr2O3 at
first prepared by Sir Henry Cavendish by the action of sulphuric 450ºC.
acid on Zinc and named by Antoine Lavoisier since it produced
Re d Hot
water on burning. H 2 O(Steam) ¾¾ ¾¾® (CO + H 2 )
Coke
water gas
(Greek : hydra= water, gennas = maker or producing)
PREPARATION OF DIHYDROGEN : Fe O , Cr O
(CO + H 2 ) + H 2 O ¾¾2¾3¾ ¾
2¾3 ® CO + 2 H
450 º C 2 2
i) From cold water : By the action of Na, K, Ca etc.
2 Na + 2H 2 O ® 2 NaOH + H 2 (highly exothermic and H2 The CO2 is removed by dissolving in water under high
pressure.
cathces fire)
ii) By Lane’s process : The superheated steam is passed over
Al-Hg and Zn-Cu couple decompose water to give nascent
heated iron at 600 - 800ºC
hydrogen
Zn + 2H 2 O ® Zn(OH) 2 + 2H 3 Fe + 4H 2 O Fe3O 4 + 4H 2 ­

Hence Couples constitute better reducing agents Iron is regenerated by passing water gas. (H2 + CO)
ii) From hot water : By the action of Mg, Zn, Al etc. Fe 3O 4 + 4CO ® 3Fe + 4CO 2
2Al + 3H 2 O ® Al 2 O 3 + 3H 2 Fe 3O 4 + 4H 2 ® 3Fe + 4H 2 O
iii) From steam : By the action of Fe, Sn etc. In actual practice steam is passed over hot iron for 10 - 15
3Fe + 4H 2 O ® Fe3O 4 + 4H 2 minutes and then water gas is blown over heated oxide for
25 - 30 minutes.
iv) From water : By the action of metallic hydrides of alkali and iii) By electrolysis of water : By the electrolysis of acidified or
alkaline earth metals. alkaline water.
CaH 2 + 2H 2 O ® Ca (OH ) 2 + H 2
H 2O H + + OH -
v) From acids : By the action of dilute acids on Zn, Mg, Fe,
etc. placed above hydrogen in electrochemical series. At cathode 2H + + 2e - ® H 2
Sn + 2HCl ® SnCl 2 + H 2
At anode 4OH - - 4e - ® 4OH ® 2H 2 O + O 2
Zn + H 2SO 4 ® ZnSO 4 + H 2
iv) As by product : When NaOH is manufactured by electrolysis
vi) From alkalies : By the action Zn, Al, Sn etc. of NaCl in Nelson or Castner Kellner cell hydrogen is
Zn + 2NaOH ® Na 2 ZnO 2 + H 2 obtained as by product.
2Al + 2 NaOH + 2 H 2 O ® 2 NaAlO2 + 3 H 2 2 NaCl ® 2 Na + + Cl -
258 Chemistry

2 Cl - - 2e - ® Cl 2 At anode APPLICATIONS OF HYDROGEN :


i) Manufacture of methyl alcohol :
2H 2 O + 2e - ® H 2 + 2OH -
+ -
2 Na + 2OH ® 2 NaOH
} At Cathode ZnO, Cr O
CO + 2H 2 ¾¾¾¾¾
2 3 ® CH OH
3
300ºC, 200 Atm
v) From hydrocarbons : By cracking of hydrocarbons
Pt / Al O
C 6 H12 ¾¾ ¾2¾
¾3 ® C H + 3H ii) Manufacture of ammonia :
6 6
10-50 Atm. 2
Fe / Mo
N 2 + 3H 2 ¾¾ ¾
¾® 2 NH 3
PURE HYDROGEN METHOD OF PREPARATION : 490º C, 210 Atm
i) By Uyeno’s method : By the action of Caustic Soda on
iii) Synthetic petrol : Fisher-Tropsch process
aluminium.
2 NaOH + 2Al + 2H 2 O ® 2 NaAlO 2 + 3H 2 a) n (CO + H 2 ) + nH 2 ® C n H 2n + nH 2 O
olefin
ii) By the action of NaH on water
NaH + H 2 O ® NaOH + H 2 b) nCO + ( 2n + 1) H 2 ® C n H 2n + 2 + nH 2 O
paraffin
iii) By treating pure Mg or Al with chemically pure H2SO4 (dil) iv) Manufacture of vegetable ghee : By hydrogenation of oils
or HCl in presence of Ni.
Mg + H 2SO 4 ® MgSO 4 + H 2 v) To produce low-temperature : It is used as Cryogenic fluid.
PHYSICAL PROPERTIES : vi) Oxy-hydrogen flame : It produces temperature of 2850ºC
and oxy-atomic hydrogen flame produces a temperature of
Hydrogen is tasteless, odourless, colourless gas. It is non
4000ºC.
poisonous but presence of AsH3 makes it poisonous. Its critical
temperature is –234.5ºC which makes its liquefaction difficult. It vii) Mixed with Helium it is used for filling balloons.
is slightly soluble in water. FORMS OF HYDROGEN :
ISOTOPES OF HYDROGEN : i) Atomic Hydrogen :
Hydrogen has three isotopes Electric arc
H 2 ¾¾¾¾¾
®2H DH = 105.4 kcal mol-1
Name 1 2 3
1H 1H or D 1H or T It is very reactive and its half life period is 0.33 seconds.
Protium Deuterium Tritium ii) Occluded Hydrogen : Hydrogen adsorbed by certain metals
Abundance 99.985% 0.016% Traces (10–15 %) eg. Pt, Pd, Fe, Ni etc is known as occluded hydrogen. One
Deuterium is known as heavy hydrogen. Tritium is formed in volume of finely divided metals adsorb the following
upper atmosphere and is b-radioactive. volumes of hydrogen.
3 0 3
2 He + -1e ® 1 H (t1/ 2 = 12.34 year) Palladium black 870, Platinum 49.5; Gold 46.3, Iron 15.7,
14 Copper 4.5, Aluminium 2.7.
7N ®6 C12 +1 H 3 (formation)
iii) Nascent hydrogen : Freshly prepared hydrogen is known
CHEMICAL PROPERTIES : as nascent hydrogen and is more reactive than ordinary
Its chemical properties are hydrogen. It causes the reduction of certain compounds
i) Reaction with non metals : which is not possible with ordinary hydrogen.

H 2 + X 2 ® 2HX(X = F, Cl, Br, I) Zn + H 2SO 4 ® ZnSO 4 + 2H


FeCl 3 + H ® FeCl 2 + HCl
2H 2 + O 2 ® 2H 2 O
KClO 3 + 6H ® KCl + 3H 2 O
3H 2 + N 2 ® 2 NH 3
iv) Ortho and Para hydrogen : The nucleus of the hydrogen
ii) Reaction with metals : 2 Na + H 2 ® NaH atom also spins like a top. When in hydrogen molecule, the
nuclear spins are in the same direction it is known as ortho
Ca + H 2 ® CaH 2 (Hydrolith) hydrogen and when the nuclear spins are in the opposite
Metals like Fe, Ni and Pd form interstitial or metallic hydrides. direction it is known as para hydrogen. The two electrons in
iii) Reducing property : a hydrogen molecule always spin in opposite direction. At
room temperature hydrogen consists of 75% ortho and 25%
CuO + H 2 ® Cu + H 2 O para. At low temperature more para is present.
Ag 2 O + H 2 ® 2Ag + H 2 O
Fe 3 O 4 + 4 H 2 ® 3Fe + 4 H 2 O P P P P
Ni Ortho-hydrogen Para-hydrogen
iv) Hydrogenation : C n H 2 n + H 2 ¾¾® C n H 2n + 2 (Parallel nuclear spin) (Anti parallel nuclear spin)
Hydrogen 259
1 1 ii) Merck process : By passing CO2 through a suspension of
Nuclear spin = + =1 (For ortho-hydrogen) BaO2 in ice cold water.
2 2
1 æ 1ö BaO 2 + H 2 O + CO 2 ® BaCO 3 + H 2 O 2
Nuclear spin = + ç- ÷ = 0 (For para-hydrogen)
2 è 2ø Manufacture :
v) Transportation of hydrogen : It is transported in the form 1) By electrolysis of 50% ice cold H2SO4
of Hydrolith (CaH2) or ammonia (NH3). Ammonia is cracked 2 H 2SO 4 H 2 + H 2 S2 O 8
by passing over heated catalysts yielding a mixture of Persulphuric acid
hydrogen (75%) and N2 (25%). ( Marshall's acid )
2 NH 3 ® 3H 2 + N 2
H 2S2 O 8 + H 2 O ® H 2SO 4 + H 2SO 5
HYDRIDES : Permonosulphuric acid
Binary compounds of hydrogen and other elements are called ( Caro's Acid )
hydrides. Hydrides are classified into the following four classes.
H 2SO 5 + H 2 O ® H 2SO 4 + H 2 O 2
1) Saline or ionic hydrides.
2) Molecular or covalent hydrides. Mechanism : H 2SO 4 H + + HSO -4
3) Metallic or intertitial hydrides.
4) Polymeric hydrides. At anode : 2 HSO -4 ® H 2S 2 O 8 + 2e -
1. Saline or ionic hydrides : These are formed by elements of
group 1, 2 (Except Be and Mg) and lanthanides by heating At cathode : 2 H + + 2e - ® H 2
the metal in hydrogen.
2) Auto oxidation : Most recent method used in America. In
These are white colourless solids (crystalline) having high
this method the anthraquinone is reduced to anthraquinol
mpt. and bpt. easily decomposed by water, alcohol, CO2 or
by dissolving in an organic solvent and passing hydrogen
SO2.
in presence of Pd. On frothing, the anthraquinol derivative
CaH 2 + 2H 2 O ® Ca (OH) 2 + 2H 2 with air, 20% solution of H2O2 is obtained and anthraquinone
CaH 2 + 2C 2 H 5 OH ® Ca (OC 2 H 5 ) 2 + 2H 2 is regenerated.
O OH
CaH 2 + 2CO 2 ® (HCOO) 2 Ca
C2H5 C2H5
They are strong reducing agents. Alkali metal hydrides are H2/Pd O2

used for making LiAlH4, NaBH4 etc and for removing last
traces of water from organic compound. O OH
2) Molecular or covalent hydrides : These are formed by 4th,
5th, 6th, 7th group elements and boron by sharing electrons O
with hydrogen atoms. eg.: NH3, HCl, B2H6, AsH3 . These C2H5
O2
are non electrolytes and are usually gases or liquids. + H2O2
3) Metallic or interstitial hydrides : The transition elements
and rare earth metals combine with hydrogen to give O
interstitial hydrides. They exhibit metallic properties and 3) By electrolysis of ammonium sulphate solution and
are powerful reducing agents. They are non stoichiometric sulphuric acid. When aqueous solution of ammonium
compounds and their composition varies with temperature sulphate and sulphuric acid in equimolar proportion is
and pressure. eg. LaH2.76, TiH1.73 electrolysed at low temperature ammonium persulphate is
4) Polymeric hydrides : These are solids containing molecules, formed. The latter on distillation with sulphuric acid gives
linked together in two or three dimensions by hydrogen 30% solution of hydrogen peroxide.
bridge bonds. e.g.: (BeH2)n, (MgH2)n and (AlH3)n
( NH 4 ) 2 SO 4 + H 2SO 4 ® 2 NH 4 .HSO 4
HYDROGEN PEROXIDE :
Discovery : French chemist Thenard 1818. NH 4 HSO 4 NH 4SO -4 + H +
Occurrence : Traces in air, rain and plants.
At anode :
PREPARATION :
i) Lab method : From true peroxide by the action of ice cold 2 NH 4SO -4 ® ( NH 4 ) 2 S 2 O8 + 2e -
dil. H2SO4.
At cathode :
Na 2 O 2 + H 2SO 4 ® Na 2SO 4 + H 2 O 2
2 H + + 2e - ® H 2 ( g )
BaO 2 + H 2SO 4 ® BaSO 4 + H 2 O 2
( NH 4 ) 2 S2 O8 + 2H 2 O ® 2 NH 4 HSO 4 + H 2 O 2
(HNO3 is not used since it will oxidise H2O2).
260 Chemistry
CONCENTRATION OF HYDROGEN PEROXIDE : ii) Oxidising agent : It is strong oxidising agent in acidic as
It is very carefully concentrated since it decomposes on heating well as in basic medium.
or on standing.
H 2 O 2 + 2 H + + 2e - ® 2 H 2 O Eº = 1.77 V
2H 2 O 2 ® 2H 2 O + O 2 (auto-oxidation)
Decomposition is catalysed by Cu, Ag, Pt, Co, Fe, MnO2 etc. H 2 O 2 + OH – + 2e - ® 3OH - Eº = 0.88 V
The methods employed for concentration are
i) Evaporation : By careful evaporation of solution on a water or H 2O 2 ® H 2O + O
bath (50% H2O2 is obtained).
PbS + 4O ® PbSO 4
ii) Dehydration in vacuum desicator : The 50% H2O2 is
dehydrated in a vacuum desicator in presence of conc. H2SO4 2KI + H 2 O + O ® 2KOH + I 2
when 90% H2O2 is obtained. 2FeSO 4 + H 2SO 4 + O ® Fe 2 (SO 4 ) 3 + H 2 O
iii) Vacuum distillation : The 90% H2O2 obtained in step (ii) is
distilled under reduced pressure to get 100% H2O2 2K 4 [Fe(CN) 6 ] + H 2 O + O ® 2K 3 [ Fe(CN ) 6 ] + 2KOH
iv) Cooling : The traces of water left are removed by freezing in
a freezing mixture when crystals of hydrogen peroxide H 2 Cr2 O7 + 4O ® 2CrO5 + H2O
Chromic Blue peroxide
separate out. acid of chromium
STRENGTH OF HYDROGEN PEROXIDE :
C 6 H 6 + O ® C 6 H 5 OH
The strength of hydrogen peroxide is indicated in terms of the
volume of oxygen at NTP that 1 volume of H2O2 gives on heating. In basic medium
For example “20 volume H2O2 means 1 volume of H2O2 at NTP
will give 20 volume of oxygen. The normality and percentage 2Cr 3+ + 3H 2 O 2 + 10OH – ® 2CrO42 - + 8H 2 O
strength of H2O2 can be calculated as follows
Mn 2+ + H 2 O 2 + 2OH – ® MnO2 + 2H 2 O
2 H 2 O 2 ® 2H 2 O + O 2
iii) Reducing agent :
2 × 34 = 68 g 1 mole O2 = 22.4 litres of O2 at NTP
22.4 litres of O2 at NTP are evolved from 68g of H2O2 a) In acidic medium

2MnO4– + 6H + + 5H2 O2 ® 2Mn 2+ + 8H2 O + 5O 2


68 Pink
x litres of O2 at NTP would be evolved from ´ x g of H 2 O 2
22.4
Cr2 O 72- + 8H + + 3H 2 O 2 ® 2Cr 3+ + 7 H 2 O + 3O 2
where x is volume strength of H2O2
68 ´ x b) In basic medium
\ Hence strength of x volume of H2O2 = g / litre
22.4 2K 3 [Fe(CN ) 6 ] + 2KOH + H 2 O 2 ®

Again Strength (g/l) = Equivalent weight × Normality ® 2K 4 [Fe(CN ) 6 ] + 2H 2 O + O 2

68 ´ x iv) Bleaching properties : Its bleaching action is due to


\ = 17 ´ N oxidation reaction.
22.4
where 17 is the equivalent weight of H2O2 H 2O 2 ® H 2O + O
STORAGE OF HYDROGEN PEROXIDE : dye + O ® dye is oxidised and bleached
It is stored in presence of traces of alcohol, acetanilide or sodium
pyrophosphate which slow down the rate of decomposition of STRUCTURE :
hydrogen peroxide. It is represented as follows
CHEMICAL PROPERTIES :
i) Acidic nature : It is weakly acidic in nature and pure In liquid and gas form In solid form
hydrogen peroxide turns blue litmus into red. (Ka = 1.57 ×
H H
10–12 at 293 K). It ionises in two steps O O
94.8 104.9
H2O2 H+ + HO -2 O O
H H
HO -2 H + + O 22 -
111.5 92.5
Hence it forms two series of salts eg. NaHO 2 sodium
hydroperoxide and Na2O2 (Sodium peroxide)
Hydrogen 261
TEST : TYPES OF HARDNESS OF WATER :
i) It liberates I2 from acidified KI It is of two kinds
2KI + H 2 O 2 ® 2KOH + I 2 i) Temporary hardness : It is due to the presence of
bicarbonates of calcium or magnesium or both.
ii) Perchromic acid :
ii) Permanent hardness : It is due to the presence of chlorides
H 2 Cr2 O 7 + 4H 2 O 2 ® 2CrO 5 + 5H 2 O and sulphates of calcium and magnesium.
Blue colour
(chromium peroxide) REMOVAL OF TEMPORARY HARDNESS :
(acidified H 2 O 2 + amyl alcohol + K 2 Cr2 O 7 ) ® blue colour It can be achieved by following methods
iii) With titanic sulphate it gives orange red pertitanic acid i) By boiling : The soluble bicarbonates are converted into
insoluble carbonates.
Ti(SO 4 ) 2 + H 2 O 2 + 2H 2 O ® 2H 2SO 4 + H 2 TiO 4
Ca(HCO3 ) 2 ® CaCO3 ¯ +H 2O + CO2 ­
iv) Black lead sulphide turned white
PbS + 4H 2 O 2 ® PbSO 4 + 4H 2 O Mg(HCO3 ) 2 ® MgCO3 ¯ +H 2O + CO2 ­
ii) By Clark’s process : By adding lime water or milk of lime
USES :
It is used as a bleaching agent, disinfectant, source of power Ca (HCO 3 ) 2 + Ca (OH ) 2 ® 2CaCO 3 ¯ +2H 2 O
(90% H2O2 as fuel in submarines, rockets and helicopters), in Mg ( HCO 3 ) 2 + 2Ca (OH ) 2 ® 2CaCO 3 ¯ + Mg (OH ) 2 ¯ +2H 2 O
restoration of old paintings in which lead oxide is used as white
paint. REMOVAL OF PERMANENT HARDNESS :
WATER : i) By adding washing soda : The calcium or magnesium salts
are precipitated as carbonates
Water is one of the most abundant substances in nature. The 4/
5th of the earth surface is covered with water. Mg ( HCO 3 ) 2 + Na 2 CO 3 ® MgCO 3 ¯ +2 NaHCO 3
Sources of water : The sources of water are MgCl 2 + Na 2 CO 3 ® MgCO 3 ¯ +2 NaCl
i) Surface water :
a) Flowing water eg.: streams and rivers Ca ( HCO 3 ) 2 + Na 2 CO 3 ® CaCO 3 ¯ +2 NaHCO 3
b) Still water eg.: ponds, lakes and reservoirs CaCl 2 + Na 2 CO 3 ® CaCO 3 ¯ +2 NaCl
ii) Underground water : water from wells ii) By adding Caustic Soda : The temporary and permanent
iii) Rain water hardness can be removed by adding caustic soda
iv) Sea water Ca ( HCO 3 ) 2 + 2 NaOH ® Ca (OH ) 2 ¯ +2 NaHCO 3
TYPES OF IMPURITIES PRESENT IN WATER :
Ca ( HCO 3 ) 2 + Ca (OH ) 2 ® 2CaCO 3 ¯ +2H 2 O
They are
i) Dissolved impurities Mg ( HCO 3 ) 2 + 2 NaOH ® Mg (OH) 2 ¯ +2 NaHCO 3
a) Inorganic salts eg. : Ca2+, Mg2+, Fe2+, Al 3+, Na+, K+
CaCl 2 + 2 NaOH ® Ca (OH ) 2 ¯ +2 NaCl
traces
_
of Zn2+ and _
Cu2+ (cations) and Cl–, SO 24 - ,

NO 3 , F or HCO 3 etc. (anions) CaSO 4 + 2 NaOH ® Ca (OH ) 2 ¯ + Na 2SO 4
b) Gases eg.: CO2, N2, O2, oxides of nitrogen, H2S etc.
MgCl 2 + 2 NaOH ® Mg (OH ) 2 ¯ +2 NaCl
c) Organic salts
ii) Suspended impurities : MgSO 4 + 2 NaOH ® Mg (OH ) 2 ¯ + Na 2SO 4
a) Inorganic : eg.: sand and clay iii) By adding Sodium phosphate (Na3PO4) : The phosphates
b) organic : eg.: animal matter, vegetable etc. of calcium and magnesium are precipited
iii) Colloidal impurities : Finely divided clay, Al(OH)3, Fe(OH)3 3Ca ( HCO 3 ) 2 + 2 Na 3 PO 4 ® Ca 3 (PO 4 ) 2 ¯ +6 NaHCO 3
colouring matter etc.
3CaCl 2 + 2 Na 3 PO 4 ® Ca 3 ( PO 4 ) 2 ¯ +6 NaCl
iv) Bacterial impurities : Micro-organisms and bacteria
Effect of impurities : The impurities effect the followings 3CaSO 4 + 2 Na 3 PO 4 ¾
¾® Ca 3 ( PO 4 ) 2 ¯ +3Na 2SO 4
a) Colour b) Taste c) Hardness
d) alkalinity e) Turbidity f) odour etc 3Mg (HCO 3 ) + 2 Na 3 PO 4 ¾
¾®

SOFT AND HARD WATER : ® Mg 3 ( PO 4 ) 3 ¯ +6 NaHCO 3


The water which produces large amount of lather with soap is
known as soft water and which forms a scum with soap is known 3MgCl 2 + 2 Na 3 PO 4 ¾
¾® Mg 3 ( PO 4 ) 2 ¯ +6 NaCl
as hard water. 3MgSO 4 + 2 Na 3 PO 4 ¾
¾® Mg 3 ( PO 4 ) 2 ¯ +3Na 2SO 4
262 Chemistry
iv) Calgon process : Calgon is sodium hexa metaphosphate. Ex. A water sample contains 204 mg of CaSO4 per litre. Calculate
The water is passed through the bed of calgon the Ca 2+ and the hardness in terms of CaCO3 equivalent.
Mg2+ form soluble complex.
Sol. CaSO 4 º CaCO 3
2CaSO 4 + Na 2 [ Na 4 (PO 3 ) 6 ] ® 136 g mol -1 100 g mol -1
2 Na 2SO 4 + Na 2 [Ca 2 (PO 3 ) 6 ]
136 mg / L of CaSO 4 º 100 mg / L of CaCO 3 eq.
2MgSO 4 + Na 2 [ Na 4 (PO 3 ) 6 ] ®
2MgSO 4 + Na 2 [Na 4 (PO3 )6 ] ® 2Na 2SO 4 + Na 2 [Mg 2 (PO3 )6 ] 204 mg/L of CaSO4 º 100 mg / L of CaCO 3 ´ 204
136
Water becomes free from Ca++ and Mg++ ions. = 150 mg/L of CaCO3
v) Permutit process : Permutit is hydrated Sodium aluminium Hence hardness = 150 mg/L or ppm.
silicate Na2 Al2 Si2 O8.xH2O. It exchanges its sodium ions
Ex. A sample of water on analysis was found to contain the
for divalent ions such as Ca2+ and Mg2+.
following impurities expressed in mg/litre.
Na 2 Al 2Si 2 O8 + CaCl 2 ® CaAl 2Si 2 O 8 + 2 NaCl
Impurity Ca(HCO3)2 Mg(HCO3)2 CaSO4 MgSO4
Na 2 Al 2Si 2 O8 + MgSO 4 ® MgAl 2Si 2 O8 + Na 2SO 4
Permutit when fully exhausted can be regenerated by Quantity 10.0 8.5 12.0 14.0
treating with 10% solution of sodium chloride Mol. wt. 162 146 136 120
Ca - permutit + 2 NaCl ® 2 Na - permutit + CaCl 2 Calculate the temporary, permanent and total hardness of
water in mg/litre.
Mg - permutit + 2 NaCl ® 2 Na - permutit + MgCl 2
Sol. Conversion into CaCO3 equivalents
It is most efficient method to get water with zero degree
Impurity Amount Multiplication CaCO3
hardness.
factor equivalt.
vi) By synthetic resins : They are of two types :
a) Cation exchange resins : These are giant molecules Ca(HCO3)2 10.0 mg/L 100/162 10.0 × 100/162
containing sulphonic acid group (–SO 3H). It is first = 6.17 mg/L
changed into sodium salt and has the general fomula Mg(HCO3)2 8.5 mg/L 100/146 8.5 × 100/146
R – Na + The hard water is passed through it when Ca2+ = 5.82 mg/L
and Mg2+ are exchanged and removed. CaSO4 12.0 mg/L 100/136 12.0 × 100/136
= 8.82 mg/L
2R - Na + + Ca 2+ ® R 2 Ca + 2 Na +
MgSO4 14.0 mg/L 100/120 14.0 × 100/120
2R - Na + + Mg 2+ ® R 2 Mg + 2 Na + = 11.67 mg/L
The resins like permutit can be regenerated with a Temporary hardness due to Ca(HCO3)2 + Mg(HCO3)2
solution of common salt. = (6.17 + 5.82) = 11.99 mg/L
R 2 Ca + 2 NaCl ® 2RNa + CaCl 2 Permanent hardnesss due to CaSO4 + MgSO4
b) Anion exchange resins : These are also giant molecules = (8.82 + 11.67) = 20.49 mg/L
and can exchange anions. They contain an amino group. Total hardness (11.99 + 20.49) = 32.48 mg/L
+ HYDRATES :
RNH 2 + H 2 O ® RNH 3OH - The substances (salts) containing water molecules are called
hydrates. These are of three types:
2RNH 3+ OH + CO 32 - (RNH 3+ ) 2 CO 3 + 2OH
– –– –

1) Cationic hydrates : When water molecules are held by


RNH 3+ OH + Cl - RNH 3+ Cl - + OH

cations by coordinate bonds, the hydrates are known as
Anion exchange resin Exhausted anion exchange resin. Cationic hydrates. eg. : MgCl2.6H2O, CaCl2.6H2O, etc.
The water is first passed through cation resins and then 2) Anionic hydrates : In this case the water molecules are held
through anion resins and pure distilled water is obtained. by anions as well as cations by coordinate bonds. eg. :
DEGREE OF HARDNESS : MgSO4.7H2O, CuSO4.5H2O.
The hardness of water is expressed in terms of ppm of calcium 3) Lattice hydrates : The water molecules occupy the lattice
carbonates. sites e.g. : Na2CO3.10H2O, K2SO4. Al2(SO4)3. 24H2O. On
heating the water molecules are lost and substances change
1 CaCO 3 º 1 MgCl 2 º 1 MgSO 4 º 1 CaCl 2 º 1 CaSO 4 to powder form.
100 ppm 95 ppm 120 ppm 111 ppm 136 ppm
Hydrogen 263
HEAVY WATER/DEUTERIUM OXIDE (D2O) : ii) Reaction with Na :
It was discovered by Urey, who showed that ordinary water 2 Na + D 2 O ® 2 NaOD + D 2
contains one part of heavy water in 6,000 parts of it. iii) with acid oxides :
PREPARATION :
P2 O 5 + 3D 2 O ® 2D 3 PO 4
It is prepared by exhaustive electrolysis of water containing alkali Heavy phosphoric acid
with nickel electrodes. About 20 litres of ordinary water gives 0.5
ml of heavy water. SO 3 + D 2 O ® D 2SO 4
Heavy sulphuric acid
PROPERTIES :
iv) with metallic carbides :
Heavy water is a colourless, odourless, tasteless mobile liquid.
Its physical properties in comparision to ordinary water are as Al 4 C3 + 12D 2 O ® 4Al(OD)3 + 3CD 4
follows : CaC 2 + 2D 2 O ® Ca (OD) 2 + C 2 D 2
Physical constants : D2O H2O
v) Deuterolysis : AlCl3 + 3D 2 O ® Al(OD) 3 + 3DCl
Melting point 276.8K 273K
vi) As water of crystallisation : It gives deuterohydrates
Boiling point 374.4K 373K
CuSO4.5D2O, MgSO4.7D2O, etc.
Sp. gravity at 20º C 1.106 0.998
Theoretically six different types of heavy water are possible
Temperature of max. density 284.6K 277K
Specific heat at 20º C 1.018 1.000 16 17 18
eg. : H – O – D, H – O – D, H – O – D,
Viscosity at 293 K 14.2 10.87
Surface tension 67.8 72.8 16 17 18
D – O – D, D – O – D, & D – O – D
Latent heat of vaporisation 2330kJkg–1 2255 kJkg–1
BIOLOGICAL AND PHYSIOLOGICAL EFFECTS :
Dielectric Constant 82 80.5 It does not support life, and is injurious to living organism. It
Solubility of NaCl at 20ºC 30.5% 35.9% checks the growth of plants and animals.
Refractive index 1.328 1.33 USES :
CHEMICAL PROPERTIES : (i) As a tracer compound
i) Electrolysis : (ii) For production of heavy hydrogen.
(iii) As moderator in nuclear reactors.
2D 2 O ®
¬ 2D 2 + O 2
264 Chemistry

Very Short/ Short Answer Questions 18. Complete the following reactions :
1. Why is 2-ethylanthraquinol preferred in the commercial (i) P4O10 + H2O ® (ii) AlCl3 + H2O ®
production of H2O2? (iii) SiCl4 + H2O ® (iv) Ca3P2 + H2O ®
2. Give an example to show that ionic hydrides forms (v) NaH + H2O ®
complexes. Multiple Choice Questions
3. Which gas is evolved when Mg3N2 (Magnesium nitride)
19. Hydrogen resembles halogens in many respects for which
is treated with H2O? Give chemical reaction.
several factors are responsible. Of the following factors
4. How would you prepare a sample of ND3? which one is most important in this respect?
5. How does heavy water react with Al4C3? (a) Its tendency to lose an electron to form a cation.
6. Arrange H2, D2 and T2 in the decreasing order of their boiling (b) Its tendency to gain a single electron in its valence
points. shell to attain stable electronic configuration.
7. Conc. H2SO4 cannot be used for drying H2, why? (c) Its low negative electron gain enthalpy value.
8. Why ice is less dense than water ? (d) Its small size.
9. Statues coated with white lead on long exposure to 20. Which of the following reactions increases production of
atmosphere turn black and original colour can be restored dihydrogen from synthesis gas?
on treatment with H2O2. Why?
1270 K
10. How will you show that H2O2 will act like oxidizing agent in (a) CH 4 ( g ) + H 2 O( g ) ¾¾¾¾ ® CO( g ) + 3H 2 ( g )
Ni
acidic as well as basic medium?
11. Why hard water is not used in industrial boilers for producing 1270K
(b) C( s ) + H 2 O( g ) ¾¾¾¾
® CO( g ) + H 2 ( g )
steam?
673K
12. Hydrogen forms compounds with elements having atomic (c) CO( g ) + H 2O( g ) ¾¾¾¾ ® CO2 (g) + H 2 (g)
Catalyst
number 9, 11, 12 and 17. What are their chemical formulae?
Compare their chemical behaviour. 1270K
(d) C 2 H 6 + 2H 2 O ¾¾¾¾
® 2CO + 5H 2
13. What happens when– Ni

(i) Water reacts with cyanamide of calcium 21. Which of the following reactions is an example of use of
(ii) Peroxosulphuric acid is hydrolysed. water gas in the synthesis of other compounds?
(iii) Moist silver oxide reacts with H2O2 1270K
(a) CH4(g) + H2O(g) ¾¾¾¾
Ni
® CO(g) + H2(g)
14. Describe the industrial application of H2 dependent on
673K
(i) the heat liberated when its atoms are made to combine (b) CO(g) + H2O(g) ¾¾¾¾
Catalyst
® CO (g) + H (g)
2 2
on the surface of a metal
(ii) its effect on unsaturated organic system is presence of 1270K
(c) CnH2n+2 + nH2O (g) ¾¾¾¾
Ni
® nCO + (2n + 1)H2
catalyst
(iii) its ability to combine with N2 under specific conditions Cobalt
(d) CO(g) + 2H2(g) ¾¾¾¾
Catalyst
® CH OH(l)
3
15. A mixture of hydrazine and H2O2 with Cu (II) catalyst is
used as a rocket propellant. Why? 22. Consider the following statements :
1. Atomic hydrogen is obtained by passing hydrogen
Long Answer Questions through an electric arc.
16. (i) What are the ways in which water molecules are bound 2. Hydrogen gas will not reduce heated aluminium oxide.
to an anhydrous salt to form hydrate? 3. Finely divided palladium absorbs large volume of
hydrogen gas.
(ii) Why water is an excellent solvent?
4. Pure nascent hydrogen is best obtained by reacting
17. How does dihydrogen react with Na with C2H5OH.
(i) O2 (ii) C Which of the above statements is/are correct ?
(iii) N2 (iv) F2 (a) 1 alone (b) 2 alone
(v) Cl2? (c) 1, 2 and 3 (d) 2, 3 and 4
Hydrogen 265
23. Elements of which of the following group(s) of periodic table Codes
do not form hydrides. (a) 1–C, 2–D, 3–B, 4–A
(a) Groups 7, 8, 9 (b) Group 13
(b) 1–B, 2–A, 3–C, 4–D
(c) Groups 15, 16, 17 (d) Group 14
24. Water contracts on heating (c) 1–B, 2–D, 3–C, 4–A
(a) to 100 °C (b) from 0°C to 4°C (d) 1–C, 2–A, 3–B, 4–D
(c) to 273 K (d) from 10°C to 20°C 26. Consider the reactions
25. Match list I with list II and select the correct answer using
(A) H2O2 + 2HI ® I2 + 2H2O
the codes given below the lists :
List I List II (B) HOCl + H2O2 ® H3O+ + Cl– + O2
1. Heavy water A. Bicarbonates of Which of the following statements is correct about H2O2
Mg and Ca in with reference to these reactions? Hydrogen peroxide is
water ______ .
2. Temporary B. No foreign ions
hard water in water (a) an oxidising agent in both (A) and (B)
3. Soft water C. D2O (b) an oxidising agent in (A) and reducing agent in (B)
4. Permanent hard D. Sulphates & (c) a reducing agent in (A) and oxidising agent in (B)
water chlorides of (d) a reducing agent in both (A) and (B)
Mg & Ca in water

1. Which of the following metal evolves hydrogen on reaction 8. Which one of the following pairs of substances on reaction
with cold dilute HNO3 ? will not evolve H2 gas ?
(a) Mg (b) Al (a) Iron and H2SO4 (aq)
(c) Fe (d) Cu (b) Iron and steam
2. The metal which gives hydrogen on treatment with acid as (c) Copper and HCl (aq)
well as sodium hydroxide is (d) Sodium and ethanol
(a) Fe (b) Zn 9. Which of the following will not displace hydrogen
(c) Cu (d) None of these (a) Ba (b) Pb
3. The metal that cannot displace hydrogen from dil HCl is (c) Hg (d) Sn
(a) Al (b) Fe 10. The adsorption of hydrogen by palladium is called
(c) Cu (d) Zn (a) hydration
4. Action of water or dilute mineral acids on active metals can (b) reduction
give (c) occlusion
(a) monohydrogen (b) tritium (d) hydrogenation
(c) dihydrogen (d) trihydrogen 11. Reaction of potassium with water is
5. The property of hydrogen which distinguishes it from other (a) exothermic (b) endothermic
alkali metals is (c) hydrolysis (d) absorption
(a) its electropositive character 12. Consider the following statements :
(b) its affinity for non-metals 1. Atomic hydrogen is obtained by passing hydrogen
(c) its reducing character through an electric arc.
(d) its non-metallic character 2. Hydrogen gas will not reduce heated aluminium oxide.
6. 2 g of aluminium is treated separately with excess of dilute 3. Finely divided palladium adsorbs large volume of
H2SO4 and excess of NaOH. The ratio of the volumes of hydrogen gas
hydrogen evolved is 4. Pure nascent hydrogen is best obtained by reacting Na
(a) 2 : 3 (b) 1 : 1 with C2H5OH
(c) 2 : 1 (d) 1 : 2 Which of the above statements is/are correct ?
7. Which of the following statements is correct ? (a) 1 alone (b) 2 alone
(a) Hydrogen has same IP as alkali metals (c) 1, 2 and 3 (d) 2, 3 and 4
(b) Hydrogen has same electronegativity as halogens 13. Metal hydride on treatment with water gives
(c) It has oxidation number of –1 and +1 (a) H2O2 (b) H2O
(d) It will not be liberated at anode. (c) acid (d) hydrogen
266 Chemistry
14. H2O2 is commonly prepared in lab. by the reaction of 27. When two ice cubes are pressed over each other, they unite to
(a) PbO2 + H2SO4 (b) MnO2 + H2SO4 form one cube. Which of the following forces is responsible to
(c) BaO2 + H2O + CO2 (d) Na2O2 + H2O hold them together ?
15. Calculate the normality of 10 volume H2O2 ? (a) Hydrogen bond formation
(a) 1.7 N (b) 12 N (b) van der Waals forces
(c) 30.3 N (d) 0.0303 N (c) Covalent attraction
16. Commercial 10 volume H2O2 is a solution with a strength of (d) Ionic interaction
approximately 28. The process used for the removal of hardness of water is
(a) 30% (b) 3% (a) Calgon (b) Baeyer
(c) 1% (d) 10% (c) Serpeck (d) Hoope
17. A, 6 volume sample of H2O2 29. Pure water can be obtained from sea water by
(a) will contain 6% V/V of H2O2 (a) centrifugation (b) plasmolysis
(b) will contain 6% W/V of H2O2 (c) reverse osmosis (d) sedimentation
(c) would give 6 volumes of oxygen per unit volume of H2O2 30. Polyphosphates are used as water softening agents because
sample at STP they
(d) would give 6 volumes of oxygen per unit weight of H2O2 (a) form soluble complexes with anionic species
sample at STP (b) precipitate anionic species
18. The structure of H2O2 is (c) form soluble complexes with cationic species
(a) planar (b) non planar (d) precipitate cationic species
(c) spherical (d) linear 31. What is heavy water ?
19. In which of the following reactions, H2O2 is acting as a (a) H2O18 (b) H2O16
reducing agent (c) H2O3 (d) D2O
(a) H 2 O 2 + SO 2 ® H 2SO 4 32. D2O is used in
(b) 2KI + H 2 O 2 ® 2KOH + I 2 (a) industry (b) nuclear reactor
(c) medicine (d) insecticide
(c) PbS + 4H 2 O 2 ® PbSO 4 + 4H 2 O 33. Match list I with list II and select the correct answer using
(d) Ag 2 O + H 2 O 2 ® 2Ag + H 2 O + O 2 the codes given below the lists :
List I List II
20. Which of the following is formed by the action of water on
1. Heavy water A. Bicarbonates of Mg
sodium peroxide
and Ca in water
(a) H2 (b) N2
2. Temporary B. No foreign ions
(c) O2 (d) CO2
hard water in water
21. The percentage by weight of hydrogen in H2O2 is
3. Soft water C. D2O
(a) 5.88 (b) 6.25
(c) 25 (d) 50 4. Permanent hard D. Sulphates & chlorides of
22. In which of the following reactions, H2O2 acts as a reducing water Mg & Ca in water
agent? Codes
(a) 1–C, 2–D, 3–B, 4–A (b) 1–B, 2–A, 3–C, 4–D
(a) PbO 2 (s ) + H 2 O 2 (aq ) ® PbO(s) + H 2 O(l) + O 2 (g)
(c) 1–B, 2–D, 3–C, 4–A (d) 1–C, 2–A, 3–B, 4–D
(b) Na 2 SO 3 (aq ) + H 2 O 2 (aq ) ® Na 2SO 4 (aq ) + H 2 O (l) 34. What is formed when calcium carbide reacts with heavy
(c) 2KI(aq ) + H 2 O 2 (aq) ® 2KOH (aq ) + I 2 (s) water?
(d) KNO 2 (aq ) + H 2 O 2 (aq ) ® KNO 3 (aq ) + H 2 O (l) (a) C2D2 (b) CaD2
(c) Ca2D2O (d) CD2
23. The low density of ice compared to water is due to
35. The correct order of the O–O bond length in O2, H2O2 and
(a) hydrogen bonding interactions
O3 is
(b) dipole – dipole interactions
(c) dipole – induced dipole interactions (a) O2 > O3 > H2O2 (b) O3 > H2O2 > O2
(d) induced dipole – induced dipole interactions (c) H2O2 > O3 > O2 (d) O2 > H2O2 >O3
24. The alum used for purifying water is 36. The hydride ion H– is stronger base than its hydroxide ion
(a) Ferric alum (b) Chrome alum OH–. Which of the following reactions will occur if sodium
(c) Potash alum (d) Ammonium alum hydride (NaH) is dissolved in water ?
25. The boiling point of water is exceptionally high because (a) H - (aq ) + H 2 O ® H 3O -
(a) there is covalent bond between H and O (b) H - (aq ) + H 2 O(l) ® OH - + H 2
(b) water molecule is linear (c) H - + H 2 O ® No reaction
(c) water molecules associate due to hydrogen bonding
(d) None of these
(d) water molecule is not linear
26. The H–O–H angle in water molecule is about 37. The component present in greater proportion in Coal gas is
(a) 90º (b) 180º (a) CH4 (b) CO2
(c) 102º (d) 105º (c) CO (d) H2
Hydrogen 267

1. The O – O – H bond angle in H2O2 is [CBSE-PMT 1994] 6. Which of the following statements in relation to the
(a) 106° (b) 109°28' hydrogen atom is correct ? [AIEEE 2005]
(c) 120° (d) 94.8° (a) 3s, 3p and 3d orbitals all have the same energy
2. When H2O2 is oxidised the product is [CBSE-PMT 1999] (b) 3s and 3p orbitals are of lower energy than 3d orbital
(a) OH– (b) O2
(c) 3p orbital is lower in energy than 3d orbital
(c) O2– (d) HO2–
3. Some statements about heavy water are given below: (d) 3s orbital is lower in energy than 3p orbital
(A) Heavy water is used as a moderator in nuclear reactors. 7. In context with the industrial preparation of hydrogen from
(B) Heavy water is more associated than ordinary water. water gas (CO + H2), which of the following is the correct
(C) Heavy water is more effective solvent than ordinary statement? [AIEEE 2008]
water. (a) CO and H2, are fractionally separated using differences
Which of the above statements are correct?
in their densities
[CBSE-PMT 2010]
(a) (A) and (C) (b) (A) and (B) (b) CO is removed by absorption in aqueous Cu2Cl 2
(c) (A), (B) and (C) (d) (B) and (C) solution
4. Which one of the following processes will produce hard (c) H2 is removed through occlusion with Pd
water ? [AIEEE 2003] (d) CO is oxidised to CO2 with steam in the presence of a
(a) Saturation of water with MgCO3 catalyst followed by absorption of of CO2 in alkali
(b) Saturation of water with CaSO4 8. The critical temperature of water is higher than that of O2
(c) Addition of Na2SO4 to water
because H2O molecule has [IIT 1997]
(d) Saturation of water with CaCO3
5. Which of the following species is diamagnetic in nature? (a) fewer electrons than oxygen
(b) two covalent bonds
(a) H -2 (b) H +2 [AIEEE 2005]
(c) v-shape
(c) H2 (d) He +2
(d) dipole moment

1. Hydrogen is not obtained when Zn reacts with 5. The m.pt. of most of the solid substances increase with an
(a) cold water (b) dil H2SO4 increase of pressure. However ice melts at a temperature
lower than its usual melting point when pressure is increased.
(c) dil. HCl (d) 20% NaOH
This is because
2. True peroxide is (a) ice is less denser than H2O
(a) BaO 2 (b) MnO 2 (b) pressure generates heat
(c) the chemical bonds break under pressure
(c) PbO 2 (d) NO2
(d) ice is not a true solid
3. An inorganic compound gives off O2 when heated, turns an
6. Acidified K 2 C r2 O 7 on oxidation by H 2 O 2 gives
acidic solution of KI violet and reduces acidified KMnO4.
The compound is (a) Blue solution (b) CrO5
(a) SO3 (b) KNO3 (c) Chromium peroxide (d) All of these
(c) H2O2 (d) All of these 7. Which reaction shows oxidising nature of H 2 O 2
4. Acidified solution of chromic acid on treatment with H2O2 (a) H 2 O 2 + 2 KI ¾
¾® 2 KOH + I 2
yields (b) Cl 2 + H 2 O 2 ¾
¾® 2 HCl + O 2
(a) CrO3 + H 2 O + O 2 (b) Cr2 O 2 + H 2 O + O 2 (c) H 2 O 2 + Ag 2 O ¾
¾® 2Ag + H 2 O + O 2
(c) CrO 5 + H 2 O + K 2SO 4 (d) H 2 Cr2 O 7 + H 2 O + O 2 (d) NaClO + H 2 O 2 ¾
¾® NaCl + H 2 O + O 2
268 Chemistry

8. The volume strength of 0.5 N H 2 O 2 solution is 20. Decomposition of H 2 O 2 is retarded by :


(a) 4.8 (b) 8.4 (a) H 3PO 4 (b) alcohol
(c) 3.0 (d) 8.0 (c) acetanilide (d) All of these
9. Calgon is an industrial name given to
21. Hydrogen can be placed in halogens group because :
(a) normal sodium phosphate
(a) H is light
(b) sodium meta aluminate
(c) sodium hexametaphosphate (b) it has isotopes D and T
(d) hydrated sodium aluminium silicate (c) it forms hydrides like halides
10. When zeolite (hydrated sodium aluminium silicate) is treated (d) H contains one electron only
with hard water the sodium ions are exchanged with 22. The structure of H 2 O 2 is :
(a) H+ ions (b) Ca2+ ions
H
(c) SO 4 2- ions (d) OH– ions
11. The species that does not contain peroxide ions (a) H O-O (b) O -O
H H
(a) PbO2 (b) H2O2
(c) SrO2 (d) BaO2 (c) H – O – O – H (d) H O -O
H
12. Hydrogen peroxide does not
23. Which is true peroxide :
(a) liberate iodide from KI
(b) turn titanium salt yellow (a) MnO 2 (b) KO 2
(c) gives silver peroxide with moist silver oxide
(c) PbO 2 (d) K 2 O 2
(d) turn mixture of aniline, KClO 3 and dil. H 2SO 4 violet
24 In the hydrogen peroxide molecule :
13. 30 volume hydrogen peroxide means
(a) O – H bonds are polar but molecule is non polar.
(a) 30% of H 2 O 2 solution (b) the four atoms are arranged in a non linear and non
(b) 30 cm3 solution contains 1g of H 2 O 2 planar manner.
(c) 1 cm3 of solution liberates 30 cm3 of O2 at STP (c) all the four atoms are in same plane.
(d) two hydrogen atoms are connected to one of the oxygen.
(d) 30 cm3 of solution contains 1 mole of H2O2
14. The molarity of a 100 ml solution containing 5.1 g of hydrogen 25. The oxide that gives H 2 O 2 on treatment with a dil. acid is
peroxide is
(a) Na 2 O 2 (b) PbO 2
(a) 0.15 M (b) 1.5 M
(c) 3.0 M (d) 50.0 M (c) TiO 2 (d) MnO 2
15. The oxidation states of most electronegative element in the
26. The metal that does not displace hydrogen from an acid is
products of reaction BaO2 with dil. H2SO4 are
(a) Al (b) Ca
(a) 0 and – 1 (b) –1 and – 2
(c) Hg (d) Zn
(c) – 2 and 0 (d) – 2 and + 1
16. Permanent hardness of water can be removed by adding 27. Nascent hydrogen consists of
Calgon (NaPO3)n. This is an example of (a) solvated protons
(a) adsorption (b) exchange of ion (b) hydrogen molecules with excess energy
(c) precipitation (d) None of these. (c) hydrogen ions in excited state
17. Water contracts on heating (d) hydrogen atoms with excess energy
(a) to 100 °C (b) from 0°C to 4°C 28. Which hydride is an ionic hydride :
(c) to 273 K (d) from 10°C to 20°C (a) H 2S (b) TiH1.73
18. 1000g aqueous solution of CaCO3 contains 10g of calcium
carbonate . Hardness of solution is (c) NH 3 (d) NaH
(a) 10 ppm (b) 100 ppm 29. The reaction,
(c) 1000 ppm (d) 10000 ppm
19. Which statement is wrong? 2 H 2 O 2 ® 2H 2 O + O 2
(a) Ordinary hydrogen is an equilibrium mixture of ortho shows that H 2 O 2 :
and para hydrogen
(b) In ortho hydrogen spin of two nuclei is in same direction (a) acts as reducing agent
(c) Ortho and para forms do not resemble in their chemical (b) acts as oxidising agent
properties (c) is decomposed
(d) In para hydrogen spin of two nuclei is in opposite (d) None of these
direction.
Hydrogen 269

EXERCISE 1 9. (c) Hg will not displace hydrogen since it is present below


1. Because it gets regenerated during the reaction hydrogen in ECS.
10. (c) Adsorption of gas by solid is called Occlusion (see text)
2. 8LiH + Al2Cl6 ¾¾ ® 2LiAlH4 + 6LiCl.
3. Mg3N2 + 6H2O ® 3Mg(OH)2 + 2NH3. 11. (a) 2 K + 2 H 2 O ® 2 KOH + H 2 . It is exothermic in nature.
4. Mg3N2 + 6D2O ® 3Mg(OD)2 + 2ND3 H2 catches fire.
Heavy Deutero 12. (c) Pure hydrogen is evolved by reacting absolute alcohol
water ammonia and Na
5. Al4C3 + 12D2O ® 4Al (OD)3 + 3CD4 C 2 H 5 OH + Na ® C 2 H 5 ONa + ½ H 2
6. T 2 > D2 > H2 . other statements are correct. See text.
7. Conc. H2SO4 on absorbing water from moist hydrogen 13. (d) Metal hydride + H 2 O ® Metal hydroxide + H 2
produces so much heat that hydrogen catches fire. 14. (c) BaO 2 + H 2 O + CO 2 ® BaCO3 + H 2 O 2 (Merck process)
9. PbO + H2S ¾¾ ® PbS + H2O
15. (a) Normality of 10V of H2O2
(black)
PbS + 4H2O2 ¾¾
® PbSO4 + 4H2O 68 ´ 10
= 17 ´ N \ N= 1.78
(white) 22.4
10. Acidic : 68 ´ 10
H2O2(aq) + 2H+ (aq) + 2e– ® 2H2O(l) 16. (b) Strength of 10V H2O2 = g / l = 3.035%
22.4
Basic : 17. (c) 6 volume H2O2 would give 6 volumes of O2 per unit
H2O2 (aq) + OH–(aq) + 2e– ® 3OH– (aq) volume of H2O2
12. Compounds are : HF, NaH, MgH2 and HCl 18. (b) See text. Structure of H2O2 is nonplanar
13. (i) CaCN2 + 4H2O ® CaCO3 + NH4OH. 19. (d) SO2 changes to H2SO4 (O.N. changes from +4 to +6
(ii) H2S2O8 + 2H2O ® 2H2SO4 + H2O2 oxidation)
(iii) H2O2 + Ag2O ® 2Ag + H2O + O2
2KI ® I2 (O.S. changes from –1 to 0 oxidation)
14. (i) Atomic hydrogen torch is used industrially to melt
refractory materials like tungstun, tantalum, etc. PbS ® PbSO 4 (O.S. changes from –2 to +6 oxidation)
Ni Ag 2 O ® 2 Ag (O.S. changes from +1 to 0 Reduction)
(ii) Oil + H 2 ¾¾ ¾ ¾ ¾® saturated vanaspati ghee
450K / 5atm 20. (c) Na 2 O 2 + 2 H 2 O ® 2 NaOH + H 2 O 2
400° C / 200atm
(iii) N2(g) + 3H2(g) ¾¾ ¾ ¾ ¾ ¾® 2NH3(g) 2 H 2 O 2 ® 2H 2 O + O 2
Fe / Mo
19. (b) 20. (c) 21. (d) 22. (c) 23. (a) 2
21. (a) % hydrogen in H2O2 = ´ 100 = 5.88%
24. (b) 25. (d) 26. (b) 34
EXERCISE 2 22. (a) PbO2 ® PbO (change in O.S. is +4 to +2 hence
reduction)
1. (a) Mg + 2HNO3 (dil.) ® Mg(NO3 ) 2 + H 2
23. (a) It is due to hydrogen bonding when H2O forms a cage
(Mg and Mn give H2 with dil HNO3) like structure in solid ice and density is reduced.
2. (b) Zn is amphoteric, it reacts with acids and alkali to give 24. (c) Potash alum is used for purifying water.
hydrogen. 25. (c) The high boiling point of water is due to H-bonding.
Zn + H 2SO 4 ® ZnSO 4 + H 2 26. (d) The hybridisation in water is sp3 and bond angle 105º
Zn + 2 NaOH ® Na 2 ZnO 2 + H 2 27. (a) Two ice cubes stick to each other due to H-bonding
3. (c) Cu is below hydrogen in E.C.S. hence cannot evolve H2 28. (a) Calgon process is used to remove permanent hardness
with acids. of water
4. (c) Active metals can react with water and dil acids to give 29. (c) Sea water is purified by reverse osmosis.
H2. 30. (c) Polyphosphates form soluble complexes with Ca2+ and
5. (d) Hydrogen is non metallic in nature. Mg2+ and are removed.
6. (b) 2Al + 3H 2SO 4 ® Al 2 (SO 4 ) 3 + 3H 2 31. (d) D2O is heavy water
32. (b) D2O is used in nuclear reactors as moderators.
2Al + 2 NaOH + 2H 2 O ® 2 NaAlO 2 + 3H 2 33. (d) Heavy water is D2O (1–C); Temporary hard water
The ratio of volumes of hydrogen evolved is 1:1. contains the bi-carbonates of Mg and Ca (2–A); Soft
7. (c) In metal hydrides the O.S. of hydrogen –1 otherwise it is water contains no foreign ions (3–B); Permanent hard
+1. water contains the sulphates and chlorides of Mg and
8. (c) Ca (4–D) therefore the answer is D.
270 Chemistry

34. (a) CaC 2 + 2D 2 O ® C 2 D 2 + Ca (OD) 2 4. (c) K2Cr2O7 + H2SO4 ® K2SO4 + H2Cr2O7


.. .. .. ..
H-O H 2 Cr2 O 7 + 4H 2 O 2 ® 2CrO 5 + 5H 2 O
35. (c) . .- O
. . - H, O
.. =O
..
5. (a) Ice occupy more volume than liquid water (Ice water).
Increase of pressure favours forward reaction ( Le-
Hence O–O bond length follows the order
Chatelier’s principle).
H2O2 > O3 > O2
36. (b) H - (aq) + H 2 O(l) ® OH - + H 2 . Since H– is a strong 6. (b) CrO5 is blue peroxide of chromium and has butterfly
base it will abstract H+ to form H2O. O O
37. (d) Composition of Coal gas is 45%, H2 , 35%, CH4, 8%, CO, structure Cr
4% gaseous hydrocarbons and 8% N2, O2 CO2. O O
O
EXERCISE 3
7. (a) H 2 O 2 + 2KI ® 2KOH + I 2 , O.S of I - ( -1) changes to
1. (d) O – O – H bond angle in H2O2 is 94.8°.
Oxidation
I2 (Zero) There is loss of electrons, hence oxidation.
2. (b) H 2 O 2 + [ O ] ¾¾ ¾ ¾¾® H 2 O + O 2 ­
68
3. (b) \ Correct choice : (b) 8. (b) Use the formula , volume strength = N × E
22 .4
4. (b) Permanent hardness of water is due to chlorides and
sulphates of calcium and magnesium i.e CaCl2, CaSO4, \ volume strength = 1.5 ´ 17 ´ 22.4 = 8.4
MgCl2 and MgSO4. 68
5. (c) A diamagnetic substance contains no unpaired 9. (c) Sodium hexametaphosphate (Na 6 P6 O18 ) is calgon, it is
electron. used to remove permanent hardness of water.
H2 is diamagnetic as it contains all paired electrons 10. (b) Na2 zeolite + CaCl2 ® Ca zeolite + 2 NaCl
H 2 = s b2 , H 2+ = s1b , H 2- = s b2 , s*1
a ; 11. (a) PbO2 is lead dioxide and does not contain O - O bonds
(diamagnetic) (paramagnetic) (paramagnetic)
and O 22- ions.
= s1b , H 2- = sb2 , He2+ = sb2 , s*1
a 12. (c) No such reaction is known (c).
agnetic) (paramagnetic) (paramagnetic) (paramagnetic) 13. (c) 30 vol of H2O2 means one volume of H2O2 on decom-
6. (a) In one electron species, such as H-atom, the energy of position will give 30 volume of oxygen.
orbital depends only on the principal quantum number,
5.1´ 1000
n. 14. (b) M= = 1 .5
Hence answer (d) 34 ´ 100
i.e., 1s < 2s = 2p < 3s = 3p = 3d < 15. (b) BaO 2 + H 2SO 4 ® BaSO 4 + H 2 O 2 .
4s = 4p Þ 4d = 4g oxygen has common O.S. as –2 and in peroxides
7. (d) On the industrial scale hydrogen is prepared from water as –1.
gas according to following reaction sequence 16. (b) There is exchange of ions
catalyst 17. (b) When water is heated from 0°C to 4°C , its density.
CO + H 2 + H 2O ¾¾¾¾
® CO 2 + 2H 2
1424 3 { æ mö
water gas (steam) increases and volume decreases. ç d = ÷
è Vø
2NaOH
¾¾¾¾
® Na 2 CO3 + H 2 O 6
18. (d) Amount of CaCO 3 in ppm is = 10 ´ 10 = 10 4 ppm.
(alkali)
From the above it is clear that CO is first oxidised to 1000
CO2 which is then absorbed in NaOH. 19. (c) Ortho and para forms of hydrogen resemble in their
8. (d) Critical temperature of water is more than O2 due to its chemical properties.
dipole moment (Dipole moment of water = 1.84 D; Dipole
20. (d) Acetanilide, alcohol and H 3PO 4 are negative catalyst
moment of O2 = zero).
and retard decomposition of H 2 O 2 .
EXERCISE 4 21. (c) Hydrogen forms hydrides like halides e.g. NaH, NaCl.
1. (a) Only elements having reduction potential less than - 0.41V 22. (a) The structure given (a) is correct.
liberate hydrogen with cold water. 23. (d) True peroxides contain O 22 - and O – O linkage.
2. (a) True peroxide contains O-O linkage and O 22 - ion. They 24. (b) Statement (b) is correct. See structure of H 2 O 2 .
give hydrogen peroxide with dil H 2SO 4 . 25. (a) Na 2 O 2 + 2HCl ® 2 NaCl + H 2 O 2
26. (c) The metals present below hydrogen in electrochemical
BaO2 + H2SO4 ( dil.) ® BaSO4 + H2 O2 series can not displace hydrogen from acids.
D
3. (c) 2 H 2 O 2 ¾¾® 2H 2 O + O 2 27. (d) Statement (d) is correct.
2KI + H 2O 2 ® 2KOH + I 2 28. (d) All metal hydrides are ionic in nature.
2KMnO4 + 3H 2SO4 + 5H 2 O2 ® 29. (c) 2H 2 O 2 ® 2H 2 O + O 2
K 2SO 4 + 2MnSO 4 + 8H 2 O + 5O 2 The reaction is decomposition of H 2 O 2 .
10 A
The s-block Elements –
Alkali Metals
THE ELEMENTS OF GROUP I ARE: (b) The ionic radii like atomic radii of all these alkali metal
Li - Lithium Na - Sodium K - Potassium ions goes on increasing on moving down the group
Rb - Rubidium Cs - Caesium Fr - Francium because of the same reason.
These elements are known as alkali metals. (iv) Density -
Lithium is known as a bridge element and was discovered by (a) These are light metals having low densities. Lithium is
Arfwedson. the lightest known metal.
Sodium and potassium were discovered by Davy, rubidium and (b) On moving down the group, both the atomic size and
caesium by Bunsen and Kirchoff while francium by Perey. atomic mass increases and since the increase in latter is
These do not occur in the native state (i.e.,do not occur free in not compensated by increase in former,consequently
nature ). density increases from Li to Cs.
(c) The density of potassium is lesser than that of sodium
GENERAL CHARACTERISTICS
because of the abnormal increase in size on moving
Physical properties of alkali metals are from Na to K.
(i) Electronic configuration :- These are s-block elements and
(v) Melting and Boiling points :-
have one electron in the valence shell in
(a) The melting and boiling points of alkali metals are quite
s-orbital. In general their electronic configuration may be
low and decreases down the group due to weakening of
represented as [noble gas ] ns1 where ‘n’ represents the
metallic bond.
valence shell.
(b) Fr is a liquid at room temperature.
Element Atomic no. Electronic Valence shell
(vi) Softness - These are soft,malleable and ductile solids which
configuration configuration
can be cut with knife. They possess metallic lustre when
Li 3 [He] 2 s1 2 s1
1
freshly cut due to oscillation of electrons.
Na 11 [Ne] 3 s 3 s1
1 (vii) Atomic volume- Atomic volume of alkali metals is the highest
K 19 [Ar] 4 s 4 s1
1 in each period and goes on increasing down the group
Rb 37 [Kr] 5 s 5 s1
Cs 55 [Xe] 6 s 1 6 s1 Eleme nt Li Na K Rb Cs
Fr 87 [Rn] 7 s 1 7 s1 Gram atomic 13 24 46 56 71
volume in cm 3
(ii) Size of the atoms - Atomic radii.
(a) The alkali metals atoms have the largest atomic radii in (viii)Ionisation energy:-
their respective periods. (a) The first ionisation energy of alkali metals is the lowest
(b) Atomic radii increases as we move down the group from amongst the elements in their respective periods and
Li to Cs due to the additon of a new shell at each step. decreases on moving down the group.
(iii) Size of the ion - Ionic radii -
Element Li Na K Rb Cs Fr
(a) The ions of the alkali metals are much smaller than their
corresponding atomic radii due to lesser number of shells IE, (kJ mol–1) 520 496 419 403 376 ----
and contractive effect of the increased nuclear charge.
272 Chemistry
(b) The second ionisation energies of all the alkali metals (xiv)Photoelectric effect :- Due to low I.E., alkali metals especially
are very large because on releasing an electron from the K and Cs show photoelectric effect (i.e. eject electrons when
elements, the resulting ions acquire noble gas (stable) exposed to light) and hence are used in photelectric cells.
configurations. (xv) Electrical conductivity :- Due to the presence of loosely
(ix) Electropositive character :- Because of their low ionisation held valence electrons which are free to move throughout
energies, alkali metals are strongly electropositive or metallic the metal structure, the alkali metals are good conductors of
in nature and this character increases from Li to Cs. heat and electricity. Electrical conductivity increases from
(x) Crystal structure :- All alkali metals possess body centred top to bottom in the order
cubic structures with co-oridination number 8
(xi) Oxidation state :- Li + < Na + < K + < Rb + < Cs +
(a) The alkali metal atoms show only +1 oxidation state, (xvi) Reducing character :-
because their unipositive ions have the stable gas (a) All the alkali metals are good reducing agents
and it is due to their low ionisation energies.
electronic configuration (ns 2 or ns 2 p 6 ) in the valence
Their reducing character, follows the order
shell.
Na < K < Rb < Cs < Li
(b) Since the alkali metal ions have noble gas configuration
with no unpaired electrons, they are diamagnetic and (b) Among the alkali metals Li has the highest negative
colourless but their permanganates and dichromates electrode potential, which depends upon its (i) heat of
compounds are coloured. vaporisation (ii) ionisation energy and (iii)heat of
(xii) Hydration of ions :- hydration and hence Li is the strongest reducing agent.
(a) All alkali metal salts are ionic (except Lithium) and soluble Elements Li Na K Rb Cs Fr
in water due to the fact that cations get hydrated by
E o (V)at –3.05 –2.71 –2.93 –2.99 –2.99 —
water molecules. The degree of hydration depends upon at 298 K
the size of the cation. Smaller the size of a cation, greater
is its hydration energy. CHEMICAL PROPERTIES :
Relative ionic radii : (i) Alkali metals form ionic compounds (Lithium can form
covalent compounds because of its high ionisation energy)
Cs + > Rb + > K + > Na + > Li + and others form ionic compounds because of their large
Relative ionic radii in water or relative degree of atomic size and low I.E.
(ii) Due to low I.E. and high electropositive character the alkali
hydration : Li + > Na + > K + > Rb + > Cs +
metals are chemically very reactive.
(b) The alkali metal ions exist as hydrated ions M + (H 2 O )x (iii) Action of Air :- On exposure to moist air, their surface is
tarnished due to the formation of their oxides, hydroxides
in the aqueous solution.
and carbonates at the surface.
(c) Since the degree of hydration of M + ions decreases as
4 Na (s)+ O 2 (g) ¾
¾® 2 Na 2 O(s)
we go down the group, the hydration energy of alkali
Sodium Sodium oxide
+ +
metal ions decreases from Li to Cs .
Na 2 O(s) + H 2 O (l ) ¾
¾® 2 NaOH(s)
(xiii) Flame colouration :- sodium hydroxide
(a) All alkali metals and their salts impart characteristic
colours to the flame because of the bonding of the ¾® Na 2 CO 3 (s) + H 2 O(l)
2 NaOH (s) + CO 2 (g ) ¾
outermost electron.The outer eletrons of these atoms sodium carbonate
are excited to higher energy levels. On returning to the Hence they are kept under inert liquid kerosene oil but lithium
original state they give out visible light of characteristic is kept wrapped in paraffin wax because it floats on
wavelength. This gives a characteristic colour to the the surface of kerosene oil due to its very low density.
flame. (iv) Action of oxygen :-
(b) On moving down the group, the ionisation energy goes (a) All the alkali metals when heated with oxygen form
on decreasing and hence the energy or the frequency of different types of oxides for example, lithium forms lithium
emitted light goes on increasing in the order Li < Na < K
oxide (Li 2 O) , sodium forms sodium peroxide ( Na 2 O 2 ) ,
< Rb < Cs. As a result, the colour shows following trend
while K, Rb and Cs form their respective superoxides
Li Na K Rb Cs
( MO 2 where M=K, Rb or Cs). The increasing stability
crimson golden pale purple sky blue
of peroxides and superoxides of alkali metals from Li to
red yellow violet (violet)
Cs is due to stabilisation of larger anions by larger cations
through lattice energy.
The s-block Elements – Alkali Metals 273
(b) Superoxides are coloured and paramagnetic as these (c) All alkali halides except LiF are freely soluble in water
... (LiF is soluble in non-polar solvents.Since it has strong
possess three electron bond (: O - O :) - where one
covalent bond.)
unpaired electron is present.Sodium peroxide acquires (d) The power of the cation to polarise the anion is known
yellow colour due to the presence of traces of superoxide as the polarising power while the tendency of the anion
as an impurity. KO 2 is orange, RbO 2 is brown and to get polarised is known as its polarisability. The
polarising power of cation and polarisability of anion
CsO 2 is orange in colour.. depends on the following factors (which are collectively
(c) All oxides, peroxides and superoxides are basic in nature. referred to as Fajan’s rules)
(d) The solubility and basic strength of oxides increase in • Size of the cation :- Smaller the size of cation greater is
its polarising power. So LiCl is more covalent than KCl.
the order Li 2 O < Na 2 O < K 2 O < Cs 2 O
• Size of the anion:-Bigger the anion, larger is its
(e) The stability of peroxides and superoxides increases in polarisability. Hence the covalent character of lithium
the order halides is in the order -
Na 2 O 2 < K 2 O 2 < Rb 2 O 2 < Cs 2 O 2 and LiI > LiBr > LiCl > LiF
• Charge of the ion and electronic configuration -Larger
NaO 2 < KO 2 < RbO 2 < CsO 2 the charge on the cation,greater is its polarising power
(v) Action with water and other compounds containing acidic Thus the covalent character of various halides is in the
hydrogen : order
(a) All the alkali metals readily react with water evolving
Na + Cl - < Mg 2+ Cl 2- < Al 3+ Cl3-
hydrogen.
when two cations have same charge and size, the one
2M + 2H 2 O ® 2MOH + H 2 having 18 electrons in their outermost shell will have
(where M=Li, Na, K, Rb or Cs) larger polarising power than a cation having 8 electrons
The reactivity with water increases on descending the in the outermost shell.For example CuCl is more covalent
group from Li to Cs as Li < Na < K < Rb < Cs due to then NaCl.
increase in electropositive character in the same order. Above rules help to predict the ionic /covalent character
(b) Alkali metals also react with alcohols and acetylene and of metal halides.
(viii)Melting points of alkali metal halides -
liberate H 2 e.g. (a) For the same alkali metal, the melting points decrease in
the order with the increase in the size of halides ion.
2Li + 2C 2 H 5OH ® 2C 2 H 5OLi + H 2
Fluorides > chlorides > bromides > iodides
Ethyl alcohol Lithium ethoxide
Thus
2M + HC º CH ® M - C º C - M + H 2
Alkali metal acetylide NaF NaCl NaBr NaI
Melting point 1268K 1081K 1028K 934K
(vi) Action of hydrogen :- Alkali metals combine with hydrogen
(b) For the same halide ion, melting points decreases with
to form ionic hydrides M + H - the increasing size of the metal but lithium halides being
2M + H 2 ® 2MH (where M = Li,Na,K etc.) covalent have lower melting point than corresponding
sodium halides. Thus
The reactivity of alkali metals towards hydrogen decreases
LiCl NaCl KCl RbCl CsCl
as we move down the group i.e. Li > Na > K > Rb > Cs, due
Melting point 883K 1081K 1045K 990K 918K
to the decreasing lattice energy of these hydrides with the
increasing size of the metal cation. Thus the stability of (ix) Reaction with nitrogen :- Only lithium reacts with nitrogen
hydrides follows the order and forms lithium nitride ( Li3 N )
LiH > NaH > KH > RbH > CsH D
(vii) Reaction with halogens :- 6Li + N 2 ¾¾® 2Li3 N
(a) Alkali metals combine readily with halogens to form ionic (x) Reaction with sulphur and phosphorus :- Alkali metals react
with sulphur and phosphorus on heating and form respective
halides M + X - sulphides and phosphides.
2M + X 2 ® 2M + X - 16M + S8 ¾¾®
D
8M 2 S
D
; 12M + P4 ¾¾® 4M 3 P
metal sulphide metal phosphide
[where M= Li,Na, K etc. and X = F,Cl, Br,I]
(b) The reactivity of alkali metals towards a particular halogen (xi) Solubility in liquid ammonia:- All alkali metals dissolve in
increases in the order : liquid ammonia giving deep blue solution, which has some
Li < Na < K < Rb < Cs characteristic properties given below due to formation of
while that of halogen towards a particular alkali metal ammoniated metal cations and ammoniated electrons in the
decreases in the order : solution.

F2 > Cl2 > Br2 > I2 M ® M+ + e-


274 Chemistry
+ The thermal stability of carbonates increases with the
M + + xNH 3 ® [M(NH 3 )x ] increasing basic strength of metal hydroxides on moving
- down the group.Thus the order is
e - + yNH3 ® [e(NH 3 )y ] Li 2 CO 3 < Na 2 CO 3 < K 2 CO 3 < Rb 2 CO 3 < Cs 2 CO 3
+ - (b) The bicarbonates of all the alkali metals are known. All
M + (x + y )NH 3 ® [ M ( NH 3 ) x ] + [e( NH 3 ) y ]
Ammoniated cation
the bicarbonates (except LiHCO3 which exits in
Ammoniated electron
(a) Colour -The blue colour is due to the excitation of solution) exist as solids and on heating form carbonates.
ammoniated electron to higher energy levels and the D
2 NaHCO 3 ¾¾® Na 2 CO 3 + CO 2 + H 2 O
absorption of photons occurs in the red region of the
spectrum.Thus the solution appears blue.But at very (c) The solubility of the carbonates and bicarbonates
increases on moving down the group due to lower lattice
high concentration the solution attains the colour like
energies. Thus order is
that of metallic copper.
(b) Conductivity :- It is highly conducting because of the LiHCO3 < NaHCO3 < KHCO3 < RbHCO3 < CsHCO3 .
presence of ammoniated electrons and ammoniated ( xv)Nature of Nitrates:- LiNO3 on heating decomposes to give
cations.However, on cooling,the conductivity increases
further. NO 2 and O 2 while the nitrates of the other alkali metals
(c) Paramagnetism :- It is paramagnetic due to the presence decompose on heating to form nitrites and O2 .
of an unpaired electrons and ammoniated
cations.However the paramagnetism decreases with D
4LiNO3 ¾¾® 2Li 2 O + 4 NO 2 + O 2
increasing concentration due to the association of D
2 NaNO3 ¾¾® 2 NaNO 2 + O 2
ammoniated electrons to yield diamagnetic species
containing electron pairs. All nitrates are soluble in water.
(xvi)Nature of sulphates :-
2[e - (NH 3 )y ] ® [e - (NH 3 )y ]2
(a) Li 2SO 4 is insoluble in water whereas the other
(d) Reducing property :- Due to the presence of ammoniated
electrons, solution is a very powerful reducing agent sulphates i.e, Na 2SO 4 , K 2SO 4 are soluble in water..
and used in organic chemistry under the name Birch (b) Lithium sulphate does not form alums and is also not
reduction. amorphous with other sulphates.
(xii)Complex formation :- Alkali metals have a weak tendency to ANOMALOUS BEHAVIOUR OF LITHIUM :
form complexes but polydentate ligands such as crown ethers Lithium, the first member of the alkali metal family shows an
and cryptands form highly stable complexes collectively anomalous behaviour because of the following main reasons:-
called as Wrap Around Complexes. Cryptands are (a) It has the smallest size in the group
macrocyclic molecules with N and O atoms and their
(b) It has very high ionization energy and highest
complexes are called cryptates. The name cryptate came from
electronegativity in the group.
the fact that metal ion is hidden in the structure.
(c) It has no vacant d-orbital in the valence shell.
(xiii)Nature of hydroxides :- Alkali metals hydroxides are very
As a result, it differs from the other member of the alkali metal
strong bases, highly soluble in water and are not decomposed
family in following respects:
on heating.However, LiOH decomposes on heating to give
(i) Lithium is harder than other alkali metals, due to strong
Li 2 O because latter is more stable than former.. metallic bond.
2LiOH ® Li 2 O + H 2 O (ii) Lithium combines with O 2 to form lithium monoxide,
Their basic strength increases from LiOH to CsOH due to a Li 2 O , whereas other alkali metals form Peroxides,
corresponding decresae in the I.E., of the metal in a group,i.e.,
M 2 O 2 and superoxides MO 2 .
the order:-
LiOH < NaOH < KOH < RbOH < CsOH (iii) Lithium, unlike the other alkali metals, reacts with nitrogen
(xiv)Nature of carbonates and bicarbonates :- to form the nitride.

(a) Li 2 CO 3 is unstable towards heat and decomposes to 6Li + N 2 ® 2Li 3 N


Lithium nitride
give Li 2 O and CO 2. (iv) LiOH is a weak base and decomposes to give the
corresponding oxide while the hydroxides of alkali metals
D
Li 2 CO 3 ¾¾® Li 2 O + CO 2 are stable to heat and sublime as such.
The s-block Elements – Alkali Metals 275
900°C Fire caused by burning of alkali metals is extinguished by
2LiOH ¾¾¾® Li 2 O + H 2 O
sprinkling CCl 4 .
(v) Li 2 CO 3 , LiF and lithium phosphate are insoluble in
A mixtureof Na 2 O 2 and dil.HCl is commercially called Oxone and
water while the corresponding salts of other alkali metals
are soluble in water. is used for bleaching delicate fibres.
(vi) LiH is the stablest among all the alkali metal hydrides. METALLURGY OF SODIUM
(vii) Li 2 CO 3 decomposes on heating to evolve CO2 whereas OCCURRENCE AND MINERALS :
other alkali metal carbonates do not. (i) Sodium does not occur in the free state because of its high
(viii) Lithium nitrate on heating evolves O2 and NO2 and reactivity.
forms Li2O while other alkali metal nitrates on heating (ii) Important minerals of sodium are -
evolve O 2 and form their respective nitrites. (a) Common salt or rock salt,NaCl
(ix) Lithium when heated with ammonia forms lithium imide (b) Chile saltpetre, NaNO3
(Li 2 NH ) while other alkali metals form amides of the (c) Sodium carbonate, Na 2 CO 3
general formula ( MNH2 where M=Na,K, Rb and S). (d) Sodium sulphate or Glauber’s salt Na2SO4. 10 H2O
(x) Only lithium combines directly with carbon to form (e) Cryolite, Na 3 AlF6
lithium carbide, Li 2 C 2 , while other alkali metals react
(f) Borax, Na 2 B 4 O 7 .10H 2 O
with ethyne to form the corresponding metal carbides.
EXTRACTION OF SODIUM :
DIAGONAL RELATIONSHIP :
(a) Sodium metal is extracted by electrolysis of fused NaCl
Lithium shows diagonal relationship with magnesium, the element
of group 2 and this resemblance is due to polarising power, i.e, containing a little CaCl 2 and KF at 873 K. This process is
known as Down process.
é Ionic charge ù
ê ú is similar for both of these elements. (b) Reactions during electrolysis -
ëê (Ionic radius ) ûú
2
NaCl Na + + Cl -
Lithium resembles magnesium in the following respects: + -
At Cathode : Na + e ® Na
(i) The atomic radius of Lithium is 1.31Å while that of At Anode : Cl - ® Cl + e - ; Cl + Cl ® Cl2
magnesium is 1.36 Å . (c) Difficulties during the process :-
(i) Sodium cannot be extracted from aqueous NaCl because
(ii) The ionic radius of Li + ion is(0.60Å) which is very close to
the metal liberated at the cathode reacts with H 2 O to
that of Mg2+ ion (0.65Å).
(iii) Lithium (1.0) and magnesium (1.2) have almost similar form metal hydroxide and H 2 .
electronegativities. (ii) NaCl melts at 800º C and it is difficult to attain and maintain
(iv) Both Li and Mg are hard metals. this high temperature.
(v) LiF is partially soluble in water like MgF2 . (iii) Molten Na forms a metallic fog (colloidal solution ) with
fused NaCl.
(vi) Both decompose water only on heating.
(vii) Alkyls of lithium and magesium are soluble in organic Above difficulties were removed by adding CaCl 2 and
solvents. KF to fused NaCl which themselves do not undergo
(viii) Both combine with O 2 to form monoxides,e.g., Li 2 O and decomposition at the voltage employed and lower the
MgO. melting point of NaCl to about 600°C .
(d) The electrodes are separated by a wire gauze to prevent the
(ix) Both LiOH and Mg (OH )2 are weak bases.
reaction between Na and Cl2 .
(x) Both LiCl and MgCl 2 are predominantly covalent.
METALLURGY OF POTASSIUM
(xi) Both Li and Mg combine with N 2 to form their respective
OCCURRENCE AND MINERALS :
nitrides, Li 3 N and Mg 3 N 2 . (i) Potassium also does not occur in free state.
(xii) The hydroxides and carbonates of both Li and Mg (ii) Important minerals of potassium are :-
decompose on heating and form their respective oxides. (a) Sylvine, KCl
(xiii) Both lithium and magnesium nitrates on heating evolve (b) Carnallite, KCl.MgCl 2 .6H 2 O
NO 2 and O 2 leaving behind their oxides.
276 Chemistry

(c) Feldspar, K 2 O.Al 2 O 3 .6SiO 2 At Cathode : Na + + e - ® Na ; Na + Hg ® Na / Hg


(d) Kainite, KCl.MgSO 4 .3H 2 O In central compartment -
Anode - Mercury
EXTRACTION OF POTASSIUM : Cathode - Iron rods
Potassium is not obtained by the electrolysis of fused KCl because Electrolyte - dil. solution of NaOH
K has lower boiling point (1039 K) than the melting point of KCl Reaction - At Anode :
(1063K) and hence it get vaporises. Therefore, K metal is extracted
OH - ® OH + e- ;
by the following methods :-
(i) By the electrolysis of fused KOH:- The reaction involved Na / Hg + OH ® NaOH + Hg
are
At Cathode : H + + e - ® H ; H + H ® H 2 ­
KOH ® K + + OH - (iii) Lowing's Process
At Cathode : K + + e- ® K
Na 2 CO 3 + Fe 2 O3 ® 2 NaFeO 2 + CO 2
At Anode : 4OH - ® 2 H 2O + O2 + 4e - Sod.ferrite
(ii) Modern method :- By the reduction of molten KCl with metallic 2 NaFeO 2 + H 2 O ® 2 NaOH + Fe 2 O 3
sodium in stainless steel vessel at 1120-1150 K.
1120 -1150 K
KCl(l) + Na(g) ¾¾¾¾¾¾
® NaCl + K(g) (iv) Pure Sodium Hydroxide
Commercial NaOH + Alcohol ®
COMPOUNDS OF SODIUM
Impurities and Na 2 CO 3 are insoluble.
SODIUM CHLORIDE, COMMON SALT OR TABLE
The filtration on evaporation give pure NaOH
SALT, NaCl :
(i) It is obtained by evaporation of sea water in sun but due to
PROPERTIES :
(i) It is a hygroscopic, deliquescent white solid, absorbs
presence of impurities like CaSO4 , CaCl 2 and MgCl 2 it is
CO2 and moisture from the atmosphere.
deliquescent It is purified by passing HCl gas through the
2 NaOH + CO 2 ® Na 2 CO 3 + H 2 O
impure saturated solution of NaCl and due to common ion
effect, pure NaCl gets precipitated. (ii) Reaction with salts:- It reacts with metallic salts to form
(ii) 28% NaCl solution is called Brine. hydroxides out of which some are unstable and decompose
to insoluble oxides,
SODIUM HYDROXIDE, CAUSTIC SODA, NaOH :
(a) Formation of insoluble hydroxides, e.g.
Preparation:-
(i) Causticizing process ( Gossage process):- A 10% solution FeCl3 + 3NaOH ® Fe(OH )3 ¯ +3NaCl

of Na 2 CO 3 is treated with milk of lime, Ca (OH )2 . (b) Formation of unstable hydroxides, e.g.
2AgNO3 + 2 NaOH ® 2 NaNO3 + 2AgOH
Na 2 CO3 + Ca (OH)2 ® CaCO3 ¯ +2 NaOH
2AgOH ® Ag 2 O ¯ + H 2 O
(ii) Electrolytic process :- In this process a concentrated solution Brown
of sodium chloride is electrolysed where Cl 2 is evolved at (c) Formation of insoluble hydroxides which dissolve in
excess of NaOH e.g. Zn, Al, Sb, Pb, Sn and As.
the anode and H 2 at the cathode. However Cl2 gas reacts
with NaOH forming NaCl and sodium hypochlorite. ZnSO 4 + 2 NaOH ® Zn (OH ) 2 ¯ + Na 2SO 4
Mercury cathode process ( Castner - Kellner cell ):- This Zn (OH) 2 + 2 NaOH ® Na 2 ZnO 2 + 2H 2 O
Sodium zincate
process is used to avoid reaction between NaOH and
Cl2 .NaOH is obtained by the electrolysis of (aqueous) (d) Formation of ammonia from ammonium salts :-
solution of brine. The cell has three compartments and D
NH 4 Cl + NaOH ¾¾® NaCl + NH 3 ­ + H 2 O
involves following reactions :-
(iii) Reaction with halogens:-
In outer compartment -
Anode - Graphite rods X 2 + 2 NaOH(cold ) ® NaX + NaXO + H 2O
Sodium hypohalite
Cathode - Mercury 3X 2 + 6 NaOH(hot ) ® 5 NaX + NaXO 3 + 3H 2 O
Electrolyte -Brine solution Sodium halate
Reaction - At Anode : (where X= Cl,Br or I)
- - (iv) Reaction with metals :- Less electropositive metals
Cl ® Cl + e ;
like Zn, Al and Sn etc. give H 2 gas with NaOH.
Cl + Cl ® Cl2 ­
Zn + 2 NaOH ® Na 2 ZnO 2 + H 2
The s-block Elements – Alkali Metals 277
(v) Reaction with sand:- Properties :-
2NaOH + SiO 2 ® Na 2SiO3 + H 2 O (i) Heating effect :- It gives CO 2 and Na 2 CO 3 .
sodium silicate 373K
(glass) 2 NaHCO 3 ¾¾¾® Na 2 CO 3 + CO 2 + H 2 O
(vi) Reaction with CO :- (ii) In aqueous medium it is alkaline due to hydrolysis:
150° –200°C NaHCO 3 + H 2 O ® NaOH + H 2 CO 3
NaOH + CO ¾ ¾ ¾ ¾ ¾ ® HCOONa
5 - 10 atm. sodium formate (iii) It is used as a constituent of baking powder and in medicine
(vii)Reaction with non-metals e.g.with P, Si, S, F, etc. to remove acidity of the stomach (as antacid).
D
Si + 4 NaOH ¾¾® Na 4SiO 4 + 2H 2 ­ (iv) It is present in Selidlitz powder.
D
P4 + 3NaOH + 3H 2 O ¾¾® PH 3 + 3NaH 2 PO 2
(v) Baking powder is a mixture of starch, sodium bicarbonate
sodium hypophosphite and potassium hydrogen tartarate.
S8 + 12 NaOH ¾¾®
D
4 Na 2S + 2 Na 2 S2 O3 + 6H 2 O (vi) Fire extinguishers contain
sodium sulphide sodium thiosulphate H 2SO 4 + Na 2 CO 3 + NaHCO 3
(viii) It breaks down the proteins of the skin flesh to a pasty mass (vii) Na 2SO 4 .10H 2 O is called Glauber’s salt, anhydrous
and hence it is commonly known as caustic soda.
Na 2SO 4 is called salt cake, NaNO3 is called chile salt-
SODIUM CARBONATE OR WASHING SODA
petre, NaHSO4 is called nitre cake, mixture of Na2O2 and
Na 2CO 3 .10H 2O :
dil. HCl is called oxone.
PREPARATION : (viii)When common salt is fused with a little Na2CO3 ,5% to
It is obtained by the following process - 10% Na 2SO4 and some sugar, it acquires a dark purple colour
(i) Solvay or ammonia - soda process :- In this process,NaCl and has a characteristic saline taste. It is used in medicine
(brine), ammonia and CO2 are taken as raw materials. The and is useful for digestion.It is called kala namak or black
involving reactions are salt or sulemani namak.
NH 3 + CO 2 + H 2 O ® NH 4 HCO 3 COMPOUNDS OF POTASSIUM
30 C
NH 4 HCO 3 + NaCl ¾¾ ¾®
o
NaHCO 3 ¯ + NH 4 Cl POTASSIUM HYDROXIDE,CAUSTIC POTASH, KOH
Sodium bicarbonate
o
PREPARATION :
250 C
2NaHCO 3 ¾¾ ¾® Na 2 CO 3 + H 2 O + CO 2 (i) It is prepared in a cell similar to that used for NaOH.In this
Sodium bicarbonate
cell electrolysis of an aqueous solution of KCl takes place.
2NH 4 Cl + Ca(OH) 2 ¾¾
® CaCl 2 + 2H 2 O + 2NH 3 (ii) It is also prepared by the action of soda lime
Amm. chloride Slaked lim e Cal. Chloride
(by product ) (NaOH + CaO) on potassium carbonate.
(ii) Electrolytic Process :- In this Nelson cell is used for the POTASSIUM CARBONATE, POTASH, PEARL ASH,
manufacture of NaOH, CO2 under pressure is blown with
steam K2CO3:
Preparation :- It is prepared by following two methods-
2 NaOH + CO 2 ® Na 2 CO 3 + H 2 O (i) By Leblanc process
(iii) Leblance Process:-This is now an absolute method. (ii) By Precht process (magnesia process)
Properties :-
(i) Sodium Carbonate crystallizes from water as POTASSIUM CYNAIDE, KCN:
decahydrate which effloresces on exposure to dry air Preparation :-
forming monohyrate which on heating change to (i) By heating potassium ferrocyanide with metallic potassium
anhydrous salt (soda-ash). K 4 [Fe(CN )6 ] + 2K ¾¾® 6KCN + Fe
D
dry air
Na 2 CO3 .10H 2 O ¾¾¾® Na 2 CO3 .H 2 O + 9H 2 O (ii) It is used in electroplating and due to the formation of soluble
D complexes with gold and silver, it is used in extraction of
Na 2 CO3 .H 2 O ¾¾ ® Na 2 CO3 + H 2 O these metals.
soda ash
POTASSIUM CHLORATE, KClO3 :
(ii) On hydrolysis it forms an alkaline solution
(i) Preparation :-
Na 2 CO 3 + 2H 2 O ® H 2 CO 3 + 2 NaOH
(weak acid) (strongbase) (a) By passing Cl 2 through boiling concentrated KOH
(iii) Aqueous sodium carbonate solution react with CO2 gas solution.
and forms sodium bicarbonate. 6KOH + 3Cl 2 ® 5KCl + KClO 3 + 3H 2 O
Na 2 CO 3 + H 2 O + CO 2 ® 2 NaHCO 3 (b) By the action of KCl on NaClO3 (obtained by
(iv) It is used as fusion mixture electrolysis of NaCl at 345-350K).
(Na 2 CO 3 + K 2 CO 3 ) and black ash (Na 2 CO 3 + CaS) NaClO 3 + KCl ® KClO 3 + NaCl
SODIUM BICARBONATE, BAKING SODA, (ii) It used as an oxidising agent and in the laboratory and
NaHCO3:- preparation of O 2 .
Preparation :- It is obtained as an intermediate product in Solvay KNO 3 is called Indian salt petre or nitre.
ammonia process.
278 Chemistry

Very Short/Short Answer Questions Multiple Choice Questions


1. Why are alkali metals used in photoelectric cells? 16. The alkali metals have low melting point. Which of the
2. Why is second ionisation energy of alkali metals higher following alkali metal is expected to melt if the room
than alkaline earth metals? temperature rises to 30°C?
3. Give two uses of sodium carbonate. (a) Na (b) K
4. Name the alkali metal which shows diagonal relationship (c) Rb (d) Cs
with magnesium? 17. Lithium is strongest reducing agent among alkali metals due
5. What happen when crystals of washing soda to which of the following factor?
(Na2CO3.10H2O) are exposed to air? (a) Ionization energy (b) Electron affinity
6. Why is the solution of alkali metals in liquid ammonia (c) Hydration energy (d) Lattice energy
conducting in nature? 18. The solubility of metal halides depends on their nature,
7. Sodium peroxide forms a white compound when it comes lattice enthalpy and hydration enthalpy of the individual
into contact with moist air. Explain. ions. Amongst fluorides of alkali metals, the lowest solubility
8. Give Reasons of LiF in water is due to
(i) Li & Mg show similar properties. (a) ionic nature of lithium fluoride
(ii) Alkali metals show +1 oxidation state. (b) high lattice enthalpy
(iii) Alkali metals have largest size in their period (c) high hydration enthalpy for lithium ion.
9. Mention a few difficulties involved in the extraction of (d) low ionisation enthalpy of lithium atom
sodium from fused sodium chloride. 19. Which of the following alkali metal ions has lowest ionic
10. How LiNO3 and NaNO3 differ from each other on the action mobility in aqueous solution ?
of water? (a) Rb + (b) Cs +
11. Explain why a pellet of sodium is covered with sodium
carbonate layer when exposed to atmosphere? (c) Li + (d) Na +
12. Alkali metals are paramagnetic but their salts are diamagnetic. 20. Which statement is false for alkali metals?
Explain. (a) Lithium is the strongest reducing agent
(b) Sodium is amphoteric in nature
Long Answer Questions (c) Li+ is exceptionally small
13. Give Reasons (d) All alkali metals give blue solution in liquid ammonia
(i) Alkali metals are good reducing agents. 21. Which of the following bicarbonates does not exist as solid?
(ii) Alkali metals can be obtained only by electrolysis of (a) KHCO 3 (b) NaHCO 3
their fused salts. (c) CsHCO3 (d) LiHCO3
(iii) Hydrogen is obtained on electrolysis of aqueous
22. On heating sodium metal in a current of dry ammonia gas
solution of salts of alkali metals.
the compound formed is
(iv) Alkali metals dissolve in liquid ammonia to give blue
(a) sodium nitrate (b) sodium hydride
solutions.
(c) sodium amide (d) sodium azide
(v) The blue colour of metal–ammonia solution disappears
23. In the synthesis of sodium carbonate, the recovery of
on standing.
ammonia is done by treating NH4Cl with Ca(OH)2. The by-
14. Give Reasons
product obtained in this process is
(i) Unlike alkali metal compounds , compounds of Li are
(a) CaCl2 (b) NaCl
much less soluble in water.
(c) NaOH (d) NaHCO3
(ii) Although Li has highest I.E. in group –1 but it is the
24. The formula of soda ash is
best reducing agent.
(a) Na2CO3.10H2O (b) Na2CO3.2H2O
(iii) Li differs from other members of its group.
(c) Na2CO3.H2O (d) Na2CO3
(iv) LiOH, LiNO3, Li2CO3 decompose readily on heating,
25. Sodium cannot be extracted by the electrolysis of brine
whereas the other alkali metal hydroxides & carbonates
solution because :
not decompose readily on heating.
(a) electrolysis cannot take place with brine solution.
(v) Alkali & some alkaline earth metals impart colour to
bunsen flame. (b) sodium reacts with water to produce NaOH + H 2
15. (a) Sodium fire in the laboratory should not be (c) sodium being more electropositive than hydrogen, H 2
extinguished by pouring water. Why?
(b) Why does table salt get wet in rainy season? is liberated at cathode not sodium.
(d) None of these
The s-block Elements – Alkali Metals 279

1. Which of the following is a man made element? 17. Which one of the following has highest electropositive
(a) Ra (b) Fr (c) Rn (d) Lr character?
2. Which one of these is most reactive ? (a) Cu (b) Cs (c) Ba (d) Cr
(a) Na (b) K (c) Pb (d) Mg 18. Which of the following has the highest conductivity ?
3. Alkali metals are generally extracted by
(a) reduction methods (a) Li + (b) Cs + (c) Na + (d) K +
(b) double decomposition methods 19. The metal that dissolves in liquid ammonia, giving a dark
(c) displacement methods blue coloured solution is
(d) electrolytic methods (a) tin (b) lead
4. The alkali metal which is a liquid at room temperature is (c) sodium (d) silver
(a) Fr (b) Cs (c) Rb (d) Na 20. Highly pure dil. solution of Na in liquid ammonia
5. The alkali metal which can emit its outermost electron under (a) shows blue colour
the influence of even candle light is (b) exhibits electrical conductivity
(a) Na (b) Rb (c) K (d) Cs (c) produces sodium amides
6. Francium was discoverd by (d) produces hydrogen gas
(a) Arfwedson (b) Davy 21. Which of the following alkali metal form complex hydrides
(c) Perey (d) Bunsen & Kirchoff (a) Li (b) Na (c) K (d) Rb
7. Sodium is usually kept under
22. Which of the following has the highest melting point?
(a) alcohol (b) kerosene oil
(a) NaCl (b) NaF (c) NaBr (d) NaI
(c) ammonia (d) water
23. Which of the following has lowest melting point?
8. Sodium metal cannot be stored under
(a) benzene (b) kerosene oil (a) Li (b) Na (c) K (d) Cs
(c) alcohol (d) toluene 24. With the increase in atomic weights melting points of the
alkali metals
9. Among KO 2 , AlO2- , BaO2 and NO +2 , unpaired (a) increase
electron is present in (b) decrease
(a) NO 2+ and BaO 2 (b) KO 2 and AlO -2 (c) remain constant
(d) do not show definite trend
(c) KO2 only (d) BaO 2 only
25. Lithium shows diagonal relationship with
10. The one with the largest size is (a) magnesium (b) beryllium
(a) Na (b) Na + (c) K (d) K + (c) aluminium (d) boron
11. The increasing order of atomic radius for the elements Na, 26. The electronic configuration of metal M is 1s2, 2s2 2p6, 3s1.
Rb, K and Mg is The formula of its oxide would be
(a) Na < K < Mg < Rb (b) K < Na < Mg < Rb
(a) MO (b) M 2 O (c) M 2 O3 (d) MO2
(c) Na < Mg < K < Rb (d) Rb < K < Mg < Na
12. Smallest among these species is 27. Which of the following electronic configurations corresponds
(a) hydrogen (b) helium to an element with the lowest ionization energy ?
(c) lithium (d) lithium ion (a) 1s 2 2s 2 2 p 3 (b) 1s 2 2s 2 2 p 5
13. Which of the following has largest size ?
(c) 1s 2 2s 2 2 p 6 (d) 1s 2 2s 2 2 p 6 3s1
(a) Na (b) Na + (c) Na - (d) None of these
14. Which of the following alkali metal ions has the lowest 28. Solution of sodium metal in liquid ammonia is strongly
mobility in aqueous solution ? reducing due to the presence of the following in solution
(a) sodium hydride (b) sodium amide
(a) Li + (b) Na + (c) K + (d) Cs +
(c) sodium atoms (d) solvated electrons
15. Which of the following has the least ionization potential? 29. Lithium is strongest reducing agent among alkali metals due
(a) Li (b) He (c) N (d) Zn to which of the following factor?
16. In view of their low ionization energies, the alkali metals are (a) Ionization energy (b) Electron affinity
(a) weak oxidising agents (b) strong reducing agents (c) Hydration energy (d) Lattice energy
(c) strong oxidising agents (d) weak reducing agents
280 Chemistry
30. Microcosmic salt is : (c) NaCl > KCl > LiCl > CsCl
(a) Na 2 HPO 4 .2H 2 O (b) (NH 4 )2 HPO 4 .2H 2 O (d) KCl > CsCl > NaCl > LiCl
42. The order of solubility of lithium halides in non polar solvents
(c) Na (NH 4 )HPO 4 .4H 2 O (d) None of these
follows the order :
31. Which of the following has lowest thermal stability ? (a) LiI > LiBr > LiCl > LiF
(a) Li2CO3 (b) Na 2CO3 (c) K 2CO3 (d) Rb2CO3 (b) LiF > LiI > LiBr > LiCl
32. Which one of these is basic ? (c) LiCl > LiF > LiI > LiBr
(a) CO 2 (b) SiO 2 (c) Na 2O (d) SO 2 (d) LiBr > LiCl > LiF > LiI
33. Sodium nitrate decomposes above 800°C to give 43. Which among the following is most soluble in water?
(a) N 2 (b) O2 (c) NO2 (d) Na 2O (a) CsClO4 (b) NaClO 4
34. Fires , that result from the combustion of alkali metals can be (c) LiClO4 (d) KClO 4
extinguished by 44. When sodium is treated with sufficient oxygen/air the
(a) CCl4 (b) sand product obtained is
(c) water (d) kerosene (a) Na 2O (b) Na 2 O 2 (c) NaO2 (d) NaO
35. The alkali metal that reacts with nitrogen directly to from 45. Choose the compound which does not possess a peroxide
nitride is group
(a) Li (b) Na (c) K (d) Rb (a) Na 2 O 2 (b) CrO5
36. Which of the following bicarbonates does not exist as solid?
(c) Fe 2 O 3 (d) BaO 2
(a) KHCO 3 (b) NaHCO 3 46. Which of the following alkali metals burns in air to form only
(c) CsHCO3 (d) LiHCO3 monoxide ?
(a) Na (b) Li (c) K (d) Cs
37. Which of the following is most stable?
47. Alkali metals form peroxides and superoxides except
(a) Na 3 N (b) Li 3 N (c) K 3 N (d) Rb3 N (a) Na (b) Rb (c) Li (d) Cs
38. LiNO3 on heating gives 48. Which is the most basic of the following?
(a) O2 (b) NO2 (a) Na 2O (b) BaO

(c) O 2 + NO2 (d) None of these (c) As 2O3 (d) Al2O3


39. Which of the following does not form an oxide on heating ? 49. Which of the following is used as a source of oxygen in
space capsules, submarines and breathing masks ?
(a) ZnCO 3 (b) CaCO 3
(a) Li2 O (b) Na 2 O 2 (c) KO2 (d) K 2O 2
(c) Li 2 CO 3 (d) Na 2 CO 3 50. Which of the following oxides of potassium is not known ?
40. Alkali metal hydrides react with H2O to give (a) K 2 O (b) K 2O 4 (c) KO3 (d) K 2O3
(a) basic solution (b) acidic solution 51. Nitrogen dioxide cannot be prepared by heating
(c) neutral solution (d) hydrogen gas
(a) KNO3 (b) Pb (NO 3 )2
41. The stability of the following alkali metal chlorides follows
the order (c) Cu (NO 3 )2 (d) AgNO 3
(a) LiCl > KCl > NaCl > CsCl 52. When NaCl is dissolved in water, the Na+ ions are
(b) CsCl > KCl > NaCl > LiCl (a) hydrated (b) hydrolysed
(c) oxidised (d) reduced

1. Which one of the following properties of alkali metals 2. Which of the following is known as fusion mixture?
increases in magnitude as the atomic number rises ? [CBSE-PMT 1994]
[CBSE-PMT 1989] (a) Mixture of Na2CO3 + NaHCO3
(a) Ionic radius (b) Melting point (b) Na2CO3.10H2O
(c) Electronegativity (d) First ionization energy (c) Mixture of K2CO3 + Na2CO3
(d) NaHCO3
The s-block Elements – Alkali Metals 281
3. Sodium is made by the electrolysis of a molten mixture of 9. Which of the following statements is incorrect?
about 40% NaCl and 60% CaCl2 because [CBSE-PMT 2011M]
[CBSE-PMT 1995] (a) Pure sodium metal dissolves in liquid ammonia to give
blue solution.
(a) Ca + + can reduce NaCl to Na
(b) NaOH reacts with glass to give sodium silicate
(b) Ca + + can displace Na from NaCl (c) Aluminium reacts with excess NaOH to give Al(OH)3
(c) CaCl 2 helps in conduction of electricity (d) NaHCO3 on heating gives Na2CO3
(d) this mixture has a lower melting point than NaCl 10. The ease of adsorption of the hydrated alkali metal ions on
4. Aqueous solution of sodium carbonate absorbs NO and NO2 an ion-exchange resins follows the order :
to give [CBSE-PMT 1996] (a) Li+ < K+ < Na+ < Rb+ [CBSE-PMT 2012S]
(a) CO2 + NaNO3 (b) CO2 + NaNO2 (b) Rb+ < K+ <Na+ < Li+
(c) NaNO2 + CO (d) NaNO3 + CO (c) K+ < Na+ < Rb+ < Li+
5. In crystals of which one of the following ionic compounds (d) Na+ < Li+ < K+ < Rb+
would you expect maximum distance between centres of 11. KO2 (potassium super oxide) is used in oxygen cylinders in
cations and anions? [CBSE-PMT 1998] space and submarines because it [AIEEE 2002]
(a) LiF (b) CsF (a) absorbs CO2 and increases O2 content
(c) CsI (d) LiI (b) eliminates moisture
6. In which of the following processes, fused sodium hydroxide (c) absorbs CO2
is electrolysed at a 330ºC temperature for extraction of (d) produces ozone.
sodium? [CBSE-PMT 2000] 12. The ionic mobility of alkali metal ions in aqueous solution
(a) Castner's process (b) Down's process is maximum for [AIEEE 2006]
(c) Cyanide process (d) Both (b) and (c) (a) Li+ (b) Na+
7. The sequence of ionic mobility in aqueous solution is : (c) K+ (d) Rb+
[CBSE-PMT 2008]
13. The products obtained on heating LiNO2 will be :
+ +
(a) K > Na > Rb > Cs+ +
[AIEEE 2011RS]
(b) Cs+ > Rb+ > K+ > Na+
(c) Rb+ > K+ > Cs+ > Na+ (a) Li2 O + NO2 + O2 (b) Li 3 N + O 2
(d) Na+ > K+ > Rb+ > Cs+ (c) Li 2 O + NO + O 2 (d) LiNO3 + O2
8. The alkali metals form salt-like hydrides by the direct
14. What is the best description of the change that occurs when
synthesis at elevated temperature. The thermal stability of
these hydrides decreases in which of the following orders ? Na2O(s) is dissolved in water ? [AIEEE 2011RS]
[CBSE-PMT 2008] (a) Oxide ion accepts sharing in a pair of electrons
(a) CsH > RbH > KH > NaH > LiH (b) Oxide ion donates a pair of electrons
(b) KH > NaH > LiH > CsH > RbH (c) Oxidation number of oxygen increases
(c) NaH > LiH > KH > RbH > CsH (d) Oxidation number of sodium decreases
(d) LiH > NaH > KH > RbH > CsH

1. On heating sodium metal in a current of dry ammonia gas the 3. Gun powder is
compound formed is (a) KNO3+ Charcoal + S (b) NaNO3 + KNO3 + S
(a) sodium nitrate (b) sodium hydride (c) NaNO3 + S (d) None of these
(c) sodium amide (d) sodium azide
4. Sodium carbonate is manufactured by Solvay process. The
2. The pair of compounds which cannot exits together in
products which can be recycled are
solution is
(a) CO2 and NH3 (b) CO2 and NH4Cl
(a) NaHCO3 and NaOH (b) Na2CO3 and NaHCO3
(c) Na2CO3+ NaOH (d) NaHCO3 and NaCl (c) NaCl and CaO (d) CaCl2 and CaO.
282 Chemistry
5. Which of the following statements is false for alkali metals? 14. Sodium thiosulphate, Na2S2O3. 5H2O is used in photography
(a) Lithium is the strongest reducing agent to
(b) Sodium is amphoteric in nature (a) reduce the silver bromide grains to metallic silver
(c) Li+ is exceptionally small (b) convert the metallic silver to silver salt
(d) All alkali metals give blue solution in liquid ammonia (c) remove undecomposed AgBr as soluble silver
6. Fusion of AgCl with Na2CO3 gives thiosulphate complex.
(a) Ag2CO3 (b) silver carbide (d) remove reduced silver
(c) Ag (d) Ag2O 15. The salt on heating does not give brown coloured gas is
7. A mixture of Al(OH)3 and Fe(OH)3 can be separated easily
(a) LiNO3 (b) KNO3
by treating it with :
(a) HCl (b) NH4OH (c) Pb(NO3)2 (d) AgNO3
(c) HNO3 (d) NaOH 16. Sodium carbonate solution in water is alkaline due to
8. When sulphur is heated with NaOH (aq). The compounds (a) hydrolysis of Na+
formed are
(b) hydrolysis of CO 32-
(a) Na 2S + H 2 O
(b) Na 2SO 3 + H 2 O (c) hydrolysis of both Na+ and CO 32- ions

(c) Na 2S + Na 2S 2 O 3 + H 2 O (d) None of these


17. A white solid reacts with dil.HCl to give colourless gas that
(d) Na 2S2 O 3 + H 2 O decolourises aqueous bromine. The solid is most likely to be
9. Identify the correct statement (a) sodium carbonate (b) sodium chloride
(a) Elemental sodium can be prepared and isolated by (c) sodium acetate (d) sodium thiosulphate
electrolysing an aqueous solution of sodium chloride
18. The chloride that can be extracted with ether is
(b) Elemental sodium is a strong oxidising agent
(a) NaCl (b) LiCl
(c) Elemental sodium is insoluble in ammonia
(d) Elemental sodium is easily oxidised (c) BaCl2 (d) CaCl2
10. Sodium sulphate is soluble in water whereas barium sulphate 19. Sodium thiosulphate is formed when
is sparingly soluble because (a) NaOH is neutralized by H2SO4
(a) the hydration energy of sodium sulphate is more than (b) Na2S is boiled with S
its lattice energy (c) Na2SO3 is boiled with Na2S and I2
(b) the lattice energy of sodium sulphate is equal to its (d) Na2SO4 is boiled with Na2S
hydration energy 20. Which one of the following is formed on dissolving I2 in
(c) the hydration energy of sodium sulphate is less than its
aqueous solution of KI?
lattice energy
(a) KIO4 (b) KIO
(d) None of these
11. Acidified solution of sodium thiosulphate is unstable (c) KI3 (d) KIO3
because in thiosulphate 21. Which of the following statements is correct for CsBr3?
(a) the sulphur atoms are at unstable oxidation state of + 2 (a) It is a covalent compound.
(b) the two sulphur atoms are at different oxidation states of (b) It contains Cs3+ and Br – ions.
+ 6 and – 2 (c) It contains Cs+ and Br 3– ions
(c) the S-S bond are unstable bonds. (d) It contains Cs+, and Br – and lattice Br2 molecule.
(d) sulphur is in zero oxidation state. 22. When a crystal of caustic soda is exposed to air, a liquid
12. Common table salt becomes moist and does not pour easily layer is deposited because :
in rainy season because (a) crystal loses water
(a) it contains magnesium chloride (b) crystal absorbs moisture and CO 2
(b) it contains magnesium carbonate
(c) crystal melts
(c) it melts slightly in rainy season
(d) sodium chloride is hygroscopic (d) crystal sublimes
13. Sodium chloride imparts a golden yellow colour to the 23. Sodium reacts with water less vigorously than potassium
bunsen flame . This can be interpreted due to because :
(a) low ionization potential of sodium (a) it is more electronegative.
(b) photosensitivity of sodium (b) it is a metal.
(c) sublimation of metallic sodium to give yellow vapours (c) it has higher atomic weight.
(d) emission of excess of energy absorbed as a radiation in (d) it is less electropositive.
the visible region.
The s-block Elements – Alkali Metals 283

24. If NaOH is added to an aqueous solution of Zn 2+ ions, a 27. Na 2SO 3 and NaHCO 3 may be distinguished by treating
white precipitate appears and on adding excess NaOH, the their aqueous solution with :
precipitate dissolves. In this solution zinc exists in the : (a) MgO (b) Mg SO 4
(a) both in cationic and anionic parts (c) litmus solution. (d) dil. acid
(b) there is no zinc left in the solution 28. Which hydride is most stable :
(c) cationic part (a) NaH (b) KH
(c) CsH (d) LiH
(d) anionic part.
29. A and B are two salts. A with dil. HCl and A & B with conc.
25. Sodium cannot be extracted by the electrolysis of brine
H 2SO 4 react to give reddish brown vapours, hence A & B
solution because
respectively are :
(a) electrolysis cannot take place with brine solution.
(a) NaNO3 , NaBr (b) NaBr, NaNO3
(b) sodium reacts with water to produce NaOH + H 2
(c) NaBr, NaNO 2 (d) NaNO 2 , NaBr
(c) sodium being more electropositive than hydrogen, H 2
30. When CO 2 is bubbled into an aqueous solution of
is liberated at cathode and not sodium.
Na 2 CO 3 the following is formed :
(d) None of these
26. The raw materials in Solvay Process are : (a) NaHCO 3 (b) H 2 O

(a) Na 2 CO 3 , CaCO 3 and NH 3 (c) NaOH (d) OH - .


31. Sodium peroxide in contact with moist air turns white due to
(b) Na 2SO 4 , CaCO 3 and NH 3 formation of :
(c) NaCl , NH 3 and CaCO3. (a) Na 2 CO 3 (b) Na 2 O
(c) NaOH (d) NaHCO 3
(d) NaOH, CaO and NH 3 .
284 Chemistry

EXERCISE 1
9. (c) In NO +2 odd ( unpaired ) electron is removed. In peroxides
1. Because they have low ionisation energy and can lose
electrons when light falls on them. (O 2 2– ) no unpaired electrons are present as the
antibonding pi M.O.’s acquired one more electron each
2. Alkali metals acquire, noble gas configuration after losing 1 for pairing. AlO2– containing Al3+ (2s22p6 configuration)
electron. and 2 oxide (O2–) ions each of which does not contain
3. (i) As washing soda in laundry. unpaired electron. Superoxide O2– has one unpaired
(ii) For softening of hard water. electron in pi antibonding M.O.
4. Li. 10. (c) The size increases down the group.
5. Monohydrate (Na2CO3 . H2O) is formed as a result of 11. (c) 12. (d) 13. (c)
efflorescence. 14. (a) Li+ is most highly hydrated [Li+(H2O)6]+. Therefore its
6. Due to ammoniated electrons and cations. effective size becomes greater than any other hydrated
alkali metal cation and hence has the lowest mobility .
7. It turns white due to formation of NaOH and Na2CO3.
15. (a)
10. NaNO3 is very readily soluble in water because it is ionic in
16. (b) Alkali metals are strong reducing agents and have less
nature while LiNO3 is not readily soluble in water because it value for I.P.
is covalent in nature.
17. (b) Alkali metals are more electropositive and this character
12. Alkali metals contain unpaired electrons and hence are increases down the group.
paramagnetic. But in the salts of alkali metals there are alkali 18. (b) Cs+ due to its very high ionic mobility has the highest
metal cations which don’t have unpaired electrons. Hence conductivity.
these are diamagnetic.
19. (c)
16. (d) 17. (c) 18. (b) 19. (c) 20. (b)
20. (b) Dilute solution of Na in liquid NH3 will not produce sodium
21. (d) 22. (c) 23. (a) 24. (d) 25. (c) amide and hydrogen gas instantaneously. These are
EXERCISE 2 metastable and when catalysed give hydrogen and amide.
21. (a) Li forms LiAlH4 and Na forms NaBH4 as the complex
1. (d) Lr. all transuranics ( 93 Np -103 Lr ) are man-made
hydrides besides simple hydrides of MH type.
elements.
22. (b) NaF due to its largest lattice energy has the highest
2. (b) melting point.
3. (d) Alkali metals are highly electropositive and thus highly 23. (d) Melting point of alkali metals decreases with increase in
reducing .Therefore reduction , double decomposition size.
and displacement methods for their extraction are not 24. (b)
suitable .Only electrolytic methods are useful for their 25. (a)
extraction.
26. (b) Electronic configuration indicates that the metal is
4. (a) univalent (alkali metal). Its oxide will be M2O.
5. (d) Cs, because of its low I.E. emits electron under the 27. (d) Electronic configuration of the alkali metals correspond
influence of even candle light. to lowest I.E.
6. (c) 28. (d)
7. (b) Alcohols , NH3 and H2O all react with sodium 29. (c) Li due to highest hydration energy among the alkali metals
is the strongest reducing agent.
2ROH + 2Na ® 2RONa + H 2 ; 30. (c)
31. (a) The weaker the base, the less stable is its carbonate.
Na(s) + NH3 (x + y) ® Na + (NH3 ) x + e - (NH3 ) y Since LiOH is the weakest base, hence Li2CO3 has the
lowest thermal stability.
2Na + 2H 2 O ® 2NaOH + H 2
32. (c)
Only kerosene does not react . D
33. (b) 2NaNO3 ¾¾¾¾
® 2NaNO2 + O 2
8. (c) above 800ºC
The s-block Elements – Alkali Metals 285
34. (a) In contrast, CCl4 is a low boiling highly non inflammable EXERCISE 3
heavy liquid . Its vapours surround the fire thereby 1. (a) Within a group, ionic radius increases with increase in
cutting off air supply. atomic number. The melting points decrease down the
35. (a) group due to weakning of metallic bond. The
36. (d) LiHCO3 is unstable and exists only in solution . electronegativity and the 1st ionization energy also
37. (b) Only Li3N is stable ,others are not formed at all . decreases down the group.
38. (c) LiNO3 like alkaline earth metal and heavy metal nitrates 2. (c) Mixture of K2CO3 and Na2CO3 is called fusion mixture
undergo decomposition as
3. (d) Sodium is obtained by electrolytic reduction of its
4LiNO3 ® 2Li2O + 4NO2 + O2
chloride. Melting point of chloride of sodium is high
39. (d) Na2CO3 does not decompose to form Na2O.
(803°C) so in order to lower its melting point(600°C),
40. (d)
calcium chloride is added to it.
41. (d) Stability of a compound depends upon its enthalpy of
4. (c) 2 Na 2 CO 3 + NO + 3 NO 2 ® 4 NaNO 2 + CO
formation DHf. The more negative value of DH f shows
more stability of a compound. Thus KCl is more stable 5. (c) As Cs+ ion has larger size than Li+ and I– has larger size
than LiCl , than F–, therefore maximum distance between centres of
cations and anions is in CsI.
DH f for LiCl = -408 kJ/mol
6. (a) In Castner process, for production of (Na) Sodium metal,
DH f for KCl = -436 kJ/mol Sodium hydroxide (NaOH) is electrolysed at temperature
42. (a) As the size of the anion increases from F– to I–, the 330ºC.
covalent character increase and hence the solubility in 7. (b) Smaller the ion more is its ionic mobility in aqueous
non -polar solvent increases solution. Ionic radii of the given alkali metals is in the
LiI > LiBr > LiCl > LiF order Na+ < K+ < Rb+ < Cs+ and thus expected ionic
43. (a) The high solubility of LiClO4is mainly due to high heat of mobility will be in the order Cs+ < Rb+ < K+ < Na+.
hydration of Li+ ion. However due to high degree of solvation (or hydration)
44. (b) Na in excess of O2 forms Na2O2. because of lower size or high charge density, the
hydrated ion size follows the order Cs+ < Rb+ < K+ < Na+
45. (c) Na2O2 is Na+ O– – O–Na+; Cr2 O5 is
and thus conductivity order is Cs+ > Rb+ > K+ > Na+ i.e.
O option (b) is correct answer.
O 8. (d) The stability of alkali metal hydrides decreases from Li
O
to Cs. It is due to the fact that M–H bonds become
Cr
weaker with increase in size of alkali metals as we move
O O
down the group from Li to Cs. Thus the order of stability

BaO2 is Ba 2+ ( -
)
O - O - while Fe2 O3 consists of
of hydrides is
LiH > NaH > KH > RbH > CsH
only Fe3+ and O2 - ions. Thus Fe2 O3 does not contain i.e. option (d) is correct answer.
9. (c) 2Al(s) + 2NaOH (aq) + 2H2O (l) ¾¾
® 2NaAlO2 + 3H2
a peroxide - O - O- linkage. sod. meta aluminate
46. (b) 10. (b) All alkali metal salts are ionic (except Lithium) and soluble
47. (c) Li + does not allow its O 2 - ion to combine with other O in water due to the fact that cations get hydrated by
atom(s) to form peroxides and superoxides . water molecules. The degree of hydration depends upon
48. (a) the size of the cation. Smaller the size of a cation, greater
49. (c) Because KO2 not only provides O2 but also removes is its hydration energy.
CO2 as follows Relative ionic radii :

4KO 2 + 2CO2 ® 2K 2 CO3 + 3O 2 Cs + > Rb+ > K + > Na + > Li +


4KO 2 + 4CO2 + 2H 2 O ® KHCO3 + 3O 2 Relative degree of hydration:

Li + > Na + > K + > Rb + > Cs +


50. (b) O 24 - ion is not possible and K 2 O4 is unknown .
11. (a) 4KO2 + 2 CO2 ® 2 K2CO3 + 3O2 .
51. (a) Only nitrates of heavy metals and lithium decompose on
KO2 is used as an oxidising agent. It is used as air
heating to produece NO 2 . purifier in space capsules. Submarines and breathing
52. (a) masks as it produces oxygen and remove carbon dioxide.
286 Chemistry
12. (d) Smaller the size of cation higher is its hydration energy 17. (d) It must be sodium thiosulphate
and greater is its ionic mobility hence the correct order Na 2S 2 O 3 + 2HCl ® 2 NaCl + SO 2 + S + H 2 O
is Li+ < Na+ < K+ < Rb+
SO 2 + Br2 + 2H 2 O ® H 2SO 4 + 2HBr
13. (a) 4LiNO 3 ® 2Li 2 O + 4NO 2 + O 2
18. (b) LiCl is covalent in nature . It is, thus soluble in polar
14. (b) solvents.
EXERCISE 4
19. (c) Na 2 S + I 2 + Na 2SO 3 ® Na 2S 2 O 3 + 2 NaI
1. (a) Na + NH3 ® NaNH 2 + 1/ 2 H 2 20. (c) Iodine is soluble in water in presence of KI forming a
Sodium
amide complex I 2 + KI ® KI 3
2. (a) NaHCO3 + NaOH ® Na 2 CO3 + H 2 O 21. (c) CsBr3 may be represented as Cs+Br3–
22. (b) It is hygroscopic and deliquescent. It absorbs moisture
(NaHCO3 is an acidic salt)
and CO 2 to form Na 2 CO 3
3. (a) Mixture of KNO 3 + Charcoal + S is gun powder . It is an
explosive 2NaOH + CO 2 ¾ ¾® Na 2 CO 3 + H 2 O
4. (a) CO2 and NH3 formed are reused (See Solvay process) 23. (d) Electropositive character and reactivity increases down
5. (b) Na is basic in nature the group.

6. (c) 2AgCl + Na 2 CO 3 ® Ag 2 CO 3 + 2 NaCl. 24. (d) Zn + + + 2NaOH ¾


¾® Zn (OH ) 2 + 2 Na +
D
Ag 2 CO 3 ¾¾® 2Ag + CO 2 + 1 / 2 O 2 Zn (OH ) 2 + 2 NaOH ¾
¾® Na 2 ZnO 2 + 2H 2 O
7. (d) Al(OH)3 is soluble in NaOH and Fe(OH)3 is insoluble.
Hence it can be separated by NaOH ¾® 2 Na + + ( ZnO 2 ) 2 -
Na 2 ZnO 2 ¾
8. (c) 4S + 6NaOH ® Na 2S2 O3 + 2 Na 2S + 3H 2 O 25. (c) The statement c is correct.
9. (d) Elemental sodium is easily oxidised ( has low I.P.) and 26. (c) NaCl (brine), NH 3 and CO 2 are raw materials. CaCO 3
acts as reductant.
is source of CO 2 .
10. (a) If lattice energy is less than hydration energy, the ionic
compound will be soluble in water
27. (b) 2Mg SO 4 + 2 Na 2 CO 3 + H 2 O ¾
¾®
11. (b) Na 2S 2 O 3 + 2HCl ® 2 NaCl + SO 2 + S + H 2 O
(Disproportionation) Mg CO 3 .Mg (OH ) 2 + 2 Na 2SO 4 + CO 2
12. (a) The table salt contains the impurity of MgCl2 which is Basic magnesium carbonate
hygroscopic in nature. Pure NaCl never moists 28. (d) The basic character and stability of hydrides decrease
13. (d) The electrons are excited to higher energy level and on down the group.
return to original state they give out visible light 29. (d) Nitrites gives NO 2 (brown) with dil. acids. The nitrites
14. (c) The unreacted AgBr is removed by soluble complex
formation and bromides give brown vapours of NO 2 and Br2
with conc. acids.
AgBr + 2 Na 2S 2 O 3 ® Na 3 [Ag(S 2 O 3 ) 2 ] + NaBr
15. (b) Potassium nitrate gives oxygen on decomposition 30. (a) Na 2 CO 3 + CO 2 + H 2 O ¾
¾® 2 NaHCO 3
2KNO 3 ® 2KNO 2 + O 2 1
31. (c) Na 2 O 2 + H 2 O ¾
¾® 2 NaOH + O2
All other nitrates give NO2 (brown vapours) and oxygen 2
16. (b) Due to anionic hydrolysis
CO32- + H 2 O ƒ HCO3– + -OH
10B

The s-block Elements –


Alkaline Earth Metals
The elements of group 2 are (iii) Density:
Be - Beryllium Mg - Magnesium Ca - Calcium (a) These are much denser than alkali metals because of
Sr - Strontium Ba - Barium Ra - Radium (Radioactive) their smaller size and greater nuclear charge.
These metals are known as alkaline earth metals as their oxides are (b) The density, however, first decreases from Be to Ca and
alkaline and occur in earth crust. then steadily increases from Ca to Ra due to difference
Radium was discovered from the ore Pitch blende by Madam Curie. in crystal structure
It is used in the treatment of cancer.
Element Be Mg Ca Sr Ba Ra
These metals do not occur in the native form (i.e., do not ocur in
free state). Density (g cm–3) 1.84 1.74 1.55 2.54 3.75 6.00
(iv) Melting and Boiling points:
GENERAL CHARACTERISTICS (i.e., PHYSICAL
PROPERTIES) OF ALKALINE EARTH METALS: (a) These have higher melting and boiling points than those
of alkali metals because the number of bonding electrons
(i) Electronic configuration: Like alkali metals, these are s-
in alkaline earth metals is two.
block elements, and have two electrons in the valence shell
in s-orbital. Hence their electronic configuration may be (b) The melting and boiling points decrease down the group
represented as [noble gas] ns2 where ‘n’ represents the with the exception of magnesium.
valence shell. (c) Melting points of halides decrease as the size of the
halogen increases. The correct order is
Element Atomic No. Electronic Valence Shell
Configuration configuration MF2 > MCl2 > MBr2 > MI2
Be 4 [He] 2s2 2s 2 (v) Metallic properties: They are silvery white metals, soft in
Mg 12 [Ne] 3s2 3s 2 nature but harder than alkali metals due to stronger metallic
Ca 20 [Ar] 4s2 4s 2 bonding.
Sr 38 [Kr] 5s2 5s 2 (vi) Atomic volume: Atomic volume of these metals increases
Ba 56 [Xe] 6s2 6s 2 considerably on moving from Be to Ra as the atomic radius
Ra 88 [Rn] 7s2 7s 2 increases.
(ii) Size of the atoms and ions (Atomic radii and ionic radii): (vii) Ionization Energy:
(a) The atomic radii of these elements are quite large but (a) The first I.E. of alkaline earth metals are higher than those
smaller than those of the corresponding elements of of the corresponding alkali metals due to smaller size
group 1, due to increased nuclear charge of these and higher nuclear charge.
elements which tends to draw the orbital electrons (b) The second I.E. values are higher than their first I.E.
inwards. values but much lower than the second I.E. values of
(b) The ionic radii are also large but smaller than those of alkali metals.
the alkali metals.
(c) On moving down the group due to increase in atomic
(c) The atomic as well as ionic radii go on increasing down size the magnitude of I.E. decreases.
the group due to the gradual addition of extra energy
(d) The ionization potential of radium is higher than that of
level and also because of the screening effect.
barium.
288 Chemistry
(viii)Electropositive character: (ii) Reaction with oxygen:
(a) These are strong electropositive elements due to their The affinity for oxygen increases down the group. Thus, Be,
large size and comparatively low ionisation energies. Mg and Ca when heated with O2 form monoxides while Sr, Ba
(b) On moving down the group, the electropositive character and Ra form peroxides.
increases due to increase in atomic radii.
D
(ix) Oxidation state: 2M + O 2 ¾¾® 2MO (M = Be, Mg or Ca )
Metal oxide
(a) Alkaline earth metals uniformly show an oxidation state
of +2 despite the presence of high ionisation energy D
because M + O 2 ¾¾® MO 2 (M = Ba, Sr or Ra )
Metal peroxide
Þ In the solid state, the dipositive ions M2+ form strong
lattices due to their small size and high charge (i.e., high (iii) Reaction with acids:
lattice energy). (a) Alkaline earth metals except Be, displace H2 from acids.
Þ In the aqueous solution, the M2+ cations are strongly
M + H 2SO 4 ¾
¾® MSO 4 + H 2
hydrated due to their small size and high charge. The
hydration energy released by the M2+ cation is very (where M = Mg, Ca, Sr or Ba)
large. (b) Reactivity, however, increases down the group from Mg
(b) The divalent ions are diamagnetic and colourless due to to Ba i.e.,
the absence of upaired electron. Mg < Ca < Sr < Ba
(x) Conductivity: These are good conductors of heat and (c) Only Mg displaces H2 from a very dilute HNO3.
electricity due to the presence of two loosely held valence (iv) Reaction with hydrogen:
electrons. (a) Except Be, all other elements combine with hydrogen on
(xi) Flame colouration: heating to form hydride (MH2).
(a) Like alkali metal salts, alkaline earth metal salts also impart
M + H2 ¾
¾® MH 2 ( where M = Mg , Ca , Sr or Ba )
characteristic flame colouration.
(b) As we move down the group from Ca to Ba, the ionisation (b) The hydride of beryllium can be prepared indirectly by
reducing beryllium chloride with lithium aluminium
energy decreases, hence the energy or the frequency of
hydride.
the emitted light increases. Thus,
Ca Sr Ba Ra 2BeCl 2 + LiAlH 4 ¾
¾® 2BeH 2 + LiCl + AlCl 3
Brick red Crimson red Apple green Crimson (c) BeH2 and MgH2 are covalent and polymeric whereas
(c) Be and Mg because of their high ionization energies, the hydrides of Ca, Sr and Ba are ionic and monomeric in
however, do not impart any characteristic colour to the nature.
bunsen flame. (d) CaH2 is also called hydrolith.
CHEMICAL PROPERTIES: (e) All the hydrides react with water to evolve H2 and thus
(a) Alkaline earth elements are quite reactive due to their low behave as strong reducing agents.
ionisation energies but are found to be less reactive than MH 2 + 2H 2 O ¾
¾® M (OH ) 2 + 2H 2
alkali metals because the alkaline earth metals have
(v) Reaction with halogens:
comparatively higher ionisation energy.
(a) All the elements of group 2 combine with halogens at
(b) Reactivity of the group 2 elements increases on moving down
high temperature and form their halides (MX2).
the group because their ionisation energy decreases.
D
(i) Reaction with water: M + X 2 ¾¾® MX 2
(a) Group 2 elements are less reactive with water as compared (b) Beryllium halides (BeF2, BeCl 2 etc.) are covalent,
to alkali metals. They react with H2O evolving H2 gas. hygroscopic and fume in air due to hydrolysis. The
halides of other alkaline earth metals are fairly ionic and
M + 2H 2 O ¾
¾® M (OH ) 2 + H 2 this character increases as the size of the metal increases.
(c) The halides are soluble in water and their solubility
where M = Mg, Ca , Sr or Ba
decreases in the order:
(b) The chemical reactivity of the metal with H2O, however MgX2 > CaX2 > SrX2 > BaX2
increases as we move from Mg to Ba, i.e., Be does not (d) BeF2 is very soluble in water due to the high solvation
react even with boiling water and Ba react vigorously
even with cold water. Thus, increasing order of reactivity energy of Be 2+ in forming [Be(H 2 O) 4 ]2 + but the
with water is fluorides of other alkaline earth metals have high melting
Mg < Ca < Sr < Ba point and they are insoluble in water.
The s-Block Elements – Alkaline Earth Metals 289
(e) BeCl2 has a polymeric structure in the solid state but (b) Both Mg2+ and Ca2+ form six coordinate complexes with
exists as a dimer in the vapour state and as a monomer at EDTA (ethylenedi-aminetetracetic acid) which are used
1200 K. to determine the hardness of water.
(vi) Reaction with nitrogen: These metals burn in nitrogen to (c) Beryllium due to small size forms complexes of type
form nitrides of the types M3N2 which are hydrolysed with [BeF3]–, [BeF4]–2 [Be (H2O)4]2+.
water to evolve NH3. (d) Mg exists as a natural complex, chlorophyll where it is
3M + N 2 ¾
¾® M 3 N 2 complexed with pyrole rings of porphyrin.
(xi) Basic strength of oxides and hydroxides:
M 3 N 2 + 6H 2 O ¾ ¾® 3M (OH ) 2 + NH 3
(a) BeO and Be(OH)2 are amphoteric while the oxides and
(a) The ease of formation of nitrides increases from Be to hydroxides of other alkaline earth metals are basic. The
Ba. (Be3N2) is volatile in nature. basic strength, however, increases from Be to Ba as the
(b) Anhydrous CaCl 2 is a good drying agent due to ionisation energy of metal decreases down the group
hygroscopic nature (CaCl2.2H2O) and cannot be used thus the order:
to dry alcohol or ammonia as it forms addition products
BeO < MgO < CaO < SrO < BaO and
with them.
Be(OH)2 < Mg(OH)2 < Ca(OH)2 < Sr(OH)2 < Ba(OH)2
(vii) Reaction with Carbon:
(b) The basic character of hydroxides of group-2 elements
(a) When heated with carbon, these form their respective
is lesser than those of group-1 hydroxides because of
carbides of the general formula MC2 (except Be) and are
the larger size elements of latter than former group.
called acetylides containing the discrete C 22- anion. (c) Aq. Ba(OH)2 is known as baryta water.
D (xii)Thermal stabilities and Nature of Bicarbonates and
M + 2C ¾¾® MC 2
Carbonates:
( where M = Mg, Ca , Sr or Ba ) (a) Bicarbonates of these metals do not exist in solid state
(b) Under these conditions beryllium, however, forms Be 2C but are known in solution only, when these solutions
called methanide containing the discrete C4– anion. are heated, these get decomposed to evolve CO2.
(c) All these carbides are ionic in nature and react with H2O
D
to form acetylene (except Be2C which gives methane). M ( HCO 3 ) 2 ¾¾® MCO 3 + CO 2 + H 2 O
(b) The carbonates of alkaline earth metals can be regarded
CaC 2 + 2H 2 O ¾
¾® Ca (OH ) 2 + HC º CH as salts of weak carbonic acid (H 2CO3) and metal
hydroxide, M(OH)2. The carbonates decompose on
Be 2 C + 2H 2 O ¾
¾® 2BeO + CH 4 heating form metal oxide and CO2.
or Be 2 C + 4H 2 O ¾
¾® 2Be(OH ) 2 + CH 4 D
MCO 3 ¾¾® MO + CO 2
(d) On heating MgC2 gives Mg2C3 called allylide which
(c) The stability of carbonates and bicarbonates increases
contains the disrete C 34- anion and gives allylene down the group.
(methyl acetylene) on hydrolysis. BaCO 3 > SrCO 3 > MgCO 3 > BeCO 3
Mg 2 C 3 + 4H 2 O ¾
¾® 2Mg (OH ) 2 + C 3 H 4 (d) Carbonates and sulphates of Ca and Mg are responsible
(viii)Reducing character: for permanent hardness of water while their bicarbonates
(a) All the alkaline earth metals, because of their low cause temporary hardness.
electrode potentials, are strong reducing agents but (xiii)Solubility of the salts: The solubility of a salt in water
these are weaker than the corresponding alkali metals. depends upon two factors.
(b) As we move down the group from Be to Ra, the reducing (1) Lattice energy: Higher the magnitude of lattice energy,
character increases due to decreasing I.E. from Be to Ra. lesser will be the solubility of the salt in the given solvent.
(ix) Solubility in liquid ammonia: (2) Hydration energy: Higher the magnitude of hydration
(a) Like alkali metals, these dissolve in liquid ammonia giving energy, higher will be the solubility of the salt in water
coloured solutions. (solvent).
(b) The tendency to form ammoniates decreases with (a) Solubility of hydroxides: As the ionic size of group 2
increase in size of the metal atom (i.e., on moving down metals increases from Be to Ba, the lattice energy
the group). decreases from Be to Ba, as follows:
(x) Complex formation: Be(OH) 2 < Mg (OH) 2 < Ca (OH) 2 < Sr (OH) 2 < Ba (OH) 2
(a) Complex formation is favoured in case of alkaline earth (b) Solubility of sulphates: The solubility of sulphates of
metals because of their small sizes as compared to the
alkaline earth metals decreases as we move down the
alkali metals.
group from Be to Ba due to the reason that ionic size
290 Chemistry
increases down the group. The lattice energy remains (iv) With conc. HNO3, both are rendered passive due to the
constant because sulphate ion is so large, so that small formation of a thin film of their respective oxides on the metal
change in cationic sizes do not make any difference. surface.
Thus the order: (v) Both react with conc. NaOH liberating H2.
BeSO 4 > MgSO 4 > CaSO 4 > SrSO 4 > BaSO 4
Be + 2 NaOH ¾
¾® Na 2 BeO 2 + H 2
The negligible solubility of BaSO4 in water is used in Sodium berylate
both qualitative and quantitative analysis.
(c) Solubility of carbonates: The solubility of the 2Al + 2 NaOH + 2H 2 O ¾
¾® 2 NaAlO2 + 3H 2
carbonates in water decreases down the group due to Sodium metaaluminate
the decrease in the magnitude of hydration energy. (vi) Oxides and hydroxides of both are amphoteric in nature.
However these insoluble metal carbonates are dissolved (vii) Carbides of both liberate methane on hydrolysis.
in water having CO2 as shown:
Be 2 C + 4H 2 O ¾
¾® CH 4 + 2 Be(OH ) 2
MgCO3 (s) + CO 2 (g) + H 2 O ¾¾
® Mg(HCO3 ) 2
Insolub le Magnesium bicarbonate Al 4 C 3 + 12H 2 O ¾
¾® 4 Al(OH ) 3 + 3 CH 4
(Soluble)
(viii)Anyhydrous chlorides of both i.e., BeCl2 and AlCl3 act as
Mg (HCO3)2 Mg2+ (aq) + 2HCO3– (aq.)
Lewis acids and dissolve in organic solvents.
The order of solubility of carbonates:
(ix) Both do not impart any colour to the flame.
BeCO3 > MgCO3 > CaCO3 > SrCO3 > BaCO3
Magnesium along with KClO3 or BaO2 is used in photography
ANOMALOUS BEHAVIOUR OF BERYLLIUM : flash blubs, fire works and as a deoxidiser in metallurgical process.
Beryllium, the first member of group 2 differs from the rest of the MgCl2.5MgO xH2O is called Sorel’s cement or Magnesia cement
members of its group due to the following reasons: and used to fill the cavities of teeth.
(i) It has a small atomic size as well as small ionic size. Mg(OH)2 in water is used in medicine as an antacid under the
(ii) It has no vacant d-orbitals in valence shell. name ‘Milk of Magnesia’, while 12 gm of MgCO3 per 100 c.c. of
(iii) It has high electronegativity value. H2O containing CO2 is known as ‘Fluid Magnesia’.
(iv) It has a high charge density. The finely divided BaSO4 is called Blanc fire and used in paints.
(v) Its hydration energy is high. Suspension of slaked lime in water is called white wash (milk of
Some points of difference are: lime).
(i) Be is harder and denser than other members of the group. A solution of MgCl2 + NH4Cl in ammonia is known as Magnesia
(ii) The m.p., b.p., and ionisation energy of Be are the highest of Mixture.
all the alkaline earth metals. Plaster of paris CaSO4.1/2 H2O is used in surgery for setting broken
(iii) Be does not react with water even at higher temperature bones.
where as other metals do. Pure Ca(H2PO4)2 is used as American baking powder.
(iv) BeO and Be(OH)2 are amphoteric in character whereas oxides Gypsum gives different products on heating as
and hydroxides of the group 2 metals are basic.
120ºC Plaster of paris [CaSO41/2 H2O]
(v) Be is least metallic of all the alkaline earth metals and forms
covalent compounds.
(vi) Be forms nitride Be3N2 with nitrogen which is volatile while CaSO4.2H2O 200ºC Dead burnt plaster [CaSO4]
nitrides of others are non-volatile. Gypsum 400ºC CaO, SO2, O2
(vii)Be does not liberate H2 from acids (HCl, H2SO4) where as
other metals do. Most abundant alkaline earth metal in the earth’s crust is Ca.
(viii)Be forms Be2C with carbon while the other members of the Be and Mg crystallize in hcp, Ca and Sr in ccp and Ba in bcc
group form ionic carbide MC2. structures.
Because of comparatively higher electronegativity both Be and
DIAGONAL RELATIONSHIP OR RESEMBLANCE
Mg form a large number of organometallic compounds.
BETWEEN Be AND Al :
CaCl2.6H2O is widely used for melting ice on roads, particularly in
The first member of group-2, Beryllium, shows similarities in the very cold countries, because a 30% eutectic mixture of CaCl2/H2O
properties with its diagonally opposite member aluminium of the freezes at –55ºC as compared with NaCl/H2O at –18ºC.
next group 13 of the next higher period, due to the similar polarizing
Magnesium perchlorate, Mg(ClO4)2 is known as anyhydrone and
power. i.e., ionic charge/(ionic radius)2 of Be and Al.
used as drying agent.
(i) Both the metals are stable in air.
Mostly kidney stones containing calcium oxalate, CaC2O4.H2O
(ii) Both have a strong tendency to form covalent compounds.
which dissolves in dil. strong acids but remains insoluble in bases.
(iii) Both form fluoro complex anions BeF42– and AlF63– in solution.
The s-Block Elements – Alkaline Earth Metals 291
METALLURGY OF MAGNESIUM MAGNESIUM CARBONATE, MAGNESITE, MgCO3:
OCCURRENCE AND IMPORTANT MINERALS : (i) It dissolves in water in the presence of CO2.
(i) Magnesium occurs in the combined state in nature and it is MgCO 3 + H 2 O + CO 2 ¾
¾® Mg ( HCO 3 ) 2
the essential constituent of chlorophyll, the green colouring
(ii) Its 12% aqueous solution is known as fluid magnesia and is
matter of the plants.
used as an antacid, laxative and in tooth pastes.
(ii) The important minerals of magnesium are.
(a) Magnesite, MgCO3 MAGANESIUM CHLORIDE, MgCl2.6H2O :
(b) Dolomite, MgCO3.CaCO3 It is a deliquescent, white crystalline solid.
(c) Carnallite, KCl.MgCl2.6H2O Heat
MgCl 2 + 6H 2 O ¾¾¾® Mg (OH ) Cl + HCl + 5H 2 O
(d) Epsom salt, MgSO4.7H2O
(e) Asbestos, CaMg3 (SiO3)4 METALLURGY OF CALCIUM
(f) Talc, Mg2(Si2O5)2.Mg(OH)2
OCCURRENCE AND IMPORTANT MINERALS :
EXTRACTION : (i) It is an important constitutent of bones and teeth (as calcium
It is extracted by the electrolysis of fused mixture of magnesium phosphate), sea shells and corals (as calcium carbonate).
chloride (which is obtained from carnallite and magnesite), NaCl (ii) The important minerals are
and CaCl2 (added to provide conductivity to the electrolyte and (a) Limestone, marble, chalk or calcite, CaCO3
to lower the fusion temperature of anhydrous MgCl2) at 700ºC in (b) Dolomite, MgCO3.CaCO3
Dow’s process. In Dow’s process, MgCl2 is obtained from sea (c) Gypsum, CaSO4.2H2O
water as MgCl2.6H2O which can be changed to anhydrous MgCl2 (d) Fluorspar, CaF2
only by passing dry HCl gas through it because even by strong (e) Anhydrite, CaSO4
heating it gets hydrolysed by its own water of crystallisation.
(f) Hydroxyapatite, 3Ca3(PO4)2.Ca(OH)2
MgCl 2 .6H 2 O + HCl ¾
¾® MgCl 2 + 6H 2 O + HCl (g) Phosphorite, Ca3(PO4)2.
Strong EXTRACTION :
MgCl 2 .6H 2 O ¾¾ ¾
¾® MgO + 2HCl + 5H 2 O
Heating It is extracted by the electrolysis of a fused mixture of calcium
Anhydrous MgCl2 is fused with anhydrous NaCl and CaCl2 and chloride and calcium fluoride (lowers the fusion temperature of
electrolysed at 700ºC. the electrolyte).

¾® Mg 2 + + 2Cl -
MgCl 2 ¾
USES OF CALCIUM :
(i) It is used to remove air from vaccum tubes, sulphur from
At Cathode : Mg2+ + 2e- ¾¾
® Mg petroleum and oxygen from molten steel.
(ii) It is used as a reducing agent in the extraction of such metals
At anode: Cl- ¾¾
® Cl + e - ; Cl + Cl ¾¾
® Cl2 ­ from their oxides where carbon is ineffective.
USES OF MAGNESIUM : COMPOUNDS OF CALCIUM
(i) Mg being a light metal forms alloys with Al and Zn which are
CALCIUM OXIDE, QUICK LIME, BURNT LIME, LIME,
used in aircraft construction. e.g., elektron (95% Mg + 5%
CaO :
Zn) used in construction of aircraft, magnalium (1-15% Mg +
85-99% Al) used in construction of aircraft and light Preparation:
instruments. (i) By the thermal decomposition of calcium carbonate.
(ii) Magnesium powder is used in flash bulbs used in 1070 -1270 K
CaCO 3 ¾¾ ¾ ¾ ¾
¾® CaO + CO 2 ; DH = +1080 kg
photography.
Properties:
COMPOUNDS OF MAGNESIUM (i) It is a basic oxide.
MAGNESIUM OXIDE, MAGNESIA, MgO : (ii) Its aqueous suspension is known as slaked lime Ca(OH)2.
With MgCl2, it forms a mixture of composition MgCl2.5MgO.xH2O hissing sound
CaO + H 2 O ¾¾¾¾¾ ® Ca(OH) 2 + Heat
which is known as Sorel’s cement or magnesia cement. Burnt lime Slaked lime
MAGNESIUM HYDROXIDE, MILK OF MAGNESIA, (iii) On heating with ammonium salts it gives ammonia
Mg(OH)2: D
Its aqueous suspension is used in medicine as an antacid. CaO + 2 NH 4 Cl ¾¾® CaCl 2 + 2 NH 3 + H 2 O
(iv) It reacts with carbon to form calcium carbide.
MAGNESIUM SULPHATE OR EPSOM SALT,
MgSO4.7H2O: CaO + 3C ¾¾
® CaC 2 + CO
It shows isomorphous nature with ZnSO4.7H2O, deliquesence Calcium carbide
and efflorescence. It is used as a purgative in medicine and as a (v) It is used as basic flux, for removing hardness of water for
stimulant to increase the secretion of bile. preparing mortar (CaO + Sand + Water).
292 Chemistry
CALCIUM HYDROXIDE, SLAKED LIME, LIME WATER, (ii) On heating at about 200ºC, it also forms dead burnt plaster of
Ca(OH)2 : paris (it has no tendency to set).
Preparation: CALCIUM CARBIDE OR CALCIUM ACETYLIDE, CaC2:
(i) By dissolving quick lime in water. Preparation:
CaO + H 2 O ¾¾ ® Ca(OH) 2 ; DH = -63kJ (i) By heating a mixture of quick lime (CaO) and powdered coke
in an electric furnace at 3300K.
Properties:
(i) Its suspension in water is known as milk of lime. 3300K
CaO + 3C ¾¾ ¾ ¾® CaC 2 + CO
(ii) It gives CaCO3 (milky) and then Ca(HCO3)2 with CO2.
Properties:
Ca(OH)2 + CO2 ¾¾
® CaCO3 + H 2O (i) It reacts with water to form acetylene.
Lime water (Milkiness)
CaC 2 + 2 H 2 O ¾
¾® Ca ( OH ) 2 + HC º CH

CaCO3 + H 2 O + CO 2 ¾
¾® Ca (HCO3 ) 2 (ii) When heated with nitrogen, it forms calcium cyanamide
(Excess) (Soluble) which on reaction with steam under pressure gives NH3.
(iii) It reacts with Cl2 to give bleaching powder CaOCl2. 1373K
CaC 2 + N 2 ¾¾ ¾
¾®CaCN 2 + C
14243
Ca (OH ) 2 + Cl 2 ¾
¾® CaOCl 2 + H 2 O Nitrolim

CALCIUM CHLORIDE, CaCl2.6H2O: CaCN 2 + 3H 2O ¾¾¾¾® CaCO3 + 2NH3


Pr essure
(i) It is a deliquescent solid which is a by-product of Solvay’s (Steam)
process.
(iii) Nitrolim (a mixture of calcium cyanamide and carbon) is used
(ii) Fused Calcium chloride is a good dessicant (drying agent),
as a fertilizer.
but it can not be used to dry alcohol or ammonia as it forms
addition product with them. BLEACHING POWDER, CaOCl 2 (CALCIUM
CALCIUM CARBONATE, LIMESTONE, MARBLE, HYPOCHLORITE, CHLORIDE OF LIME):
CHALK, SLATE, CALCITE, CaCO3: Preparation:
Preparation: By passing a current of chlorine over dry slaked lime.
(i) By passing CO2 through lime water.
Ca (OH ) 2 + Cl 2 ¾
¾® CaOCl 2 + H 2 O
Ca (OH ) 2 + CO 2 ¾
¾® CaCO 3 ¯ + H 2 O Manufacture : The manufacture of bleaching powder is carried
Properties: out in
(i) It is insoluble in H2O but dissolves in the presence of CO2, (i) Hasenclever plant or (ii) Bachmann's plant
due to the formation fo calcium bicarbonate.
Properties :
CaCO 3 + H 2 O + CO 2 ¾
¾® Ca (HCO 3 ) 2 (i) It is a mixture (mixed salt) of calcium hypochlorite
Insoluble Soluble (Ca.(OCl) 2 .4H 2 O) and basic calcium chloride
GYPSUM, CALCIUM SULPHATE DIHYDRATE, (CaCl2.Ca(OH)2.H2O).
CaSO4.2H2O : (ii) Its aqueous solution gives Ca2+, Cl– and OCl– ions.
(i) It is naturally occurring calcium sulphate and also known as (iii) With dil. H2SO4, it gives nascent oxygen which causes its
alabaster. oxidising and bleaching power.
(ii) On heating at 390K, it gives plaster of paris.
2CaOCl 2 + H 2SO 4 ¾
¾® CaCl 2 + CaSO 4 + 2HClO
(iii) It is added to cement to slow down its rate of setting.
PLASTER OF PARIS, CALCIUM SULPHATE HClO ¾¾® HCl + O
HEMIHYDATE, CaSO4.1/2 H2O:
(iv) With excess of dil. H2SO4 (or CO2), it forms Cl2 known as
(i) When it is mixed with water, it forms first a plastic mass available chlorine.
which sets into a solid mass with slight expansion due to
rehydration and its reconversion into gypsum. CaOCl 2 + H 2SO 4 ¾
¾® CaSO 4 + H 2 O + Cl 2
1 1 setting
CaSO 4 . H 2 O + 1 H 2 O ¾¾ ¾ ¾® CaSO 4 .2H 2 O CaOCl 2 + CO 2 ¾
¾® CaCO 3 + Cl 2
2 2 Gypsum (orthorhombic)
Plaster of paris
The average percentage of available chlorine is
¯ hardening 35 - 40%. Theoretically it should be 49%, which diminishes
CaSO 4 .2H 2 O on keeping the powder due to following change
Gypsum ( monoclinic)
6CaOCl 2 ® 5CaCl 2 + Ca (ClO 3 ) 2
The s-Block Elements – Alkaline Earth Metals 293
Available chlorine is estimated by Ratio of Silica and alumina
(a) Arsenite method (Penot's method)
% of SiO 2
Na 3 AsO 3 + CaOCl 2 ® Na 3 AsO 4 + CaCl 2 = 2.5 - 4
% of Al 2 O 3
(b) Iodometric method (Bunsen and Wagner's method)
Ratio of CaO and 6(SiO2 + Al2O3 + Fe2O3)
CaOCl 2 + 2CH 3COOH ® Ca (CH 3COO) 2 + Cl 2 + H 2 O % of CaO
2KI + Cl 2 ® 2KCl + I 2 = 1 .9 - 2 .1
% of SiO 2 + % of Al 2 O 3 + % of Fe 2 O 3
2 Na 2S2 O 3 + I 2 ® Na 2S 4 O 6 + 2 NaI
White Cement : It does not contain ferric oxide
(v) It gives O2 in presence of catalyst COCl2.
Process : Two processes are employed
COCl
2CaOCl2 ¾¾¾®
2
2 CaCl2 + O2 (i) Wet process (ii) Dry process
Uses : It is used for bleaching, as disinfectant and germicide in Raw material : Lime and Clay
sterlisation of water, for making wool unshrinkable and in the Manufacture :
manufacture of Chloroform. D Gypsum
Clay + lime ¾¾ ® Cement clinker ¾¾¾¾
strong 2 -3%
® Cement
CEMENT : by weight
Cement is essentially a mixture of complex silicates and aluminates
of Ca containing less than 1.0% free lime and some gypsum Gypsum regulates the setting time
(CaSO4.2H2O) Setting of Cement : When mixed with water, the cement forms a
Composition : An approximate composition is as follows : gelatinous mass sets to hard mass when three dimensional cross
links are formed between ... Si-O-Si---and ---Si-O-Al--- chains.
1. Lime CaO 60-69% 62%
The reactions involved in the setting of cement are :
2. Silica SiO2 17-25% 22%
(i) Hydration : Hydration of 3CaO.Al2O3 and 2CaOSiO2 forming
3. Alumina Al2O3 3-8% 7.5% colloidal gel.
4. Magnesia MgO 1-5% 2.5% (ii) Hydrolysis : Hydrolysis of 3CaOAl2O3 and 3CaO.SiO2
5. Iron oxide Fe2O3 0.5-5% 2.5% forming precipitates of Ca(OH)2 and Al(OH)3
6. Sulphur trioxide SO3 1-3% 1.5% Fly ash : A waste product of steel industry possess properties
similar to cement. It is added to cement to reduce its cost.
7. Sodium oxide Na2O 0.3-1.5% 1.0%
Rice Husk : It has high silica content and employed to make
8. Potassium oxide K2O 0.3-1.5% 1.0%
cement.
294 Chemistry

Very Short/Short Answer Questions Multiple Choice Questions


1. What is the function of adding gypsum to clinker cement ? 15. The first ionization energies of alkaline earth metals are
2. Why is anhydrous CaSO4 used as drying agent? Why not higher than those of the alkali metals. This is because
plaster of paris ? (a) there is an increase in the nuclear charge of the alkaline
3. How the caves are formed in limestone regions ? earth metals
4. The crystalline salts of alkaline earth metals contain more (b) there is a decrease in the nuclear charge of the alkaline
water of crystallisation than the corresponding alkali metal earth metals
salts, why? (c) there is no change in the nuclear charge
5. How is slaked lime prepared ? (d) None of the above
6. Account for the following : 16. The first ionization energy of magnesium is lower than the
Be has less negative value of the reduction potential (E°) first ionization energy of
7. What happens when (a) lithium (b) sodium
(i) Calcium is treated with nitrogen. (c) calcium (d) beryllium
(ii) Calcium is treated with dilute sulphuric acid. 17. Some of the Group 2 metal halides are covalent and soluble
(iii) Calcium is heated in the atmosphere of SO2. in organic solvents. Among the following metal halides, the
one which is soluble in ethanol is
(iv) Calcium is treated with cold water .
(a) BeCl2 (b) MgCl2
8. What happens when
(c) CaCl2 (d) SrCl2
(i) Plaster of paris is mixed with water.
18. Which of the following has correct increasing basic
(ii) Gypsum is heated at 473 K. strength?
9. Arrange the following in the order of property indicated : (a) MgO < BeO < CaO < BaO
(i) CaCl2, MgCl2, BeCl2, SrCl2 : Ionic character (b) BeO < MgO < CaO < BaO
(ii) Sr(OH)2, Ba(OH)2, Be(OH)2, Mg(OH)2 : Basic character (c) BaO < CaO < MgO < BeO
(iii) BeSO4, SrSO4, CaSO4, BaSO4 : Increasing solubility (d) CaO < BaO < BeO < MgO
in water 19. Chemical A is used for water softening to remove temporary
hardness. A reacts with Na2CO3 to generate caustic soda.
10. Alkaline earth metal ions have a tendency to form complexes, When CO2 is bubbled through A, it turns cloudy. What is
whereas it is not so in case of alkali metals. the chemical formula of A
11. Discuss thermal stability and the solubility of the oxosalts (a) CaCO3 (b) CaO
of alkaline earth metals. (c) Ca(OH)2 (d) Ca(HCO3)2
12. Why is calcium preferred over sodium to remove last traces 20. Thermal stability of alkaline earth metal carbonates decreases
of moisture from alcohol? in order
Long Answer Questions (a) BaCO3 > SrCO3 > CaCO3 > MgCO3
13. Complete the following equations for the reaction between (b) BaCO3 > SrCO3 > MgCO3 > CaCO3
(c) CaCO3 > SrCO3 > MgCO3 > BaCO3
(i) Ca + H2O (ii) Ca(OH)2 + Cl2
(d) None of these
(iii) BeO + NaOH (iv) BeCl2 + LiAlH4
21. Dehydration of hydrates of halides of calcium, barium and
(v) Ca3N2+H2O strontium i.e., CaCl2.6H2O, BaCl2. 2H2O, SrCl2 . 2H2O, can
14. Give Reasons be achieved by heating. These become wet on keeping in
(i) Alkaline earth metal hydroxides are less basic than alkali air. Which of the following statements is correct about these
metal hydroxides. halides?
(a) act as dehydrating agent
(ii) Be resembles Al.
(b) can absorb moisture from air
(iii) The fluorides of alkaline earth & alkali metal are (c) tendency to form hydrate decreases from calcium to
relatively less soluble than chlorides barium
(d) All of the above
The s-Block Elements – Alkaline Earth Metals 295
22. Which of the following statements is true about Ca(OH)2? (a) Be(OH)2 (b) Mg(OH)2
(a) It is used in the preparation of bleaching powder (c) Ca(OH)2 (d) Ba(OH)2
(b) It is a light blue solid 24. Dead burnt plaster is
(c) It does not possess disinfectant property. 1
(d) It is used in the manufacture of cement. (a) CaSO4 (b) CaSO4. H2 O
2
23. Amphoteric hydroxides react with both alkalies and acids.
(c) CaSO4. H2O (d) CaSO4. 2H2O
Which of the following Group 2 metal hydroxides is soluble
in sodium hydroxide?

1. The one which does not show variable valency is 13. The first ionization potentials (eV) of Be and B respectively
(a) barium (b) titanium are
(c) copper (d) lead (a) 8.29, 9.32 (b) 9.32, 9.32
2. Alkaline earth metals form ions of the general formula (c) 8.29, 8.29 (d) 9.32, 8.29
(a) M2+ (b) M+ (c) M (d) M 2– 14. Which of the following has maximum ionization energy
3. The number of covalent bonds formed by beryllium is
(a) 2 (b) 3 (c) 4 (d) 5 ¾® Ba + + e -
(a) Ba ¾ ¾® Be + + e -
(b) Be ¾
4. Alkaline earth metals show typical characteristics of (c) Ca ¾¾ ® Ca 2+ + 2e- (d) Mg ¾ ¾® Mg 2 + + 2e -
(a) inner transition elements (b) noble gases
(c) representative elements (d) transition elements. 15. The alkaline earth metals Ba, Sr, Ca and Mg may be arranged
5. The elements of group-2 are called alkaline earth metals in the order of their decreasing first ionisation potential as
because (a) Mg, Ca, Sr, Ba (b) Ca, Sr, Ba, Mg
(a) they are alkaline in nature (c) Sr, Ba, Mg, Ca (d) Ba, Mg, Ca, Sr
(b) they occur in earth’s crust and form alkaline salts 16. The first ionization energy of magnesium is lower than the
(c) their oxides are alkaline and occur in earth’s crust first ionization energy of
(d) these are s-block elements. (a) lithium (b) sodium
6. Alkaline earth metals are (c) calcium (d) beryllium
(a) reducing agents (b) oxidising agents 17. The most electropositive amongst the alkaline earth metals
(c) amphoteric (d) acidic is
7. Of the following, most reactive alkaline earth metal is (a) beryllium (b) magnesium
(a) beryllium (b) barium (c) calcium (d) barium
(c) calcium (d) strontium 18. The most electronegative alkaline earth metal is
8. The oxidation state shown by alkaline earth metals is (a) Be (b) Mg
(a) +2 (b) +1, +2 (c) –2 (d) –1, –2 (c) Ca (d) Ra
9. Which of the following ions has the largest ionic radius? 19. Which of the following metals has the highest melting point ?
(a) Be2+ (b) Mg2+ (a) Be (b) Mg
(c) Ca 2+ (d) Sr2+ (c) Ca (d) Ba
10. Density is lowest for 20. Which of the following has exceptionally high melting point
(a) Mg (b) Ca (c) Sr (d) Ba (a) MgO (b) NaOH
11. The first ionization energies of alkaline earth metals are higher (c) NaCl (d) KCl
than those of the alkali metals. This is because 21. The compounds of alkaline earth metals have the following
(a) there is an increase in the nuclear charge of the alkaline magnetic nature
earth metals (a) diamagnetic (b) paramagnetic
(b) there is a decrease in the nuclear charge of the alkaline (c) ferromagnetic (d) antiferromagnetic
earth metals 22. Among the alkaline earth metals, the element forming
(c) there is no change in the nuclear charge predominantly covalent compound is
(d) None of these (a) Ba (b) Sr
12. The first ionization potential of Mg is (c) Ca (d) Be
(a) less than Al 23. As the nuclear charge increases from neon to calcium, the
(b) more than Al orbital energies
(c) equal to Al (a) increase (b) increase very rapidly
(d) can be less or more than Al (c) increase very slowly (d) decreases
296 Chemistry
24. The following compounds have been arranged in order of 38. A salt on treatment with dil. HCl gives a pungent smelling
their increasing thermal stabilities. Identify the correct order. gas and a yellow precipitate. The salt gives green flame test
K2CO3 (I) MgCO3 (II) and a yellow precipitate with potassium chromate. The salt
CaCO3 (III) BeCO3 (IV) is
(a) I < II < III < IV (b) IV < II < III < I (a) NiSO4 (b) BaS2O3
(c) IV < II < I < III (d) II < IV < III < I (c) PbS2O3 (d) CuSO4
25. One which is not dissolved by dilute hydrochloric acid is 39. A metal is burnt in air and the ash on moistening smells of
(a) ZnS (b) MnS NH3. The metal is
(c) BaCO3 (d) BaSO4 (a) Na (b) Fe
26. The hydroxide which is best soluble in water is (c) Mg (d) Al
(a) Ba(OH)2 (b) Mg(OH)2 40. The carbide of which of the following metals on hydrolysis
(c) Sr(OH)2 (d) Ca(OH)2 gives allylene or propyne?
27. The order solubility in water of alkaline earth metal sulphates (a) Be (b) Ca
down the group is Be > Mg > Ca > Sr > Ba. This is due to (c) Al (d) Mg
increases in 41. Magnesium burns in CO2 to form
(a) ionization energy (b) melting point (a) MgO + C (b) MgO + CO
(c) coordination number (d) All of these (c) MgCO3 (d) MgO
28. Which of the following are arranged in increasing order of 42. Which of the following will liberate hydrogen by its reaction
solubilities? with hydrochloric acid?
(a) CaCO3 < KHCO3 < NaHCO3 (a) Copper (b) Phosphorus
(b) NaHCO3 < KHCO3 < CaCO3 (c) Mercury (d) Magnesium
43. Alkaline earth metals are not found free in nature because of
(c) KHCO3 < NaHCO3 < CaCO3
(a) their thermal instability
(d) CaCO3 < NaHCO3 < KHCO3
(b) their low melting points
29. Which of the following is least soluble in water?
(c) their high boiling points
(a) BeSO4 (b) BaSO4
(d) their greater chemical reactivity
(c) CaSO4 (d) SrSO4
44. Ionic hydride is not formed by
30. Which of the following sulphates has highest solubility in
(a) Be (b) Ca
water?
(c) Ba (d) Sr
(a) BaSO4 (b) CaSO4
45. Which of the following alkaline earth metal hydroxides is
(c) BeSO4 (d) MgSO4 amphoteric in character
31. The ion having highest mobility in aqueous solution is (a) Be(OH)2 (b) Ca(OH)2
(a) Be 2+ (b) Mg2+ (c) Sr(OH)2 (d) Ba(OH)2
(c) Ca 2+ (d) Ba2+ 46. A chloride dissolves appreciably in cold water. When placed
32. Which one of the following is least soluble in water? on a Pt wire in Bunsen flame, no distinctive colour is noted.
(a) BaF2 (b) MgF2 Which cation could be present?
(c) CaF2 (d) SrF2 (a) Be2+ (b) Ba2+
33. The characteristic outer electronic configuration of alkaline (c) Pb 2+ (d) Ca2+
earth metals is 47. Halides of alkaline earth metals form hydrates such as
(a) ns 1 (b) ns 2 MgCl2.6H2O, CaCl2.6H2O, BaCl2.2H2O and SrCl2.2H2O. This
2
(c) ns np 2 (d) ns2 np1 shows that halides of group 2 elements
34. Which of the following is strongest base? (a) are hygroscopic in nature
(a) Al(OH)3 (b) Mg(OH)2 (b) act as dehydrating agents
(c) Ca(OH)2 (d) Ba(OH)2 (c) can absorb moisture from air
35. As the alkaline earth metals (except Be) tend to lose their (d) All of the above
valence electrons readily, they act as 48. Which of the followng has lowest melting point?
(a) weak oxidising agents (b) weak reducing agents (a) SrF2 (b) BeF2
(c) strong oxidising agents (d) strong reducing agents (c) MgF2 (d) CaF2
36. Which of the alkaline earth metals is the strongest reducing 49. Amongst LiCl, RbCl, BeCl2 and MgCl2 the compounds with
agent? greatest and the least ionic character, respectively, are
(a) Ca (b) Sr (a) LiCl and RbCl (b) RbCl and BeCl2
(c) Ba (d) Mg (c) RbCl and MgCl2 (d) MgCl2 and BeCl2
37. The correct order of increasing ionic character is 50. Melting point of calcium halides decreases in the order
(a) BeCl2 < MgCl2 < CaCl2 < BaCl2 (a) CaF2 > CaCl2 > CaBr2 > CaI2
(b) BeCl2 < MgCl2 < BaCl2 < CaCl2 (b) CaI2 > CaBr2 > CaCl2 > CaF2
(c) BeCl2 < BaCl2 < MgCl2 < CaCl2 (c) CaBr2 > CaI2 > CaF2 > CaCl2
(d) BaCl2 < CaCl2 < MgCl2 < BeCl2 (d) CaCl2 > CaBr2 > CaI2 > CaF2
The s-Block Elements – Alkaline Earth Metals 297
51. Which liberates ammonia when treated with water? 63. Which of the following is incorrect?
(a) Li3N (b) Mg3N2 (a) Mg burns in air releasing dazzling light rich in UV rays.
(c) CaCN2 (d) All of these (b) CaCl2.6H2O when mixed with ice gives freezing mixture
(c) Mg cannot form complexes
52. In the reaction Mg + H 2 O ® X + H 2 ; X is
(steam)
(d) Be can form complexes due to its very small size
64. Chemical A is used for water softening to remove temporary
(a) MgO (b) Mg(OH)2 hardness. A reacts with Na2CO3 to generate caustic soda.
(c) MgH2 (d) None of these When CO2 is bubbled through A, it turns cloudly. What is
53. The value of x is maximum for the chemical formula of A
(a) MgSO4.x H2O (a) CaCO3 (b) CaO
(b) CaSO4.x H2O (c) Ca(OH)2 (d) Ca(HCO3)2
(c) BaSO4.x H2O 65. Blanc fire used in paints is
(d) All have the same value of x. (a) finely divided BaSO4
54. Which of the following shows high tendency to form a (b) paste of Mg(OH)2
peroxide? (c) suspension of slaked lime
(a) Lithium (b) Beryllium (d) MgCl2.5MgO.5H2O
(c) Magnesium (d) Radium 66. Mg is an important component of which biomolecule occurring
55. Which of the following alkaline earth metal oxides shows a extensively in living world?
coordination number four? (a) Haemoglobin (b) Chlorophyll
(a) BeO (b) MgO (c) Florigen (d) ATP
67. A metal X on heating in nitrogen gas gives Y. Y on treatment
(c) SrO (d) CaO
with H2O gives a colourless gas which when passed through
56. The basic character of the oxides MgO, SrO, K2O, NiO, Cs2O
CuSO4 solution gives a blue colour. Y is
increases in the order
(a) Mg(NO3)2 (b) Mg3N2
(a) MgO > SrO > K2O > NiO > Cs2O
(c) NH3 (d) MgO
(b) Cs2O < K2O < MgO < SrO < NiO 68. Which of the following is the component of most of the
(c) NiO < MgO < SrO < K2O < Cs2O kidney stones?
(d) K2O < NiO < MgO < SrO < Cs2O (a) (COO)2Ca (b) (COO)2Ba
57. Which of the following has correct increasing basic strength? (c) (COONa)2 (d) (COO)2Mg
(a) MgO < BeO < CaO < BaO (b) BeO < MgO < CaO < BaO 69. A firework gives out crimson coloured light. It contains a
(c) BaO < CaO < MgO < BeO (d) CaO < BaO < BeO < MgO salt of
58. Among the following oxides, the one which is most basic is (a) Ca (b) Na
(a) ZnO (b) MgO (c) Sr (d) Ba
(c) Al2O3 (d) N2O5 70. In India at the occasion of marriages, the fire works used
59. Which of the following peroxides is thermally most stable? give green flame. Which one of the following radicals may
be present?
(a) SrO2 (b) MgO2
(a) Na+ (b) K+
(c) CaO2 (d) BaO2 2+
(c) Ba (d) Ca2+
60. Mark the compound which does not contain calcium
71. Which of the following metal ions plays an important role in
carbonate?
muscle contraction?
(a) Fluorspar (b) Iceland spar
(a) K+ (b) Na+
(c) Marble (d) Limestone
(c) Mg2+ (d) Ca2+
61. Thermal stability of alkaline earth metal carbonates decreases 72. White enamel of our teeth is
in order (a) Ca3(PO4)2 (b) CaF2
(a) BaCO3 > SrCO3 > CaCO3 > MgCO3 (c) CaCl2 (d) CaBr2
(b) BaCO3 > SrCO3 > MgCO3 > CaCO3 73. The electric cookers have a coating that protects them
(c) CaCO3 > SrCO3 > MgCO3 > BaCO3 against fire. The coating is made of
(d) None of these (a) heavy lead (b) zinc oxide
62. Which of the following metal carbonates decomposes on (c) magnesium oxide (d) sodium sulphate
heating ? 74. Which of the following is used as a scavenger in metallurgy ?
(a) MgCO3 (b) Na2CO3 (a) Be (b) Mg
(c) K2CO3 (d) Rb2CO3 (c) Ca (d) Sr
298 Chemistry

1. The correct order of increasing thermal stability of K2CO3, 9. Match List – I with List –II for the compositions of
MgCO3, CaCO3 and BeCO3 is [CBSE-PMT 2007] substances and select the correct answer using the code
(a) BeCO3< MgCO3 < CaCO3 < K2CO3 given below the lists : [CBSE-PMT 2011M]
List - I List - II
(b) MgCO3 < BeCO3 < CaCO3 < K2CO3
Substances Composition
(c) K2CO3 < MgCO3 < CaCO3 < BeCO3
(A) Plaster of paris (i) CaSO4.2H2O
(d) BeCO3 < MgCO3 < K2CO3 < CaCO3 (B) Epsomite (ii) CaSO4.½ H2O
2. In which of the following the hydration energy is higher (C) Kieserite (iii) MaSO4.7 H2O
than the lattice energy? [CBSE-PMT 2007] (D) Gypsum (iv) MgSO4. H2O
(a) MgSO4 (b) RaSO4 (v) CaSO4
(c) SrSO4 (d) BaSO4 Code :
3. Which of the following oxides is not expected to react with (A) (B) (C) (D)
sodium hydroxide? [CBSE-PMT 2009] (a) (iii) (iv) (i) (ii)
(a) CaO (b) SiO2 (b) (ii) (iii) (iv) (i)
(c) BeO (d) B2O3 (c) (i) (ii) (iii) (v)
4. Which of the following alkaline earth metal sulphates has (d) (iv) (iii) (ii) (i)
hydration enthalpy higher than the lattice enthalpy? 10. Equimolar solutions of the following substances were
prepared separately. Which one of these will record the
[CBSE-PMT 2010]
highest pH value ? [CBSE-PMT 2012S]
(a) CaSO 4 (b) BeSO 4 (a) BaCl2 (b) AlCl3
(c) LiCl (d) BeCl2
(c) BaSO 4 (d) SrSO4
11. A metal M readily forms its sulphate MSO4 which is water-
5. Property of the alkaline earth metals that increases with their soluble. It forms its oxide MO which becomes inert on heating.
atomic number is [CBSE-PMT 2010] It forms an insoluble hyroxide M(OH)2 which is soluble in
(a) solubility of their hydroxides in water NaOH solution. Then M is [AIEEE 2002]
(a) Mg (b) Ba
(b) solubility of their sulphates in water
(c) Ca (d) Be.
(c) ionization energy
12. The solubilities of carbonates decrease down the magnesium
(d) electronegativity group due to a decrease in [AIEEE 2003]
6. The compound A on heating gives a colourless gas and a (a) hydration energies of cations
residue that is dissolved in water to obtain B. Excess of CO2 (b) inter-ionic attraction
is bubbled through aqueous solution of B, C is formed which (c) entropy of solution formation
is recovered in the solid form. Solid C on gentle heating (d) lattice energies of solids
gives back A. The compound is [CBSE-PMT 2010]
13. In curing cement plasters water is sprinkled from time to
(a) CaSO4.2H2O (b) CaCO3 time. This helps in [AIEEE 2003]
(c) Na2CO3 (d) K2CO3 (a) developing interlocking needle-like crystals of hydrated
7. Which of the following compounds has the lowest melting silicates
point ? [CBSE-PMT 2011] (b) hydrating sand and gravel mixed with cement
(a) CaCl2 (b) CaBr2 (c) converting sand into silicic acid
(c) CaI2 (d) CaF2 (d) keeping it cool
14. Which of the following on thermal decomposition yields a
8. Which one of the following is present as an active ingredient
basic as well as acidic oxide ? [AIEEE 2012]
in bleaching powder for bleaching action ? [CBSE-PMT 2011]
(a) NaNO3 (b) KClO3
(a) CaOCl2 (b) Ca(OCl)2
(c) CaCO3 (d) NH4NO3
(c) CaO2Cl (d) CaCl2
The s-Block Elements – Alkaline Earth Metals 299

1. Plaster of Paris hardens by 14. Mixture of MgCl2 and MgO is called


(a) giving off CO2 (b) changing into CaCO3 (a) Portland cement (b) Sorel’s cement
(c) uniting with water (d) giving out water (c) Double salt (d) None of these
2. Hesenclever’s process is a method for the manufacture of 15. The metals A and B form oxide but B also forms nitride when
(a) NaOH (b) HNO3 both burn in air. The A and B are
(c) H2SO4 (d) Bleaching powder (a) Cs, K (b) Mg, Ca
3. Oxygen is obtained from bleaching powder by (c) Li, Na (d) K, Mg
(a) the action of dilute acid (d) the action of alkali +
16. Magnesium form Mg 2+ and Mg because :
(c) heating it with lime (d) heating it with cobalt salt
4. Portland cement does not contain (a) ionic radius of Mg(II) is smaller than of Mg(I).
(a) CaSiO4 (b) Ca2SiO5 (b) hydration energy of divalent magnesium ion is higher.
(c) Ca2Al2O6 (d) Ca3(PO4)2 (c) magnesium (II) carbonate is insoluble in water.
5. Alkaline earth metal compounds are less soluble in water (d) generally higher oxidation states are preferred by metals.
than corresponding alkali metal compounds because former 17. Setting of plaster of paris involves :
have (a) combination with atmospheric CO 2
(a) lower lattice energy (b) higher I.P. (b) oxidation with atmospheric oxygen
(c) higher covalent character(d) lower covalent character. (c) hydration to yield another hydrate
6. Magnesium wire burns in the atmosphere of CO2 because (d) dehydration.
(a) Magnesium acts as an oxidising agent 18. One of the important use of quick lime is
(b) Magnesium has 2 electrons in the outermost orbit. (a) in bleaching silk (b) in drying gases & alcohol
(c) Magnesium acts as a reducing agent and removes (c) in dyeing cotton (d) as a purgative.
oxygen from CO2 19. Disodium hydrogen phosphate in presence of NH4Cl and
(d) None of these 2+
NH 4 OH gives a white precipitate with a solution of Mg
7. When hydrated MgCl2. 6H2O is strongly heated
(a) MgO is formed (b) Mg(OH)2 is formed ion. The precipitate is :
(c) Mg(OH)Cl is formed (d) Anhydrous MgCl2 is formed (a) MgNH4PO4 (b) MgHPO4
8. Which component of cement sets at the slowest rate (c) Mg(H2PO4)2 (d) Mg3(PO4)2
(a) Dicalcium silicate 20. The metal X is prepared by the electrolysis of fused chloride.
(b) Tricalcium silicate It reacts with hydrogen to form a colourless solid from which
(c) Tricalcium aluminate hydrogen is released on treatment with water. The metal is :
(d) Tetracalcium aluminoferrite (a) Ca (b) Al (c) Zn (d) Cu
9. The cation which forms a yellow precipitate with potassium 21. Amongst the following hydroxides, the one which has the
chromate in acetic acid is lowest value of K sp is :
(a) NH +4 (b) Ba 2+ (c) Ca 2+ (d) Na + (a) Ca(OH)2 (b) Mg(OH)2 (c) Be(OH)2 (d) Ba(OH)2
22. Beryllium shows diagonal relationship with aluminium. Which
10. The ionic carbide is of the following similarity is incorrect ?
(a) CaC2 (b) CaO (a) Be forms beryllates and Al forms aluminates
(c) Ca(OH)2 (d) Ca(HCO3)2 (b) Be(OH)2 like Al(OH)3 is basic.
11. Which category of salts of alkaline earth metals is not found
in solid sate, but found in solution state? (c) Be like Al is rendered passive by HNO 3 .
(a) Carbonates (b) Bicarbonates (d) Be2C like Al4C3 yields methane on hydrolysis.
(c) Hydroxides (d) Sulphates 23. A gas reacts with CaO and not with NaHCO 3 is :
12. BeF2 is soluble in water whereas fluorides of other alkaline
earth metals are insoluble because of (a) Cl 2 (b) CO 2 (c) N 2 (d) O 2
(a) ionic nature of BeF2 24. Substance which absorbs CO 2 and violently reacts with
(b) covalent nature of BeF2
H 2 O with sound is :
(c) greater hydration energy of Be2+ ion as compared to its
lattice energy (a) H 2SO 4 (b) CaCO 3 (c) ZnO (d) CaO.
(d) None of these 25. When Zeolite, (hydrated sodium Aluminium silicate), is
13. Electrolysis of fused KCl. MgCl2. 6H2O gives treated with hard water, the sodium ions are exchanged with:
(a) potassium only
(a) SO 24 - ions (b) H + ions
(b) magnesium only
(c) magnesium and chlorine (c) OH - ions (d) Ca 2+ ions
(d) potassium, magnesium and chlorine
300 Chemistry

EXERCISE 1 10. (b) Ca contains vacant 3d orbitals which leads much increase
in atomic volume of the element hence has the lowest
1. Gypsum is added to retard the rate of setting of cement. density.
2. Anhydrous CaSO4 can absorb water to form CaSO4 . 2H2O. 11. (a)
Plaster of paris is not used as a drying agent because it sets 12. (b) Mg has more IE1 than Al since the electron has to be
into a hard mass by absorbing water. removed from completely filled s-valence shell of Mg.
3. Caves in limestone regions get formed due to the dissolution 13. (d) 14. (d) 15. (a) 16. (d)
of calcium carbonate in water containing carbon dioxide. 17. (d) 18. (a)
4. This is because alkaline earth metals have smaller size and 19. (a) Being small and hard, Be atoms pack closely in the lattice
higher nuclear charges as compared to alkali metals. and hence Be has the highest m.pt.
5. By treating quick lime with water, slaked lime is formed. 20. (a) Higher the charge on the ions, greater is the lattice energy
and hence higher the m.pt. of the ionic solid.
7. (i) 3Ca(s) + N2(g) ¾¾
® Ca3N2(s)
21. (a) M2+ ions are present in alkaline earth metal compounds
(ii) Ca(s) + H2SO4(aq) ¾¾
® CaSO4(aq) + H2(g) thus leaving all paired electrons in (n-1) s 2 p 6
configuration and are diamagnetic.
(iii) 2Ca(s) + SO2(g) ¾¾
® 2CaO (s) + S(s)
22. (d)
(iv) Ca(s) + 2H2O (l) ¾¾ ® Ca(OH)2(aq) + H2(g) 23. (d) After neon (At No-18), the next electron enters into 4s
8. (i) Plaster of paris quickly solidifies to gypsum with the orbital which has lower energy than 3d orbitals.
evolution of heat and also expands slightly. 24. (b) 25. (d) 26. (a)
473K
27. (c) In crystalline sulphates, the co-ordination number of the
(ii) CaSO 4 . 2H 2 O ¾¾¾® CaSO 4 + 2H 2 O ions increases from Be to Ba and so also the lattice energy
Dead burnt plaster hence solubility in H2O decreases.
9. (i) BeCl2 < MgCl2 < CaCl2 < SrCl2 28. (d) Lesser the lattice energy, more will be the solubility in
H2O.
(ii) Be(OH)2 < Mg(OH)2 < Sr(OH)2 < Ba(OH)2
29. (b) 30. (c)
(iii) BaSO4 < SrSO4 < CaSO4 < BeSO4. 2+
31. (d) Ba is least hydrated hence its size becomes lesser than
12. Both Na and K can react with water. But Na also reacts with any other hydrated alkaline earth metal cation and
alcohol whereas calcium does not or react slowly. therefore has the highest ionic mobility.
15. (a) 16. (d) 17. (a) 18. (b) 19. (c) 32. (b) As we move down the group, the lattice energies of
fluorides decrease more rapidly than the hydration energy
20. (a) 21. (d) 22. (a) 23. (a) 24. (a) and hence the solubilities of the fluorides increase from
top to bottom within a group.
EXERCISE 2 33. (b) 34. (d)
1. (a) Barium shows fixed valency whereas Ti and Cu (transition 35. (d) Lower the I.E. or higher the negative electrode potential,
stronger is the reducing agent.
metals) and Pb (due to inert pair effect) show variable
valency. 36. (c) 37. (a)
38. (b) Since the salt on treatment with dil. HCl gives a pungent
2. (a)
smelling gas (SO2) and a yellow ppt., it must contain
3. (a) Since Be has two valence electrons, it forms only two S2O32– (thiosulphate ion) and further since it gives green
covalent bonds. flame test, it must contain Ba2+ ion. Hence salt is BaS2O3.
4. (c) 39. (c) Mg burns in air to form Mg3N2 which then reacts with
H2O to form NH3.
5. (c) Their oxides are alkaline and occur in earth’s crust.
40. (d) 41. (a)
6. (a) Because of low I.E. and high hydration energies.
42. (d) Since the electrode potential of Mg is lower than that of
7. (b) Ba is most electropositive amongst the listed and hence hydrogen (Eº = 0.0 volt), it will reduce H+ ions to H2 gas.
most reactive. 43. (d)
8. (a) 9. (d) 44. (d) BeH2 is covalent.
The s-Block Elements – Alkaline Earth Metals 301
45. (a) Be(OH)2 is amphoteric while Ca(OH)2, Sr(OH)2 and EXERCISE 3
Ba(OH)2 are all basic.
46. (a) Due to very high I.E. of Be, its valence electrons require 1. (a) As the basicity of metal hydroxides increases down the
high energy to remove (excite) which is not available in group from Be to Ba, the thermal stability of their
Bunsen flame hence Be2+ ions do not impart colour to carbonates also increases in the same order. Further group
the flame. 1 compounds are more thermally stable than group 2
47. (d) because their hydroxide are much basic than group 2
hydroxides therefore, the order of thermal stability is
48. (b) BeF2 being covalent has the lowest melting point (800ºC)
while all other fluorides melt around 1300ºC BeCO3 < MgCO3< CaCO3< K2CO3.
49. (b) 2. (a) In alkaline earth metals ionic size increases down the
50. (a) As the size of the anion increases, the covalent character group. The lattice energy remains constant because
increases and hence the mp. decrease. sulphate ion is so large, so that small change in cationic
51. (d) All nitrides react with H2O to give NH3 but CaCN2 also sizes do not make any difference. On moving down the
react with H2O to give NH3. group the degree of hydration of metal ions decreases
very much leading to decrease in solubility
CaNCN + 3H 2 O ¾
¾® CaCO 3 + NH 3
\ BeSO 4 > MgSO 4 > CaSO 4 > SrSO 4 > BaSO 4
52. (a) Mg(OH)2 is not formed because of poor solubility of
MgO in H2O. 3. (a) NaOH is a strong alkali. It combines with acidic and
53. (a) Because of smaller size, Mg2+ ions are extensively amphoteric oxides to form salts. Since CaO is a basic
hydrated. oxide hence does not reacts with NaOH.
54. (d) Larger the size of the atom, larger is the tendency to form 4. (b) Be2+ is very small, hence its hydration enthalpy is
peroxides.
greater than its lattice enthalpy
65. (a) 56. (c)
5. (a) Lattice energy decreases more rapidly than hydration
57. (b) The basic character of oxides increases down the group.
energy for alkaline earth metal hydroxides. On moving
58. (b) N2O5 is strongly acidic, ZnO and Al2O3 are amphoteric, down a group\ solubility of their hydroxides increases.
therefore, MgO is most basic.
D
59. (d) The stability of the peroxide increases as the size of the 6. (b) CaCO3( s ) ¾¾® CO 2( g ) ­ + CaO ( s )
metal increases.
A colourless B
60. (a) Fluorspar is CaF2 while other compounds listed are
different crystalline forms of CaCO3. ® Ca (OH )2(aq)
CaO (s) + H 2 O ¾¾
61. (a) B
62. (a) MgCO3 decomposes on heating to give MgO and CO2 ® Ca (HCO3 )2(aq)
Ca (OH )2 + 2CO2 ¾¾
while alkali metal carbonates do not. C
63. (c)
Ca (HCO3 )2(s) ¾¾
® CaCO3(s) + CO2(g) + H 2 O(g)
A
64. (c) Ca (HCO3 ) 2 + Ca (OH) 2 ¾
¾® 2CaCO 3 ¯ +2H 2 O
temp. hardness \ Correct choice : (b)
7. (c) Melting points of halides decreases as the size of the
Ca(OH)2 + Na 2CO3 ¾¾
® 2NaOH + CaCO3 halogen increases. The correct order is
A Caustic soda
CaF2 > CaCl2 > CaBr2 > CaI2
Ca (OH) 2 + CO 2 ¾
¾® CaCO 3 ¯ + H 2 O 8. (b) Active ingredient in bleaching powder for bleaching
A milkiness action is Ca (OCl)2
65. (a) 66. (b) 1
9. (b) (A) Plaster of paris = CaSO4. H 2O
D 2
67. (b) 3Mg + N 2 ¾¾® Mg 3 N 2 ;
X Y (B) Epsomite = MgSO4.7H2O
(C) Kieserite = MgSO4.H2O
Mg 3 N 2 + 6H 2 O ¾
¾® 3Mg(OH) 2 + 2 NH 3 ­
(D) Gypsum = CaSO4.2H2O
(colourless)
10. (a) (AlCl3, LiCl & BeCl2) ) all these solutions are acidic due
CuSO 4 + 4 NH3 ¾
¾® [Cu ( NH 3 ) 4 ] SO 4 to cationic hydrolysis, whereas BaCl2, is salt of strong
Blue complex base and strong acid, hence its solution will almost
neutral i.e., pH » 7.
68. (a) 69. (c) 70. (c) 71. (d) 72. (b)
73. (c) MgO being high melting does not catch fire and hence
protects the cooker against fire. 11. (d) Beryllium shows anomalous properties due to its small
size.
74. (c)
302 Chemistry

12. (a) The solubility is governed by DH solution 13. (d) K and Mg are formed at cathode

i.e. DH solution = DH lattice - DH Hydration K + + e - ® K, Mg 2+ + 2e - ® Mg .

Due to increase in size the magnitude of hydration energy Chlorine is formed at anode 2C l - 2e - ® Cl 2 .
decreases and hence the solubility.
14. (b) The mixture of MgCl2 and MgO is known as sorel’s
13. (a) Setting of cement is exothermic process which develops cement.
interlocking crystals of hydrated silicates
15. (d) K and Mg, both form oxides
14. (c) Calcium carbonate on thermal decomposition gives CaO
(Basic oxide) and CO2 (Acidic oxide) K + O 2 ® KO 2 ;2Mg + O 2 ® 2MgO
D Mg form nitride also 3Mg + N 2 ® Mg 3 N 2
CaCO 3 CaO + CO 2­
Basic oxide Acidic oxide K does not form nitride.
16. (b) Statement (b) is correct.

EXERCISE 4 1
17. (c) CaSO 4 . H 2 O ¾setting
¾¾ ¾® CaSO 4 .2H 2 O
1. (c) Plaster of Paris reacts with water to form gypsum which 2
is hard. Plaster of paris Gypsum (orthorhombic)
CaSO4 .1/ 2 H 2 O + 3/ 2 H 2 O ® CaSO 4 .2H 2 O hardening
¾¾ ¾¾ ¾® CaSO 4 .2H 2O
2. (d) Hasenclever and Batchmann’s plants are used for the
Gypsum (monoclinic)
manufacture of bleaching powder (See theory).
18. (b) CaO (Quick lime) is hygroscopic in nature.
3. (a) 2CaOCl2 + dil.H 2SO4 ® CaCl2 + CaSO 4 + 2HClO
19. (a) Mg + + + Na 2 HPO 4 + NH 4 OH
HClO ® HCl + O
4. (d) Portland cement is a complex mixture of silicates and
¾® Mg(NH 4 )PO 4 + 2 Na + + H 2 O
¾
aluminates of Ca.
White ppt
5. (c) The higher the covalent character, the lower the solubility
of compound in water. 20. (a) Ca + H 2 ¾¾® CaH 2 + 2H 2 O ¾ ¾® Ca (OH ) 2 + H 2
6. (c) Mg is more reducing in nature than carbon
21. (c) Be(OH)2 is insoluble in water and thus has lowest value
CO 2 + 2Mg ® 2MgO + C of K sp .
D
7. (a) MgCl 2 .6H 2 O ¾¾® D
MgCl 2 .2H 2 O ¾¾® 22. (b) The Be(OH)2 and Al(OH)3 are amphoteric in nature.

Mg (OH )Cl ¾
¾® MgO + HCl 23. (b) CaO + CO 2 ¾
¾® CaCO 3 .
8. (a) Dicalcium silicate sets at the lowest rate. But the CO 2 does not react with NaHCO 3 .
9. (b) Ba++ with CrO -4 - give yellow BaCrO4. It is insoluble in
24. (d) CaO + CO 2 ¾
¾® CaCO 3
CH3COOH.
CaO + H 2 O ¾
¾® Ca (OH) 2
(
10. (a) CaC2 calcium carbide is ionic Ca 2+ C22- . ) hissing sound and DH = – ve
11. (b) The bicarbonates of alkaline earth metals exist in solution
only 25. (d) Na 2 Z + Ca 2 + ¾
¾® CaZ + 2 Na +
12. (c) Be2+ being small in size is heavily hydrated and heat of
hydration exceeds the lattice energy. Hence BeF2 is
soluble in water.
11A
The p-Block Elements :
Boron Family
The elements of group 13 are Element B Al Ga In Tl
B - Boron Al - Aluminium Ga - Gallium M.p. (K) 2453 933 303 430 576
In - Indium Tl - Thallium
B.p. (K) 3923 2740 2676 2353 1730
Amongst the elements of this group, Al is the third most abundant
element found in the earth’s crust after oxygen and silicon. (b) The m. p. decreases from B to Ga and then increases,
due to structural changes in the elements
GENERAL CHARACTERISTICS (i.e., PHYSICAL
PROPERTIES) OF BORON FAMILY : (c) Boron has a very high m. p. because of its three
(i) Electronic configuration : The elements of group 13 belong dimensional (B12- icosahedral) structure in which B
to p-block of the periodic table and these elements contain atoms are held together by strong covalent bonds.
three electrons in the valence shell, therefore, their valence (d) Low m. p. of Ga is due to the fact that it consists of only
shell electronic configuration is ns2np1. Ga2 molecules, and Ga remains liquid upto 2273K
Element At. No. Electronic Valence therefore it is used in high temperature thermometry.
Confg. Shell Confg.
(v) Ionisation energy :
B 5 [He] 2s2, 2p1 2s 22p 1
(a) The first I.E. values of group 13 elements are lower than
Al 13 [Ne] 3s2, 3p1 3s 23p 1
the corresponding values of the alkaline earth metals,
Ga 31 [Ar]3 3d10, 4s24p1 4s 24p 1
due to the fact that removal of electron is easy in former
In 49 [Kr] 4d10, 5s25p 1 5s 25p 1
case (p-electron) than latter (s-electron)
Tl 81 [Xe] 4f14, 5d10, 6s2p 1 6s 26p 1
This results in the increase of nuclear charge.
(ii) Atomic radii and ionic radii : Consequently the valence electrons are more tightly
(a) Atoms and their ions of group 13 elements have smaller held leading to high I.E. Similarly we can explain the
size than those of alkaline earth metals of group-2, due irregularity in case of Tl on the basis of ineffective
to greater nuclear charge of former group than latter shielding of intervening electrons
group.
(b) On moving down the group IE, decreases from B to Al
(b) Atomic radii increase on going down in the group with
but the next element Ga has slightly higher IE, than Al,
an abnormaly at gallium and the unexpected decrease in
it again decreases in In and increases in the last element
the atomic size of Ga is due to the presence of electrons
Tl as follows :
in d- orbitals which do not screen the nucleus effectively.
(c) The ionic radii regularly increases from B 3+ Element B Al Ga In Tl
to Tl3+ IE, (kJ mol–1) 800 577 578 558 590
(iii) Density : It increases regularly on moving down the group
from B to Tl (c) The irregularity observed in case of Gallium is due to
(iv) Melting and boiling points : the ineffective shielding of nuclear charge because of
(a) M.P. and b. p. of group 13 elements are much higher intervening d electrons, which cause the increase in
than those of group 2 elements nuclear charge leading to high I.E.
304 Chemistry
(vi) Inert pair effect : (ii) Reaction with nitrogen : The elements of group 13 form
(a) It is the reluctance of the s-electrons of the valence their corresponding nitrides with the reaction of nitrogen.
shell to take part in bonding and occurs due to These nitrides undergo hydrolysis with steam and evolve
ineffective shielding of the ns 2 electrons by the NH3
intervening d and f electrons.
(b) It increases down a group and thus the lower elements 2B + N 2 ¾¾®
D
2BN
of group show lower oxidation states. Boron nitride
(vii) Oxidation states :
(a) B and Al show an oxidation state of +3 only while Ga, In D
2Al + N 2 ¾¾® 2AlN
and Tl show oxidation states of both +1 Aluminium nitride
and +3.
(b) As we move down in the group 13, due to inert pair BN + 3H 2 O ® H3 BO3 + NH3
effect, the tendency to achieve +3 oxidation state goes orthoboric acid
on decreasing and the tendency to acquire +1 oxidation
state goes on increasing. (iii) Action of water :
(c) Stability of +1 oxidation state follows the order Ga < In (a) Both B and Al do not react with water but amalgamated
< Tl aluminium does react with H2O evolving H2.
(d) Tl+ compounds are more stable than Tl3+ compounds. 2Al(Hg) + 6H2O ® 2Al(OH)3 + 3H2 +Hg
(viii) Electropositive (metallic) character : Aluminium amalgam
(a) These elements are less electropositive than the (b) Ga and In do not react with pure cold or hot water but Tl
elements of the group 1 and 2 due to their smaller size
forms an oxide on the surface.
and higher ionisation energies.
(b) On moving down the group, the electropositive (iv) Reaction with alkalies :
character first increases from B to Al and then decreases (a) Boron dissolves in alkalies on fusion and gives borates.
from Ga to Tl, due to the presence of d and f orbitals in
latter elements. 2B + 6NaOH ¾fuse¾¾® 2Na3BO3 + 3H2
(ix) Reducing character : It decreases down the group from Al (b) Al and Ga dissolve in concentrated alkalies on heating
to Tl because of the increase in electrode potential values and form meta-aluminate and gallate respectively.
for M3+/M. Therefore it follows the order.
D
Al + 2NaOH + 2H2O ¾¾® 2NaAlO2 + 3H2
Al > Ga > In > Tl
Element B Al Ga In Tl Sod. metaaluminate
Eº (V) at 298 K for M3+/M –1.66 –0.56 –0.34 +1.26
(x) Complex formation : Due to their smaller size and greater Ga + 2NaOH + 2H2O ¾¾® D 2NaGaO +3H
2 2
charge, these elements have greater tendency to form Sod. gallate
complexes than the s-block elements.
(xi) Nature of compounds : (v) Reaction with carbon :
(a) The tendency of formation of ionic compounds (a) B and Al form carbides with carbon on heating.
increases from B to Tl. D
(b) Boron forms only covalent compounds where as Al can 4B + C ¾¾® B4C
;
form both covalent (e.g., anhydrous AlCl3) and ionic Boron carbide
compounds (e.g., hydrated AlCl 3 ) because when
anhydrous AlCl3 is hydrated, the hydration energy 4Al + 3C ¾
¾® Al 4 C3
released is sufficient to overcome the ionisation energy Aluminium carbide
of Al.
(b) Aluminium carbide is ionic and forms methane with
(c) Gallium forms mainly ionic compounds although
anhydrous GaCl3 is covalent . water, while boron carbide is covalent having molecular
CHEMICAL PROPERTIES : formula B12C3 and it is very hard, hence used as an
Crystalline B is unreactive whereas amorphous B is reactive. It abrasive.
reacts with air at 700ºC as follows : (vi) Hyrides :
(a) 4B + 3O2®2B2O3; (a) Elements of group 13 do not combine directly with H2 to
2B + N2 ®2BN form hydrides, therefore, their hydrides have been
(b) Al is stable in air due to the formation of protective oxide prepared by indirect methods for example.
film. Some properties are given below
dry ether
(i) Action of Air : BF3 + 3LiBH4 ¾¾¾¾® 2B2H6 + 3LiF
(a) 4Al + 3O2 ®2 Al2O3 diborane
(b) Thallium is more reactive than Ga and In, due to the (b) Boron forms a number of hydrides which are called
formation of unipositive ion Tl+. boranes with general formula BnHn+4(e.g., B2H6) and
4Tl + O2 ®2 Tl2O BnHn+6 (e.g., B4H10)
The p -Block Elements – Boron Family 305
(c) Boranes catch fire in the presence of oxygen with the 373K
evolution of heat energy 2H 3 BO3 ¾¾¾® 2HBO 2
orthoboric acid - 2 H 2O Metaboric acid
B2H6 + 3O2 ® B2O3 + 3H2O ; DH = – 2008 kg/mol
(d) Boranes are hydrolysed by water as follows : ¾¾¾¾
®
red heat
B2O3 + H 2O
B2H6 + 6H2O ® 2H3BO3 + 6H2 Boric acid
or boron sesquioxide or
(e) Boranes are quite stable but the stability of hydrides of Boric anhydride
Al, Ga, In, and Tl decreases on moving down the group
because the strength of the M–H bond decreases with 2Al(OH)3 ® Al2O3 + 3H2O
increasing size of the element. 4Al(NO3)3 ® 2Al2O3 + 12NO2 + 3O2
(f) Structure of diborane : The simplest boron hydride, i.e. (d) The hardest alumina, a–Al2O3, which is stable at high
BH3 does not exist as such but exists as a dimer, B2H6, temperature, resistant to hydration and attacked by
having following structure acids, is called corundum and is used as an abrasive
where as g –Al2O3 is known as activated alumina and
Ha 11 Hb Ha used in column chromatography.
9p
m (viii) Hydroxides :
B 178pm B Amalgamated aluminium reacts with H2O and forms its hydroxide.
120º or
97º 2Al(Hg) + 6H2O ® Al(OH)3 + 3H2 + Hg
13

Aluminium amalgam
4p

Hb Ha
m

Ha However, B(OH)3 can also be prepared by the reaction of B2O3


and H2O
Ha Ha B2O3 + 3H2O ® 2B(OH)3 + 2H3BO3
Hb
(ix) Nature of oxides and hydroxides :
:

B B (a) B(OH)3 or H3BO3 is soluble in water, whereas all other


hydroxides are practically insoluble in water.
:

Hb
Ha Ha (b) On moving down the group, there is a change from
acidic to amphoteric and then to basic character of
In above structure B atoms are in sp3 hybrid state. There oxides and hydroxides of group 13 elelments.
are six B-H bonds out of which four B-Ha bonds are (x) Preparation of halides :
normal convalent bonds (two centre electron - pair (a) All the elements of boron family (except thallium which
bonds i.e, 2c–2e) present in the same plane while rest forms thallous monohalides ) form trihalides of type
two B-Hb bonds behave as bridges containing three MX3 where X= F, Cl, Br and I.
centre two electron pairs bonds i.e., 3c–2e (known as B2O3 + 3C + 3Cl2 ® 2BCl3 + 3CO
banana bonds) and present above and below the plane Al2O3 + 3C + 3Cl2 ® 2AlCl3 + 3CO
of the molecules which do not have sufficient number (b) All the boron trihalides, BX3 and aluminium trihalides
of electrons to form normal covalent bonds are called AlX 3 (except AlF 3 which is ionic) are covalent
electron - deficient molecules. Ex. B2H6. compounds whereas former exist as only monomers and
(g) Aluminium forms a polymeric hydride of general formula latter as dimers, because boron atom is too small to co-
(AlH3)x which decomposes into its elements on heating. ordinate with four large halide ions and in case of much
(h) B, Al and Ga form complex anionic hydrides, like lithium smaller F– ion , the energy released during the formation
borohydride, Li[BH4], lithium aluminium hydride , of the bridge structure is not sufficient for the cleavage
Li[AlH4] etc. due to the presence of a vacant p-orbital in of the typical pp-pp bond in BF3.
their outermost shells which accepts electron pair from
X
H– ion.
XH3 + H– ® [XH4]– where X= B, Al and Ga.
B X
(vii) Oxides :
(a) Except Tl all the elements of group 13 form oxides
(general formula, M2O3) on heating with oxygen X
BX3
4M + 3O2 ¾¾® D 2M2O3 (M = B to In)
X X X
(b) Tl forms thallium (I) oxide, Tl2O which is more stable
than thallium (III) oxide, Tl2O3 because of inert pair
Al Al
effect.
(c) B2O3 and Al2O3 can also be prepared by following
X X X
method :
Al2X6
306 Chemistry
(c) BF3 is a colourless gas, BCl3 and BBr3 are colourless (viii)It does not react with steam but other elements decompose
fuming liquids whereas BI3 is a white fusible solid at steam.
room temperature. (ix) It can be oxidised by concentrated HNO3 while aluminium is
(d) The covalent character of trihalides decreases on passive due to the formation of oxide layer on the surface.
moving from Ga to Tl. 2B +6HNO3 ® 2H3BO3 + 6NO2
(e) Hybridisation of Boron in BCl3 is sp2
Boric acid
(xi) Nature of Trihalies :
DIAGONAL RELATIONSHIP BETWEEN BORON AND
(a) Trihalides of group 13 elements behave as lewis acids
due to having a strong tendency to accept a pair of
SILICON :
electrons. Boron shows resemblance with its diagonal element silicon of
group 14. Some of the important points are given below :
(b) The relative strength of lewis acids of boron trihalides
(i) Both B and Si are non- metals.
increases in the order :
(ii) Both are semi- conductor.
BF3 < BCl3 < BBr3 < BI3. (iii) Both do not form ions, i.e., B 3+ and Si4+.
(c) The halides of group 13 elements behave as lewis acids (iv) Both B and Si form covalent hydrides : boranes i.e., B2H6,
and the acidity character decreases as follows: B4H6 etc. and silanes, i.e., SiH4, Si2H6 etc. respectively, which
BX3 > AlX3 > GaX3 > InX3 (where X=Cl, Br or I) catch fire when exposed to air.
(d) BF3 and anhydrous AlCl3 are used as a catalyst in Friedel B2H6 + 3O2 ® B2O3 + 3H2O
Crafts reactions. SiH4 + 2O2 ® SiO2 + 2H2O
(e) TlCl3 decomposes to TlCl and Cl2 above 40ºC and hence (v) Both form covalent, and volatile halides which fume in moist
acts as an oxidising agent, whereas TlBr3 converts into air due to release of HCl fumes.
Tl[TlBr4] at room temperture. BCl3 + 3H2O ® H3BO3 + 3HCl ­
SiCl4 + 4H2O ® H4SiO4 + 4HCl ­
D (vi) Both form solid oxides which are acidic and dissolve in
TlCl3 ¾¾® TlCl + Cl2 ; 2TlBr3 ® Tl[TlBr4] +Br2 alkalies forming borates and silicates respectively.
> 40 º C
B2O3 + 2NaOH ® 2NaBO2 + H2O
While TlI3 is an ionic compound containing Tl(I) and
SiO2 + 2NaOH ® Na2SiO3 + H2O
I 3- ions. (vii) Both react with electropositive metals and form binary
compounds, which yield mixture of boranes and silanes on
ANOMALOUS BEHAVIOUR OF BORON : hydrolysis. Both B and Si form hydroxides boric acid B(OH)3
Boron shows anomalous behaviour as compared to other member and silicic acid Si(OH)4 respectively which are weak acids.
of the group, due to the following reasons. D
3Mg + 2B ¾¾® Mg3B2
(i) It has smallest size in the group.
(ii) It has high ionisation energy. H O / H+
Mg3B2 ¾¾2 ¾ ¾
¾® Mixture of boranes
(iii) It has highest electronegativity in the group.
D
(iv) It does not have any vacant d-orbital in valance shell 2Mg + Si ¾¾® Mg2Si
A few points of difference are : +
Mg2Si ¾H O/H
¾2 ¾ ¾
¾® mixture of silanes
(i) It is a non-metal while other members of the group are metallic
in character. METALLURGY OF BORON
(ii) It shows allotropy as crystalline and amorphous while other OCCURRENCE AND IMPORTANT MINERALS OF
members do not. BORON :
(iii) It has the highest m. p. and b.p. amongst the elements of It does not occur in the free state in nature. It forms electron
group 13. deficient compounds.
(iv) It forms only covalent compounds while other members form Its important minerals are
both ionic and covalent compounds. (a) Borax (or Tincal), Na2B4O7.10H2O or
(v) It forms a number of hydrides which are quite stable while Na2[B4O5(OH)4]8H2O
those of other members are less stable. (b) Kernite, Na2B4O7.2H2O or Na2[B4O5(OH)4]
(vi) The halides of boron exist as monomers while AlCl3 exists (c) Orthoboric-acid H3BO3
as a dimer. (d) Colemanite, Ca2B6O11.5H2O or Ca2[B3O4(OH)3]2 2H2O
(e) Boracite, 2Mg3B8O15. MgCl2
(vii)The oxides and hydroxides of boron are weak acidic while
those of aluminium are amphoteric and those of other ISOLATION :
elements are basic. Elemental boron in the form of dark brown powder is obtained by
following methods :
The p -Block Elements – Boron Family 307
(i) By reduction of boric oxide with highly electropositve metals Heat , swells
like K, Mg, Al, Na etc. in the absence of air (iii) Action of heat : Na2B4O7.10H2O ¾¾ ¾ ¾¾®
-10H 2O
B2O3 + 6K ¾Heat
¾¾® 2B + 3K2O 740º C
(ii) By the reaction of boron halides with hydrogen at high Na2B4O7 ¾¾¾® 2NaBO2 + B2O3
temperature. Anhydrous Sodium Boric
Metaborate anhydride
2BCl3 + 3H2 ¾1270K
¾® 2B + 6HCl
¾¾ Glassy mass (Borax bead)
(iii) By thermal decomposition of boron triiodide over red hot Borax bead is used for the detection of coloured basic
tungsten filament. radicals under the name borax bead test in which on heating
W heat borax bead combines readily with a number of coloured
2BI3 ¾¾¾ ¾ ® 2B + 3I 2 transition metal oxides such as Co, Ni, Cr, Cu, Mn etc. to
(iv) By thermal decomposition of boron hydrides form the corresponding metaborates which possess
D characteristic colours.
B 2 H 6 ¾¾® 2B + 3H 2
D
USES OF BORON : CoSO4 ¾¾® CoO +SO3 ;
(i) As a semiconductor CoO +B2O3 ® Co(BO2)2
(ii) Boron steel or boron carbide rods are used to control the cobalt metaborate (blue)
nuclear reactions. Basic radical of a salt Cu Fe Co Cr Ni
10
5B + 0 n1 ®5 B11 Colours of metaborates Blue Green Blue Green Brown
Uses : It is used in making optical and hard glasses and in
COMPOUNDS OF BORON the borax bead test.
BORON HYDRIDES OR BORANES : BORIC ACID OR ORTHOBORIC ACID, H 3 BO 3
Boron forms hydrides of the type BnHn+4 and BnHn+6 which are OR B(OH) 3 :
called Boranes.
Preparation
Preparation :
(i) By treating borax with dil HCl or dil H2SO4
(i) 8BF3 + 6LiH ® B2H6 + 6LiBF4
(ii) 4BCl3 + LiAlH4 ® 2B2H6+ 3AlCl3 + 3LiCl Na2B4O7 + 2HCl + 5H2O ® 2NaCl +4H3BO3
Properties : (ii) By passing SO2 through a mixture of powdered mineral
(i) All boranes are called as electron – deficient compounds colemanite in boiling water.
because boron in boranes never completes its octet. Ca2B6O11 + 4SO2 + 11H2O ® 2Ca(HSO3)2 +6H3BO3
(ii) On reaction with water boric acid is formed. Properties :
BORAX OR SODIUM TETRABORATE DECAHYDRATE, (i) It is a very weak monobasic acid. It does not act as a proton
Na2B4O7.10H2O OR Na2[B4O5(OH)4]8H2O : donor but accepts a hydroxyl ion i.e., it behaves as a lewis
Preparation : acid.
(i) It occurs naturally as tincal in dried up lakes of Sri Lanka , H3BO3 + H2O ® [B(OH)4]– + H +
USA and India. (ii) With C2H5OH and conc. H2 SO4, it gives triethylborate.
(ii) By the boiling of mineral colemanite with a solution of
H3BO3 + 3C2H5OH¾Conc. H SO4
¾¾¾2 ¾ ¾® B(OC2H5)3 +3H2O
Na2CO3.
Ca2B6O11 + 2Na2CO3 ® 2CaCO3 + 2NaBO2 + Na2B4O7 (iii) With NaOH, it gives sodium metaborate
Colemanite Borax H3BO3 + NaOH ® NaBO2 + 2H2O
Above NaBO2 can be reconverted by passing CO2 through (iv) Heating effect :
it.
4NaBO2 + CO2 ® Na2CO3+ Na2B4O7 H3BO3¾273K
¾¾® HBO¾
433K red
2 ¾¾® H2B4O7¾¾®
¾ B2O3
hot
Properties : Orthoboric Metaboric Tetraboric Boron
(i) Its aqueous solution is basic in nature due to hydrolysis. acid acid acid trioxide
Na2B4O7+ 7H2O ® 2NaOH + 4H3BO3 (Boric anhydride)
(ii) On heating with ethyl alcohol and conc. H2SO4, it gives
Uses : As an antiseptic and eye lotion under the name Boric
volatile vapours of triethylborate which burn with a green
lotion, and as a food preservative.
edged flame.
Na2B4O7+ H2SO4 + 5H2O ® Na2SO4+ 4H3BO3 Structure : It has a layer structure in which planar BO 33-
H3BO3 + 3C2H5OH ® B(OC3H5)3 + 3H2O units are linked by H- bonding, as shown in fig.
Triethylborate
308 Chemistry
H METALLURGY OF ALUMINIUM
O OCCURRENCE AND IMPORTANT MINERALS :
H B It is the most abundant metal found in the earth’s crust. It does
O O not occur in the free state in nature.
H H Its important minerals are :
O OH H (a) Bauxite, Al2O3. 2H2O
OH B O (b) Diaspore, Al2O3. H2O
(c) Corundum, Al2O3
B HO H B (d) Cryolite, Na3 AlF6
H
O O O O (e) Alunite or alum stone, K2SO4. Al2(SO4)3. 3Al(OH)3
(f) Feldspar, K2O. Al2O3 6SiO2
H H H (g) Mica, KAl3Si3O10(OH)2
BORON HALIDES , BX3, (WHERE, X= F, Cl, Br OR I): (h) Kaolinite, Al2O3. 2SiO2 .2H2O
Preparation : By the reaction of boron and halogens at high EXTRACTION :
temperature.
Aluminium metal is extracted from bauxite. It involves following
2B + 3X2 ¾heat
¾ ¾® 2BX3 steps.
Properties : (i) Purification of bauxite : Bauxite usually contains silica as
(i) BF3 and BCl3 are gases, BBr3 is a volatile liquid and BI3 is a impurity. These impurities must be removed before
solid at room temperature. electrolysis, since aluminium, once prepared, cannot be freed
(ii) These are covalent in nature and act as lewis acids. The of other metals by refining it. The bauxite is first purified by
decreasing order of acid strength is. any of the following processes depending upon the nature
BI3 > BBr3 > BCl3 > BF3 of impurities present in it.
(a) Bayer’s process :
BORAZINE OR BORAZOLE OR TRIBORINE
Roasted
TRIAMINE, B3N3H6 : Finely powdered bauxite ¾¾ ¾¾® Roasted ore
FeO ®Fe 2O 3
It is a colourless liquid having a six membered ring of alternating
B and N atoms. It is also called inorganic benzene. It is prepared Caustic soda solution
¾¾¾¾¾¾¾¾
heat
® Al(OH) 3 ¾¾¾® Al 2O 3
by B2H6 as follows: High pressure(150ºC,80atm) pure
filtered, Fe2O3 as residue
3B2H6 + 6NH3 ¾180 ºC
¾ ¾® 2B3N3H6 + 12H2 The reactions involved are given below.
The p electrons in borazine are only partially delocalised. It is Al2O3.2H2O +2NaOH ® 2NaAlO2 + 3H2O
much more reactive than benzene, because there is a retention of
Bauxite Sod. meta
partial negative charge by nitrogen atoms in latter case. It is
aluminate
isosteric (presence of same number of atoms and electrons) with
benzene. NaAlO2 + 2H2O ® Al(OH)3 ¯ + NaOH
H
2Al(OH)3 ¾1473
¾¾
K
¾® Al2O3 + 3H2O
H B
– Alumina
B (b) Hall’s process :
H N: :N H H N+ +N H
2 3 + Na CO
Bauxite (Fine powder) ¾¾¾¾¾¾¾® solution
H B B H H B– –B H fused,extracted with water
r esidueSiO 2 , Fe 2 O 3
N +
N
CO Heat
¾¾ ¾ ¾2¾¾® ppt. Al(OH)3 ¾¾¾® PureAl2O3
H H 50 - 60 º C and filtraed
Borazine filtrate ( Na 2CO 3 )
H
The reactions involved are given below.
C Al2O3. 2H2O + Na2CO3 ®
2NaAlO2 + CO2 + 2H2O
H C C H
2NaAlO2 + 3H2O+ CO2 ¾323 -333K
¾ ¾¾ ¾®
H C 2Al(OH)3 ¯ + Na2CO3
C H
(c) Serpek’s process : This process is employed when silica
C content of ore is high.
H
Benzene Finely powdered bauxite ¾+¾coke
¾ ¾¾
+ N2
®
Heated to1800º C
The p -Block Elements – Boron Family 309
USES :
silica reduced to Si which volatalises
(i) A mixture of aluminium powder and aluminium nitrate is
+ Alumina form aluminium nitride AlN known as Ammonal and is used in bombs.
(ii) A mixture of Al powder in linseed oil is used as silver paint.
hydrolysis
(iii) The reduction of metal oxides by aluminium is called
aluminothermy or thermite process or Goldschmidt
Heated aluminothermite process.
Pure Al2O3 ¬¾ ¾
¾¾ Al(OH)3
(ppt) COMPOUNDS OF ALUMINIUM
ANHYDROUS ALUMINIUM CHLORIDE, AlCl3 (OR
Al2Cl6) :
Filtrate Preparation : It can not be prepared by heating AlCl3. 6H2O
(sod. aluminate + sod. silicate) because of its hydrolysing tendency by its own water as below.
The involving reactions are given below.
D
2AlCl3. 6H2O ¾¾® 2Al(OH)3 + 6HCl
Al2O3. 2H2O + N2 + 3C ¾2000
¾¾ ¾®
K
2Al(OH)3 ¾heat
¾¾® Al2O3 + 3H2O
2AlN + 3CO ­ + 2H2O
¾® Si ­ + 2CO ­
SiO2 + 2C ¾ However, it can be prepared by following methods:
AlN + 3H2O ¾
¾® Al(OH)3 ¯ + NH3 (i) By passing dry chlorine or HCl gas over
(ii) Electrolysis of fused alumina (Hall and Heroult’s process) heated Al.
: Since pure alumina is a bad conductor of electricity, 2Al + 3Cl2 ¾heat
¾¾® 2AlCl3
therefore it is dissolved in molten cryolite. Na3AlF6 and
2Al + 6HCl ¾heat
¾¾® 2AlCl3 + 3H2
fluorspar, CaF2 to decrease its fusion temperature. The
molten electrolyte is covered with a layer of powdered coke (ii) By heating a mixture of alumina and carbon in a current of
to prevent oxidation and loss of heat due to radiation. dry chlorine.
The reactions are : Al2O3 + 3C + 3Cl2 ¾heat
¾¾® 2AlCl3 + 3CO
Na3AlF6 3NaF + AlF3
AlF3 Al3+ + 3F– Properties :
At cathode : Al3+ + 3e– ® Al (i) It fumes in moist air due to hydrolysis
AlCl3 + 3H2O ® Al(OH)3 + 3HCl
At anode : F– ® e– + F
The resulting solution is acidic due to the formation of HCl.
2Al2O3 + 12F ® 4AlF3 + 3O2
(ii) It behaves as lewis acid.
2C + O2 ® 2CO ;
(iii) It is a covalent solid and dissolves in organic solvents like
2CO + O2 ¾
¾® 2CO2 C6H6 etc.
(iii) Refining of aluminium : It is refined by Hoope’s electrolytic Structure : It exists as dimer Al2Cl6 in which each Al atom is
process, which is carried out in a graphite lined bath which terahedrally surrounded by four Cl atoms as below.
acts as the anode and carbon cathodes are used. The refining Cl Cl Cl
cell consists of three fused layer’s of different densities. Al Al
(a) The bottom layer is of molten impure aluminium (anode) Cl Cl Cl
(b) The middle layer is of fused cryolite and barium fluoride
(c) The upper layer is of pure aluminium (cathode) USES :
There is a new method of extraction of Aluminium in which the (i) As a catalyst in Friedel - Craft reactions
purification of the oxide is not of much importance. In this method, (ii) As a mordant in dyeing
AlCl3 vapour is passed through the fused oxide at 1000ºC ALUMINIUM OXIDE OR ALUMINA Al2O3 :
2Al2O3 + 2AlCl3 ® 6AlCl + 3O2 It is the most stable compound of aluminium and occurs in nature
Aluminium monochloride as colourless corundum and several coloured oxides, (when
The above aluminium monochloride vapour is unstable when present in combination with different metal oxides) like ruby (red),
cooled and disproportionates below at 800ºC. topaz (yellow), sapphire (blue), amethyst (voilet) and emerald
3AlCl ® AlCl3 + 2Al (green), which are used as precious stones (gems).
310 Chemistry
THERMITE :
A mixture of aluminium powder and ferric oxide in the ratio 1: 3.
ALUMINIUM SULPHATE, Al2(SO4)3 :
It is used for obtaining H2S in pure form and for making fire proof clothes.
ALLOYS OF ALUMINIUM :
Alloy Composition Properties Uses

(i) Duralumin Al + Cu + Mg + Mn Light, tough, ductile Aeroplanes and automobile parts


(95%) ( 4%) ( 0.5%) (0.5%)
(ii) Aluminium Bronze Al + Cu Light, strong, golden lustre Coins, jewellery
(90%) (10%)
(iii) Alcald Duralumin coated corrosion resistant, strong aircraft industry
with pure aluminium

(iv) Magnalium Al + Mg + Ca Light, tough and strong Balance beams and machinery
(83%) (15%) ( 2%)

(v) Alnico Al + Fe + Ni + Co Highly Magnetic Permanent magnets


( 20%) ( 50%) ( 20%) (10%)

ALUMS : Examples : FeSO4, Al2(SO4)3.24H2O


The term alum is given to double sulphates of the type X2SO4, Ferrous aluminium pseudo alum
Y2(SO4)3. 24H2O where X represents a monovalent cation such MnSO4.Al2(SO4)3.24H2O
as Na+,K+ and NH +4 Rb+, Cs+, Ag+ while Y is a trivalent cation Manganese aluminium pseudo alum
such a Al3+, Cr3+ and Fe3+, Co3+, Ga3+, V3+, Ti3+. (Li+ is too small to Properties :
be accomodated in the lattice) (i) Potash alum is a white crystalline compound.
General formula :
(ii) The aqueous solution of all alums is acidic due to hydrolysis
M '2SO 4 M '2'' (SO 4 ) 3 24H 2 O or M ' M ''' (SO 4 ) 2 .12H 2 O of Al2(SO4)3, Cr2(SO4)3 or Fe2(SO4)3 as given below.
Some important alums are : Al2(SO4)3 + 6H2O ® 2Al(OH)3 + 3H2SO4
(a) Potash alum K2SO4. Al2(SO4)3. 24H2O (iii) On heating all alums lose water of crystallization
(b) Sodium alum Na2SO4. Al2(SO4)3. 24H2O and swell up. The anhydrous alum is known as burnt alum.
(c) Ammonium alum (NH4)2SO4. Al2(SO4)3. 24H2O (iv) Ionisation of aqueous solution of a double salt is as
(d) Ferric alum (NH4)2SO4. Fe2(SO4)3. 24H2O
K2SO4. Al2(SO4)2. 24H2O ®
(e) Chrome alum K2SO4. Cr2(SO4)3. 24H2O
Out of these, potash alum is the most important which is 2K+ + 2Al3+ + 3SO 24- + 24H2O
prepared in the laboratory by mixing hot solutions of
equimolar quantities of K2 SO 4 and Al 2 (SO 4 ) 3 . The Uses :
resulting solution on concentration and crystallization gives (i) In purification of water
potash alum (emperical formula is KAl(SO4)2. 12H2O). (ii) For sizing of paper
Pseudo alums : When monovalent element of ordinary alums is (iii) As a syptic to stop bleeding
replaced by a bivalent element eg Mn2+, Fe2+, Mg2+, Cu2+ or Zn2+,
the alums are called pseudo alums. (iv) As a mordant in dyeing and tanning of leather
The p -Block Elements – Boron Family 311

Very Short/ Short Answer Questions Multiple Choice Questions


1. Why boron forms electron deficient compounds? 15. The element which exists in liquid state for a wide range of
2. Why boron halides do not exist as dimer while AlCl3 exists temperature and can be used for measuring high temperature
as Al2Cl6? is
3. The +1 oxidation state is more stable than the (a) B (b) Al
+ 3 oxidation state for thallium. (c) Ga (d) In
4. Why is boron used in nuclear reactors? 16. The exhibition of highest co-ordination number depends
on the availability of vacant orbitals in the central atom.
5. Between AlF3 and AlCl3 which one will have a higher melting
Which of the following elements is not likely to act as central
point.
3-
6. BCl3 exists but BH3 does not. Explain. atom in MF6 ?
7. Explain why boron halides don’t conduct electricity in the (a) B (b) Al
liquid state? (c) Ga (d) In
8. What is the use of boron nitride? 17. Identify the statement that is not correct as far as structure
9. (a) Why BF3 forms adduct with ammonia? Explain with of diborane is concerned
reaction. (a) There are two bridging hydrogen atoms and four
(b) How does electron deficient compound BF3 achieve terminal hydrogen atoms in diborane
electronic saturation, i. e., fully occupied outer electron (b) Each boron atom forms four bonds in diborane
shells? (c) The hydrogen atoms are not in the same plane in
10. What happens when borax solution is acidified? Write a diborane
balanced equations to support your answer. (d) All, B – H bonds in diborane are similar
11. (a) Which one is more soluble in diethyl ether, anhydrous 18. Aluminium vessels should not be washed with materials
AlCl3 or hydrated AlCl3? containing washing soda because
(b) Why BBr3 is a stronger Lewis acid than BF3? (a) washing soda is expensive
12. Account for the following : (b) washing soda is easily decomposed
(a) Aluminium sulphide gives a foul odour when it becomes (c) washing soda reacts with aluminium to form soluble
damp. aluminate
(b) Although aluminium is above hydrogen in the (d) washing soda reacts with aluminium to form insoluble
electrochemical series, still it is stable in moist air. aluminium oxide
19. Which is false in case of boric acid H3BO3?
Long Answer Questions (a) It acts as a tribasic acid.
13. (i) Compound X on reduction with LiAlH4 gives a hydride (b) It has a planar structure.
Y. The compound Y reacts with air explosively resulting (c) It acts as a monobasic acid.
in boron trioxide. Identify X and Y. Give balanced
(d) It is soluble in hot water.
equations involved in the formation of
Y and its reaction with air. 20. Aluminium chloride is a/an
(ii) Assign reasons for each of the (a) Bronsted - Lowery acid
(a) Gallium (+1) undergoes dispro-portionation (b) Arhenius acid
reactions. (c) Lewis acid
(b) Unlike In+, Tl+ is more stable with respect to (d) Lewis base
disproportionation. 21. A compound X, of boron reacts with NH3 on heating to
(c) InCl undergoes disproportionation but TlCl does give another compound Y which is called inorganic benzene.
not. The compound X can be prepared by treating BF3 with
Lithium aluminium hydride. The compounds X and Y are
14. (a) Why borazine is called inorganic benzene? represented by the formulas.
(b) Why Ga has smaller atomic radii than aluminium? (a) B2H6, B3N3H6 (b) B2O3, B3N3H6
(c) Why aluminium is used as a structural material ? (c) BF3, B3N3 H6 (d) B3N3H6, B2H6
312 Chemistry
22. AlCl3 on hydrolysis gives 24. Ionisation enthalpy (DiH1kJ mol–1) for the elements of Group
(a) Al2O3. H2O (b) Al(OH)3 13 follows the order.
(c) Al2O3 (d) AlCl3.6H2O (a) B > Al > Ga > In > Tl
23. Which is not correct? (b) B < Al < Ga < In < Tl
(a) Al acts as a reducing agent (c) B < Al > Ga < In > Tl
(b) Al does not react with steam even at higher temperature (d) B > Al < Ga > In < Tl
(c) Al forms a number of alloys with other metals
(d) Al is ionic in all its compounds

1. The state of hybridization of B in BCl3 is 11. Which of the following hydroxide is acidic ?
(a) sp 3 (b) sp 2 (a) Al(OH)3 (b) Ca(OH)3
(c) sp (d) sp 3d 2 (c) Tl(OH)3 (d) B(OH)3
2. The type of hybridization of boron in diborane is 12. BF3 acts as an acid according to the concept of
(a) sp (b) sp 2 (a) Lewis (b) Bronsted
(c) sp 3 (d) sp 3d 2 (c) Arrhenius (d) None of these
3. Specify the coordination geometry around and hybridization 13. Which of the following is electron deficient ?
of N and B atoms in a 1: 1 complex of BF 3 and NH3 (a) NH3 (b) BCl3
(a) N : tetrahedral, sp3; B : tetrahedral, sp3 (c) PCl3 (d) None of these
(b) N : pyramidal, sp3; B : pyramidal, sp3 14. NH3 and BF3 form an adduct readily because they form
(c) N : pyramidal, sp3; B : planar, sp3 (a) a coordinate bond
(d) N : pyramidal, sp3; B : tetrahedral, sp3 (b) a hydrogen bond
(c) an ionic bond
4. Which one of the following elements has the highest melting
point ? (d) a covalent bond
15. BF3 is used as a catalyst in several industrial processes due
(a) Boron (b) Aluminium
to its
(c) Gallium (d) Thallium
(a) strong reducing nature
5. The element which shows least metallic character is
(b) weak reducing action
(a) Indium (b) Boron
(c) strong Lewis acid nature
(c) Aluminium (d) Gallium
(d) weak Lewis acid character
6. Which of the following does not form M3+ ion? 16. Among the halides of the elements of group 13 the one
(a) Boron (b) Aluminium which is most acidic is
(c) Indium (d) Gallium (a) BF3 (b) AlCl3
7. Which of the following does not react with aqueous (c) BCl3 (d) BBr3
NaOH ? 17. In diborane
(a) B (b) Al (a) 4–bridged hydrogens and two terminal hydrogens are
(c) Ga (d) Tl present
8. B2O3 is (b) 2– bridged hydrogens and four terminal hydrogens are
(a) acidic (b) basic present
(c) amphoteric (d) None of these (c) 3–bridged and three terminal hydrogens are present
9. Boric acid is polymeric due to (d) None of these
(a) its acidic nature 18. Identify the statement that is not correct as far as structure
(b) the presence of hydrogen bonds of diborane is concerned
(c) its monobasic nature (a) There are two bridging hydrogen atoms and four terminal
(d) its geometry hydrogen atoms in diborane
10. B(OH)3 is (b) Each boron atom forms four bonds in diborane
(c) The hydrogen atoms are not in the same plane in
(a) monobasic acid (b) dibasic acid
diborane
(c) tribasic acid (d) triacidic base
(d) All, B – H bonds in diborane are similar
The p -Block Elements – Boron Family 313
19. BCl3 does not exist as dimer but BH3 exists as dimer (B2H6) 32. A lake can be obtained by making a mixture of a coloured
because dye with
(a) chlorine is more electronegative than hydrogen (a) NH4OH (b) Ba(OH)2
(b) there is pp-pp back bonding in BCl3 but BH3 does not (c) Al(OH)3 (d) NaOH
contain such multiple bonding 33. AlCl3 is
(c) large sized chlorine atoms do not fit in between the (a) anhydrous and covalent
small boron atoms where as small sized hydrogen atoms
(b) anhydrous and ionic
get fitted in between boron atoms
(d) None of these (c) covalent and basic
20. Diborane upon hydrolysis gives (d) coordinate and acidic
(a) boric anhydride (b) metaboric acid 34. Anhydrous AlCl3 is prepared from
(c) orthoboric acid (d) boron oxide (a) conc. HCl and Al metal
21. In reaction (b) aluminium and Cl2
BF3 + 3LiBH4 ® 3LiF + X ; X is (c) dry HCl gas + heated Al metal
(a) B4H10 (b) B2H6 (d) dil. HCl and Al metal
(c) BH3 (d) B3H8 35. Aluminium chloride is a/an
22. Which of the following mineral does not contain Al ? (a) Bronsted - Lowry acid (b) Arrhenius acid
(a) Cryolite (b) Mica (c) Lewis acid (d) Lewis base
(c) Feldspar (d) Fluorspar 36. Which member of group 13 does not exhibit the group
23. In the thermite welding process we use valency in its compounds ?
(a) Al powder (b) Fe powder (a) Boron (b) Aluminium
(c) Ca powder (d) (Al + Fe) mixture
(c) Gallium (d) Thallium
24. In aluminates, the coordination number of Al is
37. The highly toxic element of group 13 is
(a) 4 (b) 6
(a) Al (b) B
(c) 3 (d) 1
(c) Ga (d) Tl
25. Aluminium is
(a) a reducing agent 38. Which of the following oxides is strongly basic ?
(b) an oxidising agent (a) B2O3 (b) Al2O3
(c) amphoteric (c) Ga2O3 (d) Tl2O3
(d) highly electronegative element 39. Which out of the following compounds does not exist?
26. Al is more reactive than Fe but Al is less easilly corroded (a) BF3 (b) TlCl3
than Fe because (c) TlCl5 (d) Both (b) and (c)
(a) it is a noble metal 40. Which of the following does not give a borax bead
(b) oxygen forms a protective oxide layer test ?
(c) iron undergoes reaction easily with water (a) Chromium (b) Ferrous salt
(d) Fe form mono and divalent ions. (c) Sodium (d) Cobalt
27. Corundum is 41. Borazole is known as
(a) Al2(SO4)3 (b) Al2O3.H2O (a) organic benzene (b) organic xylene
(c) Al2O3.2H2O (d) Al2O3
(c) inorganic benzene (d) inorganic xylene
28. Although Al has a high oxidation potential it resists
42. The I.E1 among the group 13 member follows as
corrosion because of the formation of a tough, protective
coat of (a) B > Al < Ga < Tl (b) B > Al > Ga > Tl
(a) Al(NO3 )3 (b) AlN (c) B > Ga > Al > Tl (d) B > Ga < Al < Tl
(c) Al2O3 (d) Al2(CO3)3 43. The melting pt. of group 13 follows the order
29. AlCl3 on hydrolysis gives (a) B > Al > Ga > In > Tl (b) B > Al < Ga > In > Tl
(a) Al2O3. H2O (b) Al(OH)3 (c) B > Al > Tl > In > Ga (d) B > Al < Ga < In < Tl
(c) Al2O3 (d) AlCl3.6H2O 44. The compounds of boron and hydrogen are collectively
30. Action of caustic soda on aluminium hydroxide gives a called
compound having formula (a) diboranes (b) borazoles
(a) Al2(OH)4 (b) Na2Al(OH)4
(c) boracits (d) boranes
(c) NaAlO2 (d) Na3AlO3
45. The bonds present in borazole or inorganic benzene are
31. Al(OH)3 is
(a) 9 s, 6p (b) 12 s, 3p
(a) acidic (b) basic
(c) amphoteric (d) neither acidic nor basic (c) 6 s, 9p (d) 15 s only
314 Chemistry
46. The aluminium salt commonly used to stop bleeding is (b) washing soda gets easily decomposed
(a) aluminium sulphate (b) potash alum (c) washing soda reacts with aluminium to form soluble
(c) aluminium chloride (d) aluminium fluoride aluminate
47. The nature of the solution of Potash alum is (d) washing soda reacts with aluminium to form insoluble
(a) basic (b) acidic aluminium oxide
(c) neutral (d) amphoteric 49. Which out of the following is potash alum ?
48. Aluminium vessels should not be washed with materials
containing washing soda because (a) K2SO4.Al2(SO4)3.24H2O (b) K2SO4. Cr2(SO4)3.24H2O
(a) washing soda is expensive (c) K2SO4.Fe2(SO4)3.24H2O (d) [NH4]2SO4.FeSO4.6H2O

1. In borax bead test which compound is formed? (b) alumina


[CBSE-PMT 2002] (c) alumina mixed with molten cryolite
(a) Ortho-borate (b) Meta-borate (d) molten cryolite.
(c) Double oxide (d) Tetra-borate 8. Aluminium chloride exists as dimer, Al2Cl6 in solid state as
2. Al2O3 can be converted to anhydrous AlCl3 by heating well as in solution of non-polar solvents such as benzene.
When dissolved in water, it gives [AIEEE 2004]
(a) Al2O3 with NaCl in solid state [CBSE-PMT 2006]
(b) a mixture of Al2O3 and carbon in dry Cl2 gas (a) [Al(OH) 6 ]3- + 3HCl
(c) Al2O3 with Cl2 gas
(b) [Al(H 2 O) 6 ]3+ + 3Cl -
(d) Al2O3 with HCl gas
3. The tendency of BF3, BCl3 and BBr3 to behave as Lewis acid (c) Al3+ + 3Cl -
decreases in the sequence: [CBSE-PMT 2010]
(d) Al 2 O 3 + 6HCl
(a) BCl3 > BF3 > BBr3
(b) BBr3 > BCl3 > BF3 9. Heating an aqueous solution of aluminium chloride to dryness
will give [AIEEE 2005]
(c) BBr3 > BF3 > BCl3
(d) BF3 > BCl3 > BBr3 (a) Al(OH)Cl 2 (b) Al 2 O 3
4. Aluminium is extracted from alumina (Al2O3 ) by electrolysis (c) Al 2 Cl 6 (d) AlCl3
of a molten mixture of : [CBSE-PMT 2012 S]
10. The structure of diborane ( B 2 H 6 ) contains [AIEEE 2005]
(a) Al2O3 + HF + NaAlF4
(a) four 2c-2e bonds and four 3c-2e bonds
(b) Al2O3 + CaF2 + NaAlF4
(b) two 2c-2e bonds and two 3c-3e bonds
(c) Al2O3 + Na3AlF6 + CaF2
(c) two 2c-2e bonds and four 3c-2e bonds
(d) Al2O3 + KF + Na3AlF6
(d) four 2c-2e bonds and two 3c-2e bonds
5. Which of the following structure is similar to graphite?
11. Which one of the following is the correct statement?
(a) B (b) B4C [NEET 2013]
(a) Boric acid is a protonic acid [AIEEE 2008]
(c) B2H6 (d) BN (b) Beryllium exhibits coordination number of six
6. Alum helps in purifying water by [AIEEE 2002] (c) Chlorides of both beryllium and aluminium have bridged
(a) forming Si complex with clay partiles structures in solid phase
(b) sulphate part which combines with the dirt and removes (d) B2H6.2NH3 is known as ‘inorganic benzene’
it 12. How can the following reaction be made to proceed in forward
(c) coagulaing the mud particles direction? [IIT-JEE 2006]
(d) making mud water soluble. B(OH)3 + NaOH NaBO2 + Na[B(OH)4] + H2O
7. Aluminium is extracted by the electrolysis of [AIEEE 2002] (a) addition of borax (b) addition of cis -1,2-diol
(a) bauxite (c) addition of Na2HPO4 (d) addition of trans -1,2-diol
The p -Block Elements – Boron Family 315

1. On adding ammonium hydroxide solution to 10. The two type of bonds present in B2H6 are covalent and
Al2(SO4)3 (aq) : (a) ionic (b) co-ordinate
(a) A precipitate is formed which does not dissolve in excess (c) hydrogen bridge bond (d) None of these
of ammonium hydroxide 11. Orthoboric acid when heated to red hot gives
(b) A precipitate is formed which dissolves in excess of (a) metaboric acid (b) pyroboric acid
ammonia solution (c) boron and water (d) boric anhydride
(c) No precipitate is formed 12. Anodised aluminium is
(d) None of these (a) Al obtained at anode
2. The purification method used for mineral Al 2 O 3 .2H 2 O is (b) Al prepared electrolytically
(a) froth floatation (b) leaching (c) Alloy of Al containing 95% of Al
(c) liquation (d) magnetic separation (d) Al electrolytially coated with aluminium oxide
3. The process used for purification of bauxite ore containing 13. Which statement regarding H3BO3 is not correct ?
high silica content as impurity is (a) It is a strong tribasic acid
(a) Baeyer’s process (b) Hall’s process (b) It is prepared by acidifying an aqueous solution of borax
(c) Hoope’s process (d) Serpeck’s process (c) It has a layer structure in which planar BO3 units are
4. Hydrogen forms a bridge in the chemical structure of : joined by H- bonds
(a) sodium peroxide (b) diborane (d) It does not act as proton donor but acts on lewis acid by
(c) hydrogen peroxide (d) lithium hydride accepting OH– ions
14. The precious Ruby stone is
5. The role of fluorspar ( CaF2) which is added in small quantities
in the electrolytic reduction of alumina dissolved in fused (a) alumina
cryolite (Na3AlF6) is (b) aluminium silicate
(a) as a catalyst (c) sodium aluminium silicate
(b) to make the fused mixture very conducting (d) sodium silicate
(c) to increase the temperature of the melt. 15. The hybridisation of boron atom in orthoboric acid is
(d) to decrease the rate of oxidation of carbon at the anode. (a) sp (b) sp 2
6. In Gold Schmidt reaction, certain metallic oxides are reduced (c) sp 3 (d) sp 3 d
to the metallic state by heating with 16. Which of the following statements is not correct ?
(a) metallic magnesium (b) metallic aluminium (a) Al acts as a reducing agent
(c) metallic iron (d) sodium metal (b) Al does not react with steam even at higher temperature
7. The dissolution of Al(OH)3 by a solution of NaOH results in (c) Al forms a number of alloys with other metals
the formation of (d) Al is ionic in all its compounds
17. Which of the following processes does not involve a catalyst?
(a) [Al(H 2 O) 4 (OH)]2 + (b) [Al(H 2 O) 2 (OH) 4 ]-
(a) Thermite process (b) Ostwald process
(c) [Al(H 2 O) 3 (OH) 3 ] (d) [Al(H 2 O) 6 (OH)3 ] (c) Contact process (d) Haber process
8. Which of the following is pseudo alum ? 18. The factor responsible for weak acidic nature of B–F bonds
in BF3 is
(a) (NH 4 )2 SO 4 .Fe2 (SO4 )3.24H 2O
(a) large electronegativity of fluorine
(b) K 2SO 4 .Al2 (SO4 )3 .24H 2O (b) three centred two electron bonds in BF3
(c) pp - dp back bonding
(c) MnSO 4 .Al2 (SO4 )3 .24H 2O
(d) pp - pp back bonding
(d) None of these 19. An aqueous solution of FeSO4, Al2(SO4)3 and chrome alum
9. Which of the following statements about anhydrous is heated with excess of Na2O2 and filtered. The materials
aluminium chloride is correct ? obtained are :
(a) It exist as AlCl3 molecule (a) a colourless filtrate and a green residue
(b) It is a strong lewis base (b) a yellow filtrate and a green residue
(c) It sublimes at 100°C under vaccum (c) a yellow filtrate and a brown residue
(d) It is not easily hydrolysed. (d) a green filtrate and a brown residue
316 Chemistry
20. Which of the following has the minimum heat of dissociation: 22. Which reaction cannot give anhydrous AlCl3 :
(a) Passing dry Cl2 over heated aluminium powder.
(a) (CH3 )3 N : ® BF3
(b) Heating a mixture of alumina and coke in a current of dry
Cl2.
(b) (CH3 )3 N : ® B(CH3 )2 F
(c) Passing dry HCl over heated aluminium powder.
(c) (CH3 )3 N : ® B(CH 3 ) 3 (d) Heating of AlCl3 .6H 2 O .
23. The process used for purification of bauxite ore containing
(d) (CH3 )3 N : ® B(CH 3 )F2 iron oxide impurity is known as :
21. Purification of alumina is essential because : (a) Hoope’s process (b) Serpeck’s process
(c) Baeyer’s process (d) Electrolytic process
(a) impure alumina has a very high melting point. 24. Which statement is not true about potash alum ?
(b) impure alumina is a very poor conductor of electricity. (a) On heating it melts and loses its water of crystallization.
(c) impure alumina cannot react with the oxidizing agent. (b) It’s aqueous solution is basic in nature.
(c) It is used in dyeing industries.
(d) it is difficult to purify aluminium metal.
(d) It’s empirical formula is KAl(SO 4 )2 .12H 2 O .

EXERCISE 1 10. (a)


4. Boron can absorb neutrons. 11. (d) B(OH)3 is acid because it can take OH– ions.
5. AlF3 is more ionic, therefore, has higher melting point. H3BO3 or B(OH)3 + OH– ® B(OH) -4
7. Boron halides don’t conduct electricity in the liquid state
because they are covalent compounds. 12. (a) Since BF3 is an electron deficient molecule (according
to lewis concept).
8. Boron nitride is harder than diamond and is used as an
abrassive. 13. (b) 14. (a) 15. (d) 16. (d) 17. (b)
10. Boric acid is formed on acidification of Borax (Na2B4O7). 18. (d) 19. (c) 20. (c) 21. (b)
Na2B4O7 + 2HCl + 5H2O ¾¾
® 2NaCl + 4H3BO3 22. (d) Fluorspar (CaF2) does not contain Al whereas cryolite
(Na 3AlF 6 ) mica [KAl 3.Si 3 O 10 (OH) 2 ] and feldspar
Na2B4O7 + H2SO4 + 5H2O ¾¾
® 4H3BO3 + Na2SO4
(KAlSi3O8) contain Al.
15. (c) 16. (a) 17. (d) 18. (c) 19. (a) 23. (a)
20. (c) 21. (a) 22. (b) 23. (d) 24. (d) 24. (b) In aqueous solution the probable aluminate
species is [Al(H 2 O) 2 (OH) 4 ] hence Al containing
co-ordination number 6
EXERCISE 2
25. (a) Because it can lose electrons.
1. (b) 2. (c) 3. (a) 26. (b) The protective oxide layer prevents Al from further
4. (a) Boron has a giant molecule structure like that of diamond corrosion.
therefore it has highest melting point 27. (d) 28. (c) 29. (b)
5. (b) 30. (c) Al(OH)3 + NaOH ® NaAlO2 + 2H2O
6. (a) Due to its small size and high ionization energy boron 31. (c) 32. (c) 33. (a)
does not form B3+ ion. D
34. (c) 2Al + 3Cl2 ¾¾® 2AlCl3 (anhydrous)
7. (a) 8. (a)
35. (c) 36 (c) 37. (d)
9. (b) In Boric acid each B atom is sp2 hybridized and contains
38. (d) The correct sequence of increasing basic strength is
BO 33- units which are held together by hydrogen bonds.
B2O3 < Al2O3 < Ga2O3 < In2O3 < Tl2O3
The p -Block Elements – Boron Family 317
+5 does
39. (c) Because Tl not exist 6. (c) Alum furnishes Al3+ ions which bring about coagulation
40. (c) Colourless salt or ion (ex. Na) will not give a borax bead of negatively charged clay particles, bacteria etc.
test. 7. (c) Alumina is mixed with cryolite which acts as an
41. (c) electrolyte.
42. (c) The IE1 of Ga is more than that of Al because of the small
-
atomic size and greater effective nuclear charge of Ga. 8. (b) AlCl6 + 12H 2 O 2[Al(H 2 O) 6 ]3+ + 6Cl
43. (c) Due to structural changes, melting point, increases from
9. (b) The solution of aluminium chloride in water is acidic
Ga to Tl and Ga has the lowest melting point.
due to hydrolysis.
44. (d) 45. (b) 46. (b) 47. (b) 48. (c)
49. (a) AlCl3 + 3H2O –––––® Al(OH)3 + 3HCl.
On heating it till dryness Al(OH)3 is converted into
EXERCISE 3 Al2O3
D D Al O + 3H O
1. (b) Na2B4O7. 10 H2O ¾¾¾¾
-10H O
® Al(OH)3 ¾¾ ® 2 3 2
2
10. (d) In diborane (B2H6) structure there are four 2c-2e bonds
D
Na 2 B4 O7 ¾¾® 2 NaBO2 + B2 O 3 and two 3c–2e bonds (see structure of diborane).
anhydrous sod.metaborate Boric anhydride
Structure of B2H6 :
CuO + B2 O3 ¾¾
® Cu(BO2 )2
cupric meta borate(Blue beed) Hb
2. (b) Al2O3 can be converted to anhydrous AlCl3 by heating Ht •• Ht
a mixture of Al2O3 and carbon in dry Cl2
B B
1000°C
Al2O3 + 3C + 3Cl2 2AlCl3 + 3CO
vapours
Solid anhydrous Ht •• Ht
cooled Hb
aluminium
chloride
3. (b) p-p overlap between B and F is maximum due to identical 11. (c) The correct formula of inorganic benzene is B3N3H6 so
size and energy of p-orbitals, so electron deficiency in (d) is incorrect statement
boron of BF3 is neutralized partially to the maximum OH
extent by back donation. Hence BF3 is least acidic. |
As the size of halogen atom increases from F to I, the Boric acid (H3BO3 or B - OH ) is a lewis acid so (a) is
|
extent of overlap between 2p-orbital of B and a bigger OH
incorrect statement.
p-orbital of halogen decreases. Therefore the electron
deficiency of B increases. The coordination number exhibited by beryllium is 4
4. (c) Fused alumina (Al2O3) is a bad conductor of electricity. and not 6 so statement (b) is incorrect.
Therefore, cryolite (Na3AlF6) and fluorspar (CaF2) are Both BeCl2 and AlCl3 exhibit bridged structures in solid
added to purified alumina which not only make alumina state so (c) is correct statement.
a good conductor of electricity but also reduce the
melting point of the mixture to around 1140 K. Cl m Cl Cl Cl Cl
5. (d) Boron nitride (BN) is known as inorganic graphite. The 2p
20
most stable form is hexagonal one. It has layered Be 98° Be 263 pm Be Be
structure similar to graphite. 82°
Cl Cl Cl Cl Cl
N N+

+
B– –B B Cl Cl Cl

N+ +N N Al Al
– +
–B B B– –B Cl Cl Cl

+N N N+ – + N 12. (b) cis-1,2-diol forms chelated complex ion with the


+
B– –B B product, [B(OH)4]– causing the reaction to proceed in
forward direction.
N+ +N N
– +
B B–
318 Chemistry

CH2–OH

HO–CH2
– 9. (c) AlCl3 is Lewis acid, exists as dimer (Al2Cl6) and easily
HO OH CH2–O O–CH2
+ B + B hydrolysed
CH2–OH HO OH HO–CH2 CH2–O O–CH2
10. (c) B2H6 contains hydrogen bridge bonds. These are one
Stable chelated complex ion
electron bonds also known as banana bonds.
EXERCISE 4 °C 160°C
11. (b) H 3 BO 3 ¾100
¾ ¾® HBO 2 ¾¾ ¾® H
1. (a) Al2 (SO 4 )3 + 6NH 4 OH ® 2Al(OH)3 + 3(NH 4 ) 2 SO 4

Al(OH)3 + NaOH ® Na + [Al(OH) 4 ]-Soluble complex H 2B4O7 + H 2O ¾


¾® 2B 2 O 3 + H 2 O
12. (b) Al electrolytically coated with aluminium oxide is known
it is insoluble in NH 4 OH
as anodised aluminium.
2. (b) Purification of Al 2 O 3 .2H 2 O is done by leaching. It 13. (a) H3BO3 is weak monobasic acid
dissolves the ore leaving behind impurities + –
H 3BO3+H 2O H 3O+H 2BO 2
3. (d) Serpeck’s process is employed when silica content of
ore is high 14. (a) Alumina Al2O3 is known as Ruby stone
4. (b) See text for the structure of diborane. 15. (b) The hybridizations of B in H3BO3 is sp2
5. (b) CaF2 when added to fused cryolite, lowers the m.0p0. 16. (d) Al in its compounds forms covalent bonds.
and increases the conductivity.
17. (a) In thermite process no catalyst is required
6. (b) Aluminium is reducing in nature. The reduction of Fe2O3
18. (d) It is pp - pp back bonding involving B and F. The smaller
by Al is known as Gold Schmidt alumino thermic process
atoms show more back bonding.
2Al + Fe 2 O 3 ® Al 2 O 3 + 2Fe DH = - ve (exothermic)
19. (c) The filtrate is yellow due to CrO 24- ion and residue is
7. (b) Al(OH)3 + OH - ® [Al(OH) 4 ]- brown due to Fe(OH)3.
2H O
20. (c) Due to + I effect of methyl groups the Lewis character of
¾¾¾®
2 [Al(OH) 4 (H 2 O) 2 ]- B(CH3)3 decreases and coordination becomes weaker.
8. (c) General formula for alum is 21. (d) Statement (d) is correct.

M '2 SO 4 .M "2 (SO 4 ) 3 .24 H 2 O. When M ¢ is replaced by 22. (d) 2 AlCl 3 . 6H 2 O ¾


¾® Al 2 O 3 + 6HCl + 9H 2 O
divalent ion the alum is known as pseudo alum.
23. (c) Bauxite ore containing Fe 2 O 3 is purified by Baeyer’ss
process.

24. (b) Alum form acidic solution due to hydrolysis of Al 3+ .


11B
The p-Block Elements –
Carbon Family
GENERAL CHARACTERISTICS · The compounds in +2 oxidation state are ionic in nature
1. Electronic configuration : and in +4 oxidation state are covalent in nature (Fajan’s
rule)
Element Atomic No. Electronic configuration Valence shell
configuration 7. Negative oxidation states- Carbon forms C 4- and C 22- in
certain compounds e.g.
Carbon 6 [He] 2s 2 2 p 2 2s2 2 p2
Be 2 C (Be 2+ and C 4- ), Al 4 C3 (Al3+ , C 4 - )
Silicon 14 [Ne] 3s 2 3 p 2 3s 2 3 p 2 and CaC 2 (Ca 2 + and C 22 - )
8. Ionisation energy - It decreases from C to Sn . For Pb it is
Germanium 32 [Ar] 3d 10 4s 2 4 p 2 4s 2 4 p 2 slightly higher than Sn.
9. Electronegativity values - The values decrease from C to Pb
Tin 50 [Kr] 4d 10 5s 2 5 p 2 5s 2 5 p 2
but not in a regular manner probably due to filling of d-
orbitals in Ge and Sn and f-orbitals in Pb.
Lead 82 [Xe] 4 f 14 5d 10 6 s 2 6 p 2 6s 2 6 p 2
10. Catenation - It is the tendency of an element to form long
2. Metallic character - C and Si are non metals, Ge is a metalloid chains of identical atoms. The greater the strength of element-
and Sn and Pb are metals. element bond, the greater is the strength of catenation.
3. Appearance - C is black , Si is light-brown , Ge greyish white, C >> Si > Ge » Sn > Pb (catenation )
Sn and Pb are silvery white.
4. Density - Density increases with the increase in atomic Bond ® C– C Si–Si Ge– Ge Sn–Sn
number due to increase in mass per unit volume. Bond kJ/mole 353.3 225.7 167.2 155.0
5. Melting points and Boiling points - The melting points and
11. Allotropy - All the elements except Pb show allotrpy.
boiling points decrease from carbon to lead but carbon and
silicon have very high melting and boiling points due to Allotropic forms of carbon - Diamond, Graphite and Fullerene
their giant structure. Amorphous forms of carbon - coal, charcoal etc.
6. Oxidation states - They are as follows Silicon (Si) - crystalline and amorphous
C Si Ge Sn Pb Tin (Sn) - grey tin, white tin and rhombic tin
Germanium - two crystalline forms
(+2) <+ 4 (+2 )<+ 4 +2 <+ 4 +2 <+ 4 +2> + 4
12. Valency - All elements exhibit tetravalency. In case of Carbon
· The compounds of Ge and Sn in +2 oxidation state are 406 kJ/ mole of energy is required for promotion of 2s electron
reducing in nature. Since their higher oxidation states +4 to 2p. Formation of two extra bonds provide this energy .
are more stable . 13. Inert- pair effect - On descending the group, the stability of
· The compounds of Pb in +4 oxidation state are powerful +4 oxidation state decreases and that of +2 oxidation state
oxidising in nature. Since +2 oxidation state of Pb is increases.
more stable .
320 Chemistry
14. Atomic and ionic radii - Both increase from C to Pb (iii) Oxides - They form two types of oxides
15. Atomic volume - Atomic volume shows a regular increase (a) Mono-oxides of the type MO
from C to Pb. CO(neutral) and SiO, GeO, SnO, PbO(all basic)
16. Formation of complexes - C does not give any complex due CO forms a number of coordination compounds with
to non availability of empty d orbitals in valence shell. transition metals e.g. Ni (CO )4 , Fe (CO )5 and Cr (CO) 6
The valence shell of Si and other elements contain d-orbitals (b) Dioxides of the type MO2
CO2 , SiO2 GeO2 , SnO2 and PbO2
and can accomodate more than 8 e - and can therefore form 14243 1444 424444 3
Acidic Amphoteric
complexes e.g.
CO2 is linear, gas at ordinary temperatue. Solid CO2 is
SiF4 + 2F - ® [SiF6 ]2 -
known dryice or drikold. SiO2 is solid with three
SnCl 4 + 2Cl - ® [SnCl 6 ]2 - dimensional network having Si bonded to four oxygen
The hybridisation in these complexes is sp 3 d 2 which is atoms tetrahedrally and covalently.
| |
octahedral. - Si - O - Si - O - Si -
17. Reactivity : Increases from C to Pb. | | |
O O O
18. Multiple bonding : Carbon forms pp - pp multiple bonds | | |
with itself and with S, N and O. Other elements show - Si - O - Si - O - Si -
| |
negligible tendency of this type due to their large size. Others
form dp - pp multiple bonds. The bond energy of Si - O bond is 368kJ/mol, therefore
SiO 2 is chemically inert and has high melting point.
FORMATION OF COMPOUNDS
GeO2 , SnO2 and PbO 2 all are network solids. PbO 2 is
(i) Hydrides : All form covalent hydrides .Their number and a powerful oxidising agent
ease of formation decreases down the group. PbO 2 + 2HNO 3 ® Pb(NO3 )2 + H 2 O + O 2
· Hydrides of carbon are known as Alkanes, Alkene or · Carbon also gives suboxide
Alkynes. C 3O 2 , O = C = C = C = O
· Hydrides of Si and Ge are known Silanes and Germanes · Lead also gives mixed oxide
but their number is limited. Pb 3 O 4 , 2PbO.PbO 2
· The only hydrides of Sn and Pb are SnH4 (Stannane) (iv) Acids - All elements give acids of the type H 2 MO3 e.g.
and PbH4 (Plumbane) . H 2 CO3 (carbonic acid), H 2SiO 3 (silicic acid), H 2SnO 3
· Their thermal stability decreases down the group. (meta stannic acid), H 2 PbO 3 (meta plumbic acid). Carbonic
· Their reducing character increases down the group. acid forms two series of salts, bicarbonates (HCO 3- ) and
(ii) Halides : All the elements give tetrahedral and covalent carbonates (CO 3- - ) .
halides of the type MX4 except PbBr 4 and PbI 4 , since SILICATES
-
Pb 4 + is strong oxidising and Br and I - are strong Silicates are metal derivatives of silicic acid H 2SiO 3 and can be
reducing agent. SnF4 is ionic. obtained by fusing metal oxides or metal carbonates with sand e.g.
(a) Stability - Order of thermal stability with common Fuse
Na 2 CO3 + SiO 2 ¾¾¾¾
®(Na 2SiO3 )n
halogen 1780K
CX 4 > SiX 4 > GeX 4 > SnX 4 > PbX 4 Type of silicates - Silicates contain SiO44- tetrahedra formed by
sp 3 hybridisation, depending upon the number of O-atoms shared
· Order of thermal stability with common metals
between tetrahedra and fashion, Silicates have been classified
MF4 > MCl 4 > MBr4 > MI 4
into following groups
(b) Hydrolysis - Except CX4 other tetrahalides are (i) Orthosilicates -They contain discrete SiO44- tetrahedra,
hydrolysed Examples are phenacite Be2SiO4 , willimite
SiX 4 + 2H 2 O ® SiO 2 + 4HX Zn 2SiO 4 , zircon ZrSiO4 , forsterite Mg2SiO4 .
· Ease of hydrolysis (ii) Pyrosilicates - Here two tetrahedra units are
SiX 4 > GeX 4 > SnX 4 > PbX 4 joined by one oxygen atom forming a large discrete Si 2 O 67- .
· CX4 are not hydrolysed due to absence of vacant Examples are thorteveitite Se2 (Si 2O7 ) , hemimorphite
d-orbitals in valence shell of carbon. Zn3 ( Si 2O7 ) .Zn ( OH )2 .H 2O
Dihalides -Except carbon other elements form dihalides of (iii) Chain silicates - Here two oxygen atoms per SiO4 tetrahedra
the type MX2 which are more ionic and have higher melting are shared giving polymeric anion chains. Discrete unit is
points and boiling points e.g . SnCl 2 is a solid whereas (SiO 32- ) n . Examples: synthetic sodium silicate Na 2SiO3 ,
SnCl 4 is a liquid at room temperature. lithium silicate Li 2SiO3 , natural spodumene LiAl(SiO 3 )2 ,
· C, Si and Ge form trihalides of the type MHCl3 . Pb and jodeite NaAl(SiO 3 )2 , enstalite MgSiO3 and diposide
Sn do not form trihalides of the type MHX3 CaMg (SiO 3 )2 .
The p -Block Elements –Carbon Family 321
(iv) Double chains -Here two simple chains are held together by (iii) Covalent carbides - Carbides of B and Si, B 4 C
6- and SiC are covalent .SiC is known as CARBORUNDUM,
shared oxygen atoms. The discrete unit is (Si 4 O11 )n .
Example mineral tremolde Ca 2 Mg Si O
5 14 11 (OH )2 used as abrasive and refractory material. B4C harder than
Double chains silicates is also called amphibole SiC and used as an abrasive.
(v) Cylic silicates - Here two oxygen atoms per SiO44- tetrahedra CARBON
are shared giving discrete unit Si 3O96- and Si 6 O12-
18 . Example
It is widely distributed in the free state (diamond, graphite, coal etc.)
Beryl Be3Al2Si 6 O18 . and in the combined state (oxides, carbonates hydrocarbons etc.)
Allotropic forms of carbon -The crystalline forms include
(vi) Sheet-silicates -Here three oxygen atoms per tetrahedra are
shared giving two dimensional sheet having discrete unit (i) Diamond -It is beautiful crystalline form, hardest, and has
(Si 2 O 52 - ) n . Example three dimensional polymeric structure, hybridisation of C is
Talc Mg(Si 2 O5 )2 Mg(OH )2 , Kaolin Al2 (OH)4 (Si 2 O5 ) . sp 3 . It is covalent solid, melting point 3650°C, density
3.51g / cc and bad conductor of heat and electricity
(vii)Frame work silicates - Here all four oxygen atoms of each
When heated at 1800°C - 2000°C, it is converted to graphite.
tetrahedra are shared. Example are quartz, zeolites, tridymite
and crystobalite. (ii) Graphite -Preparation
3C + SiO 2 ® SiC + 2CO 2 ® C + Si ­ (graphite)
SILICONES
It is dark grey, having hexagonal plates, hybrisation of C is
The polymeric compounds containing R 2SiO units, linear cyclic
sp 2 , good conductor of heat and electricity due to free
or cross linked are known as silicones. They are manufactured
movement of electrons. It was also known as black lead or
from alkyl substituted chlorosilanes
plumbago. It is very good lubricant.
Cu powder 2H O
2 RCl + Si ¾¾ ¾¾¾® R2 SiCl2 ¾¾¾
2¾® Aqua dag - suspensions of graphite in water
570 K
Oil dag - suspension of graphite in oil lubricants.
R R (iii) Fullerene : Fullerenes are large cage like spheroidal molecules
| |
R2 Si(OH )2 ® - O - Si - O - Si - O - with general formula C2n (where n ³ 30). Two important
| | member are C60 and C70. C60 fullerene looks like a soccer
R R
ball (so called bucky ball).
Silicone
Silicones are chemically inert, water repellent, heat resistant, good
electrical insulators .These are used as lubircants, insulators etc.
CARBIDES
Compounds of carbon with less electronegative elements e.g. Be,
B, Si etc are called carbides. These are of three types.
(i) Ionic or salt like - The carbides of elements of group 1, 2, 13,
coinage metals, Zn,Cd. Some lanthanides give ionic or salt Amorphous forms of carbon are
like carbides. Prepared by heating oxide with carbon or (i) Coal
hydrocarbon at high temperature (2350K) (ii) Coke
2Be + C ® Be2 C (iii) Charcoal or wood charcoal
CaO + 3C ® CaC2 + CO (iv) Bone-black or animal charcoal
They are further classified as (v) Lamp-black
(a) Acetylides contain ( - C º C - )2 - . These liberate (vi) Carbon-black
acetylene on hydrolysis (vii) Gas carbon
CaC2 + 2H 2 O ® Ca ( OH ) 2 + CH º CH (viii)Petroleum coke
They have NaCl type crystal lattice. Varieties of coal -
(i) Peat 60 % carbon
(b) Methanides C 4- - These react with water to give
(ii) Lignite 70% C
methane.
(iii) Bituminous 78%C
Al 4 C 3 + 12 H 2 O ® 4Al(OH )3 + 3CH 4
(iv) Semi bituminous 83% C
(c) Allylides - These react with water to give allylene.
(v) Anthracite 90%C
Mg 2 C 3 + 4H 2 O ® 2Mg (OH )2 + CH 3C º CH
Anthracite is purest - amorphous form, burns without smokey
(ii) Interstitial carbides - These carbides are formed by flame.
transition elements especially Cr, Mn and Fe group metals.
distructive distillation
These are very hard. Coke- Coal ¾¾¾¾¾¾¾¾® Coke.
It contains C = 80 - 90%
322 Chemistry
Uses - Reducing agent in Iron and steel industry for making water (iv) Lime water -
gas and graphite.
Ca (OH )2 + CO 2 ® CaCO 3 ¯ + H 2 O ¾¾¾
2 ® Ca (HCO )
CO
Wood charcoal -It is obtained by strongly heating wood without 3 2
ppt clear solution
access to air. When heated with steam it becomes more activited.
Uses -To remove colouring matters and odoriferous gases. (v) Reduction - CO2 + C(red hot coke) ® 2CO
Bone-black or animal charcoal - It is obtained by destructive
(vi) Photosynthesis-
distillation of bones in iron retort. By products are bone oil or
pyridine. Chlorophyll
6CO2 + 6H 2 O ¾¾¾¾¾® C6 H12 O6 + 6O2
Uses - As adsorbant. On burning it gives bone ash which is
calcium phosphate and used in the manufacture of phosphorous 6xCO 2 + 5xH 2 O ® (C6 H10 O5 ) x + 6xO 2
and phosphonic acid. Uses - In household as fire extinguisher. Dry powder fire
Lamp black - It is obtained by burning vegetable oils in limited
extinguisher contains ( NaHCO3 + sand ) which is decomposed
supply of air.
Uses - In the manufacture of printing ink , black paint, varnish and by heat.
carbon paper. 2NaHCO3 ® Na 2CO3 + CO 2 + H 2O
Carbon black - It is obtained by burning natural gas in limited Foamite extinguisher contains baking soda and aluminium
supply of air. sulphate and is used for oil fires.
Uses - Added to rubber mixture for making automobile tyres. Structure O = C = O . Linear, dipole moment is zero.
Gas carbon and petrolium coke - When coal is subjected to Carbon monooxide CO :
destructive distillation carbon deposited on walls is scraped and
Preparation -
called gas carbon. Similarly petroleum coke is deposited while
(i) Lab method -
distilling crude petroleum.
Uses - Both are good conductors of electricity when pressed into Conc.H SO
COOH ¾¾ ¾¾ ¾¾® CO2 + CO + H 2O
2 4
sticks they make good electrodes, known as gas electrodes. | D
COOH
Sugar charcoal - It is obtained by heating sugar in absence of air. oxalic acid
It is purest form of carbon. (ii) Manufacture -
CARBONDIOXIDE CO2 Red hot coke
(a) Air ¾¾¾¾¾¾ ® N 2 + CO (producer gas)
Preparation -
Red hot coke
(i) Lab method (b) Steam ¾¾¾¾¾® CO + H 2
CaCO3 + 2 dil. HCl ® CaCl2 + CO2 + H 2 O (water gas synthesis gas or blue gas )
(ii) Manufacture (iii) Other methods - Heating K 4 [Fe ( CN )6 ] with
(a) CaCO 3 ® CaO + CO 2
Conc.H 2SO4
(b) Fermentation C6 H12 O6 ® 2C 2 H 5OH + CO 2
(c) From fuel gases K 4 [ Fe(CN )6 ] + 6H 2SO 4 + 6H 2 O ®
K 2CO3 + CO 2 + H 2O ® 2KHCO3 2K 2SO 4 + FeSO 4 + 3( NH 4 ) 2 SO 4 + 6CO
Physical Properties -Colourless, 1.5 times heavier than air, can be
poured downwards like H 2 O .Animals die in its presence due to Properties - Neutral, colourless, poisonous, burns with blue
lack of O 2 , it is also known as black damp. flame. Sparingly soluble in water. With haemoglobin it gives
Chemical properties : “carboxy haemoglobin” which destroys its capacity to
(i) Stability - Fairly stable, decomposed at 1775K. supply oxygen to the body.
2CO2 ® 2CO + O 2 (a) Burning -Non supporter of combustion. Burns in air
(ii) Incombustible and non supporter of combustion but active with blue flame.
metals e.g. Mg, Na, K continue burning in a jar of the gas. 2CO + O 2 ® 2CO 2
Reduces ammonical AgNO3
CO 2 + 2Mg ® 2MgO + C
2[Ag(NH 3 ) 2 ]OH + CO ® 2Ag + CO 2 + H 2O + 4NH 3
CO 2 + 4Na ® 2Na 2O + C
Reduces Fehling-solution
Na 2 O + CO 2 ® Na 2 CO 3
2CuO + CO ® Cu 2O + CO 2
(iii) Acidic nature - (b) Reducing nature - Metal oxides are reduced to metals.
CO2 + H 2O H 2CO3 117 K
Fe 2 O3 + 3CO ¾¾¾® 2Fe + 3CO 2
2 NaOH + CO2 ® Na 2CO3 + H 2O
high
Na 2CO3 + H 2O + CO2 ® 2 NaHCO3
CO + H 2 O ¾¾¾ ®H 2 + CO 2
T,P
The p -Block Elements –Carbon Family 323
(c) Unsaturated nature -It gives addition products. Composition :H2 = 45-55% N2 = 2-12%
sunlight CH4 = 25-35%, CO2 = 0-3%
Cl 2 + CO ¾¾ ¾¾® COCl 2 ( phosgene )
or camphor CO = 4-11%, O2 = 1-1.5%
Ethylene, acetylene, benzene etc = 2.5-5%
CO + 2H 2 ¾
¾® CH 3OH
Uses : Used as illuminant, as fuel, to provide inert atmosphere in
NaOH + CO ¾
¾® HCOONa metallurgical processes.
CO + S ¾¾® O = C = S (carbonyl sulphide ) 5. Natural gas : It is found along with petroleum below the
D
surface of earth.
CuCl + CO + 2H 2 O ® CuCl.CO.2H 2O Composition : CH4 = 60-80%
Higher hydrocarbons = 2-14%
(d) Formation of metal carbonyles - It acts as lewis base.
C2H6 = 5-9%, C3H8 = 3-18%
M + xCO ® M (CO ) x Uses : It is used as a fuel. Its partial combustion yields carbon
M = Ni, Cr , Mo, Fe or Co black (reinforcing agent for rubber)
structure : C - ::: O + :« : C - º O + : « :C O: 6. Oil gas :
Preparation :
Uses - In metallurgy of Ni-Monds process
( Ni + 4CO ® Ni(CO) 4 ¾¾ D heated in
® Ni + 4CO) Kerosene ¾¾ ¾ ¾® mixture of simple hydrocarbons
Manufacture of methanol, phosgene, synthetic petrol, as absence of air
reducing agent. Uses : It is used as fuel in laboratories in Bunsen Burners.
Test - 7. Wood gas :
(i) Burns with blue flame.
400º C
(ii) Reduces iodine pentaoxide to I 2. Preparation : wood ¾¾ ¾
¾® CH 4 , C 2 H 6 , H 2 , CO, N 2
absence of air
I 2O5 + 5CO ® I 2 + 5CO 2
Uses : It is used as fuel
(CHCl3 or CCl4 layer terms violet) 8. Liquified Petroleum gas (LPG)
Composition : n-Butane + Iso-butane
Carbogen - (mixture of O 2 + 5 - 10%CO 2 ) is used for artificial Uses : It is used as domestic fuel.
respiration for victims of CO poisoning. 9. Gobar gas :
FUEL GASES Preparation :
1. Water gas (H2 + CO) fermentation
Cow dung ¾¾ ¾ ¾ ¾® CH 4 (main ), CO and H 2
Preparation : C + H 2 O ® CO + H 2 - 28kcal absence of air
Re d hot Steam Water
Coke gas
Uses : As domestic fuel.
British Thermal Unit (B.T.U.) : It represents the amount of
Uses : Burns with blue flame, calorific value 2700 kcal/m3. Industrial heat required to raise the temperature of one pound of water
source of hydrogen (Bosch process). Manufacture of methyl through 1ºF. One B.T.U. is equal to 252 calories.
alcohol (Patart Processs) - Synthetic petrol (Fisher-Tropsch
process). For making carburetted water gas. SILICON AND ITS COMPOUNDS
2. Semi water gas : (mixture of water gas and producer gas) Silicon does not occur free in nature . In abundance it is next to
Red hot coke O2 .
Preparation : Steam + air ¾¾¾¾¾¾ ® semi water gas
OCCURANCE
Composition :CO = 27.0%, H 2= 10.9%
CH4 = 1.28%, CO2 = 4.50% As oxide SiO 2 (Silica )in sand, quartz, flint. As silicates
N2 = 56.3% of Al, Mg , K, Fe etc. Aluminium silcate is most
Properties and uses : Its calorific value 160 - 180 B.T.U. per cubic widely distributed as Felspar ( KAlSi2O3 )and Mica
foot. As fuel in steel industry and for production of power in
internal combustion engine. [ KH 2 Al(SiO 4 )3 ] , Feldspar ( K 2 Al2O3 6SiO2 ), Kaolinite

3. Producer gas : Burning coke + limited air ® producer gas ( Al 2 O 3 .2SiO 2.2H 2 O ) .
Composition : CO = 31.7%, N2 = 65.7%, CO2 = 2.5% PREPARATION OF AMORPHOUS FORM
Properties and uses : Poisonous, combustible but non supporter It is very common and may be obtained by heating powdered
of combustion, has low calorific value. Mainly employed as fuel. quartz or finely divided silica with Mg powder.
4. Coal gas :
SiO 2 + 2Mg ® Si + 2MgO
Preparation : By destructive distillation of coal
324 Chemistry
PREPARATION OF CRYSTALLINE FORM (x) Silica garden- Aqueous solution of sodium silicate containing
crystals of various coloured salts e.g. copper sulphate, cobalt
By reduction of SiCl 4 with molten Al
nitrate , manganese chloride, nickel chloride etc.
3SiCl 4 + 4Al ® 3Si + 4AlCl3 (xi) Hydro flurosilicic acid H2SiF6
By reduction of highly purified SiCl4 with hydrogen 3SiF4 + 4H 2 O ® H 4SiO 4 ¯ +2H 2SiF6 soluble
SiCl4 + 2H 2 ® Si + 4HCl (xii)Permutit -Artificially perpared sodium aluminium silicate
containing varying composition of sodium ,aluminium and
ZONE-REFINING
silica Na 2Al2Si 2O8 .xH 2O .Used for softening hard water..
Silicon is purified by Zone-refining process because the impurities
present in it are more soluble in the liquid phase than in the solid (xiii)Zeolites -They have honey comb like structure and have the
Mx
phase.
general formula
n
( AlO2 ) x .(SiO2 ) y .zH2O
PROPERTIES
Crystalline form possesses metallic lustre. It is very hard and M = Na + , K + or Ca ++
scratches glass. Crystalline silicon is isomorphous with diamond. n = Charge on metal
CHEMICAL REACTIONS OF AMORPHOUS SILICON : They act as ion exchanger and molecular sieves. They can
be artificially perpared by heating China clay, Silica and
Burns in air Si + O 2 ® SiO 2 Na2CO3 .
Si + 2F2 ® SiF4 (xiii)Ultramarines - Zeolite type silicates, containing ions like
Si + 2Cl2 ® SiCl4 Cl - , SO 24 - , S 2 - and not water, are known as ultramarines
Si + 2NaOH + H 2 O ® Na 2 SiO 3 + 2H 2 e.g. Na 2 (Al6Si6O24 )S2 Many of them are coloured and
used as pigments and calicoprinting.
Si + 2H 2 O (steam) ® SiO 2 + 2H 2 ­
(xiv)Carborundum - silicon carbide
USES SiO 2 + 2C ® Si + 2CO
Silicon chips used for computing devices are doped with P,As, Al Si + C ® SiC
or Ga to enhance the semiconductor properties. It is nearly as hard as diamond .
COMPOUNDS OF SILICON GLASS
Amorphous, hard, brittle, transparent, transluscent super cooled
(i) SiO 2 (silica) - It exists in three crystalline forms Quartz,
solution of various silicates and borates of K, Ca and Pb . It has
Trydimite and crystobalite. Further each form has a and no definite formula but roughly can be represented as
b form. At low temperature the a form is stable and at high M '2 O.M ''O.xSiO 2 where M ' = Na, K and M " = Ba, Zn, Pb
temperature the b form is stable . Raw material used in the manufacture of glass.
(ii) Sand - It is crushed form of quartz produced in nature by (i) SiO2
weathering of rocks . (ii) Na 2CO3, K 2CO3 or NaNO3 or KNO3
(iii) Flint - It is amorphous silica associated with quartz .
(iv) Kiesulguhr - Siliceous rock composed of the remains of sea (iii) Alkaline earth metals e.g. CaCO3 , BaCO3
organisms. Used as absorbent for nitroglycerine. (iv) oxides of heavy metals
(v) Quartz or rock crystal - It is purest form of silica It is optically (v) Cullets (pieces of glass)
active. (vi) Colouring matter
(vi) Silicic acid Colouring materials -
(a) Ortho silicic acid H 4SiO 4 or Si(OH) 4 Colour Substances Colour Substances

3SiF4 + 4H 2 O ® 2H 2SiF6 + H 4SiO 4 ¯


Green Cr2 O3 , FeSO4 Red Cu2O, Selenium oxide

(b) Meta silicic acid H 2SiO 3 or SiO(OH) 2 Yellow Ferric iron, Lemon yellow CdS
uranate of Na
Na 2SiO3 + 2HCl ® 2NaCl + H 2SiO3
Purple MnO2 Black NiO
(vii) Silicic acid sol -Colloidal solution of silicic acid.
(viii)Silica gel -Transparent gel of bluish white colour Ruby AuCl3 Milky white Na 3 AlF6
SiO2 .xH 2 O . Blue CuO, CoO Peacock Blue cupric Salts

(ix) Water glass or sodium silicate-Sodium silicate containing 1200°C


Manufacture - Mixture of raw materials ¾¾¾¾ ® till CO2
excess of SiO2.
escapes ® clear liquid. After some cooling it is used for
Na 2 SiO3 .SiO 2 to NaSiO3 .3SiO 2
casting articles
The p -Block Elements –Carbon Family 325

Type of glass - Batch Properties Uses


(i) Soft or soda glass Na 2O.CaO.SiO 2 Soft Window glass and bottles

(ii) Hard or potash glass Sand+lime stone + K 2CO3 Hard Hard glass apparatus

(iii) Flint glass Sand +Red lead+ K 2CO3 High refractive index Optical instrument, prisms

(iv) Pyrex glass Na 2 O.B2 O3 Al2 O3 SiO2 Low coefficient Glass ware
(v) Jena glass Zn and Barium Borosilicate Low coefficient of expansion Glass wares
(vi) Crookes glass K 2O.PbO.CeO 2 .SiO 2 Cuts off U.V. rays Eye lenses

(vii) Quartz glass or silica glass Pure SiO 2 Low coefficient of expansion Scientific wares
(viii) Safety glass or Between two glass layers Automobile wind
shatter proof a transparent plastic layer shields, goggles etc.
glass of vinyl acetate resin is placed

(ix) Reinforced glass Has network of wires Does not shatter


(x) Ground glass Soft glass with ground surface

Annealing - The process of slow cooling of glass is known as Chemical properties - With acids
annealing .The glass becomes soft.
Sn + 2HCl ® SnCl2 + H 2 ­
TIN ( Sn ) :
Sn + dilH 2SO4 ® SnSO 4 + H 2
[Kr]4d 10 4 s 2 4 p 2 Sn + 4H 2SO 4 (Conc.) ® Sn(SO 4 ) 2 + 2SO2 + 4H 2 O
Principal ores of tin - (i) Cassiterite or Tin stone, Sn O 2 (ii)Tinpyrites, 4Sn + 10HNO3 (dil.) ® 4Sn(NO3 ) 2 + NH 4 NO3 + 3H 2 O
SnS.Cu2S. FeS.
Extraction - Sn + 4HNO3 (conc.) ® H 2SnO3 + 4NO2 + H 2 O
(i) Concentration - By gravity process washing with water
Meta stannic acid
and then magnetic separation.
(ii) Roasting -To remove volatile impurities With alkali : Sn + 2NaOH + H 2O ® Na 2SnO3 + 2H 2 ­
S + O2 ® SO2 o
WithO2 at 1500 C: Sn + O2 ® SnO2
4As + 3O2 ® 2As2O3 With steam : Sn + 2H2 O ® SnO2 + 2H 2
(iii) Smelting - It is carried out in reverberatory furnace with COMPOUNDS OF TIN :
coal (powdered anthracite )and lime stone.
SnO2 + 2C ® Sn + 2CO (i) Stannic Oxide ( SnO2 ) - It occurs naturally as cassiterite.
(slag)
CaCO3 + SiO2 ® CaSiO3 + CO2 Preparation - Sn + O 2 D
¾¾® SnO 2
Sn so obtained contains iron and other metals and called D
black tin . H 2SnO 3 ¾¾® SnO 2 + H 2 O
(iv) Refining by (a) Liquation (b) Poling and (c) Electrolytic Properties - White solid, insoluble in water and amphoteric in
Properties - Soft silvery white metal, ductile and malleable. It has nature.
maximum number of isotopes and three allotropic forms.
SnO 2 + 2H 2SO 4 ® Sn ( SO 4 ) 2 + 2H 2 O
18°C 161° C
ˆˆˆˆ
Grey ‡ˆˆˆ † ˆˆˆˆˆ
ˆ White ‡ˆˆˆˆ †
ˆ Rhombic SnO 2 + 2NaOH ® Na 2SnO3 + H 2 O
Tin cry - It produces a peculiar cracking sound on bending which is
Uses - As polishing powder, in glass and pottery manufacture.
known as tin cry.
(ii) Stannous oxide (SnO) -
Tin plague - It is the conversion of white tin to grey tin at low
temperature which crumbles into powder. D
Preparation - SnC 2 O 4 ¾¾ ® SnO + CO + CO 2
Tinning - Since tin is not attacked by organic acids the utensils are
Sn(II) oxalate
protected by thin layer of tin .A pinch of NH 4 Cl is sprinkled over
Properties : It is black solid and amphoteric in nature.
hot and clean surface, when HCl liberated removes the oxide film.Tin
then rubbed over the clean surface with the help of rag dipped in SnO + 2H + ® Sn 2 + + H 2 O
NH 4 Cl powder. The utensil is immediately dipped in water to avoid
SnO + 2OH - ® SnO 22 - + H 2 O
oxide formation .
Tin plating - It involves the depositing of thin protective layer of tin (iii) Sulphides - SnS precipitated by H 2 S .It is dark brown solid,
over sheets of iron electrolytically. soluble in yellow ammonium sulphide forming (NH 4 ) 2 SnS3
326 Chemistry

(iv) Stannous chlorides ( SnCl2 .2H 2 O ) - Chemical properties -


(i) Pb + S ® PbS
Preparation - Sn + 2HCl(conc.) ® SnCl2 + H 2
D (ii) Pb + 2Cl2 ® PbCl4
Anhydrous Sn + 2HCl(dry gas) ¾¾ ® SnCl 2 + H 2
Properties - (iii) 2Pb + O 2 ® 2PbO
· White crystalline solid, soluble in water, alcohol and ether. (iv) 6PbO + O 2 ® 2Pb3O 4
· Hydrolysed in water (v) Pb + 2HCl ® PbCl2 + H 2
SnCl 2 + H 2 O ® Sn(OH)Cl + HCl (vi) PbCl2 + 2HCl ® H 2 PbCl4
Chloroplumbous acid
· Strong reducing agent (vii) Pb+2H 2SO 4 ( conc.) ® PbSO 4 +SO 2 +2H 2 O
2HgCl2 + SnCl2 ® Hg 2Cl2 + SnCl4 (viii) 3Pb+8HNO3 ( dil.) ® 3Pb(NO3 ) 2 ++2NO+4H 2 O
Hg 2Cl 2 + SnCl2 ® 2Hg + SnCl4 (ix) Pb+4HNO3 ( conc.) ® Pb(NO3 ) 2 +2NO2 +2H 2 O
2CuCl 2 + SnCl2 ® Cu 2Cl2 + SnCl4 (x) 2Pb+4CH3COOH ® 2(CH3COO)2 Pb+2H 2 O
2FeCl3 + SnCl2 ® 2FeCl2 + SnCl4 (xi) Pb + NaOH ® Na 2 PbO 2 + H 2
(v) Butter of tin - SnCl4. .5H 2 O used as mordant in dyeing. Plumbo Solvancy - Formation of lead hydroxide with H 2 O in
(vi) Purple of cassius - Colloidal particles of gold absorbed by presence of air is known as plumbo solvancy.
stannic acid Sn (OH) 4 is known as purple of cassius. It is
2Pb + 2H 2 O + O 2 ® 2Pb(OH) 2
used for colouring glass and pottery.
(vii) Alloys of tin - Some important alloys of tin are Hence lead is readily corroded hard water has no action on lead
(a) Solder Sn 50-70% Pb 30-50% which forms a protective layer of PbCO3 and PbSO 4
(b) White metal Sn 82% Sb 12% Cu 6% Hence hard water can be conveyed through lead pipes.
(c) Brittania metal Sn 90 % Sb 7% Cu 3%
(d) Soluminium Sn 55% Zn 33% Al 11% Cu 1%
Uses - In lead accumulators, pigments such as red lead, chrome
(e) Babbit metal Sn 90% Sb 7% Cu 2% yellow, chrome red, (C 2H 5)4Pb as antiknock compound,
(f) Pewter Sn 80% Pb 20% manufacturing of sulphuric acid.
(g) Dental alloy Alloy of Sn , Ag, and Hg
COMPOUNDS OF LEAD
LEAD ( Pb ) : [ Xe ] 5d 10 6 s 2 6 p 2 (i) Lead monooxide (PbO) -
Principal ores - (i) Galena PbS (ii) Anglesite PbSO4(iii) Cerrussite Preparation -
PbCO3.
Extraction from galena - Two important processes are ( i) 2Pb(NO3 )2 ® 2PbO+4NO 2 +O 2
(a) Air reduction process (ii) Pb CO3 ® PbO+ CO 2
(b) Carbon reduction process 300°C
(iii) 2Pb+O 2 ¾¾¾®
(a) Air reduction process
(i) Concentration - By froth floatation process 2PbO (Yellow powder, known as Massicot)
(ii) Roasting - Roasted in air at 500°C – 600°C. 900°C
2PbO+O 2 ¾¾¾®
3PbS + 5O 2 ® 2PbO + PbSO 4 + 2SO 2 2PbO (Reddish yellow, known as Litharge)
2PbO + PbS ® 3Pb + SO 2
PbSO 4 + PbS ® 2Pb + 2SO 2 Properties - Insoluble in water and amphoteric in nature.
(b) Carbon reduction process : Mixed sulphides (PbS & ZnS)
are roasted to obtain oxides which are fed into blast furnace PbO + 2HNO3 ® Pb(NO3 ) 2 + H 2 O
with coke and lime. PbO + 2NaOH ® Na 2 PbO 2 + H 2 O
470K 775K
PbO + C ® Pb + CO 6PbO + O 2 ¾¾¾® 2Pb3O4 ¾¾¾® 6PbO + O 2
ZnO + C ® Zn + CO Re d lead
CaCO 3 + SiO 2 ® CaSiO 3 + CO 2
Uses - Making glass, pottery, Massicot mixed with glycerine
Molten lead tapped off from the bottom. joins glass and stone . As drier in paints and varnishes.
Purification - Electrolytic method to remove Cu, Ag, Au, Sb etc.
(ii) Lead dioxide ( PbO 2 ) -
Properties - Bluish, grey lustrous metal which acquires dull
appearance when exposed to air due to formation of basic Preparation - By any of the methods given below
carbonate (Pb(OH)2.PbCO)3. Poor conductor of electricity.
The p -Block Elements –Carbon Family 327
(i) Pb3O4 +dil.HNO3 ® 2Pb(NO3 )2 +2H 2 O+PbO 2 Properties - White crystalline solid, turns black when exposed
(ii) Pb(OH) 2 +NaOCl ® PbO 2 +H 2 O+NaCl to H 2 S due to PbS formation .It is highly poisonous.
(iii) 2Pb(OH)2 +2Ca(OCl) 2 ® 2PbO 2 +2CaCl2 +2H 2 O+O 2 Action of heat
(iv) Pb(OH)2 +CaOCl 2 ® PbO2 +CaCl2 +H 2 O(lab method) 2PbCO3 . Pb(OH) 2 ® Pb3O 4 + H 2 O + CO + CO2
(v) 3PbO+KClO3 ® 3PbO 2 +KCl
Uses - As a white paint.
(vi) PbO+KNO3 ® PbO 2 +KNO 2
(v) Halides of lead [Pb(II) halide] :
Properties - It is brown solid. Powerful oxidising in nature.
2+ -
PbO 2 + SO 2 ® PbSO 4 Preparation - Pb + 2X ® PbX 2 (X = F, Cl, Br, I)
PbO 2 + 4HCl(conc.) ® PbCl 2 + 2H 2 O + Cl 2 (soluble lead salt )
Amphoteric in nature PbI2 yellow, others are white solids . PbCl 2 and PbI2 are
PbO 2 + 2 NaOH ® Na 2 PbO 3 + H 2 O soluble in hot water.
2PbO 2 + 2H 2SO 4 ® 2PbSO 4 + 2H 2 O + O 2 · PbCl2 forms complex with conc. HCl,
PbO 2 + 2CaO ® Ca 2 PbO 4
PbCl2 + 2HCl ® H 2 [PbCl4 ] (Soluble)
Uses - Storage batteries and match industry, as oxiding agent.
(iii) Trilead tetraoxide, Red lead, minium or Sindhur - PbCl 2 + Ca(OH) 2 (lime water) ® Pb(OH)Cl + CaO + HCl
Preparation - 6PbO + O 2 ® 2Pb3O 4 PbCl + Ca(OH) (lime water) ® Pb(OH)Cl + CaO + HCl
Properties - Red crystalline solid, insoluble in water. · Covalent character
Action of heat 2 Pb3O4 ® 6 PbO + O2 ­
PbI2 > PbBr2 > PbCl2 > PbF2
With acids
Pb (IV) halides :
Pb 3O 4 + 8HCl(conc.) ® 3PbCl 2 + 4H 2 O + Cl 2 Preparation -
2Pb 3O 4 + 6H 2SO 4 ® 6PbSO 4 + 6H 2 O + O 2 273K
2Pb 3O 4 + 4HNO 3 ® 2Pb( NO 3 ) 2 + PbO 2 + 2H 2 O PbO 2 + 4HCl(conc.) ¾¾¾® PbCl 4 + 2H 2 O
Properties - It is covalent liquid and unstable.
O O
PbCl4 ® PbCl2 + Cl2
Structure Pb Pb Pb
O O · PbF4 is ionic solid.
Uses - In glass, match and pottery and as red pigment. · Sugar of lead (CH3COO) 2 Pb -It has sweet taste.
(iv) Basic lead carbonate, white lead [2PbCO3. Pb(OH)2]:
Preparation - Dutch process - It is formed by exposing thin · Chrome yellow or Lemon chrome PbCrO 4
sheets of lead to vapours of acetic acid in presence of horse · Basic lead Chromate or chrome red
dung or tan bark .
PbCrO4 .Pb(OH) 2 - It is red pigment.
2Pb + O 2 + 2H 2 O ® 2Pb(OH ) 2
· Lead tetra ethyl
Pb(OH) 2 + 2CH 3COOH ®
Pb + 4C2 H5Cl + 4Na ® Pb(C2 H5 )4 + 4NaCl
(CH 3COO ) 2 Pb + 2H 2 O
- Antiknock agent.
Pb(OH) 2 + (CH 3COO) 2 Pb ® · Lithophone (BaSO 4 . + ZnS) is substitute of white lead.
Pb(CH3COO)2 .Pb(OH) 2 Fusible Alloys of lead :
( Basic lead acetate ) 1. Type metal Pb 82% Sb 15% Sn 3 %
2. Wood metal Bi 50% Pb 25% Sn 12.5% Cd 12.5%
3[Pb(CH3COO) 2 .Pb(OH) 2 ] + 4CO 2 + 2H 2 O ® 3. Lipowitz Bi 50% Pb 27% Sn 13% Cd 10%
2[2PbCO 3 .Pb(OH ) 2 ]+ 6CH 3COOH 4. Rose metal Bi 50% Pb 28% Sn 22%
(white lead ) 5. Newton’s metal Bi 50% Pb 31% Sn 19 %
328 Chemistry

Very Short/ Short Answer Questions (ii) (a) Why N(CH3)3 is pyramidal but N(SiH3)3 is planar?
1. Explain why silicon shows a higher covalency than carbon? (b) Why (SiH3)3N is a weaker base than (CH3)3N ?
2. Why is diamond a bad conductor of electricity but a good (iii) Account for the following:
conductor of heat? PbO2 is a stronger oxidizing agent than SnO2.
3. Which oxide of carbon is an anhydride of carbonic acid ? Or
4. Why carbon forms covalent compounds whereas lead forms
ionic compounds? PbO2 can act as an oxidizing agent.
5. What name is given to the compounds formed by more 16. Glass made up of SiO2 reacts with HF to form SiF62– anion.
electropositive elements with carbon? The analogous CF62– anion does not form. Why?
6. What are silicates?
7. Silanes gets hydrolysed by water whereas alkanes do not, Multiple Choice Questions
why? 17. Which of the following is most stable?
8 (CH3)3N is basic but (CF3)3N is not basic. Explain.
9. (a) CO2 is a gas while SiO2 is a solid. Explain.
(a) Sn 2+ (b) Ge2+
(b) Give one chemical reaction to show that: (c) Si 2+ (d) Pb 2+
(i) Tin (II) is a reducing agent whereas Pb (II) is not. 18. Cement, the important building material is a mixture of oxides
(ii) Tin (II) reduces mercuric salt to mercurous salt. of several elements. Besides calcium, iron and sulphur, oxides
10. (a) Carbon monoxide is readily absorbed by ammonical of elements of which of the group (s) are present in the
cuprous chloride solution but carbon dioxide is not. mixture?
Explain. (a) Group 2
(b) Silanes are few in number whereas alkanes are large in (b) Groups 2, 13 and 14
number. Explain. (c) Groups 2 and 13
(c) Diamond is covalent, yet it has high melting point. Why? (d) Groups 2 and 14
11. Give reason : 19. The most commonly used reducing agent is
(a) Silicon form compounds with coordination number of (a) AlCl3 (b) PbCl2
5 and 6.
(c) SnCl4 (d) SnCl2
(b) Si—F bond is stronger than C—F bond although Si is
20. Lead pipes are not suitable for drinking water because
larger in size than C.
(a) lead forms basic lead carbonate
12. Arrange the following in increasing order of the property
indicated: (b) lead reacts with water containing air to form Pb(OH)2
(a) CCl2, SiCl2, GeCl2, SnCl2 and PbCl2 (c) a layer of lead dioxide is deposited over lipes
(stability) (d) lead reacts with air to form litharge
(b) CO, SiO, SnO, GeO, PbO (basicity) 21. Catenation i.e., linking of similar atoms depends on size and
(c) SiF4, SiCl4, SiI4, SiBr4 (stability) electronic configuration of atoms. The tendency of
catenation in Group 14 elements follows the order :
13. Out of CCl4 and SiCl4 which one reacts with water?
(a) C > Si > Ge > Sn (b) C >> Si > Ge » Sn
14. [SiF6]2– is known whereas [SiCl6]2– is not. Give possible
reasons. (c) Si > C > Sn > Ge (d) Ge > Sn > Si > C
22. Graphite is a soft solid lubricant extremely difficult to melt.
Long Answer Questions The reason for this anomalous behaviour is that graphite
15. (i) C and Si are almost always tetravalent but Ge, Sn and (a) is an allotropic form of diamond
Pb show bivalency. Why? (b) has molecules of variable molecular masses like
Or polymers
Why the tendency to exhibit + 2 oxidation state (c) has carbon atoms arranged in large plates of rings of
increases with increasing atomic number in group 14. strongly bound carbon atoms with weak inter plate
Or
bonds
+ 2 oxidation of lead is more stable than + 4 oxidation
state. Give reasons. (d) is a non-crystalline substance
The p -Block Elements –Carbon Family 329
23. Which statement is false (a) MeSiCl3 (b) Me2SiCl2
(a) Water gas is a mixture of hydrogen and carbon dioxide. (c) Me3SiCl (d) Me4Si
(b) Producer gas is a mixture of CO and nitrogen 25. Dry ice is
(c) Water gas is a mixture of water vapour and hydrogen. (a) solid NH3 (b) solid SO2
(d) Natural gas consists of methane, ethane and gaseous (c) solid CO2 (d) solid N2
hydrocarbons. 26. Quartz is extensively used as piezoelectric material, it
24. Silicon has a strong tendency to form polymers like silicones. contains __________ .
The chain length of silicone polymer can be controlled by (a) Pb (b) Si
adding (c) Ti (d) Sn

1. Which of the following is most electronegative? 10. The element that does not form a monoxide is
(a) Pb (b) Si (a) lead (b) tin
(c) C (d) Sn (c) germanium (d) silicon
2. The most stable +2 oxidation state is exhibited by 11. Pyrosilicate ion is
(a) Fe (b) Sn
(c) Pb (d) Si (a) SiO 22 - (b) SiO 24 -
3. The electronic configuration of four different elements is
(c) Si 2 O67 - (d) Si 2 O76 -
given below. Identify the group 14 element among these
12. Freon -12 is used as a
(a) [He] 2s1 (b) [Ne] 3s 2
(a) refrigerant (b) insecticide
(c) [Ne] 3s 2 3 p 2 (d) [Ne] 3s 2 3 p 5 (c) fungicide (d) herbicide
4. The catenation tendency of C,Si and Ge is in the order Ge 13. Which one of the following allotropic forms of carbon is
-1
isomorphous with crystalline silicon?
<Si<C.The bond energies (in kJ mol ) of C-C,Si-Si and Ge- (a) Graphite (b) Coal
Ge bonds, respectively are (c) Coke (d) Diamond.
(a) 167,180,348 (b) 180,167,348
(c) 348,167,180 (d) 348,180,167 14. The structure and hybridization of Si ( CH 3 ) 4 is
5. Ge(II)compounds are powerful reducing agents
whereas Pb(IV)compounds are strong oxidants .It is because (a) Bent, sp (b) Trigonal, sp 2
(a) Pb is more electropositive than Ge
(c) Octahedral, d 2 sp 3 (d) Tetrahedral, sp 3
(b) ionization potential of lead is less than that of Ge
(c) ionic radii of Pb2+ and Pb4+ are larger than those of Ge2+ 15. Silicon is an important constituent of
and Ge4+ (a) rocks (b) minerals
(d) of more pronounced inert pair effect in lead than (c) alloys (d) vegetables.
in Ge 16. In laboratory, silicon can be prepared by the reaction of
6. Which among CH4, SiH4, GeH4, SnH4 is most volatile? (a) SiO2 with Mg
(a) CH4 (b) SiH4 (b) by heating C in electric furnace
(c) GeH4 (d) SnH4 (c) by heating potassium fluorosilicate with potassium
7. Which of the following halides is the most stable? (d) None of these
(a) CF4 (b) CI4 17. Silica is soluble in
(c) CBr4 (d) CCl4 (a) HCl (b) HNO3
8. Which of the following conceivable structures for CCl4 will
(c) H2SO4 (d) HF
have a zero dipole moment ?
18. Quartz is a crystalline variety of
(a) Square planar
(a) Si (b) SiO2
(b) Square pyramid (carbon at apex)
(c) Irregular tetrahedron (c) Na2SiO3 (d) SiC
(d) None of these 19. Carborundum is
9. PbF4, PbCl4 exist but PbBr4 and PbI4 do not exist because of (a) Al2(SO4)3 (b) Al2O3. 2H2O
(a) large size of Br– and I– (c) AlCl3 (d) SiC
(b) strong oxidising character of Pb4+ 20. Which one of the following is a correct set for SiO2?
(c) strong reducing character of Pb4+ (a) Linear acidic (b) Linear,basic
(d) low electronegativity of Br– and I–. (c) Tetrahedral, acidic (d) Angular, basic.
330 Chemistry
21. A pseudo solid is 35. White lead is
(a) diamomd (b) glass (a) Pb3O4
(c) rock salt (d) CaCO3
(b) PbO
22. Pyrex glass is a mixture of
(a) sodium borosilicate and aluminium borosilicate (c) 2PbCO3 .Pb ( OH ) 2
(b) sodium silicate and calcium silicate
(c) sodium silicate and lead silicate (d) Pb ( CH 3COO ) 2 .Pb ( OH ) 2
(d) sodium silicate and aluminium borosilicate 36. The oxide which cannot act as reducing agent is
23. The variety of glass used in making lenses and prisms is
(a) SO2 (b) NO2
(a) soda glass (b) borosilicate glass
(c) flint glass (d) crooke’s glass (c) CO2 (d) ClO2
24. Cassiterite is an ore of
37. A solid element (symbol Y) conducts electricity and forms
(a) Mn (b) Ni
two chlorides YCln (colourless volatile liquid) and YCln - 2
(c) C (d) Sn
25. The shape of gaseous SnCl2 is (a colourless solid). To which one of the following groups of
the periodic table does Y belong?
(a) tetrahedral (b) linear
(c) angular (d) T-shaped (a) 13 (b) 14
26. Butter of tin is (c) 15 (d) 16
(a) SnCl4 .6H 2 O (b) SnCl4 .4H 2 O 38. Which of the following bonds has the most polar
(c) SnCl4 .5H 2 O (d) SnCl4 .2H 2 O character?
27. An alloy of Pb and Sn in equal proportion is called (a) C-O (b) C-Br
(a) pewter (b) type metal (c) C-S (d) C-F
(c) solder (d) constantan 39. The ion that can be precipitated by HCl as well as H2S is
28. Red lead is
(a) Pb 2 + (b) Fe3+
(a) PbO (b) Pb3O4
(c) PbO 2 (d) Pb 4 O3 (c) Zn2+ (d) Cu 2+
29. An oxide of lead which is used in lead storage battery, in 40. Which of the following is most dense?
safety matches and is a powerful oxidising agent is
(a) Fe (b) Cu
(a) PbO (b) PbO 2
(c) B (d) Pb
(c) Pb3O4 (d) 2PbO.PbO 2
41. C and Si have
30. Lead sulphate is soluble in
(a) conc. HNO3 (a) Same physical properties
(b) conc. HCl (b) Different physical properties
(c) solution of ammonium acetate (c) Same physical but different chemical properties
(d) water (d) Different chemical and physical properties
31. The percentage of lead in lead pencil is 42. Mark the oxide which is amphoteric in character
(a) zero (b) 20 (a) CO2 (b) SiO 2
(c) 80 (d) 70
32. The important ore of lead is (c) SnO2 (d) CaO
(a) chalcopyrites (b) haematite 43. Which of the following sulphate is insoluble in water?
(c) galena (d) bauxite (a) CuSO4 (b) CdSO4
33. Which of the following lead oxides is present in ‘Sindhur’?
(a) PbO (b) PbO 2 (c) PbSO4 (d) Al 2 ( SO 4 )3
(c) Pb 2 O3 (d) Pb3O4
44. Which of the following molecule has highest bond energy?
34. Which of the following reactions occurs at the cathode during
the charging of a lead storage battery? (a) F-F (b) C-C
(c) N-N (d) O-O
(a) Pb 2 + + 2e - ® Pb
45. The metal used for making radiation shield is
(b) Pb 2 + + SO 24 - ® PbSO 4 (a) Al (b) Fe
(c) Zn (d) Pb
(c) Pb ® Pb 2 + + 2e -

(d) PbSO 4 + 2H 2 O ® PbO 2 + 4H + + SO 24- + 2e -


The p -Block Elements –Carbon Family 331

1. The straight chain polymer is formed by: [CBSE-PMT 2009] 8. In silicon dioxide [AIEEE 2005]
(a) hydrolysis of CH 3 SiCl3 followed by condensation (a) there are double bonds between silicon and oxygen
polymerisation atoms
(b) hydrolysis of (CH3)4Si by addition polymerisation (b) silicon atom is bonded to two oxygen atoms
(c) hydrolysis of (CH3)2SiCl2 followed by condensation (c) each silicon atom is surrounded by two oxygen atoms
polymerisation and each oxygen atom is bonded to two silicon atoms
(d) hydrolysis of (CH3)3 SiCl followed by condensation (d) each silicon atom is surrounded by four oxygen atoms
polymerisation and each oxygen atom is bonded to two silicon atoms.
4. Name the type of the structure of silicate in which one oxygen 9. A metal, M forms chlorides in its +2 and +4 oxidation states.
2tom of [SiO4]4– is shared ? [CBSE-PMT 2011] Which of the following statements about these chlorides is
correct? [AIEEE 2006]
(a) Linear chain silicate (b) Sheet silicate
(a) MCl2 is more ionic than MCl4
(c) Pyrosilicate (d) Three dimensional (b) MCl2 is more easily hydrolysed than MCl4
3. The basic structural unit of silicates is : [NEET 2013] (c) MCl2 is more volatile than MCl4
(a) SiO 44- (b) SiO32- (d) MCl2 is more soluble in anhydrous ethanol than MCl4
10. The stability of dihalides of Si, Ge, Sn and Pb increases
(c) SiO 24- (d) SiO steadily in the sequence [AIEEE 2007]
4. Graphite is a soft solid lubricant extremely difficult to melt. (a) PbX 2 << SnX 2 << GeX 2 << SiX 2
The reason for this anomalous behaviour is that graphite
(b) GeX2 << SiX2 << SnX2 << PbX2
[AIEEE 2003]
(a) is an allotropic form of diamond (c) SiX2 << GeX2 << PbX2 << SnX2
(b) has molecules of variable molecular masses like polymers (d) SiX2 << GeX2 << SnX2 << PbX2.
(c) has carbon atoms arranged in large plates of rings of 11. In context with the industrial preparation of hydrogen from
strongly bound carbon atoms with weak interplate bonds water gas (CO + H2), which of the following is the correct
(d) is a non-crystalline substance statement? [AIEEE 2008]
5. Glass is a [AIEEE 2003] (a) CO is oxidised to CO2 with steam in the presence of a
(a) super-cooled liquid (b) gel catalyst followed by absorption of of CO2 in alkali
(c) polymeric mixture (d) micro-crystalline solid (b) CO is removed by absorption in aqueous Cu2Cl2
6. For making good quality mirrors, plates of float glass are used. solution
These are obtained by floating molten glass over a liquid metal (c) H2 is removed through occlusion with pd
which does not solidify before glass. The metal used can be (d) CO and H2, are fractionally separated using differences
[AIEEE 2003] in their densities
(a) tin (b) sodium 12. Among the following substituted silanes the one which will
(c) magnesium (d) mercury give rise to cross linked silicone polymer on hydrolysis is
7. The soldiers of Napolean army while at Alps during freezing [AIEEE 2008]
winter suffered a serious problem as regards to the tin buttons (a) R4Si (b) R2SiCl2
of their uniforms. White metallic tin buttons got converted (c) RSiCl3 (d) R3SiCl
to grey powder. This transformation is related to
13. In view of the signs of D r G ° for the following reactions :
[AIEEE 2004]
(a) a change in the partial pressure of oxygen in the air PbO 2 + Pb ® 2PbO, D r G° < 0
(b) a change in the crystalline structure of tin SnO 2 + Sn ® 2SnO , D r G° > 0
(c) an interaction with nitrogen of the air at very low Which oxidation states are more characteristics for lead and
temperature tin ? [AIEEE 2011RS]
(d) an interaction with water vapour contained in the humid
(a) For lead + 2, for tin + 2 (b) For lead + 4, for tin + 4
air
(c) For lead + 2, for tin + 4 (d) For lead + 4, for tin + 2
332 Chemistry
14. Name of the structure of silicates in which three oxygen atoms 16. The species present in solution when CO2 is dissolved in
of [SiO4]4– are shared. [IIT-JEE 2005 S] water are [IIT-JEE 2006]
(a) Pyrosilicate (a) CO 2 , H 2CO3 ,HCO3- ,CO32-
(b) Sheet silicate
(b) H 2CO3 ,CO32 -
(c) Linear chain silicate
(d) Three dimensional silicate (c) CO32- , HCO3
15. When PbO2 reacts with conc. HNO3 the gas evolved is (d) CO2, H2CO3
[IIT-JEE 2005 S]
(a) NO2 (b) O2 (c) N2 (d) N2O

1. On addition of excess of sodium hydroxide solution to 10. Which gas is essential constituent of almost all fuel gases ?
stannous chloride solution, we obtain : (a) CO2 (b) N2
(a) Sn( OH)2 (b) SnO2 .H2O (c) CO (d) H2O
(c) Na2SnO2 (d) None of these 11. Which does not exist
2. Lead pipes are readily corroded by
(a) [SnCl 6 ]2- (b) [GeCl 6 ]2-
(a) H 2SO 4 (b) HCl
(c) CH 3COOH (d) pure water (c) [SiCl 6 ]2- (d) [CCl 6 ]2-
3. A salt which gives CO2 with hot H2SO4 and also decolourises 12. Newly shaped glass articles when cooled suddenly become
acidified KMnO4 on warming is brittle, therefore these are cooled slowly, this process in
(a) bicarbonate (b) carbonate known as
(c) oxalate (d) acetate (a) tempering (b) annealing
4. Which halide is least stable and has doubtful existence (c) quenching (d) galvanising
(a) CI4 (b) GeI4 13. Mark the correct statement
(c) SnI4 (d) PbI4 (a) Water gas is used in manufacture of methyl alcohol
5. R3SiCl on hydrolysis forms (b) Water gas has highest calorific value
(c) Water gas burns with luminous flame
(a) R3SiOH (b) R 3Si - O - SiR 3
(d) The production of water gas is exothermic process.
(c) R 2Si = O (d) None of these 14. H2SO4 is not used for preparation of CO2 from marble chips
6. Which melts in boiling water ? because
(a) Gun metal (b) Wood’s metal (a) it does not react
(c) Monel metal (d) Bell metal (b) huge amount of heat is evolved
7. Incomplete combustion of petrol or diesel oil in automobile (c) the reaction is vigorous
engines can be best detected by testing the fuel gases for (d) calcium sulphate is sparingly soluble and gets deposited
the presence of on marble chips and stops the reaction.
(a) CO + H 2 O (b) CO 15. CO2 is used for extinguishing fire because
(c) NO 2 (d) SO 2 (a) it has a relatively high critical temperature
8. Which of the following is/ are methanide (s) ? (b) in solid state, it is called dry ice
(a) Be2C (b) Al4C3 (c) it is neither combustible nor a supporter of combustion
(c) Mg2C3 (d) Both (a) and (b) (d) it is a colourless gas
9. Which of the following statements is false? 16. Lead pipes are not suitable for drinking water because
(a) Water gas is a mixture of hydrogen and carbon monoxide (a) lead forms basic lead carbonate
(b) Producer gas is a mixture of CO and nitrogen (b) lead reacts with water containing air to form
(c) Water gas is a mixture of water vapour and hydrogen Pb(OH)2
(d) Natural gas consists of methane, ethane and gaseous (c) a layer of lead dioxide is deposited over pipes
hydrocarbons. (d) lead reacts with air to form litharge
The p -Block Elements –Carbon Family 333
17. CO2 and N2 are non-supporters of combustion. However for (b) PbO film is formed which resists chemical attack by acid
putting out fires CO2 is preferred over N2 because CO2 (c) PbCl2 protective coating gets formed on Pb surface
(a) does not burn (d) PbO2 film is always present on Pb surface, which resist
(b) forms non-combustible products with burning chemical attack
substances 28. Which of the folllowing is not correct?
(c) is denser than nitrogen (a) Ge (OH)2 is amphoteric
(d) is a more reactive gas (b) GeCl2 is more stable than GeCl4
18. When sodium oxide is heated in a current of CO2 at 360°C we (c) GeO2 is weakly acidic
get (d) GeCl4 in HCl forms [ GeCl2]2– ion
(a) sodium formate (b) sodium oxalate 29. Producer gas, a fuel and also a source of nitrogen is obtained
(c) sodium acetate (d) sodium carbonate by
19. Tin plague is the (a) passing a mixture of steam and air over incandescent
(a) conversion of stannous to stannic coke.
(b) conversion of white tin to grey tin (b) spraying oil into hot retorts.
(c) emission of sound while bending a tin rod (c) restricted supply of air through a bed of incandescent
(d) atmospheric oxidation of tin coke.
20. A gas does not turn lime water milky, supports combustion (d) passing steam over incandescent coke.
of burning magnesium. It has no smell and is colourless. It 30. Which of the following shows bond in silicone :
extinguishes a glowing splint but under some circumstances (a) Si – Si – Si – Si (b) – Si – O – Si – O – Si
reacts with oxygen and hydrogen. It is not poisonous. The (c) Si – C – Si – C – Si (d) Si – C – Si – O – Si
gas is likely to be 31. Which type of forces bind together the carbon atoms in
(a) water vapours (b) nitrogen diamond ?
(c) CO2 (d) helium (a) Coulombic forces (b) van der Waal’s forces
21. The correct statement with respect to CO is (c) Dipole-Dipole forces (d) Covalent forces
32. Which is the best absorbing material for carbon dioxide?
(a) it combines with H2O to give carbonic acid
(a) Cold, solid calcium chloride
(b) it reacts with haemoglobin in RBC (b) Heated charcoal
(c) it is powerful oxidising agent (c) Heated copper oxide
(d) it is used to prepare aerated drinks (d) Cold, solid calcium hydroxide.
22. Tin cry refers to 33. Water gas cannot be prepared by a continuous process
(a) conversion of white to grey tin because
(b) tin plating (a) the reaction ceases when coke is too cool.
(c) conversion of white tetrahedral tin to white rhombohedral (b) it cannot be manufactured without producer gas.
tin (c) the furnace must be allowed to cool occasionally.
(d) emission of sound while bending a tin rod. (d) hot coke must be added from time to time.
23. Plumbo - solvency means dissolution of lead in 34. A kettle which becomes furred-up in use has inside it a deposit
(a) hot water (b) acids composed mainly of :
(c) ordinary water (d) alkalies (a) magnesium bicarbonate (b) magnesium sulphate
24. The reducing power of divalent species decreases in the (c) sodium sulphate (d) calcium carbonate
order 35. The substance used to impart green colour to glass is :
(a) Ge > Sn > Pb (b) Sn > Ge > Pb (a) Cu 2 O (b) SnO2
(c) Pb > Sn > Ge (d) None of these
(c) Cr2 O 3 (d) CdS
25. Which is formed when SiCl4 vapours are passed over
36. On doping Ge metal with a little of In, one gets :
hot Mg
(a) insulator (b) rectifier
(a) SiCl 2 + MgCl 2 (b) Si + MgCl 2 (c) n-type semiconductor (d) p-type semiconductor.
(c) Mg 2Si + Cl 2 (d) MgSiCl 6 37. Water gas is produced by :
(a) saturating hydrogen with moisture.
26. Mg2C3 possess which of the following characteristics? (b) passing steam through a red hot coke bed.
(a) Is called magnesium allylide (c) mixing oxygen and hydrogen in the ratio of 1 : 2
(b) It contain Mg2+ and C 34- ions (d) heating a mixture of CO 2 and CH 4 in petroleum
(c) It on hydrolysis gives propyne refineries.
(d) All of these 38. Which of the following is similar to graphite
27. Lead is not affected by dil. HCl in cold because (a) B (b) BN
(a) Pb is less electronegative than H (c) B 2 H 6 (d) B4C
334 Chemistry

EXERCISE 1 16. (a) SiO2 + 2Mg ® MgO + Si ( lab method ) .


1. Si because of the presence of vacant d–orbitals can show a 17. (d) SiO 2 + 6HF ® H 2SiF6 + 2H 2 O .
covalency upto six while C because of the absence of d–
orbitals cannot have a covalency of more than four. 18. (b) Purest form of SiO 2 is quartz.
3. Since carbonic acid decomposes to give CO2 and H2O, 19. (d) Carborundum is SiC.
therefore, CO2 is regarded as an anhydride of carbonic acid: 20. (c) SiO2 being oxide of non metal is acidic in nature and silicon
H2CO3 ¾¾ ® H2O + CO2. is bonded to O atoms tetrahedrally. It has giant structure
5. Ionic compounds. 21. (b) Glass is pseudo solid.
22. (a) Pyrex glass is a mixture of sodium borosilicate and
6. Silicates are minerals which consist of SiO 44 - units arranged aluminium borosilicate.
in different ways. 23. (c) Flint glass is used for optical (for making lenses) and
7. Silane gets hydrolysed because silicon has vacant d-orbital prism.
whereas alkane cannot due to non-availability of vacant d- 24. (d) Cassiterite is an ore of tin.
orbital in carbon. Cl
17. (d) 18. (b) 19. (d) 20. (b) 21. (b) 25. (c) Shape of SnCl2 is angular Sn , Hybridisation is
Cl
22. (c) 23. (a) 24. (c) 25. (c) 26. (b)
sp 3 .
EXERCISE 2 26. (c) Butter of tin is SnCl4 .5H 2 O .
27. (c) Solder (See alloys).
1. (c) Electronegativity decreases down the group.
2. (c) Inert pair effect increases down the group. Hence for 28. (b) Red lead is Pb3O4 .
Pb2+ ,O.S. is most stable. 29. (b) PbO 2 .
3. (c) Valence shell electronic configuration of group 14 30. (c) PbSO 4 + 2CH3COONH 4 ® ( CH 3COO ) 2 Pb + ( NH 4 ) 2 SO 4
elements is ns p 2 2.
31. (a) Lead pencils do not contain lead. Lead pencils contain
4. (d) The more the bond energy, the more is the catenation. graphite.
5. (d) Ge(II) tends to acquire Ge (IV) state by loss of electrons. 32. (c) Galena PbS.
Hence it is reducing in nature. Pb (IV) tends to acquire Pb 33. (d) Pb3O4 is also known as Sindhur..
(II) O.S. by gain of electrons. Hence it is oxidising in
nature. This is due to inert pair effect. 34. (d) (See chapter on electrochemistry)
6. (a) The lower the mol. wt. The more is the volatile character. 35. (c) Basic lead carbonate is white lead 2PbCO3 , Pb ( OH ) 2 .
7. (a) Since bond energy of C-F >C-Cl > C-Br > C-I
36. (c) CO2 , C is in +4 O.S. which is maximum.
Hence CF4 is most stable.
8. (d) CCl4 is tetrahedral in nature. 37. (b) SnCl4 is colourless volatile liquid and SnCl2 is
9. (b) F and Cl are more oxidising in nature and can achieve Pb colourless solid Sn conducts electricity and it belongs to
14 group.
in (IV) O.S. but Br2 and I 2 can not achieve Pb in (IV) 38. (d) C-F is most polar due to highest electronegativity
O.S. secondly Pb 4+ is strong in oxidising nature and in difference.
its presence, Br - and I - can not exist. 39. (a) Pb2+ can be precipitated as PbCl2 in cold water and as
10. (d) Silicon does not form mono oxide. PbS in II group by H2S in acid medium.
40. (d) Pb is most dense by virtue of its property.
11. (d) Si 2 O76 - ion represent pyrosilicate (see text). 41. (d) C and Si have different physical and chemical properties.
12. (a) Freon -12 is used as refrigerant. 42. (c) CO 2 , SiO 2 are acidic, CaO is basic and SnO 2 is
13. (d) Diamond and crystalline silicon are isomorphous.
amphoteric.
1
14. (d) Hybridisation = ( 4 + 4 + 0 - 0 ) = 4, sp3 tetrahedral. 43. (c) PbSO4 is insoluble in water.
2 44. (b) F-F C-C N-N O-O
15. (a) SiO 2 , hence silicon, is an important constituent of Bond energy 37.0 81.6 39.0 34.9
rocks. 45. (d) Since lead can stop harmful radiation. Hence it is used
for making radiation shield.
The p -Block Elements –Carbon Family 335

EXERCISE 3 9. (a) Metal atom in the lower oxidation state forms the ionic
bond and in the higher oxidation state the covalent bond
1. (c) Hydrolysis of substituted chlorosilanes yield because higher oxidation state means small size and great
corresponding silanols which undergo polymerisation. polarizing power and hence greater the covalent character.
Hence MCl2 is more ionic than MCl4.
CH3 Cl H OH – 2HCl CH3 OH
Si + Si 10. (d) Reluctance of valence shell electrons to participate in
CH3 Cl H OH CH3 OH bonding is called inert pair effect. The stability of lower
Dialkyl silandiol oxidation state (+2 for group 14 element) increases on
going down the group. So the correct order is
Polymerisation of dialkyl silandiol yields linear
SiX2 < GeX2 < SnX2 < PbX2
thermoplastic polymer.
11. (a) On the industrial scale hydrogen is prepared from water
CH3 CH3 gas according to following reaction sequence
catalyst 2NaOH
HO — Si — OH + H O —Si — OH CO + H 2 + H 2 O ¾¾¾¾
® CO 2 + 2H 2 ¾¾¾¾ ® Na 2 CO3 + H 2 O
1424 3 (alkali)
water gas
CH3 CH3
From the above it is clear that CO is first oxidised to CO2
which is then absorbed in NaOH. So the correct answer
CH3 CH3 in option (a)
12. (c) The cross linked polymers will be formed by RSiCl3
HO — Si — O — Si — OH
OH
CH3 CH3 |
3nH O
nRSiCl3 ¾¾¾¾
2 ® nR - Si - OH ¾¾
®
-3nHCl |
O– O– OH
| |
2. (c) O O
Si Si | |

O O O– R - Si - O - Si
| |
O
– O– O O
| |

Pyrosilicate [Si2O7]6– R - Si - O - Si - R
| |
3. (a) SiO44– is basic structural unit of silicates. O O
| |
4. (d) Statement (c) is correct (Cross linked polymer)
5. (a) Glass is a transparent or translucent super cooled liquid. The correct choice is (c)
6. (d) It is because mercury exists as liquid at room °
13. (c) Negative D r G value indicates that + 2 oxidation state
temperature.
is more stable for Pb. Also it is supported by inert pair
7. ˆˆ† white tin. Grey tin is brittle and crumbles
(b) Grey tin ‡ˆˆ effect that + 2 oxidation state is more stable for Pb and
as DrG° value is positive in second reaction it indicates
down to powder in very cold climate.
that + 4 oxidation state is more stable for Sn.
8. (d) In SiO2 (quartz), each of O-atom is shared between two
i.e., Sn++ < Pb++, Sn4+ > Pb4+
SiO44– tetrahedra.
14. (b) Two dimensional sheet structures of the formula
— —

— —

— —

— —

— Si — O —Si — O — Si — O —Si — (SiO5 ) n2n - are formed when tetrahedral are shared.

O O O O 15. (b) PbO2 is a powerful oxidizing agent and liberate O2 when


treated with acids.

— Si — O —Si — O — Si — O —Si — 2PbO 2 + 4HNO3 ¾¾


® 2Pb(NO3 ) 2 + 2H 2 O + O 2 ­

O O O O 16. (a)
— —

— —

— —

— —

— Si — O —Si — O — Si — O —Si — + - ˆˆˆ + 2-


CO2 + H 2 O ‡ˆˆ
ˆˆˆ
† ˆ H + HCO3 ‡ˆˆ
ˆˆˆ
†
ˆ H 2 CO3 ‡ˆˆ ˆ 2H + CO3
†

Structure of SiO2
336 Chemistry

EXERCISE 4 18. (d) CO 2 + Na 2 O ® Na 2 CO 3


19. (b) It is the conversion of white tin to grey tin at low
1. (d) SnCl 2 + 2 NaOH ® Sn (OH) 2 + 2 NaCl
temperature which crumbles into powder.
Sn (OH ) 2 + 2 NaOH ® Na 2SnO 2 + 2H 2 O 20. (b) All mentioned properties are for nitrogen.
2. (c) Lead pipes are readily corroded by water containing 21. (b) CO react with haemoglobin, forms carboxy haemoglobin
organic acids. and stopes the supply of O2
3. (c) Oxalate gives CO and CO2 with hot H2SO4 and 22. (d) Tin cry is the emission of sound on bending the tin.
decolourises KMnO4
23. (c) Plumbo- solvency is formation of Pb(OH ) 2 by Pb with
2KMnO 4 + 3H 2SO 4 + 5H 2 C 2 O 4 ® ordinary water.
K 2SO 4 + 2MnSO4 + 8H 2 O + 10CO 2 2Pb + 2H 2 O + O 2 ® 2Pb(OH ) 2
24. (a) The stability of +2O.S. follows the order
4. (d) In nature Pb 4+ is strong oxidant and I - is strong
reductant Pb 2 + > Sn 2 + > Ge 2 +

Hence PbI 4 cannot exist. Hence reducing power Ge > Sn > Pb

25. (b) SiCl 4 + 2Mg ® Si + 2MgCl 2


5. (b) R 3SiCl + HOH ® R 3SiOH + HCl
26. (d) Mg 2 C3 + 4H 2 O ® 2Mg(OH) 2 + CH3C º CH
R 3SiOH + HOSiR 3 ® R 3Si - O - SiR 3 + H 2 O
6. (b) Wood metal m.p.t. 70ºC . (See fusible alloys lead.) 27. (c) Pb with dil HCl forms protective coating of PbCl2
7. (b) The exhaust of auto gives CO due to incomplete 28. (b) Ge4 + is more stable than Ge2+ . Hence GeCl 4 is more
combustion of petrol or diesel.
stable than GeCl 2
8. (d) Be2C and Al4C3 give methane with water.
29. (c) Producer gas is mixture of CO + N 2 . It is prepared by
9. (a) Water gas is CO + H 2
incomplete combustion of coal in limited supply of air.
10. (c) CO is essential constituent of almost all fuel gases.
30. (b) Silicone has Si – O – Si – O – Si linkage.
11. (d) Carbon, due to absence of d-orbitals can not extend its
coordination number beyond four. 31. (d) In diamond each carbon is sp 3 hybridised and linked to
12. (b) Annealing makes the glass soft. other C-atoms by covalent bonds.
13. (a) Methyl alcohol is manufactured using water gas. Patart- 32. (d) CO 2 is acidic in nature and it require most basic
process.
substance for absorption.
ZnO + Cr O + CuO
(CO + H 2 ) + H 2 ¾¾¾¾¾¾¾®
2 3
CH 3OH 33. (a) CO 2 + H 2 CO + H 2 O (endothermic process)
300°C

14. (d) Sparingly soluble CaSO4 deposits on marble and stops 34. (d) The bicarbonates of Ca and Mg are decomposed to Ca
the reaction. and Mg carbonates by boiling.
15. (c) CO2 is incombustible and non supporter of combustion. 35. (c) Cr2 O 3 impart green colour to glass.
16. (b) 2Pb + 2H 2 O + O 2 ® 2Pb(OH ) 2 36. (d) Doping of Ge with In produces p-type semiconductor.

17. (c) CO2 being more dense covers the igniting material more 37. (b) H 2 O (steam) + C (coke) ¾
¾® CO + H 2
effectively than N2.
38. (b) Graphite and boron nitride have similar structure.
12A
Purification,
Qualitative and Quantitative
Analysis of Organic Compounds
The organic compounds obtained from natural sources or conditions of temperature and pressure without
synthesised in the lab are often contaminated with impurities. decomposition and containing non volatile impurities are
They are purified by specialised methods purified by simple distillation.
METHODS OF PURIFICATION OF SOLIDS (ii) Fractional distillation: It is employed for separating two or
(i) Cystallisation: A saturated solution of impure substance is more volatile liquids having boiling points close to each
prepared in hot solvent and is heated with vegetable or other eg. acetone (bpt 60ºC) and methanol (bpt 65ºC). The
animal charcoal which adsorbs the impurities . The solution vapours of the liquids pass through the fractionating column
is filtered while hot through a hot water funnel. The filtrate which provides greater space for their cooling. The vapours
on cooling deposits crystals of pure compound. Success of of high boiling fractions condense and fall back into
the process depends upon the selection of the solvent. The distillation flask. The process is repeated till fractions of
impurities must be least soluble while hot and most-soluble high volatility go up followed by of lower volatility. They
while cold. The quick cooling gives tiny but pure crystals are collected separately.
while slow cooling gives bigger but impure crystals. When (iii) Distillation under reduced pressure or vacuum distillation:
crystal formation is initiated by adding crystals of substance Some liquids decompose when heated to their boiling points
it is called “Seeding”. eg. glycerol. Such liquids can be purified by distillation under
(ii) Fractional crystallisation : It is based on the differential reduced pressure much below their boiling points. The lower
solubilities of different compounds in a solvent. The the pressure lower is the boiling point and vice versa.
compound having less solubility crystallises out first on (iv) Steam distillation: The liquids insoluble in water, steam
cooling leaving behind others in solution. Sometimes mixture volatile in nature, having high molecular weight and having
of two solvents eg. Alcohol & water; Chloroform & high vapour pressure are purified by steam distillation
Petroleum ether, Alcohol or ether give better results. provided the impurities present are not steam volatile.
(iii) Sublimation: Some solids directly pass into vapour when Theory of steam distillation: The liquid boils when its
heated and vapour directly pass into solid when cooled vapour pressure becomes equal to the atmospheric
without the intermediate formation of liquid. This is known pressure. For a mixture of liquid (pl) and steam (ps) we have
as sublimation. The substances which sublime can be at boiling point
purified by this method provided the impurities present do P = pl + ps \ pl = (P – ps)
not sublime.
Hence liquid will boil at lower temperature than its normal
Camphor, napthalene, anthracene, benzoic acid, phthalic
boiling point. Examples Aniline, o-nitrophenol,
anhydride and anthraquinone are purified by sublimation.
bromobenzene, salicyaldehyde, essential oils etc.
METHODS OF PURIFICATION OF LIQUIDS
CHROMATOGRAPHIC METHODS
(i) Simple distillation: The vapourisation of a liquid by heating
It was discovered by Tswett (1906) and developed by Kuhn,
and subsequent condensation of vapours by cooling is
Winterstein and Karrer.
known as distillation. The liquids boiling under ordinary
338 Chemistry
(i) Principle: Selective adsorption or distribution of various EXTRACTION
components of mixture beween the two phases - fixed phase The process of removing a substance from its aqueous solution
and mobile phase. by shaking with a suitable solvent is known as extraction. (partition
(ii) Classification: law) The greater the number of operations the greater is the
(a) Adsorption chromatography: Fixed phase - Solid or recovery of substance. Soxhlet extractor is used for continuous
ion exchange resin extraction.
Mobile phase - Liquid or gas DRYING OF ORGANIC SUBSTANCES
Hence it is further known as liquid - solid Stable solids may be dried in a steam or air oven; by heating on a
chromatography, gas-solid chromatography or ion free flame. In a vacuum desicator using anhy. CaCl , conc H SO
2 2 4
exchange chromatography. or P2O5 etc.
(b) Partition Chromatography: Fixed phase - liquid Alcohols are dried over : Quick lime, anhy. K CO , MgSO or
2 3 4
supported on inert solid Na2SO4
Mobile phase - liquid or gas Aldehydes and Ketones : Anhy. MgSO4, Na2SO4 or CaCl2
Hence we have liquid - liquid partition chromatography Organic acids: Anhy. MgSO , Na SO or P O
4 2 4 2 5
and liquid gas partition chromatography
Organic bases: Solid KOH, Quick lime or Soda lime
Paper chromatography is the example of partition
chromatography CRITERIA OF PURITY
(iii) Column chromatography: (i) Mixed melting point: The purified substance is mixed with
Example of Adsorption chromatography pure substance (100%) and melting point is taken. If it is
sharp the substance is pure otherwise not. Impurities lower
Adsorbents used are: alumina, silica gel, cellulose powder,
the melting point.
sucrose, animal charcoal, magnesium oxide or keiselguhr
etc. (ii) Boiling Point : The boiling point of a liquid is defined as the
temperature at which its vapour pressure becomes equal to
Liquid Solvents used are : benzene, petrolium ether,
atomospheric pressure. Sharp and concordant boiling point
chloroform, alcohol etc.
is the criteria of purity of organic substance. Impurities
When the solvent is poured over the mixture present at the increase the boiling point.
top of a column packed with adsorbent the components are
separated into number of layers called Zones, bands or DETECTION OF CARBON AND HYDROGEN
chromatograms due to preferential adsorption. The process Strong
(C) organic compound + CuO ¾¾ ¾ ¾®
being known as development. D

Elution : The continuous pouring of solvent from the top Ca (OH ) 2


® CO 2 ¾¾ ¾ ¾ ¾® CaCO3 ¯ H 2 O
of column is known as elution or running of column. Colourless
milky
Solvent : It is known as eluent. Strong
(H ) organic compound + CuO ¾¾ ¾ ¾®
The most weakly adsorbed component is eluted first by D
least polar solvent while more strongly adsored component
Anhy. CuSO4
is eluted later by highly polar solvents. ® H 2 O ¾¾ ¾ ¾ ¾¾ ® CuSO 4 .5H 2 O
Colourless
blue
CHEMICAL METHODS
DETECTION OF NITROGEN
The substance to be purified is treated with a suitable chemical
reagent to form a stable derivative (impurities being unreacted). (i) Soda-lime test:
It is then separated by suitable method and decomposed to get organic compound + NaOH ® smell of NH3 ­
the pure compound. Examples: (Not reliable since –NH2, –NO2 and – N = N-groups do not
(i) Mixture of amines (1º, 2º and 3º) is separated by Hinsberg’s respond).
method and Hoffmann’s method. (ii) Lassaigne’s test: Common for N, S and X (halogens)
(ii) Acetic acid from pyroligneous acid is separated by forming Lassaigne’s filtrate or sodium extract is prepared by fusing
Ca salt. the organic compound with Na in ignition tube. Fused mass
(iii) Acids are separated by forming Na derivatives with NaHCO3. is dissolved in water, boiled and filtered. The filtrate is
(iv) Commercial benzene contains thiophene which is removed sodium extract which contains
by forming sulphonic acid derivative. Na + C + N ® NaCN sodium cyanide
(v) Carbonyl compounds are purified by forming bisulphite 2Na + S ® Na2S Sodium sulphide
derivative. Na + X ® NaX (Sodium halide)
(vi) Absolute alcohol is obtained from rectified spirit by quick Na + C + N + S ® NaCNS
lime process and azeotropic distillation. Sodium Sulphocyanide
Purification, Qualitative and Quantitative Analysis of Organic Compounds 339
Test for nitrogen : Sod. extract + freshly prepared FeSO4 (b) Layer test for bromine and iodine:
solution + FeCl3 solution + dil. H2SO4 ® green or blue Sod. extract + dil. HNO3 + CHCl 3 or CCl 4 or CS 2
colouration or sometimes blood red colour
Shake
2 NaCN + FeSO4 ® Fe(CN)2 + Na2SO4 + Cl 2 water ¾¾ ¾® Brown layer Br
4 NaCN + Fe(CN)2 ® Na4 [Fe(CN)6] ® Violet layer I
3 Na4[Fe(CN)6] + 4 FeCl3 ® Fe4[Fe(CN)6]3+12NaCl 2 NaBr + Cl 2 ® 2 NaCl + Br2
ferric ferocyanide Br2 + Solvent - Brown layer
(prussian blue) 2 NaI + Cl 2 ® 2 NaCl + I 2
FeCl3 + 3 NaCNS ® Fe(CNS)3 + 3NaCl I 2 + Solvent - Violet layer
Ferric sulphocyanide
(c) Beilstein's test:
(Blood red)
Organic compounds containing halogens when
(NH2.NH2 and diazo compounds do not give this test, diazo
heated over Cu wire loop, give blue or green colour
compounds are decomposed to give N2 and NH2.NH2 does
flame due to formation of volatile copper halides.
not contain C)
(Not reliable since thiourea, urea, pyridine and
Test for sulphur : organic acids also give this test)
Na 2S + Na 2 [Fe(CN) 5 NO] ® Na 4 [Fe(CN) 5 NOS] DETECTION OF PHOSPHOROUS
Sodium nitropruside Purple coloured complex
Organic compound + fusion mixture (Na2CO3 + KNO3) or Na2O2
H 2O
Na 2S + (CH 3COO) 2 Pb ® PbS + 2CH 3COONa D
¾¾® fused mass ¾¾¾® water extract + conc. HNO3 +
black ppt. ammonium molybdate ® yellow ppt.
Test for halogens: Na 3 PO 4 + 3HNO 3 ® H 3 PO 4 + 3NaNO 3
Sod. extract + Conc. HNO3 + AgNO3 Solution H 3 PO 4 + 12 (NH 4 ) 2 MoO 4 + 21 HNO 3 ® (NH 4 )3 PO 4 .12MoO 3 + 21
(i) If white precipitate soluble in NH4OH ® Cl– Amm. molybdate
present
® ( NH 4 ) 3 PO 4 .12MoO3 + 21NH 4 NO 3 + 12H 2 O
NaCl + AgNO 3 ® AgCl ¯ + NaNO 3 Amm. phosphomolybdate (yellow ppt.)
AgCl + 2NH 4 OH ® [Ag(NH 3 ) 2 ]Cl + 2H 2 O ESTIMATION OF ELEMENTS
Soluble Complex (i) Estimation of carbon and hydrogen (Liebig’s method)
CuO
(ii) If light yellow precipitate sparingly soluble in NH4OH C x H y + O 2 ¾¾¾® x CO 2 + y / 2 H 2O
® Br– present
By knowing the amount of CO2 and H2O from known weight
NaBr + AgNO 3 ® AgBr ¯ + NaNO 3 of organic compound, the percentage of carbon and
hydrogen can be calculated.
AgBr + 2 NH 4 OH ® [Ag ( NH 3 ) 2 ] Br + 2H 2 O • The water is absorbed in anhydrous CaCl2 or anhydrous
Sparingly Soluble magnesium perchlorate
Complex
• The carbon dioxide is absorbed in Saphnolite (a resion)
(iii) If yellow precipitate insoluble in NH4OH ® I– present or Ascarite (NaOH on asbestos) or KOH
NaI + AgNO 3 ® AgI ¯ + NaNO 3 • When organic compound contains nitrogen, oxides of
nitrogen (NO and N2O etc) are formed and absorbed by
AgI + NH 4 OH ® No Complex caustic potash. These are removed by the use of bright
(a) Function of Conc. HNO3 : It decomposes NaCN copper gauge
and Na2S to avoid their interference 4Cu + 2 NO 2 ® 4CuO + N 2
NaCN + HNO 3 ® NaNO 3 + HCN ­ Cu + N 2 O ® CuO + N 2
Na 2S + 2HNO 3 ® 2 NaNO 3 + H 2S ­ Nitrogen is not absorbed by KOH solution.
• When organic compound contains halogens they are
NaCN + AgNO3 ® AgCN ¯ + NaNO3 removed by using silver gauge by forming non volatile
white ppt. silver halide
• When sulphur is present, it is removed by forming lead
Na 2S + 2AgNO 3 ® Ag 2S + 2 NaNO 3 sulphate by using fused lead chromate and halogens
Black ppt. form lead halides.
340 Chemistry
• Percentage of carbon (iii) Estimation of halogens (Carius method)
(a) Organic compound + Fuming HNO3 + AgNO3
12 Wt. of CO 2 ´ 100
= ´ ® AgX.
44 Wt. of organic substance
It is estimated gravimetrically
• Percentage of hydrogen Percentage of halogen =
2 Wt. of H 2 O ´ 100
= ´ wt . of halogen atom ´ wt. of AgX ´ 100
18 Wt. of organic substance =
Mol. wt. of AgX ´ wt . of organic compound
(ii) Estimation of nitrogen: It is estimated by Dumas or (b) Schiff's and Piria method :
Kjeldahl’s method
organic compound (small amount)
(a) Duma’s Method: This method can be applied to all
nitrogenous compounds. Though tedious better than Pt Crucible
+ (CaO + Na 2 CO 3 ) ¾¾ ¾ ¾¾® cooled +
Kjeldahl's method. D Strong

CuO y z Pt Crucible Filter


C x H y N z ¾¾¾® x.CO(CaO 2 + +H +3) N
Na22OCO ¾¾¾¾¾
2 ® cooled + dil HNO 3 ¾¾¾® Filtrate + AgNO3 ® AgX
CO2 2 2
Cu
+ traces of oxides of Nitrogen ¾¾ ¾®
AgX is estimated as in Carius method
4Cu + 2 NO 2 ® 4CuO + N 2 This method is not suitable for volatile halides for iodides
2Cu + 2 NO ® 2CuO + N 2 only Na2CO3 is used as lime froms calcium iodate which
is insoluble in water.
Cu + N 2 O ® CuO + N 2
(c) Stepanov's method:
The mixture of gases is passed in KOH all gases except
Reflux
N are absorbed. Nitrogen is collected over KOH and R - X + C2 H5OH + Na ¾¾¾® RH + Nax + C2 H5ONa
2
its volume at NTP is measured. Re flux
Rx + C2 H 2 OH + Na ¾¾¾¾
® RH + NaX + C 2 H 5ONa
Percentage of Nitrogen
Excess of alcohol distilled off and solution treated with
28 ´ volume of N 2 at NTP ´ 100 6N HNO3 and filtered. AgX is precipitated by adding
=
22400 ´ wt. of organic compound AgNO3 which is filtered, washed and weighted.

wt of nitrogen ´100 (iv) Estimation of sulphur: sulphur can be estimated by


= (a) Messenger’s method, (b) Carius method or
wt. of organic substance
(c) Asboth’s method.
(b) Kjeldahl’s method: Organic compound + oxidising agent (fuming HNO3 in
Organic compound +Conc H2SO4 + (small amount of Carius method) ® H 2 SO 4 ¾BaCl 2 ® BaSO . It is
¾¾¾ 4
K 2SO 4 and CuSO 4 or Hg) ® ( NH 4 ) 2 SO 4 estimated gravimetrically (Alk.KMnO4 in Messenger's
2 NaOH method)
¾¾ ¾¾® Na 2SO 4 + 2 NH 3 + 2 H 2 O
(Na2O2 + Na2CO3 in Asboth's method)
Ammonia is passed into H2SO4 or HCl of known volume
and normality. The volume of acid neutralised by NH3 is 32 ´ wt . of BaSO 4 ´ 100
Percentage of Sulphur =
calculated by neutralising the acid left by NaOH solution. 233 ´ wt. of organic compound

1 .4 ´ N ´ V (v) Estimation of phosphorous: By Carius method


Percentage of Nitrogen =
wt. of organic compound Organic Compound + Fuming nitric acid ® H 3PO 4

N = normality of acid, V = volume of acid in ml. Magnesia


¾¾ ¾ ¾
¾® Mg NH4PO4
neutralised by ammonia mixture
( MgSO 4 + NH 4OH + NH 4Cl)
In practice K2SO4 is added to raise the bpt of H2SO4
and CuSO4 or Hg is added to catalyse the reaction) ignition
¾¾¾¾® Mg2 P2O7 (Magnesium phyrophosphate)
Kjeldahl's method is not reliable. The results obtained
are generally low. It cannot be applied to compounds in Percentage of phosphorous
which nitrogen is directly linked to oxygen such as nitro, 62 ´ wt. of Mg 2 P2 O7 ´ 100
nitroso, azo and nitrogen present as in pyridine. It is =
222 ´ wt. of organic compound
used for estimating nitrogen in food , fertilizers and
agricultural products.
Purification, Qualitative and Quantitative Analysis of Organic Compounds 341
DETERMINATION OF MOLECULAR WEIGHT (B) Chemical methods:
(A) Physical methods (i) Silver Salt method for Acids
(i) Vapour Density method: by (a) Victor's Meyer, (b) (ii) Platinichloride method for Bases
Dumas and (c) Hofmann's (iii) Volumetric method for Acids and Bases
(ii) Freezing point (i) Silver salt method for acids: Mol. wt. of an acid is
(iii) Boiling method. given by the following expression
(i) Vapour Density method: by (a) Victor's Meyer, (b) Mol. wt. of acid = Equivalent wt × basicity
Dumas and (c) Hofmann's
æ wt. of silver salt ´108 ö
(a) Victor Meyer’s method: for volatile substances: A Mol. wt. of acid = n ç - 107 ÷
è wt. of silver ø
known weight of the volatile substance is vaporised
by heating in Victor Meyer’s tube and the equivalent n = basicity of the acid
volume of the air displaced is calculated at NTP
(ii) Platinichloride method for bases: Mol wt. of a base is
after proper correction of aqueous tension. Vapour
given by the following expression
density is given by following relation.
Mol. wt. of base = Eq. wt. of base × Acidity
Weight of certain volume of vapour
V.D. =
Weight of same volume of hydrogen Mol. wt. of base = n æç wt. of Pt. salt ´195 - 410 ö÷
2è wt. of Pt. ø
At N.T.P. 22.4 Litre of Hydrogen = 2 g.
n = Acidity of the base
M. Wt = 2 × VD
(iii) Molecular weight of an Acid and a Base by
(b) Duma's method : Weight of known volume of the
vapour of the substance in a glass bulb at an Volumetric method:
elevated temperature is noted. Equivalent weight of an acid is that weight of acid which
(c) Hofmann's method: (i) A known weight of the is neutralised by 1 geq. of a base Equivalent weight of a
substance is vaporised above a mercury column in base is that weight of base which is neutralised by 1
a barometric tube and volume of the vapours formed geq of an acid
is noted. N´V
In Dumas and Hofmann method the calculations geq of acid or base =
1000
are same as Victor Meyer’s method.
where N = Normality of acid of base
(ii) Freezing point method or cryoscopic method:
V = Volume of acid or base in ml.
K f ´ w ´1000 EMPIRICAL AND MOLECULAR FORMULA
M=
DTf ´ W (i) Empirical formula: It represents the relative number of
M = molecular weight of the substance atoms of each element present in one molecule of the
compound.
Kf = molal depression constant
(ii) Molecular formula: It represents the actual number of atoms
w = Weight of the substance
of each element present in one molecule of the compound.
DTf = depression in freezing point
CALCULATION OF EMPIRICAL FORMULA
W = weight of the Solvent
(i) Divide the percentage of each element by the atomic weight
(iii) Boiling point method or Ebulioscopic method:
of the element.
K b ´ w ´ 1000 (ii) Divide all the numbers by lowest number obtained in the
M=
DTb ´ W first step.
(iii) Convert them in whole numbers if obtained in fractions by
M = molecular weight of the substance
multiplying them by common factor.
Kb = molal elevation constant
CALCULATION OF MOLECULAR FORMULA
w = weight of the substance
W = weight of solvent Mol. wt.
Molecular formula = (Empirical formula)
DTb = Elevation in boiling point Empirical wt.
342 Chemistry
EUDIOMETRY
Determination of molecular formula of gaseous hydrocarbon by exploding with excess of oxygen in a eudiometer is known as
Eudiometry.
Hydrocarbon (known volume) + O2 (Exces known volume)

(i ) Exploded (ii ) Coo l Contraction in volume (iii) KOH contraction in (iv) Remaining
¾¾ ¾ ¾ ¾® CO 2 ¾¾ ¾¾® ¾¾ ¾ ¾ ¾ ¾ ¾ ¾ ¾® ¾¾ ¾ ¾ ¾ ¾ ¾ ¾¾® ® ¾¾¾¾¾¾¾¾¾¾®
gives volume of H 2O volume gives volume of CO 2 volume is of uncreated O2
H 2O
O 2 (unreacted)
It is absorbed by alkaline pyrogallol. Hence volume of O2 reacted can be known and molecular formula can be calculated with the
help of following equation

æ yö y
C x H y + ç x + ÷ O 2 ® XCO 2 + H 2 O
è 4 ø 2

DENTIFICATION OF SOME ORGANIC COMPOUNDS


(i) Aldehydes : Give silver mirror with Tollen’s reagent and red precipitate with Fehling soln. and pink colour with Schiffs
reagent.
Remember benzaldehyde do not respond to Fehling solution.
(ii) Ketones : Give orange ppt with 2, 4 dinitrophenyl hydrazine solution.
(iii) Acids : Give effervescence with NaHCO 3
(iv) Alcohols : Give wine red colour with cerric ammonium nitrate.
(v) Phenols : Give deep colour change with 1% FeCl 3 sol.
(vi) Ethers : with conc. HI give alcohol and alkyl iodide. With excess HI, RI is the product.
(vii) Methyl ketones : Give iodoform with I 2 and NaOH.
(viii) Esters : When added to very dil soln. of NaOH containing phenolphthalein, the pink colour is discharged.
(ix) Aliphatic 1° amines : Give alcohols with NaNO 2 and HCl.

(x) Aromatic 1° amines : Give diazo compound with NaNO 2 and HCl.

(Note : NH 2 must be linked to benzene nucleus)

(xi) 2° amines : Give highly nitrosamine with NaNO 2 and HCl.

(xii) 3° amines : Salt only salt with NaNO 2 and HCl

(xiii) Cyanides : On reduction with Na + C 2 H 5 OH give 1° amines. On hydrolysis give acids

(xiv) Isocyanide : On reduction with Na + C 2 H 5 OH give 2° amines. On acid hydrolysis give 1° amine and formic acid.

(xv) 1° amine : Give isocyanides with CHCl 3 + NaOH, which have unpleasant odour..
(xvi) Nitro compounds : Give 1° amine with Sn + HCl.
(xvii) Amides : Give NH 3 when heated with NaOH.
(xviii) Anilides : They are hydrolysed to 1° amines.
Purification, Qualitative and Quantitative Analysis of Organic Compounds 343

1. 0.280 gm. of an organic substance gave on combustion 0.560 4. 0.147 gm. of an organic compound containing chlorine was
gm. of carbon dioxide and 0.14 gm. of water. Calculate the heated with nitric acid and silver nitrate which gave 0.287
percentage of carbon, hydrogen and oxygen present in the gm. of silver chloride. Calculate the percentage of chlorine
compound. in the compound.
Sol. Percentage of carbon Sol. Percentage of chlorine

12 Wt. of CO 2 ´ 100 12 ´ 0.560 ´ 100 35.5 ´ Wt. of AgCl ´ 100 35.5 ´ 0.287 ´ 100
= ´ = = 54.54% = = = 48.29%
44 Wt. of substance 44 ´ 0.280 143.5 ´ Wt. of substance 143.5 ´ 0.147

Percentage of hydrogen 5. 0.2595 gm. of an organic compound when treated by Carius


method gave 0.35 gm. of BaSO4. Calculate the percentage of
2 Wt. of H 2 O ´ 100 2 ´ 0.14 ´100 sulphur in the compound.
= ´ = = 5.55 %
18 Wt. of substance 18 ´ 0.280 Sol. Percentage of Sulphur
Percentage of oxygen = 100 – (54.54 + 5.55) 32 Wt. of BaSO 4 ´ 100 32 ´ 0.35 ´ 100
= (100 – 60.09) = 39.91% = 233 ´ Wt. of substance = 233 ´ 0.2595 = 18.52%
2. 0.300 gm. of an organic substance gave 37.9 ml. of dry N2 at
17°C and 770 mm of Hg pressure. Calculate the percentage 6. 0.35 gm of an organic compound was Kjeldahlised and
of N2 in the compound. ammonia evolved was absorbed in 100 ml. of N/5 H2SO4.
Sol. Convert the volume of N2 at NTP The residual acid required 154 ml. of N/10 NaOH for
neutralisation. Calculate the percentage of nitrogen in the
P1V1 P2 V2 770 ´ 37.9 760 ´ V2 compound.
= ; = ; \ V2 = 36.14 ml
T1 T2 (273 + 17) 273
1 .4 ´ N ´ V
Sol. Percentage of nitrogen =
Percentage of N2 = Wt. of substance
28 Vol. of N 2 at NTP ´ 100 where N V = Meq of acid neutralised by ammonia
´
22400 Wt. of substance
1
Total Meq of acid taken = N.V. = ´ 100 = 20
28 ´ 36.14 ´ 100 5
= = 15.06%
22400 ´ 0.300 1
Meq of NaOH used to neutralise acid = ´ 154 = 15.4
3. 0.4422 gm. of an organic substance was treated according 10
to Kjeldahl's method. The ammonia evolved was absorbed Meq of acid neutralised by ammonia = 20.0 – 15.4 = 4.6
in 50 cc of 1N H2SO4. The residual acid required 65.5 cc of
N/2 alkaline solution. Find the percentage of nitrogen in the 1.4 ´ 4.6
Percentage of nitrogen = = 18.4%
compound. 0.35
Sol. Meq of the acid taken = 50 × 1 = 50 7. 0.6 gm. of the silver salt of a dibasic acid when ignited gave
M eq of the acid neutralised by alkaline solution = a residue of 0.370 gm. of metallic silver. Find its molecular
weight.
1
65.5 ´ = 32.75 Sol. Molecular weight of an acid
2
Meq of the acid neutralised by ammonia æ Wt. of silver salt ´ 108 ö
= Basicity ç –107÷
= 50 – 32.75 = 17.25 è Wt. of silver ø
Percentage of nitrogen
æ 0.60 ´108 ö
1.4 ´ N ´ V 1.4 ´ 17.25 = 2ç - 107 ÷ = 136.24
= = = 54.61% è 0.37 ø
Wt. of substance 0.4422
344 Chemistry
8. 0.596 gm. of chloroplatinate of a monoacidic base gave on 13. 20 ml. of gaseous hydrocarbon was exploded with excess of
ignition 0.195 gm. of Platinum. Find the molecular weight of oxygen in an eudiometer tube. On cooling volume was
the base. reduced by 50 ml. On further treatment with KOH solution
Sol. Molecular weight of a base there was further contraction of 40 ml. What is the molecular
Acidity æ Wt. of platinum salt ´195 ö formula of hydrocarbon ?
= çç - 410 ÷÷
2 è Wt. of platinum ø æ yö y
Sol. C x H y + ç x + ÷O 2 ¾
¾® xCO 2 + H 2 O
1 æ 0.596 ´ 195 ö è 4 ø 2
= 2 ç 0.195 - 410 ÷ = 93
è ø
9. 0.22 gm. of a substance when vaporised by Victor Meyer's æ yö y
1 vol. çè x + ÷ø vol. x vol. vol.
method displaced 45.0 ml. of air measured over water at 4 2
20°C and 755 mm pressure. Calculate the molecular weight
of the substance (vapour pressure of water at 20°C is 17.4 æ yö
20 vol. 20ç x + ÷ vol. 20 x vol.
mm). è 4 ø
Sol. Calculation of volume at NTP
CO2 produced is 40 ml. Therefore 20x = 40 Þ x = 2
P1V1 P2 V2 (755 - 17.4) ´ 45 760 ´ V2
= ; = Contraction produced on explosion and cooling
T1 T2 (273 + 20) 273
æ yö
\ V2 = 40.70 ml. 20 + 20ç x + ÷ - 20 x
è 4 ø
Wt. of 40.7 ml. of vapour at NTP = 0.22 gm.
\ Wt. of 22400 ml. of vapour at NTP Contraction on explosion as given in question is 50.

0.22 æ yö
= ´ 22400 = 121.1 gm. \ 20 + 20ç x + ÷ - 20 x = 50
40.7 è 4 ø
Hence molecular weight of the substance = 121.1
10. The molal freezing point constant of benzene is 4.90 and its y
20 ´ = 30 \y = 6
melting point is 5.51°C. A solution of 0.816 g of a compound 4
A dissolved in 7.5 g of benzene freezes at 1.59°C. Calculate the Hence formula of hydrocarbon = C2H6.
molecular weight of compound A. 14. A dibasic acid gave the following results on analysis
Sol. DTf = 5.51°C - 1.59°C = 3.92°C (a) 0.177 gm. of acid gave on combustion 0.2642 gm. of CO2
K f ´ w ´ 1000 4.9 ´ 0.816 ´ 1000 and 0.081 gm. of water.
m= = = 136 (b) 0.2988 gm. of its Ag salt left 0.1944 gm. of Ag on ignition.
DTf ´ W 3.92 ´ 7.5
Determine the molecular formula of the acid and write
11. The boiling point of pure acetone is 56.38°C at normal down the structural formula of its possible isomers.
pressure. When 0.707 gm. of a substance is dissolved in 10
gm. acetone, the solution boils at 56.88°C. Calculate the 12 0.2642 ´ 100
Sol. % of C = ´ = 40.70
molecular weight of the substance. Ebullioscopic constant 44 0.177
Kb for acetone is 1.67.
Sol. Molecular weight M 2 0.081´ 100
% of H = ´ = 5.07
K b ´ w ´ 1000 1.67 ´ 0.707 ´1000 18 0.177
= = = 236.13 % of O = 100 – (40.70 + 5.07) = 54.23
DTb ´ W (56.88 - 56.38) ´ 10
Element % At. wt. Relative Simple
12. 0.177 gm. of a monobasic acid requires 30 cc of N/10 NaOH
for complete neutralisation. Calculate the molecular weight no. of atoms ratio
of the acid. 40.70 3.39
C 40.70 12 = 3.39 = 1´ 2 = 2
1 1 12 3.39
Sol. geq. of NaOH used = 30 ´ ´ = 0.003
10 1000 5.07 5.07
H 5.07 1 = 5.07 = 1.5 ´ 2 = 3
geq. of acid neutralised by NaOH = 0.003 1 3.39
0.003 geq. º 0.177 g. of acid 54.23 3.39
O 54.23 16 = 3.39 = 1´ 2 = 2
16 3.39
1 geq. º 0.177 = 59 g of acid,
0.003 Empirical formula of dibasic acid = C2H3O2
Hence equivalent of acid = 59 Empirical formula weight of the acid = 59
Since the acid is monobasic. Therefore molecular weight of
acid = 59 × 1 = 59.
Purification, Qualitative and Quantitative Analysis of Organic Compounds 345
Mol. wt. of acid
æ Wt. of Ag Salt ´ 108 ö
Mol. wt. of acid = basicity × çç - 107 ÷÷ æ Wt. of Silver salt ´ 108 ö
è Wt. of Ag ø = Basicity × çç - 107 ÷÷
è Wt . of Silver ø
æ 0.2988 ´ 108 ö
= 2ç - 107 ÷ = 118 æ 0.91´108 ö
è 0.1944 ø = 2´ç - 107 ÷ =150
è 0.54 ø
Molecular formula =
Molecular weight Mol.wt 150
(Empirical formula) n= = =2
Empirical formula weight Empirical wt 75

118 Molecular formula = (C2H3O3 )2 = C 4 H 6 O 6


= (C 2 H 3 O 2 ) = C 4 H 6 O 4 The compound is Tartaric acid
59
Since the acid is dibasic it must contain two –COOH groups COOH
which may be present on the same or different C-atoms. H OH
COOH H
CH 2 .COOH OH
| CH3 –CH
COOH
CH 2 COOH COOH
Succinic acid Methyl malonic acid 18. 0.236 g organic compound on combustion gives 0.528 g

15. 0.56 g of an organic compound gave 0.589 g of silver iodide. CO 2 and 0.324 g H 2 O . 0.295 g compound gives 56 cc
Calculate the % of I in the compound. N 2 by Dumas method . The compound is monoacidic
127 wt. of AgI base and its platinichloride contains 36.89% platinum. Find
Sol. % I = × × 100
235 wt. of organic compound the mol. formula of the compound.
12 0.528
127 0.589 Sol. %C= × × 100 = 61.02
= × × 100 = 56.84 44 0.236
235 0.56
16. 2.79 g of an organic compound was heated in a Carius tube 2 0.324
%H= × × 100 = 15.25
with conc HNO 3 and H 3PO 4 formed was 18 0.236
converted into Mg NH 4 PO 4 ppt. The precipitate on heating 28 56´ 100
%N= × = 23.74
gave 1.332 g of Mg 2 P2 O 7 . What is the % of P in the 22400 0.295
compound? Element % At. wt. Relative Simple
62 wt. of Mg 2 P2 O 7 no. of atoms ratio
Sol. % P = × × 100
222 wt. of organic compound 61.02 5.08
C 61.02 12 = 5.08 =3
12 1.69
62 1.332
= × × 100 = 13.33%
222 2.79 15.25 15.25
H 15.25 1 = 15.25 =9
17. A dibasic acid contain C = 32%, H = 4%. 0.91 g silver salt of 1 1.69
the acid gives 0.54 g silver on ignition. Determine the
molecular formula of the compound. 23.74 1.69
N 23.74 14 = 1.69 =1
Sol. 14 1.69
Empirical formula of compound C 3 H 9 N .
Element % At. wt. Relative Simple
no. of atoms ratio Mol. wt. of compound

32 2.66 Acidity æ wt. of Pt salt ´ 195 ö


C 32.0 12 =2.66 çç - 410 ÷÷
12 2.66 = 1´ 2 = 2 =
2 wt . of Pt
è ø
4.0 4 .0
H 4.0 1 =4.0 = 1.50 ´ 2 = 3 1 æ 100 ´ 195 ö
1 2.66 = - 410÷ = 59
2 çè 36.89 ø
64 4 .0
O 64.0 16 =4.0 = 1.50 ´ 2 = 3 Mol. wt
16 2.66 59
n= = =1
Empirical formula = C 2 H 3O 3 Empirical wt. 59
346 Chemistry
\ Mol. formula of compound = C 3 H 9 N SOME PRACTICE PROBLEMS ON THE
It can have any one of the following structures : ESTIMATION OF ELEMENTS
CH 3 1. 0.4950 g of CO 2 and 0.2025 g of H 2 O were obtained
CH - NH 2 by combustion of 0.275 g of an organic compound. Find
CH 3CH 2 CH 2 NH 2 CH 3 the percentage of C and H
n propylamine Isopropylamine (Ans C = 54.54%, H = 9.09%)
H 3C 2. 0.236 g of an organic compound gave 0.528 g CO 2 and
C2 H5
NH 0.324 g H 2 O on complete combustion. Determine the
CH 3 H 3C N
% of C and H in the organic compound.
H 3C (Ans C = 61.02 %, H = 15.25%)
3. 1 g of an organic compound was analysed by Kjeldahl’s
Ethylmethylamine Trimethylamine method. Ammonia evolved was absorbed in 80 ml
19. Find the molecular formula of the compound from the N N
following data H 2SO 4 . The residual acid required 35 ml NaOH
5 10
C = 16.27%, H = 0.677%, Cl = 72.20% for neutralisation. Calculate the % N in the compound.
In victor Meyer’s method 0.2950 g of compound displaces (Ans 17.64%)
50.44 ml moist air at 24°C and 735.7 mm pressure. The 4. In Duma’s experiment 0.204 g of an organic
aqueous tension at 24°C is 22.2 mm compound gave 18.6 ml moist N 2 at 17°C and 756 mm
Sol. Element % At. wt. Relative Simple pressure. Calculate the % N in the compound. (Aq.
no. of atoms ratio tension at 17°C = 14.5 mm) (Ans 10.468%)
5. 0.2046 g of an organic compound in Dumas
16.27 1.356
C 16.27 12 = 1.356 =2 experiment gave 30.4 ml N 2 at 15°C and 732 mm
12 0.677 pressure. Aq. tension at 15°C is 12.7 mm. Find the % N
in the compound. (Ans 16.68%)
0.677 0.677
H 0.677 1 = 0.677 =1 6. Ammonia obtained in Kjeldahl’s method neutralised 20
1 0.677
N
ml H SO . What is the % N in the compound.
72.20 2.03 10 2 4
Cl 72.20 35.5 = 2.03 =3
35.5 0.677 (Ans 14.0%)
7. In carius method 0.378 g of an organic compound gave
10.853 0.678 0.64 g of AgBr. Calculate the % Br in the
O 10.853 16 =0.678 =1 compound. (Ag = 108, Br = 80) (Ans 73.60%)
16 0.677
8. 0.75 g of an organic compound containing phosphorous
Empirical formula of compound C 2 HCl 3O
was heated with conc HNO 3 in the carius tube. The
Mol. wt. of compound by victor Meyer's method
H 3PO 4 formed was precipitated with Magnesia mixture
2´ W
= as MgNH 4 PO 4 . The latter when ignited after washing
V0 ´ 0.00009
and drying left a residue of 1.2545 g of Mg 2 P2 O 7 .
Where W = wt. of organic compound Calculate the % P in the compound (Mg = 24, P = 31, O
V0 = volume of dry air at NTP = 16). (Ans 46.71%)
9. 0.32 g of an organic compound containing sulphur gave
273 (753.7 - 22.2) after suitable treatment 0.2334 g of BaSO 4 . Calculate
V0 = × × 50.44 = 44.62
760 24 + 273 the % S in the compound. (Ba = 137.4 , S = 32, O = 16)
(Ans 10 %)
2 ´ 0.2950
M. wt. = = 146.9 10. An organic compound containing C, H, N and O gave
44.62 ´ 0.00009 following results on analysis.
Mol. wt. 146.9 (i) 0.50 g of the compound gave 0.726 g CO 2 and
n= = =1
Emperical wt. 147.5 0.450 g H 2 O .
(ii) Same compound gave 62.4 ml N 2 at NTP. Find the
Molecular formula of organic compound = C 2 HCl 3O
percentage of all the elements presents in the
The compound is Cl 3 C.CHO chloral. compound.
(Ans C = 39.6%, H = 10.0%, N = 15.6%, O = 34.8%)
Purification, Qualitative and Quantitative Analysis of Organic Compounds 347

Very Short/ Short Answer Questions Multiple Choice Questions


1. Two volatile compounds A and B differ in their boiling points 17. The principle involved in paper chromatography is
by 15K. Suggest a suitable method for their separation. (a) Adsorption (b) Partition
2. How will you separate a mixture of o– and p– nitrophenols? (c) Solubility (d) Volatility
3. Name two methods which can be safely used to purify 18. The fragrance of flowers is due to the presence of some
aniline. steam volatile organic compounds called essential oils.
4. What type of organic compounds cannot be Kjeldahlised? These are generally insoluble in water at room temperature
5. Name the process of separating benzoic acid and but are miscible with water vapour in vapour phase. A
naphthalene. suitable method for the extraction of these oils from the
6. What is the suitable adsorbent in the process of column flowers is:
chromatography? (a) Distillation
7. What is the value of ‘x’ in an organic compound with (b) Crystallisation
molecular formula CxH12 with vapour density 42? (c) Distillation under reduced pressure
8. Why are melting point and refractive index used as the (d) Steam distillation
criteria of purity of organic compounds? 19. During hearing of a court case, the judge suspected that
some changes in the documents had been carried out. He
9. It is advisable to prepare Lassaigne’s extract in distilled water.
asked the forensic department to check the ink used at two
Why?
different places. According to you which technique can give
10. Explain why an organic liquid vapourizes at a temperature the best results?
below its boiling point in its steam distillation?
(a) Column chromatography
11. What type of compounds are purified by sublimation?
(b) Solvent extraction
12. What are the advantages of using the chromatography
(c) Distillation
method over other methods of separation?
(d) Thin layer chromatography
13. In Carius method of estimation of halogen, 0.15 g of an
20. Styrene can be purified by
organic compound gave 0.12 of AgBr. Find the percentage
of bromine in the compound. (a) Simple distillation (b) Fractional distilation
14. A sample of 0.50 g of an organic compound was treated (c) Steam distillation (d) Vacuum distillation
according to Kjeldahl’s method. The ammonia evolved was 21. In Kjeldahl’s method for the estimation of nitrogen, the
absorbed in 50 ml of 0.5 MH2SO4. The residual acid required foumula used is
60 ml of 0.5 M solution of NaOH for neutralization. Find the 1.4VW 1.4VW
percentage composition of nitrogen in the compound. (a) %N = (b) %N =
N V
Long Answer Questions VNW 1.4 VN
(c) %N = (d) %N =
15. (i) On combustion, 0.3650 g of compound (A) gave 0.88 g 1.8 W
CO2 and 0.495 g H2O. Further, 0.1186 g of the same
compound gave, on combustion by Dumas’ method, 22. Chromatography is a valuable method for the separation,
20 ml of nitrogen measured at 27°C and 779 mm Hg isolation, purification and identification of the constituents
pressure. (Aqueous tension at 27°C = 19 mm Hg). What of a mixture and it is based on general principle of
is the percentage composition of the compound? (a) phase rule (b) phase distribution
(ii) A 3.87 mg sample of organic compound give 5.80 mg (c) interphase separation (d) phase operation
CO2 and 1.58 mg H2O on combustion. What is the 23. In Carius method 0.099 g organic compound gave 0.287 g
percentage composition of this compound? The organic AgCl. The percentage of chlorine in the compound will be
compound contains only C, H and O. (a) 28.6 (b) 71.7 (c) 35.4 (d) 64.2
16. (i) Give reaction for test of sulphur with sodium 24. Which of the following compounds gives blood red
nitroprusside in organic compound. colouration when its Lassaigne’s extract is treated with alkali
(ii) What is the role of and ferric chloride ?
(a) K2SO4 in Kjeldahl solution during digestion. (a) Thiourea (b) Diphenyl sulphide
(c) Phenyl hydrazine (d) Benzamide
(b) Dil. HNO3 in Lassaigne's extract.
348 Chemistry

1. For the separation of two immiscible liquids which method (a) sodium hydroxide (b) sodium sulphate
(or apparatus) is used? (c) calcium chloride (d) sodium bicarbonate
(a) chromatography (b) fractionating column 12. In sodium fusion test of organic compounds, the nitrogen
(c) fractional distillation (d) separating funnel of the organic compound is converted into
2. A mixture of benzene and chloroform is separated by (a) sodamide (b) sodium cyanide
(a) sublimation (b) separating funnel (c) sodium nitrite (d) sodium nitrate
(c) crystallization (d) distillation 13. The Lassaigne’s extract is boiled with dil. HNO3 before
3. Which of the following has molecular weight of 92 ? testing for halogens because
(a) Toluene (b) Benzene (a) silver halides are soluble in HNO3
(c) Methylene (d) Propene (b) Na2S and NaCN are decomposed by HNO3
4. Aniline is usually purified by (c) Ag2S is soluble in HNO3
(a) chromatographic technique (d) AgCN is soluble is HNO3
(b) steam distillation 14. Which of the following compounds does not show
Lassaigne’s test for nitrogen ?
(c) by addition of oxalic acid
(a) Urea (b) Hydrazine
(d) fractional crystallization
(c) Phenylhydrazine (d) Azobenzene
5. An organic compound contains 49.3% carbon, 6.84%
hydrogen and its vapour density is 73. Empirical formula of 15. When pieces of human hair are heated strongly with soda-
the compound is lime, smell of ammonia can be detected. Which one of the
following conclusion can be drawn form this observation?
(a) C3H5O2 (b) C6H10O4
(a) Ammonia is present in the human hair
(c) C3H10O2 (d) C4H10O4
(b) Ammonium salt is present in hair
6. Distillation under reduced pressure is employed for
(c) Hair contains amino acids
(a) C6H6
(d) None of the above
(b) petrol
16. Which of the following is the best scientific method to test
(c) CH2OHCHOHCH2OH
presence of water in a liquid ?
(d) organic compounds used in medicine
(a) Smell
7. Impure glycerine is purified by
(b) Taste
(a) steam distillation (b) simple distillation
(c) Use of litmus paper
(c) vacuum distillation (d) None of these
(d) Use of anhydrous copper sulphate
8. Absolute alcohol is prepared by
17. Liebig’s method is used for the estimation of
(a) fractional distillation (b) Kolbe’s method
(a) nitrogen (b) sulphur
(c) azeotropic distillation (d) vacuum distillation
(c) carbon and hydrogen (d) halogens
9. The latest technique used for purification of organic
18. 0.765 g of an acid gives 0.535 g of CO2 and 0.138 g of H2O.
compounds is
Then the ratio of the percentage of carbon and hydrogen is
(a) chromatography (b) vacuum distillation
(a) 19 : 2 (b) 18 : 11
(c) fractional distillation (d) crystallization
(c) 20 : 17 (d) 1 : 7
10. In paper chromatography
19. Nitrogen in an organic compound can be estimated by
(a) moving phase is liquid and stationary phase in solid
(a) Kjeldahl’s method only (b) Duma’s method only
(b) moving phase is liquid and stationary phase is liquid
(c) Both (a) and (b) (d) Neither (a) nor (b)
(c) moving phase is solid and stationary phase is solid
20. Duma's method involves the determination of nitrogen
(d) moving phase is solid and stationary phase is liquid content in the organic compound in form of
11. A is a lighter phenol and B is an aromatic carboxylic acid. (a) NH3 (b) N2
Separation of mixture of A and B can be carried out easily by
(c) NaCN (d) (NH4)2SO4
using a solution of
Purification, Qualitative and Quantitative Analysis of Organic Compounds 349
21. Molecular mass of a volatile substance may be obtained by (a) 65.0 (b) 64.0
(a) Kjeldahl’s method (b) Duma’s method (c) 63.8 (d) 62.5
(c) Victor-meyer’s method (d) Liebig’s method 33. The equivalent weight of an acid is equal to
22. The percentage of sulphur in an organic compound whose (a) Molecular weight × acidity
0.32 g produces 0.233 g of BaSO4 [At. wt. Ba = 137, S = 32] is (b) Molecular weight × basicity
(a) 1.0 (b) 10.0 (c) Molecular weight/basicity
(c) 23.5 (d) 32.1 (d) Molecular weight/acidity
23. If 0.2 gram of an organic compound containing carbon, 34. Simple distillation can be used to separate liquids which
hydrogen and oxygen on combustion, yielded 0.147 gram differ in their boiling points at least by
carbon dioxide and 0.12 gram water. What will be the content (a) 5ºC (b) 10ºC
of oxygen in the substance ? (c) 30º – 50ºC (d) 100ºC
(a) 73.29% (b) 78.45% 35. Empirical formula of a compound is CH2O. If its vapour
(c) 83.23% (d) 89.50% density is 90, then the molecular formula of the compound
24. If a compound on analysis was found to contain is
C = 18.5%, H = 1.55%, Cl = 55.04% and O = 24.81.%, then its (a) C5H10O5 (b) C3H6O3
empirical formula is (c) C6H12O6 (d) C4H8O4
(a) CHClO (b) CH2ClO 36. The empirical formula of an acid is CH2O2, the probable
(c) C2H2OCl (d) ClCH2O2 molecular formula of the simplest acid may be
25. An organic compound contains C = 40%, H = 13.33% and N (a) CH2O (b) CH2O2
= 46.67%. Its empirical formula would be (c) C2H4O4 (d) C3H6O6
(a) CHN (b) C2H2N 37. A gaseous hydrocarbon has 85% carbon and vapour
(c) CH4N (d) C3H7N density of 28. The possible formula of the hydrocarbon will
be
26. Two elements X (atomic weight = 75) and Y (atomic weight
(a) C3H6 (b) C2H4
= 16) combine to give a compound having 75.8% X. The
formula of the compound is (c) C2H2 (d) C4H8
(a) XY (b) X2Y 38. A hydrocarbon has C= 85.72% and remaining H. The
hydrocarbon is
(c) X2Y2 (d) X2Y3
(a) C2H4 (b) C2H6
27. On analysis, a saturated hydrocarbon is found to contain
(c) C2H2 (d) CH4
83.70% carbon and 16.30% hydrogen. The empirical formula
39. An organic compound containing carbon, hydrogen and
will be (at wt. of C= 12; at. wt. of H = 1)?
oxygen contains 52.2% carbon and 13.04% hydrogen.
(a) C3H6 (b) C3H8
Vapour density of the compound is 23. Its molecular formula
(c) C3H4 (d) C6H12 will be
28. Empirical formula of a hydrocarbon containing 80% carbon (a) C2H6O (b) C3H8O
and 20% hydrogen is (c) C4H8O (d) C5H10O
(a) CH (b) CH2 40. Percentage of Se(at. wt. = 78.4) in peroxidase anhydrase
(c) CH3 (d) CH4 enzyme is 0.5% by weight, then minimum molecular weight
29. An organic compound with C = 40% and H = 6.7% will have of peroxidase anhydrase enzyme is
the empirical formula (a) 1.568 × 104 (b) 1.568 × 103
(a) CH4 (b) CH2O (c) 15.68 (d) 2.136 × 104
(c) C3H6O3 (d) C2H4O2 41. The empirical formula of a compound is CH 2. One mole of
30. In a hydrocarbon, mass ratio of hydrogen and carbon is 1:3, the compound has a mass of 42 g, its molecular formula is
the empirical formula of hydrocarbon is (a) CH2 (b) C2H2
(a) CH4 (b) CH2 (c) C3H6 (d) C3H8
(c) C2H (d) CH3 42. In a compound, C, H and N atoms are present in 9 :1: 3.5 by
31. 0.24 g of a volatile liquid on vaporization gives 45 ml of weight . If molecular weight of the compound is 108, then
vapours at NTP. What will be the vapour density of the molecular formula of the compound is
substance ? (Density of H2 = 0.089 g L–1) (a) C2H6N2 (b) C3H4N
(a) 95.39 (b) 39.95 (c) C6H8N2 (d) C9H12N3
(c) 99.53 (d) 59.93 43. The following is the percentage composition of a compound
32. 0.1914 g of an organic acid is dissolved in about 20 ml of : Na = 16.08%, C = 4.19%, O = 16.78% and H2O = 62.95% . Its
water. 25 ml of 0.12 N NaOH is required for the complete molecular formula is
neutralization of the acid solution. The equivalent weight of (a) Na2CO3 (b) Na2CO3H2O
the acid is (c) Na2CO3.10H2O (d) Na2CO3.5H2O
350 Chemistry

1. Lassaigne’s test is used in qualitative analysis to detect 7. The best method for the separation of naphthalene and
[CBSE-PMT 1989] benzoic acid from their mixture is:
(a) Nitrogen (b) Sulphur [CBSE-PMT 2005]
(c) Chlorine (d) All of these (a) distillation (b) sublimation
2. Kjeldahl’s method is used in the estimation of
(c) chromatography (d) crystallisation
[CBSE-PMT 1990]
8. The compound formed in the positive test for nitrogen with
(a) Nitrogen (b) Halogens the Lassaigne solution of an organic compound is
(c) Sulphur (d) Oxygen
[AIEEE-2004]
3. In sodium fusion test of organic compounds, the nitrogen of
(a) Fe4[Fe(CN)6]3 (b) Na3[Fe(CN)6]
the organic compound is converted into
[CBSE-PMT 1991] (c) Fe(CN)3 (d) Na4[Fe(CN)5NOS]
(a) Sodamide (b) Sodium cyanide 9. In a compound C, H and N atoms are present in 9 : 1 : 35 by
weight. Molecular weight of compound is 108. Molecular
(c) Sodium nitrite (d) Sodium nitrate
formula of compound is [AIEEE 2002]
4. A is a lighter phenol and B is an aromatic carboxylic acid.
Separation of a mixture of A and B can be carried out easily (a) C2H6N2 (b) C3H4N
by using a solution of [CBSE-PMT 1992] (c) C6H8N2 (d) C9H12N3.
(a) Sodium hydroxide 10. The ammonia evolved from the treatment of 0.30 g of an organic
(b) Sodium sulphate compound for the estimation of nitrogen was passed in 100
(c) calcium chloride mL of 0.1 M sulphuric acid. The excess of acid required 20 mL
of 0.5 M sodium hydroxide solution for complete neutralization.
(d) Sodium bicarbonate
The organic compound is [AIEEE 2004]
5. The most suitable method for separtion of a 1 : 1 mixture of ortho
and para nitrophenols is [CBSE-PMT 1994] (a) urea (b) benzamide
(a) Sublimation (b) Chromatography (c) acetamide (d) thiourea
(c) Crystallization (d) Steam distillation 11. Sodium nitroprusside, when added to an alkaline solution
6. The first organic compound, synthesized in the laboratory, of sulphide ions, produces purple colour ion due to the
was [CBSE-PMT 1995] formation of [IIT 1995, Orissa JEE 2008]
(a) alcohol (b) acetic acid (a) Na[Fe(H2O)5 NOS] (b) Na2[Fe(H2O)5 NOS]
(c) urea (d) none of these (c) Na3[Fe(CN)5NOS] (d) Na4[Fe(CN)5NOS]
Purification, Qualitative and Quantitative Analysis of Organic Compounds 351

1. In Kjeldahl’s method nitrogen present is estimated as 11. Liquid benzene burns in oxygen according to
(a) N2 (b) NH3
2C6H6 (l) +15O2 (g) ®12CO2 (g) + 6H2O(g) . How many litre
(c) NO2 (d) None of these
2. Lassaigne’s test for the detection of nitrogen will fail in of O2 at STP are needed to complete the combustion of 39g
case of of liquid benzene:
(a) 11.2 litre (b) 22.4 litre
(a) NH 2 CONH 2 (b) H 2 NCONHNH 2 .HCl
(c) 84 litre (d) 74 litre
(c) H 2 N.NH 2 .2HCl (d) H 6 C5 NHNH 2 .2HCl 12. 0.75 g platinic chloride of a mono- acidic base on ignition
3. The most suitable method of separation of 1:1 mixture of gives 0.245 g platinum. The molecular weight of the base is
ortho and para nitrophenols is (a) 75.0 (b) 93.5
(a) distillation (b) crystallisation (c) 100 (d) 80.0
(c) sublimation (d) chromatography 13. An organic compound has C and H percentage in the ratio 6
4. In kjeldahl's method of estimation of nitrogen, K2SO4 acts as : 1 and C and O percentage in the ratio 3:4 the compound is
(a) oxidising agent (b) catalytic agent (a) HCHO (b) CH3OH
(c) hydrolysing agent (d) boiling point elevator (c) CH3CH2OH (d) (COOH)2
5. For detection of sulphur in an organic compound sodium 14. Before testing halogens the sodium extract is boiled with
nitroprusside is added to the sodium extract. A violet colour conc. HNO 3 to
is obtained due to formation of (a) bring common ion effect
(a) Fe(CN ) 2 (b) K 3 Fe(CN ) 5 NS (b) make solution clear
(c) destroy CN– and S– – ions
(c) Na 4 [Fe(CN ) 5 NOS] (d) Na 4 Fe(CN ) 6
(d) make the solution acidic
6. Which of the following reagents is useful for separating 15. The sodium extract prepared from sulphanilic acid, contains
aniline and nitrobenzene?
SCN - . It gives blood red colouration with
(a) NaOH (aq) (b) H 2 O
(a) a mixture of Na 2S and CS 2
(c) NaHCO3 (d) HCl (aq)
7. 0.5g of an organic compound containing nitrogen on (b) FeCl 3
Kjeldahlising required 29 mL of N/5 H2SO4 for complete
neutralization of ammonia. The percentage of nitrogen in the (c) FeSO 4
compound is (d) Na 2SO 3
(a) 34.3 (b) 16.2
(c) 21.6 (d) 14.8 16. An organic compound is treated with NaNO 2 and dil. HCl
8. Kjeldahl’s method cannot be used for the estimation of at 0°C. The resulting solution is added to an alkaline solution
nitrogen in of b -naphthol where by a brilliant red dye is produced. It
(a) pyridine (b) nitro compounds shows the presence of
(c) azo compounds (d) All of these
(a) – NO 2 group (b) aromatic – NH 2 group
9. A compound contains 38.8% C, 16% H and 45.2% N. The
formula of compound would be : (c) – CONH 2 group (d) aliphatic – NH 2 group
(a) CH 3 NH 2 (b) CH 3CN 17. Methanol and ethanol can be distinguished by
(a) iodoform reaction (b) Esterification
(c) C 2 H 5 CN (d) C H2(NH2)2
(c) Victor Meyer test (d) Acrolein test
10. 0.22 g of organic compound C x H y O which occupied 112 ml 18. 0.532 g of the chloroplatinate of an organic base (M. wt. 264)
gave 0.190 g of Pt on ignition. Then the number of nitrogen
at NTP and on combustion gave 0.44 g CO2. The ratio of x to
atoms per molecule of the base is
y in the compound is
(a) 4 (b) 3
(a) 1 : 1 (b) 1 : 2
(c) 1 (d) 2
(c) 1 : 3 (d) 1 : 4
352 Chemistry
19. 0.59 g of an organic substance when treated with caustic 27. To detect iodine in presence of bromine, the sodium extract
soda evolved ammonia which required 20 cc of N/2 sulphuric is treated with NaNO 2 + glacial acetic acid + CCl 4 . Iodine
acid for neutralisation. The percentage of nitrogen is
is detected by the appearance of
(a) 26.32% (b) 40%
(c) 53.6% (d) 63.6% (a) yellow colour of CCl 4 layer
20. Which of the following compounds will give foul odour (b) purple colour of CCl 4
of isocyanide when heated with chloroform and alcoholic
KOH ? (c) brown colour in the organic layer of CCl 4
(a) Sulphanilic acid (b) Para toluidine
(d) deep blue colour in CCl 4
(c) Glycine (d) Anthranilic acid
21. 10.0 ml mixture of methane and ethylene was exploded with 28. Which of the following compounds will give silver mirror
30 ml (excess) of oxygen. After cooling the volume was 21.0 test ?
ml. On treatement with KOH solution the volume was reduced (a) CH 3COOH (b) HCOOH
to 7.0 ml. The amount of methane and ethylene in the mixture
respectively are (c) CH 3 CO. COOH (d) None of these
(a) 5 ml CH 4 + 5 ml C 2 H 4 29. Benzaldehyde and acetone can be distinguished by

(b) 3 ml CH 4 + 7 ml C 2 H 4 (a) NaHSO3 test (b) nitropruside test

(c) 6 ml CH 4 + 4 ml C 2 H 4 (c) Fehling solution test (d) iodoform test


30. In the estimation of carbon and hydrogen by combustion
(d) 4 ml CH 4 + 6 ml C 2 H 4 method which of the following is/are correct ?
22. In steam distillation the vapour pressure of volatile organic I. A spiral of copper is introduced at the right extreme of
compound is combustion tube if the organic compound contains
(a) equal to atmospheric pressure nitrogen.
(b) double the atmospheric pressure II. A spiral of silver is introduced if the organic compound
(c) less than atmospheric pressure contains halogens.
(d) more than atmospheric pressure III. The copper oxide in the combustion tube is replaced by
23. Which of the following compounds will give orange red ppt. lead chromate if the organic compound contains sulphur.
with 2, 4-dinitrophenylhydrazine reagent ? (a) I and II are correct (b) I and III are correct
(a) C 6 H 5 OH (b) CH 3CH 2 COOH (c) II and III are correct (d) All are correct

(c) CH 3COOC 2 H 5 (d) CH 3COCH 3 31. In Kjeldahl’s method for the estimation of N 2 , potassium
24. Beilstein’s test is given by which of the following? sulphate and copper sulphate are used. On the basis of their
(a) Halogens (b) Thiourea functions which of the following statement(s) is/are correct?
(c) Pyridine (d) All of these I. Potassium sulphate raises the bpt. and ensures complete
25. The molecular mass of an organic compound which contains reaction.
only one nitrogen atom can be II. Copper sulphate acts as catalyst.
(a) 152 (b) 146 III. Potassium sulphate acts as catalyst and copper sulphate
(c) 76 (d) 73 raises the bpt.
26. Which of the following observations is correct and useful in (a) Only III is correct (b) I and II are correct
identifying carboxylic acids ? (c) Only II is correct (d) None is correct
(a) Carboxylic acids liberate CO 2 gas from NaHCO 3 32. 2.79 g of an organic compound when heated in Carius tube
solution with conc. HNO 3 and H 3PO 4 formed converted into
(b) Acids liberate I 2 from a mixture of KIO 3 and KI MgNH 4 .PO 4 ppt. The ppt. on heating gave 1.332 g of
(c) They produce fruity smell of esters when heated with Mg 2 P2 O 7 . The percentage of P in the compound is
alcohol in presence of conc. H 2SO 4
(a) 23.33% (b) 13.33%
(d) All of these (c) 33.33% (d) 26.66%
Purification, Qualitative and Quantitative Analysis of Organic Compounds 353

EXERCISE 1 Na 2S + 2AgNO 3 ¾
¾® 2 NaNO 3 + Ag 2S ¯
1. By fractional distillation. Black
2. o–nitrophenol is steam volatile while p–nitro phenol is not NaCN + AgNO 3 ¾
¾® NaNO3 + AgCN ¯
and hence these can be separated by steam distillation. White
3. Vacuum distillation and steam distillation. NaCl + AgNO 3 ¾
¾® NaNO3 + AgCl ¯
5. Fractional crystallisation using benzene as a solvent. white
6. Al2O3 (alumina). boil
Na 2S + 2HNO 3 ¾¾¾® 2 NaNO 3 + H 2S ­
7. V.D. is 42, Mol. wt. = 2 × V.D. = 2 × 42 = 84 therefore, molecular boil
formula is C6H12. NaCN + HNO 3 ¾¾
¾® NaNO 3 + HCN ­
8. Every pure substance has fixed melting point and refractive 14. (b) Hydrazine (NH2NH2) does not contain carbon and hence
index. Impurities reduce the melting point and change on fusion with Na metal, it cannot form NaCN;
refractive index. consequently hydrazine does not show Lassaigne’s test
9. Tap water usually contains dissolved chlorine which gives for nitrogen.
test for halogens. 15. (c) Hair contains amino acids which upon fusion with soda
13. Percentage of Br = 34.04% lime (NaOH + CaO) evolve NH3.
14. Percentage of nitrogen = 56% 16. (d) Hydrated CuSO4 is blue in colour, while anhydrous is
17. (b) 18. (d) 19. (d) 20. (d) white.
21. (d) 22. (b) 23. (b) 24. (a) CuSO 4 + 5H 2 O ¾
¾® CuSO 4 .5H 2 O
White Blue
EXERCISE 2 17. (c)
1. (d)
2 (d) Benzene (b.p. 80ºC) and chloroform (b.p. 61.5ºC) have 18. (a) % of C = 12 ´ 0.535 ´ 100 = 19.07 %
44 0.765
different boiling points.
2 0.138
3. (a) Mol. wt. of toluene, C6H5CH3 = 72 + 5 + 12 + 3 = 92 % of H = ´ ´ 100 = 2.004 %
4. (b) Aniline is miscible in steam and has b.p. lower than water, 18 0.765
hence it is collected as distillate along with steam. Ratio of C : H = 19 : 2
5. (a) % of O = 100 – (49.3 + 6.84) = 43.86 19. (c) 20. (b) 21. (c)

Relative ratio of C : H : O =
49.3 6.84 43.86
: : 22. (b) % of S = 32 ´ 0.233 ´ 100 = 10 %
12 1 16 233 0.32
= 4 : 6.84 : 2.74 12 0.147
23. (a) % of C = ´ ´ 100 = 20.045 %
= 1.49 : 2.49 : 1 44 0 .2
= 3 : 5 : 2 (multiplying by 2) 2 0.12
\ Empirical formula = C3H5O2 % of H = ´ ´ 100 = 6.666 %
18 0.2
6. (c) Glycerol decomposes at its boiling point, hence it should
be purified by distillation under reduced pressure. % of O = 100 - (20.045 + 6.666) = 73.29 %
7. (c) Vaccum distillation means distillation under reduced 24. (a) Relative ratio of wts.
pressure. C H Cl O
8. (c) Since alcohol and water form a constant boiling mixture 18.5 1.55 55.04 24.81
(azeotrope). 35.5 16
12 1
9. (a) 10. (b) = 1.55 = 1.55 = 1.55 = 1.55
11. (d) Carboxylic acids dissolve in sodium bicarbonate, while
\ Relative No. of atoms
phenol does not.
12. (b) 1 1 1 1
13. (b) Na2S and NaCN, formed during fusion with metallic Hence empirical formula is CHClO
sodium, must be removed before adding AgNO 3 , 25. (c) As in above question,
otherwise black ppt. due to Na2S or white precipitate due
to AgCN will be formed and thus white precipitate of 40 13.33 46.67
C= = 3.33; H = = 13.33; N = = 3.34
AgCl will not be identified easily. 12 1 14
354 Chemistry
Relative No. of atoms, 85.72 14.28
38. (a) C : H = : = 7.15 : 14.28 = 1 : 2
3.33 13.33 3.34 12 1
C= = 1; H = = 4; N = =1
3.33 3.33 3.33 \ Empirical formula = CH2
\ Empirical formula = CH4N From the give options, simple multiple of CH2 is only C2H4
75 . 8 24 . 2 39. (a) % of O = 100 – (52.2 + 13.04) = 34.76%
26. (d) Relative No. of atoms : X = = 2;Y = = 3 Relative ratio of
75 16
C H O
\ Empirical formula = X2Y3
27. (b) Relative No. of atoms 52.2 13.04 34.76
C H 12 1 16
83.70 16.30 4.35 13.04 2.17
=7 = 16.30
12 1 2 6 1
\ Empirical formula = C2H6O
7 16.30
or =1 = 2.33
7 7 2 ´ 23
Mol. formula = ´ C 2 H 6O = C 2 H 6 O
or 1 × 3 = 3 2.33 × 3 = 8 24 + 6 + 16
\ Empirical formula = C3H8 At. wt. of Se
40. (a) % of Se = ´100
80 20 6.66 20 Min. mol. wt.
28. (c) C : H or = 6.66 ; = 20 or =1; =3
12 1 6.66 6 .66
\ Empirical formula = CH3 \ Min. mol. wt. = At. wt. of Se ´100
29. (b) % of O = 100 – (40 + 6.7) = 53.33 % of Se
78.4 ´ 100
40 6.7 53.33 = = 1.568 ´ 10 4
C:H:O = : : = 3.33 : 6.7 : 3.33 = 1 : 2 : 1 0.5
12 1 16 41. (c) Mol. wt. of the compound
\ Empirical formula = CH2O = Mass of 1 mole of the compound = 42
30. (a) Mass ratio of H : C = 1 : 12 empirical formula wt. of CH2 = 12 + 2 = 14
However, given mass ratio of H : C = 1 : 3
Therefore, for every C atom, there are 4 H atoms, hence 42
\ Molecular formula = CH 2 ´ = C3 H 6
empirical formula = CH4 14
42. (c) Relative ratio of wts. of C : H : N = 9 : 1 : 3.5
31. (d) V.D. = Wt. of 45 ml. of vapours at NTP
Wt. of 45 ml. of H 2 at NTP 9 1 3.5
Relative No. of atoms : : =0.75:1: 0.25=3: 4:1
12 1 14
0.24 g \ Empirical formula = C3H4N
= = 59.93
45 ml. ´ 0.000089 g ml -1 Mol. wt.(given) = 108
32. (c) 25 ml of 0.12 N NaOH = 0.1914 g of acid 108
\ Mol. formula = ´ C3 H 4 N = C 6 H8 N 2
0.1914 ´ 1000 36 + 4 + 14
1000 ml of 1 N NaOH = = 63.8 43. (c) Relative No. of atoms or molecule
25 ´ 0.12
\ Eq. wt. of the acid = 63.8 16.08 4.19 16.78 62.95
Na : C : O : H 2 O = : : :
33. (c) 34. (c) 23 12 16 18
35. (c) Mol. wt. = 2 × V.D. = 2 × 90 = 180 = 0.70 : 0.35 : 1 : 3.50
Empirical formula wt. = 12 + 2 + 16 = 30 = 2 : 1 : 3 : 10
Mol. wt. \ Empirical formula = Na2CO3.10H2O
\ Molecular formula = EF wt. ´ empirical formula
EXERCISE 3
180 1. (d) Nitrogen, sulphur and halogens are tested in an organic
= ´ CH 2 O = C6 H12 O6
30 compound by Lassaigne's test. The organic compound
36. (b) The simplest acid is HCOOH whose mol. formula is CH2O2 is fused with sodium metal as to convert these elements
which coincides with the given empirical formula. into ionisable inorganic substances,
85 15 Na + C + N ¾¾
® NaCN
37. (d) C : H = : = 7 : 15 » 1 : 2
12 1
\ Empirical formula = CH2 2Na + S ¾¾
® Na 2S
Mol. wt. = 2 × 28 = 56
2Na + X 2 ¾¾
® 2NaX
56
Mol. Formula = ´ CH 2 = C 4 H 8 The cyanide, sulphide or halide ions can be confirmed in
12 + 2 aqueous solution by usual test.
Purification, Qualitative and Quantitative Analysis of Organic Compounds 355
2. (a) Kjeldahl's method is suitable for estimating nitrogen in 4. (d) To increase the bpt of H2SO4,K2SO4 is added
those compounds in which nitrogen is linked to carbon
and hydrogen. The method is not used in case of nitro, 5. ( c) Na 2S + Na 2 [Fe(CN)5 NO] ¾¾
® Na 4[Fe(CN)5 NOS]
azo and azoxy compound. This method is basically used Sodium nitroprusside Sodium thionitroprusside
for estimating nitrogen in food fertilizers and agricultural + –
products. 6. (d) C6 H 5 NH 2 +HCl ® C6 H 5 NH 3 Cl
Soluble salt
3. (b) Sodium cyanide (Na + C + N ® NaCN).
(Lassaigne's test) Nitrobenzene remains unaffected
4. (d) Carboxylic acids dissolve in NaHCO 3 but phenols do 1.4 ´ N ´ V 1.4 ´ 29 ´ 1/ 5
7. (b) N% = = = 16.24%
not. wt.of organic compound 0.5
NaHCO3
RCOOH ¾¾¾¾ ® RCOONa + H 2O + CO2 8. (d) Kjeldahl’s method is not suitable for heterocylclic ring
by evolving CO2 gas. with N atom, azo, azoxy and nitro compounds
5. (d) The boiling point of o-nitrophenol is less than para-
9. (a) % of element Relative Simple ratio
nitrophenol due to presence of intramolecular hydrogen
bonding. Since p-nitrophenol is less volatile in than o- no. of atoms
nitrophenol due to presence of inter molecular hydrogen 38.8
bonding hence they can be separated by steam distillation. C 38.8 = 3.2 1
12
6. (c)
7. (b) Among the given compounds naphthelene is volatile 16
H 16.0 = 16.0 5
but benzoic acid is non-volatile (it forms a dimer). So, 1
the best method for their separation is sublimation, 45.28
which is applicable to compounds which can be N 45.28 = 3.2 1
14
converted directly into the vapour phase from its solid
state on heating and back to the solid state on cooling. 10. (b) M.wt of compound = 0.22 ´ 22400 = 44
Hence it is the most appropriate method. 112
8. (a) Prussian blue Fe 4 [Fe(CN ) 6 ]3 is formed in lassaigne 12 0.44 ´ 100
test for nitrogen. % of C = ´ = 54.54
44 0.22
3Na 4 [Fe(CN)6 + Fe3+ ¾¾
®
Fe 4 [Fe(CN) 4 ]3 + 12Na +
Amount of C in compound = 44 ´ 54.54 = 24
Prussian blue 100
9. (c) According to given molecular weight.
\ molecular formula is C2HyO or C2H4O
10. (a) H 2SO 4 is dibasic.
It corresponds to M.wt 44. Hence x : y is 1 : 2
0.1 MH 2SO 4 = 0.2 NH 2SO 4 11. (c) 2 × 78 g C6H6 requires 15 × 22.4 litre O2
Meq of H 2SO 4 taken = = 100 ´ 0.2 = 20
Meq of H 2SO 4 neutralised by NaOH = 20 ´ 0.5 = 10 \ 39 g C6H6 will require = 15 ´ 22.4 ´ 39 = 84 litre
2 ´ 78
Meq of H 2SO 4 neutralised by NH3 = 12 - 10 = 10
12. (b) Apply the formula M.wt
1.4 ´ Meq of acid neutrialised by NH3
% of N 2 = Acidity æ wt of Pt salt ´195 ö
wt. of organic compound = ç - 410 ÷÷
1.4 ´ 10 2 çè wt of Pt ø
= = 46.6
0.3
% of nitrogen in urea = 14 ´ 2 ´ 100 = 46.6 \ M.wt = 1 æç 0.75 ´ 195 - 410 ö÷ = 93.5
60 2 è 0.245 ø
11. (d) Na 2 [Fe(CN)5 NO] + S2- ® Na 4 [Fe(CN)5 NOS] 13. (a) % ratio of C : H is 6 : 1 and C : O is 3 : 4 or 6 : 8
Sodium thio-nitroprusside
(violet) \ % ratio of C : H : O is 6 : 1 : 0 ( Total = 15)

EXERCISE 4 % of C = 6 ´ 100 = 40 40 / 12 = 3 . 33 = 1
15
1. (b) In Kjeldahl’s method nitrogen is converted into
1
(NH4)2 SO4, then to NH3 %H= ´ 100 = 6.6 6.6 / 1 = 6.6= 2
2. (c) For detection of nitrogen by Lassaigne’s method, the 15
8
compound must contain C and N both to form NaCN. % O = ´100 = 53.3 53.3 / 16 = 3 . 3= 1
3. (a) o- nitrophenol is more volatile due to intramolecular H- 15
bonding . p - nitrophenol is less volatile due to simple ratio CH2O
intermolecular H-bonding \ The compound is H.CHO
356 Chemistry

14. (c) It decomposes NaCN and Na 2S to avoid their Vol. of CO 2 produced + O 2 left = 21 ml
interference Vol. of CO 2 produced = 21 – 7 = 14 ml
NaCN + HNO 3 ¾
¾® NaNO3 + HCN ­ CO 2 produced is
Na 2S + 2 HNO 3 ¾
¾® 2 NaNO3 + H 2S ­ a + 2(10 – a) = 14
\ a=6
NaCN + AgNO 3 ¾
¾® AgCN ¯ + NaNO3 \ Vol. of CH 4 = 6 ml and vol. of C 2 H 4 = 4 ml
22. (d)
Na 2S + 2 AgNO 3 ¾
¾® Ag 2S ¯ + 2 NaNO3
NO 2
- +++
15. (b) SCN + Fe ¾
¾® Fe(SCN ) 3
blood red ppt. 23. (d) C = O + H 2 N.NH NO 2 ¾
¾®
OH
16. (b) N 2 Cl + NO 2
¾
¾®

C = NNH NO 2
OH
N=N orange red ppt.
24. (d) Pyridine and thiourea form volatile cupric cyanide and
Red dye also give bluish green flame like volatile cupric halides.
I 2 / NaOH
25. (d) The compounds with odd number of N-atoms have odd
17 (a) CH 3COCH 3 ¾¾¾¾ ¾® CHI3 (iodoform) masses and with even number of N-atoms have even
18. (a) The equivalent wt. of base masses. This is “nitrogen rule”.
26. (d) All the points are correct.
1 æ 0.532 ö
E= ç ´ 195 - 410 ÷ = 68
2 è 0.190 ø 27. (b) 2NaI + 2 NaNO 2 + 4 CH 3COOH ¾
¾®
I 2 + 2NO + 4 CH 3COONa + 2 H 2 O
264
Acidity = =4 28. (b) HCOOH is reducing in nature
68
Acidity of base = number of N atoms per molecule of Ag 2 O + HCOOH ¾ ¾® 2Ag + H 2 O + CO 2
base = 4 silver mirror
19. (a) Percentage of nitrogen by Kjeldahl’s method 29. (d) Benzaldehyde does not give iodoform test while acetone
1.4 ´ N ´ V 1.4 ´ 20 1 forms CHI3 .
= = ´ = 26.32%
Wt. of organic compound 0.532 2 30. (d) When organic compound contains nitrogen, upon
combustion it will produce oxides of nitrogen soluble in
20. (b) H 3 C – NH 2 + CHCl3 + 3KOH ¾
¾®
KOH solution. The copper will convert them into N 2

H 3C – N = C + 3KCl + 3 H 2 O 2NO + 2Cu ¾


¾® 2CuO + N 2
Isocyanides have foul odour.
2 NO 2 + 4Cu ¾ ¾® 4CuO + N 2 etc.
21. (c) Let the vol. of CH 4 be = a ml Halogens will be removed as AgX. In case of sulphur
\ Vol. of C 2 H 4 = (10 – a) ml SO 2 formed will be removed as PbSO 4 .
Equation for combustion of CH 4 and C 2 H 4 31. (b) K 2SO 4 raises bpt. and CuSO 4 acts as catalyst.

CH 4 + 2 O 2 ¾
¾® CO 2 + 2 H 2 O (liquid) 62 wt. of Mg 2 P2 O 7
32. (b) Percentage of P = × × 100
a ml 2a ml a ml negligible volume 222 wt. of compound

C2H 4 + 3 O2 ¾ ¾® 2 CO 2 + 2 H 2O 62 1.332
= × × 100 = 13.33%
(10 – a) ml 3(10 – a) ml 2(10 – a) ml negligible 222 2.79
12B
Classification and
Nomenclature of Organic
Compounds
CLASSIFICATION OF ORGANIC COMPOUNDS (iii) Homocyclic : The ring system is made up of one type of
The ability of carbon to combine with large number of elements atoms generally carbon
especially O, N, S, X etc, to undergo catenation to form chains of (a) Alicyclic : The cyclic compounds resembling open chain
varying lengths and shapes and existance of isomers has led to aliphatic compounds.
the formation of more than five million organic compounds. These For example: Cycloalkanes
have been classified into the following main groups (b) Aromatic : The benzene, napthalene and their derivatives
etc are homocyclic aromatic compounds
Organic
Compounds
NH2

Acyclic, Open Cyclic, Closed chain


chain or Aliphatic or Ring Compounds

Homocyclic
Benzene Aniline
Heterocyclic

Alicyclic Aromatic Alicyclic Aromatic OH COOH

(i) Acyclic or open chain compounds : For example


Phenol Benzoic acid

OH
n-Butane Isobutane n-Butanol

Napthalene Anthracene

2, 4-Dimethyl pent-1-ene

(ii) Cyclic or closed chain compounds.: For example


O Phenanthrene

(iv) Heterocyclic : The ring system is made up of two or more


Cyclobutane than two types of atoms. They may be
Cyclohexane Cyclopentanone
358 Chemistry
(a) Alicyclic : eg. HOMOLOGOUS SERIES
A group of a particular class of compounds where a preceeding or
succeeding member differ by one–CH2. The members of the series
••
N are known as homologues. The homologues
O
•• H (i) have the same general formula CnH2n+2 or CnH2n+1 X
Tetrahydrofuran Pyrrolidine (ii) molecular weight differing by 14 of two successive members
O (iii) can be prepared by general methods of preparation
O O (iv) have almost similar chemical properties
N (v) show regular gradation in physical properties such as mpt,
H Oxirane
1,4-Dioxane bpt, density etc
Piperidine or Epoxyethane
(b) Aromatic : eg. NOMENCLATURE
The most widely accepted and the latest system of naming organic
compounds is IUPAC (International Union of Pure and Applied
O N S
Furan H Chemists) system, according to which the name essentially
Thiophene
Pyrrole consists of three parts.
(i) Word root : It indicates the nature of the basic carbon
skeleton. From C1 to C4 common names have been retained
N and from C5 upwards Greek number roots have been used
Pyridine N
Chain length Word root Chain length Word root
Quinoline
C1 Meth- C7 Hept-
CLASSIFICATION BASED ON FUNCTIONAL GROUPS
On the basis of functional groups which confer characteristic C2 Eth- C8 Oct-
properties on them, the organic compounds have been classified C3 Prop- C9 Non-
as follows C4 But- C10 Dec-
Class Functional group C5 Pent- C11 Undec-
Halides X (Cl, Br, I) Halo C6 Hex- C12 Dodec
The generic word root for any carbon chain is “alk”.
O
|| (ii) Suffix : These are of two types
Esters — C— O — R
(a) Primary suffix : It is added to the word root to designate
Olefins Alkenes >C = C< saturation or unsaturation in a carbon chain
O Type of Primary Generic name
||
Acid halides — C— X Carbon chain Suffix
Acetylenes / Alkynes —C º C — Saturated - ane Alkane
O O Unsaturated with - ene Alkene
|| || one C=C
Anhydrides — C— O — C— Unsaturated with - yne Alkyne
Alcohols —OH (Hydroxy) one C º C
Amines —NH2 (b) Secondary suffix : It is added to indicate the functional
group present in the compound. The terminal ‘e’ is
O
Aldehydes || dropped, if secondary suffix begins with a vowel (a, e, i,
— C— H o, u, y) but it is retained if the secondary suffix begins
with a consonant.
O
||
Ketones C — C— C Functional Secondary Generic name
Sulphonic acid —SO3H group suffix

O – OH – ol Alkanol
|| –CHO – al Alkanal
Acids — C— O — H
>C = O – one Alkanone
O – COOH – oic acid Alkanoic acid
||
Amides –COX – oyl halide Alkanoyl halide
— C — NH 2
– CONH2 – amide Alkanamide
Classification and Nomenclature of Organic Compounds 359
– COOR – alkyl -- – oate Alkyl alkanoate – NO2 Nitro
– (CO)2O – oic anhydride Alkanoic anhydride – NO Nitroso
– CN – nitrile Alkane nitrile + ..
-Nº N Diazo
– SH – thiol Alkanethiol
– NH2 – amine Alkanamine
æR = ö
(iii) Prefix : They are of two types – OR ç CH3 , C2 H5 , C3 H 7 , etc. ÷ Alkoxy
è ø
(a) Primary prefix : It is for cyclic nature of the compound
and primary prefix cyclo is used immediately before the –R (CH3, C2H5, C3H7, etc) Alkyl
word root. eg.: Thus the complete IUPAC name of an organic
compound consists of the following parts
Sec. prefix Prim. prefix
word root Prim. suffix
Sec. suffix
Primary Word Prim. Sec. IUPAC
prefix root suffix suffix name 2 3
Cyclo hex ane – Cyclohexane For example: HO 4 Br
1
(b) Secondary prefix : The certain atoms and groups which 6 5
are not considered as functional groups but are treated 4-Bromo-cyclo-Hex-2-ene-1-ol
as substituents are called secondary prefixes. They are
IUPAC name is 4-Bromocyclohex-2-ene-1-ol or 4-
added before the word root in case of acyclic compounds
Bromo-2-Cyclohexenol
and before the primary prefix in case of cyclic compounds
in alphabetical order. word root = hex
The important secondary prefixes are Prime suffex = ene
Sec. suffex = ol
Substituent Sec. prefix
Prime prefex = Cyclo
– X (F, Cl, Br, I) Halo Sec. prefex = 4bromo

ALKYL GROUPS :
Univalent groups formed by the removal of one hydrogen atom from an alkane are known as alkyl groups or alphyl groups. Their
names are obtained by changing the suffix –ane of parent hydrocarbon by –yl.
Alkane Group Short-hand notation IUPAC name
Methane Methyl CH3– Me Methyl
Ethane ethyl C2H5– Et Ethyl
Propane n-propyl CH3CH2CH2– n-Pr, Pra or Pr 1-propyl

|
Iso propyl CH 3 – C H – CH 3 Iso- Pr , Prb , Pri 2-propyl

Butane n-butyl CH3CH2–CH2–CH2– –n-Bu, Bu a or Bu 1-butyl


|
s-butyl CH 3CH 2 – C H – CH 3 s-Bu, Bu b or BuS 1-methyl propyl
Iso-butyl (CH3)2CH–CH2– Iso-Bu, or Bui 2-methyl propyl
t-butyl (CH3)3C– t-Bu, But 1,1-dimethyl ethyl
NOMENCLATURE OF SIMPLE ALIPHATIC COMPOUNDS
Class Formula General Common IUPAC group IUPAC
formula name name
1. Alkanes CH4 CnH2n+2 Methane Alkane Methane
or paraffins C2H6 Ethane Ethane
2. Alkenes C2H4 CnH2n Ethylene Alkene Ethene
or Olefins C3H6 Propyline Propene
360 Chemistry
3. Alkynes C2H2 CnH2n–2 Acetylene Alkyne Ethyne
or Acetylenes C3H4 Methyl acetylene Propyne
4. Alkyl CH3Cl R–X Methyl chloride Haloalkanes Chloromethane
halides C2H5Cl Ethyl chloride Chloroethane
5. Dihalogen CH2Cl CnH2nX2 Ethylene 1,2-dichloroethane
derivatives (Alkylene halide) chloride
CH2Cl
(vicinal)

CHCl 2
Ethylidine Chloride 1,1-dichloroethane
CH 3
Geminal
6. Trihalogen CHCl3 CnH2n–1X3 Chloroform 1,1,1-trichloromethane
derivatives
7. Tetra halogen CCl4 CnH2n–2X4 Carbon tetra 1,1,1,1-tetra-
derivatives chloride chloromethane
8. Alcohols CH3OH CnH2n+1OH Methyl Alkanol Methanol
(Monohydric) alcohol

Dihydric CH 2OH CnH2n(OH)2 Ethylene glycol Ethane-1, 2-diol


CH 2OH
Trihydric CH2OH CnH2n–1(OH)3 Glycerol Propane-1,2,3-triol
CHOH
CH2OH
9. Monocarboxylic HCOOH R–COOH Formic Acid Alkanoic acid Methanoic acid
Acids or Fatty CH3COOH Acetic acid Ethanoic acid
acids C3H7.COOH Butyric acid Butanoic acid
C4H9.COOH Valeric acid Pentanoic acid
COOH
Dicarboxylic CH2 CnH2n(COOH)2 Malonic acid Propane-1, 3 dioic
COOH
acids acid

CH2COOH Succinic acid Butane-1, 4-dioic


CH2COOH acid

CH2COOH Glutaric acid Pentane-1,5-dioic


CH2
CH2COOH
acid

C H 2 .CH 2 COOH Adipic acid Hexane-1,6-dioic acid


|
CH 2CH 2COOH

C OOH oxalic acid Ethane-1,2-dioic acid


|
COOH
10. Ethers CH3OC2H5 R–O–R' Ethyl methyl Alkoxy alkane Methoxyethane
ether
CH3OCH2–CH2–CH3 Methyl Propyl 1-methoxypropane
ether
2 3 2 3
Epoxides CH2 CH2 RCH CH–R Alkene Oxides Oxirane or Oxirane or
O O Epoxyalkane (Ethylene Oxide)
Classification and Nomenclature of Organic Compounds 361

Propylene Oxide 2-methyl Oxirane


1,2-Epoxypropane

11. Acid chlorides CH3COCl R–COCl Acetyl chloride Alkanoyl halide Ethanoyl chloride
O
12. Esters CH3 – C – OC2H5 RCOOR' Ethyl acetate Alkyl alkanoate Ethyl ethanoate
(here alkyl
O O attached to oxygen)

13. Anhydrides C2H5–C–O–C–C2H5 (RCO)2O Propionic Alkanoic


Propanoic anhydride anhydride
anhydride
O
14. Amides CH3–C–NH2 RCONH2 Acetamide Alkanamide Ethanamide
15. Amines
(a) 1°Amine CH3NH2 RNH2 Methyl amine Alkanamine Methanamine

(b) 2°Amine (CH3)2NH RNHR Dimethyl amine N-alkylalkan-amine N-methyl-


methanamine
(c) 3°Amine (CH3)3N R - N- R Trimethyl amine N,N-dialkyl N,N-dimethyl
|
R alkanamine methanamine

16. Aldehydes CH3CHO RCHO Acetaldehyde Alkanal Ethanal


CH3–CH2CHO Propional- ' Propanal
dehyde
CHO Glyoxal Ethan-1, 2-dial
CHO

O O
||
17. Ketones CH3–C–CH3 R – C– R Acetone Alkanone Propan-2-one
O
CH3–C–CH2–CH3 Ethylmethyl Butan-2-one
ketone
18. Cyanides HC º N R–CN Hydrogen cyanide Alkanenitrile Methanenitrile
CH3C º N Methyl cyanide Ethanenitrile
CH2 – CN Butan-1, 4 dinitrile
CH2 – CN

Propanl-1, 2,
CH2 – CN
3-tricarbonitrile
CH – CN
CH2 – CN
362 Chemistry
LINE ANGLE FORMULA (iv) If two or more of the same alkyl groups are present, use the
Bonds are represented by lines, carbon atoms are assumed to be prefixes di, tri etc to avoid repetition
present at the start and finish of a line. Nitrogen, oxygen and 5 1
halogens are labelled, but hydrogens are only shown when 3
bonded to a drawn atom. Each atom is assumed to have sufficient 4 2
hydrogen atoms around it to make it neutral. For example: 3, 3-Dimethylpentane
6 4 2 (v) Alphabetical order : The side chains are cited in alphabetical
n-Hexane CH3(CH2)4CH3 order
7 5 1
5 3 1
4 3

CH2 O 6 2
6 O
CH2 C 5
1 4-Ethyl-3, 3-dimethylheptane
2-Cyclohexenone CH2 CH 4 2
(vi) Longest chain with maximum number of side chains : If two
CH 3
or more chains of the same length are possible, choose the
one with maximum number of side chains.
But-1, 3-diene CH 2 = CH – CH = CH 2
2 4 6
1
3
3
1 5
4 2

CH3 OH
| |
3-methyl but-2-ol CH 3 – C H – CH – CH 3
(vii) Locant sum : Sum of the locants must be minimum. But out
of two sets of the sum of the locants, the set having the
3 1 lowest number when compared by term is preferred. For
4 2 example out of (2+6+7=15) and (3+4+7=14), the first set is
OH correct.
4
NOMENCLATURE OF COMPLEX HYDROCARBONS 6
3 2
The following rules are followed
5 1
(i) Longest chain rule : The longest continuous chain of carbon
atoms is picked up which forms the base name of the 2,3, 5-Trimethylhexane (not 2,4,5-Trimethylhexane)
compound. (viii) The name of a complex radical is considered to begin with
(ii) Numbering : The longest chain is numbered by arabic the first letter of its complete name
numerals beginning with the end nearest a substituent.
i.e. including the numerical affix (di, tri, tetra etc are numerical
(iii) If two or more side chains are in equivalent positions, then affix) for alphabetical order.
the one cited first in the name is assigned the lower number.
12 10 8 6 4 2
11 9 7
8 6 4 2 13 3 1 5 3 1
5
5 2
7 3 1 4

4-Ethyl-5-methyloctane 7-(1,2-Dimethyl pentyl)-5-ethyl tridecane


(not 4-Methyl-5-ethyloctane)
(ix) When the side chains have the identical name the priority is
given to side chain having lowest locant

8 6 2 2 4
4
4 3 2 1
5
1 3
7 3 1
12 10 8 6 4 2
4-isopropyl-5-propyl octane 13 11 9 7 5 3 1
(not 5-Isopropyl-4-propyloctane) 6-(1-Methylbutyl)-8-(2-methyl butyl) tridecane
(not 6-(2-Methylbutyl)-8-(1-Methylbutyl) tridecane)
Classification and Nomenclature of Organic Compounds 363
(x) The numerical prefixes bis, tris tetrakis are used to indicate NOMENCLATURE OF CYCLOALKANES : (Alicyclic
the multiplicity of substituted substituent Compounds)
(i) The base name is decided by the number of carbon atoms
2 4 6 which the cyclic or acyclic portion contains. If the ring
3 5 contains more or equal number of carbon atoms as alkyl then
1 7
it is regarded as derivative of cycloalkane
2-Methyl-4-bis(1-methylethyl)heptane
Nomenclature of complex alkenes and alkynes : (ii) Carbons are numbered to give lowest numbers to substituted
(i) Selection of longest chain containing maximum number of carbons. For example
double or triple bonds (sometimes longest chain rule is
violated) 2
CH3
3
1

4 6
5
3-n-propylhexa-1,5-diene methyl cyclo pentane
It contains longest chain of 7C atoms, but both the double
bonds are not included. Hence longest chain of 6C catoms CH3 CH3
is picked up)
(ii) If both, the double and triple bonds, are present the — CH — CH — CH3
compound is regarded as derivative of alkyne. In such cases 1 2 3
the terminal 'e' of -ene is dropped if it is followed by suffix (1, 2-dimethyl propyl) cyclohexane
starting with a,i,o,u,y. For example :
(iii) When there are more acyclic than cyclic carbons the cyclic
6 4 2
part becomes cycloalkyl substituent
7 5 3 1
Hept-4-en-1-yne 2 6 8
(not Hept-4-ene-1-yne) 4
3
(iii) If double and triple bonds are at equidistance from either 1 5 7
side, the preference is given to double bond. 4-Cyclopropyl-3-methyloctane
2 4 6 8 (iv) When acyclic portion contains a multiple bond or a functional
1 3
group, the cyclic portion is treated as substituent.
5 7
oct-1-en-7-yne
(not oct-7-en-1-yne)
O

(iv) If the compound contains two or more double or triple bonds CH = CH – C – CH3
4 3 2 1
a terminal "a" is added to the word root.
2 4 6 2 4 6
4-Cyclohexyl but-3-en-2-one
1 3 5 1 3
5 7
Hexa-2,4-diene Hepta-2,5-diyne
(v) In case when both contain the same functional group, the
(not Hex-2,4-diene) (not Hept-2,5-diyne) base name is decided by the number of c-atoms.
(v) The terminal 'a' is not added to the word root when the OH
complete primary suffix do not start with a numerical affix 1 3
1 2
2 4 6 2 4 6 6
2
5 3 OH
1 3 5 7 1 3
5 7 4
Hepta-1,6-dien-3-yne Hept-3-ene-1,6-diyne 2-(2-Hydroxy propyl) cyclohexan-1-ol
(Note that di, tri, tetra.. are numerical affix) (vi) When both contain the different functional groups, the base
(vi) Side chains containing multiple bonds are named as follows name is decided by principal characteric group
CH2= CH2=CH–CH2– CH3–CH= CH2=CH–
Allyl Ethylidene Vinyl 5 6
3 2 1
HO 4 CH2–CH2–COOH
2 6 2 6 1
8
4 4 2
3
Examples : 1 3 7 5 1 3 5 7 3-(4-Hydroxyclohexyl)propanoic acid
4-Vinylocta-1,7-diene 4-Methylidenhepta-1,6-diene
364 Chemistry
(vii) When the acylic ring is directly attached to benzene ring, it Numbering of C-atoms in fused rings and bridge rings : The
is named as derivative of benzene numbering starts from bridge head carbon, proceeds along the
longest bridge passing through the second bridge head atom,
proceeds to the next longest bridge and completed along the
shortest path.
Numbering of C-atoms in spiro compounds : The numbering
Cyclohexyl benzene
(viii)Presence of certain groups starts from the carbon atom, next to spiro atom, present in the
Functional gp. Suffix smaller ring giving minimum number to atoms containing functional
–COOH Carboxylic acid groups.
–COOR Alkyl carboxylate (i) Fused Rings : Fused rings share two adjacent carbon atoms
and the bond between them eg. :
–COX Carbonyl halide
–CONH2 Carboxamide
-C º N Carbonitrile
–CHO Carbaldehyde Bicyclo [4.4.0]decane
(ii) Bridged Rings : These share two non adjacent carbon atoms
(the bridgehead carbons) and one or more carbon atoms
between them 7
1
6
2

5 4
3
CONH 2
CHO Bicyclo [2.2.1]heptane
6 1
2
CH 3 More examples of bridge Compounds
2
5 3 1
OH 3
4 6
2-Methylcyclohexane-1-carboxamide 3-hydroxy cyclohexane carbaldehyde
C N COCl 7
4
5
CH 3
2, 6, 6-Trimethyl bicyclo[3.1.1]hept-2-ene
1
7
2-Methyl cyclohexane carbonitrile cyclohexane-1-carbonyl chloride 1
2 4 2
NOMENCLATURE OF POLYCYCLIC ALKANES 6
There are three ways that rings can be joined. 3 4
3 5
Bicyclo [1.1.0] butane Bicyclo[2.2.1]hept-2-ene
(i) Fused rings (iii) Spirocyclic Compounds : The two rings share one carbon
atom 1 5 6
4
(ii) Bridged rings 2 7

3 8
Spiro [4.3] octane
(iii) Spirocyclic compounds
More examples of spiro Compounds
The carbon atoms common to both the rings are called bridge
7 6 1
head atoms. The chain of carbon atoms connecting the bridge 2 Br
5
head atoms is called a bridge.
3
8 9 4
2-Bromo spiro[4.4] nonane

7 6 1
2 CH3
1 5
5 6 8
4 3
2 7 10
9 4
3 9
spiro atom
8 2-methyl spiro[4.5]deca-1, 6-diene
Classification and Nomenclature of Organic Compounds 365
Nomenclature of compounds containing identical cyclic units (i) Presence of only two similar terminal groups : The carbon
joined by a single bond atoms of such groups are included in the principal chain. For
No. of cyclic hydrocarbon units Two Three Four example
Prefix bi ter quarter
Numbering of C-atoms : The numbering starts from the
C-atom joining the rings.
2 1 1' 2'
2 2'
1 1'
O
4
O
3 4' 3' 3 3'
1,1'-Bicyclopropane 3
1,1'-Bicyclobutane
1 5
H 5C 2O OC2H5
4' 3' 2 4
Diethyl pentanedioate
2 1 1'' 2''
1' 2' O O
3 4 4'' 3'' 3
1,1',2',1''-Tercyclobutane 1 5
H 2N NH2
2 4
3 2 2' 3' Pentanediamide
1 1'
4 4'
(ii) Presence of more than two similar terminal groups attached
5 6 6' 5' to the main principal chain : In this case special suffixes are
1,1'-Biphenyl
used and carbons of terminal groups are not counted in the
principal chain
Functional groups Suffix
–COOH - Carboxylic acid
–CHO - Carbaldehyde
–COX - Carbonylhalide
–CONH2 - Carboxamide
NOMENCLATURE OF COMPOUNDS CONTAINING
TERMINATING FUNCTIONAL GROUPS –COOR - Alkyl carboxylate
–CN - Carbonitrile
If only one group such as –COOH, –CHO, –COOR, –CONH2, –
COCl or – C º N is present in the molecule it is always given HOOC 2 4
COOH
3
number 1 and 1 is never writen when there is no ambiguity.
1
COOH 5
5 3 Pentane-1,3,5-tricarboxylic acid
3 5
4 2 2
6 4 NC 2 4
1 CN
O 1 OH 3
OH 2-Ethylpent-2-enoic acid 1
O CN
5
Pentane-1,3,5-tricarbonitrile
2-Ethylhaxenoic acid
Presence of more than two similar terminal groups not
NH2 directly attached to the principal chain : In such case the
2 longest chain with two similar terminal groups is selected
1
3 and carbons of groups are counted in the principal chain.
O
HOOC 1 3
7
5
2-Methylpropanamide COOH 4
2 6
4 2 CH2COOH
4 2 OMe 5
1 4-(Carboxymethyl)heptanedioic acid
3 1 3 CHO
Pent-2-enal CH2COOH
5 (terminal e of ene is removed
HOOC 3 5
O as it comes before a) COOH
1 4
Methylpentanoate 2 CH2COOH

Nomenclature of compounds containing two or more than two 3,3-Bis(Carboxymethyl)


pentanedioic acid
similar terminal groups
366 Chemistry
NOMENCLATURE OF COMPOUNDS Naming of substituted substituents : In this case the subsidiary
containing substituents (not regarded principal functional groups) substituents are named as prefixes. For example

Br –CH 2 - CHO = CH - CH3


2 4 6
4
Formyl methyl Ethylidene
3
2 5
3 1
1 º C - CH 2 - CH3 -CH 2 - COOH
Cl NO2
Propylidyne Carboxymethyl
2-Bromo-3-Chlorobutane 3- Nitromethylhexane
(follow alphabetical order) -CH 2 Cl -CH2 - C º N
Chloromethyl Cyanomethyl
NOMENCLATURE OF COMPOUNDS CONTAINING
MORE THAN ONE TYPE OF FUNCTIONAL GROUPS -CH 2 - CH 2 - CO - NH 2
In such case the compound is regarded as derivative of senior Carbomoylethyl
funtional group and the other functional groups are regarded as Examples :
substituents. The numbering of the parent chain is done in such OH Cl
a way so that the functional group of highest priority gets the 1. CH3–CH2–CH–CH2–CH–CHO
lower number and the chain contains the maximum number of 6 5 4 3 2 1
functional groups. 2-Chloro-4-hydroxyhexanal
The seniority of functional groups ( highest prirority) follow the
OH O
following order
2. CH3–CH–CH2–C–CH3 4-hydroxypentan-2-one
5 4 3 2 1
Group Prefix name 2º Suffix name
– SO3H Sulpho Sulphonic acid OH O
– COOH Carboxy Oic acid 3. CH3–C–CH2–C–H 3-hydroxy-3-methylbutan-1-al
4 3 2 1
– COOR alkoxy carbonyl Oate CH3
– COX Halo carbonyl/Haloformyl –oyl halide
3 2 1
– CONH2 Carbamoyl amide 4. CH2–CH–CH2OH 2,3-epoxypropan-1-ol
– CHO Aldo or formyl al O
– CN Cyano nitrile 1 2 3
– NC Isocyano Isonitrile 5. CH2–CH–CH2Cl 3-Chloro-1, 2-epoxypropane
>C = O Keto or oxo one O
– OH Hydroxy ol CH3
4 3 2 1
– SH Mercapto thiol 6. CH3–CH–C–COOH 3-methyl-2-oxobut-1-oic acid
– NH2 Amino Amine O
– OR Alkoxy – 3 2 1
7. NC–CH2–CH2–COOH 3-cyanoprop-1-oic acid
Epoxy –
OH O
>C=C< – ene 8. CH3–CH–CH2–C–OCH2CH3
4 3 2 1

–C º C – – yne Ethyl-3-hydroxybutanoate
–N=N Azo – CH3
6 4 3 2 1
– NO2 Nitro – 9.
5
HO–CH2–C–CH=CBr–CH2–CHO
– NO Nitroso –
– X (Cl, Br, I) Halo (Cl, Br, I)
The terminal e of the primary suffix is replaced by the suffix
name of functional group.
3-Bromo-5-cyclopentyl-6-hydroxy-5-
Alphabetical order for substituents : These should be placed in methylhex-3-en-1-al
alphabetical order.
Classification and Nomenclature of Organic Compounds 367

OH OH Aromatic hydrocarbons (Arenes).


(i) Containing one ring:
10. HO–H2C– –CH–CH–CH3 CH3
1 2 3
1-(p-hydroxymethylphenyl)prop-1, 2-diol

O OEt Benzene Methyl benzne


11. (Toluene)
2 OH C2H5
4 1
5 3
O

Benzene
4-Carbethoxypent-3-en-1-oic acid Methyl benzne Ethyl benzne

O NH2
CH3 CH3
1 CH3
12. Br 4
2
3
5
CH3
O
1, 2-dimethylbenzene 1, 3-dimethylbenzene
2-(Bromomethyl)-3-oxopentanamide (o-xylene) (m-xylene)
13. MeO 2 OEt CH3
3 1

1-ethoxy-3-methoxypropane CH
3
2 4 1, 3-dimethylbenzene CH3
14. 1, 4-dimethylbenzene
1 5
6 (p.xylene)
CH3 H3 C – CH – CH3

2-Ethenyl-3-methylcyclohexa-1, 3-diene
3 OH
4 H3C CH3
2
15. 1, 3, 5-trimethylbenzene 2-propylbenzene
5 1
6 (mesitylene) (cummene)
H – – CH CH = CH2
4-ethylhex-4-en-2-ol

NOMENCLATURE OF AROMATIC COMPOUNDS


Generally Benzene and its derivatives are known as aromatic
ethenylbenzene
compounds. They are of two types (styrene, vinyl benzene)
(i) Nuclear substituted : The functional group is directly
attached to the benzene nucleus e.g. phenol, toluene, (ii) Containing more than one ring
chlorobenzene etc.
(ii) Side chain substituted : The functional group is present in
the side chain e.g. Benzyl alcohol, Benzylamine etc.
In the first case the compounds are named as derivatives of
benzene and in the second case as derivatives of aliphatic
Naphthalene Anthracene
compounds (except arenes).
The IUPAC name of benzene is cyclohex-1,3,
5-triene, but now aromatic compounds have their popular
common name adopted by IUPAC. In IUPAC system the
position of functional groups are indicated by arabic
numerals i.e. 1, 2, 3 instead ofNaphthalene
o, m and p. Phenanthrene
368 Chemistry
(iii) Aryl groups:
OH OH

CH2– CH OH

OH
1, 2-dihydroxy benzene 1, 3-dihydroxybenzene
(Catechol) (Resorinol)
Phenyl Benzyl Benzal
OH

C CH3 CH3 CH3


OH HO OH
1, 3, 5-trihydroxybenzne
(Phloroglucinol)
Benzo 2-Tolyl or 3-Tolyl (m-Tolyl) 4-Tolyl (p-Tolyl)
o-Tolyl (b) Side chain substituted hydroxy derivatives are known
as alcohols
(iv) Halogen dervivatives:

Cl Cl CH2OH CH2 – CH2 – CH2OH


Cl

Chlorobenzene 1, 2-dichlorobenzene Phenylmethanol 3-phenyl prop-1-ol


(o-dichlorobenzene) (Benzyl alcohol)
Cl
3 2 1
CH2Cl CH2 – CH2 – CH2 (vi) Aromatic ethers:
Cl
OCH3 OC2H5

orobenzene phenylchloro methane 1-chloro-3-phenyl propane


(Benzyl chloride) (g-phenylpropyl chloride)
(v) Phenols- Methoxybenzene Ethoxybenzene
(Anisole) (phenetole)
(a) Nuclear substituted hydroxy derivatives are known as
phenols
(vii) Amines:
CH3
OH NH2 NH2
CH3

Phenol 2-methylphenol
(o-cresol) Benzenamine 2-methylbenzenamine
(Aniline) (o-Toluidine)

NH2
CH3 CH3
NH2
OH

OH Benzenamine 2-methylbenzenamine Benzene-1, 2-diamine


3-methylphenol OH (o-Phenylenediamine)
(m-cresol) 4-methylphenol
(p-cresol)
Classification and Nomenclature of Organic Compounds 369
(viii) Nitro compounds: (xi) Acids:

NO2 OH COOH COOH


O2N NO2 CH3

Nitro benzene NO2 Benzoic acid 2-methylbenzoic acid 2-Hydroxybenzoic acid


2, 4, 6-trinitrophenol (o-Toluic acid)
(Picric acid)
COOH
OH COOH
CH3
O2N NO2 COOH
2-methylbenzoic acid 2-Hydroxybenzoic acid Benzene-1, 2-dicarboxylic acid
(Salicylic acid) (Phthalic acid)
NO2
(xii)Acid derivatives:
6-trinitrophenol 2, 4, 6-trinitrotoluene
(T.N.T.) CONH2 COCl

(ix) Aldehydes:

CHO CHO Benzamide Benzoyl chloride


OH
COOCH3 O O
C–O–C
Benzaldehyde 2-hydroxybenzaldehyde
(Salicyaldehyde)
Methyl benzoate Benzoic anhydride
(x) Ketones: (xiii)Sulphonic acids:
COCH3 SO3H
CO

Methyl phenyl Ketone Diphenyl Ketone Benzene sulphonic acid


(Acetophenone) (Benzophenone)
370 Chemistry

Very Short/ Short Answer Questions Long Answer Questions


1. Write the metamer of diethyl ether. What is its IUPAC name? 11. Name each of the following :
2. What is the functional group of (i) an aldehyde and (ii) a
nitro compound?
3. What primary and secondary suffixes are as applied to
IUPAC nomenclature? (i) (ii)
4. Give the IUPAC name of the compound: I
CH2 = CH – CH(CH3)2
5. Give the IUPAC names of the following compound:
CH3—CH—C—CH—CH3



Br O CH3 (iii) (iv)

6. Write the IUPAC name of


CH3—CH—CH2—CH—COOH

(v) Br OH
CHO CH3
7. Write IUPAC names for 12. Give IUPAC names of
(i) CH3 – CH = C(NO2) CH2CH3 CH3 O
CH3 +
Cl N -
| OH O
(ii) H 2 C = CHCH - CHCH - CHCHMe2 (a) Cl (b)
|
CH(Me)CH2CH3
CH3
HO
CH3

(iii) (c)
C2H5
OH
(iv) (H2C = CH)2 CHCH2CH = CHCH3
8. Write the condensed structural formula of
Multiple Choice Questions
(a) 4-ethyl-3-methyl heptane 13. Correct IUPAC name of
(b) 3, 3-dimethyl octane H3C - CH - CH - CH 3 is _________ .
| |
9. Write the structural formula of C2 H5 C2 H5
(i) o-methoxyethylbenzene
(a) 2 –ethyl-3-methylpentane
(ii) 2,3-dibromo-1-phenylpentane
(b) 3,4-dimethylhexane
(iii) 4-ethyl-1-fluoro-2-nitrobenzene
(c) 2-sec-butylbutane
10. Name the compounds whose line formulae are given below:
(d) 2, 3-dimethylbutane
O NO2 14. Which of the following is the correct IUPAC name?
(a) 3-Ethyl-4, 4-dimethylheptane
(i) (ii) (b) 4,4-Dimethyl-3-ethylheptane
(c) 5-Ethyl-4, 4-dimethylheptane
(d) 4, 4-Bis(methyl)-3-ethylheptane
Classification and Nomenclature of Organic Compounds 371
15. The IUPAC name for (a) 3, 3-dimethyl - 1- cyclohexanol
(b) 1, 1-dimethyl-3-hydroxy cyclohexane
O O
|| || (c) 3, 3-dimethyl-1-hydroxy cyclohexane
CH3 — C — CH 2 — CH 2 — C — OH
(d) 1, 1-dimethyl-3-cyclohexanol
is ___. 19. The IUPAC name of the compound shown below is :
(a) 1-hydroxypentane-1, 4-dione
Cl
(b) 1,4-dioxopentanol
(c) 1-carboxybutan-3-one
(d) 4-oxopentanoic acid Br
16. The IUPAC name for (a) 3-bromo-1-chlorocyclohexene
(b) 1-bromo-3-chlorocyclohexene
Cl
(c) 2-bromo-6-chlorocyclohex-1-ene
NO2
(d) 6-bromo-2-chlorocyclohexene
20. Name of the following compound is :

CH3 CH 3CH 2 CH 3
C
(a) 1-Chloro-2-nitro-4-methyl benzene CH 3 CH 2 OH
(b) 1-Chloro-4-methyl-2-nitrobenzene (a) 2-ethylbutan-2-ol
(c) 2-Chloro-1-nitro-5-methyl benzene (b) 1-ethyl-1-methylpropan-1-ol
(d) m-Nitro-p-chlorotoluene (c) 3-methyl pentan-3-ol
17. Which of the following compounds has wrong IUPAC name? (d) diethylethanol
(a) CH3–CH2–CH2 –COO–CH2CH3 ® ethyl butanoate 21. The IUPAC name for

(b) CH3 - CH - CH 2 - CHO ® 3-methyl-butanal C1


| |
CH3 CH3 - C - CH 2 - CH = CH - CH3 is
|
H
(c) CH3 - CH - CH - CH ® 2-methyl-3-butanol
3 (a) 5–chlorohex–2–ene
| |
OH CH3 (b) 2–chlorohex–5–ene
(c) 1–chloro–1–methylpent–3–ene
O (d) 5–chloro–5–methylpent–2–ene
||
(d) CH3 - CH - C - CH 2 - CH3 ® 2-methyl-3-pentanone 22. The correct name for the following hydrocarbon is
|
CH3

18. The IUPAC name of the compound is


(a) Tricyclo [4.1.0] heptane (b) Bicyclo [5.2.1] heptane
(c) Bicyclo [4.1.0] heptane (d) Bicyclo [4.1.0] hextane
HO
372 Chemistry

1. The kind of valency that exists in CaH2 & C2H2 is 8. The IUPAC name of this compound
(a) electrovalency in CaH2 and covalency in C2H2 CH 3 - C H - CH 2CH 2CH 3 is
(b) electrovalency in both |
CH(CH3 ) 2
(c) covalency in CaH2 and electrovalency in C2H2
(a) 2- isopropylpentane (b) 2, 3 -dimethylhexane
(d) covalency in both
(c) Isononane (d) 2, 4- dimethylhexane
2. The compound which has one isopropyl group is
9. The IUPAC name of the compound
(a) 2, 2, 3, 3 - Tetramethylpentane
CH 3 - CH - CH2- CH 2 - Cl
(b) 2, 2 - Dimethylpentane |
(c) 2, 2, 3- Trimethylpentane CH 3
(d) 2- Methypentane (a) 1-chloro -3- methylbutane
3. Which of the following statements is false for isopentane ? (b) 2- methyl-4- chlorobutane
(a) It has three CH3 groups (c) 2-methyl-4-chlorobutane
(b) It has one CH2 group (d) 1-chloropentane
(c) It has one CH group 10. IUPAC name of the given compound will be
(d) It has a carbon which is not bonded to hydrogen (CH3)2C(CH2CH3)CH2CH(Cl)CH3
4. Which of the following IUPAC names is correct? (a) 5-chloro-3-, 3-dimethylhexane
(a) 2-Methyl-3-ethylpentane (b) 4-chloro-2-ethyl-2-methylpentane
(b) 3-Ethyl-2-methylpentane (c) 2-chloro-4-ethyl-4-methylpentane
(c) 2-Ethyl-3-methylpentane (d) 2-chloro-4, 4- dimethylhexane
(d) 3-Methyl-2-ethylpentane 11. The IUPAC name of (CH3)3 C – CH = CH2 is
5. The correct IUPAC name of the compound with molecular (a) 2 , 2- dimethylbut -3-ene
formula, (CH3)3 C – CH3 is (b) 2, 2- dimethylpent -4-ene
(a) Pentane (c) 3, 3- dimethylbut - 1- ene
(b) 1, 1, 1 -trimethylethane (d) Hex -1-ene
(c) 2, 2- dimethylpropane 12. The IUPAC name of the following
(d) Neopentane CH3C(CH3)2CH2CH=CH2 is
6. The IUPAC name of (a) 2, 2- dimethyl -4- pentene
CH 3 - CH - CH - CH 2 CH 2 - C H - CH 3 (b) 4, 4- dimethyl -1- pentene
| | | (c) 1, 1, 1- trimethyl -3- butene
CH 3 CH 3 CH 3
(d) 4, 4, 4- trimethyl -1- butene
(a) 2, 5, 6- trimethylhexane 13. The IUPAC name of the compound
(b) 2, 3, 6- trimethylheptane
(c) 2, 3, 6- trimethylhexane CH3 – CH – C – CH 2 CH 3
| ||
(d) 2, 5, 6- trimethylheptane CH 3 CH 2
7. The IUPAC name of
(a) 2- ethyl -3- methylbut -1- ene
H C 4H9 (b) 2- isopropylbut -1- ene
| |
CH 3CH 2 - C - C - CH 3 is (c) 2- methyl - 3 ethyl - 3- butene
| | (d) 2- (1- methylethyl) but -1- ene
CH 3 CH3
14. CH3CH2CH2CH(CH=CH2)CH2CH2CH3 is
(a) 3, 4, 4- trimethylheptane (a) 4-ethenylheptane
(b) 3, 4, 4- trimethyloctane (b) 3-n-propyl-1-hexene
(c) 2- butyl -2-methyl-3-ethylbutane (c) 4-ethenylhexane
(d) 2-ethyl-3, 3- dimethylheptane (d) 3-ethenylheptane
Classification and Nomenclature of Organic Compounds 373
15. IUPAC name of the following compound will be 24. IUPAC name of the compound
CH 3 - CH = C - CH 2 - CH 3
| CH 3 - C H - CH 2 - CHOH - CH 3 is
CH 2 -CH 2 -CH 3 |
CH 2 -CH 3
(a) 3-propyl-3-hexene (b) 3-propyl-2-hexene
(c) 3-ethyl-2-hexene (d) 4-ethyl-4-hexene (a) 4- methyl -3- hexanol (b) Heptanol
(c) 4- methyl - 2- hexanol (d) None of these
16. The IUPAC name of CH 3 - C = CHCH3 is
| 25. The correct nomenclature (IUPAC) for the following alcohol
C2 H 5 is
(a) 2-ethylbutene (b) 2-ethylbut-2-ene CH3CH2 CH3
(c) 3-methylpent-2-ene (d) 2-ethylpent-2-ene C
17. The IUPAC name of CH3–CºC–CH(CH3)2 is OH
CH3CH2
(a) 4- methyl -2-pentyne
(b) 4, 4 - dimethyl -2 butyne (a) 2-ethyl-2-butanol
(c) Isopropylmethylacetylene (b) 3-methyl-3-pentanol
(d) 2-Methyl-4-pentyne (c) 3-ethyl-3-methyl-3-pentanol
18. The IUPAC name of (d) 1, 1 - diethylethanol
26. The IUPAC name of
CH 3 - C= C - C H - CH 2 - C º CH is
| | |
Cl CH3 C2H5 H Br
| |
(a) 6-chloro-4-ethyl-5-methylhept-5-en-1-yne CH 3 - C - CH 2 - CH 2 - CH 2 - C - CH 3 is
| |
(b) 6-chloro-4-ethyl-5-methylhept-1-yn-5-ene OH Br
(c) 2-chloro-4-ethyl-3-methylhept-2-en-6-yne
(d) 2-chloro-4-ethyl-3-methylhept-6-yn-2-ene (a) 6, 6- dibromoheptan -2- ol
19. The IUPAC name of CH3OC2H5 is (b) 2, 2- dibromoheptan -6- ol
(a) Methyl ethyl ether (b) Ethyl methyl ether (c) 6, 6- dibromoheptan -2- al
(c) Methoxyethane (d) Ethoxymethane (d) None of these
20. Which of the following represents the correct IUPAC name 27. The IUPAC name of the compound
of the compound
CH2 = CH – CH2 Cl?
CH3
|
(a) Allyl chloride (b) 1- chloro -3- propene CH3 - CH - CH2 - C - CH3 is
(c) 3- chloro-1- propene (d) Vinyl chloride | |
OH OH
21. The name of ClCH 2 - C = C - CH 2 Cl according to
| | (a) 1, 1- dimethylbutane -1, 3-diol
Br Br (b) 1, 3. 3- trimethylpropane -1, 3-diol
IUPAC nomenclature system is (c) 2- methylpentane -2, 4- diol
(a) 2, 3- dibromo -1, 4- dichlorobutene-2 (d) 1, 3, 3- trimethyl -1, 3- propanediol
(b) 1, 4- dichloro-2, 3- dibromobutene-2 28. Choose the correct IUPAC name for
(c) Dichlorodibromobutene
(d) Dichlorodibromobutane CH3 - C H - CHO
|
22. IUPAC name of (C2H5)2CHCH2OH is CH2 -CH 3
(a) 2- ethylbutanol - 1 (b) 2- methylpentanol -1
(a) Butan - 2- aldehyde
(c) 2- ethylpentanol -1 (d) 3- ethylbutanol -1
(b) 2- methylbutanal
23. The IUPAC name of
(c) 3- methylisobutyraldehyde
OH
| (d) 2- ethylpropanal
CH 3 - C - CH 2 - C H - CH 3 is
| | 29. The IUPAC name of the compound having the molecular
CH 3 CH 3 formula Cl3C –CH2CHO is
(a) 2, 4- dimethylpentanol -2 (a) 3, 3, 3- trichloropropanal
(b) 2, 4- dimethylpentanol -4 (b) 1, 1, 1- trichloropropanal
(c) 2, 2- dimethylbutanol-2 (c) 2, 2, 2- trichloropropanal
(d) Butanol -2 (d) Chloral
374 Chemistry
30. The incorrect IUPAC name is 36. The IUPAC name of CH3CH=CHCOOC2H5 is
(a) CH 3 - C - C H - CH 3 (a) Ethyl but-1-enoate (b) Ethyl but-2-enoate
|| | (c) Ethyl prop-2-enoate (d) None of these
O CH3
37. The IUPAC name of the compound
2-methyl-3-butanone
CH 3CH = CHCH 2 C HCH 2 COOH is
(b) CH3 – C H – C H – CH 3 |
| | NH 2
CH3 CH 2 CH 3
(a) 5- amino - 2 - heptenoic acid
2, 3 -dimethylpentane (b) b - amino- d - heptanoic acid
(c) CH3– C º CCH(CH3)2 (c) 5- amino hex - 2- enecarboxylic acid
4-methyl-2-pentyne
(d) 3- amino -5- heptenoic acid
(d) CH3 - CH - CH - CH3 38. Indicate the wrongly named compound
| |
Cl Br (a) CH 3- C H - CH 2 - CH 2 - CHO
3-chloro-2-bromobutane |
CH 3
31. The systematic name of (CH3)2 CH – COOH is
(a) 2- propanoic acid (4-methyl -1- pentanal)
(b) isobutanoic acid (b) CH 3- CH - C º C - COOH
(c) 2- methylpropanoic acid |
CH 3
(d) 2- methylbutanoic acid
32. The IUPAC name of the formula, (4- methyl -2- pentyne -1- oic acid)

CH3 H (c) CH 3CH 2 CH 2- C H - COOH


| | |
CH 3 - C = C - COOH is CH 3

(a) 2-methyl-2-butenoic acid (2- methyl -1- pentanoic acid)


(b) 3-methyl-3-butenoic acid O
(c) 3-methyl-2-butenoic acid ||
(d) CH3 - CH2 - CH = CH - C- CH3
(d) 2-methyl-3-butenoic acid
33. IUPAC name of CH2 = CHCN is (3- hexen -5- one)
(a) Ethenenitrile (b) Vinyl cyanide 39. The IUPAC name of tert–butylchloride is
(c) Cyanoethene (d) 2-propenenitrile (a) 4-chlorobutane
34. The compound (b) 2-chlorobutane
(c) 1-chloro-3-methylpropane
is (d) 2-chloro-2-methylpropane
known by which of the following names ? 40. The IUPAC name of acraldehyde is
(a) Bicyclo [2.2.2] octane (a) Prop-2-en-1-al (b) Propenylaldehyde
(b) Bicyclo [2.2.1] octane (c) But-2-en-1-al (d) Propenal
(c) Bicyclo [1.2.1] octane 41. Vinylcarbinol is
(d) Bicyclo [1.1.1] octane (a) HO–CH2–CH=CH2
35. The IUPAC name of the compound, (b) CH3C(OH)=CH2
(c) CH3–CH=CH–OH
CH3
| (d) CH3–C(CH2OH)= CH2
CH3CH2 - CH - CH2COCl is 42. IUPAC name of 4- isopropyl-m-xylene is
(a) 3-Methylpentanoyl chloride (a) 1-isopropyl-2-, 4 -dimethylbenzene
(b) 3- Methylbutanoyl chloride (b) 4-isopropyl-m-xylene
(c) 1- Chloro - 3- ethylbutanone (c) 1-isopropyl -3, 5- dimethylbenzene
(d) 1- Chloro - 3- methylpentanone (d) 4- isopropyl-3, 5- dimethylbenzene
Classification and Nomenclature of Organic Compounds 375
43. The IUPAC name of (a) 5- Oxo-4-hydroxy -2- pentanone
(b) 4- hydroxy -5-al-2- pentanone
OHC - CH = CH - C H - CH = CH2 is
| (c) 2-hydroxy -4-oxopentanal
CH 2CH 2CH 2CH3
(d) 1-al-4-oxo-2- pentanol
(a) 5- vinyloct -3- en -1 -al 49. The structural formula of cyclohexylalcohol is
(b) 4- butyl- 2,5 - hexadien -1-al
(c) 5- vinyloct - 5- en - 8- al
OH OH
(d) 3- butyl -1, 4 - hexandien - 8 -al (a) (b)
44. The IUPAC name of

CH3 O CH2OH
| ||
CH 3- C H - C - CH 2 - CH 2 OH is (c) (d) None of these

(a) 1- hydroxy -4- methyl -3- pentanone


50. The structural formula of 2-methyl-2-butene is
(b) 2- methyl -5- hydroxy -3- pentanone
(a) CH3 – CH(CH3) – CH = CH2
(c) 4- methyl -3- oxo -1- pentanol
(b) CH3 – CH2 – C(CH3) = CH2
(d) Hexanol -1 - one -3
(c) CH3 – CH = CH – CH3
45. The correct IUPAC name of
(d) CH3 – CH = C(CH3) – CH3
O 51. An organic compound X (molecular formula C6H7O2N) has
||
H - C - CHO is six carbon atoms in a ring system, two double bonds and a
nitro group as substituent , X is
(a) Formylmethanal (b) 1, 2-ethanedione
(a) Homocyclic but not aromatic
(c) 2-oxoethanal (d) 1, 2-ethanedial
(b) Aromatic but not homocyclic
46. The IUPAC name of the compound
(c) Homocyclic and aromatic
CH 2 - CH - COOH is (d) Heterocyclic and aromatic
| |
OH NH 2 52. Name of the compound given below is

(a) 2- amino-3- hydroxypropanoic acid


(b) 1- hydroxy -2- aminopropan -3- oic acid
(c) 1- amino -2- hydroxypropanoic acid
(d) 3- hydroxy -2- aminopropanoic acid
47. The IUPAC name of the compound (a) 3- methyl -4- ethyloctane
(b) 2, 3- diethylheptane
CH 3 - C = CH - CH 2 - COOH
| (c) 5- ethyl - 6- methyloctane
OH (d) 4- ethyl- 3- methyloctane
(a) Hydroxypentenoic acid 53. The correct IUPAC name of the compound,
(b) 4- hydroxy -3- pentenoic acid
(c) 4- hydroxy -4- pentenoic acid is
(d) 3- hydroxy -4- methyl-3-ene pentenoic acid
48. The IUPAC name of (a) 5, 6 diethyl - 8- methyldec - 6 - ene
O OH (b) 6- Butyl - 5 - ethyl - 3- methyloct - 4- ene
|| |
is (c) 5, 6- diethyl - 3 - methyldec - 4- ene
CH 3 - C - CH 2 - CH - CHO
5 4 3 2 1 (d) 2, 4, 5- triethylnon - 3- ene.
376 Chemistry

1. 2-Methyl 2-butene will be represented as [CBSE-PMT 1992] 6. IUPAC name for the compound [CBSE-PMT 1998]

CH 3 Cl CH2CH3
| C=C
(a) CH 3 - CH - CH 2 CH 3 is
H3C I
(b) CH 3 - C = CH - CH 3 (a) trans-3-iodo-4-chloro-3-pentene
|
CH 3 (b) cis-2-chloro-3-iodo-2-pentene
(c) trans-2-chloro-3-iodo-2-pentene
(c) CH 3 - CH 2 - C = CH 2
(d) cis-3-iodo-4-chloro-3-pentene
|
CH 3 7. The correct structure of trans-2 hexenal is
[CBSE-PMT 1999]
(d) CH 3 - CH - CH = CH 2
| CHO
CH 3
(a) (b) CHO
2. The IUPAC name of [CBSE-PMT 1992]
(c) CHO (d)
CHO
CH 3 - CH - CH = C - CHO 8. The incorrect IUPAC name is [CBSE-PMT 2001]
| |
OH CH 3 (a) CH 3 - C - C H - CH 3
|| |
(a) 4-Hydroxy-1-methylpentanal O CH 3
(b) 4-Hydroxy-2-methylpent-2-en-1-al 2-Methyl-3-butanone
(c) 2-Hydroxy-4-methylpent-3-en-5-al
(b) CH 3 - CH - C H - CH 3
(d) 2-Hydroxy-3-methylpent-2-en-5-al | |
3. Which of the following IUPAC names is correct for the CH3 CH 2CH 3
compound? [CBSE-PMT 1994] 2,3-Dimethylpentane
H 3C - C H - CH - CH 2 - CH 3
| | (c) CH 3 C H - C H - CH 3
CH3 CH 2CH3 | |
Cl Br
(a) 2-Methyl-3-ethylpentane 2-Bromo-3-chlorobutane
(b) 3-Ethyl-2-methylpentane
(d) CH 3 - C º CCH (CH 3 ) 2
(c) 2-Ethyl-3-methylpentane
4-Methyl-2-pentyne
(d) 3-Methyl-2-ethylpentane
9. Name of the compound given below is [CBSE-PMT 2003]
4. The first organic compound, synthesized in the laboratory,
was [CBSE-PMT 1995]
CH3
(a) alcohol (b) acetic acid
H3C
(c) urea (d) none of these CH3
5. The IUPAC name of [CBSE-PMT 1996]
H 3C — C H — CH — CH 2 —CH 2— C H — CH 3 is CH3
| | |
CH3 CH3 CH3
(a) 5-ethyl-6-methyloctane
(a) 1, 3-isopropyl-3-methylpropane
(b) 4-ethyl-3-methyloctane
(b) 2, 3, 6-trimethylheptane
(c) 3-methyl-4-ethyloctane
(c) 2, 5, 6-trimethylheptane
(d) 2, 3-diethylheptane
(d) 2, 6, 3-trimethylheptane
Classification and Nomenclature of Organic Compounds 377
10. Names of some compounds are given. Which one is not
correct in IUPAC system? [CBSE-PMT 2005]
COOH
(c)
CH3
| OH
(a) CH 3 - CH 2 - CH 2 - C H - CH - CH 2 CH 3
|
CH 2CH3 OH

3-Methyl-4-ethyl heptane
(d) COOH
(b) CH3 - CH - CH - CH3
| |
OH CH3
3- Methyl - 2- butanol 14. The structure of isobutyl group in an organic compound is

(c) CH3 - CH 2 - C - CH - CH 3 (a) CH3 - CH - CH 2 - CH3 [NEET 2013]


|| | ½
CH2 CH3
2- Ethyl -3- methyl - but -1- ene (b) CH3 - CH 2 - CH 2 - CH 2 -

(d) CH 3 - C º C - CH - (CH 3 ) 2 CH3


4 - Methyl- 2 - pentyne |
(c) CH3 - C-
|
11. The correct IUPAC name of the compound
CH3

CH 3
is [CBSE-PMT 2011] (d) CH - CH 2 -
CH 3

(a) 4-Ethyl-3-propyl hex-1-ene 15. Which of the following compounds has wrong IUPAC name?
(b) 3-Ethyl-4-ethenyl heptane [AIEEE 2002]
(c) 3-Ethyl-4-propyl hex-1-ene (a) CH3–CH2–CH2 –COO–CH2CH3 ® ethyl butanoate
(d) 3-(1-ethylpropyl) hex-1-ene
(b) CH3 - CH - CH 2 - CHO ® 3-methyl-butanal
12. The IUPAC name of the following compound is |
CH3
Cl CH2CH3
(c) CH3 - C H - CH - CH3 ® 2-methyl-3-butanol
C C | |
OH CH3
CH3 I

(a) trans-2-chloro-3-iodo-2-pentene [CBSE-PMT 2011 M] O


||
(b) cis-3-iodo-4-chloro-3-pentene (d) CH3 - C H - C - CH 2 - CH3 ® 2-methyl-3-pentanone
|
(c) trans-3-iodo-4-chloro-3-pentene CH3
(d) cis-2-chloro-3-iodo-2-pentene
16. The IUPAC name of CH3COCH(CH3)2 is [AIEEE 2003]
13. Structure of the compound whose IUPAC name is 3-ethyl-2-
hydroxy-4-methylhex-3-en-5-ynoic acid is : [NEET 2013] (a) 2-methyl-3-butanone
(b) 4-methylisopropyl ketone
OH
(c) 3-methyl-2-butanone
(a) COOH (d) Isopropylmethyl ketone
17. The general formula C n H 2n O 2 could be for open
chain [AIEEE 2003]
OH (a) carboxylic acids (b) diols
COOH (c) dialdehydes (d) diketones
(b)
378 Chemistry

18. The IUPAC name of the compound is 21. The IUPAC name of is
HO

[AIEEE 2004] (a) 3-ethyl-4, 4-dimethylheptane [AIEEE 2007]


(a) 3, 3-dimethyl - 1- cyclohexanol (b) 1, 1-diethyl-2,2-dimethylpentane
(b) 1, 1-dimethyl-3-hydroxy cyclohexane (c) 4, 4-dimethyl-5,5-diethylpentane
(d) 5, 5-diethyl-4,4-dimethylpentane.
(c) 3, 3-dimethyl-1-hydroxy cyclohexane
22. The correct decreasing order of priority for the functional
(d) 1, 1-dimethyl-3-cyclohexanol groups of organic compounds in the IUPAC system of
19. Which of the following will have a mesoisomer also? nomenclature is [AIEEE 2008]
[AIEEE 2004] (a) – COOH, – SO3H, – CONH2, – CHO
(a) 2, 3- Dichloropentane (b) – SO3H, – COOH,– CONH2, – CHO
(b) 2, 3-Dichlorobutane (c) – CHO, – COOH, – SO3H, – CONH2
(c) 2-Chlorobutane (d) – CONH2, – CHO, – SO3H, – COOH
(d) 2-Hydroxypropanoic acid 23. The IUPAC name of neopentane is [AIEEE 2009]
20. The IUPAC name of the compound shown below is : (a) 2, 2-dimethylpropane (b) 2-methylpropane
[AIEEE 2006] (c) 2, 2-dimethylbutane (d) 2- methylbutane
24. The IUPAC name of the following compound is
Cl [IIT-JEE 2009]
OH

Br
CN
(a) 3-bromo-1-chlorocyclohexene Br
(b) 1-bromo-3-chlorocyclohexene (a) 4-Bromo-3-cyanophenol
(c) 2-bromo-6-chlorocyclohex-1-ene (b) 2-Bromo-5-hydroxybenzonitrile
(d) 6-bromo-2-chlorocyclohexene (c) 2- Cyano-4-hydroxybromobenzene
(d) 6-Bromo-3-hydroxybenzonitrile

OH (a) 2 - cyclohexylbutane (b) 2 - phenylbutane


(c) 3 - cyclohexylbutane (d) 3 - phenylbutane
1. IUPAC name of is :
4. The IUPAC name of CH = C H is
| |
OH OHC NH2
(a) But – 2 ene – 2, 3– diol (a) 1 - amino prop - 2 - enal
(b) Pent – 2 –ene – 2, 3 – diol (b) 3 - amino prop - 2 - enal
(c) 2 – methylbut – 2 – ene – 2, 3 – diol (c) 1 - amino - 2 - formylethene
(d) Hex – 2 – ene – 2, 3 – diol (d) 3 - amino - 1 - oxoprop - 2 - ene
OH 5. The given compound in IUPAC may be called,
2. IUPAC name of CH is :
3
NH 2
(a) 5 - methylhexanol (b) 2 - methylhexanol |
(c) 2 - methylhex - 3 - enol (d) 4 - methylpent - 2 - enol (CH3 )2 -C-CH 2 -CO-CH3
3. IUPAC name of following compound is :
(a) Diacetone
H (b) Acetoneamine
CH
CH33 - C - CH
CH22 -- CH
CH3 (c) Diacetoneamine
(d) 4 - amino - 4 - methylpentane - 2 - one
Classification and Nomenclature of Organic Compounds 379
10. The acceptable IUPAC name for the molecule including E –
O Z designation
CH2– C – OH
OH
6. The IUPAC name of compound C is:
COOH
(a) Z – 3 – Propyl – 1, 3 – pentadiene
CH2– COOH (b) E – 3 – Propyl – 1, 3 – pentadiene
(c) Z – 3 – Ethenyl – 1, 3 – hexadiene
(a) 1, 2, 3 - tricarboxy - 2, 1 - propane (d) E – 3 – Ethenyl – 1, 3 – hexadiene
(b) 3 - carboxy - 3 hydroxy - 1 , 5 - pentanedioic acid 11. What is the IUPAC name of the compound ?
(c) 3 - hydroxy - 3 - carboxy - 1, 5 - pentanedioic acid
(d) 2 - hydroxy propane -1, 2, 3 - tricarboxylic acid.
H
7. What is the IUPAC name of the following compound ? CH 3 Cl
NH 2 (a) 2 – chloro – 2 – butene
(b) 3 – chloro – 1 – butene
(c) 3 – methyl – 3 – chloropropene – 1
(d) 3 – chloro – 3 – methyl – 1 – propene
(a) 2-methyl-4-hexanamine 12. What is the IUPAC name of the following compound ?
(b) 5-methyl-3-hexanamine
(c) 2-methyl-4-amino hexane
(d) 5-methyl-3-amino hexane
8. Which of the following is the correct IUPAC name of the (a) 3 – methyl – 5 – heptanone
compound ? (b) 5 – methyl – 3 – heptanone
(c) 5 – ethyl – 3 – hexanone
N (CH 3 ) 2 (d) 2 – ethyl – 4 – hexanone
13. Identify the correct IUPAC name of the compound given
below

Cl
Cl H

(a) 1, 2-dichloro-4-(N, N-dimethyl) aniline


(b) Dimethyl – (3, 4-dichlorophenyl) amine
(c) 3, 4-dichloro - N, N-dimethyl aniline (a) 4 – benzyl – 5 – methyl hexanal
(d) N, N-dimethylamino - 3, 4-dichlorobenzene (b) 2 – methyl – 3 – phenyl hexanal
9. Which one of the following is ethyl-4-(dimethyl amino) (c) 5 – isopropyl – 5 – phenyl butanal
butanoate ? (d) 5 – methyl – 4 – phenyl hexanal
14. Which of the following is the IUPAC name of the compound
CHO
Br
(a) (MeNH) 2 OEt
Cl
(a) o – bromo – m – Chlorobenzaldehyde
(b) 2 – bromo – 5 – Chlorobenzaldehyde
(b)
OEt (c) 6 – bromo – 3 – Chlorobenzaldehyde
NH 2 (d) 1 – bromo – 4 – Chlorobenzaldehyde
15. The IUPAC name of the following epoxide is
H 3C H

(c) H 2 N H CH 2 CH 3
OEt O
(a) cis – 2 – ethyl – 3 – methyloxirane
(b) trans – 1 – ethyl – 2 – methyl oxycyclopropane
(c) trans – 2 – ethyl – 3 – methyloxirane
(d) Me 2 N (d) trans – 1 – ethyl – 2 – methyl ethane epoxide
OEt
380 Chemistry
16. The IUPAC name of the following compound (a) 1, 1 – dimethyl – 1 – cyclopentyl methane
(b) 2 – cyclopentyl propane
(c) 1 – (1 – methyl) ethyl cyclopentane
(d) Cumene.
(a) E – 1 – cyclopentyl – 2 – hexene 22. The accepted IUPAC name of the following compound.
(b) E – 2 – cyclohexyl – 2 – hexene
(c) Z – 1 – cyclopentyl – 1 – hexene
(d) Z – 2 – cyclopentyl – 2 – hexene
17. The IUPAC name of the following compounds is
O
N (a) Propanoic anhydride
| (b) Butanoic propanoic anhydride
H O (c) Ethyl butanoate
(a) N – phenyl ethanamide (b) N – phenyl ethanone (d) 4, 6 – dioxo heptane
(c) N – phenyl methanamide (d) None of these 23. The IUPAC name of the compound
18. What is the name of the following compound ?
H COOH
O
||
O O
H CHO
Cl
(a) 4-formyl-3-oxo cyclohexane-1-carboxylic acid
(a) 2 – chlorohexyl ethanoate
(b) 2, 4-dioxo cyclohexanoic acid
(b) ethyl – 2 chloro hexanoate
(c) 2, 4-dioxo heptanoic acid
(c) 1 – chlorohexyl ethanoate
(d) None of these
(d) ethyl – 1 chloro hexanoate
24. The correct IUPAC name of the compound
19. The IUPAC name of the following compound.
HC CH 3 CH 3
||
HO CH 3
O
OH
(a) 2, 6 – dimethyl octa – 2, 6 – dien – 1 – al
(a) 4 – hydroxy – 3 – methyl butanoic acid
(b) 3, 7 – dimethyl hepta – 2, 6 – dien – 1 – al
(b) 1 – hydroxy – 2 – methyl butanoic acid
(c) 3, 7 – dimethyl octa – 2, 6 – dien – 1 – al
(c) 3 – hydroxy – 2 – methyl butanoic acid
(d) 2, 6 – dimethyl – 2, 6 – dien – 8 – al
(d) 3 – (hydroxymethyl) butanoic acid
25. The IUPAC name of the compound
20. What is the IUPAC name of the following compound ?
CONH 2
COOH CHO
Br

(a) 5 – formyl – 2 – methyl pent – 3 – en – 1 – amide


(b) 2 – carbamoyl hexanal
CH 2 CH3
(c) 2 – carbamoyl hex – 3 – en – 1 – al
(a) 6 – bromo – 4 – ethylbenzene carboxylic acid (d) 6 – Keto – 2 – methyl hexanamide
(b) 2 – bromo – 4 – ethylbenzene carboxylic acid 26. The correct name of the compound (IUPAC)
(c) Ortho – bromo – paraethyl benzoic acid Cl
(d) 4 – bromo – 3 – ethyl benzoic acid Cl 3C.CH
Cl
21. What is the IUPAC name of the compound ?
(a) D. D. T.
CH(CH 3 ) 2 (b) Gammexane
(c) p, p’-dichloro diphenyl trichloroethane
(d) 1, 1, 1-trichloro-2, 2-bis (4-Chlorophenyl) ethane
Classification and Nomenclature of Organic Compounds 381
27. The IUPAC name of the compound 33. The correct IUPAC name of
CH 3

(a) 3 – methyl spiro [4, 5] dec – 2 – ene


(b) 2 – Methyl spiro [4, 5] dec – 1 – ene
(a) tri cyclopropane
(c) 2 – Methyl [5, 4] dec – 1 – ene
(d) 3 – Methyl spiro [5, 4] dec – 2 – ene (b) 1, 1’, 2’, 1” – tercyclopropane
28. The IUPAC name of the compound (c) tricyclopropyl propane
(d) 1, 1’, 2, 2’ – tercyclo butane
34. IUPAC name of
O O
O
(a) 1, 2, 3 – triformyl propane (a) spiro [4, 5] decane (b) spiro [3, 5] nonone
(b) Propane – 1, 2, 3 – tricarbaldehyde (c) bicyclopentyl hexane (d) bicyclo [5, 4, 0] decane
(c) 3 – formyl – 1, 5 – pentane dial
(d) Propane – 1, 2, 3 – trial O
29. The correct name (IUPAC) of the compound 35. Aspirin has the structure O . It IUPAC name
OH
O
O O
C - O - C2H 5 is
(a) Carbomethoxy benzoic acid
(a) 2 – carbethoxy cyclopentan – 1 – one (b) Methyl – 2 – carboxy benzoate
(b) ethyl – 2 – oxo cyclopentanoate (c) 2 – carboxy phenyl ethanoate
(c) carbethoxy cyclopentanone (d) O – carboxy phenyl acetate
(d) None of these 36. IUPAC name of the compound
30. The accepted IUPAC name of the camphor is O
||
O O

(a) 1, 7, 7 – trimethyl bicyclo [2. 2. 1] heptan – 2 – one NO 2


(b) 1, 7, 7, – trimethyl bicyclo [2. 1. 2] heptan – 2 – one (a) ethyl-3-methyl-2-(3-nitro) phenyl butanoate
(c) 1, 2, 2 – trimethyl bicyclo [2. 2. 1] heptan – 6 – one
(b) ethyl-2-methyl-2-(m-nitro) phenyl propanoate
(d) None of these
(c) ethyl-2-methyl-2-(3-nitro) phenyl propanoic acid
31. The IUPAC name of
(d) None of these
NH 2 COOH 37. The IUPAC name of the compound

COOH NH 2
(a) o, o’ – diamino – o, o’ – dicarboxy biphenyl (a) Tetraphenyl methane
(b) 6, 6’ – diamino diphenyl – 2, 2’ – dicarboxylic acid (b) 1, 1, 1, 1 – tetra phenyl methane
(c) diamino dicarboxy biphenyl (c) 1, 1, 1, 1 – tetra cyclohexyl methane
(d) None of these (d) Neophenyl
32. The IUPAC name of 38. Which of the following numberings is correct ?
OH FF 1 2 FF 3 2
C A 3 B 1
|| Br Br
5 4 4 5
O
(a) 3 – phenyl prop – 2 – enoic acid F 5 4 FF 5 1
(b) Cinnamic acid C 3 D 2
1 Br Br
(c) 1 – Styrene acid 2 4 3
(d) 3 – Cyclohexyl – 2 – propenoic acid (a) A (b) B (c) C (d) D
382 Chemistry

EXERCISE 1 Further, in writing the IUPAC name, prefix should be


1. 1-Methoxypropane, CH3OCH2CH2CH3 or arranged in the alphabetic order, i.e., 3-ethyl should be
written first followed by 2-methyl.
2-methoxypropane, CH3-OCH(CH3)2. 5. (c) In such questions, first draw the structure of the molecule
2. (i) –CHO (II) –NO2 and then name the molecule observing the required rules
3. The primary suffix indicates whether the carbon chain is for IUPAC nomenclature.
saturated or unsaturated while the secondary suffix indicates
CH3
the functional group present in the molecule. 1 2| 3
4. 3–Methylbut–l–ene. C H 3 - C - CH 3
|
5. 2-Bromo-4-methylpentan-3-one. CH3
6. 2, 4-dimethyl-5-oxopentanoic acid.
1 2 3 4 5 6 7
10. (i) 3-Ethyl-4-methylheptan-5-en-2-one 6. (b) CH 3 CH CH CH 2 CH 2 CH CH 3
(ii) 3-Nitrocyclohex-1-ene. | | |
CH3 CH3 CH3
13. (b) 14. (b) 15. (d) 16. (b) 17. (c)
18. (a) 19. (a) 20. (c) 21. (a) 22. (c) CH3
1 2 3 4
7. (b) CH3 CH2 – CH – CCH3
EXERCISE 2 5 6 7 8
CH3 CH2CH2 CH2CH3
1. (a) CaH2 is formed by the donation of two electrons from Ca
atom (having two electrons in the outermost shell) to 1 2
two H atoms (each having one electron in the outermost
8. (b) CH3–CH–CH3
4 5 6
shell). CH3 – CH – CH2CH2CH3
3
CH3 CH 3 9. (a)
| |
2. (d) CH 3 - C - C - CH 2 CH 3 5 6
| | CH2 CH3
CH3 CH 3
(a )
10. (d) CH3 –4C – CH
3 2 1
2 CH CH3
CH3 Cl
CH3
| CH3
CH 3 – C – CH 2 CH 3 4 3 2 1
| 11. (c) CH3 – C – CH = CH2
CH3
( b) CH3

CH 3 CH3
CH3 CH3 5 4 3 2 1
| | |
12. (b) CH3 CCH2CH = CH2
CH 3 C - C HCH 2 CH 3 CH 3 C HCH 2 CH 2CH 3
| CH3
CH3 (d )
(c ) 4 3 2
13. (a) CH3 – CH – C – CH2CH3
3. (d) In isopentane, (CH3)2CH CH2 CH3, every carbon is having
hydrogen atom(s). CH3 CH2
1
4. (b) When the sum of locants is same, viz. 2 + 3 = 5, lower
Here numbering should be done in a way that simpler
alkyl group should be given lower numeral number, i.e.,
alkyl group(s) is (are) present on the main chain
here it should be 2-methyl and 3-ethyl and not the reverse.
Classification and Nomenclature of Organic Compounds 383
2 1 O CH3
CH=CH2 1 2 3 4
14. (b) 6 5 4 3 30. (a) CH3 – C – CH – CH3
CH3CH2CH2CHCH2CH2CH3
3-Methyl-2-butanone
4 5 6
CH2 CH2CH3 31. (c) 32. (c)
15. (c)
1 2 3 3 2 1
CH3 – CH = C – CH2CH3 33. (d) CH = CH – CN
2
Double bond should be present on the minimum possible Nitrile carbon is also counted in the carbon chain
number in the lengthiest possible carbon chain. 34. (a) 35. (a)
4 5
16. (c) CH2CH3 4 3 2 1
3 2 1
36. (b) C H 3 - C H = C H - C OOC2 H 5
CH3 – C = CH CH3 37. (d)
1 2 3 4 5
17. (a) CH3 C º C CH CH3 O
6 5 || 1
4 3 2
CH3 38. (d) CH CH CH = CH – C–C H
3 2 3
7 6 5 4 3 2 1
18. (a) CH3 – C = C – CH – CH2 – C º CH 1
CH3
Cl CH3 CH2CH3 39. (d) tert - Butyl chloride is CH3 – C – Cl
2

Remember a triple bond gets priority over the double 3


bond, hence it is principal functional group here, thus CH3
suffix should be –yne and not –ene 2
3 1
19. (c) 40. (a) CH = CH – CHO (acraldehyde)
2
1 2 3
20. (c) CH = CH–CH Cl 41. (a) Carbinol is methyl alcohol (CH3OH), hence vinylcarbinol
2 2
3 2 1
21. (a) Since b (from bromo) comes earlier in alphabetical order should be CH = CHCH OH (prop-2-enol)
2 2
than c (from chloro), the correct name should be 2, 3-
dibromo-1, 4-dichlorobutene-2 and not 1,4-dichloro-2, 42. (a) 4 - Isopropyl -m-xylene CH3
3-dibromobutene-2. 4
CH2 CH3 5 3
4 3 2 1 2
22. (a) CH3CH2CH CH2 OH 6
1
CH3
OH
CH(CH3)2
1 2 3 4 5
23. (a) CH3 – C – CH2 – CH – CH3
1 2 3 4 5 6
43. (b) CHO.CH = CH CH CH = CH2
CH3 CH3
CH2 CH2 CH2 CH3
Select the lengthiest possible chain having functional (–
OH) group. Carbon chain should include all functional groups,
4 3 2 1 although it may be smaller in length, –CHO should be
24. (c) CH3 – CH – CH2 – CH – CH3 given lowest number.
5 6 CH3 O
CH2CH3 OH
5 4 3 2 1
1 2 44. (a) CH3 – CH – C – CH2 – CH2OH
25. (b) CH3CH2 3
CH3
5 4 C
CH3CH2 OH 45. (d)
26. (a)
46. (a)
OH CH3
5 4 3 2 1
5 4 3 2 1
27. (c) CH3 – CH – CH2 – C – CH3 47. (b) CH3- C H = CH - CH 2 - C OOH
|
OH OH
2 1 O OH
28. (b) CH3 – CH – CHO 5 4 3 2 1
3 4 48. (c) CH – C – CH – CH – CHO
CH2 – CH3 3 2
– CHO gets priority over – CO
29. (a)
49. (a) 50. (d) 51. (a) 52. (d) 53. (c)
384 Chemistry

EXERCISE 3 1 2 3 4
8. (a) CH3 C CH CH3
CH3
| O CH3
1. (b) 1 CH3 - 2C = 3CH - 4 CH3 3 Methyl-2- butanone
2-Methyl-2-butene 1 2 3
CH3 CH CH CH3
2. (b) 5 CH - 4 CH - 3CH = 2
C - 1CHO 4 5
3 CH3 CH2 CH3
| |
OH CH 3 2,3dimethyl pentane
4-Hydroxy-2-methylpent-2-en-1-al 4 3 2 1
CH3 CH CH CH3
3. (b) 1 CH 3 - 2 CH - 3CH - 4 CH 2 - 5 CH 3 Cl Br
| |
H 3C CH 2 CH 3 2 Bromo 3 chlorobutane
5CH
3-Ethyl-2-methylpentane 3
1 2 3 4
4. (c) The vital force theory suffered first death blow in 1828 CH3 C C CH
when Wohler synthesized the Ist organic compound
urea in the laboratory from inorganic compounds CH3
reported below : 4 - Methyl-2-pentyne
to isomeric charge 9. (b) CH3 CH3 5 7
NH 4 CNO ¾¾ ¾ ¾ ¾ ¾ ¾¾® NH 2 CONH 2 1 3
4
rearrangement leading CH3
Urea 2 6
Later on a further blow to vital force theory was given CH3
by Kolbe (1845) who prepared acetic acid, the first
organic compound, in laboratory from its elements. 4 ethyl - 3 methyl octane
10. (a) Correct IUPAC name of
5. (b) When many substituents are present, the numbering is
done from the end where the sum of locants is the lowest CH 3
|
(lowest sum rule)
CH 3 CH 2 CH 2 - C H - C H - CH 2 CH 3
|
7 6 5 4 3 2 1
CH 2CH 3
C H3 - CH - CH - C H 2 - C H 2 - C H - C H3
| | | is 4-Ethyl-3–methyl heptane
CH3 CH3 CH3
11. (a) The given compound is
2, 5, 6 trimethyl heptane (wrong)
2 1
2 + 5 + 6 = 13 CH = CH2
1 2 3 4 5 6 7
CH3 — CH — CH — CH 2 — CH 2 — CH — CH3 Ch3 — CH2 — CH2 — CH — CH — CH2 — CH3
| | | 3 4
CH3 CH3 CH3 CH2 – CH3
5 6
2 + 3 + 6 = 11
4 ethyl- 3-propyl hex – 1 – ene
2, 3, 6 TriMethyl Heptane (correct)
4 5
Cl CH2 – CH3 12. (a) Cl CH2CH3
6. (c) C C C=C
H3C I 2 3
CH3 I
1
As in this compound the common groups i.e highly
electronegative halogen atoms are on opposite side, Correct IUPAC name of above compound is trans-2-
hence it is a trans isomer. chloro-3-iodo-2-pentene
Thus its name is trans-2-chloro-3-iodo-2-pentene. OH
7. (a) When similar atoms are on opposite side the compound 1
3 2
is in trans-form H3C COOH
13. (a) 4
6
H CHO CH 3
5
C C
IUPAC name of the structure is 3-ethyl-2-hydroxy-4-
C3H7 H methylhex-3-en-5-ynoic acid
Classification and Nomenclature of Organic Compounds 385

CH 3 EXERCISE 4
14. (d) CH - CH 2 - (iso-butyl group) 1. (b) The compound contains longest chain of 5C - atoms and
CH 3 e of ene is retained as the suffix name starts with constant
2. (d) The compound is an enol containing chain of 5C atoms.
15. (c) The correct name is 3 - methylbutan 2-ol
3. (b) The compound is a derivative of butane
O CH3 4. (b) It contains longest chain of three C atoms having CHO
1 2| | 3| 4
16. (c) CH3 - C - CH - CH3 ; 3- methyl-2-butanone and double bond
5. (d) The compound contains longest chain of 5 C - atoms
having > C = O and - NH2 groups
17. (a) CnH2nO2 is general formula for carboxylic acid
6. (d) The compound contains longest chain of 3 C - atoms and
three -COOH groups and one -OH group attached to it
(latest convention).
18. (a) 1 3
2 7. (b) The compound contains longest chain of 6C atoms and
HO amino group. Hence it is an alkanamine.
IUPAC name – 3, 3-Dimethyl -1 cyclohexanol 8. (c) The compound is derivative of aniline. The positions of
19. (b) The compounds containing two similar assymmetric C- groups are shown by numbering the nuclear C-atoms.
atoms have plane of symmetry and exist in meso from 9. (d) The compound is an ester. Its IUPAC name is derived
from alkyl alkanoate.
10. (a) Select the longest chain of C-atoms which includes both
plane of symmetry the double bonds. Further ethenyl and methyl groups at
C3 and C 4 are on the same side. Hence it is Z-isomer..
11. (a) It contains chain of 4C atoms. Double bond is given
meso 2, 3 dichlorobutane preference over Cl-atom.
12. (b) The compound is a ketone containing longest chain of
Cl 7C-atoms and side chains.
1
6 2 13. (d) The compound is an aldehyde containing longest chain
20. (a)
5 of 6 C-atoms and side chains.
3 Br
4 14. (b) The compound is a derivative of benzaldehyde. Start
3-bromo-1chlorocyclohexene numbering C-atoms of benzene nucleus from C-atom
bearing – CHO group.
CH 3
7 6 5 4| 3 2 1 *
15. (c) The compound is derivative of oxyrane CH * .
2 - CH 2
21. (a) CH3 - CH 2 - CH 2 - C - CH - CH 2 - CH3
| |
CH3 CH 2 O
| *
CH3 The hydrogen atoms attached to C –atom are on the
3- ethyl - 4,4 - dimethylheptane opposite side hence it is trans isomer.
16. (b) When acyclic portion contains a multiple bond or
22. (a) The correct order of priority for the given functional functional group, the cyclic portion is treated as
group is substituent.
O O 17. (a) It is derivative of ethanamide having N-phenyl group.
|| ||
–COOH > –SO3 H > – C - NH 2 > - C - H 18. (b) The compound is an ethyl ester of hexanoic acid.
19. (a) The compound is a derivative of butanoic acid.
23. (a) CH3 20. (b) The compound is a derivative of benzoic acid. The
|
1
positions of substituents attached to benzene nucleus
3 2
H3C- C - CH3 are represented by number of C-atoms and not by ortho,
|
CH3 meta and para.
Neopentane 21. (c) The cyclic portion contains more C-atoms than
or 2, 2- Dimethylpropane acyclic portion. Hence it is derivative of cyclopentane.
22. (b) The compound is anhydride of butanoic and propanoic
24. (b) – CN has highest priority. Further the sum of locants is acid.
7 in (b) and 9 in (d). 23. (a) It is derivative of cyclohexane. The C-atom bearing
– COOH group is to be assigned the number 1.
386 Chemistry
24. (c) The compound is an aldehyde containing longest chain 32. (a) It is derivative of propanoic acid
of 8 C-atoms. O
25. (a) The compound is an amide containing longest chain of 5 ||
C-atoms – CHO is substituent group. CH = CH - C - OH
3 2 1
26. (d) The compound is derivative of ethane.
3-phenyl prop-2-enoic acid
27. (b) It is spiro compound. The numbering begins with the
33. (b) In such compounds the numbering is done as follows
ring atom next to the spiro atom around the smaller ring
and then to the spiro atom and finally around the large 3'
ring. The total number of C-atoms decides the parent 2 2' '
hydrocarbon. 1 1' 2' 1' '

1 2 CH 3 3' '
7 6 3
1, 1' , 2' , 1' ' tercyclopropane
8
5 2 1 6 7
9 10 4 3
34. (a) 5 8 spiro [4.5] decane
Hence it is 2-methyl spiro[4.5] deca-1-ene. 3 4 10 9
28. (b) When – CHO is treated as side chain, its name is 35. (c) It is an ester of ethanoic acid
carbaldehyde.
O
3 2 1 6 || 2
C H2 - C H - C H2 1 O - C - CH 3
5
| | | 1
Hence CHO CHO CHO is propane – 1, 2, 3 tri 2
4 COOH
carbaldehyde. 3
29. (a) It is derivative of cyclopentanone having 2-carboxy phenyl ethanoate
36. (a) It is an ethyl ester of butanoic acid
O
|| CH 3 O
– C– O – C2 H5 carbethoxy group at 2C. | ||
CH 3 - CH - CH - C - O - C 2H 5
30. (a) It is a bridge compound 4 3 2 1
1 6 1 2
6 O
7 2 3
5 NO 2
5 3 4
4 ethyl-3-methyl-2-(3-nitro) phenyl butanoate
1, 7, 7-trimethyl bicyclo [2. 2. 1] heptan-2-one 37. (c) It is derivative of methane 1, 1, 1, 1-tetracyclohexyl
31. (b) It is biphenyl derivative methane.
38. (d) F *1
NH 2 COOH 5
5' 6' 2 3
Br
2
4' 4 **
1' 1 4 3
3' 2' 6 5 * **
The numbering of C-atom starts from C or C . But
COOH NH 2 *
numbering from C give minimum locant (2) to Br which is
correct.
12C
Hybridisation & Shapes
of Organic Molecules
HYBRIDISATION
Sigma bonds are the most common bonds in organic chemistry.
All single bonds are sigma (s) bonds and formed by the
C
overlapping between s-s, s-p and p-p (head on) atomic orbitals
present on different atoms. A pi (p) bond results from the overlap
of two p-orbitals that are oriented perpendicular to the axis of the
nuclei. A p bond is not cylindrically symmetrical. A s bond is These hybrid orbitals can overlap with four s atomic orbitals
stronger than p bond due to better overlap. All multiple bonds provided by four hydrogen atoms to form methane molecule.
contain one s bond and others p bond(s).
To have more efficient overlapping and to provide more H
symmetrical structure to the molecule the atomic orbitals on the H
same atom interact to provide hybrid atomic orbitals and the
interaction is known as hybridisation. The hybrid atomic orbitals H 109.5°
have enhanced electron density. C
9Å C
HYBRIDISATION OF CARBON
H 1. 0 H
H
The ground state electronic configuration of carbon is
H
2 2 H
1s , 2s 2p1x 2p1y 2p 0z . The electronic configuration of carbon in

excited state is 1s 2 , 2s1 2p1x 2p1y 2p1z . Bonding in Methane

sp3 hybridisation:
H H
If we superimpose one s and three p atomic orbitals we get 4sp3
hybrid orbitals.

3 4 H
s-atomic p-atomic C C
hybrid H
orbital orbital orbital
Each hybrid orbital contains single electron, has 25% s character
and 75% p character. They are directed towards the four corners H H
of a regular tetrahedron with the carbon located in the centre. The
angle between any two sp3 hybrid orbitals is 109º 28' (109.5º). Bonding in Ethane
388 Chemistry
sp hybridisation:
H H
If we superimpose one s and one p atomic orbitals we get 2sp
hybrid orbitals.
109.5°
1.54Å
9Å C C
1. 0 H 1
H s-atomic orbital p-atomic orbital
H H
sp2 hybridisation: If we superimpose one s and two p atomic 1 2
orbitals we get 3sp2 hybrid orbitals
hybrid orbital
2 Each sp hybrid orbital has 50% s character and 50% p character.
s-atomic orbital p-atomic orbital They are diagonally present with their axis forming an angle of
180º. The unhybridised 2p y and 2p z atomic orbitals are
perpendicular to each other and perpendicular to hybrid orbitals
2 3 also.
hybrid orbital pz
Each sp2 hybrid orbital has 33% s character and 67% p character. • • py
They lie in the same plane with their axis directed towards the • •
sp
corner of an equilateral triangle and are 120º apart from each other.
The unhybridised pz atomic orbital is perpendicular to the plane
of sp2 hybrid orbitals. BONDING IN ACETYLENE

pz
• • • • py
H • • • • H
C
sp

H—C C—H

pz
HYBRIDISATION OF NITROGEN
The ground state electronic configuration of nitrogen is
BONDING IN ETHYLENE
Consider two sp2 hybridised carbon atoms approaching to each 7N = 1s 2 , 2s 2 2p1x 2p1y p1z
other and four hydrogen atoms which provide four s atomic
orbitals One s and three p atomic orbitals superimpose and give 4 sp3 hybrid
orbitals. These are tetrahedrally present.

C C
N
N H
H H H 107°
H

H H H

H p H sp2 hybridisation: when nitrogen attaches itself to two other atoms


C C C C it is present in the sp2 hybridised form. Consider the formation of
H s H methylimine CH2 = NH in which carbon and nitrogen both are in
H H sp2 hybrid state
Hybridisation & Shapes of Organic Molecules 389

H BOND LENGTHS
Some importants bond lengths are as follows

C N
C–C sp3 – sp3 1.54 Å C–O sp3 – O 1.41 Å
sp3 – sp2 1.50 Å sp2 – O 1.34 Å
sp3 – sp 1.46 Å C=O sp2 – O 1.20 Å
H H
sp2 – sp2 1.48 Å sp – O 1.16 Å
Carbon Nitrogen
sp2 – sp 1.43 Å C–N sp3 – N 1.47 Å
H sp – sp 1.38 Å sp2 – N 1.36 Å
p
s C=C sp2 – sp2 1.34 Å C=N sp2 – N 1.28 Å
C N
sp2 – sp 1.31 Å CºN sp – N 1.16 Å
H H sp – sp 1.28 Å
sp hybridisation: when nitrogen is attached to only one atom its CºC sp – sp 1.21 Å
hybridisation is sp. In H - C º N both carbon and nitrogen are in C–H sp3– H 1.11 Å
sp hybridised form
sp2 – H 1.10 Å
• • • •
sp – H 1.08 Å
H • • • •• H—C N BOND ANGLES IN SELECTED MOLECULES
H Cl
Carbon Nitrogen
HYBRIDISATION OF OXYGEN C C
H H Cl Cl
2 2 109.5° 109.5°
The electronic configuration of oxygen is 1s 2s 2p 2x 2p1y 2p1z . H Cl
sp3hybridisation : When oxygen is attached to two atoms the CH3 Cl
hybridisation is sp3.
112° 112°
C C
H CH3 H Cl
106° 112°
H H

.. CH3
O .O.
N
111° H H
O
H H 104.5° H CH3 H 107°

H H H H
sp2 hybridisation: When oxygen is attached to one atom as in 104.5° 118° C C
O
case of aldehydes and ketones e.g. in Formaldehyde carbon and H H 121° H
oxygen, both are in sp2 hybrid form.
Cl Cl 120°
CH3
H
114° C C 120° C O
Cl 123° Cl CH3

C O 127° 121°
CH3 CH3
110° C O 114° C O
Cl 123° CH3HN 125°
H
AROMATICITY AND AROMATIC COMPOUNDS
Carbon Oxygen Aromatic indicates a stable system which undergoes substition
H rather than addition, retaining the closed p-electron system. Many
..
C O such systems contain only six p electrons, but generally they
.. contain (4n+2) p electrons, where n is an integer.
H
390 Chemistry
In general, higher polyclic aromatic compounds are somewhat
n=1 less stable than benzene.
n = 0 cyclopropenyl cation contains 2p - electrons and is aromatic

H
Benzene Cyclopentadienyl
anion H
+

ANTIAROMATICITY
H The less stability of monocyclic compounds containing (4n)p
electrons than their acyclic analogues is called anti aromaticity.
H For example
Cycloheptatrienyl
cation Cyclobutadiene is less stable than 1,3-Butadiene
(Tropylium cation)
Nonbenzenoid heterocyclic compounds with 6p electrons are Here Resonance is the cause of destabilisation (hence the concept
aromatics of antiaromaticity)

¬¾
®

N O S N
More examples of antiaromatic compounds
Pyridine H
Furan Thiophene Pyrrole
H
The hetero atom contributes to non bonded electrons, to complete –
the sextet.

n=2

Cyclooctatetraene
(8 p electrons) Cycloheptatrienyl anion
Naphthalene Cyclo octatetraenyl (8p electrons)
dianion

H + H –

n=3

Cyclopentadienyl cation Cyclopropenyl anion


Anthracene Phenanthrene (4p electrons) (4p electrons)
Hybridisation & Shapes of Organic Molecules 391

Very Short/ Short Answer Questions 9. Will the following compound be aromatic. Explain.
1. Out of ethylene and acetylene which is more acidic and
why?
2. Write the type of hybridisation involve in CH4, C2H4 and
C2H2.
3. What types of hybridisation are involved with the central
atom of a molecule having following shapes:
(i) planar, (ii) a regular tetrahedral, (iii) an equilateral Multiple Choice Questions
triangle. 10. Butyne-2 contains :
4. State the hybridisation of the carbon labelled (x) and (y) in
(a) sp hybridised carbon atoms only
acetic acid.
(b) sp 3 hybridised carbon atoms only
H O
H— C — C — O —H (c) both sp and sp 2 hybridised carbon atoms
y x
H (d) both sp and sp 3 hybridised carbon atoms
5. How hybridization of ‘C’ atom changes when ethene is 11. The correct order towards bond length is
hydrogenated? (a) C - C < C = C < C º C (b) C º C < C = C < C - C
6. Ethane is stable while ethene is quite reactive, explain why?
(c) C = C < C º C < C - C (d) C = C < C - C < C º C
7. Give hybridisation state of each carbon atom in the following
compounds. 12. The C–H bond length is minimum in the bond formed by
(i) CH2 = C = O (a) sp-s overlapping (as in alkynes)
(ii) CH3CH = CH2 (b) sp2-s overlapping (as in alkenes)
(iii) (CH3)2 CO (c) sp3-s overlapping (as in alkanes)
(iv) CH3CH2– (d) None of these
13. Triple bond of ethyne is made of or Cylindrical shape of an
Long Answer Questions alkyne is due to
8. The ring systems having following characteristics are (a) Three s – bonds
aromatic. (b) Three p – bonds
(i) Planar ring containing conjugated p bonds. (c) Two s and one p – bond
(ii) Complete delocalisation of the p-electrons in ring (d) Two p and one s – bond
system i.e, each atom in the ring has unhybridised p-
14. The hybridization of carbon atom in benzene is:
orbital, and
(iii) Presence of (4n + 2) p-electrons in the ring where n is (a) sp (b) sp2 (c) sp3 (d) dsp2
an integer (n = 0, 1, 2 ...........) [Huckel rule] 15. The conditions for aromaticity is :
Using this information classify the following compounds (a) molecule must have cyclic clouds of delocalised p
as aromatic /non-aromatic. electrons
(b) molecule must contain (4n + 2) p electrons
(c) Both (a) and (b)
N – (d) None of the above
+
(A) (B) (C) 16. Which one of the following has the shortest carbon carbon
bond length ?
(a) Benzene (b) Ethene (c) Ethyne (d) Ethane
17. Shape of methane molecule is
(a) tetrahedral (b) pyramidal
+
(D) (c) octahedral (d) square planar
(E)
392 Chemistry
18. Which of the following has the shortest C – C bond length? (a) sp3 orbital (b) sp2 orbital
(a) C2H5OH (b) C2H6 (c) sp orbital (d) sp3 and sp respectively
(c) C2H2 (d) C2H4 21. In ethene, the bond angle(s) is/ are
19. The compound 1, 2 - butadiene has (a) 109°28 (b) 120°
(a) only sp hybridized carbon atoms (c) 180° (d) Both (a) and (c)
(b) only sp2 hybridized carbon atoms 22. Electronegativity of carbon atoms depends upon their state
(c) both sp and sp2 hybridized carbon atoms of hybridisation. In which of the following compounds, the
(d) sp, sp2 and sp3 hybridized carbon atoms carbon marked with asterisk is most electronegative?
20. In the following molecule, the two carbon atoms marked by (a) CH3 – CH2 – *CH2 – CH3
asterisk (*) possess the following type of hybridized orbitals (b) CH3 – *CH = CH – CH3
(c) CH3 – CH2 – C º *CH
H3C - C*º C*- CH3
(d) CH3 – CH2 – CH = *CH2

1. How many primary carbon atoms are there in 7. 2- Pentene contains


CH3CH2CH(CH3)C(CH3)2CH2CH2CH3 ? (a) 15 s- and one p- bond (b) 14 s-and one p- bond
(a) 3 (b) 4 (c) 15 s- and two p- bonds (d) 14 s- and two p- bonds
(c) 5 (d) 6 8. The number of pi-bonds in
2. The compound which contains all the four 1º, 2º, 3º and 4º CH2 = CH– CH = CH – C º CH is
carbon atoms is
(a) 2 (b) 5
(a) 2, 3-dimethylpentane
(c) 4 (d) 3
(b) 3-chloro-2, 3-dimethylpentane
9. Which of the following is not aromatic ?
(c) 2, 3, 4-trimethylpentane
(a) Benzene (b) Pyridine
(d) 3, 3- dimethylpentane
(c) Pyrimidine (d) 1, 4- Dioxane
3. Which of the following statements is not correct?
10. Toluene has
(a) Double bond is shorter than a single bond
(a) 6 s-and 3 p-bonds (b) 9 s-and 3 p-bonds
(b) Sigma bond is weaker than a p (pi) bond
(c) 9 s-and 6 p-bonds (d) 15 s-and 3 p-bonds
(c) Double bond is stronger than a single bond
11. The ratio of p- to s- bonds in benzene is
(d) Covalent bond is stronger than hydrogen bond
4. Which of the following statements is not correct for sigma- (a) 1 : 4 (b) 1 : 2
and pi- bonds formed between two carbon (c) 3 : 1 (d) 1 : 6
atoms ? 12. Which one is false in the following statements?
(a) Bond energies of sigma-and pi-bonds are of the order (a) Each carbon in ethylene is in sp2- hybridization
of 264 kJ/mol and 347 kJ/mol, respectively (b) Each carbon in acetylene is in sp-hybridization
(b) Free rotation of atoms about a sigma-bond is allowed (c) Each carbon in benzene is in sp2-hybridization
but not in case of pi-bonds
(d) Each carbon in ethane is in sp2 -hybridization
(c) Sigma-bond determines the direction between carbon
atoms but a pi-bond has no primary effect in this regard 13. The hybridization involved in six carbon atoms of benzene is
(d) Sigma-bond is stronger than a pi-bond (a) three sp3 and three sp2 (b) three sp2 and three sp
5. The percentage of s- character of the hybrid orbitals in (c) all six sp (d) all six sp2
ethane, ethene and ethyne are respectively. 14. In which of the compounds given below there is more than
(a) 50, 75, 100 (b) 10, 20. 40 one kind of hybridization (sp, sp2, sp3) for carbon ?
(c) 25, 33, 50 (d) 25, 50, 75 (i) CH3CH2CH2CH3 (ii) CH3CH = CHCH3
6. Select the molecule which has only one p-bond (iii) CH2=CH–CH=CH2 (iv) H –C º C – H
(a) CH º CH (b) CH2 = CHCHO (a) (ii) (b) (iii) and (iv)
(c) CH3CH = CH2 (d) CH3CH=CHCOOH (c) (i) and (iv) (d) (ii) and (iii)
Hybridisation & Shapes of Organic Molecules 393
15. In compound III II I
CH2 = CH– CH2– CH2 – C º CH, the C2 – C3 bond is of the CH3 - CH 2 - C º CH
type (a) I
(a) sp –sp2 (b) sp 3 –sp 3 (b) II
(c) sp –sp3 (d) sp 2 –sp 3 (c) III
16. The type of hybridizations exhibited by carbons in (d) All are equally electronegative
CH = CH – CH2 – COOH 23. In benzene, all the six C –C bonds have the same length
because of
are (a) tautomerism (b) sp2-hybridization
(c) isomerism (d) inductive effect
24. Bond length between C–C in ethane (I), ethene (II), acetylene
(a) sp3 and sp (b) sp2 only (III) and benzene (IV) follows the order:
3
(c) sp and sp 2 (d) sp3, sp2 and sp (a) I > IV > II > III (b) I > II > IV > III
17. The hybrid orbitals of carbon atoms numbered 1 and 2 in (c) I > II > III > IV (d) III > IV > II > I
the compound.
25. The bond length between sp2-hybridized carbon atoms is
1 2 (a) 1.20 Å (b) 1.34 Å
H 2 C = C = CH 2 are
(c) 1.54 Å (d) 1.40 Å
(a) sp only (b) sp and sp3 respectively 26. In which of the following, the bond length between two
(c) sp and sp respectively (d) sp2 and sp respectively
2 2
carbons is equal
18. Which of the following represents the given mode of (a) 2- Butene (b) Benzene
hybridisation sp2–sp2 – sp – sp from left to right? (c) 1- Butene (d) Propyne
(a) H2C = CH – C º N (b) CH º C – C º CH 27. The bond length between sp3 - hybridized carbon and other
CH2 carbon atom is minimum in
(a) propane (b) propyne
(c) H2C = C = C = CH2 (d) CH2
(c) propene (d) butane
19. In which of the following species, all types of hybrid carbons 28. Carbon atoms in benzene molecule are inclined at an angle
are present ? of
(a) 120º (b) 180º
(a) CH2 = C = CH2 (b) CH3–CH=CH– CH +2
(c) 109º–28 (d) 60º
(c) CH3–CºC– CH +2 (d) CH3–CH=CH– CH -2 29. Which of the following compounds is not aromatic?

20. The chemical system that is non-aromatic is + –

(a) (b)
(a) (b)

+
+
(c) (d)
+ –
(c) (d) N
30. As the s - character of hybrid orbital increases, the bond
21. The change in the state of hybridization of the asterisked angle
carbon in the following reaction. (a) increases (b) decreases
(c) does not change (d) becomes zero
* H O *
CH3 CN ¾¾¾
2 ® CH CONH
3 2 is 31. Which of the following bonds is strongest ?
(a) sp3 to sp2 (b) sp3 to sp (a) – C – C– (b) > C = C <
(c) sp to sp2 (d) sp2 to sp3
22. Which of the following carbon atoms is most |
(c) - C - C = (d) – C º C –
electronegative? | |
394 Chemistry
32. sp3-hybridization
leads to which shape of the molecule ? 34. The maximum number of carbon atoms arranged linearly in
(a) Tetrahedral (b) Octahedral the molecule,
(c) Linear (d) Trigonal planar CH3 – C º C – CH = CH2 is
33. The cylindrical shape of alkynes is due to (a) 5 (b) 4
(a) three sigma C – C bonds (c) 3 (d) 2
(b) three p C – C bonds 35. Which of the following is planar in shape ?
(c) two s C – C and p C – C bonds (a) Methane (b) Acetylene
(d) one sigma C – C and two p C – C bonds (c) Benzene (d) Isobutane

1. The Cl – C – Cl angle in 1,1,2,2- tetrachloroethene and (a) CH3C º CCH2 – CH = CHCH = CH2
tetrachloromethane respectively will be about (b) CH3CH2– CH = CHCH = CHC º CH
[CBSE PMT 1988] (c) CH3CH = CHCH2 – C º CCH = CH2
(a) 120° (b) 90° and 109.5° (d) CH3CH = CHCH2 – CH = CH = C º CH.
(c) 109.5° and 90° (d) 120° and 109.5° 8. When the hybridization state of carbon atom changes from
2. Which of the following possesses a sp-carbon in its sp3 to sp2 and finally to sp, the angle between the hybridized
structure? [CBSE PMT 1989] orbitals [CBSE PMT 1993]
(a) CH2 = CCl – CH = CH2 (a) decreases gradually
(b) CCl2 = CCl2 (b) decreases considerably
(c) CH2 = C = CH2 (c) is not affected
(d) CH2 = CH – CH = CH2. (d) increases progressively.
3. Cyclic hydrocarbon ‘A’ has all the carbon and hydrogen
9. The restricted rotation about carbon carbon double bond in
atoms in a single plane. All the carbon carbon bonds have
2-butene is due to [CBSE PMT 1993]
the same length, less than 1.54 Å, but more than 1.34 Å. The
C – C – C bond angle will be [CBSE PMT 1989] (a) Overlap of one s- and sp 2 - hybridized orbitals
(a) 109°28' (b) 100°
(b) Overlap of two sp 2 - hybridized orbitals
(c) 180° (d) 120°
4. An organic compound X (molecular formula C 6 H 7 O 2 N ) (c) Overlap of one p- and one sp 2 - hybridized orbitals
has six carbon atoms in a ring system, two double bonds and (d) Sideways overlap of two p- orbitals.
a nitro group as substituent, X is [CBSE PMT 1990] 10. Huckel's rule states that a monocyclic conjugated compound
(a) Homocyclic but not aromatic will be aromatic if it contains [CBSE PMT 1996]
(b) Aromatic but not homocyclic (a) (4n + 2p) electrons
(c) Homocyclic and aromatic
(b) (4p + 2n) electrons
(d) Heterocyclic and aromatic
(c) 4p electrons
5. The shortest C – C bond distance is found in
(d) (4n + 2)p electrons
(a) Diamond (b) Ethane [CBSE PMT 1991]
11. The structural formula of a compound is CH3 – CH = C = CH2.
(c) Benzene (d) Acetylene
The types of hybridization at the four carbons from left to
6. An sp3 hybrid orbital contains [CBSE PMT 1991]
right are [CBSE PMT 1999]
(a) 1/4 s-character (b) 1/2 s-character
(a) sp , sp , sp , sp (b) sp , sp , sp2, sp2
2 2 2 3 2 3
(c) 1/3 s-character (d) 2/3 s-character.
(c) sp3, sp2, sp, sp2 (b) sp3, sp2, sp2, sp2
7. A straight chain hydrocarbon has the molecular formula
C8H10. The hybridization of the carbon atoms from one end 12. The correct order regarding the electronegativity of hybrid
of the chain to the other are respectively sp3, sp2, sp2, sp3, orbitals of carbon is [CBSE PMT 2006]
sp2, sp2, sp and sp. The structural formula of the hydrocarbon (a) sp > sp2 > sp3 (b) sp < sp2 > sp3
would be : [CBSE PMT 1991] (c) sp < sp2 < sp3 (d) sp > sp2 < sp3
Hybridisation & Shapes of Organic Molecules 395
13. In the hydrocarbon .
15. The radical, CH2 is aromatic because it has :
CH3 – CH = CH – CH2 – C º CH
6 5 4 3 2 1 [NEET 2013]
(a) 7 p-orbitals and 6 unpaired electrons
The state of hybrization of carbons 1, 3 and 5 are in the (b) 7 p-orbitals and 7 unpaired electrons
following sequence : [CBSE-PMT 2008] (c) 6 p-orbitals and 7 unpaired electrons
(a) sp2, sp, sp3 (b) sp, sp3, sp2 (d) 6 p-orbitals and 6 unpaired electrons
16.. In which of the following species is the underlined carbon
(c) sp, sp2, sp3 (d) sp3, sp2, sp
having sp3 hybridisation? [AIEEE 2002]
14. The state of hybridization of C2, C3, C5 and C6 of the (a) CH3 COOH (b) CH3 CH2 OH
hydrocarbon, [CBSE-PMT 2009] (c) CH3 COCH3 (d) CH2 = CH –CH3

CH3 CH3 17. Which one of the following does not have sp2 hybridized
| | carbon ? [AIEEE 2004]
CH3 C CH = CH CH C º CH (a) Acetonitrile (b) Acetic acid
7 6| 5 4 3 2 1
CH3 (c) Acetone (d) Acetamide
18. In allene (C3H4), the type(s) of hybridisation of the carbon
is in the following sequence: atoms is (are) : [IIT-JEE 2012]
(a) sp3, sp2, sp2 and sp (b) sp, sp2, sp2 and sp3 (a) sp and sp3 (b) sp and sp2
(c) sp, sp2, sp3 and sp2 (d) sp, sp3, sp2 and sp3 (c) only sp3 (d) sp2 and sp3

1. The bond between carbon atom (1) and carbon atom (2) in 6. Allyl isocyanide has
1 2 (a) 9 s and 4 p - bonds
compound N º C - C H = CH 2 involves the hybridisation (b) 8 s and 5 p - bonds
(a) sp2 and sp2 respectively (a) sp3 and sp respectively (c) 9 s , 3 p and 2 non- bonded electrons
(c) sp and sp2 respectively (d) sp and sp respectively (d) 8 s , 3 p and 4 non - bonded electrons
x 7. During elimination reactions, the hybrid state of carbon
2. The compound in which C uses its sp3 - hybrid orbitals for atoms involved change as shown below:
bond formation is (a) sp3 to sp2 nature
X X
(a) HCOOH (b) (H2 N)2 CO (b) sp3 to sp nature
(c) No change in hybridised state
X X (d) Either (a) or (b)
(c) (CH 3 )3COH (d) CH 3C HO
8. In the dehydration reaction,
3. Of the following compounds which will have a zero dipole
P O
5 ® CH C º N ,
moment CH 3CONH 2 ¾¾

3
(a) 1, 1 - dichloroethylene the hybridisation state of carbon changes from
(b) Trans -1, 2 - dichloroethylene (a) sp3 to sp2 (b) sp to sp
(c) Cis - 1 , 2 - dichloroethylene (c) sp2 to sp (d) sp to sp3
(d) None of these ·
4. The number of s - and p - bonds in but- 1 - ene - 3 - yne are 9. Unpaired electron in CH3 occupies
(a) 5s - and 5p - (b) 7s - and 3p - (a) sp - hybrid orbital (b) sp3 - hybrid orbital
(c) p - orbital (d) sp2 - hybrid orbital
(c) 8s - and 2p - (d) 8s - and 4p - 10. The hybridisation of carbon atom in C - C single bond of
5. The species which use sp2 - hybrid orbitals in its bonding H - C º C - CH = CH 2 is
(a) PH 3 (b) NH 3 (a) sp3 - sp3 (b) sp2 - sp2
(c) CH 3+ (d) CH 4 (c) sp - sp2 (d) sp3 - sp
396 Chemistry
11. Which of the following represents the given mode of 15. Which of the following species would be expected to exhibit
hybridisation sp2 – sp2 – sp – sp from left to right? aromatic character ?
(a) H 2 C = CH – C º CH . –. +
(b) HC º C – C º CH
(c) H 2 C = C = C = CH 2 (i) (ii)
CH2
(d) H C . –. +
2
12. The structure of H2C = C = CH2 is
(a) linear (iii) (iv)
(b) planar
(c) non-planar (a) (i) and (iv) (b) (ii) and (iv)
(d) has several resonance structures (c) (i) and (iii) (d) (ii) and (iii)
13. Aromatic character of benzene is proved by 16. Which among the following is aromatic ?
(a) resonance theory (b) aromatic sextet theory 2– –
(c) orbital theory (d) All of these
14. Which of the following will show aromatic behaviour ?
I II III IV V
(a) I (b) II and V
(a) (b) (c) III (d) IV
17. Which of the following is not aromatic ?
+ (a) Benzene
(b) Cyclooctatetraenyl dianion
(c) Tropylium ion
(c) (d) (d) Cyclopentadienyl cation

EXERCISE 1 EXERCISE 2
1. Acetylene, due to greater electronegativity of the

sp–hybridized carbon. CH3
1º 2º 3º 4º 2º 2º 1º
2. CH4 = sp3 1. (c) CH – CH – CH – C – CH – CH – CH ;
3 2 2 2 3
C2H4 = sp2 CH3 CH3
1º 1º
C2H2 = sp
Thus there are five 1º carbon atoms.
3. (i) sp2, (ii) sp3, (iii) sp2.
4. The carbon atom x is sp2 and carbon atom y is sp3 hybridised. 1º 1º
CH3 CH3
5. Ethene (C2 H 4 ) has sp 2 hybridized carbon atoms. On 1º 4º 2º 1º
2. (b) CH3 – CH – C – CH2 – CH3 ;
hydrogenation, it changes to ethane (C2H6) in which carbon 3º
atoms are sp3 hybridized. Cl

10. (d) 11. (b) 12. (a) 13. (d) 14. (b) 15. (c) Thus all the four types of carbon atoms are present in
16. (c) 17. (a) 18. (c) 19. (d) 20. (c) 21. (b) this compound.

22. (c) 3. (b) 4. (a) 5. (c) 6. (c)


Hybridisation & Shapes of Organic Molecules 397
H H H 28. (a)
| | | 29. (c) Species (a), (b) and (d) have 2, 6 and 6 delocalized p
H - C- C = C- C- C- H electrons in a cyclic structure, hence all these are aromatic.
7. (b) | | | | | ;
H H H H H Species (c) having 4p electrons is not aromatic
2 - Pentene 30. (a) 31. (d) 32. (a) 33. (d)
34. (b) The carbon atoms attached to the triple bond lie in a
No. of s bonds = 14, No. of p bonds = 1
straight line, while the carbon atom of the CH2 group is
8. (c) 9. (d) 10. (d)
inclined at an angle of 120º. Hence, only 4 carbon atoms
are arranged in a straight line (linear arrangement).
11. (a)

No. of s bonds = 12 ; No. of p bonds = 3


\ Ratio of p : s bonds = 3 : 12 = 1 : 4 35. (c)
12. (d) 13. (d) EXERCISE 3
14. (a) In compounds (i), (iii) and (iv), all carbon atoms are sp3,
1. (d) Tetrachloroethene being an alkene has sp2 - hybridized
sp2 and sp hybridised, respectivley. However, compound
C– atoms and hence the angle Cl – C – Cl is 120° while in
(ii) has sp2 and sp3 hybridised carbon atoms;
tetrachloromethane, carbon is sp3 hybridized, therefore
sp 3 sp 2 sp 2 sp 3 the angle Cl – C – Cl is 109.5°.
CH 3 - CH = CH - CH 3
1 2 3 4 5 6 sp 2 sp sp 2
15. (d) CH = CH - CH - CH - C º CH 2. (c) CH 2 = C = CH 2
2 2 2
3. (d) All the properties mentioned in the question suggest
16. (c) In the compound, all carbon atoms except CH2 are sp2 that it is a benzene molecule. Since in benzene all carbons
hybridised. are sp2–hybridized, therefore, C – C – C angle is 120°
sp2 sp sp2
17. (d) H 2 C = C = CH2 H H
1.09Å 1.40Å 120°
18. (a) C C
sp 3 sp sp 2
19. (c) CH 3 - C º C - CH 2 + H C 120° C H
Remember that carbocations are sp2 hybridised. C C
20. (c) Only species (c) is not aromatic.
sp sp 2 H H
H O
21. (c) H 3C - C º N ¾¾2¾® H 3C - C - NH 2
|| NO 2
O NO 2
22. (a) Greater the s character, more electronegative is the carbon,
i.e. sp hybridised (having 50% s character) carbon having 4. (a) or
H atom is more electronegative in comparison to C-atom
which does not have H atom. Hence it is homocyclic (as the ring system is made of
sp3 sp 2 sp sp
one type of atoms, i.e. carbon) but not aromatic.As it
does not have (4n +2)p electron required for aromaticity.
3 C - CH 2 - C º CH
23. (b) 5. (d) Shortest C – C distance (1.20 Å) is in acetylene.
24. (a) Greater the bond order smaller the bond length. As acetylene has sp hybridisation, the bond length
Molecule H3C–CH3 > C6H6 > H2C = CH2 > HC º CH increases in the order
Bond order 1 1.5 2 3 C º C < C = C < C-C
(due to resonance) sp(1.20Å) sp 2 (1.34Å) sp(1.54Å)
25. (b) 26. (b) 6. (a) sp3 orbital has 1/4(25%) s-character. & 75% p character.
sp3 sp 3 sp 3
sp3 sp 2 sp 2 sp3 sp 2 sp sp
27. (b) (a) H3 C - CH 2 - CH3
7. (d) CH3 CH = CH CH 2 - CH– C º CH
sp 3 sp sp 8. (d) Angle increases progressively
(b) CH 3 - C º C H
sp3 ( 109°28' ), sp2 (120°), sp (180°)
sp 3 sp 2 sp 2 9. (d) Rotation around p bond is not possible. If any attempt
(c) H 3 C - CH = CH 2 is made to rotate one of the carbon atoms, the lobes of
sp3 sp3 sp3 sp3 p-orbital will no longer remain coplanar i.e no parallel
(d) CH3 - CH 2 - CH 2 - CH3 overlap will be possible and thus p-bond will break .
This is known as concept of restricted rotation. In other
Since sp orbital has the smallest size, sp3 carbon attached
words the presence of p-bonds makes the position of
to sp carbon, i.e., sp3- sp bond should have minimum
two carbon atom.
carbon-carbon single bond length.
398 Chemistry
10. (d) Huckel’s rule states that for aromaticity 6. (c) Allyl isocyanide has the structure
there must be (4n + 2) p electron present in compound ®
H 2 C = CH - CH 2 - N = C
where n is an integer it has 9 s and 3p bonds along with 2 non- bonded
sp3 sp 2 sp sp 2 electrons
11. (c) CH - CH = C = CH
3 2 7. (d) CH3 - CHX 2 ® CH 2 = CHX ® CH º CH
12. (a) Among the three given hybrid orbitals, sp hybrid orbital sp3 sp2 sp
is most electronegative. Contribution of s in sp hybrid
orbital is maximum (50%) so this orbital is closer to O
||
nucleus. Naturally it will have greater tendency to pull 8. (c) CH 3 - C - NH 2 ® CH 3 - C º N
electron towards it. Hence it becomes more ­ ­
electronegative and sp3 becomes least electronegative sp 2 sp

as it have only 25% S character. 9. (b) CH3 is sp3 hybridised and unpaired electron is present
13. (b) C – 1 is sp hybridized (C º C) in sp3 hybridised orbital
C – 3 is sp3 hybridized (C– C) 10. (c) Count s and p bonds to get answer
C – 5 is sp2 hybridized (C = C) sp 2 sp 2 sp sp
Thus the correct sequence is sp, sp3, sp2. H 2C = CH - C º CH
11. (a) ¾¾ ¾ ¾ ¾ ¾ ¾ ¾®
CH3 CH3 from left to right
3|
sp sp sp |
2 sp 3
14. (d) CH 3 C CH = CH CH — C º C H
12. (c) The hydrogen atoms on terminal C-atoms lie in
7 6| 5 4 3 2 1 perpendicular planes and hence the molecule as a whole
CH3 is non-planar. H2C = C = CH2
15. (d) Presence of 6p orbitals, each containing one unpaired 13. (d)
electron, in a six membered cyclic structure is in
accordance with Huckel rule of aromaticity.
16. (b) In molecules (a), (c) and (d), the carbon atom has a 14. (b)
multiple bond, only (b) has sp3 hybridization.
O Non-aromatic
sp3 || sp3 (unconjugated 6p electrons)
17. (a) H3C - C - CH3
sp 2
Acetone

O O
sp3 || sp3 sp sp3 ||
; CH3 - C - OH ; CH3 - C º N ; C H 3 - C - NH 2
2
sp 2
sp
Aromatic Alicyclic
Acetic acid Acetonitrile Acetamide (conjugated 6p electrons)

sp 2 sp sp 2 +
18. (b) Allene (C3H4) is H 2 C = C = CH 2

EXERCISE 4
1. (c) The C1 forms 2s bonds and 2p - bonds (sp-hybridisation) Antiaromatic
(conjugated 4p electrons)
and C2 forms 3s bonds and 1p bond (sp2 hybridisation)
x 15. (d) (ii) and (iii) have delocalized six p electrons and hence
2. (c) See the number of s bonds formed by C in each case. In
x x x x these are aromatic; (i) has 8 conjugated p electrons, while
HCOOH , (H 2 N)2CO and CH3C HO, C forms 3s (iv) has 4 conjugated p electrons.
bonds and 1 p bond, hybridisation is sp 2 . In 16. (a) Only structure I has 8 + 2 = 10p electrons, hence it is
x x aromatic; II has 4, III has 8, IV has 9 and V has 4 electrons,
(CH3 )3C OH , C forms 4s bonds, hence hybridisation thus II to V are not aromatic.
is sp3 17. (d) No. of delocalized p electrons in benzene (a),
s- cyclooctatetraenyl dianion (b), and tropylium ion (c) are
Cl H
3. (b) C= C s- has zero dipole moment 6, 10 and 6 respectively. Further all the three species are
H Cl cyclic, hence all of these are aromatic. Cyclopentadienyl
cation (d) has 4 p electrons, hence it is not aromatic but
4. (b) H2C = CH- C º CH has 7s and 3p actually it is antiaromatic.
+
5. (c) CH3 is sp2 hybridised
12D
General Organic
(Basic Concepts)
ELECTRONEGATIVITY Hence transmission can be ignored after the second C-atom. An
A covalent bond, where the electrons are shared equally is called atom or group which attracts electrons more strongly than
a non-polar bond (eg H–H) and an unequal sharing of the pair of hydrogen is said to have a negative inductive effect (–I). An atom
bonding electrons results in a polar bond. The unequal sharing of or group which attracts electrons less strongly than hydrogen is
electrons is due to the ability of an atom to attract electrons said to have a positive inductive effect (+I).
towards itself which is known as Electronegativity. + +
Elements with higher electronegativity values have greater N R 3 > N H 3 > NO 2 > CN > COOH > F > Cl > Br > I
attraction for bonding electrons.
> OAr > OR > OH > C º CR > Ar > H
H
2.2 ¬
¾¾ Increasing – I effect (Electrons attracting )
Li Be B C N O F
1.0 1.6 1.8 2.5 3.0 3.4 4.0 O - > COO - > CR 3 > CHR 2 > CH 2 R > CH 3 > D > H
Na Mg Al Si P S Cl
0.9 1.3 1.6 1.9 2.2 2.6 3.2 ¬
¾¾ Increasing + I effect (electrons repelling)
Br Inductive effect does not change the covalency. The more the
3.0 inductive effect between a bond, the more is the ionic character of
I the bond.
2.7 Applications :
Electronegativity increases from left to right and decreases from (I) Acid character of Acids : Formic acid is stronger than acetic
top to bottom. acid.
INDUCTIVE EFFECT (I)
O d+ O
The displacement of shared pair of electrons towards the more H–C CH3 C d–
electronegative atom in a molecule is called inductive effect. It is O–H O–H
a permanent effect e.g. Formic acid Acetic acid
d+ d- The oxygen atom in acetic acid holds the hydrogen atom
H ® Cl more tightly after acquiring negative charge due to +I effect
• It develops polarity in a bond or molecule. of methyl group. Hence it is less ionised
• It is transmitted along a chain of atoms but the intensity Acid character of halogens substituted acids. Chloro
goes on decreasing with the increase in the size of chain. For substituted acetic acids follow the following order for acid
example,
character.
d+
C ® C ®® C ®®® X d -
400 Chemistry
(II) Reactivity of Alkyl halides. It follows the following order
d–
Cl R R
O
d– –
Cl C C O H R C Cl R C Cl
d–
Cl R H
3° 2°
Trichloro acetic acid
H H
d– R C Cl H C Cl
Cl
O
d– – H H
Cl C C O H 1°

H Due to +I effect the intermediate carbonium ions (R 3C+ )


are stabilised in the order t > s > p > methyl. Hence the order
Dichloro acetic acid
of reactivity of alkyl halides decreases from right to left.
H O O (III) Basic character of Amines. It follows the order
d– – d d+ d–
Cl C C O H CH3 C O–H R H

H R .N. H R N:
.. H
Monochloro acetic acid Acetic acid
¬
¾¾ Increasing acid character 2ºamine 1ºamine

The O – H bond is more ionic in nature in trichloro acetic acid R H


and its ionic character decreases from right to left. or
R .N. R H N
.. H
Dispersal of the negative charge after ionisation decreases
from left to right which is the cause of decreasing the acid
2ºamine 1ºamine 3ºamine ammonia
character.
..
Cl Although electron density on nitrogen is maximum in R 3 N
O
– +
Cl C C O +H but due to steric hinderance it is less basic.
(IV) Inductive effect and dipole moment :
Cl Inductive effect leads to a dipole moment. The measured
Dissociation constants (×10–5) for acids dipole moments of some alkyl halides are given below
CH3Br C2H5Br (CH3)2CHBr (CH3)3CBr
Acetic Monochloro Dichloro Trichloro
1.79 1.88 2.04 2.21
acid acetic acid acetic acid acetic acid
CH3I C2H5I (CH3)2CHI (CH3)3CI
1.8 155 500 13,000
1.64 1.78 1.84 2.13
Monofluoro Monochloro Monobromo Monoiodo
+I effect increases from –CH3 to –C(CH3)3 gp. –I effect of Br
acetic acid acetic acid acetic acid acetic acid
is more than I.
217 155 138 75
(V) Estimation of percentage ionic character of bonds :
Inductive effect of halogens F > Cl > Br > I % Ionic character of covalent bond
Dissociation constant (×10 –5 ) of a, b and g mono
Observed D.M.
chlorobutyric acids = ´ 100% .
Calculated D.M.
n-Butyric acid a-Monochloro butyric acid
1.5 139 INDUCTOMETRIC EFFECT
b-Monochloro butyric acid g-Monochloro butyric acid d+ d–
8.8 3.0 Consider the inductive effect in a bond A ®— B . When some
The transmission of the inductive effect along a chain of negatively charged ion approaches A, the inductive effect between
carbon atoms weakens as the chain gets longer. A – B is temporarily increased which is known as inductometric
effect.
General Organic (Basic Concepts) 401
ELECTROMERIC EFFECT (E)
It involves the complete transference of p pair of electrons to one
of the atoms joined a multiple bond. It is temporary effect and
takes place at the requirement of attacking reagent. Consider the Resonance Hybrid
addition of HCl to propene. Feature of resonance
(i) Resonance is a permanent effect.
(ii) It involves the delocalisation of electrons, lone pair of
electrons and p pair of electrons.
+ Nucleophilic (iii) The number of unpaired (not lone pairs) electrons must stay
CH3 – CH – CH3 – CH 3 - C H - CH 3 the same.
attack of Cl |
Secondary carbonium ion Cl (iv) Resonating structure with lowest energy contributes more
2- Chloroprop ane
towards resonance.
Addition takes place according to Markownikoff's rule which
states that the negative portion of attacking reagent goes to carbon (v) Negative charges are most stable on electronegative atoms.
atom containing lesser number of hydrogen atoms. The reason (vi) Resonating structures with maximum bonds and little charge
for this is that it results in the formation of more stable intermediate contribute more.
secondary carbonium ion. (vii) Real structure resemble the major contributor more than the
PEROXIDE EFFECT, KHARASCH EFFECT : minor contributor.
(viii)All the atoms participating in resonance must lie in the same
In presence of oxygen or peroxide the addition of HBr to
plane or must atleast nearly do so.
unsymmetrical alkene takes place anti to Markownikov's rule
which is known as peroxide effect or Kharasch effect. Applications of resonance :
I. Acid character of phenols : Phenols are acidic and Alcohols
(I) O 2 or peroxide ¾¾® R (free radical)
(II) H - Br + R ¾¾® R - H + Br } Chain initiation Steps are neutral.
.. ..
• :O – H :O:
}
(III) R - CH = CH 2 + Br • ¾¾® R - CH - CH 2 Br Chain
• propagation
(IV) R - CH - CH 2 Br + HBr ¾¾® R.CH 2 .CH 2 Br + Br •
steps +
+ H
(V) Br + Br ¾¾® Br2
Phenol Phenoxide ion
(VI) 2R C H - CH 2 Br ¾¾® R - C H - CH 2 Br Chain Resonance stabilisation of Phenoxide ion
|
R - CH - CH 2Br termination
steps ..
(VII) R C H - CH 2 Br + Br ¾¾® RCHBrCH 2 Br :O: :O: :O:

·
The attack of Br on terminal carbon atom (see step III) results in
..
the formation of more stable secondary free radical. This is the
reason that addition takes place anti to markownikoff's rule. HCl ..

and HI do not show peroxide effect. HCl does not give Cl atoms
and HI gives molecular I2. ..
RESONANCE :O: :O: :O: :O: :O:
Representation of certain molecules by various electronic ..
configurations is known as Resonance. Electronic configurations
differ only in location of electrons, the atoms must stay in the
same conditions. e.g.

Resonance stabilisation of Phenol molecule :


.. .. ..
:O – H O–H O–H
(I) Benzene (II) ..

The real structure is a combination of the resonance forms and is


called Resonance hybrid. ..
402 Chemistry

.. .. V. Resonance and bond lengths :


O–H :O – H
..
Benzene

Normal C – C = 1.54 Å, C = C = 1.34 Å,


All C – C bonds in benzene 1.39Å.
Resonance stabilisation of phenoxide ion is more than the

Resonance stabilisation of Phenol molecule itself. Hence O O
Phenol will ionise to give phenoxide and H+ ions. Carboxylate ion H – C H– C
II. Basic character of amines : Aromatic amines are less basic O O –
than aliphatic amines
Normal C = O = 1.22Å, C – O = 1.4Å,
.. All C – O bonds in carboxylate ion = 1.28Å
NH2 NH2 NH2
.. O O
Nitro group – N –N
O O
..
Aniline Normal N – O = 1.36Å, N = O = 1.15Å
All N – O bonds in nitro group 1.21 – 1.23 Å
VI. Resonance and bond order : It is obtained by the following
NH2 :NH2 relation.
.. Total number of bonds between two atoms
Bond order =
Total number of major resonating forms

(a) NO 3- ion
Due to resonance the unshared pair of electrons present on
nitrogen atom is delocalised within benzene nucleus and not
available for protonation (to accept H+). Hence basic
character is suppressed.
III. Stability of cations : Methoxy methyl cation is more stable
by 76 kcal/mole than methyl Cation.
4
H H B.O. = = 1.33
.. | Å | 3
CH 3 - O
. . - C- H ¬¾® CH 3 - O
. . = C- H
Å
(b) CO3- - ion
More stable (every atom has octet)

IV. Unexpected addition products : – –


O O O
+ – –
H / I- C O C O C O
CH 2 = CH - Cl – –
O O O
..
CH 3 - CH - Cl
. . :¾¾® CH 3 - CHI - Cl 4
Å B.O. = = 1.33
(A)
3

+ .. VII. Resonance and Dipole moment : Resonance affects the


CH 2 - CH 2 - Cl
. . :¾
¾® ICH 2 - CH 2Cl Dipole moment. D.M. of ethyl chloride is 2.05 Debye.
(B) d+ d–
In vinyl chloride CH2 CH Cl
A is more stable than B Inductive effect
Å .. Å .. - +
CH 3 - CH - Cl
.. :¬¾® CH 3 - CH = Cl : CH 2 = CH - Cl
.. : ¬
¾® CH 2 - CH = Cl
..
:
..
Every atom has octet Resonance
General Organic (Basic Concepts) 403
Inductive and resonance induced D.M. operate in opposite
+
direction, hence value is 1.44 Debye. CH 3 - CH - CH = CH 2
Electrophilic Nucleophilic
In chlorobenzene Inductive and Resonance induced D.M. –
operate in opposite direction. + attack of Br
CH 3 - CH = CH - C H 2
d– +
d+ Cl : Cl
-80 °
¾¾¾® CH 3CHBr.CH = CH 2 + CH 3 - CH = CHCH 2 Br
1,2 Addition 80% 1,4 -addition 20%
Inductive effect Resonance 40%
The value of DM is 1.55 Debye. 40°
¾¾¾® CH 3 - CHBrCH = CH 2 + CH 3 - CH = CHCH 2 Br
VIII.Resonance Energy (E) : It is given by the equation 1,2Addition 20% 1,4-addition 80%

ER = E0 – EC
ER = Resonance energy, E0 = Observed heat of formation (II) Extra stability : Each C-atom in 1, 3-butadiene is sp2
and EC = Calculated heat of formation of the most stable of hybridised and contains one pz atomic orbital parallel to each
the resonating structures. other and perpendicular to the plane of hybrid atomic orbitals.
In case of unsaturated compunds, Resonance energy is the By sidewise overlapping these pz atomic orbitals form a
difference between a measured and calculated heat of delocalised p molecular orbital which provides the extra
hydrogenation e.g., stability to the molecule.

E R (CO 2 ) = E 0 (383 kcal) = E C (346 kcal) = 37 kcal / mol


C1– C2 C1– C2
C3– C4 C3– C4

+ 3 H2 + 49.8 kcal. (Observed) delocalised p MO

Benzene Cyclohexane
(III) Bond length : Conjugation affects the bond length. The C2 –
C3 bond length in 1,3-butadiene is 1.47 Å and C1 – C2 bond
Calculated value of heat of hydrogenation of benzene length is 1.35Å due to conjugation.
= 28.6 × 3 = 85.8 kcal/mole, RE = 36.0 kcal / mole. (IV) Heat of hydrogenation :
The greater the RE, the more is the stability. Calculated heat of hydrogenation of 1,3 butadiene
MESOMERIC EFFECT (ME) : = 28.6 × 2 = 57.2 kcal.
It is a permanent effect and similar to electromeric effect. Like Observed heat of hydrogenation of 1,3-butadiene
Inductive effect it may be +ME or –ME. = 53.7 kcal.
+ME atoms or groups donate electrons to the double bond or R.E. = 57.2 – 53.7 = 3.5 kcal/mole
conjugated system e.g. –Cl, –Br, –I, NH2, –NHR, –NR2, –OH, – Thus due to conjugation 1,3-butadiene is stabilised by 3.5
OR, –SH, –SR etc. kcal/mol.
.. – + HYPERCONJUGATION :
CH2 = CH – Cl:
.. CH2 – CH = Cl
. .:
Introduced by Baker and Nathan (1935). The electron release by
–ME atoms or groups withdraw electrons eg. –NO2, CN, COOH, C–H bond by the effect similar to electromeric effect is known as
CHO, HSO3. hyperconjugation. It is a permanent effect.
Å ..
CH 2 = CH - C º N ¾
¾® C H 2 - CH = C = N
.. H
| H+ ..
- C - C = C- ¬¾® - C = C - C-
CONJUGATION : | | | | |
The compounds containing alternate single and double bonds (I) (II)
Hyperconjugated forms
are known as conjugated compounds. Such compounds exhibit
certain abnormal properties due to interaction between single and Since there is no apparent bond between C and H + , the
double bonds, known as conjugation hyperconjugation is also known as No bond Resonance. The
(I) Abnormal addition reactions : Addition of HBr to 1,3- magnitude of inductive effect and hyperconjugation follows the
butadiene. order.
–CH , - C H , - CH (CH ) , - C ( CH )
Increasing Hyperconju gation ¬¾ ¾3 ¾ ¾
2¾5 ¾¾ ¾¾
3 2¾ ¾ ¾ ¾

H + / Br -
CH 2 = CH - CH = CH 2 ¾¾ ¾¾®
Electrophilic Increasing Inductive effect
+
attack of H
404 Chemistry
Effects of hyperconjugation : Types of reagents :
(I) Heat of hydrogenation of Substituted Olefins : (I) Electrophilic reagents : They have high affinity for electrons
+ + + + + + ..
The greater the number of H.C. forms the more is the stability. H + , H 3O + , N O 2 , H S O 3 , X (Cl, Br, I), R + , R C O, NO, - N º N
Compound Heat of hydrogenation They may be neutral in nature also AlCl3, BF3, ZnCl2, FeCl3,
CH2 = CH2 Ethylene 32.8 kcal/mol SnCl4 etc.
CH3–CH = CH2 Propylene 30.1 kcal/mol (II) Nucleophilic reagents : Electron rich species and have
(Due to H.C. forms) affinity towards nucleus (which is positively charged).

CH 3 CH 3 - OH - , RO - , X - (Cl, Br, I), RCOO - , NH 2- , NO 2- , SH


| | SH - , CN - , C 6 H 5 O - , R - C º C -
CH 3 - C = C - CH 3 Tetramethyl ethylene 26.6 kcal mol. Neutral nucleophiles are capable of donating a pair of
(Due to more H.C. forms) electrons e.g.
(II) Bond lengths : Dimethyl acetylene .. .. .. .. .. ..
. ., R - O
H2 O . . - H, N H 3 , R - S
. .- H, R N H 2 , R - O
. . - R , R - Mg - X
H H H
| | H+ | (III) Nucleophilicity is defined by the rate of attack on an
H - C - C º C - C - H ¬¾® H - C = C = C - C - H electrophilic carbon atom.
|1 2 3 | 4 | 1 2 3 | 4
1. Species with negative charge are stronger nucleophiles
H H H H
than analogous species without a negative charge.
Normal C–C = 1.54Å; C1 – C2 = 1.46Å OH - > H 2 O, SH - > H 2S, NH 2- > NH 3
(III) Hyperconjugation and dipole moment : 2. Nucleophilicity decreases from left to right across the
Calculated dipole moment of nitro methane is 2.59 Debye periodic table eg.
and the observed value is 3.15 Debye. OH - > F - , NH 3 > H 2 O
H – 3. Nucleophilicity increases down the periodic table.
| O H+ O
H - C- N ¬¾® H - C = N I - > Br - > Cl - > F - , HSe - > HS - > HO -
| O | O Reaction intermediates :
H H +
(I) Carbocations (Carbonium ions) C . These are the
Cleavage of covalent Bond : species carrying positive charge on the carbon atom, which
(I) Heterolytic cleavage / heterolysis : shared pair of electrons is sp2 hybridised, with planar structure. The vacant p-orbital
is retained by one atom lies perpendicular to the plane of the other atoms. They are
strong electrophiles. They are stabilised by alkyl substituents
Å by
A - B or A : / B ¾
¾® A : + B (a) Inductive effect :
R R R H
+ + + +
or A - B or A / : B ¾ Å
¾® A + :B C C C C
R R R H H H H H
Species carrying negative charge are known as anions; they 3° 2° 1° methyl
stability
are rich in electrons hence nucleophilic in nature. In a (b) Hyperconjugation : Partial overlapping of filled orbitals
chemical reaction such species always attack at the point of with empty ones
low electron density. Species carrying positive charge are H
known as cations, they are electrons deficient hence | Å H+
electrophilic in nature. In a chemical reaction they attack at H - C - CH 2 ¬¾® H - C = CH 2
| |
the point of high electron density. H H
(II) Homolytic cleavage / homolysis : The atoms retain one H H
electron each. + | |
¬¾® H C = CH 2 ¬¾® H - C = CH 2
| H+
A - B or A : B ¾
¾® A° + B° H
(c) Resonance Stabilisation : Unsaturated carbocations are
The resulting species are neutral, contain at least one stabilised by Resonance. e.g.
unpaired electron hence known as free radicals, electron Allyl Carbonium ion
deficient hence electrophilic in nature, very reactive, + +
paramagnetic in nature, hydrogen abstractor. Homolytic CH 2 = CH - CH 2 ¬¾® CH 2 - CH = CH 2
cleavage usually occurs in non polar bonds at high Benzyl Carbonium ion
temperature or in presence of UV radiations.
General Organic (Basic Concepts) 405
+ (b) Resonance Stabilisation :
CH2 CH2 CH2
Allyl Carbanion :

.. ..
CH 2 = CH - C H 2 ¬¾® C H 2 - CH = CH 2
Benzyl Carbanion
+
CH2 CH2
:CH2 CH2 CH2

:
The order of stability of different carbonium ions
+ + +
(C 6 H 5 ) 3 C > (C 6 H 5 ) 2 C H > (C 6 H 5 ) C H 2 >
+ + + + + CH2 :CH2
CH 2 = CH . CH 2 > R 3 C > R 2 CH > R CH 2 > C H 3
:

(II) Reactions of carbocations :


(a) Combination with a nucleophile :
+
H O Resonance stabilisation is more effective than other factors.
MeCH 2 - C H 2 ¾¾2¾® MeCH 2 CH 2 OH
(C 6 H 5 ) 3 C - > (C 6 H 5 ) 2 C H > C 6 H 5 - C H 2 >
(b) Elimination of Proton :
+ CH 2 = CH - C H 2 > C H 3 > C H 2 R > C HR 2 > C R 3
¾® MeCH = CH 2 + H +
MeCH 2 - C H 2 ¾
¬
¾¾ Stability
(c) Rearrangement : If 1,2-shift of hydrogen or alkyl can
form a more stable carbocation, then such a (IV) Free radicals C• : They are sp2 hybridised and planar..
rearrangement takes place The perpendicular p-orbital contains an odd electron. They
H lack in octet hence electrophilic in nature. The order of stability
| + + is
CH 3 - C - C H 2 ¾
¾® CH 3 - C H - CH 3
| (a) Inductive effect :
H
1° cation 2° cation R R R H
C° C° C° C°
CH 3 CH 3 R R R H H H H H
3º 2º 1º methyl
| + |
CH 3 - C - C H 2 ¾
¾® CH 3 - C - CH 2 CH 3 stability (Inductive stabilisation)
| +
CH 3 (b) No bond resonance stabilisation / hyperconjugation:
3° cation
1° cation
H
– | g Hg
(III) Carbanions (carbo anions) C : The species carrying H - C - CH 2 ¬¾® H - C = CH 2
| |
negative charge on the carbon atom which is sp3 hybridised H H
and tetrahedral. They are nucleophilic in nature and their
structure resembles an amine. The stability order is
H H
(a) Inductive effect : | |
g
¬¾® H C = CH 2 ¬¾® H - C = CH 2
H H R R | Hg
| | H
H– C: R– C: R– C: R– C:
| | | (c) Resonance stabilisation :
H H H R Allyl free radical
1º 2º 3º
g g
Methyl Stability
CH 2 = CH - CH 2 ¬¾® CH 2 - CH = CH 2
+ I effect of R increases electron density on C making it less Benzyl free radical
stable.
406 Chemistry

CH 2 CH2 CH2 2pz


H
Triplet carbenes : C C
H

CH2 CH2
Multiplicity is A = 2 × 1 + 1 = 3

æ 1 1 ö
ç S = + = 1, Two unpaired electrons have parallel spin ÷
è 2 2 ø
Order of stability of various free radicals
Triplet is more stable than singlet.
g g g g g g g g
(C6 H5 )3 C > (C6 H 5 ) 2 C H > C6 H 5 CH 2 > CH 2 = CH - CH 2 > (CH 3 )3 C > (CH 3 ) 2 CH
. . > CH 3 CH 2 > CH 3
Nitrenes : R - N
. . Nitrenes are nitrogen analogs of carbenes,
g g g g g g
C6 H5 CH 2 > CH 2 = CH - CH 2 > (CH3 )3 C > (CH3 ) 2 CH > CH3 CH 2 > CH 3
nitrogen is sp2 hybridised.
¬
¾¾ Stability
.. .. ..
D or hv
Generation R - N
..- N º N
. . ¾¾ ¾¾® R - N
. . + N2
CARBENES C : These are uncharged reactive intermediates
Azide
that contain a divalent carbon atom which is sp2 hybridised. There
is a perpendicular vacant p-orbital. Structure singlet :
Generation :
2pz
U.V.
(I) CH 2 N 2 ¾¾¾® : CH 2 + N 2
Diazomethane Carbene
R R
N OR N N
U.V.
(II) CH 2 = C = O ¾¾¾® : CH 2 + CO
Ketene

OH - Multiplicity A = 2s + 1 = 2 × 0 + 1 = 1
(III) CHCl3 ¾¾¾® : CCl 2 + HCl
Chloroform Dichloro carbene

Singlet carbenes :
R
2pz Triplet : N OR N

C Or C

Multiplicity A = 2s + 1 = 2 × 1 + 1 = 3
Arynes : The derivatives of benzyne are called arynes :
°

Z
H H
C Or C Benzyne Aryne
H H
Generation :
H
NH2 –
+ NH3 + Cl
æ 1 1 ö
Multiplicity is A = 2s + 1 = 2 × 0 + 1 = 1 çs = + 2 - 2 = 0÷ Cl
è ø
General Organic (Basic Concepts) 407
Types of reactions : (III) Addition reactions :
(I) Substitution reactions or replacement reactions : (a) Electrophilic Addition reactions, initiation by electrophile
(a) S N 1 : Substitution nucleophilic unimolecular reactions : e.g.
Such reactions take place in two stages H + / X-
+
X-
R - CH = CH 2 ¾¾ ¾¾® R - C H - CH 3 ¾¾®
¾ R - C H - CH 3
Slow + - + – Fast First attack
R – X ¾¾¾® R + X ; R + Y ¾¾
¾® R - Y by erectrophile
|
X
(b) S N 2 : Substitution nucleophilic bimolecular reactions :
(b) Nucleophilic addition reactions, initiation by nucleophile
d- d+ d- e.g.
Y - + R - X ¾¾¾®[ Y × × × × × R × × × × × X ] ¾¾
¾® Y - R + X -
Slow Fast
Transition State
H + / CN - H+
¾® > C - O - ¾¾
> C = O ¾¾ ¾ ¾ ¾® > C - OH
First attack by | |
SN 1 SN 2 Nucleophile
CN CN
• 3°> 2°> (1° and CH3 X do not go) CH 3 X >1°> 2°> 3°
(c) Free radical addition reactions, initiation by free radical
• Strength of nucleophile Strong nucleophiles required
Peroxide
not important R - CH = CH 2 + H · / Br · ¾¾¾¾® R - CH 2CH 2 Br
• Good ionising solvent May go faster in less polar (IV) Rearrangement reactions :
required solvent
• Rate = K [RX] Rate = K [RX] [Nu–] NHOH NH2
• Possible rearrangements No rearrangements
• Lead to recemisation Lead to inversion.
(c) S N 1 : Substitution nucleophilic internal. Example
Phenyl hydroxylamine OH p-amino phenol
ROH + SOCl 2 ¾
¾® RCl + SO 2 + HCl O
NH4CNO NH–
2 C–NH2
ROH + SOCl 2 ¾
¾® RO.SOCl + HCl
Amm. Cynate Urea

Slow + Fast
ROSOCl ¾¾¾® R + O ¾¾
¾® R - Cl + SO 2 (V) Polymerisation reactions :
S=O
Cl nCH 2 = CH 2 ¾¾
® -(CH 2 - CH 2 -) n
Ethylene Polythene
(II) Elimination reactions :
DIRECTIVE INFLUENCE OF ATOMS AND GROUPS :
(a) E1 : Elimination unimolecular (For electrophilic substitution reactions)
When monosubstitution product of benzene is converted into di-
X substitution product, the position of second incoming group is
| | + |
- C - C - ¾¾¾® X - + - C- C - ¾¾ ¾
Slow :B Fast decided by the atom or group already present in the benzene
¾® > C = C < + B : H
| | | | nucleus. This is known as directive influence of atoms and groups.
H H
A carbocation Directive influence is governed by three effects :
(I) Inductive effect (I) (II) Electromeric effect (E)
(b) E2 : Bimolecular elimination (III) Resonance (M)
Any effect that pushes the electrons towards the benzene nucleus
X is taken as positive and activates the benzene nucleus for further
| | substitution. The effect that pushes the electrons away from
- C - C - ¾¾® X - + > C = C < + H : B
:B
benzene nucleus is taken as negative and deactivates the benzene
| |
H nucleus for further substitution.
Here we will consider Inductive effect and Mesomeric effect
E1 (comparison) E2 (Resonance) to decide the directive influence of atoms and groups.
The electromeric effect is similar to Mesomeric effect and always
• Good ionising solvent Solvent polarity not so
operate in the same direction, the only difference is the former is
required important temporary and latter is permanent.
• Base strength not Strong bases are required (I) Directive influence of OH group :
important (a) Inductive effect (I)
OH
• Rate = K [RX] Rate = K [RX] [B–]
• Saytzeff orientation Saytzeff orientation –I deactivation
• Rearrangements are No rearrangements
common
408 Chemistry
(b) Mesomeric effect (M) (b) Mesomeric effect (M)
:O – H O–H O–H
C N C N C N
+

O–H O–H :O – H
C N C N
+M
+ +
activation –M
deactivation

Ortho and para positions become the points of high The o,p-positions become the points of low electron
electron density as + M > > – I. The electrophilic reagent density, therefore the electrophilic reagent will attack at
will attack at o- and p- positions. Hence OH gp. is o, p- the m position. Hence CN is meta directing in nature
directing in nature with activation of benzene nucleus.
with deactivation of benzene nucleus.
.. .. .. ..
Other examples are - NH 2 , - NHR, - NR2 , - OR Other examples are : - NO 2 , COOH , HSO 3 , CHO
..
(IV) Directive influence of –Cl atom :
(II) Directive influence of –CH3 group :
(a) Inductive effect (I)
(a) Inductive effect (I) ..
:Cl:
CH3
– I deactivation
+ I activation

(b) Mesomeric effect (M)


(b) Hyperconjugation (HC)
.. .. ..
:Cl: :Cl :Cl +
H H H
..
H–C–H H–C H H–CH

..
:

.. .. .. ..
:Cl + :Cl :Cl:

H H +M
H–CH H–C–H activation
..
+ HC
activation By mesomeric effect the o,p positions become the points
of high electron density. Further –I > +M, hence Cl is
The o, p-positions become the points of high electron o, p directing in nature with deactivation of benzene
density. The electrophilic reagent will attack at o- and p- nucleus.
positions. Hence methyl group is o,p-directing in nature Other examples are : –F, Br, I
with activation of benzene nucleus.
EASE OF ELECTROPHILIC SUBSTITUTION OF
Other examples are : -C 2 H 5 , - C3 H 7 , - C 4 H 9 etc. BENZENE AND ITS DERIVATIVES :
(III) Directive influence of –CN group : 1. Strongly activating (o, p directing) :
(a) Inductive effect (I) .. .. ..
CN
- N H 2 , - NHR, - N R 2 , - O
..
H
2. Moderately activating (o, p directing) :
– I deactivation
.. .. ..
-O
..
CH 3 , - O C H , -N
.. 2 5 ..
HCOCH3
General Organic (Basic Concepts) 409
3. Weakly activating (o, p-directing) : (IV) Friedel Craft's alkylation
-CH 3 , - C 2 H 5 , - C 3 H 7 , C 6 H 5 ¾® R + + AlCl -4
RX + AlCl 3 ¾ Electrophile R+
4. Benzene itself. (V) Friedel crafts acylation
5. Deactivating (o, p - directing) : F, Cl, Br, I +
¾® R C O + AlCl 4-
RCOCl + AlCl3 ¾
6. Deactivating (m-directing) : +
+ Electrophile R C O
NO 2 , - N (CH 3 ) 3 , CN , COOH , COOR , SO 3 H, CHO, - COR
Again NUCLEOPHILIC SUBSTITUTION OF BENZENE
It does not occur with benzene itself, but it does occur with some
NH 2 > OH > OCH 3 > NHCOCH 3 > C 6 H 5 > CH 3 substituted benzenes.
¬
¾¾ activation of benzene nucleus

O2N + N(C6H5)2
COMMON ELECTROPHILIC SUBSTITUTION
REACTIONS
(I) Nitration –
O2N N(C6H5)2 + H
+ +
® NO 2 + H 3O + 2HSO -4
HNO3 + 2H 2SO 4 ¾¾
+
Electrophile NO 2 –
O2N + OH
(II) Sulphonation
® HSO 4- + H 2 O + HS+ O3
2H 2SO 4 ¾¾
+ –
Electrophile HSO 3 or SO3 O2N OH + H
(III) Halogenation
H– much less stable hence some oxidising reagent with which H–
¾® FeCl 3 X - + X +
X 2 + FeCl 3 ¾ Electrophile X +
can react facilitate the nucleophilic substitution.
410 Chemistry

Very Short/ Short Answer Questions Long Answer Questions


1. Is neopentyl radical, 4°? 13. What are reaction intermediates ? How are they generated
by bond fission ?
2. Arrange the following:
14. (i) Select electrophiles out of the following:
(i) –NO2, –COOH, –F, –CN, –I, in increasing order of–I
H+, Na+, Cl–, C2H5OH, AlCl3, SO3, CN–, CH3CH2+,:
effect.
CCl2, R–X
(ii) –CH 3, –D, –C(CH 3 ) 3, –CH(CH 3 ) 2, –CH 2CH 3 in
(ii) Select nucleophiles from the following:
decreasing order of +I effect. BF3, NH3, –OH, R–X, C2H5OH.
3. What do you mean by (i) Homolytic fission (ii ) Heterolytic
fission. 15. Categorise the following species as electrophiles or
4. Arrange the following: nucleophiles

g
HS– , BF3, ROH
g g
(i) C6H5 CHCH3 , C6H5 C HCH = CH2, C6H5CH2CH 2 , Multiple Choice Questions
g
C6H5 C(CH 3) 2 in order of increasing stability.. 16. Select the most stable carbocation from amongst the
following
(ii) CH3CH2+, C6H5CH2+, (CH3)3C+, CH2 = CHCH2+ in
order of decreasing stability. +
(a)
(iii) HC º C–, CH2 = CH–, CH3CH2–, CH3–, (CH3)2CH–,
C6H5CH2– in order of increasing stability. (b)
5. What are carbocations (carbonium ion)? Discuss their types. +
6. What are free radicals ? Discuss their types.
7. How does (i) an electron withdrawing group (EWG) and (ii)
an electron donating group (EDG) influence the acid strength (c) +
of carboxylic acid?
8. Which of the following species behaves as (i) a nucleophile,
(ii) an electrophile, (iii) both, or (iv) neither? (d) +
....I ..– , H N, BeCl , NO+ , CH C º N :,
.. 3 2 2 3 17. What is the correct order of decreasing stability of the
... following cations ?
H , H C = C., CH
2 2 4 Å
9. What are carboanions? Discuss their types. I. CH3 — CH— CH3
10. The structure of triphenylmethyl cation is given below. This
Å
is very stable and some of its salts can be stored for months. II. CH3 — CH— OCH3
Explain the cause of high stability of this cation.
Å
III. CH3 — CH— CH 2 — OCH3
(a) II > I > III (b) II > III > I
C (c) III > I > II (d) I > II > III
18. The order of decreasing stability of the carbanions
(CH3 )3 C - (I) ; (CH3 ) 2 CH - (II) ; CH3CH -2 (III);
11. Identify the most stable species in the following set of ions C 6 H 5 CH -2 ( IV ) is
giving reasons: (a) I > II > III > IV (b) IV > III > II > I
+ + + + (c) IV > I > II > III (d) I > II > IV > III
(i) CH 3 , CH 2 Br , CH Br2 , CBr3 19. The most stable free radical among the following is
– – – – · ·
(ii) CH 3 , CH 2Cl , CHCl2 , CCl3 (a) C6 H 5 CH 2 CH 2 (b) C 6 H 5 CHCH 3

12. Why does SO3 act as an electrophile? · ·


(c) CH3CH 2 (d) CH 3CHCH 3
General Organic (Basic Concepts) 411
20. Which of the following is not a nucleophile? 22. Which of the following is an electrophile ?
(a) CN– (b) OH– (a) Lewis acid
(b) Lewis base
(c) NH3 (d) BF3 (c) Negatively charged species
21. The kind of delocalization involving sigma bond orbitals is (d) None of the above
called 23. Which of the following pairs represent electrophiles?
(a) Inductive effect (a) AlCl3, H2O (b) SO3, NO2+
(b) Hyperconjugation effect (c) BF3, H2O (d) NH3, SO3
(c) Electromeric effect 24. Electromeric effect is a
(a) permanent effect (b) temporary effect
(d) Mesomeric effect
(c) resonance effect (d) inductive effect

1. Which of the following is correct regarding the – I effect of 8. Select the most stable carbocation from amongst the following
the substituents?
+
(a) – NR2 < – OR < – F (b) – NR2 > – OR < – F (a)
(c) – NR2 < – OR > – F (d) – NR2 > – OR > – F
2. Polarization of electrons in acrolein may be written as (b)
d- d+ d- d+ +
(a) CH 2 = CH - CH = O (b) CH 2 = CH - CH = O
d- d- d+ d-
(c) CH 2 = CH- CH = O (d) CH 2 = CH - CH = O (c) +
3. Heterolytic fission of a covalent bond in organic molecules
gives
(a) free radicals (b) cations and anions
(c) only cations (d) only anions (d) +
4. In which of the following homolytic bond fission takes place ?
(a) Alkaline hydrolysis of ethyl chloride 9. The most stable carbonium ion among the following is
(b) Addition of HBr to double bond
(c) Photochlorination of methane + +
(d) Nitration of benzene (a) C6 H5 CHC6 H5 (b) C6 H5CH 2
5. Homolytic fission of C–C bond in ethane gives an
+ +
intermediate in which carbon is (c) CH3 CH 2 (d) C6 H5CH2CH 2
(a) sp3-hybridised (b) sp2-hybridised
(c) sp-hybridised (d) sp2d-hybridised 10. Which of the following is most stable?
6. Among the following, the true property about (a) Ph3C+ (b) Ph2CH+
CH3 (c) PhCH2+ (d) Tropylium cation
+
C – CH3 is 11. Consider the following carbocations
CH3 + +
I. C6 H5 CH 2 II. C6 H5 CH 2CH 2
(a) it is non-planar
(b) its C+ is sp2-hybridized
+ +
(c) an electrophile can attack on its C+ III. C6 H5CHCH3 IV. C6 H5C(CH3 )2
(d) it does not undergo hydrolysis
7. Which of the following is the most stable carbocation The correct sequence for the stability of these carbocations
(carbonium ion)? is
+
(a) CH 3CH 2 + (b) (CH ) CH (a) II < I < III < IV (b) II < III < I < IV
3 2
+ +
(c) III < I < II < IV (d) IV < III < I < II
(c) (CH 3 )3 C (d) C6 H5CH 2
412 Chemistry
12. The most stable carbanion among the following is 22. Which of the following has the highest nucleophilicity?
(a) F - (b) OH -
CH2 – CH2– CH2–
(c) CH 3- (d) NH -2
23. The correct nucleophilicity order is
(a) (b)
(a) CH 3- < NH -2 < HO - < F -
(b) CH 3- ~
- NH -2 > HO - ~- F-
CH2– CH2– (c) CH 3- > NH -2 > HO - > F -
(d) NH -2 > F - > HO - > CH 3-
24. What is the decreasing order of strength of the bases
(c) (d)
OH - , NH -2 , HC º C - and CH 3CH -2 ?
OCH3 NO2 (a) CH 3 CH -2 > NH -2 > HC º C - > OH -
13. The order of decreasing stability of the carbanions (b) HC º C - > CH 3CH -2 > NH -2 > OH -
(CH 3 )3C - (I) ; (CH 3 ) 2 CH - (II); CH 3CH 2- (III); (c) OH - > NH 2- > HC º C - > CH 3CH -2

C 6 H 5 CH -2 ( IV ) is (d) NH -2 > HC º C - > OH - > CH 3CH -2

(a) I > II > III > IV (b) IV > III > II > I 25. Which is the correct symbol relating the two Kekule
structures of benzene ?
(c) IV > I > II > III (d) I > II > IV > III
14. The most stable free radical among the following is (a) ®¬ (b) ®
(c) º (d) «
· ·
(a) C 6 H 5 CH 2CH 2 (b) C6 H 5 CHCH 3 26. Point out the incorrect statement about resonance?
(a) Resonance structures should have equal energy
· ·
(c) CH 3CH 2 (d) CH 3CHCH 3 (b) In resonating structures, the constituent atoms must be
in the same position
15. Intermediate involved in Reimer-Tiemann reaction is (c) In resonating structures, there should not be same
(a) carbocation (b) carbanion number of electron pairs
(c) carbene (d) free radical (d) Resonating structures should differ only in the location
16. For the reaction of phenol with CHCl3 in presence of KOH, of electrons around the constituent atoms
the electrophile is 27.
-
: CH 2 - C - CH3 and CH 2 = C - CH 3 are
(a) +
CHCl 2 (b) : CCl2 || |
O. .: :O -
. .:
·
(c) CHCl (d) CCl4 (a) resonating structures (b) tautomers
2
17. In the mechanism of Hofmann reaction which intermediate (c) geometrical isomers (d) optical isomers
rearranges to alkyl isocyanate? 28. The most unlikely representation of resonance structures of
(a) Bromamide (b) Nitrene p-nitrophenoxide ion is
(c) Nitroso (d) Amide –O
+ O –O O–
18. Which of the following is an electrophile? +
N N
(a) H2O (b) NH3
(c) AlCl3 (d) C2H5NH2
19. Which of the following is not a nucleophile? (a) (b)
(a) CN– (b) OH–
(c) NH3 (d) BF3 O– O
20. Which of the following is not a nucleophile?
(a) H2O (b) CH3OH –O
O + O O
(c) H2 (d) NH3 +
N N
21. Which of the following behaves both as a nucleophile and
as an electrophile ?
(c) (d)
(a) CH 3C º N (b) CH 3OH

(c) CH 2 = CHCH 3 (d) CH 3 NH 2 O– O
General Organic (Basic Concepts) 413
29. In which of the following, resonance will be possible? 32. Resonance in most of the organic molecules
(a) CH3 - CH 2 - CH 2 - CHO (a) increases stability (b) decreases stability
(c) increases reactivity (d) None of these
(b) CH 2 = CH - CH = O
33. The kind of delocalization involving sigma bond orbitals is
(c) CH 3COCH 3 called
(a) inductive effect
(d) CH 2 = CH - CH 2 - CH = CH 2
(b) hyperconjugation effect
30. Which of the following statements regarding the resonance (c) electromeric effect
energy of benzene is correct? (d) mesomeric effect
(a) Resonance energy is the energy required to break the 34. The reaction,
C–H bond in benzene
¾® CH 3CHBrCH 3 is
CH 2 = CH - CH 3 + HBr ¾
(b) Resonance energy is the energy required to break the
(a) nucleophilic addition
C–C bond in benzene
(b) electrophilic substitution
(c) Resonance energy is a measure of stability of benzene
(c) electrophilic addition
(d) Resonance energy is the energy required to convert
(d) free radical addition
35. The addition of HCN to a carbonyl compound is an example
of
(a) nucleophilic substitution
31. The heat of hydrogenation of 1-hexene is 126 kJmol–1, When (b) electrophilic addition
a second double bond is introduced in the molecule, the (c) nucleophilic addition
heat of hydrogenation of the resulting compound is 230 kJ (d) electrophilic substitution
mol–1. The resulting compound (diene) is 36. Acetaldehyde is the rearrangement product of
(a) 1, 3-Hexadiene (b) 1, 4-Hexadiene (a) methyl alcohol (b) allyl alcohol
(c) 1, 5-Hexadiene (d) Nothing certain (c) vinyl alcohol (d) All are correct

1. For (i) I–, (ii) Cl–, (iii) Br–, the increasing order of nucleophilicity 4. Base strength of : [CBSE-PMT 2008]
would be [CBSE-PMT 2007] (A) H 3CCH 2 ,– (B) H 2C = CH and
(a) Cl– < Br– < I– (b) I– < Cl– < Br– (C) H - C º C
(c) Br– < Cl– < I– (d) I– < Br– < Cl– is in the order of :
2. The order of decreasing reactivity towards an electrophilic (a) (B) > (A) > (C) (b) (C) > (B) > (A)
reagent, for the following would be [CBSE-PMT 2007]
(c) (A) > (C) > (B) (d) (A) > (B) > (C)
(i) benzene (ii) toluene 5. The stability of carbanions in the following :
(iii) chlorobenzene (iv) phenol
(a) (ii) > (iv) > (i) > (iii) (b) (iv) > (iii) > (ii) > (i) (I) RC = C (II)
(c) (iv) > (ii) > (i) > (iii) (d) (i) > (ii) > (iii) > (iv)
(III) R 2C = CH (IV) R 3C - CH 2
3. Which one of the following is most reactive towards
electrophilic attack ? [CBSE-PMT 2008] is in the order of : [CBSE-PMT 2008]
CH2OH (a) (I) > (II) > (III) > (IV) (b) (II) > (III) > (IV) > (I)
(a) (b) (c) (IV) > (II) > (III) > (I) (d) (I) > (III) > (II) > (IV)
6. Cyclohexanol (I), acetic acid (II), 2, 4, 6-trinitrophenol (III)
Cl and phenol (IV) are given. In these the order of decreasing
OH acidic character will be : [CBSE-PMT 2010]
(a) III > II > IV > I (b) II > III > I > IV
(c) (d)
(c) II > III > IV > I (d) III > IV > II > I
414 Chemistry
7. The correct order of increasing reactivity of C – X bond 12. Some meta-directing substituents in aromatic substitution
towards nucleophile in the following compounds is: are given. Which one is most deactivating? [NEET 2013]
[CBSE-PMT 2010] (a) –SO3H (b) –COOH
X (c) –NO2 (d) –C º N
X
NO2 13. Arrangement of (CH3)3 – C –, (CH3)2 – CH –, CH3 – CH2 –
(CH3)3 C – X, (CH3)2CH – X when attached to benzyl or an unsaturated group in
increasing order of inductive effect is [AIEEE 2002]
(a) (CH3)3 –C – < (CH3)2 – CH – < CH3 – CH2–
NO2 (b) CH3 –CH2– < (CH3)2– CH – < (CH3)3 –C –
(I) (II) (III) (IV) (c) (CH3)2 – CH– < (CH3)3 –C – < CH3—CH2–
(a) I < II < IV < III (b) II < III < I < IV (d) (CH3)3 – C– < CH3 –CH2 – < (CH3)2 –CH –
(c) IV < III < I < II (d) III < II < I < IV 14. The reaction:
H O
8. Among the given compounds, the most susceptible to (CH3)3C – Br ¾¾2¾® (CH3)3 – C –OH is a/an
nucleophilic attack at the carbonyl group is: [AIEEE 2002]
[CBSE-PMT 2010] (a) elimination reaction (b) substitution reaction
(a) CH3COOCH3 (b) CH3CONH 2 (c) free radical reaction (d) displacement reaction.
15. In the anion HCOO– the two carbon-oxygen bonds are found
(b) CH3COOCOCH3 (d) CH 3 COCl
to be of equal length. what is the reason for it ?
9. Which one of the following is most reactive towards [AIEEE 2003]
electrophilic reagent ? [CBSE-PMT 2010, 2011] (a) The C = O bond is weaker than the C — O bond
CH3 CH3 (b) The anion HCOO– has two resonating structures
(a) (b) (c) The anion is obtained by removal of a proton from the
OCH3 OH acid molecule
(d) Electronic orbitals of carbon atom are hybridised
CH3 CH3 16. Rate of the reaction [AIEEE 2004]
(c) (d) O O
NHCOCH3 CH2OH R – C + Nu R–C +Z
Z Nu
10. Which one is a nucleophilic substitution reaction among the
is fastest when Z is
following ? [CBSE-PMT 2011]
(a) OC2H5 (b) NH2
(a) CH3 – CH = CH2 + H2O CH3 – CH – CH3 (c) Cl (d) OCOCH3
OH 17. Due to the presence of an unpaired electron, free radicals
(b) RCHO + R¢ MgX R – CH – R¢ are: [AIEEE 2005]
(a) cations (b) anions
OH (c) chemically inactive (d) chemically reactive
CH 3 18. The decreasing order of nucleophilicity among the
(c) CH3 – CH2–CH–CH2Br + NH 3 nucleophiles [AIEEE 2005]
(a) CH 3C - O -
CH3
||
CH 3–CH 2–CH–CH2NH 2 O
(b) CH 3O -
(d) CH3CHO + HCN ¾¾ ® CH3CH (OH) CN
11. Which of the following compounds undergoes nucleophilic (c) CN -
O
substitution reaction most easily ? [CBSE-PMT 2011 M] ||
(d) H 3C S – O – is
Cl ||
Cl O
(a) (b) (a) (c), (b), (a), (d) (b) (b), (c), (a), (d)
NO2 (c) (d), (c), (b), (a) (d) (a), (b), (c), (d)
CH3 19. The reaction [AIEEE2005]
O

O
||

||

Cl R–C + Nu R–C +X
Cl X Nu
is fastest when X is
(c) (d)
(a) OCOR (b) OC 2 H 5

OCH3 (c) NH 2 (d) Cl


General Organic (Basic Concepts) 415
20. The increasing order of stability of the following free radicals 26. The order of stability of the following carbocations :
is [AIEEE 2006] Å
• • • • CH 2
(a) (C6H5)2 C H < (C6H5)3 C < (CH3)3 C < (CH3)2 C H
Å Å
• • • •
(b) (CH3)2 C H < (CH3)3 C < (C6H5)2 C H < (C6H5)3 C CH 2 = CH - C H 2 ; CH3 - CH 2 - CH 2 ; is :
• • • • I II
(c) (CH3)2 C H < (CH3)3 C < (C6H5)2 C H < (C6H5)3 C III
• • • •
(d) (C6H5)3 C < (C6H5)2 C H < (CH3)3 C < (CH3)2 C H [JEE M 2013]
21. Presence of a nitro group in a benzene ring [AIEEE 2007] (a) III > II > I (b) II > III > I
(a) deactivates the ring towards electrophilic substitution (c) I > II > III (d) III > I > II
(b) activates the ring towards electrophilic substitution 27. Among the following, the least stable resonance structure is
[IIT-JEE 2007]
(c) renders the ring basic
(d) deactivates the ring towards nucleophilic substitution. Å O
(a) Å
Å
22. The electrophile, E attacks the benzene ring to generate N
the intermediate s-complex. Of the following, which s-
O
complex is lowest energy? [AIEEE 2008]

NO2
O
H Å
(b)
+ E Å N
(a) (b) +
O
H E

NO2 NO2 O
H (c) Å
E Å N
(c) + (d) + H
E O
23. Arrange the carbanions, [AIEEE 2009]

(CH3 )3 C , C Cl3 , (CH3 ) 2 CH ,C6 H5 CH 2 O


(d) Å
in order of their decreasing stability : Å N
(a) (CH3 ) 2 C H > C Cl3 > C6 H5 C H 2 > (CH3 )3 C O

(b) CCl3 > C6 H5 CH2 > (CH3 )2 CH > (CH3 )3 C 28. Hyperconjugation involves overlap of the following orbitals
[IIT-JEE 2008]
(c) (CH3 )3 C > (CH3 )2 CH > C6 H5 CH 2 > C Cl3 (a) s-s (b) s - p (c) p- p (d) p-p
29. The correct stability order for the following species is
(d) C6 H5 CH2 > CCl3 > (CH3 )3 C > (CH3 )2 CH
[IIT-JEE 2008]
24. The correct order of increasing basicity of the given conjugate
+ + +
bases (R = CH3) is [AIEEE 2010]
O O +
(a) RCOO < HC º C < R < NH 2 (I) (II) (III) (IV)
(a) (II) > (IV) > (I) > (III) (b) (I) > (II) > (III) > (IV)
(b) R < HC º C < RCOO < NH 2
(c) (II) > (I) > (IV) > (III) (d) (I) > (III) > (II) > (IV)
(c) RCOO < NH 2 < HC º C < R 30. In the following carbocation, H/CH3 that is most likely to
migrate to the positively charged carbon is [IIT-JEE 2009]
(d) RCOO < HC º C < NH 2 < R H H
1 2 4 5
25. A solution of ( – ) – 1 – chloro –1– phenylethane in toluene H3C — C
+
—3C — C — CH3
racemises slowly in the presence of a small amount of SbCl5,
due to the formation of : [JEE M 2013] HO H CH3
(a) carbanion (b) carbene
(a) CH3 at C-4 (b) H at C-4
(c) carbocation (d) free radical
(c) CH3 at C-2 (d) H at C-2
416 Chemistry
31. Among the following compounds, the most acidic is O
[IIT-JEE 2011] Cl
H3C – Cl Cl Cl
(a) p-nitrophenol (b) p-hydroxybenzoic acid
(c) o-hydroxybenzoic acid (d) p-toluic acid
32. KI in acetone, undergoes SN2 reaction with each of P, Q, R P Q R S
and S. The rates of the reaction vary as (a) P > Q > R > S (b) S > P > R > Q
(JEE Advanced 2013) (c) P > R > Q > S (d) R > P > S > Q

1. In E2 elimination, some compounds follow Hofmann’s rule (a) Carbanion (b) Carbene
which means (c) Free radical (d) Carbocation
(a) the double bond goes to the most substituted position 8. Which of the following is singlet carbene ?
(b) the compound is resistant to elimination ..
(c) no double bond is formed (a) (CH3)3C+ (b) C 2 H 5 C CH 3
(d) the double bond goes mainly towards the least – +
substituted carbon (c) CH3CHCH 3 (d) CH 2 = CH - CH 2
2. SN1 reaction on optically active substrates mainly gives 9. The reaction,
(a) retention in configuration
C 2 H 5 I + KOH ® C 2 H 5 OH + KI is called
(b) inversion in configuration
(c) racemic product (a) hydroxylation substitution
(d) No product (b) electrophilic substitution
3. Which of the following contains only three pairs of electrons? (c) nucleophilic substitution
(a) Carbocation (b) Carbanion (d) dehydroiodination
(c) Free radical (d) None of these 10. An organic compound C5H11X an dehydrohalogenation
4. The addition of HBr on butene -2 in presence of peroxide gives pentene - 2 only. What is halide
follows the (a) CH 3CH 2 CHXCH 2 CH 3
(a) electrophilic addition (b) free radical addition
(c) nucleophilic addition (d) None of these (b) (CH 3 )2CHCHXCH3
5. The stability of 2, 3 - dimethyl-but - 2- ene is more than 2- (c) CH3CH 2 CH 2 CHXCH3
butene. This can be explained in terms of
(a) resonance (b) hyperconjugation (d) CH3CH 2 CH 2 CH 2 CH 2 X
(c) electromeric effect (d) inductive effect 11. Sulphur trioxide is
6. (CH3)4N+ is neither an electrophile, nor a nucleophile because (a) an electrophile (b) a nucleophile
it (c) a homolytic reagent (d) a base
(a) does not have electron pair for donation as well as cannot 12. Intermediate product formed in the acid catalysed
attract electron pair dehydration of n- propyl alcohol is
(b) neither has electron pair available for donation nor can
(a) CH 3 - CH 2 - CH 3 (b) CH 3 - CH = CH 2
accommodate electron since all shells of N are fully
occupied + +
(c) can act as Lewis acid and base (c) CH 3 - C H - CH 3 (d) CH 3 - CH 2 - C H 2
(d) None of these 13. Which of the following is most stable
7. A solution of (+) - 2 - chloro - 2 - phenylethane in toluene
(a) CCl 3CH (OH ) 2 (b) (CH3)2.C.(OH)2
racemises slowly in the presence of small amounts of SbCl5
due to the formation of (c) CH3C.Cl(OH)2 (d) CH3CH(OH)2
General Organic (Basic Concepts) 417
14. Chlorine in vinyl chloride is less reactive because 21. The intermediate during the addition of HCl to propene in
(a) sp2 - hybridised carbon has more acidic character than the presence of peroxide is
· +
sp3 - hybridised carbon (a) CH3CHCH 2 Cl (b) CH 3CHCH 3
· +
(b) C - Cl bond develops partial double bond character (c) CH3CH 2CH 2 (d) CH3CH 2CH 2
(c) of resonance
22. Which of the following acids has the smallest dissociation
(d) All are correct constant ?
15. The compound which gives the most stable carbonium ion (a) CH3CHFCOOH (b) FCH2CH2COOH
on dehydration :
(c) BrCH2CH2COOH (d) CH3CHBrCOOH
CH3 23. One of the equivalent energy resonance structures for
| formate anion is shown below
(a) CH3 - CH - CH 2 OH (b) H 3 C - C - OH
|

: :
| O
A
CH3 CH3
H –C

: :
(c) CH3– CH 2 – CH 2 – CH 2 OH (d) CH3 - CH - CH 2 - CH3 O:
| B
OH Which C – O bond is longer ?
16. Among the following compounds which is most reactive (a) A
towards nitration (b) B
(a) Benzene (b) Nitrobenzene (c) Both are equal
(c) Toluene (d) Chlorobenzene
(d) Structure shown is not correct
17. Example of chlorinolysis is
24. Which of the following resonance structure is lowest in
(a) CH 2 = CH 2 ¾¾
® C 2 H 4 Cl 2 energy?
(b) CCl4 + H 2 O ¾¾
® COCl2 + 2HCl
H O

:
H O : : : :
(c) CHCl3 + 4NaOH | || |
(A) H – C – C (B) H – C – C
¾¾
® HCOONa + 3NaCl + 2H 2 O +
| |
: :

Cl2 O– H O– H
(d) C3 H8 ¾¾¾ ® CCl 4 + C 2 Cl6 + 8HCl HB HB

:
18. During debromination of meso - dibromobutane, the major
:

H O : :
compound formed is : | |
(a) n - butane (b) 1 - butene (C) H – C – C
| +
: :

(c) Cis - 2 - butene (d) Trans - 2 - butene O– H


HB
19. In the following compounds
(a) A (b) B
O
(c) C (d) All have same energy
(I) (II) (III) (IV)
N N 25. The structure that does not have an error is
N N
H H H OH
the order of basicity is : |
(A) H - C - N - Cl
(a) IV > I > III > II (b) III > I > IV > II |
(c) II > I > III > IV (d) I > III > II > IV H
20. In the following groups (B) CH 3 - CH 2 -CH 2 - Cl
|
-OAc -OMe Cl
(I) (II) (C) H 2 C = CH H
Cº C–N
-OSO 2 Me -OSO 2 CF3 H H
(III) (IV) H
(D) H C= N
the order of leaving group ability is OH
(a) I > II > III > IV (b) IV > III > I > II (a) A (b) B
(c) III > II > I > IV (d) II > III > IV > I (c) C (d) D
418 Chemistry

26. How many degrees of unsaturation are there in benzene 31. Which of the followings is most stable ?
CH 3
CH3
? |
(a) 2 (b) 3 (a) H (b) +
(c) 4 (d) More than 4
Y
27. Which is incorrect for the following pairs ? H Y
Column I Column II CH 3 CH3
| H |
(a) resonance +
(c) Y (d)
+
O O
|| H Y
(b) equilibrium
OH OH
Å 32. The stable resonating form of vinyl methyl ketone is
: O: -

:
(c) equilibrium Å |
(a) CH2 – CH = C –CH3
O OH : O:
|| |
(d) equilibrium Å - ||
(b) C H 2 – C H – C – CH 3
28. What is the mechanistic intermediate in the following reaction
:O: -

:
O O O O |
|| (i ) NaOMe || (c) CH 2 = CH - C - C H 3
Å
¾¾¾¾¾® OEt
OEt (ii ) MeI | :O:
:

(d) Å
:

O O CH 2 - CH - C - C H 3
O– O
|| || | || 33. Hyperconjugation involves
(a) OEt (b) OEt
| (a) s - p conjugation (b) s - p delocalisation
OMe (c) no bond resonance (d) All of these
34. The following reaction
O O
Br
|| + HBr ¾
¾®
(c) OEt (d) None of these
| is an example of
(a) elimination reaction (b) nucleophilic subsitution
29. In the reaction shown below the six membered ring is (c) electrophilic addition (d) nucleophilic addition
generated by shifting which bond 35. Which of the following reactions is elimination reaction?

(a) O + Ph 3 P = CH 2 ¾
||

B ¾®
CD HBr
¾¾¾® Br
A
| CH 2 + Ph 3 PO
||

H
OH
(b) C 2 H 5 Cl + KOH ® C 2 H 5 OH + KCl
(a) A (b) B (c)
(c) C (d) D H
30. When HI is added on to propene in presence of peroxide the |
Alc KOH
:

C2 H5 - C - CH3 ¾¾¾¾¾® CH3CH = CH - CH3 + N (CH 3 ) 3


intermediate formed is |
+ + N(CH3 )3
(a) CH3CHCH3 (b) CH3CH 2 C H2 +
· · (d) None of them
(c) CH3CHCH 2ICl (d) CH3CH 2CH 2
General Organic (Basic Concepts) 419
41. The change occuring with conversion of configuration
36. –C-O- electrophilic substitution H
|| aq
O (a) D Cl ¾¾®
KOH
will occur at Me
(a) meta in second ring
H
(b) ortho/para in the first ring
(b) D CONH 2 ¾Br - KOH
(c) meta in the first ring ¾2¾¾¾®
(d) ortho/para in the second ring Me
37. The correct order of the stability of the following carbanions H
Br2
is (c) D CH 2 CONH 2 ¾¾¾¾ ®
KOH
- - Me
(i) RCH 2 (ii) H
(d) D CH 2 Cl ¾aq KOH
¾ ¾¾®
- - Me
(iii) (iv)
+
42. In which case the NO 2 will attack at the meta position
(a) III > IV > I > II (b) I > II > III > IV
(c) I > III > II > IV (d) IV > III > II > I CCl3 NO2
38. The major product of dehydration of the following

CH 3
+ (ii)
H3 O (i)
CH - CH 3 ¾¾¾ ® Product.
| D + –
OH NH3 O

CH 3
CH 3
(a) (b) CH = CH 2
CH 3 (iii) ( iv)

CH = CH 2 (a) I, II, III (b) II, IV


– CH = CH - CH 3
(c) (d) (c) II and III only (d) II only
CH 3
43. Arrange the following in order of decreasing basicity
39. Which among the following is substitution reaction ?
A. CH3CH2MgBr B. HC º CMgBr
OMgX
(a) C = O + RMg X ¾
¾® C C. CH3CH2OMgBr
R
(a) A > C > B (b) A > B > C
O
O (c) C > B > A (d) B > A > C
|| ||
44. Which of the following is not a resonance form of the enolate
| O ¾
¾® O ion formed in the following acid base equilibrium
||

(b) +
|| || O
O O
CH 3
Anhy. ZnCl 2 + NaOH
(c) R 3COH ¾¾ ¾ ¾ ¾
¾® Turbidity
Conc. HCl O
(d) All of these
O O–
40. Which of the following reactions cannot proceed by SN1
mechanism ? CH 3 CH 2
:

(a) (b)
CH 2 Cl O
O
| O– O–
(a) (b) Cl CH 3 CH 3
(c) (d)
(c) (d) Cl O O–
Cl
420 Chemistry

EXERCISE 1 EXERCISE 2
1. No. It is 1°. 1. (a) Greater the electronegativity, higher is the -I effect. Thus
2. (i) –NO2 > –CN > –COOH > –F > –I – F > –OR > – NR2
(ii) –C(CH 3 ) 3 > –CH(CH 3 ) 2 > –CH 2 CH 3 > 2. (d) Due to – I effect of the – CHO group, oxygen acquires-d
–CH3 > –D. - charge and the terminal carbon acquires d + charge.
. . d+ d-
4. (i) C6H5CH2 C H 2 < C6H5CHCH3 < C6H5 C (CH3)2 < CH2 = CH – C = O
. H
C6H5 CH –CH=CH2.
(ii) (CH3)3C+ > C6H5CH2+ > CH2 = CHCH2+ > CH3CH2+ 3. (b) 4. (c)
5. (b) Homolytic fission of the C – C bond gives free radicals
(iii) (CH3)2CH– < CH3CH2– < CH3– < CH2 = CH–
in which carbon is sp2- hybridised.
< C6H5CH2– < HC º C–. 6. (b) In carbocations, carbon bearing positive charge is
.. –
8. (i) Nucleophiles : ....I .. , H3N .. always sp2-hybridised
7. (d) Higher the possibility of delocalisation of the positive
(ii) Electrophiles : BeCl2 and NO +2 charge, greater is stability of the species. Thus

.. + + +
(iii) Both : CH3C º N.. and H2C = C .. C6H5 C H2 > (CH3 )3 C+ > (CH3 ) 2 C H > CH3 C H2
(iv) Neither : CH4 and H2. Benzyl carbocation is more stable than tert-butyl due to
10. Stabilised due to nine possible canonical structures. resonance in the former.
8. (b) Structure (b) is a 3º carbocation, while (a) is 2º and (c)
and (d) are 1º carbocations; thus (b) is the most stable.
9. (a) Higher the possibility of delocalisation, greater is its
C +
stability; in C6 H5CHC6 H5 , +ve charge can delocalise
over two benzene rings.
10. (a) Same as above, in Ph3C+, +ve charge can delocalise over
three benzene rings.
Other + +
C canonical 11. (a) I. C6 H5 CH 2 II. C6 H5 CH 2CH 2
structures 1º Benzylic 1º

+ +
16. (b) 17. (a) 18. (b) 19. (b) 20. (d) 21. (b) III. C6 H 5 CHCH3 IV. C6 H 5 C(CH3 )2
2º benzylic 3º benzylic
22. (a) 23. (b) 24. (b)
3º Benzylic (IV) > 2º Benzylic (III) > 1º Benzylic
(I) > 1º (II)
12. (d) –NO 2 group, being strong electron-withdrawing,
disperses the –ve charge, hence stabilizes the concerned
carbanion.
General Organic (Basic Concepts) 421
13. (b) C6H5CH2– > CH3CH2– > (CH3)2 CH– >(CH3)3C– 31. (a) Since the given heat of hydrogenation of the diene is
Dispersal of –ve Intensification of –ve charge due less (230 kJ mol–1) than the heat of hydrogenation of
charge due to to + I effect of CH3 gps. two isolated double bonds, i.e. 126 × 2 = 252 kJ mol–1,
the diene must be conjugated, i.e., it should be 1,3 -
resonance and - I effect
hexadiene, CH2 = CH – CH= CH CH2 CH3.
. 32. (a) 33. (b) 34. (c)
14. (b) C6 H 5CHCH3 is a 2º benzylic free radical, hence
stabilized most due to resonance.
15. (c) 35. (c)
16. (b) Dichlorocarbene, : CCl2 (a carbene) is the electrophile
formed as an intermediate in Reimer-Tiemann reaction. Since first attack is nuclephilic in nature (by CN –) hence
17. (b) 18. (c) the addition is nuclophilic addition.
19. (d) BF3 has electron deficient B, hence it is an electrophile.
H
20. (c) H2 has no lone pair of electrons, hence cannot function |
rearrangement or Tautomersin
36. (c) H 2 C = CH OH ¾¾¾¾¾¾¾¾¾¾¾® H3C- C = O
as a nucleophile. Acetaldehyde
Vinyl alcohol
21. (a) Due to the presence of a lone pair of electrons on N,
CH3C º N: acts as a nucleophile. Further due to greater
electronegativity of N than C, the C atom of – C º N EXERCISE 3
carries a positive charge and hence behaves as an
1. (a) Nucleophilicity increases down the periodic table.
electrophile.
22. (c) I - > Br - > Cl- > F-
23. (c) Nucleophilicity increases with the decrease in 2. (c) Electrophiles have high affinity for electrons. They attack
electronegativity of the central atom. Since at the site where electron-density is highest. Electron
electronegativity follows the order: F > O > N > C; donating groups increases the electron density. The
nucleophilicity of the concerned group will follow the electron donating tendency decreases in the order :
reverse order i.e., –OH > –CH3 > –H > –Cl
Therefore, the correct order of reactivity towards
CH 3 - > NH 2 - > OH - > F -
electrophile is
24. (a) Stronger the acid, weaker the conjugate base. Since acid C6H5OH > C6H5CH3 > C6H6 > C6H5Cl
character follows the order
3. (c) Out of the given compounds the most reactive towards
H 2 O > NH 3 > HC º CH > CH 3 - CH 3 OH
(Acid character), electrophilic attack is . Phenol forms phenoxide
the basic character of their conjugate bases decreases
in the reverse order, i.e., ion which is stable due to resonance i.e. the correct
answer is option (c).
CH 3CH -2 > HC º C - > NH 2 - > OH - (Basic character)
4. (d) The amount of s-character in various hybrid orbitals is
25. (d) as follows.
26. (c) All resonating structures should have same number of sp = 50%, sp2 = 33% and sp3 = 25%
electron pairs. Therefore s character of the C – H bond in acetylene (sp)
27. (a) The two structures involve only movement of electrons is greater than that of the C – H bond in alkene (sp2
and not of atoms or groups, hence these are resonating hybridized) which in turn has greater s character of the C
structures. – H bond than in alkanes. Thus owing to a high s
28. (c) In structure (c), nitrogen has 10 electrons in the valence character of the C – H bond in alkynes, the electrons
shell which is not possible. constituting this bond are more strongly held by the
carbon nucleus with the result the hydrogen present on
29. (b) Only structure (b) has a conjugated system, which is
such a carbon atom can be easily removed as proton.
necessary for resonance.
The acidic nature of three types of C – H bonds follows
30. (c) the following order
422 Chemistry

-C º C - H > - C = C - H > - C - C - H III IV

Further, as we know that conjugate base of a strong acid X


is a weak base, hence the correct order of basicity is X
NO2
- - - >
H - C º C < CH2 = CH < CH2 - CH3
NO2
5. (d) The stability of carbanions is affected due to resonance,
II I
inductive effect and s - character of orbitals. Greater the
number of groups having + I group (alkyl group) lesser or I < II < IV < III
stable would be the carbanion. 8. (d) Cl– is the weakest base and hence better leaving group
Further stability of carbanion decreases with decrease 9. (b) Due to + M effect of – OH group and hyperconjugation
in s-character. Benzene carbanions are stablized due to of – CH3 group.
resonance, hence the correct order is 10. (c) Because of high electronegativities of the halogen atom,
the carbon halogen (C – X) is highly polarised covalent
R - C = C > R 2 C = CH> > R C – CH bond. Thus, the carbon atom of the C – X bond becomes
3 2
a good site for attack by nucleophiles (electron rich
The correct order of stability of given carbanion is in the species). Nucleophilic substitution reactions are the
order I > III > II > IV. most common reactions of alkyl halides.
11. (a) In SN Ar reactions, a carbanion is formed as an intermediate,
OH so any substituent that increases the stability of carbanion
6. (a) ON2 and hence the transition state leading to its formation will
NO2 > CH3COOH >
enhance the SNAr reactions. To compare the rates of
substitution in chlorobenzene, chlorobenzene having
NO2 electron-withdrawing group, and chlorobenzene having
electron-releasing group, we compare the structures
(III) (II) carbanion I (from chlorobenzene), II (from chlorobenzene
containing electron-withdrawing group) and III (from
OH OH chlorobenzene containing electron-releasing group).
Z Cl Z Cl
>
– –
(IV) (I)
Explanation : Presence of three — NO 2 groups in G
o–, p– positions to phenolic groups (in III) makes phenol I II
strongly acidic because its corresponding conjugate G withdraws electrons, neutralises (disperses) –ve
base is highly stabilised due to resonance. Conjugate charge of the ring, stabilises carbanion, facilitates SN
reaction (activation effect)
base of CH3COOH, II (i.e. CH3COO- ) is resonance
Z Cl
hybrid of two equivalent structures. The conjugate base
of phenol, IV is stabilized due to resonance (note that –
here all resonating structures are not equivalent). The
conjugate base of cyclohexanol, I does not exhibit
resonance, hence not formed. G
7. (a) Tertiary alkyl halide is most reactive towards nucleophilic III
substitution because the corresponding carbocation (3°) G releases electrons, intensifies –ve charge, destabilizes
is most stable. Aryl halide is least reactive due to partial carbanion, retards SN reaction (deactivation)
double bond character of the C – Cl bond. NO2 is activating group and CH3 and OCH3 are deactiving
Presence of — NO 2 groups in ortho and para positions group.

increases the reactivity of the – Cl towards nucleophiles. Hence, the correct order of nucleophilic substitution
reactions
(CH3)3 – C – X > (CH3)2 – CH2 – X >
General Organic (Basic Concepts) 423
Cl Cl Cl
Cl
Cl – – – –
23. (b) C Cl > C6H5CH2 > (CH3)2 CH > (CH3)3C
> > >
Cl
–ve charge –M effect +I effect of CH3 group
highly dispersed delocalises intensifies the –ve charge
NO2 CH3 OCH3 –ve charge
due to – I effect
12. (c) Decreasing order of deactivating effect of the given
m-directing group is 24. (d) The correct order of basicity is

> NO2 > – CN > – SO3H > – COOH RCOO - < CH º C - < - NH 2 < R -

—NO2 group is most deactivating group due to strong 25. (c) Carbocations are planar hence can be attacked on
– E, – I and – M effects. either side to form racemic mixture.
13. (b) –CH3 group has +I effect, as number of –CH3 group Å
SbCl
5 ® Ph - C H - CH + SbCl- ¾¾
increases, the inductive effect increases. Cl - CH - CH3 ¾¾¾¾ 3 6 ®
Toluene
| (carbocation)
14. (b) The reaction is nucleophilic sustitution reaction
Ph
(-)
O O
15. (b) H–C–O H–C=O
Ph - CH - CH3 + SbCl5
16. (c) Cl– is the best leaving group among the given options. |
Cl
17. (d) Free radicals are electrically neutral, unstable and very (d + l) mixture
reactive on account of the presence of odd electrons.
26. (d) Higher stability of allyl and aryl substituted methyl
18. (b) The stronger the base the more is the nucleophilic
carbocation is due to dispersal of positive charge due
character and vice versa.
to resonance
Basic character
+ +
CH 2 = CH - C H 2 ¬¾® CH 2 - CH = CH 2
CH 3O - > CN - > CH 3 COO - > H 3C - Resonating structures of allyl carbocation

SO3

+
Hence the nucleophilic character. CH2 CH2 CH2 CH2
Å Å
O
||
19. (d) R - C - X ; when X is Cl the C–X bond is more polar
Å
and ionic which leaves the compound more reactive for Resonating structures of benzyl carbocation
nucleophilic substitution reaction.
20. (b) The order of stability of free radicals whereas in alkyl carbocations dispersal of positive charge
on different hydrogen atoms is due to hyperconjugation.
• • • •
(C6 H5 )3C > (C6 H5 ) 2CH > (CH3 )3C > (CH3 ) 2CH Hence the correct order of stability will be
Å
The stabilisation of first two is due to resonance and last CH2
two is due to inductive effect. Å Å
21. (a) Nitro group is electron withdrawing group, so it > CH 2 = CH - CH2 > CH3 - CH 2 - CH 2
Allyl, I Propyl, II
deactivates the ring towards electrophilic substitution. Benzyl, III
22. (b) In option (b) the complex formed is with benzene 27. (a) Due to similar charges on adjacent atoms, the
whereas in other cases it is formed with nitrobenzene structure (a) is least stable.
with –NO2 group in different position (o-, m-, p-). The
28. (b) Alkyl groups with at least one hydrogen atom on the
complex formed with nitrobenzene in any position of
a-carbon atom, attached to an unsaturated carbon
–NO2 group is less stable than the complex formed
atom, are able to release electrons in the following
with benzene so the correct answer is (b). The most
way.
stable complex has lowest energy.
424 Chemistry
32. (b)

O
Cl
Compounds : CH3 – Cl : Cl : Cl
Note that the delocalisation involves s and p bond orbitals S P R Q
(or p orbitals in case of free radicals) ; thus it is also known
Relative reactivities 1,00,000 : 200 : 79 : 0.02
as s – p conjugation. This type of electron release due to towards SN 2 reaction
the presence of the system H—C—C = C is known as
hyperconjugation
29. (d) The correct stability order is EXERCISE 4
I > III > II > IV 1. (d) During E2 elimination the double bond goes mainly
toward the least hindered carbon
+ +
O O 2. (c) The intermediate planar carbonium ion has equal chances
I III to be attacked from either side leading to racemisation
+ M effect, 6 hyper- +
conjugative H’s, + M, 3 hyperconjugative
. H’s, one CH 3 group 3. (a) R 3 C has three pairs of electrons in three bonds
2 CH 3 groups
(+ I effect) 4. (b) It is free radical addition (See mechanism of peroxide
effect)
+
+
IV CH3 CH3
II | |
5 hyperconjugative 5 hyperconjugative H’s 5. (b) H 3C - C = C - CH 3 . The more substituted alkenes
H’s 2 R groups one R group
have greater number of hyperconjugated forms. Hence
30. (d) Migrating tendency of hydride is greater than that of
they are more stable
alkyl group. Further migration of hydride from C–2 gives
more stable carbocation (stabilized by +R effect of OH 6. (b) The octet around N is complete, hence it has no
group and +I and hyperconjugative effects of methyl electrophilic character. N has no unshared pair of electrons
group). to act as nucleophile

H H 7. (d) SbCl5 pulls Cl– to form SbCl 6- leaving behind planar


1 2 3+ 4 5
1, 2 hydride
H3C – C — C– C – CH3 ¾¾¾¾¾ ® +
shift C 6 H 5 - CH - CH 3 carbonium ion. It can be attacked
OH H CH3 from either side leading to racemic mixture.

8. (b) When in R 2 C : (substituted carbene ), the two electrons


H H are present in the same orbital with opposite spin it is
+
H3C – C — C– C – CH3 called singlet state.
9. (c) It is nucleophilic substitution reaction. The nucleophile
..OH H CH3
..

is OH– obtained from KOH


More stable carbocation
10. (a) 3 - Halopentane will always give pentene -2 on dehydro-
31. (c) o-Hydroxybenzoic acid is strongest acid and the halogenation
decreasing order of acidity is 11. (a) SO3 is an electrophile since it can accept a lone pair of
electrons in a- subshell
COOH COOH COOH OH 12. (c) Dehydration of alcohol always leads to the intermediate
OH formation of carbonium ion. In case of n- propyl alcohol,
> > > +
the intermediate CH3 - CH 2 - CH 2 (1°) rearranges to
+
OH CH3 NO2 more stable CH3 - CH 2 - CH3 (2°) ion.
General Organic (Basic Concepts) 425
13. (a) Chloral hydrate is stabilised by H - bonding
O O O O
|| || ||
OEt OEt

:

O O O O
|| || || ||
- MeI
.. .. +
OEt ¾¾
¾® OEt
14. (c) CH 2 = CH - Cl: « CH 2 - CH = Cl: a double bond is
.. ..
Na
formed between C and Cl. Hence it is less reactive due to
resonance 29. (a) Five membered ring is being converted into six membered
ring. Hence the sixth bond must come from within the
15. (b) (CH 3 ) 3 COH gives tert (CH 3 ) 3 C + . The order of ring.
+ 30. (a) HI does not add on to propene by peroxide effect. Even
stability of carbonium ions is 3° > 2° > 1° > CH 3
in presence of peroxide the addition is according to
16. (c) Due to hyperconjugation toluene is most reactive for Markownikoff’s rule forming intermediate carbonium ion.
electrophilic substitution reaction ( see theory)
17. (d) Chlorinolysis is substitution by chlorine CH 3
18. (d) It gives trans - 2 - butene
+
19. (d) In I the unshared pair of electrons on N always available 31. (d) The structure carring +ve charge nearest to
for protonation in III due to presence of electronegative
O atom the electron density on N is decreased . In II and
IV resonance suppresses the basic character H Y
20. (b) The more the electrons withdrawing groups attached to
electrons repelling -CH 3 group is most stable.
- O atom, the more is the ease of leaving
21. (b) The addition of HCl to propene proceeds by ionic 32. (a) In (a) the separation of charges is maximum and
mechanism and not by free radical mechanism. Hence it electronegative atom oxygen carry –ve charge.
forms intermediate carbonium ion 33. (c) correct. (See text.)
22. (c) BrCH2CH2CO2H is the weakest acid as Br has less -I 34. (c) Addition reactions of alkenes are known as electrophilic
effect compared to F and is also far from -CO2H group. addition reactions.
23. (c) Formate ion 35. (c) The reaction (c) is Hoffmann elimination (see Reaction
Mechanisms.)
O O – 36. (d) The second ring has o/p directing O-atom and first ring
H–C H–C
O – O has m-directing -C - group. o/p directing group activate
||
Hence A, B bonds are of same length. O
24. (a) The order of stability of resonating structures carrying the benzene nucleus.
no charge > carrying minimum charge and each atom
having octet complete. – –
37. (a) ¬
¾® ¬
¾® ¬
¾®
25. (d) In A nitrogen is bivalent, in B and C carbon is pentavalent.

26. (c) The degree of unsaturation is equal to number of
hydrogen molecules needed to convert the given –
compound into an alkane. – –
¬
¾® ; ¬
¾®
27. (b) In (b) the pair has resonance. –

O O
|| || MeO- –
28. (b) ¾¾¾¾® –
OEt - MeOH RCH 2 (1°); (2°)
426 Chemistry

41. (a) CH 3CHDCl is 1° alkyl halide, react with aq. KOH by


CH 3 CH 3
+ SN2 mechanism which always leads to inversion of
CH – CH 3 ¾H CH - CH 3
38. (a) ¾
¾® | configuration.
|
+ O -H
OH +
|
H 42. (a) -CCl 3 ,- NO 2 and –NH3 are m - directing in nature.

-
CH 3 43. (b) CH3C H 2 > HC º C - > CH 3CH 2 O - basic character..
CH 3
-H O CH - CH 3 + The acid character being
¾¾¾
2¾®
+
2° CH 3 CH 3CH 2 OH > HC º CH > CH 3CH 3 . The weaker the
3° H
acid, the stronger the base (conjugate pair)
44. (b) (a),(c) and (d) are resonating structures.
+
CH 3
-H
O
–O
CH 3
CH 3 CH 3
Anhy. ZnCl
39. (c) R 3COH ¾¾ ¾ ¾ ¾
¾2®
R 3C.Cl
Conc. HCl O O

It is substitution reaction. R 3C.Cl being insoluble in O


H 2 O give turbid solution. CH 3
40. (d) Aliphatic halides react by SN2 mechanism.
–O
12E
Isomerism
ISOMERISM
CH3 CH3 CH3
Berzelius introduced the term Isomer (Gr. Isos=Same, Mers=parts)
to different compounds with the same molecular formula and the CH3
phenomenon was called Isomerism.
Types of isomerism : There are two main types of Isomerism
(i) Structural or constitutional isomerism : It is due to CH3
difference in the arrangement of atoms within the molecule. o-xylene m-xylene CH3
(ii) Stereo isomerism or space isomerism : It is due to different p-xylene
spatial arrangement of some atoms and groups.
Types of structural isomerism Position Isomers are also Known as regiomers.
(i) Chain isomerism : This is due to difference in the structure (iv) Metamerism : This is due to different alkyl groups attached
of the carbon chains. Examples. to the same functional group.
CH3 CH3
CH3CH2.CH2.CH2–CH3 CH–CH2–CH3
CH CH3–NH–CH2CH2CH3 CH 3 - NH - CH C 2H 5NH.C 2H 5
n-Pentane 3
Iso-Pentane CH3
Diethyl amine
Methyl n-propyl amine Methyl Isopropyl amine
CH3
|
CH 3 – C – CH 3 CH3
CH 3 - O - CH 2CH 2CH 3 CH 3 - O - CH C 2H 5 - O - C 2H 5
|
CH3 CH3 Diethyl ether
Neo-Pentane Methyl n-propyl ether Methyl Isopropyl ether

(ii) Functional isomerism : This is due to difference in the (v) Ring chain isomerism : Cyclic/acyclic Isomerism
functional groups
C2H5OH CH 3OCH 3 CH 3 - CH = CH 2
ethyl alcohol dimethyl ether Propene
Cyclopropane
CH3COOH HCOOCH3
Acetic acid Methyl formate
CH 3CH 2 CHO CH 3COCH 3 CH 3 - CH 2 - CH = CH 2
Propionaldehyde Acetone. But -1-ene Cyclobutane
(iii) Position or region isomerism : This is due to difference in
the positions of the same functional groups (vi) Tautomerism (Tauto = Same, Mers = Parts) : It is due to the
presence of a mobile atom in the molecule and the same
OH
| substance behaves in such a way as if it is a mixture of two or
CH 3 – CH 2 – CH 2 OH CH 3 – C H – CH 3 more compounds. Further we have
n- Propyl alcohol Iso - Propyl alcohol
428 Chemistry
(a) Dyad system : When the mobile atom oscillates between
S SH
two adjascent atoms eg || |
H-C º N H–N C H 2 N - C- NH 2 HN = C - NH 2
Thio Urea Enolic form
Hydrocyanic acidIso-hydrocyanic acid
(b) Triad system : When the mobile atom oscillates between
atoms one position ahead eg. NO O= N – OH
HO
O OH
|| | p-Nitrosophenol Keto form
CH 3 - C - CH 2 .COOC 2 H 5 CH3 - C = CH.COOC2 H5
Types of stereo isomerism :
93% Keto form 7% Enol form
(I) Optical Isomerism
Aceto acetic ester Aceto acetic ester
(II) Geometrical Isomerism and
Aceto acetic ester reacts with HCN, NH 2 OH,
(III) Conformational Isomerism
C6H5NHNH2 showing the properties of a ketone and
(I) Optical isomerism : The compounds having the same
also reacts with CH 3COCl, PCl5, Na showing the
molecular formula, the same structural formula but different
properties of OH group.
behaviour towards the plane polarised light are known as
It gives colour change with 1% FeCl3 a characteristic Optical Isomers.
Terminology used in optical isomerism.
æ OH ö
test of ç | ÷ (enol group) • Plane polarised light : Light having vibrations restricted
çè - C| = C| -÷ø to one plane only is called plane polarised light.
• Optically active compounds : The compounds capable
There exists an equilibrium between the two forms which of rotating the plane of polarisation of plane polarised
is dynamic in nature. light are known as optically active compounds.
• Optical activity : It is the ability of a substance to rotate
O OH the plane of polarisation of plane polarised light.
|| |
CH 3 - C - CH 3 CH 3 - C = CH 2 • Dextrorotatory compounds (d or +) : The compounds
which rotate the plane of polarisation of plane polarised
Acetone 99.5% Acetone (enol form) 0.5% light towards the right hand side are called dextro rotatory.
• Laevo rotatory compounds (l or –) : The compounds
O OH
|| | which rotate the plane of polarisation of plane polarised
CH3 - C- CH2 .COCH3 CH 3 - C = CH.CO CH 3 light towards the left hand side are called laevo rotatory.
• Specific rotation : The rotatory power of optically active
Acetyl acetone (keto form) Acetyl acetone (enol form)
compounds is compared in terms of specific rotation.
Specific rotation
OH O
Observed rotation in degrees
H =
Length of tube in decimeter ´ Concentration of solution in g/ml
H
q
[a]Dt =
Phenol l´C
Keto form of Phenol
• D corresponds to D line of Sodium light (l = 5893Å)
Triad system containing nitrogen • t corresponds to temperature
• Rotation is observed and measured with a polarimeter
O O–H
CH 3 - CH 2 - N CH3 – CH = N The specific rotation varies with light l and temperature.
O O • Optical activity due to crystalline strcuture : Some
Nitro ethane Aci Nitro ethane compounds are optically active only in crystalline form.
It dissolves in NaOH on account of aci form. Tautomeric They loss their optical activity when dissolved or fused
form which is less stable is called labile form. e.g. Quartz.
• Optical activity due to molecular structure : Some
O OH compounds are optically active in solid as well as in
|| |
H 2 N - C - NH 2 HN = C - NH 2 solution e.g. tartaric acid. Hence their optical activity is
Urea ( Keto form ) Enolic form due to their molecular structure which remains the same
in solid form and in solution.
Isomerism 429
• Asymmetric carbon atom : A carbon atom attached to
four different atoms and groups is called asymmetric COOH COOH
*
carbon atom. e.g. CH C . H OH HO H
3 H OH COOH
• Chirality : If the mirror image of the molecule is different
from the molecule it is said to be a chiral molecule. In CH3 CH3
such case if one configuration of the molecule is placed Dextrorotatory (d or +) Laevorotatory (lor –)
above its mirror image configuration, the similar atoms Lactic acid Lactic acid
and groups do not fall over each other and the (Source meat extract) (Obtained during
fermentation of
configurations are said to be nonsuperimposable.
Sucrose)
If object and mirror image configurations are superimposable
Racemic Lactic acid : It is an equimolar mixture of
(similar atoms and groups fall over each other) the molecule
d- and l- forms. It is optically inactive due to external
is said to be "achiral”.
Chirality is the necessary condition for a compound to be compensation of optical rotation. It is present in sour milk. It
optically active. can be resolved.
Enantiomers : Pairs of nonsuperimposable mirror images Examples of optically active compounds containing one
are called enantiomers. asymmetric C-atom.
Fischer Projections : Fischer projections are drawn with a
COOH COOH C2H5
cross, with chiral atom at the centre of the cross.
H OH H OH H CH2OH
A A A
D CH2 COOH C6H5 CH3
B C B C Malic acid Mandelic acid Amyl alcohol
B C D
D Number of optically active forms is given by 2n where n is
The horizontal line represents wedges (bonds) coming out number of asymmetric C-atoms different in nature.
of the plane of the paper. The vertical line represents dashed Resolution : The separation of d- and l- forms present in a
lines (bonds) in the plane of the paper (Bow-tie convention). racemic mixture is known as resolution.
The carbon chain is drawn along the vertical line of the Conditions for Chirality : Absence of (I) Plane of symmetry
projection with most highly oxidised carbon substituent at (II) Centre of symmetry and (III) Alternating axis of
the top. Fischer projections are very useful to determine symmetry.
chirality of a compound. Plane of Symmetry : A plane which divides the molecule in
Lebel and Van't Hoff’s theory about optical isomerism :
two portions in such a way that one portion is the mirror
The tetrahedral structure of a compound containing
asymmetric carbon atom (*Cabed) gives two configurations image of the other eg. Tartaric acid.
related to each other as object and its mirror image but are COOH
non-superimposable. H OH
Plane of symmetry
H OH
d d
COOH
c c
e b b e Meso tartaric acid
a a It is optically inactive due to internal compensation of optical
d or + l or – rotation. It can not be resolved.
Enantiomers or Enantiomorphs
Centre of Symmetry : It is a point from which lines, when
drawn on one side to meet the groups and produced to an
d d
equal distance on the other side of the point will meet the
same original groups.
e b b e
H NH CO CH3
a a
d or + l or –
(I) (II) C C

Enantiomers or Enantiomorphs CO NH
CH3 H
* Trans:1,4-dimethyl diketo piperazino (Inactive)
Optical isomerism of Lactic acid : CH 3 CHOH.COOH
Contains centre of symmetry
430 Chemistry

CH CH3 CH3 • When the configuration II is rotated through an angle


NH CO
180° the configuration I is obtained hence they are not
C C enantiomers but represent one single compound.
• d-tartaric acid is obtained from grapes and tamarind. Its
CO NH
H H mpt is 170°C.

erazino (Inactive) cis: 1,4-dimethyldiketo piperazino (Active) • l-tartaric acid is prepared by resolving racemic acid. Its
No centre of symmetry mpt is 170°C.

Alternating axis of symmetry : If a molecule is rotated • Meso-tartaric acid is obtained by oxidation of maleic
acid, heating d-tartaric acid with water at 170°C. Its mpt
360° is 143°C.
through an angle of about the axis and then reflected
n
• Racemic tartaric acid (dl or ±). It is obtained from Argol.
in a plane perpendicular to the axis, gives back the original Its mpt is 206°C. It is an equimolar mixture of d and l
molecule it is said to possess the n fold alternating axis of forms.
symmetry.
• Racemic tartaric acid can be resolved into d and l forms.
90° It is a mixture of two compounds.
CH3 H H CH3
• Meso-tartaric acid cannot be resolved. It is a single
2 1 1 4 compound.
H CH3 CH3 H
Optical Isomerism of the compound containing two dissimilar
H H3C H3C H C-atoms : Example a,b-dibromo cininamic acid and 2,3-
3 4 2 3 dihydroxy butanoic acid.
CH3 H H CH3
1,2,3,4-tetramethyl cyclo butane Mirror
CH3 H

H CH3

H H3C

CH3 H

Optical Isomerism of tartaric acid : (It contains two similar


asymmetric C–atoms).
COOH COOH

H OH HO H

H OH HO H

COOH COOH
Meso tartaric acid
(I) (II)
l
COOH COOH COOH
Let optical rotation due to chiral centre C3 and C2 be A and B
and further A > B. In the above case I-II and III-IV are pairs of
H OH HO H enantiomers where as I-III; I-IV, II-III, and II-IV are pairs of
HO H H OH diastereo-isomers.
Diastereo Isomers : Stereo isomers which are not mirror
COOH COOH COOH images of each other are called diastereo isomers. They have
different physical properties (mpt, bpt, solubility) and are
d-tartaric acid l-tartaric acid
often easy to separate by distillation, recrystallisation,
(III) (IV)
chromatography etc.)
Isomerism 431
The same compound pair are called the meso diastereoisomer (II) Substituted allenes : Unsymmetrically substituted allene
(I-II in case of Tartaric acid see above). Most diastereo- (CH2 = C = CH2) are optically active.
isomers are either geometric isomers or compounds with two a x a a
or more chiral atoms. C=C=C C=C=C
b y b b
Enantiomers : Enantiomers are non superimposable mirror (I) (II)
image isomers. They have identical physical properties (bpt, active Inactive
mpt, density etc.) except for their rotation of plane polarised
a x a a a x a b
light. They are much more difficult to separate. In nature C=C=C C=C=C
very often only one enantiomer
b y
is produced. Living organisms
b b b x b a
(I)
are one of the best sources of optically active compounds (III) (IV)
Inactive Active
(plants, enzymes, animals, cells etc.).
Enantiomeric excess (Optical Purity) : It is given by
Asymmetric synthesis : The synthesis of an optically active Optical purity = O.P.
compound from optically inactive compound under the
Observed rotation d -l
influence of an optically active compound without resolution = ´ 100% Or O.P. = ´ 100%
Pure enantiomer rotation d+l
is known as asymmetric synthesis.
where d and l are ratio of two forms
Geometrical isomerism : Alkenes with double bonds cannot
ROH
CH 3COCOOH ¾¾¾¾¾¾® CH 3COCOOR ¾¾¾
2H undergo free rotation
® CH 3CHOH.COOR ¾¾¾
H Oand can have different geometrical shapes
® CH CHOH.COOH
Pyruvic acid Optically active alcohol Optically active ester with two
Active lac different groups on3 each end of the double bond. e.g.
molecules C2a2b2, C2a2bd or C2abde.
2H H O a a a b a a
OCOOR ¾¾¾
® CH 3CHOH.COOR ¾¾¾
2 ® CH CHOH.COOH
3
y active ester Active lactic ester l -lactic acid
C=C C=C ; C=C
b b b a b d
(I) (II) (III)
Racemisation : The transformation of an optically active
a
isomer under the influence a light
of heat, a or some breagents
a a a d a d a e
into an inactive isomer is called racemisation. C=C ; C=C C=C
b b b a b d b a b e b d
Walden inversion / Optical (I)inversion : The conversion of (III) (IV) (V) (VI)
an enantiomer into another is called Walden inversion. I-II, III-IV and V-VI are pairs of geometrical isomers.
Nomenclature :
CH2 COOH CH2 COOH Cis Isomer : Contains the similar atoms or groups on the same
PCl5 side.
HO H H Cl Trans Isomer : Contains the similar atoms or groups on the
KOH
COOH COOH opposite side.
Examples :
d-Malic acid l-Chlorosuccinic acid CH3 CH3 CH3 H
C=C C=C
H H H CH3
CH2COOH
Cis-But-2-ene Trans But-2-ene
AgOH
H OH
H–C–COOH H–C–COOH
COOH
H–C–COOH HOOC–C–H
-Chlorosuccinic acid l-Malic acid
Cis-But-2-ene Trans But-2-ene Maleic acid Fumaric acid
Geometrical Isomerism of Oximes
Optical isomerism due to restricted rotation : Aldoximes :
(I) Diphenyls :
R H2NOH R
C=O C = NOH
H –H2O H
SO3H SO3H
H H
Aldehyde Aldoxime

R R H R H
C = NOH C C
SO3H SO3H H
H H
N N
OH HO
Enantiomers of diphenyl disulphonic acid
Syn Aldoxime Anti Aldoxime
432 Chemistry
Ketoximes : (III) By Oxidation :
COOH
R H2NOH R H COOH
C=O C = NOH C H OH
–H2O Dil. aq. KMnO4
R' R'
Unsymmetrical Ketone Ketoxime C or OsO4
H OH
H COOH
Maleic acid COOH
R R R' R R' meso-Tartaric acid
C = NOH H COOH
C C C Dil. aq. KMnO4
N N C or OsO4
OH HO HOOC H
Syn (R', OH) Anti (R', OH) Fumaric acid
COOH
COOH
Geometrical isomerism of azo compounds : Example azobenzene
COOH
H OH HO H
+
C6H5 HO H H OH
N=N HOOCN = N
C6H5 C6H5 C6H5 COOH COOH
d-Tartaric acid l-Tartaric acid
Syn-azo benzene Anti-azo benzene
Hence Maleic must be cis and fumaric must be trans.
E,Z system of Nomenclature for Geometrical Isomers:
Geometrical isomerism in cyclo alkanes : Cyclo alkanes also
If two high-priority groups are on the same side, the configuration
cannot undergo free rotation.
is Z (German, Zus ammen = togehter). If they are on opposite side,
the configuration is E (German; entgegen = opposite)

H H H CH3 (1) Br F (2) (2) Cl F (2)


C=C C=C
CH3 CH3 CH3 H
(2) Cl I (1) (1) Br I (1)
cis-1,2-dimethyl cyclo pentane trans-1,2-dimethyl cyclo pentane E Z
(2) CH3 CH3 (1) (2) CH3 H (2)
Determination of configuration of geometrical isomers :
C=C C=C
(I) Physical methods : In general the cis isomer has low mpt, (1) Br H (2) (1) Br CH3(1)
higher bpt, high density higher dipole moment, greater trans or E cis or Z
solubility, higher refractive index, higher heat of combustion.
Assignment of Priority : Atoms with higher atomic numbers receive
(II) By Cyclisation : higher priorites
I > S > O > N > 13C > 12 C > Li > 3 H(T) > 2 H(D) > H
O R and S Assignments : By Cahn-Ingold-Prelog. Enantiomers are
designated as (R) and (S) according to following rules
H – C – COOH D H–C–C
O (I) Atoms with higher atomic numbers receive higher priorities.
H – C – COOH H–C–C (II) When the same atom is bound directly to the chiral carbon,
we go to the next atom along the chain.
O (III) Double and triple bonds are treated as if each bond were to
a separate atom. e.g.
Maleic acid Maleic anhydride
H H
| |
- C = O º - C- O ;
| |
D
H - C - COOH ¾¾
® No anhydride O C
||
HO HOOC - C - H C C
Fumaric acid | |
- C º C - H º - C- C- H
| |
Hence Maleic acid must be cis isomer. C C
Thus between –CHO (O, O, H) and –CH2OH (O, H, H) the
former will have priority.
Isomerism 433
(IV)The molecule is drawn in three dimensions in such a way that Molecules with two or more chiral atoms.
the bond between the chiral carbon and the lowest priority 1 CH3
group heeds back into the paper. CH3
2
(V) Draw an arrow from the group of highest priority, to the H Br
second, to the third priority group. H Br
3
(VI)If the arrow is clockwise, the chiral carbon is assigned (R). If H Cl
CH.Cl.CH3
the arrow is anticlockwise the chiral carbon is assigned (S). 4
CH3
Example : Alanine
NH2 NH2 (3)
COOH H
Cl CH3 CH3 Cl
C C (1)
H CH3 CH3 COOH HC Br Br CH (2)
CH3 H H CH3
R-enantiomer
2S
NH2 CH3
H
CH.Br.CH3
C C H Br
COOH HOOC CH3 H Cl
H Cl CH3
(S) enantiomer CH3
(Always exchange the groups twice to get the same
compound. If you exchange the groups once you get the CHBrCH3 CHBrCH3
enantiomer).
CH3 Cl Cl CH3
By using Fischer Projections :
H H
NH2 NH2 3R
H COOH CH3 COOH
CH3
CH3 H
H Br
Hence the compound is (2S, 3R).
H Cl
NH2
COOH HOOC CH3 CH3

H (2S, 3R)

(S) enantiomer

Move the group of lowest priority to the bottom.

1. Write down all possible isomers of C 4 H 8 O 2 and type of Esters :-


isomerism existing between them HCOOC 3 H 7 CH 3COOC 2 H 5 C 2 H 5 COOCH 3
Sol. Acids :-
Propyl formate Ethyl acetate Methyl propionate
CH 3CH 2 CH 2 COOH (CH 3 ) 2 CH . COOH
*
n - Butyric acid Isobutyric acid Aldol :- CH3CHOHCH 2CHO

b - hydroxybutyric aldehyde
434 Chemistry
(i) Acids are chain isomers 4 Write all possible isomers of each formula and report what
(ii) Acids,esters and aldol are functional isomers type of isomerism exists in each
(iii) Aldol contains asymmetric C-atom and can exist in (a) C4H11N (b) C4H8
two optically active forms and one racemic form Sol.

NH 2
OH |
|
CH 3 - CH 2 - CH 2 - CH 2 - NH 2 CH3 - CH 2 - C H - CH3
CH 3 - C*- CH 2 .CHO
| 1- amino butane (I) 2 - amino butane (II)
H
CH 3
2. Write the possible isomers of C 4 H 8 O and type of CH 3 |
CH 3 - C - CH3
isomerism | |
CH 3 - C H - CH 2 - NH 2 NH 2
CH 3 CH 3
Sol. C = O n - C3H 7 | 1 - amino 2- methyl propane (III) 2- amino 2- methyl propane (IV)
C2 H5 C=O CH 3 - CH - CHO
H CH3
|
(I) (II) (III)
CH 3 - CH 2 - CH 2 - NH - CH 3 CH3 - CH - NH - CH3
Ethyl methyl Ketone n -butyraldehyde Isobutyraldehyde
Methyl propylamine (V) methyl isopropylamine (VI)
Functional isomers I and II, I and III
Chain isomers II and III CH3
3. Write the possible isomers of C3 H 6 O 3 and type of isom- |
C2H5 - NH - C2 H5 CH3 - N - C 2 H5
erism existing between them
diethyl amine (VII) ethyl dimethylamine (VIII)
Sol. (HCHO) 3 CH 3CHOHCOOH CH 2 OH.CH 2 COOH Position isomers I and II, III and IV
(I) (II) (III) Chain isomers I and III, I and IV
Functional isomers 1°, 2° and 3° amines are functional iso-
Meta formaldehyde a - hydroxy b- hydroxy mers
propionic acid propionic acid Metamers :- VI, V, VII
Functional isomers I and II, I and III
C 4H8 :
Position isomers II and III
Ring chain isomers I and II, 1 and III (i) CH 3CH 2 .CH = CH 2 but -1-ene
(ii) CH 3CH = CH CH 3 but - 2 - ene
CH 2
O O CH 3 H CH 3 CH 3
C=C C= C
CH 2 CH 2 H CH 3 H H
O trans (A) cis (B)
CH 3
Trioxane (meta formaldehyde) (iii) C = CH 2 2 - methyl propene
Stereoisomers :- II will show optical isomerism due to the CH 3
presence of an asymmetric C- atom.
(iv) Cyclobutane
COOH COOH
CH 3
H OH HO H
(v) methyl cyclopropane
CH 3 CH 3 Chain isomers I and III, II and III
Position isomers I and II
Ring chain isomers I and IV, II and IV
It will have the one racemic formed (dl, ± ) Geometrical isomers A and B forms of II
Isomerism 435

5. Write are possible isomers of C 7 H 9 N 7. Write all possible isomers of C 4 H 6 O 4


Sol.
NH 2 NH 2 NH 2 COOH C H 2 - COOH
CH 3 Sol. CH 3 - HC |
COOH CH 2 - COOH

CH3 methyl malonic acid Succinic acid


CH 3
8. Write all possible metamers of C 4 H10 O .
o- Toluidine m - Toluidine p- Toluidine
Sol. C2H5 - O - C2H5 CH 3 - CH 2 - CH 2 - O - CH 3
NH CH 3 CH 2 NH 2 diethyl ether methyl n-propyl ether

CH 3
CH - O - CH 3
CH 3
N -methyl aniline Benzylamine
methyl iso-propyl ether
6. Write possible isomers of C8 H 8 O 2
9. Write Keto-enol tautomers of phenol C 6 H 5 OH

CH 3 CH 3 CH 3 O
OH
COOH H
Sol. Sol. H
COOH
COOH enol form Keto form
o- Toluic acid m - Toluic acid p - Toluic acid 10. Write possible isomers of C 4 H 4 O 4 and type of isomerism
exhibited by them
COOCH 3 OCOCH 3 CH 2COOH
Sol. H - C - COOH HOOC - C - H
|| ||
H - C - COOH H - C - COOH
Maleic acid Fumaric acid
methylbenzoate phenyl acetate phenylacetic acid
These are Geometrical isomers.
436 Chemistry

Very Short/ Short Answer Questions CH3 CH3


1. What type of isomerism is shown by butane and isobutane? (b) H OH
and
HO H
2. A compound is formed by the substitution of two chlorine HO H H HO
CH3 CH3
atoms for two hydrogen atoms in propane. What is the
number of structural isomers possible ? CH3 CH3
3. An alcohol with no carbon-carbon double bond has the H OH HO H
(c) and
formula C4H8O. What are the possible structures? HO H HO H
4. Draw all possible isomers for a compound with molecular CH3 CH3
formula C8H10 containing benzene ring. Also, give IUPAC
CH3 CH3
name to these isomers.
H OH H OH
5. An aliphatic amine has a molecular mass of 59. Draw all its (d) and
HO H H OH
possible isomers. CH3 CH3
Long Answer Questions 14. Among the following four structures I to IV,
6. Draw all possible isomers of a compound with molecular CH3 O CH3
formula, C4H10O. Classify them as chain and position | || |
C 2 H 5 - CH - C3 H 7 , CH 3 - C - CH - C 2 H 5 ,
isomers.
( I) ( II )
Multiple Choice Questions
7. In which of the following, functional group isomerism is not CH3
H |
possible? | Å
H- C C2H5 - CH- C2H5
(a) Alcohols (b) Aldehydes
| ,
(c) Alkyl halides (d) Cyanides H ( IV )
8. Which of the following compounds will show metamerism? ( III)
(a) CH3–CO–C2H5 (b) C2H5–S–C2H5 it is true that
(c) CH3–O–CH3 (d) CH3–O–C2H5 (a) only I and II are chiral compounds
9. The number of possible open chain (acyclic) isomeric (b) only III is a chiral compound
compounds for molecular formula C5H10 would be (c) only II and IV are chiral compounds
(a) 8 (b) 7
(d) all four are chiral compounds
(c) 6 (d) 5
15. Which of the following compounds is optically active ?
10. Which are isomers ?
(a) CH3CHClCOOH (b) CH3CH2CH2CH3
(a) Ethyl alcohol and dimethyl ether
(c) (CH3)2CHOH (d) (CH3)3CCl
(b) Acetone and acetaldehyde
16. Keto-enol tautomerism is observed in :
(c) Propionic acid and propanone
(d) Methyl alcohol and dimethyl ether O O
r || ||
11. C6H5C º N and C6H5N = C are which type of isomers? (a) H 5 C 6 - C - CH 2 - C - C 6 H 5
(a) Position (b) Functional
(c) Tautomerism (d) Linkage O
||
12. Which of the following compounds exhibits stereoisomerism ? (b) H 5 C 6 - C - CH 3
(a) 2-methylbutene-1
(b) 3-methylbutyne-1 O
(c) 3-methylbutanoic acid ||
(c) H 5 C 6 - C - H
(d) 2-methylbutanoic acid
13. Which of the following pairs of compounds are enantiomers? (d) Both (a) and (b)
CH3 CH3 17. The following compounds differ in
H Cl H Cl
(a) HO H
and
HO H
C=C C=C
H OH HO H H Cl Cl H
CH3 CH3
Isomerism 437
(a) configuration (b) conformation 18. The molecular formula of diphenyl methane,
(c) structure (d) chirality
CH2 , is C13H12.
How many structural isomers are possible when one of the
hydrogens is replaced by a chlorine atom?
(a) 6 (b) 4 (c) 8 (d) 7

1. Which organic structure among the following is not an


9. Which of the following compounds is isomeric with 2, 2, 4,
isomer of the compound
4- tetramethylhexane?
CH3–CO–CH2CH2CH2CH3 ?
(a) 3-ethyl -2, 2- dimethylpentane
(a) CH3CH2OCH =CHCH2CH3
(b) 4-isopropylheptane
(b) CH3CH = CHCH2CH2CHO
(c) 4-ethyl-3-methyl-4-n propyloctane
(c) (CH3)2CH–CO–CH2CH3
(d) 4, 4-diethyl-3-methylheptane
(d) CH3CH2COCH2CH2CH3 10. The pair of structures given below represent
2. The least number of carbon atoms in alkane showing CH3 CH3
isomerism is H Cl H H
(a) 3 (b) 1
(c) 2 (d) 4
H H H H
3. The number of possible alkynes with molecular formula C5H8
is
CH3 CH2Cl
(a) 2 (b) 3
(a) enantiomers
(c) 4 (d) 5
(b) diastereomers
4. The total number of isomers for C4H8 is
(c) structural isomers
(a) 5 (b) 6 (d) two molecules of the same compound.
(c) 7 (d) 8 11. Which of the following compounds will show metamerism?
5. The number of possible open chain (acyclic) isomeric (a) CH3–CO–C2H5 (b) C2H5–S–C2H5
compounds for molecular formula C5H10 would be (c) CH3–O–CH3 (d) CH3–O–C2H5
(a) 8 (b) 7 12. The number of ether metamers represented by the formula
(c) 6 (d) 5 C4H10O is
6. Consider the following statements : A hydrocarbon of (a) 4 (b) 3
molecular formula C5H10 is a (c) 2 (d) 1
I. monosubstituted alkene 13. A compound with molecular formula, C7H16 shows optical
isomerism, the compound will be
II. disubstituted alkene
(a) 2, 3-dimethylpentane (b) 2, 2-dimethylpentane
III. trisubstituted alkene
(c) 2-methylhexane (d) None of these
Which of the following statement(s) is(are) correct? 14. Which are isomers ?
(a) I, II and III (b) I and II (a) ethyl alcohol and dimethyl ether
(c) II and III (d) I and III (b) acetone and acetaldehyde
7. How many cyclic isomers of C5H10 are possible? (c) propionic acid and propanone
(a) 4 (b) 3 (d) methyl alcohol and dimethyl ether
(c) 2 (d) 5 15. Methoxyethane and propanol are the examples of isomerism
8. How many isomers can C5H12 have ? of the type
(a) structural (b) position
(a) 3 (b) 2
(c) functional (d) tautomerism
(c) 4 (d) 5
438 Chemistry
16. The compound C4H10O can show 28. Which of the following compounds exhibits geometrical
(a) metamerism (b) functional isomerism isomerism ?
(c) position isomerism (d) All of these (a) C2H5Br (b) (CH)2(COOH)2
17. Isomers of propionic acid are (c) CH3CHO (d) (CH2)2(COOH)2
(a) HCOOC2H5 and CH3COOCH3 29. Which is a pair of geometrical isomers ?
(b) HCOOC2H5 and C3H7COOH
Cl Br Cl Br
(c) CH3COOCH3 and C3H7OH
I. C=C II. C=C
(d) C3H7OH and CH3COCH3 H Br H CH3
r
18. C6H5C º N and C6H5N = C are which type of isomers? Br
Cl CH3 H
(a) Position (b) Functional III. C=C IV. Cl C=C
Br H CH3
(c) Tautomerism (d) Linkage
19. A functional isomer of 1-butyne is (a) I and II (b) I and III
(a) 2-butyne (b) 1-butene (c) II,III and IV (d) III and IV
(c) 2-butene (d) 1, 3-butadiene 30. Which of the following compounds will exhibit geometrical
20. The number of primary amines of formula C4H11N is isomerism ?
(a) 1-phenyl-2-butene (b) 3-phenyl-1-butene
(a) 1 (b) 2
(c) 2-phenyl -1-butene (d) 1,1-diphenyl-1-propene
(c) 3 (d) 4
31. But-2-ene exhibits cis-trans-isomerism due to
21. The number of isomers possible for C7H8O is
(a) rotation around C3 – C4 sigma bond
(a) 2 (b) 3
(b) restricted rotation around C = C bond
(c) 4 (d) 5
(c) rotation around C1 – C2 bond
22. The total number of isomeric alcohols with the molecular
(d) rotation around C2 – C3 double bond
formula C4H9OH is
32. The restricted rotation about carbon-carbon double bond
(a) 2 (b) 3
in 2- butene is due to
(c) 4 (d) 5
(a) overlap of one s- and one sp2-hybridized orbitals
23. The number of possible acyclic structural isomers of C4H10O is (b) overlap of two sp2-hybridized orbitals
(a) 5 (b) 6 (c) overlap of one p-and one sp2-hybridized orbitals
(c) 7 (d) 8 (d) sideways overlap of two p-orbitals
24. Tautomerism will be exhibited by 33. Which of the following has zero dipole moment?
(a) (CH3)3CNO (b) (CH3)2NH (a) cis-2-butene (b) trans-2-butene
(c) R3CNO2 (d) RCH2NO2 (c) 1-butene (d) 2-methyl-1propene
25. IUPAC name for the compound 34. Which of the following has zero dipole moment?
Cl CH 2CH 3 (a) 1, 1-dichloromethane
C=C is (b) cis-1, 2-dichloroethene
H 3C I (c) trans-1, 2-dichloroethene
(a) E-2-chloro-3-iodopentene-2 (d) 1-chloroethane
(b) Z-2-chloro-3-iodo-2-pentene 35. The number of geometrical isomers of
(c) E-3-iodo-4-chloro-3-pentene CH3CH=CH–CH=CH–CH=CHCl is
(d) Z-3-iodo-4-chloro-3-pentene (a) 2 (b) 4
26. The prefixes syn and anti are used to denote (c) 6 (d) 8
(a) structural isomers (b) conformational 36. The number of isomers for the compound with molecular
isomers formula C2BrClFI is
(a) 3 (b) 4 (c) 5 (d) 6
(c) geometrical isomers (d) optical isomers
37. Fischer projection indicates
27. How many structural (including stereoisomers) isomers
could be obtained by replacing one hydrogen of propene (a) horizontal substituents above the plane
with chlorine ? (b) vertical substituents above the plane
(a) 2 (b) 4 (c) both horizontal and vertical substituents below the plane
(c) 3 (d) 5 (d) both horizontal and vertical substituents above the plane
Isomerism 439
38. An organic compound will show optical isomerism if 49. An organic molecule necessarily shows optical activity if it
(a) four groups attached to C atom are different (a) contains asymmetric carbon atoms
(b) three groups attached to C atom are different (b) is non-polar
(c) two groups attached to C atom are different (c) is non-superimposable on its mirror image
(d) all the groups attached to C atom are same (d) is superimposable on its mirror image
39. Which of the following molecules contain asymmetric carbon 50. Optically active isomers but not mirror images are called
atom ? (a) enantiomers (b) mesomers
(a) CH3CHClCOOH (b) CH3CH2COOH (c) tautomers (d) diastereomers
(c) ClCH2CH2COOH (d) Cl2CHCOOH 51. The two compounds shown in the figure below are
40. Which of the following compound is not chiral?
O O
(a) DCH2CH2CH2Cl (b) CH3CHDCH2Cl
or
(c) CH3CHClCH2D (d) CH3CH2CHDCl
41. Consider the following
1 2 3 4 5 6 7 (a) diastereomers (b) enantiomers
C H 3 - C H 2 - C H 2 - C H 2 - C H 2 - C H 2 - C H3
(c) epimers (d) regiomers
To make it a chiral compound, the attack should be on carbon 52. Which of the following is optically active ?
(a) 1 (b) 3 (c) 4 (d) 7 (a) Oxalic acid (b) Ethylene glycol
42. The (R)- and (S)- enantiomers of an optically active (c) Glycerol (d) Tartaric acid
compound differ in 53. How many stereoisomers are possible for tartaric acid?
(a) their reactivity with achiral reagents (a) 2 (b) 3
(b) their optical rotation of plane polarized light (c) 4 (d) 5
(c) their melting points 54. Meso- tartaric acid is optically inactive due to the presence
(d) None of these of
43. (+)-Mandelic acid has a specific rotation of + 158º. What (a) molecular symmetry (b) molecular asymmetry
would be the observed specific rotation of a mixture of 25% (c) external compensation (d) two asymmetric C atoms
(–)- mandelic acid and 75% (+)-mandelic acid ? 55. The correct statement about the compounds A, B, and C
(a) + 118.5º (b) –118.5º
COOCH3 COOH COOH
(c) – 79º (d) + 79º H OH H OH H OH
44. Optical activity is measured by H OH H OH HO H
is
(a) Polarimeter (b) Refractometer COOH COOCH3 COOCH3
(c) Spectrograph (d) Tracer technique (A) (B) (C)

45. Which one of the following compounds is not optically (a) A and B are identical
active ? (b) A and B are diastereomers
(a) CH3CH2CH(CH3)CH2Cl (b) CH3CH2CH(CH3)2 (c) A and C are enantiomers
(c) CH3–CHOH–COOH (d) CH3–CHCl–CH2Br (d) A and B are enantiomers
46. An optically active compound is 56. How many optically active stereoisomers are possible for
(a) 1-bromobutane butane-2, 3- diol ?
(b) b-bromobutyric acid (a) 1 (b) 2
(c) 2-bromo-2-methylpropane (c) 3 (d) 4
(d) 1-bromo -2-methylpropane 57. The number of possible enantiomeric pairs that can be
produced during monochlorination of 2-methylbutane is
47. Which of the following is optically active ?
(a) 2 (b) 3
(a) butane (b) 4-methylheptane
(c) 4 (d) 1
(c) 3-methylheptane (d) 2-methylpentane
58. The number of enantiomers of the compound
48. Which of the following compounds exhibits stereoisomerism ? CH3CHBrCHBrCOOH is
(a) 2-methylbutene-1 (b) 3-methylbutyne-1 (a) 0 (b) 1
(c) 3-methylbutanoic acid (d) 2-methylbutanoic acid (c) 3 (d) 4
440 Chemistry

59. Which of the following has a chiral carbon? 67. Which one of the following is the stablest structure of
(a) 2,3-dimethylpentane cyclohexatriene ?
(b) CH3–CH2–CH2–CHO (a) Chair form (b) Boat form
(c) (CH3)3CCH2CH2CH3 (c) Half chair form (d) Planar form
(d) (CH3)2CHCH2CH2CH2CH3 68. The isomers which can be converted into another form by
rotation of the molecule around single bond are
60. Me (a) geometrical isomers (b) conformers
Me H H (c) enantiomers (d) diastereomers
69. The most stable conformation of n-butane is
(a) skew-boat (b) eclipsed
Me H
(c) gauche (d) staggered -anti
Hydrogenation of the above compound in the presence of 70. In the following structures, which two forms are staggered
poisoned palladium catalyst gives conformations of ethane ?
(a) optically active compound
(b) an optically inactive compound H
H
H H H
(c) a racemic mixture H
H
(d) a diastereomeric mixture H
H
H H H
61. HO– H H
– CH = CH – CH – COOH will show.. H H
H H
OH (1) (2) (3)
(a) geometrical isomerism
(b) optical isomerism
H
(c) geometrical and optical isomerism H
(d) neither geometrical nor optical isomerism H
H H H
62. Number of stereoisomers of the compound, 2-chloro-4-
(4)
methylhex-2-ene is/are
(a) 1 (b) 2 (a) 1 and 4 (b) 2 and 3
(c) 4 (d) 6 (c) 1 and 2 (d) 1 and 3
63. The number of stereoisomers for pent-3-en-2-ol is 71. Which of the following is the most stable form of
(a) 2 (b) 4 cyclohexane ?
(c) 3 (d) 5 (a) boat (b) chair
H (c) twist boat (d) half chair
|
64. C6 H 5 CHO + HCN ¾¾
® C6 H 5 - C - CN 72. In the boat conformation of cyclohexane, the most
| destabilizing interaction is
OH
The product would be (a) eclipsing (b) 1, 3-diaxial
(a) a racemate (c) 1, 3-diequatorial (d) flagpole -flagpole
(b) optically active 73. The stability of the compounds
(c) a meso compound
(d) a mixture of diastereomers
65. The process of separation of a racemic modification into d -
and l- enantiomers is called
(i) (ii) (iii) (iv)
(a) resolution (b) dehydration
(c) revolution (d) dehydrohalogenation (a) (iv) > (iii) > (i) > (ii) (b) (i) > (iii) > (ii) > (iv)
66. An important chemical method to resolve a racemic mixture (c) (ii) > (iii) > (i) > (iv) (d) (iv) > (i) > (iii) > (ii)
makes use of the formation of 74. The Baeyer’s angle strain is expected to be maximum in
(a) a meso compound (b) enantiomers
(a) cyclodecane (b) cyclopentane
(c) diastereomers (d) racemate
(c) cyclohexane (d) cyclooctane
Isomerism 441

1. CH3 – CHCl – CH2 – CH3 has a chiral centre. which one of the 7. Which of the following does not show geometrical
following represents its R-configuration? [CBSE-PMT 2007] isomerism? [AIEEE 2002]
(a) 1,2-dichloro-1-pentene (b) 1,3-dichloro-2-pentene
C 2 H5 C2 H5 (c) 1,1-dichloro-1-pentene (d) 1,4-dichloro-2-pentene
| |
(a) H - C - CH3 (b) Cl - C - CH3 8. Racemic mixture is formed by mixing two [AIEEE 2002]
| | (a) isomeric compounds (b) chiral compounds
Cl H (c) meso compounds (d) optical isomers.
9. Among the following four structures I to IV, [AIEEE 2003]
CH3 C2 H5 O CH3
| | CH3 || |
|
(c) H - C - Cl (d) H 3C - C - Cl C 2 H 5 - CH - C3 H 7 , CH 3 - C - C H - C 2 H 5 ,
| |
C 2 H5 ( I) ( II )
H

2. How many stereoisomers does this molecule have ? H


| Å CH3
[CBSE-PMT 2008] H- C |
| C 2 H 5 - C H- C 2 H 5
CH 3CH = CHCH 2 CHBrCH3 H ,
( IV )
(a) 4 (b) 6 (c) 8 (d) 2 ( III)

3. Which of the following compounds will exhibit cis-trans it is true that


(geometrical) isomerism? [CBSE-PMT 2009]
(a) only I and II are chiral compounds
(a) Butanol (b) 2-Butyne (c) 2-Butenol(d) 2-Butene
4. The order of stability of the following tautomeric compounds (b) only III is a chiral compound
is : [NEET 2013] (c) only II and IV are chiral compounds
(d) all four are chiral compounds
OH O
| || 10. Amongst the following compounds, the optically active
CH 2 = C - CH 2 - C - CH 3 alkane having lowest molecular mass is [AIEEE 2004]
I H

<
O O |
|| || (a) CH3 - C -
CH3 - C- CH 2 - C- CH 3 |
C2H5
II
CH3
OH O |
| || (b) CH3 - CH 2 - CH - CH3
CH 3 - C = CH - C - CH 3
III (c) CH 3 - CH 2 - CH 2 - CH 3
(d) CH 3 - CH 2 - C º CH
(a) III > II > I (b) II > I > III
(c) II > III > I (d) I > II > III 11. Which of the following compounds is not chiral?
5. The type of isomerism present in nitropentamine chromium [AIEEE 2004]
(III) chloride is [AIEEE 2002] (a) 1-chloro-2-methyl pentane
(a) linkage (b) ionization (b) 2-chloropentane
(c) polymerisation. (d) optical (c) 1-chloropentane
6. A similarity between optical and geometrical isomerism is (d) 3-chloro-2-methyl pentane
that [AIEEE 2002] 12. Increasing order of stability among the three main
(a) each forms equal number of isomers for a given conformations (i.e. Eclipse, Anti, Gauche) of 2-fluoroethanol
compound is [AIEEE 2006]
(b) if in a compound one is present then so is the other
(c) both are included in stereoisomerism (a) eclipse, anti, gauche (b) anti, gauche, eclipse
(d) they have no similarity. (c) eclipse, gauche, anti (d) gauche, eclipse, anti
442 Chemistry
13. Which of the following molecules is expected to rotate the (a) S, S (b) R, R (b) R, S (c) S, R
plane of plane-polarised light? [AIEEE 2007] (a) 3-methyl-2-pentene (b) 4-methyl-1-pentene
COOH CHO (c) 3-methyl-1-pentene (d) 2-methyl-2-pentene
16. The alkene that exhibits geometrical isomerism is :
(a) H2N H (b) HO H
[AIEEE 2009]
H (a) 2- methyl propene (b) 2-butene
CH2OH
(c) 2- methyl -2- butene (d) propene
H2N NH2
17. The number of stereoisomers possible for a compound of the
(c) (d) H H molecular formula
Ph Ph CH3 – CH = CH – CH(OH) – Me is: [AIEEE 2009]
SH
14. Which one of the following conformations of cyclohexane is (b) 2 (c) 4 (d) 6 (d) 3
chiral? [AIEEE 2007] 18. Out of the following, the alkene that exhibits optical isomerism
(a) Boat (b) Twist boat is [AIEEE 2010]
(c) Rigid (d) Chair. (a) 3-methyl-2-pentene (b) 4-methyl-1-pentene
15. The absolute configuration of [AIEEE 2008] (c) 3-methyl-1-pentene (d) 2-methyl-2-pentene
HO2C CO2H 19. The number of stereoisomers obtained by bromination of
trans-2-butene is [IIT-JEE 2007]
OH
(a) 1 (b) 2 (c) 3 (d) 4
HO H H

1. The molecular formula of a saturated compound is 5. The maximum number of stereoisomers possible for
C 2 H 4 Br2 . This formula permits the existence of 3 - hydroxy - 2 -methyl butanoic acid is
(a) 1 (b) 2
(a) functional isomers (b) optical isomers
(c) positional isomers (d) cis-trans isomers (c) 3 (d) 4
2. The production of an optically active compound from a 6. The optically active tartaric acid is named as D - (+) - tartaric
symmetric molecule without resolution is termed acid because it has a positive
(a) Walden inversion (b) Partial racemisation (a) optical rotation and is drived from D - glucose
(c) Asymmetric synthesis (d) Partial resolution (b) pH in organic solvent
3. Which of the following is optically active (c) optical rotation and is derived from D - (+) -
(a) n - propanol (b) 2 - chlorobutane glyceraldehyde
(c) n - butanol (d) 3 - hydroxy pentane (d) optical rotation when substituted by deuterium
4. Geometrical isomerism is not shown by 7. The enol form of acetone after treatment with D2O gives:
CH3 O
OD
| | ||
(a) CH 3CH 2 C = C CH 2 CH 3 (a) CH 3 - C = CH 2 (b) CH 3 - C - CD 3
|
CH3
OH OD
| |
(b) C 2 H 5 - C = C - CH 2 I (c) CH 2 = C - CH 2 D (d) CD 2= C - CD 3
| |
H H 8. Two geometrical isomers are given by the following
compound
(c) CH 2 = C(Cl)CH3
(a) ethylidene bromide (b) acetylene tetrachloride
(d) CH 3 - CH = CH - CH = CH 2 (c) acetylene tetrabromide (d) acetylene dibromide
Isomerism 443
9. Only two isomeric monochloro derivatives are possible for : 16. How many stereoisomers are possible for the molecule?
(a) n - butane
(b) 2, 4 - dimethylpentane H CH 3
(c) benzene C=C OH
H C=C
(d) 2 - methylbutane
10. Keto-enol tautomerism is observed in CH 3
HO–C
O O
|| ||
(a) C 6 H 5 - C - H (b) C 6 H 5 - C - CH 3
O
(a) 2 (b) 4
(c) C 6 H 5 - C - C 6 H 5 (d) (d) None of these
|| (c) 6 (d) 8
O 17. The configuration of the compound
11. Ethoxy ethane and methoxy propane are
Cl
(a) geometrical isomers (b) optical isomers
(c) functional group isomers (d) metamers
12. The number of structural and configurational isomers of a (a) E (b) Z
bromo compound, C5H9Br, formed by the addition of HBr to (c) Both (a) and (b) (d) Neither (a) nor (b)
2 - pentyne respectively are 18. The alcohol product(s) of the reduction of 2-methyl-3-
(a) 1 and 2 (b) 2 and 4 pentanone with LiAlH4 is (are)
(c) 4 and 2 (d) 2 and 1 O
13. The structure, (i ) LiAlH diethyl ether
¾¾ ¾ ¾4¾ ¾ ¾ ¾¾®
CH3 H (ii) H 2O
C=C H
CH3 C (a) a single enantiomer (b) racemic mixture
COOH shows :
CH3 (c) two diastereoisomers (d) two structural isomers
19. How many stereoisomers of phenyl propylene oxide are there ?
(a) geometrical isomerism
H O H
(b) optical isomerism
(c) geometrical and optical isomerism
(d) tautomerism CH 3
14. Which of the alkenes shown below are member of a pair of E
- Z isomers ? (a) 1 (b) 2
H Cl (c) 4 (d) None of these
20. What is the R and S configuration for each stereogenic centre
(A) Cl (B) Cl in this sugar from top to bottom ?
O H
H H
H Cl H OH
HO H
(C) (D) H OH
H H CH 2 OH
H Br (a) R, R, S (b) R, S, S
(a) A and C only (b) B and C only (c) R, S, R (d) S, S, R
(c) A and D only (d) None of these 21. How many isomers have the compound bromomethyl
15. The correct priorities for the substituents shown below, cyclopentane (ignoring chirality) ?
according to the E-Z sequence rule is (a) 4 (b) 5
I. – CN II. – CBr (CH 3 ) 2 (c) 6 (d) 7
O 22. The structure shown below has the configuration
|| CH 2 CH 3
III. – COOH IV. – CH 2 - C - OCH3
|
O
C C(CH 3 ) 3
||
V. – C - H CH(CH 3 ) 2
(a) II, III, V, I, IV (b) V, II, I, IV, III (a) R (b) S
(c) III, IV, I, II, V (d) II, V, I, IV, III (c) Both (a) and (b) (d) Neither (a) nor (b)
444 Chemistry

23. The structure of allene is 29. Which of the following optically active compounds racemizes
CH 2 = C = CH 2 in dil. KOH/ CH 3OH solution ?
(a) staggered (b) planar O O
(c) eclipsed (d) None of these H CH 3
(a) (b)
24. The compound showing geometrical isomerism
CH 3 C6 H5
Br
(a) C 6 H 5 CH = NOH (b)
Br O
O
H 3C C6 H 5
(c) (d) All of these (c) (d)
H CH 3
25. Which of the following will show tautomerism ?
30. How many stereoisomeric aldohexoses are there ?
O (a) four (b) eight
(c) sixteen (d) eighteen
31. What is the relationship between open chain forms of D-
(a) glucose and D-altrose ?
(a) enantiomers
O (b) constitutional isomers
(c) diastereomers
(d) different conformations of the same compound
(b) O = = =O 32. Which of the following compounds will not have diastereo
isomers ?
O (a) COOH (CHOH ) 2 COOH
(b) CH 3 CHOH.COOH
(c)
(c) CH 2 OH (CHOH ) 4 CHO

O (d) CH 2 OH (CHOH ) 4 CH 2 OH
33. The correct number of chiral centres in

O C 2 H5 C 2 H 5

(d)
(a) 1 (b) 3
O (c) 4 (d) 2
34. Give the correct relationship among the followings :
26. Identify the Keto form of the following enol
CHO CH 2 OH CHO
H OH H OH H OH
OH H OH H OH HO H
(a) 1-penten-3-one (b) (E)-3-penten-2-one CH 2 OH CH 2 OH CH 2 OH
(c) 2-pentanone (d) (E)-3-pentanal
(A) (B) (C)
27. What is the relationship between keto and enol tautomers? (a) A and B are identical
(a) Resonance forms (b) A and B are diastereomers
(b) Constitutional isomers (c) A and C are diastereo isomers
(c) Stereoisomers (d) A and C are enantiomers
(d) Different conformations of the same compound 35. The number of metamers given by the compound
28. Which of the following has the highest percentage of enol in
a Keto-enol equilibrium ? – O – CH 2 .CH 3
(a) Hexanal (b) 2-hexanone (a) 3 (b) 4
(c) 2, 4-hexanedione (d) 2, 5-hexanedione (c) 5 (d) 6
Isomerism 445

36. The number of asymmetric C-atom created and number of 39. The stereo isomers formed when cis-2-butene is reacted with
possible stereoisomers when benzil (Ph.CO.CO.Ph) is reduced Br2 .
with LiAl H 4 . (a) meso-2, 3-dibromo butane
(a) 2, 3 (b) 2, 2 (b) racemic 2, 3-dibromo butane
(c) pair of diastereomers
(c) 2, 4 (d) 3, 2
(d) cannot be predicted
37. An optically active compound (A) is treated with NaI/acetone 40. Which of the followings is Z-isomer ?
(B) is obtained
Cl CH 3
(a) C=C
CH 3 CH 3 FF C 2 H5
NaI
H I ¾¾ ¾
¾® I H FF Br
acetone (b) C=C
C2 H5 C2 H 5 Cl H
O
(A) (B)
What is true about (B) ? CHO C–H
(c)
(a) optically active C=C
H H
(b) optically inactive and racemic form
CH 3OOC CH 3
(c) optically inactive
(d) C=C
(d) None of these H OH
38. The following compounds A, B, C have R or S configurations 41. The two compounds given below are

OH OH CºN HO COOH and HO COOH

HOOC H H CH 3 CH 3 C º CH (a) identical (b) enantiomers


(c) diastereo isomers (d) regiomers
CH 3 CH = CH 2 CH 2 OH 42. Which of the following have an enol form ?

(A) – CHO (B) (CH 3 ) 3 C.CHO


(A) (B) (C)
(a) R, R, S (b) R, S, R (C) CH 3CCl 2 .CHO
(c) R, R, R (d) S, R, S (a) Only A (b) Only B
(c) A and C only (d) None have enol form
446 Chemistry

EXERCISE 1 6. (a) As sketched in the above question , C5H10 may be


1. Chain or nuclear isomerism. monosubstituted (i) and (iv), disubstituted as in (ii), (iii)
and (v) and trisubstituted as in (vi)
2. Four: 1, 1–dichloropropane (CH3CH2CHCl2),
7. (d) C5H12 has 1º unstauration, so isomeric cyclic compounds
1, 2–dichloropropane (CH3CHClCH2Cl),
(having 1º unsaturation) may be in the form of five-four,
2, 2–dichloropropane (CH3CCl2CH3) and or three - membered ring compounds.
1, 3–dichloropropane (ClCH2CH2CH2Cl). CH2CH3
7. (c) 8. (b) 9. (c) 10. (a)
11. (b) 12. (d) 13. (b) 14. (a)
15. (a) 16. (d) 17. (a) 18. (b)
cyclopentane methylcyclobutane ethylcyclopropane
H H H CH3
EXERCISE 2
1. (b) Structures (a), (c) and (d) have the same molecular formula
(C6H12O) while (b) has C6H10O as molecular formula cis trans
2. (d) CH3 CH3 H
CH3
3. (b) CH3CH2CH2CºCH, (CH3)2CHCºCH, CH3CH2CºCCH3 cis- and trans - 1, 2-dimethylcyclopropane
I II III CH3
|
4. (b) CH3CH2CH=CH2 CH3CH=CHCH3 (CH3)2C=CH2 CH 3 - CHCH 2 CH 3
8. (a) CH3CH2CH2CH2CH3
1-butene (i) 2- butene (ii), (iii) 2-methylpropene
(cis,- trans) (iv) n- pentane 2-methylbutane
CH3
CH3
|
CH 3- C - CH 3
|
CH3
cyclobutane (v) methylcyclopropane (vi) 2, 2-dimethylpropane

5. (c) C5H10 has 1º degree of unsaturation since the isomers 9. (b) 2, 2, 4, 4 - Tetramethylhexane has 10 carbon atoms, only
are acyclic, all of these are alkenes. For writing the 4-isopropylheptane has also 10 carbon atoms so these
isomers, first introduce the double bond at different two are isomers.
possible positions, and then consider the possibility of 10. (c) Convert these Newmann projections into open chain
branching in the alkyl group. structures.
CH3CH2CH2CH =CH2 CH3CH2CH=CHCH3 CH3 CH3
1-pentene (i) 2- pentene, (cis,- trans) (ii), (iii) H Cl H H
H H H H
CH 3 CH3
| | CH3 CH2Cl
CH 3- C H - CH = CH 2 CH 3CH 2 C = CH 2 Both structures have same molecular formula C4H9Cl,
thus these are isomers. However, the two have different
3-methyl-1-butene, (iv) 2-methyl-1-butene, (v)
groups, viz CH 3 and CH 2Cl, so these are neither
CH 3 enantiomers nor diastereomers. Hence these are
| structural isomers.
CH 3 - C = CHCH 3
11. (b) 12. (b) 13. (a) 14. (a) 15. (c)
2-methyl-2-butene, (vi) 16. (d) 17. (a) 18. (b) 19. (d)
Isomerism 447
20. (d) Introduce - NH2 at various possible positions to get
C6H5
different isomers which is possible only, on C1 and C2. |
Now observe isomerism in the alkyl part C 6 H 5 CH 2 CH = CHCH 3 CH 2 = CH CHCH3
(C4H9–) of the amine which is three
1-phenyl-2-butene 3-phenyl-1-butene
NH 2
| C6 H 5
CH3CH2CH2CH2NH2 CH 3CH 2 C HCH 3 |
CH 2= C CH 2 CH 3 (C6H5)2C=CHCH3
(CH3)2CHCH2NH2 (CH3)3CNH2
2-phenyl-1-butene 1, 1- diphenyl -1-propene
21. (d) Molecular formula C7H8O points out the presence of
one benzene ring (relatively less number of H atoms); so Note that only in first structure both of the doubly
it can be C6H5CH2OH and its other isomers. bonded carbon atoms have different groups.
C6H5CH2OH C6H5OCH3 OH.C6H4.CH3 31. (b) 32. (d) 33. (b) 34. (c)
Benzylalcohol (i) Anisole (ii) o -, m-, p- Cresols, (iii), 35. (d) The given structure has three double bonds whose each
(iv) and (v) carbon atom is differently substituted hence number of
22. (c) Alcohol may be 1º, 2º and 3º; further analyse the geometrical isomers will be 2n = 23’ = 8, where n is the
possibility of isomerism in each type of alcohol number of double bonds whose each carbon atom is
differently substituted.
CH 3 36. (d) Consider one doubly bonded carbon atom first, attach
|
CH3CH2CH2CH2OH CH 3CH 2 C HO H any one halogen atom (say F) on this C on one bond,
and then attach the three remaining halogen atoms, of
1-butanol, 1º 2-methylpropanol, 1 º course one by one, on the other bond.

OH CH3 F F F
| | C=C ; C=C ; C=C
CH 3CH 2 C HCH 3 CH 3- C - OH Cl Br I
|
CH3 Now attach the remaining two halogen atoms in each of
the above structures, for which there are two possibilities
2-butanol, 2º 2-methyl-2-propanol, 3º for every structure; so on the whole there will be 6
23. (c) There are four isomeric alcohols (mentioned in geometrical isomers.
the above question) and three isomeric ethers
F Br F I F Cl
corresponding to the formula C4H10O; CH3OC3H7–n; C=C C=C C=C
CH3OC3H7–iso and C2H5OC2H5 Cl I Cl Br Br I
24. (d) Only RCH2NO2 has a -hydrogen atom (I) (II) (III)
O OH F I F Cl F Br
+ + C=C C=C C=C
RCH2 – N RCH = N Br Cl I Br I Cl
O– O– (IV) (V) (VI)
25. (a) 26. (c) 37. (a) 38. (a) 39. (a)
27. (b) Replace three different types of hydrogen atoms,one by 40. (a) None of the carbon atoms in DCH2CH2CH2Cl is chiral
one. i.e., each carbon atom is achiral (symmetric).
1 2 3 4 5 6 7
CH3 C=CH2 CH3 CH=CH2 CH2CH=CH2 41. (b) C H 3 - C H 2 - C H 2 - C H 2 - C H 2 - C H 2 - C H 3

Cl Cl Replacement of one hydrogen atom at C2,C3,C5 or C6


(ii) (i) can make the resulting compound chiral.
CH3CH=CHCl
cis- and trans, (iii) and (iv) 42. (c)
43. (d) Specific rotation of the mixture
28. (b) (CH)2(COOH)2 is HOOCCH = CH COOH, hence here
geometrical isomerism is possible. æ 75 ö æ 25 ö
=ç ´158 ÷ + ç ´ -158 ÷ = + 79 º
29. (c) è 100 ø è 100 ø
30. (a) Draw the structure, and observe whether the two doubly 44. (a) 45. (b) 46. (b) 47. (c) 48. (d)
bonded carbon atoms are differently substituted or not. 49. (c) 50. (d)
448 Chemistry
51. (a) Since the configuration at the chiral centre is same , the
CH3 CH3
two structures (optical isomers) are not mirror images, |
hence these are diastereomers. *|
59. (a) CH 3 - CH - CH - CH 2 CH 3
52. (d) 53. (b) 54. (a) 2, 3 - dimethylpentane
55. (d) Rotation of B through 180º within the plane of the paper (has chiral C)
gives D which is an enantiomer of A, hence A and B are
enantiomers CH3
|
COOH CH 3 - C - CH 2 CH 2CH 3
|
H OH CH3
Rotate through 180º
H OH ¾¾¾¾¾¾¾¾¾ ®
with the plane of the paper 2, 2 - dimethylpentane
COOCH3 (No chiral carbon)
(B)

CH3
COOCH3 COOCH3 |
CH 3 - CH - CH 2 CH 2 CH 2 CH 3
OH H H OH
HO H H OH 2 - Methylhexane
(No chiral carbon)
COOH COOH
(D) (A)

56. (b) Butane-2, 3-diol (CH3CHOHCHOHCH3) has two similar Me


Me H H
chiral carbon atoms (similar to tartartic acid), hence like H2
tartaric acid it has three optical isomers (d,-l-, and meso-), 60. (b) Pd-BaSO4
of which only two (d– and l– ) are optically active. H
Me
57. (d) First draw possible different structures obtained on
monochlorination of 2-methylbutane,
CH3CH(CH3)CH2CH3. Me
Me Me H
H2 H
CH 3 CH3
| | Pd-BaSO H
*
(i) ClCH 2 - C HCH 2 CH 3 (ii) CH 3 - C - CH 2 CH 3
| H H
Optically active
Cl
Optically inactive Due to cis-addition of H2 to the triple bond, the reduced
product has a plane of symmetry and hence is optically
inactive.
CH 3 Cl CH 3 61. (c) Geometrical isomerism due to the presence of a double
| |* | bond whose each carbon has two different substituents.
(iii) CH 3 - CH - CH CH 3 (iv) CH 3 - CH - CH 2 CH2 Cl
Optical isomerism due to presence of a chiral carbon,
Optically active Optically inactive
*
- C H OH COOH .
Thus structures (i) and (iii) are optically active, each has
one chiral carbon; so each structure will give one *
62. (c) CH 3 - C = CH - C H - CH 2 CH 3
enantiomeric pair; thus total enantiomeric pairs will be | |
two. Cl CH3
2 - Chloro - 4 - methylhex - 2 - ene
* *
58. (d) The structure CH 3 C H Br C H Br COOH has two
Geometrical isomerism due to C = C, each C has two
different chiral carbon atoms, hence number of different groups. Each geometrical isomer (cis–, or trans)
enantiomers (optically active forms) is 2n = 22 = 4 will show optical isomerism due to the presence of chiral
C. Hence total number of isomers will be 2 × 2 = 4
Isomerism 449

* EXERCISE 3
63. (b) CH 3 - CH - CH = CH CH 3 ;
|
OH 2
Pent - 3 - en - 2 - ol C2H5
3
As in above question, total number of stereoisomers
1. (b) Cl — C — CH3
=2×2=4 1
64. (a) Since during the reaction, a chiral carbon is created and
further since the CN– ion can attack the planar aldehyde H 4
group both from the top and the bottom face of the R-configuration
aldehyde group with equal ease, therefore, a 50:50 mixture
of the two enantiomers, i.e. a racemic mixture is obtained. 2. (a) In the molecule
65. (a)
CH3CH = CH CH 2 CH - CH3,
66. (c) Diastereomers have different solubility, m.p. and b.p., |
hence they can be separated by fractional crystallisation. Br
67. (d) Since all the six carbon atoms of cyclohexatriene the number of stereoisomers is given by sum of
(benzene) are sp2 hybridized, therefore its stablest geometrical isomers (because of presence of C = C) and
structure is planar form. optical isomers (because of presence of chiral carbon
68. (b) 69. (d) atom).
70. (c) Note that in structures 1 and 2, every two adjacent Number of geometrical isomers = 2 (one C = C is present).
hydrogen atoms are at maximum possible distance from Number of optical isomers = 2 (one chiral carbon atom).
each other (staggered conformation). Total number of stereoisomers = 2 + 2 = 4
71. (b) 3. (d) Alkenes with double bonds cannot undergo free rotation
72. (d) Boat conformation of cyclohexane has two types of and can have different geometrical shapes with two
destabilizing interactions, i.e. eclipsing and flagpole- different groups on each end of the double bond.
flagpole. Out of these flagpole-flagpole interaction is
the most destabilizing. CH3 CH3 CH3 H
C=C C=C
73. (a) Cyclohexane (iv) is non-planar and has chair H H H CH3
conformation. In this conformation, the bond angle is cis-But-2-ene trans-But-2-ene
the normal tetrahedral angle (109°, 28’) and thus has no
4. (a) Enolic form predominates in compounds containing
angle strain and hence is most stable. The rest of the
two carbonyl groups separated by a – CH2 group. This
molecules are nearly planar and hence their stability
is due to following two factors.
depends upon the angle strain in accordance with
Baeyer’s strain theory. Since cyclopropane has higher (i) Presence of conjugation which increases stability.
(ii) Formation of intramolecular hydrogen bond
æ 109° 28'– 60° ö
angle strain ç = 24°, 44' ÷ than between enolic hydroxyl group and second
è 2 ø carbonyl group which leads to stablisation of the
molecule. Hence the correct answer is III > II > I.
æ 109° 28'–108° ö
cyclopentane ç = 0°, 44' ÷ . Therefore 5. (b) The nitro group can attach to metal through nitrogen as
è 2 ø (–NO2) or through oxygen as nitrito (–ONO).
cyclopentane (iii) is more stable than cyclopropane (i). 6. (c) Optical and geometrical isomers are stereoisomers
Further, because of the presence of a double bond in a
three membered ring, cyclopropene (ii) is the least stable. Cl
7. (c) C = CH-CH 2 - CH 2 CH3 does not show geometrical
Thus the order of stability is (iv) > (iii) > (i) >(ii). Cl
74. (a) As the size of the ring increases, the internal angle isomerism due to presence of two similar Cl atoms on the
increases accordingly. As a result, the deviation from same C-atom.
the tetrahedral angle and hence the angle strain increases 8. (d) Racemic mixture is formed when enantiomers are mixed in
as the size of the ring increases. Thus, planar equimolar proportion
cyclodecane is expected to have the maximum angle
9. (a) A chiral object or structure has four different groups
strain.
attached to the carbocation.
450 Chemistry
10. (a) Only 2- cyclopropyl butane has a chiral centre. 18. (c) For a compound to show optical isomerism, presence
11. (c) 1-chloropentane is not chiral while others are chiral in of chiral carbon atom is a necessary condition.
nature
H
Cl Cl |
C–C–C–C–C; C–C–C–C–C; H 2 C = CH — C* — CH 2 - CH3
1–chloropentane *
2–chloropentane |
CH3
3- methyl-1-pentene
Cl C C Cl
C–C–C–C–C C–C–C–C–C
*
1–chloro-2-methyl pentane
*
3–chloro-2-methyl pentane
19. (a) Anti addition of Br2 on trans alkene provides meso
compound.
12. (c) The greater the distance between the atoms or groups
CH3
the more is the stability of conformers. Hence the order
H 3C H H Br
of stability is C= C + Br2 ¾CCl
¾¾ 4®
H CH 3 H Br
F F H F CH3
H H HO H
Therefore, no. of stereoisomers = 1
HO
H H H H H H
OH H H EXERCISE 4
anti or staggered eclipsed Skew or
Gauche 1. (c) C 2 H 4 Br2 represents 1, 1-dibromoethane and 1, 2 -
dibromethane which are position isomers
13. (b) The organic compounds which have chiral carbon atom 2. (c) It is definition of asymmetric synthesis
(a carbon atom attached to four different group or atoms
and do not have plane of symmetry rotate plane Cl
|
polarised light. 3. (b) CH3 - CH - CH 2 CH3 contains asymmetric C - atom
CHO
| 4. (c) The condition for geometrical isomerism is
HO - C*- H (C* is asymmetric carbon) a a a e
| c=c
CH 2 OH
or c=c
b b b d

14. (b) Chiral conformation will not have plane of symmetry.


OH CH3
Since twist boat does not have plane of symmetry it is | |
chiral. 5. (d) H 3C - C - C - COOH contains two asymmetric
15. (b) The absolute configuration is (R, R) | |
H H
(using priority rules to get the absolute configuration)
So the correct answer is (b)
C - atoms . Hence number of optical isomers is 2 2 = 4
16. (b) H3C CH3 H3C H 6. (c) Positive sign is for optical rotation (dextro rotatory)
C=C C=C and D - is for configuration. It is derived from
H H H CH3 CHO
|
* H– C – OH
17. (b) CH 3 – CH = CH – CHCH 3 D (+) glyceraldehyde
|
|
OH CH2OH
exhibits both geometrical as well as optical isomerism.
CHO
cis - R cis - S |
trans - R trans - S L (–)glyceraldehyde is HO– C – H
|
CH2OH
Isomerism 451

OH N C CH 3
| |
7. (a) The enol form of acetone is CH3 - C = CH 2 with | |
15. (a) - C - N , - C - CH 3 ,
| | |
OD N C Br
|
D2O it gives CH 3 - C = CH 2
(i) (ii)
a a
8. (d) BrCH = CHBr . It fulfills the condition of c=c OH H O-C
b b | | |
- C - O, - C - C - OCH 3 ,
| | | |
Cl O C H O
|
9. (a) CH3 - CH 2 CH - CH3 , CH3CH 2 CH 2CH 2 - Cl (2 from (iii) (iv)
n-butane)
O-C
Cl | |
-C-O
(1from benzene) (All positions are the same) |
H
C C
(v)
C–C–C–C–C
o Arrange (NNN), (BrCC), (OOO), (CHH), (OOH) in
(3 from 2, 4 - dimethylpentane) ( there are three different increasing atomic number. The order is ii, iii, v, i, iv.
positions, ·, c, and o) 16. (a)

·C
17. (a) The groups of higher priority Cl and – C 2 H 5 are on
|
opposite sides.
· C- C - C- C
* o
18. (b) The resulting compound is C3 H 7 .CHOH.C 2 H 5 which
(4 from 2 - methyl butane) ( there are 4 different is optically inactive and reaction leads the racemisation.
positions, ·, c, o and *)

O OH 19. (c) H H and H C6 H5


| H
10. (b) C6H5–C–CH3 H 3C C6 H5 H 3C
C6H5– C = CH2

11. (d) There are metamers Each structure will have two configurations.

Br
| O H C - O
12. (b) C - C = C - C - C structural : 1, geometrical : 2 C 2
| H OH O - C - H
H 20. (c) HO H 4 H - C - OH 1
H OH HO - C - H
H CH 2 OH
| H - C - OH 3
C - C = C - C - C structural : 1, geometrical : 2.
| CH 2 OH
Br

Hence 2 structural and 4 geometrical isomers 2 1


13. (b) It contains one asymmetric C- atom. Hence exhibits optical
isomerism. 4 C 1 º 3 C 2 R configurations
14. (c) The structures B and C contain similar atoms on ethylenic 3 4
C-atoms. Hence do not exhibit isomerism.
Arrange the groups in order of priority by following the
text.
452 Chemistry

Br 30. (c) Aldohexose, contain four asymmetric carbon atoms and


Br CH 3
CH 3 exist in 2 4 = 16 stereo isomers.
31. (b) Constitutional isomers
21. (c) Each structure , ,
Br CHO
|
CHO H - C - OH
| |
CH 3 will give two configurations. Altrose is HO - C - H , Glucose is HO - C - H
| |
(CHOH) 3 (CHOH) 2
C | |
| 4 CH 2 OH CH 2 OH
H – C –H
C 32. (b) For diastereo isomerism the compound must have two
C C 4
| | | different asymmetric carbon atoms.
22. (a) 1 C – C – C – C–C 2 1 C 2 33. (d) The molecule contains two similar chiral centres.
| | | | 3 34. (c) A and C are diastereo isomers.
C C C C–C
– C
| 35. (b) Metamers are CH 2 - OCH 3 ,
H 3

3 OCH 3 OCH 3
CH 3 , ,
2 C 1 R configuration CH 3
4
H 3C – OCH 3
H H
23. (a) Allene has the structure C=C=C staggered
H H OH OH
| |
24. (d) Write cis and trans configuration for each. 36. (a) Ph.CO.CO. Ph ¾LiAlH
¾ ¾¾ 4®
Ph - C x - C x - Ph The
| |
O OH H H
product contains two similar asymmetric carbon atoms
25. (a) are tautomers. and two optically active and one optically inactive meso
form.

O O CH3
|
37. (b) The reaction proceeds through planar CÅ
3 5 H C2 H 5
26. (b) 1
| 2 4 leading to racemisation.
OH O 38. (d)
The keto form has E configuration. Br Br
27. (b) The keto enol tautomers are constitutional isomers. CH 3 CH 3 | |
39. (b) Br2
C=C ¾¾
¾® CH3 - C - C - CH3
28. (c) Enol form of 2, 4-hexane dione
H H | |
O H O H H

CH 3C = CH - C - CH 2 - CH 3 Racemic form
40. (c)
is stabilised by H-bonding.
41. (a) The given compounds are identical.
O 42. (d) None can have the enol form due to absence of H-atom
||
29. (d) The compounds containing - C - group adjacent to O
||
asymmetric carbon atom carrying a hydrogen easily on the carbon adjacent to - C - group.
racemise.
13
Hydrocarbons

SATURATED HYDROCARBONS, PARAFFINS OR (II) Derived names : In this system the higher hydrocarbons are
ALKANES : considered as derived from methane, eg.
They are open chain compounds of carbon and hydrogen having
all the atoms linked together by single covalent bonds, least H H CH3
reactive in nature | |
CH 3 - C - CH 2 - CH 3 CH 3 - C - CH
(Parum = little; affins = affinity) and having general formula CnH2n+2 | | CH3 ;
;
H H
NOMENCLATURE : ethyl m ethyl m ethane methyl iso propyl methane
(I) Trivial names : The first four members have the trivial names
derived from their preparation from corresponding alcohols (III) IUPAC system
containing same number of carbon atoms eg. methane from Types of carbon atoms :
methyl alcohol, ethane from ethyl alcohol and so on. After
(I) Primary (1°) - It is attached to one carbon atom only.
butane they are named according to Latin or Greek numerals
(II) Secondary (2°) - It is attached to two carbon atoms.
of the number of Carbon atoms present in them with class
suffix-ane. eg. Pentane (penta = 5) : hexane (hexa = 6) and so (III )Tertiary (3°) - It is attached to three carbon atoms.
on. (IV) Quaternary (4°) - It is attached to four carbon atoms.
The straight chain hydrocarbons are called normal (n) and
1° 1°
contain 1° or 1° & 2° carbon atoms. The branched chain C C
1° 2° 1°
hydrocarbons containing the group (CH 3 ) 2 CH - are called C – C 4°
–C–C–C

Iso and contain 1° & 3° or 1°, 2° and 3° carbon atoms. The C1°
hydrocarbons containing a quaternary carbon atom are
called neo (new). Isomerism : They exhibit chain isomerism

CH 3 - CH 2 - CH 2 - CH 3 CH 3 - C H - CH 3
; | ;
n -Bu tan e
CH 3
Iso - butane n-Pentane Isopentane Neopentane

Number of isomers of alkanes :


CH 3
|
CH 3 - C - CH 3 C 4 H10 C5 H12 C 6 H14 C 7 H16 C8 H18 C10 H 22
| ; 2 3 5 9 18 75
CH 3
Neopentane
454 Chemistry
Occurrence : The crude petroleum and natural gas contain (IX) By Wolf. Kishner reduction : From Carbonyl compounds
hydrocarbons from C1 to C40. Ozokerite (a neutral wax) is a mixture
NH .NH
of higher solid hydrocarbons. Waxes of some plants and animals C = O + 4H ¾¾ ¾
2 ¾¾
2® CH 2 + H 2 O
KOH / glycol
also contain some higher paraffins.
GENERAL METHODS OF PREPARATION OF (X) By Berthlot's reaction :
ALKANES Electric arc
C + 2H 2 ¾¾ ¾ ¾¾® CH 4
(I) From unsaturated hydrocarbons : (Sabatier and Senderen's 1200°C
reaction)
(XI) From metal carbides :
H 2 / Ni
C n H 2 n + H 2 ¾¾ ¾
¾® C n H 2 n + 2
200°-300°C Al 4 C 3 + 12 H 2 O ¾
¾® 4Al(OH ) 3 + 3CH 4

Ni / H Be 2 C + 4H 2 O ¾
¾® 2Be(OH ) 2 + CH 4
C n H 2n - 2 + 2H 2 ¾¾ ¾
¾2® C H
n 2n + 2
300°C
(XII) From carbon monooxide
(II) By heating anhydrous sodium salt of fatty acid with soda
lime (Lab method). Ni + C
CO + 3H 2 ¾¾¾® CH 4 + H 2 O
255°C
CaO
RCOONa + NaOH ¾¾¾ ® RH + Na 2 CO 3 (XIII) By reduction of alcohols, aldehydes, ketones and
Δ
carboxylic acids :
(III) By reduction of alkyl halides with nascent hydrogen.
Red P
ROH + 2HI ¾¾ ¾® RH + I 2 + H 2 O
RX + 2H ¾
¾® RH + HX 200°C

Reducing agents used are Na/C2H5OH; Zn/HCl;


Red P
RCHO + 4HI ¾¾ ¾® RCH 3 + 2I 2 + H 2 O
Zn–Cu couple; Hg–Al couple, HI/P. 150°C
(IV) Wurtz synthesis :
Red P
ether
RX + 2 Na + XR ¾¾¾® R – R + 2 NaX RCOR + 4HI ¾¾¾ ® RCH 2 R + 2I 2 + H 2 O
150º C
• Alkanes with odd number of carbon atoms can not be
prepared by this method. Red P
RCOOH + 6HI ¾¾ ¾® RCH 3 + 3I 2 + 2H 2 O
• Different alkyl halides give mixture of hydrocarbons viz. 150°C
R – R, R' – R' & R – R'. (XIV) Corey-House alkane synthesis :
• 3° alkyl halides give alkenes by dehydrohalogenation.
ether
(V) By Frankland's method : RX + 2Li ¾¾¾ ® RLi + LiX
Alkyl lithium
R - I + Zn + I - R ¾
¾® R - R + ZnI 2
(VI) From Grignard's reagent : 2RLi + CuI ¾¾
® LiR 2 Cu + LiI
Compound H + RMgX ¾
¾® Compound MgX + RH Lithium dialkyl copper

H must be attached to O, N, S or sp hybridised C-atom. (i.e.


active H-atom). R ' Br + LiR 2 Cu ¾
¾® R '- R + RCu + LiBr
(VII) Kolbe's electrolytic method : The reaction is particularly useful for preparing
unsymmetrical alkanes.
– +
RCOOK RCOO + K Properties : C1–C4 gases; C5–C17 colourless liquids, higher
Anode Cathode
solids. Insoluble in water soluble in organic solvents

–e Branched chain alkanes boil at lower temperature than
Fractionation
RCOO R + CO2; R + R R–R isomeric straight chain alkanes. The latter have higher
van der Waals forces of attraction.
(VIII) By Clemmensen's reduction : From carbonyl compounds
Melting points of hydrocarbons containing even number of
Zn - Hg C-atoms are relatively more than those containing odd
C = O + 4H ¾¾ ¾
¾® CH 2 + H 2 O
Conc. HCl number of carbon atoms.
Hydrocarbons 455
Halogenation (Cl2)
CH 3Cl . CH 2 Cl 2 . CHCl 3 . CCl 4 (from methane)
UV light / or D
Direct Sun Light
C + 4HCl (explosive reaction) (from methane)
2Cl2

• Ease of replacement of hydrogen atom 3° > 2° > 1°. The relative rates being 5:3.8:1
• Order of reactivity of halogens F2 > Cl2 > Br 2 > I 2
• Iodination is reversible CH4 + I 2 CH3I + HI
5HI + HIO3 3I2 + 3H2O
It requires the use of oxidising agent such as HNO3, HIO3 etc.
• Fluro compounds are obtained from bromo or chloro compounds
2RBr + HgF2 2RF + HgBr2
• Chlorination and bromination proceed by free radical mechanism.

Sulphuryl Chloride
R – Cl + SO2 + HCl (Chlorination)
SO2Cl2+ light or peroxide
NO2
Nitration HNO3 |
CH 3CH 2 CH 2 NO 2 + CH 3 C H - CH 3 + CH 3 CH 2 NO 2 + CH 3 NO 2 (from propane)
450°C vapour phase
2CH 3 COOH + CO 2 + H 2 O (from neopentane)
Sulphonation
RSO3H + H 2O
H2SO4 400°C Alkyl sulphonic acid

,
Oxidation O2 Cu tube 100 atm.
CH 3OH ü
200°C ï
MoO / O2 ï
ï
RH HCHO + H 2 O ý From methane
Alkane (CH3COO)2 Mn/O2 ï
ï
HCOOH + H 2 Oþï
Alk. KMnO4 [O]
R 3COH (From Isobutane)
Alkanes containing 2° and 1° H atoms are generally not
oxidised by K2Cr2O7 and KMnO4
Combustion
CO 2 + H 2 O
O2 Excess D
CH 3
Isomerisation |
Anhy. AlCl3 + Conc. HCl CH 3 - C H - CH 3 (from n - butane)

Aromatisation Cr2O3
+ 4H2 (from n-Hexane)
450–500°C

Dehydrogenation
CH 3 - CH = CH 2 + H 2 (from propane)
Cr2O3 / Al2O3 D
CH 3 CH3
Alkylation | |
CH 3 - C - CH 2 - C H - CH 3
Isobutane + Isobutylene |
CH 3
2,2,4 - trimethyl pentane (common name isoctane)

Cracking D
CH 3 - CH = CH 2 + CH 4 + CH 2 = CH 2 + CH 3 - CH3
n-butane gives

CH3 CH3
(CH 3 ) 3 CH + CH3 COCH 3 | |
CH 3 - C - C - CH 3
Conc. H2SO4 | |
CH3 OH
2,3,3- trimethyl butanol - 2
456 Chemistry
CONFORMATIONAL ANALYSIS :

H H
Ethane – H
H H H
C
C
H H H
H H H
Sawhorse projection Newman projection
Different arrangements of atoms in a molecule convertible into one another by rotation about single bond are called conformers
or conformations. Their study is known as conformational analysis. If the energy barrier to the rotation is nil or small, the rotation
is said to be free or almost free. Ethane can exist in an infinite number of conformations. They are

HH H H H
H H
H
H H
H HH H
H H H H
Eclipsed q = 60° Staggered q < 60° > 0 Skew
q = It is dihedral angle between C–H bonds on the front of a Newman projection and those on the back.
Order of stability : Staggered > Skew > Eclipsed
Energy barrier between eclipsed and staggered is 2.8 kcal/mol.
Conformations of Propane C3H8 :

H3C H CH3 H3C H


H H
H
H H
H HH H
H H H H
Eclipsed Staggered q = 60° Skew q < 60° > 0
Order of stability : Staggered > Skew > Eclipsed
Energy barrier between eclipsed and staggered is 3.3 kcal/mol
Conformations of n-Butane C4H10 :

H3C CH
3
H3C CH
3
CH3 CH3 H CH3
H CH3 H H
H H
H H H H
HH CH3
H H H H H
H
CH3
H H
q = 0° q < 60° > 0 q = 60° Gauche q = 180° Anti or Trans q= 120 ° Eclipsed
Fully Eclipsed Skew
Staggered

Order of stability : Anti > Gauche > Skew > Eclipsed > Fully Eclipsed
Energy barrier between fully eclipsed and fully staggered (anti) is 5.3 kcal / mol or 22 kJ/mol.
Conformations of Cyclohexane : It exists in two nonplanar, strainless forms, the boat and the chair form.

a Axial a a
e e
e e e
e
a a equatorial
a a
e
e
e e
e e
a a
Move up Move Down
Move Down a a

Chair form Boat form Chair form


Hydrocarbons 457
Equatorial hydrogens lie in the plane of the ring carbons. Nomenclature :
Axial hydrogens lie (up or down) the plane of the ring. (I) Common System : The suffix -ane of alkane is replaced
There are six equatorial and six axial hydrogens. In the flipping by -ylene, and named as Alkylenes eg. ethylene,
and reflipping between conformations, axial becomes propylene etc.
equatorial and vice versa. Chair form has the lowest energy. (II) IUPAC System : The suffix -ane of alkane is replaced
Cyclohexane can assume other shapes also. by -ene and hence named as Alkenes.
(III) Derived Names : They are named as substituted
derivatives of ethylene. For example
CH 3 - CH = CHCH 3 Common
IUPAC Derived
Name
Name name
Chair form Half Chair a -Butylene
But-2-ene s-Dimethyl
Most Stable or 2-Butylene ethylene
Isomerism : Alkenes show four types of isomerism
(I) Chain Isomerism :
5 3 1 2 3 1
4
6 4 2 5 3 1 4 2
Twist Boat Boat form Hex-1-ene 4-methyl pent-1-ene 3,3-dimethyl but-1-ene
(Least Stable)
Baeyer's Strain Theory : When ring compounds are formed, (II) Position Isomerism :
the bonds deviate from normal positions which produces a But-1-ene But-2-ene
condition of strain in the molecule. The strain is directly (III) Ring Chain Isomerism :
proportional to angle of deviation and can be calculated as
follows. The more the angle of deviation, the less is the
stability. But-1-ene Cyclo butane
Angle of deviation of cyclic compound (IV)Geometrical Isomerism :

1 H3C CH3 H3C H


= (109 °28 '- angle of cyclic compound)
2 H H H CH3
1 cis But-2-ene trans But-2-ene
For Cyclo propane = (109°28'-60°) = 24°44'
2 GENERAL METHODS OF PREPARATIONS OF
1 ALKENES
Cyclo butane = (109°28'-90°) = 9°44' (I) By dehydration of alcohols :
2
Dehydratin g agent
1 ROH ¾¾ ¾ ¾ ¾ ¾¾® Alkene + H 2 O
Cyclopentane = (109°28'-108°) = 0.44'
2 Dehydrating agents Conc. H2SO4; P2O5; H3PO4; Al2O3.
Cyclohexane Anhy. ZnCl2. Anhy. Oxalic acid.
1 Ease of dehydration of alcohols 3° > 2° > 1°.
(109°28'-120°) = -5°16' (deviation outside )
= (II) By Dehydrohalogenation of Alkyl halides.
2
Cycloheptane | |
- C - C - + KOH (alcoholic) ¾
¾® > C = C < + KX + H 2 O
1 | |
= (109°28'-128°34' ) = -9°33' (deviation outside ) X H
2
Limitations : Baeyer's strain theory fails to explain the • Dehydrohalogenation and dehydration follows
Saytzeff's rule (Hydrogen is removed from C-atom
stability of large ring alicyclic compounds.
containing lesser number of H-atoms or more
Sachse Mohr theory of Strainless rings :
substituted alkene is formed)
A ring with six or more carbon atoms can assume "puckered" • More substituted alkenes are more stable.
structure and there is a little distortion of normal tetrahedral
angle. Thus there is little or negligible angle strain in the R 2 C = CR 2 > R 2 C = CHR > R 2 C = CH 2 >
molecule. RCH = CHR > RCH = CH 2 > CH 2 = CH 2
UNSATURATED HYDROCARBONS, OLEFINS OR • Ease of dehydrohalogenation 3° > 2° > 1°.
ALKENES : • Ease of dehydrohalogenation : Iodides > Bromides >
They are open chain compounds of carbon and hydrogen Chlorides
having double bonds also known as Olefins (oil forming) (III) By dehalogenation of vicinal halides :
Olefiant gas is Dutch name of ethylene which formed oily | | | |
CH OH
ethylene chloride with chlorine. Their general formula is - C - C - + Zn ¾¾ ¾
3 ¾®- C = C - + ZnX
2
| | D
CnH2n. X X
458 Chemistry
(IV)By dehalogenation of gem. halides : Alkenes with odd or even number of carbon atoms and
having any position of double bond can be prepared.
RCHX 2 + 2 Zn + X 2 CHR ¾
¾® R - CH = CHR + 2 ZnX 2
(VI)By partial reduction of alkynes :
(V) By electrolysis of Sodium or potassium salt of succinic Pd - CaCO Lindlar 's Catalyst
acid or its derivatives R - C º CR + 2H ¾¾ ¾ ¾ ¾
3 ¾ ¾ ¾ ¾¾
¾® R - CH = CHR
Quinoline (always cis Isomer )

C H 2 COOK Electrolysis C H 2 COO - + 2K +


Na / liq. NH
| | R - C º C - R ¾¾¾¾¾
¾3® RCH = CHR
CH 2COOK CH 2COO- Birch reduction (always trans Isomer)
Pot. Succinate Anode Cathode
– (VII) Decomposition of quaternary ammonium hydroxide:
–2e
D
C H COO• (C 2 H 5 ) 4 NOH ¾¾® CH 2 = CH 2 + (C 2 H 5 ) 3 N + H 2 O
2
|
CH 2COO• (VIII) By cracking of alkanes :
600°C
CH 3 - CH 2 - CH 2 CH 3 ¾¾ ¾
¾® CH 3 - CH = CH 2 + CH 4
• CH CH2 Properties : C2–C4 gases; C4 – C15 liquids C16 onwards
Fractionation |
2
¾
¾® ||
• CH 2 Solids. Less volatile than alkanes and possess anaesthetic
–2 CO2 CH 2
properties.
Chemical Properties : Addition reactions given by alkenes are known as electrophilic addition reactions.
H2/Ni 200 – 300°C
CH3 - CH3 (Sabatier - Senderen reaction) (Syn addition)
Br2/CCl4 or CHCl 3
CH 2Br.CH 2 .Br (Colour of bromine discharged) ( Anti addition )

HX
CH 3 .CH 2 X (Order of reactivity of HX HI > HBr > HCl)
(Anti and Syn addition)
HOX hypohalous acid
CH 2 OH.CH 2 Cl ethylene chlorohydrin

Cold conc. H2SO4 HOH


CH 3 .CH 2 H SO 4 ¾ ¾¾ ® CH 3 CH 2 OH
Ethylhydro gen sulphate Ethyl alcohol

CH2Cl . CH2NO
Ethylene nitroso chloride
O3 O
CH2 CH2 H2O
Ozone 2HCHO + H2O2 2HCOOH
O O H2/Zn
2HCHO + H2O

R–CH=CH2 — Addition of Zn powder removes H2O2 and aldehyde is obtained


Zn + H2O2 ZnO + H2O
Addition of HNO3
CH2(OH)CH2NO2
b-Nitro ethyl alcohol
Addition of acyl halide
CH2(Cl)CH2COCH3
e.g. CH3COCl
Methyl b-chloro ethyl Ketone

CH2
O
CH2
Ethylene Oxide
1% alk. KMnO4 Cold CH2OH
(Test for unsaturation)
Baeyer's reagent CH2OH
Hydroxylation
Ethylene glycol (Syn. addition)
Hydrocarbons 459
[O]acid KMnO4
or acid K2Cr2O7 CH2 OH [O]
2HCOOH
CH2OH

The double bond is completely cleaved; the molecule is split into two
molecules. If carbon has two hydrogens it will become CO 2. If carbon has one
hydrogen it will become carboxylic acid. If carbon has no hydrogens it will
become a ketone. For example,
CH3 + CH3
KMnO4/H
C = CH2 C = O + CO2
CH3 CH3

CH3 H +
KMnO4/H CH3 OH
C=C C=O+O=C
Periodic acid HIO4 CH3 CH3 CH3 CH3
or lead tetra acetate
(H2O + O) CH3 CH3
COH C = O + HCHO
CH3 CH3
CH2 OH
H2O2 / CH3COOH CH2OH

CH2OH
C6H5.CO.OOH H2 O
C–C C–C
Perbenzoic acid 160
O OH OH
Oxide Glycol

Addition of sulphur monochloride b a a' b'


Cl. CH 2 . CH 2 - S - CH 2 . CH 2 Cl + S
S2Cl2 Mustard gas

Hydroboration B2H6 /ether Hydrolysis


or (CH3 CH2 )3B – 3CH3CH2OH
H2O2,OH
BH–3 THF Triethyl borate (Addition of H2O anti to
Markownikoff’s rule)

Oxo process carbonylation


CH.3CH.2CHO
CO + H2 [Co(CO)]4 Propionaldehyde

Oxy-mercuration - demercuration
R-CH-CH3 (from R-CH= CH)2
OH
or (Addition of H2O according to
H2O + acid or Catalyst BF3,Mn phosphate Markownikoffs rule)

(i) OsO4 Osmium tetraoxide R– CH – OH


R– CH – OH
cis glycol

Cl(500–
2 600ºC)
Cl CH2– CH = CH2 (from propene)
Allylic substitution Allyl chloride

BrCH2– CH = CH2 (from propene)


Allylic substitution Allylbromide
(Wohl - ziegler reaction)

CH2
methyl cyclopropane
460 Chemistry

Isomerisation Al2(SO4)3
at 200 – 300ºC
CH 3 - CH 2 - CH = CH 2 ¾
¾® CH 3 - CH = CH - CH 3
or D above 500º – 700º C
But-1-ene gives But-2-ene

Polymerisation
–(CH2 – CH2)n polythene
O2 D

Combustion
CO2 + H2O + heat

ALKYNES OR ACETYLENES : They are characterised by the (III) By dehalogenation of tetrahalides


presence of triple bond having general formula CnH2n–2. X X
NOMENCLATURE : According to trivial system they are regarded D
–C–C– + 2 Zn ¾¾® – C º C– + 2Zn X2
as derivatives of acetylene. In the IUPAC system their names are
derived by replacing suffix -ane by -yne. X X
Formula Common Name Derived name IUPAC name (IV) From lower alkynes
CH º CH Acelylene Acetylene Ethyne Na 2RX
CH3C º CH Allylene Methyl acetylene Propyne
HC ºCH ¾¾® NaC º CNa ¾¾ ¾® R – C º C – R +
liq.NH3
CH3C º C– CH3 Cretonylene Dimethyl acetylene But - 2- yne
2NaX
ISOMERISM : They exhibit four types of Isomerism (V) Kolbe's electrolytic method
(I) Chain Isomerism
– +
CH. COOK Electrolysis CHCOO + 2K

CH. COOK CHCOO – at anode
Hex–1–yne 4–methyl pent –1– yne Pot. fumarate –2e
(II) Position Isomerism CHCOO•
CHCOO•

Hex – 1–yne Hex – 3 – Yne Fractionation •CH CH


+2CO2®
(III) Functional Isomerism •CH CH

(VI) From haloform


But – 1 –yne But –1,3 – diene
CHI3 + 6Ag + I3CH ¾¾®D CH º CH + 6AgI
(IV) Ring chain Isomerism
(VII) Acetylene from calcium carbide (wohler's reaction)

CaC2 +2H2O ¾ ¾® Ca(OH)2 + CH º CH


But – 2 –yne cyclobutene (VIII) Berthlot's reaction
GENERAL METHODS OF PREPARATION : 1200°C
2C + H 2 ¾¾ ¾
¾® HC º CH
(I) By dehydrohalogenation of vicinal halides Arc.
(IX) From methane :
NaNH2
H–C–C–H – C º C– + 2HX 2CH 4 ¾¾ ¾
1500°C
¾® HC º CH + 3H 2
or Alc. KOH Electric arc.
X X
PROPERTIES : C2–C4 gases C5–C12 liquids. C13 onwards solids.
(II) By dehydrohalogention of gem halides
Acetylene has garlic odour due to phosphene and hydrogen
AlcKOH AlcKOH sulphide impurity.
H–C–C–X H– C = C – X – C º C–
– HX – HX Chemical Properties: Alkynes are less reactive then alkenes for
H X
electrophilic addition reactions.
Hydrocarbons 461
(A) Electrophilic Addition Reactions :

H2/Ni 300°C H / Ni 200°C


CH 2 = CH 2 ¾¾2 ¾ ¾ ¾¾® CH3 - CH 3

Cl2 / SbCl5 CHCl CHCl 2 CCl 2


Cl 2 BaCl or
|| ¾¾¾® | ¾¾ ¾2 ¾¾® || + HCl
Kiesulguhr CHCl CHCl 2 Lime CH.Cl
Acetylene Acetylene Trichloro ethylene
dichloride tetrachloride (Westrosol)
(Westron)

Reactivity of halogens Cl > Br > I

Addition of HX CH 2 HX
CH 3
|| ¾¾¾® |
CHX CHX 2
Ethylidene halide

Addition of 2HOCl CHCl 2 - H 2O


| ¾¾ ¾ ® Cl 2 CH.CHO
CH(OH) 2 Dichloro acetaldehyde
(Unstable)

CH 3 CH 3
H O
R–C CH | ¾¾2¾® |
CH(HSO 4 ) 2 CHO
Ethylidene hydrogen Sulphate ( Acetaldehyde)

(B) Nucleophilic Addition Reactions :

10% HgSO4 Tautomerisation


[CH2 = CHOH] CH3CHO
400% dil. H2SO4
Vinylalcohol (Kucherov's Reaction)

HCN / Ba (CN)2
CH2= CHCN
Acrylonitrile

CH3COOH / Hg2+
CH 2 = CHOOC.CH 3 ¾
¾® CH 3 - CH(OOCCH 3 ) 2
Vinyl acetate Ethylidene acetate

CH3OH / CH3OK CH2= CHOCH3


Methyl Vinyl ether
200°C high P.

AsCl3 anhy. AlCl3


ClCH = CH . AsCl 2
Lewisite
462 Chemistry

Addition of CO + HO
2
CH2= CH.COOH
Ni (CO)4 Acrylic acid

Cl
CuCl (aq.) + NH4Cl HCl
|
CH 2 = CH - C º CH ¾¾
¾® CH 2 = CH - C = CH 2
Vinyl acetylene Chloroprene
( monomer of Neoprene rubber )

Polymerisation
hot copper tube
Benzene
HCHO under pressure Partial
HC º C.CH 2 OH ¾¾¾
¾® CH 2 = CH.CH 2 OH
Propargyl alcohol reduction Allyl alcohol

Oxidation Acid KMnO4


HCOOH
Formic acid

Alk. KMnO4[O] COOH


|
COOH
Oxalic acid

K2Cr2O7 + H2SO4[O]
CH3COOH
Acetic acid
SeO2 CHO
| glyoxal
CHO
N2 electric spark
2HCN

Na or NaNH2 (alc.)
HC º CNa ¾
¾® NaC º CNa
Sod. acetylide di Sod. acetylide

Cu2Cl 2/ NHO4H
Cu.C º C.Cu
Copper acetylide (Red ppt.)

AgNO3 / NH4OH Ag.C º C – Ag


Silver acetylide (white ppt.)
O
O3 / Ozonolysis H2O
CH CH HC – C – H + H2O2 2HCOOH

O O O O Formic acid
Ozonide Glyoxal

Combustion
CO2 + H2O

Iron pyrites 300°C

S
Thiophene

NH3 high temp.

H
Pyrrole

All the products obtained in the above reactions are from acetylene.
Hydrocarbons 463
AROMATIC HYDROCARBONS : distill
Benzene C6H6 ¾¾¾® Benzene + Some toluene + Thiophene
110°C
A Benzene
Preparation - A Small scale preparation : Again distill
¾¾ ¾ ¾ ¾® Almost pure benzene
(I) From acetylene : at 80-82°C

Impurity of thiophene is removed by heating with


Re d hot tube hydrogen under pressure at 400°C in presence of catalyst.
3HC º CH ¾¾ ¾ ¾ ¾
¾®
500°C Catalyst
C 4 H 4S + 4H 2 ¾¾ ¾¾® C 4 H10 + H 2S
400°C n - butane
(II) From benzoic acid :
20. Properties - Colourless Liquid : bpt 80.1°C. Insoluble in H2O.
COOH
(A) Electrophilic Substituton Reactions :
NO 2
NaOH + CaO + Na2CO3
Soda lime
+ + H2O

(III) From Phenol : Nitrobenzene

OH SO3H
Sulphonation Conc. HS2O4
Zn dust + ZnO +
distillation Electrophile SO3 or HSO3
Benzene
Sulphonic acid
(IV) From Chlorobenzene : Cl
Cl Halogenation Cl / Fe2Cl3
+
Mg/dil. HCl Electrophile Cl Chlorobenzene
+ 2H + HCl

CH 3
Friedel Crafts's Alkylation
(V) From benzene sulphonic acid : CH3Cl/Anhy Al3Cl
+
SO3H Electrophile CH
(Carbonium ion) Toluene

200°C + H2SO4
+ 2H2O COCH 3
Friedel Crafts's Acylation
Steam CH3COCl/Anhy AlCl3
(VI) From benzene diazonium chloride : +
Electrophile CH3CO
acylium ion Acetophenone
N2Cl
(B) Addition reaction :
H3PO2
+ N2 + HCl 3H2/Ni
+
Cu
Cyclohexane
(B) Large Scale preparation :
(I) From Petroleum : n-hexane fraction of petroleum
Cl
3Cl2, UV light Cl Cl
Cr2O3 / Al2O3
CH3 (CH 2 )CH3 ¾¾¾¾¾¾
® + 4H2
n - Hexane 550°C Cl Cl
Cl
(II) From light-oil fraction of Coal tar BHC - Benzene Hexa Chloride,
Gammexane,Lindane used as insecticide.
Conc. H SO
2 4 ® ¾¾¾¾ NaOH
Light oil fraction ¾¾¾¾¾¾ ® ¾¾¾® + +
- pyridene - phenol 110°
464 Chemistry

HC = O (VI) Claus and Dewar's structures :


(I) O3 (II) H 2O /Zn
3 |
Ozonolysis HC = O
glyoxal

O (VII) Baeyer and Armstrong's centric structure :


Oxidation V2O5
O + 4H2O + 4CO2
450°C
O
Maleic anhydride

(VIII) Robinson's sixtet structure :


700–800°C
Red hot iron tube
Biphenyl

STRUCTURE OF BENZENE : (IX) Thiele's structure of partial valency :


(I) Open chain structures proposed.
(a) CH 2 = C = CH - CH = C = CH 2
(X) Benzene and resonance : Benzene is a resonance hybrid of
(b) HC º C - CH 2 - CH 2 - C º CH the following structures.
(c) H 3C - C º C - C º C - CH 3

(II) Ring structure by Kekule :


Kekule's
Objections against Kekule structure :
(a) Two ortho di substituted derivatives
X X
X X Resonance
Dewar's hybrid
and All C–C bonds in the benzene are of the same length 1.39Å.
(XI) Benzene and aromaticity : Benzene is an aromatic compond
since it obeys Huckel rule of (4n + 2)p electrons. The value
(b) About Stability
of n must be an integer.
(III) Explanation against above objections : Presence of two 4n + 2 = 6p electrons of benzene
structures undergoing quick interconverson.
\n =1
(XII) Molecular orbital structure of benzene : Each C-atom in
benzene is sp2 hybridised and forms 3s bonds. The pz atomic
orbitals left on each C-atom form a delocalised p-molecular
orbital which stabilises the structure and renders all C-C bonds
(IV) Levine and Cole : Levine and Cole confirmed the presence of equal in length.
above structures by ozonolysis of o-Xylene and obtaining
the products.
(a) Dimethyl glyoxal
(b) Methyl glyoxal and or
(c) Glyoxal
(V) Ladenburg's prism structure :
6 2
1
X-ray crystallographic methods showed
5 3

4 or
benzene a planar compound whereas above structure
is non planar.
delocalised p
. M.O.
Hydrocarbons 465
PETROLEUM
CaC 2 + 2H 2 O ¾
¾® C 2 H 2 + Ca (OH ) 2
Petroleum (petra = rock, oleum = oil). A thick dark coloured
complex liquid, mixture of organic compounds obtained from below Polymerisa tion
3C 2 H 2 ¾¾ ¾ ¾ ¾¾® C 6 H 6 ;
the surface of the earth is petroleum. The chief components of
petroleum are hydrocarbons, aliphatic, alicyclic (naphthalenes) C2H 2 + H 2 ¾
Polymerisa tion
¾® C 2 H 4 ¾¾ ¾ ¾ ¾¾® C 6 H12
or aromatic in varying proportions and 1 to 6 percent of Sulphur,
Nitrogen and Oxygen compounds. The theory was supported by Moissan, Sabatier and
Senderens. It fails to explain the presence of optically active
Natural Gas : Found along with petroleum and roughly contains
compounds, Compounds of N and S, chlorophyll and haemin
60 to 80 percent methane 5 to 9 percent ethane, 3 to 18 percent
derivatives.
propane, 2 to 14 percent higher hydrocarbons. It is used as fuel.
(II) Engler's Theory : Petroleum is a product of slow
Partial combustion of natural gas yields Carbon blocks (reinforcing
decomposition of dead marine animals under high
agents for rubber).
temperature and pressure. It explains the presence of brine,
Theories of origin :
fossils, compounds of N and S.
(I) Mendeleef 's Carbide Theory :
It explains the presence of all the above mentioned
¾® Al 4 C 3 ;
4Al + 3C ¾ compounds including chlorophyll. Hence petroleum is of
animal as well as vegetable origin.
Ca + 2C ¾
¾® CaC 2 (III)Modern Theory : Petroleum is produced by partial
decomposition of marine animals and sea weeds etc.
¾® 3CH 4 + 4Al(OH ) 3 ;
Al 4 C 3 + 12 H 2 O ¾ Mining and Refining : Recovery from oil wells and separation
of individual components.

Products of Petroleum Fractionation


Fractions Composition Boiling Range Uses
1. Uncondensed gases C1–C4 0-30°C
(a) Cymogene Used in the manufacture of Ice
(b) Rhigolene Used as local anaesthetic
2. Crude Naphtha Refractionation C5–C10 30-150°C
(a) Petroleum ether C5–C6 30-70°C Solvent for oils and fats and rubber, dry cleaning
(b) Gasoline or Petrol C6–C8 70-90°C Fuel and dry cleaning
(c) Ligroin or Light Petroleum C6–C8 90-120°C In drycleaning as solvent
(d) Benzene or Benzolene C8–C10 120-150°C Solvent (oil and paints industry), dry cleaning
3. Kerosene Oil C11–C18 150-300°C As fuel, for illumination, making oil gas
4. Diesel or Heavy Oil C18–C42 Above 300°C Fuel in heavy automobiles
5. Residue Refractionation under
reduced pressure C30–C45 Above 400°C
(a) Paraffin Wax For Candles and boot polishes
(d) Lubricating Oils For lubrications
(c) Vaseline In ointments and toilet goods
(d) Pitch In paints and varnishes, as fuel
(e) Petroleum Coke As fuel.

Flash Point : The minimum temperature at which an oil gives Anti Knock Compounds : 59% Tetra ethyl lead, 13% Ethylene
off sufficient vapours to form an explosive mixture with air is bromide, 24% Ethylene chloride, 4% Kerosene and dye is
called Flash Point. anti knock mixture.
Knocking : A sharp metallic sound emitted by internal In presence of aromatic compounds tetramethyl lead is more
combustion engine owing to immature ignition of the air effective.
gasoline mixture. Knocking is maximum in presence of straight Octane number : The percentage of iso octane (2, 2, 4-
chain hydrocarbons and minimum in presence of aromatic trimethyl pentane) in a mixture of iso octane and n-heptane
and branched chain hydrocarbons. having the same knocking properties as the fuel under
consideration.
466 Chemistry
The higher the octane number, the better is the fuel. (V) Aromatisation :
Octane number of compunds : Aromatic Compounds > Cyclo
alkanes > Olefins > Branched chain alkanes > Straight
Chain alkanes. CH 3 - (CH 2 ) 4 - CH 3 ¾
¾® + 4H2
Cetane number : It is the percentage of cetane
(n-hexadecane) in a cetane and a-methylnaphthalene mixture
that has the same ignition qualities as the fuel. (VI) Chain Fission :
Cracking : The conversion of less valuable higher fraction
C16 H 34 ¾
¾® C 8 H18 + C 8 H16 or C 6 H14 + C10 H 20
to the more valuable lower fraction by the application of heat
is known as cracking. Synthetic Petrol :
(I) Liquid Phase Cracking : (I) Bergins Process :
475° -530°C
Heavy Oil (C18 - C 43 ) ¾¾ ¾ ¾ ¾ ¾ ¾ ¾
¾®(C 4 - C10 )
Catalyst 1000-1200 lbs psi
Sn, Pb or Mo Compounds
Catalyst : Silica, titanium dioxide, zinc oxide, ferric oxide, Powdered Coal + Heavy Oil ¾¾ ¾ ¾ ¾ ¾ ¾ ¾ ¾
¾®
H 2 400 -500°C, 200 -700 Atm
alumina etc. High pressure keeps the oil in liquid state.
Octane number of product : 65-70.
(II) Vapour phase Cracking :
600-800°C Petrol
Kerosene oil or gas oil ¾¾¾ ¾¾® Lower fractions
50-150 lbs psi
F.D. Middle Oil
(III) Cracking in presence of hydrogen : Product ¾ ¾¾ ®
Catalyst
R - R + H 2 ¾¾ ¾¾® 2RH ü Heavy Oil
ï
ROH + H 2 ¾ ¾® RH + H 2 O ïï
ý Product free from O, S & N Heavy oil is reused.
¾® 2RH + H 2S ï
R 2S + 2H 2 ¾
ï (II) Fischer-Tropsch Process :
¾® 2RH + NH 3 þï
R 2 NH + 2H 2 ¾
S team + Coke (Red hot) ¾¾® CO + H 2
water gas
Reactions taking place during cracking :

(I) Carbonisation : CH 4 ¾1000 °C


¾¾ ¾® C + 2H 2 Catalyst
n(CO + H 2 ) + nH 2 C n H 2n + nH 2 O (Olefin )
(II) Dehydrogenation : 200–250 °C
450°C 910afm.
CH 3 - CH 3 ¾¾ ¾
¾® CH 2 = CH 2 + H 2 nCO + (2n + 1)H 2 ® C n H 2n + 2 + nH 2 O (Paraffin )
(III) Polymerisation : Catalyst Cobalt (100 parts), thoria (5 parts), magnesia
(8 parts), Kieselguhr (200 parts).
2(CH 3 ) 2 C = CH 2 ¾
¾®(CH 3 ) 3 C - CH 2 - C = CH 2 l Reforming : It is increasing of anti knock properties by
|
CH3 special type of cracking which includes Alkylation,
Isomerisation, Aromatisation, Cyclisation,
H
¾¾®
¾2 (CH 3 ) 3 C - CH 2 CH(CH 3 ) 2 Dehydrogenation, Fractionation etc.
Iso octane l No lead petrol : It does not contain lead and obtained by
reforming.
(IV) Alkylation :
l Petro chemicals : Chemicals derived from petroleum
(CH 3 ) 2 CH - CH 3 + CH 2 = CH 2 ¾
¾®(CH 3 ) 3 C - CH 2 - CH 3 sources.
Hydrocarbons 467

Very Short/Short Answer Questions Long Answer Questions


1. What does LPG stand for ? 14. (i) Why is the Wurtz synthesis not a good method for
2. Name the products formed when an ethereal solution preparing propane?
containing ethyl iodide and methyl iodide is heated with (ii) What alkanes would be expected from the reaction of
sodium metal. sodium with 50 : 50 mixture of 1-chloropropane and
2-chloropropane ?
3. Out of ethylene and acetylene which is more acidic and
why? 15. (i) Arrange the following compounds according to the
increasing order of boiling point: Hexane, heptane, 3-
4. Can eclipsed and staggered conformations of ethane be
methyl pentane, 2, 2-dimethyl butane.
isolated? Give reason.
(ii) Account for the following :
5. Why is dipole moment of trans-1, 2-dichloroethene zero?
(a) The boiling points of hydrocarbon decreases with
6. How will you distinguish between increase in branching.
CH3CH2C º CH and CH3 —C º C —CH3 (b) Hydrocarbons with odd number of carbon atoms
7. What is halogen carrier? Give one example. have a melting point lower than expected.
8. Arrange the following in increasing order of their release of (c) Boiling point of n-pentane is greater than that of
energy on combustion. neo-pentane but melting point of neo-pentane is
greater than that of n-petane.
16. The ring systems having following characteristics are
(i) (ii)
aromatic.
(i) Planar ring containing conjugated p bonds.
(ii) Complete delocalisation of the p-electrons in ring
(iii) (iv) system i.e, each atom in the ring has unhybridised p-
orbital, and
9. Why is cyclopropane is more reactive as compared to (iii) Presence of (4n + 2) p-electrons in the ring where n is
cyclohexane? an integer (n = 0, 1, 2 ...........) [Huckel rule]
10. What happens when Using this information classify the following compounds
as aromatic /non-aromatic.
(i) A mixture of steam and methane is passed over heated
nickel (supported over catalyst at 1273 K)
(ii) Methane is heated to high temperature (1500 K).
(iii) Isobutane is treated with alkaline KMnO4. N – +
11. What is the role of red phosphorus in the reduction of alkyl (A) (B) (C)
halide using hydroiodic acid?
12. Write down the structural formulae of the product obtained
in the following reaction:

cold dilute
+
cold dilute (D)
(i) (ii) KMnO4
(E)
KMnO4
17. Convert the following :
hot (a) Ethyne to ethanal
(iii) K2Cr2O7/H2SO4 (b) Tert. butyl chloride to 2,2,3,3 tetra methyl butane.
(c) Ethyne to benzene.
13. You are given three jars containing ethane, ethylene and
(d) Benzoic acid to benzene
acetylene, respectively. How will you identify each of them
by chemical test? (e) But 1–ene to butane 1,2– diol.
468 Chemistry
Multiple Choice Questions 23. Arrange the following alkyl halides in decreasing order of
the rate of b–elimination reaction with alcoholic KOH.
18. Arrange the following in decreasing order of their boiling
points. H

— —
(A) n–butane (B) 2-methylbutane
(C) n-pentane (D) 2, 2–dimethylpropane (A) CH3—C—CH2Br
(a) A > B > C > D (b) B > C > D > A CH3
(c) D > C > B > A (d) C > B > D > A
19. Arrange the halogens F2, Cl2, Br2, I2, in order of their (B) CH3—CH2—Br
increasing reactivity with alkanes. (C) CH3—CH2—CH2—Br
(a) I2 < Br2 < Cl2 < F2 (b) Br2 < Cl2 < F2 < I2 (a) A > B > C (b) C > B > A
(c) F2 < Cl2 < Br2 < I2 (d) F2 < I2 < Cl2 < F2 (c) B > C > A (d) A > C > B
20. An alkene having molecular formula C7H14 was subjected
24. The addition of HBr to 1-butene gives a mixture of products
to ozonolysis in the presence of zinc dust. An equimolar
A, B and C
amount of the following two compounds was obtained
CH3 CH3 Br C2H5
C = O and C=O
CH3 CH3CH2 C C
(A) H5C2 CH3 (B) H CH3
The IUPAC name of the alkene is H Br
(a) 3, 4-dimethyl-3-pentene (b) 3, 4-dimethyl-2-pentene
(C) CH3—CH2—CH2—CH2—Br
(c) 2, 3-dimethyl-3-pentene (d) 2, 3-dimethyl-2-pentene
21. Which of the following reactions of methane is incomplete The mixture consists of
combustion ? (a) A and B as major and C as minor products
Cu / 523K /100atm 2CH OH
(a) 2CH4 + O2 ¾¾¾¾¾¾¾® (b) B as major, A and C as minor products
3
Mo2O3 HCHO + H O (c) B as minor, A and C as major products
(b) CH4 + O2 ¾¾¾¾ ® 2
(d) A and B as minor and C as major products
(c) CH4 + O2 ® C(s) + 2H2O(l)
25. Among the following compounds, the decreasing order of
(d) CH4 + 2O2 ® CO2 (g) + 2H2O(l)
22. The correct IUPAC name of following alkane is reactivity towards electrophilic substitution is

H2C—CH2—CH—CH2—CH2—CH—CH2—CH3

— —

CH CH2
CH3 CH3 CH3
CH3 OCH3 CF3
(a) 3,6 – Diethyl – 2 – methyloctane I II III IV
(b) 5 – Isopropyl – 3 – ethyloctane
(c) 3 – Ethyl – 5 – isopropyloctane (a) III > I > II > IV (b) IV > I > II > III
(d) 3 – Isopropyl – 6 – ethyloctane (c) I > II > III > IV (d) II > I > III > IV

1. Successive alkanes differ by 4. When petroleum is heated gradually, the first batch of
(a) CH2 (b) CH vapours evolved will be rich in
(c) CH3 (d) C2H4 (a) kerosene (b) petroleum ether
2. An alkyne has the general formula (c) diesel (d) lubricating oil
(a) CnH2n (b) CnH2n+2 5. The order of appearance of the following with rising
temperature during the refining of crude oil is
(c) CnH2n–2 (d) CnH2n+1
(a) kerosene oil, gasoline, diesel
3. Which of the following is not a mixture of hydrocarbons?
(b) diesel, gasoline, kerosene oil
(a) Candle wax (b) Kerosene (c) gasoline, diesel, kerosene oil
(c) Vegetable oil (d) Paraffin oil (d) gasoline, kerosene oil, diesel
Hydrocarbons 469
6. LPG contains 20. Formation of alkanes by action of Zn on alkyl halides is
(a) methane (b) ethane called
(c) butane (d) None of these (a) Frankland's reaction (b) Cannizaro's reaction
7. Petrol for aviation purposes must contain (c) Wurtz's reaction (d) Kolbe’s reaction
(a) straight chain hydrocarbons 21. For preparing an alkane, a concentrated solution of sodium or
(b) aromatic hydrocarbons potassium salt of a saturated carboxylic acid is subjected to
(c) olefinic hydrocarbons (a) hydrolysis (b) oxidation
(d) highly branched chain paraffins (c) hydrogenation (d) electrolysis
8. In commercial gasoline, the type of hydrocarbons which are 22. Pure methane can be produced by
desirable is (a) Wurtz reaction
(a) Branched hydrocarbons (b) Kolbe’s electrolytic method
(b) Straight chain hydrocarbons (c) Soda-lime decarboxylation
(c) Linear unsaturated hydrocarbons (d) Reduction with H2
(d) All of these 23. Which of the following liberates methane on treatment with
9. Fischer-Tropsch process is used in the manufacture of water ?
(a) synthetic petroleum (b) ethanol (a) Silicon carbide (b) Calcium carbide
(c) benzene (d) ethanoic acid (c) Beryllium carbide (d) Magnesium carbide
10. Octane number is zero for 24. Which of the following methods is most appropriate for the
(a) isoheptane (b) n-heptane manufacture of methane ?
(c) isooctane (d) n-octane (a) Reduction of CH2Cl2
11. Which of the following has lowest octane number ? (b) Wurtz reaction
(a) Iso-octane (b) n-Heptane (c) Liquefaction of natural gas
(c) n-Hexane (d) n-Hexadecane (d) None of these
12. Which of the following has highest octane number ? 25. The reaction/method that does not give an alkane is
(a) n-Hexane (b) n-Heptane (a) catalytic hydrogenation of alkenes
(c) n-Pentane (d) 2, 2, 4-Trimethylpentane (b) dehydrohalogenation of an alkyl halide
13. The process in which higher hydrocarbons are broken down
(c) hydrolysis of alkylmagnesium bromide
into lower hydrocarbons by controlled pyrolysis is called
(d) Kolbe’s electrolytic method
(a) Hydrolysis (b) Cracking
26. Ethyl bromide on treatment with alcoholic KOH gives
(c) reforming (d) Both (a) and (b)
(a) ethylene (b) ethanol
14. A fuel has the same knocking property as a mixture of 70%
(c) acetic acid (d) ethane
isooctane (2,2,4-trimethylpentane) and 30% n-heptane by
volume. The octane number of the fuel is 27. When n-propyl iodide is heated with alcoholic KOH, one of
the products is
(a) 100 (b) 70
(c) 50 (d) 30 (a) Propene (C3H6) (b) Cyclopropane (C3H6)
15. Cetane number of a diesel fuel will increase with the addition (c) C3H4 (d) C3H8
of 28. 1-Chlorobutane, on reaction with alcoholic potash (KOH),
(a) n-Decane (b) n-Hexadecane gives
(c) n-Pentane (d) a-Methylnaphthalene (a) 1-Butene (b) 1-Butanol
16. Which one is used as an antiknock in petrol fuel ? (c) 2-Butene (d) 2-Butanol
(a) Basic lead carbonate (b) Lead tetraacetate 29. In preparation of alkene from alcohol using Al2O3 which is
(c) Tetraethyl-lead (d) Basic lead sulphate effective factor ?
17. Wurtz reaction involves the interaction of alkyl halides in (a) Porosity of Al2O3 (b) Temperature
dry ether with (c) Concentration (d) Surface area of Al2O3
(a) sodium (b) zinc 30. Which one of the following heptanols can be dehydrated to
(c) copper (d) platinum hept-3-ene only ?
18. Ethane is formed by the reaction of methyl iodide and (a) Heptan-3-ol (b) Heptan-4-ol
sodium metal in dry ether solution. The reaction is known (c) Heptan-2-ol (d) Heptan-1-ol
as 31. Which of the principle is applied in the following reaction ?
(a) Clemmensen reduction (b) Kolbe’s reaction alc. KOH
CH 3CHBrCH 2 CH 3 ¾¾ ¾ ¾®
(c) Wurtz reaction (d) Cannizzaro's reaction
19. Which one of the following cannot be prepared by Wurtz CH 3CH = CHCH 3 + CH 2 = CHCH 2 CH 3
reaction ? ( Major ) ( Minor )
(a) CH4 (b) C2H6 (a) Markovnikov’s rule (b) Saytzeff's rule
(c) C3H8 (d) C4H10 (c) Kharasch's effect (d) Hofmann's rule
470 Chemistry
32. The conversion of 2, 3-dibromobutane to 2-butene with Zn 44. How many monochlorobutanes will be obtained on
and alcohol is chlorination of n-butane?
(a) redox reaction (b) a-elimination (a) 5 (b) 2
(c) b-elimination (d) Both (a) and (b) (c) 3 (d) 4
33. Coal-tar is a main source of 45. Conversion of CH4 to CH3Cl is an example of ......... reaction
(a) aromatic compounds (b) aliphatic compounds (a) electrophilic substitution
(c) cycloalkanes (d) heterocyclic compounds (b) nucleophilic substitution
34. Which of the following metal powder is used to convert (c) free radical substitution
trichloromethane into acetylene by heating the latter with it ? (d) free radical addition
(a) Na (b) Mg 46. The molecule having dipole moment is
(c) Ca (d) Ag (a) 2,2-dimethylpropane
35. Tetrabromoethane on heating with Zn gives (b) trans-2-pentene
(a) ethyl bromide (b) ethane (c) hexane
(c) ethene (d) ethyne (d) 2,2,3,3-tetramethylbutane
36. What is formed when calcium carbide reacts with heavy 47. Olefins can be hydrogenated by
water ? (a) zinc and hydrochloric acid
(a) C2D2 (b) CaD2 (b) raney nickel and hydrogen
(c) Ca2D2O (d) CD2 (c) nascent hydrogen
37. Monosodium acetylide reacts with an alkyl halide to form (d) lithiumaluminium hydride in ether
(a) an alkane 48. Which one of the following compounds would have the
(b) an alkene highest heat of hydrogenation ?
(c) an unsymmetric higher alkyne (a) CH 2 = CH 2
(d) a symmetric higher alkyne
38. Fractional distillation of coal-tar produces the following (b) CH 3 - CH 2 - CH = CH 2
fractions. Phenol is the main component of which fraction ? (c) CH 3CH = CHCH 3
(a) Light oil (b) Middle oil
(c) Heavy oil (d) Green oil (d) (CH3 ) 2 C = C(CH3 ) 2
39. Benzene is obtained by fractional distillation of 49. The reaction,
(a) light oil (b) middle oil CH 2 = CHCH 3 + HBr ® CH 3CHBrCH 3 is a type of
(c) anthracite oil (d) heavy oil
(a)electrophilic addition reaction
40. Heating a mixture of sodium benzoate or benzoic acid and
(b)nucleophilic addition reaction
soda-lime gives
(c)free radical addition reaction
(a) benzene (b) methane
(d)electrophilic substitution reaction
(c) sodium benzoate (d) calcium benzoate
41. Crude naphtha is a mixture of 50. CH 3 - CH = CH 2 + HI ¾ ¾® X , X is
(a) alkanes (b) conjugated dienes (a) CH3–CH2–CH2–I (b) CH3–CHI–CH3
(c) alkynes (d) alkyl halides (c) CH3CH2CH3 (d) None of these
42. Hydrocarbon which is liquid at room temperature is 51. When propene is treated with HBr in the dark and in absence
(a) pentane (b) butane of peroxide, the main product is
(c) propane (d) ethane (a) 1-Bromopropane (b) 2-Bromopropane
43. Which of the following isomers will have the highest boiling (c) 1,2-Dibromopropane (d) 1,3-Dibromopropane
point ? 52. Pentene-1 with HCl gives
(a) 3-Chloropentane (b) 2-Chloropentane
(a) CH 3 - CH 2 - CH 2 - CH 2 - CH 2 - CH 3 (c) 1,2-Dichloropentane (d) 1-Chloropentane
(b) CH 3 - CH - CH 2 - CH 2 - CH 3 53. Addition of HI to double bond of propene yields isopropyl
| iodide and not n-propyl iodide as the major product, because
CH3 addition proceeds throughs
(a) a more stable carbonium ion
(c) CH 3 - CH - CH - CH 3
| | (b) a more stable carbanion
CH 3 CH 3 (c) a more stable free radical
(d) homolysis
CH 3 54. Propene is reacted with HBr in presence of peroxides, the
|
(d) CH 3 - C - CH 2 - CH 3 product is
| (a) 2-Bromopropane (b) 1-Bromopropane
CH 3 (c) 3-Bromopropane (d) None of these
Hydrocarbons 471
55. The principal organic product formed in the reaction, 64. Ozonolysis of C7H14 gave 2-methyl-3-pentanone. The alkene
peroxide is
CH 2 = CH(CH 2 )8 COOH + HBr ¾¾¾ ¾® is
(a) 2-ethyl-3-methyl-1-butene
(a) CH3CHBr(CH2)8COOH (b) 3-ethyl-2-methyl-3-butene
(b) CH2 = CH(CH2)8COBr (c) 2,5-dimethyl-3, 4-dimethylhex-3-ene
(c) CH2BrCH2(CH2)8COOH (d) 3-ethyl-2-methyl-1-butene
(d) CH2 = CH(CH2)7CHBrCOOH 65. Which one of the following is a free-radical substitution
56. Markovnikov’s rule is applicable to reaction ?
(a) CH2 = CH2 (b) CH3–CH2–CH3
CH3 CH3 Anh. AlCl3 CH3
(c) CH3–CH = CH–CH3 (d) C=C (a) + CH3Cl
CH3 C2H5
57. Which of the following compound gives similar products
obeying Markovnikov’s rule and peroxide effect ? CH2Cl CH2NO2
(b) + AgNO2
(a) CH3–CH=CH2 (b) CH3CH = CH–CH3
(c) C2H5–CH = CH–CH3 (d) C2H5–CH = CH2
58. The addition of HBr is easiest in (c) CH 3CHO + HCN ® CH 3CH (OH)CN
(a) CH2=CHCl (b) ClCH=CHCl
CH3 Boiling CH2Cl
(c) CH3–CH=CH2 (d) (CH3)2C=CH2
(d) + Cl2
59. 1, 3-Butadiene when treated with Br 2 gives
(a) 1, 4-dibromo-2-butene (b) 1, 3-dibromo-2-butene
(c) 3, 4-dibromo-1-butene (d) 2, 3-dibromo-2-butene 66. Ethylene reacts with 1% cold alkaline KMnO4 to form
60. Ozonolysis of 2, 3-dimethyl-1-butene followed by reduction (a) oxalic acid (b) ethylene glycol
with zinc and water gives (c) ethyl alcohol (d) HCHO
(a) methanoic acid and 3-methyl-2-butanone 67. Baeyer’s reagent is used in the laboratory for
(b) methanal and 2-methyl-2-butanone (a) detection of double bond
(c) methanal and 3-methyl-2-butanone (b) reduction process
(d) methanoic acid and 2-methyl-2-butanone (c) oxidation process
61. Position of double bond in alkenes can be identified by (d) detection of glucose
(a) bromine water 68. Baeyer’s reagent is
(b) ammonical silver nitrate solution (a) saturated KMnO4 soln. (b) neutral KMnO4 soln.
(c) ozonolysis (c) alkaline KMnO4 soln. (d) acidic KMnO4 soln.
(d) None of these 69. Which of the following has the lowest dipole moment ?
62. Which alkene on ozonolysis gives CH3 CH3
CH3CH2CHO and CH3COCH3 (a) C=C (b) CH 3C º CCH 3
H H
CH3
(a) CH3CH2CH = C (c) CH 3CH 2 C º CH (d) CH 2 = CH - C º CH
CH3 70. 3-Hexyne reacts with Na/liq. NH3 to produce
(b) CH3CH2CH = CHCH2CH3 (a) cis-3-Hexene (b) trans-3-Hexene
(c) CH3CH2CH=CHCH3 (c) 3-Hexylamine (d) 2-Hexylamine
(d) CH 3 - C = CHCH 3 71. The most suitable catalyst for the hydrogenation of
| 2-Hexyne ¾ ¾® 2-cis-Hexene is
CH 3 (a) Pd–BaSO4 (b) (Ph3P)3RhCl
63. An alkene having molecular formula C7H14 was subjected (c) 10% Pd—C (d) Raney Ni
to ozonolysis in the presence of zinc dust. An equimolar 72. Lindlar’s catalyst is
amount of the following two compounds was obtained (a) Na in alcohol (b) Raney nickel
CH3 CH3 (c) Pd/BaSO4 (d) Na/liq. NH3
C = O and C=O 73. Acetylene when reacts with two molecules of HBr, then it
CH3 CH3CH2
gives
The IUPAC name of the alkene is (a) Br - C º C - Br (b) Br - CH = CH - Br
(a) 3, 4-dimethyl-3-pentene (b) 3, 4-dimethyl-2-pentene
(c) 2, 3-dimethyl-3-pentene (d) 2, 3-dimethyl-2-pentene (c) Br2 CH - CHBr2 (d) CH 3 - CHBr2
472 Chemistry
74. In the following reaction, 84. Propyne and propene can be distinguished by
(a) conc. H2SO4 (b) Br2 in CCl4
H 2O
C2H2 X CH3CHO (c) dil. KMnO4 (d) AgNO3 in ammonia
HgSO 4/H 2SO 4, 60ºC 85. Identify the reagent from the following list which can easily
what is X ? distinguish between 1-butyne and 2-butyne
(a) bromine, CCl4
(a) CH3CH2OH (b) CH3–O–CH3 (b) H2, Lindlar catalyst
(c) CH3CH2CHO (d) CH2 = CHOH (c) dilute H2SO4, HgSO4
75. When 2-pentyne is treated with dil. H2SO4 and HgSO4, the (d) ammonical Cu2Cl2 solution
product formed is 86. An organic compound, on treatment with Br2 in CCl4 gives
bromoderivative of an alkene. The compound will be
(a) 1-pentanol (b) 2-pentanol
(a) CH3 - CH = CH 2 (b) CH 3 CH = CHCH 3
(c) 2-pentanone (d) 3-pentanone
(c) HC º CH (d) H 2 C = CH 2
76. In which of the following hydrogen is most acidic?
87. Which of the following will yield a mixture of 2-chlorobutene
(a) Acetylene (b) Methane
and 3-chlorobutene on treatment with HCl ?
(c) Ethane (d) Ethylene
77. Match List I (Reagents used with ethyne) with List II (a) CH 2 = C = CH – CH3 (b) H 2 C = C - CH = CH 2
|
(products) and select the correct answer using the codes CH3
given below in the lists.
(c) CH 2 = CH - CH = CH 2 (d) HC º C - CH = CH 2
List I List II
88. An unknown compound A has a molecular formula C4H6,
I. Hydrogen chloride A Benzene
when A is treated with an excess of Br2, a new substance B
II. Hydrogen in presence B Acetaldehyde with formula C4H6Br4 is formed. A forms a white precipitate
of Ni at 573 K with ammonical silver nitrate solution. A may be
III. Hydrogen in presence C Ethene (a) Butyne-1 (b) Butyne-2
of Pd and BaSO4 at (c) Butene-1 (d) Butene-2
473 K 89. The hydrocarbon which decolourises alkaline KMnO 4
IV. Water in presence of D Ethane solution, but does not give any precipitate with ammonical
silver nitrate is
H2SO4 and HgSO4
(a) benzene (b) acetylene
Codes :
(c) propyne (d) butyne-2
(a) I-B, II-C, III-D, IV-A (b) I-A, II-D, III-C, IV-B 90. Select the true statement about benzene from amongst the
(c) I-A, II-D, III-B, IV-C (d) I-D, II-A, III-C, IV-B following
78. In its reaction with silver nitrate, acetylene shows (a) Because of unsaturation benzene easily undergoes
(a) oxidising property (b) reducing property addition
(c) basic property (d) acidic property (b) There are two types of C—C bonds in benzene molecule
(c) There is cyclic delocalisation of pi-electrons in benzene
79. When an alkyne, RC º CH , is treated with cuprous ion in
(d) Monosubstitution of benzene gives three isomeric
an ammonical medium, one of the products is
products
(a) RC º CCu (b) CuC º CH 91. Aromatic hydrocarbons undergo
(c) CuC º CCu (d) RC º CR (a) nucleophilic addition reactions
80. Which of the following does not give a white precipitate (b) electrophilic addition reactions
with AgNO3 solution ? (c) electrophilic substitution reactions
(d) None of these
(a) Propyne (b) 1-Butyne
92. Benzene on treatment with a mixture of conc. HNO3 and
(c) 2-Butyne (d) 1-Pentyne conc. H2SO4 at 100ºC gives
81. A compound is treated with NaNH2 to give sodium salt. (a) Nitrobenzene (b) m-Dinitrobenzene
Identify the compound (c) p-Dinitrobenzene (d) o-Dinitrobenzene
(a) C2H2 (b) C6H6 93. Which species represents the electrophile in aromatic
(c) C2H6 (d) C2H4 nitration ?
82. KMnO4 will oxidise acetylene to (a) NO -2 (b) NO +2
(a) ethylene glycol (b) ethyl alcohol (c) NO 2 (d) NO 3-
(c) oxalic acid (d) acetic acid 94. In the Friedel-Craft's synthesis of toluene, reactants in
83. Which one of the following gives Tollen’s reagent test ? addition to anhydrous AlCl3 are
(a) 1-Butanol (b) 2-Butanol (a) C6H6 + CH4 (b) C6H6 + CH3Cl
(c) 1-Butyne (d) 2-Butyne (c) C6H5Cl + CH3Cl (d) C6H5Cl + CH4
Hydrocarbons 473
95. When CH3Cl and AlCl3 are used in Friedel-Craft's reaction, 106. The correct order of reactivity towards the electrophilic
the electrophile is substitution of the compounds aniline (I), benzene (II) and
nitrobenzene (III) is
(a) Cl + (b) AlCl-4
(a) II > III > I (b) I < II > III
(c) CH 3+ (d) AlCl+2 (c) I > II > III (d) III > II > I
96. Benzene reacts with CH3COCl + AlCl3 to give 107. Addition of Br2 to 1-butene would give 1, 2-dibromobutane
(a) chlorobenzene (b) toluene which is
(c) benzyl chloride (d) acetophenone (a) achiral (b) racemic
97. Which of the following is not a meta-directing group ? (c) meso (d) optically active
(a) SO3H (b) NO2
(c) CN (d) NH2 108. Addition of Br2 to trans-2-butene would give a product
98. In the reaction of C 6 H 5 Y, the major product which is
( > 60%) is m-isomer, so the group Y is (a) chiral (b) meso
(a) —COOH (b) —NH2 (c) racemic (d) optically active
(c) —OH (d) —Cl 109. Name of following reaction is
99. A group which deactivates the benzene ring
towards electrophilic substitution but which directs the
incoming group principally to the o- and p-positions is + || ¾
¾®
(a) –NH2 (b) –Cl CN CN
(c) –NO2 (d) –C2H5 (a) Claisen Condensation
100. The most reactive compound for electrophilic nitration is (b) Diel’s Alder reaction
(a) benzene (b) nitrobenzene
(c) Dieckmann cyclisation
(c) benzoic acid (d) toluene
101. Which of the following will be most easily attacked by an (d) Michael addition reaction
electrophile? 110. The cycloalkane having the lowest heat of combustion per
Cl CH 2 group

(a) (b)
(a) (b)
OH CH3
(c) (d) (c) (d)

102. Among the followng compounds (I – III), the correct order


in of reactivity with an electrophile is 111. Which of the following will form alkynide ?
OCH3 NO2 C º C - CH 3
(a)

C º C-H
(b)
I II III
(a) II > III > I (b) III < I < II
(c) I > II > III (d) I = II > III CºC
103. The most reactive among the following towards (c)
sulphonation is
(a) toluene (b) chlorobenzene
(c) nitrobenzene (d) m-Xylene (d) CH 3 - C º C - CH 3
104. Which of the following is the most reactive towards ring 112. Which of the following will give alkene in Kolbe’s electrolytic
nitration ? method ?
(a) Benzene (b) Mesitylene
CH. COOK
(c) Toluene (d) m-Xylene ||
105. Aromatic compounds burn with a sooty flame because (a) CH 3 - C. COOK (b) CH 3CH 2 COOK
(a) they have a ring structure of carbon atoms
(b) they have a relatively high percentage of hydrogen CH 2 COOK
(c) they have a relatively high percentage of carbon |
(c) CH 3 - CH COOK (d) All of these
(d) they resist reaction with oxygen of air
474 Chemistry

1. Predict the product C obtained in the following reaction of


butyne-1. [CBSE-PMT 2007] CH3 CH3
CH3 H
HI
CH 3 CH 2 - C º CH + HCl ¾¾
® B ¾¾® C
(c) (d)
I
| H H
H H H
(a) CH3 - CH 2 - CH 2 - C - H HH H3 C
|
Cl 8. Which of the following conformers for ethylene glycol is most
stable? [CBSE-PMT 2010]
I
|
(b) CH 3 - CH 2 - CH - CH 2 Cl OH OH
H OH H H
I
|
(c) CH3CH 2 - C - CH3 (a) (b)
|
Cl
H H H H
(d) CH3 - CH - CH 2 CH 2 I
| H OH
Cl
2. Which of the compounds with molecular formula C5H10 OH OH
yields acetone on ozonolysis? [CBSE-PMT 2007] OH H
(a) 3-methyl-1-butene (b) cyclopentane
(c) 2-methyl-1-butene (d) 2-methyl-2-butene. (c) (d)
3. The IUPAC name of the compound having the formula
CH º C – CH = CH2 is : [CBSE-PMT 2009] H HO
(a) 1-butyn-3-ene (b) but-1-yne-3-ene H H H H H H
(c) 1-butene-3-yne (d) 3-butene-1-yne
9. In the following reactions, [CBSE-PMT 2011]
4. Liquid hydrocarbons can be converted to a mixture of
gaseous hydrocarbons by : [CBSE-PMT 2010] CH3
H + /Heat
(a) oxidation (a) CH
CH3–CH–CH–CH
–CH–CH–CH33 A + B
Major Minor
(b) cracking OH products products
(c) distillation under reduced pressure
HBr,dark
(d) hydrolysis (b) A ¾¾¾¾¾¾¾¾
A¾¾¾¾¾¾¾¾ ® ® CC ++ D
D
in absenceof peroxide
in absenceof peroxide æMajor ö æMinor ö
1.Mg, Ether èproduct ø èproduct ø
5 In the following reaction, C6H5CH2Br ¾¾¾¾¾
+
® X, the
2.H3O the major products (A) and (C) are respectively :
product ‘X’ is [CBSE-PMT 2010] CH3 CH3
(a) C6H5CH2CH2C6H5 (b) C6H5CH2OCH2C6H5 (a) CH22= C – CH22– CH33 and CH22– CH– CH22– CH33
(c) C6H5CH2OH (d) C6H5CH3
6. The IUPAC name of the compound Br
CH3CH = CHC º CH is [CBSE-PMT 2010] CH3 CH3
(a) Pent-l-yn-3-ene (b) Pent-4-yn-2-ene (b) CH3–3 C = CH–CH33 and CH3– C – CH2– CH3
(c) Pent-3-en-1-yne (d) Pent-2-en-4-yne
Br
7. In the following the most stable conformation of n-butane is:
[CBSE-PMT 2010] CH3 CH 3
CH3 (c) CH3– C = CH – CH3 and CH3 – CH – CH – CH 3
CH3
H CH3 H H Br
(a) (b) CH3 CH3
(d) CH22= C – CH22– CH33 and CH33– C – CH22 – CH33
H H H H
CH3 Br
H
Hydrocarbons 475

10. In the following reaction : [CBSE-PMT 2012 S] 17. Acid catalyzed hydration of alkenes except ethene leads to
the formation of [AIEEE 2005]
CH3
+
(a) mixture of secondary and tertiary alcohols
H 2O/H (b) mixture of primary and secondary alcohols
C CH —
— CH2
(c) secondary or tertiary alcohol
CH3 (d) primary alcohol
18. Which one of the following conformations of cyclohexane
A B is chiral? [AIEEE 2007]
Minor Product + Major Product
(a) Boat (b) Twist boat
The major product is :
(c) Rigid (d) Chair.
CH3 19. The compound formed as a result of oxidation of ethyl
benzene by KMnO4 is [AIEEE 2007]
(a) C CH CH3 (a) benzyl alcohol (b) benzophenone
(c) acetophenone (d) benzoic acid.
OH CH3 20. Which of the following reactions will yield
CH3 2, 2-dibromopropane? [AIEEE 2007]
(a) CH3 – CH = CH2 + HBr ®
(b) C CH2 CH3 (b) CH3 – C º CH + 2HBr ®
(c) CH3CH = CHBr + HBr ®
OH CH3 (d) CH º CH + 2HBr ®
21. The reaction of toluene with Cl2 in presence of FeCl3 gives
CH3
predominantly [AIEEE 2007]
(c) (a) m-chlorobenzene (b) benzoyl chloride
C CH CH3
(c) benzyl chloride (d) o- and p-chlorotoluene.
CH3 OH 22. Toluene is nitrated and the resulting product is reduced
with tin and hydrochloric acid. The product so obtained is
CH3 diazotised and then heated wth cuprous bromide. The
reaction mixture so formed contains [AIEEE 2008]
(d) C CH2 CH2 (a) mixture of o- and p-bromotoluenes
(b) mixture of o- and p-dibromobenzenes
CH3 OH (c) mixture of o- and p-bromoanilines
11. Which of these will not react with acetylene? [AIEEE 2002] (d) mixture of o- and m-bromotoluenes
(a) NaOH (b) ammonical AgNO3 23. In the following sequence of reactions, the alkene affords
(c) Na (d) HCl. the compound ‘B’
12. What is the product when acetylene reacts with hypochlorous O H O
acid? [AIEEE 2002] CH 3 - CH = CH - CH 3 ¾¾3¾
® A ¾¾¾
2 ® B.
Zn
(a) CH3COCl (b) ClCH2CHO The compound B is [AIEEE 2008]
(c) Cl2CHCHO (d) ClCHCOOH. (a) CH3CH2CHO (b) CH3COCH3
13. On mixing a certain alkane with chlorine and irradiating it (c) CH3CH2COCH3 (d) CH3CHO
with ultraviolet light, it forms only one monochloroalkane. 24. The hydrocarbon which can react with sodium in liquid
This alkane could be [AIEEE 2003] ammonia is [AIEEE 2008]
(a) pentane (b) isopentane
(a) CH 3CH 2 CH 2 C º CCH 2 CH 2CH3
(c) neopentane (d) propane
14. Butene-1 may be converted to butane by reaction (b) CH 3CH 2 C º CH
with [AIEEE 2003]
(c) CH 3CH = CHCH3
(a) Sn – HCl (b) Zn – Hg
(c) Pd/H2 (d) Zn – HCl (d) CH 3CH 2 C º CCH 2CH3
15. Which one of the following has the minimum boiling point ?
25. The treatment of CH3MgX with CH 3C º C - H produces
[AIEEE 2004]
(a) 1 - Butene (b) 1 - Butyne [AIEEE 2008]
(c) n- Butane (d) isobutane (a) CH 3 - CH = CH 2
16. 2-Methylbutane on reacting with bromine in the presence of
(b) CH 3C º C - CH3
sunlight gives mainly [AIEEE 2005]
(a) 1-bromo-3-methylbutane H H
(b) 2-bromo-3-methylbutane | |
(c) 2-bromo-2-methylbutane (c) CH 3 - C = C - CH3
(d) 1-bromo-2-methylbutane (d) CH4
476 Chemistry
26. One mole of a symmetrical alkene on ozonolysis gives two
moles of an aldehyde having a molecular mass of 44 u. The
alkene is [AIEEE 2010] (a) CHO (b) CHO
(a) propene (b) 1-butene
(c) 2-butene (d) ethene CO 2 H
27. Ozonolysis of an organic compound 'A' produces acetone (c) COOH (d)
CO 2 H
and propionaldhyde in equimolar mixture. Identify 'A' from
the following compounds: [AIEEE 2011] 31. The synthesis of 3-octyne is achieved by adding a
(a) 1 – Pentene bromoalkane into a mixture of sodium amide and an alkyne.
(b) 2 – Pentene The bromoalkane and alkyne respectively are
(c) 2 – Methyl – 2 – pentene [IIT-JEE 2010]
(d) 2 – Methyl – 1 – pentene (a) BrCH2CH2CH2CH2CH3 and CH3CH2C º CH
28. The non aromatic compound among the following is : (b) BrCH2CH2CH3 and CH3CH2CH2C º CH
[AIEEE 2011RS] (c) BrCH2CH2CH2CH2CH3 and CH3C º CH
(d) BrCH2CH2CH2CH3 and CH3CH2C º CH
32. The bond energy (in kcal mol–1) of a C–C single bond is
(a) (b) approximately [IIT-JEE 2010]
S (a) 1 (b) 10
(c) 100 (d) 1000
33. The number of optically active products obtained from the
(c) (d) complete ozonolysis of the given compound is :
[IIT-JEE 2012]

CH3 H
29. 2-Hexyne gives trans-2-Hexene on treatment with :
(a) Pt/H2 (b) Li / NH3 [AIEEE 2012]
CH3 CH CH C CH CH C CH CH CH3
(c) Pd/BaSO4 (d) Li AlH4
30. Cyclohexene on ozonolysis followed by reaction with zinc
dust and water gives compound E. Compound E on further H CH3
treatment with aqueous KOH yields compound F. (a) 0 (b) 1
Compound F is [IIT-JEE 2007] (c) 2 (d) 4

1. To prepare a pure sample of n-hexane using sodium metal as 5. An alkane C7H16 is produced by the reaction of lithium di(3-
one reactant, the other reactant will be pentyl) cuprate with ethyl bromide. The structural formula
(a) Ethyl chloride and n-butyl chloride of the product is
(b) Methyl bromide and n-pentyl bromide (a) 3-ethylpentane (b) 2-ethylpentane
(c) n-Propyl bromide (c) 3-methylhexane (d) 2-methylhexane
6. When ethyl alcohol is heated with conc. H2SO4 at 443 K,
(d) Ethyl bromide and n-butyl bromide
ethylene is formed by
2. Indicate the expected structure of the organic product when
(a) intramolecular hydration
ethyl magnesium bromide is treated with heavy water (D2O)
(b) intermolecular hydration
(a) C2H5–C2H5 (b) C2H5OD (c) intermolecular dehydration
(c) Sodium benzoate (d) C2H5D (d) intramolecular dehydration
3. (CH3)3CMgCl on reaction with D2O produces 7. Electrolysis of cold concentrated aqueous solution of
(a) (CH3)3CD (b) (CH3)3COD potassium succinate yields
(c) (CD)3CD (d) (CD)3COD (a) ethane (b) ethyne
4. The reagent used for the conversion, (c) ethene (d) ethane-1, 2-diol
8. When CH3CH2CHCl2 is treated with NaNH2, the product
CH 3CH 2 COOH ® CH 3CH 2 CH 3 is formed is
(a) LiAlH4 (a) CH 3 – CH = CH 2 (b) CH 3 – C º CH
(b) Soda-lime
NH2 Cl
(c) Red P and concentrated HI (c) CH3CH2CH (d) CH3CH2CH
(d) Zn – Hg/conc. HCl NH2 NH2
Hydrocarbons 477
9. 1,2-Dibromopropane on treatment with X moles of NaNH2 17. Which of the following alkenes will react fastest with H2
followed by treatment with ethyl bromide gave pentyne. under catalytic hydrogenation conditions ?
The value of X is R R R H
(a) 1 (b) 2 (a) (b)
H H R H
(c) 3 (d) 4
R R R R
10. In the reaction, (c) (d)
R H R R
oxidation NaOH Soda - lim e 18. The conversion of ClCH=CHCl to Cl2CH–CHCl2 can be
C6 H 5CH3 ¾¾¾¾® A ¾¾¾® B ¾¾¾¾¾
®C,
D carried out with
the product C is (a) Cl2 (b) Cl2/hn
(a) C6H5OH (b) C6H6 (c) Cl2/AlCl3 (d) Cl2/aq. NaOH
(c) C6H5COONa (d) C6H5ONa 19. The products formed by the action of chlorine on ethene in
11. Reactivity of hydrogen atoms attached to different carbon saturated solution of KBr is/are
atoms in alkanes has the order (a) ClCH2CH2Cl + ClCH2CH2Br
(a) Tertiary > Primary > Secondary (b) ClCH2CH2Cl
(c) ClCH2CH2Cl + BrCH2CH2Br
(b) Primary > Secondary > Tertiary
(d) ClCH2CH2Cl + BrCH2CH2Br + ClCH2CH2Br
(c) Tertiary > Secondary > Primary
20. In methyl alcohol solution, bromine reacts with ethylene to
(d) Both (a) and (b) yield BrCH2CH2OCH3 in addition to 1, 2-dibromoethane
12. In the free radical chlorination of methane, the chain initiating because
step involves the formation of (a) the ion formed initially may react with Br – or CH3OH
(a) chlorine atom (b) hydrogen chloride (b) the methyl alcohol solvates the bromine
(c) methyl radical (d) chloromethyl radical (c) the reaction follows Markovnikov’s rule
13. Consider the following reaction, (d) this is a free-radical mechanism
·
21. The intermediate during the addition of HCl to propene in
H 3C - CH - CH - CH 3 + Br ®' X '+ HBr the presence of peroxide is
| |
D CH3 ·
+
(a) CH3CHCH 2 Cl (b) CH 3CHCH 3
Identify the structure of the major product ‘X’ +
·
(c) CH3CH 2CH 2 (d) CH 3CH 2C H 2
· ·
(a) CH3 - CH - CH - CH 2 (b) CH 3 - CH - C - CH 3 22. In presence of peroxide, hydrogen chloride and hydrogen
| | | |
D CH3 iodide do not give anti-Markovnikov’s addition to alkenes
D CH 3
because
· ·
(a) both are highly ionic
(c) CH 3 - C - C H - CH 3 (d) CH3 - C H - CH - CH3 (b) one is oxidising and the other is reducing
| | |
D CH 3 CH3 (c) one of the steps is endothermic in both cases
(d) all steps are exothermic in both cases
14. A hydrocarbon with molecular formula C8H18 gives only 23. Arrange the following compounds in increasing order of
one monochloro derivative. The hydrocarbon is reactivity towards the addition of HBr.
(a) n-Octane RCH=CHR, CH2=CH2, R2C=CHR, R2C=CR2
(b) 2-Methylheptane (a) CH2=CH2<RCH=CHR<R2C=CHR <R2C=CR2
(c) 2,2,4-Trimethylpentane (b) R2C=CHR<RCH=CHR<CH2=CH2 <R2C=CR2
(d) 2,3,3,3-Tetramethylbutane (c) RCH=CHR<R2C=CR2<R2C=CHR <CH2=CH2
15. Which of the following compounds has the lowest boiling (d) R2C=CR2<R2C=CHR<RCH=CHR <CH2=CH2
point ? 24. Which of the following is the predominant product in the
reaction of HOBr with propene?
(a) CH3CH2CH2CH2CH3
(a) 2-Bromo-1-propanol (b) 3-Bromo-1-propanol
(b) CH3CH = CHCH2CH3
(c) 2-Bromo-2-propanol (d) 1-Bromo-2-propanol
(c) CH3CH = CH–CH = CH2
25. The reaction of propene with HOCl proceeds via the addition
(d) CH3CH2CH2CH3 of
16. Among the following alkenes : 1-butene(I), cis-2-butene(II), (a) H+ in the first step
trans-2-butene(III), the decreasing order of stability is (b) Cl+ in the first step
(a) III > II > I (b) III > I > II (c) OH– in the first step
(c) I > II > III (d) II > I > III (d) Cl+ and OH– in a single step
478 Chemistry
26. The products obtained via oxymercuration (HgSO4 + H2SO4) 34. Indicate the organic structure for product expected when 2-
of 1-butyne would be methylpropene is heated with acetyl chloride in the presence
of anhydrous ZnCl2 ?
(a) CH 3 - CH 2 - CO - CH 3
CH 3
(b) CH 3 - CH 2 - CH 2 - CHO |
(a) CH 3 - C - C = CH 2 (b) CH 3 - C - C - CH 3
(c) CH 3 - CH 2 - CHO + HCHO || | || |
O CH 3 O CH 3
(d) CH 3CH 2 COOH + HCOOH
27. Which set of products is expected on reductive ozonolysis CH 3 H
| |
of the following diolefin ? (c) CH 3 - C - CH 2 COCH 3 (d) CH 3 - C - CH 2 - COCH 3
| |
CH3 Cl CH 3
|
CH3CH = C - CH = CH 2 Hg2+ / H +
35. Ph - C º CCH3 ¾¾¾¾
¾® A , A is
(a) CH 3CHO ; CH 3COCH = CH 2 O
(a) Ph (b) Ph
(b) CH3CH = C - CHO ;CH 2O O
|
CH3 (c) Ph OH (d) Ph
OH
(c) CH 3CHO ; CH 3COCHO ; CH 2 O
36. Excess of CH3COOH is reacted with CH º CH in presence
(d) CH 3CHO ; CH 3COCH 3 ; CH 2 O of Hg2+, the product is
28. The alkene C6H10 producing (a) CH 3CH (OOCCH 3 ) 2
OHC(CH2)4CHO on ozonolysis is
(a) Hexene-1 (b) Hexene-3 (b) CH 2 = CH (OCOCH 3 )
(c) Cyclohexene (d) 1-Methylcyclohexene (c) (CH 3COO )CH 2 - CH 2 (OOCCH 3 )
29. 2.8 g of pure alkene containing only one double bond per (d) None of these
molecule, react completely with 8 g of bromine (in an inert 37. When acetylene reacts with arsenic trichloride in the
solvent). What is the molecular formula of the alkene ? presence of anhydrous aluminium chloride it produces
(a) C2H4 (b) C4H8 (a) b-Chlorovinyldichloroarsine
(c) C3H4 (d) C6H12 (b) Lewisite
(c) Nitrobenzene
CH 3 (d) Both (a) and (b)
|
30. The compound CH 3 - C = CH - CH 3 on reaction with 38. Which one of the following compounds will react with
methylmagnesium iodide ?
NaIO4 in the presence of KMnO4 gives
(a) CH3CH2CH2CH2CH3 (b) CH3CH=CH–CH=CH2
(a) CH3COCH3 + CH3COOH
(c) CH 3C º CCH 2 CH 3 (d) CH 3 CH 2 CH 2 C º CH
(b) CH3COCH3 + CH3CHO
(c) CH3CHO + CO2 39. Propyne on polymerization gives
(a) mesitylene (b) benzene
(d) CH3COCH3
(c) ethylbenzene (d) propylbenzene
+
CH3 H Br2, CCl4
40. 1-Butyne reacts with cold alkaline KMnO4 to produce
[F] C4H8Br2 (a) CH3CH2COOH
31. H3C OH –H 2O
(b) CH3CH2CH2COOH
5 such products
are possible (c) CH3CH2COOH + CO2
How many structures of F are possible ? (d) CH3CH2COOH + HCOOH
41. Two gases, P and Q decolourise aqueous bromine but only
(a) 2 (b) 5
one of them gives a white precipitate with aqueous ammonical
(c) 6 (d) 3 silver nitrate solution. P and Q are likely to be
32. When CH2 = CH – COOH is reduced with LiAlH4, the (a) ethane and ethyne (b) But-1-yne and but-2-yne
compound obtained will be (c) ethane and but-2-yne (d) ethyne and propyne
(a) CH3–CH2–COOH (b) CH2=CH–CH2OH 42. The compound X (C5H8) reacts with ammonical AgNO3 to
(c) CH3–CH2–CH2OH (d) CH3–CH2–CHO give a white precipitate, and on oxidation with hot alkaline
33. Ethyne can be oxidised to oxalic acid by using KMnO4 gives the acid, (CH3)2CHCOOH. Therefore, X is
(a) chromic acid (b) alkaline KMnO4 (a) CH2 = CHCH = CHCH3 (b) CH 3 (CH 2 ) 2 C º CH
(c) hypochlorous acid (d) Any of these
(c) (CH 3 ) 2 CH - C º CH (d) (CH3)2C = C = CH2
Hydrocarbons 479
43. Choose the compound which can react with [Ag(NH3)2 ]+ 52. Two organic compounds A and B both containing only
and on treatment with alk. KMnO4 gives (CH3)3CCOOH carbon and hydrogen, on quantitative analysis gave the
(a) CH 3CH 2 CH 2 - C º C - CH 3 same percentage composition by weight :
C = (12/13) × 100%, H = (1/13) × 100%
(b) (CH 3 ) 2 CHCH 2 - C º CH
A decolourises bromine water but B does not. A and B
(c) (CH 3 ) 3 C - C º CH respectively are
(a) C2H2 and C6H6 (b) C6H6 and C2H2
(d) (CH 3 ) 3 C - C º C - CH 3
(c) C2H4 and C2H6 (d) C2H2 and C2H6
44. Which of the following possesses the highest melting point
53. What is the correct product of reaction ?
?
(a) Chlorobenzene (b) o-Dichlorobenzene CH 3
(c) m-Dichlorobenzene (d) p-Dichlorobenzene 1. BH , THF
¾¾ ¾3¾ ¾® ?
45. The function of AlCl3 in the Friedel-Craft's reaction is 2. H 2O 2 , OH -
(a) to absorb water (b) to absorb HCl
(c) to produce electrophile (d) to produce nucleophile CH 3 CH 3
46. Which one of the following will undergo metasubstitution
(a) H (b)
on monochlorination ? H
(a) Ethoxybenzene (b) Chlorobenzene H
(c) Ethyl benzoate (d) Phenol H
47. Which of the following deactivates benzene towards further
substitution reaction ? CH 3 CH 3
(a) —NHR (b) —OH (c) (d) H
H H
(c) —COOR (d) —OR
48. Which of the following will have fastest rate of reaction H
with Br2/FeBr3? 54. Choose the correct alkyne and reagents for the preparation
NO2
(a) (b) of

CH3 OCH3
(a) , HgSO 4 , H 2SO 4 , H 2 O
(c) (d)

49. The order of activity of the various o- and p-directors is


(a) –O– > –OH > –OCOCH3 > –COCH3 (b) , HgSO 4 , H 2SO 4 , H 2 O
(b) –OH > –O– > –OCOCH3 > –COCH3
(c) –OH > –O– > –COCH3 > –OCOCH3
(d) –O– > –COCH3 > –OCOCH3 > –OH (c) , BH 3 , H 2 O 2 , NaOH
50. Identify the correct order of reactivity in electrophilic
substitution reactions of the following compounds :

CH3 Cl NO2 (d) , BH 3 , H 2 O 2 , NaOH

55. Of the three isomeric C3 H 4 hydrocarbons shown below


how many can exist with all carbon and hydrogen nuclei
1 2 3 4 located in a single plane
(a) 1 > 2 > 3 > 4 (b) 4 > 3 > 2 > 1 CHO
(c) 2 > 1 > 3 > 4 (d) 2 > 3 > 1 > 4 H 2C = C = CH 2 , CH 2 , H 3C - C º CH
51. n-Butylbenzene on oxidation will give CHO
(a) benzoic acid (b) butanoic acid (a) 0 (b) 1
(c) benzyl alcohol (d) benzaldehyde (c) 2 (d) 3
480 Chemistry
56. Which sets of reagents would give the correct product ? 59. The product of the following reaction

CH 2
¾
¾® ||
CH I , Zn (Cu )
¾¾ ¾
2 2¾ ¾ ¾
¾®
Diethyl ether
O
||
(a) Hg (O - C - CF3 ) 2

(b) Hg(OAC)2 in THF/ H 2 O , NaBH4 (a) (b)

(c) BH 3 , NaOH / H 2 O 2

(d) BH 3 , H 2 O 2 / OH - , NaH, CH 3 I (c) (d)


57. For the synthesis of the following compound :
60. The major product of the following reaction
CH 3
CH CH MgCl
O
H 3C - C º CH ¾¾¾¾¾¾
3 2 ®
ether H 3O +
CH 2 OH
CH 3 CHCH 3
HO CH 2 CH 3
||
Which method is best ?
(a) (b)
Method I :

CH 3 C º CH C CH 3
CH 2 HO C
NBS, CCl ( heat )
HO
NaOH
¾¾ ¾ ¾ ¾
4 ¾¾®
¾¾¾¾®
benzoyl peroxide
CH 3 (c) (d)
CH 3
61. The major product obtained in the following reaction
Method II : O
|| Br
CH 3 ¾¾
¾2®
?
FeBr3

Br / FeBr Mg / ethane
(i ) H C = O
3 ® ¾¾ ¾¾¾®
¾¾2¾¾¾ ¾¾ ¾2¾ ¾¾® O
(ii ) H 3O + ||
(a) Br
CH 3
O
(a) Method I ||
(b) Method II (b) Br
(c) Both (I) and (II) are equally good
(d) Neither (I) nor (II) O
||
58. The reaction of cis-2-butene with CH 2 I 2 and Zn(Cu) to
(c) Br Br
give cis-1, 2-dimethyl cyclopropane is
(a) stereospecific reaction
O
(b) enantiospecific reaction ||
(c) diastereoselective reaction (d)
(d) regioselective reaction Br
Hydrocarbons 481
62. The unsaturated hydrocarbon which on ozonolysis gives 69. Bromination of ethyl benzene in presence of light gives
one mole each of formaldehyde, acetaldehyde and methyl
C2 H5 C2 H5
glyoxal ( CH 3 CO.CHO)
Br
(a) CH 3 – CH = C( CH 3 ) – CH 3 (a) (b)
(b) CH 2 = CH – CH 2 – CH = CH 2
(c) CH 2 = CH – C( CH 3 ) = CH – CH 3 Br

(d) (CH 3 ) 2 C = CH - CH 3 Br
63. Which of the following is not the mechanistic step during
CH - CH 3
chlorination of methane by Cl2 / hn ? (c) (d) Both (a) and (b)
·
(a) Cl 2 ® 2Cl
· ·
70. Pair of enantiomers of 1, 2-dimethylcyclopentane having
(b) Cl + CH4 ® CH3 + HCl structures
· ·
(c) Cl + CH 4 ® CH 3Cl + H
· · H CH 3 H H
(d) Cl + CH3 ® CH3Cl CH 3 H

64. Which is correct for the following changes ? H CH 3 CH 3 H CH 3 CH 3


Me
(I) (II) (III)
X
H OH
Baeyer's reagent (a) I and III (b) I and II
A ¾¾® B ¾¾¾¾¾¾
¾®
H OH (c) II and III (d) I, II and III
Me 71. Which of the following is correct
(a) X is Lindlar Catalyst, B is Cis-2-butene
(b) A is 2-butyne, X is Na-liq. NH 3 C H COOH
(a) ¾¾
6¾5 ¾¾
¾® H H
(c) B is trans-2-butene, X is Na-liq. NH 3
H H O
(d) A is 2-butene, X is SeO 2

65. NH 2 + (excess) of Et. MgBr ® ?


(i ) O
(a) 1 mol of Ethane (b) 3 mol of ethane (b) ¾¾ ¾
¾3®
OHC.CH2.CH2.CHO
(ii) H 2O 2
(c) EtC º C.CH 2CH 2 NHEt (d) 4 mole of ethane
H H
66. Addition of Br2 to alkene proceeds by the intermediate
formation of
(a) carbonium ion KMnO OH
(c) ¾¾ ¾¾

(b) carbanium ion with NaOH
(c) bridged (non classical) carbonium ion H
H H OH H
(d) All of these
67. In the following change,
O O
|| ||
(d) Alk .
X+2 C=C ¾¾®
Pd
+2 CH – CH ¾¾
¾® O - C - (CH 2 ) 3 - C - O
100°C KMnO 4

The X may be H H
(a) cyclohexene (b) cyclohexadiene
(c) cyclogexatriene (d) None of these HgSO 4 / H 2SO 4
X
Na liquid NH 3 72. 1- Pentyne
68. R - CH = CH 2 ¾¾ ¾ ¾ ¾¾ ® R. CH 2 . CH 3 Y
Et. OH BF3 , THF, H 2 O 2 , OH -
The reaction is called X and Y can be distinguished by
(a) Birch's reduction (b) Clemensen's reduction (a) silver-mirror test (b) iodoform test
(c) Catalytic reduction (d) None of these
(c) Both (a) and (b) (d) Neither (a) nor (b)
482 Chemistry
73. Which of the following change is correct 76. PhCH 3 on reaction with Cl2 / hn followed by Na/ether will
I 2 , CH3COOAg give
(a) C=C ¾¾ ¾ ¾¾ ¾
¾® C -C
wet CH3COOH
OH OH
OH (a)
I , CH COOAg
(b) C=C ¾¾
2 ¾¾
3 ¾¾
¾® C -C
dry CH3COOH Me Me
OH
(c) Both (a) and (b) one correct (b)
(d) Neither (a) nor (b)
74. Choose the correct among the following (c) Me Me
H
H (d) - CH 2 - CH 2 -
(a) H
H 77. In the following changes which is correct.
S - trans (transoid), less stable Reagent HCOOOH
A ¾¾ ¾¾® B ¾¾ ¾¾
¾® Erythroracemic
R
(b) H H
H H (a) A is 2-hexyne and R is Na-liquid NH 3
S - cis (cisoid), more stable (b) A is 3-hexyne and R is H 2 – Lindlar
(c) S-refers to geomectrical isomerism with respect to single
(c) B is 2-hexene and R is H 2 – Lindlar
bond.
(d) All are correct (d) A is 3-hexyne and R is H 2 /Ni
75. Choose the correct product of the following reaction
O
O ||
||
Cl 78. The compound reacts with MeMgI to give A and
(1) OOH
CH 3
? Cl
(2) H3 O +
CH 3 with Me 2 CuLi to give B. What are A and B ?

O O Me OMgI O
CH 3 || || ||
OH
(a) CH 3 (b) OH
(a) (b)
CH 3 OH
H CH 3
Me Me Me Cl Me

CH 3 OH Me OMgI Me Me OMgI Me O Li
(c) OH (d) CH 3
CH 3 H (c) (d)
OH CH 3
Cl Me Me Cl
Hydrocarbons 483

EXERCISE 1 8. (a) 9. (a) 10. (b)


11. (d) Longer the straight chain alkane, lower is the octane
1. Liquefied petroleum gas. number. Thus, n-hexadecane has even lower octane
2. A mixture of ethane, propane and butane is formed. number than n-heptane
3. Acetylene, due to greater electronegativity of the sp– 12. (d) 2,2,4-Trimethylpentane is the most branched
hybridized carbon. hydrocarbon among the four given options
4. No, because the difference of energy between these two 13. (b) Liquid hydrocarbons (a mixture of relatively higher
conformations is very small. hydrocarbons) is converted into gaseous hydrocarbons
5. because two Cl’s in it have equal and opposite bond (a mixture of lower hydrocarbons) by strong heating, this
moments. process is known as cracking
6. CH 3 - C º C - CH 3 + NaNH 2 ¾ ¾ ® No reaction 14. (b) Since the fuel mixture contains 70% isooctane, its octane
-+ number is 70
CH3CH2 º CH + NaNH2 ¾¾ ® CH 3CH 2 C º CNa
15. (d) 16. (c) 17. (a) 18. (c)
7. A halogen compound used during halogenation of benzene
19. (a) CH4 has only one carbon atom, hence it can’t be prepared
is called halogen carrier e.g. FeCl3.
by Wurtz reaction, which involves two molecules of alkyl
8. (iii) < (iv) < (i) < (ii).
halide
Ni
10. (i) CH4 + H2O ¾¾¾¾
1273K
® CO + 3H
2
20. (a)
21. (d)
1500 K
(ii) CH4 ¾¾¾¾
Carbon
® C + 2H2 22. (c) Other three methods can be used for the preparation of
hydrocarbons having at least 2 carbon atoms
CH3 CH3 23. (c)
| | 24. (c)
Alkaline
(iii) CH3 - C -H + [O] ¾¾¾¾ ® CH 3 - C - OH
| KMnO 4 | 25. (b) Dehydrohalogenation of an alkyl halide gives an alkene
CH3 CH3 26. (a) 27. (a) 28. (a)
Isobutane tert - Butyl alcohol 29. (d) The amount of alcohol absorbed depends upon the
surface area of Al2O3
11. red phosphorus removes the iodine liberated in reaction
30. (b) Heptan-4-ol has similar alkyl group on both ends of the
OH carbon atom bearing –OH group, hence dehydration of
12. (i) (Hydroxylation) (ii) alcohol will form only one alkene, not a mixture of two
OH OH OH alkenes
O OH
O |
(iii) CH3CH 2 CH 2 CH CH 2 CH 2 CH3 ¾¾¾® CH3CH 2 CH = CHCH
+ - H 2O
OH Heptan - 4- ol Hept -3- en
Butanone Ethanoic acid
18. (d) 19. (a) 20. (d) 21. (c) 22. (a) ® CH 3CH 2 CH = CHCH 2 CH 2 CH 3
Hept -3- ene
23. (d) 24. (a) 25. (a)
31. (b)
EXERCISE 2
32. (c) CH 3CHBrCHBrCH 3 ¾Zn / alcohol
¾ ¾ ¾¾® CH 3CH = CHCH 3
1. (a)
Since two bromine atoms are lost from different carbon
2. (c)
atom, the reaction is known as b-elimination.
3. (c) Vegetable oil is a mixture of triglycerides of higher fatty
33. (a) 34. (d) 35. (d)
acids.
4. (b) Petroleum ether has lower boiling point than all the other 36. (a) CaC 2 + 2D 2 O ® C 2 D 2 + Ca (OD) 2
three products 37. (c) HC º CNa + XR ® HC º CR
5. (d) Gasoline has the lowest boiling point, while diesel has Since the atom or group attached to two acetylenic carbon
the highest. atoms are different, the alkyne formed (higher alkyne) is
6. (c) unsymmetrical
7. (d) Highly branched chain paraffins have high octane 38. (b) 39. (a) 40. (a) 41. (a)
number
484 Chemistry
42. (a) Recall that first four alkanes are gaseous in nature at CH3 CH3 CH3 CH3
room temperature. 63. (d) C=O+O=C C=C
CH3 C2H5 CH3 CH2CH3
43. (a) Among isomeric alkanes, boiling point decreases with
2,3-Dimethyl-2-pentene
increase in branching
1 2 2 1 64. (a) Since the ozonolysis product, 2-methyl-3-pentanone,
44. (b) n-Butane, C H 3 C H 2 C H 2 C H 3 has two different contains only six carbon atoms, while the alkene has seven
hydrogen atoms, marked by 1 and 2. carbon atoms, the other ozonolysis product should be
45. (c) 46. (b) 47. (b) CH2O, the only carbonyl compound having one carbon
48. (a) The heat of hydrogenation of an alkene depends upon atom. Hence, the structure of the alkene C7H14 can be
its stability. Higher the stability, lower the heat of established as below
hydrogenation. Since CH2=CH2 has no substituent, it is
CH 3 CH 2CH 3
the least stable alkene and hence has the highest heat of | |
hydrogenation CH 3 - C H - C = O + O = CH 2
49. (a) 50. (b) 51. (b) 52. (b) 2- Methyl -3pentanone
53. (a) All are examples of electrophilic addition reactions and
involve the formation of stable carbocation leading toCH3 CH 2CH3 CH3 CH 2CH3
the formation of addition product according to| |
ozonolysis
| |
Markovnikov’s rule CH 3 - CH - C = O + O = CH 2 ¬¾¾¾¾ ¾ CH 3 - CH - C = CH 2
2- Methyl- 3pentanone 2 - Ethyl - 3- methyl -1- butene
54. (b)
55. (c) (See Anti Markovnikov's rule) 65. (d) 66. (b) 67. (a) 68. (c)
56. (d) Markovnikov’s rule is applicable only to unsymmetrical
69. (b) CH 3C º CCH 3 , being symmetrical, has the lowest dipole
alkenes, which is option (d) here
57. (b) Symmetrical alkenes (option b) give similar products moment
whether the addition is according to Markovnikov’s rule 70. (b) Reduction of alkynes with Na/liq. NH3 gives
or according to peroxide effect. trans-alkenes
58. (d) Addition of HBr on alkenes is an electrophilic reaction, 71. (a) Reduction of alkynes with Lindlar’s catalyst (Pd-BaSO )
4
hence more the number of electron-releasing groups on gives cis-alkenes
the double bond easier will be the addition
72. (c)
59. (a) The intermediate 2º carbocation shows resonance
HBr
é +
Br + ù 73. (d) CH º CH ¾¾¾® CH 2 = CHBr
CH 2 = CH - CH = CH 2 ¾¾¾
® êCH 2 Br - CH - CH = CH 2 ú
ë û HBr
¾¾¾ ¾ ¾¾ ¾ ¾® CH 3CHBr2
Markovnikov addition
+ ù
« CH 2 Br - CH = CH - CH 2 úû 74. (d) 75. (c) 76. (a) 77 (d)
78. (d) 79. (a)
80. (a) 2-Butyne (CH3CºCCH3 ) , being a non-terminal alkyne,
-
Br
¾¾
¾® CH 2 Br - CH = CH - CH 2 Br does not have acidic hydrogen atom
1, 4 - Dibromo - 2 - butene 81. (a) Only terminal alkynes, react with bases like NaNH2,
60. (c) In such questions, break the molecule at double bond by CH3MgI, ammonical AgNO3 and ammonical cuprous
adding oxygen atom at each doubly bonded carbon chloride
82. (c)
CH3
| 83. (c) Tollen’s reagent (ammonical silver nitrate) reacts with
i) O terminal alkynes
CH 2 = C - CHCH3 ¾¾¾
3
® CH 2O + O = C - CHCH3
| ii) Zn/H 2O Methanal | | 84. (d) Ammonical silver nitrate and ammonical cuprous chloride
CH3 CH3 CH3
2,3- Dimethyl -1- butene 3- Methyl - 2 - butanone
give precipitate only with terminal alkynes (propyne,
CH 3C º CH )
61. (c)
62. (a) In such questions follow the following points: (i) write 85. (d) Ammonical cuprous chloride gives precipitate (red) only
down the structure of the given carbonyl compound(s), with terminal alkynes, i.e. alkynes having acidic hydrogen
(ii) remove oxygen atoms of the two carbonyl compounds, atom, such as butyne-1 ( CH º CCH 2 CH 3 )
(iii) join the two carbon atoms of the two carbonyl groups
to each other to get the structure of the parent alkene 86. (c) Only alkynes, which is CH º CH in the present question,
react with Br2 to give bromoderivative of an alkene
CH 3CH 2 C = O + O = C - CH 3 ¬
¾¾ CH 3CH 2 C = C - CH 3
| | | |
H CH3 H CH3 CH º CH + Br2 ¾
¾® BrCH = CHBr
Hydrocarbons 485

+
101. (c) Phenol as well as toluene have electron-releasing groups,
87. (a) CH 2 = C = CH - CH 3 ¾H
¾
¾® however –OH group, in phenol, is more electron-releasing
(due to +R effect) than the –CH3 group in toluene, so
+ + phenol is more easily attacked by an electrophile.
é ù
= C = CH - CH3 ¾¾¾
® êCH 2 = C- CH 2CH3 + CH 2 = CH - CHCH3 ú
ë û 102. (c) OCH3 NO2
Cl-
¾¾¾
® CH 2 = C - CH 2CH3 + CH 2 = CH - CHCH3
| | > >
Cl Cl
2- Chlorobutene 3- Chlorobutene
Electron-releasing No group Electron withdrawing
88. (a) Conversion of C 4H 6 to C 4H 6Br 2 indicates that the group present is present group is present
compound is either butyne-1 or butyne-2. However, white
precipitate with ammonical silver nitrate solution indicates
that it is a terminal alkyne, i.e. butyne-1 and not butyne-
2 103. (d) Cl NO2 CH3
89. (d) Since the hydrocarbon decolourises KMnO4 solution, it
must be unsaturated, i.e. alkene or alkyne. However, it
does not give any precipitate with ammonical AgNO3, it
must be a non-terminal alkyne, i.e. butyne-2 (b) (c) (a)
90. (c) Electron-withdrawing One electron-
91. (c) groups are present releasing group
92. (b) Mononitration occurs at about 60º C, but at 100ºC
dinitration occurs. Since –NO 2 is m-directing,
m-dinitrobenzene is formed at 100ºC CH3
93. (b)
94. (b)
95. (c) RCl + AlCl3 ® R+ + AlCl -4
Electrophi le CH3
(a) (d)
96. (d)
One electron- Two electron-releasing groups, hence
97. (d) releasing group most reactive towards sulphonation
98. (a) Since the major product (>60%) is a m-isomer, the group
Y must be m-directing which is –COOH
104. (a) Mesitylene (1,3,5-trimethylbenzene) has three electron-
99. (b) –Cl group is o-, p-directing due to +R effect ; however it
releasing (–CH3) groups, while m-xylene, toluene and
is deactivating due to strong –I effect of Cl (difference
from other o-, p-directing groups which are activating). benzene have 2, 1 and 0 electron-releasing (–CH3) groups
The net result is that chlorobenzene undergoes o-, p- 105. (c)
substitution, but with difficulty 106. (c)
100. (d) NO2 COOH 107. (b) CH3 - CH 2 - CH = CH 2 + Br2 ¾
¾®

Br
|
CH3 - CH 2 - CH - CH 2 Br Racemic mixture.
Benzene Nitrobenzene Benzoic acid
108. (b) Always remember cis 2-butene + Br2 ® Racemic 2, 3-
No group is present Deactivating group present
dibromobutane \ Trans 2-butene + Br2 ® Meso 2, 3-
CH3 dibromobutane.
109. (d) The given reaction is Diel’s Alder reaction
110. (a) More is the stability, lower is the heat of combustion
(See Baeyer’s strain theory). Cyclohexane is more stable.
111. (b) Only 1-Alkynes form alkynides
Benzoic acid Toluene 112. (c) K salt of succinic acid or its derivatives give alkenes by
ivating group present Activating group present Kolbe’s electrolytic method.
486 Chemistry
EXERCISE 3 8. (a) Due to hydrogen bonding between the two OH groups,
1. (c) This reaction occurs according to Markownikoff’s rule gauche conformation of ethylene glycol (a) is the most
which states that when an unsymmetrical alkene undergo stable conformation.
hydrohalogenation, the negative part goes to that
C-atom which contain lesser no. of H-atom. H
O
CH3 - CH 2 - C º CH + HCl H O
H
¾¾
® CH3 - CH 2 - C = CH 2
|
Cl
H H
I H
|
HI
¾¾® CH3 - CH 2 - C - CH3 \ Correct choice : (a)
|
Cl

CH3
2. (d) H3C — C = CH — CH3 9. (b) CH3 — CH — CH — CH3 H+/ Heat

CH3 OH

H3C O H
O3
C C CH3 CH3
H3C CH3 H / Heat
O CHO — CH — CH — CH CH3 — C = CH — CH3 + CH3 — CH — CH = CH2
2 methyl 3 methyl
O O butene-1 (20%)
H 2O
butene-2
CH3 – C – CH3 + CH3 – C – H (80%) (B)
–H2O 2 acetone acetaldehyde (A)

In this case dehydration is governed by Saytezeff’s rule


3. (c) If both the double and triple bonds are present, the according to which hydrogen is preferentially eliminated
compound is regarded as derivative of alkyne. Further from the carbon atom with fewer number of hydrogen atoms
if double and triple bonds are at equidistance from either i.e., poor becomes poorer. Thus, 2 methyl butene-2 is the
side, the preference is given to double bond. major product.
4. (b) During cracking higher hydrocarbons (liquid) are
converted to lower gaseous hydrocarbons. CH3
ether HBr, dark
5. (d) C6H5CH2Br + Mg ¾¾¾® C6H5CH2MgBr CH3 — C = CH — CH3 (CH — CH — CH — CH
in absence
(A) of peroxide
(in ether as solution)

+
H
C 6 H 5 CH 2 Mg Br ¾¾¾ CH3
® C H CH + MgBr(OH)
H OH 6 5 3 HBr, dark
(CH3)2 — CH — CH — CH3 + CH3 — C — CH2 – CH3
6. (c) When both double and triple bonds are present, then Br Br
triple bond is considered as the principal group. (Minor) (Major)

This reaction is governed by Markownikoff’s rule according


CH3 - CH = CH- C º CH to which when an unsymmetrical reagent e.g. HBr adds to
5 4 3 2 1
an unsymmetrical alkene, then the negative part of the
7. (b) The bulky methyl groups are at maximum distance from reagent is added to that carbon atom of the double bond
each other. which bears the least number of hydrogen atom. Thus, in
above case. 2-methyl 2-bromo butane will be the major
product.
Hydrocarbons 487

CH3 H / Pd
14. (c) Butene - 1 ¾¾2 ¾ ¾® Butane
15. (d) Bpt. follows the order
10. (a) C CH CH2 Alkynes > Alkenes > Alkanes (Straight chain) > Alkanes
(branched chain) of comparable molecular weights.
CH3
CH3 CH3
| |
+ Br
H 16. (c) CH3 - CH - CH 2 - CH3 ¾¾¾
2 ® CH - C - CH CH
3 2 3
sunlight |
Br
2 - bromo - 2 - methyl butane
CH3
H2O
Ease of replacement of H-atom 3° > 2° > 1°.
C CH CH3 +
Å –H
H SO
CH3 17. (c) CH3 - CH = CH 2 + H 2 O ¾¾¾¾
2 4 ® CH - CH - CH
3 3
|
1,2-methyl shiftting OH
2° alcohol

CH3 CH3
CH3 CH3 | |
H 2SO 4
Å
CH3 - C = CH 2 + H 2 O ¾¾¾¾
® CH3 - C - CH3
CH3 C CH CH3 |
C CH CH3 OH
3° alcohol
CH3 H3C OH Addition follows Markownikoff’s rule.
3° carbocaction
(more stable)
A (Minor) 18. (b) Chiral conformation will not have plane of symmetry. Since
twist boat does not have plane of symmetry it is chiral.
H2O 4 1

OH CH3
3 6
C CH CH3

CH3
B (Major) 5 2

11. (a) Acetylene reacts with the other three as: 19. (d) When alkyl benzene are oxidised with alkaline KMnO4,
(strong oxidising agent) the entire alkyl group is oxidised
CH2
CH º CNa Na
CH º CH +HCl to –COOH group regardless of length of side chain.
liq. NH3
CHCl CH2CH3 COOH
CH2 CH3 ( O ), KMnO4 / OH -
+HCl ¾¾¾¾¾¾¾¾ ®
CHCl CHCl2 Benzoic acid
Ethyl benzene
[AgNO3+NH4OH]
CH º CH AgCºCAg + NH4NO3 20. (b) The reaction follows Markownikov’s rule which states
white ppt. that when unsymmetrical reagent adds across
CHOH unsymmetrical double or triple bond the negative part
12.(c) CH º CH + HOCl adds to carbon atom having lesser number of hydrogen
® ||
¾¾¾¾ atoms.
CHCl
HBr
CH 3 - C º CH + HBr ® CH3 - C = CH2 ¾¾¾ ®
+ HOCl é CH(OH) 2 ù -H O CHO |
¾¾¾¾® ê | 2 ® |
ú ¾¾¾¾ Br
êëCHCl2 úû CHCl2
Br
dichloroacetaldehyde |
13. (c) In neopentane all the H atoms are same (1º). CH3 - C - CH3
|
CH3 Br
|
CH 3 - C - CH 3 2, 2-dibromopropane
|
CH3
488 Chemistry
21. (d) FeCl 3 is Lewis acid. In presence of FeCl 3 o- and 25. (d) Writing the reaction we get
p-chlorotoluene is formed.
CH 3 MgX + CH 3 – C º C – H ¾¾
®
CH3 CH3 CH3
Cl CH3 – C º CMgX + CH 4 ( g )
FeCl3
+ Cl2 ¾¾¾® + So we find that CH4 is produced in this reaction.
Toluene o-chlorotoluene
Cl 26. (c) The given molecular formula suggests that the aldehyde
p-chloro toluene formed will be acetaldehyde hence the alkene will be
CH3CH = CHCH3
CH3 2-butene

22. (a) Toluene ( ) contains –CH3 group which is O


O H H
¾¾®
3
C C
o-, p- directing group so on nitration of toluene the
O O
–NO2 group will occupy o-, p- positions.

CH3 CH3 CH3


NO2
(HNO 3 + H2SO 4) Zn / H 2O
+ ¾¾¾¾® 2CH3CHO
27. (c) From the products formed it is clear that the compound
NO2 has 5 carbon atoms with a double bond and methyl
o- p-
group on 2nd carbon atom.
on reduction with Sn/HCl they will form corresponding CH3
anilines in which –NO2 group changes to –NH2. The | O3 / Zn, H2O
CH3 - C = CH - CH 2 - CH3 ¾¾¾¾¾ ¾®
CH3 CH3
NH2 (2–Methyl–2–pentene)
mixture now contains and . These (A)

CH3 O
NH2
|
anilines when diazotized and then treated with CuBr forms CH3 - C = O + CH 3 - CH 2 - C
o-, p- bromotoluenes.
23. (d) Completing the sequence of given reactions, H
Acetone Propionaldehyde

CH 3 – CH = CH - CH 3 ¾¾3¾
®
O
H H
28. (d) sp3 Carbon
O
Zn / H O
CH3– CH CH – CH3 ¾¾¾¾®
2

Cyclopentadiene is non aromatic, as it has sp3 carbon


O O in the ring.
‘A’ 29. (b) Anti addition of hydrogen atoms to the triple bond
(ozonide)
occurs when alkynes are reduced with sodium (or
lithium) metal in ammonia, ethylamine, or alcohol at
2CH3CHO+ H 2 O + ZnO
'B' low temperature. This reaction called, a dissolving
Thus ‘B’ is CH3CHO metal reduction, produces an (E)- or trans-alkene.
Hence (d) is correct answer. Sodium in liq. NH3 is used as a source of electrons in
24. (b) Alkynes having terminal –C º H react with Na in liquid the reduction of an alkyne to a trans alkene.
ammonia to yield H2 gas. CH3CH2C º CH can react with CH3 CH2 CH2 C C CH3
Na in liquid NH3 so the correct answer is (b). 2-Hexyne
Na in
CH3CH 2 C º CH ¾¾¾¾¾
® Li/NH3
liquid NH3 CH3 CH2 CH2 H
Birch reduction
C C
1 H CH3
CH3CH 2 C º C – Na + + H 2 ( g ) Trans-2-Hexene
2
Hydrocarbons 489
8. (b)
1.O3 CHO
30. (a) 9. (c) CH 2 BrCHBrCH3 ¾¾ ¾ ¾¾
2® i ) xNaNH
Pentyne
2.H2O/Zn CHO
ii ) C 2 H5Br ( No. of C atoms=5)
[E]
The reaction can be summed up as below
KOH(aq), D
CHO 2 NaNH NaNH
– H 2O CH 2 BrCHBrCH3 ¾¾ ¾ ¾
2 ® CH º CCH ¾¾ ¾
3 ¾2 ® NaC
aldol condensation 1, 2 - Dibromopropane
[F]
BrCH CH
31. (d) Only (d) can form 3-Octyne NaC º CCH 3 ¾¾ ¾2¾¾
3 ® CH CH C º CCH
3 2 3
Pentyne- 2
® CH 3CH 2 C º C - Na +
NaNH
CH 3 CH 2 C º CH ¾¾¾¾ 2
- NH 3 10. (b)
CH CH CH CH Br 11. (c) Reactivity of H-atoms depends upon the stability of free
¾¾¾¾¾¾¾
3 2 2
(S 2)
2
® radicals (3º > 2º > 1º) produced by them
N

CH3 CH 2 C º CCH 2 CH 2 CH 2 CH3 + NaBr hν


Cl : Cl ¾¾® Cl. + Cl.
12. (a) (chain-initiation
Chlorine Chlorine atoms or chlorine
348 molecule free radicals
32. (c) C – C bond energy = 348 kJ/mol = kcal/mol
4.2 step)
= 82.85 kcal/mol » 100 kcal/mol. 13. (b) Since Br• is less reactive but more selective, therefore the
33. (a) most stable 3º free radical will be the major product
14. (d) Since the hydrocarbon C 8 H 18 gives only one
CH3 H
monochloroderivative, its all 18 hydrogen atoms are
equivalent, which is possible only in case of 2,2,3,3-
H3C CH CH C CH CH C CH CH CH3 tetramethylbutane

CH 3 CH 3
H CH3 | |
CH 3 - C - C - CH 3
O3/Zn/H2O | |
CH 3 CH3
15. (d) Option (d) has the lowest number of carbon atoms
CH3
16. (a)
17. (a) See the order of stability of substituted alkenes. Lesser
2 H3C CH O + 2O CH C CH O the stability more is the reactivity.
(achiral) 18. (a)
H 19. (a) Reaction of chlorine on ethene is an electrophilic addition
(achiral) and involves the formation of carbocation which is

EXERCISE 4 CH 2 ClCH +2 which may be attacked by Cl– (from Cl2) or


1. (c) Br– (from KBr) forming ClCH2CH2Cl and ClCH2CH2Br
2. (d) C2 H 5 MgBr + DOD ® C2 H 5 D + MgBr(OD) respectively
20. (a) Same as above
3. (a) (CH3 )3 CMgCl + DOD ® (CH 3 )3 CD + MgCl(OD)
21. (b) Peroxide effect (where free-radicals are formed) is
4. (c) observed only in addition of HBr ; addition of HCl is an
ionic reaction and thus CH3+CHCH3 (2º carbocation) is
C2H5 formed as an intermediate.
5. (a) CH – CuLi + BrCH2CH3
C2H5 22. (c) The reaction of HCl with carbon radical in case of HCl,
2
and addition of iodine radical to double bond in case of
Lithium di(3-pentyl) cuprate
HI are endothermic steps
CH2CH3 23. (a) Same as above
CH3CH2CHCH2CH3 + (C2H5)2CHCu + LiBr CH 2 = CH 2 < RCH = CHR < R 2 C = CHR < R 2 C = CR 2
(no+ I group) (Two +I groups) (3 + I groups) (4 + I groups)
3-Ethylpentane
24. (d) Since O is more electronegative than Br, therefore
6. (d) heterolytic fission of HOBr gives HO– & Br+ ions
7. (c) 25. (b)
490 Chemistry
26. (a) Oxymercuration leads to hydration in Markovnikov’s 42. (c) Since the compound X(C5H8) reacts with ammonical
manner AgNO3 to give a white precipitate, it must be a terminal
é OH ù
alkyne, i.e. it should have -C º CH grouping. Further,,
ê |
HgSO4 + H 2SO 4 ú since X gives (CH3)2CHCOOH on oxidation, X must have
CH 3CH 2 C º CH ¾¾ ¾ ¾¾ ¾
¾® ê CH 3CH 2 C = CH 2 ú
ê ú (CH 3 ) 2 CHC º C - group. Thus the compound X is
ë û
(CH 3 ) 2 CHC º CH
® CH 3CH 2 COCH 3 43. (c) Similar to that of above question
CH3 CO 2 + (CH 3 ) 3 C - COOH ¬¾
alk.
¾¾ (CH 3 ) 3 C - C º CH
|
i ) O3 KMnO 4
27. (c) CH 3CH = C - CH = CH 2 ¾¾¾® CH 3CHO +
ii ) Zn / H 2O
[ Ag ( NH ) ]+
CH ¾¾ ¾ ¾3 ¾
2¾® (CH ) C - C º CAg
3 3
CH3 ( white ppt.)
|
O = C - CHO+ CH 2 O 44. (d) p-Dichlorobenzene, being symmetrical, fits closely in the
28. (c) Since the alkene on ozonolysis gives a single compound, crystal lattice and hence will have the highest melting
having two aldehydic groups, it must be cyclic point
45. (c)
46. (c) OC2H5 Cl OH
CH = O CH
(CH2)4 (CH2)4 or
CH = O CH
Cyclohexene

29. (a) 8g Br2 react with 2.8g of alkene Ethoxybenzene Chlorobenzene Phenol
\ 80g Br2 react with 28g alkene
Thus the molecular wt. of alkene is 28 which indicates o-p-directing
C2H4 COOC2H5
30. (a) 31. (d) 32. (b) 33. (b)
é + ..
+ - ù
34. (c) CH 3COCl + ZnCl 2 ® êCH 3 - C = O
. . « CH 3 - C º O ú + ZnCl 3
êë úû
Acylium ion

CH3 Chlorobenzene Ethyl benzoate


+
m-directing
+
CH3– C = CH2 + CH3CO CH3– (CH3)C – CH 2COCH3
o-, p-directing

– CH3 47. (c) Due to strong –R effect, –COOR is deactivating group,


ZnCl 3 while all others are activating groups. Recall that all m-
C – CH2COCH3 CH3 – C – CH2COCH3
(–ZnCl2) directing groups are deactivating groups
Cl 48. (d)
35. (a)
CH COOH –
36. (a) CH º CH ¾¾¾
3 ¾¾® CH = CHOCOCH
2 3 49. (a) O OH
CH COOH
¾¾¾
3 ¾¾® CH CH(OCOCH )
3 3 2
37. (d) 38. (d) 39. (a) >

é OH OH ù
Most electron- Electron releasing
ê
cold alk.
| | ú releasing due to due to –OH group
40. (c) CH 3CH 2 C º CH ¾¾ ¾¾®ê CH 3CH 2 C - C - H ú –ve charge on O
KMnO 4 ê | | ú
ë OH OH û O O

oxidation OH O – C – CH3 C – CH3
® CH 3CH 2 COCHO ¾¾¾¾® CH 3CH 2COOH + CO 2
41. (b) Since both gases decolourise KMnO4 solution, both must
be unsaturated, i.e. alkene or alkyne ( >option b or d). > >
However, only one of them gives a white precipitate with
ammonical AgNO3 solution, it must be terminal alkyne
Electron releasing Less electron releasing due to presence of Electron-
and thus the other should be either a non-terminal alkyne –CO grouping which shows resonance withdrawing group
or an alkene. The datas coincide with the option b with the electrons on O
Hydrocarbons 491

50. (c) CH3 Cl CH3


CH3

> > 57. (b) Br / FeBr Mg / ether


¾¾2¾¾¾

¾¾ ¾ ¾
¾®
Br
Electron-releasing No group Electron-releasing
group as well as electron-
CH 3 CH3
withdrawing group*

NO2
CH3 CH3

> H C = O
¾¾2 ¾ ¾
¾®
MgBr H 3O +
Electron-releasing Strong electrons
CH 2OH
as welll as electron- withdrawing group CH3 CH3

* In C6H5Cl, –Cl exerts electron-releasing effect due to


+R effect, but simultaneously it also exerts –I effect, the CH2Br
result is that the molecule is deactivated due to –I effect
but undergoes o, p-substitution due to powerful +R effect
Method (1) will give
51. (c)
52. (a) Determine the empirical formula of the two compounds CH2Br
CH2Br
æ 12 ö æ1 ö
C : H = ç ´ 100 ÷ : ç ´ 100 ÷ = 92.3 : 7.7 which is allylic bromination.
è 13 ø è 13 ø 58. (a) The reaction results in the formation of one isomer, hence
92.3 7.7 it is stereospecific reaction.
= : = 7.7 : 7.7 = 1 : 1
12 1 ZnCu
59. (a) CH 2 + CH 2 I 2 ¾¾ ¾®
\ The empirical formula of the two compounds is CH. diethyl ether
Thus the molecular formula of A and B will be the simple
multiple of CH, i.e. C2H2, C3H3, C4H4, C6H6, etc. CH2
Thus options (c) and (d) are discarded. Further since A
decolourises bromine water and B does not, A and B CH2
should be C 2 H 2 (acetylene) and C 6 H 6 (benzene)
respectively. 60. (d) H 3C - C º C - H + CH 3CH 2 MgCl ¾
¾®
53. (d) The reaction is hydroboration which is addition of H 2 O
to alkene see anti to Makownikoff's's rule H 3C - C º C -MgCl + CH 3CH 3

54. (b) HgSO H SO


¾¾ ¾4¾ ¾
2 ¾
¾4®
H 2O OH
H
Tautomerisation
¾¾¾¾¾¾
¾®
C º C - CH 3

55. (a) In H 2 C = C = CH 2 hybridisation of C atoms is sp2


and sp. The molecule is monoplanar but H–atoms lie in O
different planes. See allene structure. ||
61. (a) - C - CH 2 - B The ring B has o/
H
56. (d) ¾¾¾
3® BH
p directing and (A) has m-directing group. Hence the
H 2 O 2 / OH -
hydroborat ion
product will be

O
NaH
¾¾¾® ||
CH3I - C - CH 2 - - Br
492 Chemistry

O O
62. (c) CH 2 = CH - C(CH 3 ) = CH - CH 3 ¾¾3¾
® ||
Zn H 2O 72. (c) In the reactions X is CH 3 - CH 2 - CH 2 - C - CH 3

CH 2 O + O = CH - C - CH 3 - CH 3CHO and Y is CH 3CH 2 CH 2 CH 2 CHO X will give +ve


|| iodoform test and Y will give +ve Silver mirror test.
O
73. (c) Both (a) and (b) are correct. Wet CH 3COOH gives cis
For solving such question remove the doubles bonds
and join the carbons to oxygen by double bonds. addition and dry CH 3COOH gives trans addition
· · products.
63. (c) Cl + CH 4 ® CH3Cl + H is not mechanistic step. 74. (c) Trans is more stable, cis is less stable. Hence only (c) is
64. (a) Only Lindlar’s catalyst converts alkyne to alkene (cis correct.
addition) and alkenes with Baeyer’s reagent give cis 75. (c) Peracids gives trans hydroxylated products with
glycols. alkenes.76. (d) Side chain substitution followed by
Wurtz reaction
65. (b) HC º C - CH 2 - CH 2 NH 2 it contains three active
1 2 3 4 5 6 Na / liquid NH
hydrogen atoms (one attached to sp hybridised C-atom 77. (a) CH 3 - C º C - CH 2 - CH 2 CH 3 ¾¾¾¾¾¾
¾3®

and two attached to N). Hence Et.MgBr will give three


moles of C 2 H 6 . CH 3
CH 3 |
H H - C - OH
Br Å
C=C ¾
¾® |
66. (c) CH 2 = CH 2 + Br2 ® CH 2 CH 2 H C3 H 7 H - C - OH
|
67. (a) Two moles of alkene is reduced thus X must be C3H 7

Trans addition Erythroracemic


+ 4H
O Me OMgI
which give 4H atoms needed for alkene. ||
78. (b) MeMgI
68. (a) The given reaction is Birch reduction. ¾¾¾
¾®
69. (a) Bromination in presence of light leads to side chain
| |
substitution. For nuclear substitution we require halogen Cl Cl
carrier.
70. (b) I and II bear mirror image relationship, hence are
enantiomers. O O
|| ||
71. (a) Peracids give epoxides (alkene oxide) with alkene. In (b), Me2CuLi
(c), and (d) the products are wrong. ¾¾¾¾¾¾¾¾
®
Corey House reaction

| Me
Cl
14
Environmental
Chemistry
POLLUTION (iv) Mesosphere : The region of the earth’s atmosphere above
Pollutant :- Any material affecting the life is known as pollutant the stratosphere and below the thermosphere. It is the
and the phenomenon is known as pollution. The pollutants may coldest region (temp. –2 to –92ºC) of atmosphere.
be inorganic, biological or radiological in nature. (v) Exosphere : The uppermost region of atmosphere. It
(i) Primary pollutants : These are emitted directly from the contains atomic and ionic O2, H2 and He.
sources. eg. inorganic gases such as H2S, SO2, CO, NO, HF, (b) Hydrosphere :- The aqueous envelop of the earth eg. oceans,
NH3 radioactive substancs or particulates such as smoke, lakes etc.
ash, dust, fumes. (c) Lithosphere :- The solid rocky portion of the earth.
(ii) Secondary pollutants : These are formed in the atmosphere (d) Biosphere :- The biological envelop which supports the life.
by chemical interactions among primary polluntants eg. SO3, AIR POLLUTION :
NO2, CH4, aldehydes, ketones, nitrates, sulphates, phenols.
(I) Major air pollutants: The major air pollutants are following:-
(iii) Bio-degradable pollutants : These are domestic wastes which
(a) Carbon monoxide (CO) :- It is produced by incomplete
are rapidly decomposed by micro-organisms.
combustion of gasoline in motor vehicles, wood, coal,
(iv) Non-degradable pollutants : These include chemicals,
inceneration and forest fires.
mercuric salts, lead compounds, pesticides etc.
It is treacherous and deadly poisonous gas. It induces
(v) Natural pollution : It is caused by radioactive substances,
headache, visual difficulty coma and death. It blocks
volcanic eruptions, forests and mines fires, floods etc.
the normal transport of oxygen from the lungs to other
(vi) Artificial pollution : It is caused by industries, thermal plants, parts of the body.
automobile exhausts, sewage etc.
(b) Sulphur dioxide ( SO 2 ) :- It is produced by petrol
ENVIRONMENT :
combustion, coal combustion, petrol refining and
The conditions existing around animal or human life. It is further
smelting operations.
classified as
It hinders the movement of air in and out of lungs. It is
(a) Atmosphere :- The gaseous envelop surrounding the earth.
particularly poisonous to trees causing chlorosis and
(i) Stratosphere : The layer of the earth’s atmosphere above
the troposphere and below the mesosphere. dwarfing. In presence of air it is oxidised to SO 3 which
(ii) Troposphere : The lowest region of the atmosphere is also irritant.
extending from earth’s surface to the lower boundary of
2 SO 2 + O 2 (air) ¾
¾® 2 SO 3
the stratosphere. It contains water vapour and is greatly
affected by air pollution. In presence of moisture SO 3 is converted into highly
(iii) Thermosphere : The upper region of the atmosphere corrosive sulphuric acid.
above the mesosphere. It is the hottest region (temp
upto 1200ºC). SO 3 + H 2 O (moisture ) ¾
¾® H 2SO 4
494 Chemistry
It attacks marble, limestone,vegetation, paper and textiles Hydrocarbons + O2, NO2, NO, O, O3 ® Peroxides,
and injurious to human beings. formaldehyde, peroxyacetylnitrate (PAN), acrolein etc.
(c) Oxides of nitrogen :- NO 2 and NO It is oxidising in nature and causes irritation to eyes,
lungs, nose, asthamatic attack and damage plants.
(Source - combustion of coal, gasoline, natural gas,
(IV) Acid rain :- The oxides of C, N and S present in the atmosphere,
petroleum refining, chemical plants manufacturing
dissolve in water and produce acids and lower the pH of
explosives and fertilizers, tobacco smoke.
water to below 5.5.
Breathing NO 2 causes chlorosis to plants and
2-
H 2 O + CO 2 ¾
¾® H 2 CO 3 2 H + + CO3
chronic lung conditions leading to death. NO 2 reacts
with moisture to form acids. 2 H 2 O + 2 SO 2 + O 2 ® 2 H 2SO 4

2 NO 2 + H 2 O ¾
¾® HNO 2 + HNO 3 2 H + + SO 24 -

3 HNO 2 ¾ ¾® 2 NO + HNO 3 + H 2 O -
2 H 2 O + 4 NO 2 + O 2 ® 4 HNO 3 4H + + 4NO3
(d) Smoke, dust :-
The acids are toxic to vegetation, react with marble and
Sources foundries, cement works, iron and steel works,
damage buildings.
gas works, power generating stations.
(e) Ammonia : Ammonia works CaCO 3 + H 2SO 4 ¾
¾® CaSO 4 + H 2 O + CO 2
(f) Chlorine and hydrogen chloride : Chlorine works Acids corrode water pipes and produce salts with heavy
(g) Chlorinated hydrocarbons : Dry cleaning works metals ions viz Cu, Pb, Hg and Al toxic in nature.
(h) Mercaptans : Oil refineries, coke ovens (V) Green House effect :- The retention of heat by the earth and
(i) Zn and Cd : Zinc industries atmosphere from the sun and its prevention to escape into
(j) Freon : Refrigeration works. the outer space is known as green house effect. Green house
(II) Photochemical pollutants :- The nitrogen dioxide by gases such as CO 2 , ozone, methane, the chlorofluoro carbon
absorbing sunlight in blue and U. V. region decomposes into compounds and water vapour form a thick cover around the
nitric oxide and atomic oxygen followed by a series of other earth which prevents the IR rays emitted by the earth to
reactions producing O 3 formaldehyde, acrolein and escape. It gradually leads to increase in temperature of
peroxyacylnitrates. atmosphere.
Ozone causes bronchial irritation even at 1 ppm level. Ozone Consequences of green house effect
affects tobacco plants, spinach, tomato, potato etc. The (i) Global warming would result in rise in sea level due to
photochemical pollutants are powerful eye irritants. The increased rate of melting of glaciers and floods.
reactions are as follows (ii) Increase in infectious diseases like malaria, dengue etc.
(VI) Ozone layer and its depletion :- The ozone layer, existing
NO 2 + h u ¾
¾® NO + O, O + O2 ¾
¾® O 3
between 20 to 35 km above the earth’s surface, shield the
earth from the harmful U. V. radiations from the sun. The U. V.
¾® RO · ,
RH + O ¾ ¾® RO ·3
RO · + O 2 ¾
radiations cause skin cancer, cataract of eye, and harmful to
RO ·3 + RH ¾
¾® Aldehydes + Ketones vegetation.
Depletion of ozone is caused by oxides of nitrogen
RO ·3 + NO ¾
¾® RO ·2 + NO 2 N 2O + h u ¾ ¾® NO + N
reactive nitric oxide
RO ·2 + NO 2 ¾
¾® Peroxyacylnitrate
NO + O 3 ¾
¾® NO 2 + O 2
(III) Smog : It is a mixture of smoke (composed to tiny particles of
carbon, ash and oil etc. from coal combustion) and fog in O3 + h u ¾
¾® O 2 + O
suspended droplet form. It is of two types :
(a) London smog or classical smog : It is coal smoke plus NO 2 + O ¾
¾® NO + O 2
fog. The fog part is mainly SO2 and SO3. It has sulphuric 2 O3 + h u ¾
¾® 3 O 2 (Net reaction)
acid aerosol. It causes bronchial irritation and acid rain.
The presence of oxides of nitrogen increase the
It is reducing in nature.
decomposition of O 3 . Depletion of ozone by chlorofluoro
(b) Photochemical smog or Los Angeles smog : The
oxidised hydrocarbons and ozone in presence of carbons.
humidity cause photochemical smog.
Environmental Chemistry 495
(iii) Sulphates : Sulphates of Na, K, Mg cause diarrihoea
C F2 Cl 2 + h u ¾
¾® CF2 Cl + Cl
(iv) Sodium chloride : It imparts bad taste to water
CFCl 3 + h u ¾
¾® CFCl 2 + Cl (v) Iron and manganese : Stain fabrics, bad taste, modify
colours
Cl + O 3 ¾
¾® ClO + O2
(vi) Lead : It damages kidney, liver, brain and central nervous
ClO + O ¾
¾® Cl + O 2 system
(vii) Cadmium and Mercury : Cause kidney damage
O3 + O ¾
¾® 2 O 2 (Net reaction)
(viii) Zn : Causes vomitting, dizziness and diarrhoea
(VII) Control of air pollution :- It can be controlled by
(ix) Arsenic can cause cramps, paralysis and death
1. Dissolving HCl, HF, SiF4 in water and SO 2 , Cl 2 , H 2S (x) Phosphates from fertilizers : They promote algae growth
in alkaline solution. and reduce D.O. concentration of water. This process is
2. Adsorbing gas and liquids molecules over activated known as eutrophication.
charcoal and silica gel. (xi) Anionic detergents (eg. alkyl benzene sulphonates, ABS)
3. Chemical reactions : They produce stable foam, stabilise colloidal impurities
and inhibit oxidation of organic compounds like phenol.
2CuO + O 2 + 2 S O 2 ¾
¾® 2 CuSO 4 ABS is not bio-degradable.
(xii) Hydrogen sulphide : Acidic, rotten-egg odour and
2 CaCO 3 + O 2 + 2 SO 2 ¾
¾® 2 CaSO 4 + 2 CO 2
corrosive to metals.
4 NO + 4 NH 3 + O 2 ¾ ¾® 2 N 2 + 6 H 2 O (xiii) Polychlorinated biphenyls : They are resistant to
4. Use of precipitators to settle charge particles oxidation and cause skin disorders and are carcinogenic.
5. Use of settling chambers under the action of gravity (xiv) Acid polluted water (pH < 3) : H2SO4 produced by
oxidation of Iron pyrites (FeS2) harmful to life.
6. Use of natural gas in place of diesel, petrol etc.
Suspended impurites :
WATER POLLUTION
(i) Parasitic worms : They cause infections
A. Water Pollution : The contamination of water by foreign
(ii) Bacterias : Cause dysentery, typhoid, cholera
substances which would constitute a health hazard and make
it harmful for all purposes (domestic, industrial or agriculture (iii) Viruses : Cause entroviral infections
etc.) is known as water pollution. The polluted water may (iv) Algae : Cause foul odour, taste, turbidity
have affensive odour, bad taste, unpleasant colour, murky D. International standards for drinking water :
oily etc. S.No. Characteristics Acceptable Rejection
B. Sources of water pollution : (mg/l) limit limit
(i) Domestic sewage : Discharges from kitchens, baths, 1. pH value 7 - 8.5 6.5 - 9.5
lavatories etc. 2. Total dissolved solids 500 1500
(ii) Industrial waters : Wastes from manufacturing 3. Total hardness (as CaCO3) 200 600
processes which includes acids, alkalies, pesticides, 4. Fluorides 1.0 1.5
insecticides, metals like copper, Zinc, lead, mercury,
5. Chlorides 200 1000
fungicides etc.
6. Sulphates 200 400
(iii) Oil : from oil spills or washings of automobiles
7. Nitrates 45 45
(iv) Atomic explosion and processing of radioactive
8. Magnesium 30 150
materials
9. Calcium 75 200
(v) Suspended particles (organic or inorganic) viruses,
10. Zinc 5.0 15.0
bacterials algae protozoa etc.
11. Anionic detergents 0.2 1.0
(vi) Wastes from fertilizer plants such as phosphates,
nitrates ammonia etc. 12. Iron 0.1 1.0
13. Manganese / Copper 0.05 0.5
(vii) Clay : Ores, minerals, fine particles of soil.
14. Phenolic compounds 0.001 0.002
C. Effects of impurities in water : Dissolved substances
15. Toxic Materials
(i) Hardness : Corrossive effect on boils, alkalinity, laxative
eg. As, Cd, Cr, CN–, Pb, Se 0.05 - 0.001 0.05 - 0.001
effect
(ii) Fluorides : Motting of teeth enamel, nervous and
skeleton disorders, above 1 mg/litre causes fluorosis.
496 Chemistry
AEROBIC AND ANAEROBIC OXIDATION : SEWAGE TREATMENT :
The oxidation of organic compounds present in sewage in The artificial treatment is called sewarage and involves the
presence of good amount of dissolved or free oxygen (approx. 8.5 following steps :
ml/l) by aerobic bacterials is called aerobic oxidation. When (i) Preliminary process : Passing sewage through screens to
dissolved or free oxygen is below a certain value the sewage is remove large suspended matter and then through mesh
called stale anaerobic bacterias bring out purification producing
screens to remove solids, gravels, silt etc.
H2S, NH3, CH4, (NH4)2S etc. This type of oxidation is called
anaerobic oxidation. (ii) Settling process (sedimentation) : The residual water when
The optimum value of D.O. for good quality of water is 4-6 ppm (4- allowed to stand in tanks, the oils and grease float on the
6 mg/l). The lower the concentration of D.O., the more polluted is surface and skimmed off and solids settle down. The colloidal
the water. meterial is removed by adding alum, ferrous sulphate etc.
BIOLOGICAL OXYGEN DEMAND (BOD) : and we get primary sludge.
It is defined as the amount of free oxygen required for biological (iii)Secondary treatment or Biological treatment : It is aerobic
oxidation of the organic matter by aerobic conditions at 20°C for chemical oxidation or aeration which converts carbon of the
a period of five days. Its unit is mg/l or ppm. An average sewage organic matter to CO2, nitrogen into NH3 and finally into
has BOD of 100 to 150 mg/l. nitrites and nitrates, dissolved bases form salts such as
CHEMICAL OXYGEN DEMAND (COD) : NH4NO2, NH4NO3 and Ca(NO3)2 etc. and secondary sludge
It is a measure of all types of oxidisable impurities (biologically is obtained.
oxidisable and biologically inert organic matter such as cellulose) The primary and secondary sludge (combined) is
present in the sewage. COD values are higher than BOD values.
subjected to anaerobic digestion to CO2 and CH4
COD DETERMINATION :
A known volume of sample is refluxed with known volume of 2[CH 2 O] ¾
¾® CO 2 + CH 4
standard K2Cr2O7 + dil. H2SO4 in presence of Ag2SO4 (catalyst)
(iv) Tertiary treatment : It is treatment of waste water with lime
for 1½ hours. The unreacted K2Cr2O7 is then titrated against
FeSO4.(NH4)2SO4.6H2O solution. The oxygen equivalent of for removal of phosphate which is coagulated by adding
K2Cr2O7 consumed is taken as a measure of COD. alum and ferric chloride and removed by filtration. Water is
1 ml. of 1 N K2Cr2O7 º 0.008 g oxygen disinfected by adding chlorine.

SOIL OR LAND POLLUTION : Secondarty sludge forms a good fertilizer for soil as it contains
nitrogen and phosphorous compounds.
The addition of substances in an indefinite proportion changing
the productivity of the soil is known as soil or land pollution. PESTICIDES :
Sources of soil pollution : The chemical substances used to kill or stop the growth of
(i) Agricultural pollutants : Chemicals like pesticides, fertilizers, unwanted organisms are called pesticides. They are further
fumigants, insecticides, herbicides, fungicides. classified as
(ii) Domestic refuge and industrial wastes (a) Insecticides : They are used to kill insects. The most common
(iii) Radioactive wastes from research centres, and hospitals insecticides are
(iv) Soil conditioners containing toxic metals like Hg, Pb, As, Cd
Cl
etc. Cl
Cl
(v) Farm wastes from poultries, dairies and piggery forms Cl
(vi) Improper disposal of human and animal excreta. (i) Cl3C.CH (ii)
Cl Cl
(vii) Pollutants present in air from chemical works Cl Cl
CONTROL OF WATER POLLUTION : BHC, 666, gamexane
D.D.T.
The water pollution can be reduced by following techniques OCONHCH3
(i) Recycling of waste water : by aeration and use of trickling OCONHCH3
filter (iii) CH(CH3)2 (iv)
(ii) Use of chemicals : Effective filtration and chlorination
Sevin, Carbaryl
(iii) Special techniques : Such as adsorption, ion-exchangers, Baygon
reverse osomosis, electrodialysis etc. S
||
(iv) Waste-water reclamation : Sewage water can be directly (v) (C 2 H 5 ) 2 - P - O - NO
used for irrigation and fish farms. Since it contains N, P and K.
Parathion
Environmental Chemistry 497
(c)Fungicides : They are used to stop or kill fungus e.g.
OCH3
OH
Cl Cl

(vi) Cl3C.CH CuSO 4


(i) copper sulphate (ii)
Cl
2,4,6-trichlorophenol

OCH3 (d) Rodenticides : They are used to kill rodents e.g.


Methoxychlor
Zn 3 P2 Tl.SO 4
Cl (i) Zincphosph ide
(ii) Thalium sulphate

(vii) Aldrin Cl (iii) ClCH 2 COONa


CCl2 CH2 Sodium monochloro acetate
Cl
Cl
(b) Herbicides : They are used to kill weeds e.g.
O.CH2COOH
Cl

(i) (ii) Triazines


Cl
2,4-dichlorophenory acetic acid (2,4D)

(iii) NaClO 3 (iv) Na 3 AsO 3


Sodium chlorate Sodium arsenite
The (iii) and (iv) are not used now a days.
498 Chemistry

Very Short/ Short Answer Questions 19. Which of the following is not a consequence of green-
house effect?
1. In what regions of the atmosphere, the temperature (a) Climatic conditions will be changed resulting in
increases with altitude and in which regions it decreases? (b) Plants in warmer climates with adequate rainfall would
2. What is the most important sink of CO pollutant? grow faster
3. What is marine pollution? (c) The incidence of infectious diseases is likely to
increase
4. Why COD is preferred over BOD ?
(d) Malaria will be controlled as the mos
5. What should be the tolerable limit of fluoride ions in drinking quitoes will not survive.
water? What happens if it is higher than10 ppm ? 20. Which of the following statements is not true about classi-
6. Name any four methods for waste management. cal smog?
7. Write the name of gas produced in Mathura refineries which (a) Its main components are produced by the action of
can damage the great historical monument “Taj Mahal”? sunlight on emissions of automobiles and factories.
8. What are 'asbestosis' and 'silicosis'? (b) Produced in cold and humid climate.
9. (i) Why there is ozone depletion mainly over Antarctica? (c) It contains compounds of reducing nature.
(ii) In which season the depletion of ozone on Antarctica (d) It contains smoke fog and sulphur dioxide
takes place and when is it replenished? 21. Which of the following statements is wrong?
10. Depending upon the nature of pollutants, how can the (a) Ozone is not responsible for green house effect.
pollution be classified into different types ? (b) Ozone can oxidise sulphur dioxide present in the atmo-
sphere to sulphur trioxide.
11. How plant nutrients and pesticides acts as water pollutants?
(c) Ozone hole is thinning of ozone layer present in strato-
12. Fish do not always grow in warm as well as in cold water, sphere.
why?
(d) Ozone is produced in upper stratosphere by the action
13. On the basis of chemical reactions involved, explain how of UV rays on oxygen.
do chlorofluorocarbons cause thinning of ozone layer in 22. Which of the following statements is correct?
stratosphere.
(a) Ozone hole is a hole formed in stratosphere from which
14. How can lead poisoning be cured? ozone oozes out.
(b) Ozone hole is a hole formed in the troposphere from
Long Answer Questions which ozone oozes out.
15. What is the strategy for control of Environmental pollution? (c) Ozone hole is thinning of ozone layer of stratosphere
16. List industrial wastes which contaminate water. at some places.
Multiple Choice Questions (d) Ozone hole means vanishing of ozone layer around
the earth completely.
17. Dinitrogen and dioxygen are main constituents of air but
these do not react with each other to form oxides of nitrogen 23. Which among the following statements is false?
because __________. (a) Oil slick in sea water increases D.O. value.
(a) the reaction is endothermic and requires very high (b) The main reason for river water pollution is industrial
temperature. and domestic sewage discharge.
(b) the reaction can be initiated only in presence of a (c) Surface water contains a lot of organic matter mineral
catalyst. nutrients and radioactive materials.
(c) oxides of nitrogen are unstable. (d) Oil spill in sea water causes heavy damage to fishery.
(d) N2 and O2 are unreactive 24. Sewage containing organic waste should not be disposed
18. The pollutants which came directly in the air from sources in water bodies because it causes major water pollution.
are called primary pollutants. Primary pollutants are Fishes in such a polluted water die because of
sometimes converted into secondary pollutants. Which of (a) large number of mosquitoes
the following belongs to secondary air pollutants?
(b) increase in the amount of dissolved oxygen
(a) C O
(b) Hydrocarbon (c) decrease in the amount of dissolved oxygen in water
(c) Peroxyacetyl nitrate (d) clogging of gills by mud.
(d) NO
Environmental Chemistry 499
25. Which of the following practices will not come under green 26. Water is often treated with chlorine to
chemistry? (a) remove hardness
(a) If possible, making use of soap made of vegetable oils (b) increase oxygen content
instead of using synthetic detergents. (c) kill germs
(b) Using H2O2 for bleaching purpose instead of using (d) remove suspended particles
chlorine based bleaching agents.
(c) Using bicycle for travelling small distances instead of
using petrol/ diesel based vehicles.
(d) Using plastic cans for neatly storing substances.

1. The uppermost region of the atmosphere is called 11. Formation of photochemical smog takes place in
(a) ionosphere (b) stratosphere (a) winter during day time
(c) troposphere (d) exosphere (b) summer during morning time
2. Which of the following is the coldest region of atmosphere? (c) summer during day time
(a) thermosphere (b) mesosphere (d) winter during morning time
(c) troposphere (d) stratosphere 12. Which of the following statement is false?
3. The region which is greatly affected by air pollution is (a) London smog is oxidising in nature
(a) thermosphere (b) stratosphere (b) Photochemical smog causes irritation in eyes
(c) troposphere (d) mesosphere (c) London smog is a mixture of smoke and fog
4. The region containing water vapour is (d) Photochemical smog results in the formation of PAN
(a) thermosphere (b) stratosphere
13. The viable particulate among the following is
(c) troposphere (d) mesosphere
(a) fumes (b) algae
5. In which of the following regions hydrogen and helium are
found (c) smoke (d) mist
(a) stratosphere (b) mesosphere 14. The non-viable particulate among the following is
(c) exosphere (d) troposphere (a) dust (b) bacteria
6. The substance which is a primary pollutant? (c) moulds (d) fungi
(a) H2SO4 (b) CO 15. Which of the following acts as a sink for CO?
(c) PAN (d) Aldehydes (a) Plants
7. Which of the following is most abundant hydrocarbon (b) Haemoglobin
pollutant? (c) Microorganisms present in the soil
(a) Butane (b) Ethane
(d) Oceans
(c) Methane (d) Propane
16. Which forms the part of hazy fumes of photochemical smog?
8. The aromatic compounds present as particulates are
(a) SO2 (b) Nitrogen dioxide
(a) polycyclic aromatic hydrocarbons
(b) benzene (c) PAN formation (d) Aldehydes
(c) toluene 17. Depletion of ozone layer causes
(d) nitrobenzene (a) breast cancer (b) blood cancer
9. The biggest particulate matter is (c) lung cancer (d) skin cancer
(a) HNO3 droplets (b) soot 18. The gas responsible for ozone depletion :
(c) H2SO4 droplets (d) fly ash (a) NO and freons (b) SO2
10. Formation of London smog takes place in (c) CO2 (d) CO
(a) winter during day time 19. The reaction caused by U.V. radiations from sun produces
(b) summer during day time (a) ozone (b) carbon monoxide
(c) summer during morning time
(c) sulphur dioxide (d) fluorides
(d) winter during morning time
500 Chemistry
20. Which of the following chemical, harmful to ozone, is 28. Water pollution is caused by
released by chlorofluoro carbon? (a) pesticides (b) fly ash
(a) Sulphur dioxide (b) Fluorine (c) auto exhausts (d) aeroplanes
(c) Chlorine (d) Nitrogen dioxide 29. Which causes death of fishes in water bodies polluted by
sewage?
21. Ozone hole refers to
(a) Foul smell (b) Pathogens
(a) increase in concentration of ozone
(c) Clogging of gills by silt (d) Decrease in D.O.
(b) hole in ozone layer 30. Phosphate pollution is caused by
(c) reduction in thickness of ozone layer in troposphere (a) sewage and agricultural fertilizers
(d) reduction in thickness of ozone layer in stratsophere (b) weathering of phosphate rocks only
22. Ozone layer of stratosphere requires protection from (c) agricultural fertilizers only
indiscriminate use of (d) phosphate rocks and sewage
(a) baloons 31. Sewage water is purified by
(b) pesticides (a) aquatic plants (b) micro-oganisms
(c) atomic explosions (c) light (d) fishes
(d) aerosols and high flying jets 32. Water is often treated with chlorine to
(a) remove hardness
23. Phosphate fertilizers when added to water leads to
(b) increase oxygen content
(a) increased growth of decomposers
(c) kill germs
(b) reduced algal growth
(d) remove suspended particles
(c) increased algal growth
33. Ozone is an important constituent of stratosphere because it
(d) nutrient enrichment (eutrophication)
(a) destroys bacteria which are harmful to human life
24. The type of pollution caused by spraying of DDT
(b) prevents the formation of smog over large cities
(a) air and soil (b) air and water (c) removes poisonous gases of the atmosphere by reacting
(c) air (d) air, water and soil with them
25. Pollution in large cities can be checked only by (d) absorbs ultraviolet radiation which is harmful to human
(a) shifting of factories out of the residential area life
(b) less use of insecticides 34. Which is the major air pollutant
(c) proper disposal of organic wastes, sewage and industrial (a) He (b) O2
effluents (c) CO2 (d) CO
(d) All the above 35. The ozone layer is present in
26. Sewage mostly constitutes (a) stratosphere (b) troposphere
(a) non-biodegradable pollutants (c) thermosphere (d) mesosphere
(b) biodegradable pollutants 36. The green house effect is caused by
(c) effluents (a) CO2 (b) NO2
(d) air pollutants (c) NO (d) CO
27. Chief source of soil and water pollution is 37. Surface water contains
(a) mining (b) agro industry (a) suspended impurity (b) salt + organic matter
(c) thermal power plant (d) All the above (c) only salt (d) organic matter
Environmental Chemistry 501

1. The greenhouse effect is because of the 7. When rain is accompanied by a thunderstorm, the collected
[CBSE-PMT 1996] rain water will have a pH value
[AIEEE 2003]
(a) presence of gases, which in general are strong infrared (a) slightly lower than that of rain water without
absorbers, in the atmosphere thunderstorm
(b) presence of CO2 only in the atmosphere (b) slightly higher than that when the thunderstorm is not
(c) pressure of O3 and CH4 in the atmosphere there
(d) N2O and chlorofluorohydrocarbons in the atmosphere (c) uninfluenced by occurrence of thunderstorm
2. Which of the following is/are the hazardous pollutant(s) (d) which depends upon the amount of dust in air
present in automobile exhaust gases? [CBSE-PMT 1996] 8. The smog is essentially caused by the presence of
(i) N2 (ii) CO (a) Oxides of sulphur and nitrogen [AIEEE 2004]
(iii) CH4 (iv) Oxides of nitrogen (b) O2 and N2
(a) (ii) and (iii) (b) (i) and (ii)
(c) O2 and O3
(c) (ii) and (iv) (d) (i) and (iii)
(d) O3 and N2
3. Green chemistry means such reactions which :
9. Identify the wrong statement in the following: [AIEEE 2008]
(a) produce colour during reactions [CBSE-PMT 2008]
(b) reduce the use and production of hazardous chemicals (a) Chlorofluorocarbons are responsible for ozone layer
depletion
(c) are related to the depletion of ozone layer
(d) study the reactions in plants (b) Greenhouse effect is responsible for global warming
4. Which one of the following statement is not true ? (c) Ozone layer does not permit infrared radiation from the
[CBSE-PMT 2011] sun to reach the earth
(a) pH of drinking water should be between 5.5 – 9.5. (d) Acid rain is mostly because of oxides of nitrogen and
(b) Concentration of DO below 6 ppm is good for the sulphur
growth of fish. 10. Identify the incorrect statement from the following :
(c) Clean water would have a BOD value of less than 5 [AIEEE 2011RS]
ppm. (a) Ozone absorbs the intense ultraviolet radiation of the
(d) Oxides of sulphur, nitrogen and carbon are the most sun.
widespread air pollutant.
(b) Depletion of ozone layer is because of its chemical
5. Which one of the following statements regarding reactions with chlorofluoro alkanes.
photochemical smog is not correct? [CBSE-PMT 2012]
(c) Ozone absorbs infrared radiation.
(a) Carbon monoxide does not play any role in
(d) Oxides of nitrogen in the atmosphere can cause the
photochemical smog formation.
depletion of ozone layer.
(b) Photochemical smog is an oxidising agent in character.
11. What is DDT among the following ? [AIEEE 2012]
(c) Photochemical smog is formed through photochemical
(a) Greenhouse gas
reaction involving solar energy.
(b) A fertilizer
(d) Photochemical smog does not cause irritation in eyes
and throat. (c) Biodegradable pollutant
6. Frequent occurrence of water blooms in a lake indicates (d) Non-biodegradable pollutant
(a) nutrient deficiency [AIEEE 2003] 12. The gas leaked from a storage tank of the Union Carbide
plant in Bhopal gas tragedy was : [JEE M 2013]
(b) oxygen deficiency
(a) Methyl isocyanate (b) Methylamine
(c) excessive nutrient availability
(c) Ammonia (d) Phosgene
(d) absence of herbivores in the lake
502 Chemistry

1. The substance which is not regarded as a pollutant? 9. Minamata disease is due to pollution of
(a) NO2 (b) CO2 (a) aresenic into the atmosphere
(c) O3 (d) Hydrocarbons (b) organic waste into drinking water
2. The greatest affinity for haemoglobin is shown by which of (c) oil spill in water
the following : (d) industrial waste mercury into fishing water
(a) NO (b) CO 10. BOD is connected with
(c) O2 (d) CO2 (a) microbes and organic matter
3. The substance having the largest concentration in acid rain? (b) organic matter
(a) H2CO3 (b) HNO3 (c) microbes
(c) HCl (d) H2SO4 (d) None of the above
4. Which of the following is not involved in the formation of 11. Eutrophication causes reduction in
photochemical smog? (a) dissolved oxygen (b) nutrients
(a) Hydrocarbon (b) NO (c) dissolved salts (d) All the above
(c) SO2 (d) O3 12. Which among the following statements is false?
5. The statement which is not true (a) Oil slick in sea water increases D.O. value
(a) NO2 does not play any role in photochemical smog (b) The main reason for river water pollution is industrial
(b) SO3 is more harmful air polluntant than SO2 and domestic sewage discharge
(c) SO2 dos not affect larynx (voice box) (c) Surface water contains a lot of organic matter mineral
(d) NO is more toxic to living tissues than NO2 nutrients and radioactive materials
6. The false statement among the followings : (d) Oil spill in sea water causes heavy damage to fishery
(a) The average residence time of NO is one month 13. Presence of which of the following fuel gas in the exhaust
(b) Limestone acts as a sink for SOx fumes shows incomplete combustion of fuel?
(c) SOx can be removed from flue gases by passing through (a) Sulphur dioxide
a solution of citrate ions (b) Carbon monoxide and water vapour
(d) Ammonia acts as a sink for NOx (c) Carbon monoxide
7. The statement which is not correct about control of (d) Nitrogen dioxide
particulate pollution 14. Which one of the following statements about ozone and
(a) In electrostatic precipitator, the particulates are made to ozone layer is true?
acquire positive charge which are then attracted by the (a) Ozone layer is beneficial to us because ozone cuts out
negative electrode and removed the ultraviolet radiation of the sun
(b) Gravity settling chamber removes larger particles from (b) The conversion of ozone to oxygen is an endothermic
the air reaction
(c) Cyclone collector removes fine particles in the diameter (c) Ozone has a triatomic linear molecule
range 5-20 microns (d) None of the above
(d) Wet scrubbers are used to wash away all types of
particulates
8. Which of the following statements about polar stratosphere
clouds (PSCs) is not correct?
(a) PSCs do not react with chlorine nitrate and HCl
(b) Type I clouds are formed at about –77ºC and contain
solid HNO3 . 3H2O
(c) Type II clouds are formed at about –85ºC and contain
some ice
(d) A tight whirlpool of wind called Polar Vortex is formed
which surrounds Antarctica
Environmental Chemistry 503

EXERCISE 1 hn •
20. (c) CF2 Cl 2 ¾¾® CF2 Cl + Cl
1. Temperature increases in stratosphere and thermosphere,
• •
it decreases in troposphere and mesosphere. Cl + O 3 ¾
¾® Cl O + O 2
2. Soil micro–organism.
21. (d) Ozone hole is reduction in ozone layer in stratosphere.
3. Pollution of sea water due to discharge of wastes into it. 22. (d) Aerosols use CFC an flying jets release NO. Both causes
4. COD can be found in a few minutes whereas BOD requires depletion of ozone layer.
at least 5 days. 23. (d) Addition of phosphate fertilizers to water leads to
5. 1 ppm or 1 mg dm–3. Higher concentration is harmful to nutrient enrichment (eutrophication)
bones and teeth. 24. (d) DDT causes water, air and soil pollution.
6. Recycling, burning incineration and sewage treatment. 25. (d)
7. Sulphur dioxide 26. (c) Domestic sewage constitute biodegradable pollutants.
8. The lung disease caused by particulates of asbestos is 27. (d)
'asbestosis' and that caused by silica is 'silicosis'. 28. (a) Pesticides cause water pollution.
29. (d) Decrease in D.O causes death of fish
14. By feeding the patient with an aqueous solution of calcium
complex of EDTA. 30. (a) Phosphate pollution is caused by sewage and
agricultural fertilizers.
17. (a) 18. (c) 19. (d) 20. (a) 21. (a)
31. (b) Sewage water is purified by micro-organisms.
22. (c) 23. (a) 24. (c) 25. (d) 26. (c)
32. (c) Water is often treated with Cl2 to kill germs.
EXERCISE 2 33. (d) Ozone absorbs U.V. radiations harmful to human life.
1. (d) The uppermost region of atmosphere is exosphere. 34. (d) CO is major air pollutant.
2. (b) The coldest region is mesosphere 35. (a) Ozone layer is present in stratosphere.
(temp. –27ºC to –92ºC) 36. (a) CO2 causes green house effect.
3. (c) Air pollution greatly affect the troposphere. 37. (b) Surface water contains salts + organic matter.
4. (c) Troposphere contains water vapour. EXERCISE 3
5. (c) H2, He and ionic oxygen are present in exosphere. 1. (a) Green house gases such as CO2, ozone, methane, the
6. (b) CO is primary pollutant. chlorofluorocarbon compounds and water vapour form
7. (c) Most abundant hydrocarbon pollutant is methane. a thick cover around the earth which prevents the IR
8. (a) PAH (Poly Aromatic Hydrocarbon) rays emitted by the earth to escape. It gradually leads
to increase in temperature of atmosphere.
9. (d) Fly ash.
2. (c) CO and oxides of Nitrogen are poisnous gases present
10. (d) London smog is formed in morning during winter.
in automobile exhaust gases.
11. (b) Photochemical smog is formed in noon in summer.
3. (b) Green chemistry may be defined as the programme of
12. (a) London smog is reducing in nature. developing new chemical products and chemical
13. (b) Algae processes or making improvements in the already
14. (a) Dust existing compounds and processes so as to make less
15. (c) CO is converted into CO2 by microorganism present in harmful to human health and environment. This means
soil. the same as to reduce the use and production of
16. (b) NO2 forms a part of photochemical smog. hazardous chemicals.
i.e. correct answer is option (b).
17. (d) Depletion of ozone layer causes skin cancer.
18. (a) NO and freons are responsible for ozone depletion. 4. (b) The ideal value of D.O for growth of fishes is 8 mg/ l .
7mg /l is desirable range, below this value fishes get
¾® O + O ; O + O 2 ¾
19. (a) O 2 ¾ ¾® O 3 susceptible to disease. A value of 2 mg/ l or below is
lethal for fishes.
504 Chemistry

5. (d) The oxidised hydrocarbons and ozone in presence of 11. (d) DDT is a non-biodegradable pollutant.
humidity cause photochemical smog. 12. (a) Methyl isocyanate, CH3 – N = C = O
Hydrocarbons + O2, NO2, NO, O, O3 ® Peroxides, EXERCISE 4
formaldehyde, peroxyacetylnitrate (PAN), acrolein etc.
1. (b) CO2 is generally not regarded as an pollutant.
It is oxidising in nature and causes irritation to eyes,
lungs, nose, asthamatic attack and damage plants. 2. (a) Haemoglobin has great affinity for NO.
6. (b) 3. (d) Acid rain contains H2SO4 > HNO3> HCl.
4. (c) Photochemical smog does not involve SO2.
7. (a) Normal rain water has pH 5.6 Thunderstorm results in
the formation NO and HNO3 which lowers the pH. 5. (b) SO3 is more harmful pollutant than SO2.
8. (a) Smog is caused by oxides of sulphur and nitrogen. 6. (a) The average residence time of NO is 4 days.
9. (c) Ozone layer acts as a shield and does not allow 7. (a) Particulates acquire negative charge and are attracted
ultraviolet radiation from sun to reach earth. It does by the positive electrode.
not prevent infra-red radiation from sun to reach earth. 8. (a) PSCs react with chlorine nitrate and HCl to give HOCl
Thus option (c) is wrong statement and so it is the and Cl2.
correct answer. 9. (d) Minamata is caused by Hg poisoning.
10. (c) The ozone layer, existing between 20 to 35 km above 10. (a) BOD is connected with microbes and organic matter
the earth’s surface, shield the earth from the harmful 11. (a) Eutrophication causes reduction in D.O
U. V. radiations from the sun.
12. (a) Oil slick in sea water decreases D.O value.
Depletion of ozone is caused by oxides of nitrogen
13. (c) Presence of CO in the exhaust fumes shows incomplete
N 2O + h u ¾ ¾® NO + N
combustion.
reactive nitric oxide
14. (a) Statement (a) is correct.
NO + O 3 ¾
¾® NO 2 + O2
qqq
O3 + h u ¾
¾® O 2 + O
NO 2 + O ¾
¾® NO + O 2
2 O3 + h u ¾
¾® 3 O 2 (Net reaction)
The presence of oxides of nitrogen increase the
decomposition of O3.
15
The Solid State

SOLIDS : TYPES OF CRYSTALLINE SOLIDS :


Solid substances are characterised by definite shape, volume, Character Ionic Covalent Molecular Metallic
rigidity, high density, low compressibility, mechanical strength, Constituent Positive and Atoms Molecules Positive ions
slow diffusion and hard nature. The constituent particles (atoms, particles negative ions

molecules or ions) are closely packed and held together by strong Binding Electrostatic Shared Vander Waals' Electrostatic
forces attraction electrons Dipole-dipole between +ve
forces. ions and
negative
CRYSTALLINE SOLIDS : electrons
The constituent particles are arranged in definite geometrical Properties High melting Very high Low mpt Moderate to
pattern i.e. they have lattice structure. points mpt high mpt.
Hard & brittle Very hard Very soft Hard & Soft
AMORPHOUS SOLIDS : Semi Non Insulators good
They have random arrangement of the constituent particles Conductors Conductors Conductors

(amorphous - means without any form). Examples NaCl, CaF2 Diamond, H2O, CO2 Cu, Fe
Silica
DIF FERENCES BETWEEN CRYSTALLINE AND
CRYSTAL LATTICE :
AMORPHOUS SOLIDS ARE :
The three dimensional orderly arrangement of constituent particles
Crystalline Amorphous of a crystalline solid is known as Crystal lattice.
• Have lattice structure • Do not have lattice structure UNIT CELL :
• Have sharp melting point • No sharp mpt The smallest geometrical portion of the crystal lattice which can
be used as repetitive unit to build up the whole crystal is called
• They are anisotropic i.e. • They are isotropic i.e. their
unit cell.
their physical properties are physical properties are the
different along different same along all directions TYPES OF UNIT CELL :
(i) Simple or primitive : Unit cell in which the particles are
directions
present at the corners only
• They are symmetrical • They are non symmetrical.

They are supercooled liquids

• Show clean cleavage • No clean cleavage

Examples: Diamond, NaCl, etc. Examples: Glass, Rubber, Plastic


506 Chemistry
(ii) Face centred : In which the particles are present at the corners a
& at the centre of each of six faces. Atoms touch each other along edges. Hence r =
2
( r = radius of atom and a = edge length)
4
pr
Therefore P.F. = 3 3 = 0.52
( 2r )
(ii) Face centred cubic unit cell : Number of atoms per unit cell
(iii) Body centred : In which the particles are present at the corners
=4
and in the centre.
G B
A
H

(iv) End face centred : In which the particles are present at the F
corners and at the centre of two opposite faces.
E D
Atoms touch each other along the face diagonal. Hence r =
2a
(length of face diagonal = 2a )
4

NUMBER OF PARTICLES PER UNIT CELL : 4


4 ´ pr 3
Therefore P.F. = 3 = 0.74
Unit Cell No. of particles and their Total 3
æ 4r ö
contribution çç ÷÷
Corner Face Centre è 2ø
(iii) Body centred cubic unit cell : Number of atoms per unit cell
1 = 2.
1. Simple Cubic 8× – – 1
8 G B
1 1
2. Face Centred 8× 6× – 4 H A
8 2
1
3. Body Centred 8× – 1 2 c a
8

1 1 F C
4. End face Centred 8 × 2× – 2 b
8 2 a
E a D
PACKING FRACTION : Atoms touch each other along the body diagonal. Hence
It is defined as the ratio of the volume of the unit cell that is
occupied by the spheres to the volume of the unit cell. 3a
r= (length of body diagonal = 3 a )
(i) Primitive cubic unit cell : Number of atoms per unit 4
cell = 1
4
G 2 ´ pr 3
B 3 = 0.68
Therefore P.F. = 3
æ 4r ö
H A çç ÷÷
è 3ø
(iv) Hexagonal primitive unit cell : Each corner atom is being
F shared by 6 other unit cells therefore their contribution to one
C
1
E D unit cell would be .
6
The Solid State 507

1 1 MILLER INDICES :
Total number of atoms per unit cell ´ 12 + 3 + ´ 2 = 6 Reciprocals of Weiss indices and multiplying throughout by
6 2
smallest number to make integers are known Miller indices and
a = 2r represented by h, k and l and the plane as (hkl). For example, in
(112) plane h = 1, k = 1 and l = 2.
The distance between the parallel planes in a crystal is designated
as dhkl. For different cubic lattices the interplanar distances are
C a
given by d hkl =
h + k 2 + l2
2

where a = length of cube side and h, k and l are Miller indices.


2 CRYSTALLOGRAPHIC AXES :
Height of unit cell C = 4r The three mutually perpendicular lines meeting at a common point
3
Length of unit cell a = 2r are called crystallographic axes. The angles between the axes
Area of base = area of six equilateral triangles opposite to the intercepts a, b, c on crystallographic axes are
represented by a, b and g. If axes coincide with or are parallel to
æ 3ö the edges of crystal faces, then a, b, g will be the same as interfacial
= 6 ´ çç 4 ÷÷(2r)
2

è ø angles.

æ 3 ö MORE ABOUT THE LAW OF RATIONAL INDICES :


2
Volume of unit cell = 6 ´ çç (2r ) 2 ÷ ´ 4r
÷ Consider a cubic unit cell, crystallographic axes, the intercepts,
è 4 ø 3
the faces represented as follows :
4 X, Y and Z are crystallographic axes meeting at O. Let LMN be a
6 ´ pr 3
3 unit plane having intercepts a, b and c on X, Y and Z axis.
PF = = 0.74
3 2
6´ ( 2r ) 2 ´ 4 r Z
4 3 B
C
SIZE AND SHAPE OF CRYSTALS :
The slower the rate of formation of crystal, the bigger is the size of A D
L
crystal and vice versa.
c b
MICRO CRYSTALLINE : Y
a O M
Crystals can be seen only under powerful microscope.
INTERFACIAL ANGLES : H N E
The angle between any two faces is called an interfacial angle.
Interfacial angle remains the same in the same substance although G F
the size of faces or even shape of crystal may vary. X
LAW OF RATIONALITY OF INDICES OR INTERCEPTS: (1) Intercepts of axis X :
The ratio between intercepts on crystallographic axes for the (i) On face ADFG will be multiple of a
different faces of a crystal can always be expressed by rational (ii) On face BCEH will be multiple of 'a' with negative sign
numbers. since it is in opposite direction (negative value is
Z represented by putting a bar over it)
(iii) On the faces ABHG and DCEF will be ¥ since X-axis
N can never cut these faces.
(2) Intercepts of axis Y :
Cc (i) On face DCEF will be multiple of b
b (ii) On face ABHG will be multiple of b with negative sign
a Y
O B M since in opposite direction
A (iii) On faces ABCD and EFGH will be ¥ since Y can never
cut these faces
X L
The plane LMN has intercepts OL, OM and ON along X, Y and Z (3) Intercepts of axis Z :
axes of lengths 2a, 4b and 3c respectively. When OA = a, OB = b (i) On face ABCD will be multiple of C.
and OC = c are taken as unit distances, the intercepts are in the (ii) On face EFGH will be multiple of C with negative sign
ratio 2a : 4b : 3c and coefficients of a, b and c are known as WEISS since in opposie direction.
INDICES. (iii) On faces ABHG and DCEF will ¥ since Z axis can never
Weiss indices may have any values integers, fractional or infinity. cut these faces.
508 Chemistry
CRYSTALLOGRAPHY AND X-RAY DIFFRACTION Now for maximum reflection condition (defined above)
(BRAGG'S EQUATION) : path difference = n × wavelength
When X-rays strike the parallel planes of the crystal at an angle q, 2d . sin q = nl
they are reflected from different planes. The resulting reflected This is Bragg's equation.
beam will be strong if all reflected rays are in phase. Now as SYMMETRY IN CRYSTAL SYSTEMS :
2p (i) Plane of symmetry : Which divides a crystal in two parts in
Phase difference = ´ path difference
l such a way that one part is the mirror image of the other.
If path difference is an integral multiple (say n) of wavelength, (ii) Axis of symmetry : It is a line about which the crystal may be
then phase difference becomes an integral multiple of 2p or 360° rotated such that it presents the same appearance more than
and hence the reflected rays will be in same phase once during complete revolution.
(iii) Centre of symmetry : It is a point such that any line drawn
Q’ through it intersects the surface of the crystal at equal
Q P’ distances in both directions.
ELEMENTS OF SYMMETRY :
P q O
Total number of planes, axes and centre of symmetries present in
q d a crystal are termed as elements of symmetry. A cubic crystal of
A B NaCl possesses 23 elements of symmetry.
X-ray reflection from crystals • Planes of symmetry = 3 + 6 = 9
Now from figure the path difference between waves reflected from • Axes of symmetry = 3 + 4 + 6 = 13
upper planes = AO + OB • Centre of symmetry = 1
From simple trignometry, if d is the distance between successive • Total number of symmetry elements = 23
OA SEVEN CRYSTAL SYSTEMS :
layers then sinq =
d There are about 230 crystal forms, which have been grouped into
OA = d sin q 14 types of space lattices called BRAVAIS LATTICES on the
Similarly OB = d sin q basis of their symmetry and 7 different crystal systems on the
Thus net path difference = OA + OB = 2d sin q basis of interfacial angles and axes.

Crystal System Parameters of unit Cell Bravais


Intercepts/axes angles Lattices
1. Cubic a=b=c a = b = g = 90° 3 Primitive, Face Centred, Body centred
2. Tetragonal a =b¹c a = b = g = 90° 2 Primitive, Body centred
3. Rhombohedral a=b= c a = b = 90° g ¹ 90° 1 Primitive
4. Ortho Rhombic a¹b¹c a = b = g = 90° 4 Primitive, Face centred,
Body centred, End centred
5. Monoclinic a¹b¹c a = b = 90°, g ¹ 90° 2 Primitive, End Centred
6. Triclinic a¹b¹c a ¹ b ¹ g ¹ 90° 1 Primitive
7. Hexagonal a =b ¹ c a = b = 90°, g = 120° 1 Primitive

TYPES OF UNIT CELLS OCCURING IN CRYSTALS :

c c c

b a
g
b a b a
b b
g g a b

a a
Simple Cubic Tetragonal Rhombohedral
The Solid State 509
c c (b) Hexagonal close packing : Space occupied by spheres
is 60.4%. Hence it is more efficient.

b a b a
b
g g
a a
Orthorhombic Monoclinic b
c c

b b a
a b (ii) Close packing in three dimensions : Following the
b g
g hexagonal close packing here, we have two layers.
a
a
Triclinic Hexagonal
Bravais showed from geometrical considerations that there
are only seven shapes in which unit cells can exist. c
c c c
These are
(i) Cubic a a a
a
(ii) Orthorhombic
c c c
(iii) Rhombohedral
(iv) Hexagonal
(v) Tetragonal
(vi) Monoclinic
(vii) Triclinic
On the basis of presence of particles there are four types For third layer there are two types of voids, marked a
of unit cells tetrahedral voids and marked c octahedral voids. If spheres
(i) Primitive or simple are placed over a, first layer is repeated and we have AB- AB
(ii) Face centred arrangement. This is known as hexagonal close packing
(iii) Body centred (hcp). If spheres for third layer are placed over c, a different
(iv) End centred
layer is formed and we have ABC-ABC arrangement. This is
He also showed that out of possible twenty eight unit cells. (Seven
knwon as cubic close packing (ccp).
shapes × four types in each shape = 28) only fourteen actually
Both hcp and ccp are equally efficient and space occupied
exist. These are called BRAVAIS LATTICES.
by spheres is 74.0%. The cubic close packing has face
CO-ORDINATION NUMBER : centered cubic (fcc) unit cell.
It is defined as the number of particles immediately adjacent to Following the square close packing in three dimensions,
each particle in the crystal lattice. we have AB, AB arrangement of spheres.
In simple cubic lattice C.N. is 6
In body centred lattice C.N. is 8 and
In face centred cubic lattice C.N. is 12.
High pressure increases C.N. and high temperature decreases
the C.N.
PACKING OF CONSTITUENT PARTICLES IN
CRYSTALS :
Generally the particles of larger size do the packing and particles
The resulting structure in this arrangement is referred to as
of smaller size occupy interstitial places
body-centred cube (bcc). In this arrangement only 68% of
(i) Close packing in two dimensions : It is of two types
(a) Square close packing : Space occupied by spheres is the available space is occupied by spheres and occur in
52.4% many solids.
INTERSTITIAL VOIDS :
The space left after hcp and ccp (74.0% in each case) is known as
interstitial voids. They are of two types:
(i) Tetrahedral voids : The vacant space touching four spheres
is called tetrahedral void. The radius ratio is
rvoid
= 0.225
rsphere
510 Chemistry

rvoid
= 0.414
rsphere

(a) (b)
(a) (b)
(a) Eight tetrahedral voids per unit cell of ccp structure
Location of octahedral voids per unit cell of ccp or fcc lattice
(b) one tetrahedral void showing the geometry.
(a) at the body centre of the cube and
The number of tetrahedral voids is twice the number of (b) at the centre of each edge (only one such void is shown).
spheres (atoms). The number of octahedral voids is equal to the number of
(ii) Octahedral voids : The vacant space touching six spheres is spheres (atoms).
called octahedral void. The radius ratio is

LIMITING RADIUS RATIO :

Cations tend to get surrounded by the largest number of anions therefore the larger the rvoid ratio, the larger is the coordination
rsphere
number.
Limiting radius ratio(r+/r–) Shape Coordination number Example
0.155 – 0.225 Planar triangle 3 B2O3
0.225 – 0.414 Tetrahedral 4 ZnS
0.414 – 0.732 Square planar 4
0.414 – 0.732 Octahedral 6 NaCl
0.732 – 1.000 Body Centred Cubic 8 CsCl

STRUCTURE OF SOME SIMPLE IONIC COMPOUNDS :


It can be summarised in the following table, for cubic ionic solids.
S. Crystal Structure Packing C.No. No. of formula Examples
No. Units per unit
cell

1. Rock Salt (NaCl) Cl– in ccp 6 4 Halides of Li, Na


Na+ in octahedral voids K, Rb, AgCl, AgBr, NH4Cl
2. Zinc blends (ZnS) S– – in ccp 4 4 CuX (Cl, Br, I), AgI, BeS
Zn++ alternate
tetrahedral voids
3. Wurtzite (ZnS) Zn++ hcp 4 4
S––alternate
tetrathedral voids
4. CsCl Cs+ in Cubic Voids 8 1 CsBr, CsI, CsCl, TlCl,
Cl– at the corners TlBr, TlI, TlCN
of a cube or vice versa
5. Fluorite CaF2 Ca++ in ccp, F– Ca++ F– 4 SrF2, BaF2, BaCl2, CaF2,
tetrahedral voids (8:4) CdF2, PbF2
6. Antifluorite Li2S S– – in ccp, Li+ Li+S2– 4 Na2O, Li2O, K2O, Na2O,
tetrahedral voids (4:8) K2S
The Solid State 511
PRESENCE OF CATIONS AND ANIONS IN UNIT CELL :
S.No. Crystal Structure Anion Cation
1. NaCl 8 at corners 6 at faces 12 at edges, 1 in body
2. ZnS 8 at corners 6 at faces 4 in body
3. CsCl 8at corners 1 in body
4. CaF2 8 in body 8 at corners, 6 at faces

Lattices of some elements : DENSITY OF UNIT CELL :


(i) Cubic close packed lattice : Cu, Ag, Au, Fe, Ni, Al It is given by
(ii) Hexagonal close packed lattice : Zn, Cd, Hg, Be, Mg, Ti
Z´M
In both the above structures each atom is surrounded by d= g/cm3
twelve nearest touching neighbours hence coordination N A ´ a 3 ´ 10 -30
number is 12. where Z = Number of formula units present in unit cell. It is also
(iii) Body centred cubic lattice : Li, Na, K, Rb, Cs, W, Cr known as rank or unit cell content.
This packing is not so tight as the first two are, each atom is M = Molecular weight, NA = Avogadro number
surrounded by eight nearest touching neighbours, hence a = edge length of unit cell in pm
coordination number is 8. IMPERFECTIONS IN SOLIDS :
RELATION BETWEEN EDGE OF UNIT CELL AND They are of two types.
RADIUS OF CATION AND ANION IN FCC (i) Electronic imperfections : In Crystals of pure Silicon or
STRUCTURE : Germanium some electrons are released from covalent bonds
at temperatures above 0K which are now free to move in the
2r+ + 2r- = a crystal. The electron deficient bonds produced are known
as holes. Electrons (e) and holes (h) in solids are considered
a a 2
\r+ + r- = or r = = a to be electronic imperfections. Concentrations of electrons
2 2 2 4 (e) and holes (h) is represented by n and p.
In NaCl str ucture, distan ce between cation an d an ion (ii) Atomic imperfections : Further they may be divided as
(a) Point imperfections : The periodic arrangement of
=
2
(
a + -
r +r ) constituent particles is disturbed.
(b) Lattice imperfections : The deviations from periodic
BODY DIAGONAL : arrangement extend over microscopic regions of the
crystal.
In CsCl ions touch along body diagonal :
(c) Line defects / dislocations : The lattice imperfections
2r + + 2r - = 3.a (length of body diagonal) extend along lines.
(d) Plane defects : The lattice imperfections extend along
3.a 3 surfaces of crystals.
\ r + + r- = or r = a
2 4 STOICHIOMETRIC POINT DEFECTS :
FACE DIAGONAL : They include
(i) Schottky defect : It is due to missing of ion pairs (one cation
If there is cube of edge length a then face diagonal is 2 a , Body and one anion) from their lattice sites. It maintains the
electrical neutrality. Density of crystal decrease. It is more
diagonal is 3.a .
common in compounds with high coordination number and
COORDINATION NUMBER OF AN ATOM IN : when size of positive and negative ions is almost equal.
(i) Primitive Cubic unit cell = 6 Example NaCl, KCl, CsCl and KBr etc.
(ii) Body centred cubic unit cell = 8 (ii) Frenkel defect : It is due to dislocation of ion from its usual
(iii) Face centred cubic unit cell = 12 site to interstitial position. Density remains the same. It is
common in compounds having low coordination number and
NORMAL SPINEL STRUCTURE :
large difference in size of +ve and –ve ions. Examples ZnS,
The mineral MgAl2O3 is known as spinel. The oxide ions are AgCl, AgBr and AgI.
arranged in ccp, Mg++ ions occupy tetrahedral voids and Al+++ AgBr shows both of the above defects.
ions occupy the octahedral voids. Ferrite ZnFe2O4 has spinel
NON STOICHIOMETRIC POINT DEFECTS :
structure. Being magnetic these are used in telephones and
memory loops in computers. They include
INVERSE SPINEL STRUCTURE : (i) Metal excess (due to anion vacancy) : It is due to absence of
anion from lattice site leaving a hole which is occupied by
Divalent ions occupy octahedral voids and trivalent ions are
electron to maintain electrical neutrality
equally distributed in tetrahedral and octahedral voids. e.g. Fe3O4
A+ B– A+ B–
or FeO. Fe2O3. B– A+ e– A+
512 Chemistry
F-Centres - Holes occupied by electrons are called F-Centres DONORS :
(Colour centres). The greater the number of F-centres the Impurities capable of donating electrons are known as donors
greater is the intensity of colour of solid and solids are e.g. P, As, Sb.
paramagnetic in nature. Due to F-centres
NaCl is yellow ACCEPTORS :
KCl is violet Impurities capable of accepting electrons are known as acceptors
LiCl is pink. e.g. B, Al, Ga.
Crystals showing schottky defect also show this defect. n-TYPE SEMI CONDUCTORS :
(ii) Metal excess due to interstitial cations : It is due to extra
Silicon doped with phosphorous is called n-type semi conductor.
positive ion and an electron occupying an interstitial position
The conductivity is due to presence of negative charge
A+ B– A+ B–
A+ e– (electrons).
B– A+ B– A+ Si Si Si
Crystals exhibiting Frenkel defect show this defect eg ZnO. +
(iii) Metal deficiency due to cation vacancy : It is due to absence Si P
of a metal ion from its lattice site and charge is balanced by –
ion having higher positive charge. Si Si Si
A+ B– O B–
B – A ++ B– A+ p-TYPE SEMI CONDUCTORS :
Transition metals exhibits this defect due to metal deficiency, Silicon doped with gallium is called p-type semi conductor. The
the compound obtained are non stoichiometric. e.g. it is conductivity is due to the presence of positive holes.
difficult to prepare ferrous oxide with the ideal composition
Si Si Si
FeO. What we actually obtain is Fe0.95O or FexO with x=0.93 +
to 0.96 Si Ga
(iv) Metal deficiency due to interstitial negative ion : It is due to
presence of negative ion in interstitial position and presence Si Si Si
of metal ion with extra charge.
Some typical 13-15 comounds are InSb, AlP and GaAs and some
A+ B– A+ B–
B – typical 12-16 compounds are ZnS, CdS, CdSe and HgTe.
B– A+ B– A++ These exhibit electrical and optical properties of great use in
This defect is not common electronic industry.
NON STOICHIOMETRIC DEFECT DUE TO PRESENCE PIEZOELECTRIC EFFECT AND PIEZOELECTRICITY :
OF FOREIGN ATOMS : The charge developed on the crystal due to mechanical stress is
In AgCl crystals two Ag+ ions may be missing while one Cd++ called piezoelectric effect and the current thus produced is called
occupies one of the vacant sites. piezo electricity.
Ag+ Cl– Ag+ Cl– PYROELECTRICITY :
Cl– Cd++ Cl– The electric current produced by heating on the faces of a crystal
FOOL'S GOLD : is called pyroelectricity.
Some samples of Iron pyrites shine like gold due to exchange of FERROELECTRIC EFFECT :
electrons between Fe2+ and Fe3+, which leads to metallic lustre. The reversal of direction of permanently lined up dipoles of piezo
Such samples are known as fool's gold.
electric crystal by applying electric field is called ferro electric
SEMI CONDUCTORS : effect.
Electronic conductors having electrical conductivity in the range Examples Rochelle Salt, KH2PO4 and Barium titanate BaTiO3.
of 104 – 10–7 ohm–1 cm–1 are known as semi-conductors. Examples
ANTIFERRO ELECTRIC SOLID :
Si, Ge, Sn (grey) Cu2O, SiC and GaAs.
When the dipoles in alternate lattices point are in opposite direction
INTRINSIC SEMI CONDUCTORS :
and net dipole moment is zero, the crystals are referred to as
Pure substances those are semi-conductors known as intrinsic
antiferro electric. Example is lead Zirconate PbZrO3.
semi conductors e.g. Si, Ge.
DIAMAGNETIC :
EXTRINSIC SEMI CONDUCTORS :
Their conductivity is due to the presence of impurities. They are The substances which are weakly repelled by magnetic fields e.g.
formed by doping e.g. Si doped with P. KCl, NaCl and TiO2. It is independent of temperature.
DOPING : PARAMAGNETIC :
It is defined as addition of impurities to a semiconductor to increase The substances which are attracted by magnetic field and loss
the conductivity. Doping of Si or Ge is carried out with P, As, Sb, magnetism in the absence of a magnetic field e.g. O2, Cu++, Fe+++,
B, Al or Ga. Na, K etc.
The Solid State 513
FERROMAGNETIC : ISOMORPHISM :
The substances which are strongly attracted by magnetic field Two or more substances having the same crystalline form are
and retain magnetism even when the magnetic field is removed known as isomorphous and the phenomenon is called isomorphism
e.g. CrO2, Fe, Co & Ni show ferro magnetism at room temperature. K2SO 4.Cr 2(SO 4)3.24H2O and K 2SO 4.Al 2(SO4) 3.24H2O are
The alignment of magnetic moments is spontaneous in such isomorphous.
substances ­ ­ ­ ­ ­ . MITSCHERLICH'S LAW :
It states that isomorphous substances possess an equal number
ANTIFERRO MAGNETISM : of atoms united in a similar way.
It is due to compensatory alignment of moments ­¯ ­¯ ­¯ . ISOPOLYMORPHISM :
The net moment is zero e.g. MnO, MnSe and KMnFe3. When each form of a polymorphic substance is isomorphous with
FERRIMAGNETISM : a form of another polymorphic substance the phenomenon is
known as Isopolymorphism e.g. antimony trioxide and arsenious
The alignment of moments are parallel and anti-parallel unequal in
oxide both exist in rhombic and octahedral form. The rhombic
number, there is net moment e.g. Fe3O4, ferrites of formula M2+
forms of two oxides are isomorphous and octahedral forms of the
Fe2O4 (M = Mg, Cu, Zn)
oxides are also isomorphous.
CLASSIFICATION OF SOLIDS ON THE BASIS OF SILICATES :
ELECTRICAL CONDUCTIVITY :
Silicates are metal derivatives of silicic acid H4SiO4 obtained by
(i) Metals : 108 ohm–1 cm–1 fusing metal oxides or metal carbonates with sand.
(ii) Insulators : 10–12 ohm–1 cm–1
Fuse SiO
(iii) Semiconductors : 104 – 10–7 ohm–1 cm–1 Na 2 CO 3 ¾¾ ¾ ¾
¾2 ® Na SiO ( Na SiO )
4 4 2 3 n
1770K
BERTHOLLIDES :
CLASSIFICATION OF SILICATES :
Non Stoichiometric compounds are called Berthollides. The
ORTHO SILICATES :
common are sulphides and oxides e.g. FeO, CuO.
They contain discrete SiO44– units.
LIQUID CRYSTALS (MESOMORPHIC OR PARA
Examples : Phenacite Be2SiO4, willemite Zn2SiO4. Metal cations
CRYSTALLINE) : are tetrahedrally surrounded by four O atoms.
The substances having the properties of crystalline solids and
PYRO SILICATES :
flow properties of liquids. Examples : Para-azoxy anisole and Normal
Two tetrahedral units are joined by one oxygen atom and contain
Octyl-para azoxy cinnamate. Such solids undergo two sharp phase
Si2O76– unit.
transformations on fusion, first yielding turbid liquid and at higher
Examples : Thortiveitite Se2 (Si2O7)
temperature a clear liquid. The turbid liquids are called liquid
Hemimorphite Zn3 (Si2O7) . Zn (OH)2. H2O.
crystals and are anisotopic in nature (a property of crystalline
solid). CHAIN SILICATES :
(i) (SiO32–)n Two oxygen atoms per SiO4 tetrahedron are shared
TRANSITION POINT :
giving polymeric anion chains.
It is the temperature at which a substance changes sharply into Example : Spodumene LiAl (SiO3)2, Diposide CaMg(SiO3)2.
the state of liquid crystals. (ii) (Si4O116–)n contain double chain.
POLYMORPHISM : Example : Tremolite Ca2Mg5 (Si4O11) (OH)2.
Existence of a substance in more than one crystalline form is known CYCLIC SILICATES :
as polymorphism e.g. Carbon, Sulphur, Tin etc. Two oxygens per SiO4 tetrahedron are shared containing Si3O96–
ALLOTROPY : and Si6O1812– units.
The polymorphism in elements is known as allotropy. Example : Benyl : Be3Al2Si6O18
ENANTIOTROPIC SUBSTANCE : SHEET SILICATES :
Polymorphic form of a substance whose transition temperature is Here three oxygens of each SiO4 tetrahedron are shared. Containing
lower than the fusion temperature is known enantiotropic (Si2O52–)n
substance and phenomenon is called enantiotropy. Example : Clay.

MONOTROPY : FRAME WORK SILICATES :


All oxygen atoms of each SiO4 tetrahedron are shared. They
When fusion temperature of both the forms is below the transition
contain three dimensional net work.
temperature the phenomenon is known as monotropy.
Examaple : Quartz, zeolite.
514 Chemistry

1. Sodium metal crystallises in a body centred cubic lattice with Sol. The arrangement of atoms shows a simple cubic cell.
the cell edge a = 4.29 Å. What is the radius of sodium atom.
1
Sol. In bcc the atoms touch along body diagonal Number of X atoms = ´ 8 = 1 (at corners)
8
3 ´ 4.29
\ 2r + 2r = 3 . a ; r = = 1.857 Å 1
4 Number of O atoms = ´12 = 3 (at edges)
2. Chromium metal crystallizes with a body centred cubic lattice. 4
The length of the unit cell edge is found to be 287 pm. Calculate Number of Na atoms = 1 (at centre)
the atomic radius. What would be the density of chromium Hence formula NaXO3
in g/cm3.
+ –
3 .a 3 ´ 287 6. A solid A B has NaCl type close packed structure. If the
Sol. In bcc lattice r = = = 124.24 pm anion has a radius of 250 pm; What should be the ideal
4 4
radius for the Cation? Can a cation C+ having radius 180 pm
Z´M + –
Density = N ´ a 3 be slipped into the tetrahedral void of the crystal A B ? Give
A
reasons for your answer.
2 ´ 51.99
r= = 7.30 g / ml + –
6.023 ´ 10 23 ( 287 ´ 10 -10 ) 3 Sol. Since A B has NaCl type close packed structure it must
3. Gold has closed packed structure which can be viewed as belong to a crystal system having coordination number 6.
spheres occupying 0.74 of the total volume. If the density of Hence limiting radius ratio
gold is 19.3 g/ml. Calculate the approximate radius of gold r+
ion in the solid (gold = 197 amu). = 0.414 - 0.732
Sol. Packing fraction = 0.74. It has fcc type cell. r-
So number of gold ions per cell = 4
r– = 250 pm ; r + ³ 0.414 ´ 250 = 103.5 pm
Z´M 4 ´ 197 Ideal radius for cation.
r= = = 19 .3 ;
NA ´ a3 a 3 ´ 6 .023 ´ 10 23 (II) Cation C+ having radius 180 pm cannot be slipped into
\ a = 4.07 ´10 -8 cm + – r+
the tetrahedral site of crystal A B because should be
For fcc cell, r-
2 .a 2 ´ 4.07 ´ 10 -8 r+
r= = = 1.439 ´10 -8 cm 0.225 – 0.414. In the given case
180
=
= 0.72 .
4 4 r 250-
4. A metal crystallizes into two cubic phases. (fcc and bcc)
7. In an ionic crystalline solid AB, r + = 0.74 Å and
whose unit cell lengths are 3.5 and 3.0 Å respectively.
r– = 1.84Å. The atomic weights of A and B are 70 and 40 amu
Calculate the ratio of densities of fcc and bcc.
respectively. Find (a) Coordination number of A & B (b)
Z´M Density in g/cc of solid.
Sol. r =
NA ´ a3
r+ 0.74
Sol. Given = = 0.402 < 0.414 therefore coordination
4´ M 2´ M r - 1.84
For fcc, r1 = ; For bcc, r2 =
N A (3.5 ´10-8 ) 3 N A (3.0 ´10 -8 )3 number = 4 (tetrahedral)
2r+ + 2r– = a
r1 4 (3.0 ´10 -8 ) 3
= = 1.259 \ a = 2(0.74 + 1.84) = 5.16Å
r 2 2 (3.8 ´10 -8 )3
5. A solid has a structure in which X atoms are located at the 4 ´ 110
= = 5.33 g / cc
cube corners of the unit cell. O atoms are located at the 6.023 ´ 10 23 (5.16 ´ 10-8 )3
centre of cube edges and Na atom at the cube centre. What
8. What is the concentration of cation vacancies when NaCl is
type of lattice is represented by this compound. Also suggest
a molecular formula for the compound. doped with 10–3 mole % of SrCl2.
The Solid State 515
Sol. The number of cation vacancies created in the lattice of NaCl l 154
is equal to the number of divalent Sr2+ ion added d 200 = = = 280 pm
2 sin q 2 sin 16°6'
Concentration of Sr 2+ ions = 10 –3 mole per cent
a
= 10 -3 100 = 10 -5 mol d 200 =
1 mole of Sr 2+ contains 6.023 × 1023 Sr2+ ions 22 + 02 + 02
10–5 mole of Sr2+ ions contains 6.023 × 1023 × 10–5 ions. \ a = 2 ´ 280 = 560 pm
Therefore the number of cation vacancies in NaCl crystal is + -
6.023 × 1018. 15. A solid A B has an NaCl type closest packed structure. If
9. In Fe0.93O, find the % of Fe+++ ions. the radius of the anion is 250 pm. What is the radius of the
Sol. Iron 93% and O is 100%. cation?
Let Fe+++ be x% then Fe++ (93 – x)% then rc
Positive charge = Negative charge Sol. For NaCl type structure, = 0.414
ra
2(93 - x ) + 3x = 2 ´100
\ rc = (0.414) ra = 0.414 × 250 pm = 103.5 pm
\ x = 14
14
16. KF has NaCl structure. What is the distance between K +
+++
\ % of Fe = ´ 100 = 15.2% .
93 and F - in KF, if the density of KF is 2.48 g cm - 3 .
10. X-rays of wavelength 1.54Å strike a crystal and are observed [At. wt. of K = 39 and F = 19]
to be deflected at an angle of 22.5°. Assuming that n = 1. Calculate Sol. For NaCl type structure the rank of unit cell = 4
the placing between the planes of atoms that are responsible Z´ M 4 ´ 58
a3 = =
for the reflection. r´ NA 2.48 ´ 10 - 30
´ 6.023 ´ 10 23
Sol. Bragg's equation is nl = 2d sin q Edge length a = 538 pm
1.54 ´ 1 a
\d = = 2.01Å So the distance between K + and F - ion =
= 269.0 pm
2 sin 22.5° 2
17. The unit cell of an element of atomic mass 96 and density
11. At what glancing angle would be first order diffraction from
(110) plane of KCl observed using X-ray of wavelength of 10.3 g cm - 3 is a cube with edge length of 314 pm. Find the
154 pm. The dimensions of the unit cell is 315 pm. structure of the crystal lattice.
Sol. For the first order diffraction; l = 2d sin q
r ´ N A ´ a 3 10.3 ´ 6.023 ´10 23 ´ (314 ´ 10 -10 ) 3
l = 154 pm ; d = 315 pm Sol. Z= = =2
M 96
154 = 2 ´ 315 sin q ; \ q = 14.1° The number of atoms per unit cell is two. Hence the crystal
lattice has bcc unit cell.
12. The planes in a crystalline solid intersect the crystal axes at 18. KCl crystallises in the same type of lattice as does NaCl.
(2a, b, c) (–a, b, c) (a, 2b, 3c), (3a, b, c), (–a, b, ¥ ). Calculate
the miller indices for these planes. rNa + r
Na +
Sol. Given that, = 0.5 and = 0.7
rCl- r
Intercepts 2a, b, c –a, b, c a, 2b, 3c 3a, b, c –a, b, ¥ K+
Weiss indices 211 –111 123 311 –11 ¥ Calculate (a) the ratio of the side of the unit cell for KCl to
Reciprocal of weiss indices that for NaCl and (b) the ratio of density of NaCl to that of
KCl.
1 11 1 11 11 1 1 11 1 11 Sol. NaCl crystallises in the fcc unit cell such that
2 11 -1 1 1 123 3 11 -1 1 ¥ a
Clear fractions 122 –111 632 133 –110 ( rNa + + rCl- =
2
)
Miller indices 122 111 632 133 110
Where a is the edge of length of unit cell. Since
13. Determine the interplanar spacing between the (220) planes
of a cubic lattice of length 450 pm. r r
Na + Na +
= 0.5 and = 0.7
a 450 450 r r
Sol. d hkl = = = = 159 pm Cl- K+
h2 + k 2 + l 2 22 + 22 + 02 8
r +r
Na + Cl -
14. First order X-ray reflection (l = 154) maximum from a set of We have, = 1.5 ....(i)
r
(200) planes of a body centred cubic lattice was observed at Cl -
16°6'. Calculate the length of the edge of unit cell.
r rK + 0. 5 0.5
Sol. For first order reflection n = 1. K+
and = = =
r rNa + / 0.5 r + /r 0.7
\ l = 2d sin q Cl- Na K+
516 Chemistry
Adding 1 on both sides
Z´ M 4 ´ 108
rK + + rCl - Hence density r = =
=
1.2 NA ´ a 3 6.023 ´ 10 23 (3.53 ´ 10- 8 )3
rCl - 0. 7 ...(ii)
= 16.3 g cm - 3
Dividing (ii) by (i)
22. LiI occurs as a cubical closest packing . If the edge length of
r +r
K+ Cl -
=
1.2
×
1 the unit cell is 624 pm, determine the ionic radii of Li + and
r +r 0.7 1 .5
Na + Cl - I - ions
a KCl / 2 1.2 a KCl 1 .2 Sol. The cubical closest packing has fcc unit cell. I - ions occupy
or = or = the corner and the face centers. These ions touch each other
a NaCl / 2 0.7 ´ 1.5 a NaCl 1 . 05
along the face diagonal of the cube. Hence
Z´ M 4r - =
Now since r = I
2 .a
NA ´ a3
a 624pm
3 or r- = = = 220.65 pm
I
r NaCl æa ö æ M NaCl ö 2 2 2(1.414)
We will have = çç KCl ÷÷ ç ÷
ç M ÷
r KCl è a NaCl ø è KCl ø Along the edge we have I - Li + I - arrangement where I -

æ 1.2 ö æ 58.5 ö
3 are at the corners and Li + at the centre of the edge
=ç ÷ ´ç ÷ = 1.172 (Octahedral void)
è 1.05 ø è 74.5 ø
\ 2r - +2r =a
-3 I Li+
19. The density of CsBr which has bcc structure is 4.4 g cm .
The unit cell has edge length a pm. Calculate the inter ionic a æ 624 ö
r = – rI- = ç - 220.65 ÷ pm = 91.35 pm
distance in crystals of CsBr (Atomic mass Cs = 133, Br = 80) Li+ 2 è 2 ø
é Z´ M ù 23. Calculate the packing fraction for Ca unit cell, given that Ca
3 2 ´ 213 êFormula r = ú
Sol. a = crystallises in face centred cube unit cell
4.4 ´ 6.023 ´10 23 êë N A ´ a 3 úû
Sol. The rank of fcc unit cell = 4

a = 5.438 × 10 - 8 cm 2.a
The atomic radius r =
For bcc lattice the interionic distance 4
(The atoms touch along face diagonal)
3 1.732 ´ 5.438 ´ 10- 8
r+ + r- = ×a= = 470.9 pm Volume of 4 atoms
2 2
3
20. If the radius of Cs + is 169 pm and that of Cl - is 181 pm. 4 3 4 æ 2 .a ö÷ 2 pa3
= 4 × p r = 4 × pç =
Predict the structure of CsCl. What is the coordination 3 3 çè 4 ÷ø 6

number of Cs + ion?
2 p a3 3
rc 169 Packing fraction = a = 0.74
Sol. = = 0.94 6
ra 181
24. The first order reflection of a beam of X -rays of wavelength
Since the value is greater than 0.732, the structure of CsCl is 1.54Å from the (100) plane of a crystal of the simple cubic
bcc. Its coordination number is 8. type occurs at an angle of 11.29. Calculate the length of the
21. The nearest neighbour Ag atoms in the silver crystal are unit cell (sin 11.29° = 0.1991)
2.5 × 10 - 8 cm. Presuming fcc structure of the crystal, what Sol. nl = 2 d sin q

will be the density of the silver? [At. wt. of Ag = 108 mol -1 ] 1 × 1.54 × 10 - 8 cm = 2 × d × sin11.29
Sol. We know that the nearest neighbour distance between the d = 3.68 × 10 - 8 cm
silver atoms is half of the face diagonal which is related to Further
2.a a
the edge length a by a
2 d= or 3.68 × 10 - 8 =
h 2 + k 2 + l2 12 + 0 2 + 0 2
2.a
= 2.5 × 10 - 8 cm or a = 3.53 × 10 - 8 cm
2 or a = 3.68 × 10 - 8 cm
The Solid State 517

Very Short/Short Answer Questions Multiple Choice Questions


1. How can you convert NaCl structure into CsCl structure 18. How many Cl– ions are there around Na+ ion in NaCl crystal?
and vice versa? (a) 3 (b) 4
2. What are coordination number of each ion present in the (c) 6 (d) 8
cubic close packed structure of Na2O at ordinary temperature 19. Which of the following is the incorrect statement?
and pressure? (a) NaCl has 6 : 6 coordination and CsCl has
3. AgI crystallises in cubic close packed ZnS structure. What 8 : 8 coordination.
fraction of tetrahedral sites are occupied by Ag+ ions?
(b) In Na2O each oxide ion is coordinated by 8Na+ ions
4. How does the electrical conductivity of superconductors
and each Na+ ion by 4 oxide ions
vary with temperature?
5. What makes the crystal of KCl appear sometimes violet? (c) NaCl structure transform to CsCl structure on heating
6. Pure silicon is an insulator. Silicon doped with phosphorus (d) In CaF2 structure each F– ion is coordinated by 4 Ca2+
is a semiconductor. Silicon doped with gallium is also a and vice-versa.
semiconductor. What is the difference between the two 20. Which of the following is a correct statement ?
doped silicon semiconductors? (a) Bonding in metallic crystals is directional
7. Account for following: (b) Diamond has two dimensional crystal lattice
(a) Silicon is an insulator but silicon doped with (c) Co-ordination number of bcc lattice is 12
phosphorous acts as a semiconductor. (d) A ccp structure has 8 tetrahedral and 4 Octahedral
(b) Some of glass objects recovered from ancient interstices.
monuments look milky instead of being transparent. 21. The r+ / r– ratio of ZnS is 0.402. Pick out the false statements
8. Copper crystallises in fcc lattice and has density of of the following
8×930 g cm–3 at 293 K. Calculate the radius of copper atom. (a) ZnS is 4 : 4 coordination compound.
Atomic mass of Cu = 63×55 amu, Avogadro’s constant, (b) ZnS does not crystallize in rock salt type lattice because
NA = 6×02 × 1023.
r+ / r– is too small to avoid overlapping of S2– ions.
9. A compound is formed by two elements M and N. The
(c) Zn2+ ion is too small to fit precisely into the octahedral
element N forms ccp and atoms of M occupy 1/3 rd of
voids of S2– ions.
tetrahedral voids. What is the formula of the compound?
10. Which of these two CdCl2 and NaCl will produce schottky (d) Zn 2+ ion is too large to fit into the octahedral voids of
defect, if added to a AgCl crystal? S2– ions.
11. Why is Fe3O4 ferrimagnetic at room temperature but 22. The unit cell dimensions of a cubic lattice (edges a, b, c and
becomes paramagnetic at 850 K? the angles between them, a, b and g) are
12. Why does LiCl acquire pink colour when heated in Li (a) a = b = c, a = b = g = 900
vapours? (b) a = b ¹ c, a = b = g = 900
13. Which stoichiometric defect in crystals increases the density (c) a = b = c,a = g = 900, b ¹ 900
of a solid? (d) a ¹ b ¹ c,a = b = 900, g ¹ 900
14. How many atoms constitute one unit cell of a face-centered 23. A NaCl crystal is changed into CsCl type structure by:
cubic crystal? (a) increasing temperature and decreasing pressure.
15. (a) What type of semiconductor is obtained when silicon is (b) increasing pressure and decreasing temperature.
doped with boron? (c) increasing both temperature and pressure
(b) What type of magnetism is shown in the following (d) None of these.
alignment of magnetic moments?
24. M 2 X has anti fluorite structure. In such structure
­­­­­­ --
(a) X ions occupy all the 8 octahedral voids
(c) What type of point defect is produced when AgCl is
doped with CdCl2? (b) Each X - - is surrounded by 4 M + in tetrahedral
16. The unit cell of an element of atomic mass 108 u and arrangement
density 10.5 g cm–3 is a cube with edge length, 409 pm. Find (c) Larger cations occupy the position of F - ions and
the type of unit cell of the crystal.
smaller anions that of C + + ions
[Given: Avogadro’s constant = 6.023 × 1023 mol–1]
17. The radius of Na+ ion is 94 pm and that of Cl– ion is 181 pm. (d) Smaller cations occupy the position of F - ions and
Predict whether the coordination number of Na+ ion is 6 or 4. larger anions that of C + + ions
518 Chemistry
25. For two ionic solids CaO and KI, identify the wrong 26. For a covalent solid, the units which occupy lattice points
statement amongst the following : are:
(a) The lattice energy of CaO is much large than that of KI (a) atoms (b) ions
(c) molecules (d) electrons
(b) KI is more soluble in water
27. Graphite is a
(c) KI has higher melting point (a) molecular solid (b) covalent solid
(d) CaO has higher melting point (c) ionic solid (d) metallic solid

1. Which of the following is not a crystalline solid? 10. A match box exhibits
(a) KCl (b) CsCl (a) cubic geometry
(c) Glass (d) Rhombic S (b) monoclinic geometry
2. Among solids, the highest melting point is exhibited by (c) tetragonal geometry
(a) Covalent solids (b) Ionic solids (d) orthorhombic geometry.
(c) Pseudo solids (d) Molecular solids
11. When molten zinc is cooled to solid state, it assumes hcp
3. The major binding force of diamond, silicon and quartz is
structure. Then the number of nearest neighbours of zinc
(a) electrostatic force
atom will be
(b) electrical attraction
(a) 4 (b) 6
(c) covalent bond force
(d) non-covalent bond force (c) 8 (d) 12
+
4. The maximum proportion of available volume that can be 12. Co-ordination number of Na in NaCl is
filled by hard spheres in diamond is (a) 4 (b) 3
(a) 0.52 (b) 0.34 (c) 6 (d) 5
(c) 0.32 (d) 0.68 13. A crystal lattice with alternate +ve and –ve ions has radius
5. The packing fraction for a body-centred cubic is ratio 0.524. Its co-ordination number is
(a) 0.42 (b) 0.53 (a) 4 (b) 3
(c) 0.68 (d) 0.82 (c) 6 (d) 12
6. In a solid AB having the NaCl type structure, ‘A’ atoms 14. How many unit cells are present in a cube-shaped ideal
occupy the corners of the cubic unit cell. If all the face- crystal of NaCl of mass 1.00 g?
centred atoms along one of the axes are removed, then the [Atomic masses : Na = 23, Cl = 35.5]
resultant stoichiometry of the solid is
(a) 2.57 × 1021 unit cells (b) 5.14 × 1021 unit cells
(a) AB2 (b) A2B 21
(c) A4B3 (d) A3B4 (c) 1.28 × 10 unit cells (d) 1.71 × 1021 unit cells
7. A substance AxBy crystallizes in a face centred cubic (fcc) 15. Which of the following is example of body centred
lattice in which atoms ‘A’ occupy each corner of the cube cube ?
and atoms ‘B’ occupy the centres of each face of the cube. (a) Mg (b) Zinc
Identify the correct composition of the substance AxBy (c) Copper (d) Sodium
(a) AB3 16. Which of the following describes the hexagonal close packed
(b) A4B3 arrangement of spheres?
(c) A3B (a) ABCABA (b) ABCABC
(d) Composition can’t be specified (c) ABABA (d) ABBABB
8. Crystals can be classified into basic crystal habits, equal to 17. A metallic crystal crystallizes into a lattice containing a
(a) 7 (b) 4 sequence of layers AB AB AB......Any packing of spheres
(c) 14 (d) 2 leaves out voids in the lattice. What percentage of volume
9. Tetragonal crystal system has the following unit cell of this lattice is empty space?
dimensions (a) 74% (b) 26%
(a) a = b = c, a = b = g = 90° (c) 50% (d) none of these.
(b) a = b ¹ c, a = b = g = 90° 18. In crystal structure of sodium chloride, the arrangement of
Cl– ions is
(c) a ¹ b ¹ c, a = b = g = 90° (a) fcc (b) bcc
(d) a = b c, = b = 90° = 120° (c) Both fcc and bcc (d) None of these.
The Solid State 519
19. The intermetallic compound LiAg crystallizes in cubic lattice 32. When electrons are trapped into the crystal in anion vacancy,
in which both lithium and silver have co-ordination number the defect is known as :
of eight. The crystal class is (a) Schottky defect (b) Frenkel defect
(a) simple cubic (b) body centred cubic (c) Stoichiometric defect (d) F-centres
(c) face-centred cubic (d) None of these 33. In the laboratory, sodium chloride is made by burning sodium
20. In the crystals of which of the following ionic compounds in the atmosphere of chlorine. The salt obtained is yellow in
would you expect maximum distance between the centres of colour. The cause of yellow colour is
the cations and anions? (a) Presence of Na+ ions in the crystal lattice
(a) LiF (b) CsF
(b) Presence of Cl– ions in the crystal lattice
(c) CsI (d) LiI
21. Potassium crystallizes with a (c) Presence of electrons in the crystal lattice
(a) body-centred cubic lattice (d) Presence of face centred cubic crystal lattice.
(b) face-centred cubic lattice 34. Schottky defect in crystals is observed when
(c) simple cubic lattice (a) unequal number of cations and anions are missing from
(d) orthorhombic lattice the lattice
22. The range of radius ratio (cationic to anionic) for an (b) equal number of cations and anions are missing from the
octahedral arrangement of ions in an ionic solid is lattice
(a) 0 – 0.155 (b) 0.155 – 0.225 (c) an ion leaves its normal site and occupies an interstitial
(c) 0.225 – 0.414 (d) 0.414 – 0.732 site
23. The interionic distance for cesium chloride crystal will be (d) density of the crystal is increased
a 35. Which of the following has Frenkel defects?
(a) a (b) (a) Sodium chloride (b) Graphite
2
(c) Silver bromide (d) Diamond
3a 2a
(c) (d) 36. Which of the following crystals does not exhibit Frenkel
2 3 defect?
24. Which of the following has hcp crystal stucture? (a) AgBr (b) AgCl
(a) NaCl (b) CaCl (c) KBr (d) ZnS
(c) Zn (d) RbCl 37. Due to Frenkel defect, the density of ionic solids
35. In the fluorite structure, the coordination number of Ca2+ ion (a) decreases (b) increases
is :
(c) neither (a) nor (b) (d) does not change
(a) 4 (b) 6
38. A solid with high electrical and thermal conductivity from
(c) 8 (d) 3
the following is
26. For an ionic crystal of the general formula AX and co-
ordination number 6, the radius ratio value will be (a) Si (b) Li
(a) greater than 0.73 (b) between 0.73 and 0.41 (c) NaCl (d) Ice
(c) between 0.41 & 0.22 (d) Less than 0.22 39. If we mix a pentavalent impurity in a crystal lattice of
27. In CsCl, if coordination number of Cs+ is 8, then coordination germanium, what type of semiconductor formation will occur?
number of Cl– ion is : (a) p-type (b) n-type
(a) 4 (b) 6 (c) both (a) and (b) (d) None of the two.
(c) 8 (d) 12 40. The addition of arsenic to germanium makes the latter a
28. The number of unit cells in 58.8 g of NaCl is nearly (a) metallic conductor (b) intrinsic semiconductor
(a) 6 × 1020 (b) 3 × 1022 (c) mixed conductor (d) extrinsic semiconductor
23
(c) 1.5 × 10 (d) 0.5 × 1024
41. A semiconductor of Ge can be made p-type by adding
29. An element has bcc structure having unit cells 12.08×1023.
The number of atoms in these cells is (a) trivalent impurity (b) tetravalent impurity
(a) 12.08 × 1023 (b) 24.16 × 1023 (c) pentavalent impurity (d) divalent impurity
(c) 48.38 × 10 23
(d) 12.08 × 1022 42. On doping Ge metal with a little of Ga, one gets
30. An element (atomic mass = 100 g / mol) having bcc structure (a) p-type semi conductor (b) n-type semi conductor
has unit cell edge 400 pm. Then, density of the element is (c) insulator (d) rectifier
(a) 10.376 g/cm3 (b) 5.188 g/cm3 43. The pure crystalline substance on being heated gradually
3
(c) 7.289 g/cm (d) 2.144 g/cm36 first forms a turbid liquid at constant temperature and still at
31. The edge length of face centred unit cubic cell is 508 pm. If higher temperature turbidity completely disappears. The
the radius of the cation is 110 pm, the radius of one anion is behaviour is a characteristic of substance forming.
(a) 144 pm (b) 288 pm (a) Allotropic crystals (b) Liquid crystals
(c) 618 pm (d) 398 pm (c) Isomeric crystals (d) Isomorphous crystals.
520 Chemistry
44. If one end of a piece of a metal is heated the other end becomes 48. The existence of a substance in more than one solid
hot after some time. This is due to modifications is known as
(a) Energised electrons moving to the other part of the metal (a) Isomorphism
(b) resistance of the metal (b) Polymorphism
(c) mobility of atoms, in the metal
(c) Amorphism
(d) minor perturbation in the energy of atoms.
45. Which of the following is ferroelectric compound? (d) Allotropy
(a) BaTlO3 (b) K4[Fe(CN)6] 49. The second order Bragg diffraction of X-rays with l = 1.0Å
(c) Pb2O3 (d) None of these from a set of parallel planes in a metal occurs at an angle of
46. Which of the folliowing metal oxides is anti-ferromagnetic in 60°. The distance between the scattering planes in the crystal
nature? is
(a) MnO2 (b) TiO2 (a) 0.575Å (b) 1.00Å
(c) VO2 (d) CrO2
(c) 2.00Å (d) 1.15Å
47. Among the following which compound will show the highest
lattice energy? 50. Superconductors are derived from compounds of
(a) KF (b) NaF (a) p-block elements (b) lanthanides
(c) CsF (d) RbF (c) actinides (d) transition elements

1. If NaCl is doped with 10– 4 mol % of SrCl2, the concentration 7. A solid compound XY has NaCl structure. If the radius of the
of cation vacancies will be (NA = 6.02 × 1023 mol–1) cation is 100 pm, the radius of the anion (Y–) will be :
[CBSE-PMT 2007] [CBSE-PMT 2011 M]
16
(a) 6.02 × 10 mol –1 (b) 6.02 × 1017 mol–1 (a) 275.1 pm (b) 322.5 pm (c) 241.5 pm (d) 165.7 pm
14
(c) 6.02 × 10 mol –1 (d) 6.02 × 1015 mol–1 8. A metal crystallizes with a face-centered cubic lattice. The
2. The fraction of total volume occupied by the atoms present edge length of the unit cell is 408 pm. The diameter of the
in a simple cube is [CBSE-PMT 2007] metal atom is : [CBSE-PMT 2012]
(a) 288 pm (b) 408 pm
p p p p
(a) (b) (c) (d) (c) 144 pm (d) 204 pm
3 2 4 2 4 6
9. A metal has a fcc lattice. The edge length of the unit cell is 404
3. If ‘a’ stands for the edge length of the cubic systems : simple pm. The density of the metal is 2.72 g cm-3. The molar mass of
cubic, body centred cubic and face centred cubic, then the the metal is : (NA Avogadro’s constant = 6.02 × 1023 mol-1)
ratio of radii of the spheres in these systems will be [NEET 2013]
respectively, [CBSE-PMT 2008] (a) 30 g mol -1 (b) 27 g mol -1

1 3 1 1 1 (c) 20 g mol-1 (d) 40 g mol-1


(a) a: a: a (b) a : 3a : a 10. Which of the following statements about the interstitial
2 4 2 2 2 2
compounds is incorrect ? [NEET 2013]
1 3 3 (a) They are chemically reactive.
(c) a: a: a (d) 1a : 3a : 2a (b) They are much harder then the pure metal.
2 2 2
4. With which one of the following elements silicon should be (c) They have higher melting points than the pure metal.
doped so as to give p-type of semiconductor ? (d) They retain metallic conductivity.
[CBSE-PMT 2008] 11. The number of carbon atoms per unit cell of diamond unit cell
(a) Germanium (b) Arsenic (c) Selenium (d) Boron is : [NEET 2013]
5. AB; crystallizes in a body centred cubic lattice with edge (a) 8 (b) 6
length ‘a’ equal to 387 pm. The distance between two (c) 1 (d) 4
oppositely charged ions in the lattice is : [CBSE-PMT 2010] 12. Na and Mg crystallize in bcc and fcc type crystals respectively,
(a) 335 pm (b) 250 pm (c) 200 pm (d) 300 pm then the number of atoms of Na and Mg present in the unit
6. Among the following which one has the highest cation to cell of their respective crystal is [AIEEE 2002]
anion size ratio? [CBSE-PMT 2010] (a) 4 and 2 (b) 9 and 14
(a) NaF (b) CsI (c) CsF (d) LiF (c) 14 and 9 (d) 2 and 4.
The Solid State 521
13. How many unit cells are present in a cube-shaped ideal 21. Lithium forms body centred cubic structure. The length of
crystal of NaCl of mass 1.00 g ? the side of its unit cell is 351 pm. Atomic radius of the lithium
[Atomic masses : Na = 23, Cl = 35.5] [AIEEE 2003] will be : [AIEEE 2012]
21
(a) 5.14 × 10 unit cells 21
(b) 1.28 × 10 unit cells (a) 75 pm (b) 300 pm (c) 240 pm (d) 152 pm
21
(c) 1.71 × 10 unit cells (d) 2.57 × 1021 unit cells 22. Which of the following exists as covalent crystals in the solid
14. What type of crystal defect is indicated in the diagram below? state ? [JEE M 2013]
[AIEEE 2004] (a) Iodine (b) Silicon
(c) Sulphur (d) Phosphorus
Na + Cl - Na + Cl - Na + Cl -
23. The packing efficiency of the two-dimensional square unit
Cl– c Cl– Na+ c Na+
Na+Cl– c Cl– Na+ Cl– cell shown below is : [IIT-JEE 2010]
Cl– Na+ Cl–Na+
c Na+
(a) Interstitial defect
(b) Schottky defect
(c) Frenkel defect
(d) Frenkel & Schottky defects
15. An ionic compound has a unit cell consisting of A ions at the L
corners of a cube and B ions on the centres of the faces of
the cube. The empirical formula for this compound would be
(a) 39.27% (b) 68.02% (c) 74.05% (d) 78.54%
[AIEEE 2005]
24. A compound MpXq has cubic close packing (ccp) arrangement
(a) A 3B (b) AB3 of X. Its unit cell structure is shown below. The empirical
(c) A 2 B (d) AB formula of the compound is [IIT-JEE 2012]

16. Total volume of atoms present in a face-centred cubic unit


cell of a metal is (r is atomic radius) [AIEEE 2006]

12 3 16 3
(a) pr (b) pr
3 3 M
X
20 3 24 3
(c) pr (d) pr
3 3
17. In a compound, atoms of element Y form ccp lattice and those
of element X occupy 2/3rd of tetrahedral voids. The formula
of the compound will be [AIEEE 2008]
(a) X4Y3 (b) X2Y3 (c) X2Y (d) X3Y4
18. Copper crystallises in fcc with a unit cell length of 361 pm.
(a) MX (b) MX2 (c) M2X (d) M5 X14
What is the radius of copper atom? [AIEEE 2009]
25. The arrangement of X–ions around A+ion in solid AX is
(a) 127 pm (b) 157 pm
given in the figure (not drawn to scale). If the radius of X– is
(c) 181 pm (d) 108 pm 250 pm, the radius of A+ is [JEE Advanced 2013]
19. The edge length of a face centered cubic cell of an ionic
substance is 508 pm. If the radius of the cation is 110 pm, the
radius of the anion is [AIEEE 2010] –
X
(a) 288 pm (b) 398 pm (c) 618 pm (d) 144 pm
20. Percentages of free space in cubic close packed structure A+
and in body centered packed structure are respectively
[AIEEE 2010]
(a) 30% and 26% (b) 26% and 32%
(c) 32% and 48% (d) 48% and 26% (a) 104 pm (c) 183 pm (b) 125 pm (d) 57 pm
522 Chemistry

9. Argon crystallizes in a structure in which the atoms are


1. The radius of Li+ ion is 60pm and that of F- is 136 pm.
located at the postions (0, 0, 0)
Structure of LiF and coordination number is
(a) Like NaCl, C.No. = 6 (b) Like CsCl, C.No. = 8 æ 1 1ö æ1 1ö æ1 1 ö
ç 0, , ÷, ç ,0, ÷and ç , ,0 ÷ . The unit cell is
(c) Anti fluoride, C.No. = 8 (d) None of these è 2 2ø è2 2ø è2 2 ø
2. Atoms A are arranged in ccp array. Atoms B occupy all (a) simple cubic
octahedral voids and half of the tetrahedral voids. The formula (b) body-centred cubic
of compound AB is (c) face-centred cubic
(a) AB2 (b) A2B (d) hexagonal close packed
(c) AB (d) Al2B3 10. In NaCl, the centre-to-centre nearest-neighbour distance of
3. If Germanium crystallises in the same way as diamond, then ions is
which of the following statement is not correct? 1 3
(a) Every atom in the structure is tetrahedrally bonded to 4 (a) a (b) a
4 2
atoms.
(b) Unit cell consists of 8 Ge atoms and co-ordination number 1 1
(c) a 2 (d) a
is 4. 2 2
(c) All the octahedral voids are occupied. 11. Consider the radii 0.095 nm (Na+), 0.181 nm (Cl – ),
(d) All the octahedral voids and 50% tetrahedral voids remain 0.074 nm (Zn 2+), 0.184 nm (S 2– ), 0.068 nm (Ti 4+),
unoccupied. 0.140 nm (O2–), 0.169 nm (Cs+). Choose the correct option
from among the following. (Use radius ratio rules)
4. Amorphous solid may be classified as
(a) Na+ ions are packed in octahedral holes between the
(a) Isotropic and superheated solid planes of close-packed Cl– ions.
(b) Isoenthalpic and superheated liquid (b) Zn 2+ ions are packed in tetrahedral holes
(c) Isotropic and supercooled liquids (c) Cs + ions are packed in a simple cubic array of
(d) Anisotropic and supercooled liquids Cl– ions
5. In a solid lattice the cation has left a lattice site and is located (d) All of these
at an interstitial position, the lattice defect is : 12. Consider the following fcc unit cells choose the correct option
(a) Interstitial defect (b) Valency defect
(c) Frenkel defect (d) Schottky defect
6. Which of the following will not adopt CsCl structure ?
(a) CsF (b) CsBr
(c) CsS (d) CsCN
7. Sodium metal crystallises in bcc lattice with the cell edge, a = I II
4.29 Å. What is the radius of sodium atom?
(a) 1.79 Å (b) 1.89Å
(c) 4Å (d) 3.2Å
8. Which of the following statements is wrong ?
(a) The coordination number of each type of ion in CsCl
crystal is 8.
(b) A metal that crystallizes in bcc structure has a
III IV
coordination number of 12.
(c) A unit cell of an ionic crystal shares some of its ions with (a) I and II represent tetrahedral holes
other unit cells. (b) II, III and IV represent tetrahedral holes
(d) The length of the unit cell in NaCl is 552 pm. (c) I and II represent octahedral holes
(rNa+ = 95 pm; rCl–=181 pm). (d) I, II and IV represent octahedral holes
The Solid State 523
13. Which of the following is the incorrect statement 21. Which of the following is a correct statement ?
(a) NaCl has 6 : 6 coordination and CsCl has 8 : 8 coordination. (a) Bonding in metallic crystals is directional
(b) Diamond has two dimensional crystal lattice
(b) In Na 2O each oxide ion is coordinated by 8 Na + ions
(c) Co-ordination number of bcc lattice is 12
and each Na + ion by 4 oxide ions (d) A ccp structure has 8 tetrahedral and 4 Octahedral voids.
(c) NaCl structure transform to CsCl structure on heating 22. M 2 X has anti fluorite structure. In such structure
-
(d) In CaF2 structure each F ion is coordinated by 4 --
(a) X ions occupy all the 8 octahedral voids
Ca 2 + and vice versa.
(b) each X - - is surrounded by 4 M + in tetrahedral
14. The compound having the lowest lattice energy arrangement
(a) NaF (b) CsF
(c) KF (d) RbF (c) larger cations occupy the position of F - ions and smaller
anions that of C + + ions
15. In the unit cell of KCl, Cl - ions constitute ccp and K + ions
fall into the octahedral holes. These holes are (d) smaller cations occupy the position of F - ions and larger
(a) one at the centre and 12 are at the centres of the edges.
anions that of C + + ions
(b) one at the centre and 6 at the centres of the faces.
23. The coordination number X (........) of each ion in KBr is
(c) 8 at the corners of 8 small cubes forming the unit cell
changed to Y (..........) by.............
(d) None is correct
(a) X = 6 to Y = 8 applying higher temperature
16. The non stoichiometric compound Fe 0.94O is formed when (b) X = 8 to Y = 6 applying high pressure
(c) X = 6 to Y = 8 applying high pressure
x % of Fe 2 + ions are replaced by as many 2 Fe 3 + ions, x is (d) None of these
3
24. The unit cell of diamond is made up of
(a) 18 (b) 12
(a) 6 C atoms, 4 atoms constitute ccp and 2 atoms occupy
(c) 15 (d) 6
the half of octahedral voids
17. The incorrect statement for the sulphur atom of ZnS is
(b) 12 C atoms, 4 atoms form fcc lattice and 8 atoms occupy
(a) As S 2 - is larger than Zn 2 + only 4 rather than 6 or 8 all tetrahedral holes
(c) 8 C atoms, 4 atoms constitute ccp and 4 atoms occupy
S 2 - can be packed around Zn 2 + all the octahedral voids
(b) Its structure is similar to diamond except that alternate (d) 8 C atoms, 4 atoms form fcc lattice and 4 atoms occupy
atoms are Zn and S half of the tetrahedral voids alternately
(c) As S 2 - is larger than Zn 2 + only 6 rather than 8 or 4 25. CaO and NaCl have the same crystal structure and nearly the
same ionic radii. If x is the lattice energy of NaCl, the lattice
sulphide ions can be placed around Zn 2 + ions energy of CaO is very nearly
(d) ZnS is a covalent compound (a) x (b) 2x
18. In KBr crystal structure, the second nearest neighbour of
(c) x (d) 4 x
K + ion will be.............. and its number is 2
(a) K + , 12 (b) K+ ,6 26. Doping of AgCl crystals with CdCl 2 results in
(c) Br - , 12 (d) Br - , 6 (a) Frenkel defect
19. Which of the following expression is correct for CsCl unit (b) Schottky defect
cell with lattice parameter a (c) Substitutional cation vacancy
(d) Formation of F - centres
3a 3a
(a) r +r = (b) r +r = 27. NaCl is doped with 2 × 10 - 3 mole % of SrCl 2 . The
Cs+ Cl- 2 Cs+ Cl - 2
concentration of cation vacancies is
a r +r = 2a
(c) rCs+ + rCl- = (d) Cs + Cl- (a) 12.04 ´10 20 per mol
2
20. For a cubic crystal, the lattice parameter, a is 300 pm. The (b) 3.01´ 1018 per mol
spacing (d) for (111) plane will be
(c) 6.02 ´ 1018 per mol
(a) 212.1 pm (b) 259.8 pm
(c) 173.2 pm (d) 300 pm (d) 12.04 ´ 1018 per mol
524 Chemistry

28. NH 4 Cl crystallises in a bcc lattice with edge length of unit 32. Al (at. wt 27) crystallizes in the cubic system with a cell edge
of 4.05 Å . Its density is 2.7 g per cm 3 . Determine the unit
-
cell equal to 387 pm. If the radius of the Cl ion is 181 pm, cell type calculate the radius of the Al atom
+ (a) fcc, 2.432 Å (b) bcc, 2.432 Å
the radius of N H 4 ions is (c) bcc, 1.432 Å (d) fcc, 1.432 Å
33. The ratio of inner planar distancess of three types of planes
(a) 366.3 pm (b) 154.1 pm (d100 , d110 , d111 ) for simple cubic lattice are
(c) 92.6 pm (d) None of these
1 1
(a) 1 : 1 : 1 (b) : :1
29. A metal crystallizes in 2 cubic phases fcc and bcc whose unit
2 3 3 2
cell lengths are 3.5 Å and 3.0Å respectively. The ratio of their
densities is (c) 3 : 2 :1 (d) 1 : 2 : 3
(a) 0.72 (b) 2.04 34. In mineral MX 2 , M 2 + does ccp and X - occupy the
(c) 1.46 (d) 3.12 tetrahedral voids. The number of cations and anions per unit
cell, the coordination number of cation and percent of
30. In a cubic closed packed structure of mixed oxides, the lattice tetrahedral voids occupied are
is made up of oxide ion, 20% of tetrahedral voids are occupied (a) 8, 4, 8, 100 % (b) 8, 4, 8, 50%
(c) 4, 8, 8, 50% (d) 4, 8, 8, 100%
by divalent X 2 + ions and 50% of the octahedral voids are
35. Iron crystallizes in several modifications. At about 911°C,
occupied by trivalent Y 3 + ions. The formula of the oxide is the bcc ' a ' form undergoes a trasition to fcc ' g ' form. If the
distance between the two nearest neighbours is the same in
(a) X.Y2O 4 (b) X 4 Y5O10 the two forms at the transition temperature, the ratio of the

(c) X 5 Y4 O10 (d) X 2 YO 4 density of iron in fcc form (r2 ) to that of iron of bcc form
(r1 ) at the transition temperature
31. The ranks of atoms in face centred cubic and hexagonal unit
cells respectively are r1 r1
(a) = 0.918 (b) = 0.718
(a) 6, 4 (b) 4, 6 r2 r2
(c) 1, 6 (d) 2, 6 r1 r1
(c) = 0.518 (d) = 0.318
r2 r2
The Solid State 525

EXERCISE 1
14. (a) Number of unit cells = N A ´ Weight of subs tan ce
High Pressure M. Wt. ´ Z
¾¾¾¾¾
NaCl structure ¬¾¾¾¾¾ ® CsCl structure
1.
(CN 6 : 6) Heat to 760K (CN8 : 8) 6.023 ´ 10 23 ´ 1
= = 2.57 ´ 10 21 unit cells .
3. In the face-centred unit cell, there are eight tetrahedral voids. 58.5 ´ 4
Of these, half are occupied by silver cations. 15. (d) Na has Body centred cubic lattice.
4. Electrical conductivity increases with decrease of 16. (c) ABAB is hexagonal close packing.
temperature. 17. (b) In AB AB packing spheres occupy 74%. 26% is empty.
8. 128 pm. 18. (a) Arrangement of Cl– ions is fcc in NaCl
9. The formula of the compound will be = N3 M2 or M2N3. 19. (b) In bcc each atom has C. No. = 8.
18. (c) 19. (c) 20. (d) 21. (d) 22. (a) 20. (c) In CsI since Cs+ and I– have largest size.
23. (b) 24. (d) 25. (c) 26. (a) 27. (b) 21. (a) Potassium crystallises in BCC lattice.
22. (d) For octahedral r+ / r– = 0.414 – 0.732.
EXERCISE 2
23. (c) CsCl is body centred. Therefore 2r + + 2r – = 3.a body
1. (c)
Glass is amorphous solid. diagonal.
2. (a)
Covalent as in case of diamond.
3a
3. (c)
Covalent bond force (see text). \ r+ + r - = .
4. (b)
The volume to be filled by hard spheres in diamond is 2
0.34. 24. (c) Zn has hcp lattice.
5. (c) The p.f. for body centred cube = 0.68 (see text). 25. (c) In fluorite structure the C.No. of Ca++ is 8. See structure
6. (d) In NaCl structure we have particles at (8 corners + 6 faces) of simple ionic compounds.
one type. (12 edges + 1 in body) other type.
r+
A atoms are at eight corners. Removal of atoms along one 26. (b) Since C.No. is 6 it is octahedral and = 0.414 - 0.73 .
axis from face means removal of two A atoms. r-
27. (c) It is bcc hence, C. No. of each ion is 8.
1 1
\ Number of A atoms = ´8 + ´4 = 3
8 2 28. (c) Number of unit cells = N A ´ Wt. of subs tan ce
M. Wt. ´ Z
1
Number of B atoms = ´12 + 1 = 4 . \ A 3B 4 . 6.023 ´ 10 23 ´ 58.8
4 = = 1.5 ´10 23
58.8 ´ 4
1
7. (a) Number of A atoms = ´8 =1 Total number of molecules
8 29. (b) Number of unit cells =
(Z)
1
Number of B atoms = ´6 = 3 \Number of atoms = 12.08 ´ 1023 ´ 2 = 24.16 ´ 10 23
2
\ Formula AB3 (for bcc Z = 2)
8. (a) Seven crystal systems.
30. (b) r = Z ´ M = 2 ´ 100
9. (b) For tetragonal a = b ¹ c , a = b = g = 90° 3
NA ´ a 6.023 ´ 10 23
´ (400 ´ 10 -10 ) 3

10. (d) Match box is orthorhombic a ¹ b ¹ c; a = b = g = 90° = 5.188 g / cm 3


11. (d) Number of nearest neighbours means coordination 31. (a) 2r+ + 2r – = a; (edge length)
number = 12.
12. (c) Coordination number 6. (See Text) \ 2 ´110 + 2r - = 508 \ r - = 144 pm
32. (d) F-centres (see text.)
r+ 33. (c) Due to presence of F-centres.
13. (c) = 0.524 . It is in between 0.414 – 0.732.
r- 34. (b) It is stoichiometric defect and it is observed when equal
Hence C. No. = 6. number of cations and anions are missing from the lattice
site.
526 Chemistry
35. (c) AgBr exhibit Frenkel defect. 4. (d) The semiconductors formed by the introduction of
36. (c) KBr does not exhibit Frenkel defect. impurity atoms containing one elecron less than the
37. (d) No change in density (see text). parent atoms of insulators are termed as p-type
38. (b) From the given substances Li has high thermal & semiconductors. Therefore silicon containing 14
Electrical conductivity (due to metallic bond). electrons is to be doped with boron containing 13
39. (b) n-type, since electron is set free. electrons to give a p-type semi-conductor.
40. (d) Extrinsic semiconductor (see text).
5. (a) For bcc lattice body diagonal = a 3 .
41. (a) p-type semiconductors are formed by doping 14 group
elements with 13 group elements (valency 3). The distance between the two oppositely charged ions
42. (a) p-type semi-conductor. a
= 3
43. (b) It is the property of liquid crystal. 2
44. (a) It is due to movement of energised electrons 387 ´ 1.732
= = 335pm
(KE µ T). 2
45. (a) BaTlO3 (see text.) M+
46. (a) MnO2 (see text). 6. (c) is highest in CsF
47. (b) The smaller the size of cation and anion the more is X-
lattice energy Na+ has smaller size than others. \ correct choice : (c)
48. (b) Polymorphisms (see text). 7. (c) Radius ratio of NaCl like crystal
49. (d) nl = 2d sin q ; r+
= = 0.414
r-
æ 3ö 100
2 ´1Å = 2 ´ d sin 60 Þ d = 1.15Å ç sin 60 =
ç ÷ r- = = 241.5 pm
è 2 ÷ø 0.414
50. (a) Superconductors are derived from compound of p block 8. (a) For ccp 2 a = 4r
elements.
2 ´ 408
= 2r (2r = Diameter)
EXERCISE 3 2
1. (b) Since each Sr++ ion provides one cation vacancy, hence Diameter = 288.5
Concentration of cation vacancies = mole % of SrCl2 9. (b) Density is given by
added
Z´M
= 10–4 mole % d= ; where Z = number of formula units
-4 NA a 3
10
= ´ 6.023 ´ 1023 = 6.023 ´ 1017 mol–1 present in unit cell, which is 4 for fcc
100
2. (d) Number of atoms per unit cell = 1 a = edge length of unit cell. M = Molecular mass
a
Atoms touch each other along edges. Hence r = 4´M
2 2.72 =
( )
3
( r = radius of atom and a = edge length) 6.02 ´10 23 ´ 404 ´ 10 -10
4
pr
p
Therefore % fraction =
3 = (Q 1pm = 10 -10 cm)
3 6
(2 r )
3. (a) Following generalization can be easily derived for 2.72 ´ 6.02 ´ (404)3
various types of lattice arrangements in cubic cells M= = 26.99
4 ´107
between the edge length (a) of the cell and r the radius
of the sphere. = 27 gm mole–1
a 10. (a) In interstitial compounds small atoms like H, B & C
For simple cubic : a = 2r or r =
2 enter into the void sites between the packed atoms
For body centred cubic : of crystalline metal. They retain metallic conductivity
4 3 and are chemically inert.
a= r or r = a 11. (a) Diamond is like ZnS. In diamond cubic unit cell, there
3 4
are eight corner atoms, six face centered atoms and
For face centred cubic :
four more atoms inside the structure.
1
a = 2 2r or r = a Number of atoms present in a diamond cubic cell
2 2
Thus the ratio of radii of spheres for these will be 1 1
= 8´ + 6 ´ + 4 = 8
simple : bcc : fcc 8 2
a 3 1 (corners) (face (inside
= : a: a i.e. option (a) is correct answer.. centered) body)
2 4 2 2
The Solid State 527
12. (d) bcc - points are at corners & one in centre of the unit 22. (b) Among the given crystals, only silicon exists as a
cell. covalent solid. It has diamond like structure.
1 23. (d) Packing efficiency
Number of atoms per unit cell = 8 ´ + 1 = 2 .
8
fcc - points are at the corners and also centre of the six = Area occupied by circles within the square
Area of square
faces of each cell.
2pr 2 2pr 2
´ 100 = p ´100 = 78.54%
1 1
Number of atoms per unit cell = 8 ´ + 6 ´ = 4 . = ´ 100 =
8 2 L2 2( 2 r ) 2 4
13. (d) Number of formulas in cube shaped crystals
1
1. 0 24. (b) No. of M atoms = ´ 4+1=1+1=2
= ´ 6.02 ´ 10 23 since in NaCl type of structure 4 4
58.5
1 1
formula units form 'a' cell No. of X atoms =×6+ ×8=3+1=4
2 8
23
\ units cells = 1.0 ´ 6.02 ´10 = 2.57 × 1021 unit cells. So, formula = M2X4 = MX2
58.5 ´ 4 25. (a) The arrangement given shows octahedral void
14. (c) arrangement-limiting radius ratio for octahedral void is
15. (b) Each corners is shared by 8 cubes and each face is r
shared by 2 faces A+
= 0.414
r
1 X–
Number of A ions in the unit cell. = ´ 8 = 1
8 rA+ = 0.414 × rX– = 0.414 × 250 = 103.5 » 104 pm.
1
Number of B ions in the unit cell =´6 =3 EXERCISE 4
2
Hence empirical formula of the compound = AB3 r+ 60
1. (a) ratio is = 0.441, Hence LiF has NaCl structure
r- 136
16. (b) The face centered cubic unit cell contains 4 atom
with CN = 6.
4 16 2. (a) 3. (c)
\ Total volume of atoms = 4 ´ pr 3 = pr 3
3 3 4. (c) Amorphous solids are isotropic and supercooled liquids.
17. (a) From the given data, we have 5. (c) Frenkel defect is due to dislocation of ion from its usual
Number of Y atoms in a unit cell = 4 lattice site to interstitial position.
2 16
Number of X atoms in a unit cell = 8 ´ = 6. (a) CsF will not adopt CsCl structure due to small size of F- .
3 3
From the above we get the formula of the compound as 7. (b) For bcc lattice we have 2r + + 2r - = 3 a. In question
X16 / 3Y4 or X 4 Y3 we have 4r = 3 .a
18. (a) For fcc unit cell, 4r = 2a 3 ´ 4.29
2 ´ 361 \ r= = 1.89Å .
r= = 127 pm 4
4 8. (b) The crystals of CsCl has body - centred cubic unit cell. Hence,
19. (d) For an fcc crystal each ion in this structure has coordination number of eight.
edge length In case of crystals of NaCl two interpenetrating face-centred
rcation + ranion =
2 cubic lattices are present out of these one composed entirely
508 of Na+ ions and the other of Cl– ions. Each Na+ ion is located
110 + ranion =
2 half way between two Cl– ions and vice versa. A unit cell of
ranion = 254 – 110 = 144 pm NaCl crystal has Cl- ions at the corners as well as at the face
20. (b) Packing fraction is defined as the ratio of the volume centres and Na+ ions are located in octahedral voids. On
of the unit cell that is occupied by the spheres to the each edge of cubic unit cell, there are two Cl– ions and one
total volume of the unit cell.
P.F. for ccp and bcc are 0.74 and 0.68 respectively. (
Na+ ions. Hence a = 2 r + + r -
Na Cl
)
So, the free space in ccp and bcc are 26% & 32% = 2 (95 pm + 181 pm) = 552 pm
respectively. 9. (c) The planes indicate, the unit cell is fcc.
21. (d) For bcc structure 3 a = 4r 10. (d) In NaCl the Cl– and Na+ touch along edge of cube the
3 3 a
r= a= ´ 351 = 152 pm. distance between ions is
4 4 2
528 Chemistry
r+ 3
11. (d) Calculate the ratio to get the limiting ratio value and d1 (a 2 )3 z1 æ 3 ö 4
r- 29. (c) = ´ =ç ÷ ´ = 1.46
d 2 (a1 )3 z 2 è 3.5 ø 2
consult the table. All are correct.
12. (c) Examine the nearest number of neighbours in (I) and (II). 30. (b) Sphere Tetrahedral void Octahedral void
The number is six which is octahedral. O 2- X 2+ Y 3+
13. (c) CsCl (8 : 8 coordination) transform to NaCl (6 : 6
coordination) on heating but reverse is not possible. O OO O
14. (b) The coulombic attractions between Cs + and F - ions is 2 ´ 20 50
+ 1
lowest due to larger size of Cs . 100 100
15. (a) One octahedral void is present at the body centre of the 1 0.4 0.5
cube and 12 octahedral voids are present on the centres 10 4 5
of the cube. Hence formula X 4 Y5O10
16. (a) The number of Fe 3+ ions replacing x Fe2+ ions = 2 x 31. (b) No. of particles in face centered cubic unit cell
3 1 1
2x = ´ 8 (corners) + ´ 6 (faces) = 4
vacancies of cations = x – = x /3 8 2
3 No. of particles in hexagonal unit cell
But x/3 = 1 – 0.94 = 0.06, x = 0.06 × 3 = 0.18 = 18% 1 1
17. (a) Statement (a) is correct. = ´ 12 (corners) + ´ 2(faces ) + 3 (in the body) = 6
6 2
18. (a) The first nearest neighbour of K+ ion will be 6 Br - ions Z´ M
at a distance of a/2 and the second nearest neighbours 32. (d) r = ,
No ´ a3
a 2
will be 12 K+ ions at a distance of. Z ´ 27
2 2.7 = \ Z=4
19. (b) CsCl has a bcc structure ions touching along body 6.02 ´ 10 ´ ( 4.05) 3 ´ 10 -24
23

diagonal Hence it is face centred cubic unit lattice.


a a 3 300 3 Again 4r = a 2 = 5.727 Å
20. (c) d111 = = = = 173.2 pm
(12 + 12 + 12 )1/ 2 3 3 \ r = 1.432 Å
21. (d) The statement (d) is correct. a
33. (a) d hkl = 2
22. (d) The statement (d) is correct. ( h + k + l 2 )1 / 2
2
23. (c) The increase in pressure results in decrease in size of
1 1
ions (more in case of anion than cation), the r + / r - Hence d100 = a , d110 = , d111 =
increases and the coordination number also increase. 2 3
24. (d) The statement (d) is correct. 34. (d) The number of formula units in ccp (rank) = 4
Product of charges \ 4MX 2
25. (d) Lattice energy =
Interionic distance Hence number of cations (M 2 + ) = 4 ,
In NaCl the product of charges = 1 × 1 = 1 and -
In CaO it is 2 × 2 = 4 Anions (X ) = 8
The interionic distance is almost same. C. N. of cation = 8; All tetrahedral voids are occupied.
26. (c) The statement (c) is correct. 35. (a) In a – from distance between nearest neighbour atom is
27. (d) The addition of one Sr 2+ replaces 2 Na + and one 3a1
cationic vacancy is created. .
2
No. of cationic vacancy = 2 ´ 10 -3 mole % of NaCl In g form distance between nearest neighbour atom is
a2
2 ´10 -3 .
= mol -1 of NaCl 2
100
3 a1 a 2
= 2 ´ 10 -5 ´ 6.02 ´ 10 23 mol -1 \ = (given)
2 2
= 12.04 ´ 1018 mol -1 of NaCl
28. (b) For bcc lattice a2 3
or =
a1 2
2 r + +2 r = a 3
3 3
a 3 387 ´ 1.732 r1 z1 æ a2 ö 1 æ 3 ö÷
\r +r = = = 335.1 = ç ÷ = ç = 0.918
NH4
+
Cl- 2 r2 z2 ça ÷ 2 çè 2 ÷ø
2 è 1ø
or, r = 335 . 1 – 101 . 0 = 154 . 1 pm
Cl -
16
Solutions
SOLUTION : Soluble substances : Have solubility greater than
Homogeneous mixture of two or more non reacting components 1g per 100 ml
whose composition can be varied within certain limits is known as Insoluble substances : Have solubility less than
solution. 0.001 g per 100 ml
SOLUTE : Sparingly soluble substances : Have solubility less than 1g
The component which is present relatively in smaller proportion and more than 0.001 g per 100 ml.
in the solution is called solute.
(iii) Factors influencing solubility :
SOLVENT :
The component which is present in larger proportion in solution (a) Temperature : If the dissolution is endothermic the
is called solvent. solubility increases with temperature e.g. NaNO2, KNO3,
NaCl.
BINARY SOLUTION :
Containing two components only. If the dissolution is exothermic the solubility decreases
TYPES OF SOLUTIONS : with temperature e.g. Li2CO3, Na2CO3, Na2SO4, CeSO4
Depending upon the physical state of solute and solvent, the (b) Nature of solute/solvent.
solutions may be of the following types. SOLUBILITIES OF IONIC SOLIDS :
S.No. Solute Solvent Example The smaller the value of lattice energy and more the value of heat
1. Gas Gas gases in air of hydration more is the solubility of ionic compounds. Lattice
2. Gas Liquid Aerated Water energy depends upon force of attraction (F) between oppositely
3. Gas Solid Occluded hydrogen
qq'
4. Liquid Gas Moist air charged ions and is given by F = .
r 2D
5. Liquid Liquid Alcohol in water, Toluene + Benzene
6. Liquid Solid Mercury in Zinc amalgam q, q' are charges, r distance between charges, D is the dielectric
7. Solid Gas Smog constant of medium. Dielectric constant of water is 80, methanol
8. Solid Liquid Salt solution, Sugar solution 33.5, Benzene 2.3
9. Solid Solid Alloys Ionic solids dissolve more in a solvent having high dielectric
constant.
SOLUTIONS OF SOLIDS IN LIQUIDS :
(Solute = Solid and Solvent = Liquid) SOLUBILITIES OF MOLECULAR SOLIDS (NON
(i) Saturated solution : A solution which remains in contact IONIC SOLIDS) :
with excess of the solute is said to be saturated. Molecular solids containing polar groups (e.g. OH) are soluble in
(ii) Solubility : The amount of solute dissolved in 100 g of a polar solvents like water, methanol etc. Others are soluble in non
solvent to form a saturated solution at a given temperature is polar solvents.
termed as the solubility of solute.
530 Chemistry
SOLUBILITY OF GASES : (v) Molality (m) : Number of moles of solute dissolved in 1000 g
It is generally expressed in terms of absorption coefficient which of the solvent.
is defined as the volume of gas, reduced to NTP, dissolved by Molality (m)
unit volume of solvent at particular temperature and 1 atmospheric wt. of solute ´1000
pressure of the gas. =
M. wt of solute ´ wt. of solvent in grams
Absorption co-efficients of some gases at 20°C.
(vi) Mole fraction : The ratio of the number of moles of one
Solvent Hydrogen Nitrogen Oxygen Carbon dioxide component to the total number of moles of solution. For
Water 0.017 0.015 0.028 0.88 components A & B.
Ethanol 0.080 0.130 0.143 3.00 nA nB
Benzene 0.060 0.104 0.165 xA = ; xB = ; x A + x B = 1 always.
nA + nB nA + nB
FACTORS INFLUENCING THE SOLUBILITY OF A
(vii) Mass fraction :
GAS :
(i) Nature of the gas : Easily liquifiable gases are more soluble. Mass of A
Mass fraction of A = ;
e.g. CO2 is more soluble than O2 and H2 Total mass of solution
(ii) Nature of solvent : Gases capable of forming ions in aqueous
solution are more soluble in water than in other solvents. eg. Mass of B
Mass fraction of B =
HCl, NH3 etc. Total mass of solution
(iii) Temperature : Solubility decreases with rise of temperature (viii) ppm (parts per million) : It is the mass of the solute present
at constant pressure. in million (106) parts by mass of the solution.
(iv) Pressure of the gas (Henry's Law) Mass of A ´10 6
Henry's law : The mass of a gas dissolved per unit volume of ppm A =
Mass of solution
solvent is proportional to the pressure of the gas at constant
Molality, mole fraction and mass fraction do not change
temperature. with temperature.
m µ P or (ix) Demal (D) : When one mole of solute is present in one litre
m = k.P where k is Henry's constant. of the solution at 0°C, it is known as one demal.
Henry's Law in terms of volume : The volume of a gas dissolved (x) Formality (f) : Number of formula mass in grams present in
in a solvent at a given temperature is independent of the pressure. one litre of solution.
Most gases obey Henry's law : SOME IMPORTANT RELATIONS :
(a) when pressure is not very high
(b) temperature is not very low md æ1 M ö
(i) M= or d = Mç + 2 ÷
(c) gas is not highly soluble M è m 100 ø
1+ m 2
(d) gas does not form any compound with solvent 1000
(e) gas does not undergo dissociation. 1000x 2
(ii) m =
EXPRESSION OF THE CONCENTRATION OF A x1M1
SOLUTION :
1000 dx 2
(i) Percentage : (iii) M =
(a) By weight - weight of solute per 100 g of solution. x1M1 + x 2 M 2
(b) By volume - weight of solute per 100 ml of solution.
Volume of solute per 100 ml of solution. m
(iv) Mole fraction ( x 2 ) =
(ii) Strength : Number of grams of solute dissolved per litre of 1000
m+
solution. (g/litre) M = molarity M1
Strength = Equivalent weight × Normality m = molality
(iii) Normality (N) : Number of gram equivalents of the solute
d = density of solution
dissolved per litre of the solution.
x1 = mole fraction of solvent
wt. of solute ´1000 x2 = mole fraction of solute
Normality (N) =
E. wt of solute ´ volume in mL M1 = molecular mass of solvent
(iv) Molarity (M) : Number of moles of the solute dissolved in M2 = molecular mass of solute
one litre of solution. Normality = n × Molarity
wt. of solute ´1000 Molecular Mass
Molarity (M) = where n =
M. wt of solute ´ volume in mL Equivalent mass
Solutions 531
VAPOUR PRESSURE : when components are mixed to form the solution (III) DVmix = 0
It is defined as the pressure of the vapour in equilibrium with the i.e. no change in volume. In ideal solution the A–B intermolecular
liquid at that temperature interactions are the same as A–A and B–B inter-molecular
interactions. Strictly, there is no ideal solution. The following pairs
(i) Vapour pressure of a liquid increases with rise in temperature. almost behave as ideal solutions.
(ii) When intermolecular forces are weak, the liquid is more (a) benzene and toluene
volatile and has more vapour pressure. (b) ethyl bromide and ethyl chloride
VARIATION OF VAPOUR PRESSURE WITH (c) n-Heptane and n-hexane
TEMPERATURE : (d) chlorobenzene and bromobenzene
(e) ethylene chloride and ethylene bromide.
It is given by Clapeyron-Clausius Equation
For ideal solutions the vapour pressure of the solution always
P2 DH v é1 1 ù lies between the vapour pressure of the pure components and
log = ê - ú generally dilute solutions behave as ideal solutions.
P1 2.303R T
ë 1 T2û
NON IDEAL SOLUTIONS :
DHv = Enthalpy of vapourisation of liquid; R = gas constant; P2 =
vapour pressure at T2 and P1= vapour pressure at T1. Solutions which (I), do not obey Raoult's law, (II) DHmix ¹ 0 (III)
DVmix ¹ 0.
VAPOUR PRESSURE OF SOLUTION :
The vapour pressure of a solution is the sum of the partial vapour TYPES OF NON IDEAL SOLUTIONS :
pressures of the components of the solution. (Dalton's law) They are of two types
(i) Showing positive deviations : For such solutions (a) A–B
P = p A + p B + p C ......... etc . inter-molecular interactions are weaker than A–A and B–B
In general, the vapour pressure of a component in solution is less intermolecular interactions (b) DHmix is +ve (c) DVmix is +ve
than that of pure solvent. Examples :
Carbon tetrachloride + benzene
RAOULT'S LAW :
Carbon tetrachloride + chloroform
(i) The partial pressure of a volatile component of a solution is Carbon tetrachloride + Toluene
directly proportional to its mole fraction in solution at any
Acetone + Carbon disulphide
temperature.
Acetone + Ethyl alcohol
p A µ x A and p B µ x B and Acetone + Benzene
Methyl alcohol + Water
p A = p oA .x A , p B = p oB .x B Ethyl alcohol + Water
where pA = partial pressure of component A, (ii) Showing negative deviations : For such solutions (a) A–B
intermolecular interactions are stronger than A–A and B–B
p oA = vapour pressure of component A in pure form, intermolecular interactions (b) DHmix is –ve (c) DVmix is –ve.
xA = mole fraction of component A in solution. Examples :
Chloroform + Acetone
Also remember, p A = p ´ g A , where p = Total pressure Chloroform + Benzene
and gA = Mole fraction of component A in vapour phase Chloroform + Diethyl ether
(ii) Raoult's law for solution containing non-volatile Acetone + Aniline
solute : The relative lowering of vapour pressure of a solution HCl + Water
containing non volatile solute is equal to the mole fraction of HNO3 + Water
p oA - p A n AZEOTROPIC MIXTURE OR CONSTANT BOILING
non volatile solute = xB =
p oA n+N . MIXTURE :
A mixture of two or more components which like a pure chemical
For very dilute solution :
compound boils and distills over completely at the same
poA - pA n temperature without change in their chemical composition is called
Alternate formula = (Q n << N ) an azeotrope. Non ideal solutions form azeotropes.
poA N
AZEOTROPIC MIXTURE WITH MINIMUM BOILING
A = Volatile component of solution,
POINT :
B = non-volatile component of solution or the solute
n = moles of B, N = moles of A. It is formed by liquids showing positive deviation. An intermediate
composition of liquids having highest vapour pressure, hence
AN IDEAL SOLUTION : lowest boiling point gives this azeotrope. Such azeotropes have
The solution which (I) Obey Raoult's law at all temperatures and boiling points lower than either of the pure component e.g.
concentrations (II) DHmix = 0 i.e. no heat is evolved or absorbed Rectified spirit (ethanol 95.5% + H2O 4.50%) bpt 351.5 K.
532 Chemistry
AZEOTROPIC MIXTURE WITH MAXIMUM BOILING These are
POINT : (i) Relative lowering of vapour pressure
It is formed by liquids showing negative deviation. An intermediate (ii) Osmotic pressure
composition of liquids having minimum vapour pressure, hence (iii) Elevation in boiling point
highest boiling point gives this azeotrope. Such azeotropes have (iv) Depression in freezing point
boiling points higher than either of the pure components e.g.
Water & HNO3 (HNO3 68% + H2O 32%) bpt 393.5K. RELATIVE LOWERING OF VAPOUR PRESSURE :
It is measured by Ostwald and Walker's method (Gas saturation
GRAPHICAL REPRESENTATION OF IDEAL AND
method)
NON IDEAL SOLUTIONS :
Ideal Solution po - p w2
= mole fraction of non volatile solute = w + w
po 1 2

w2 = loss in weight of solvent


w1 = loss in weight of solution
0
pA OSMOSIS :
V.P. p = p A + pB
pB
0
pA The spontaneous flow of the solvent through a semi-permeable
membrane from pure solvent to solution or from a dilute solution
to concentrated solution is known as osmosis.
pB
Some natural semipermeable membranes are pig's bladder, skin
xA = 0 xA = 1 round white of an egg, membrane round the red blood corpuscle
xB = 1 Mole fraction xB = 0 and in the cell of the plant.
Cupric ferrocyanide Cu2[Fe(CN)6] is artificially prepared
Positive Deviation semi-permeable membrane
(CuSO4 + K4 [Fe(CN)6])

2Cu + + + Fe (CN ) 46 - ¾
¾® Cu 2 [ Fe( CN ) 6 ] ¯
It is not suitable for non aqueous solution as it gets dissolved.
p = pA + pB 0
pA Another artificial semipermeable membrane is Ca3(PO4)2.
V.P.
pB
0
pA OSMOTIC PRESSURE (p) :
The hydrostatic pressure developed on the solution which just
pB prevents the osmosis of pure solvent into the solution through a
semipermeable membrane is called osmotic pressure.
xA = 0 xA = 1
xB = 1 Mole fraction xB = 0
p

Negative Deviation h p = hrg

Solvent
0
V.P. pA Solution
pB
0
p = p A + pB
pA Semipermeable
membrane
pB p = hrg
xA = 0 xA = 1
xB = 1 Mole fraction xB = 0
=hrg
COLLIGATIVE PROPERTIES : Or
Properties which depend upon the number of particles irrespective The minimum external pressure applied to the solution to just
of their nature (ions or molecules) are called colligative properties. stop the process of osmosis is known as osmotic pressure.
Solutions 533
RELATION BETWEEN OSMOTIC PRESSURE (p) AND
patm p patm LOWERING OF VAPOUR PRESSURE :
Relative lowering of vapour pressure is directly proportional to
osmotic pressure
po - p p´M po - p æ M ö
o
= \ o
µ p ç since Constant ÷
p dRT p è dRT ø
SPM

OSMOLARITY :
Solution Solvent Osmotic behaviour of solutes undergoing association or
dissociation is equal to molarity × number of particles produced
Or per formula unit of solute known as osmolarity.
The minimum external pressure applied on solution to make its REVERSE OSMOSIS :
vapour pressure equal to that of solvent is known as osmotic When the external pressure applied on the solution is more than
pressure. osmotic pressure, the solvent flows from the solution to the pure
DETERMINATION OF OSMOTIC PRESSURE : solvent which is called reverse osmosis. Desalination of sea water
(i) Pfeffer's method is done by reverse osmosis to make it potable.
(ii) Berkeley and Hartley's method (most common) PLAMOLYSIS :
(iii) Morse and Frazer's method The flow of the fluid from the plant cell when placed in a hypertonic
(iv) Townsend's negative pressure method solution is called plamolysis. The plant cell undergoes shrinkage.
(v) De. Vries method EXO-OSMOSIS :
ISOTONIC SOLUTIONS : Outward osmotic flow of fluid from a cell through semipermeable
Solutions having the same osmotic pressure. 0.91% NaCl solution memberane e.g. grape in conc. NaCl solution.
is isotonic with human RBC's. ENDO-OSMOSIS :
HYPOTONIC SOLUTION : Inward osmotic flow of fluid from a cell through semipermeable
Solution having lower Osmotic pressure. membranes e.g. grape in water.
HYPERTONIC SOLUTION : SILICA GARDEN :
Solution having higher osmotic pressure. When coloured salts e.g. Copper Sulphate, Cobalt Nitrate,
Manganese Chloride, Nickel Chloride and Ferrous sulphate etc.
VAN'T HOFF'S THEORY OF DILUTE SOLUTIONS : are placed in Sodium silicate solution of density 1.1, a colloidal
Dilute solutions behave like gases and gas laws can be applied to and semipermeable shell of Silicates is formed round the crystal.
them. Inside this, there is strong solution of salt and outside weak
BOYLE-VAN'T HOFF LAW : solution of sodium silicate. Water permeates into the shell, pressure
rises and the shell bursts out, the salt solution escapes but
Osmotic pressure of a solution is directly proportional to its
immediately reacts with sodium silicate to form again
concentration at constant given temperature.
semipermeable shell of the metallic silicate. The process is repeated
1 again and again and projection of coloured silicates continues to
pµCµ
V grow.

CHARLE'S-VAN'T HOFF LAW : KONOWALOFF'S RULE :


Osmotic pressure of a solution is directly proportional to its The mole fraction of the more volatile component is always greater
temperature at fixed concentration. in the vapour phase than in the solution phase.
pµT RELATION BETWEEN THE TOTAL PRESSURE AND
MOLE FRACTIONS OF VOLATILE COMPONENTS IN
VAN'T HOFF EQUATION FOR SOLUTIONS :
LIQUID PHASE :
p µ CT \ p = SCT
S = Solution constant having same value as gas constant P = p oA .x A + p oB .x B
C = Molar concentration RELATION BETWEEN THE TOTAL PRESSURE AND
T = Absolute temperature and p = Osmotic pressure MOLE FRACTIONS OF VOLATILE COMPONENTS IN
AVOGADRO-VAN'T HOFF LAW FOR SOLUTIONS : VAPOUR PHASE :
Equal volumes of solutions having the same osmotic pressure
1 YA YB
and temperature contain equal number of moles i.e. have same = +
concentration. P p oA p oB
534 Chemistry
BOILING POINT AND ELEVATION IN BOILING Carbon tetrachloride 5.03
POINT : Chloroform 3.63
The temperature at which the vapour pressure of a liquid becomes Ethyl alcohol 1.20
equal to atmospheric pressure is known as the boiling point of the Ethyl ether 2.11
liquid.
Addition of non volatile solute in a solvent lowers the vapour
DETERMINATION OF MOLECULAR WEIGHT FROM
pressure and increases the boiling point. The increase in boiling BOILING POINT ELEVATION :
point is directly proportional to the molality of solution,
K b ´ w ´1000
Boiling point of M=
DTb ´ W
Solvent
Solution Kb = molal elevation constant,
1.013 bar
or 1Atm w = weight of solute,
nt W = weight of solvent,
olve n
S io DTb = Elevation in bpt.
lut
Vapour pressure

So The method employed is known as ebullioscopic method. The


common methods are
(i) Landsberger's method
(ii) Coltrell's method
FREEZING POINT AND DEPRESSION IN FREEZING
Temperature/ K POINT :
The temperature at which the vapour pressure of the solvent in its
The diagram shows that DTb denotes
liquid and solid phase becomes the same is known as freezing
the elevation of boiling point of a
point of solvent.
solvent in solution.

nt
K ´ w ´ 1000 solve
DTb µ m or DTb = K b ´ m = b id
M´W liqu tion
Kb = molal elevation constant or ebullioscopic constant. It is solu
nt
characteristic for given solvent. When olve
s
Vapour pressure

m = 1, DTb = Kb. zen


fr o
Thus molal elevation constant or ebullioscopic constant is defined
as the elevation in boiling point caused by one molal solution. Its
value is given by

RT02 M A .RT02 Temperature/ K


Kb = =
1000 l v DH vap ´1000 Diagram showing DTf, depression
of the freezing point of a solvent
Units of Kb = K kg mol–1 in a solution.
R = gas constant, T0 = normal boiling point lv = latent heat of
vaporisation per gram of pure solvent. Kb is constant for given When non volatile solute is dissolved in a solvent, the freezing
solvent. point decreases. Depression in freezing point is given by
MA = molecular mass of solvent, DH v = Enthalpy of vaporisation DTf µ m
of solvent
MOLAL ELEVATION CONS TANTS OF S OME K f ´ w ´ 1000
DTf = K f ´ m =
COMMON SOLVENTS : M´ W
Solvents Value of Kb in K kg mol–1 Kf is molal depression constant or molal Cryoscopic constant.
Water 0.52 When m (molality) = 1 then DTf = K f
Methyl alcohol 0.80 Hence molal depression constant or molal Cryoscopic constant
Acetone 1.72 is equal to depression in freezing point caused by one molal
Benzene 2.53 solution. Its value is given by
Solutions 535

RT02 M A RT02 1
Kf = = Units of Kf = K . Kg mol–1 Colligative property µ , hence higher
1000 lf DH fusion ´ 1000 Molecular mass of solute
values are obtained for molecular masses in case of association.
R = gas constant,
Acetic acid in benzene associates and its observed molecular
T0 = normal freezing point, mass is 120.
lf = heat of fusion per gram of solvent.
O--H—O
Kf is freezing point constant for a given solvent.
MOLAL DEPRESSION CONSTANTS OF SOME CH3–C C–CH3
COMMON SOLVENTS :
O—H--O
Water 1.86 Naphthalene 6.90
Acetic acid
Acetic acid 3.90 Bromoform 14.40
Phenol 7.27 Cyclohexane 20.00 Benzoic acid in benzene associates and its observed molecular
mass is 244.
Benzene 5.12 Camphor 37.70
DETERMINATION OF MOLECULAR WEIGHT BY O--H—O
DEPRESSION IN FREEZING POINT :
C6H5–C C–C6H5
K ´ w ´1000
M= f O—H--O
DTf ´ W
Benzoic acid
Kf = Molal depression constant, w = weight of non-volatile solute,
DTf = depression in freezing point,W = weight of solvent KCl in water dissociates and its observed molecular mass
The method employed is known as Cryoscopic method. The is 37.25.
common methods are + –
KCl K + Cl
(i) The Beckmann method
These observed values are corrected by multiplying by Van't Hoff
(ii) The Rast Method
factor i.
RELATION BETWEEN OSMOTIC PRESSURE AND
VAN'T HOFF FACTOR (i ) :
OTHER COLLIGATIVE PROPERTIES :
Its value is obtained by any of the following expression
æ p o - p ö dRT
(i) p = ç A o A ÷´ Normal molecular mass
ç p ÷ M Relative lowering of vapour pressure (i) i=
è A ø B
Observed molecular mass

dRT Observed Value of Colligative property


(ii) p = DTb ´ Elevation in boiling point (ii) i = Calculated Value of Colligative property
1000K b

dRT No. of particles after association or dissociation


(iii) p = DTf ´ Depression in freezing point (iii) i = No. of particles before association or dissociation
1000K f
p = Osmotic pressure, DEGREE OF DISSOCIATION :
d = Density of solution at temperature T, Fraction of total number of molecules undergoing dissociation,
R = Gas constant, i -1
a=
MB = Molecular mass of solute, m -1
Kb = Molal elevation constant of solvent, m = number of particles of electrolyte given in solution.
Kf = Molal depression constant of solvent. i = Van't Hoff factor i.
ABNORMAL MOLECULAR MASS AND COLLIGATIVE DEGREE OF ASSOCIATION :
PROPERTIES : Fraction of the total number of molecules undergoing association.
When solutes undergo association or dissociation in solution, i -1
there is decrease or increase in number of particles and there are a=
1
discrepencies between observed and calculated values of -1
m
colligative properties.
536 Chemistry
RELATION BETWEEN RELATIVE LOWERING OF CONJUGATE SOLUTIONS :
VAPOUR PRESSURE AND MOLALITY OF SOLUTION: When two partially miscible liquids A and B are mixed with each
other, two layers are formed. First, Solution of A in B and Second,
pº -p m ´ M1 Solution of B in A. Such solutions co-existing together are called
=
pº 1000 conjugate solutions. For example solution of phenol and water.
CRITICA L SOLUTION TEM PERATURE O R
Where m = molality of solution CONSOLUTE TEMPERATURE :
M1 = Molecular weight of solvent On heating the conjugate solution the mutual solubility of A and
B increases. A temperature is reached when two phases disappear
RELATION BETWEEN WEIGHTS OF TWO
and one homogeneous phase is left. This temperature is known
IMMISCIBLE LIQUIDS IN THE DISTILLATE :
as Critical solution temperature or Consolute Temperature. It is
of two types :
w A pA ´ M A
= (i) Upper Consolute temperature : For example water-phenol
wB pB + p B
system.
where w = weight of liquid (ii) Lower Consolute temperature : In some cases it is also
possible to have a lower temperature of similar kind. For
p = partial pressure
example Trimethylamine-water is a lower consolute
M = Molecular weight temperature.

1. The solublity of KCl in water is 45.5 g per 100 g water at 55°C.


50
What is the mass percent of KCl. Moles of ethanoic acid = = 0.833
60
mass of KCl
Sol. Mass % of KCl = ´100 Total moles nA + nB + nC = 1.388 + 0.543 + 0.833 = 2.764
Total mass of all components
1.388
Mole fraction of water = = 0.5022
45.5 2.764
= ´ 100 = 31.27
45.5 + 100
0.543
Mole fraction of ethanol = = 0.196
2.764
2. Find the molarity and molality of a 15% solution of H2SO4
(density of H2SO4 = 1.10 g/ml). 0.833
Mole fraction of ethanoic acid = = 0.301
Wt ´ 1000 15 ´ 1000 2.764
Sol. Molarity = = = 1.53 M 4. The density of a 3M Sodium thiosulphate (Na2S2O3) is
M. Wt ´ Volume 98 ´ 100
1.25 g/l. Calculate (I) the percentage by mass of sodium
Wt of solute ´1000 thiosulphate (II) the mole fraction of sodium thiosulphate
Molality = M. Wt ´ Wt. of solvent (III) molalities of Na+ and S2O32– ions.
Sol. Mass of 1000 ml. of Na2S2O3 solution = 1000 × 1.25 = 1250 g
Mass of 100 ml soln. = 1.10 × 100 = 110 g (I) Mass of Na2S2O3 in 1000 ml = 3 × 158 (M. Wt) = 474 g
Mass of solvent=110 – 15 = 95 g
474 ´ 100
Mass % of Na2S2O3 = = 37.92
15 ´ 1000 1250
m= = 1.64
98 ´ 95 (II) Moles of Na2S2O3 = 3
3. A solution contains 25% water, 25% ethanol and 50% ethanoic Mass of water = (1250 – 474) = 776 g
acid by mass. Calculate the mole fraction of each component. 776
Moles of water = = 43.1
25 18
Sol. Moles of water n A = = 1.388
18 Total moles = 3 + 43.1 = 46.1

25 3
Moles fraction of Na2S2O3 = = 0.065
Moles of ethanol n B = = 0.543 46.1
46
Solutions 537
(III) Number of moles of Na+ ions = 2 × 3 = 6
17.3555 - 17.2350 10 18
6 ´ 1000 or = ´
Molality of Na+ = = 7.73 m 17.3555 m 100
776
Number of moles of S2O32– ions = 3 1.8
0.006943 =
3 ´ 1000 m
Molality of S2O32– = = 3.86 m
776 \ m = 259.25
5. 0.080575 kg of Glauber's Salt is dissolved in water to obtain
1 dm3 of solution of density 1.0772 kgdm–3. Calculate the p o - ps w´M
(b) =
molarity, molality and mole fraction of Na2SO4 in solution. p o m´ w
Sol. Molecular mass of Glauber's salt = 322 g/mol = 0.322
kg mol–1 23.62 - Ps 1.5 ´ 18
or =
0.080575 23.62 60 ´ 50
Moles of Glauber's salt = = 0.2502
0.322
\ Ps = 23.407 mm Hg
Moles of Na2SO4 will be same i.e. 0.2502.
8. The vapour pressure of water at 20°C is 17.54 mm Hg. When
0.2502 20g of solute was added in 50 g water, the V.P. was found to
\ Molarity of solution = = 0.2502 M
1 lower by 0.30 mm Hg. Calculate M wt of solute.
Mass of 1 litre solution = V × d = 1 dm 3 × 1.0772 kg dm–3
= 1.0772 kg po - ps w´M
Sol. =
Mass of Na2SO4 = 0.2502 × 142 (M. wt of Na2SO4 = 142) p o m´ W
= 0.03552 kg
Mass of solvent = (1.0772 – 0.0355) Kg = 1.0417 kg 0.30 20 ´ 18
or =
0.2502 17.54 m ´ 50
Molality m = = 0.24 m
1.0417
\ m = 420.96
1041.7 9. Find the molality of a solution containing non volatile solute
Moles of solvent = = 57.87
18 if the vapour pressure is 2% below the vapour pressure of
pure water. (DHANBAD 1990)
0.2502
Mole fraction of Na2SO4 = = 4.3 ´ 10 -3
0.2502 + 57.87 p o - ps m´M
Sol. Apply o
=
6. How much concentrated H2SO4 of density 1.84 g cm–3 which p 1000
contains 98 mass percent of H2SO4 must be diluted to prepare
where m = molality of solution;
100 cm3 of the 20 mass percent H2SO4 of density 1.14 g cm–
3.
M = Molecular weight of solvent.

Sol. Molarity of first sample of H2SO4 2 ´1000


\m = = 1.111
100 ´18
98 ´ 1000 ´ 1.84
M= = 18.4
98 ´ 100 2 m ´18
or =
Molarity of second sample of H2SO4 100 1000

20 ´ 1000 ´ 1.14 2 ´1000


M= = 2.32 ; M1V1 = M2V2 \m = = 1.111
98 ´ 100 100 ´18
10. An aqueous solution of glucose boils at 100.01°C. The Kb
V × 18.4 = 2.32 × 100 V = 12.6 cm3
for water is 0.5 K mol–1 Kg. What is the number of glucose
7. (a) 10g of a certain non-volatile solute were dissolved in
molecules in the solution containing 100 g water.
100 g H2O at 20°C. The vapour pressure was lowered
from 17.3555 mm Hg to 17.2350 mm Hg. Calculate K b ´ w ´ 1000 0.5 ´ w ´ 1000
Sol. DTb = ; 0.01 =
molecular wt. of solute. M´W 180 ´ 100
(b) The vapour pressure of pure water at 25°C is 23.62 mm Hg w = 0.36 g
what will be vapour pressure of a solution of 1.5 g of 180 g glucose contains = 6.023 × 1023 molecules
urea in 50 g of water.
6.023 ´1023
0.36 g glucose contains = ´ 0.36
p o - ps n w M 180
Sol. (a) = = ´
o N m W
p = 1.2 ´ 1021 molecules
538 Chemistry
11. The freezing point of a solution containing 50 cm3 of ethylene 15. A 1.2% solution (wt/volume) of NaCl is isotonic with 7.2%
glycol in 50 g of water is found to –34°C. Assuming ideal solution (wt/volume) of glucose. Calculate degree of
behaviour calculate the density of ethylene glycol. ionisation and Van't Hoff factor of NaCl.
Kf for water = 1.86 K kg mol–1 Sol. Since glucose is non electrolyte its osmotic pressure is
Sol. Ethylene glycol = 50 cm3
Weight of ethylene glycol = 50 × d
æ 7.2 ´ 1000 ö æ 7.2 ´ 1000 ö
Molecular weight of ethylene glycol = 62 g p1 = CRT = ç ÷ ´ 0.0821 ´ T çQ C = ÷
è 180 ´ 100 ø è 180 ´ 100 ø
Weight of water = 50 g
NaCl is electrolyte and let a be the degree of ionisation then
K ´ w ´ 1000 1.86 ´ 50 ´ d ´ 1000
DTf = f ; 34 = NaCl Na+ + Cl–
M´W 62 ´ 50
d = 1.133 g/cm3 1–a a a
12. Calculate the freezing point of an aqueous solution of non 1- a + a + a
electrolyte having an osmotic pressure 2.0 atm at 900 K. Van't Hoff's factor i = = 1+ a
1
Kf = 1.86 K mol–1 kg and S = 0.0821 litre atm K–1mol–1.
Osmotic pressure of NaCl is
Sol. p = CST where C = Molar concentration, p = 2 atm,
S = 0.0821 litre atm K–1 mol–1, T = 300 K æ 1.2 ´ 1000 ö
p2 = (1 + a) ç ÷ ´ 0.0821´ T
\ 2 = C × 0.0821 × 300 \ C =0.0812 mol lit–1 è 58.5 ´ 100 ø
DTf = Kf × molality
For isotonic solutions p1 = p 2
For dilute solutions molarity = molality
DTf = 1.86 × 0.0812 = 0.151 æ 7.2 ´ 1000 ö æ 1.2 ´ 1000 ö
ç ÷ ´ 0.0821´ T = ç ÷ ´ 0.0821´ T (1 + a)
Freezing point = T – DTf = 0 – 0.151 = –0.151°C è 180 ´ 100 ø è 58.5 ´ 100 ø
13. A decimolar solution of potassium ferrocyanide is 50%
dissociated at 300K. Calculate the osmotic pressure of the \ a = 0.95 and i = 1.95
solution. Given S = 8.314 JK–1 mol–1. 16. The degree of dissociation of Ca(NO3)2 in a dilute aqueous
solution containing 7g of salt per 100 g of water at 100°C is
0.1
Sol. Given C = 0.1 mol lit–1 = = 10 2 mol m -3 70%. Calculate the vapour pressure of solution.
10 -3
Sol. Ca(NO3)2 Ca++ + 2 NO3–
p = 102 ´ 8.314 ´ 300 Nm -2 1 – 0.7 0.7 2 × 0.7
But, K4 [Fe(CN)6] 4K+ + [Fe(CN)6]4– Van't Hoff factor i = 1 – 0.7 + 0.7 + 1.4 = 2.4

1 – 0.5 4×0.5 0.5 Normal M.wt


Van't Hoff factor i =
Obs. M.wt
1 - 0.5 + 2.0 + 0.5
Van't Hoff's factor i = =3
1 164
\ Obs. M.wt = = 68.33
-2 2.4
\p = 10 ´ 8.314 ´ 300 ´ 3 = 7.483 ´ 10 Nm
2 5

p o - ps w M 7 ´ 18
14. 0.85%, aqueous solution of NaNO3 is apparently 90% Now = ´ = = 0.0184
p o m W 68.33 ´ 100
dissociated at 27°C. Calculate the osmotic pressure.
(R = 0.0821 atm K–1 mol–1)
Sol. Molecular weight of NaNO3 = 85 po - ps
= 0.0184
po
Wt ´1000 0.85 ´ 1000
Molarity = = = 0.1 mol lit -1
M. wt ´ V 85 ´ 100 760 - ps
= 0.0184
NaNO3 solution is 90% dissociated 760
NaNO3 Na+ + NO3– \ ps = 746.01 mm Hg
1 – 0.9 0.9 0.9 17. To 500 cm3 of water 3.0 × 10–3 Kg of acetic acid is added. If
23% acetic acid is dissociated, what will be the depression in
Van't Hoff factor i = 1 – 0.9 + 0.9 + 0.9 = 1.9
freezing point? Kf and density of water are 1.86K kg–1 mol–1
\ p = 1.9 ´ 0.1 ´ 0.0821 ´ 300 = 4.68 atm and 0.997 g cm–3 respectively.
Solutions 539
Sol. Mass of acetic acid = 3 × 10–3 kg = 3.0 g
x 2 ´1000
Molecular mass of acetic acid = 60 g now apply the formula m =
x1M1
Mass of Solvent = 500 × 0.997 = 498.5 g
CH3COOH CH3COO– + H+ 1000 ´ 0.012
m= ;
1 – 0.23 0.23 0.23 i = 1.23 0.988 ´ 78
3 ´ 1000 m = 0.156 mol kg -1
\ DTf = K f ´ molality ´ i = 1.86 ´ ´ 1.23 = 0.229
60 ´ 498.5
21. Addition of 0.643 g of a compound to 50 ml of benzene
18. Calculate the molality of a 1 litre solution of 93%
(density 0.879 g mol -1 ), lower the freezing point from 5.51°C
H 2SO 4 (mass/volume). The density of the solution is 1.84
g/ml to 5.03°C. If Kf for benzene is 5.12 kg mol -1 , Calculate the
molecular mass of the compound
93 ´ 1000
Sol. Molarity of 93% H 2SO 4 is = = 9.48
98 ´ 100 K f ´ w ´ 1000
Sol. Molecular mass =
The relation between M and m is as follows DTf ´ W
m´ d 5.12 ´ 0.643 ´ 1000
M= ; =
m M2 0.48 ´ 50 ´ 0.879
1+
1000 = 156.06 g/mol
m ´1.84 ( DTf = (5.51 – 5.03) = 0.48 and Mass of solvent
9.48 =
m ´ 98
1+ = V ´ d = 50 × 0.879)
1000 22. At 100°C benzene and toluene have vapour pressure of 1375
On solving and 558 torr, respectively. Assuming these two form an ideal
m = 10.30 solution calculate the composition of the solution that boils
19. What mass of non volatile solute urea ( NH 2 CONH 2 ) must at 1 atm and 100°C. What is the composition of vapour issuing
at these conditions.
be dissolved in 100 g of water in order to decrease the vapour
pressure of water by 25%. What will be the molality of the Sol. Let xAand xB be the mole fraction of benzene and toluene
solution? respectively
Sol. Let the mass of urea be w g o
then, P = x A p A + (1 - x A )p Bo
Now use the alternate formula
760 = x A × 1375 + (1 – x A ) 558
p0 - p n
= x A = 0.247
p N
Mole fraction of benzene in the vapour phase
100 - 75 W / 60 o
= x ApA 0.247 ´ 1375
75 100 / 18 = = = 0.447
P 760
6000 Mole fraction of toluene in solution
or w = = 111.11 g
54 (1 – 0.247) = 0.753
111.11´ 1000 Mole fraction of toluene in vapour
Molality of urea = (1 – 0.447) = 0.553
60 ´ 100
23. The vapour pressure of pure benzene at a certain temperature
= 18.5 mol kg -1 is 640 mm Hg. The non volatile solid weighing 2.175 g is
20. The vapour pressure of pure benzene at 25°C is 639.7 mm Hg added to 39.0 g of benzene. The vapour pressure of the
and the vapour pressure of a solution of a solute in benzene solution is 600 mm Hg. What is the molar mass of the solid
at the same temperature is 631.9 mm Hg. Calculate the molality substance?
of the solution. Sol. Apply the formula
o
Sol. From Raoult’s law p A = x A PA P° - P n
=
P N
631.9
\ x1 = = 0.988 640 - 600 2.175 / M
639.7 =
600 39 / 78
\ Mole fraction of solute = ( 1 – 0.988) = 0.012 = x 2
M = 65.25 g/mol
540 Chemistry
24. A 0.1 molar solution of NaCl is found to be isotonic with 1%
NaCl Na + + Cl -
urea solution. Calculate
(a) degree of dissociation of sodium chloride. 1– a a a
(b) Van’t Hoff factor and 1- a + a + a
i= ; 1 + a = 1.87
-3 1
Assume density of 1% urea equal to 1 g cm
\ a = 0.87 or 87%
Sol. (a) NaCl Na + + Cl - 27. The freezing point of a solution containing 0.2 g of ethanoic
C(1 – a ) Ca Ca acid in 20 g of benzene is lowered by 0.45°C. Calculate the
Total concentration of species in solution degree of association of ethanoic acid in benzene. Assume
= C(1 + a ) = 0.1(1 + a )
ethanoic acid dimerises in the solvent benzene. Kf for
Osmotic pressure of NaCl
= 0.1 (1 + a ) RT
T benzene = 5.12 K mol -1 kg
Osmotic pressure of urea solution
K f ´ w ´ 1000 5.12 ´ 0.2 ´ 1000
1 Sol. M = = = 113.77
= RT
T DTf ´ W 0.45 ´ 20
60 ´ 0.1
Since two solutions are isotonic, then Van’t Hoff factor

1 Normal M.wt 60
0.1 (1 + a ) RT = RT
T i= = = 0.53
60 ´ 0.1 Observed M.wt 113.77
\ a = 0.667 2 CH 3COOH (CH3COOH)2
(b) Again Van’t Hoff's factor
a
C(1 + a) 1–a
i= = 1.667 2
C
25. What minimum weight of ethane -1, 2-diol must be mixed 1- a + a / 2
Van’t Hoff factor i = = 0.53
with 10 gallons of water to protect it from freezing at – 24°C? 1
Ethane 1, 2 - diol is an antifreeze addition
a
(1 gallon = 3.785 liters, Kf for H 2 O = 1.86 K mol -1 kg) or 1 – = 0.53
2
Sol. DTf = Kf × m (m = Molality) \ Degree of association a = 0.94 = 94%
28. The degree of dissociation of calcium nitrate in dilute solution
24 containing 7 g per 100 g of water at 100°C is 70%. Calculate
\ m= = 12.90
1.86 the vapour pressure of solution.
Freezing point of water = 0.0°C
\ 1 kg of water should contain 12.90 mole of Sol. Ca ( NO 3 ) 2 Ca + + + 2 NO 3-
ethane-1, 2-diol 1– a a 2a
10 gallons of water = 10 × 3.785 Moles at equilibrium
= 37.85 liters 1 – a + a + 2 a = 1 +2 a
Moles of ethane – 1, 2-diol in 37.85 liters = 1 + 2 × 0.7 = 2.4
= 12.9 × 37.85mol Again

12.9 ´ 37.85 ´ 62 æQ molar mass of ethane ö M normal


çç ÷÷ = 1 + 2 a = 2.4;
= kg -1 M observed
1000 è 1, 2 - diol = 62 g mol ø
= 30.272 kg 164
M observed = = 68.33
26. A decinormal solution of sodium chloride exerts an osmotic 2.4
pressure of 4.6 atm at 300 K Calculate the degree of From Raoult's law
dissociation
Po - Ps n
(R = 0.0821 atm litre K -1mol -1 ) = ;
o N
Sol. p = CRTT P
p calculated = 0.1 × 0.0821 × 300 = 2.46 atm 7
p observed = 4.6 atm
Ps 68.33
Van’t Hoff factor 1– = = 0.0184
Po 100 / 18
4.6 atm Ps = (1 – 0.0184) 760
i= = 1.87
2.46 atm = 746.016 mm Hg
Solutions 541

Very Short/Short Answer Questions Long Answer Questions


1. Why is the osmotic pressure considered to be a colligative 14. Define osmotic pressure. Arrange the following in increasing
property? order of osmotic pressure and give reasons in support of
2. If glycerine [C3H5 (OH)3] and ethylene glycol (C2H6O2) are your answer:
sold at same price per kg, which would be cheaper for (a) 34×2 g per litre of sucrose (M . Wt = 342)
preparing an antifreeze solution for the radiator of an (b) 90×0 g per litre of glucose (M. Wt = 180)
automobile?
(c) 5×85 g per litre of NaCl (M . Wt = 58×5)
3. Why is liquid ammonia bottle first cooled in ice before opening
it? 15. (a) In a solution of three components A, B and C, mole
4. At the same temperature, hydrogen is more soluble in water fraction of A is 0×5 and that of C is 0×2. What is mole
than helium. Which of them will have a higher value of KH fraction of B?
and Why ? (KH = Henry’s constant). (b) Under what condition, molality and molarity of a solution
5. Two liquids X and Y have boiling points 110 °C and 130 °C are equal?
respectively. Which one of them has higher vapour pressure (c) Calculate the osmotic pressure of mixture of 100 mL of
at 50 °C ? 6% urea solution and 100 mol of 18% glucose solution
6. CCl4 and H2O are immiscible whereas C2H5OH and H2O are at 27°C. [R = 0×0821 L atm mol–1 K–1].
miscible in all proportions. Correlate this behaviour with the 16. (a) Define the following terms:
molecular structures of the three compounds. (i) Mole fraction
7. An aqueous solution freezes at 272×4 K while pure water at (ii) Ideal solution
273 K. Determine
(b) 15.0 g of an unknown molecular material is dissolved in
(a) the molality of solution
450 g of water. The resulting solution freezes at – 0.34°C.
(b) boiling point of solution
What is the molar mass of the material?
(c) lowering of vapour pressure of water at 298 K.
(Kf for water = 1.86 K mol–1)
[Given Kf = 1×86 K kg mol–1, Kb = 0×512 K mg mol–1,
vapour pressure of pure water is 23×756 mm Hg]. 17. (a) Explain the following:
8. Vapour pressure of pure water at 35 °C is 31. 82 mm Hg. When (i) Henry’s law about dissolution of a gas in a liquid
27. 0 g of solute is dissolved in 100 g of water (at same (ii) Boiling point elevation constant for a solvent
temperature), vapour pressure of solution thus formed is 30. (b) A solution of glycerol (C3H8O3) in water was prepared
95 mm Hg. Calculate molecular mass of the solute. by dissolving some glycerol in 500 g of water. This so-
9. What do you expect to happen when Red Blood Corpuscles lution has a boiling point of 100.42°C. What mass of
(RBC’s) are placed in following solutions: glycerol was dissolved to make this solution? (Kb for
(i) 1% NaCl solution water = 0.512 K kg mol–1)
(ii) 0.5% NaCl solution 18. (a) State Raoult’s law for a solution containing volatile com-
10. Find the boiling point of a solution containing 0.520 g of ponents.
glucose (C6H12O6) dissolved in 80.2 g of water. How does Raoult’s law become a special case of Henry’s
[Given Kb for water = 0.52 K/m] law?
11. Calculate the mole fraction of benzene in solution (b) 1.00 g of a non-electrolyte solute dissolved in 50 g of
containing 30% by mass in carbon tetrachloride. benzene lowered the freezing point of benzene by 0.40
12. A 1.00 molal aqueous solution of trichloroacetic acid K. Find the molar mass of the solute. (Kf for benzene =
(CCl3COOH) is heated to its boiling point. The solution has 5.12 kg mol–1)
the boiling point of 100.18°C. Determine the Van’t Hoff factor 19. (a) Define the following terms:
for trichloro-acetic acid. (Kb for water = 0.512 K kgmol–1) (i) Ideal solution (ii) Azeotrope
13. 18 g of glucose, C6H12O6 (Molar Mass = 180 g mol –1) is dis-
(iii) Osmotic pressure
solved in 1 kg of water in a sauce pan. At what
temperature will this solution boil? (b) A solution of glucose (C6H12O6) in water is labelled as
(Kb for water = 0.52 K kg mol–1, boiling point of pure water = 10% by weight. What should be the molality of the so-
373.15 K). lution? (Molar mass of glucose = 180 g mol–1)
542 Chemistry
Multiple Choice Questions 24. Osmotic pressure of a sugar solution at 24°C is 2.5
20. When mercuric iodide is added to the aqueous solution of atmosphere. The concentration of the solution in gm mole
potassium iodide then per litre is
(a) freezing point is raised. (a) 10.25 (b) 1.025
(c) 102.5 (d) 0.1025
(b) freezing point is lowered.
25. A solution is obtained by dissolving 12 gm of urea (mol. wt.
(c) freezing point does not change.
60) in a litre of water. Another solution is obtained by
(d) boiling point does not change. dissolving 68.4 gm of cane sugar (mol. wt. 342) in a litre of
21. Which one of the following salts will have the same value of water at the same temperature. The lowering of vapour
van’t Hoff factor (i) as that of K4[Fe (CN)6]. pressure in the first solution is
(a) same as that of 2nd solution
(a) Al2(SO4)3 (b) NaCl (b) nearly one-fifth of the 2nd solution
(c) Al (NO3)3 (d) Na2SO4. (c) double that of 2nd solution
22. For a dilute solution, Raoult’s law states that (d) nearly five times that of 2nd solution
(a) the lowering of vapour pressure is equal to the mole 26. Freezing point of an aqueous solution is (–0.186)°C. Elevation
fraction of the solute of boiling point of the same solution is Kb = 0.512°C,
(b) the relative lowering of vapour pressure is equal to the Kf = 1.86°C, find the increase in boiling point.
mole fraction of the solute (a) 0.186°C (b) 0.0512°C
(c) the relative lowering of vapour pressure is proportional (c) 0.092°C (d) 0.2372°C.
to the amount of solute in solution 27. The relationship between osmotic pressure at 273 K when
(d) the vapour pressure of the solution is equal to the 10g glucose (P1), 10 g urea (P2), and 10g sucrose (P3) are
mole fraction of the solvent dissolved in 250 ml of water is
23. The normality of 10% (weight/volume) acetic acid is (a) P1 > P2 > P3 (b) P3 > P1 > P2
(a) 1 N (b) 10 N
(c) P2 > P1 > P3 (d) P2 > P3 > P1
(c) 1.7 N (d) 0.83 N

1. Which of the following modes of expressing concentration (a) more than the theoretical weight
is independent of temperature? (b) less than the theoretical weight
(a) Molarity (b) Formality (c) same as theoretical weight
(c) Normality (d) Molality (d) None of the above
2. The solubility of a gas in water depends on
7. What is the molarity of H2SO4 solution that has a density
(a) nature of the gas (b) temperature
1.84 gm/cc at 35°C and contains 98% H2SO4 by weight?
(c) pressure of the gas (d) All of these
(a) 4.18 M (b) 8.14 M
3. If one mole of a substance is present in 1 kg of solvent, then
its concentration is called (c) 18.4 M (d) 18 M
(a) Molar conc. (b) Molal conc. 8. The amount of oxalic acid (mol. wt. 63) required to prepare
(c) Normality (d) Strength wt/wt. 500 ml of its 0.10 N solution is
4. Equal volumes of 0.1 M HCl and 0.1M NaOH are mixed. The (a) 0.315 g (b) 3.150 g
concentration of the resulting solution will be (c) 6.300 g (d) 63.00 g
(a) 0.1 M (b) 0.05 M 9. The molarity of the solution containing 7.1 g of Na2SO4 in
(c) 0.2 M (d) 0.0 M 100 ml of aqueous solution is
5. When the solute is present in trace quantities the following (a) 2 M (b) 0.5 M
expression is used (c) 1 M (d) 0.05 M
(a) gram per million (b) milligram percent 10. A solution of CaCl2 is 0.5 mol/litre, then the moles of chloride
(c) microgram percent (d) parts per million ions in 500 ml. will be
6. For preparing 0.1 N solution of a compound from its impure (a) 0.25 (b) 0.50
sample of which the percentage purity is known, the weight
(c) 0.75 (d) 1.00
of the substance required will be
Solutions 543
11. What will be the molality of a solution having 18 g of glucose 20. The vapour pressure of a solvent A is 0.80 atm. When a non-
(mol. wt. = 180) dissolved in 500 g of water? volatile substance B is added to this solvent its vapour
(a) 1 m (b) 0.5 m pressure drops to 0.6 atm. the mole fraction of B in the solution
(c) 0.2 m (d) 2 m is
12. 50 ml of 10N H2SO4, 25 ml of 12 N HCl and 40 ml of 5 N HNO3 (a) 0.25 (b) 0.50
were mixed together and the volume of the mixture was made (c) 0.75 (d) 0.90
1000 ml by adding water. The normality of the resulting 21. At a particular temperature, the vapour pressures of two
solution will be liquids A and B are respectively 120 and 180 mm of mercury.
(a) 1 N (b) 2 N If 2 moles of A and 3 moles of B are mixed to form an ideal
(c) 3 N (d) 4 N solution, the vapour pressure of the solution at the same
13. An aqueous solution of 6.3 g oxalic acid dihydrate is made temperature will be (in mm of mercury)
upto 250 ml. The volume of 0.1 N NaOH required to completely (a) 156 (b) 145
neutralize 10 ml of this solution is (c) 150 (d) 108
22. At room temperature, the mole fraction of a solute is 0.25 and
(a) 40 ml (b) 20 ml
the vapour pressure of a solvent is 0.80 atm. Then the lowering
(c) 10 ml (d) 4 ml
of vapour pressure is
14. 2.5 litre of 1 M NaOH solution are mixed with another 3 litre
(a) 0.75 (b) 0.60
of 0.5 M NaOH solution. Then the molarity of the resulting (c) 0.20 (d) 0.80
solution is 23. Which of the following is not correct for ideal solution?
(a) 0.80 M (b) 1.0 M (a) DSmixing = 0 (b) DVmixing = 0
(c) 0.73 M (d) 0.50 M (c) DHmixing = 0 (d) It obeys Raoult's law
15. 12g of urea is dissolved in 1 litre of water and 68.4 g of 24. Which of the following does not show positive deviation
sucrose is dissolved in 1 litre of water. The lowering of vapour from Raoult's law?
pressure of first case is (a) Benzene-Chloroform
(a) equal to second (b) greater than second (b) Benzene-Acetone
(c) less than second (d) double that of second (c) Benzene-Ethanol
16. Which of the following statement is correct if the (d) Benzene-Carbon tetrachloride
intermolecular forces in liquids A, B and C are in the order 25. Identify the mixture that shows positive deviation from
A < B < C? Raoult's law
(a) B evaporates more readily than A (a) CHCl3 + (CH3)2CO
(b) B evaporates less readily than C (b) (CH3)2CO + C6H5NH2
(c) A and B evaporates at the same rate (c) CHCl3 + C6H6
(d) A evaporates more readily than C (d) (CH3)2CO + CS2
26. Azeotropic mixture of HCl and H2O has
17. The aqueous solution that has the lowest vapour pressure
(a) 48% HCl (b) 22.2% HCl
at a given temperature is
(c) 36% HCl (d) 20.2% HCl
(a) 0.1 molal sodium phosphate 27. Which of the following is not a colligative property?
(b) 0.1 molal barium chloride (a) Depression in freezing point
(c) 0.1 molal glucose (b) Elevation of boiling point
(d) 0.1 molal acetic acid (c) Osmotic pressure
18. Vapour pressure of CCl4 at 25°C is 143 mm Hg. 0.5 g of a non- (d) Modification of refractive index
volatile solute (mol. wt. 65) is dissolved in 100 ml of CCl4. 28. Which of the following is a colligative property?
Find the vapour pressure of the solution. (Density of CCl4 = (a) Change in free energy (b) Change in pressure
1.58 g/cm3) (c) Heat of vapourisation (d) Osmotic pressure
29. Which of the following colligative properties can provide
(a) 141.93 mm Hg (b) 94.39 mm Hg
molar mass of proteins (or polymers or colloids) with greatest
(c) 199.34 mm Hg (d) 143.99 mm Hg precision?
19. The vapour pressure of solvent decreases by 10 mm of (a) Depression in freezing point
mercury when a non-volatile solute was added to the solvent. (b) Osmotic pressure
The mole fraction of the solute in the solution is 0.2. What (c) Relative lowering of vapour pressure
should be the mole fraction of the solvent if the decrease in (d) Elevation of boiling point
vapour pressure is to be 20 mm of mercury ? 30. Isotonic solutions have same
(a) 0.8 (b) 0.6 (a) molar concentration (b) molality
(c) 0.4 (d) 0.4 (c) normality (d) None of these
544 Chemistry
31. When mango is placed in dilute aqueous solution of 42. If the solution boils at a temperature T1 and the solvent at a
hydrochloric acid, it temperature T2 the elevation of boiling point is given by
(a) shrinks (b) swells (a) T1 + T2 (b) T1 – T2
(c) bursts (d) Nothing happens (c) T2 – T1 (d) T1 + T2
32. As a result of osmosis, the volume of the solution
43. Which of the following will have highest boiling point at 1
(a) gradually decreases (b) gradually increases
atm pressure?
(c) is not affected (d) Any of the three
33. The relationship between osmotic pressure at 273 K when (a) 0.1 M NaCl (b) 0.1 M Sucrose
10g glucose (P1), 10 g urea (P2), and 10g sucrose (P3) are (c) 0.1 M BaCl2 (d) 0.1 M Glucose
dissolved in 250 ml of water is 44. A solution of 1 molal concentration of a solute will have
(a) P1 > P2 > P3 (b) P3 > P1 > P2 maximum boiling point elevation when the solvent is
(a) ethyl alcohol (b) acetone
(c) P2 > P1 > P3 (d) P2 > P3 > P1
(c) benzene (d) chloroform
34. 0.1 M NaCl and 0.1 M CH3COOH are kept in separate
45. The normal boiling point of water is 373 K (at 760 mm Hg).
containers. If their osmotic pressures are P 1 and P 2
Vapour pressure of water at 298 K is 23 mm Hg. If enthalpy of
respectively then what is the correct statement?
vaporisation is 40.656 kJ mol–1, the boiling point of water at
(a) P1 > P2 (b) P1 = P2
23 mm Hg pressure will be
(c) P1 < P2 (d) P1 = P2 = 0 atm.
35. What happen when isotonic solution of A (mol. wt. 342) and (a) 250 K (b) 294.4 K
B (mol. wt. 60) are put into communication through (c) 51.6 K (d) 12.5 K
semipermeable membrane? 46. If the elevation in boiling point of a solution of 10 gm of
(a) Transference of solvent from solution of A to that of B solute (mol. wt. = 100) in 100 gm of water is DTb, the
takes place ebullioscopic constant of water is
(b) Transference of solvent from solution of B to that of A (a) 10 (b) 10 DTb
takes place
(c) No transference of solvent from solution of A to that of DTb
(c) DT b (d)
B takes place 10
(d) Change in temperature of the solutions takes place 47. The rise in the boiling point of a solution containing 1.8 g of
36. Which among the following will show maximum osmotic glucose in 100 g of solvent is 0.1°C. The molal elevation
pressure? constant of the liquid is
(a) 1 M NaCl (b) 1 M MgCl2 (a) 0.01 K/m (b) 0.1 K/m
(c) 1 M (NH4)3PO4 (d) 1 M Na2SO4 (c) 1 K/m (d) 10 K/m
37. Isotonic solutions have 48. The boiling point of a solution of 0.11 g of a substance in 15
(a) same boiling point (b) same vapour pressure g of ether was found to be 0.1°C higher than that of pure
(c) same melting point (d) same osmotic pressure ether. The molecular weight of the substance will be (Kb =
38. An aqueous solution of sucrose, C12H22O11, containing 34.2 2.16°K kg mol–1)
g/ litre has an osmotic pressure of 2.38 atmospheres at 17°C. (a) 148 (b) 158
For an aqueous solution of glucose, C6H12O6 to be isotonic (c) 168 (d) 178
with this solution, it would have 49. The boiling point of 0.1 molal K4[Fe(CN)6] solution will be
(a) 34.2 g/lit of glucose (b) 17.1 g/lit of glucose (Given Kb for water = 0.52°K kg mol–1)
(c) 18.0 g/lit of glucose (d) 36.0 g/lit of glucose (a) 100.52°C (b) 100.104°C
39. The osmotic pressure of 5% (mass-volume) solution of cane (c) 100.26°C (d) 102.6°C
sugar at 150°C (mol. mass of sugar = 342) is 50. The freezing point of equimolal aqueous solution will be
(a) 4 atm (b) 5.07 atm highest for
(c) 3.55 atm (d) 2.45 atm (a) C6H5NH3+Cl– (b) Ca(NO3)2
40. A 1% aqueous solution (mass by volume) of a certain (c) La(NO3)2 (d) C6H12O6
substance is isotonic with a 3% solution of dextrose i.e. 51. If all the following four compounds were sold at the same
glucose (molar mass 180) at a given temperature. The molar price, which would be cheapest for preparing an antifreeze
mass of the substance is solution for a car radiator?
(a) 60 (b) 120 (a) CH3OH (b) C2H5OH
(c) 180 (d) 360 (c) C2H4(OH)2 (d) C3H5(OH)3
41. A 5% solution of cane sugar (mol. wt. = 342) is isotonic with 52. Which of the following 0.10 m aqueous solution will have
1% solution of substance X. The molecular weight of X is the lowest freezing point?
(a) 34.2 (b) 171.2 (a) Al2(SO4)3 (b) C6H10O5
(c) 68.4 (d) 136.8 (c) KI (d) C12H22O11
Solutions 545
53. Which of the following aqueous solution will have highest 61. The van't Hoff factor for 0.1 M Ba(NO3)2 solution is 2.74.
depression in freezing point? The degree of dissociation is
(a) 0.1 M Urea (b) 0.1 M Sucrose (a) 91.3% (b) 87%
(c) 0.1 M AlCl3 (d) 0.1 M K4 [Fe(CN)6] (c) 100% (d) 74%
54. The depression in freezing point for 1 M urea, 1 M glucose 62. Formation of a solution from two components can be
and 1 M NaCl are in the ratio considered as
(a) 1 : 2 : 3 (b) 3 : 2 : 2 (i) pure solvent ® separated solvent molecules, DH1
(c) 1 : 1 : 2 (d) None of these
(ii) pure solute ® separated solute molecules, DH2
55. Which one of the following aqueous solutions will have the
lowest freezing point? (iii) separated solvent and solute molecules ®
(a) 0.1 molal solution of urea solution, DH3
(b) 0.1 molal solution of sucrose Solution so formed will be ideal if
(c) 0.1 molal solution of sodium chloride
(a) DHsoln = DH1 + DH 2 - DH3
(d) 0.1 molal solution of calcium chloride
56. The depression of freezing point is directly proportional to (b) DHsoln = DH1 - DH 2 - DH3
(a) mole fraction of the solution
(b) molarity of the solution (c) DHsoln = DH3 - DH1 - DH 2
(c) molality of the solution
(d) molarity of the solvent (d) DHsoln = DH1 + DH 2 + DH3
57. A 0.5 molal solution of ethylene glycol in water is used as 63. If liquids A and B form an ideal solution
coolant in a car. If the freezing point constant of water be
(a) the enthalpy of mixing is zero
1.86°C per mole, the mixture shall freeze at
(a) 0.93°C (b) –0.93°C (b) the entropy of mixing is zero
(c) 1.86°C (d) –1.86°C (c) the free energy of mixing is zero
58. An aqueous solution freezes at –0.186°C (Kf = 1.86, Kb = (d) the free energy as well as the entropy of mixing are each
0.512) what is the elevation in boiling point? zero
(a) 0.186 (b) 0.512
64. 25 ml of a solution of barium hydroxide on titration with a 0.1
(c) 0.86 (d) 0.0512
molar solution of hydrochloric acid gave a titre value of 35
59. The molal freezing point constant for water is 1.86°C/m.
Therefore, the freezing point of 0.1 M NaCl solution in water ml. The molarity of barium hydroxide solution was
is expected to be (a) 0.07 (b) 0.14
(a) –1.86°C (b) –0.186°C (c) 0.28 (d) 0.35
(c) –0.372°C (d) + 0.372°C 65. During depression of freezing point in a solution the
60. The molecular weight of benzoic acid in benzene as
following are in equilibrium
determined by depression in freezing point method
corresponds to (a) liquid solvent, solid solvent
(a) ionization of benzoic acid (b) liquid solvent, solid solute
(b) dimerization of benzoic acid (c) liquid solute, solid solute
(c) trimerization of benzoic acid (d) liquid solute, solid solvent
(d) solvation of benzoic acid

1. A 0.0020 m aqueous solution of an ionic compound (a) Addition of NaCl (b) Addition of Na 2SO4
Co(NH3)5(NO2)Cl freezes at – 0.00732 °C. Number of moles
of ions which 1 mol of ionic compound produces on being (c) Addition of 1.00 molal KI (d) Addition of water
dissolved in water will be (Kf = – 1.86°C/m) 3. A solution of sucrose (molar mass = 342 g mol–1) has been
[CBSE-PMT 2009] prepared by dissolving 68.5 g of sucrose in 1000 g of water.
The freezing point of the solution obtained will be
(a) 3 (b) 4 (c) 1 (d) 2
(Kf for water = 1.86 K kg mol–1). [CBSE-PMT 2010]
2. An aqueous solution is 1.00 molal in KI. Which change will
cause the vapour pressure of the solution to increase? (a) – 0.372°C (b) – 0.520°C
[CBSE-PMT 2010] (d) + 0.372°C (d) – 0.570°C
546 Chemistry

4. 25.3 g of sodium carbonate, Na 2 CO3 is dissolved in enough 12. In mixture A and B components show -ve deviation as
water to make 250 mL of solution. If sodium carbonate (a) D Vmix > 0 [AIEEE 2002]
dissociates completely, molar concentration of sodium ions, (b) D Hmix < 0
Na + and carbonate ions, CO 32– are respectively (Molar mass (c) A – B interaction is weaker than A – A and B – B interaction
of Na2CO3 = 106 g mol–1) [CBSE-PMT 2010] (d) A – B interaction is stronger than A – A and B – B
(a) 0.955 M and 1.910 M (b) 1.910 M and 0.955 M interaction.
(c) 1.90 M and 1.910 M (d) 0.477 M and 0.477 M 13. A pressure cooker reduces cooking time for food because
[AIEEE 2003]
5. The freezing point depression constant for water is
– 1.86ºC m–1. If 5.00 g Na2SO4 is dissolved in 45.0 g H2O, the (a) boiling point of water involved in cooking is increased
freezing point is changed by – 3.82ºC. Calculate the van’t (b) the higher pressure inside the cooker crushes the food
Hoff factor for Na2SO4 [CBSE-PMT 2011] material
(a) 2.05 (b) 2.62 (c) 3.11 (d) 0.381 (c) cooking involves chemical changes helped by a rise in
temperature
6. The van’t Hoff factor i for a compound which undergoes
(d) heat is more evenly distributed in the cooking space
dissociation in one solvent and association in other solvent
is respectively : [CBSE-PMT 2011] 14. In a 0.2 molal aqueous solution of a weak acid HX the degree
of ionization is 0.3. Taking kf for water as 1.85, the freezing
(a) less than one and greater than one.
point of the solution will be nearest to [AIEEE 2003]
(b) less than one and less than one. (a) – 0.360º C (b) – 0.260º C
(c) greater than one and less than one. (c) + 0.480º C (d) – 0.480º C
(d) greater than one and greater than one. 15. Which one of the following aqueous solutions will exihibit
7. Mole fraction of the solute in a 1.00 molal aqueous solution is highest boiling point ? [AIEEE 2004]
[CBSE-PMT 2011] (a) 0.015 M urea (b) 0.01 M KNO3
(a) 0.1770 (b) 0.0177 (c) 0.01 M Na2SO4 (d) 0.015 M glucose
(c) 0.0344 (d) 1.7700 16. 6.02 × 1020 molecules of urea are present in 100 ml of its
solution. The concentration of urea solution is
8. A 0.1 molal aqueous solution of a weak acid is 30% ionized. If
[AIEEE 2004]
Kf for water is 1.86°C/m, the freezing point of the solution will
be : [CBSE-PMT 2011 M] (a) 0.02 M (b) 0.01 M
(a) – 0.18°C (b) – 0.54°C (c) 0.001 M (d) 0.1 M
(Avogadro constant, NA = 6.02 × 1023 mol–1)
(c) – 0.36°C (d) – 0.24°C
17. To neutralise completely 20 mL of 0.1 M aqueous solution of
9. 200 mL of an aqueous solution of a protein contains its 1.26 g.
phosphorous acid (H3PO3), the value of 0.1 M aqueous KOH
The osmotic pressure of this solution at 300 K is found to be
solution required is [AIEEE 2004]
2.57 × 10–3 bar. The molar mass of protein will be (R = 0.083 L
(a) 40 mL (b) 20 mL
bar mol–1 K–1) [CBSE-PMT 2011 M]
(c) 10 mL (d) 60 mL
(a) 51022 g mol–1 (b) 122044 g mol–1
18. For which of the following parameters the structural isomers
(c) 31011 g mol–1 (d) 61038 g mol–1 C2H5OH and CH3OCH3 would be expected to have the same
10. PA and PB are the vapour pressure of pure liquid components, values? [AIEEE 2004]
A and B, respectively of an ideal binary solution. If XA (Assume ideal behaviour)
represents the mole fraction of component A, the total pressure
(a) Boiling points
of the solution will be. [CBSE-PMT 2012]
(b) Vapour pressure at the same temperature
(a) PA + XA (PB – PA) (b) PA + XA (PA – PB)
(c) Heat of vapourization
(c) PB + XA (PB – PA) (d) PB + XA (PA – PB)
(d) Gaseous densities at the same temperature and pressure
11. Freezing point of an aqueous solution is 19. Which of the following liquid pairs shows a positive
(–0.186)°C. Elevation of boiling point of the same solution is deviation from Raoult’s law ? [AIEEE 2004]
Kb = 0.512°C,Kf = 1.86°C, find the increase in boiling point.
(a) Water - nitric acid
[AIEEE 2002]
(b) Benzene - methanol
(a) 0.186°C (b) 0.0512°C
(c) Water - hydrochloric acid
(c) 0.092°C (d) 0.2372°C.
(d) Acetone - chloroform
Solutions 547
20. Which one of the following statements is FALSE? 28. A 5.25% solution of a substance is isotonic with a 1.5% solu-
[AIEEE 2004] tion of urea (molar mass = 60 g mol–1) in the same solvent. If the
(a) The correct order of osmotic pressure for 0.01 M densities of both the solutions are assumed to be equal to
aqueous solution of each compound is 1.0 g cm–3, molar mass of the substance will be [AIEEE 2007]
BaCl 2 > KCl > CH3COOH > Sucrose (a) 210.0 g mol–1 (b) 90.0 g mol–1
(b) The osmotic pressure (p) of a solution is given by the (c) 115.0 g mol–1 (d) 105.0 g mol–1.
equation p = MRT, where M is the molarity of the solution 29. At 80° C, the vapour pressure of pure liquid ‘A’ is 520 mm Hg
(c) Raoult’s law states that the vapour pressure of a and that of pure liquid ‘B’ is 1000 mm Hg. If a mixture solution
component over a solution is proportional to its mole of ‘A’ and ‘B’ boils at 80° C and 1 atm pressure, the amount of
fraction ‘A’ in the mixture is (1 atm = 760 mm Hg) [AIEEE 2008]
(d) Two sucrose solutions of same molality prepared in
(a) 52 mol percent (b) 34 mol percent
different solvents will have the same freezing point
depression (c) 48 mol percent (d) 50 mol percent
30. The vapour pressure of water at 20°C is 17.5 mm Hg. If 18 g of
21. If a is the degree of dissociation of Na 2SO 4 , the Vant
glucose (C6H12O6) is added to 178.2 g of water at 20°C, the
Hoff’s factor (i) used for calculating the molecular mass is
vapour pressure of the resulting solution will be [AIEEE 2008]
[AIEEE 2005]
(a) 1 – 2 a (b) 1 + 2 a (a) 17.325 mm Hg (b) 15.750 mm Hg
(c) 1 – a (d) 1 + a (c) 16.500 mm Hg (d) 17.500 mm Hg
22. Benzene and toluene form nearly ideal solution. At 20°C, the 31. A binary liquid solution is prepared by mixing n-heptane and
vapour pressure of benzene is 75 torr and that of toluene is ethanol. Which one of the following statements is correct
22 torr. The partial vapour pressure of benzene at 20°C for a regarding the behaviour of the solution? [AIEEE 2009]
solution containing 78 g of benzene and 46 g of toluene in (a) The solution is non-ideal, showing – ve deviation from
torr is [AIEEE 2005]
Raoult’s Law.
(a) 53.5 (b) 37.5
(b) The solution is non-ideal, showing + ve deviation from
(c) 25 (d) 50
Raoult’s Law.
23. Two solutions of a substance (non electrolyte) are mixed in
the following manner. 480 ml of 1.5 M first solution + 520 ml (c) n-heptane shows + ve deviation while ethanol shows
of 1.2 M second solution. What is the molarity of the final – ve deviation from Raoult’s Law.
mixture ? [AIEEE 2005] (d) The solution formed is an ideal solution.
(a) 2.70 M (b) 1.344 M 32. Two liquids X and Y form an ideal solution. At 300 K, vapour
(c) 1.50 M (d) 1.20 M pressure of the solution containing 1 mol of X and 3 mol of Y
24. Equimolar solutions in the same solvent have is 550 mm Hg. At the same temperature, if 1 mol of Y is further
[AIEEE 2005] added to this solution, vapour pressure of the solution
(a) different boiling and different freezing points
increases by 10 mm Hg. Vapour pressure ( in mm Hg) of X and
(b) same boiling and same freezing points
Y in their pure states will be, respectively: [AIEEE 2009]
(c) same freezing point but different boiling point
(d) same boiling point but different freezing point (a) 300 and 400 (b) 400 and 600
25. Density of a 2.05M solution of acetic acid in water is (c) 500 and 600 (d) 200 and 300
1.02 g/mL. The molality of the solution is [AIEEE 2006] 33. If sodium sulphate is considered to be completely dissociated
(a) 2.28 mol kg–1 (b) 0.44 mol kg–1 into cations and anions in aqueous solution, the change in
(c) 1.14 mol kg –1 (d) 3.28 mol kg–1
freezing point of water (DTf ), when 0.01 mol
26. 18 g of glucose (C6H12O6) is added to 178.2 g of water. The
of sodium sulphate is dissolved in 1 kg of water, is (Kf = 1.86
vapour pressure of water for this aqueous solution at 100ºC
is [AIEEE 2006] K kg mol–1) [AIEEE 2010]
(a) 76.00 Torr (b) 752.40 Torr (a) 0.372 K (b) 0.0558 K (c) 0.0744 K (d) 0.0186 K
(c) 759.00 Torr (d) 7.60 Torr 34. On mixing, heptane and octane form an ideal solution. At
27. A mixture of ethyl alcohol and propyl alcohol has a vapour 373 K, the vapour pressures of the two liquid components
pressure of 290 mm Hg at 300 K. The vapour pressure of (heptane and octane) are 105 kPa and 45 kPa respectively.
propyl alcohol is 200 mm Hg. If the mole fraction of ethyl Vapour pressure of the solution obtained by mixing 25.0 g of
alcohol is 0.6, its vapour pressure (in mm Hg) at the same heptane and 35 g of octane will be (molar mass of heptane
temperature will be [AIEEE 2007] = 100 g mol–1 and of octane = 114 g mol–1) [AIEEE 2010]
(a) 360 (b) 350 (c) 300 (d) 700 (a) 72.0 kPa (b) 36.1 kPa (c) 96.2 kPa (d) 144.5 kPa
548 Chemistry
35. A 5% solution of cane sugar (molar mass 342) is isotonic with [IIT-JEE 2009]
1% of a solution of an unknown solute. The molar mass of
(a) 4.0× 10– 4 (b) 4.0 × 10–5
unknown solute in g/mol is : [AIEEE 2011RS]
(c) 5.0 × 10– 4 (d) 4.0 × 10–6
(a) 171.2 (b) 68.4 (c) 34.2 (d) 136.2
40. Dissolving 120 g of urea (mol. wt. 60) in 1000 g of water gave
36. Kf for water is 1.86 K kg mol–1. If your automobile radiator
a solution of density 1.15 g/mL. The molarity of the solution
holds 1.0 kg of water, how many grams of ethylene glycol
is [IIT-JEE 2011]
(C2H6 O2) must you add to get the freezing point of the
solution lowered to –2.8ºC ? [AIEEE 2012] (a) 1.78 M (b) 2.00 M (c) 2.05 M (d) 2.22 M

(a) 72 g (b) 93 g (c) 39 g (d) 27 g 41. The freezing point (in °C) of a solution containing 0.1 g
of K3[Fe(CN)6] (Mol. wt. 329) in 100 g of water (Kf = 1.86 K
37. The molarity of a solution obtained by mixing 750 mL of 0.5(M)
kg mol–1) is [IIT-JEE 2011]
HCl with 250 mL of 2(M) HCl will be : [JEE M 2013]
(a) –2.3 × 10–2 (b) –5.7 × 10–2
(a) 0.875 M (b) 1.00 M (c) 1.75 M (d) 0.975 M
(c) –5.7 × 10–3 (d) –1.2 × 10–2
38. When 20 g of naphthoic acid (C11H8O2) is dissolved in 50 g
of benzene (Kf = 1.72 K kg mol–1), a freezing point depression 42. For a dilute solution containing 2.5 g of a non-volatile non-
of 2 K is observed. The Van't Hoff factor (i) is [IIT-JEE 2007] electrolyte solute in 100 g of water, the elevation
in boiling point at 1 atm pressure is 2°C. Assuming
(a) 0.5 (b) 1 (c) 2 (d) 3
concentration of solute is much lower than the concentration
39. The Henry’s law constant for the solubility of N2 gas in water of solvent, the vapour pressure (mm of Hg) of the solution is
at 298 K is 1.0 × 105 atm. The mole fraction of N2 in air is 0.8. (take Kb = 0.76 K kg mol–1) [IIT-JEE 2012]
The number of moles of N 2 from air dissolved in
(a) 724 (b) 740 (c) 736 (d) 718
10 moles of water at 298 K and 5 atm pressure is

1. An X molal solution in carbon tetrachloride show the mole (a) 6.2 g (b) 8.5 g
fraction of solute equal to 0.23527 . The value of X is (c) 12.6 g (d) 14.0 g
(a) 1.55 (b) 1.82 6. At 30°C and 40°C the vapour pressure of a liquid are 0.025
(c) 2.00 (d) 2.16 and 0.050 atm respectively. Calculate the heat of vaporization
2. 3.0 molal sodium hydroxide solution has a density of of the liquid.
1.110 gm L-1 . The molarity of this solution is (a) 14.06 cal/mol (b) 13060 cal/mol
(a) 2.97 (b) 3.05 (c) 140.8 cal /mol (d) 2160 cal/ mol
(c) 3.65 (d) 4.11 7. The equilibrium vapour pressure of water vapour over a 0.20
mole sample of water (l) is 35 torr at 27°C. If the amount of the
3. For a solution of two liquids A and B it was proved that
liquid water is decreased to 0.10 mole, the equilibrium vapour
PS = x A (p°A - p°B ) + p°B . The resulting solution will be pressure of it would be
(a) Non -ideal (b) ideal (a) 17.5 torr (b) 35.0 torr
(c) semi-ideal (d) None of these (c) 51.5 torr (d) None of these
4. The vapour pressure of water at 50°C is 92.5 torr. What will 8. A flask is partially evacuated to 400 torr pressure of air. A
be the vapour pressure of solution which consists of 1 mole small amount of benzene is introduced into the flask in order
of nonvolatile solute in 100 g of water at 50°C that some liquid will remain after equilibrium has been
(a) 906 .5 torr (b) 94.2 torr established. The vapour pressure of benzene at 25°C is 220
torr. . What is the total pressure in the flask at equilibrium at
(c) 91.8 torr (d) 90.8 torr.
25°C ?
5. When 200g of 10% solution was cooled part of the solute
(a) 120 torr (b) 510 torr
precipitated and the concentration of solution become 6% .
The mass of the precipitated solute is (c) 620 torr (d) 480 torr
Solutions 549
9. Which of the following solutions will have the highest 15. Consider the following statements
boiling point?
1. Isotonic solutions have the same molar concentration at
(a) 0.5 molal BaCl2 a given temperature
(b) 1.0 molal KBr 2. The molal elevation constant Kb is a characteristic of a
(c) 1.8 × 1024 glucose molecules per litre. solvent, and is independent of the solute added
3. The freezing point of a 0.1 M aqueous KCl solution is
(d) 100 g powdered glucose in one litre water
more than that of a 0.1 M aqueous AlCl3 solution.
10. Two 1-litre flask A and B are connected to each other by a
Which of these statements is correct?
valve which is closed. Flask A has benzene in equilibrium
with its vapours at 30°C. The flask B, is evacuated, and the (a) 1 and 2 (b) 2 and 3
valve is opened. Which of the following is true. If temperature (c) 1 and 3 (d) 1, 2 and 3
is kept constant.’ 16. The vapour pressure of pure liquid is 70 torr at 300 K. It forms
(a) Some of the benzene molecules would move to flask B an ideal solution with another liquid Y. The mole fraction of Y
from flask A. in the solution is 0.2 and total pressure of solution is 84 torr
at 300 K. The vapour pressure of pure liquid Y at 27° C
(b) Vapour pressure will be half the initial value.
(a) 70 torr (b) 560 torr
(c) The vapour pressure remains unchanged
(c) 140 torr (d) 154 torr
(d) Some more of the liquid benzene in flask A would
17. The molecular mass of a solute cannot be calculated by which
evaporate.
of the following?
11. The vapour pressure of ethanol and methanol are 42.0 mm
and 88.5 mm Hg respectively . An ideal solution is formed at WB ´ RT po WB MA
the same temperature by mixing 46.0 g of ethanol with 16.0 g (a) M B = (b) M B =
pV (po - p)WA
of methanol. What is the mole fraction of methanol vapour ?
(a) 0.467 (b) 0.502 DTb WB ´ 1000 K b WB ´1000
(c) 0.513 (d) 0.556 (c) M B = (d) M B =
K b WA DTb ´ WA
12. Which of the following solutions will have the maximum
lowering of vapour pressure at 300 K 18. At certain hill station pure water boils at 99.725 ° C. If K b for

(a) 1 M CaCl2 (b) 1 M NaCl water is 0.513 ° C kg mol-1 , the boiling point of 0.69 m solution
(c) 1 M Phenol (d) 1 M sucrose of urea will be

13. Two Aqueous solutions S1 and S2 are separated by a (a) 103 °C (b) 100.079 °C

semipermeable membrane . Solution S1 has got a greater (c) 100.359 °C (d) unpredictable

vapour pressure than solution S2 . Water will be flowing 19. Two elements A and B form compounds of formula AB2 and

(a) from S1 to S2 AB4 . When dissolved in 20.0 g of benzene 1.0 g of AB2

(b) from S2 to S1 lowers F. pt. by 2.3° C whereas 1.0 g of AB4 lowers F. pt. by
(c) in both the directions 1.3° C. The K f for benzene is 5.4. The atomic masses of A
(d) in either direction depending upon the nature of the and B are
solute (a) 25, 42 (b) 42, 25
14. What is the degree of dissociation of sodium chloride, if the (c) 52, 48 (d) 48, 52
molar mass determined by a cryoscopic method was found to
20. The heat of vaporisation of benzene is 7353 cal mol -1 . The
be 31.80 g mol-1 [ Atomic mass Na = 23 g mol–1 Cl = 35.5 g mol–
1] ? approximate bpt. of benzene is

(a) 0.58 (b) 0.73 (a) 77.1 °C (b) 87.1 °C

(c) 0.84 (d) 0.92 (c) 101 °C (d) 95 °C


550 Chemistry
21. The vapour pressure of a solution of the liquids A 23. How many grams of sucrose (M. wt. 342) must be dissolved
(p° = 80 mm Hg and x A = 0.4) and B (p°= 120 mm Hg and in 100 g water to produce a solution with a 105.0°C difference
between the freezing point & the boiling point temperature ?
x B = 0.6) is found to be 100 mm Hg. It shows that the (a) 460 g (b) 342 g
solution exhibits (c) 72 g (d) 34.2 g
(a) positive deviation from ideal behaviour 24. Coolent used in car radiator is aqueous solution of ethylene
glycol. In order to prevent the solution from freezing at – 0.3
(b) negative deviation from ideal behaviour
°C. How much ethylene glycol must be added to 5 kg of
(c) ideal behaviour
-1
(d) positive deviation for lower conc. and negative for higher water ? ( K f = 1.86 K kg mol )
conc. (a) 50 kg (b) 55 g
(c) 45 g (d) 40 g
1 1 25. The total vapour pressure of a solution of components A
22. Plot of Vs ( x A mole fraction of A in liquid state
xA YA and B is 600 torr. The mole fraction of component A in liquid
and vapour phase are 0.70 and 0.35 respectively. The vapour
and YA in vapour state) is linear whose slope and intercept
pressure of pure A and B are
respectively are given (a) 300 torr, 130 torr (b) 1300 torr, 300 torr
(c) 300 torr, 1300 torr (d) 300 torr, 300 torr
p oB - p oA
(a) p oB / p oA , 26. A current of dry air is passed through a solution of 2.64 g of
p oB a non volatile solute in 30.0 g of ether and then through pure
ether. The loss in weight of solution was 0.645 g and of the
p oA - p oB ether 0.0345 g. The molecular weight of the solid is
(b) p oA - p oB ,
p oB (a) 122 g (b) 12.2 g
(c) 244 g (d) 135 g
27. An ideal solution of 1 mol A and 2 mol B has vapour pressure
p oB - p oA
(c) p oB - p oA , 250 torr at 50° C. If 1 mole of A is added to the solution, the V.
p oB P. of solution becomes 300 torr. What will be the V. P. if 1 mole
of B is added to the solution instead of 1 mole of A
p oA - p oB (a) 150 torr (b) 225 torr
(d) p oB / p oA ,
p oB (c) 275 torr (d) 175 torr
Solutions 551

EXERCISE 1
50 ´ 10 + 25 ´ 12 + 40 ´ 5
= = 1N
7. (a) 0×322 mol/kg. 1000
(b) 373×165 K.
(c) 0×137 mm Hg. 13. (a) Normality of oxalic acid = 6.3 ´ 1000 = 0.4
63 ´ 250
8. 177×38 g mol–1
N1V1 = N2V2; 0.1 ×V1 = 0.4 × 10. \ V1 = 40 ml
9. (i) They will shrink due to plasmolysis.
14. (c) M3V3 = M1V1 + M2V2
(ii) They will swell and may even burst.
\ M3 × 5.5 = 2.5 × 1 + 3 × 0.5. \ M3 = 0.727.
14. Sucrose < NaCl < Glucose
12
20. (a) 21. (a) 22. (c) 23. (c) 24. (d) 15. (a) Moles of urea = = 0.2
60
25. (a) 26. (b) 27. (c)
68.4
EXERCISE 2 Moles of sucrose = = 0. 2
342
1. (d) Molality involves weight of solute and solvent, hence it Both are non electrolyte hence lowering of V.P. will be
is independent of temperature. same.
2. (d) All are correct. See Henry's law. 16. (d) Lesser the intermolecular forces, the more the volatile
3. (b) If one mole of solute is present in 1 Kg of solvent the character.
conc. is 1 m. 17. (a) 0.1 m Na3PO4 = 0.4 m (Van't Hoff factor i = 4)
4. (b) Solution will be neutral. Concentration of each will be \ Lowering of vapour pressure will be maximum.
0.05 M since volume got doubled. 18. (a) Relative lowering of V.P. = mole fraction of Solute
5. (d) For very dil. solution the concentration is expressed in
p o - ps
ppm. = x2
po
6. (a) Since the compound is impure more than theoretical
weight is required. 143 - p s 0.5 / 65
= ; p s = 141.93 mm Hg
143 158 / 154
Wt ´1000 98 ´ 1000
7. (c) M = = ´1.84 = 18.4
M. Wt ´ V 98 ´100 19. (b) Dp = x ; Dp = x 2 ; 10 = 0.2 ;
2
po Dp ' x '2 20 x '2
Wt ´ 1000 Wt ´1000
8. (b) M = ; 0.10 = ;
E.wt. ´ V 63 ´ 500 \ x '2 = 0.4, x '1 = 1– 0.4 = 0.6
sum of mole fractions is always 1
Wt = 3.150 g
20. (a) 0.80 - 0.6 = x B ; x B = 0.25
9. (b) M = Wt ´ 1000 ; M = 7.1 ´ 1000 = 0.5M 0.80
M.Wt . ´ V 142 ´ 100
21. (a) Vapour pressure of solution = PA + PB
10. (b) CaCl2 Ca++ + 2Cl–. 120 ´ 2 180 ´ 3
= p oA x A + p oB x B = + = 156 mm Hg
Moles of CaCl2 in 500 ml = 0.25 and Cl– = 0.50. 5 5

Wt. ´1000g Dp
11. (c) Apply m = 22. (c) = xB; \ Dp = 0.25 ´ 0.80 = 0.20
M.Wt ´ Wt. of Solvent po

N1V1 + N 2 V2 + N 3 V3 23. (a) DSmixing ¹ 0 for ideal solution. On mixing two solutions
12. (a) N =
Total volume the entropy increases.
552 Chemistry
24. (a) Benzene + chloroform, show negative deviation.
P2 DH v æ T2 - T1 ö
25. (d) CH3COCH3 + CS2 45. (b) Clapeyron Clausius equation log = ç ÷.
P1 2.303R è T1T2 ø
26. (d) Azeotrope of HCl + H2O contains 20.2% HCl.
27. (d) Modification of refractive index is not a colligative 760 40656 æ 373 - T1 ö
log = ç ÷
property. 23 2.303 ´ 8.314 çè 373T1 ÷ø
28. (d) Osmotic pressure.
29. (b) Osmotic pressure. T1 = 294.4K
30. (a) Isotonic solutions have same molar concentration at K b ´ w ´ 1000 DTb ´ 100 ´ 100
given temperature provided the Van't Hoff factor (i) is 46. (c) DTb = ; \Kb = = DTb
M´W 10 ´ 1000
same
31. (b) Mango swells due to osmosis. 0.1 ´ 180 ´ 100
47. (c) K b = =1 K/ m
32. (b) Increases 1.8 ´1000

33. (c) Moles of glucose 10 = 0.05, 48. (b) M =


K b ´ w ´ 1000 2.16 ´ 0.11´ 1000
= = 158.4
180 DTb ´ W 0.1 ´15

10 10 49. (c) DTb = K b ´ m ´ i = 0.52 ´ 0.1´ 5 = 0.26;


urea = = 0.16, Sucrose = = 0.029 .
60 342
\b.pt. = 100.26°C
Hence osmotic presure p2 > p1 > p3 (p µ c) 50. (d) Glucose is non electrolyte hence depression in freezing
34. (a) NaCl is strong electrolyte and CH3COOH weak electrolyte. point will be minimum, hence freezing point will be highest.
\ p1 > p 2 (value of i will be more in case of NaCl) K f ´ w ´ 1000
51. (a) DTf = . Other things being equal, the
35. (c) Since solutions are isotonic, hence no transference of M´W
solvents. smaller the molecular weight, the more is the DTf
36. (c) Vant Hoff factor i = 4 in case of (NH4)3 PO4,
52. (a) DTf = K f ´ w ´ i van't Hoff factor i = 5 for Al2(SO4)3
+
(NH4)3 PO4 3 N H 4 + PO34- hence lowest freezing point.
53. (d) For K 4 éëFe(CN)6 ùû, i = 5 hence lowest freezing point.
37. (d) For isotonic solutions p1 = p 2
54. (c) Urea and glucose are non electrolytes and 1 M NaCl
38. (c) For isotonic solutions p1 = p 2 or C1 = C2;
º 2M non electrolyte. \1 : 1 : 2 . (for NaCl i =2)
34.2 18.0 55. (d) 0.1 m CaCl2. Concentration of particles = 0.3 m (Since i = 3).
= 0.1 ; = 0. 1 .
342 180 56. (c) DTf µ m (m = molality of solution)
Hence 18 gL–1 glucose is 0.1 M. 57. (b) DTf = K f ´ m = 1.86 ´ 0.5 = 0.93; Tf = -0.93°C
5 1000 58. (c) 0.186 = 1.86 × m; \ m = 0.1;
39. (b) p = CRT p = ´ ´ 0.0821 ´ 423 = 5.07 atm.
342 100 \ DTb = 0.512 ´ 0.1 = 0.0512° C

30 10 59. (c) DTf = 2 ´ K f ´ m = 2 ´1.86 ´ 0.1 = 0.372;


40. (a) = \ M 2 = 60
180 M 2 \Tf = -0.372°C
60. (b) Benzoic acid forms a dimer in benzene.
10 50
41. (c) = ; \ m = 68.4 61. (b) Ba(NO3)2 Ba++ + 2NO3–
m 342
1–a a 2a
42. (b) Solution always boil at higher temperature T1 - T2 = DTb
1 - a + a + 2a
i= = 2.74; \ a = 0.87 = 87%
43. (c) Conc. of particles will be highest in BaCl2 (Van't Hoff 1
factor i = 3). 62. (d)
44. (d) DTb = Kb×m
63. (a) For ideal solution DH mix = 0 .
when m = 1 DTb = Kb. Higher the value of Kb, the more is
64. (b) M1V1 = M2V2 M1 × 25 = 0.1 × 35;
value of DTb.
\ M1 = 0.14
65. (a) Liquid solvent and solid solvent are in equilibrium.
Solutions 553
EXERCISE 3
WB
1. (d) DTf = 0 – (0.00732°C) = 0.00732 11. (b) DTb = K b ´1000 ;
M B ´ WA
DTf = i × Kf × m
DT f 0.00732 WB
i= = = 1.97 » 2 DTf = K f ´1000 ;
K f ´m 1.86 ´ 0.002 M B ´ WA
2. (d) When the aqueous solution of one molal KI is diluted
with water, concentration decreases, therefore the DTb K b DTb 0.512
= = = = 0.0512°C .
vapour pressure of the resulting solution increases. DTf Kf - 0.186 1.86
1000W2 1.86×1000×68.5
3. (a) DT f = K f = = 0.372 12. (b) [DH mix < 0]
M 2W1 342×1000
13. (a) On increasing pressure, the temperature is also increased.
Tf = T °f –DTf
Tf = – 0.372°C Thus in pressure cooker due to increase in pressure the
b.p. of water increases.
25.3 1000
4. (b) Concentration of Na 2CO3 = ´ = 0.955 M 14. (d) DTf = Kf × m × i ;
106 250
DTf = 1.85 × 0.2 × 1.3 = 0.480º C
+
[Na ] = 2 × 0.955 = 1.91 M
\ Tf = 0 - 0.480 = -0.480o C
é CO32– ù = 0.955 M
ë û
+ -
5. (b) Given Kf = – 1.86ºC m–1, mass of solute = 5.00 g, ( HX H + X , i = 1.3)
1- 0.3 0. 3 0.3
mass of solvent = 45.0 g and DTf = – 3.82°C
DTf = i × Kf .m
i = 1 - 0.3 + 0.3 + 0.3 = 1.3
5 ´ 1000
3.82 = i × 1.86 ×
142 ´ 45 15. (c) DTb = K b ´ m ´ i .
\ i = 2.62 (Molecular mass of Na2SO4 = 142 g)
6. (c) If compound dissociates in solvent i > 1 and on The value of i for Na 2SO 4 = 3,
association i < 1.
7. (b) 1 molal solution means 1 mole of solute dissolved in for KNO3 = 2, for urea = 1 and for glucose = 1
1000 gm solvent.
\ nsolute = 1 wsolvent = 1000 gm 6.02 ´ 10 20 ´ 1000
16. (b) M = = 0.01M
6.02 ´ 10 23 ´ 100
1000
\ nsolvent = = 55.56
18 17. (a) N1V1 = N 2 V2 (H 3PO 3 is dibasic \ M = 2N)
1 20 ´ 0.2 = 0.1´ V
xsolute = = 0.0177
1 + 55.56 \ V = 40 ml
8. (d) Given a = 30% i.e., 0.3
18. (d) Gaseous densities of ethanol and dimethyl ether would
HA ¾¾ ® H+ + A–
be same at same temperature and pressure. The heat of
1–a a a
vaporisation, V.P. and b.pt. will differ due to H - bonding
1 – 0.3 0.3 0.3
i = 1 – 0.3 + 0.3 + 0.3 in ethanol.
i = 1.3 19. (b) A mixture of benzene and methanol show positive
DTf = 1.3 × 1.86 × 0.1 = 0.2418 deviation from Raoult’s law
Tf = 0 – 0.2418 = – 0.2418 °C
20. (d) DTf = K f ´ m ´ i . Since Kf has different values for
WRT 1.26 ´ 0.083 ´ 300
9. (d) M = = different solvents, hence even if the m is the same DTf
pV 2.57 ´10-3 ´ 200 ´ 10 -3
will be different
31.374 ´ 10 6
= = 61038 g
514 ˆˆ† 2Na + SO 4-
21. (b) Na 2SO 4 ‡ˆˆ 2a
10. (d) p = pAxA + pBxB 1-a a
= pAxA + pB (1 – xA)
Þ pAxA + pB – pBxA 1 - a + 2a + a
Vant. Hoff’s factor i = = 1 + 2a
Þ pB+ xA (pA – pB) 1
554 Chemistry
22. (d) Given 28. (a) Osmotic pressure (p) of isotonic solutions are equal.
Vapour pressure of benzene= 75 torr For solution of unknown substance C1(concentration).
Vapour pressure of toluene = 22 torr 5.25 / M
mass of benzene in = 78g C1 =
V
78 Where M represents molar mass.
hence moles of benzene = = 1mole
78 1.5 / 60
For solution of urea, C2 (concentration) =
(mol.wt of benzene = 78) V
mass of toluene in solution = 46g Given, p1 = p2 Q p = CRT
46 5.25 / M 1.5 / 60
\ C1RT = C2RT or C1 = C2 or =
hence moles of toluene = = 0.5 mole V V
92
\ M = 210 g/mol
now partial pressure of benzene = Pbo .X b 29. (d) At 1 atmospheric pressure the boiling point of mixture
is 80°C.
1 1 2 At boiling point the vapour pressure of mixture, pT = 1
= 75 × = 50 torr = 75 × = 78 × atmosphere = 760 mm Hg.
1 + 0.5 1.5 3
Using the relation,
23. (b) From the molarity equation.
pT = p A° x A + p B° xB , we get
M1V1 + M2V2 = MV
pT = 520 x A + 1000(1 - x A )
M1V1 +M V2
2
M= where V = V1 + V2 {Q pAo = 520 mm Hg ,
V

480 ´ 1.5 + 520 ´ 1.2 pB° = 1000 mm Hg , x A + xB = 1 }


M= = 1.344 M
480 + 520 or 760 = 520 x A + 1000 - 1000 x A or 480 x A = 240
24. (d) Equimolar solutions of normal solutes in the same
solvent will have the same b. pts and same f. pts. 240 1
or x A = = or 50 mol. percent
480 2
æ1 M ö
25. (a) Apply the formula d = M ç + 2 ÷ i.e., the correct answer is (d)
è m 1000 ø 30. (a) The vapour pressure of a solution of glucose in water
can be calculated using the relation
æ1 60 ö
\ 1.02 = 2.05 ç + ÷ p° - ps Moles of glucose in solution
è m 1000 ø =
ps Moles of water in solution
On solving we get, m = 2.288 mol/kg
17.5 - ps 18/180
or = [Q p° = 17.5 ]
18 ps 178.2/18
26. (b) Moles of glucose = = 0. 1
180
0.1´ ps
178.2 or 17.5 – ps = or ps = 17.325 mm Hg.
Moles of water = = 9.9 9.9
18 Hence (a) is correct answer.
Total moles = 0.1 + 9.9 = 10 31. (b) For this solution intermolecular interactions between
n-heptane and ethanol are weaker than n-heptane-n-
pH2O = Mole fraction × Total pressure = 9.9 ´ 760 heptane & ethanol-ethanol interactions hence the
10 solution of n-heptane and ethanol is non-ideal and
= 752.4 Torr shows positive deviation from Raoult’s law.
32. (b) p = pA° xA + pB° xB
27. (b) pA° = ? , Given p B° = 200 mm of Hg, xA = 0.6, total
1 3
xB = 1 – 0.6 = 0.4, P = 290 of Hg 550 = pA° ´ + pB° ´
4 4
P = PA + PB = PA°x A + PB°xB pA° + 3 pB° = 550 ´ 4 ...(i)
Þ 290 = PA° × 0.6 + 200 × 0.4 \ p°A = 350 mm of Hg. In second case
1 4
ptotal = pA° ´ + pB° ´
5 5
Solutions 555

pA° + 4 pB° = 560 ´ 5 ...(ii) 750 ´ 0.5 + 250 ´ 2


=
Subtract (i) from (ii) 1000
\ pB° = 560 ´ 5 - 550 ´ 4 = 600 = 0.875 M
38. (a) Molecular weight of naphthoic acid
Q pA° = 400
C11H8O2 = 172 g mol–1.
33. (b) Sodium sulphate dissociates as The theoretical value of depression in freezing point
® 2Na + + SO -4 -
Na 2SO 4 ¾¾
20×1000
hence van’t hoff factor i = 3 = Kf × molality = 1.72× = 4K
172×50
Now DT f = i K f .m Van't Hoff factor,
= 3 × 1.86 × 0.01 = 0.0558 K
Observed value of colligative property
p Total = p °A x A + p °B x B
i=
34. (a) Theoretical value of colligative property
= p°Heptane xHeptane + p°Octane x Octane 2
= = 0.5 .
4
25 /100 35 /114 PN 2 = k H x N
= 105 ´ 25 35 + 45 ´ 25 35 39. (a) 2
+ + 0.8 × 5 = 1 × 105 × x N
100 114 100 114 2

0.25 0.3 \ x N = 4 × 10–5


= 105 ´ + 45 ´ 2
0.25 + 0.3 0.25 + 0.3 Solubility in 10 moles = 4 × 10–4.
105 ´ 0.25 45 ´ 0.3 26.25 + 13.5 120
= + = 40. (c) Number of moles of urea = =2
0.55 0.55 0.55 60
= 72 kPa Total mass of solution = 1000 + 120 = 1120 g
35. (b) For isotonic solutions Mass
Total volume of solution (in L) =
p1 = p 2 Density
1120 112
C1 = C2 = = L
3 115
1.15 ´ 10
5 / 342 1/ M
= Number of moles
0.1 0.1 Molarity of the solution =
Volume of solution in litre
5 1 2×115
= = = 2.05 mol L–1
342 M 112
41. (a) DTf = i × Kf × m
342
Þ M = = 68.4 gm/mol Where m = Molality of the solution
5
(i.e. number of moles of solute per 1000 g of the solvent)
36. (b) DTf = i × Kf × m
0.1
Given DTf = 2.8, Kf = 1.86 K kg mol–1 i = 1 Here m = ´10
329
(ethylene glygol is a non- electrolyte)
0.1´10
wt. of solvent = 1 kg Thus DTf = 4 × 1.86 × = 2.3 × 10–2
329
Let of wt of solute = x
Thus Tf = 0 – 2.3 × 10–2 = – 2.3 × 10–2 ºC
Mol. wt of ethylene glycol = 62
42. (a) From Raoult law
x
2.8 = 1 × 1.86 × p° – p No.of moles of solute
62 ´ 1 =
p° No. of moles of solvent+ No. of moles of solute
2.8 ´ 62 When the concentration of solute is much lower than
or x = = 93 gm
1.86 the concentration of solvent,
37. (a) From molarity equation : p° - p No. of moles of solute
M1V1 + M2V2 = M × V =
p° No. of moles of solvent
M V + M 2V2 DTb = Kb × m
M= 1 1 where V = total volume
V
556 Chemistry

Number of moles of the solute 6. (b) Apply claypeyron equation,


m= ´ 1000
Mass of solvent in grams 0.050 DH v æ 313 - 303 ö
log = ç ÷ = 13060cal / mol
Number of moles of the solute 0.25 2.303R è 313 ´ 303 ø
DTb = K b ´ × 1000
Mass of solvent in grams 7. (b) The equilibrium vapour pressure will remain the same. It
Number of moles of solute is independent of the quantity of substance
DTb ´ Mass of solvent in grams 8. (c) The total pressure will be the sum of pressures of air and
= benzene
K b ´ 1000
9. (c) For electrolytes multiply the concentration by Van't Hoff’s
2 ´100 factor i
= = 0.26,
0.76 ´1000 (a) 0.5 × 3 = 1.5 for BaCl2
100 (b) 1.0 × 2 = 2 for KBr,
Number of moles of solvent = = 5.56
18
1.8 ´ 10 24
From equation (i) we get, 760 - p = 0.26 (c) = 2.98 for glucose
760 5.56 6.023 ´ 10 23
On solving, p = 724.46 » 724 100
(d) = 0.55 for glucose
180
EXERCISE 4
Hence highest bpt will be observed in case of (c)
1000X 2 10. (c) There is no change in vapour pressure.
1. (c) Use the formula, m = ;
X1M1
11. (c) C2 H5OH CH3OH
Given X2 =0.23527,X1 =(1-0.23527) =0.76473 42.0 mm Hg 88.5 mm Hg
Molecular weight of CCl4 = 153.82, 46.0g 16.0 g
1 Mole 0.5 Mole
1000 ´ 0.23527 Total Moles 1 + 0.5 = 1.5
m= = 2.00
0.76473 ´ 153.82
2. (a) Use the formula Total pressure P = 42 ´ 1 + 88.5 ´ 0.5 ;
1.5 1.5
md 3.1 ´1.110 o
( p A = pA ´ xA )
M= ;M = ; M = 2.97
m.M 2 3 ´ 40
1+ 1+ p = 28 + 29.5 = 57.5
1000 1000
Mole fraction in vapour phase pA
3. (b) ps = X A (poA – poB ) + pBo ; = xA × Total pressure
o
ps = pA ´ x A - poB ´ x A + poB ; 29.5
\ xA = = 0.513
57.5
o
ps = pA ´ x A - pBo (1 - x B ) + pBo 12. (a) Maximum lowering of vapour pressure will be given by
the substance giving maximum number of particles in
\ ps = poA ´ xA + pBo ´ xB. solution. In case of electrolytes do not forget. To multiply
This is condition for ideal solution. the molarity by Van’t Hoff factor i.
13. (a) In case of osmosis the flow of the solvent, from lower
1000
4. (d) Moles of water = 55.5; concentration to higher concentration.
18
14. (c) Let a be the give of dissociation then
55.5 NaCl Na + Cl -
Mole fraction of water = = 0.982
55.5 + 1 1- a a +a

pA = 0.982 × 92.5 ; pA = 90.8 torr 1- a + a + a


Van’t Hoff's factor i = = 1+ a
5. (b) 10% soln contains 10g solute in 90 gm solvent or 1
20 g solute in 180 g solvent Again Van’t Hoff’s factor
6% solution contains 6 g solute in 94g solvent. For 60%
the amount of Normal mol . wt 58.5
= = = 1.83
Observed mol.wt 31.8
solute in 180g solvent. = 6 ´ 180 = 11.48;
94 Equating to both values of i, \1 + a = 1.83
Amount precipitated = 8.52 g \ a = 0.83
Solutions 557

15. (d) All the statements are correct (see text) 1 po æ 1 ö


o o – 1 = Ao ç - 1÷
16. (c) P = p A .x A + p B .x B , xA p B è yA ø
84 = 0.2 poY + 70.0 × 0.8
p oA 1 po
= – Ao
\ poY = 140 torr p oB yA pB

DTb ´ WB ´1000 1 po 1 æ po ö
17. (c) M B = is wrong. The correct form is or, = Ao . + çç 1 - Ao ÷÷
K b ´ WA xA p B yA è pB ø
K b ´ WB ´1000
MB = p oA 1 po - po
DTb ´ WA = . + B o A
p oB yA pB
18. (b) DTb = K b × m = 0.513 × 0.69 = 0.3539° C
This is equation of straight line.
\ bpt. of solution = 99.725° C + 0.3539° C
= 100.079° C p oA p oB - p oA
Slope = , Intercept =
19. (a) Let the masses of A and B be a and b. The mass of AB2 p oB p oB

will be (a + 2b) g mol -1 and AB4 will be (a + 4b) g mol -1 23. (c) B. pt. = 100 + D T b = 100 + K b m
F. pt. = 0 – D Tf = 0 – Kf m
5.1´1´1000
For AB2, 2.3 = ............ (I)
(a + 2b) ´ 20 Tb – Tf = 100 + K b m – (– K f m)
105 = 100 + 0.51 m + 1.86 m
5.1´1´1000
For AB4, 1.3 = ............ (II) 5
(a + 4b) ´ 20 \ m= = 2.11
2.37
On solving (I) and (II),
Weight of sucrose to be dissolved in 100 g water
a = 25.49 and b = 42.64
2.11´ 342 ´ 100
D H evp = = 72 g
-1 -1 1000
20. (a) Trouton’s rule is = 21 cal. K mol
Tb
24. (b) DTf = 0.3° C
7353
Tb = = 350.1 K = 77.1° C K f ´ WB ´ 1000
21 DTf = 0.3° C =
M B ´ WA
o o
21. (b) Ptotal = p A ´ x A + p B ´ x B
1.86 ´ WB ´ 1000
= 80.0 × 0.4 + 120.0 × 0.6 = 104 mm Hg =
62 ´ 5000
The observed Ptotal is 100 mm Hg which is less than
\ WB = 50 g
104 mm Hg. Hence the solution shows negative deviation.
The amount used should be more than 50 g.
o
22. (c) p A = pA ´ x A = Total pressure × yA 25. (c) XA = 0.70 \ X B = 0.30;
p B = poB ´ x B = Total pressure × yB YA = 0.35 \ YB = 0.65
Where x and y represent mole fraction in liquid and vapour
600 ´ 0.35
phase respectively. p A = 600 × 0.35 \ = p oA
0.70
p oB x B yB p oA = 300 torr
\ = , \
p oB xA yA
600 ´ 0.65
p B = 600 × 0.65 \ = p oB
1 - yA 0.30
poB (1 - x A )
=
poA x A yA
\ p oB = 1300 torr
558 Chemistry
26. (a) Ostwald and Walker’s method When 1 mole of A is added

2 1 2 1
po - ps W2 W2 / m xA = = and x B = =
= = 4 2 4 2
po W1 + W2 W2 / m + W1 / M
1 o 1
W1 = loss of weight of solution, \ p oA . + p B . = 300
2 2
W2 = loss of weight of solvent
\ p oA + p oB = 600 ............. (ii)
0.0345 2.64 / m From (i) and (ii),
\ =
0.645 + 0.0345 2.64 / m + 30 / 74
p oB = 150 and p oA = 450
m = 122 g
When 1 mole of B is added
o o
27. (b) V. P. of solution P = p A x A + pB xB
1 3
xA = , xB =
1 2 4 4
or 250 = p oA . + p oB .
3 3 3 1
\ P = 150 × + 450 × = 225
4 4
p oA + 2 p oB = 750 ............... (i)
17

Electrochemistry
ELECTROLYTES AND ELECTROLYSIS : Z is a constant known as Electrochemical equivalent
A substance which decomposes as a result of the passage of When I = 1 amp., t = 1 sec. then Q = 1 coulomb, then W = Z.
electric current is called an electrolyte and phenomenon of Thus Electrochemical equivalent is the amount of the
decomposition by electricity is called electrolysis. substance deposited or liberated by 1 ampere current
passing for 1 second (i.e. 1 coulomb, I × t = Q)
ELECTROLYTIC CELL :
(II) Second law : When the same quantity of electricity is passed
The apparatus used to carry out electrolysis is known as
through different electrolytes, the amounts of the products
electrolytic cell. The main features of electrolytic cell are :
obtained at the electrodes are directly proportional to their
Feature Cathode Anode chemical equivalents or equivalent weights. Thus
Sign Negative since attached to Positive since attached to Mass of A Eq. wt of A m1 E1 Z1Q Z1
negative end of external positive end of external = ; = = =
battery battery
Mass of B Eq. wt of B m 2 E 2 Z 2 Q Z 2
Direction of into the cell out of the cell Hence electrochemical equivalent µ Equivalent wt.
movement of
electrons ONE FARADAY :
Direction of Cations Anions E µ Z; E = FZ F = Known as Faraday's constant
movement of ions
Half-Reaction Reduction Oxidation One Faraday is the quantity of charge carried by one mole of
electrons
OXIDATION :
Loss of electrons is called oxidation. IF = 1.6023 ´ 10 -19 ´ 6.023 ´ 10 23 = 96500 Coulombs

¾® O 2 + 4H + + 4e -
2H 2 O ¾ AMOUNT OF THE SUBSTANCE 'm' LIBERATED OR
DEPOSITED AT AN ELECTRODE :
REDUCTION :
Gain of electrons is called reduction. Eq . wt of the Sub . ´ I ´ t Eq. wt ´ Q
m= =
96500 96500
2 H 2 O + 2e - ¾
¾® H 2 + 2OH -
Ex.1: Three electrolytic cells A, B and C containing electrolytes
FARADAY'S LAWS OF ELECTROLYSIS : AlCl3, CuSO4 and AgNO3 respectively were connected in
(I) First law : The amount of the substance deposited or series. A steady current of 1.287 ampere was passed through
liberated at an electrode is directly proportional to the the cells for a definite time. After electrolysis 2.158 g of
quantity of electricity passed through an electrolyte silver were deposited at the cathode of cell. Calculate.
W µ I´ t = I´ t ´ Z = Q´ Z (I) total charge passed during electrolysis.
I = Current strength in amp., t = time in sec., (II) time period for which the current is passed
Q = Quantity of charge (coulombs) (III) Mass of aluminium deposited at cathode of cell A
(IV) Mass of Copper deposited at cathode of cell B
560 Chemistry
Eq. of Zn++ deposited =
Eq. wt ´ total charge (Q)
Sol.: (I) Amt. deposited =
96500 i´ t 1.70 ´ 90 ´ 230
= = 3.646 ´ 10 -3
96500 100 ´ 96500
107.9
2.158 = ´ total charge Meq. of Zn++ deposited = 3.646
96500
Initial Meq of Zn++ = 300 × 0.160× 2 = 96
2.158 ´ 96500 (Q M × 2 = N for Zn++)
\Q = = 1930 C
107.9 Meq. of Zn++ left in solution 96 – 3.646 = 92.354
1930 C 92.354
(II) Charge = Current × time; \ time = = 1499.6 s [ ZnSO 4 ] = = 0.154M
1.287 A 2 ´ 300

mass of Al Eq. wt of Al COMPARISION OF ELECTROLYTIC AND METALLIC


(III) = ; CONDUCTORS :
mass of Ag Eq. wt of Ag
Property Metallic Conductor Electrolytic Conductor
9 ´ 2.158
\ mass of Al = = 0.18 g 1. Conduction due to movement of electrons due to movement of ions
107.9
2. Transfer of matter No transfer of matter there is transfer of matter

31.75 ´ 2.158 3. Chemical property No change in chemical property there is chemical change
(IV) Mass of copper deposited = = 0.635 g 4. Resistance Resistance is offered Resistance is offered by
107.9 by atomic Kernels interionic attractions,
viscosity of solvent
Ex.2 : The same quantity of electrical charge that deposited
5. Temperature decreases with increase increases with increase of
0.583 g of Ag was passed through solution of gold salt and of temperature temperature
0.355 gold was deposited. Find the oxidation state of gold in 6. Magnitude generally high generally low
the solution. 7. Solvation No solvation Ions are solvated

Sol.: Au n + + ne- ¾
¾® Au FACTORS AFFECTING ELECTROLYTIC
CONDUCTION :
197 Please refer to the properties discussed above viz. viscosity,
Eq. wt. of Au =
n temperature, solvation of ions etc.
m1 E1 0.583 107.9 OHM'S LAW :
= ; = ; The potential difference across the conductor is directly
m 2 E 2 0.355 197
proportional to the current flowing through it.
n
Potential difference µ current
\ n = 3, \ Oxidation state of gold = 3
V µ I; V = RI
Ex.3: Calculate the quantity of electricity that would be
where R is a constant and known as resistance of the conductor
required to reduce 12.3 g of nitrobenzene to aniline if current
efficiency is 50%. If the potential drop across the cell is 3.0 V
R=
volt, how much energy will be consumed. I
Law is applicable to metallic as well as electrolytic conductors. R
Sol.: C 6 H 5 NO 2 + 6H + + 6e - ¾
¾® C 6 H 5 NH 2 + 2H 2 O
is expressed in Ohms also designated as W.
M 123 RESISTANCE (R) :
Eq. wt of nitrobenzene = =
6 6 It offers obstruction to the passage of electric current. It is directly
proportional to the length (l) and inversely proportional to the
E´i´t 123 i ´ t ´ 50 æ 2 ´ 50 ö
W= Þ 12.3 = ´ çi = ÷ area of cross section (a) of the conductor.
96500 6 100 ´ 96500 è 100 ø
l l
(given current efficiency is 50 %) Rµ
= r´
a a
Q = i × t = 115800 coulombs where r is a constant, called resistivity or specific resistance.
Now energy used = Q × V = 115800 × 3 = 347.4 kJ
when l = 1 cm and a = 1 cm2 then r = R
Ex.4: A current of 1.70 amp. is passed through 300 ml of 0.160 M
solution of ZnSO4 for 230 sec with a current efficiency of SPECIFIC RESISTANCE (r) :
90%. Find the molarity of Zn++ after the deposition of Zn The resistance offered by 1 cm3 of the conductor is known as
assuming no change in volume during electrolysis. specific resistance.
1.70 ´ 90 R.a Ohm ´ cm 2
Sol. i = amp. Units of r = = = Ohm cm or W cm
100 l cm
Electrochemistry 561
CONDUCTANCE (C) : RELATION BETWEEN EQUIVALENT CONDUCTIVITY
It is ease of flow of electric current through the conductor ( L eq ) AND SPECIFIC CONDUCTIVITY ( k)
and reciprocal of resistance R.
k ´ 1000 k ´ 1000
C=
1
units ohm–1, mhos or W–1 L eq = k ´ V = =
R Normality of solution N

SPECIFIC CONDUCTIVITY k (KAPPA) : where V is the volume in cm3 or ml containing 1 gev. of the
It is the reciprocal of specific resistance. electrolyte.
UNITS OF EQUIVALENT CONDUCTIVITY :
1 l l æl ö Ohm–1 cm2 eq–1 or W–1 cm2 eq–1 or S cm2 eq–1
k= = = C ´ = C ´ cell constant ç = cell constant ÷
r R.a a èa ø EFFECT OF DILUTION ON EQUIVALENT
Hence specific conductivity k (Kappa) = Conductance × cell CONDUCTIVITY :
constant Since the degree of dissociation of the electrolyte increases with
Units k = Ohm–1 cm–1 = W–1 cm–1 = S cm–1 (W–1 = S, Seimens) dilution, the equivalent conductivity also increases. The increase
is more in case of weak electrolytes than strong electrolytes.
EFFECT OF DILUTION ON CONDUCTANCE :
The equivalent conductivity increases and specific conductivity
The number of current carrying particles or ions per ml decrease
decreases with dilution.
on dilution and specific conductivity, being the conductance of
IMPORTANCE OF EQUIVALENT CONDUCTIVITY :
one centimetre cube of solution, decreases with dilution.
It helps to compare the conductivity of different electrolytes,
CELL CONSTANT AND ITS DETERMINATION : since solutions of different electrolytes having 1 gram equivalent
l each in the same volume will have the same total charge of
The quantity is known as cell constant. Its direct measurement electricity. One mole of NaCl yields ions carrying 2 faradays of
a
electricity and one mole of Na2SO4 yields ions carrying 4 faradays
is very difficult. It is measured by using standard solution of KCl
of electricity. But 1 gram equivalent of each will produce ions
whose conductivity is known at different concentrations and
carrying 2 faradays of electricity
temperatures. Specific conductance of KCl solutions :
Concentration Specific conductance (W–1 cm–1) at Mol wt. of Na 2SO 4
(Equivalent wt. of Na2SO4 = ).
0°C 18°C 25°C 2
0.1M 0.007154 0.011192 0.012886 Hence conductivity of different electrolytes can only be compared
0.01M 0.0007751 0.0012227 0.0014114 if their solutions have equivalent concentrations.
MOLAR CONDUCTIVITY (Lm) :
Specific conductance
Hence cell constant = Measured conductance The conductivity of all the ions produced when 1 mole of an
electrolyte is dissolved in V ml of solution is known as molar
Ex.5: Specific conductance of a decinormal solution of KCl at conductivity.
18°C is 0.0112 Ohm–1 cm–1. The resistance of the cell or
containing the solution at 18°C was found to be 55 Ohm. 'The conductance of a solution containing one gram-mole of
What is the cell constant. electrolyte placed between two large electrodes one centimeter
apart'.
Specific conductance
Sol. Cell constant = RELATION BETWEEN MOLAR CONDUCTIVITY (Lm)
Measured conductance
AND SPECIFIC CONDUCTIVITY ( k) :
0.0112
= 0.616 cm -1 k ´ 1000
= Lm = k ´ V =
1 M
55 where V is the volume in cm3 or ml containing 1 mole of the
electrolyte.
EQUIVALENT CONDUCTIVITY ( L eq ) :
UNITS OF MOLAR CONDUCTIVITY (Lm) :
The conductivity of all the ions produced when 1 gram equivalent Ohm–1 cm2 mol–1 or W–1 cm2 mol–1 or S cm2 mol–1
of an electrolyte is dissolved in V ml of solution is known as
equivalent conductivity EFFECT OF DILUTION ON MOLAR CONDUCTIVITY:
or Since the degree of dissociation of electrolyte increases with
'The conductance of a solution containing 1 gm. equivalent of dilution, L m also increases but less in case of strong electrolytes
electrolyte placed between two large electrodes one centimetre and more in case of weak electrolytes.
apart'.
562 Chemistry
DEBYE-HUCKEL ONSAGAR EQUATION : 1
Sol. Conductance of solution C = ohm–1
Relation between molar conductivity L m at a particular 45
¥
concentration and molar conductivity L m at infinite dilution l 2.3
is given by Cell constant = = 0.6052
a 3.8
L m = L ¥m - b C Specific conductance
where b is constant. It depends upon nature of solvent and
1
temperature. = 0.6052 ´ = 0.01345 Ohm-1cm-1
45
FACTORS AFFECTING THE MOLAR
0.01345 ´1000
CONDUCTIVITY Lm : L eq = = 26.90 Ohm -1cm 2 eq -1
1
(I) Nature of electrolyte : The strong electrolytes like KNO3,
KCl, NH4NO3, HCl, H2SO4, NaOH, KOH etc are completely 2
ionised in aqueous solution and have high values of Lm. N
Ex.7: The specific conductance of an solution of KCl at
The weak electrolytes are ionised to lesser extent in aqueous 50
solution and have lower values of Lm. 25°C is 0.002765 mho. If the resistance of a cell containing
(II) Concentration of the solution : The concentrated solutions this solution is 400 ohms, what is the cell constant.
of strong electolytes have significant inter-ionic attractions,
which reduce the speed of ions and lower the value of Lm. Sol.: Cell constant=
0.002765
= 0.002765 ´ Resistance
The dilution decreases such attractions and increases the Conductance
value of L m. The limiting value L m( L¥m the molar = 0.002765 ´ 400 = 1.106
conductivity at zero concentration or at infinite dilution) KOHLRAUSCH'S LAW :
can be obtained by extrapolating the graph.
At infinite dilution the molar conductivity of an electrolyte is the
L¥m sum of the ionic conductivities of the cations and anions. e.g. for
KCl AxBy.
Lm
CH3 COOH
( )
L ¥m (A x By ) = xl¥ A+ + yl¥ B- ( )
It is important to note that the source of cations (A+) and anions
C (B–) may be electrolyte itself or any other electrolyte.
¥ ¥
In case of weak electrolytes, the degree of ionisation Thus L¥m (CH3COOH) = l CH3COO- + l H +
increases which increases the value of Lm. The limiting value
L¥m cannot be obtained by extrapolating the graph. The ¥ ¥ ¥
(II) L m (Al2 (SO4 )3 ) = 2l Al3+ + 3lSO24-
limiting value, L¥m , for weak electrolytes is obtained by APPLICATIONS OF KOHLRAUSCH'S LAW :
Kohlrausch law. (I) Calculation of molar conductivities of weak electrolytes at
(III) Temperature : The increase of temperature decreases inter- infinite dilution.
ionic attractions, solvation of ions, viscosity and increases Ex.8:Calculate the degree of dissociation of 0.01 M NH4OH
kinetic energy of ions and their speed. Thus Lm increases solution when its molar conductivity is 9.4 Ohm–1 cm2 mol–
with temperature. 1. The molar conductivities at infinite dilution of NaCl, NH Cl
4
(IV) Viscosity of solvent : The higher the value of viscosity the and NaOH are 126, 130 and 217 Ohm–1 cm2 mol –1
lower is the value of Lm. respectively.
(V) Dielectric constant of solvent : The higher the value of
Sol. Lm (NaCl) = lNa+ + lCl- = 126 Ohm cm mol ( I)
¥ ¥ ¥ -1 2 -1
dielectric constant of solvent, the more is the value of Lm.
The former decreases interionic attractions.
L¥m (NH4Cl) = l¥NH+ + lCl
¥ -1
- = 130 Ohm cm mol
2 -1 (II)
DEGREE OF DISSOCIATION (a) : 4

For weak electrolytes L ¥m (NaOH) = l ¥Na + ¥


+ l OH - = 217 Ohm
-1
cm 2 mol -1 (III)
L cm Molar conductivity at a given concentration Add II and III and subtract I
a= =
L ¥m Molar conductivity at infinite dilution
L ¥m (NH 4 OH) = (130 + 217) – (126) = 221 Ohm–1cm2mol–1
N Degree of dissociation
Ex.6: The resistance of solution of an electrolyte in a cell
2
was found to be 45 ohm. Find the equivalent conductance of the L cm (NH 4 OH) 9.4
a= = = 4.25%
solution if the electrodes in the cell are 2.3 cm apart and have a L ¥m (NH 4 OH) 221
area of cross section 3.8 cm2.
Electrochemistry 563
(II) Calculation of the solubility of sparingly soluble salts:
speed
Very small amount of sparingly soluble salts like AgCl or Ionic Mobility U =
potential gradient
BaSO4 is present in their saturated solutions in completely
ionised form. Their L eq equal to k V can be regarded as POTENTIAL GRADIENT :
Potential difference applied at the electrodes divided by the
L¥eq at infinite dilution. According to Kohlrausch law its distance between the electrodes is known as Potential gradient.
value is given by GALVANIC CELL, ELECTROCHEMICAL CELL OR
VOLTAIC CELL :
kV = L ¥eq = l ¥a + l ¥c
A device which converts chemical energy into electrical energy
Hence knowing the value of k (Kappa), V can be known is called Galvanic, Electrochemical or voltaic cell. Main features
which is the volume containing 1 gev of substance and of the cell are
solubility can be known. General representation of an electrochemical cell
(III) Calculation of degree of ionisation :
e
Lcm Molar conductivity at particular concentration
a= =

n+ n+
m
Molar conductivity at infinite dilution M(s) | M (aq) (1M, 1 atm, 298 K) | | M (aq) (1M, 1 atm, 298 K) | M’(s)
Anode Oxidation Salt Reduction Cathode
(IV) Calculation of absolute ionic mobilities : half cell bridge half cell
The ionic mobility ( U ¥ ) and ionic conductance ( l¥ ) at
Cathode Anode
infinite dilution are related to each other as below Sign Positive due to Negative due
consumption of electrons to release of electrons
l¥ l¥
U¥ = = (IF = 96500 coulombs) Reaction Reduction Oxidation
F 96500 Movement Into the cell Out of cell
of electrons
IONIC MOBILITY :
Other features of the electrochemical cell are :
The distance travelled by an ion per second under a potential (i) There is no evolution of heat
gradient of 1 volt per cm is known as ionic mobility.
(ii) The solutions remain neutral on both side
(iii) The reaction and flow of electrons stops after some time.

DIFFERENCE BETWEEN ELECTROCHEMICAL CELL AND ELECTROLYTIC CELL :


Features Electrochemical cell Electrolytic cell

– +

+ –
– + –
– +
– +

– +
M ® M n+ + n e M n+ + ne - ® M
Anode Cathode

1. Assembly It is a combination of two half cells, It is a single cell containing the same electrodes
containing the same or different present in the same electrolyte.
electrodes in the same or different electrolytes.
2. Nature of electrodes Anode is negative, Cathode is positive Anode is positive, Cathode is negative
3. Movement of From anode to cathode in Electrons enter through cathode and
electrons external circuit leave through anode.
4. Energy It converts chemical energy It converts electrical energy into chemical
into electrical energy, produced energy. Energy is supplied to the electrolytic
as a result of redox reaction. solution to bring about the redox reaction.
5. Spontaneity Cell reaction is spontaneous. Cell reaction is non-spontaneous.
6. Salt bridge Salt bridge is required. No salt-bridge is required.
564 Chemistry
DANIELL CELL : Cr3+/Cr Cr 3+ + 3e - ¾
¾® Cr –0.74
An electrochemical cell of Zinc and copper metals is known as
Daniell Cell. It is represented as Fe3+/Fe Fe3+ + 3e - ¾
¾® Fe –0.41

Cd++/Cd Cd + + + 2e - ¾
¾® Cd –0.40
Zn plate Voltmeter Cu plate
Ni + + + 2e - ¾
Anode Cathode Ni++/Ni
Salt ¾® Ni –0.25
bridge
0.1 M 0.1 M Sn++/Sn Sn + + + 2e - ¾
¾® Sn –0.14
ZnSO4 CuSO 4
1 1
H+/ H 2 H + + e- ¾
¾® H 2 0.00
2 2
Zn (s) | Zn++ (aq.) | | Cu2+ (aqs.) | Cu (s)
Cu++/Cu Cu + + + 2e - ¾
¾® Cu 0.34
¾® Zn 2+ + 2e -
LHS oxidation : Zn ¾
1 1
I2 / I I2 + e- ¾
¾® I 0.54
RHS reduction : Cu 2+ + 2e - ¾
¾® Cu 2 2
Overall reaction :
Ag+/Ag Ag + + e - ¾
¾® Ag 0.80
2+ 2+
Zn + Cu (aq) ¾¾
® Zn (aq) + Cu
Hg++/Hg Hg + + + 2e - ¾
¾® Hg 0.85
By convention cathode is represented on the RHS and anode
on the LHS. Two vertical lines represent the salt bridge.
1 1
Br2 / Br - Br2 + e - ¾
¾® Br - 1.09
FUNCTION OF SALT BRIDGE : 2 2
(I) Completes the circuit and allows the flow of current (II) It
1
maintain the electrical neutrality on both sides. Salt-bridge Cr2 O 72- / Cr 3+ 1
Cr2 O72- + 7H + + 3e - ¾¾
7
® Cr 3+ + H 2 O 1.33
generally contains solution of strong electrolyte such as KNO3, 2 2 2
KCl etc. KCl is preferred because the transport numbers of K+
1 1
and Cl– are almost same. Cl 2 / Cl - Cl 2 + e - ¾
¾® Cl - 1.36
2 2
ELECTRODE POTENTIAL :
When an electrode is in contact with the solution of its own ions in MnO -4 / Mn + + MnO 4- + 8H + + 5e - ¾
¾® Mn + + + 4H 2 O 1.51
a half cell, it has a tendency to lose or gain electrons which is known
as electrode potential. It is expressed in volts. H2O2/H2O H 2 O 2 + 2H + + 2e - ¾
¾® 2H 2 O 1.77
Some standard electrode potentials (E° Red) at 298K (ECS)
1 1
Couple Half-Reaction E° Volts F2 / F - F2 + e - ¾
¾® F - 2.87
2 2
Li+/Li Li + + e - ¾
¾® Li –3.05
Hg +2 + / 2Hg Hg 2+ + + 2e - ¾
¾® 2Hg 0.79
M+/M M + + e- ¾
¾® M –2.94
NO 3- / NO, H 2 O NO 3- + 4H + + 3e - ¾
¾® NO + 2H 2 O 0.97
(K, Rb or Cs)
Ba++/Ba Ba + + + 2e - ¾
¾® Ba –2.90 Au3+/Au Au 3+ + 3e - ¾
¾® Au 1.42

OXIDATION POTENTIAL :
Sr++/Sr Sr + + + 2e - ¾
¾® Sr –2.89
The tendency to lose electrons in above case is known as
Ca++/Ca Ca + + + 2e - ¾
¾® Ca –2.76 Oxidation potential.
Oxidation potential of a half-cell is inversely proportional to
Na+/Na Na + + e - ¾
¾® Na –2.71 the concentration of ions in the solution.
REDUCTION POTENTIAL :
Mg++/Mg Mg + + + 2e - ¾
¾® Mg –2.38
The tendency to gain electrons in above case is known as
1 1 reduction potential.
H2 / H- H 2 + e- ¾
¾® H - –2.23 It is not possible to determine the absolute value of electrode
2 2
potential. For this a reference electrode is required. The
Al3+/Al Al3+ + 3e - ¾
¾® Al –1.67 electrode potential is only the difference of potentials between
two electrodes that we can measure by combining them to give
Zn++/Zn Zn + + + 2e - ¾
¾® Zn –0.76 a complete cell.
Electrochemistry 565
Reduction :
STANDARD REDUCTION POTENTIAL ( Eo ):
Mn+ /M
Hg 2 Cl 2 (s) ® Hg 22 + + 2Cl -
According to latest convention the electrode potential is always
represented as reduction potential. If its value is negative, it means Hg 22+ + 2e - ® 2Hg(l)
electrode has oxidation potential. The standard conditions are 1 Overall reaction :
molal solution, 298K temperature and 1 atm. pressure. According
2Hg(l) + 2Cl
– –
to IUPAC convention, the reduction potential alone be called as Hg2Cl2(s) +2e E ºRe d = 0.2422 V
the electrode potential unless it is specifically mentioned. (iii) Silver-silver salt electrode
REFERENCE ELECTRODE : Half cell is Ag, AgCl(s), Cl– (KCl or HCl)
The following electrodes are used as reference electrodes for Oxidation :
determining the standard reduction potentials.
Ag ®Ag+ + e-
(i) Standard hydrogen electrode (SHE) : Standard hydrogen
Ag + + Cl - ® AgCl(s)
electrode (SHE) also known as Normal Hydrogen Electrode
(NHE), consists of platinum wire, carrying platinum foil
coated with finely divided platinum black. The wire is sealed Ag + Cl - ® AgCl(s) + e -
into a glass tube, placed in a beaker containing 1M HCl. The
hydrogen gas at 1 atm pressure is bubbled through the Reduction :
solution at 298K. Half cell is Pt, H2(1 atm)/H+ (1M)
AgCl(s) ® Ag + + Cl -
In SHE, at the surface of platinum, either of the following
reactions can take place Ag + + e - ® Ag

AgCl(s) + e - ® Ag + Cl -

EoRed = 0.2225 V
H2(g) at
1 bar ELECTROMOTIVE FORCE (EMF) OF A CELL :
It is the difference between the electrode potentials of two half-
cells and cause of flow of current from electrode at higher
potential to electrode at lower potential. It is also the measure of
free energy change. Standard EMF of a cell
E ocell = E ocathode - E oanode = E oright - E oleft

1.00 MH
+ Finely divided
platinum coated
ELECTROCHEMICAL SERIES :
on platinum foil It is the arrangement of electrodes in the increasing order of their
o
+ - Standard Reduction potential E M n + / M .
2H (aq) + 2e ® H 2 (g) Reduction
APPLICATIONS OF ELECTROCHEMICAL SERIES :
H 2 (g) ¾¾ ® 2H + (aq) + 2e - Oxidation
(I) The lower the value of E°, the greater the tendency to form
The electrode potential of SHE has been fixed as zero.
cation
All other single electrode potentials are referred to as
potentials on hydrogen scale. M – ne ¾ ¾® M n +
Drawbacks of SHE (II) Replacement (or evolution) of H2 from hydro acids by metals.
1. It is difficult to maintain H2 at 1 atm. pressure.
Metals placed below hydrogen in E.C.S. can not replace
2. It is difficult to maintain H+ ion concentration at 1M
hydrogen from dil. acids but metals placed above hydrogen
3. The platinum electrode is easily poisoned by traces of
can replace hydrogen from dil. acids.
impurities.
Hence calomel and silver chloride electrodes are Ca + H 2SO 4 ¾
¾® CaSO 4 + H 2 ­
conveniently used as reference electrodes.
possible
(ii) Calomel electrode (Hg, Hg2Cl2, KCl)
Half cell is Hg, Hg2Cl2(s) KCl(solution) (Ca + 2H + ® Ca + + + H 2 )
Oxidation :
Cu + H 2SO 4 ¾
¾® CuSO 4 + H 2 ­
2Hg(l) ® Hg 22+ + 2e - not possible
Hg 22 + + 2 Cl -
® Hg 2 Cl 2 (Cu + 2H + ® Cu + + + H 2 )
(III) Metals placed above hydrogen evolve H2 with H2O or steam,
2Hg (l) + 2Cl ® Hg 2 Cl 2 (s) + 2e - but metals placed below hydrogen cannot.
566 Chemistry
(IV) Oxides of metals placed above hydrogen are not reduced
by H2 but oxides of iron and metals placed below iron are Ag(s) + Br - (aq) ® AgBr(s) + e -
reduced by H2
SnO, PbO, Fe3O4, CuO are reduced with H2 AgBr(s) + e - ¾¾
® Ag(s) + Br - (aq)
CaO, MgO, K2O are not reduced with H2 Cl–(aq) / Hg2Cl2(s) / Hg (Pt) :
(V) Oxides of Ag and metals below Ag are decomposed
2Hg(l) + 2Cl- ® Hg 2 Cl2 (s) + 2e -
2HgO ¾
¾® 2Hg + O 2
Hg 2 Cl 2 (s) + 2e - ¾¾
® 2Hg(l) + 2Cl - (aq)
2 Ag 2 O ¾
¾® 4Ag + O 2
OH–(aq) / Cu(OH)2(s) / Cu :
(VI) Reducing character decreases down the series.
(VII) Reactivity decreases down the series. Cu(s) + 2OH - (aq) ® Cu(OH) 2 + 2e -
(VIII)Feasibility of redox reaction : A redox reaction can occur if
the species losing the electrons lie above that which gains Cu(OH) 2 (s) + 2e - ¾¾
® Cu(s) + 2OH - (aq)
the electrons.
(IX) Determination of emf : Emf is the difference of reduction SO--
4 (aq) / PbSO 4 (s) / Pb :
potentials of two half cells.
Eemf = ERHS – ELHS
Pb(s) + SO 4-- (aq) ® PbSO 4 (s) + 2e -
If the value of e.m.f. is positive the reaction can take place
spontaneously, otherwise not.
PbSO 4 (s) + 2e- ¾¾
® Pb(s) + SO --
4 (aq)
FUNCTIONING OF ELECTROCHEMICAL CELL :
With the passage of time the electrode potential of the cathode (IV) Redox electrodes half cells : An inert metal such as platinum
decreases and that of anode increases the difference becomes wire dipped in a solution of ions of the same metal in different
zero, the driving force of emf becomes zero and reaction stops. oxidation states
TYPES OF ELECTRODES AND HALF-CELLS : Fe3+(aq), Fe2+(aq) / Pt : Fe 2+ (aq) ® Fe3+ (aq) + e-
(I) Gas electrodes or gas Ion half cells :
Half-cell Reaction Fe3+ (aq) + e - ¾¾
® Fe 2 + (aq)
Oxidation
Sn4+(aq), Sn2+(aq) / Pt: Sn 2+ (aq) ® Sn 4+ (aq) + 2e-
H + (aq) / H 2 (g) / Pt : H 2 (g ) 2H+ + 2e–

2H + (aq) + 2e - H2(g) Sn 4 + (aq) + 2e - ¾¾


® Sn 2+ (aq)
Reduction Cr2O72–(aq), Cr3+(aq), H+(aq) / Pt :
Cl - (aq) / Cl 2 (g) / Pt : Cl (aq) Cr2 O72- (aq) + 14H + (aq) + 6e - ¾¾
® 2Cr 3+ (aq) + 7H 2 O

½Cl(g) + e–
1 MnO–4 (aq); Mn2+(aq), H+(aq) / Pt :
Cl 2 (g) + e - Cl - (aq)
2
(II) Metal-Metal ion electrode or half-cell : A metal rod dipped MnO 4- (aq) + 8H + (aq) + 5e- ¾¾
® Mn 2+ (aq) + 4H 2 O(l )
in the solution of its own ions
NERNST EQUATION FOR ELECTRODE POTENTIAL:
Zn++(aq) / Zn : Zn ® Zn + + + 2e - The relationship between the concentration of ions and electrode
Zn + + + 2e - ¾
potential is given by Nernst equation.
¾® Zn
Ag+(aq) / Ag : Ag ® Ag+ + e- M n + + ne- ® M

2.303RT 1
Ag + + e - ¾
¾® Ag E M n + / M = E oM n + / M - log
nF [M n + ]
Cu++(aq) / Cu : Cu ® Cu ++ + 2e-
2.303 ´ 8.314 ´ 298 1
Cu + + + 2e - ¾
¾® Cu E M n + / M = E oM n + / M - log
n ´ 96500 [M n + ]
(III) Metal-metal insoluble Salt-Salt anion :
Cl–(aq) / AgCl / Ag:
0.059 1
E M n + /M = E oM n+ /M - log n +
Ag(s) + Cl – (aq) ® AgCl(s) + e - n [M ]

AgCl(s) + e - ¾¾
® Ag(s) + Cl - (aq) If we write the elcetrode reaction, in general, as
Br–(aq) / AgBr / Ag : Oxidised state + ne– Reduced State
Electrochemistry 567
The potential of electrode is given by RELATIONSHIP BETWEEN FREE ENERGY CHANGE
AND EQUILIBRIUM CONSTANT :
RT [Reduced state]
E cell = E cell
o
- 2.303 log
nF [Oxidised state] DGo = -2.303RT log Kc

2.303RT [Oxidised state] CONCENTRATION CELLS :


or E cell = E cell
o
+ log They are of two types
nF [Reduced state]
(I) Electrode concentration cells : Two electrodes of different
Ex.9: What is the electrode potential of a half cell for concentrations are dipped in the same solution of electrolyte
magnesium electrode dipped in a 0.1 M MgSO4 solution at e.g.
25°C. (E° = –2.36).
Pt, H 2 (PH 2 = p1 ) | H + | H 2 (PH 2 = p 2 )Pt p1 > p 2
0.0591 1
Sol.: E = -2.36 - log = –2.389V 2.303RT p
Mg ++ /Mg 2 0.1 E cell = log 1
nF p2
NERNST EQUATION FOR CELL POTENTIAL : (II) Electrolyte concentration cells : Electrodes are the same
Consider a general cell reaction involving n electrons. but electrolyte solutions have different concentrations e.g.

aA + bB ¾
¾® cC + dD Zn, Zn + + (c1 ) | | Zn + + (c 2 ); Zn c 2 > c1

2.303 c 0.0591 c
2.303RT [C]c [D]d E cell = RT log 2 = log 2
E cell = E cell
o
- log nF c1 n c1
nF [A]a [B]b
Concentration of solids and liquids is taken as unity, APPLICATIONS OF THE CONCENTRATION CELLS :
concentration of ions Mol L–1 and concentration of gases as (I) Determination of valency
partial pressures in Atmosphere. (II) Determination of solubility of sparingly soluble salts.
NERNST EQUATION AND EQUILIBRIUM (III) Determination of transition point.
CONSTANT (Kc) : OVERVOLTAGE :
When Ecell drops to zero the concentration of ions will be The difference between the voltage at which a gas is actually
equilibrium concentrations. We have for general cell reaction evolved and theoretical value at which it ought to have been
evolved during electrolysis is known as overvoltage.
aA + bB cC + dD
REVERSIBLE CELL :
2.303RT [C] [D] c d When the cell reaction can be stopped or reversed by applying
E cell = E cell
o
- log an emf exactly equal to or infinitesimally greater than that of the
nF [A]a [B]b
cell, it is called reversible cell.
Examples of reversible electrodes
2.303RT
E ocell = log K c [Since at equilibrium Ecell = 0] 1. Metal-metal ion electrode : Metal rod dipped into a solution
nF
of its own ions.
0.0591 + –
E ocell = log Kc at 298K. M M +e
n The negative electrode (electrode reaction involving
RELATION BETWEEN ELECTRICAL ENERGY AND oxidation) increases M+ ions in solution
FREE ENERGY (DG) : The positive electrode (electrode reaction involving
reduction) decreases M+ ions in solution.
If n is the number of electrons liberated (or taken up) in a particular
Thus, electrode is reversible with respect to M+ ions
cell reaction, then n faradays (nF) of electricity will be generated
in the complete reaction. If E is the EMF of the cell, then 2. Gas electrodes :
(a) Hydrogen electrode : Pt, H2(g), H+
Electrical energy supplied by the cell = nFE
Reaction
According to Gibbs and Helmholtz, the decrease of free energy
1 H (g) H+ + e–
(–DG) of the reaction occuring in the cell is equal to electrical 2 2
energy The electrode is reversible with respect to H+ ions
Hence, –DG = nFE (b) Chlorine electrode : Pt, Cl2(g), Cl–
Reaction
The standard free energy and E ocell are related as
1 – –
Cl (g) + e Cl (aq)
2 2
DG = o
- nFE ocell The electrode is reversible with respect to Cl– ion.
568 Chemistry
(c) Metal - Metal Salt Ion Electrode : (I) Electrolysis of aqueous NaOH :
¾® Na + + OH -
i) Calomel Electrode
NaOH ¾ Completely dissociated
Hg, Hg2Cl2(s) ; KCl(solution)
Oxidation : ¾® H + + OH - Slightly dissociated
H 2O ¾

2Hg(l )
2+
Hg 2 + 2e
– At cathode : 2 H + + 2e - ¾
¾® 2H ¾
¾® H 2
2+ –
Hg 2 + 2Cl Hg2Cl2(s) At anode : 2OH - - 2e - ¾ ¾® H 2 O + O; O + O ¾
¾® O 2
2Hg + 2Cl

Hg2Cl2(s) + 2e
– Current is carried by Na+ and OH– ions.
(II) Electrolysis of aqueous H2SO4
Oxidation decreases concentration of chloride ions.
¾® 2H + + SO 24 -
H 2SO 4 ¾ Completely dissociated
Reduction :
2+ – ¾® H + + OH -
H 2O ¾ Slightly dissociated
Hg2Cl2(s) Hg + 2e

2
At cathode : 2H + + 2e - ¾
¾® 2H ¾
¾® H 2
2Hg(l )
2+
Hg + 2e
2
At anode : 2OH - - 2e - ¾
¾® 2OH ¾
¾® H 2 O + O;
2Hg(l ) + 2Cl
– –
Hg2 Cl2(s) + 2e
O+O¾
¾® O 2
Reduction increases the concentration of chloride ions
(III) Electrolysis of aqueous NaCl
The electrode is reversible with respect to Cl– ions.
ii) Silver-Silver Chloride Electrode : Ag, AgCl(s), Cl–(KCl ¾® Na + + Cl -
NaCl ¾ Completely dissociated
or HCl)
¾® H + + OH -
H 2O ¾ Slightly dissociated
Oxidation :
+ – At cathode : 2 H + + 2e - ¾
¾® 2H ¾
¾® H 2
Ag Ag + e
+
Ag + Cl

AgCl(s) At anode : 2Cl - - 2e - ¾
¾® 2Cl ¾ ¾® Cl 2
– Although the oxidation potential of OH– is more than Cl–
Ag + Cl AgCl(s)+ e– yet Cl2 is formed at anode due to overvoltage.
Oxidation decreases the concentration of Cl– ions. (IV) Electrolysis of aqueous CuSO4 using Pt electrodes
Reduction : ¾® Cu + + + SO -4 -
CuSO 4 ¾
+ –
AgCl(s) Ag + Cl ¾® H + + OH -
H 2O ¾
+ –
Ag + e Ag At cathode Cu + + + 2e - ¾
¾® Cu
– –
AgCl(s) + e Ag + Cl At anode 2OH - - 2e - ¾
¾® 2OH ¾
¾® H 2 O + O;
Reduction increases the concentration of Cl– O+O¾ ¾® O 2
(d) Oxidation - Reduction Electrodes : Such electrodes (V) Electrolysis of CuSO4 solution using copper electrodes
are set up by inserting unattackable metal (eg Platinum)
into a solution of ions in different oxidation states. The ¾® Cu + + + SO -4 - ;
CuSO 4 ¾ ¾® H + + OH -
H 2O ¾
metal acquires a potential due to tendency of ions in
one oxidation state to change into another stable
At Cathode Cu + + + 2e - ¾
¾® Cu
oxidation state. At Anode ¾® Cu + + + 2e -
Cu ¾
Electrode reaction :
Oxidation potential Cu > OH - > SO 4- -
n+ – (n+m)+ - 3. 4 -1.23 - 2.01
M – me M
2+ – 3+
Fe – e Fe FACTORS GOVERNING THE DISCHARGE
2+ – 4+
Sn – 2e Sn POTENTIAL :
(I) Position in the electrochemical series
IRREVERSIBLE CELL : (II) Concentration of ions in the solution
When the cell reaction cannot be stopped or reversed it is called (III) Nature of electrodes
irreversible cell e.g. cell of Zn and Ag electrodes immersed in Discharge potential of some ions
solution of H2SO4.
Criteria of the formation of products in Electrolysis - Li + < K + < Na + < Ca + + < Mg + + < Al + + < Zn + + < Fe + +

< Ni + + < H + < Cu + + < Hg + + < Ag +


Preferential discharge theory : Ions present in large excess
conduct electricity and ions having lower discharge potentials
are discharged at respective electrodes. < Au + + + < SO -4 - < NO 3- < OH - < Cl - < Br - < I -
Electrochemistry 569
TRANSPORT NUMBER : (b) Nickel-cadmium storage cell
The fraction of the total current carried by each ion is called its Anode - Cadmium
transport number. Cathode - metal grid containing NiO2
The amount of electricity carried by a particular ion µ speed of Electrolyte - KOH solution
particular ion. Anode Reaction Cd ( s ) + 2OH - ® Cd(OH) 2 ( s ) + 2e -
Transport number of the cation nc Cathode Reaction :

=
Current carried by the cation
=
Uc
NiO 2 ( s ) + 2H 2 O ( l ) + 2e - ® Ni ( OH )2 ( s ) + 2OH - ( aq )
Total current U c + Ua
Net Reaction :
Ua Cd ( s ) + NiO2 ( s ) + 2H 2 O ( l )
Transport number of anion na =
Uc + Ua
® Cd(OH)2 ( s ) + Ni(OH)2 ( s )
Uc = speed of cation and Ua = speed of anion.
Cell potential = 1.4 V.
Further na + nc = 1
FUEL CELLS :
SOME COMMERCIAL CELLS AND THEIR TYPES :
They convert chemical energy into electrical energy and the
They are broadly classified into two groups. reactants are continuously fed and products are removed.
(I) Primary cells : They cannot be recharged and used again. (a) Hydrogen-Oxygen-Fuel Cell : Electrodes are made of porous
Examples are graphite impregnated with catalyst (Pt, Ag or a metal oxide).
(a) Dry cell or Leclanche cell : Particulars are
Anode - Zinc Container H2O
Cathode - graphite rod surrounded by MnO2 powder Anode Cathode
Electrolyte - paste of NH4Cl + ZnCl2
Aqueous electrolyte
Cathode Reaction : MnO2 + NH +4 + e– ¾¾® MnO(OH) + NH3

Anode Reaction : Zn – 2e - ¾
¾® Zn + +
H2 O2
Zn 2 + + 2 NH 3 ¾
¾®[ Zn ( NH 3 ) 2 ]2+
Cell potential 1.25V to 1.5V
(b) Mercury Cell :
Anode - Zn-Hg amalgam
Cathode - paste of (HgO + C) Fuel cell using H2 and O2 produces electricity.
Electrolyte - moist paste of KOH-ZnO
Cathode Reaction : Electrolyte-aqueous solution of KOH or NaOH
- -
HgO(s) + H 2 O( l ) + 2e ¾¾
® Hg(l) + 2OH Oxygen and hydrogen are continuously fed into the cell.
Oxidation (half-cell reaction) :
Anode Reaction : Zn + 2OH - ¾¾
® ZnO + H 2 O(l) + 2e - Reaction : H 2 + 2OH - ¾
¾® 2H 2 O + 2e -
® ZnO(s) + Hg(l)
Net Reaction : Zn(Hg) + HgO(s) ¾¾ Reduction (half-cell reaction) :
cell potential = 1.30 volt O 2 + 2H 2 O + 4e - ¾
¾® 4OH -
(II) Secondary cells : It can be recharged and can be used again
and again. Examples Net Reaction : 2H 2 + O 2 ¾
¾® 2H 2 O
(a) Lead storage battery : EMF of the cell 1 volt.
Anode - Spongy lead (b) Hydrocarbon-oxygen fuel cell : Based upon the combustion
Cathode - grid of lead packed with PbO2 of hydrocarbons such as methane, ethane propane etc.
Electrolyte - 38% H2SO4 by mass Oxidation (half cell reaction) :
Anode Reaction : Pb + SO 4- - ¾
¾® PbSO 4 + 2e - C 3 H 8 + 20 OH - ¾¾
® 3CO 2 + 14H 2 O + 20e -
Cathode Reaction : Reduction (half cell reaction) :
( O 2 + 2 H 2 O + 4e - ¾
¾® 4OH - ) ´ 5
PbO 2 + SO 4- - + 4H + + 2e - ¾
¾® PbSO 4 + 2H 2 O
Net Reaction : Net Reaction C 3 H 8 + 5O 2 ¾
¾® 3CO 2 + 4 H 2 O

Pb + PbO 2 + 4H + + 2SO 4- - ¾
¾® 2PbSO 4 + 2H 2 O ADVANTAGES OF FUEL CELLS :
When recharged the cell reactions are reversed. I. Pollution free II. High efficiency
570 Chemistry
THERMODYNAMIC EFFICIENCY OF FUEL CELLS : 1
Net Reaction, Fe + 2H + + ¾® Fe + + + H 2 O
O2 ¾
-DG -nFEocell 2
h= =
DH DH
1
For H2–O2 fuel cells it is 95%. At surface 2Fe + + + O 2 + 2H 2 O ¾
¾® Fe 2 O 3 + 4H +
2
CORROSION :
Slow formation of undesirable compounds such as oxides, Fe 2 O 3 + xH 2 O ¾
¾® Fe 2 O 3 .xH 2 O
sulphides or Carbonates at the surface of metals by reaction with
moisture and other atmospheric gases is known as corrosion. Net reaction at surface
FACTORS AFFECTING THE CORROSION : 1
2Fe + + + O 2 + ( x + 2)H 2 O ¾
¾® Fe 2 O 3 xH 2 O+ 4H +
(I) Reactivity of metals 2 Rust
(II) Presence of moisture and atmospheric gases like CO2, SO2
etc. PREVENTION OF CORROSION :
(III) Presence of impurities (I) Barrier protection : By painting, coating, electroplating
(IV) Strains in the metal (II) Sacrificial protection : By galvanization, Sherardising
(V) Presence of electrolyte (III) Electrical protection
RUSTING OF IRON-ELECTROCHEMICAL THEORY : (IV) Use of anti rust compounds.
An electrochemical cell known as corrosion cell is developed at
CALCULATION OF POTENTIAL OF INTERMEDIATE
the surface of iron.
Anode ® Pure iron
REACTION :
Cathode ® Impure Surface When two half-reactions having potential E1 and E2 which are
Electrolyte : combined to yield a third half reaction having potential E3. Then
E3 is given by
CO 2 + H 2 O ¾
¾® H 2 CO 3 H + + HCO 3-

SO 2 + H 2 O ¾
¾® H 2SO 3 H + + HSO 3- n1E1 + n 2 E 2
E3 =
n
Anode Reaction ¾® Fe + + + 2e -
Fe ¾
Remember : The cell potentials are not thermodynamic functions
Cathode Reaction 2 H + + 2e - ¾
¾® 2H
and should not be added.
1
2H + O2 ¾
¾® H 2 O
2
Electrochemistry 571

Very Short/Short Answer Questions Long Answer Questions


1. Which is a better conductor, aqueous solution of NaCl or 12. Explain why electrolysis of aqueous solution of NaCl gives
iron? H2 at cathode and Cl2 at anode. Write overall reaction.
2. Can a nickel spatula be used to stir a solution of CuSO4? E° =- 2×71V, E°H =- 0×83V,
Na + / Na 2O / H 2
( E º 2+ = – 0 × 25 V;; E 2+ = + 0 × 34 V) E° =+ 1×36V, E °O =1×23V
Cl2 / Cl-
Ni /Ni Cu /Cu
2 / H2O
3. Why do the electrochemical cells stop working after
sometime? 13. (a) What type of a battery is the lead storage battery?
Write the anode and cathode reactions and the overall
4. Can we find out the value of L °CH COOH by extrapolating a reaction occurring in a lead storage battery when cur-
3
graph of molar conductivity of CH 3COOH solution rent is drawn from it.
against c ? (b) In the button cell, widely used in watches, the follow-
5. Electrolysis of NaBr and NaI in aqueous solution liberate ing reaction takes place
Br2 and I2 respectively while that of NaF liberates O2 instead Zn ( s ) + Ag 2O( s ) + H2O ® Zn 2+ ( aq )
of F2. Explain.
6. In each of the following pairs, which will provide greater +2 Ag( s ) + 2OH -( aq )
conduction of electricity and why? Determine E° and DG° for the reaction.
(i) Copper wire at 30 ºC or same copper wire at 100 ºC.
(ii) KCl solution at 25 ºC or same KCl solution at 50 ºC.
( given : E°Ag + /Ag = + 0.80V
(iii) 0×1 M NH4OH at 25ºC or 2M NH4OH at 25 ºC. E°
Zn 2 + /Zn
= - 0.76 V )
7. Consider a cell composed of two half-cells:
14. (a) Define molar conductivity of a solution and explain
(i) Cu(s) |Cu2+ (aq), and
how molar conductivity changes with change in con-
(ii) Ag(s) |Ag+ (aq). centration of solution for a weak and a strong electro-
Calculate lyte.
(a) the standard cell potential, and (b) The resistance of conductivity cell containing 0.001 M
(b) the cell potential when concentration of Cu2+ is 2M KCl solution at 298 K is 1500 W. What is the cell con-
and concentration of Ag+ is 0×05 M, at 298 K. stant if the conductivity of 0.001 M KCI solution at 298
K is 0.146 × 10–3 S cm–1?
Given : E º = + 0.34V, E º = 0×80 V,
Cu 2+ /Cu Ag + /Ag 15. (a) State Kohlrausch law of independent migration of ions.
R = 8×314 JK–1 mol–1 F = 96500 C mol–1 Write an expression for the molar conductivity of ace-
8. Silver is electrodeposited on a metallic surface area of 800 tic acid at infinite dilution according to Kohlrauch law.
cm2 by passing a current of 0×2A for 3 hours. Calculate the (b) Calculate
thickness of silver deposited. Density of silver is 10×47 g Ù°m for acetic acid,
cm3 and atomic mass of Ag = 108. Given that
9. Express the relation among cell constant, resistance of the
Ù°m (HCl) = 426 S cm2 mol–1
solution in the cell and conductivity of the solution. How is
molar conductivity of a solution related to its conductivity? Ù°m (NaCl) = 126 S cm2 mol–1
10. Calculate the emf of the following cell at 298 K: Ù°m (CH3COONa) = 91 S cm2 mol–1
Fe(s) | Fe2+ (0.001 M) || H+ (1 M) | H2(g) (1bar), Pt (s) 16. (a) Write the anode and cathode reactions and the overall
reaction occurring in a lead storage battery.
[ Given E°Cell = + 0.44 V ] (b) A copper-silver cell is set up. The copper ion
11. For the cell concentration is 0.10 M. The concentration of silver
Zn (s) | Zn2+ (2M) || Cu2+ (0.5 M) | Cu (s) ion is not known. The cell potential when measured
(a) Write equation for each half-reaction. was 0.422 V. Determine the concentration of silver ions
(b) Calculate the cell potential at 25°C in the cell.

éGiven E° 2+ æ Given E ° = + 0.34 V ö


= - 0.76 V; E° 2+ = + 0.34 V ù è Ag + / Ag
= + 0.80V, E ° 2 +
ø
ë Zn /Zn Cu /Cu û Cu / Cu
572 Chemistry

Multiple Choice Questions 21. A hypothetical electrochemical cell is shown below


O_ Å
17. The standard electrode potentials of four elements A, B, C A | A + ( xM) || B+ ( yM) | B
and D are –3.05, –1.66, –0.40 and +0.80. The highest
The emf measured is +0.20 V. The cell reaction is
chemical reactivity will be exhibited by : (a) A+ + e–® A; B++ e– ® B
(a) A (b) B (b) The cell reaction cannot be predicted
(c) C (d) D (c) A + B+ ® A+ + B
18. The aqueous solution of which of the following (d) A+ + B ® A + B+
decomposes on passing electric current 22. On heating one end of a piece of a metal, the other end
(a) cane sugar (b) urea becomes hot because of
(c) methanol (d) potassium iodide (a) resistance of the metal
19. The standard electrode potential of the half cells are given (b) mobility of atoms in the metal
below (c) energised electrons moving to the other end
Zn 2 + + 2e - ¾
¾® Zn ; E = – 7.62 V
(d) minor perturbation in the energy of atoms
23. 4.5 g of aluminium (at. mass 27 amu) is deposited at
Fe 2 + + 2e - ¾
¾® Fe ; E = – 7.81 V cathode from Al 3+ solution by a certain quantity of
electric charge. The volume of hydrogen produced at
The emf of the cell, Fe 2 + +Zn ¾¾® Zn 2 + + Fe is STP from H+ ions in solution by the same quantity of
(a) 1.54 V (b) – 1.54 V electric charge will be
(c) – 0.19 V (d) +0.19 V (a) 44.8 L (b) 22.4 L
20. Two electrolytic cells, one containing acidified ferrous (c) 11.2 L (d) 5.6 L
chloride and another acidified ferric chloride, are connected 24. On passing 0.5 Faraday of electricity through molten sodium
in series. The ratio of iron deposited at cathodes in the two chloride, sodium deposited at cathode will be :
cells will be : (a) 29.25 g (b) 11.50 g
(a) 3 : 1 (b) 2 : 1 (c) 58.50 g (d) 0.00 g
(c) 1 : 1 (d) 3 : 2

1. Which of the following is the use of electrolysis? 6. A current of 2.0 A passed for 5 hours through a molten metal
(a) Electrorefining (b) Electroplating salt deposits 22.2 g of metal (At wt. = 177). The oxidation
(c) Both (a) and (b) (d) Neither (a) nor (b) state of the metal in the metal salt is
2. Which of the following will form the cathode with respect to
(a) +1 (b) +2
iron anode in an electrolytic cell?
(a) Mg (b) Al (c) +3 (d) +4
(c) Cu (d) Zn 7. A 5 ampere current is passed through a solution of zinc
3. The number of coulombs required for the deposition of sulphate for 40 minutes. Find the amount of zinc deposited
107.870 g of silver is at the cathode
(a) 96500 (b) 48250 (a) 40.65 g (b) 4.065 g
(c) 193000 (d) 10000
(c) 0.4065 g (d) 65.04 g
4. An electrolytic cell contains a solution of Ag2SO4 and has
platinum electrodes. A current is passed until 1.6 gm of O2 8. On passing a current of 1.0 ampere for 16 min and 5 sec
has been liberated at anode. The amount of silver deposited through one litre solution of CuCl2, all copper of the solution
at cathode would be was deposited at cathode. The strength of CuCl2 solution
(a) 107.88 gm (b) 1.6 gm was (Molar mass of Cu = 63.5, Faraday constant = 96500 C
(c) 0.8 gm (d) 21.60 gm mol–1).
5. 1.08 g of pure silver was converted into silver nitrate and its (a) 0.07 M (b) 0.2 N
solution was taken in a beaker. It was electrolysed using
platinum cathode and silver anode. 0.01 Faraday of electricity (c) 0.005 M (d) 0.02 N
was passed using 0.15 volt above the decomposition 9. In a solution of CuSO4 how much time will be required to
potential of silver. The silver content of the beaker after the precipitate 2 g copper by 0.5 ampere current ?
above shall be (a) 12157.48 sec (b) 102 sec
(a) 0 g (b) 0.108 g (c) 510 sec (d) 642 sec
(c) 1.08 g (d) None of these
Electrochemistry 573
10. What is the amount of chlorine evolved when 2 amperes of 21. The unit of equivalent conductivity is
current is passed for 30 minutes in an aqueous solution of (a) ohm cm
NaCl? (b) ohm–1 cm+2 (g equivalent)–1
(a) 66 g (b) 1.32 g (c) ohm cm2 (g equivalent)
(c) 33 g (d) 99 g (d) S cm–2
11. When 9.65 coulombs of electricity is passed through a 22. The conductivity of a saturated solution of BaSO 4
solution of silver nitrate (atomic mass of Ag = 108 is 3.06 × 10–6 ohm–1 cm–1 and its equivalent conductance is
g mol–1), the amount of silver deposited is
1.53 ohm–1 cm2 equiv–1. The Ksp for BaSO4 will be
(a) 16.2 mg (b) 21.2 mg
(a) 4 × 10–12 (b) 2.5 × 10–9
(c) 10.8 mg (d) 6.4 mg –13
12. The charge required to deposit 9 g of Al from Al3+ solution (c) 2.5 × 10 (d) 4 × 10–6
is (At. wt. of Al = 27.0) 23. The resistance of 0.1 N solution of a salt is found to be 2.5×
(a) 3216.3 C (b) 96500 C 103 ohm . The equivalent conductance of the solution is
(c) 9650 C (d) 32163 C (cell constant = 1.15 cm–1)
13. The quantity of electricity needed to deposit 127.08 g of (a) 4.6 (b) 5.6
copper is (c) 6.6 (d) 7.6
(a) 1 Faraday (b) 4 Coulombs 24. Specific conductance of 0.1 M sodium chloride solution is
(c) 4 Faraday (d) 1 Ampere 1.06 × 10 –2 ohm –1 cm –1 . Its molar conductance in
14. Silver is monovalent and has atomic mass of 108. Copper is ohm–1 cm2 mol–1 is
divalent and has an atomic mass of 63.6. The same electric (a) 1.06 × 102 (b) 1.06 × 103
current is passed for the same length of time through a (c) 1.06 × 10 4 (d) 5.3 × 102
silver coulometer and a copper coulometer. If 27.0 g of silver 25. Molar conductivity of a solution is 1.26 × 102 W–1 cm2 mol–1.
is deposited, then the corresponding amount of copper Its molarity is 0.01. Its specific conductivity will be
deposited is (a) 1.26 × 10–5 (b) 1.26 × 10–3
(a) 63.60 g (b) 31.80 g (c) 1.26 × 10 –4 (d) 0.0063
(c) 15.90 g (d) 7.95 g
26. Molar ionic conductivities of a two-bivalent electrolytes
15. By passing 0.1 Faraday of electricity through fused sodium
chloride, the amount of chlorine liberated is x 2+ and y 2- are 57 and 73 respectively. The molar
(a) 35.45 g (b) 70.9 g conductivity of the solution formed by them will be
(c) 3.545 g (d) 17.77 g (a) 130 S cm2 mol–1 (b) 65 S cm2 mol–1
16. The unit of specific conductivity is 2 –1
(c) 260 S cm mol (d) 187 S cm2 mol–1
(a) ohm cm–1 (b) ohm cm–2
27. The equivalent conductivity of 0.1 M weak acid is 100 times
(c) ohm–1 cm (d) ohm–1 cm–1
less than that at infinite dilution. The degree of dissociation
17. The cell constant of a given cell is 0.47 cm–1. The resistance
of a solution placed in this cell is measured to be 31.6 ohm. of weak acid will be
The conductivity of the solution (in S cm–1 where S has (a) 100 (b) 10
usual meaning) is (c) 0.01 (d) 0.001
(a) 0.15 (b) 1.5 28. An electrochemical cell is set up as follows :
(c) 0.015 (d) 150 Pt (H2, 1 atm)/0.1 M HCl/0.1 M acetic acid/(H2, 1 atm) Pt
18. The specific conductivity of N/10 KCl solution at 20°C is EMF of this cell will not be zero because
0.212 ohm–1 cm–1 and the resistance of the cell containing (a) the temperature is constant
this solution at 20°C is 55 ohm. The cell constant is (b) the pH of 0.1 M HCl and 0.1 M acetic acid is not the
(a) 4.616 cm–1 (b) 11.66 cm–1 same
(c) 2.173 cm–1 (d) 3.324 cm–1 (c) acids used in the two compartments are different
19. The resistance of 1 N solution of acetic acid is 250 ohm, (d) EMF of a cell depends on molarities of the acids used
when measured in a cell of cell constant 1.15 cm–1. The 29. Which one of the following reaction occurs at the cathode?
equivalent conductance (in ohm–1 cm2 equiv–1) of 1 N acetic
(a) 2OH - ¾ ¾® H 2 O + O + 2e -
acid will be
(a) 4.6 (b) 9.2 ¾® Ag + + e -
(b) Ag ¾
(c) 18.4 (d) 0.023
(c) Fe 2 + ¾
¾® Fe 3+ + e -
20. The equivalent conductance at infinite dilution of a weak
acid such as HF (d) Cu 2 + + 2e - ¾
¾® Cu
(a) can be determined by extrapolation of measurements of 30. Which of the following statements is true for an
dilute solutions of HCl, HBr and HI electrochemical cell?
(b) can be determined by measurement of very dilute HF
(a) Reduction occurs at H2 electrode
solutions
(b) H2 is cathode and Cu is anode
(c) can be determined from measurements of dilute
solutions of NaF, NaCl and HCl (c) H2 is anode and Cu is cathode
(d) is an undefined quantity (d) Oxidation occurs at Cu electrode
574 Chemistry
31. On the basis of the information available from the reaction 39. The standard electrode potential (E°) for OCl– /Cl– and
1
Cl - / Cl 2 respectively are 0.94 V and –1.36 V. The E° value
4 2
Al + O 2 ¾¾® Al 2 O 3 , D G = - 827 kJmol - 1 of O , the
3 3 2 2
minimum e.m.f. required to carry out electrolysis of Al2O3 is 1
(F = 96500 C mol–1) for OCl - / Cl 2 will be
2
(a) 4.28 V (b) 6.42 V (a) –0.42 V (b) –2.20 V
(c) 8.56 V (d) 2.14 V (c) 0.52 V (d) 1.04 V
32. In the electrochemical reaction 40. The standard reduction potential for Fe 2+ / Fe and
Sn2+/ Sn electrodes are –0.44 and –0.14 volt respectively.
2Fe3+ + Zn ¾ ¾® Zn 2+ + 2Fe 2+ , For the cell reaction
on increasing the concentration of Fe2+
(a) increases cell emf Fe 2+ + Sn ¾
¾® Fe + Sn 2+
(b) increases the current flow the standard emf will be
(a) +0.30 V (b) –0.58 V
(c) decreases the cell emf
(c) +0.58 V (d) –0.30 V
(d) alters the pH of the solution
41. The emf of the cell
33. In the cell reaction
Ni / Ni 2+ (1.0 M ) | | Au 3+ (1.0 M) / Au is [E° for Ni2+ / Ni = –
+ 2+
Cu(s) + 2Ag (aq) ¾¾
® Cu (aq) + 2Ag(s) , 0.25 V; E° for Au3+ / Au = 1.5 V]
Eo 0.46 V. By doubling the concentration of Cu2+, Eocell (a) +1.25 V (b) +1.75 V
cell =
will become (c) –1.25 V (d) –1.75 V
(a) doubled 42. An unknown metal M displaces nickel from nickel (II)
sulphate solution but does not displace manganese from
(b) halved
manganese sulphate solution. Which order represents the
(c) increases but less than double
correct order of reducing power?
(d) decreases by a small fraction (a) Mn > Ni > M (b) Ni > Mn > M
34. If salt bridge is removed from two half-cells the voltage (c) Mn > M > Ni (d) M > Ni > Mn
(a) drops to zero (b) does not change 43. Electrode potentials (E° red ) of four elements A, B, C, D are
(c) increases gradually (d) increases rapidly
–1.36, –0.32, 0, –1.26 V respectively. The decreasing reactivity
35. In a salt bridge KCl is used because order of these elements is
(a) it is an electrolyte (a) A, D, B and C (b) C, B, D and A
(b) it is good conductor of electricity (c) B, D, C and A (d) C, A, D and B
(c) the transport number of K+ and Cl– ions are nearly same 44. Chlorine cannot displace
or both have same ionic mobility (a) Fluorine from NaF (b) Iodine from NaI
(d) it is ionic compound (c) Bromine from NaBr (d) None of these
36. The reference electrode is made by using 45. Standard potentials (E°) for some half-reactions are given
(a) ZnCl2 (b) CuSO4 below :
(c) HgCl2 (d) Hg2Cl2 (I) Sn 4 + + 2e ¾¾
® Sn 2 + ; E° = +0.15V
37. The standard hydrogen electrode potential is zero, because
(a) there is no potential difference between the electrode (II) 2Hg 2 + + 2e ¾
¾® Hg 22+ ; E° = 0.92 V
and the solution
(III) PbO 2 + 4H + + 2e ¾
¾® Pb 2+ + 2 H 2 O;
(b) hydrogen ions acquire electrons from a platinum
electrode E° = +1.45V
(c) it has been measured accurately based on the above, information which one of the following
(d) it has been defined that way statements is correct?
38. The standard reduction potentials E° for the half reactions (a) Sn4+ is a stronger oxidising agent than Pb4+
are as (b) Sn2+ is a stronger reducing agent than Hg22+
(c) Pb2+ is a stronger oxidising agent than Pb4+
¾® Zn 2+ + 2e - ; E° = 0.76 V
Zn ¾ (d) Pb2+ is a stronger reducing agent than Sn2+
46. The oxidation potentails of A and B are +2.37 and +1.66 V
¾® Fe 2+ + 2e - ; E° = 0.41V
Fe ¾ respectively. In chemical reactions
(a) A will be replaced by B
The EMF for the cell reaction will be
(b) A will replace B
(a) –0.3 V (b) 0.35 V
(c) A will not replace B
(c) 1.17 V (d) –1.17 V (d) A and B will not replace each other
Electrochemistry 575
47. Electrode potential data are given below : 53. The e.m.f. of a Daniell cell at 298 K is E1

Fe +3 (aq) + e - ¾¾
® Fe +2 (aq); E° = +0.77 V ZnSO 4 CuSO 4
Zn Cu
(0.01M) (1.0 M)
3+ -
Al (aq) + 3e ¾¾
® Al (s) ; E° = – 1.66 V
When the concentration of ZnSO4 is 1.0 M and that of CuSO4
Br2 (aq) + 2e - ¾¾
® 2Br - (aq); E° = + 1.08V
is 0.01 M, the e.m.f. changed to E2. What is the relationship
between E1 and E2?
Based on the data, the reducing power of Fe2+, Al and Br–
(a) E1 < E2 (b) E1 = E2
will increase in the order
(c) E2 = 0 ¹ E1 (d) E1 > E2
(a) Br– < Fe2+ < Al (b) Fe2+ < Al < Br–
54. The metal that cannot displace hydrogen from dilute
(c) Al < Br– < Fe2+ (d) Al < Fe2+ < Br–
hydrochloric acid is
48. Choose the correct statement from the following which is
related to the electrochemical series (a) aluminium (b) Iron
(a) Electrochemical series is not the arrangement of metals (c) copper (d) zinc
and ions according to their reactivity 55. Which reaction is not feasible?
(b) The metal ions at the top of the electrochemical series (a) 2 KI + Br2 ¾
¾® 2KBr + I 2
are highly electronegative
(c) Strongly electropositive metals can displace weakly (b) 2 KBr + I 2 ¾
¾® 2KI + Br2
electropositive metals from their salt solution
(d) All metals above hydrogen in the series do not displace (c) 2KBr + Cl 2 ¾
¾® 2KCl + Br2
hydrogen from dilute acids
49. The standard reduction potentials of four elements are given (d) 2H 2 O + 2F2 ¾
¾® 4HF + O 2
below. Which of the following will be the most suitable
reducing agent? 56. Which of the following will form a cell with the highest
I = –3.04 V II = – 1.90 V voltage?
III = 0 V IV = 1.90 V (a) 1 M Ag + ,1 M Co 2+ (b) 2 M Ag + , 2 M Co 2+
(a) III (b) II
(c) I (d) IV (c) 0.1 M Ag + , 2 M Co 2+ (d) 2 M Ag + , 0.1 M Co 2+
50. The standard reduction potentials at 298K for the following
half reactions are given against each 57. E° of a cell aA + bB ¾
¾® cC + dD is

Zn 2+ (aq) + 2e Zn(s) ; –0.762 V (a) E + RT ln


[a]A [b]B
(b) E +
RT [C]c [D]d
ln
[c]C [d]D nF [A]a [B]b
3+
Cr (aq) + 3e Cr (s); –0.740 V
RT [C]c [d]D RT [a]A [B]B
2H + (aq) + 2e (c) E + ln (d) E + ln
H2 (g) ; 0.00 V nF [A]A [B]B nF [C]C [d]D

Fe3+ (aq) + e Fe2+ (aq) ; 0.770 V 58. The standard EMF for the cell reaction,

Which is the strongest reducing agent? Zn + Cu 2+ ¾


¾® Cu + Zn 2+ is 1.1 volt at 25°C.
(a) Zn (s) (b) Cs (s) The EMF for the cell reaction, when 0.1 M Cu2+ and
(c) H2(g) (d) Fe3+ (aq) 0.1 M Zn2+ solutions are used, at 25°C is
51. Zn gives H2 gas with H2SO4 and HCl but not with HNO3 (a) 1.10 V (b) 0.10 V
because (c) –1.10 V (d) –0.110 V
(a) Zn acts as oxidizing when reacts with HNO3
59. What is the potential of half-cell consisting of
(b) HNO3 is weaker acid than H2SO4 and HCl zinc electrode in 0.01 M ZnSO 4 solution at 25°C
(c) In electrochemical series Zn is above hydrogen
(d) NO3– is reduced in preference to hydronium (E o
ox = 0.763 V )
52. A smuggler could not carry gold by depositing iron on the
(a) 0.8221 V (b) 8.221 V
gold surface since
(a) gold is denser (c) 0.5282 V (d) 9.282 V
(b) iron rusts 60. The oxidation potential of 0.05 M H2SO4 is
(c) gold has higher reduction potential than iron (a) –2 × 0.0591 (b) –0.01 × 0.0591
(d) gold has lower reduction potential than iron (c) –2.321 × 0.0591 (d) +1 × 0.0591
576 Chemistry
61. For the cell reaction, 70. Which of the following reaction occurs at the cathode during
the charging of lead storage battery?
® Zn ( C 2 ) + Cu ( s )
2+
Cu 2+ [C1 (aq)] + Zn(s) ¾¾
(a) Pb 2+ + 2e - ¾
¾® Pb
of an electrochemical cell, the change in free energy, DG at a
given temperature is a function of (b) Pb 2 + + SO 24 - ¾
¾® PbSO 4

æ C2 ö ¾® Pb 2+ + 2e -
(c) Pb ¾
(a) ln (C1) (b) ln çç C ÷÷
è 1ø ¾® 2PbO 2 + 4H + + SO 24- + 2e -
(d) PbSO 4 + 2H 2 O ¾
(c) ln (C1 + C2) (d) ln (C2) 71. Reaction that takes place at graphite anode in dry cell is
62. The relationship between standard reduction potential of a (a) Zn 2 + + 2e - ¾¾
® Zn(s)
cell and equilibrium constant is shown by
® Zn 2 + + 2e -
(b) Zn(s) ¾¾
n 0.059
(a) E ocell = log k c
o
(b) E cell = log k c (c) Mn 2 + + 2e - ¾¾
0.059 n ® Mn(s)

log k c ® Mn + + e - + 1.5V
(d) Mn(s) ¾¾
(c) E ocell = 0.059 n log k c (d) E cell =
o
n 72. Which one of the following cells can convert chemical energy
63. E° for the cell, of H2 and O2 directly into electrical energy?
(a) Mercury cell (b) Daniell cell
Zn | Zn 2+ (aq) | | Cu 2+ (aq)| Cu is 1.10 V at 25°C. The (c) Fuel cell (d) Lead storage cell
equilibrium constant for the cell reaction 73. Hydrogen-Oxygen fuel cells are used in space craft to supply
(a) power for heat and light (b) power for pressure
Zn + Cu 2+ (aq) Cu + Zn 2+ (aq) (c) oxygen (d) water
is of the order of 74. As lead storage battery is charged
(a) 10–37 (b) 1037 (a) lead dioxide dissolves
(c) 10 –17 (d) 1017 (b) sulphuric acid is regenerated
64. The standard EMF of Daniell cell is 1.10 volt. The maximum (c) lead electrode becomes coated with lead sulphate
electrical work obtained from the Daniell cell is (d) the concentration of sulphuric acid decreases
(a) 212.3 kJ (b) 175.4 kJ 75. The thermodynamic efficiency of cell is given by
(c) 106.15 kJ (d) 53.07 kJ (a) DH/DG (b) nFE/DG
65. The emf of Daniell cell is 1.1 volt. If the value of Faraday is (c) nFE/DH (d) nFE
96500 coulombs per mole, the change in free energy in kJ is 76. The electroplating with chromium is undertaken because
(a) 212.30 (b) –212.30 (a) electrolysis of chromium is easier
(c) 106.15 (d) –106.15 (b) chromium can form alloys with other metals
66. Pure water does not conduct electricity because it (c) chromium gives protective and decorative coating to
the base metal
(a) has low boiling point (b) is almost unionised
(d) of the high reactivity of metallic chormium
(c) is neutral (d) is readily decomposed
77. Prevention of corrosion of iron by Zn coating is called
67. At cathode, the electrolysis of aqueous Na2SO4 gives
(a) galvanization (b) cathodic protection
(a) Na (b) H2
(c) electrolysis (d) photoelectrolysis
(c) SO3 (d) SO2
78. Which of the following statements is correct?
68. At anode in the electrolysis of fused NaCl
(a) Oxidation number of oxygen in KO2 is +1
(a) Na+ is oxidized (b) Cl– is oxidized (b) The specific conductance of an electrolyte solution
(c) Cl is reduced (d) Na is reduced decreases with increase in dilution
69. In electrolysis of NaCl when Pt electrode is taken then H2 is (c) Sn2+ oxidises Fe3+
liberated at cathode while with Hg cathode, it forms sodium (d) Zn/ZnSO4 is a reference electrode
amalgam 79. In the electrolytic cell, flow of electrons is from
(a) Hg is more inert than Pt (a) cathode to anode in solution
(b) More voltage is required to reduce H+ at Hg than at Pt (b) cathode to anode through external supply
(c) Na is dissolved in Hg while it does not dissolve in Pt (c) cathode to anode through internal supply
(d) Conc. of H+ ions is larger when Pt electrode is taken (d) anode to cathode through internal supply
Electrochemistry 577

1. Zn gives H2 gas with H2SO4 and HCl but not with HNO3 M
because [CBSE-PMT 2002] 9. The equivalent conductance of solution of a weak
32
(a) Zn acts as an oxidising agent when it reacts with HNO3 2
monobasic acid is 8.0 mho cm and at infinite dilution is 400
(b) HNO3 is weaker acid than H2SO4 and HCl mho cm2. The dissociation constant of this acid is:
(c) in electrochemical series, Zn is above hydrogen
[CBSE-PMT 2009]
(d) NO3– is reduced in preference to hydronium ion
(a) 1.25 × 10–6 (b) 6.25 × 10–4
2. The efficiency of a fuel cell is given by [CBSE-PMT 2007]
(c) 1.25 × 10–4 (d) 1.25 × 10–5
DG DS DH
(a) DG (b) (c) (d) 10. For the reduction of silver ions with copper metal, the standard
DS D H DG DG cell potential was found to be + 0.46 V at 25°C.
3. The equilibrium constant of the reaction: The value of standard Gibbs energy, D G° will be
Cu( s) + 2Ag+ (aq) ‡ˆˆ
ˆˆ† Cu 2+ (aq) + 2Ag( s) ;
(F = 96500 C mol –1 ) [CBSE-PMT 2010]
E° = 0.46 V at 298 K is [CBSE-PMT 2007]
(a) – 89.0 kJ (b) – 89.0 J (c) – 44.5 kJ (d) – 98.0 kJ
(a) 2.0 × 1010 (b) 4.0 × 1010 (c) 4.0 × 1015 (d) 2.4 × 1010
4. On the basis of the following E° values, the strongest oxidizing 11. An increase in equivalent conductance of a strong electrolyte
agent is : [CBSE-PMT 2008] with dilution is mainly due to: [CBSE-PMT 2010]
[Fe(CN)6]4– ® [Fe(CN)6]3– + e– ; E° = – 0.35 V (a) increase in ionic mobility of ions
Fe2+ ® Fe3+ + e–; E° = – 0.77 V (b) 100% ionisation of electrolyte at normal dilution
(a) [Fe(CN)6]4– (b) Fe2+ (c) increase in both i.e. number of ions and ionic mobility of
(c) Fe3+ (d) [Fe(CN)6]3– ions
5. Kohlrausch’s law states that at : [CBSE-PMT 2008] (d) increase in number of ions
(a) finite dilution, each ion makes definite contribution to 12. Which of the following expressions correctly represents the
equivalent conductance of an electrolyte, whatever be equivalent conductance at infinite dilution of Al2(SO4)3,
the nature of the other ion of the electrolyte.
° 2-
(b) infinite dilution each ion makes definite contribution to Given that L °Al3+ and LSO are the equivalent
4
equivalent conductance of an electrolyte depending on
the nature of the other ion of the electrolyte. conductances at infinite dilution of the respective ions?
(c) infinite dilution, each ion makes definite contribution to [CBSE-PMT 2010]
conductance of an electrolyte whatever be the nature of
(b) 2L°Al3+ + 3LSO
° 2-
1 ° 1
the other ion of the electrolyte. (a) L 3+ + L° 2-
3 Al 2 SO4 4
(d) infinite dilution, each ion makes definite contribution to
æ ° 2- ö ´ 6
equivalent conductance of an electrolyte, whatever be (c) L°Al3+ + LSO
° 2- (d) èL °Al3+ + LSO
4 4 ø
the nature of the other ion of the electrolyte.
6. Standard free energies of formation (in kJ/mol) at 298 K are 13. Consider the following relations for emf of a electrochemical
– 237.2, – 394.4 and – 8.2 for H2O(l), CO2(g) and pentane (g), cell: [CBSE-PMT 2010]
respectively. The value E°cell for the pentane-oxygen fuel cell (i) emf of cell = (Oxidation potential of anode) –
is : [CBSE-PMT 2008] (Reduction potential of cathode)
(a) 1.968 V (b) 2.0968 V (c) 1.0968 V (d) 0.0968 V (ii) emf of cell = (Oxidation potential of anode) + (Reduction
7. Given: [CBSE-PMT 2009] potential of cathode)
(i) Cu2+ + 2e– ® Cu, Eo = 0.337 V (iii) emf of cell = (Reduction potential of anode) + (Reduction
(ii) Cu2+ + e– ® Cu+, Eo = 0.153 V potential of cathode)
Electrode potential, Eo for the reaction, (iv) emf of cell = (Oxidation potential of anode) – (Oxidation
Cu + + e– ® Cu, will be : potential of cathode)
(a) 0.90 V (b) 0.30 V (c) 0.38 V (d) 0.52 V
Which of the above relations are correct?
8. Al2O3 is reduced by electrolysis at low potentials and high
currents. If 4.0 × 104 amperes of current is passed through (a) (ii) and (iv) (b) (iii) and (i)
molten Al 2 O 3 for 6 hours, what mass of aluminium is (c) (i) and (ii) (d) (iii) and (iv)
produced? (Assume 100% current efficiency. At. mass of 14. Standard electrode potential of three metals X, Y and Z are
Al = 27 g mol–1) [CBSE-PMT 2009] – 1.2 V, + 0.5 V and – 3.0 V, respectively. The reducing power
(a) 8.1 × 104 g (b) 2.4 × 105 g of these metals will be : [CBSE-PMT 2011]
(c) 1.3 × 104 g (d) 9.0 × 103 g (a) Y > Z > X (b) X > Y > Z (c) Z > X > Y (d) X > Y > Z
578 Chemistry
15. The electrode potentials for [CBSE-PMT 2011] 23. At 25°C molar conductance of 0.1 molar aqueous solution
Cu2+(aq) + e– ¾¾ ® Cu+(aq) of ammonium hydroxide is 9.54 ohm-1 cm2mol-1 and at
and Cu+(aq) + e– ¾¾ ® Cu(s) infinite dilution its molar conductance is 238 ohm-1cm2 mol-1.
are + 0.15 V and + 0.50, respectively. The value of The degree or ionisation of ammonium hydroxide at the
E°Cu 2+ / Cu will be : same concentration and temperature is : [NEET 2013]
(a) 20.800% (b) 4.008%
(a) 0.500 V (b) 0.325 V (c) 0.650 V (d) 0.150 V
(c) 40.800% (d) 2.080%
16. Standard electrode potential for Sn4+ / Sn2+ couple is + 0.15 V
24. A button cell used in watches functions as following
and that for the Cr3+ / Cr couple is – 0.74 V. These two couples
in their standard state are connected to make a cell. The cell Zn(s) + Ag2O(s) + H2O(l)
potential will be : [CBSE-PMT 2011] 2Ag(s) + Zn2+(aq) + 2OH–(aq)
(a) + 1.19 V (b) + 0.89 V (c) + 0.18 V (d) + 1.83 V If half cell potentials are :
17. If the E°cell for a given reaction has a negative value, then Zn2+(aq) + 2e– ® Zn(s); Eo = – 0.76 V
which of the following gives the correct relationships for the Ag 2 O(s) + H 2 O (l) + 2e – ® 2Ag(s) + 2OH – (aq);
values of DG° and Keq ? [CBSE-PMT 2011] Eo = 0.34 V
(a) DG° > 0 ; Keq > 1 (b) DG° < 0 ; Keq > 1 The cell potential will be : [NEET 2013]
(c) DG° < 0 ; Keq < 1 (d) DG° > 0 ; Keq < 1 (a) 0.42 V (b) 0.84 V
18. A solution contains Fe2+, Fe3+ and I– ions. This solution (c) 1.34 V (d) 1.10 V
was treated with iodine at 35°C. E° for Fe3+ / Fe2+ is + 0.77 V 25. Conductivity (unit Siemen’s S) is directly proportional to
and E° for I2/2I– = 0.536 V. The favourable redox reaction is : area of the vessel and the concentration of the solution in it
(a) I2 will be reduced to I– [CBSE-PMT 2011 M] and is inversely proportional to the length of the vessel
(b) There will be no redox reaction then the unit of the constant of proportionality is
(c) I– will be oxidised to I2 [AIEEE 2002]
(d) Fe2+ will be oxidised to Fe3+ (a) Sm mol–1 (b) Sm2 mol–1
19. Limiting molar conductivity of NH4OH [CBSE-PMT 2012 S] (c) S–2m2 mol (d) S2m2 mol–2.

(i.e., L° ) is equal to :
26. EMF of a cell in terms of reduction potential of its left and
m(NH4OH right electrodes is [AIEEE 2002]
(a) E = Eleft - Eright (b) E = Eleft + Eright
(a) L°m( NH Cl) + L°m( NaCl) -L°m NaOH (c) E = Eright - Eleft (d) E = -(Eright + Eleft).
4 ( )
27. If f denotes reduction potential, then which is true?
(b) L°m( NaOH ) + L°m( NaCl) -L°m( NH Cl) [AIEEE 2002]
4
(a) E°cell = f right – f left (b) E°cell = f left + f right
(c) L°m( NH 4OH ) + L °m( NH 4Cl) -L°m( HCl)
(c) E°cell = f left – f right (d) E°cell = – ( f left + f right).
(d) L°m( NH 4Cl) + L°m( NaOH)-L°m( NaCl) 28. What will be the emf for the given cell
Pt | H2 (P1) | H+ (aq) | | H2 (P2) | Pt [AIEEE 2002]
20. Standard reduction potentials of the half reactions are given P P
RT RT
below : [CBSE-PMT 2012 M] (a) log 1 (b) log 1
F2(g) + 2e– ® 2F– (aq); E° = + 2.85 V f P2 2f P2
Cl2(g) + 2e– ® 2Cl–(aq); E° = + 1.36 V RT P
Br2(l) + 2e– ® 2Br–(aq); E° = + 1.06 V (c) log 2 (d) None of these.
f P1
I2(s) + 2e– ® 2I–(aq); E° = + 0.53 V
The strongest oxidising and reducing agents respectively 29. Which of the following reaction is possible at anode?
are : [AIEEE 2002]
(a) F2 and I– (b) Br2 and Cl– (a) 2 Cr3+ + 7H2O ® Cr2O72– + 14H+
(c) Cl2 and Br – (d) Cl2 and I2 (b) F2 ® 2F –
21. Molar conductivities ( L° m ) at infinite dilution of NaCl, HCl (c) (1/2) O2 + 2H+ ® H2O
and CH3COONa are 126.4, 425.9 and 91.0 S cm2 mol–1 (d) None of these.
respectively. L°m for CH3COOH will be : 30. For a cell reaction involving a two-electron change, the
[CBSE-PMT 2012 M] standard e.m.f. of the cell is found to be 0.295 V at 25ºC. The
(a) 425.5 S cm2 mol–1 (b) 180.5 S cm2 mol–1 equilibrium constant of the reaction at 25ºC will be
(c) 290.8 S cm2 mol–1 (d) 390.5 S cm2 mol–1 (a) 29.5 × 10–2 (b) 10 [AIEEE 2003]
22. A hydrogen gas electrode is made by dipping platinum (c) 1 × 1010 (d) 1 × 10–10
wire in a solution of HCl of pH = 10 and by passing hydrogen 31. Standard reduction electrode potentials of three metals A, B
gas around the platinum wire at one atm pressure. The & C are respectively + 0.5 V, – 3.0 V & –1.2 V. The reducing
oxidation potential of electrode would be ? [NEET 2013] powers of these metals are [AIEEE 2003]
(a) 0.59 V (b) 0.118 V (a) A > B > C (b) C > B > A
(c) 1.18 V (d) 0.059 V (c) A > C > B (d) B > C > A
Electrochemistry 579
32. When during electrolysis of a solution of AgNO3 9650 41. Aluminium oxide may be electrolysed at 1000°C
coulombs of charge pass through the electroplating bath, to furnish aluminium metal (At. Mass = 27 amu; 1 Faraday =
the mass of silver deposited on the cathode will be 96,500 Coulombs). The cathode reaction is–
(a) 10.8 g (b) 21.6 g [AIEEE 2003]
(c) 108 g (d) 1.08 g Al3+ + 3e- ® Al
33. For the redox reaction : [AIEEE 2003] To prepare 5.12 kg of aluminium metal by this method we
Zn(s) + Cu 2+ (0.1 M) ® Zn 2+ (1 M) + Cu (s) require [AIEEE 2005]
º
taking place in a cell, E cell is 1.10 volt. Ecell for the cell will (a) 5.49 × 101 C of electricity
4
(b) 5.49 × 10 C of electricity
be æç 2.303 RT = 0.0591 ö÷
è F ø 7
(c) 1.83 × 10 C of electricity
(a) 1.80 volt (b) 1.07 volt
(c) 0.82 volt (d) 2.14 volt (d) 5.49 × 10 7 C of electricity
34. Several blocks of magnesium are fixed to the bottom of a
ship to [AIEEE 2003] Electrolyte: KCl KNO3 HCl NaOAc NaCl
42.
(a) make the ship lighter 2
^¥ (S cm mol ) : 149.9
–1
145 426.2 91 126.5
(b) prevent action of water and salt
(c) prevent puncturing by under-sea rocks Calculate L ¥
HOAC using appropriate molar conductances
(d) keep away the sharks
35. In a hydrogen-oxygen fuel cell, combustion of hydrogen of the electrolytes listed above at infinite dilution in H 2 O
occurs to [AIEEE 2004] at 25°C [AIEEE 2005]
(a) produce high purity water (a) 217.5 (b) 390.7
(b) create potential difference between two electrodes
(c) 552.7 (d) 517.2
(c) generte heat
o o
(d) remove adsorbed oxygen from electron surfaces 43. The molar conductivities L NaOAc and L HCl at infinite
36. Consider the following Eº values [AIEEE 2004]
dilution in water at 25ºC are 91.0 and 426.2 S cm2/mol
Eº = +0.77V ; E º Sn 2+ / Sn = -0.14V o
Fe3+ /Fe2 + respectively. To calculate L HOAc , the additional value
Under standard conditions the potential for the reaction required is [AIEEE 2006]
Sn (s) + 2Fe3+ (aq) ® 2Fe 2+ (aq) + Sn 2+ (aq) is o o
(a) L NaOH (b) L NaCl
(a) 0.91 V (b) 1.40 V
o o
(c) 1.68 V (d) 0.63 V (c) LH (d) L KCl
2O
37. The standard e.m.f. of a cell involving one electron change
is found to be 0.591 V at 25ºC. The equilibrium constant of 44. Resistance of a conductivity cell filled with a solution of an
the reaction will be electrolyte of concentration 0.1 M is 100 W. The conductivity
(F = 96,500 C mol–1; R = 8.314 JK–1 mol–1) [AIEEE 2004] of this solution is 1.29 S m–1. Resistance of the same cell
(a) 1.0 × 1010 (b) 1.0 × 105 when filled with 0.2 M of the same solution is 520 W. The
(c) 1.0 × 101 (d) 1.0 × 1030 molar conductivity of 0.02 M solution of electrolyte will be
38. The limiting molar conductivities L º for NaCl, KBr and KCl [AIEEE 2006]
are 126, 152 and 150 S cm2 mol–1 respectively. The L º for (a) 1.24 × 10–4 S m2 mol–1 (b) 12.4 × 10–4 S m2 mol–1
NaBr is [AIEEE 2004] (c) 124 × 10–4 S m2 mol–1 (d) 1240 × 10–4 S m2 mol–1
(a) 278 S cm2 mol–1 (b) 176 S cm2 mol–1 45. The equivalent conductances of two strong electrolytes at
(c) 128 S cm2 mol–1 (d) 302 S cm2 mol–1 infinite dilution in H2O (where ions move freely through a
39. The E º values for Cr, Mn, Fe and Co are – 0.41, + solution) at 25°C are given below : [AIEEE 2007]
M 3+ / M 2 +
1.57, + 0.77 and + 1.97V respectively. For which one of these LoCH3COONa = 91.0 S cm2 / equiv.
metals the change in oxidation state from +2 to +3 is easiest?
L o HCl = 426.2 S cm 2 / equiv.
(a) Fe (b) Mn [AIEEE 2004]
(c) Cr (d) Co What additional information/ quantity one needs to calcu-
40. The highest electrical conductivity of the following aqueous late L o of an aqueous solution of acetic acid?
solutions is of [AIEEE 2005] (a) L o of chloroacetic acid (ClCH2COOH)
(a) 0.1 M difluoroacetic acid
(b) L o of NaCl
(b) 0.1 M fluoroacetic acid
(c) 0.1 M chloroacetic acid (c) L o of CH3COOK
(d) 0.1 M acetic acid (d) the limiting equivalent coductance of H + (l° ).
H+
580 Chemistry
54. Resistance of 0.2 M solution of an electrolyte is 50 W. The
46. The cell, Zn | Zn 2+ (1 M) || Cu 2+ (1 M) | Cu ( E °cell = 1.10 v)
specific conductance of the solution is 1.3 S m–1. If resistance
was allowed to be completely discharged at 298 K. The of the 0.4 M solution of the same electrolyte is 260 W, its molar
æ [Zn 2+ ] ö conductivity is : [AIEEE 2011RS]
relative concentration of Zn2+ to Cu2+ çç 2+ ÷
÷ is (a) 6.25 × 10–4 S m2 mol–1 (b) 625 × 10–4 S m2 mol–1
è [Cu ] ø (c) 62.5 S m2 mol–1 (d) 6250 S m2 mol–1
[AIEEE 2007] 55. The standard reduction potentials for Zn 2+ /Zn,
(a) 9.65 × 104 (b) antilog (24.08) Ni2+/Ni and Fe2+/Fe are –0.76,–0.23 and –0.44 V respectively.
(c) 37.3 (d) 1037.3. The reaction X + Y 2 + ¾¾ ® X 2+ + Y will be spontaneous
47. Given Eº = –0.72 V, Eº 2+ = – 0.42 V. The when : [AIEEE 2012]
Cr 3+ / Cr Fe / Fe (a) X = Ni, Y = Fe (b) X = Ni, Y = Zn
potential for the cell (c) X= Fe, Y = Zn (d) X= Zn, Y = Ni
Cr|Cr3+ (0.1M)|| Fe2 + (0.01 M)| Fe is [AIEEE 2008]
(a) 0.26 V (b) 0.336 V (c) – 0.339 (d) 0.26 V 56. Given : E° 3+ = -0.74 V; E° - = 1.51 V
Cr / Cr MnO4 / Mn 2 +
48. In a fuel cell methanol is used as fuel and oxygen gas is used
as an oxidizer. The reaction is E° 2- 3+ = 1.33 V; E° - = 1.36 V
Cr2O7 / Cr Cl / Cl
CH 3OH(l ) + 3/2O 2 (g) ¾¾ ® CO 2 (g) + 2H 2 O(l ) Based on the data given above, strongest oxidising agent
At 298 K standard Gibb’s energies of formation for CH3OH(l), will be : [JEE M 2013]
H2O(l) and and CO2 (g) are –166.2 –237.2 and –394.4 kJ mol–1 (a) Cl (b) Cr3+
respectively. If standard enthalpy of combustion of methonal (c) Mn2+ (d) MnO4 –
is – 726 kJ mol–1, efficiency of the fuel cell will be: 57. Four successive members of the first row transition elements
[AIEEE 2009] are listed below with atomic numbers. Which one of them is
(a) 87% (b) 90% (c) 97% (d) 80% expected to have the highest E° 3+ 2+ value ?
49. Given: M /M
[JEE M 2013]
E° 3+ = –0.036V, (a) Cr(Z = 24) (b) Mn(Z = 25)
Fe / Fe
(c) Fe(Z = 26) (d) Co(Z = 27)
E° = –0.439V
Fe2+ / Fe 58. Electrolysis of dilute aqueous NaCl solution was carried out
The value of standard electrode potential for the change, by passing 10 milli ampere current. The time required to liber-
ate 0.01 mol of H2 gas at the cathode is (1 Faraday = 96500
Fe3+ ( aq ) + e – ¾¾
® Fe 2+ ( aq ) will be: [AIEEE 2009]
C mol–1)
(a) 0.385 V (b) 0.770 V (c) –0.270 V (d) –0.072 V [IIT-JEE 2008S]
50. The Gibbs energy for the decomposition of Al2O3 at 500°C is (a) 9.65 × 104 sec (b) 19.3 × 104 sec
as follows : (c) 28.95 × 104 sec (d) 38.6 × 104 sec
2 4 59. AgNO3(aq.) was added to an aqueous KCl solution gradually
Al2 O3 ® Al + O2 , D r G = + 966 kJ mol -1 and the conductivity of the solution was measured. The plot
3 3
The potential difference needed for electrolytic reduction of of conductance ( L ) versus the volume of AgNO3 is
Al2O3 at 500°C is at least [AIEEE 2010] [IIT-JEE 2011]
(a) 4.5 V (b) 3.0 V
(c) 2.5 V (d) 5.0 V
51. The correct order of E ° 2+ values with negative sign for
M /M
the four successive elements Cr, Mn, Fe and Co is
[AIEEE 2010]
(a) Mn > Cr > Fe > Co (b) Cr < Fe > Mn > Co
(c) Fe > Mn > Cr > Co (d) Cr > Mn > Fe > Co
52. The reduction potential of hydrogen half-cell will be negative
if : [AIEEE 2011]
(a) p(H2) = 1 atm and [H+] = 2.0 M
(b) p(H2) = 1 atm and [H+] = 1.0 M
(c) p(H2) = 2 atm and [H+] = 1.0 M
(d) p(H2) = 2 atm and [H+] = 2.0 M
53. Consider the following cell reaction: [AIEEE 2011]
2Fe( s) + O2 ( g) + 4H+ (aq) ® 2Fe2+ (aq) + 2H2O(l);E° = 1.67V
At [Fe2+] = 10–3 M, p(O2) = 0.1 atm and pH = 3, the cell
potential at 25ºC is (a) (P) (b) (Q)
(a) 1.47 V (b) 1.77 V (c) 1.87 V (d) 1.57 V (c) (R) (d) (S)
Electrochemistry 581

1. Which of the following solutions of KCl will have the highest 7. What is the reaction taking place at the anode when an
value of specific conductance? aqueous solution of copper sulphate is electrolysed using
(a) 1.0 N (b) 0.1 N Pt–electrodes (inert) ?
(c) 1.0 ×10–2N (d) 1.0 ×10–3N (a) Cu 2 + + 2e - ® Cu
2. Which of the following statements is not correct? (b) 2 H + + 2 e - ® H 2
(a) The equivalent conductance of an electrolyte increases
(c) 2H 2 O ® O 2 + 4H + + 4e -
on dilution.
(b) The equivalent conductance of an electrolyte decreases (d) 2SO 24 - ® S 2 O 82 - + 2e -
on dilution 8. The specific conductance at 298 K of AgCl is
(c) The degree of ionization of a weak electrolyte is given -1 -1
1.826 × 10–6 ohm cm . The ionic conductances of Ag+
by a = l c / l 0 , where l c and l 0 are equivalent
conductances at concentration c and zero respectively and Cl– are 61.92 and 76.34 respectively. What is the solubility
(d) In case of a weak electrolyte on dilution,, specific of AgCl in water ?
conductance decreases but its equivalent conductance (a) 1.1´10 -2 g -1 (b) 1.9 ´ 10 -3 g L-1
increases
3. Which of the following statements is wrong ? (c) 2.1´ 10 -5 g L-1 (d) 2.1´ 10 -6 g L-1
(a) Electrolysis of an aqueous sodium hydroxide solution 9. The potential of a hydrogen electrode at pH=10 is
liberates H2 gas at the cathode and O2 gas at the anode. (a) 0.59 V (b) zero volt
(b) Electrolysis of dil. H2SO4 liberates H2(g) at cathode (c) –0.59 V (d) 0.059 V
10. Equivalent conductance at infinite dilution, l° of NH4Cl,
and O2 (g) at the anode
NaOH and NaCl are 128.0, 217.8 and 109.3 ohm–1 cm2 eq–1
(c) DG°= nFE° for a spontaneous reaction
respectively. The equivalent conductance of 0.01 N NH4OH
0.059 is 9.30 ohm–1 cm2 eq–1, then the degree of ionization of
(d) E=E° - log Q , Where Q = reaction quotient.
n NH4OH at this temperature would be
4. In the electrolysis of CuCl2 solution, the mass of the cathode (a) 0.04 (b) 0.1
increased by 3.2g. What occured at the copper anode ? (c) 0.39 (d) 0.62
(a) 0.12 litre of Cl2 was liberated 11. What is the standard cell potential E° for an electrochemical
cell in which the following reaction takes place
(b) 0.56 litre of O2 was liberated spontaneously ?
(c) 0.1 mol Cu2+ passed into the solution.
Cl2 (g) + 2Br - ® Br2 (aq) + 2Cl- DG° = -50.6 kJ
(d) 0.05 mol of Cu2+ passed into the solution.
(a) 1.2 V (b) 0.53 V
5. Copper can be deposited from acidified copper sulphate (c) 0.26 V (d) –0.53 V
and alkaline copper cyanide both. If the same current is 12. When electric current is passed through acidified water, 112
passed for the definite time, which of the following is correct? ml of hydrogen gas at STP collected at the cathode in 965
(a) The amount of copper deposited from acidic copper seconds. The current passed in amperes is
sulphate will be higher (a) 1.0 (b) 0.5
(b) The amount of copper deposited from alkaline copper (c) 0.1 (d) 2.0
-
cyanide will be more. 13. If the half cell reaction is A + e ® A- has a large negative
(c) The same amount of Cu will be deposited reduction potential , it follows that.
(d) No Cu will be deposited (a) A is readily reduced (b) A is ready oxidised
6. The EMF of the cell Tl/Tl+ (0.001M) || Cu2+ (0.01M) /Cu is (c) A- is readily reduced (d) A- is readily oxidised
0.83. The cell EMF can be increased by 14. The electrochemical equivalent of silver is 0.001180 g. When
(a) Increasing the concentration of Tl+ ions. an electric current of 0.5 amp is passed through an aqueous
silver nitrate solution for 200 sec., the amount of silver
(b) Increasing the concentration of Cu2+ ions. deposited is
(c) Increasing the concentration of Tl + and Cu2+ ions. (a) 1.1180 g (b) 0.1180 g
(d) None of these (c) 5.590 g (d) 0.5598 g
582 Chemistry
15. A gas X at 1 atm is bubbled through a solution containing a 21. The mathematical expression for law of independent
mixture of 1 M Y– and M Z– at 25°C. If the reduction potential migration of ions and Ostwald’s dilution law are given by
of Z > Y > X, then,
(a) Y will oxidize X and not Z (a) L = Lom - BC½ (b) L o = F( U + + U - )
(b) Y will oxidize Z and not X
(c) Y will oxidize both X and Z Lo 1 Lmc
(c) Lom = n+ l + + n- l- (d) L = o +
(d) Y will reduce both X and Z m L m K a (L om )2
16. For the electrochemical cell, M | M + || X - | X, 22. The variation of equivalent conductance of a weak electrolyte
o – with (concentration)½ is represented as
E o M + / M = 0.44V and E (X/X ) = 0.33V..
From this data one can deduce that
(a) M+X®M++X– is the spontaneous reaction
(b) M++X–®M+X is the spontaneous reaction (a) l (b) l
(c) Ecell = 0.77 V
(d) Ecell = - 0.77 V Öc Öc
17. Standard electrode potential data are useful for understanding
the suitability of an oxidant in a redox titration. Some half cell
reactions and their standard potentials are given below:
(c) l (d) l
MnO -4 (aq.) + 8H + (aq.) + 5e - ® Mn 2 + (aq.) + 4H 2 O(l)
Öc Öc
E° = 1.51 V
23. The ionic mobility of ions at infinite dilution is related to
Cr2 O 72 - (aq.) + 14H + (aq.) + 6e - ® 2Cr 3+ (aq.) + 7H 2 O(l) ionic conductance by
E° = 1.38 V (a) L o = Fk (b) L o F = U++U–
Fe3+ (aq.) + e - ® Fe2 + (aq.) E° = 0.77 V (c) L o = U++ U– (d) L o = F(U++ U–)
24. The variation of equivalent conductance of strong
Cl2 (g) + 2e- ® 2Cl- (aq.) E° = 1.40 V electrolyte with (concentration)½ is represented by
Identify the only incorrect statement regarding the
quantitative estimation of aqueous Fe(NO3)2
(a) MnO-4 can be used in aqueous HCl
(a) l (b) l
(b) Cr2O72 - can be used in aqueous HCl c c

(c) MnO-4 can be used in aqueous H2SO4

(d) Cr2O72 - can be used in aqueous H2SO4


18. A galvanic cell is constructed as follows. A half-cell consists (c) l (d) l
of a platinum wire immersed in a solution containing 1.0 M c c
of Sn2+ and 1.0 M of Sn4+, and another half-cell has a thallium 25. The value of molar conductance of HCl is greater than that
rod immersed in a 1.0 M solution of Tl+. of NaCl at a given temperature and concentration because
Given : Sn 4+ (aq) + 2e- ¾¾
® Sn 2+ (aq); Eo = + 0.13V (a) ionic moblility of HCl is greater than that of NaCl
(b) the dipole moment of NaCl is greater than that of HCl
and Tl+ (aq) + e - ¾¾ ® Tl(s); Eo = –0.34V,, (c) NaCl is more ionic than HCl
what is the cell voltage if the Tl+ concentration is increased (d) HCl is Bronsted acid and NaCl is a salt of a strong acid
tenfold? and strong base
(a) 0.411V (b) 4.101V 26. At pH = 2, E o (quinhydron) = 1.30 V, E(quinhydron) will be
(c) 0.492V (d) 0.222V
19. The unit of ionic mobility is
OH O
(a) m–2V–1s–1 (b) m2V–1s–1
+ 2 H + + 2e -
(c) m–2Vs–1 (d) m2V–2s–1
20. If r is the resistance in ohm of a centimeter cube, generally OH O
called the specific resistance of the substance constituting
the conductor, the resistance r of the layer containing ''a'' (a) 1.20 V (b) 1.42 V
cubes is given by (c) 1.36 V (d) 1.30 V
27. Consider the following reactions
1 1 1 1 1 1
(a) = + + ...... (b) = + + ... (i) Cd 2 + (aq ) + 2e - ¾
¾® Cd (s) , E° = – 0.40 V
r r r r ra ra
(c) r = a / r (d) r = r + r +...... (ii) Ag + (aq) + e- ¾¾® Ag(s), E° = 0.80 V
Electrochemistry 583
For the galvanic cell involving the above reactions. Which 34. Absolute ionic mobility is the speed of ion under the electric
of the following is not correct ? field of and its dimension
(a) Molar concentration of the cation in the cathodic
compartment changes faster than that of the cation in (a) 5 V across a distance of 5 cm, m 2 v -1s -1
the anodic compartment. -1 -1
(b) 10 V across a distance of 5 cm, mv s
2+
(b) E cell increase when Cd solution is diluted.
(c) 5 V across a distance of 10 cm, m 2 v - 2 s -1
(c) Twice as many electrons pass through the cadmium
electrode as through silver electrode. (d) None of these
35. Given the cell reactions
(d) E cell decreases when Ag + solution is diluted.
MX(s) + e - ¾¾
® M(s) + X - (aq), E ° = 0.207V
28. Conductance of 0.1 M KCl (conductivity = X Ohm-1cm -1 ) and M + (aq ) + e - ¾
¾® M (s ) , E° = 0.799 V
filled in a conductivity cell is Y Ohm-1 . If the conductance The solubility of MX (s) at 298 K is
-1
of 0.1 M NaOH filled in the same cell is Z Ohm-1 , the molar (a) 1. 10 -5 mol L-1 (b) 1.0 × 10 - 4 mol L
conductance of NaOH will be (c) 1.0 ´ 10 -10 mol L-1 (d) 1.0 ´ 10 -9 mol L-1
(a) 10 3 XZ (b) 10 4 XZ 36. Electrolysis is carried out in three cells
Y Y
(A) 1.0 M CuSO 4 Pt electrode
XZ XZ
(d) 10 (d) 0.1
Y Y (B) 1.0 M CuSO 4 copper electrodes
29. In electrolytic reduction of a nitroarene with 50% current (C) 1.0 M KCl Pt electrodes
efficiency 20.50 g of the compound is reduced by 2 × 96500 C If volume of electrolytic solution is maintained constant in
of electric charge. The molar mass of the compound is each of the cell, which is correct set of pH changes in (A), (B)
(a) 123.0 g (b) 61.5 g and (C) cell respectively ?
(c) 10.2 g (d) 20.5 g (a) decrease in all the three
30. On electrolysing K 2SO 4 solution using inert electrodes, (b) increase in all the three
(c) decrease, constant, increase
1.68 L (STP) of gases was obtained. How many moles of MnO -4
(d) increase, constant, increase
could be reduced to Mn 2 + by the same quantity of 37. The reversible reduction potential of pure water is – 0.414 V
electricity ? under 1 atm H 2 pressure. If the reduction is considered to
(a) 0.02 (b) 0.15
(c) 0.20 (d) 0.10 be 2 H + + 2e ¾
¾® H 2 . Calculate the [H+ ] of pure water
31. The standard reduction potential for Cu 2+ /Cu is + 0.34. (a) 1.02 ´ 10 -7 (b) 1.02 ´ 10 -9
Calculate the reduction potential at pH = 14 for the above
(c) 2.01 ´ 10 - 7 (d) 2.01 ´ 10 -9
couple. (Ksp Cu (OH) 2 = 1 × 10 -19 )
(a) – 0.22 V (b) + 0.22 V 38. 1. 0 L each of a buffer containing 1 mole NH 3 and 1 mol of
(c) – 0.44 V (d) + 0.44 V
NH +4 were placed in the cathodic and anodic half-cells and
32. A 1.0 M with respect to each of the metal halides AX 3 , BX 2 ,
965 C of electricity was passed. If anodic and cathodic half
CX 3 and DX 2 is electrolysed using platinum electrodes. If cells reactions involve oxidation and reduction of water only
E oA3+ / A = 1.50 V, E oB2 / B = 0.3 V, E Co 3+ / C = -0.74 V, as
E oD 2 / D = -2.37 V. The correct sequence in which the ¾® 4H + + O 2 - 4e - ;
2H 2 O ¾
various metals are deposited at the cathode is ¾® H 2 + 2OH -
2 H 2 O + 2e ¾
(a) A, B, C, D (d) A, B, C
Then pH of
(c) D, C, B, A (b) C, B, A
33. Electrolysis of NaCl solution with inert electrodes for certain (a) cathodic solution will increase
(b) anodic solution will decrease
period of time gave 600 cm 3 of 1.0 M NaOH in the electrolytic (c) both the solutions will remain practically constant
cell. During the same period 31.80 g of copper was deposited
(d) both the solutions will increase
in a copper voltmeter in series with the electrolytic cell. What
39. The pH of 0.5 L of 1.0 M NaCl after the electrolysis for 965
is the percent current efficiency in the electrolytic cell ? (At.
wt. of Cu = 63. 6) seconds using 5.0 A current (100% efficiency) is
(a) 40 (b) 25 (a) 1.0 (b) 13.0
(c) 60 (d) 50 (c) 12.7 (d) 1.3
584 Chemistry
40. The emf of a particular voltaic cell with the cell reaction 44. A lead storage battery containing 5.0 L of (1N) H 2SO 4
Hg 22+ + H2 2Hg + 2H +
solution is operated for 9. 65 × 10 5 s with a steady current
is 0.65 V. The maximum electrical work of this cell when 0.5 g
of 100 mA. Assuming volume of the solution remaining
of H 2 is consumed.
constant, normality of H 2SO 4 will
(a) – 3.12 × 10 4 J (b) –1.25´105 J (a) remain unchanged (b) increases by 0.20
(c) 25.0 ´ 10 6 J (d) None (c) increase by unity (d) decrease by 0.40
41. 48250 C of electricity was required to deposit all the copper
45. The emf of the cell Pt Cl 2 (g) ( p1 atm)/ Cl - (aq) (1M)/ Cl 2 (g)
present in 0.5 L of CuSO 4 solution using inert electrodes.
(p 2 atm) Pt will be positive when
The molarity of solution was (Assume volume constant)
(a) 0.50 M (b) 2.50 M (a) p1 = p 2 (b) p1 < p 2
(c) 0.25 M (d) 1.00 M
42. At 298 K, the conductivity of saturated solution of AgCl in (c) p1 > p 2 (d) None of these
-6 46. The equilibrium constant for the disproportionation reaction
water is 2.6 × 10 Scm-1 . Its solubility product at 298 K
+
2+
(given l¥ (Ag + ) = 63.0 Scm2 mol-1 , l¥ (Cl - ) = 67.0 ¾® Cu (s) + Cu (ag)
2C u (aq) ¾

S cm 2 mol.-1 ) at 25°C (Eo Cu + / Cu = 0.52V, EoCu 2+ / Cu = 0.16V) is

(a) 4.0 ´ 10 -10 M 2 (b) 4.0 ´ 10 -16 M 2 (a) 6 ´ 10 4 (d) 6 ´ 10 6

(c) 2 ´ 10 -8 M 2 (d) 2.0 ´ 10 -5 M 2 (c) 1.2 ´ 10 6 (d) 1.2 ´ 10 -6


43. In acidic medium MnO 2 is an oxidant as 47. How long a current of 3 amperes has to be passed through a
solution of AgNO 3 to coat a metal surface
MnO 2 (s) + 4H + + 2e - ¾ ¾® Mn 2+ + 2H 2 O
of 80 cm 2 and 0.005 mm thick layer. Density of Ag is 10.5
If the pH of solution is decreased by one unit, the electrode
potential of the half cell Pt : MnO 2, Mn 2+ will change by g cm -3
(a) 0.236 V (b) – 0.236 V (a) 125.1 seconds (b) 12.5 seconds
(c) – 0.118 V (d) 0.118 V (c) 155.2 seconds (d) 200 seconds
Electrochemistry 585

EXERCISE 1 E´i´ t
10. (b) Apply m =
1. Iron ions. 96500
2. No, we cannot use nickel spatula to stir a solution of CuSO4.
7. (a) Standard electrode potential = 0.46V E´Q
11. (c) Apply m = where Q = coulombs
(b) Ecell = 0×37 V. 96500
8. 2×886 × 10–3 cm. E´Q 27
17. (a) 18. (d) 19. (c) 20. (d) 21. (c) 12. (b) Apply m = . E. wt of Al = =9.
96500 3
22. (c) 23. (d) 24. (b)
127.08
EXERCISE 2 13. (c) geq of copper to be deposited is = =4
31.75
1. (c) Electrorefining and electroplating are done by 1F deposits 1 geq, 4F will deposit 4 geq.
electrolysis.
WA WB W
2. (c) In cell the cathode is of higher reduction potential. 14. (d) = ; 27 = Cu ; \ WCu = 7.95g
3. (a) Amount of substance deposited EA EB 108 31.8
15. (c) 0.1 F will liberate 0.1 gev of Cl2 = 3.545 g
= m = E. wt ´ Q ; 16. (d) Ohm–1 cm–1
96500
1 0.47
107.870 17. (c) k = ´ Cell constant = = 0.01487
107.870 = ´ Q; R 31.6
96500
\ Q = 96500C 18. (b) Cell constant = k ´ R
= 0.212 ohm -1 cm -1 ´ 55 ohm
4. (d) WA = WB ; 1.6 = Wt. of Ag
EA EB 8 108 = 11.66 cm -1
\ Wt. of Ag = 21.6 g Cell constant 1.15
19. (a) k = =
5. (a) Ag + + e - ¾ R 250
¾® Ag
1F 108 g 1.15 1000
L eq = ´ = 4.6
1 F = 1 mole of electrons = 96500 C 250 1
0.01F = 1.08 g Ag; Ag left = 1.08 – 1.08 = 0
20. (c) Kohlrausch Law. Leq (NaF)+ Leq (HCl)– Leq (NaCl) =
6. (c) m = E.Wt ´ Q ;
96500 Leq (HF)
21. (b) Ohm–1 cm2 geq–1
m ´ 96500 22.2 ´ 96500
\ E. wt = = = 60.3
Q 2 ´ 5 ´ 60 ´ 60 k ´ 1000 3.06 ´ 10-6 ´ 1000
22. (d) Solubility = = = 2 × 10–3
L eq 1.53
At wt . 177
Oxidation state = = =3 Ksp = S2 = 4 × 10–6
Eq. wt. 60.3
23. (a) Specific conductance = Conductance × Cell constant
7. (b) m = E ´ Q = 32.68 ´ 5 ´ 40 ´ 60 = 4.065 g
96500 96500 1
k= ´1.15 ;
2.5 ´103
31.75 ´ 1´ 965 .3175
8. (c) m = = 0.3175 g = = 0.005 moles in
96500 63 1.15 1000
L eq = ´ = 4.6
one litre. \ 0.005 M. 2.5 ´ 10 3 0.1
E´i´ t 31.75 ´ .5 ´ t
9. (a) m = ;2= , \ t = 12157.48 sec . k ´ 1000 1.06 ´ 10-2 ´ 1000
96500 96500 24. (a) L m = = = 1.06 ´ 10 2
M 0.1
586 Chemistry
47. (a) Fe Al Br
L m ´ M 1.26 ´ 10 -2 ´ 0.01
25. (b) k = = = 1.26 ´ 10 -3 0.77 –1.66 1.08 E°Red
1000 1000 –0.77 1.66 –1.08 E°Oxi
26. (a) L¥m = 57 + 73 = 130 S cm 2 mol -1 X 2+ Y 2 - Hence Reducing power Al > Fe > Br
48. (c) On the basis of ECS strongly electropositive metals can
x displace less electropositive metals from their salt
Lcm solutions.
27. (c) Degree of dissociation = ¥ = 100 = 0.01 . 49. (c) I has the highest oxidation potential hence most reducing.
Lm x
50. (a) Since oxidation potential of Zn is highest hence strongest
28. (b) It is an example of concentration cell, Ecell cannot be zero reducing agent.
since [H+] are different (HCl is strong and CH3COOH
51. (d) The reduction potential of NO3- ion is more than H+
weak acid).
29. (d) Reduction occurs at cathode. ion. (see ECS).
30. (c) Consult ECS. Reduction potential of Cu is more 52. (c) Gold having higher E oRed and oxidises Fe ¾
than H2. ¾® Fe + + .
53. (d) Cell reaction Zn + Cu + + ¾ ¾® Zn + + + Cu
31. (d) 4 2 DG = –827 kJ mol–1
¾® Al 2 O 3 ;
Al + O 2 ¾ 0.059 0.01
3 3 o
E1 = E cell - log
2 1.0
æ2 ö o
Number of electrons involved is 4 ç ´ 6 = 4 ÷ \ E1 = (E cell + 0.059) V
è 3 ø
o 0.059 1.0
E 2 = E cell - log
827000 2 0.01
DGo = –nFE o or = Eo
4 ´ 96500 o
\ E 2 = (Ecell - 0.059) V E1 > E2.
o
\ E = 2.14V 54. (c) Copper is below in ECS hence cannot displace hydrogen
2+ 2 from HCl.
RT ln[Fe ] é 2+ ù
32. (c) Nernst equation Ecell = E ocell - Zn HCl H+ + Cl–
nF [Fe3+ ]2 ë û
2H + 2e ® H2
+ –
increasing [Fe2+] will decrease the Ecell. Cu ® Cu++ + 2e–
33. (d) Emf will decrease. not possible. Hence evolution of H2 is not possible.
34. (a) Salt bridge allows the flow of current by completing 55. (b) I2 is less reactive than Br2.
circuit. No current will flow and voltage will drop to zero, 56. (d) Oxidation potential of Co is more than Ag, hence cell
if salt bridge is removed. reaction will be
35. (c)
Co + 2 Ag + ¾
¾® Co 2 + + 2 Ag
36. (d) Calomel electrode is used as reference electrode.
37. (d) Though it is not zero but has been fixed as zero. o RT ln [Co 2 + ] , the lesser the value of
E cell = E cell -
38. (b) E ocell = E ored (Zn) + E oxi
o
(Fe) = 0.76 - 0.41 = 0.35V nF [Ag + ]2
2+
39. (a) OCl - ¾
¾® Cl - ; E° = 0.94 V (I) the factor [Co ] greater will be value of Ecell. .
[ Ag + ]2
1
Cl - ¾
¾® Cl 2 + e - ; E° = 1.36 V (II)
2 o RT [C]c [D]d
57. (b) E cell = E cell - ln .
nF [A]a [B]b
- 1
Add (I) + (II) OCl ¾¾® Cl 2 ; E° = 0.94 – 1.36 = –0.42 V
2 RT [C]c [D]d
Hence E ocell = E cell + ln
40. (d) E ocell = E oRe d (Fe) + E oxi
o
(Sn) = –0.44 + 0.14 = –0.30V nF [A]a [B]b
58. (a) Since concentration of ions is the same hence
41. (b) E ocell = E oRe d (Au) + E oxi
o
(Ni) = 1.5 + 0.25 = 1.75V
Ecell = E o cell.
42. (c) Oxidation potential of M is more than Ni and less than
Mn. Hence reducing power Mn > M > Ni 59. (a) The Half cell reaction is Zn + + + 2e - ¾
¾® Zn .
43. (a) The greater the oxidation potential greater is reactivity o 0.059 1
E cell = E cell - log
A > D > B > C. 2 [Zn ++ ]
44. (a) Chlorine is less reactive than fluorine.
45. (b) Follow ECS, more the reduction potential, the stronger = - 0.763 - 0.059 log 1 = -0.822 V
the oxidising power and vice versa. 2 0.01
46. (b) Follow ECS, A will replace B. Eoxi = 0.822 V
Electrochemistry 587

60. (d) 2 H + + 2e - ¾
¾® H 2 o 0.0591
3. (c) As Ecell = log Kc
n
0.059 1
E Red = E oRe d - log + 2 ; 0.0591
n [H ] \ 0.46 = log Kc
2
0.059 1 2 ´ 0.46
E Re d = 0 - log ; ERed = –0.059 V,,
2 (0.1) 2 \ log K c = = 15.57
0.0591
Eoxi = 0.059 V. or Kc = Antilog 15.57 = 3.7 × 1015 » 4 × 1015
RT C 4. (c) From the given data we find Fe 3+ is strongest
o
61. (b) E cell = E cell - ln 2 and DG = –nFE°cell oxidising agent. More the positive value of E°, more is
nF C1
the tendency to get oxidized. Thus correct option is
C (c).
Hence DG is a function of ln 2 . 5. (d) Kohlrausch’s Law states that at infinite dilution, each
C1
ion migrates independent of its co-ion and contributes
0.059 to the total equivalent conductance of an electrolyte a
o
62. (b) At equilibrium Ecell = 0. E cell = log K C definite share which depends only on its own nature.
n
From this definition we can see that option (d) is the
o 0.059 1.10 ´ 2 correct answer.
63. (b) E cell = log K C or = log K C
6. (c) Writing the equation for pentane-oxygen fuel cell at
2 0.059
respective electrodes and overall reaction, we get
\ K C = 1.9 ´ 1037 At Anode:
64. (a) DG = –nF E°cell = –2 × 96500 × 1.1 J = 212.3 kJ. C5 H12 + 10H 2 O ® 5CO 2 + 32H + + 32e -
65. (a) Q Change in free energy = maximum electrical work done (pentane)
66. (b) Water is weak electrolyte and remains almost unionised.
At Cathode:
67. (b) Cathode Anode
Aqs. soln. of Na2SO4 2 Na+ SO4– – 8O 2 + 32H + + 32e - ® 16H 2O
Ionisation of H2O ® H+ OH– Overall :C5H12 + 8O2 ® 5CO 2 + 6H 2O
+ - 1 Calculation of DG° for the above reaction
E°Red of H is more hence H + e ¾ ¾® H 2
2 DG° = [5×(–394.4) + 6× (–237.2)]
68. (b) Cl– is oxidised to Cl2 at anode. – [–8.2]
69. (b) 70. (d) 71. (b) 72. (c) = – 1972.0 – 1423.2 + 8.2 = – 3387.0 kJ
73. (b) H2 – O2 fuel cell supply power for pressure. = – 3387000 Joules.
74. (b) H2SO4 regenerated follow text. From the equation we find n = 32
-nFE ° and substituting
75. (c) Thermodynamic efficiency is given by Using the relation, DG° = – nFEcell
DH
76. (c) various values, we get
77. (a) Galvanization ° (F = 96500C)
– 3387000 = –32×96500× Ecell
78. (b) Specific conductance decreases with dilution
(follow text). 3387000
° =
or Ecell
79. (d) In electrolytic cell the flow of electrons is from anode to 32 ´ 96500
cathode through internal supply.
3387000 3387
EXERCISE 3 = or V = 1.0968 V
3088000 3088
1. (d) Zinc gives H2 gas with dil H2SO4/HCl but not with HNO3 Thus option (c) is correct answer.
because in HNO3, NO3– ion is reduced and give NH4NO3, 7. (d) Cu2+ + 2e– ® Cu; DGo = – nEoF
N2O, NO and NO2 (based upon the concentration of = – 2 × F × 0.337
HNO3) = – 0.674 F ....(i)
4Zn+10HNO 3 ¾¾ ® 4Zn(NO 3 ) 2 +NH 4 NO 3 +3H 2 O Cu+ ® Cu2+ + e– ; DGo = – nEoF
= – 1 × F × – 0.153
Zn is on the top position of hydrogen in electrochemical
= 0.153 F ....(ii)
series. So Zn displaces H2 from dilute H2SO4 and HCl
On adding eqn (i) & (ii)
with liberation of H2.
Cu+ + e– ® Cu ;
Zn + H2SO4 ® ZnSO4 + H2
DGo = – 0.521 F = – nEoF;
DG Here n = 1 \ Eo = + 0.52 V
2. (b) Efficiency of a fuel cell (h ) =
DH
588 Chemistry
8. (a) Q Q = i × t 16. (b) Given ESn +4 = + 0.15 V
Sn +2
\ Q = 4.0 × 104 × 6 × 60 × 60 C
= 8.64 × 10 8 C E Cr+ 3 Cr = – 0.74 V
3+ -
Al + 3e ¾¾
® Al
3´96500C 27g E °cell = E °ox + E °red
[3 × 96500 C liberates = 27 g of Al] = 0.74 + 0.15
\ 96500 C liberates 9 g of Al = 0.89 V
9 17. (d) Standard Gibbs free energy is given as DG° = – nE°F
8.64 × 108 C liberates = ´ 8.64 ´ 108 g Al
96500 If E°cell < 0 i.e., – ve
= 8.1 × 104 g of Al DG° > 0
9. (d) Degree of dissociation Further DG° = – RT ln Keq
L 8.0 \ DG° > 0 and Keq < 0
a= = = 2 ´ 10-2 18. (c) Given Fe+3/Fe2+ = + 0.77 V
L ¥ 400
and I2/2I– = 0.536V
2 (e– + Fe+3 ¾¾ ® Fe+2) E° = 0.77 V
ca 2 1
( )
2
Ka = » ca 2 = ´ 2 ´ 10 -2 2I – ¾¾ ® I2 + 2e – E° = – 0.536 V
(1 - a ) 32
2Fe+3 + 2I– ¾¾ ® 2Fe+2 + I2
= 1.25 ´ 10-5 E° = E°ox + E°red
10. (a) Cu + 2 Ag+ (aq) ¾¾
® Cu 2+ (aq ) + 2 Ag (s ) = 0.77 – 0.536
= 0.164 V
Here, n = 2 , E°cell = + 0.46 V \ Since value of E° is + ve reaction will take place.

DG° = – nE°F 19. (d) L°m( NH ° °


) = L m NH4+ + L mCl-
4 Cl
– 2×0.46×96500
= kJ ; – 89 kJ
1000 L°m( NaOH) = L°m + L°m
Na + OH -
11. (a) Dilution of strong electrolytes increases ionisation,
hence ionic mobility of ions which in turn increases L°m( NaCl) = L°m + L°m
equivalent conductance of the solution. Na + Cl-
12. (c) Conductivity of an electrolyte depends on the mobility
\ L°m °
of ions and concentration of ions. The motion of an
ionic species in an electric field is retarded by the
( NH+4 ) + Lm(OH- )
oppositely charged ions due to their interionic
= L°m ° °
attraction. On dilution, concentration of electrolyte
decreases and the retarding influence of oppositely
( NH4+ ) + Lm(Cl- ) + L m( Na + )
charged ions decreases. Therefore mobility of ions
é ù
increases. +L°m - ê L°m + + L°m - ú
13. (a) Option (ii) and (iv) are correct (OH ) -
ëê
Na ( )Cl ú
û ( )
14. (c) As the value of standard reduction potential decreases
the reducing power increases i.e., L°m( NH L )
°
(
°
) + L m( NaOH) -L°m( NaCl)
4 OH = m NH 4 Cl
Z > X > Y
( -3.0) ( -1.2) ( +0.5) 20. (a) Higher the value of reduction potential higher will be
the oxidising power whereas the lower the value of
o o o
15. (b) Cu 2 + + 1e - ® Cu + E1 = 0.15V; DG1 = - n1E1 F reduction potential higher will be the reducing power.

21. (d) L°CH3COOH = L °CH3COONa + L °HCl - L°NaCl


Cu + + 1e - ® Cu E2o = 0.50V; DG2o = - n2 E2o F
= 91 + 425.9 – 126.4 = 390.5 S cm2mol–1
22. (a) H2 ¾¾ + -
® 2H + 2e
Cu 2 + + 2e - ® Cu DG° = DG°1 + DG° 2
1 atm 10-10
– nE°F = –1 n1 E1o F + (–1) n2 E2oF
– nE°F = –1 (n1 E1o F + n2 E2oF)
( )
2
E 0.059 10-10
H 2 /H = 0 –
+ log
n1E1o + n2 E2o 0.15 ´ 1 + 0.50 ´ 1 2 1
E° = =
n 2
E = +0.59 V
E° = 0.325 V H 2 /H +
Electrochemistry 589
LM 9.54 0.059
23. (b) a = L ¥ = = 0.04008 = 4.008 %. 37. (a) Ecell = log K c
M 238 n
24. (d) E°Cell = E°OP + E°RP
0.591 = 0.059 log K c or 10 = log K c \ K c = 1´1010
= 0.76 + 0.314 = 1.10 V

25. (b) S =
K m 2 mol
\ K = Sm 2 mol-1
( )
38. (c) L o NaCl = l o Na + + lCl- ....(i)
3
m´m
26. (c) Ecell = Reduction potential of cathode (right)
Lo KBr = lo ( K ) + l ( Br )
+ o -
....(ii)
- reduction potential of anode (left)
= Eright - Eleft. L o KCl = l o ( K ) + l ( Cl )
+ -
....(iii)
27. (a) Ecell = Eright (cathode) - Eleft (anode). (i) + (ii) - (iii)
28. (d) If p1 > p2 then E cell =
2.303
nF
p
RT log 1
p2
(
L o NaBr = l o Na + + lo Br - ) ( )
29. (a) 2Cr 3+ + 7H 2 O ® Cr2 O 72- + 14H + = 126 + 152 - 150 = 128 S cm 2 mol -1
O.S. of Cr changes from + 3 to + 6 by loss of electrons. 39. (c) The given values show that Cr has maximum oxidation
At anode oxidation takes place. potential, therefore its oxidation will be easiest. (Change
30. (c) The equilibrium constant is related to the standard emf the sign to get the oxidation values)
of cell by the expression 40. (a) Thus difluoro acetic acid being strongest acid will furnish
maximum number of ions showing highest electrical
n 2
log K = Eº cell ´ = 0.295 ´ conductivity. The decreasing acidic strength of the
0.059 0.059
carboxylic acides given is difluoro acetic acid >
590 fluoro acetic acid > chloro acetic acid > acetic acid.
log K = = 10 or K = 1 × 1010
59 41. (d) 1 mole of e– = 1F = 96500 C
31. (d) A B C 27g of Al is deposited by 3 × 96500 C
+ 0.5C –3.0V –1.2V 5120 g of Al will be deposited by
The higher the negative value of reduction potential, 3 ´ 96500 ´ 5120
the more is the reducing power. Hence B > C > A. = = 5.49 ´ 10 7 C
27
9650 1
32. (a) No. of moles of silver = = moles 42. (b) L ¥
96500 10 HCl = 426.2 (i)

\ Mass of silver deposited = 1 ´ 108 = 10.8 g L¥


AcONa = 91.0 (ii)
10

NaCl = 126.5 (iii)
+2
0.059 [Cu ]
33. (b) E cell = E º cell + log L¥
AcOH = (i) + (ii) - (iii) = [426.2 + 91.0 - 126.5] = 390.7
n [ Zn + 2 ]
o
0.059 43. (b) L CH is given by the following equation
3COOH
= 1.10 + log [0.1] = 1.10 - 0.0295 = 1.07 V
2
34. (b) Magnesium provides cathodic protection and prevent
o
L CH
3COOH
( o
= L CH
3COONa
)(
+ L oHCl - L oNaCl )
rusting or corrosion.
o
35. (b) In H 2 - O 2 fuel cell, the combustion of H2 occurs to Hence L NaCl is required.
create potential difference between the two electrodes l
44. (a) Conductivity (kappa) =
36. (a) Fe3+ + e ® Fe2 + , DG ° = -1 ´ F ´ 0.77 R.a.

Sn 2+ + 2e ® Sn(s), DG° = -2 ´ F(-0.14) ælö


cell constant. ç ÷ = k ´ R = 1.29 ´ 100 = 129
for è aø

Sn (s) + 2Fe 3+ (aq) ® 2Fe 2+ (aq) + Sn 2 + (aq) 1


Again conductivity of 0.02M solution k = ´ 129
DG = -(-2F ´ (-0.14) + 2(-1 ´ F ´ 0.77) = -1.82F 520
\ Standard potential for the given reaction k ´ 1000 129 1000
Lm = = ´ = 1.24 ´ 10 -4 S m -2 mol -1
-1.82F M 520 0.02
= = 0.91V
- 2´ F
590 Chemistry
45. (b) According to Kohlrausch’s law, molar conductivity of 49. (b) Given
weak electrolyte acetic acid (CH3COOH) can be
Fe3+ + 3e - ® Fe ,
calculated as follows:
… (i)
(
LoCH3COOH = LoCH3COONa + L oHCl - Lo NaCl ) E°
Fe3+ / Fe
= –0.036 V

\ Value of L o NaCl should also be known for Fe 2+ + 2e - ® Fe ,


E° = –0.439V … (ii)
calculating value of L oCH3COOH . Fe2 + / Fe

46. (d) Ecell = 0; when cell is completely discharged. we have to calculate


Fe3+ + e - ® Fe2 + , DG = ?
æ é Zn 2+ ù ö
0.059
log ç ë û÷ To obtain this equation subtract equ (ii) from (i) we
Ecell = E°cell - ç é 2+ ù ÷
2 ç Cu ÷ get
èë ûø
Fe3+ + e – ® Fe 2 + … (iii)
æ é Zn 2+ ù ö As we know that DG = – nFE
0.059
or 0 = 1.1 - log ç ë û÷ Thus for reaction (iii)
2 ç é 2+ ù ÷
ç Cu ÷ DG = DG1 - DG2
èë ûø
– nFE° = – nFE1 – (–nFE2)
æ é Zn 2+ ù ö –nFE° = nFE2 – nFE1
log ç ë û ÷ = 2 ´1.1 = 37.3 –1FE° = 2× (– 0.439) – 3 × (– 0.036)
ç é 2+ ù ÷ 0.059 –1 FE° = – 0.770 F
ç Cu ÷
èë ûø \ E° = + 0.770V
50. (c) DG = – nFE
é Zn 2+ ù For 1 mol of Al, n = 3
ë û = 1037.3
\
é Cu 2+ ù 4 4
ë û For mol of Al, n = ´ 3 = 4
3 3
47. (d) From the given representation of the cell, Ecell can be
found as follows. DG 966´103
or E = = = -2.5 V
-nF -4´96500
[ Cr3+ ]
2
Ecell = ( E o 2+
Fe / Fe
- E o 3+
Cr / Cr ) -
0.059
6
log
[Fe 2+ ]3
\ The potential difference needed for the reduction
= 2.5 V.
[Nernst -Equ.] 51. (a) The value of E ° for given metal ions are
M 2+ M
0.059 (0.1) 2
= –0.42 – (–0.72) - log E° = -1.18 V,
Mn 2+ Mn
6 (0.01)3
0.059 0.1´ 0.1 E° = - 0.9 V,
= –0.42 + 0.72 - log Cr 2+ Cr
6 0.01´ 0.01´ 0.01
E° = - 0.44 V and
Fe 2+ Fe
-2
0.059 10 0.059
= 0.3 - log = 0.3 - ´4 E° = - 0.28 V.
6 -6 6 Co 2+ Co
10
= 0.30 – 0.0393 = 0.26 V
Hence option (d) is correct answer. The correct order of E ° values without
M 2+ M

3 considering negative sign would be


48. (c) CH3OH (l) + O2 (g) ® CO2 (g) + 2H2O (l) Mn2+ > Cr2+ > Fe2+ > Co2+.
2
1
DGr = [DGf (CO 2 , g ) + 2DGf (H 2O, l)] - 52. (c) H+ + e– ¾¾ ® H2
2
é 3 ù 0.059 [P(H 2 )]1/ 2
êë DGf (CH3OH, l )+ 2 DGf (O 2 , g ) úû E = Eº – log
1 [H + ]
= – 394.4 + 2 (–237.2) – (–166.2) – 0
= – 394.4 – 474.4 + 166.2 = – 702.6 k J Now if p H2 = 2 atm and [H+] = 1M

702.6 0.059 21/ 2 -0.059


% efficiency = ´ 100 = 97% then E = 0 – log = log 2
726 1 1 2
Electrochemistry 591
53. (d) Here n = 4, and [H+] = 10– 3 (as pH = 3) Q=I×t
Applying Nernst equation Q 1930 C
t= = = 19.3´10 4 sec
2+ 2 I 10 ´10-3 A
0.059 [Fe ]
E = Eº – log + 4
n [H ] ( pO2 ) 59. (d) AgNO 3 ( aq ) + KCl( aq ) ¾¾
® AgCl( s ) + KNO 3 ( aq )
Conductivity of the solution is almost compensated
0.059 (10-3 )2
= 1.67 - log due to formation of KNO3(aq). However, after end
4 (10-3 ) 4 ´ 0.1 point, conductivity increases more rapidly due to
addition of excess AgNO3 solution.
0.059
= 1.67 - log107 = 1.67 – 0.103 = 1.567
4 EXERCISE 4
1 l 1. (a) The specific conductance increases with concentration.
54. (a) k= ´
R A The number of ions per cm -3 increase with increase of
1 l concentration.
1.3 = ´ 2. (a) The equivalent conductance increase with dilution, since
50 A
interionic attractions decrease with dilution
l
= 65m-1 3. (c) The correct-relation is DG° = -nFE°
A 4. (d) The amount. of copper deposited at cathode by
k ´ 1000 3.2
L= reduction of Cu 2+ ions is = 0.05 moles. The same
molarity 63
[molarity is in moles/litre but 1000 is used to convert
liter into cm3] amount 0.05 mole of Cu 2+ must pass into solution from
anode by oxidation
æ 1 ö 5. (c) Cu(CN) 2 in basic medium forms a complex which
çè ´ 65 m -1 ÷ ´ 1000 cm3
260 ø 2 ++
= maintains the constant supply of Cu ions
0.4 moles
1
6. (b) The oxidation potential µ and
650 m -1 1 Concentration of ions
= ´ m3
260 ´ 4 mol 1000 reduction potential µ concentration of ions. The cell
= 6.25 × 10–4 S m2 mol–1 voltage can be increased by decreasing the concentration
55. (d) For a spontaneous reaction DG must be –ve of ions around anode or by increasing the concentration
Since DG = – nFE° of ions around cathode
Hence for DG to be -ve DE° has to be positive. Which 7. (c) Electrodialysis of aqueous solution of
is possible when X = Zn, Y = Ni CuSO 4 Cu ++ + SO --
4
Zn + Ni++ ¾¾ ® Zn++ + Ni
Around anode we have SO 4- - and H 2 O . The oxidation
E° + E°
Zn / Zn +2
Ni / Ni= 0.76 + (–0.23)
2+
Potential of H 2 O(1.23V) is lower than oxidation
= +0.53 (positive)
56. (d) Higher the value of standard reduction potential, potential of SO 4-- (2.0V)
stronger is the oxidising agent, hence MnO4– is the \ 2H 2 O ® O2 + 4H + + 4e-
strongest oxidising agent.
57. (d) E°
Cr 3+ / Cr 2 + = – 0.41 V ( ) ( )
8. (b) L ¥m (AgCl) = l ¥ Ag + + l ¥ Cl - = 61.92 + 76.34 = 138.26

E° k ´ 1000 1.826 ´ 10 -6 ´ 1000


= + 0.77 V Lm = \M = = 1.32 ´ 10 -5 M
Fe3+ / Fe2 + M 138.26

Mn 3+ / Mn 2 + = + 1.57 V,, Hence solubility = 1.32 ´ 10 -5 ´ 143.5g / l
E° = + 1.97 V = 1.9 ´ 10 -3 g / l
Co3+ / Co 2 +
58. (b) ˆˆ† H + + OH-
H 2O ‡ˆˆ 9. (a) The equation is H+ + e– 1/2H2 ;
0.159 1
1 E = E° - log
H+ + e- ¾¾ ® H2 1 [H + ]
2
\ 0.5 mole of H2 is liberated by 1 F = 96500 C Since
0.01 mole of H2 will be liberated by 0.059 1
pH = 10, [ H + ] = 10 -10 ; E = 0 - log ; E = 0.59
96500
´ 0.01 = 1930 C
1 10 -10
=
0.5
592 Chemistry
18. (a) The reaction is 2Tl(s) + Sn4+ ® 2Tl+ + Sn2+
10. (a) Λ ¥ ¥ ¥ ¥
eq (NH 4 OH) = Λ eq (NH 4 Cl) + Λ eq (NaOH) - Λ eq (NaCl)
The cell is Tl(s) | Tl+(1.0M) | | Sn4+(1.0M) | Sn2+(1.0M) 1Pt
–0.34 V +0.13V
= 129.8 + 217.8 - 109.3 = 238.3 ohm -1cm 2 eq -1
0.0592 [Tl + ]2 [Sn 2 + ]
L eq 9.30 E = 0.13 - ( -0.34) - log
a= = = 0.04 or 4% 2 [Sn 4 + ]
L¥aq 238.3
= 0.47 V – 0.0296 log (10)2 [Tl+ conc. increases tenfold)
= 0.47 – 0.0592 = 0.411V
11. (c) DG ° = -nFE°; E° = -DG ° ; 19. (b) Unit of ionic mobility is cm2 s–1 volt–1
nF 20. (b) Reciprocal of resistance is conductance. We can add
the conductance and not resistance. Hence
-(-50.61kJ)
E° = = 0.26V
2 ´ 96500 ´10-3 1 1 1
= + + .........
r r r
2 ´ 112g
12. (a) 112 ml of H2 at STP = (Since 22400 ml at STP= 21. (c) At infinite dilution each ion makes a definite contribution
22400 towards molar conductance which is given by
M.wt)
Lom = V+ l + + V- l -
Eq.wt ´ i ´ t 22. (a) The equivalent conductance of weak electrolytes
Amount deposiled =
96500 increase with dilution ( c ).
2 ´ 112 1 ´ 965 ´ i 23. (d) The relation between ionic mobility and ionic
\ = ; l°
22400 96500 conductance is U º =
F
i = 1 amp
Where Uº is ionic mobility and lº ionic conductance
13. (a) A + e- ® A- has large negative potential . It means A- is \ Lo = F (U+ + U- )
readily oxidised the reaction proceeds towards left hand 24. (a) In case of equivalent conductance of strong electrolyte
side there is little increase with dilution.
14. (b) The electrochemical equivalent is the amount deposited 25. (a) The greater the ionic mobility, the greater the molar
by one coulomb of electricity conductance. The ionic mobility of HCl is greater than
that of NaCl.
coulomb of electricity = 0.5 ´ 200 = 100o C
0.059
\ Amount deposited = 0.001180´100 = 0.1180g 26. (b) DE = E° - log [H + ]2
2
15. (a) The given order of reduction potentials (or tendencies)
0.059
is Z > Y > X. A spontaneous reaction will have the = 1.30 - log (10 -2 ) 2
following characteristics 2
Z reduced and Y oxidised = 1.418 V
Z reduced and X oxidised 27. (c) Cd(s) + 2 Ag + (aq) ¾
¾® Cd 2 + (aq) + Ag (s),
Y reduced and X oxidised
Hence, Y will oxidise X and not Z. 0.059 [Cd 2 + ]
E cell = E °cell – log
16. (b) For, M + + X - ¾ o 2 [Ag + ]2
¾® M + X , E cell = 0.44 – 0.33 = 0.11V is
positive, hence reaction is spontaneous. 28. (b) Conductivity (X) = conductance (c) × cell constant

17. (a) MnO-4 will oxidise Cl - ion according to the equation X


\ Cell constant =
Y
2MnO -4 + 16H + + 10Cl - ¾
¾® 2Mn 2 + + 8H 2 O + 5Cl 2 ­
X
The cell corresponding to this reaction is as follows: Conductivity of NaOH = .Z
Y
Pt, Cl2 (1 atm) | Cl- || MnO4- , Mn 2 + , H + | Pt
X 1000 XZ 4
D m (NaOH) = .Z´ = 10
o Y 0.1 Y
E cell = 1.51 -1.40 = 0.11 V

Eocell being +ve, DG ° will be -ve and hence the above 29. (a) Ar NO 2 + 6H + + 6e - ¾
¾® ArNH 2 + 2H 2 O

reaction is feasible. MnO-4 will not only oxidise Fe 2 + Ewt ´ Q M 2 ´ 96500 50


W= ; 20.50 = ´ ´
96500 6 96500 100
ion but also Cl - ion simultaneously.. \ M = 123.0 g
Electrochemistry 593

30. (a) K 2SO 4 2K + + SO24- E °cell = 0.207 – 0.799 = – 0.592 V


H 2O H+ + OH - MX ( s ) ® Ms+ ( aq ) + X s- ( aq )
+ ¯ 4e - – ¯ 4e - Ksp = éM+ ù éX- ù = S ´ S = S2
ë ûë û
2H 2 O 2 + 2H 2 O
o 0.592
E cell = log K sp
For 22.4 × 3 L º 4 e - n
4 ´ 1.68 - 0.0591
or 1.68 L at STP º e = 0.1 mole e - -0.592 = log K sp
22.4 ´ 3 1
MnO 4- + 8H + + 5e - ¾
¾® Mn 2 + + 4H 2 O log Ksp = – 10, Ksp = 10 -10 ,

5e - reduce 1 mole MnO -4 Solubility = Ksp = 1.0 × 10 -5


36. (c) Anode :
1
0.1 will reduce = ´ 0.1 = 0.02 Moles 1
5 ¾® 2H + +
CuSO 4 / Pt. H 2 O ¾ O 2 + 2e -
2
31. (a) When pH = 14 [H + ] = 10-14 and [OH - ] = 1 M

Ksp = [Cu 2+ ] [OH - ]2 = 10 -19 ¾® Cu 2+ (aq) + 2e -


CuSO 4 / Cu Cu(s) ¾

10 -19 KCl/Pt 2Cl - ¾


¾® Cl 2 (aq) + 2e -
\ [Cu 2+ ] = = 10 -19
[OH - ]2 Cathode :
The half cell reaction Cu 2+ (aq) + 2e - ¾
¾® Cu(s) (pH = decrease)
Cu 2+ + 2e - ¾
¾® Cu -
Cu 2+ (aq) + 2e ¾¾® Cu(s) (pH = constant)
0.059 1 2 H 2 O + 2e - ¾
¾® H 2 + 2OH - (aq) (pH = increase)
E = E° – log
2 [Cu 2+ ]
37. (a) 2 H + + 2e - ¾
¾® H 2 ; E ° = 0 V
0.059 1
= 0.34 – log = – 0.22 V 0.059 1
2 10 -19 E = Eo - log
H + /H 2 H + /H 2 2 [H + ]2
32. (b) The more the reduction potential, the more is the
deposition of metals at cathode. Cation having E° value – 0.414 = 0 + 0.059 log [H + ]
less than – 0.83 V (reduction potential of H 2 O ) will not
deposit from aqueous solution. \ [H + ] = 1.02 ´10 -7 M
38. (c) Due to buffer action the pH will remain practically
600 ´ 1 constant.
33. (c) Moles of NaOH formed = = 0.6
100
or geq of NaOH = 0.6 39. (b) Anode : 2 Cl - ¾
¾® Cl 2 + 2e -

31.8 Cathode : 2H 2 O + 2e - ¾
¾® H 2 + 2OH -
geq of Cu deposited = = 1 .0
63.6 / 2
1´ 5 ´ 965
Moles of OH - formed = Z × i × t = = 0.05
0.6 ´100 96500
\ Current efficiency = = 60 %
1
0.05
34. (d) Ionic mobility is the velocity of an ion under a potential [OH - ] = = 1 ´ 10 -1
0 .5
gradient of 1Vm -1 .
[H + ] = 10 -13 pH = 13
- -
35. (a) MX(s) + e ¾
¾® M(s) + X (aq); E° = 0.207 V .......(i)
40. (a) Wmax = – n.FE;
+ - ......(ii)
M ( aq ) + e ¾
¾® M (s ); E ° = 0 .799 V Wmax = – 2 × 96500 × 0.65 = – 1.25 ´ 10 5 J
From (i) – (ii),
0.5g H 2 = 0.25 mole.
¾® Ms+ (aq) + Xs- (aq);
MX(s) ¾
Hence Wmax = – 1.25 ´ 10 5 ´ 0.25 = –3.12 ´ 10 4 J
594 Chemistry

1 ´ 48250 2 × 96500 C consumed 4 equiv. of H 2SO 4


41. (a) Moles of Cu 2+ deposited = = 0.25
2 ´ 96500 -3
and 100 × 10 × 9.65 ´ 10 5 C consumed
0.25
\ Molarity = = 0.5 4 ´ 100 ´ 10 -3 ´ 9.65 ´ 105
0.5 = = 2 equiv. H 2SO 4
2 ´ 96500
42. (a) L m (AgCl) = 63 + 67 = 130
2
k ´ 1000 \ Decrease in normality = = 0.40
Lm = 5
solubility 45. (b) The cell reaction is
-3
2.6 ´ 10 ´ 1000 Cl 2 (g) ( P2 atm) ¾
¾® Cl 2 (aq) (P1atm)
\ Solubility S = = 2 ´105 mol -1
130
0.0592 P 0.0592 P
10 E cell = – log 1 = log 2
\ K sp = 4 ´ 10 2 P2 2 p1

[Mn 2+ ] E cell will be positive when p 2 > p1 .


0.0592
43. (d) E = E° - log 46. (c) The reaction
2 [ H + ]4
2 Cu + (aq) ¾
¾® Cu(s) + Cu 2+ (aq)
0.0592 ´ 4 [Mn 2+ ] 1 / 4
= E° - log
[Cu 2 + ]
2 [H + ] E cell = E° cell –
0.0592
log
1 [Cu + ]2
= E° – 0.0592 ´ 2 (log[Mn 2+ ]1 / 4 + pH)
At equilibrium E cell = 0 \ E° cell = 0.0592 log K c
D E = E 2 - E1 = 0.0592 × 2 (pH1 - pH2 )
= 0.188 × 1V 0.52 – 0.16 6
or, log K c = \ K c = 1.2 ´ 10
0.0592
44. (d) Pb + SO 4- - ¾
¾® PbSO 4 + 2e -
47. (a) Volume = Area × thickness
Mass = Volume × density
PbO 2 + 4H + + SO 4- - + 2e - ¾
¾® PbSO 4 + 2H 2 O
\ Mass of Ag to be deposited
80 ´ 0.005
Pb + PbO 2 + 2H 2SO 4 ¾
¾® 2PbSO 4 + 2H 2 O = ´ 10.5 = 0.42 g
10
The reaction indicates that 2 moles of H 2SO 4 E.wt ´ i ´ t
Amount deposited =
corresponds to 2 × 96500 C and 2 mol H 2SO 4 º 4 96500
equiv. of H 2SO 4 . 108 ´ 3 ´ t
\ 0.42 = \ t = 125.1 seconds
98500
18
Chemical Kinetics
CHEMICAL KINETICS : rinst d[P]
Deals with rate, mechanism and factors affecting the chemical dt =slope
Concentration of reactants

reactions.
RATE OF REACTION : d[P]
It is defined as the change in concentration of a reactant or a dt
product in a particular interval of time. The change in concentration [P1]
of a substance say A is represented by D[A] D[P]
D[A] = [Final concentration – Initial concentration] [P2] rav D[P] {[P2]–[P1]}
For a reactant : Final concentration < Initial concentration Dt Dt (t2 – t1)
Thus D[A] will have negative sign
For a product : Final concentration > Initial concentration
Thus D[A] will have positive sign t Time t1 t2
Average rate in terms of products (b)
D[A ] Total change in concentration Instantaneous and average rate of a reaction
= = The minus sign indicates the decrease in concentration and plus
Dt Total change in time sign increase in concentration. The above expressions give the
D[ A ] average rate of reaction.
Average rate in terms of reactants = - FEATURES OF THE RATE OF REACTION :
Dt
The negative sign in the expression make the rate positive. Hence (I) Rate of reaction is proportional to the concentration of the
rate of a reaction is never negative. reactants
Consider the decomposition of N2O5 (II) Rate of a reaction is always a positive quantity
(III) Rate of a reaction is determined by measuring the
2 N 2 O 5 (g ) ¾
¾® 4 NO 2 (g ) + O 2 (g )
concentration of a reactant or a product as a function of time
1 DN 2 O 5 1 DNO 2 DO 2 (IV) Property related to concentration is selected e.g. volume,
Rate = - . = =
2 Dt 4 Dt Dt pressure, thermal or electrical conductivity, colour change,
pH, etc.
[R]0 UNITS OF RATE OF REACTION :
Mol L–1 time–1 and for gaseous reaction atm time–1
INSTANTANEOUS RATE :
Concentration of reactants

[R1] As the reaction progresses the concentration of reactants keeps


–D[R] –{[R2]–[R1]}
D[R] rav = on falling. Hence the rate of reaction keeps on falling with time.
[R2] d[R] Dt (t2 – t1)
The rate of reaction at any given instant will be given by the
Dt –d[R]
rinst –slope dx 1 dN 2 O 5 1 dNO 2 dO 2
dt expression r = =- = . =
dt dt 2 dt 4 dt dt
where dx is the infinitesimally small change in the concentration
t1 t2 t Time of x in infinitesimally small interval of time dt.
(a)
596 Chemistry
RELATION BETWEEN AVERAGE AND Increase in temperature or decrease in the activation energy
INSTANTANEOUS RATE: will increase the rate of reaction and exponential increase in
Instantaneous rate = Average rate as Dt approaches to zero the rate constant
Dx ö At temperature T1 the equation (ii) is
Instantaneous rate = æç
dx
÷ =
è Dt ø Dt ®0 dt - Ea
lnk1 = + log A (iii)
FACTORS AFFECTING THE RATE OF REACTION : RT1
(i) Nature of the reactants : Reactions involving lesser bond and at temperature T2
rearrangement proceed much faster than those which involve
larger bond rearrangement. - Ea
(ii) Temperature : In most cases the rate of reaction in a lnk2 = + log A (iv)
RT2
homogeneous system is approximately doubled or even
tripled by an increase in temperature of 10°C. Since A is constant for a given reaction
(iii) Concentration of reactants : At fixed temperature and in Subtracting (iii) from (iv)
absence of catalyst, the rate increases with increased
Ea E
concentration of reactants. lnk2 – lnk1 = - a
(iv) Surface area : The more the surface area, the more is the RT1 RT2
rate of reaction.
(v) Positive catalyst : Increases the rate by lowering the energy k Ea é T2 - T1 ù
log 2 = ê ú
of activation. k1 2.303R ë T1T2 û
(vi) Presence of light : Photochemical reactions are influenced
by radiations of specific wavelength. Knowing the value of slope Ea can be calculated.
THE RATE CONSTANT : (b) Catalyst : Presence of positive catalyst lowers the energy of
acitvation and hence influences the rate constant.
Consider a reaction A + B ¾ ¾® Products .
UNITS OF RATE CONSTANT :
At a particular temperature rate is given by
Units depend upon the order of a reaction
dx units = (Concentration)1 – n . time–1
r= µ C A .C B = kCA .C B
dt = (Mol L–1)1–n . time–1
where k is rate constant, velocity constant or specific reaction where n = order of reaction
rate. When CA = CB = 1 then k= r (rate) Units of rate constant for zero order reaction
In general rate constant may be defined as the rate of the
= Mol L–1 time–1
reaction when the concentration of each of the reactants is
unity. It is characteristic of a particular reaction. Units of rate constant for first order reaction = time–1
FACTORS INFLUENCING RATE CONSTANT : CHARACTERISTICS OF RATE CONSTANT :
(a) Temperature : Variation of rate constant with temperature (I) The value of k is different for different reactions.
is given by (II) At fixed temperature the value of k is constant.
Arrhenius equation : k = Ae - E a / RT (i) (III) It is independent of concentration but depends on
A = Constant known as frequency factor temperature.
Ea = Energy of activation. Both A and Ea are characteristic of (IV) The larger the value of k the faster is the reaction and vice
a particular reaction. versa.
e - E a / RT = known as Boltzmann factor
THE DIFFERENCE BETWEEN RATE CONSTANT AND
Taking natural logarithm of (i) RATE OF REACTION :
Rate Constant Rate of reaction
- Ea
lnk = + log A (ii) (a) It is equal to rate of (a) It is the change in
RT
reaction when the concentration of reactants or
1 concentration of reacting products per unit time.
The plot of lnk vs gives a straight line
T species is unity
(b) Units depends on order of –1 –1
Intercept (b) Units are Mol L Sec
–1 –n
reaction = (Mol L )1 time–1
–Ea (n = Order of reaction)
Slope =
lnk R (c) It depends upon (c) It depends upon
temperature temperature and concentration

1
T
Chemical Kinetics 597
TEMPERATURE COEFFICIENT : FEATURES OF ORDER OF REACTION :
It is the ratio of two rate constants differing by a temperature of (I) The order of a reaction is experimentally determined
10 °C. Generally the temperatures are 298K & 308 K. quantity.
(II) It cannot be written from balanced chemical equation.
Rate constant at 308K
Temperature Coefficient = (III) It can be written from the rate law equation.
Rate constant at 298K
(IV) It depends upon the molecules undergoing change in
The value of temperature coefficient is generally 2 and 3. In rare concentration.
cases the value is 4, 5 etc. (V) Order may be zero, whole number, fractional or negative
RATE LAW EQUATION : even
The mathematical expression, which practically relates the rate of (VI) Reaction with order ³ 3 are rare.
a chemical reaction and concentration of reactants is called rate RATE DETERMINING STEP :
law equation e.g. for a hypothetical reaction.
Some reactions take place in more than one step. Each step has
aA + bB cC + dD its own rate. The slowest step is called the rate determining step
Rate µ [A]a [B]b. It is law of mass action or rate controlling step e.g.
Rate µ [A]x [B]y It is rate law..
2 NO 2 + F2 ¾
¾® 2 NO 2 F
If rate actually vary according to this equation practically then
this is rate law equation. For decomposition of N2O5.
Mechanism NO 2 + F2 ¾
¾® NO 2 F + F (slow )
2N2O5 2N2O4 + O2
NO 2 + F ¾
¾® NO 2 F (fast )
Rate µ [ N 2 O 5 ] and not Rate µ [ N 2 O 5 ]2
\ Rate law equation is, Rate = k[N2O5] 2 NO 2 + F2 ¾
¾® 2 NO 2 F
Rate law equation for reversible reaction
First step is the rate determining step. The rate law equation is
k1 always written from slow step.
H2 + I2 2HI
k2 \ The rate law is given as rate = k [ NO2 ][ F2 ]
1 d[HI] Remember that reaction intermediate are never shown in rate law
Rate = = k1[H 2 ][I 2 ] - k 2 [HI]2
2 dt equation.

éRate of forwardù éRate of backwardù MOLECULARITY OF REACTION :


Rate = ê úû - êëreaction úû The number of molecules of reactants which take part in a single
ëreaction
actual step of the reaction, whether their concentration is
Rate law equation involving side reactions
appreciably changed or not is known as molecularity of reaction.
k1 227 Molecularity is a theoretical concept and is always a whole number.
227 Th
Ac 223 NH 4 NO 2 ¾
¾® N 2 + 2H 2 O Unimolecular
Fr
k2
2HI ¾
¾® H 2 + I 2 Bimolecular
Rate of fromation of Th 227
= k1[Ac 227
]
2 NO + O 2 ¾
¾® 2 NO 2 trimolecular
Rate of formation of Fr 223 = k 2 [Ac 227 ]
The reactions with molecularity more than 3 are rare and generally
\ Rate = (k1 + k 2 )[Ac 227
] not possible. Such reactions proceed through more than one
steps and are termed as complex reactions. Each step has its own
ORDER OF REACTION : molecularity e.g.
The sum of all the powers to which the concentration terms are
raised in rate law equation is known as the order of a reaction. For 4HBr + O 2 ¾
¾® 2H 2 O + 2Br2
a general reaction Probable mechanism is
aA + bB cC + dD
The rate law equation is HBr + O 2 ¾
¾® HOOBr Molecularity two

dx
( Rate) = [ A ] x [ B] y ; HOOBr + HBr ¾
¾® 2HOBr Molecularity two
dt
( HOBr + HBr ¾
¾® H 2 O + Br2 ) ´ 2 Molecularity two
Order w.r.t. A is x, Order w.r.t. B is y
Overall order is x + y
4HBr + O 2 ¾
¾® 2H 2 O + 2Br2 Molecularity five
598 Chemistry
PSEUDO UNIMOLECULAR REACTIONS : The half-life period of the first order reaction is independent
Reactions like hydrolysis of an ester or cane sugar, which though of the initial concentration of reacting substances.
bimolecular and yet following the kinetics of first order are called The time taken for the completion of any fraction of the
pseudo unimolecular reactions. reaction is independent of the initial concentration.
First order growth kinetics : It is used in population growth
CH 3COOC 2 H 5 + H 2 O CH 3COOH + C 2 H 5 OH
and bacteria multiplication
dx
= k[CH 3 COOC 2 H 5 ] 2.303 a+x
k= log
dt t a
C12 H 22 O11 + H 2 O C 6 H12 O 6 + C 6 H12 O 6 where a is initial population and (a + x) population after time
t.
dx (2) Second order reaction : The reaction rate is determined by
= k[C12 H 22 O11 ] .
dt the variation of two concentration terms. They are of two
In above reactions the rate is independent of concentration types
of H2O being present in excess. Hence [H2O] is constant. (a) 2A ¾
¾® Products
DIFFERENCES BETWEEN ORDER AND
MOLECULARITY OF REACTION : Rate µ [A]2

Order of reaction Molecularity (b) A + B ¾


¾® Products
1. It is experimentally It is a theoretical concept. Rate µ [A][B]
determined quantity
2. It can have integral, Always integral values only, 1 x
For the (a) type of reaction k = .
fractional or negative never zero or negative t a(a - x)
values Unit of k = conc –1 time –1 = (mol L –1 ) –1 time –1
3. It cannot be obtained It can be obtained. i.e. mol–1 L. s–1
from balanced or
stoichiometric equation. When t = t1 / 2 , x = a / 2 .
4. It tells about the slowest It does not tell anything
1 æ 1ö
step in the mechanism about mechanism Half-life period (t1/2) = ç t1 / 2 µ ÷
ka è aø
5. It is sum of the powers It is the number of reacting
of the concentration species undergoing The time taken for the completion of half of the reaction is
terms in the rate law simultaneous collision in the inversely proportional to the initial concentration of the
equation. reaction. reactant.
6. It is changed when one It undergoes no change. Note: The time taken for the completion of half or any fraction
reactant is taken in of the reaction is inversely proportional to the initial
large excess concentration of the reactant.
KINETIC EQUATIONS OF DIFFERENT ORDERS : (3) Third order reaction : The reaction rate is determined by
(1) First order reaction : The reaction rate is determined by the variation of three concentration terms. They are of three
one concentration variable term. types
A ¾ ¾® products (a) 3A ¾
¾® Products
dA Rate µ [A ]3
= k[A]
dt
Let 'a' moles per unit volume be the initial concentration and (b) 2A + B ¾
¾® Products
x moles have disappeared after time t. Then the concentration
left will be (a – x) moles per unit volume. Rate µ [A]2 [B]

1 a 2.303 a (c) A + B + C ¾
¾® Products
k = .ln = log
t (a - x) t a-x Rate µ [A][B][C]
This is called the kinetic equation of a first order reaction.
Units of k = time–1 eg h–1, m–1 or s–1 x ( 2a - x )
Consider the type (a) k =
Half-life period (t1/2). The time taken for the completion of 2 t a 2 (a - x ) 2
half of the reaction is known as half-life period. When Unit of k = (mol L–1)–2 s–1 = mol–2 L2 s–1
t = t1 / 2 , x = a / 2 . When t = t1/2, x = a/2
0.693 3 1
t1/ 2 = Half-life period (t1/2) = (Q t1/ 2 µ )
2
k 2ka a2
Chemical Kinetics 599
The time taken for the completion of half of the reaction is (II) Conversion of para hydrogen to ortho hydrogen at high
inversely proportional to the square of the initial temperature.
concentration of reactants.
Rate µ (PH )3/2
Note :The time taken for the completion of any fraction of 2
the reaction is inversely proportional to the square of the
initial concentration of reactants. \ Rate = k(PH 2 )1.5 . Order w.r.t. H2 is 3/2.
(4) Zero order reaction : The reaction rate is independent of NEGATIVE ORDER REACTIONS :
the concentration of the reactants.
Conversion of ozone into oxygen.

¾® Product Rate = K [O3]2 [O2]–1
Order with respect to oxygen is –1.
dx
Rate = k[A]0 or = k on integration DETERMINATION OF ORDER OF REACTION :
dt
(1) Fractional change method : The time required to complete
x = kt + c
a definite fraction of reaction is given by
when t = 0, x = 0 so c = 0
t1 / 2 µ (a)1- n where n is order of reaction
x
Hence x = kt or k = The following graphs are obtained between t1/2 and initial
t
concentration 'a'.
Unit of k = Conc. time–1 = mol L–1 s–1
When the reaction is complete, C = 0
Co
\ E completion = I. t1/2 II. t1/2
k
Co - C
For zero order reaction k = a
t a
Zero order t1/2 µ a First order t1/2 µ a0
Co = initial concentiation
C = conc. at any time t
III.
When t = t1 / 2 , x = a / 2
t1/2
a
Half life period (t1/2) = (t µ a )
2k 1 / 2
n–1
The time taken for the completion of half of the reaction is a
directly proportional to the initial concentration of reactants. Second & Third Order
For zero order reaction, where n = 2 & 3
(I) Rate is independent of concentration of reactant (s) 1 1
(II) Concentration of reactants do not vary with time t1 / 2 µ
and t1 / 2 µ
a a2
(III) Rate does not vary with time. (2) Graphical method : (a) The graphs between concentration
(IV) Rate is always equal to rate constant. of any reactant Vs. time or concentration of any product Vs.
(V) t ½ < a time are plotted. The slope of the tangent to the curve gives
(VI) If we double the amount of reactants then time for the rate of reaction at that time.
completion will be doubled.
HALF-LIFE PERIOD FOR THE nTH ORDER Product
REACTION : dx
When the order of reaction is n, t1/2 is given by dt
Concentration
1 é 2n -1 - 1 ù
t1/ 2 = ê ú
k(n - 1) êë a n -1 úû Reactant
dx
1 dt
\ t1 / 2 µ
n -1
a time
Unit of k = (conc)1–n. time–1
FRACTIONAL ORDER REACTIONS : dx
The new graphs between Vs. (a – x), (a – x)2 or
(I) Reaction between H2 and D2. dt
Rate = k PH (PD )1/2. Order w.r.t. D2 is 1/2. (a – x)3 are plotted.
2 2
600 Chemistry

1
For II order slope = ; k can be calculated
Zero order k
dx dx
dt dt 1
For III order slope = ; k can be calculated
I order 2k
(3) Use of differential rate equations.
(a – x) (a – x)
(4) Use of integral rate equations – hit and trial method
(5) Ostwald's isolation method.
Examples of reactions of different order :
(a) Reactions of zero order :
dx dx
hn
dt dt (I) H 2 (g ) + Cl 2 (g ) ¾¾® 2HCl (g )
II order III order
MO
(II) 2NH 3 (g) ¾¾¾
® N 2 (g) + 3H 2 (g)
hn
2 3
(a – x) (a – x) Au
(III) 2HI(g) ¾¾¾ ® H 2 (g) + I 2 (g)
(b) Making use of integrated form of rate expression : (b) Reactions of Ist order :
(I) 2 N 2O5 ¾
¾® 4 NO 2 + O 2

(II) 2H2 O2 ¾¾
® 2H2 O + O2
t t I order
Zero order
(III) NH 4 NO 2 ¾
¾® N 2 + 2H 2 O

x Log (a – x) (IV) SO 2 Cl 2 ¾
¾® SO 2 + Cl 2
x
(V) (CH 3 ) 3 CO . OC(CH 3 ) 3 ¾¾® 2CH 3COCH 3 + C 2 H 6
x
Zero order t = COOH
k
(VI) O2N NO2
a
For first order kt = 2.303 log
a-x
2.303 2.303
or t= log a - log(a - x) NO2
k k
O2N NO2
+ CO2

t II order t III order NO2

(VII) CH 3COOC 2 H 5 + H 2 O ¾¾® CH 3COOH + C 2 H 5OH


1 1
(a - x ) (a - x ) 2 (VIII) C12 H 22 O11 + H 2 O ¾¾® C 6 H12 O 6 + C 6 H12 O 6

(IX) (CH3 ) 2 CHN = NCH(CH 3 ) 2 ¾


¾® N 2 + C 6 H14
azo isopropane
x 1 x(2a - x)
2nd order kt = 3rd order kt = (c) Reactions of II order :
a(a - x) ; 2 a 2 (a - x) 2
1 1 1 1 (I) 2 NO 2 + F2 ¾
¾® 2 NO 2 F
t= - t= -
k(a - x) k ( a ) - 2k ( a )
2 2
2k(a x)
(II) 2HI ¾
¾® H 2 + I 2
Evaluation of k
III) 2CH 3CHO ¾
¾® 2CH 4 + 2CO
1
For zero order slope = (IV) CH 3COOC 2 H 5 + NaOH ¾
k ¾®

-2.303 ® CH 3COONa + C 2 H 5 OH
For I order slope = ; k can be calculated
k
(V) NO + O 3 ¾
¾® NO 2 + O 2
Chemical Kinetics 601

(VI) 2 NO 2 ¾ COLLISION FREQUENCY (z) :


¾® 2 NO + O 2
Total number of collisions which occur among the reacting
(VII) C 2 H 4 + H 2 ¾
¾® C 2 H 6 molecules per second per unit volume is called collision
frequency. Its value is given by
(VIII) 2 N 2 O ¾
¾® 2 N 2 + O 2
z= 2 p n s2 n 2
(IX) S2 O82 -
+ 2I -
® 2SO 42 -
¾¾ + I2
(d) Reactions of III order : n = average velocity, s = molecular diameter in cm.,
n = number of molecules per cc.
(I) 2 NO + O 2 ¾
¾® 2 NO 2
RATE OF REACTION FROM COLLISION THEORY :
(II) 2 NO + Br2 ¾
¾® 2 NOBr It is given by
Rate of reaction = f × z
(III) 2 NO + Cl 2 ¾
¾® 2 NOCl
z = collision frequency, f = fraction of effective collisions
(IV) 2 NO + H 2 ¾
¾® N 2 O + H 2 O
Dn
= = e - E a / RT
(V) 2 NO + D 2 ¾
¾® N 2 O + D 2 O N

(VI) 2FeCl 3 + SnCl 2 ¾


¾® 2 FeCl 2 + SnCl 4 \ Rate (k) = Ze - E a /RT

(VII) Fe 3+ + 3I - ¾
¾® FeI 3
GRAPHICAL REPRESENTATION OF EXOTHERMIC
REACTION :
(e) Reactions of fractional order :
(I) H 2 + Br2 ¾
¾® 2HBr A(Reactant) ® B (Product)
E a(F.R.) + DH = E a(B.R.)
Rate = k[H 2 ][Br2 ]1/ 2 Order = 1.5
DH = - ve
(II) CO + Cl 2 ¾
¾® COCl 2 where F.R. = Forward reaction and B.R. = Backward reaction
Rate = k[CO] [Cl 2 ]
2 1/ 2
Order = 2.5
Threshold energy
(III) COCl 2 ¾
¾® CO + Cl 2

Rate = k[COCl 2 ]3 / 2 Order = 1.5


(f) Reaction of IV order : Ea(FR) Ea’ (B.R.)
Ea
4KClO 3 ¾
¾® 3KClO 4 + KCl
DH
COLLISION THEORY : B
Chemical reaction occurs as a result of effective collisions
between reacting molecules. For this two things are important. Progress of Reaction
(I) Proper orientation of reacting molecules.
(II) Possession of certain minimum amount of energy by reacting
GRAPHICAL REPRESENTATION OF ENDOTHERMIC
molecules
REACTION :
THRESHOLD ENERGY :
The minimum amount of energy possessed by the reacting A(Reactant) ® B (Product)
molecules to have effective collisions, resulting in the formation DH = + ve
of product, is called threshold energy. E a(F.R.) = DH + E 'a(B.R.)
ACTIVATION ENERGY (Ea) :
Threshold energy
The excess energy over and above the average potential energy
possessed by reacting molecules to have effective collisions
resulting in the formation of product is known as activation Ea(FR) Ea’ (B.R.)
energy.
Activation Energy = threshold energy B
Ea DH
– average energy of reactants
For fast reaction the activation energy is low.
For slow reaction the activation energy is high.
Progress of Reaction
602 Chemistry
GRAPHICAL REPRESENTATION OF THE EFFECT OF QUANTUM EFFICIENCY OR QUANTUM YIELD :
TEMPERATURE ON RATE OF REACTION : Number of moles reacting per Einstein of the light absorbed. It is
Fraction of the molecules having energy equal to activation expressed as
energy is shown by shaded portion. The fraction of such Number of molecules reacting in a given time
molecules become almost double for 10°C rise of temperature f=
Number of quanta of light absorbed in the same time
and the rate of reaction almost doubles for 10°C rise of
temperature. Number of moles reacting in a given time
f=
Number of Einsteins absorbed in the same time
T1 T2>T1
CHEMILUMINESCENE :
T2 Emission of light in a chemical reaction at ordinary temperature is
DN called chemiluminescence.
N
FLUORESCENCE :
The absorption of energy and instantaneous re-emitting of the
energy is called fluorescence.
Energy
PHOSPHORESCENCE :
The continuous glow of some substances even after the cutting
PHOTOCHEMICAL REACTIONS : of source of light is called phosphorescene e.g. ZnS.
Reactions which take place by the absorption of radiations of BIOLUMINESCENCE :
suitable wavelength e.g. The phenomenon of chemiluminescence exhibited by certain
light living organisms is called bioluminescence e.g. light emission by
H 2 (g ) + Cl 2 (g ) ¾¾¾® 2HCl (g )
fire flies.
Photosynthesis of carbohydrates in plants takes place in presence
EFFECT OF CATALYST ON REACTION RATES :
of chlorophyll and sunlight.
A catalysed reaction provides a new pathway or a mechanism by
light
6CO 2 + 6H 2 O ¾¾¾® C 6 H12 O 6 + 6O 2 which the potential energy barrier between the reactants and
products is lowered. The graphical representation is as follows :
FREE ENEREGY CHANGE IN PHOTOCHEMICAL
REACTIONS : Activated Complex
The free energy change of a photochemical reaction may not be
negative. In the synthesis of carbohydrates and formation of Ea Ea
HCl, DG is +ve.
PHOTOSENSITISATION : Ea(c)
E
Certain molecules absorb light energy and transfer it to another Ea (c)
molecule which may undergo a reaction. The process is called Reactants
DH
photosensitisation and the substance doing so is called
photosensitizer e.g. chlorophyll acts as photosensitizer in Products
photosynthesis. Progress of Reaction
VISION : (I) DH (enthalpy) of reaction remains the same for catalysed as
The isomerisation of retina by absorbing photon of light in the well as uncatalysed reaction.
retina of eye and reconversion to original form by releasing energy (II) Energy of activation is lowered considerably for catalysed
is known as vision. forward and backward reactions.
Study of kinetics of some reactions :
GROTHUS-DRAPER LAW :
(I) Hydrolysis of an ester
Only those rays which are absorbed by the substance are effective
CH3COOC2H5 + H2O CH3COOH + C2H5OH
in producing a chemical change. A part of light is reflected and
transmittted also. It is biomolecular and 1 order. The rate constant is given by

LAW OF PHOTOCHEMICAL EQUIVALENCE OR 2.303 V - v0


k= log ¥
EINSTEIN'S LAW (STARK-EINSTEIN LAW) : t V¥ - v t
In a photochemical reaction, one quantum of active light (photon) Initial conc. of ester µ V¥ - v 0 and conc. of ester present
is absorbed per molecule of the reacting substance which
disappears. after time t µ V¥ - v t .
Chemical Kinetics 603
(II) Inversion of cane sugar : Hence velocity constant k is given by
C12 H 22 O11 + H 2 O C 6 H12 O 6 + C 6 H12 O 6 2.303 (r - r )
k= log 0 ¥
Sucrose (d) Glucose (d) Fructose (l) t (rt - r¥ )
Bimolecular I order Laevorotatory (III) Saponification of ethyl acetate :
The change is noted by rotation of plane polarised light in a
polarimeter. CH 3COOC 2 H 5 + NaOH ¾
¾® CH 3COONa + C 2 H 5 OH

a µ r0 - r¥ Bimolecular II order reaction.

(a - x ) µ rt - r¥ a
The value of k = t.a (a - x)

Pressure due to a moles = 114 mm Hg


1. The reaction 2N 2 O5 (g) ¾¾
® 4NO 2 (g) + O 2 (g) takes
x
place in a closed flask. The concentration of NO2 increases Pressure due to a + moles =133 mm Hg
by 20 × 10–3 moles lit–1 in 5 sec. 2
(a) Calculate the rate of the reaction (b) the rate of change of x
concentration of N2O5. Hence pressure due to moles
2
Sol. (a) Rate of reaction
= 133 – 114 = 19 mm Hg
1 dNO 2 1 20 ´10 -3 and due to x moles 19 × 2 = 38 mm Hg.
= . = . = 10 -3 mol. lit -1s -1
4 dt 4 5 \ Pressure due to (a – x) moles = 114 – 38 = 76 mm Hg
(b) Rate of change of concentration of N2O5 Change in pressure 114 – 76 = 38 mm Hg.
38 -1
1 dN 2 O5 1 dN 2 O5 = 0.002 atm min
; 10 -3 =
Average rate =
r=- 25 ´ 760
2 dt 2 dt
Again PV = nRT.
= 2.0 ´ 10-3 mol lit -1s -1
n 0.002
2. A first order reaction is 20% complete in 10 minutes. Calculate \ = = 8.58 ´ 10 - 7 mol lit -1s -1
V 0.0821´ 473 ´ 60
(I) the rate constant of the reaction (II) time taken for the
reaction to go to 75% completion. 4. The rate constant of a reaction is given by lnk (sec–1)
= 14.34 – (1.25 × 104)/T. Calculate (a) the energy of activation
2.303 a
Sol. (I) For Ist order reaction k = log (b) the rate constant at 500 K (c) At what temperature will its
t a-x half-life period be 256 minute.
Sol. (a) Given ln k = 14.34 – (1.25 × 104)/T.
2.303 100
k= log = 0.0223 min -1
10 100 - 20 Ea
Arrhenius equation ln k = ln A –
RT
2.303 100
(II) t = log = 62.18 min
0.0223 100 - 75 Ea
Comparing both the equations = 1.25 ´ 104
R
3. The decomposition of 2 N 2 O 5 ¾
¾® 2 N 2 O 4 + O 2 is at
200°C. If the initial pressure is 114 mm Hg and after 25 min.
\ E a = 1.25 ´ 104 R = 1.25 ´104 ´1.987 = 24.83 kcal mol -1
of the reaction the total pressure of gaseous mixture is 133
(1.25 ´ 104 )
mm Hg. Calculate the average rate of the reaction in (b) ln k = 14.34 - (Q T = 500K)
(a) atm. m–1 (b) mol s–1 500

Sol. 2 N 2 O 5 ¾
¾® 2 N 2 O 4 + O 2 \ k = 2.35 ´10-5 sec-1
a -x x x/2
604 Chemistry

0.693 0.693 1.25 ´ 10 4 3 ´10 -3 [0.15]m [0.05]n


(c) k = \ log = 14.34 - = \ 3 = (3) m \ m =1
256 ´ 60 256 ´ 60 T 1´10 -3 [0.05]m [0.05]n
\ T = 513K Order with respect to Cl2 is 1.
Divide III by I,
5. The rate constant of reaction is 1.5 ´ 10 7 s -1 at 50°C and
4.5×107 s–1 at 100°C. Evaluate Arrhenius parameter and Ea. 9 ´ 10 -3 [0.05]m [0.15]n
= \ 9 = (3) n \n = 2
-3
k2 E a (T2 - T1 ) 1´ 10 [0.05] m
[0.05] n

Sol. log k = 2.303R T T Order with respect to NO is 2.


1 1 2
(b) Rate = k [Cl2] [NO]2
4.5 ´ 10 7 Ea (373 - 323) (c) Rate constant, 1 ´ 10-3 = k[0.05][0.05]2
log =
1.5 ´ 10 7 2.303 ´ 8.314 373 ´ 323
\ k = 8 lit2 mol–2 sec–1
E a = 2.2 ´ 104 J mol-1 (d)Rate = k [Cl2] [NO]2 = 8 [0.2] [0.4]2
= 0.256 mol lit–1 time–1
Again k = Ae–Ea/RT 8. In oxidation of oxalic acid when 2 ml of
4
\ 4.5 ´ 107 = Ae -2.2´10 /8.314´373 ( COOH ) 2 ¾
¾® CO + CO 2 + H 2 O
solution were withdrawn and titrated against standard
\ A = 5.42 ´ 1010 solution of KMnO4, the following results were obtained.
6. The rate constant for an isomerisatrion reaction A ® B is Time Vol. of KMnO4 (in ml) required for
4.5 × 10 –3 min–1. If the initial concentration of A is the titration of oxalic acid
1 M. Calculate the rate of reaction after 1 hour. 0 22.0
2.303 a 300 17.0
Sol. For I order reaction k = log 450 15.0
t a-x
600 13.4
2.303 1
4.5 ´ 10 -3 = log Show that the reaction is of first order.
60 a-x
2.303 22
Sol. k = log = 8.6 ´ 10 -4
\ (a - x ) = 0.7634 300 17
Then the rate after 60 minutes = k(a – x)
2.303 22
= 4.5 × 10–3 × 0.7634 = 3.4354 × 10–3 M min–1 k= log = 8.34 ´ 10 -4
7. The data given below are for the rection NO and Cl2 to form 450 15
NOCl at 295 K Since the value of k is the same hence the reaction is of Ist
[Cl2] [NO] Initial rate mol L–1 Sec–1 order.
0.05 0.05 1 × 10–3 9. A first order reaction has a specific reaction rate of
0.15 0.05 3 × 10–3 2.31 × 10–3 s–1. Calculate t1/2.
0.05 0.15 9 × 10–3 0.693 0.693
Sol. t1/ 2 = = = 300 sec
(a) What is the order with respect to NO and Cl2 in the k 2.31 ´ 10-3
reaction.
10. Show that the time required for the completion of
(b) Write rate expression
3/4th of reaction of Ist order is twice the time required for
(c) Calculate the rate constant. the completion of 1/2 of the reaction.
(d) Determine the rate when [Cl2] = 0.2 M
and [NO] = 0.4 M. 2.303 1 2.303
Sol. t 3/ 4 = log = log 4
Sol. The reaction is 2 NO + Cl 2 ¾ k (1 - 3 / 4) k
¾® 2 NOCl

\ Rate = k[Cl 2 ]m [NO]n 2.303 1 2.303


t1/ 2 = log = log 2
k (1 - 1/ 2) k
1´10 -3 = [0.05]m [0.05]n (I)
t 3 / 4 log 4
-3 = ; t 3 / 4 = 2t1 / 2
3 ´ 10 = [0.15] [0.05]
m n (II) t1 / 2 log 2

9 ´10 -3 = [0.05]m [0.15]n (III) 11. When the initial concentration is changed from 0.50 to 1.0
mole lit–1, the time of half completion for a certain reaction is
Divide II by I, found to change from 50 sec. to 25 sec. Calculate the order
of reaction.
Chemical Kinetics 605

n -1
Sol. k = 2.303 log a
Sol. T1 = æç a 2 ö÷ t a-x
T2 çè a1 ÷ø
2.303 5.0
n -1 6= log
50 æ 1.0 ö t 0.05
=ç ÷ 2 = (2)n – 1 or 21 = (2)n – 1
25 è 0.5 ø 2.303
t= log100 = 46.06 sec.
\ 1 = n – 1. Hence n = 1 + 1 = 2. Reaction is of second order. 6 min -1
12. Find the 3/4 th life (t 3/4) of a first order reaction. The rate
16. For the reaction, 2A + B + C ® A 2 B + C , rate law has been
constant k = 7.4 ´ 10 -5 sec-1
determined to be rate = k[A][B] 2. If the rate constant
2.303 a 2.303 1 k = 2.0 ´ 10 -6 mol L sec , what will be initial rate
–2 2 –1
Sol. t 3 = log = log
4
k a-x k 1-
3 of the reaction with [A] = 0.1 mol L–1 and [B] = 0.2 mol L–1
4 and [C] = 0.8 mol L–1.
2.303 Sol. For 2A + B + C ® A 2 B + C
t3 = log 4 = 1.84 ´104 sec
4
7.4 ´ 10-5 Rate = k[A][B]2

13. For a reaction it takes 10 min. for the initial concentration of = 2.0 ´10 -6 ´ 0.1´ [0.2]2
2.0 mol L–1 to become 1.0 mol L–1 and another 10 min to
become 0.50 mol L–1. Calculate the rate constant of the = 8.0 ´ 10 -9 mol L-1 s -1
reaction. 17. At some temperature, the rate constant for
Sol. The time taken for the concentration to fall to half the decomposition of HI on the surface of gold is 0.08 mol L –
1 s–1
concentration is constant in both the cases. Hence the
reaction is of first order 2HI (g) ® H 2 (g ) + I 2 (g)
0.693 0.693 what is the order of the reaction. How long will it
\ k= = = 0.0693 min -1
t½ 10 take for the concentration of HI to drop from 1.50 M to 0.30
M.
14. The rate of reaction 2 NO + Cl 2 ® 2 NOCl is doubled when
Sol. From the unit of k, the reaction is of zero order.
concentration of Cl2 is doubled and it becomes 8 times
when concentration of both NO and Cl2 are doubled. Deduce C0 - C
k=
the order of reaction. 2t
Sol. Rate is doubled when [Cl2] is doubled
1.5 - 0.30 1.2
Rate w.r.t. Cl2 = k[Cl2]1 0.08 = or t=
2t 0.08 ´ 2
Rate becomes 8 times when concentration of both NO and
Cl2 is doubled \ t = 7.5 s
Rate = k[Cl2 ][NO]x 18. The following data were obtained at a certain temperature
for the decomposition of ammonia in contact with tungsten
8 = k[2][2]x p (mm Hg) 50 100 200
\x=2 t½ 3.64 1.82 0.90
Find the order of the reaction.
Rate law is Rate = K[Cl 2 ][ NO]2
n -1 n -1
( t½)1 æ a1 ö 3.64 æ 100 ö
Total order 1 + 2 = 3 Sol. =ç ÷ or =ç ÷
15. A reaction that is of the first order with respect to ( t½) 2 çè a 2 ÷ø 1.82 è 50 ø
reactant A has a rate constant of 6 min–1. If we start with [A]
= 5.0 mol L–1 when would [A] reach the value of 0.05 mol L–1. 2 = (2) n -1 or n -1 = 1
\n = 2
606 Chemistry

Very Short/Short Answer Questions [ N 2O5 ] ( M ) Time ( min )


1 Hydrolysis of ethyl acetate with KOH is a second order 0.400 0.00
reaction while with HCl, it is first order reaction., Why ?
0.289 20.0
2. A reaction, P + Q ¾¾ ® products, has rate law equation,
1/3 3
rate = k [P] [Q] . Suggest the suitable changes in initial 0.209 40.0
concentration of P and Q so as to make the initial rate 16 0.151 60.0
times. 0.109 80.0
3. For the homogeneous decomposition of N2O5 into NO2
(a) Calculate the rate constant. Include units with your an-
and O2
swer.
N2O5 (g) ¾¾ ® 4NO2 (g) + O2 (g)
(b) What will be the concentration of N 2O5 after 100
1 d [N2 O5 ] minutes?
Rate = – = k [N2 O5 ]
2 dt (c) Calculate the initial rate of reaction.
Find out the order of reaction with respect to N2O5. 11. (a) A reaction is first order in A and second order in B.
4. Give an expression for time required to complete the nth (i) Write differential rate equation.
fraction of a first order reaction. (ii) How is rate affected when concentration of B is tripled?
5. For the reaction, 2A + 2B ¾¾ ® Products, on doubling the (iii) How is rate affected when concentration of both A and
[A] and keeping the [B] constant, the reaction rate increases B is doubled?
by four times. On doubling the [B] and keeping the [A] (b) What is molecularity of a reaction?
constant, the reaction rate increases by two times. Write the 12. (a) For a reaction A + B ® P, the rate law is given by,
rate law.
r = k [A] [ B]2
1/2
6. The reaction, 2NO(g) + O2 (g) ¾¾ ® 2NO2 (g) and
2CO (g) + O2 (g) ¾¾ ® 2CO2 (g) What is the order of this reaction?
look to be similar. Let the former is faster than the latter at (b) A first order reaction is found to have a rate constant
the same temperature. Explain why. k = 5.5 × 10–14 s–1, Find the half life of the reaction.
7. In general, it is observed that the rate of a chemical reaction
doubles with every 10° rise in temperature. If this Long Answer Questions
generalisation holds good for the reaction in the temperature
13. The Arrhenius equation is k = k¥ exp. (–Ea/RT). The energy
range 295 K to 305 K, then what would be the value of
of activation, Ea for two reactions X and Y are 100 kJ mol –1
activation energy for this reaction ? (R = 8×314 JK–1 mol–1).
and 125 kJ mol–1 respectively. At 227 °C, the rate constant kx
8. The catalytic decomposition of hydrogen peroxide was
and ky are equal. Draw systematic plots of log k versus 1/T,
studied by titrating it at different intervals with KMnO4.
covering a temperature range below and above 227 °C.
Calculate the rate constant from the following data,
14. (a) With the help of a labelled diagram explain the role of
assuming the reaction to be of first order:
activated complex in a reaction.
t (sec) 0 600 1200
(b) A first order reaction is 15% completed in 20 minutes.
KMnO4 (mL) 22×8 13×8 8×2
How long will it take complete 60% of the reaction?
9. The decomposition of N2O5 in CCl4 solution follows the
15. (a) What is the physical significance of energy of
first order rate law. The concentrations of N2O5 measured at
activation? Explain with diagram
different time internals are given below: Find the rate constant
(b) In general, it is observed that the rate of a chemical
for decomposition.
reaction doubles with every 10 degree rise in tempera-
Time in ture. If the generalization holds good for the reaction in
0 80 160 410 600 1130 1720 the temperature range of 295 K to 305 K, what would be
seconds (t)
the value of activation energy for this reaction?
[N2O5] in
5.5 5.0 4.8 4.0 3.4 2.4 1.6 [R = 8.314 J mol–1 K–1]
mol/L
16. (a) A reaction is second order in A and first order in B.
10. Nitrogen pentoxide decomposes according to equation: (i) Write the differential rate equation.
2N 2O5 ( g ) ® 4NO2 ( g ) + O2 ( g ) (ii) How is the rate affected on increasing the concentra-
tion of A three times?
This first order reaction was allowed to proceed at 40°C and
(iii) How is the rate affected when the concentration of both
the data below were collected:
A and B are doubled?
Chemical Kinetics 607
(b) A first order reaction takes 40 minutes for 30% decom- 20. Order of reaction can be
position. Calculate t1/2 for this reaction. (a) 0 (b) fraction
(Given log 1.428 = 0.1548) (c) whole number (d) integer, fraction, zero
17. (a) For a first order reaction, show that time required for 21. The rate of reaction between two reactants A and B
99% copletion is twice the time required for the comple- decreases by a factor of 4 if the concentration of reactant
tion of 90% of reaction. B is doubled. The order of this reaction with respect to
(b) Rate constant ‘k’ of a reaction varies with temperature reactant B is:
‘T’ according to the equation: (a) 2 (b) -2
Ea æ 1 ö (c) 1 (d) -1
log k = log A - ç ÷ 22. The rate of reaction between A and B increases by a factor
2.303R è T ø
of 100, when the concentration of A is increased 10 folds,
Where Ea is the activation energy. When a graph is
the order of reaction with respect to A is
1 (a) 10 (b) 1
plotted for log k Vs. , a straight line with a slope of
T (c) 4 (d) 2
– 4250 K is obtained. Calculate ‘Ea’ for the reaction. ˆˆ† 2HBr(g) , the rate law
23. For the reaction H 2 (g) + Br2 (g) ‡ˆˆ
(R = 8.314 JK–1mol–1)
is rate = k [H 2 ][Br2 ]1/ 2 . Which of the following statement
Multiple Choice Questions is true about this reaction
18. The rate of a reaction does not depend upon (a) The reaction is a second order one
(a) Temperature (b) Concentration (b) Molecularity of the reaction is 3/2
(c) Catalyst (d) None of these (c) The unit of k is s–1
19. Which of the following statements is incorrect? (d) Molecularity of the reaction is 2
(a) Activation energy for the forward reaction equals 24. The following data are for the decomposition of ammonium
activation energy for the reverse reaction nitrite in aqueous solution :
(b) For a reversible reaction, an increase in temperature Vol. of N2 in cc Time (min)
increases the reaction rate for both the forward and 6.25 10
the backward reaction 9.00 15
(c) The larger the initial reactant concentration for a 11.40 20
second order reaction, the shorter its 13.65 25
half-life. 35.65 Infinity
(d) When Dt is infinitesimally small, the average rate equals The order of rection is :
the instantaneous rate (a) zero (b) one
(c) two (d) three

1. In the reversible reaction (c) temperature effects


k1 (d) solvent effects
2 NO2 N2O4,
k2 +d[B]
3. 3A ¾
¾® 2B , rate of reaction, is equal to
the rate of disappearance of NO2 is equal to dt

2k1 3 d[ A] 2 d[ A]
(a) [ NO 2 ]2 (a) - (b) -
k2 2 dt 3 dt

(b) 2k1[ NO 2 ]2 - 2k 2 [ N 2 O 4 ] 1 d[ A ] d[ A ]
(c) - (d) + 2
3 dt dt
(c) 2k1[ NO 2 ]2 - k 2 [ N 2 O 4 ] 4. Consider the chemical reaction,
(d) (2k1 – k2) [NO2]
N 2 (g ) + 3H 2 (g ) ¾¾® 2 NH 3 (g ) . The rate of this reaction
2. The rate of the reaction intermediates can be determined by
the study of can be expressed in terms of time derivative of concentration
of N2 (g), H2 (g) or NH3 (g). Identify the correct relationship
(a) catalyst effects
amongst the rate expressions
(b) concentration of the reactants
608 Chemistry
13. In Arrhenius plot, intercept is equal to
-d[ N 2 ] -1 d[H 2 ] 1 d[ NH 3 ]
(a) Rate = = =
dt 3 dt 2 dt Ea
(a) - (b) ln A
R
-d[ N 2 ] d[ H 2 ] d[ NH 3 ]
(b) Rate = = -3 =2 (c) ln k (b) log10a
dt dt dt
14. A chemical reaction was carried out at 300 K and 280 K. The
d[ N 2 ] 1 d[H 2 ] 1 d[ NH 3 ] rate constants were found to be k1 and k2 respectively. then
(c) Rate = = =
dt 3 dt 2 dt (a) k1 = 4k1 (b) k2 = 2k1
(c) k2 = 0.25 k1 (d) k2 = 0.5 k1
d[ N 2 ] d[ H 2 ] d[ NH 3 ]
(d) Rate = - =- = 15. The decomposition of a substance follows first order kinetics.
dt dt dt
Its concentration is reduced to 1/8th of its initial value in 24
5. The rate of a gaseous reaction is given by the expression minutes. The rate constant of the decomposition process is
k(A)(B). If the volume of the reaction vessel is suddenly
reduced to 1/4 of the initial vol. the reaction rate relative to 0.692
(a) 1/24 min–1 (b) min -1
the original rate will be 24
1 1
2.303 æ1ö 2.303 æ8ö
log ç ÷ min -1 log ç ÷ min -1
(a) (b)
16 8 (c) (d)
24 è8ø 24 è1ø
(c) 8 (d) 16
6. The units for the rate constant of first order reaction is 16. The rate of chemical reaction is doubled for every 10°C rise
in temperature because of
(a) s –1 (b) mol L–1 s–1
(c) mol s –1 (d) L mol–1 s–1 (a) increase in the activation energy
7. The rate constant of reaction depends upon (b) decrease in the activation energy
(a) temperature (b) pressure (c) increase in the number of molecular collisions
(c) volume (d) All the these (d) increase in the number of activated molecules
8. The rate constant of a reaction is 10.8 × 10 –5 17. The temperature coefficient of most of the reactions lies
mol dm–3 s–1. The order of the reaction is between
(a) zero (b) 1 (a) 1 and 3 (b) 2 and 3
(c) 2 (d) 3 (c) 1 and 4 (d) 2 and 4
9. If concentration of reactants is increased by 'x', then k 18. The velocity of a reaction is doubled for every 10°C rise in
becomes temp. If the temp. is raised to 50°C the reaction velocity
k k increases by about
(a) ln (b) (a) 12 times (b) 16 times
x x
(c) k + x (d) k (c) 32 times (d) 50 times
10. Rate constant in the case of first order reaction is
19. For the reaction A ¾ ¾® B , the rate law expression is : rate
(a) inversely proportional to the concentration units
= k [A]. Which of the following statements is incorrect?
(b) independent of concentration units
(a) The reaction follows first order kinetics
(c) directly proportional to the concentration units
(b) The t1/2 of reaction depends on initial concentration of
(d) inversely proportional to the square of the concentration
reactants
units
11. The Arrhenius equation expressing the effect of temperature (c) k is constant for the reaction at a constant temperature
on the rate constant of the reaction is (d) The rate law provides a simple way of predicting the
conc. of reactants and products at any time after the
Ea
(a) k = e - E a / RT (b) k = start of the reaction.
RT
20. For a reaction of type A + B ¾¾® products , it is observed
Ea
(c) k = log e (d) k = Ae - E a / RT that doubling concentration of A causes the reaction rate to
RT
be four times, but doubling amount of B does not affect the
12. For the following homogeneous reaction, rate. The rate equation is
k
A + B ¾¾ ®C k
(a) Rate = k [A] [B] (b) Rate = [A]2
the unit of rate constant is 4
(a) sec –1 (b) sec–1 mol L–1 (c) Rate = k [A]2 [B] (d) Rate = k [A]2 [B]2
(c) sec–1 mol–1 L (d) sec–1 mol–2 L2
Chemical Kinetics 609
21. Select the rate law that corresponds to the data shown for 29. A first order reaction does not depend upon
the following reaction A + B ¾
¾® C (a) volume (b) pressure
(c) temperature (d) All of these
Expt. No. (A) (B) Initial Rate
1 0.012 0.035 0.10 30. 2A ¾¾ ® B + C, would be a zero order reaction when
2 0.024 0.070 0.80 (a) the rate of reaction is proportional to square of conc.
3 0.024 0.035 0.10 of A
4 0.012 0.070 0.80 (b) the rate of reaction remains same at any conc. of A
(a) Rate = k[B]3 (b) Rate = k [B]4 (c) the rate remains unchanged at any conc. of B and C
(c) Rate = k [A] [B]3 (d) Rate = k [A]2 [B]2 (d) the rate of reaction doubles if conc. of B is increased to
22. The chemical reaction 2O 3 ¾
¾® 3O 2 proceeds as follows double
31. Which of the following is correct for a first order reaction?
:
Fast Slow (a) t1 / 2 µ a (b) t1 / 2 µ 1 / a
O 3 ¾¾ ¾® O 2 + O ; O + O 3 ¾¾¾® 2O 2 the rate law
expression should be (c) t1 / 2 µ a 0 (d) t1 / 2 µ 1 / a 2
(a) r = k[O3]2 (b) r = k [O3]2[O2]–1 32. Order of reaction is decided by
(c) r = k3 [O3][O2]2 (d) r = [O3][O2]2 (a) temperature
23. For the reaction 2 H 2 O 2 ¾¾® 2H 2 O + O 2 , r = k [H2O2]. It (b) mechanism of reaction as well as relative concentration
of reactants
is
(c) molecularity
(a) zero order reaction (b) first order reaction
(d) pressure
(c) second order reaction (d) third order reaction
24. If the rate of the reaction is equal to the rate constant, the 33. Half-life of a reaction is found to be inversely proportional to
order of the reaction is the cube of initial concentration. The order of reaction is
(a) 1 (b) 2 (a) 4 (b) 3
(c) 3 (d) 0 (c) 5 (d) 2

25. For the reaction A + B ¾ ¾® C , it is found that doubling 34. The reaction L ¾ ¾® M is started with 10.0 g of L. After 30
the concentration of A increases the rate 4 times, and and 90 minutes 5.0 g and 1.25 g of L respectively are left. The
doubling the concentration of B doubles the reaction rate. order of the reaction is
What is the overall order of the reaction? (a) 0 (b) 1
(a) 3/2 (b) 4 (c) 2 (d) 3
(c) 1 (d) 3 35. If initial concentration is reduced to 1/4th in a zero order
26. For a first-order reaction, the half life period is independent reaction, the time taken for half the rection to complete
of (a) remains same (b) becomes 4 times
(a) initial concentration (c) becomes one-fourth (d) doubles
(b) cube root of initial concentration 36. A first order reaction is half-completed in 45 minutes. How
(c) first power of final concentration long does it need for 99.9% of the reaction to be completed?
(d) square root of final concentration (a) 20 hours (b) 10 hours
27. For a first order reaction, the plot of log K against 1/T is a
straight line. The slope of the line is equal to 1
(c) 7 hours (d) 5 hours
2
Ea 2.303
(a) - (b) - E R 37. A reaction which is of first order w.r.t. reactant A, has a rate
R a constant 6 min–1. If we start with [A] = 0.5 mol L–1, when
-E a would [A] reach the value of 0.05 mol L–1
Ea
(c) - (d) 2.303R (a) 0.384 min (b) 0.15 min
2.303
(c) 3 min (d) 3.84 min
28. For a first order reaction, a plot of log (a – x) against time is 38. The rate constant for the reaction
a straight line with a negative slope equal to
-k 2 N 2O5 ¾ ¾® 4 NO 2 + O 2 , is 3.0 × 10–5 sec–1. If the rate is
(a) (b) – 2.303 k 2.40 × 10–5 mol litre–1 sec–1, then the concentration of N2O5
2.303
(in mol litre–1) is
2.303 Ea (a) 1.4 (b) 1.2
(c) (d) - 2.303 R (c) 0.04 (d) 0.8
k
610 Chemistry
39. The reaction, 47. According to the collision theory of reaction rates, the rate
1 of reaction increases with temperature due to
N 2 O 5 (ln CCl 4 ) ¾¾® 2 NO 2 + O 2 (g) (a) greater number of collision
2
(b) higher velocity of reacting molecules
is first order in N2O5 with rate constant 6.2 × 10–4 s–1. What
(c) greater number of molecules having the activation
is the value of rate of reaction when [N2O5] = 1.25 mol L–1? energy
(a) 7.75 × 10–4 mol L–1 s–1 (b) 6.35 × 10–3 mol L–1 s–1 (d) decrease in the activation energy
(c) 5.15 × 10–5 mol L–1 s–1 (d) 3.85 × 10–4 mol L–1 s–1 48. Which of the following has been used to explain the subject
40. The time taken for 90% of a first order reaction to complete of chemical kinetics
is approximately (a) Collision theory of bimolecular reactions
(a) 1.1 times that of half-life (b) The activated complex theory
(b) 2.2 times that of half-life (c) Arrhenius equation
(c) 3.3 times that of half-life (d) All of these
(d) 4.4 times that of half-life 49. Which of the following statements is incorrect?
41. The rate of a certain reaction at different time intervals are (a) Activation energy for the forward reaction equals to
as follows activation energy for the reverse reaction
Time Rate (b) For a reversible reaction, an increase in temperature
0 sec. 1.8×10–2 mol l–1 s–1 increases the reaction rate for both the forward and the
10sec. 1.82×10–2 mol l–1 s–1 backward reaction
(c) The larger the initial reactant concentration for a second
20 sec. 1.79×10–2 mol l–1 s–1
order reaction, the shorter is its half-life.
The reaction is of
(d) When Dt is infinitesimally small, the average rate equals
(a) zero order (b) first order the instantaneous rate
(c) second order (d) third order 50. Activation energy of a chemical reaction can be determined
42. A substance initial concentration (a) reacts according to by
zero order kinetics. What will be the time for the reaction to (a) evaluating rate constant at standard temperature
go to completion (b) evaluating velocities of reaction at two different
a k temperatures
(a) (b) (c) evaluating rate constants at two different temperatures
k a
(d) changing concentration of reactants
a 2k 51. A catalyst increases rate of reaction by
(c) (d) (a) decreasing enthalpy
2k a
(b) decreasing internal energy
43. The hypothetical reaction A 2 + B 2 ¾
¾® 2AB ; follows (c) decreasing activation energy
(d) increasing activation energy
Fast
the following mechanism A 2 ¾¾¾® A + A , 52. The activation energy for a simple chemical reaction A ® B
Fast is Ea in forward direction. The activation energy for reverse
A + B 2 ¾¾¾® AB + B , A + B ¾¾
¾® AB .
Slow
reaction
The order of the overall reaction is (a) is always less than Ea
(a) 0 (b) 1 (b) can be less than or more than Ea
(c) 2 (d) 3/2 (c) is always double of Ea
44. The reaction (d) is negative of Ea
2N2O5 2N2O4 + O2 is 53. A graph plotted between log k Vs 1/T for calculating
activation energy is shown by
(a) bimolecular and of second order
(b) unimolecular and of first order
(c) bimolecular and of first order log k
log k
(d) bimolecular and of zero order (a) (b)
45. The given reaction

2FeCl 3 + SnCl 2 ¾
¾® 2 FeCl 2 + SnCl 4 1/T 1/T
is an example of
(a) first order reaction (b) second order reaction
(c) third order reaction (d) None of these log k
log k
46. Collision theory is applicable to
(c) (d)
(a) first order reactions (b) zero order reactions
(c) bimolecular reactions (d) intra-molecular reactions
1/T
1/T
Chemical Kinetics 611
54. Activation energy of the reaction is 58. The temperature dependence of rate constant (k) of a
(a) the energy released during the reaction chemical reaction is written in terms of Arrhenius equation, k

(b) the energy evolved when activated complex is formed = A . e - E a Activation energy (Ea) of the reaction can be
calculated by plotting
(c) minimum amount of energy needed to overcome the
potential barrier 1 1
(a) k vs. log T (b) log k vs
(d) the energy needed to form one mole of the product T
55. The activation energy for a hypothetical 1
reaction, A ® Product, is 12.49 kcal/mole. If temperature is (c) log k vs. log T (d) k vs. T
raised from 295 to 305, the rate of reaction increased by
59. The reaction A ® B follows first order kinetics. The time
(a) 60% (b) 100% taken for 0.8 mole of A to produce 0.6 mole of B is 1 hour.
(c) 50% (d) 20% What is the time taken for conversion of 0.9 mole of A to
produce 0.675 mole of B?
56. In a reaction, the threshold energy is equal to
(a) 0.5 hour (b) 0.25 hour
(a) activation energy + normal energy of reactants (c) 2 hours (d) 1 hour
(b) activation energy - normal energy of reactants 60. According to the adsorption theory of catalysis, the speed
(c) normal energy of reactants - activation energy of the reaction increases because
(a) in the process of adsorption, the activation energy of
(d) average kinetic energy of molecules of reactants
the molecules becomes large
57. When two reactants A and B are mixed to give products C (b) adsorption produces heat which increases the speed of
and D, the reaction quotient, 'Q', at the initial stages of the the reaction
reaction (c) adsorption lowers the activation energy of the reaction
(a) is zero (b) decreases with time (d) the concentration of product molecules at the active
(c) is independent of time (d) increases with time centres of the catalyst becomes high due to adsorption.

1. The reaction of hydrogen and iodine monochloride is given


log 2 log 2 ln 2 0.693
as: [CBSE-PMT 2007] (a) (b) (c) (d)
k k 0.5 k 0.5k
H 2 ( g ) + 2ICl( g ) ¾¾
® 2HCl( g ) + I 2 ( g )
3. If 60% of a first order reaction was completed in 60 minutes,
The reaction is of first order with respect to H2(g) and ICI(g), 50% of the same reaction would be completed in aproximately
following mechanisms were proposed. [CBSE-PMT 2007]
Mechanism A: (a) 45 minutes (b) 60 minutes
H 2 ( g ) + 2ICl( g ) ¾¾
® 2HCl( g ) + I 2 ( g ) (c) 40 minutes (d) 50 minutes
Mechanism B: 4. The rate constants k1 and k2 for two different reactions are
H 2 ( g ) + ICl( g ) ¾¾
® HI( g );slow 1016 . e–2000/T and 1015 . e–1000/T, respectively. The temperature
HI( g ) + ICl( g ) ¾¾
® HCl( g ) + I 2 ( g );fast at which k1 = k2 is : [CBSE-PMT 2008]
Which of the above mechanism(s) can be consistent with the
2000 1000
given information about the reaction? (a) 1000 K (b) K (c) 2000 K (d) K
2.303 2.303
(a) A and B both (b) neither A nor B
(c) A only (d) B only 5. The bromination of acetone that occurs in acid solution is
2. In a first-order reaction A ® B, if k is rate constant and inital represented by this equation.
concentration of the reactant A is 0.5 M, then the half-life is CH3COCH3 (aq) + Br2 (aq)
[CBSE-PMT 2007] ® CH3COCH2Br (aq) + H+ (aq) + Br– (aq)
612 Chemistry
These kinetic data were obtained for given reaction (a) 6.25 × 10–3 mol L–1s–1 and 6.25 × 10–3 mol L–1s–1
concentrations. (b) 1.25 × 10–2 mol L–1s–1 and 3.125 × 10–3 mol L–1s–1
(c) 6.25 × 10–3 mol L–1s–1 and 3.125 × 10–3 mol L–1s–1
Initial Initial rate, (d) 1.25 × 10–2 mol L–1s–1 and 6.25 × 10–3 mol L–1s–1
Concentrations, M disappearance of
11. For an endothermic reaction, energy of activation is Ea and
Br2, Ms–1
[CH3 COCH3] [Br2] [H+] enthalpy of reaction of DH (both of these in kJ/mol). Minimum
0.30 0.05 0.05 5.7×10–5 value of Ea will be. [CBSE-PMT 2010]
0.30 0.10 0.05 5.7 × 10–5
0.30 0.10 0.10 1.2 × 10–4 (a) less than DH (b) equal to DH
0.40 0.05 0.20 3.1 × 10–4 (c) more than DH (d) equal to zero
Based on given data, the rate equations is:[CBSE-PMT 2008] 12. During the kinetic study of the reaction, 2A + B ® C + D,
following results were obtained:
(a) Rate = k[CH3COCH3][H+]
(b) Rate = k [CH3COCH3][Br2]
Run [A](mol L–1 ) [B](mol L–1 ) Initial rate of
(c) Rate = k [CH3COCH3] [Br2] [H+]2
formation of
(d) Rate = k [CH3COCH3][Br2] [H+] –1 –1
D (mol L min )
6. For the reaction, N2 + 3H2 ® 2NH3, [CBSE-PMT 2009] –3
I 0.1 0.1 6.0 × 10
d [ NH3 ] – d [H2 ] II 0.3 0.2 7.2 × 10
–2
= 2 × 10–4 mol L–1 s–1 , the value of
dt dt III 0.3 0.4 2.88 × 10
–1

would be : –2
IV 0.4 0.1 2.40 × 10
(a) 4 × 10–4 mol L–1 s–1 (b) 6 × 10–4 mol L–1 s–1
(c) 1 × 10–4 mol L–1 s–1 (d) 3 × 10–4 mol L–1 s–1 Based on the above data which one of the following is correct?
7. In the reaction [CBSE-PMT 2009] [CBSE-PMT 2010]
(a) rate = k [A]2 [B] (b) rate = k[A] [B]
BrO3– (aq) + 5Br - ( aq ) + 6H + ( aq) ® 3Br2 (l ) + 3H 2O(l )
2 2
The rate of appearance of bromine (Br2) is related to rate of (c) rate = k [A] [B] (d) rate = k [A] [B]2
disappearance of bromide ions as following: 13. The rate of the reaction 2NO + Cl 2 ¾¾ ® 2NOCl is given
by the rate equation rate = k [NO]2 [Cl2] [CBSE-PMT 2010]
d[Br2 ] 5 d[Br – ] d[Br2 ] 5 d[Br – ] The value of the rate constant can be increased by:
(a) =– (b) =
dt 3 dt dt 3 dt (a) increasing the concentration of NO.
(b) increasing the temperature.
d[Br2 ] 3 d[Br – ] d[Br2 ] 3 d [Br – ] (c) increasing the concentration of the Cl2
(c) = (d) =-
dt 5 dt dt 5 dt (d) doing all of these
8. Half life period of a first-order reaction is 1386 seconds. The 14. Which one of the following statements for the order of a
specific rate constant of the reaction is : [CBSE-PMT 2009] reaction is incorrect ? [CBSE-PMT 2011]
(a) 0.5 × 10–2 s–1 (b) 0.5 × 10–3 s–1 (a) Order can be determined only experimentally.
–2 –1 (b) Order is not influenced by stoichiometric coefficient of
(c) 5.0 × 10 s (d) 5.0 × 10–3 s–1
the reactants.
9. For the reaction A + B ¾¾ ® products, it is observed that: (c) Order of reaction is sum of power to the concentration
[CBSE-PMT 2009] terms of reactants to express the rate of reaction.
(d) Order of reaction is always whole number.
(1) On doubling the initial concentration of A only, the rate
15. The rate of the reaction 2N2O5 ® 4NO2 + 2O2 can be written
of reaction is also doubled and
in three ways : [CBSE-PMT 2011 M]
(2) On doubling the initial concentrations of both A and B,
- d[N 2O5 ]
there is a change by a factor of 8 in the rate of the reaction. = k [N 2O5 ]
dt
The rate of this reaction is given by:
(a) rate = k [A] [B]2 (b) rate = k [A]2 [B]2 d[NO 2 ]
= k ¢ [N 2 O5 ]
(c) rate = k [A] [B] (d) rate = k [A]2 [B] dt

10. For the reaction [ N 2 O5 (g) ¾¾


® 2NO2 (g) + 1/2 O 2 (g)] d[O 2 ]
= k ¢¢ [N 2O5 ]
dt
the value of rate of disappearance of N 2 O5 is given as The relationship between k and k' and between k and k¢¢ are :
6.25 × 10–3 mol L–1s–1. The rate of formation of NO 2 and O 2 (a) k¢ = 2k ; k¢ = k (b) k¢ = 2k ; k¢¢ = k / 2
(c) k¢ = 2k ; k¢¢ = 2k (d) k¢ = k ; k¢¢ = k
is given respectively as : [CBSE-PMT 2010]
Chemical Kinetics 613
16. The unit of rate constant for a zero order reaction is 24. The integrated rate equation is [AIEEE 2002]
[CBSE-PMT 2011 M] Rt = log C0 - logCt.
(a) mol L–1 s–1 (b) L mol–1 s–1 The straight line graph is obtained by plotting
(c) L2 mol–2 s–1 (d) s –1 1
(a) time Vs logCt (b) Vs Ct
17. In a zero-order reaction for every 10° rise of temperature, the time
rate is doubled. If the temperature is increased from 10°C to
100°C, the rate of the reaction will become : 1 1
(c) time Vs Ct (d) Vs
[CBSE-PMT 2012 S] time Ct
(a) 256 times (b) 512 times (c) 64 times (d) 128 times
18. Activation energy (Ea) and rate constants (k1 and k2) of a 25. In respect of the equation k = Ae - E a / RT in chemical kinetics,
chemical reaction at two different temperatures (T1 and T2) which one of the following statements is correct ?
are related by : [CBSE-PMT 2012 M] [AIEEE 2003]
(a) A is adsorption factor
E æ1 1ö E æ1 1ö
= - a çç - ÷÷÷ = - a çç - ÷÷÷
k2 k2 (b) Ea is energy of activation
(a) ln (b) ln
k1 ç
R è T1 T2 ø÷ k1 R èç T2 T1 ø÷ (c) R is Rydberg’s constant
(d) k is equilibrium constant
E æ1 1ö E æ1 1ö
= - a çç + ÷÷÷ = a çç - ÷÷÷
k2 k2 26. The rate law for a reaction between the substances A and B
R èç T1 T2 ÷ø
(c) ln (d) ln
k1 ç
R è 2T T1 ø÷ k1 is given by [AIEEE 2003]
n
Rate = k [A] [B] m
19. What is the activation energy for a reaction if its rate doubles
On doubling the concentration of A and halving the
when the temperature is raised from 20°C to 35°C? (R = 8.314
concentration of B, the ratio of the new rate to the earlier rate
J mol–1 K–1) [NEET 2013]
–1 –1
of the reaction will be as
(a) 269 kJ mol (b) 34.7 kJ mol
(a) (m + n) (b) (n – m)
(c) 15.1 kJ mol–1 (d) 342 kJ mol–1
20. The formation of gas at the surface of tungsten due to 1
(c) 2(n – m) (d) (m+ n )
adsorption is the reaction of order [AIEEE 2002] 2
(a) 0 (b) 1 27. For the reaction system :
(c) 2 (d) insufficient data. 2 NO(g) + O 2 (g) ® 2 NO 2 (g) volume is suddenly reduced
21. Units of rate constant of first and zero order reactions in to half its value by increasing the pressure on it. If the reaction
terms of molarity (M) unit are respectively [AIEEE 2002] is of first order with respect to O2 and second order with
(a) sec–1, Msec–1 (b) sec–1, M respect to NO, the rate of reaction will [AIEEE 2003]
(c) Msec–1, sec–1 (d) M, sec–1. (a) diminish to one-eighth of its initial value
22. For the reaction A + 2B ® C, rate is given by R = [A][B]2 (b) increase to eight times of its initial value
then the order of the reaction is [AIEEE 2002] (c) increase to four times of its initial value
(a) 3 (b) 6 (d) diminish to one-fourth of its initial value
(c) 5 (d) 7. 28. In a first order reaction, the concentration of the reactant,
23. The differential rate law for the reaction [AIEEE 2002] decreases from 0.8 M to 0.4 M in 15 minutes. The time taken
for the concentration to change from 0.1 M to 0.025 M is
H2 + I 2 ® 2HI is [AIEEE 2004]
d[ H 2 ] d[ I 2 ] d[ HI] (a) 7.5 minutes (b) 15 minutes
(a) - =- =-
(c) 30 minutes (d) 60 minutes
dt dt dt
29. The rate equation for the reaction 2A + B ® C is found to
d[ H 2 ] d[ I 2 ] 1 d[ Hl] be : rate = k[A][B]. The correct statement in relation to this
= =
(b) dt dt 2 dt reaction is that the [AIEEE 2004]
(a) rate of formation of C is twice the rate of disappearance
1 d[ H 2 ] 1 d[ I 2 ] d[ Hl] of A
(c) 2 dt = 2 dt = - dt
(b) t1 / 2 is a constant
d[ H 2 ] d[ I 2 ] d[HI] (c) unit of k must be s–1
(d) - 2 = -2 = (d) value of k is independent of the initial concentrations of
dt dt dt
A and B
614 Chemistry
30. Consider an endothermic reaction X ® Y with the activation (c) the energy above which all the colliding molecules will
energies E b and E f for the backward and forward reactions, react
(d) the energy below which colliding molecules will not
respectively. In general [AIEEE 2005]
react
(a) there is no definite relation between E b and E f 36. The following mechanism has been proposed for the
(b) E b = E f reaction of NO with Br2 to form NOBr :
(c) E b > E f NO(g) + Br2(g) NOBr2(g)
(d) E b < E f NOBr 2 ( g ) + NO (g ) ¾
¾® 2 NOBr ( g )
31. A reaction involving two different reactants can never be If the second step is the rate determining step, the order of
[AIEEE 2005] the reaction with respect to NO(g) is [AIEEE 2006]
(a) bimolecular reaction (a) 3 (b) 2
(b) second order reaction
(c) first order reaction (c) 1 (d) 0
(d) unimolecular reaction 37. The energies of activation for forward and reverse reactions
32. A schematic plot of ln k eq versus inverse of temperature for A2 + B2 ƒ 2AB are 180 kJ mol–1 and 200 kJ mol–1
respectively. The presence of a catalyst lowers the activation
for a reaction is shown below [AIEEE 2005]
energy of both (forward and reverse) reactions by 100 kJ
6.0 mol–1. The enthalpy change of the reaction (A2 + B2 ® 2AB)
in the presence of a catalyst will be (in kJ mol–1)
In keq

[AIEEE 2007]
(a) 20 (b) 300 (c) 120 (d) 280
2.0 38. Consider the reaction, 2A + B ® products. When
1.5 ´10 -3 1 (k -1 ) 2.0 ´10 -3 concentration of B alone was doubled, the half-life did not
T change. When the concentration of A alone was doubled, the
The reaction must be rate increased by two times. The unit of rate constant for this
(a) highly spontaneous at ordinary temperature reaction is [AIEEE 2007]
(b) one with negligible enthalpy change (a) s –1 (b) L mol–1 s–1
(c) endothermic
(c) no unit (d) mol L–1 s–1.
(d) exothermic
33. t1/4 can be taken as the time taken for the concentration of a 39. A radioactive element gets spilled over the floor of a room. Its
half-life period is 30 days. If the initial velocity is ten times the
3
reactant to drop to of its initial value. If the rate constant permissible value, after how many days will it be safe to enter
4
the room? [AIEEE 2007]
for a first order reaction is K, the t1/4 can be written as
[AIEEE 2005] (a) 100 days (b) 1000 days
(a) 0.75/k (b) 0.69/k (c) 300 days (d) 10 days.
(c) 0.29/k (d) 0.10/k
1
34. A reaction was found to be second order with respect to the 40. For a reaction A ® 2B, rate of disappearance of ‘A’ is
concentration of carbon monoxide. If the concentration of 2
carbon monoxide is doubled, with everything else kept the related to the rate of appearance of ‘B’ by the expression
same, the rate of reaction will [AIEEE 2006] [AIEEE 2008]
(a) increase by a factor of 4
d[A] 1 d[B]
(b) double (a) – = (b) – d[A] = 1 d[B]
(c) remain unchanged dt 2 dt
dt 4 dt
(d) triple
35. Rate of a reaction can be expressed by Arrhenius equation d[A] d[B]
(c) d[A ] d[B]
– =
(d) – =4
as : [AIEEE 2006] dt dt dt dt
k = A e–E/RT
41. The half life period of a first order chemical reaction is 6.93
In this equation, E represents
minutes. The time required for the completion of 99% of the
(a) the total energy of the reacting molecules at a
temperature, T chemical reaction will be (log 2 = 0.301) [AIEEE 2009]
(b) the fraction of molecules with energy greater than the (a) 23.03 minutes (b) 46.06 minutes
activation energy of the reaction (c) 460.6 minutes (d) 230.03 minutes
Chemical Kinetics 615

42. The time for half life period of a certain reaction 47. The rate of a reaction doubles when its temperature changes
A ¾¾ ® Products is 1 hour. When the initial concentration from 300 K to 310 K. Activation energy of such a reaction will
of the reactant ‘A’, is 2.0 mol L–1, how much time does it take be : (R = 8.314 JK–1 mol–1 and log 2 = 0.301) [JEE Main 2013]
for its concentration to come from 0.50 to 0.25 mol L–1 if it is (a) 53.6 kJ mol –1 (b) 48.6 kJ mol–1
a zero order reaction ? [AIEEE 2010] (c) 58.5 kJ mol –1 (d) 60.5 kJ mol–1
(a) 4 h (b) 0.5 h (c) 0.25 h (d) 1 h 48. Consider a reaction aG + bH ® Products. When
43. Consider the reaction : concentration of both the reactants G and H is doubled, the
rate increases by eight times. However, when concentration
Cl2 (aq ) + H 2S( aq ) ® S( s) + 2H + ( aq) + 2Cl - ( aq) of G is doubled keeping the concentration of H fixed, the rate
is doubled. The overall order of the reaction is
The rate equation for this reaction is
[IIT-JEE 2007]
rate = k[Cl2 ][H 2S] (a) 0 (b) 1
Which of these mechanisms is/are consistent with this rate (c) 2 (d) 3
equation? [AIEEE 2010] 49. Under the same reaction conditions, initial concentration of
1.386 mol dm–3 of a substance becomes half in 40 seconds
A. Cl2 + H 2S ® H + + Cl - + Cl+ + HS- (slow)
and 20 seconds through first order and zero order kinetics,
Cl + + HS- ® H + + Cl - + S (fast) respectively. Ratio (k1/k0) of the rate constant for first order
(k1) and zero order (k0) of the reaction is [IIT-JEE 2008]
B. H 2S ƒ H + + HS- (fast equilibrium) (a) 0.5 mol–1 dm3 (b) 1.0 mol dm–3
(c) 1.5 mol dm –3 (d) 2.0 mol–1 dm3
Cl2 + HS- ® 2Cl - + H + + S (Slow) 50. For a first order reaction A®P, the temperature (T) dependent
(a) B only (b) Both A and B rate constant (k) was found to follow the equation
(c) Neither A nor B (d) A only 1
log k = – (2000) + 6.0 . The pre-exponential factor A and
44. A reactant (A) froms two products : [AIEEE 2011RS] T
k the activation energy Ea, respectively, are [IIT-JEE 2009]
A ¾¾
1 ® B,
Activation Energy Ea1
(a) 1.0 × 106 s–1 and 9.2 kJ mol–1
k2
A ¾¾® C, Activation Energy Ea2 (b) 6.0 s–1 and 16.6 kJ mol–1
(c) 1.0 × 106 s–1 and 16.6 kJ mol–1
If Ea2 = 2 Ea1, then k1 and k2 are related as : (d) 1.0 × 106 s–1 and 38.3 kJ mol–1
51. Plots showing the variation of the rate constant (k) with
(a) k2 = k1e Ea1 / RT (b) k2 = k1e Ea2 / RT
temperature (T) are given below. The plot that follows
(c) k1 = Ak2 e Ea1 / RT (d) k1 = 2k2 e Ea2 / RT Arrhenius equation is [IIT-JEE 2010]

45. The rate of a chemical reaction doubles for every 10°C rise of
temperature. If the temperature is raised by 50°C, the rate of
the reaction increases by about : [AIEEE 2011]
(a) 10 times (b) 24 times (c) 32 times (d) 64 times (a) (b)
46. For a first order reaction (A) ® products the concentration of
A changes from 0.1 M to 0.025 M in 40 minutes.
The rate of reaction when the concentration of A is 0.01 M is :
[AIEEE 2012]
(a) 1.73 × 10–5 M/min (b) 3.47 × 10–4 M/min (c) (d)
(c) 3.47 × 10–5 M/min (d) 1.73 × 10–4 M/min
616 Chemistry

1. Two substances R and S decompose in solution Rate in terms of N2. Rate in terms of H2
independently, both following first order kinetics. The rate
(mol L–1sec–1 ) (mol L–1sec–1 )
constant of R is twice that of S. In an experiment, the solution
initially contained 0.5 millimoles of R and 0.25 millimoles of (a) 2×10–4 2×10–4
S. The molarities of R and S will be equal just at the end of (b) 3×10–4 1×10–4
time equal to (c) 1×10–4 3×10–4
(a) twice the half life of R (b) twice the half life of S (d) 2×10–1 2×10–3
(c) the half life of S (d) the half life of R 8. The rate constant , the activation energy and the Arrhenius
2. For a reaction, the rate constant at particular temperature parameter of a chemical reaction at 25°C are 3×10–4 s–
has a value of 2.0 × 10–3 mol lit-1s -1 . The order of the reaction
1,104.4kJ /mol and 6×1014 s–1 respectively. The value of the

is rate constant as T ® ¥ is
(a) –1 (b) 0 (a) 2.0×1018 s–1 (b) 6.0×1014 s–1
(c) 1 (d) 2 (c) Infinity (d) 3.6×1030 s–1
3. For the gas phase decomposition A ® 2B, the rate 9. A substance ‘A’ decomposes in solution following first order
constant is 6.93 × 10–3 min–1 at 300 K. The percentage of a kinetics. Flask I contains l L of a 1M solution of A and flask
remaining at the end of 300 minutes is II contains 100 ml of a 0.6 M solution. After 8 hours the
concentration of A in flask I has become 0.25. What will be
(a) 75 (b) 50
the time taken for concentration of A in flask II to become
(c) 25 (d) 12.5 0.3M ?
4. For a first order reaction t 0.75 is 1368 seconds, therefore, (a) 0.4 h
the specific rate constant in sec–1 is (b) 2.4 h
(c) 4.0 h
(a) 10–3 (b) 10–2 (d) Can’t be calculated since rate constant is not given
(c) 10–9 (d) 10–5 10. A reaction rate constant is given by
5. Consider the following reactions at 300 K k = 1.2 ´ 1014 e-25000/RT sec-1 . It means
(a) log k versus log T will give a straight line with a slope as
X ¾¾ ¾
¾® Y (uncatalysed reaction)
–25000
catalyst
X ¾¾¾
¾® Y (catalysed reaction) (b) log k versus T will give a straight line with slope as
25000
The energy of activation is lowered by 0.314 KJ mol -1 for (c) log k versus log 1/T will give a straight line with slope as
the catalysed reaction. The rate of reaction is –25000
(a) 38 times (d) log k versus 1/T will give a straight line
(b) 15 times 11. Consider a general chemical change 2A + 3B ® products.
(c) 25 times The rate with respect to A is r1 and that with respect to B is
(d) 22 times that of uncatalysed reaction. r2. The rates r1 and r2 are related as
6. For a reaction A ® B , the rate increases by a factor of 2.25 (a) 3 r1=2r2 (b) r1 = r2
when the concentration of A is increased by 1.5. What is the (c) 2r1 = 3r2 (d) r12 = 2r22
order of the reaction?
12. In case of first order reaction,the ratio of the time required for
(a) 3 (b) 0
99.9% completion to 50% completion is
(c) 2 (d) 1 (a) 2 (b) 5
7. The reactions rate N 2 (g) + 3H 2 (g) ® 2 NH 3 (g) was (c) 10 (d) None of these
13. For the exothermic reaction A + B ® C + D, DH is the heat
measured d[ NH 3 ] = 2 ´ 10 - 4 mol sec –1 . The rates of of reaction and Ea is the energy of activation. The energy of
dt
activation for the formation of A + B will be
reactions expressed in terms of N2 and H2 are (a) Ea (b) DH
(c) Ea + DH (d) DH – Ea
Chemical Kinetics 617
14. The following statement(s) is (are) correct : 20. For a reaction following first-order kinetics, which of the
(i) A plot of log kp versus 1/T is linear following statements are correct?
(ii) A plot of log [X] versus time is linear for a first order (a) The time taken for the completion of 50% of the reaction
is twice of t½
reaction, X ® P
(b) A plot of the reciprocal of the concentration of the
(iii) A plot of log p versus 1/T is linear at constant volume
reactants against time gives a straight line.
(iv) A plot of p versus 1/V is linear at constant temperature (c) The degree of dissociation is equal to 1–e–kt.
(a) (i) only (b) (ii) only (d) A plot of [A]0/[A] versus time gives a straight line.
(c) (i) & (iv) (d) (i), (ii) & (iv) 21. An endothermic reaction has a positive internal energy
15. The decomposition of A follows two parallel first order change DU. In such a case, what is the minimum value that
reactions the activation energy can have ?
–4 –1
(a) DU (b) DU = DH + DnRT
k1 1.26 ´ 10 sec (c) DU=DH–DnRT (d) DU=Ea+RT
B
–5 –1 22. At 400 K the half-life of a sample of a gaseous compound
A k2 3.8 ´ 10 sec
C initially at 56.0 kPa is 340 s. When the pressure is 28.0 kPa, the
The percentage distribution of B and C are half-life is 170s. The order of the reaction is
(a) 0 (b) 2
(a) 90 % B and 10 % C
(c) 1 (d) ½
(b) 80 % B and 20 % C
23. Consider the reaction mechanism
(c) 60 % B and 40 % C
A2 2A (fast)
(d) 76.83 of B and 23.17 % C A + B ® P (slow)
16. In a first order reaction the concentration of where A is the intermediate. The rate for the reaction is
reactant decreases from 800 mol/dm3 to 50 mol/dm3 in 2 x 104 (a) k2[A][B] (b) k2k½[A2]½[B]
sec. The rate constant of reaction in sec-1 is: ½
(c) k2k [A][B] (d) k2k½[A]2[B]
(a) 2 × 104 (b) 3.45 × 10-5 24. N2O2(g) ® 2NO(g) is a first-order reaction in terms of the
(c) 1.386 × 10-4 (d) 2 × 10-4 concentration of N2O2(g). Which of the following is valid,
17. Which of the following graphs best describes the rate at [N2O2] being constant ?
which N2O4 decomposes to NO2 if the reaction is first-order (a) [NO] = [N2O2]0 e–kt (b) [NO] = [N2O2]0 (1–e–kt)
in N2O4? (c) [NO] = [N2O2]0 (e–kt–1) (d) [NO] = [N2O2]0 (1–e–kt)
25. The reaction A(g) + 2B(g) ® C(g) + D(g) is an elementary
process. In an experiment involving this reaction, the initial
(a) [N2O4] (b) [N2O4] partial pressures of A and B are pA = 0.60 atm and pB= 0.80
atm respectively. When pC= 0.20 atm, the rate of the reaction
relative to the initial rate is
Time Time
(a) 1/6 (b) 1/12
(c) 1/36 (d) 1/18

(c) [N2O4] (d) [N2O4] 26. In acidic medium the rate of reaction between BrO3 - and

Br - ion is given by the expression


Time Time
d ( BrO3 - ) - - + 2
18. The rate constant for a zero-order reaction is = k[BrO3 ][Br ][H ]
dt
C0 C0 - Ct Which of the following is correct ?
(a) k = (b) k =
2t t
(a) Doubling the concentration of H + ions will increase
C0 - Ct C0 the reaction rate by 4 times.
(c) k = ln (d) k = (b) Rate constant of overall reaction is 4 sec–1.
2t 2t
(c) Rate of reaction is independent of the conc. of acid
19. The rate expression for the reaction A(g) + B(g)®C(g) is rate (d) The change in pH of the solution will not affect the
= kC 2A C½ rate.
B . What changes in the initial concentrations of A
27. The rate constant of the reaction increases by
and B will cause the rate of reaction to increase by a factor of (a) carrying out the reaction for longer period
eight? (b) increasing the temperature
(a) CA×2; CB×2 (b) CA×2; CB×4 (c) increasing the conc. of reactants
(c) CA×1; CB×4 (d) CA×4; CB×1 (d) None is correct.
618 Chemistry
34. For a zero order reaction, the plot of conc. Vs time is linear
28. The activation energy of the reaction A ¾ ¾® X is 12.49
with
kcal mol–1. If temperature is raised to 305 K from 295 K, the
(a) –ve slope and non zero intercept
reaction rate increases by
(b) +ve slope and zero intercept
(a) » 50 % (b) » 60 %
(c) –ve slope and zero intercept
(c) » 100 % (d) unpredictable.
(d) + ve slope and non zero intercept
29. For the reaction A + B ¾ ¾® C + D. The variation of the 35. The half life decomposition of gaseous CH3CHO at constant
concentration of the products is given by the curve
temperature but at initial pressure of 364 mm and 170 mm of
Hg were 410 sec, and 880 sec respectively. The order of the
Y
reaction is
Z
(a) 1.0 (b) 3.0
Conc (c) 2.0 (d) 1.5
36. The racemisation of a - pinene is first order reaction. In the
W
X gas the specific reaction rate was found to be 2.2 ´ 10 -5 mm
Time
Hg–1 at 457.6 K and 3.07 ´ 10 -3 mm Hg–1 at 510.1 K. The
(a) Z (b) Y energy of activation is
-3
(c) W (d) X (a) K (510.1 – 457.6) ln (3.07 × 10 / 2.2 ´ 10 -5 )
30. A gaseous reaction
1 (b) 3. 048 ´10 -3
X 2 (g) ¾
¾® Y + Z(g)
2
There is increase in pressure from 100 mm Hg to 120 mm Hg (c) ln 2. 2 ´10 -5 ´ 457.6 k cal.
in 5 minutes. The rate of disappearance of X 2 is (d) (510 .1 ´ 457 .6 / 52 .5) R ln (307)/2.2) cal.

(a) 8 mm Hg min -1 (b) 2 mm Hg min -1 37. The half life of the first order reaction

(c) 16 mm Hg min -1 (d) 4 mm Hg min -1 CH 3 .CHO (g ) ¾


¾® CH 4 (g ) + CO (g )
31. For an exothermic chemical process occuring in two steps as If initial pressure of CH 3CHO (g) is 80 mm Hg and the total

(i) A + B ¾¾® X (slow) (ii) X ¾ ¾® AB (fast) pressure at the end of 20 minutes is 120 mm Hg
(a) 80 min (b) 120 min
The progress of the reaction can be best described by
(c) 20 min (d) 40 min

38. A drop of a solution (volume = 0.05 ml) contains 6 ´ 10 -7


(a) A+ B AB (b) A+ B AB
mol of H + . If rate of disappearnace of H + is 6.0 ´ 10 5 mol

lit -1 s -1 , how long will it take for the H + in the drop to


disappear ?
(c) A+ B AB
(d) None of these
(a) 2.0 ´ 10 -2 s (b) 2 . 0 ´ 10 - 8 s
32. The rate of certain reaction increases by 2.3 times when the
temperature is raised from 300 K to 310 k. If k is the rate (c) 8.0 ´ 10 -8 s (d) 6.0 ´10 -6 s
constant at 300 K then rate constant at 310 k will be
39. The rate constant for two parallel reactions were found to be
2
(a) 3 k (b) 2.3 k 1.0 ´ 10 -2 dm 3 mol -1s -1 and 3.0 ´ 10 -2 dm 3 mol -1s -1 .
(c) 2 k (d) k If the corressponding energies of activation of the parallel
-1
33. The half life period for catalytic decomposition of AB3 at 50 reactions are 60.0 kJ mol-1 and 70.0 kJ mol respectively
mm Hg is 4 hrs and at 100 mm Hg it is 2 hrs. The order of what is the apparent overall energy of activation
reaction is
(a) 1 (b) 3 (a) 130.0 kJ mol -1 (b) 65.0 kJ mol -1
(c) 2 (d) 0 (c) 67.5 kJ mol -1 (d) 100.0 kJ mol -1
Chemical Kinetics 619

EXERCISE 1 6. (a) Remember units of rate constant =(Mol L –1 ) 1–n.


2. Increase the concentration of P to eight times and that of Q time–1. For 1st order n = 1 \ units = time–1.
to two times. 7. (a) Rate constant depends upon temperature see Arrhenius
3. The order of reaction with respect to N2O5 is one. equation.
8. (a) mol dm–3 s–1 units are for zero order.
2×303 æ 1 ö
log ç
è 1 - 1/ n ÷ø
4. t1/n = 9 (d) Rate constant does not change with concentration.
k
5. Rate = k [A]2 [B]. 10. (b) k = time–1 for 1st order. It is independent of concentration
7. Ea = 51×86 kJ mol–1. terms.
8. k = 8×455 × 10–4 s–1. 11. (d) k = Ae - E a / RT
9. k(at 410 s) = 0.77 × 10–4s–1. 12. (c) k = (mol lit–1)1–n time–1. For given reaction n = 2.
18. (d) 19. (a) 20. (d) 21. (b) 22. (d) \ k = mol–1 lit sec–1
23. (d) 24. (b)
Ea
EXERCISE 2 13. (b) ln k = ln A –
RT
, intercept is ln A.

k1 14. (c) The rate constant doubles for 10º C rise in temperature.
1. (b) 2 NO2 N2O4,
k2 For 20º C rise, the rate constant will be 4 times
\ k1 = 4k2 or k2 = 0.25 K1
For forward reaction rate = - 1 dNO2 = dN2 O4
2 dt dt
2.303 a 2.303 1 2.303
15. (d) k = log = log = log 8
= k1[NO 2 ] 2 t a-x 24 1 24
8
For backward reaction
16. (d) With temperature the number of activated molecules
dN 2 O 4 1 dNO 2
Rate = - = = k 2 [N 2 O 4 ] . increase.
dt 2 dt
17. (b) Between 2 and 3.
Rate of disappearance of NO2 = (Rate of disapperance of 18. (c) There are 5 tens hence (2)5 = 32.
NO2 – Rate of appearence of NO2)
19. (b) For 1st order rection t 1/2 independent of initial
= 2k1[NO2]2 – 2k2[N2O4].
concentration.
2. (b) By concentration of reactants.
20. (c) Let the rate law equation be r = k[A]x[B]y................(1)
3. (b) 3A ® 2B
when A = 2A, 4r = k[2A]x[B]y........................(2)
Rate of reaction - 1 dA = 1 dB or - 2 dA = dB
4r k[2A]x [B] y
3 dt 2 dt 3 dt dt Divide(2) by (1) =
r k[A]x [B] y
4. (a) N 2 (g ) + 3H 2 (g ) ¾
¾® 2 NH 3 (g )
\ 4 = (2)x, Hence x = 2

Rate = -
dN 2 1 dH 2 1 dNH 3 when B = 2B, r = k[A]x [2B]y.........................(3)
=- =
dt 3 dt 2 dt
r k[A]x [2B] y
5. (d) If volume is reduced to 1/4th concentration will \ 1 = Divide (3) by (1) = (2)y \ y = 0
be increased 4 times hence reaction rate will r k[A]x [B]y
increase by 16. Rate = k[A][B].
\ Rate = k[A]2[B]0
New rate = k[4A][4B] = 16k[A][B]
620 Chemistry
21. (a) Let the rate law be r =k [A]x[B]y
1
34. (b) After every 30 minutes the amount is reduced to
x y 2
0.10 [0.024] [0.035]
Divide (3) by (1) =
0.10 [0.012] x [0.035] y therefore t1/2 is 30 minutes. In 90 minutes the amount is

\ 1 = [2]x, x = 0 1 1
reduced to i.e. . Here n = 3. True for 1st order reaction.
8 2n
0.80 [0.024] x [0.070] y
Divide (2) by (3) = a
0.10 [0.024]x [0.035] y 1 1
35. (c) t = for zero order, t = and t1 = , \ t1 = 2t
k 4k 2k
\ 8 = (2)y , y = 3
Hence rate equation, R = k[A]0[B]3 =k[B]3 0.693 2.303 100
36. (c) k = further t = ´ 45 log
45 0.693 100 - 99.9
22. (b) O 3 ¾Fast
¾
Slow
¾® O 2 + O; O + O 3 ¾¾¾® 2O 2
2.303 ´ 45 ´ 3 1
= = 7 hours .
[O ][O] 0.693 2
k= 2 (I), Rate = k' [O3][O] put [O] from (I)
[O 3 ]
2.303 a 2.303 0 .5
37. (a) t = log = log = 0.384 min .
k '[O3 ]k[O3 ] k a-x
= k[O3 ]2 [O2 ]-1
6 0.05
r=
[O2 ] 38. (d) Rate = k [N2O5] ;
Note intermediates are never represented in rate law 2.4 ´10 -5 = 3.0 ´ 10 -5 [ N 2 O 5 ]
equation.
23. (b) 1st order reaction. \ [ N 2 O 5 ] = 0 .8
24. (d) Rate = k [A]0 39. (a) Rate = k [N2O5]
\ Rate = k for zero order. = 6.2 ×10–4×1.25 = 7.75 × 10–4 mol L–1s–1
25. (d) Order with respect to A is 2 and B is 1.
2.303 100
\ Order of reaction = 2+1 = 3 40. (c) t 90% = log (I)
k 100 - 90
26. (a) t1/2 for 1st order is independent of initial concentration.
2.303 100
Ea t 50% = log (II)
27. (d) k = Ae - Ea / RT log k = log A – .
1 k 100 - 50
2.303R T
t log10
-E a Dividing 90% = \ t 90% = 3.3t 50%
Equation of straight line slope = . t 50% log 2
2.303R
41. (a) Since rate is constant throughout. It is zero order.
2.303 a
28. (a) t = log 42. (a) For zero order reaction tcompletion = a/k.
k a-x
43. (d) A 2 + B 2 ¾
¾® 2AB ; A 2 ¾
¾® A + A ( Fast );
2.303 2.303
or t = log a - log(a - x)
k k A + B2 ¾
¾® AB + B (Slow )
29. (d) Order of reaction is independent of given factors.
Rate law = k[A][B 2 ] put value of [A] from Ist reaction
30. (b) For zero order the rate does not change with
concentration. since A is intermediate k[A 2 ] = A
31. (c) t1/2 µ aº. t1/2 is independent of initial concentration.
32. (b) The order of a chemical reaction is given by \ Rate law equation = K k[A 2 ][B2 ]
concentration of reactants appearing in the lowest step.
1 3
\ Order = +1 =
1
33. (a) t1 / 2 µ a1- n
2 2
t1 / 2 µ hence n = 4.
a3 44. (c) It is bimolecular first order reaction since Rate µ [N2O5]
Chemical Kinetics 621
45. (c) Third order
a 0.9
46. (c) Applicable to bimolecular reactions. In second case = =4
a - x 0.225
47. (a)
Since the ratio is the same, hence time will be same.
48. (d) All the statements are correct (see text).
Therefore 1 hr.
49. (a) E a (F.R .) ¹ E a (B.R .) Ea can be calculated. 60. (c) Adsorption lowers the activation energy.
EXERCISE 3
k2 E a (T2 - T1 )
50. (c) log = ; Ea can be calculated.
k1 2.303R T1T2
1. (d) As the slowest step is the rate determining step thus
51. (c) Activation energy is lowered in presence of +ve catalyst. the mechanism B will be more consistent with the given
52. (b) Since the nature of reaction (i.e. exothermic or information also because it involve one molecule of
endothermic) not given therefore Ea for reverse reaction H2 and one molecule of ICl it can expressed as
can be more or less. r = k [H2][ICl]
53. (b) In accordance with Arrhenius equation. Which shows that the reaction is first order w.r.t. both
54. (c) Statement (c) see text. H2 & ICl.
55. (b) For 10°C rise of temperature the rate is almost doubled. 2. (c) For a first order reaction
56. (a) Threshold Energy = Energy of activation + Internal 2.303 a
energy k= log
t a- x
[C][D] [Products] when t = t½
57. (d) Q = = ; Q = increases with time
[A][B] [Reactants]
2.303 a
k= log
Ea 1 t½ a - a/2
58. (b) k = Ae - Ea / RT log k = log A - .
2.303R T
2.303 ln 2
or t½ = log 2 =
1 k k
Plot of log k Vs.
T 3. (a) For a first order reaction

-E a 2.303 a
Straight line Slope = k= log
2.303R t a- x

59. (d) A ® B when t = 60 and x = 60%

t= 0 0.8 0 2.303 100 2.303 100


k= log = log
t= 1 0.8 – 0.6 0.06 60 100 - 60 60 40
t= 0 0.9 0 Now,
t= ? 0.9 – 0.675 0.675
2.303 100 2.303
t1 = log = ´ log 2
2.303 a 2 0.0153 100 - 50 0.0153
1st order t = log
k a-x
2.303
= ´ 0.3010 = 45.31 min .
2.303 0.8 0.0153
2.303
1= log k= log 4
k 0.2 1 2000
-
k = 2.303 × 0.6020 4. (d) Given, k1 = 1016.e T

2.303 0.9 2.303 ´ 0.6020


t= log = = 1 hr.. -
1000
2.303 ´ 0.6020 0.675 2.303 ´ 0.6020 and k2 = 1015.e T

Second Approach when k1 and k2 are equal at any temperature T, we


have
a 0.8
In first case = =4
a - x 0 .2 -
2000
-
1000
1016.e T 15
= 10 .e T
622 Chemistry

2000 1000 M to 0.20 M but the rate has changed from 1.2× 10 –4 to
- -
or 10 ´ 10 .e
15 T 15
= 10 .e T 3.1×10–4. This is due to change in concentration of
CH3COCH3 from 0.30 M to 0.40 M. Thus the rate is
2000 1000 directly proportional to [CH3 COCH3]. We now get
- -
T T
or 10.e =e
rate = k [CH3COCH3]1[Br2]0[H+]1
2000 1000 = k [CH3COCH3][H+].
or ln 10 - =-
T T 6. (d) Rate of disappearance of H2 = rate of formation of
NH3.
2000 1000
or ln 10 = - 1 d[H2 ] 1 d[NH3 ]
T T – =
3 dt 2 dt
1000
or 2.303 log 10 = – d[H 2 ] 3 d[NH3 ] 3 –4
T Þ = = × 2×10
dt 2 dt 2
or 2.303 ×1×T=1000 [\ log 10= 1]
= 3×10 –4 mol L–1s –1
1000
or T = K 7. (d) Rate of disappearance of Br–
2.303
= rate of appearance of Br2
5. (a) Rewriting the given data for the reaction
1 d[Br – ] 1 d[Br2 ]
H+ Þ– =
CH 3COCH 3 ( aq ) + Br2 ( aq ) ¾¾® 5 dt 3 dt
CH3COCH 2 Br(aq) + H + (aq) + Br - ( aq) 1 d[Br2 ] 1 d[Br – ]
Þ =-
3 dt 5 dt
S. Initial concent Initial concentr Initial concentr Rate of
d[Br2 ] 3 d[Br – ]
No. -ration of -ation of Br2 -ation of H + disappearance Þ =-
dt 5 dt
CH3 COCH3 in M in M of Br2 in Ms -1
in M d dx 8. (b) For a first order reaction
i.e. - [Br2 ]or
dt dt
0.693 0.693
1 0.30 0.05 0.05 5.7 ´ 10-5 t1/2 = ; k= = 0.5 × 10–3s–1
k 1386
2 0.30 0.10 0.05 5.7 ´ 10-5
3 0.30 0.10 0.10 1.2 ´ 10-4 9. (a) When concentration A is doubled, rate is doubled.
4 0.40 0.05 0.20 3.1 ´ 10-4 Hence order with respect to A is one.
When concentrations of both A and B are doubled,
rate increases by 8 times hence order with respect to B
Actually this reaction is autocatalyzed and involves
is 2.
complex calculation for concentration terms.
\ rate = k [A]1 [B]2
We can look at the above results in a simple way to
find the dependence of reaction rate (i.e., rate of Total order = 1 + 2 = 3
disappearance of Br2).
10. (b) N 2 O5 (g) ¾¾
® 2NO2 (g) + 1/2 O 2 (g)
From data (1) and (2) in which concentration of
CH3COCH3 and H+ remain unchanged and only the
[ N2 O5 ] = + [ NO2 ] = 2 [O2 ]
concentration of Br2 is doubled, there is no change in d 1d d

rate of reaction. It means the rate of reaction is dt 2 dt dt
independent of concentration of Br2.
[ NO2 ] = 1.25 ´ 10-2 mol L–1s–1 and
d
Again from (2) and (3) in which (CH3CO CH3) and (Br2)
dt
remain constant but H+ increases from 0.05 M to 0.10
i.e. doubled, the rate of reaction changes from 5.7×10–5
[O2 ] = 3.125 ´ 10-3 mol L–1s–1
d
to 1.2 × 10–4 (or 12 × 10–5), thus it also becomes almost
dt
doubled. It shows that rate of reaction is directly
proportional to [H+]. From (3) and (4), the rate should 11. (b)
have doubled due to increase in conc of [H+] from 0.10
Chemical Kinetics 623
12. (d) In case of (II) and (III), keeping concentration of [A]
k Ea æ 1 1ö
constant, when the concentration of [B] is doubled, 19. (b) log 2 = ç - ÷
the rate quadruples. Hence it is second order with k1 2.303R è T1 T2 ø
respect to B. In case of I & IV Keeping the
concentration of [B] constant. when the concentration Ea é 1 1 ù
-
2.303 ´ 8.314 ë 293 308 úû
ê
of [A] is increased four times, rate also increases four log 2 =
times. Hence, the order with respect to A is one. hence

Rate = k [A] [B]2 Ea 15


0.3 = ×
2.303 ´ 8.314 293 ´ 308
13. (b) 2 NO (g) + Cl2(g) ƒ 2 NOCl(g)
Rate = k [NO]2 [Cl] 0.3 ´ 2.303 ´ 8.314 ´ 293 ´ 308
Ea = .
The value of rate constant can be increased by 15
increasing the temperature. = 34673 J mole-1
\ Correct choice : (b) = 34.7 J mole-1
14. (d) order of reaction may be zero, whole number or 20. (a) It is zero order reaction
fractional.
21. (a) k = (mol L–1)1–n sec–1, n = 0, 1.
15. (b) Rate of disappearance of reactant = Rate of appearance
22. (a) Order is the sum of the power of the concentrations terms
of products
in rate law expression.
1 d[N 2O5 ] 1 d[NO 2 ] d[O 2 ] \ Order of reaction = 1+2 = 3
- = =
2 dt 4 dt dt 23. (d)
24. (a)
1 1
k[N 2 O5 ] = k ¢[N 2O5 ] = k ¢¢[N 2O5 ]
2 4 25. (b) In equation k = Ae - E a / RT ; A = Frequency factor
k = velocity constant, R = gas constant and
k k¢
= = k ¢¢ Ea = energy of activation
2 4
26. (c) Rate1 = k [A]n [B]m; Rate2 = k [2A]n [½B]m
k
k¢ = 2k, k ¢¢ = Rate 2 k[2A ]n [½B]m
2 \ =
Rate1 k[A]n [B]m
16. (a) Rate = k[A]°
= [2]n [½]m = 2n.2–m = 2n–m
Unit of k = mol L–1 sec–1
27. (b) r = k [O2][NO]2. When the volume is reduced to 1/2, The
æ T2 -T1 ö æ 100-10 ö conc. will double
r
100°C çè
10 ø
÷ çè
10 ø
÷
17. (b) =2 =2 = 29 = 512 (where 2 is \ New rate = k [2O2][2NO]2 = 8 k [O2][NO]2
r
10°C
The new rate increases by eight times.
temperature coefficient of reaction) 28. (d) The t1 / 2 is 15 minutes. To fall the concentration from
18. (b, d) 0.1 to 0.025 we need two half lives i.e., 30 minutes.
According to Arrhenius equation 29. (d) The velocity constant depends on temperature only. It is
independent of concentration of reactants.
k 2 Ea æ 1 1 ö
= -
k1 R çè T1 T2 ÷ø
ln 30. (d) Enthalpy of reaction (DH) = Ea – Ea
(f) (b)
for an endothermic reaction DH = +ve hence for DH to be
Ea æ 1 1 ö positive
= – -
R çè T2 T1 ÷ø Ea < Ea
(b) (f)
31. (d) The molecularity of a reaction is the number of reactant
molecules taking part in a single step of the reaction.
k1 E æ1 1ö
ln =- aç - ÷ Thus the reaction involving two different reactant can
K2 R è T1 T2 ø
never be unimolecular.
624 Chemistry
32. (d) The graphs show that reaction is exothermic. 39. (a) Since initial velocity is ten times the permissible value
\ A0 = 10A
-DH
log k = +I
RT 0.693 0.693
l= =
For exothermic reaction DH < 0 or t1/ 2 30

1 2.303 A 2.303 10A


\ log k Vs would be negative straight line with t1/2 = log 0 = log
T l A 0.693 / 30 A
positive slope.
2.303 ´ 30
= ´ log10 = 100 days.
2.303 1 2.303 4 0.693
33. (c) t1/4 = log = log
k 3/ 4 k 3
40. (b) The rates of reactions for the reaction
2.303 2.303 1
= (log 4 - log 3) = (2 log 2 - log 3) A ¾¾
® 2B
k k 2
can be written either as
2.303 0.29
= (2 ´ 0.301 - 0.4771) = d
k k -2 [A] with respect to ‘A’’
dt
34. (a) Since the reaction is 2nd order w.r.t Co. Thus, rate law is
given as. 1 d
or [B] with respect to ‘B’
r = k [CO]2 2 dt
Let initial concentration of CO is a i.e. [CO] = a From the above, we have
\ r1 = k (a)2 = ka2 d 1 d
when concentration becomes doubled, i.e.[CO] = 2a –2 [A] = [B]
dt 2 dt
\ r2 k (2a)2 = 4ka2 \ r2 = 4r1
d 1 d
So, the rate of reaction becomes 4 times. or – [A] = [B]
dt 4 dt
35. (c) In Arrhenius equation K = A e–E/RT, E is the energy of
activation, which is required by the colliding molecules i.e., correct answer is (b)
to react resulting in the formation of products. 41. (b) For first order reaction,
36. (b) (i) NO(g) + Br2(g) NOBr2(g) 0.693 0.693
k= =
(ii) NOBr 2 (g ) + NO (g ) ¾
¾® 2 NOBr (g ) t1/ 2 6.93
Rate law equation = k[NOBr2] [NO] 2.303 100
But NOBr2 is intermediate and must not appear in the k= log
t 100 - 99
rate law equation
0.693 2.303 100
[NOBr2 ] = log
from Ist step K C = 6.93 t 1
[NO] [Br2 ]
0.693 2.303 ´ 2
\ [NOBr2] =KC [NO] [Br2] =
6.93 t
\ Rate law equation = K . KC [NO]2 [Br2]
t = 46.06 min
hence order by reaction is 2 w.r.t. No.
42. (c) For the reaction
37. (a) Presence of catalyst does not affect enthalpy change
A ® Product
of reaction DH R = E f - E b = 180 – 200 = – 20 kJ/mol given t1/ 2 = 1 hour
38. (b) Since doubling the concentration of B does not for a zero order reaction
change half life, the reaction is of 1st order w.r.t. B.
Order of reaction with respect to A = 1 because rate of t1/ 2 =
[ A0 ]
reaction doubles when concentration of A is doubled 2k

[ A0 ]
keeping concentration of B constant.
2
\ Order of reaction = 1 + 1 = 2 and units of second or k = = = 1 mol lit –1 hr–1
2 t1/ 2 2 ´1
order reaction are L mol–1 sec–1.
Chemical Kinetics 625
Further for a zero order reaction 46. (b) For a first order reaction

dx change in concentration 2.303 a 2.303 0.1


k= = k= log = log
dt time t a- x 40 0.025

0.50 - 0.25 2.303 2.303 ´ 0.6020


1= = log 4 =
time 40 40

\ time = 0.25 hr. = 3.47 ´ 10 –2 min


–1

43. (d) Since the slow step is the rate determining step hence Rate = k[A] = 3.47 × 10–2 × 0.01
if we consider option (A) we find
= 3.47 × 10–4 M/min
Rate = k [ Cl2 ][ H 2S] 47. (a) Activation energy can be calculated from the equation
Now if we consider option (B) we find log k2 -Ea æç 1 1ö
= çç - ÷÷÷÷
log k1 2.303R è T2 T1 ø
Rate = k [ Cl2 ] éë HS ùû
-
...(i)

For equation, k
Given 2 = 2 ; T2 = 310 K ; T1 = 300 K
ˆˆ† H+ + HS–
H S ‡ˆˆ k1
2

é H + ù é HS- ù -Ea æ 1 1 ÷ö
= log 2 = çç - ÷
K = ë ûë 2.303´8.314 èç 310 300 ÷ø
û
H 2S
Ea = 53598.6 J/mol = 53.6 kJ/mol.
- K [ H 2S ] 48. (d) Overall order = sum of orders w.r.t each reactant.
or éë HS ùû =
H+ Let the order be x and y for G and H respectively

Substituting this value in equation (i) we find


[G]mole [H]mole rate(mole
Rate = k [ Cl2 ] K
[ H 2S] =k'
[ Cl2 ][ H 2S] Exp.No.
litre -1 litre -1 litre - time-1 )
+ éH+ ù
H 1 a b r
ë û
2 2a 2b 8r
Thus slow step should involve 1 molecule of Cl2 and
3 2a b 2r
1 molecule of H2S.
hence only , mechanism (A) is consistent with the
given rate equation. Q For (1) and (3), the rate is doubled when conc. of
G is doubled keeping that of H constant i.e.,
- Ea / RT
44. (c) k1 = A1e 1 .........(i) rate µ [G] \ x = 1
From (2) and (3), y = 2
- Ea / RT
k2 = A2 e 2 ........(ii) \ Overall order is 3.
On dividing eqn (i) from eqn. (ii) 49. (a) The values of rate constants k0, k1 for zero order and
first order reaction, respectively, are given by the
k1 A1 following equation:
= ( E - Ea1 ) / RT ........(iii)
k2 A2 a2
A0
k0 = [where A0 = initial concentration,
Given Ea2 = 2 Ea1 2×t1/2

On substituting this value in eqn. (iii) and t1/2 = half-life period]

Ea1 / RT 0.693
k1 = k2 A ´ e and k1 =
t1/ 2
45. (c) Since for every 10ºC rise in temperature rate doubles Substituting various given values, we get
for 50ºC rise in temperature increase in reaction rate
= 25 = 32 times 1.386 mol litre –1
k0 = ... (i)
2 ´ 20 sec
626 Chemistry

0.693
and k1 =
40sec
... (ii) 4. (a) k = 2.303 log 100 On solving we get . k = 10–3
1386 100 - 75
Dividing (ii) by (i), we get
5. (c) k un cat = A e E uncat / RT ......... (i)
k1 0.693 2´ 20 0.693
= ´ k cat = A e E cat / RT ......... (ii)
1.386 mol dm-3
40 = = 0.5
k0 1.386
Let the Ea for uncatalysed reaction be A then for catalysed
= 0.5 mol–1 dm3
reaction it will be (A - 8.314 ´ 103 ) cal/mole
Thus the correct answer is (a).
Dividing (ii) by (i),
Ea
50. (d) log k = log A - …(1) 3
2.303RT k cat e ( A -8.314´10 ) / RT
8.314´10 3
/ RT
= =e
k un cat e A / RT
1
Also given log k = 6.0 - (2000) …(2)
T k cat
= e 3.33 = 28 (take log then antilog)
On comparing equations, (1) and (2) k un cat
log A = 6.0 Þ A = 106 s–1
6. (c) Let the rate law equation be
Ea
and 2.303 R = 2000 ; r = k[A]x ...................(i)

2.25 ´ r = = k[1.5A]x ...................(ii)


ÞEa = 2000 × 2.303 × 8.314 = 38.29 kJ mol–1
Divide (ii) by (i) 2.25 = (1.5)x \ x = 2 ; Second order
51. (a) As per Arrhenius equation (k = Ae - Ea / RT ) , the rate
7. (c) N2+3H2 2NH3 ;Rate is given by any of the
constant increases exponentially with temperature.
expressions

EXERCISE 4 -d[ N 2 ] 1d[ H 2 ] 1 d ( NH 3 ]


=- =
1. (a) Substance R Substance S dt 3dt 2 dt
2k k rate constant
1
t½ 2 t½ Half life period Rate of disappearance of N 2 = the rate of formation
2
T = n × t1/2 where n = number of half life period
of NH3 = 1 × 10–4
0.5 Rate of disappearance of H2 =3/2 the rate of formation of
Amount of R left = ;
(2) T/t½ NH3 = 3 × 10–4

8. (b) k = Ae- Ea /Rt When T ® ¥ Then k = A


0.25
Amount of S left = 9. (c) The concentration of A remains 1/4 th in 8 hours. Therefore
(2) T/ 2t½
1/(2)n, n = 2
T and E½ is 8 = n × t½ ; t½ = 4 ; In 4 hours 0.6 will become 0.3
0 .5 (2) / t1/ 2 T /t
Equating both = or 2 = (2) 1/ 2
T
0.25 (2) 1/ 2
/ 2 t 10. (c) k = 1.2 ´ 1014 e -25000/RT sec -1 or

25000 1
\ T = 2 t 1 / 2 . 2t1/2 is half life of S and twice the half - log k = log 1.2 × 10 -
14
.
R T
life of R
2. (c) Units of k = (Mol lit–1) time –1 ; Hence n = 1 order

0.693 Logk
3. (d) For first order t½ = = 100 min ; 300 = 100 × n
-3
6.93 ´10
1/T
100 Equation of straight line
Percentage left = = 12.5%
( 2) 3
2500
slope = -
R
Chemical Kinetics 627

11. (a) For the change 2A + 3B ® products 17. (d) N 2 O 4 2NO 2 . Decomposition of N2O4 is first
order and its concentration will will decrease with time.
1 d[ A ] 1 d[ B] 1 1
- =- , r1 = r2 ; 3r1 = 2r2
2 dt 3 dt 2 3 18. (b) For zero order

2.303 100 Co - C t
12. (c) t 99.9% = log k=
k 100 - 99.9 t

2.303 100 2.303 2.303 19. (b) Rate = kC 2A C½


= log = log1000 = ´3 B
k 0.1 k k
Rate = k[2C A ]2 [4C B ]½ = 4 ´ 2[C A ]2 [C B ]½
2.303 100
t 50.0% = log
k 100 - 50
20. (c) k = 1 ln a or ln a = kt
t a-x a-x
2.303 2.303
= log 2 = ´ 0.3010
k k
a-x
ln = - kt or a - x = a e - kt
t 99.9% 2.303 ´ 3 k 3 a
= ´ = = 10
t 50.0% k 2.303 ´ 0.3010 0.3
x
\ x = a(1 - e - kt ) or = (1 - e - kt )
t 99.9% = 10 ´ t 50.0% a

13. (c) For the exothermic reaction the energy of products is U2


always less than the reactants. If Ea is the energy of Ea
activation for the forward reaction, the energy of 21. (a) U1 DU = U2 – U1 = +ve
activation for backward reaction is Ea + DH
14. (d) The relevant expressions are as follows. 22. (a) For zero order reaction the t1/2 µ a.
DH 1 23. (b) A 2
Choice (A) log Kp = - +I 2A (fast) ....................(i)
R T
K
Choice (B) log [X] = log [X]0 + kt A + B ¾¾®
¾2 P(slow ) .............(ii)
Choice (C) P/T = constant
Rate law = k2[A][B]
(V constant)
But A is intermediate
Choice (D) PV = constant
(T constant) [A]2
15. (d) For two parallel reactions \ from k = \ [A] = k ½ [A 2 ]½
[A 2 ]
k average = k 1 + k 2 = 1.26 × 10 -4 + 3.8 × 10 - 5 put in rate law equation

= 1.64 × 10 - 4 sec -1 Rate law = k 2 k ½ [A 2 ]½ [B]

kB 1.26 ´ 10 - 4 24. (d)


The fractional yield of B = =
k av 1.64 ´ 10 - 4 25. (a) A (g ) + 2B(g ) ¾
¾® C(g ) + D(g )
0.7683 = 76.83 %

-5
Rate = k[A][B]2 = k(0.60)(0.80)2 .......................(i)
kC 3.8 ´ 10
The fractional yield of C = =
k av 1.64 ´ 10 - 4 when p c = 0.20 atm p A is reduced to 0.40 and
pB = 0.40 (See stoichiometric representation)
= 0.2317 = 23.17 %
Rate = k[0.40] [0.40]2..............................(ii)
2.303 [A]0 2.303 800
16. (c) k = log = log = 1.386 ´ 10 4 s -1 0.40 ´ 0.40 ´ 0.40 1
t [A]t 2 ´ 10 4 50 \ (ii) divide by (i) = =
0.60 ´ 0.80 ´ 0.80 6
628 Chemistry

26. (a) The rate law equation contains [H + ]2 , term. The rate k2 E a (T2 - T1 )
36. (d) ln =
k1 R T2 ´ T1
will change with change in pH and new rate will be [2]2
= 4 times the old rate 3.07 ´10-3 E a (510.1 - 457.6)
ln =
Ea 2.2 ´10 -5 R 510.1´ 457.6
1
27. (b) log k = log A – .
2.303 T \ Ea = (510.1 × 457.6) / 52.5 × R ln 307/2.2 cal.
The value of k increases with the increase of temperature.
37. (c) CH 3CHO (g ) ¾
¾® CH 4 (g) + CO (g)
28. (c) Generally the rate of reactions is doubled by 10°C rise in
temperature i.e. the increase is » 100 %.
When t = 0 p0 0 0
29. (b) The curve Y shows the increase in concentration of
products with time. When t = t p0 - p p p
30. (a) The increase in pressure shows the increase in conc. of Z.
120 - 100 \ p 0 - p + p + p = 120 mm Hg
Rate of appearance of Z = = 4 mm Hg min -1
5
or, p 0 + p = 120 mm Hg;
Rate of disappearance of X 2 = 2 × rate of appearance of
p = 120 – 80 = 40 mm Hg
Z = 2 × 4 mm Hg min -1 = 8 mm Hg min -1
31. (c) A dip in the curve shows the formation of X (reaction 1 p0 1 80 1
k= ln = ln = ln 2
taking place in two steps). Since the reaction is exothermic t p -p
0 20 80 - 40 20
Ea of X must be less.
ln2
K 310 Again, t1 / 2 =
32. (b) Temperature coefficient = = 2.3 k
K 300
ln2
K 310 = 2.3 K 300 \ t1 / 2 = ´ 20 = 20 min.
ln2
1
33. (c) t1 / 2 µ
6 ´ 10 -7
where n is the order of reaction
( p) n -1 38. (b) [H + ] = = 1.2 ´ 10 -2 M
-3
0.05 ´ 10
n -1 n -1
2 æ 50 ö 1 æ1ö
=ç ÷ or =ç ÷ DX DX 1.2 ´ 10 -2 M
4 è 100 ø 2 è2ø Rate r = ; Dt = =
Dt r 6 ´ 105 M s -1
\ n=2
-8
C0 - C \ T = 2 ´ 10 s
34. (a) For zero order reaction k = or C = C0 – kt
t
k 2 E 2 + k 1 E1
The equation has negative slope and non zero intercept. 39. (c) Ea =
k 2 + k1
n -1
t1 / 2 (1) é p (2) ù
35. (c)
t1 / 2 (2) êë p (1) úû
= , 1.0 ´10 - 2 ´ 60 + 3.0 ´10 - 2 ´ 70
=
1.0 ´10 - 2 + 3.0 ´10 - 2
n -1
880 é 364 ù Ea = 67.5 k J mol -1
= ê ú ;
410 ë 170 û

[2.14]1 = [2.1]n -1
19
Surface Chemistry

Thomas Graham divided the solids into two types. TYPES OF COLLOIDAL SYSTEMS :
CRYSTALLOIDS : Dispersed Dispersion Name Examples
The substances whose solution readily diffuse through a phase medium
parchment membrane eg sugars, salts, acid and base. Gas Liquid Foam Shaving Cream,
COLLOIDS : Soda Water froth
The substance whose solution diffused at very slow rate through Gas Solid Solid foam Pumice stone,
a parchment membrane eg glue, gelatin, gum arabic etc. corck, foam rubber
Later on it was found that crystalloids and colloids differed only Liquid Gas Aerosol Mist , cloud, fog
in their size of particles. They can be interconverted, hence Liquid Liquid Emulsion Hair cream, milk
colloidal is a state.
Liquid Solid Gel Cheese, butter
On the basis of the size of particles we have
Solid Gas Smoke Occluded gases,
1nm 100nm dust soot in air
1Å 10Å 1000Å 10,000Å Solid Liquid Sol Colloidal gold
Solid Solid Solid sol alloys

True solution Colloidal Solution Suspension HYDROSOLS OR AQUASOLS :


For such sols the dispersion medium is water.
COLLOIDAL SYSTEM : SOLVOSOLS :
Finely divided particles of any substance with diameters lying Depending upon the nature of the dispersion medium sols may be
between 10Å - 2000Å dispersed in any medium constitute a named as alcosols (in alcohol), benzosols (in benzene). Aerosols
colloidal system. It is heterogenous system consisting of two (in air) and in general solvosols.
phases.
LYOPHILIC SOLS ( Solvent loving) :
DISPERSED PHASE :
When dispersed phase has certain affinity for the dispersion
Discontinuous phase of the colloidal system is known as medium it is known as lyophilic sol. eg sols of gum, starch etc.
dispersed phase.
LYOPHOBIC SOLS (Solvent Hating) :
DISPERSION MEDIUM :
When dispersed phase has no affinity for dispersion medium
The continuous phase of the colloidal system is known as it is known as lyophobic sol. eg. sols of gold, iron and sulphur
dispersion medium. etc.
630 Chemistry
CHARACTERISTICS OF LYOPHILIC AND LYOPHOBIC AgNO + Tannic acid ® Ag Sol
SOLS :
AuCl3 + Tannic acid ® Au Sol
Property Lyophilic Sol Lyophobic Sol (iv) Hydrolysis - Ferric hydroxide sol is prepared by pouring
1 . Preparation Can be prepared easily Prepared by special methods dilute solution of ferric chloride into boiling water
2 . Charge May or may not carry Always carry positive or
any charge negative charge. FeCl3 + 3H 2 O ¾¾
® Fe(OH)3 + 3HCl
3. Solvation Sol particles are Not solvated. Red Sol
(Positive in nature)
heavily hydrated
4. Precipitation High concentration Precipitated by low Sols of chromium and aluminium can also be prepared
of the electrolytes concentration by this method.
required to cause . of electrolytes.
precipitation
(v) Change of solvent - When ethanolic solution of sulphur is
5. Viscosity Very high Same as that of dispersion added to an excess of water, the sol of sulphur is obtained.
medium. This is physical method.
6. Surface tension Low Same as that of dispersion (b) Dispersion methods :
medium. Here lumps of the substances broken down to colloidal size
7. Reversibility Reversible Irreversible
in presence of dispersion medium and suitable stabilizer.
8. Migration in May or may Always migrate in electric
(i) Mechanical dispersion - In this method colloid mill, ball mill
electric field not migrate field. or ultrasonic disintegrator are used.
9 . Tyndall effect No Yes (ii) Bredig’s arc method (electrical disintegration). An arc is
10. Stability More stable Less stable struck between two metal electrodes of silver, gold or platinum
CLASSIFICATION ON THE BASIS OF SIZE OF SOL held at the surface of cold water containing traces of alkali
when sol of metal is obtained.
PARTICLES :
(i) Multimolecular colloids - In this case the colloidal particles Electrodes
consists of aggregates of atoms or small molecules with
diameters of less than 1nm eg gold sol, sulphur sol.
(ii) Macromolecular colloids - In this case dispersed particles
are of colloidal size and are called macromolecules, usually
Dispersion
polymers such as starch, cellulose, proteins or synthetic medium
polymers etc. Ice-bath
(iii) Associated colloids - At low concentrations they behave as
electrolytes and at higher concentrations as colloids. They
are also known as micelles. Soaps and detergents are the
Bredig’s Arc method
examples. The micelles may contain about 100 molecules or
more. (iii) By peptization - The dispersal of freshly precipitated
PREPARATION OF COLLOIDAL SOLUTIONS : substance into colloidal solution by the addition of some
Lyophobic sols and lyophilic sols are prepared by different electrolyte having one ion common is known as peptization.
methods. The electrolyte used is called peptizing agent e.g.
Preparation of lyophobic sols - The methods employed are : (i) Ferric hydroxide Fe(OH)3 is peptized by ferric chloride
(a) Condensation methods (b) Dispersion methods. giving sol. of [Fe(OH)3]Fe3+
(a) Condensation methods : (ii) Silver chloride AgCl is peptized by HCl giving negative
In these methods small ions or molecules are induced to combine sol of [AgCl]Cl–
together to form aggregates of colloidal size and chemical or (iii) Cadmium sulphide CdS is peptized by H2S giving
physical methods may be employed. negative sol of [CdS]S– –
(iv) Cellulose nitrate is peptized by a mixture of ethanol and
Chemical methods :
water. The product obtained is called “colloidon”.
(i) Double decomposition - An arsenic sulphide (As2S3) sol is
prepared by passing H2S through cold solution of As2O3 till PURIFICATION OF COLLOIDAL SOLUTIONS :
yellow colour deepens to its maximum The removal of the particles of electrolytes from sols is known as
purification of sols. It can be achieved by any of the following
As 2 O 3 + 3H 2S ® As 2S3 (Sol ) + 3H 2 O
methods.
(ii) Oxidation - A sol of sulphur is prepared by passing H2S into a) Dialysis b) Electrodialysis c) Ultra Filtration
solution of SO2 Dialysis : A mixture containing colloidal particles and particles of
2H 2S + SO 2 ® 2 H 2 O + S ¯ true solution is placed in a parchment bag. The bag is hanged in
water vessel through which it is continously flowing. The particles
(iii) Reduction - Sols of gold, platinum and silver are prepared of true solution come out of membrane leaving behind colloidal
by reduction of their compounds in water using, solution.
formaldehyde or hydrazine or tannic acid
Surface Chemistry 631
(d) Electrical properties
Water + Crystalloid (i) Presence of charge - All the particles of dispersed phase
carry a positive or negative charge and dispersion medium
carry the opposite charge. Sol as a whole is neutral. The
origin of the charge may be due to
Water (a) Preferential adsorption of the common ion at the surface
of sol particles eg.
Electrodialysis : The process is the same as above. The vessel is Fe(OH)3 + Fe 3+ ® [Fe(OH )3 ]Fe 3+ (+ ve sol )
fitted with electrodes which make the removal of electrolytes
quick. On applying the EMF ions of electrolyte migrate out to the As 2S3 + S 2 - ® [ As 2S 3 ]S 2 - ( - ve sol )
oppositely charged electrodes while the colloidal particles are In Bredig’s Arc method the negative charge on the metal sol
is due to adsorption of hydroxyl ions furnished by alkali.
held back.
(b) Ionisation of surface groups
(i) Charge on protein sols - Protein sols may be negative,
Water + Electrolyte positive or neutral depending upon the pH value of the
+ solution
Anode + + Cathode
+
Water +

Ultrafiltration : The pores of ordinary filter paper are made smaller


in size by treating with solution of gelatin or collodion (4% Positive sol. (Low pH) Neutral sol. (Zwitter ion)
Isoelectric point
solution of nitro cellulose in mixture of alcohol and ether)
hardening by formaldehyde and drying. The colloidal particles
left on the filter paper are then stirred with fresh dispersion medium
to get pure colloidal solution.
Negative sol. (High pH)
Pressure At Low pH, concentration of H+ is large, COOH group is
not ionised and NH2 group get protonated, hence
positive sol.
Colloide At high pH, concentration of OH– is large, COOH group
is ionised and NH2 group is not protonated, hence
Ultra filter
negative sol.
At intermediate pH, protein molecule exist as zwitter
ion, hence no charge.
+
(ii) Charge on soaps and detergents - In soaps -COONa
Electrolyte +
and in detergents -OSO3- N a constitute the surface.
Thus due to their ionisation soaps and detergents acquire
PROPERTIES OF SOLS : negative charge.
The following are the properties given by sols (iii)Charge on basic and acidic dyes - An acidic dye ionises
(a) Physical (b) Colligative (c) Optical to give H+ ions and acquire negative charge and a basic
(d) Electrical and (e) Kinetic dye ionises to give OH– ions and acquire positive charge.
(a) Physical properties (c) Frictional electrification : It is due to rubbing of particles of
(b) Colligative properties - Except osmotic pressure other dispersed phase with particles of dispersion medium.
Some examples of positively and negatively charged sols.
colligative properties are little affected.
Positively Charged sols Negatively charged sols
(c) Optical properties - tyndal effect - The phenomenon of the
1. Hydroxide of Al, Cr and Fe Metallic Sols eg. Pt, Cu, Ag and
scattering of light by sol particles in all possible directions Al(OH)3, Cr(OH)3 and Au etc.
is called Tyndal effect. Fe(OH)3
2. Basic dyes eg. methylene blue. Sulphur, sulphides (CdS, As2S3),
Silicic acid etc
3. Proteins in acid solution Gelatin, Starch, acid dyes
eg. haemoglobin (eg. eosin) clay
(ii) Helmholtz electrical double layer - Each sol particle is
The colloidal particles absorb the incident light energy, become surrounded by either positive or negatve ions in the form of
self-luminious, and scatter the light in all directions. Blue colour fixed layer or compact layer. The second layer is diffuse or
of sky and sea water, visibility of projector path etc are due to mobile layer consisting of ions of both the signs but net
this effect. charge being equal and opposite to the fixed layer. This is
known as Helmholtz electrical double layer.
632 Chemistry
Zeta potential - The potential difference developed between Factors affecting Brownian movement
the two layers is known as zeta potential or electric kinetic (i) Size of particles - Small particles execute more rapid, brisk
potential. and vigrous motion than larger particles.
(ii) Temperature - It is increases with temperature.
- + - Importance
- - (i) Stability of colloidal solution
+ +
+ + +
(ii) Confirmation of kinetic theory
- Positive -
-
(iii) Determination of avogadro number
- + sol
+
+ +
Brownian movement is due to the bombardment of the
+ - particles of the dispersion medium on the particles of
-
- + - dispersed phase.
(iii) Electrophoresis - The movement of sol particles under an COAGULATION, PRECIPITATION
applied electric potential is called electrophoresis or OR FLOCCULATION:
cataphoresis. The phenomenon helps in The settling down of the discharged sol particles is called
Reservoir coagulation or precipitation of the sol. It can be achieved by
(i) Electrophoresis
(ii) Addition of electrolytes
Anode Cathode (iii) By boiling
(iv) Mixing two sols of opposite charge
(v) By persistant dialysis
HARDY SCHULZE RULE :
Initial level
It states that the precipitating effect of an ion increases with the
Water valency of the ion.
(dispersion Colloidal For precipitating a negative sol the precipitating power of cations
medium) solution follow the order
Al 3+ > Ba 2 + > Na +
For precipitating a positive sol the precipitating power of anions
follow the order
Stop cock
[Fe(CN)6 ]4 - > PO34- > SO24 - > Cl-
Electrophoresis Blood is positively charged sol (pH=7.4) and is coagulated by
alum, Al2(SO4)3 or FeCl3. These salts lower the pH and denaturate
(a) removing suspended impurities
globular proteins.
(b) removing smoke from chimney gases
(c) electroplating of rubber FLOCCULATION VALUE :
(d) painting metals with colloidal pigments The minimum concentration in millimoles per litre required to cause
(e) coagulation of sols the precipitation of a sol in 2 hours. The smaller the flocculation
(f) determination of charge value, the higher is the precipitating power of an ion.
(iv) Electro-osmosis - The movement of the dispersion medium PROTECTIVE ACTION OF LYOPHILIC SOLS :
under the influence of applied electric potential is known as The lyophobic sols are less stable than lyophilic sols. The lyophilic
electro osmosis. The phenomenon helps in sols are thus used to protect the lyophobic sols. This property of
(a) Removing water from peat lyophilic sols is known as protective action of lyophilic sols. A
(b) Dewatering of moist clay little gelatin stabilises As2S3 sol.
(c) Drying dye pastes.
GOLD NUMBER :
(v) Kinetic properties - Brownian movement - The zig - zag
movement of colloidal particles in the dispersion medium is The number of milligrams of a hydrophilic colloid that will just
called Brownian movement. prevent the precipitation of 10 ml of a gold sol on the addition of
1 ml of 10% sodium chloride solution is known as gold number.
The smaller the value of gold number of lyophilic sol, the greater
is the protective action.
Gold number of some lyophilic sols.
1. Gelatin 0.005 - 0.01
2. Caseinate 0.01
3. Haemoglobin 0.03
4. Gum arabic 0.15
5. Sodium Oleate 0.4
Brownian movement 6. Potato starch 25
Surface Chemistry 633
STABILITY OF SOLS : (iii) Isoelectric point - The concentration at which the colloidal
It is mainly due to two factors particles have no charge is known as the isoelectric point.
(i) Presence of like charge on sol particles. It prevents them The isoelectric point of gelatin is at pH 4.7.
from aggregating and settling down under the influence of (iv) U - numbers - The number of mgs of a hydrophilic sol which
gravity. is sufficient to produce the colour change from red to blue in
(ii) Solvation of sol particles - In case of lyophilic sols a 10 cc of gold sol.
protective layer of solvent is formed around sol particles in (v) Congo - rubin number - The number of mgs of protective
addition to charge. Hence they are more stable than lyophobic colloid which prevents the colour change in 100 ml of 0.01 %
sols. congo rubin dye to which 0.16 g equivalent of KCl is added
EMULSIONS : ASSOCIATED COLLOIDS :
Emulsion is a liquid liquid system. They are of two types The molecules of certain substances are smaller than colloidal
(i) Oil dispersed in water (O/W type) eg milk and vanishing particles but aggregate spontaneously in a given solvent to form
cream particles of colloidal size. The aggregates are known as micelles
(ii) Water dispersed in oil (W/O type) eg butter and cream. and the substances as associated colloids. The soaps and
The dispersed phase is always in the form of small droplets detergents are the examples.
which increases the surface area and makes the emulsion
unstable. Preparation of an emulsion is called Emulsification.
Emulsions are generally unstable and are stabilized by the +
Na
use of emulsifying agents (emulsifiers). For O/W emulsions
Insoluble tail Polar head
the emulsifiers are proteins, gums, natural and synthetic Soap molecule
soaps. For W/O emulsions the emulsifiers are heavy metal e.g. C17H35 COONa
salts of fatty acids, and long chain alcohols etc. The
emulsifier concentrates at the interface, reduces surface A soap micelle
tension on the side of one liquid.
PROPERTIES OF EMULSIONS : The tails being insoluble in water are directed towards the centre
(i) They show Tyndal effect and Brownian movement. and soluble polar heads are on the surface in contact with water.
(ii) They can be demulsified (broken) by heating, adding The charge stabilises the micelle. The minimum concentration
electrolytes, freezing and centrifuging required for the micelle formation is called critical micellisation
(iii) They can be diluted with dispersion medium. concentration (CMC).
GELS : CLEANSING ACTION OF SOAP :
The liquid solid system is called gel. They are of two types It may be due the following two factors
(a) Elastic gels - They can be temporarily deformed by applying (i) Solubilisation of grease into the micelle
force eg Gelatin, starch and soaps. (ii) Emulsification of grease
(b) Non elastic gels - They are rigid eg silica gel. When soap is applied on to a fabric, the tails of the soap anions
PROPERTIES OF GELS : are pegged ino the grease stains and polar head form a charged
(a) Syneresis - Shrinkage of gels on standing by exudation of layer around it. By mutual repulsion the grease droplets are
solvent is known as syneresis suspended in water (formation of emulsion) and are washed away
(b) Thixotropy - Certain gels when shaken form a sol and on with water.
standing are converted into the form of gel They are known APPLICATIONS OF COLLOIDS :
as thixotropic gels and sol - gel transofrmation is known as (1) Industrial applications
thixotropy. (a) Purification of drinking water - By adding alum, the
(c) Swelling or imbibition of gels - The property of adsorbing suspended impurities are coagulated and removed.
definite amount of water and causing the volume of gel to (b) Electrical precipitation of smoke - Smoke carry negative
increase is known as swelling or imbibition. charge and is removed by the principle of electrophoresis
SOME USEFUL DEFINITIONS : in cottrell’s precipitator.
(i) Streaming potential - When liquid is forced through a porous (c) Sewage disposal - It is passed through big tanks fitted
material or a capillary tube, a potential difference is set up with electrodes. The colloidal particles lose their charge
between the two sides. This is called streaming potential. and settled down and removed.
(ii) Sedimentation potential or dorn effect - When a particle is (d) Electroplating of rubber - Latex is colloidal suspension
forced to move in a resting liquid a potential difference is set of negatively charged rubber particles in water and can
up which is known as sedimentation potential or dorn effect. be deposited on metals by electrophoresis.
634 Chemistry
(e) Artificial rains - Clouds are aerosols (water dispersed (ii) Heterogeneous catalysis - When the catalyst is in
in air). Aggregates of particles of water cause the rain a different phase than that of reactants and products the
fall which can be artificially achieved by throwing process is known as heterogeneous catalysis eg.
electrified sand or AgI on clouds and cause the artificial (a) Formation of ammonia by Haber’s process
rain. AgI has similar crystal structure as that of ice. N 2 + 3H 2 + [Fe] ® 2 NH3 + [Fe]
(f) Leather tanning - Skin of animals is positively charged gas gas solid gas solid
colloidal system. Extract of barks, wood leaves is (b) Formation of sulphur trioxide (contact process for
negatively charged colloidal solution of tannin. When sulphuric acid)
latter is applied on the surface of skin (leather) it becomes
2SO2 + O2 + [Pt] ® 2SO3 + [Pt]
hard and does not putrefy. gas gas solid gas solid
(g) In warfare - Animal charcoal is used in gas masks to
(c) In solution decomposition of hydrogen peroxide by
adsorb poisonous gases. Smoke screens are titanium
MnO2 or Pt
oxide particles dispersed in air.
2H 2 O 2 + [Pt ] ® 2H 2 O+ O 2 + [Pt ]
(2) In everyday life - Blood, milk, butter, cheese, clothes, shoes liquid solid
all are colloidal system.
(3) In medicines - Colloidal medicines are easily adsorbed and CLASSIFICATION OF CATALYSTS :
assimilated, hence are widely used. Colloidal antimony is According to their mode of action they have been classified into
effective medicine for kalazar. Blood is coagulated by FeCl3. four groups.
Colloidal sols of Ag (argyrol and protargol) are used as eye (i) Positive Catalysts - They always accelerate the speed of a
lotions. chemical reaction eg.
(a) Decomposition of potassium chlorate in presence of
(4) In nature - Blue colour of sky, tails of comets are due to
manganese dioxide
scattering of light.
2KClO + [MnO 2 ] ® 2KCl + O 2 + [MnO 2 ]
Formation of deltas in rivers is due to coagulation of
negatively charged sand particles by Na +, Mg2+ etc present (b) Manufacture of ammonia in presence of iron
in sea water. N 2 + 3H 2 + [ Fe] ¾
¾® 2 NH 3 + [ Fe ]
(ii) Negative Catalysts :
CATALYSIS They always retard the speed of chemical reaction eg
CATALYST : (a) Oxidation of chloroform in presence of ethanol
A substance which when added in very small amount to a chemical ®
4CHCl3 + 3O 2 + [C 2 H 5OH] ®
reaction change the speed of that reaction without itself
undergoing any chemical change is known as catalyst and the 4COCl 2 + 2Cl 2 + 2H 2 O + [ C 2 H 5 OH ]
phenomenon is known as catalysis. (b) Decomposition of hydrogen peroxide in presence of
dilute acids or glycerol
TYPES OF CATALYSIS :
2H 2O 2 + [H 3PO 4 ] ® 2H 2 O + O 2 + [H 3 PO 4 ]
The catalysis may be of two types
(i) Homogeneous Catalysis - When catalyst, reactants and the (iii) Auto catalyst - In this case one of the products act as a
products all are present in one phase, the process is known catalyst eg.
as homogeneous catalysis eg. Oxidation of oxalic acid by acidic permangnate
(a) Oxidation of sulphur dioxide to sulphur trioxide with
nitric oxide 2KMnO 4 + 3H 2SO 4 ® 2MnSO 4 + K 2SO 4 + 3H 2O + 5O

2SO 2 + O 2 + [ NO] ® 2SO 3 + [ NO] COOH


gas gas gas gas gas 5 | + 5 O ® 10 CO 2 + 5H 2 O
COOH
(b) Combination of hydrogen and chlorine in presence of The speed of the reaction is slow in the begining but
steam increases rapidly due to the formation of Mn ++ ions which
H 2 + Cl2 + [H 2O] ® 2HCl+ [H 2O] act as auto catalyst.
gas gas gas gas gas (iv) Induced Catalyst - A substance which induces the similar
reaction in another which is otherwise is not possible act an
(c) Hydrolysis of an ester in presence of acid or alkali
induced catalyst eg sodium sulphite solution oxidises in air
to sodium sulphate but sodium arsenite is not oxidised by
CH 3COOC 2 H 5 + H 2O air. In presence of sodium sulphite it is also oxidised.
liquid liquid
[O]
Na 2SO 3 + Na 3 AsO 3 ¾¾®
¾ Na 2SO 4 + Na 3 AsO 4
H+ /OH- air
CH3COOC 2 H5 + H 2O ¾¾¾¾¾® CH 3COOH +C 2H 5OH Therefore sodium sulphite is induced catalyst.
liquid liquid liquid liquid
Surface Chemistry 635
(v) Catalyst promoters - The substances which increase the (i) Intermediate theory - A catalyst combines with one of the
activity of catalyst are known as catalyst promoters eg reactants and forms the unstable intermediate compound
Molybdenum (Mo) or aluminium oxide (Al2O3) increase the which in turn combines with another reactant and generates
activity of iron catalyst in Haber ’s process for the the catalyst eg. consider the oxidation of SO2 by O2 in
manufacture of ammonia presence of NO as catalyst

2 N 2 + 3H 2 ¾¾ ¾
Fe / Mo
¾® 2 NH 3 2SO 2 + O 2 + [ NO] ® 2SO 3 + [ NO]
(vi) Catalyst poisoning - The substance which destroys the Intermediate mechanism
activity of a catalyst is called a poison and process is called O2 + 2NO ® 2NO 2
catalytic poisoning eg. Catalyst Intermediate
pt
2SO 2 + O 2 ¾¾ ¾ ¾ ¾
¾® 2SO 3 [SO 2 + NO 2 ® SO 3 + NO] ´ 2
poisoned by As 2O 3
2SO 2 + O 2 + [2NO] ® 2SO 3 + 2[ NO]
pt
2H 2 + O 2 ¾¾ ¾ ¾
¾® 2H 2 O
poisoned by CO Catalytic oxidation of hydrochloric acid by atmospheric
oxygen in presence of cupric chloride (Deacon’s process)
CHARACTERISTICS OF CATALYST :
CuCl
The following are some important features for a catalyst or HCl + O 2 ¾¾¾2 ® 2Cl 2 + 2H 2 O
catalysis. Mechanism - (2CuCl2 ® Cu2Cl2 + Cl2) × 2
(i) Small quantity - Generally small amount of a catalyst is 2Cu 2Cl 2 + O 2 ® 2Cu 2 .OCl 2
required but sometimes the large quantity of catalyst is Intermediate
effective eg anhydrous AlCl3 in Friedel Craft’s reaction is
needed upto 30%. (Cu 2 OCl 2 + 2HCl ® 2CuCl 2 + H 2 O) ´ 2
(ii) Unchangeabiliy - Although there is no change in the chemical 4HCl + O 2 + [4CuCl 2 ] ® 2Cl 2 + 2H 2 O + [4CuCl 2 ]
compostion and mass, there may be a change in physical Limitations of intermediate theory-
state of catalyst eg granular MnO2 changes to fine powder (a) Action of promoters and poisons.
in decomposition of potassium chlorate (b) Participation of catalyst in chemical reaction.
(iii) Specific action of catalyst - From the same reactants the (ii) Adsorption theory - By the adsorption of the reactants at the
different products are obtained with different catalysts eg. surface of a catalyst, their concentrations are increased. The
(a) C 2 H 5 OH ¾Al
¾2¾
O3
¾® C 2 H 4 + H 2 O ( Dehydratio n ) rate being directly proportional to concentration, is increased.
D (iii) Modern theory - It is the combination of intermediate theory
C2H5OH ¾¾¾
hot
® CH3CHO + H 2 (Dehydrogenation) and adsorption theory. At the surface of a catalyst there are
Cu,300ºC residual or free valencies and reactant molecules are retained
at the surface in distorted or strained positions. These
(b) 4 NH 3 + 3O 2 ¾Cu
¾® 2 N 2 + 6H 2 O molecules then react vigrously to give products and make
500º C
space for other reactant molecules. The greater the surface
Pt
4 NH 3 + 5O 2 ¾¾® 4 NO + 6 H 2 O area of the catalyst, the more is its activity. Further catalysts
800º C
with rough surface, having greater number of corners, peaks,
(iv) Initiation of reaction - In general a catalyst does not start a cracks have more active centres and more effective e.g. finely
chemical reaction, but there are reactions initiated by catalysts divided nickel or platinum
eg.
Active spot
H 2 + O 2 ¾Room
¾ ¾Temp
¾¾.® No reaction
Pt Black Ni
2H 2 + O 2 ¾¾ ¾¾® 2 H 2 O
(v) Equilibrium - A catalyst does not affect the state of Ni Ni Active spot Ni
equilibrium, it simply helps to attain it quickly.
Ni Ni Active spot Ni Ni Ni Ni
(vi) Optimum temperature - The rate of reaction is maximum at a
particular temperature which is fixed for every reaction. Ni Ni Ni Ni Ni Ni
(vii) Physical state of catalyst - Finely divided catalyst gives better
yield in a short period than lumps.
(viii) Activation energy - A positive catalyst lowers the activation ACTION OF PROMOTERS :
energy of a chemical reaction. It can be assumed that a loose compound is formed between the
catalyst and the promoter which possesses an increased capacity
THEORY OF CATALYSIS : of adsorption.
There are following main theories Action of catalytic poison - It may be due to the preferential
(i) Intermediate theory adsorption of poison on the active spots of the catalyst thus
(ii) Adsorption theory reducing the number of free active spots for reacting molecules.
(iii) Modern theory
636 Chemistry
ACID BASE CATALYSIS :
A large number of reactions catalysed by acids or bases are known
ADSORPTION
eg. ADSOPRTION :
(a) Hydrolysis of an ester in presence of an acid Adsorption is the phenomenon of higher concentration of
H+
molecular species at the surface than in the bulk of a solid or
CH 3COOC 2 H 5 + H 2 O ¾¾
¾® CH 3COOH + C 2 H 5 OH
liquid. Forces involved are van der waal’s forces.
(b) Cyanides are hydrolysed by acids or alkalis The unbalanced or residual forces acting along the surface of a
solid or liquid attract the moleculs of other species when brought
H+ +H O
RCN + H2O ¾¾¾
®RCONH2 ¾¾¾
2 ¾
®RCOOH + NH3 into their contact. These molecules remain at surface and do not
-
or OH
go into bulk, hence making the concentration more at surface.
Such reactions are called acid base catalysis and H+ or OH–
ADSORBATE :
ions act as catalyst.
The substance that concentrates at the surface is called adsorbate.
MODERN CONCEPT OF ACID BASE CATALYSIS:
ADSORBENT :
According to this concept
(i) A reaction which is catalysed by an acid is also catalysed by Adsorbent is the solid or liquid at whose surface the concentration
all substances having tendency to lose protons (H+) occurs.
(ii) A reaction which is catalysed by a base is also catalysed by ABSORPTION :
all substances having a tendency to gain protons. Absorption is the phenomenon of uniform distribution of a
ENZYMES OR BIOLOGICAL CATALYSTS : substance throughout the body of a solid or liquid. For eg water
Many complex chemical reactions like oxidation, reduction or vapour is absorbed by a sponge or anhydrous CaCl2 while it is
hydrolysis take place in presence of enzymes which are highly adsorbed by silicagel. The pressure of gases (SO2, Cl2, NH3)
complex, nitrogenous non living organic substances. They are pressure decreases when treated with powdered charcoal in a
highly specific in nature eg. closed vessel due to adsorption of gas on charcoal surface.
(a) Starch is catalysed by diastase maltose is formed and then SORPTION :
maltose by maltase
Adsorption and absorption go hand in hand thus making it tough
Diastase
2(C 6 H10 O5 ) n + n H 2 O ¾¾ ¾¾® n C12 H 22 O11 to distinguish between them. The general term sorption was
Maltose introduced by McBain to include both processes.

Maltase
CHARACTERISTICS OF ADSORPTION :
C12H 22O11 + H 2O ¾¾¾¾
® 2C6H12O6
(i) It is a surface phenomenon.
Glucose
(ii) It is a spontaneous process.
(b) Urea is converted into ammonium carbonate by Urease (iii) Adsorption is accompanied by evolution of heat as the
H 2 N.CO.NH 2 + 2 H 2 O ¾¾ ¾
Urease
¾®( NH 4 ) 2 CO 3 residual forces acting along the surface of adsorbent
decrease.
MECHANISM OF ENZYME ACTION : Heat evolved on adsorption of 1 mole of a gas is molar heat
Enzyme can increase the rate of biochemical reactions by 106 to of adsorption.
1012 times. They are highly selective and specific and act on
(iv) Adsorption is accompanied by decrease in enthalpy. i.e. D H
certain molecules called substrates and leave the rest of the system
= -ve
unaffected (LOCK AND KEY THEORY). Michaelis and Menten
proposed the following mechanism for enzyme catalysis (v) Adsorption is accompanied by decrease in entropy i.e. TDG
is negative.
E + S ES ¾
¾® P + E
Enzyme Substrate Activated Complex Product Enzyme Also, DG = DH - TDS. As D H is negative and so is DS,
Þ DG is negative. But experimentally DH becomes less
CHARACTERISITCS OF ENZYME CATALYSIS :
negative as the adsorption progresses and ultimately
1. Enzymes are most efficient catalysts.
becomes zero as the equilibrium is reached.
2. Enzymes are very selective and specific.
Adsorption of gases at the surface of metals like Ni, Pt, Pd
3. Enzymes require optimum condition of temperature and pH etc is called Occlusion.
of the medium.
4. Enzymes are easily poisoned by addition of other substances PHYSICAL ADSORPTION :
particularly heavy metal ions (Hg2+ or Ag+) It is also called van der Waal’s adsorption which involves weak
5. Activity of enzymes is increased by activators (certain metal forces, physical in nature, with small heat of adsorption. For
ions Co2+, Mn2+, Na+, Cu++ etc) or coenzymes. example adsorption of H2 or O2 on charcoal.
Surface Chemistry 637
CHEMICAL ADSORPTION :
x
Forces of attraction between adsorbate and adsorbent being of The relation = k ( P )T solid, gas is called adsorption
m
the same strength as chemical bonds, given the name
isotherm. Such isotherms are obtained when adsorbate forms
chemisorption. It may involve covalent or ionic bonds. e.g. H2 is
a unimolecular layer on surface of adsorbent.
chemisorbed on Ni. Hydrogen is first adsorbed by van der waal’s
forces and then dissociates. Hence hydrogen atoms are
chemisorbed on Ni.

x/m

Ps
Often adsorption involves combination of two types of P
adsorption
At low pressure, x/m µ p 1 or x/m = kp 1 where
POINTS OF DISTINCTION BETWEEN PHYSICAL
k = constant
AND CHEMICAL ADSORPTION :
At high pressure, beyond saturation x/m is independent of p
Physical Chemical and x/m µ pº.
(i) Forces van der waals Chemical bonds At intermediate pressures, x/m µ p1/n where n > 1
(ii) Temperature at low temperature at high temperature
x 1
(iii) Layer multimolecular monomolecular or log = log k + log p where k = constant.
m n
(iv) Pressure increases with decrease with P. Infact

high pressure change of pressure has


no effect.
= 1/n
pe
(v) Liquification related to liquifi- not related to liquification
slo
cation of gases of gases log x/m
(vi) Enthalpy 20 - 40 kj/mol 40 - 400 kj/mol
(vii) Activation Energy low high intercept = log K
(viii) Reversibility reversible irreversible
log P
(ix) Specificity not specific very specific

FACTORS AFFECTING ADSORPTION OF GASES BY Above equation is called FREUNDLICH ISOTHERM


SOLIDS : (iv) Effect of temperature - According to Le chatelier’s principle,
(i) Nature of gas - Easily liquifiable gases get adsorbed readily. physical adsorption occurs rapidly and lower temperature
Thus 1g of activated charcoal adsorbs 380 ml of SO2 (critical and decreases with increasing temperature. Chemisorption
temp. 157ºC). 16 ml. of methane (critical temp. = –83º ), 4.5 ml of increases with temperature. Rise of temperature might cause
H2 (critical temp. –20º C) physical adsorption to change to chemical adsorption e.g.
But chemisorption is more specific and occurs if there is a N2 is physically adsorbed on Fe at 190ºC but chemisorbed to
possibility of reaction between gas adsorbed and the solid. form nitride at 500ºC.
(ii) Nature of adsorbent - Different solids adsorb same gas at x
given temperature and pressure to different extent. Adsorbing The relation = k ( T )P liquid, solid is called adsorption
m
power depends upon chemical nature, surface area
isobar.
cleanliness of surface, distribution of capillaries or forces
acting on surface. Finely divided metals (Ni, Pt) and porous
substances (charcoal, silica gel) are best solid adsorbents as
they provide large surface area.
(iii) Effect of pressure - Extent of adsorption increases with x/m x/m
pressure as adsorption is a reversible process and is
accompanied by decrease of pressure x/m is the extent of
adsorption where m is the mass of adsorbent and x that of
Physical T Chemical T
adsorbate at equilibrium.
638 Chemistry
(v) Surface area - Larger specific area (surface area per g of ACTIVATION OF ADSORBENT :
adsorbent) means greater adsorption. Activation or increasing the power of adsorption can be achieved
(vi) Heat of adsorption - Energy released when 1g mole of a gas in different ways.
is adsorbed on solid surface is called heat of adsorption.
Adsorption is an exothemic process similar to condensation. (i) by increasing surface area
Heat of adsorption is small as attractive forces between gas (ii) by making surface rough.
molecules and solid surface are due to weak van der waal’s Small size of particles makes the interparticle space too small
forces. to allow penetration of gas molecules and thus may decrease
ADSORPTION OF SOLUTES FROM SOLUTIONS: the extent of adsorption.
Porous and finely divided solid substances can also adsorb APPLICATIONS :
dissolved substances from solution eg activated charcoal adsorbs (i) High vacuum can be generated if a partially evacuated vessel
dye stuffs. When solution of acetic acid is shaken with activated is connected to a container of activated charcoal cooled with
charcoal, part of acid is removed by adsorption and concentration
liquid air. This is due to adsorption of gas molecules in vessel.
of the soluion is decreased. Some precipitates act as adsorbents.
For e.g. Mg(OH)2 when precipitated in the presence of dyestuff (ii) Gas masks contain activated charcoal or adsorbents to
magnesium forms a blue lake. Adsorption from solution follows remove poisonous gases and thus purify the air.
same principles as that for adsorption of gases by solids. Thus (iii) Heterogenous catalysis (see chapter on catalysis)
(i) Some adsorbents specifically adsorb some solute more (iv) Chromatographic analysis is based on principle of selective
effectively than others. adsorption. Mixture of gases can also be separated by
(ii) Increase in surface area causes increase in adsorption. selective adsorption of gases by liquids (gas
(iii) Adsorption increases with temperature. chromatography).
(iv) It involves establishment of equilibrium between the amount (v) Colouring matter can be removed from solutions by animal
adsorbed and the concentration of solute in solution. charcoal which adsorbs the colouring matter.
Freundlich isotherm using concentration can be written as: (vi) Froth floatation process which involves adsorption of mineral
particles wetted with oil is used to free low grade sulphide
x ores (PbS, ZnS, Cu2S) from silica.
= k × c1/n
m not in AIEEE syllabus but given for additional information.
where x = mass of solute adsorbed.
m = mass of adsorbent DISTRIBUTION LAW
c = equilibrium concentration of solution.
NERNST DISTRIBUTION LAW OR PARTITION LAW:
x 1
log = log k + log c is a straight line. When a solute is shaken with two immiscible liquids, having
m n
solubility in both, the solute distributes itself between the two
LANGMUIR’S ADSORPTION ISOTHERM : liquids in such a way that the ratio of its concentrations in two
Langmuir exploited theoretical considerations based on kinetic liquids is constant at a given temperature, provided the molecular
theory of gases to derive an adsorption isotherm. If w is mass of state of the solute remains the same in both the liquids.
gas adsorbed per unit mass of adsorbent and p is the pressure of
the gas then Solute X

p 1 æ Bö C1
= + ç ÷p where A and B are constant A
w A èAø
B X C2
p/w q
B 1 Concentiation of X in A C1
slope = , intercept = = a constant or = KD
A A Concentiation of X in B C2
p
COMPETING ADSORPTION : (Where KD = distribution coefficient or partition coefficient or
Different adsorbates compete among themselves to adsorb on distribution ratio)
the adsorbent. Strongly adsorbable substance can displace a Distribution of Succinic acid between ether and water illustrates
weakly adsorbable substance. For e.g. Cl2 and NH3 can replace the consistancy of the ratio of concentrations in each layer
O2 and N2 already adsorbed on charcoal. Moisture is also strongly C1 in ether C2 in water C1 /C2
adsorbed by silica though it is present in small proportion in air
and charcoal adsorbs polluting gases present in air in small 0.013 0.069 0.188
amounts. 0.022 0.119 0.184
Surface Chemistry 639
Since the solubility also represents concentration, the HENRY’S LAW :
distribution law can also be written as It states that at a constant temperature the solubility of a gas in
a liquid is directly proportional to the pressure of the gas above
C1 S1
= = KD it
C 2 S2
C a P or C = KHP
Where S1 and S2 are the solubilities of the solute in two solvents. Where C is the solubility or concentration of the gas, KH is a
LIMITATIONS OF DISTRIBUTION LAW : constant known as Henry’s constant and p is the pressure of
The law holds good when the following conditions are fulfilled. the gas.
(i) Molecular State : The molecular state of the solute must • The law holds good for dilute solutions of gases which do
remain the same in both the solvents i.e. there should be no not react with the solvent.
association or dissociation of solute molecules. • When mixture of gases is there, the solubility of each gas is
(ii) Constant temperature : The temperature is kept contant proportional to its partial pressure.
(iii) Non-miscibility of solvents : The two solvents must be APPLICATIONS OF DISTRIBUTION LAW :
immiscible and the non miscibility must not change on (i) Solvent extraction : The separation of organic substances
addition of solute. from aqueous solutions by the use of organic solvent
(iv) Dilute solutions : The law is applicable to dilute solutions generally ether is based on partition law. The aqueous
only solution of organic substance is shaken with ether. The
(v) Equilibrium concentrations : The C1 and C 2 are the organic substance distributes itself in both the liquids
equilibrium concentrations which are separated by separating funnel. The solvent layer
on evaporation leaves behind organic substance.
MODIFICATIONS :
For more recovery of the substance, the number of
The following modifications are applied under different
operations should be more. The amount of organic
conditions
substance left in aqueous solution is given by
(i) When solute associates : Apply the following formula
n
æ KV ö
No change
xn = A ç ÷
è KV + v ø
C1 X Solvent A
= k ( constant ) n = Number of operations
n C2 nX Xn Solvent B
Association K = Distribution constant
V= Volume of aqueous solution
Where n is the number of molecules undergoing association
v = Volume of organic solvent
(ii) When solute dissociates : Apply the following formula
A = Amount of solute

No change (ii) Determination of degree of association

X Solvent A (iii) Determination of degree of dissociation


C1
= k ( constant ) (iv) Determination of solubility
C2 (1 - a) X A+B Solvent B
Dissociation (v) Purification of organic substances by chromatography
(vi) Desilverisation of lead (Parke’s proces)
Where a is the degree of dissociation
(vii) Layer test for Bromide and Iodide
(iii) When temperature varies : Variation of distribution
(viii)Study of complex formation
coefficient with change of temperature is given by following
relation (ix) Distribution Indicators
(xi) Determination of molecular weight in different solvents
d log K DH
=
dT RT 2
640 Chemistry

1. If the solubility of iodine in water is 0.8 g/l at 25º C, calculate 3. The solubility of pure oxygen in water at 20º C and 1.00 atm
the solubility of I2 in CCl4. The kD of iodine in between CCl4
pressure is 1.38 ´ 10 -3 mole/litre. Calculate the concentration
and water is 85 in favour of CCl4 at 25º C.
of O2 (mole/litre) at 20º C and partial pressure of 0.21 atm.
[I 2 ]in CCl 4 layer Sol. Calculation of Henry’s constant
Sol. k D = = Let the solubility of I2 in CCl4 be
[I 2 ]in water layer
Conc. of O 2 1.38 ´10 -3
a g/litre kH or k = mole / litre
pressure of O 2 1.00 atm
a
85 = mole / litre
0.8 = 1.38 ´ 10 - 3
atm
a = 68 g litre–1 Calculation of concentration of O2
2. Succinic acid has normal molecular weight in water. Succinic
mole / litre
acid was allowed to distribute in water and benzene by C = kP = 1.38 ´ 10 - 3 ´ 0.21 atm
atm
shaking. The concentiation of acid in water and benzene
are C1 and C2 having values. = 2.9 × 10–4 mole/litre
(i) (ii) (iii) 4. The distribution coefficient of an alkaloid between
C1 g/100 ml 0.150 0.195 0.289 chloroform and water is 20 in favour of chloroform. Compare
the weights of the alkaloid remaining in aqueous solution
C2 g/100 ml 2.420 4.120 9.600
when 100 ml containing 1 gram has been shaken with (a)
Find the molecular weight of acid in benzene 100 ml chloroform and (b) two successive 50 ml portion.
Sol. Calculating the value of kD by applying the relation C1/C2
C water 1
and C1/ÖC2 Sol. K = =
C chloroform 20
C1 0.152 C1 0.150
In (i) = = 0.062 ; = = 0.096 (a) When 100 ml is used in one lot, the amount unextracted is
C 2 2.420 C2 2.420
æ KV ö
X1 = 1 ´ ç ÷
C1 0.195 C1 0.195 è KV + v ø
In (ii) = = 0.047 ; = = 0.096
C 2 4.120 C2 4.120
æ 1 / 20 ´ 100 ö 5
= 1´ ç ÷= = 0.0476 g
è 1 / 20 ´ 100 + 100 ø 105
C1 0.289 C1 0.289
In (iii) = = 0.030 ; = = 0.093 (b) When 50 ml chloroform is used in each of two stages the
C 2 9.600 C2 9.600 amount unextracted is
Since constant value of K D is obtained by relation 2 2
C1/ÖC2 hence succinic acid forms a dimer in benzene æ 1 / 20 ´ 100 ö æ 5 ö
X 2 = 1´ ç ÷ = ç ÷ = 0.0083 g
è 1 / 20 ´ 100 + 50 ø è 55 ø
Therefore its molecular weight in benzene = 2 × 118 = 236
Surface Chemistry 641

Very Short Answer Questions 19. Which one of the following in not a colloidal solution?
(a) Smoke (b) Ink
1. Which of the following is adsorbed most preferably:
(c) Blood (d) Air
O2, NH3, CO?
2. Can crystalloid be brought into colloidal state? Give one 20. Which of the following statements is not true about the oil-
example. in-water type emulsion?
3. Why are colloids good adsorbents? (a) On addition of small amount of water, no separate layer
4. Write the names of two gases which are adsorbed by of water appears
transition metals such as nickel and cobalt. (b) On addition of oil, separate layer of oil is formed
5. Can a gas mixed with another gas form a colloidal solution? (c) Addition of an electrolyte causes the conductivity of
Give reasons. the emulsion to increase
6. Account for the following: (d) Addition of small amount of oil soluble dye renders the
(a) Ferric hydroxide sol is positively charged. entire emulsion coloured.
(b) The extent of physical adsorption decreases with rise
21. Suspensions are
in temperature.
(a) Visible to naked eye
(c) A delta is formed at the point where river enters the sea.
(b) Not visible by any means
7. Detergents with straight chain hydrocarbons are preferred
(c) Invisible under electron microscope
to branched chain hydrocarbons. Explain.
(d) Invisible through microscope
8. Smoke is passed through charged plates before allowing it
to pass through chimneys in industries. 22. Which of the following terms is not related with colloids ?
9. Comment on sign of DH, DS and DG for adsorption process. (a) Brownian movement
10. What is demulsification? Name two demulsifiers. (b) Dialysis
11. Out of NH3 and CO2, which gas will be absorbed more readily (c) Ultrafiltration
on the surface of activated charcoal and why? (d) Wavelength
12. What is meant by chemisorption? 23. A colloid always :
13. Explain the following terms : (a) Contains two phases
(i) Electrophoresis (b) Is a true solution
(ii) Dialysis (c) Contains three phases
(iii) Tyndall effect. (d) Contains only water soluble particles
14. What is meant by coagulation of colloidal solution? 24. When a colloidal solution is observed under an
Describe briefly and three methods by which coagulation of ultramicroscope, we can see
lyophobic sols can be carried out. (a) Light scattered by colloidal particles
15. Explain the following terms giving a suitable example for each: (b) Size of the particle
(i) Aerosol (c) Shape of the particle
(ii) Emulsion (d) Relative size
(iii) Micelle 25. One desires to prepare a positively charged sol of silver
16. Write the dispersed phase and dispersion medium of the iodide. This can be achieved by
following colloidal systems: (a) adding small amount of AgNO3 solution to KI solution
(i) Smoke in slight excess
(ii) Milk (b) adding small amount of KI solution to AgNO3 solution
17. What are lyophilic and lyophobic colloids? Which of these in slight excess
sols can be easily coagulated on the addition of small (c) mixing equal volumes of equimolar solutions of AgNO3
amounts of electrolytes? and KI
Multiple Choice Questions (d) None of these
26. Cod Liver oil is
18. Which one of the following statements is incorrect in the (a) fat dispersed in water (b) water dispersed in fat
case of heterogeneous catalysis ?
(c) water dispersed in oil (d) fat dispersed in fat.
(a) The catalyst lowers the energy of activation
27. Colloid of which one of the following can be prepared by
(b) The catalyst actually forms a compound with the
electrical dispersion method as well as reduction method ?
reactant
(a) Sulphur (b) Ferric hydroxide
(c) The surface of the catalyst plays a very important role
(c) Arsenious sulphide (d) Gold
(d) There is no change in the energy of activation.
642 Chemistry

1. The size of colloidal particles is between 13. When H 2 S gas is passed thr ough nitric acid, the
(a) 10-7 - 10-9 cm (b) 10-9 - 10-11 cm product is
-5
(c) 10 - 10 cm-7 (d) 10-2 - 10-3 cm (a) Rhombic S (b) Prismatic S
2. The size of colloidal particles is of the order (c) Amorphous S (d) Monoclinic S
(a) > 0.1 m (b) 1mm to 0.2m 14. Colloid of which one of the following can be prepared by
(c) < 1 mm (d) > 10 mm electrical dispersion method as well as reduction method ?
3. Butter is a colloid formed when (a) Sulphur (b) Ferric hydroxide
(a) fat is dispersed in water (c) Arsenious sulphide (d) Gold
(b) fat globules are dispersed in water 15. The formation of colloid from suspension is
(c) water is dispersed in fat (a) Peptisation (b) Condensation
(d) None of the these (c) Sedimentation (d) Fragmentation
4. Milk is a colloid in which 16. Which one of the following substances is not used for
(a) A liquid is dispersed in liquid preparing lyophillic sols ?
(b) A solid is dispersed in liquid (a) Starch (b) Gum
(c) A gas is dispersed in liquid (c) Gelatin (d) Metal sulphide
(d) some sugar is dispersed in water 17. Blood may be purified by
5. Cod Liver oil is (a) Dialysis (b) Electro osmosis
(a) Fat dispersed in water (b) Water dispersed in fat (c) Coagulation (d) Filtration
(c) Water dispersed in oil (d) Fat dispersed in fat. 18. The cause of Brownian movement is
6. Cloud or fog is a colloidal system in which the dispersed (a) Heat changes in liquid state
phase and the dispersion medium are
(b) Convectional currents
(a) Gas, Liquid (b) Liquid, gas
(c) The impact of molecules of the dispersion medium on
(c) Liquid, liquid (d) Solid, Liquid
the colloidal particles.
7. Which one among the following sols is hydrophobic ?
(d) Attractive forces between the colloidal particles and
(a) Gum (b) Gelatin
molecules of dispersion medium.
(c) Starch (d) Sulphur
19. Brownian movement is found in
8. Which of the following constitutes irreversible colloidal
(a) Colloidal solution (b) Suspension
system in water as dispersion medium?
(c) Saturated solution (d) Unsaturated solution
(a) Clay (b) Platinum
20. Random motion of colloidal particles is known as
(c) Fe(OH)3 (d) All of these
9. Which of the following is a lyophillic colloid ? (a) Dialysis (b) Brownian movement
(a) Milk (b) Gum (c) Electroosmosis (d) Tyndall effect
(c) Fog (d) Blood 21. Milk can be preserved by adding a few drops of
10. Surface tension of lyophillic sols is (a) Formic acid solution (b) Formaldehyde solution
(a) lower than that of H2O (c) Acetic acid solution (d) Acetaldehyde solution.
(b) more than that of H2O 22. In which of the following Tyndall effect is not
(c) equal to that of H2O observed ?
(d) can either lower or more than H2O. (a) Suspensions (b) Emulsions
11. Peptization denotes (c) Sugar solution (d) Gold sol
(a) Digestion of food 23. Point out the false statement
(b) Hydrolysis of proteins (a) Brownian movement and Tyndall effect are shown by
(c) Breaking and dispersion into the colloidal state colloidal systems.
(d) Precipitation of solid from colloidal dispersion (b) Gold number is a measure of the protective power of a
12. Colloidal gold is prepared by lyophillic colloid
(a) Mechanical dispersion (b) Peptisation (c) The colloidal solution of a liquid in liquid is called gel
(c) Bredig’s Arc method (d) Hydrolysis (d) Hardy - Schulze rule is related with coagulation.
Surface Chemistry 643
24. Tyndall effect is shown by 37. The Rubin number which was proposed by Ostwald as an
(a) Sol (b) Solution alternative to the Gold number in order to measure the
(c) Plasma (d) Precipitate protective efficiency of a lyophillic colloid may be defined as
25. Gold number is associated with the
(a) Electrophoresis (b) Purple of caslus (a) Mass in milligrams of a colloid per 100 c.c. of solution
(c) Protective colloids (d) Amount of pure gold which just prevents the colour change of standard sol
of dye Congo - Rubin from red to violet when 0.16 g eq.
26. In Brownian movement or motion, the paths of the particles
KCl is added to it.
are
(b) Mass in grams of a colloid per 100 c.c. of solution which
(a) Linear (b) Zig-zag
just prevents the colour change of standard sol of dye
(c) Uncertain (d) Curved
Congo - Rubin from red to violet when 0.1 M KCl is
27. The migration of colloidal solute particles in a colloidal added to it
solution, when an electric current is applied to the solution is
(c) Mass in grams of a colloid per 100 c.c. of solution which
known as
just prevents the colour change of standard sol of dye
(a) Brownian movement (b) Electroosmosis Congo - Rubin from red to violet when 0.2 M KCl is
(c) Electrophoresis (d) Electrodialysis added to it.
28. When a strong beam of light is passed through a colloidal (d) Mass in grams of a colloid per 100 c.c. of solution which
solution, the light will just prevents the colour - change of standard sol of dye
(a) be reflected (b) be scattered Congo - Rubin from red to violet when 1 M KCl is added
(c) be refracted (d) give a rainbow to it
29. The simplest way to check whether a system is colloidal is by 38. The function of gum arabic in the preparation of an Indian
(a) Tyndall effect (b) Brownian movement ink is
(c) Electrodialysis (d) Finding out particle size (a) Coagulation (b) Peptization
30. A colloidal solution is subjected to an electrical field. The (c) Absorption (d) Protective action
particles move towards anode. The coagulation of same sol 39. At CMC the surfactant molecules
is studied using NaCl, BaCl2 and AlCl3 solutions. Their (a) Decompose
coagulating power should be (b) Become completely soluble
(a) NaCl > BaCl2 > AlCl3 (b) BaCl2 > AlCl3 > NaCl (c) Associate
(c) AlCl3 > BaCl3 > NaCl (d) BaCl2 > NaCl > AlCl3 (d) Dissociate
31. In coagulating the colloidal solution of As2S3 which has the 40. Which one of the following is correctly matched?
minimum coagulating value ? (a) Emulsion - curd (b) Foam - mist
(a) NaCl (b) KCl (c) Aerosol - smoke (d) Solid sol - cake
(c) BaCl2 (d) AlCl3 41. Alum helps in purifying water by
32. Which of the following ions can cause coagulation of (a) forming Si complex with clay particles
proteins ?
(b) sulphate part which combines with the dirt and removes
(a) Ag + (b) Na+ it
(c) Mg2+ (d) Ca2+ (c) aluminium which coagulates the mud particiles
33. Which is used for ending charge on colloidal solution (d) making mud water soluble
(a) Electrons (b) Electrolytes 42. The process which is catalysed by one of the products is
(c) Positively charged ions (d) Compounds called
34. Which of the following is most effective in causing the (a) Acid - base catalysis (b) Auto catalysis
coagulation of ferric hydroxide sol?
(c) Negative catalysis (d) Positive catalysis
(a) KCl (b) KNO3
43. Which one of the following is an example of homogeneous
(c) K2SO4 (d) K3[Fe(CN)6]
catalysis ?
35. Arsenic sulphide is a negative sol. The reagent with least
(a) Haber’s process of synthesis of ammonia
precipitating power is
(a) AlCl3 (b) NaCl (b) Catalytic conversion of SO2 to SO3 in contact process
(c) CaF2 (d) Glucose (c) Catalytic hydrogenation of oils
36. The stablity of lyophillic colloids is due to which of the (d) Acid hydrolysis of methyl acetate
following? 44. In the case of auto catalysis
(a) Charge on their particles (a) reactant catalysis
(b) Large size of their particles (b) heat produced in the reaction catalysis
(c) Small size of their particles (c) product catalysis
(d) A layer of dispersion medium (d) solvent catalysis
644 Chemistry
45. Which of the following statements about a catalyst is/are 57. Which one of the following is an incorrect statement for
true ? physisorption ?
(a) A catalyst accelerates the reaction by bringing down the (a) It is reversible process
free energy of activation (b) It requires less heat of absorption
(b) A catalyst also takes part in the reaction mechanism (c) it requires activation energy
(c) A catalyst makes the reaction more feasible by making (d) It takes place at low temperature
the DGº more negative 58. Which is adsorbed in maximum amount by activated charcoal
(d) A catalyst makes the equilibrium constant of the reaction ?
more favourable for the forward reaction. (a) N2 (b) CO2
46. The role of a catalyst in a reversible reaction is to (c) Cl2 (d) O2
(a) Increase the rate of forward reaction 59. Which among the following statements is false?
(b) Decrease the rate of backward reaction (a) Increase of pressure increases the amount of adsorption
(c) alter the equilibrium constant of the reaction (b) Increase of temperature may decrease the amount of
(d) Allow the equilibrium to be achieved quickly adsorption.
47. Which of the following kind of catalysis can be explained by (c) The adsorption may be monolayered or multilayered
the adsorption theory ? (d) Particle size of the adsorbent will not affect the amount
(a) Homogeneous catalysis of adsorption.
(b) Acid - base catalysis 60. Rate of physisorption increases with
(c) Heterogeneous catalysis (a) decrease in temperature (b) increase in temperature
(d) Enzyme catalysis (c) decrease in pressure (d) decrease in surface area
48. A biological catalyst is essentially 61. Distribution law was given by
(a) An enzyme (b) A carbohydrate (a) Van’t Hoff (b) Henry
(c) An amino acid (d) A nitrogenous base (c) Nernst (d) Ostwald
49. In Zeigler - Natta polymerisation of ethylene, the active species 62. When is the Distribution Law applicable?
is (a) Temperature remains constant
(a) AlCl3 (b) Et3Al (b) Dilute solutions are employed
(c) Two solvents are mutually insoluble
(c) CH2CH2 (d) Ti3+
(d) All of these
50. Adsorbed acetic acid on activated charcoal is:
63. The relation between pressure and solubility of a gas was
(a) Adsorber (b) Absorber given by
(c) Adsorbent (d) Adsorbate (a) Henry (b) Nernst
51. Which of the following is less than zero during adsorption. (c) Both (a) and (b) (d) Neither (a) nor (b)
(a) D G (b) D S 64. The solubility of iodine in water is 0.35 g/ litre at 25°C. If
(c) D H (d) All of these distribution coefficient of iodine between CS2 and water is
52. The heats of adsorption in physisorption lie in the range (in 600, the solubility of iodine in CS2 in g/ litre will be
kJ/mol) (a) 210 (b) 510
(a) 40 - 400 (b) 40 - 100 (c) 195 (d) 250
(c) 10 - 40 (d) 1 - 10 S1
65. Partition coefficient, K = is valid
53. In physical adsorption, gas molecules are bound on the solid S2
surface by (a) when dissociation takes place
(a) Chemical forces (b) Electrostatic forces (b) when association takes place
(c) Gravitational forces (d) van der waal’s forces (c) when there is a complex formation
54. How many layers are adsorbed in chemical adsorption ? (d) in case of normal distribution
(a) One (b) Two 66. When a solute is shaken with two immiscible liquid, it
(c) Many (d) Zero associates in one of the liquids. The distribution constant is
55. In physical adsorption, the forces associated are given by
(a) ionic (b) covalent C1 C1
(a) (b)
(c) van der Waal (d) H - bonding C2 C 2 (1 - a )
56. Adsorption due to strong chemical forces is called
S1 C1
(a) Chemisorption (b) Physiosorption (c) (d)
S2 n C2
(c) Reversible adsorption (d) Both (b) and (c)
Surface Chemistry 645
67. The condition under which Nernst distribution law will not 72. You are given 100 ml of CCl4 to extract iodine from 200 ml of
hold true is its aqueous solution. For extracting the maximum amount of
(a) The molecular state of the solute is same in both iodine, which one of the following process would you use
solvents (a) Use 100 ml of CCl4
(b) Temperature is constant (b) Use 50 ml of CCl4 twice
(c) The solute does not cause any change in the mutual (c) Use 10 ml of CCl4 ten times
solubility of the two solvents
(d) Use 25 ml of CCl4 4 times
(d) None of these
68. When benzoic acid is shaken with a mixture of 100 ml water 73. Distribution law cannot be applied to I2 distributed between
and 50 ml ether, the ether layer was found to contain 0.127 g (a) H2O and C2H5OH (b) H2O and CCl4
of acid and water layer 1.843 g of the acid. The partition (c) H2O and CS2 (d) H2O and C2H5OC2H5
coefficient between water and ether is 74. Which of the following applications is not shown by
(a) 72.5 (b) 7.25 distribution law?
(c) 725 (d) 0.725 (a) Pallison’s process
69. If in a pair of immiscible liquids, a common solute dissolves (b) Partition Chromatography
in both and equilibrium is reached, then the concentration (c) Solvent extraction
of the solute in the upper layer is (d) Parke’s process
(a) same as in lower 75. Which one of the following statements is false for
(b) in fixed ratio with that in the lower layer distribution law?
(c) lower than in the lower layer (a) The two solvents must be mutually immiscible
(d) higher than in the lower layer (b) The concentration of the solute in both the solvents
70. The partition coefficient of a solute for ether and water is in must be kept high
the ratio 5:1. The amount of solute extracted from 100 cc of (c) The substrate must not react with any of the solvents
its aqueous solution with 100 cc of ether in one time will be (d) The temperature should not change during
(a) 90% (b) 87.3% experimentation
(c) 91.8% (d) 83.3% 76. When solution of 5g of iodine in CS2 was shaken with the
71. The partition coefficient of X between liquids A and B is 10. same volume of water. The amount of iodine in water is
The partition coefficient of X between B and A is (Distribution coefficient C CS2 / C H 2O = 420.0)
(a) 100 (b) 1 (a) 1.19 (b) 0.0019
(c) 0.1 (d) 10 (c) 0.0119 (d) 0.119

1. The Langmuir adsorption isotherm is deduced by using the 3. In Freundlich Adsorption isotherm, the value of 1/n is :
assumption [CBSE-PMT 2007] (a) between 0 and 1 in all cases [CBSE-PMT 2012S]
(b) between 2 and 4 in all cases
(a) the adsorption sites are equivalent in their ability to adsorb
(c) 1 in case of physical adsorption
the particles
(d) 1 in case of chemisorption
(b) the heat of adsorption varies with coverage 4. Which one of the following, statements is incorrect about
(c) the adsorbed molecules interact with each other enzyme catalysis? [CBSE-PMT 2012S]
(d) the adsorption takes place in multilayers. (a) Enzymes are mostly proteinous in nature.
(b) Enzyme action is specific.
2. If x is amount of adsorbate and m is amount of adsorbent,
(c) Enzymes are denaturated by ultraviolet rays and at high
which of the following relations is not related to adsorption
temperature.
process ? [CBSE-PMT 2011] (d) Enzymes are least reactive at optimum temperature.
(a) x / m = f (p) at constant T. 5. The protecting power of lyophilic colloidal sol is expressed in
(b) x / m = f (T) at constant p. terms of : [CBSE-PMT 2012S]
(c) p = f (T) at constant (x / m). (a) coagulation value
(b) gold number
x (c) critical miscelle concentration
(d) p T
m (d) oxidation number
646 Chemistry
6. Which of the following is not the property of hydrophilic 10. Gold numbers of protective colloids A, B, C and D are 0.50,
solutions ? 0.01, 0.10 and 0.005, respectively. The correct order of their
(a) High concentration of dispersed phase can be easily protective powers is [AIEEE 2008]
obtained (a) D < A < C < B (b) C < B < D < A
(b) Coagulation is reversible (c) A < C < B < D (d) B < D < A < C
11. Which of the following statements is incorrect regarding
(c) Viscosity and surface tension are nearly the same as
physisorptions? [AIEEE 2009]
that of water.
(a) More easily liquefiable gases are adsorbed readily.
(d) The charge of the particle depends on the pH of the
(b) Under high pressure it results into multimolecular layer
medium and it may be positive, negative or zero. on adsorbent surface.
7. Which one of the following characteristics is not correct for (c) Enthalpy of adsorption ( DH adsorption) is low and positive.
physical adsorption ? [AIEEE 2003] (d) It occurs because of van der Waal’s forces.
(a) Adsorption increases with increase in temperature 12. According to Freundlich adsorption isotherm which of the
(b) Adsorption is spontaneous following is correct? [AIEEE 2012]
(c) Both enthalpy and entropy of adsorption are negative x
(d) Adsorption on solids is reversible (a) µ p°
m
8. The disperse phase in colloidal iron (III) hydroxide and
x
colloidal gold is positively and negatively charged, (b) µ p1
respectively. Which of the following statements is NOT m
correct ? [AIEEE 2005] x
(a) Coagulation in both sols can be brought about by (c) µ p1/n
m
electrophoresis
(d) All the above are correct for different ranges of pressure
(b) Mixing the sols has no effect 13. The coagulating power of electrolytes having ions Na+, Al3+
(c) Sodium sulphate solution causes coagulation in both and Ba2+ for arsenic sulphide sol increases in the order :
sols [JEE M 2013]
(d) Magnesium chloride solution coagulates, the gold sol (a) Al3+ < Ba2+ < Na+ (b) Na+ < Ba2+ < Al3+
more readily than the iron (III) hydroxide sol (c) Ba2+ < Na+ < Al3+ (d) Al3+ < Na+ < Ba2+
9. In Langmuir's model of adsorption of a gas on a solid surface 14. Among the following, the surfactant that will form micelles in
[AIEEE 2006] aqueous solution at the lowest molar concentration at ambient
(a) the mass of gas striking a given area of surface is condition is : - [IIT-JEE 2009S]
proportional to the pressure of the gas (a) CH3(CH2)15N+(CH3)3Br– (b) CH3(CH2)11OSO–3 Na+
(c) CH3(CH2)6COO–Na+ (d) CH3(CH2)11N+(CH3)3Br–
(b) the mass of gas striking a given area of surface is
15. Among the electrolytes Na 2 SO4, CaCl 2, Al 2 (SO4 )3 and
independent of the pressure of the gas
NH4Cl, the most effective coagulating agent for Sb2S3 sol is
(c) the rate of dissociation of adsorbed molecules from [IIT-JEE 2009S]
the surface does not depend on the surface covered (a) Na2SO4 (b) CaCl2
(d) the adsorption at a single site on the surface may (c) Al2(SO4)3 (d) NH4Cl
involve multiple molecules at the same time

1. For the adsorption of a gas on a solid, the plot of 3. Adsorption is accompanied by the evolution of heat. So
log(x/m) versus log p is linear with slope equal to according to Le-Chatelier principle the amount of substance
(a) k (b) log k adsorbed should
(c) n (d) 1/n (a) increase with decrease in temperature
(where, p= pressure of gas, m = mass of the adsorbent, (b) increase with increase in temperature
x = mass of the gas adsorbed) (c) decrease with decrease in temperature
2. Which of the following gas molecules have maximum value (d) decrease with increase in temperature
of enthalpy of physisorption? 4. At low pressure, the fraction of the surface covered follows
(a) C2H6 (b) Ne (a) zero-order reaction (b) second-order reaction
(c) H2O (d) H2 (c) first-order reaction (d) fractional order
Surface Chemistry 647
5. At high pressure, the entire surface gets covered by a 9. The critical micelle concentration (CMC) is
monomolecular layer of the gas follows
(a) the concentration at which micellization starts
(a) three-halved order (b) second-order
(b) the concentration at which the true solution is formed
(c) first-order (d) zero-order
6. Which of the following statements is incorrect ? (c) the concentration at which one molar electrolyte is
(a) Adsorption always leads to a decrease in enthaply and present per 1000 g of the solution
entropy of the system (d) the concentration at which DH=0
(b) Adsorption arises due to unsaturation of valence forces 10. The isoelectric-point of a colloidally dispersed material is
of atoms or molecules on the surface
the pH value at which
(c) Adsorption increases with rise in the temperature
(d) Adsorption decreases the surface energy (a) the dispersed phase migrate in an electric field
7. During adsorption (b) the dispersed phase does not migrate in an electric field
(a) TDS is positive (b) DH–TDS is negative (c) the dispersed phase has pH equal to 7
(c) DH is positive (d) TDS and DG become zero (d) the dispersed phase has pH equal to zero
8. Which of the following statements is not correct?
11. Peptization involves
(a) Decrease of temperature and increase of pressure, both
tend to cause increase in the magnitude of adsorption of (a) precipitation of colloidal particles
a gas on a solid. (b) disintegration of colloidal aggregates
(b) The easily liquefiable gases adsorb more on solid (c) evaporation of dispersion medium
(c) Greater the surface area per unit mass of the adsorbent,
(d) impact of molecules of the dispersion medium on the
the greater is its capacity of adsorption
colloidal particles
(d) None of these
648 Chemistry

EXERCISE 1 20. (b) Brownian movement is random motion.


1. NH3 is adsorbed most preferably. 21. (b) Formaldehyde is emulsifier.
2. Sodium chloride (NaCl) can be brought in to colloidal state 22. (c) Sugar forms homogeneous solution hence no Tyndall
in benzene. effect is exhibited
3. They have enormous surface area per unit mass and are, 23. (c) Liquid - liquid system is known as emulsion.
thus, good adsorbents. 24. (a) Tyndall effect is shown by sols.
4. Hydrogen and oxygen. 25. (c) Gold number is associated with protective colloids
5. No, this is because gases mix homogeneously in all
26. (b) Brownian movement is zig zag motion of sol particles.
proportions.
9. DH Þ – Ve 27. (c) It is electrophoresis (see definition of electrophoresis)
DS Þ – Ve 28. (b) It is Tyndall effect
DG Þ – Ve 29. (a) Tyndall effect is the simplest way to check colloidal system
18. (c) 19. (d) 20. (d) 21. (a) 22. (d) since path becomes visible due to scattering of light.
23. (a) 24. (a) 25. (b) 26. (c) 27. (d)
30. (c) The movement towards anode shows that sol is negative.
EXERCISE 2 For coagulation of negative sol. Cation with higher charge
1. (c) Size of colloidal particles is 10-5 - 10-7 cm. is more effective.
31. (d) As2S3 is negative sol. For coagulating negative sol. Al3+
2. (b) 1 micron m = 10–4 cm and 1 millimicron = 10–7 cm.
is most effective. The higher the magnitude of the charge,
3. (c) Butter (liquid - solid) water dispersed in fat the lower is coagulating value.
4. (a) Milk is emulsion, liquid dispersed in liquid 32. (a) Proteins are coagulated by Ag+ ions.
5. (c) W/O emulsion water dispersed in oil 33. (b) Charge carried by dispersed phase and dispersion medium
6. (b) Cloud and fog are colloidal system having dispersed is neutralised by electrolytes and coagulation takes place.
phase liquid and dispersion medium gas 34. (d) Fe(OH)3 is positive sol. K3[Fe(CN)6] will provide
7. (d) Sulpher is hydrophobic sol. [Fe(CN)6]3– for coagulation.
8. (d) All are irreversible colloidal systems. 35. (d) Glucose is non electrolyte hence least effective in
9. (b) Gum is lyophilic colloid. precipitating As2S3 (negative sol)
10. (a) Surface tension of lyophilic sols is lower than water 36. (d) The stability of lyophilic colloids is due to layer of
(dispersion medium). dispersion medium around sol particles.
11. (c) Peptization comes under dispersion methods of 37. (a) See definition in text
preparation of colloids 38. (d) Alum is good coagulating agent because of multivalent
12. (c) Colloidal gold is prepared by Bredig's arc method. ions.
13. (c) Sol of amorphous sulpher is formed. 39. (c) CMC is critical micellization concentration, the surfactants
at this concentration associate to form micelles. For soaps,
14. (d) Gold by Bredig’s meth od (Dispersion method)
the CMC value is 10-3 mole/litre
and by reduction method AuCl3 + Tannic acid ® Gold sol
40. (c) Smoke is solid gas system, solid (D. P) and gas (D.M.)
15. (a) Formation of colloid from suspension is known as
peptization. Note : D.P. : Dispersed phase
16. (d) Metal sulphide give lyophobic sol not lyophilic sol. D.M. : Dispersion medium

17. (a) Blood is purified by dialysis. 41. (c) Alum. coagulates mud particles and helps in purifying
water.
18. (c) It is due to impact of molecules of dispersion medium on
the colloidal particles. 42. (b) In auto catalysis one of the products act as catalyst
43. (d) In acid hydrolysis of methyl acetate all are present in one
19. (a) Brownian movement is exhibited by colloidal system.
phase (liquid)
Surface Chemistry 649
44. (c) 45. (a) 46. (d)
a
47. (c) Adsorption theory is applied to heterogeneous catalysis 70. (d) Let the amount extracted be a then, k D = = 5,
100 - a
48. (a) Enzymes are biological catalysts.
\ a = 83.3%
49. (d) Ti3+ Ziegler Natta catalyst is a mixture of
(C2H5)3 Al + TiCl3. C ether
71. (c) Partition coefficient k D = , k D is in favour of
50. (d) The substance which is adsorbed is called adsorbate C w ater
51. (a) Adsorption is spontaneous process. Hence DG must be ether. The value is reversed if we change it in favour of
negative water.
52. (c) For physiosorption the D H value is 10 - 40 kJ/mol. 72. (c) The greater the number of operations, the more is the
53. (d) In physisorption, gas molecule is held on the solid surface recovery of the substances to be extracted.
by weak van der Waal’s forces. 73. (a) H2O and C2H5OH are miscible liquids, the law is applicable
54. (a) Chemical adsorption is monolayer to immiscible liquids.

55. (c) In physical adsorption the forces between adsorbate and 74. (a) Pallison process is not based on distribution law.
adsorbent are weak van der waal’s forces. 75. (b) The concentration of the solute in both the solvents must
56. (a) Chemisorption involves strong chemical forces. be kept low and not high.

57. (c) Physical adsorption does not require any activation 76. (c) Let the amount of iodine in water be a gm., C1 = 5g in x ml
energy. of CS2, C2 = 5 – a g in x ml of H2O. Then

58. (b) The gases having higher values for critical temperature 5-a
= 420, \= 0.0118
are easily liquified and are adsorbed to the greater extent. a
59. (d) Statement (d) is wrong (see text) EXERCISE 3
1 1. (a) Langmuir adsorption isotherm is based on the
60. (a) Physical adsorption µ . assumption that every adsorption site is equivalent
Temperature
and the ability of a particle to bind there is independent
61. (c) The distribution law was given by Nernst. of whether nearby sites are occupied or not.
62. (d) All the conditons are correct. 2. (d)
63. (a) Henry gave the relation between pressure and solubility 3. (a) According to Freundlich Adsorption isotherm
of a gas as, solubility a pressure. 1
x
64. (a) Let the solubility of iodine in CS2 be a then = KP n
m
S1 a 1
= KD , = 600 , \ a = 210 at low pressure =1
S2 0.35 n
x
S \ µ P1
65. (d) K D = 1 is valid only when their is no association or m
S2
dissociation of solute dissolved and no change of 1
at high pressure =0
temperature. There is no complex formation. n
x
C1 µ P°
66. (d) In case when solute associate, k D = m
n C2 i.e., the value of n varies between 0 to 1
67. (d) All the statements are correct. 4. (d) Enzymes are most reactive at optimum temperature. The
optimum temperature for enzyme activity lies between
C1 1.843 0.127 1.843 / 100
68. (b) k D = , C1 = and C 2 = ,\ k D = = 7.25 40°C to 60°C.
C2 100 50 0.127 / 50
5. (b) The lyophobic sols are less stable than lyophilic sols.
1.843 0.127 The lyophilic sols are thus used to protect the
1= and C 2 = ,
100 50 lyophobic sols. This property of lyophilic sols is known
as protective action of lyophilic sols which is
1.843 /100
\ kD = = 7.25 represented by gold number.
0.127 / 50
6. (c) See properties of hydrophilic sols.
C
69. (b) At equilibrium stage 1 = k D ; where C1 & C2 are the 7. (a) As adsorption is an exothermic process.
C2
concentrations of the solute in the upper and lower layer. \ Rise in temperature will decrease adsorption.
650 Chemistry
8. (b) When oppositely charged sols are mixed their charges 14. (b)
are neutralised. Both sols may be partially or completely 15. (c) As Sb2S3 is a negative sol, so Al2(SO4)3 will be the
precipitated.
most effective coagulant due to higher positive charge
9. (a) According to Langmuir's model of adsorption of a gas
on a soild surface the mass of gas adsorbed(x)per gram on Al (Al3+) – Hardy-Schulze rule.
of the adsorbent (m) is directly proportional to the EXERCISE 4
pressure of the gas (p) at constant temperature i.e.
x x 1
µp 1. (d) Freundlich equation is log = log K + P where
m m n
10. (c) For a protective colloid lesser the value of gold number 1
= slope .
more will be the protective power. Thus the correct n
order of protective power of A, B, C and D is 2. (c) The more the liquefiable nature of a gas, the more is the
Þ (A) < (C) < (B) < (D) enthalpy of adsorption. Water is more liquefiable.
Gold number 0.50 0.10 0.01 0.005 3. (a) The extent of adsorption increases with decrease in
temperature.
Hence (c) is the correct answer
4. (c) At low pressure the extent of adsorption is directly
11. (c) Adsorption is an exothermic process, hence DH will
proportional to pressure which follows first order kinetics.
always be negative.
5. (d) At high pressure the extent of adsorption follows zero
12. (d) The Freundlich adsorption isotherm is mathematically order kinetics.
represented as 6. (c) Adsorption increases with decrease in temperature.
x 7. (b) DG = DH – TDS. Adsorption is spontaneous process DG
= kP1/n
m should be negative. Hence DH – TDS should be negative.
At high pressure 1/n = 0. Hence, x / m µ P° 8. (d) All statements are correct
At low pressure 1/n = 1 Hence, x/m µ P1 9. (a) CMC is the concentration at which micellization starts
13. (b) According to Hardy Schulze rule, greater the charge i.e. surfactant molecules aggregate.
on cation, greater is its coagulating power for negatively 10. (b) At isoelectric point there is no migration of dispersed
charged sol (As2 S3 ), hence the correct order of phase in an electric field.
coagulating power : Na+ < Ba2+ < Al3+ 11. (b) Peptisation is disintegration of colloidal aggregate.
20
General Principles
& Processes of
Isolation of Elements
METALLURGY : CONCENTRATION OR DRESSING :
The science and technology of isolation of pure metals from their The process of the removal of gangue or matrix from the ore is
ores and preparing them for practical use. The process includes. known as concentration. It is achieved by
(i) Mining - getting the ore out of ground. (i) Hand picking
(ii) Concentration - preparing for further treatment. (ii) Gravity separation (hydraulic -washing )
(iii) Reduction - to obtain the metals in zero oxidation state. (iii) Magnetic separation
(iv) Refining - to obtain the pure metal. (iv) Electrostatic separation
(v) Mixing with other metals - to form an alloy. (v) Froth flotation process -for sulphide ores.
MINERALS : (vi) Leaching
Most metals are found in nature in the form of solid inorganic Froth flotation process - Finely divided ore is mixed with oil (pine
compounds called minerals. Names of minerals are based on the oil, eucalyptus oil or camphor oil) and agitated with water
location of their discovery, the person who discovered them or containing a detergent (foaming agent). When air is bubbled
some characteristics of the mineral. through the mixture, the air bubbeles are stabilised by the detergent
.These adsorb mineral particles wetted with oil and rise to the
ORE : surface. The earthy matter wetted by water settles down at the
The mineral from which the metal can be extracted economically. bottom.
Hence all minerals can not be classified as ores. The most Collectors - Which increase the non wettability of ore particles
important ores are oxide,sulphide and carbonate minerals. e.g. pine oil, xanthates and fatty acids.
GANGUE OR MATRIX : Froth stabiliser -Which stabilise the froth e.g. cresoles and aniline.
The unwanted rocky,earthy or sandy materials almost always Depressants - Depressants prevent the formation of froth eg
associated with the ores as impurities are called gangue or matrix. NaCN, when added to ore containing ZnS and PbS form a complex
FLUX AND SLAG : with ZnS as Na 2 [ Zn (CN )4 ] and prevent it from forming froth.
Flux is a substance added to the ores before heating which PbS is then easily separated from ZnS.
combines chemically with earthy impurities (gangue) and form a Leaching- Leaching is the selective dissolution of the desired
fusible mass known as slag . mineral leaving behind the impurities in a suitable dissolving agent
Flux can be Acidic e.g. SiO2 (silica), Na2 B4O7 .10H 2O (borax); eg bauxite when treated with strong solution of
Basic e.g. CaO, MgO ; or Neutral - neutral compounds, and they NaOH, Al 2O 3 dissolves leaving behind Fe 2O 3 .SiO 2 .
decrease the melting point and make the order conducting in an
Al2O3 + 6NaOH ® 2Na 3AlO3 + 3H 2O
electrolytic cell e.g. CaF2 , Na3 AlF6 , KF etc.
Slag consists mostly of molten silicates ,aluminates, phosphates, Na 3 AlO 3 + 3H 2 O ® 3 NaOH + Al(OH )3
fluorides and another inorganic materials. The formation of slag is 2Al(OH )3 ® Al 2 O 3 + 3H 2 O
known as slagging .
652 Chemistry
Leaching is also employed in Ag ore and native gold. (iii) Self reduction -Cu, Pb and Mg are obtained by self
reduction in roasting.
Ag2S + 4NaCN® 2Na[Ag(CN)2 ] + Na2S
3
4Au + 8KCN + 2H2O + O2 ® 4K[Au(CN)2 ] + 4KOH Cu 2S + O2 ® Cu 2O + SO2
2
CALCINATION : Cu 2S + 2Cu 2O ® 6Cu + SO 2
It is the heating of the ore in a suitable furnace in absence of air 2PbS + 3O 2 ® 2PbO + 2SO 2
much below its melting point to cause decomposition and PbS + 2PbO ® 3Pb + SO 2
elimination of volatile products.
(iv) Electrolytic reduction -Oxides of very active metals like
PbCO 3 ® PbO + CO 2 alkali or alkaline earth are not easily reduced by chemical
The process is generally applied to hydrated oxide or carbonate reducing agents e.g. Na, Mg, Al etc. They are obtained
ores. by electrolytic reduction.
Fusion
ROASTING : ® 2Na + + 2Cl -
2NaCl ¾¾¾¾
It is the heating of the concentrated ore in a suitable furnace At anode : 2Cl - - 2e - ® Cl 2
strongly in presence of air with or without certain substances
below the melting point which causes chemical reaction to expel At Cathode : 2Na + + 2e- ® 2Na
volatile impurities eg oxides of S, As and Sb. (v) Amalgamation process - Ag and Au are obtained by
leaching process using solution of KCN or NaCN to
2ZnS + 3O 2 ® 2ZnO + 2SO 2 form argento cyanide or aurocyanide . Ag and Au is
2MoS 2 + 7O 2 ® 2MoO 3 + 4SO 2 precipitated by adding Zn dust.
REDUCTION : 2Na[Ag(CN )2 ] + Zn ® Na 2 [Zn(CN )4 ] + 2Ag
After calcination or roasting, the metal oxides are reduced and 2K[Au(CN )2 ] + Zn ® K 2 [Zn(CN )4 ]+ 2Au
impurities are removed as slag. Reduction and slagging take place Soluble complex
together.
(a) Removal of impurities as slag - For acidic impurities viz. REFINING :
SiO 2 , P2 O5 basic flux is added. The methods employed for the refining of metals are
SiO 2 + CaO ® CaSiO 3 (i) Liquation -Impurities present must be less fusible than the
metal to be purified . Impure metal is placed on the slopping
For basic impurities like MnO acidic flux is added.
hearth of reverberatory furnace at a temperature just above
MnO + SiO 2 ® MnSiO3 the melting point of the metal . The pure metal flows down
(b) Reduction of oxides - leaving behind the impurities . Sn ,Pb, Bi are purified by this
(i) Decomposition of oxides by heating - For thermally method.
unstable oxide. (ii) Distillation - Zn and Hg are purified by distillation under
HgS + O 2 ® Hg + SO 2 reduced pressure provided the impurities are non volatile.
Hg is obtained from its sulphide ore cinnabar directly in (iii) Fractional crystallisation - (Zone refining ).
the roasting step.
(ii) Chemical Reduction -
• Reduction by carbon - Sn from oxide ore cassiterite Molten zone
SnO2 is obtained by heating with coke.
SnO 2 + C ® Sn + 2CO Recrystallized metal Metal rod
Zn from sulphide ore Zinc blende (ZnS).
Roasting Circular heater
2ZnS + 3O 2 ¾¾ ¾¾® 2ZnO + 2SO 2
ZnO + C ® Zn + CO
Impure metal rod is heated with the help of circular heater at
Iron is obtained from oxide ore haematite ( Fe2O3 ). one end. The metal melts and on cooling the pure metal gets
Fe 2 O 3 + 3C ® 2Fe + 3CO solidified while impurities pass on into the molten zone. The
• Reduction by H2 and CO- process is repeated twice or thrice to get the pure metal.
(iv) Polling -The molten metal is stirred with green poles of wood,
MoO 3 + 3H 2 ® Mo + 3H 2 O
which liberates gas like methane. The latter reduces any oxide
CuO + CO ® Cu + CO 2
present in the metal eg CuO in the blister copper is reduced
• Reduction by other metals e.g. Al and Mg - to copper.
Cr2 O 3 + 2Al ® 2Cr + Al 2 O 3 (v) Electrolytic refining- The blocks of impure metal form the
3Mn 3 O 4 + 8Al ® 4Al 2 O3 + 9Mn anode and pure metal form the cathode. Aqueous
Rb 2 O 3 + 3Mg ® 3MgO + 2Rb
General Principles
General Principles & Processes of Isolation of Elements 653
solution of appropriate salt is then electrolised. On oxidised and carried away by blast air .
electrolysis at a suitable voltage the pure metal is deposited (viii)Desilverisation of lead -Lead obtained from galena (PbS)
at cathode. contains impurities of silver, removal of which is called
desilverisation .The processes employed are
At anode M - ne- ® M n + (a) Parke’s process -Lead containing silver is melted in iron
At cathode M n + + ne- ® M pots and 1% Zn is added then cooled. Zn -Ag alloy
solidifies and being light floats over molten lead and
Cu, Sn, Ag, Pb, Cr, Ni are refined by this process. removed .
(vi) Vapour phase refining - Metal is removed as volatile ( Pb and Ag ) + Zn ® ( Zn - Ag ) alloy + Pb
compound which is then decomposed by heating to get pure
(b) Pattison’s process -Lead containing less than 2.5% of
metal. e.g.
Ag melts at lower temperature than lead. Thus when an
(a) Mond’s process - Ni is purified by this process. alloy of Pb-Ag containing more lead is melted then
allowed to cool slowly, pure lead separates.
Ni(s ) + 4CO(g) ¾¾¾ ¾¾® Ni (CO) 4 (g)
300 -350K
450- 470K (ix) Chromatographic methods - It is based on the preferential
Ni(CO) 4 (g) ¾¾ ¾¾¾® Ni(s) + 4CO(g) adsorption of different compunds on an adsorbent. The
(b) Van Arkel process-Zr, Hf , Si,Ti and Be are refined by mixture is put in the liquid or gaseous medium which is moved
this process. through the adsorbent. The different compounds are
adsorbed at different levels on adsorbent in column
Zr s + 2 I 2 g ® ZrI 4 g chromatography and recoverd by using suitable solvent
ZrI4(g ) On hot
Zr(s) + 2 I 2(g) (eluent). The common adsorbent as Al2O3 or SiO2 (silica).
filament The least adsorbed component is recoverd first. The method
(vii) Cupellation - This is the method of purifying silver containing is very useful for purification of elements available in minute
lead as impurity. Impure silver is heated in a shallow (Cupel) quantities.
which is made of bone ash under blast of air. Lead is easily

Flow meter

Detector

Mixture of
compounds
C
O
L
U Oven
Stationary Coloured
M
N phase bands

Flow injector
Pump
Solvent
tank (a) (b)
Industrial method Laboratory method
Schematic diagrams showing column chromatography
There are several chromatographic techniques such as gas chromatography, paper chromatography, etc.
METAL SOURCE MAIN METHOD OFEXTRACTION
Metal Occurrence (ore) Common method of extraction
Li Spodumene LiAl (SiO 3 ) 2 Electrolysis of fused LiCl with KCl .
K Carnallite KCl MgCl2 . 6H2O ; Kaenite KCl MgSO4 3H2O, Electrolysis of fused KCl with CaCl 2 added to it.
Sylvine KCl ; Nitre KNO3 ; Feldspar K2O Al2O3 6SiO2 ;
Mical KH2Al3(SO4)3 ;
654 Chemistry

Ba Witherite BaCO3 ; Barytes BaSO4 ; Electrolysis of fused BaCl 2 .


Ca Lime stone, chalk, marble, calcite CaCO3 ; Electrolysis of fused CaCl 2 and CaF2 mixture.
Gypsum CaSO4 H2O; Dolomite CaCO3 .MgCO3 ;
Fluorspar CaF2; Phosphorite Ca 3 (PO 4 )2 ;
Na Rock salt, Common Salt NaCl ; Chile Saltpetre NaNO3 ; Electrolysis of fused NaCl with CaCl2 added to it.
Borax or Sodium borate Na2B4O7 10H2O,
Sodium Carbonate Na2CO3;
Mg Carnallite KCl.MgCl2 6H2O Magnesite MgCO 3 ; Electrolysis of fused MgCl2 with KCl added to it .
Dolomite MgCO3CaCO3 ; Kieserite MgSO 4 .H 2 O;
Epsom salt MgSO 4 .7H 2 O :
Be :Beryl 3BeO.Al 2O 3 .6SiO 2 ; Chrysoberyl BeO.Al 2 O3 ; Electrolysis of fused BeF2 with NaF added to it .
Al Bauxite Al 2O 3.2H 2 O; Potash feldspar K2OAl2O3. 6SiO3 , Electrolysis of molten Al2PO3 in molten Na3AlF6.
Corundum Al 2O 3 ; Cryolite Na 3AlF6 ;
Kaolin Al2O3 2SiO2. 2H2O
Mn Pyrolusite MnO 2 ; Hausmannite Mn 3O 4 ; Reduction of oxide with Al or C :
Braunite Mn2O3; Manganite Mn2O3. H2O : 3Mn 3O 4 + 8Al ® 9Mn + 4Al 2 O 3.
Ti Ilmenite TiO2. FeO; RutileTiO2 : Reduction of TiCl4 with Mg or Na :
TiCl 4 + 2Mg ® Ti + 2MgCl 2 .
Zn Zinc blende ZnS;Calamine ZnCO3 , Zincite ZnO; Reduction of ZnO with C or electrolysis of
Willemite Zn 2SiO 4 : ZnSO 4 : ZnO + C ® Zn + CO .
Cr Chromite FeO. Cr2 O 3 ;Chrome Ore Cr2 O 3 : Reduction of Cr2 O 3 with Al: Cr2O 3 + 2Al ® 2Cr + Al 2 O 3
Fe Haematite Fe 2 O3 ; Magnetite Fe3O 4 ; Reduction of oxides with CO:
Limonite 2Fe 2 O3 .3H 2 O; Iron Pyrites FeS2 ; Fe 2O 3 + 3CO ® 2Fe + 3CO 2 .
Siderite Spathic Iron ore FeCO 3 ; Copper pyrites CuFeS 2 :
Co Smaltite (CoNiFe) As 2 ; Cobaltite Co AsS; Reduction of
Co 3O 4 with Al : 3Co3O 4 + 8Al ® 9Co + 4Al 2O 3
Ni Millerite NiS; Reduction of NiO with CO: NiO + CO ® Ni + CO 2
Sn Cassiterite or Tin stone SnO 2 ; Stannite Cu 2S .FeS. SnS2 : Reduction of SnO 2 with C: SnO 2 + 2C ® Sn + 2CO.
Pb Galena PbS ; Cerussite PbCO 3 ,Anglesite PbSO 4 : Reduction of PbO with C: PbO + C ® Pb + CO .
Bi Native; Bismuth glance Bi 2S3 ; Bismuthite Bi 2O 3 : Reduction of Bi 2O 3 with arbon Bi2O3 + 3C ® 2Bi + 3CO .
Cu Native; Copper pyrites or
Chalcopyrites CuFeS2 or Cu2SFeS3 ; Partial oxidation of sulphide ore :
Cuprite Cu2O; Malachite Cu (OH )2 .CuCO 3 ; 2Cu 2O + Cu 2S ® 6Cu + SO 2 .

Copper glance Cu 2S; Azurite 2CuCO3 .Cu ( OH )2 :


Ag Native, Argentite, Silver glance Ag 2S ; Special method with
Horn Silver (Chlorargyrite) AgCl; Ruby silver NaCN: Ag 2S + 4 NaCN ® 2 Na [ Ag (CN )2 ] + Na 2 S;
(pyrargyrite) Ag3SbS3 ; 2 Na [ Ag (CN )2 ] + Zn ® 2 Ag + Na 2 [ Zn (CN )4 ]

Hg Cinnabar HgS; direct reduction of (by heat alone)HgS:


HgS + O 2 ® Hg + SO 2
General Principles
General Principles & Processes of Isolation of Elements 655
Pt Native; Sperrylite PtAs 2 : Thermal decomposition of (NH 4 )2 Pt.Cl 6
(NH 4 )2 PtCl 6 ® Pt + 2 NH 4 Cl + 2Cl 2 .
Au Bismuth aurite BiAu; Sylvanite (Au Ag) Te2 Cyanide process:

4Au + 8KCN + 2H 2O + O2 ® 4K éë Au ( CN )2 ùû + 4KOH;

2K éë Au ( CN ) 2 ùû + Zn ® K 2 éë Zn ( CN ) 4 ùû + 2Au.

U Pitch blende or Uraninite U 3O8 or UO2 2UO3 ; Reduction of U3O8 by C :


Carnotite K 2 O.2UO 3.V2O 5 .3H 2 O. U 3O8 + 8C ® 3U + 8CO 98-99% pure Uranium is obtained
Antunite ( UO2 ) PO4 .8H 2O : by reducing U 3O8 by Mg, Ca or Al.
3U3O8 + 16Al ® 9U + 8Al2O3

C Free as diamond and graphite


(also known as plumbago i.e.black lead)
Si Sand SiO 2 ; Clay; Talc 2MgSi 2 O 5 Mg (OH )2
or Mg 3Si 4 O11.H 2O; Quatrz SiO 2 ; Feldspar KAlSi3O8 ;
Asbestos (CaMg3Si 4 O12 ); beryl Be 3Al 2Si 6 O18 ;
Mica KAlSi3O10(OH)2
N Free in air ; Ammonia: Chile Saltpetre; nitre.
P Phosphorite Ca 3 (PO 4 )2 ; Chlorapatite 3Ca 3 (PO 4 )2 .CaCl 2 ;
Fluorapatite 3Ca 3 (PO 4 )2 .CaF2 .
S Native; Gypsum CaSO 4 .2H 2O ;Celestine SrSO 4 ;
Galena PbS ; Zinc bledne ZnS ; Copper pyrites
Cu 2S.Fe 2S3 and Iron pyrites FeS2
F Fluorspar CaF2; Cryolite Na3AlF6,
Fluorapatite CaF2.3Ca3(PO4)2.
Cl In sea water, rock salts as NaCl;
Carnallite MgCl 2 .KCl.6H 2O.
Br Carnallite KCI.MgCl2 .6H 2 O contains 0.01to 0.1% as
Mg Br2 ; sea water as MgBr2 .
I Sea weeds as NaI .In Caliche (Chili saltpetre NaNO3 )
as NaIO3 (Sodium iodate).
Sb Stibnite Sb2S3 .

SOME IMPORTANT TERMS USED IN aqueous solution . It includes


METTALLURGY : (a) Leaching
(i) Pyrometallurgy - In this process decomposition of the (b) Reduction
minerals and the extraction of the metal is brought about in Ag and Au are extracted by this process.
dry state at high temperature by the action of heat. The steps (iii) Electrometallurgy is the process of obtaining metals
employed are through electrolysis. The electropositive metals are obtained
(a) Calcination (b) Roasting by this method e.g. Na, Mg etc.
(c) Smelting (d) Refining (iv) Amalgamation process In this process the metal is extracted
Smelting is melting process that causes the materials to by using mercury. Most of the metals dissolve in Hg to form
separate into two or more layers. amalgams which when distilled in iron retorts leave behind
Two important kinds of layers are slag and molten metal. Iron free metal and Hg distills over
is obtained by pyrometallurgy. distilled
Ore + Hg ® amalgam ¾¾ ¾¾® Hg + metal
(ii) Hydrometallurgy is the extraction of metals from ores using
Ag, Au, Pt form amalgam.
656 Chemistry

(v) Thermit - It is mixture of Fe2O3 (3 parts ) and Al powder (e) Metals obtained by precipitation method are - Ag ,Au
(f) Metals obtained by reduction with Co, Fe
(1part) when ignited with the help of barium peroxide, Fe2O3
(g) Metals obtained by reduction with water gas are - Ni
is reduced to iron and an enormous amount of heat is
produced due to exothermic nature of the reaction. (xv) Alloys -Alloys contain more than one element and have the
characteristics of metals.
Fe2 O3 + 2Al ® Al2 O3 + 2Fe + 2500o C · Pure metals and alloys have different physical properties.
The molten is utilised for welding and the process is known · Solution alloys are homogeneous mixtures and they are of
as thermit welding known as Goldschmidt Alumino thermic two types
process. (a) Subtitutional alloys (Solute atoms take the positions of
(vi) Refractory materials - The substance capable of solvent atom)
withstanding at very high temperature without undergoing (b) Interstitial alloys (Solute atoms occupy interstitial sites)
any deformation is called refractory material. · Heterogeneous alloys - Components are not dispersed
Acidic refractories -silica, quartz and sand stone uniformally e.g. pearlite steel .
Basic refractories - lime ,dolomite and magnesite RELATIVE ABUNDANCE :
Neutral refractories - chromite, bone ash and graphite. Abundance of elements in the earth’s crust (by weight)
(vii) Matte or regulus - Artificially obtained sulphides are known O > Si > Al > Fe > Ca > Na > K > Mg > H > Ti > Cl > F
as matte or regulus e.g. Cu 2S in extraction of copper..
Abundance of elements in the earth’s crust (in terms of number of
(viii)Sulphating roasting - It is partial oxidising roasting. Roasting atoms per 100 atoms)
of galena gives mixture of lead oxide and lead sulphate O > Si > Al > H > Na > Ca > Fe > Mg >
2 PbS + 3O 2 ® 2 PbO + 2SO 2 > K > Ti > C > Mn > S > F > Cl > Cr
PbS + 2O 2 ® 2 PbSO 4 THERMODYNAMIC PRINCIPLES OF METALLURY :
Theory of metallurgical transformations can be interpreted by
(ix) Chlorinating roasting- Silver ores mixed with common salt
Gibb’s free energy change at any specified temp.
when heated in presence of air, the chloride is obtained.
DG = DH - TDS or DGº = - RT lnK
Ag 2S + 2 NaCl ® 2AgCl + Na 2S
where, DH = enthalpy change and
(x) Bessemerisation -The oxidation of impurities by passing the
DS = entropy change
hot blast of air through molten metal in bessemer converter
is called bessemerisation . Pig iron and copper are purified K = equilibrium constt. at temp. T
by this method. The reducing agent is oxidised and metal oxide is reduced. The
(xi) Sintering-The conversion of small pieces of a substances role of reducing agent is to provide DGº negative.
into larger one by partial fusion is known as sintering. During reduction the metal oxide decomposes.
(xii)Pulverisation- The conversion of large pieces of a substance
M x O(s) ¾¾® xM (Solid or liquid) +12 O 2 (g)
into small fine pieces or powder is known as pulverisation.
(xiii)Anodizing-The process of forming an oxide coating on metal The reducing agent (C or CO) is oxidised
surface by making it an anode by electrolytic method in called 1
anodizing. C(s) + O 2 (g) ¾¾
® CO(g)
2
(xiv)Mode of extraction of some metals- 1
CO(g) + O 2 (g) ¾¾
® CO 2 (g)
(a) Metals obtained by electrolytic reduction are - Li, Na, K, 2
Mg, Ca , Al, Sr, Ba
C(s) + O 2 (g) ¾¾
® CO 2 (g)
(b) Metals obtained by reduction of oxides by carbon are -
Zn from ZnO, Sn from SnO2. If net DG of two possible reactions (Reduction/Oxidation) is
negative, the overall reaction will occur.
(c) Metals obtained by reduction of oxides by thermite
process, (Alumino thermic process) are Cr from Cr 2O3, H.J.T. Elligham diagram (plots of DG Vs T) provides a sound basis
Mn from Mn3O4. for considering the choice of reducing agent in the reduction of
oxides.
(d) Metals obtained by air reduction method are - Hg from
HgS, Pb from PbS.
General Principles
General Principles & Processes of Isolation of Elements 657
Note: Although thermodynamically feasible the magnesium metal,
is not used for the reduction of aluminas. The temperature required
would be so high and the process would be technologically difficult
and uneconomic.
EXTRACTION OF IRON FROM ITS OXIDES:
The two simple reactions are
1
(i) ® Fe(s / l) + O 2 (g); DG(FeO, Fe)
FeO (s) ¾¾
2

1
(ii) C(s) + O (g) ® CO(g); DG(C, CO)
2 2
Adding the two reactions, we get
FeO(s) + C(s) ® Fe(s/l) + CO(g)
DG(FeO, Fe) + DG (C, CO) = DG
The resultant DG is –ve above 1073K (approx).
If the metal is obtained in liquid state the reduction becomes easier.
Fig. Gibbs energy (DGº) vs T plots (schematic) for formation of (entropy increases and DG decreases).
some oxides (Ellingham diagram) LIMITATIONS OF ELLINGHAM DIAGRAM :
When alumina is reduced by magnesium, the two equations are 1. It fails to predict the kinetics of reduction processes i.e. how
(i) ® 2Al2 O 3 (ii) 2 Mg + O 2 ¾¾
4 Al + 3O 2 ¾¾ ® 2MgO fast it could be
2. When the reactant/product are solid DG° cannot be
DG becomes ZERO at the point of intersection of the Al2O3 and
interpreted by the equation DG° = – RT log K
MgO curves (marked “A”). Above this point the magnesium can
reduce alumina.
658 Chemistry

Very Short/Short Answer Questions 14. Explain the role of each of the following:
1. Write the names of two oxides which cannot be reduced with (i) NaCN in the extraction of silver.
carbon but can be reduced with aluminium powder to get the (ii) SiO2 in the extraction of copper.
corresponding metals. 15. (a) Which solution is used for the leaching of silver metal
2. Why are metal carbonates and sulphides first converted into in the presence of air in the metallurgy of silver?
corresponding oxides for finally obtaining the corresponding (b) Out of C and CO, which is a better reducing agent at the
metals? lower temperature range in the blast furnace to extract
3. Copper and silver lie below in the electrochemical series and iron from the oxide ore?
yet they are found in the combined state as sulphides in 16. (a) Which of the following ores can be concentrated by
nature. Comment. froth floatation method and why?
4. Indicate the temperature at which carbon can be used as a Fe2O3, ZnS, Al2O3
reducing agent for FeO.
(b) What is the role of silica in the metallurgy of Copper?
5. What is the metal extracted from malachite and azurite? Give
one important use of the metal.
Multiple Choice Questions
6. Name two metals each of which can be obtained 17. The metal always found in the free states is
(a) by thermite process (a) Au (b) Ag
(b) by electrolytic reduction (c) Cu (d) Na
(c) refined by liquation, and
18. Bronze is a mixture of
(d) refined by electrolysis.
(a) Pb + Sn (b) Cu + Sn
7. Name the type of reduction processes by which the following
metals are obtained from their oxides: (c) Cu + Zn (d) Pb + Zn
(a) Metal x which is low in the reactivity series. 19. Which reagent is used in Bayer’s process?
(b) Metal y which is in the middle of the reactivity series. (a) Na2CO3 (b) Carbon
(c) Metal z which is high up in the reactivity series.
(c) NaOH (d) Silica
8. What method for concentration of ore is preferred in each of
20. Matrix is defined as –
the following cases and why:
(a) The ore has higher density particles interspersed with a (a) the unwanted foreign material present in the ore
large bulk of low density impurities. (b) the flux added to remove the unwanted impurities from
(b) The ore consists of copper sulphide intermixed with ore
clay particles. (c) the slag formed as a result of the reaction of flux with
9. At a site, low grade copper ores are available and zinc and gangue
iron scraps are also available. Which of the two scraps will (d) the material used in the reduction of metal oxide to
be more suitable for reducing the leached copper ore and metal
why? 21. The process to heat the ore in the presence of excess supply
10. Although thermodynamically feasible, in practice, of air below its melting point is called –
magnesium metal is not used in for the reduction of alumina (a) roasting (b) calcination
in the metallurgy of aluminium. Why?
(c) smelting (d) liquation
11. Differentiate between a mineral and an ore.
22. Thomas slag is
12. Describe the principle controlling each of the following
processes: (a) Ca3(PO4)2
(i) Zone refining of metals (b) CaSiO3
(ii) Electrolytic refining of metals (c) Mixture of (a) and (b)
13. Which methods are usually employed for purifying the (d) FeSiO3
following metals? 23. Blister copper is
(i) Nickel
(a) Impure Cu (b) Cu alloy
(ii) Germanium
(c) Pure Cu (d) Cu having 1% impurity
General Principles
General Principles & Processes of Isolation of Elements 659
24. Which of the following fluxes is used to remove acidic Codes:
impurities in metallurgical process? (a) I-C, II-A, III-D, IV-B
(a) Silica (b) I-D,II-B,III-C,IV-A
(b) Lime stone (c) I-C,II-B,III-D, IV-A
(c) Sodium chloride (d) I-D,II-A,III-C,IV-B
(d) Sodium carbonate 26. In forth flotation process many chemicals (frother , collector,
25. Match list I with list II and select the correct answer using activator, and depressant) are used . Which is called a frother:
the codes given below the lists: (a) CuSO 4 (b) NaCN+ alkali
List I List II
(c) Pine oil (d) Potassium xanthate.
I. Cyanide process A. Ultrapure Ge
II. Floatation process B. Pine oil
III. Electrolytic reduction C. Extraction of Al
IV. Zone refining D. Extraction of Au

1. An example of an oxide ore is 9. Which of the following fluxes is used to remove acidic
(a) bauxite (b) malachite impurities in metallurgical process?
(c) zinc blende (d) feldspar (a) Silica (b) Lime stone
2. The natural materials from which an element can be extracted (c) Sodium chloride (d) Sodium carbonate
economically are called 10. Cryolite is
(a) ores (b) minerals
(a) Na 3AlF6 and used in the electrolysis of alumina for
(c) gangue (d) None of these
3. The impurities associated with mineral used in decreasing electrical conductivity
metallurgy are called collectively? (b) Na 3AlF6 and used in the electrolysis of alumina for
(a) Slag (b) Flux lowering the melting point of alumina
(c) Gangue (d) Ore
(c) Na 3AlF6 and used in the electrolytic purification of
4. The most abundant metal on the surface of the earth is
alumina
(a) Fe (b) Al
(c) Ca (d) Na (d) Na 3AlF6 and used in the electrolysis of alumina
5. The most abundant element in the earth’s crust (by weight) 11. Flux is used to
is
(a) remove all impurities from ores
(a) Si (b) Al
(b) reduce metal oxide
(c) O (d) Fe
(c) remove silica
6. During smelting an additional substance is added which
combines with impurities to form a fusible product. It is known (d) remove silica and undesirable metal oxide
as 12. Cassiterite is concentrated by
(a) slag (b) mud (a) levigation
(c) gangue (d) flux (b) electromagnetic separation
7. When a metal is to be extracted from its ore and the gangue (c) floatation
associated with the ore is silica, then (d) liquefaction
(a) an acidic flux is needed 13. Froth floatation process is used for the metallurgy of
(b) a basic flux is needed
(a) chloride ores (b) amalgams
(c) both acidic and basic fluxes are needed
(c) oxide ores (d) sulphide ores
(d) Neither of them is needed
14. Electromagnetic separation is used in the concentration of
8. A basic lining is given to a furnace by using
(a) calcined dolomite (b) lime stone (a) copper pyrites (b) bauxite
(c) haematite (d) silica (c) cassiterite (d) cinnabar
660 Chemistry
15. For which ore of the metal, froth floatation method is used 29. Use of electrolysis is
for concentration? (a) Electroplating (b) Electrorefining
(a) Horn silver (b) Bauxite (c) Both (a) and (b) (d) Neither (a) nor (b)
(c) Cinnabar (d) Heamatite 30. In electrorefining of copper some gold is deposited as
16. Which of the following metal is leached by cyanide process (a) anode mud (b) cathode mud
(a) Ag (b) Na (c) electrolyte (d) None of these
(c) Al (d) Cu 31. Purification of silicon element used in semiconductors is
17. Which one of the following ores is not concentrated by done by
(a) zone refining (b) heating
froth floatation process?
(c) froth floatation (d) heating in vacuum
(a) Copper pyrites (b) Pyrargyrite
32. Silver containing lead as an impurity is removed by
(c) Pyrolusite (d) Zinc blende
(a) poling (b) cupellation
18. Calcination is used in metallurgy for removal of?
(c) lavigation (d) distillation
(a) Water and sulphide (b) Water and CO2
33. Nickel is purified by thermal decomposition of its
(c) CO2 and H 2S (d) H 2 O and H 2S (a) hydride (b) chloride
19. Which of the following reactions is an example for calcination (c) azide (d) carbonyl
process ? 34. The chief source of iodine, in which it is present as sodium
(a) 2Ag + 2HCl + (O ) ® 2AgCl + H 2 O iodate, is
(a) Carnallite (b) Sea weeds
(b) 2Zn + O 2 ® 2 ZnO (c) Caliche
(d) Iodine never exists as sodium iodate.
(c) 2ZnS + 3O 2 ® 2ZnO + 2SO 2
35. Malachite is an ore of
(d) MgCO3 ® MgO + CO 2 (a) iron (b) copper
20. Heating of ore in the absence of air below its melting point is (c) mercury (d) zinc
called 36. The important oxide ore of iron is
(a) leaching (b) roasting (a) siderite (b) haematite
(c) smelting (d) calcination (c) pyrites (d) bauxite
21. Heating pyrites to remove sulphur is called 37. In the commercial electrochemical process for aluminium
(a) smelting (b) calcination extraction the electrolyte used is
(c) liquation (d) roasting
22. The process of converting hydrated alumina into (a) Al(OH )3 in NaOH solution
anhydrous alumina is called (b) An aqueous solution of Al 2 (SO 4 )3
(a) roasting (b) smelting
(c) dressing (d) calcination (c) A molten mixture of Al2 O 3 and Na 3 AlF6
23. Mac Arthur process is used for
(a) Ag (b) Fe (d) A molten mixture of Al 2 O 3 and Al(OH )3
(c) Cl (d) O2 38. Cassiterite is an ore of
24. In Goldschmidt aluminothermic process, reducting agent is (a) Mn (b) Ni
used
(c) Sb (d) Sn
(a) coke (b) Al powder
39. Which of the following element is extracted commercially by
(c) Na (d) Ca
the electrolysis of an aqueous solution of its compound?
25. Aluminothermic process is used for metallurgy of
(a) Pb (b) Ag (a) Chlorine (b) Bromine
(c) Al (d) None of these (c) Sodium (d) Aluminium
26. Which metal can’t be obtained from electrolysis? 40. Galena is an ore of
(a) Ca (b) Mg (a) Pb (b) Hg
(c) Cr (d) Al (c) Zn (d) Zn
27. Which of the following metals is obtained by 41. Which one of the following element does not exits in the
electrolytic reduction process? native form?
(a) Fe (b) Cu (a) Au (b) Pt
(c) Ag (d) Al (c) Fe (d) S
28. The common method of extraction of metals from oxide ores 42. The most electropositive metals are isolated from their ores by
is (a) high temperature reduction with carbon
(a) reduction with carbon
(b) self reduction
(b) reduction with hydrogen
(c) thermal decomposition
(c) reduction with aluminium
(d) electrolytic method (d) electrolysis of fused ionic salts
General Principles
General Principles & Processes of Isolation of Elements 661
43. The metal always found in the free states is 49. Match list I with list II and select the correct answer using
(a) Au (b) Ag the codes given below the lists:
(c) Cu (d) Na List I List II
44. Pyrolusite is a/an I. Cyanide process A. Ultrapure Ge
(a) oxide ore (b) sulphide ore II. Floatation process B. Pine oil
(c) carbide ore (d) Not an ore III. Electrolytic reduction C. Extraction of Al
45. Which of the following metals is extracted by the IV. Zone refining D. Extraction of Au
electrometallurgical method ? Codes:
(a) Cu (b) Fe (a) I-C, II-A, III-D, IV-B (b) I-D,II-B,III-C,IV-A
(c) Na (d) Ag (c) I-C,II-B,III-D, IV-A (d) I-D,II-A,III-C,IV-B
46. Among the following statements the incorrect one is 50. Which of the following condition favours the reduction of
a metal oxide to metal?
(a) Calamine and siderite are carbonates
(a) DH = +ve, TDS = + ve at low temperature
(b) Argentite and cuprite are oxides
(b) DH = +ve, TDS = – ve at any temperature
(c) Zinc blende and iron pyrites are sulphides (c) DH = –ve, TDS = – ve at high temperature
(d) Malachite and azurite are ores of copper. (d) DH = –ve, TDS = + ve at any temperature
47. When an aqueous solution of sodium chloride is electrolysed 51. Ellingham diagram normally consists of plots of
using platinum electrodes, the ion (a) DSº vs T (b) D fGº vs DSº
discharged at the electrodes are (c) DGº vs T (d) DHº vs DT
(a) sodium and hydrogen (b) sodium and chloride 52. A coupled reaction is takes place as follow–
(c) hydrogen and chloride (d) hydroxyl and chloride A + B ––––® C + D, D Gº = + x kj
48. In the extraction of iron , slag is produced.Slag is D + E ––––® F D Gº = – y kj
(a) CO (b) FeSiO3 for the spontaneity of reaction A + B + E –––® C+F,
which of the following is correct?
(c) MgSiO 3 (d) CaSiO 3 (a) 2x = y (b) x < y
(c) x > y (d) x = (y)× TDS

1. Which of the following statements, about the advantage of (a) Manganese (b) Carbon
roasting of sulphide ore before reduction is not true?
[CBSE-PMT 2007] (c) Silicon (d) Phosphorus
4. Which of the following pairs of metals is purified by van
(a) The DG of of the sulphide is greater than those for CS2 Arkel method ? [CBSE-PMT 2011]
and H2S. (a) Ga and In (b) Zr and Ti
(b) The DG of is negative for roasting of sulphide ore to oxide. (c) Ag and Au (d) Ni and Fe
5. The following reactions take place in the blast furnace in the
(c) Roasting of the sulphide to the oxide is thermodynamically
feasible. preparation of impure iron. Identify the reaction pertaining to
the formation of the slag. [CBSE-PMT 2011 M]
(d) Carbon and hydrogen are suitable reducing agents for
reduction of metal sulphides. (a) Fe2O3(s) + 3 CO(g) ®2 Fe (l) + 3 CO2 (g)

2. Sulphide ores of metals are usually concentrated by froth (b) CaCO3 (s) ®CaO (s) + CO2 (g)
flotation process. Which one of the following sulphide ores (c) CaO (s) + SiO2(s) ® CaSiO3(s)
offer an exception and is concentrated by chemical leaching?
(d) 2C(s) + O2 (g) ®2 CO(g)
[CBSE-PMT 2007]
6. Aluminium is extracted from alumina (Al2O3 ) by electrolysis
(a) Galena (b) Copper pyrite
of a molten mixture of : [CBSE-PMT 2012 S]
(c) Sphalerite (d) Argentite
(a) Al2O3 + HF + NaAlF4 (b) Al2O3 + CaF2 + NaAlF4
3. Which of the following elements is present as the impurity to
the maximum extent in the pig iron ? [CBSE-PMT 2011] (c) Al2O3 + Na3AlF6 + CaF2 (d) Al2O3 + KF + Na3AlF6
662 Chemistry
7. Which one of the following is a mineral of iron ? (c) Metal sulphides are less stable than the corresponding
[CBSE-PMT 2012 S] oxides
(a) Malachite (b) Cassiterite (d) CO2 is more volatile than CS2
(c) Pyrolusite (d) Magnetite 12. Pb and Sn are extracted from their chief ore by
8. In the extraction of copper from its sulphide ore, the metal is [IIT-JEE 2004S]
finally obtained by the reduction of cuprous oxide with : (a) carbon reduction and self reduction.
(a) Copper (I) sulphide (Cu2S) [CBSE-PMT 2012 S] (b) self reduction and carbon reduction.
(b) Sulphur dioxide (SO2) (c) electrolysis and self reduction.
(c) Iron sulphide (FeS) (d) self reduction and electrolysis.
(d) Carbon monoxide (CO) 13. Extraction of zinc from zinc blende is achieved by
9. Which one of the following ores is best concentrated by (a) electrolytic reduction [IIT-JEE 2007]
froth-flotation method ? [AIEEE 2004] (b) roasting followed by reduction with carbon
(a) Galena (b) Cassiterite (c) roasting followed by reduction with another metal
(c) Magnetite (d) Malachite (d) roasting followed by self-reduction
10. Heating mixture of Cu 2 O and Cu 2 S will give 14. In the cyanide extraction process of silver from argentite ore,
[AIEEE 2005] the oxidising and reducing agents used are [IIT-JEE 2012]
(a) Cu2SO3 (b) CuO + CuS (a) O2 and CO respectively
(c) Cu + SO3 (d) Cu + SO2 (b) O2 and Zn dust respectively
11. Which of the following factors is of no significance for (c) HNO3 and Zn dust respectively
roasting sulphide ores to the oxides and not subjecting the (d) HNO3 and CO respectively
sulphide ores to carbon reduction directly? [AIEEE 2008] 15. Sulfide ores are common for the metals
(a) Metal sulphides are thermodynamically more stable than (JEE Advanced 2013)
CS2 (a) Ag, Cu and Pb (c) Ag, Mg and Pb
(b) CO2 is thermodynamically more stable than CS2 (b) Ag, Cu and Sn (d) Al, Cu and Pb

1. Which process represents the change, (a) absorbance of ultraviolet light- and reemission of white light
Ti + 2I 2 ® TiI 4 ® Ti + 2I 2 (b) shock cooling by contact with a shower of molten lead.
(a) Cupellation (b) Van Arkel (c) X-ray method
(c) Polling (d) Zone Refining (d) smelting.
2. In froth flotation process many chemicals (frother , collector, 6. Among the following groups of oxides, the group containing
activator, and depressant) are used . Which of the folloiwng oxides that cannot be reduced by carbon to give the
is a frother:
respective metals is
(a) CuSO 4 (b) NaCN+ alkali
(a) Cu 2 O, SnO 2 (b) Fe 2O 3 , ZnO
(c) Pine oil (d) Potassium xanthate.
3. Froth flotation process is based on (c) CaO, K 2O (d) PbO, Fe3O 4
(a) wetting properties of ore particle
7. The electrolytic reduction technique is used in the extraction of
(b) specific gravity of ore particles
(c) magnetic properties of ore particles (a) highly electronegative elements
(d) electrical properties of ore particles . (b) highly electropositive elements
4. In the electrolysis of alumina, cryolite is added to: (c) metalloids
(a) lower the melting point of alumina and to increase the (d) transition metals.
electrical conductivity
8. In electro-refining of metal the impure metal is made the
(b) minimise the anode effect
anode and a strip of pure metal, the cathode, during the
(c) remove impurities from alumina
electrolysis of an aqueous solution of a complex metal salt.
(d) None of these
This method cannot be used for refining of
5. In the metallurgy of Zn the Zn dust obtained from roasting
and reduction of zinc sulphide contains some ZnO. It is (a) silver (b) copper
removed by (c) aluminium (d) sodium
General Principles
General Principles & Processes of Isolation of Elements 663
9. In the froth flotation process of concentration of ores, the 18. D G° Vs T plot in the Ellingham’s diagram slopes downward
ore particles float because they: for the reaction
(a) are light
1 1
(b) are insoluble (a) Mg + O 2 ® MgO (b) 2Ag + O 2 ® Ag 2O
2 2
(c) have the surface which is not wetted easily
(d) have a constant electrical charge 1 1
(c) C + O2 ® CO (d) CO + O 2 ® CO 2
10. Thomas slag is 2 2
(a) calcium silicate 19. Which of the following reactions taking place in the bast
(b) calcium phosphate furnace during extraction of iron is endothermic?
(c) tricalcium phosphate and calcium silicate (a) CaCO3 ® CaO + CO 2
(d) calcium ammonium phosphate
(b) 2C + O2 ® 2CO
11. Extraction of Ag from commercial lead is possible by
(a) Parke’s process (b) Clarke’s process (c) C + O 2 ® CO 2
(c) Pattinson’s process (d) Electrolytic process
(d) Fe 2O3 + 3CO ® 2Fe + 3CO 2
12. Iron is obtained on large scale from Fe2O3 by
20. Consider the following reactions at 1000°C
(a) Reduction with CO (b) Reduction with Al
1
(c) Calcination (d) Passing H2 A. Zn(s) + O 2 (g) ® ZnO(s); DG° = -360 kJ mol-1
13. After partial roasting the sulphide of copper is reduced by 2
(a) cyanide process (b) electrolysis
1
(c) reduction with carbon (d) self reduction B. C(gr) + O 2 (g) ® CO(g); DG° = -460 kJ mol –1
2
14. Before introducing FeO in blast furnace , it is converted to
Choose the correct statement at 1000°C
Fe2O3 by roasting so that
(a) zinc can be oxidised by carbon monoxide.
(a) it may not be removed as slag with silica
(b) zinc oxide can be reduced by graphite
(b) it may not evaporate in the furnace
(c) carbon monoxide can be reduced by zinc.
(c) presence of it may increase the m.pt. of charge (d) both statements (a) and (b) are true
(d) None of these. 21. The substance not likely to contain CaCO3 is
15. The phenomenon of removing layers of basic oxides from (a) calcined gypsum (b) sea shells
metals before electroplating is called (c) dolomite (d) a marble statue
(a) galvanising (b) anodising 22. The value of Df Gº for formation of Cr2 O3 is – 540 kJmol–1 and
(c) pickling (d) poling. that of Al2O3 is – 827 kJ mol–1 What is the value of DrG° for
16. Aluminothermic process is used for the extraction of metals the reaction?
, whose oxides are
4 2 2 4
(a) fusible Al(s) + Cr2 O 3 (s) ® Al 2 O3 (s) + Cr(s).
3 3 3 3
(b) not easily reduced by carbon
(c) not easily reduced by hydrogen (a) – 574 kJ mol –1
(b) –287kJ mol–1
(c) + 574 kJ mol –1
(d) +287kJ mol–1
(d) strongly basic.
23. Which of the following statement is not correct about
17. When the sample of copper with zinc impurity is to be purified
Ellingham diagram?
by electrolysis, the appropriate electrodes are
cathode anode (a) D G increases with an increase in temperature
(a) pure zinc pure copper (b) It consists of plots of DfGº Vs T for formation of
oxides
(b) impure sample pure copper
(c) A coupling reaction can be well expressed by this
(c) impure zinc impure sample diagram
(d) pure copper impure sample. (d) It express the kinetics of the reduction process
664 Chemistry

EXERCISE 1 15. (c) Cinnabar is sulphide ore (HgS). Hence purified by froth
floatation process.
1. Manganese oxide (MnO2) and chromium oxide (Cr2O3). 16. (a) Ag is leached by cyanide process (see text).
2. Reduction of oxides is easier 17. (c) Pyrolusite is MnO2. Hence not concentrated by froth
3. This is due to high polarising power of Cu and Ag ions floatation process.
4. Carbon can be used as a reducing agent above 1123 K. 18. (b) Calcination is used for removal of volatile impurities and
5. Copper is extracted from malachite and azurite. decompose carbonates.
19. (d) Decomposition of carbonates and hydrated oxides.
6. (a) Chromium and manganese
20. (d) It is definition of calcination (see text).
(b) Sodium and mangnesium
21. (d) Pyrites are sulphur ores and are converted into oxide by
(c) Tin and lead
roasting.
(d) Copper and silver
22. (d) Al2O 3.2H 2O ® Al 2O3 + 2H 2O is calcination.
7. (a) x is obtained by heating metal oxide alone.
23. (a) Mac Arthur process is used for Ag and Au (see text).
(b) y is obtained by the reducing its oxide with carbon. 24. (b) In Goldschmidt aluminothermic process reducing agent
(c) z is obtained by electrolytic reduction. is Al powder.
25. (d)
10. Because magnesium is a more costlier metal than aluminium
26. (c) Chromium is obtained by reduction with Al.
17. (a) 18. (b) 19. (c) 20. (a) 21. (a) 27. (d) Al is obtained by electrolytic reduction of Al2O3 in
presence of cryolite (see text).
22. (c) 23. (d) 24. (b) 25. (b) 26. (c) 28. (a) The common method for the extraction of metal from oxide
EXERCISE 2 ore is by reduction with carbon.
29. (c) Electrolysis is used for electroplating and
1. (a) Bauxite ore of aluminium is Al 2 O3 .2H 2 O . electrorefining both.
2. (a) Ores (see text). 30. (a) Gold is deposited as anode mud (see refining of copper).
3. (c) Impurities associated with minerals are called gangue or 31. (a) Silicon is purified by zone refining . Metals of high purity
matrix. are always purified by zone refining.
4. (b) Al, see text. 32. (b) Silver containing lead is purified by cupellation (see text).
5. (c) O, see text. 33. (d) Ni is purified by Mond’s process by decomposition of
6. (d) Flux is added which combines with impurities to form Ni(CO)4.
slag.
34. (c) Caliche is crude chilesaltpetre (NaNO3) and contains
7. (b) Since silica is acidic impurity the flux must be basic.
NaIO3, source of iodine.
CaO + SiO2 ® CaSiO3
8. (a) Dolomite on calcination gives CaO.MgO which provides 35. (b) Malachite is an ore of copper Cu ( OH ) 2 .CuCO3 .
basic lining in furnace. 36. (b) Important ore of iron is Haematite Fe 2 O3 .
9. (b) To remove acidic impurities basic flux is added which
37. (c)
is CaCO3 .
38. (d)
Cassiterite is an ore of Sn also known as tin stone SnO2.
10. (b) Na 3 AlF6 is cryolite and used in the electrolysis of 39. (a)
Cl2 is obtained by electrolysis of (aqueous) NaCl.
alumina to lower the melting point and increase electrical 40. (a)
Galena is an ore of lead. It is PbS.
conductivity. 41. (c)
Fe does not exist native.
11. (d) Flux removes acidic or basic impurities e.g. silica and 42. (d)
Most electropositive metals are obtained by electrolysis
other metal oxides etc. of their fused ionic salts.
12. (b) Cassiterite contains the magnetic impurities of FeSO 4 43. (a) Gold being least reactive found native.
and concentrated by electromagnetic separation. 44. (a) MnO2 is pyrolusite (oxide ore).
13. (d) Froth floatation process is used for the concentration of 45. (c) Na is obtained by elctrolytic reduction being
sulphide ores. electropositive in nature.
14. (c)
46. (b) Cuprite is Cu2O and Argentite is Ag 2 S .
General Principles
General Principles & Processes of Isolation of Elements 665
5. (c) In blast furnace at about 1270 K, calcium carbonate is
47. (c) almost completely decomposed to give CaO which acts
as a flux and combines with SiO2 present as impurity
(gangue) in the ore to form calcium silicate (fusible
slag)

(At cathode) (At anode) CaO(s) (basic flux) + SiO2 (s) (acidic flux) ¾¾ ®
CaSiO3 (s) (slag)
Reduction potential of H is more than Na. 6. (c) Fused alumina (Al2O3) is a bad conductor of electricity.
Therefore, cryolite (Na3AlF6) and fluorspar (CaF2) are
48. (d) Slag formed in the extraction of iron is CaSiO3 .
added to purified alumina which not only make alumina
49. (b) Cyanide process is for gold (I-D); floatation process - a good conductor of electricity but also reduce the
pine oil (II-B); Electrolytic reduction - Al (III-C); Zone melting point of the mixture to around 1140 K.
refining -Ge (IV-A).
7. (d) Fe3O4 – Magnetite
50. (d)
CuCO3 · Cu(OH)2 – Malachite
51. (c) Ellingham diagram normally consists of plots of DfGº
Vs T for the formation of oxides of elements. Pyrolusite – MnO2 and Cassiterite – SnO2.
52. (d) For a spontaneous reaction , D Gº must be negative 8. (a) Cuprous oxide formed during roasting of cuprous
and it can be possible only in this case when x < y sulphide is mixed with few amount of cuprous sulphide
and heated in a reverberatory furnace to get metallic
EXERCISE 3
copper.
1. (d) The sulphide ore is roasted to oxide before reduction
because the DGof of most of the sulphides are greater 2Cu 2O + Cu 2S ® 6Cu + SO2
than those of CS2 and H2S, therefore neither C nor H 9. (a) Galena is PbS and purified by froth floatation method
can reduce metal sulphide to metal. Further, the standard 10. (d) During bessemerisation, cuprous sulphide
free energies of formation of oxide are much less than
is oxidised which combines with remaining cuprous
those of SO2. Hence oxidation of metal sulphides to
sulphide to form free copper metal
metal oxide is thermodynamically favourable.
2. (d) Leaching is the selective dissolution of the desired 2Cu 2 O + Cu 2S ¾
¾® 6Cu + SO 2
mineral leaving behind the impurities in a suitable
dissolving agent e.g., 11. (c) The reduction of metal sulphides by carbon reduction
process is not spontaneous because DG for such a
Argentitie or Silver glance, Ag2S is an ore of silver.
process is positive. The reduction of metal oxide by
Silver is extracted from argentite by the mac-Arthur
carbon reduction process is spontaneous as DG for
and Forest process (leaching process).
such a process is negative. From this we find that on
Ag 2S + 4NaCN ® 2Na[Ag ( CN )2 ] + Na 2S thermodynamic considerations CO2 is more stable than
CS2 and the metal sulphides are more stable than
4Au + 8KCN + 2H 2O + O 2 ® 4K[Au ( CN ) 2 ] + 4KOH corresponding oxides.
3. (b) Pig iron or cast iron contains 3 – 5% carbon and varying In view of above the factor listed in choice (c) is incorrect
amounts of Mn, Si, P and S which makes the iron hard and so is of no significance.
and brittle. 12. (b) PbO & PbSO4 get reduced by PbS itself which is already
4. (b) Zr and Ti are purified by van Arkel method. present in mixture so because the reduction took place
870K by itself, hence is known as self reduction.
Zr( s ) + 2I 2 ( g ) ¾¾¾® ZrI 4 ( g )
D
2PbO + PbS ¾¾
® 3Pb + SO 2 ­
2075K
ZrI4 ( g ) ¾¾¾¾¾¾¾ ® Zr(s ) + 2I 2 ( g )
Tugsten filament
D
PbSO 4 + PbS ¾¾
® 2Pb + 2SO 2 ­
523K
Ti(s) + 2I2(s) ¾¾¾ ® TiI4(g) 13. (b) Extraction of Zn from ZnS (Zinc blende) is achieved by
1700K roasting followed by reduction with carbon.
2ZnS + 3O2 ¾¾
® 2ZnO + 2SO2
Ti(s) + 2I2(g)
Pure titanium ZnO + C ¾¾
® Zn + CO
666 Chemistry
14. (b) The reactions involved in cyanide extraction process 10. (c) Tricalcium phosphate and calcium silicate is Thomas slag
are : 11. (a) See Parke’s process
12. (a) Fe2O3 + 3CO ® 2Fe + 3CO2 ( in blast furnace)
Ag 2 S + 4NaCN ® 2Na [Ag(CN)2] + Na2S
(argentite) 13. (d) 2CuO + CuS ® 3Cu + SO 2 (Self - reduction)
14. (a) FeO is capable forming slag with SiO2
4Na2S + 5O 2 + 2H2O ® 2Na2SO4 + 4NaOH + 2S
Oxiding SiO 2 + FeO ® FeSiO 3
agent
15. (c) Pickling is removel of basic oxide layers on metals before
electroplating.
2Na[Ag(CN)2] + Zn ® Na2 [Zn(CN)4] + 2 Ag ¯ 16. (b) When reduction by carbon is not satisfactory in case of
(reducing
agent) metals having high m.pt., aluminothermic process is used
.
15 (a) Silver, copper and lead are commonly found in earth's
17. (d) Pure metal always deposits at cathode.
crust as Ag2S (silver glance), CuFeS2 (copper pyrites)
18. (c) 19. (a) 20. (b)
and PbS (galena)
21. (a) Gypsum is CaSO4.2H2O
EXERCISE 4 22. (b) The two equation are:
1. (b) The given reaction is the method named as Van Arkel for 4 2
the purification of titanium Al(s) + O 2 (g) ® Al2 O3 (s), D f Gº = -827kJ mol -1
3 3
2. (c) Froth reduces the surface tension of water and the
… (1)
solution forms froth.
3. (a) Froth flotation process is based on wetting properties of 4 2
Cr(s) + O 2 (g) ® Cr2 O3 (s), D f Gº = -540kJ mol -1
ore particles. 3 3
4. (a) Cryolite lowers the m.p of alumina and increases the … (2)
electrical conductivity Subtracting equation (ii) from equation (i) we have,
5. (d) See metallurgy of Zn 4 2
6. (c) Ca and K are strong reducing agents, hence their oxides Al(s) + Cr2 O 3 (s),
3 3
cannot be reduced with carbon
7. (b) Highly electropositive elements are obtained by 2 4
® Al2 O3 (s) + Cr(s), D r G° = -287kJ mol -1
electrolytic reduction. 3 3
8. (d) Na reacts vigorously with water (exothermic process ) 23. (d) Ellingham diagrams are based on thermodynamic con-
9. (c) The surface of particles not wetted hence they float at cepts. It does not tell anything about the kinetics of the
the surface reduction process.
21A

The p-Block Elements -


Nitrogen Family
GENERAL CHARACTERISTICS : 9. Allotropy : All the elements (except bismuth) show allotropy.
The group 15 of the periodic table consists of nitrogen, Nitrogen - a-nitrogen, b-nitrogen
phosphorous, arsenic, antimony and bismuth. These elements Phosphorous - white, Red, scarlet, violet, a-black, b-black
are known as pnicogens and their compounds as pnicomides. Arsenic - Grey, Yellow, Black
1. Electronic configuration : Antimony - Metallic, Yellow, Explosive
Element At. No. Electronic configuration Valence shell 10. Oxidation state :
N P As Sb Bi
electronic
configuration
–3 to +5 –3, +3, +4, +5 +3, +5 +3, +5 +3, +5
Nitrogen has a wide range of oxidation states
Nitrogen 7 [He] 2s2 2p3 2s2 2p3
Oxidation state Example
Phosphorou s 15 [Ne] 3s2, 3p 3 3s2 3p3

Arsenic 33 [Ar] 3d10, 4s2 4p3 4s2 4p3 +5 N2O5, HNO3, NO 3-


An timo ny 51 [Kr] 4d10, 5s2 5p3 5s2 5p3 +4 NO2, N2O4
Bismuth 83 [Xe] 4f14, 5d10 6s2 6p3 6s2 6p3 +3 HNO2, NO -2 , NF3
2. Metallic character : N, P(non metals), As, Sb(metalloids),
+2 NO
Bi(metal)
0 N2
3. Physical state : Nitrogen is first element after hydrogen to
–1 NH2OH, NH2F
be a diatomic gas in normal form. All other elements in the
group are normally solids. –2 N2H4
4. Atomicity : N2 is diatomic while others are tetra-atomic M4 –3 NH3, NH +4 , NH -2
5. Melting and boiling points : The melting point increases 11. Negative oxidation states : –3 oxidation state is exhibited by
from nitrogen to arsenic. The boiling points increase other elements also. Ca3P2, Na3As, Zn3Sb2
regularly on moving down the group. 12. Inert pair effect : Inert pair effect increases down the group
6. Density : Density increases down the group. and due to this effect the stability of +3 oxidation state
increases and stability of +5 oxidation state decreases on
7. Atomic radii : Atomic radii increases with increase in atomic moving down the group.
number. 13. Ionisation energy : Ionisation energy of nitrogen is very
8. Covalent radii : Covalent radii increases in a regular fashion high due to small atomic radius. The ionisation energy
down the group. decreases down the group.
668 Chemistry
14. Electronegativity : The electronegativity decreases from 2. Phosphine :
nitrogen to bismuth. a) Any metal phosphide + H 2O ® PH3
15. Catenation : They exhibit the property of catenation but
due to weak M–M bond to less extent than 14 group Ca 3P2 + 6H 2O ® 3Ca (OH) 2 + 2PH 3
elements.
b) P4 + 3KOH + 3H 2O ® PH 3 + 3KH 2 PO 2
Bond C–C N–N P–P As–As
kJ/mol 353.3 163.7 201.6 147.4 c) PH 4 I + NaOH ® NaI + H 2O + PH 3
16. Reactivity : Elemental nitrogen is higly unreactive largely 3. Other hydrides :
because of its strong triple bond. (almost as inert as noble
(a) Zn 3M 2 (s) + 6HCl ® 2MH 3 (g) + 3ZnCl 2 (aq)
gases).
While phosphorus is extremely reactive and kept in water. It M = As, Sb, Bi
is inflamable and can be ignited at 45ºC. It shows green (b) Halides : All the elements of this group form trihalides
luminiscence or glow in dark on account of its slow of th e type MX 3 and except nitrogen all form
oxidation. Th is glow phenomenon is known as pentahalides of the type MX5
phosphorescence. MX3 M = N, P, As, Sb, Bi and X = F, Cl, Br or I
17. Multiple bond formation : Only nitrogen has a tendency to MX5
form pp—pp multiple bonds. Others forms dp–pp multiple when X = F, M can be P, As, Sb and Bi
bonds easily. when X = Cl, M can be P, As and Sb
COMPOUNDS OF GROUP 15 ELEMENTS : when X = Br, M can be P
a) Hydrides : All the elements of this group form hydrides of NF3 is a colourless, odourless gas and the most stable
the type MH3 which are covalent and pyramidal in shape. of this series. It has low reactivity.
Some properties follow the order which are NCl3 is a yellow oily liquid that reacts with water to form
NH3 > PH3 > AsH3 > SbH3 > BiH3 ammonia and hypochlorous acid.
Ammonia Phosphine Arsine Stabene Bismuthene NI3 is shock sensitive and decomposes explosively
1. Ease of formation when touched.
2. Stability Hydrolysis :
3. Basic character NCl3 + 3H 2O ® NH 3 + 3HOCl
4. Solubility
PCl3 + 3H 2O ® H3PO3 + 3HCl
5. Bond angle (NH3 107.5º ; PH3 92º, AsH3 91, SbH3 90º)
6. Strength of M – H bond 2AsCl 3 + 3H 2 O ® As 2O 3 + 6HCl
7. Dipole moment SbCl 3 + H 2 O ® SbOCl + 2HCl
8. Decomposition temperature
Some properties follow the order BiCl 3 + H 2O ® BiOCl + 2HCl
NH3 < PH3 < AsH3 < SbH3 < BiH3 * Ease of hydrolysis BiCl3 > SbCl3 > AsCl3 > PCl3 > NCl3
1. Reducing character * Trihalides except BiF3 are covalent in nature
2. Covalent character * Trihalides have pyramidal structure.
3. Poisonous character (c) Pentahalides : As nitrogen does not contain-vacant d-
4. Rate of combustion orbitals in the second shell and cannot expand its
Boiling points : BiH3 > SbH3 > NH3 > AsH3 > PH3 outer shell hence it does not form pentahalides.
Preparation of hydrides : Some hydrides can be prepared as * The hybridisation in pentahalides is sp3d (trigonal
follows : bipyramidal)
1. Ammonia : * Thermally less stable than trihalides
(a) Any ammonium salt + metal oxide or * Act as Lewis acids PCl5 + Cl - ® [PCl -6 ]
D
hydroxide ¾¾® NH3 * On complete hydrolysis they produce acids
eg. NH 4 Cl + Ca (OH ) 2 ® CaCl 2 + H 2 O + NH 3 PCl5 + 4H 2 O ® H 3PO 4 + 5HCl
2 NH 4 NO 2 + K 2 O ® 2 NH 3 + 2KNO 2 + H 2 O (d) Oxides : All the elements of this group form oxides of
or (NH4)3PO4, NH4NO3, (NH4)2SO4, the type M2O3 and M2O5. either by direct combination
(NH4)2SO3, (NH4)2S or (NH4)2 (C2O4) with O2 or indirectly.
(b) Any nitride + H 2 O ® NH 3 ­ Oxides of N N2O5 , N2O4 , N2O3 , NO, N2O
strongly acidic
14243
BN + 3H 2 O ® H 3 BO 3 + NH 3 neutral

AlN + 3H 2 O ® Al(OH ) 3 + NH 3 Oxides of P P2 O 5 ( P4 O10 ) P2 O 3 ( P4 O 6 )


strongly acidic
The p-Block Elements - Nitrogen Family 669

Oxides of As As 2 O 5 As 2 O 3 Uses :
Amphoteric i) For manufacturing of nitric acid
ii) For detection of Oxygen
Oxides of Sb Sb 2 O 5 Sb 2 O 3 iii) For manufacturing of sulphuric acid as catalyst (Lead
Amphoteric chamber process)
Bi 2O 5 Bi 2 O3 Structure :
Oxides of Bi . .. .-. +.
Basic :N =O
..¬¾® N
.. =O:
N2O (dinitrogen oxide or nitrous oxide) known as laughing NO2 (nitrogen dioxide)
gas Preparation :
Preparation :
i) 2NO + O 2 ® 2 NO 2
(i) Priestley’s method :
ii) 2Pb ( NO 3 ) 2 ® 2PbO + 4 NO 2 + O 2
2 NO + H 2 O + Fe ® N 2O + Fe(OH) 2
(Common method)
(ii) Bertholet’s (common method) :
iii) Cu + 4HNO3 (conc.) ® Cu( NO3 )2 + 2H2O + 2NO2
D
NH 4 NO 3 ¾¾® N 2 O + 2 H 2 O Properties : Highly toxic, paramagnetic, reddish brown gas
(iii) 2NaNO3 + ( NH 4 ) 2 SO 4 ® N 2O + 2H 2O + Na 2SO 4 with choking odour, acidic
Reactions :
Properties :
Colourless, fairly unreactive, pleasing odour, sweet taste, i) 2NO 2 + H 2O ® HNO 2 + HNO 3 (Hence it is mixed
supports combustion. It is neutral. anhydride of HNO2 and HNO3)
D
2 N 2 O( g ) ¾¾® 2 N 2 (g ) + O 2 ( g ) ii) 2NO 2 N 2O4
brown colourless, solid / liquid, acidic
.-. + .. .. + .. -
.. = N =O
Structure : Linear N .. ¬
¾® N º N — O
.. : hn
iii) NO 2 ¾¾® NO + O
Uses : Mixed with oxygen it is used as anaesthetic iv) It is combustible and supports the combustion of
NO (nitrogen oxide or nitric oxide) : burning P, Mg or charcoal. Burning S or candle is
Preparation : extinguished.
Common method v) It is oxidising and reducing in nature
2NaNO2 + 2FeSO 4 + 3H 2SO4 Uses :
i) For Manufacturing of HNO3
® Fe 2(SO 4 )3 + 2NaHSO 4 + 2H 2O + 2NO
ii) As Catalyst in lead chamber process for Sulphuric acid
(i) D
N 2 (g ) + O 2 ( g ) ¾¾® 2 NO (g ) (commercial) Structure :
(ii) 3Cu (s) + 8HNO 3 (dil) ® 3Cu ( NO 3 ) 2 + 4H 2 O + 2 NO
(Lab method)
D , Pt N2O3 (dinitrogen trioxide) Nitrogen sesquioxide
(iii) 4NH3 + 5O2 ¾¾¾¾¾
750°C,6atm
® 4NO + 6H 2O
253K
Common method : 2NO + N 2 O 4 ¾¾¾ ® 2N 2 O 3
(Ostwald’s process)
Properties : Colourless, paramagnetic, slightly toxic gas, Preparation : NO + NO 2 ® N 2 O 3
blue in liquid state. It is combustible and supports Properties : It is blue solid, acidic
combustion. It is neutral.
N 2O 3 + H 2O ® 2HNO 2
D
FeSO 4 + NO ® FeSO 4 .NO ¾¾® FeSO 4 + NO Hence it is anhydride of HNO2
dark brown pure gas
Absorbed by sulphuric acid
D N 2 O 3 + 2H 2SO 4 ® 2 NO[HSO 4 ]2 + H 2 O
[Fe (H 2 O) 6 ]SO 4 + NO ® [Fe(H 2 O)5 NO]SO 4 ¾¾®
Hydrated nitrosyl Nitroso
complex sulphuric acid

Uses and Structure : Shape and structure is not definitely


® FeSO4 + NO + 5H 2 O :
O O
:

:
:

2 NO + O 2 ® 2 NO 2 (brown ) known N—N


: :
It is reducing as well as oxidsing in nature. : O
:
670 Chemistry
N2O5 (dinitrogen pentaoxide) (d) Oxyacids of N and P : Both form a number of oxy acids
Preparation : which are as follows :
i) 2HNO 3 + P2O 5 ® N 2 O5 + 2HPO 3 (common method) Oxidation Basicity
90º C number
ii) 4AgNO 3 + 2Cl 2 ¾¾¾® 4AgCl + 2 N 2 O 5 + O 2
1. Hyponitrous acid H2N2O2 +1
iii) N 2 O 4 + O 3 ® N 2 O 5 + O 2
2. Nitroxylic acid H4N2O4 +2
Properties : Colourless crystalline solid and sublimes
3. Nitrous acid HNO2 +3
N 2O 5 + H 2 O ® 2HNO 3
4. Nitric acid HNO3 +5
It is anhydride of nitric acid
5. Peroxy nitric acid HNO4 +5
N 2 O 5 + 2NaOH ® 2 NaNO3 + H 2 O
6. Hydronitrous acid H2NO2 +2
N 2 O 5 + H 2O 2 ® HNO 4 + HNO 3
7. Hypophosphorous acid H3PO2 +1 1
Pernitric acid

Uses : It is powerful oxidising agent 8. Phosphorous acid H3PO3 +3 2


9. Orthophosphoric acid H3PO4 +5 3
..
.. .

O:
O
.

.. 10. Pyrophosphoric acid H4P2O7 +5 4


:
Structure : N — .O. — N
: 11. Meta phosphoric acid HPO3 +5 1
.. .

O:
O
.
..

12. Hypophosphoric acid H4P2O6 +4 4


It’s ionic structure (by X-ray) is NO +2 NO 3- . It is also called O O O
nitronium nitrate. P P P
P2O3 or P4O6 (phosphorous trioxide) H H H OH HO OH
OH OH OH
D Hypophosphorous Phosphorous Phosphoric
Preparation : P4 + 3O 2 ¾¾® P4 O 6 acid acid acid
white limited
O O O O
Properties : white solid like wax, garlic odour, highly
poisonous. P P P
OH HO O OH HO O
P OH OH
Pyrophosphoric Metaphosphoric
O O acid acid
O
Structure : P P O O O O
O
P OH P P
P
O O HO P OH HO OH
P2 O 5 or P 4 O 10 (phosphorus pentaoxide) flower of OH O O
O P
phosphorous Hypophosphoric
acid O OH
Preparation : i) P4 + 5O 2 ® P4O10
white excess (HPO3) 3
Properties : White crystalline solid smells like garlic, Cyclo meta phosphoric
acid
sublimes
P4 O10 + 2 H 2 O( cold ) ® 4 HPO 3 O O O
meta phosphoric acid
P P P
O O O O
P4 O10 + 6H 2 O( hot ) ® 4H 3 PO 4 HO
ortho phosphoric acid
HO OH
Polymetaphosphoric
Uses : Powerful dehydrating agent acid (HPO3)n

O NITROGEN :
|| Discovered by Daniel Rutherford. Abundance in air is 78.15% by
P volume. It occurs in combined state as saltpetre (KNO3) and Chile
O O Saltpetre (NaNO3). It is also known as Azote (without life)
O Preparation :
Structure : O=P P=O
O i) NH 4+ + NO 2- ® N 2 + 2H 2 O
P
O || O ii) 2 NH 3 + 3CuO ® N 2 + 3H 2 O + 3Cu
O iii) 8 NH 3 + 3Cl 2 ® N 2 + 6 NH 4 Cl
The p-Block Elements - Nitrogen Family 671
D NITRIC ACID (HNO3) (ALSO KNOWN AS AQUA
iv) ( NH 4 ) 2 Cr2 O 7 ¾¾® N 2 + 4H 2 O + Cr2 O 3 . It is violent
FORTIS) :
reaction with flashes of light (volcano experiment)
Preparation :
v) H 2 NCONH 2 + 2HNO 2 ® 2N 2 + CO 2 + 3H 2 O Lab method :
Preparation of very pure N2 : By heating Sodium azide 2NaNO3 + H 2SO 4 (conc.) ® Na 2SO 4 + 2HNO 3
2 NaN3 ® 2 Na + 3N 2 Manufacturing :
Manufacturing : By Linde’s or Claude’s process (a) Birkland - Eyde process : Air is passed through an electric
Atmospheric air is compressed and then released into a bigger arc (3000ºC) when N2 combines with O2 to form NO. It is
area when liquid air is obtained (Joule Thomson effect) which is cooled and allowed to combine with O2 to form NO2. The
mainly mixture of N2 and O2. They are separated by fractional latter is passed in water in presence of excess of air to give
distillation. HNO3.
Properties : It is colourless, odourless, tasteless, slightly lighter N 2 + O 2 ® 2 NO
than air, slightly soluble in water, non poisonous gas.
It is incombustible and non supporter of combustion. It combines 2NO + O 2 ® 2 NO 2
with metals and non metals to form number of compounds. 4 NO 2 + 2H 2 O + O 2 ® 4HNO 3
Uses : To decrease concentration of oxygen in air and make
(b) Ostwald’s Process : From ammonia
combustion less rapid. To create inert atmosphere and in the
preparation of NH3, HNO3, CaCN2, etc. Pt gauge
4NH 3 + 5O 2 ¾¾¾¾® 4NO + 6H 2O
800ºC
AMMONIA (NH3) :
Preparation : Lab method : 2NO + O 2 ® 2 NO 2
2NH 4Cl + Ca (OH) 2 ® CaCl 2 + 2 NH 3 + 2H 2O 4 NO 2 + 2H 2 O + O 2 ® 4HNO 3
Manufacturing : Properties : Syrupy, colourless, pungent liquid usually
(a) Haber’s process : available as 68% and 15.7 M. Aqueous solution is often
Fe/Mo
yellow due to small concentrations of NO2
N 2 + 3H 2 ¾¾¾¾
® 2NH 3
450ºC, 245 Atm. FUMING NITRIC ACID (HNO3 + NO2)
Other catalysts employed are Chemical properties : As an acid - It is a strong acid and in
aqueous solution the ionisation is virtually complete.
(i) finely divided Os or U
(ii) Finely divided Ni deposited over pumice stone HNO 3 + H 2 O H 3 O + + NO 3-
(iii) Fe(OH)3 with traces of SiO2 and K2O Thus it reacts with basic oxides, hydroxides, carbonates etc.
(b) Cyanamide process :
CaO + 2H 3 O + ® Ca 2+ + 3H 2 O
CaC2 + N 2 ® (CaCN 2 + C)
Nitrolim As oxidising agent
Mixture of Calcium Cyanamide and graphite under the name
NO 3- + 2 H 3O + + e - ¾¾¾® NO 2 + 3H 2 O
conc .
of nitrolim is used as fertilizer.
NO 3 + 4H 3O + + 3e - ¾¾® NO + 6 H 2 O
dil.
CaCN 2 + 3H 2O ® CaCO 3 + 2 NH 3
(c) Serpeks process : As by product during hydrolysis of AlN or 2HNO 3 ® H 2O + 2 NO 2 + O (concentrated HNO3)
AlN + 3H 2O ® Al(OH )3 + NH 3
2HNO 3 ® H 2 O + 2 NO + 3O (dil. HNO3)
Properties : Colourless gas, characteristic pungent odour,
brings tears into eyes, collected by downward displacement Oxidation of non metals, - ic acids are formed eg.
of air. Extremely soluble in water due to H-bonding. It is a S + H 2O + 3O ® H 2SO 4 (Sulphuric acid)
strong lewis base.
C + H 2O + 2O ® H 2 CO 3 (Carbonic acid)
* Ammonia is dried over any metal oxide but CaO is
cheaper. 2P + 3H 2 O + 5O ® 2H 3PO 4 (Phosphoric acid)
* When passed through alkaline solution of Nesslers
I 2 + H 2 O + 5O ® 2HIO 3 (Iodic acid)
reagent a brown coloured complex known as Millon’s
base is formed. 3As + 3H 2 O + 5O ® 2H 3AsO 4 (Arsenic acid)
2K2HgI4 + 3KOH + NH3
Sn + H 2 O + 2O ® H 2SnO3 (meta stannic acid)
I
Hg Hg + 7KI + 2H2O Se + H 2 O + 2O ® H 2SiO 3 (selenious acid)
I O NH2
Millon’s base Te + H 2 O + 2O ® H 2TeO 3 (Tellurous acid)
672 Chemistry
Oxidation of compounds : Conc. HCl). eg.:
2HNO 3 ® H 2O + 2 NO 2 + O , SO 2 + H 2 O + O ® H 2SO 4 HNO 3 + 3HCl ® NOCl + 2 H 2 O + 2Cl
nascent
H 2S + O ® H 2 O + S , 2HBr + O ® H 2O + Br2 chlorine

FeS + 4O ® FeSO 4 Au + 3Cl ® AuCl3


C12H 22O11 + 18O ® 6(COOH) 2 + 5H 2 O AuCl3 + HCl ® HAuCl 4
The action of Nitric acid on metals : Armstrong’s theory - The Chloro auric acid
metal first displaces nascent hydrogen from acid which further
Similarly platinum forms - Pt + 4Cl ® PtCl 4
reacts with acid to give secondary reactions.
Metal + HNO3 ® Metal nitrate + H (Primary reaction) PtCl 4 + 2HCl ® H 2 PtCl 6
Chloropla tan ic
acid
2HNO3 + 2H ® 2NO2 + 2H 2O ü
ï Structure of nitric acid :
2HNO3 + 6H ® 2NO + 4H 2 O ï .. ..

..
H O H O
ï .. .. .. ..
2HNO3 + 8H ® N 2 O + 5H 2O ý secondary reactions O N
.. O
.. N
2HNO3 + 10H ® N 2 + 6H 2 O ï

.. .
O

.. .
O
ï

.
..

.
2HNO3 + 16H ® 2NH3 + 6H 2O ïþ Uses : In the manufacture of fertilizers ii) For purification of silver
and gold iii) In the manufacture of explosives iv) oxidising reagent
Factors affecting the secondary reactions v) As nitrating reagent
i) Nature of the metal ii) Concentration of the acid NITROUS ACID (HNO2) :
iii) Temperature iv) Presence of impurities Preparation :
Action of nitric acid on zinc under different conditions
(i) Cold and very dil. acid evolves ammonia which reacts with (i) Ba ( NO 2 ) 2 + H 2SO 4 (dil.) ® BaSO 4 ¯ +2HNO 2
HNO3 forming ammonium nitrate (ii) 2NaNO 2 + H 2SO 4 ® Na 2SO 4 + 2HNO 2
4Zn + 10HNO 3 ® 4Zn( NO3 ) 2 + 3H 2 O + NH 4 NO3
(iii) N 2O 3 + H 2O ® 2HNO 2
(ii) Cold and dil HNO3
(iv) NH 3 + 3H 2O 2 ® HNO 2 + 4H 2 O
4Zn + 10HNO 3 ® 4Zn ( NO 3 ) 2 + 5H 2O + N 2 O
Properties : It has slight bluish colour in solution may be due to
(iii) Cold and moderately conc.
anhydride N2O3. It is very unstable.
3Zn + 8HNO 3 ® 3Zn( NO3 ) 2 + 4H 2O + 2 NO (i) Decomposition :
(iv) Cold and concentrated
3HNO 2 ® 2 NO + HNO 3 + H 2 O (auto oxidation)
Zn + 4HNO 3 ® Zn( NO3 ) 2 + 2H 2O + 2NO 2
(ii) Action of heat :
Action of nitric acid on Copper under different conditions
2HNO 2 ® H 2O + N 2 O3 ® NO 2 + NO
(i) Cold and dil.
4Cu + 10HNO 3 ® 4Cu( NO3 ) 2 + 5H 2O + N 2O (iii) Oxidising nature : 2HNO 2 ® 2 NO + H 2O + O
(ii) Cold and moderately concentrated H + + HNO 2 + e - ® H 2 O + NO (electron acceptor)
3Cu + 8HNO 3 ® 3Cu ( NO 3 ) 2 + 4H 2O + 2NO
(iv) Reducing nature : HNO 2 + O ® HNO 3
(iii) Cold and concentrated
Cu + 4HNO 3 ® Cu( NO3 ) 2 + 2H 2O + 2 NO 2 H 2 O + HNO 2 ® HNO 3 + 2H + + 2e - (electron donor)
(iv) Hot and conc. (v) Reaction with ammonia :
5Cu + 12HNO 3 ® 5Cu ( NO 3 ) 2 + 6H 2O + N 2 NH 3 + HNO 2 ® NH 4 NO 2 ® N 2 + 2H 2O
Metals like Mg and Mn give hydrogen with dil. HNO3 (vi) Formation of diazonium compounds :
Mg + 2HNO 3 ® Mg( NO3 ) 2 + H 2 C 6 H 5 NH 2 HCl + HNO 2 ® C6 H 5 N = NCl + 2H 2O
Uses : In the manufacture of azo dyes.
Mn + 2HNO 3 ® Mn ( NO 3 ) 2 + H 2
Structure : It is a tautomeric mixture of the following forms
Passivity : Metals like, Fe, Cr, Ni, Al or Co become inactive or ..
passive due to stable oxide layers. O
..

.. .. .. ..
Noble Metals like Pt, Pd, Os, Ir and Au do not react with nitric H O
.. N O
.. H N
.. .

acid. They react with aqua regia (1 vol. Conc. HNO3 + 3 vol. O
.
The p-Block Elements - Nitrogen Family 673
PHOSPHOROUS : (ii) With metals :
Discovered by Brand 3Na + P ® Na 3P 3Ca + 2P ® Ca 3P2
Occurence : It occurs in combination only as phosphates (iii) With compounds :
(i) Phosphorite Ca3(PO4)2
4P + 3NaOH + 3H 2 O ® 3NaH 2 PO 2 + PH 3
(ii) Chlorapatite CaCl2.3Ca3(PO4)2
(iii) Fluorapatite CaF2.3Ca3(PO4)2. P + 5HNO 3 ® H 3PO 4 + H 2O + 5NO 2
In phosphoproteins of brain, bones, teeth, milk, egg, nervous 2P + 5H 2SO 4 ® 2H 3PO 4 + 2H 2O + 5SO 2
tissues of animal and plants.
Uses :
Manufacturing : By reduction of calcium phosphate with carbon
In matches, explosives, as rat poison and in fertilizers and alloys.
in presence of SiO2 in an electric furnance
Match box :
2Ca 3 (PO 4 ) 2 + 6SiO 2 ® 6CaSiO 3 + P4 O10 Side contains : Red P or P2S3 + Sand + Glue
P4O10 + 10C ® P4 + 10CO On tip : Red P + Oxidising agent like KClO3 or KNO3 or Pb3O4
+ glass powder or chalk for friction + glue
Purification : By melting under acidified solution of K2Cr2O7.
The impurities are oxidised and redistilled. PHOSPHINE (PH3) :
Properties : Freshly prepared phosphorous is colourless. On Preparation :
standing acquires pale lemon colour due to formation of red (i) Any phosphide +H2O ® PH3
phosphorus on the surface. It is therefore called yellow
phosphorous. Due to its poisonous nature the jaw bones decay Ca3P2 + H2O ® 2PH3 + Ca(OH)2
and disease is known as “Phossy jaw” Na3P + 3H2O ® PH3 + 3NaOH
Allotropic forms of phosphorous and their preparation :
i) Red phosphorous : It is prepard by carefully heating yellow 2AlP + 3H 2SO 4 ® 2PH 3 + Al 2 (SO 4 )3
phosphorous in an inert atmosphere for about 8 days. (ii) Decomposition of H3PO3 :
ii) Violet phosphorous : By crystallisation of white 4H 3 PO 3 ® 3H 3PO 4 + PH 3
phosphorous from molten lead
(iii) Lab. method :
iii) Scarlet :
1. By exposing the solution of red P in PBr 3 to light or by 4P + 3NaOH + 3H 2 O ® 3NaH 2 PO 2 + PH 3
boiling.
(iv) Pure PH3 : PH 4 I + NaOH ® PH 3 + NaI + H 2 O
2. By heating PBr 3 with Hg at 513 K
Properties : Colourless, highly poisonous, with rotten fish odour
4PBr3 + 6Hg ® 6HgBr2 + P4 gas, slightly soluble in water
iv) Black : By heating white P to 473K under 1000kg/sq. cm.. It Chemical properties :
is the most stable form, good conductor of electricity.
(i) Basic nature : PH 3 + HI ® PH 4 I
Some points of distinction between white and red phosphorous
Property White Red (ii) Decomposition : 4PH 3 ¾311K
¾¾® P4 + 6 H 2
P P P P

1 . Structure P P P P P P P P (iii) Combustibility : 4PH 3 + 8O 2 ® P4O10 + 6H 2 O


P P P P
2. Colour Light yellow Violet Pure PH3 is not spontaneously inflammable. Ordinary PH3
3. Odour Garlic Odourless is spontaneously inflammable due to the presence of P2H4
4. Conductivity Bad Conductor Semi Conductor (iv) With metallic salts :
5. Physiological action Poisonous Non poisonous
6. Density 1.81 2.1 3CuSO 4 + 2PH 3 ® Cu 3P2 + 3H 2SO 4
7. Melting point 44ºC 500ºC
(v) With chlorine : PH 3 + 3Cl 2 ® PCl 3 + 3HCl
8. Ignition temperature 35ºC 240ºC
9. Hardness Soft Brittle Uses :
10. Solubility in CS2, Soluble Insoluble (i) Holme’s signals : A mixture of CaC2 and Ca3P2 when treated
CCl4 & benzene
with water, phosphine is liberated which catches fire and
11. Action of KOH PH 3 No action
12. Action of CuSO4 Cu3P2 No action
lights up acetylene. Burning gases serve the purpose of a
13. Action of Cl2 PCl3 or PCl5 On heating PCl3 or PCl5 signal. They are used in ships.
14. In dark Shines does not (ii) Smoke screen : Ca3P2 is used smoke screen. PH3 obtained
from it catches fire to give the needed smoke.
Chemical properties :
(iii) Cellphos : It is trade name of AlP, aluminium phosphide and
(i) With non metals :
used as fumigant. In presence of moisture it gives PH3 which
4P + 3O 2 ® 2P2O 3 , 4P + 5O 2 ® 2P2O 5 kills insects and pests
(iv) Rat poison : Zinc phosphide Zn3P2 is a rat poison, which
2P + 3Cl 2 ® 2PCl3 , 2P + 5Cl 2 ® PCl 5
gives PH3
674 Chemistry
ORTHOPHOSPHORIC ACID (H3PO4) : FERTILIZERS :
Preparation : Fertilizers are the chemical substances which are added to soil in
(i) P4 O10 + 6 H 2 O ® 4H 3 PO 4 order to make up the deficiency of nutrients required by plants.
Nutrients are classified as
(ii) D
Red P + 5HNO 3 ¾¾® H 3 PO 4 + H 2 O + 5 NO 2 (i) Primary nutrients : which are consumed in large quantities
(iii) Ca 3 (PO 4 ) 2 + 3H 2SO 4 ® 3CaSO 4 + 2H 3 PO 4 eg. Nitrogen, Phosphorous and Potassium
(ii) Secondary nutrients : Calcium and Magnesium
(iv) PCl 5 + 4 H 2 O ® H 3 PO 4 + 5HCl
(iii) Micro nutrients : which are required in minute quantities
Properties : Colourless syrupy liquid eg. Copper, Zinc, Manganese, Boron, Molybdenum, chlorine
(i) Action of heat : and Iron
250 º C
2H 3 PO 4 ¾¾ ¾
¾® H 4 P2 O 7 + H 2 O ¾¾ ¾®
316º C TYPES OF FERTILIZERS :
Pyrophosph oric (i) Nitrogenous fertilizers : These provide nitrogen to the
acid
plants. They are
® 2 HPO 3 + H 2 O (a) Urea :
meta phosphoric 200 Atm.
acid 2 NH 3 + CO 2 ¾¾ ¾ ¾® H 2 NCOONH 4
(ii) It is tribasic and ionises in three steps 2H 2 NCOONH 4 ® H 2 N.CONH 2 + H 2O
H 3 PO 4 ® H + + H 2 PO -4 (Readily) It contains about 47% N2
(b) Ammonium sulphate (Sindri fertilizer) :
H 2 PO 4- ® H + + HPO -4 - (Weakly) NH 3 + H 2 O ® NH 4 OH

HPO 4- - ® H + + PO -4 -- (very weak ionisation) 2 NH 4OH + CO 2 ® ( NH 4 ) 2 CO 3 + H 2O

(iii) H 3 PO 4 + 12( NH 4 ) 2 MoO 4 + 21HNO 3 ® ( NH 4 ) 2 CO 3 + CaSO 4 ® ( NH 4 ) 2 SO 4 + CaCO 3 ¯


Ammonium (c) Basic Calcium nitrate : (nitrate of lime or Norwegian
molybdate saltpetre)
(NH 4 ) 3 PO 4 .12MoO3 + 21NH 4 NO3 + 12H 2O CaCO3 + 2HNO 3 ® Ca ( NO 3 ) 2 + H 2O + CO 2
Ammonium
phosphomolybdate Ca ( NO 3 ) 2 + CaO ® Ca ( NO3 ) 2 .CaO
Uses : (d) Calcium cyanamide :
(i) For preparation of HBr, HI in laboratory 850 -1000ºC
CaC 2 + N 2 ¾¾¾¾¾® [CaCN 2 + C]
(ii) For preparing metaphosphoric acid Nitrolim
(iii) stabiliser for H2O2 CaCN 2 + 3H 2O ® CaCO 3 + 2 NH 3
ORTHOPHOSPHOROUS ACID (H3PO3) : or CaCN2 + 2H 2O + CO2 ® CaCO3 + H 2 N.CONH 2
Preparation :
H 2 N.CO.NH 2 + H 2 O ® CO 2 + 2 NH 3
(i) P4 O 6 + 6H 2 O ® 4H 3 PO 3
(ii) Phosphatic fertilizers : These provide phosphorous to
(ii) PCl 3 + 3H 2 O ® H 3 PO 3 + 3HCl plants
Properties : It crystallises as deliquescent white solid (a) Super phosphate of lime :
(i) Acidic nature : Ca 3 (PO 4 ) 2 + 2H 2SO 4 ® Ca (H 2 PO 4 ) 2 + 2CaSO4
14444244443
+
H 3 PO 3 H + (H 2 PO 3- ) +
H + ( HPO 3 ) 2-
available phosphorous 16 - 18% P2O5
(ii) Decomposition : 4H 3 PO 3 ® 3H 3PO 4 + PH 3 (b) Phosphatic slag or Thomas slag :
This reaction is disproportionation 4P + 5O 2 ® P4 O10
(iii) Reducing nature :
6CaO + P4 O10 ® 2Ca 3 (PO 4 ) 2
SO 2 + 2 H 3 PO 3 ® 2H 3 PO 4 + S ¯ It is by product of steel industry, available phosphorous
3PCl5 + H 3PO 3 ® PCl3 + 3POCl 3 + 3HCl 14-18% P2O5
(c) Triple super phosphate :
H 3PO 3 + I 2 + H 2O ® H 3PO 4 + 2HI
Ca 3 (PO 4 ) 2 + 4H 3PO 4 + 3Ca (H 2 PO 4 ) 2
(colour of I2 discharged)
(d) Nitrophos :
2KMnO 4 + 3H 2SO 4 + 5H 3PO 3 ® Ca 3 (PO 4 ) 2 + 4HNO3 ® Ca (H 2 PO 4 ) 2 + 2Ca ( NO3 ) 2
144444244444 3
® K 2SO 4 + 3H 2 O + H 3PO 4 + 2MnSO 4 (iii) Potash fertilizers : These provide potassium to plants. Eg.
CuSO 4 + H 3PO 3 + H 2O ® Cu + H 2SO 4 + H 3PO 4 : KCl, KNO3, K2SO4
(iv) Mixed fertilizers : a) Ammoniated superphosphate : It is
Hg 2Cl 2 + H 3PO 3 + H 2 O ® 2Hg + 2HCl + H 3PO 4 prepared by spraying ammoniation solution (NH4NO3 =
Uses : As reducing agent 65.0%, NH3 = 21.7% and H2O = 13.27%) on superphosphate.
The p-Block Elements - Nitrogen Family 675

Very Short/Short Answer Questions 19. If you touch concentrated HNO3 with your finger and
immediately wash it with water, then the skin at the place
1. On being slowly passed through water, PH3 forms bubbles where it came into contact with HNO3 becomes yellow. This
but NH3 dissolves. Why is it so? is because of :
2. What is the difference between the nature of pi-bonds present (a) the formation of Xanthoprotein
in H3PO3 and HNO3? (b) the absorption of HNO3 by skin
3. NO2 is coloured but its dimer is colourless. Why? (c) the blood absorbing HNO3
4. N atom possesses 5 valence electrons but does not form (d) the absorption of yellow/brown NO2 gas
20. PCl5 is possible but NCl5 does not exist :
NCl5. Why?
(a) in N, d-sub-shell is absent
5. NF3 is an exothermic compound but NCl3 is an endothermic (b) ionization energy of N is very high
compound. Explain. (c) it does not like Cl
6. What is laughing gas? How is it prepared? (d) none of these
7. PCl5 is ionic in nature in the solid state. Why? 21. The boiling points of the following hydrides follow the order
8. In trimethylamine, nitrogen has a pyramidal geometry of
whereas in trisilylamine N(SiH3)3 it has a planar geometry? (a) SbH 3 > NH 3 > AsH 3 > PH 3
9. Molecular nitrogen N2 is not particularly reactive. Explain. (b) NH 3 > AsH 3 > PH 3 > SbH 3
10. NH3 is more basic than PH3. Explain.
(c) SbH 3 > AsH 3 > PH 3 > NH 3
11. Present a comparative account of proton affinities of NH3
and PH3. (d) NH 3 > PH 3 > AsH 3 > SbH 3
12. Which of the following is/are not known and why: PCl3, 22. Which of the following statements is not correct for
AsCl3, SbCl3, NCl5, BiCl5 nitrogen ?
(a) Its electronegativity is very high
13. Both PCl+4 and SiCl4 have tetrahedral structures. Explain.
(b) d-orbitals are available for bonding
Long Answer Questions (c) It is a typical non-metal
(d) Its molecular size is small
14. In what ways nitrogen differs in its chemical behaviour from 23. The structural formula of hypophosphorous acid is
that of its heavier congeners? O O
15. Discuss the pattern of variation in the oxidation states of P P P
to Bi (a) H (b) H
OH OH
H OH
Multiple Choice Questions O O
16. Which one of the following statements is true for HNO2 ? P P
(c) HO (d) H
(a) It can not act as reducing agent OH OOH
(b) It can not act as oxidising agent OH OH
24. Producer gas is a mixture of :
(c) It can not act both as oxidant reductant
(a) CO and N 2 (b) CO 2 and H 2
(d) It is stable only in aqueous solution.
17. Of the following which is paramagnetic and has three (c) N 2 and O 2 (d) CH 4 and N 2
electron bond in its structure 25. Which of the following oxides of nitrogen is a coloured
(a) N2O (b) NO gas?
(a) N2O (b) NO
(c) N2O3 (d) N2O5
(c) N2O5 (d) NO2
18. With respect to protonic acids, which of the following 26. Which is the most thermodynamically stable allotropic form
statements is correct ? of phosphorus?
(a) PH3 is more basic than NH3 (a) Red (b) White
(b) PH3 is less basic than NH3 (c) Black (d) Yellow
(c) PH3 is equally basic as NH3 27. The number of P – O – P bonds in cyclic metaphosphoric
acid is
(d) PH3 is amphoteric while NH3 is basic.
(a) zero (b) two (c) three (d) four
676 Chemistry

1. Which one of the following elements is most metallic ? 13. The one which does not form pentachloride is
(a) P (b) As (a) nitrogen (b) phosphorus
(c) Sb (d) Bi (c) arsenic (d) antimony
2. In NH3 and PH3 the common is 14. Which of the following compounds does not exist?
(a) odour (b) combustibility (a) AsCl5 (b) SbCl3
(c) basic nature (d) None of these (c) BiCl5 (d) SbCl5
3. Which element from group 15 gives most basic compound 15. What is hybridization of P in PCl5 ?
with hydrogen (a) sp3 (b) sp3d 2
(a) Nitrogen (b) Bismuth
(c) sp3d (d) sp2
(c) Arsenic (d) Phosphorus
16. Nitrous oxide is
4. Ionic radii (in Å) of As3+, Sb3+ and Bi3+ follow the order
(a) soluble in cold water (b) soluble in hot water
(a) As3+ > Sb3+ > Bi3+ (b) Sb3+ > Bi3+ >As3+
3+ 3+
(c) Bi > As > Sb 3+ (d) Bi3+ > Sb3+ > As3+ (c) acidic in nature (d) None of these
5. The three important oxidation states of phosphorus are 17. Which of the following oxides of nitrogen reacts with FeSO4
(a) –3, +3 and +5 (b) –3, +3 and –5 to form a dark brown compound
(c) –3, +3 and +2 (d) –3, +3 and +4 (a) N2O (b) NO
6. The basic character of hydrides of the group 15 elements (c) NO2 (d) N2O3
decreases in the order 18. Which of the following oxides is the most acidic?
(a) SbH3 > PH3 > AsH3 > NH3 (a) N2O5 (b) P2O5
(b) NH3 > SbH3 > PH3 > AsH3 (c) As2O 5 (d) Sb2O5
(c) NH3 > PH3 > AsH3 > SbH3 19. In the reaction
(d) SbH3 > AsH3 > PH3 > NH3 4HNO 3 + P4 O10 ® 4HPO 3 + X , the product X is
7. The stability of the hydrides follows the order
(a) NH3 > PH3 > AsH3 > SbH3 (a) N2O5 (b) N2O3
(b) NH3 < PH3 < AsH3 < SbH3 (c) NO2 (d) H2O
(c) PH3 > NH3 > AsH3 > SbH3 20. Which oxide of nitrogen is obtained on heating ammonium
(d) AsH3 > NH3 > PH3 > SbH3 nitrate at 250ºC ?
8. In nitrogen family, the H-M-H bond angle in the hydrides (a) Nitric oxide (b) Nitrous oxide
gradually becomes closer to 90º on going from N to Sb. This (c) Nitrogen dioxide (d) Dinitrogen tetraoxide
shows that gradually
(a) the basic strength of the hydrides increases 21. Which of the following can be used as an anaesthesia ?
(b) almost pure p-orbitals are used for M-H bonding (a) N2O (b) NO
(c) the bond energies of M-H bonds increases (c) NCl3 (d) NO2
(d) the bond pairs of electrons become nearer to the central
atom 22. The brown ring test for NO -2 and NO 3- is due to the
9. The boiling points of the following hydrides follow the order formation of complex ion with a formula
(a) SbH3 > NH3 > AsH3 > PH3 (a) [Fe(H2O)6]2+ (b) [Fe(NO)(CN)5]2+
(b) NH3 > PH3 > AsH3 > SbH3 (c) [Fe(H2O)5NO] 2+ (d) [Fe(H2O)(NO)5]2+
(c) NH3 > AsH3 > PH3 > SbH3 23. A deep brown gas is formed by mixing two colourless gases
(d) SbH3 > AsH3 > NH3 > PH3 which are
10. Which of the following has highest dipole moment? (a) NO2 and O2 (b) N2O and NO
(a) NH3 (b) PH3 (c) NO and O2 (d) NH3 and HCl
(c) AsH3 (d) SbH3 24. Which one of the following oxides of nitrogen is blue solid ?
11. The correct sequence of decrease in the bond angle of the (a) NO (b) N2O3
following hydrides is
(c) N2O (d) N2O5
(a) NH3 > PH3 > AsH3 > SbH3
25. Of the following compounds, the most acidic is
(b) NH3 > AsH3 > PH3 > SbH3
(a) As 2O30 (b) P2O5
(c) SbH3 > AsH3 > PH3 > NH3
(d) PH3 > NH3 > AsH3 > SbH3 (c) Sb2O3 (d) Bi2O3
12. Among the trihalides of nitrogen which one is most basic ? 26. Which of the following is a cyclic phosphate ?
(a) NF3 (b) NCl3 (a) H3P3O10 (b) H6P4O13
(c) NI3 (d) NBr3 (c) H5P5O15 (d) H7P5O16
The p-Block Elements - Nitrogen Family 677
27. The structural formula of hypophosphorous acid is 39. Ammonia can be dried by
O O (a) conc. H2SO4 (b) P4O10
(c) CaO (d) anhydrous CaCl2
P P 40. The shape of ammonia molecule is
(a) H OH (b) H OH (a) tetrahedral (b) pyramidal
H OH
(c) planar triangle (d) octahedral
O O 41. P4O10 is not used to dry NH3 gas because
P (a) P4O10 reacts with moisture in NH3
P
(c) HO OH (d) OH OOH (b) P4O10 is not a drying agent
OH OH (c) P4O10 is acidic and NH3 is basic
28. P—O—P bond is present in (d) P4O10 is basic and NH3 is acidic
(a) H4P2O6 (b) H4P2O5 42. When ammonia is heated with cupric oxide, a molecule of
(c) Both (a) and (b) (d) Neither (a) nor (b) ammonia will
29. Orthophosphoric acid is (a) gain 3 electrons (b) lose 3 electrons
(a) monobasic (b) dibasic (c) gain 2 electrons (d) lose 2 electrons
(c) tribasic (d) tetrabasic 43. Brown colour in HNO3 can be removed by
30. The oxyacid of phosphorous in which phosphorous has the (a) adding Mg powder
lowest oxidation state is (b) boiling the acid
(a) hypophosphorous acid (c) passing NH3 through acid
(b) orthophosphoric acid (d) passing air through warm acid
(c) pyrophosphoric acid 44. HNO2 acts as an/a
(d) metaphosphoric acid (a) acid (b) oxidising agent
31. The number of P—O—P bonds in cyclic metaphosphoric (c) reducing agent (d) All the above
acid is 45. Which of the following phosphorus is most reactive ?
(a) zero (b) two (a) Red phosphorus (b) White phosphorus
(c) three (d) four (c) Scarlet phosphorus (d) Violet phosphorus
32. The triple bond between N atoms of nitrogen molecule 46. White phosphorus is
( N º N ) consists of (a) a monoatomic gas
(b) P4, a tetrahedral solid
(a) three s-bonds
(c) P8, a crown
(b) two s-bonds and one p-bond
(d) a linear diatomic molecule
(c) one s-bond and two p-bonds
47. One mole of calcium phosphide on reaction with excess of
(d) three p-bonds
water gives
33. The molecule having bond order 3 is
(a) one mole of phosphine
(a) H2 (b) N2
(b) two moles of phosphoric acid
(c) O2 (d) He +2 (c) two moles of phosphine
34. An element (X) forms compounds of the formula XCl3, X2O5 (d) one mole of phosphorus oxide
and Ca3X2 but does not form XCl5. Which of the following is 48. PH3, the hydride of phosphorus is
the element X ? (a) metallic (b) ionic
(a) B (b) Al (c) non-metallic (d) covalent
(c) N (d) P 49. The acid which forms two series of salts is
35. Which of the following is obtained when N2 reacts with (a) H3PO4 (b) H3PO3
calcium carbide ? (c) H3BO3 (d) H 3 PO -2
(a) Calcium cyanate (b) Calcium acetate 50. The equivalent weight of phosphoric acid (H3PO4) in the
(c) Calcium cyanamide (d) Calcium carbonate reaction
36. On heating ammonium dichromate, the gas evolved is
(a) oxygen (b) ammonia NaOH + H 3PO 4 ® NaH 2 PO 4 + H 2O is
(c) nitrous oxide (d) nitrogen (a) 25 (b) 49
37. What causes nitrogen to be chemically inert ? (c) 59 (d) 98
(a) Multiple bond formation in the molecule 51. In the following reaction
(b) Absence of bond polarity H O
PCl 5 ¾¾2¾® HCl + A
(c) Short internuclear distance
(d) High bond energy the product ‘A’ is
38. In which the NH3 is not used ? (a) H2P2O4 (b) H2P2O7
(a) Cold storage (c) H3PO4 (d) H3PO3
(b) Anaesthetic 52. Which of the following undergoes sublimation
(c) Manufacture of rayon and plastic (a) ZnCl2 (b) CuCl2
(d) None of these (c) AgCl (d) NH4Cl
678 Chemistry
53. BCl3 is a planar molecule whereas NCl3 is pyramidal because 63. The nitride ion in lithium nitride is composed of
(a) BCl3 has no lone pair of electrons but NCl3 has a lone (a) 7 Protons + 10 electrons (b) 10 Protons + 10 electrons
pair of electrons (c) 7 Protons + 7 electrons (d) 10 Protons + 7 electrons
(b) B—Cl bond is more polar than N—Cl bond 64. Which of the following leaves no residue on heating ?
(c) nitrogen atom is smaller than boron atom (a) Pb(NO3)2 (b) NH4NO3
(d) N—Cl bond is more covalent than B—Cl bond (c) Cu(NO3)2 (d) NaNO3
54. The electronic configuration of an elements is 1s2 2s2 2p6 65. Which of the following molecules is linear ?
3s2 3p6 3d10 4s2 4p3. Its properties would be similar to which (a) SO2 (b) NO +2
(c) NO 2 - (d) SCl2
of the following elements ?
(a) Boron (b) Oxygen 66. A metal X on heating in nitrogen gas gives Y. Y on treatment
(c) Nitrogen (d) Chlorine with H2O gives a colourless gas which when passed through
55. Which of the following has least covalent P—H bond ? CuSO4 solution gives a blue colour. Y is
(a) Mg(NO3)2 (b) Mg3N2
(a) PH3 (b) P2 H 62+ (c) NH3 (d) MgO
67. Which one of the following molecules will have bonds of
(c) P2 H 5+ (d) PH +4 unequal lengths ?
56. Among the followng species, identify the isostructural pairs (a) NF3 (b) BF3
(c) PF5 (d) SF6
NF3 , NO 3- , BF3 , H 3 O + , HN 3 68. In which of the following the bond angle is maximum ?
(a) [ NF3 , NO 3- ] and [BF3 , H 3 O + ] (a) NH3 (b) NH +4
(c) PCl3 (d) SCl2
(b) [ NF3 , HN 3} and [ NO3- , BF3 ]
69. One of the oxidants used with liquid propellants is
+
(c) [ NF3 , H 3O ] and [ NO3- , BF3 ] (a) ammonium perchlorate (b) nitrocellulose
(c) sulphuric acid (d) nItrogen tetroxide (N2O4)
(d) [ NF3 , H 3O + ] and [HN 3 , BF3 ] 70. The aqueous solution/liquid that absorbs nitric oxide to a
considerable extent is
57. In PO 34- ion, the formal charge on each oxygen atom and (a) lead nitrate (b) nitric acid
P—O bond order respectively are (c) ferrous sulphate (d) sodium hydroxide
(a) –0.75, 1.25 (b) –3, 1.25 71. Which one of the following arrangements of molecules is
(c) –0.75, 1.0 (d) –0.75, 0.6 correct on the basis of their dipole moments ?
58. In nitroprusside ion, the iron and NO exist as Fe++ and NO+ (a) BF3 > NF3 > NH3 (b) NF3 > BF3 > NH3
(c) NH3 > BF3 > NF3 (d) NH3 > NF3 > BF3
rather than Fe+++ and NO. These forms can be differentiated
72. Sodium hexametaphosphate is known as
by
(a) calgon (b) permutit
(a) estimating the concentration of iron
(c) natalite (d) nitrolim
(b) measuring the concentration of CN - 73. The oxidation number of As in H 2 AsO -4 is
(c) measuring the solid state magnetic moment (a) +6 (b) +5
(d) thermally decomposing the compound (c) –7 (d) +9
59. Nitrogen atom has an atomic number of 7 and oxygen has an 74. Which of the following compounds is explosive in nature ?
atomic number 8. The total number of electrons in a nitrate (a) Phosphorus trichloride (b) Nitrogen trichloride
ion will be (c) Hyponitrous acid (d) Nitrosyl chloride
(a) 8 (b) 16 75. Among the 15th group elements, as we move from nitrogen
(c) 32 (d) 64 to bismuth, the pentavalency becomes less pronounced and
60. If HNO3 changes into N2O, the oxidation number is changed trivalency becomes more pronounced due to
by (a) Non metallic character (b) Inert pair effect
(a) 2 (b) 6 (c) High electronegativity (d) Large ionization energy
(c) 0 (d) +4
61. The hybridized state of N in R2NH is 76. In NO 3- ion, the number of bond pair and lone pair of
(a) sp3 (b) sp2 electrons on nitrogen atom are
(c) sp (d) dsp 2 (a) 2,2 (b) 3, 1
(c) 1, 3 (d) 4, 0
62. The hybridization of atomic orbitals of nitrogen in NO +2 , 77. Pentavalence in phosphorus is more stable when compared
to that of nitrogen even though they belong to same group.
NO 3- and NH +4 are
This is due to
(a) sp, sp3 and sp2 respectively (a) dissimilar electronic configuration
(b) sp, sp2 and sp3 respectively (b) due to presence of vacant d-orbitals
(c) sp2, sp and sp3 respectively (c) reactivity of phosphorus
(d) sp2, sp3 and sp respectively (d) inert nature of nitrogen
The p-Block Elements - Nitrogen Family 679

1. Which of the following is a nitric acid anhydride? 11. H3PO2 is the molecular formula of an acid of phosphorus. Its
(a) NO (b) NO2 [CBSE PMT 1988] name and basicity respectively are [CBSE PMT 1992]
(c) N2O5 (d) N2O3. (a) Phosphorus acid and two
2. Which of the following metal evolves hydrogen on reacting (b) Hypophosphorous acid and two
with cold dilute HNO3 ? [CBSE PMT 1989] (c) Hypophosphorous acid and one
(a) Mg (b) Al (d) Hypophosphoric acid and two
(c) Fe (d) Cu. 12. Which of the following fertilizers has the highest nitrogen
3. Each of the following is true about white and red phosphorus percentage ? [CBSE PMT 1993]
except that they [CBSE PMT 1989] (a) Ammonium sulphate
(a) Are both soluble in CS2 (b) Calcium cyanamide
(b) Can be oxidised by heating in air (c) Urea
(c) Consist of the same kind of atoms (d) Ammonium nitrate
(d) Can be converted into one another 13. Which of the following oxides will be the least acidic?
4. When orthophosphoric acid is heated to 600°C, the product (a) As4O 6 (b) As 4O10 [CBSE PMT 1996]
formed is [CBSE PMT 1989]
(c) P4O10 (d) P4O6
(a) PH3 (b) P2O5
14. Which of the following oxy-acids has the maximum number
(c) H3PO3 (d) HPO3
of hydrogens directly attached to phosphorus?
5. Which of the following statements is not correct for nitrogen?
(a) H4P2O7 (b) H3PO2 [CBSE PMT 1999]
[CBSE PMT 1990]
(c) H3PO3 (d) H3PO4
(a) Its electronegativity is very high
15. Nitrogen forms N2, but phosphorus is converted into P4 from
(b) d-orbitals are available for bonding
P, the reason is [CBSE PMT 2001]
(c) It is a typical non-metal
(a) Triple bond is present between phosphorus atom
(d) Its molecular size is small
(b) pp – pp bonding is strong
6. P2O5 is heated with water to give [CBSE PMT 1991]
(c) pp – pp bonding is weak
(a) Hypophosphorous acid
(d) Multiple bond is formed easily
(b) Phosphorous acid
(c) Hypophosphoric acid 16. Oxidation states of P in H4 P2O5 , H4 P2O6 , and H4 P2O7 ,
(d) Orthophosphoric acid are respectively: [CBSE-PMT 2010]
7. PCl3 reacts with water to form [CBSE PMT 1991] (a) + 3, + 5, + 4 (b) + 5, + 3, + 4
(a) PH3 (b) H3PO3, HCl (c) + 5, + 4, + 3 (d) + 3, + 4, + 5
(c) POCl3 (d) H3PO4 17. In which of the following compounds, nitrogen exhibits
8. PH4I + NaOH forms [CBSE PMT 1991] highest oxidation state ? [CBSE-PMT 2012 S]
(a) PH3 (b) NH3 (a) N2H4 (b) NH3
(c) P4O6 (d) P4O10 (c) N3H (d) NH2OH
9. Pure nitrogen is prepared in the laboratory by heating a 18. Which of the following statements is not valid for oxoacids
mixture of [CBSE PMT 1991] of phosphorus? [CBSE-PMT 2012 S]
(a) NH4OH + NaCl (b) NH4 NO3 + NaCl (a) Orthophosphoric acid is used in the manufacture of triple
(c) NH4 Cl + NaOH (d) NH4 Cl + NaNO2. superphosphate.
10. Sugarcane on reaction with nitric acid gives (b) Hypophosphorous acid is a diprotic acid.
(a) CO2 and SO2 [CBSE PMT 1992] (c) All oxoacids contain tetrahedral four coordinated
(b) (COOH)2 phosphorus.
(c) 2 HCOOH (two moles) (d) All oxoacids contain atleast one P = O and one P — OH
(d) No reaction. group.
680 Chemistry
19. In case of nitrogen, NCl3 is possible but not NCl5 while in 24. The decreasing values of bond angles from NH3 (106º) to
case of phosphorous, PCl3 as well as PCl5 are possible. It is SbH3 (101º) down group-15 of the periodic table is due to
due to [AIEEE 2002] (a) decreasing lp-bp repulsion [AIEEE 2006]
(a) availability of vacant d orbitals in P but not in N (b) decreasing electronegativity
(b) lower electronegativity of P than N (c) increasing bp-bp repulsion
(c) lower tendency of H-bond formation in P than N (d) increasing p-orbital character in sp3
(d) occurrence of P in solid while N in gaseous state at room 25. Regular use of which of the the following fertilizers increases
temperature.
the acidity of soil? [AIEEE 2007]
20. Number of sigma bonds in P4O10 is [AIEEE 2002]
(a) Ammonium sulphate
(a) 6 (b) 7
(b) Potassium nitrate
(c) 17 (d) 16.
(c) Urea
21. What may be expected to happen when phosphine gas is
(d) Superphosphate of lime.
mixed with chlorine gas ? [AIEEE 2003]
(a) PCl3 and HCl are formed and the mixture warms up 26. The reaction of P4 with X leads selectively to P4O6. The X is
(b) PCl5 and HCl are formed and the mixture cools down (a) Dry O2 [IIT-JEE 2009]
(c) PH3 . Cl2 is formed with warming up (b) A mixture of O2 and N2
(d) The mixture only cools down (c) Moist O2
22. Which one of the following substances has the highest (d) O2 in the presence of aqueous NaOH
proton affinity? [AIEEE 2003] 27. Extra pure N2 can be obtained by heating [IIT-JEE 2011]
(a) H2S (b) NH3 (a) NH3 with CuO (b) NH4NO3
(c) PH3 (d) H2O (c) (NH4)2Cr2O7 (d) Ba(N3)2
23. The number of hydrogen atom(s) attached to phosphorus 28. Concentrated nitric acid, upon long standing, turns yellow
atom in hypophosphorous acid is [AIEEE 2005] brown due to the formation of [JEE Advanced 2013]
(a) three (b) one (a) NO (b) NO2
(c) two (d) zero (c) N2O (d) N2O4

1. Phosphine on reaction with hydrobromic acid gives 5. The nitrate which when heated gives off a gas or a mixture of
(a) PBr3 (b) PH4Br gases which cannot relight a glowing splinter is
(c) PBr5 (d) P2H4 (a) sodium nitrate (b) ammonium nitrate
2. An inorganic compound producing organic compound on (c) lead nitrate (d) potassium nitrate
heating is 6. Liquid ammonia bottles are opened after cooling them in ice
(a) sodamide for sometime. It is because liquid NH3
(b) ammonium cyanate (a) Brings tears to the eyes
(c) sodalime (b) Has a high vapour pressure
(d) potassium cyanide (c) Is a corrosive liquid
3. Which is not correct for N2O ? (d) Is a mild explosive
(a) It is laughing gas and is used as anaesthetic agent. 7. Ammonia is generally manufactured for fertilizers by the
(b) It is nitrous oxide reaction
(c) It is not a linear molecule. (a) 2 NH 4 Cl + Ca (OH ) 2 ® CaCl 2 + 2H 2 O + 2 NH 3
(d) It is least reactive of all the oxides of nitrogen
(b) By passing an electric discharge in a mixture of N 2 and
4. An important method of fixation of atmospheric N2 is
H2
(a) Fisher- Tropsch’s process
(c) By passing a mixture of N2 and H2 under high pressure
(b) Haber’s process
and moderate temperature over a catalyst
(c) Frasch’s process
(d) None of these
(d) Solvay’s process
The p-Block Elements - Nitrogen Family 681
8. Nitrogen can be purified from the impurities of oxides of 19. Ammonia, on reaction with hypochlorite anion, can form
nitrogen and ammonia by passing through (a) NO (b) NH4Cl
(a) conc. HCl (c) N2H4 (d) HNO2
(b) alkaline solution of pyrogallol 20. The catalyst used in the manufacture of HNO 3 by Ostwald’ss
(c) a solution of K2Cr2O7 acidified with H2SO4
process is :
(d) a solution of KOH
(a) platinum gauze (b) vanadium pentoxide
9. Ammonium dichromate is used in some fire works. The green
(c) finely divided nickel (d) platinum black .
coloured powder blown is
21. The deep blue colour produced on adding excess of ammonia
(a) CrO3 (b) Cr2O3
to copper sulphate is due to presence of
(c) Cr (d) CrO(O2)
2+
10. Phosphine is not obtained by which of the following reaction (a) Cu 2 + (b) Cu ( NH 3 ) 4
(a) White P is heated with NaOH
(c) Cu ( NH 3 ) 26 + (d) Cu( NH3 ) 22 +
(b) Red P is heated with NaOH
(c) Ca3P2 reacts with water 22. Ammonia on catalytic oxidation gives an oxide from which
(d) Phosphorus trioxide is boiled with water nitric acid is obtained. The oxide is :
11. Phosphine is not evolved when (a) N 2 O3 (b) NO
(a) white phosphorus is boiled with a strong solution of
(c) NO 2 (d) N 2 O5
Ba(OH)2
(b) phosphorus acid is heated 23. The dipole moment of NF3 is less than NH 3 because
(c) calcium hypophosphite is heated
(a) NH 3 forms associated molecules.
(d) metaphosphoric acid is heated.
(b) F is more reactive than H.
12. Pure phosphine is not combustible while impure phosphine
is combustible, this combustibility is due to presence of (c) The resultant of bond polarity is less.
(d) The resultant of individual polarities is opposed by the
(a) P2 H 4 (b) N2 polarity of lone pair.
(c) PH 5 (d) P2 O5 24. Which property of white phosphorus is common to
red P
13. A white precipitate is obtained on hydrolysis of
(a) It burns when heated in air.
(a) PCl5 (b) NCl3 (b) It reacts with hot caustic soda solution to give
(c) BiCl 3 (d) AsCl3 phosphine.
(c) It shows chemiluminescence.
14. Calcium cyanamide on treatment with steam under pressure
gives NH3 and (d) It is soluble in carbon disulphide.
25. Fixation of nitrogen means :
(a) CaCO3 (b) Ca (OH) 2
(a) reaction of nitrogen with oxygen.
(c) CaO (d) CaHCO3 (b) conversion of free atmospheric nitrogen into nitrogen
15. Which of the following statements is not true ? compounds.
(a) NO2 can be prepared by heating Pb(NO3)2 (c) the action of denitrifying bacteria on nitrogen
compounds.
(b) NO2 is red - brown gas
(d) decomposition of nitrogenous compounds to yield free
(c) NO2 is diamagnetic
nitrogen.
(d) NO2 readily dimerises to N2O4
16. Hydrolysis of PI3 yields 26. Conc. HNO 3 is heated with P2 O5 to form :
(a) monobasic acid and a salt (a) NO 2 (b) NO
(b) monobasic acid and dibasic acid
(c) dibasic acid and tribasic acid (c) N 2 O5 (d) N 2 O
(d) monobasic acid and tribasic acid 27. Superphosphate of lime is obtained from the reaction of :
17. If 20% nitrogen is present in a compound its minimum (a) Bones with gypsum
molecular weight can be (b) Calcium phosphate with sulphuric acid
(a) 144 (b) 70 (c) Calcium phosphate with HCl
(c) 100 (d) 140 (d) Calcium carbonate with phosphoric acid
18. Blasting of TNT is done by mixing it with 28. Orthophosphoric acid on heating gives :
(a) NH4Cl (b) NH4NO3 (a) metaphosphoric acid (b) phosphine
(c) NH4NO2 (d) (NH4)2SO4 (c) phosphorus pentoxide (d) phosphorus acid
682 Chemistry

EXERCISE 1 14. (c) Due to inert pair effect the lower O.S. is more common at
the bottom of group
1. This is because PH3 is much weaker base than NH3.
NH3 dissolves in water forming NH4OH. 1
15. (c) Hybridisation in PCl5 = (5 + 5 + 0 - 0) = 5 sp3d
NH + H O ‡ˆˆˆˆ† NH + + OH– 2
3 2 4
2. In H3PO3, there is pp – dp bond whereas in HNO3 there is 16. (a) N2O is soluble in cold and not in hot water
pp – pp bond. pp – pp bond in HNO3 is stronger than pp – 17. (b) FeSO 4 + NO ® FeSO 4 .NO
dp bond in H3PO3.
18. (a) In N2O5 nitrogen is present in the highest O.S. and it is
3. NO2 has unpaired electron and due to this, it has brown most electronegative in nature than others
colour. N2O4 does not have unpaired electron and thus it is
19. (a) See preparation of N2O5
colourless.
20. (b) N2O (see text)
4. N atom does not have d-orbitals and thus it cannot have all
21. (a) N2O is used as anaesthetic
five valence electrons as unpaired.
22. (c) [Fe(H2O)5NO]2+ ion is formed
5. F is highly electronegative and N – F bond energy is higher
than N – Cl bond energy. 23. (c) 2NO + O 2 ® 2 NO 2 brown
6. N2O is laughing gas prepared by heating NH4NO 3 . 24. (b) N2O3 is blue solid
D 25. (b) P2O5 ;P is electronegative and present in highest O.S. of
NH 4 NO3 ¾¾® N 2 O + 2H 2 O
5, hence P2O5 is most acidic
7. PCl5 is ionic in the solid state because it exists as [PCl4]+
26. (c) H5P5O15 (HPO3)5. It is metaphosphoric acid which is a
[PCl6]–.
cyclic phosphate.
16. (d) 17. (b) 18. (b) 19. (a) 20. (a)
27. (b) See structures of oxy acids of phosphorous
21. (a) 22. (b) 23. (a) 24. (a) 25. (d)
28. (b) H4P2O5 is pyrophosphorous acid it contains P–O–P
26. (c) 27. (c) bond
EXERCISE 2 29. (c) H3PO4 is tribasic
30. (a) Hypophosphorous acid is H3PO2 in which O.S. of P is
1. (d) Metallic character increases down the group, Bi is most +1
metallic
31. (c) See structure of metaphosphoric acid.
2. (c) Basic nature
32. (c) One s and two p
3. (a) Nitrogen; NH3 is most basic .. ..
4. (d) Ionic radii increases down the group 33. (b) It is N º N
5. (a) –3, +3, +5 (see text) 34. (c) Nitrogen can form NCl3, N2O5 Ca3N2 and not NCl5 since
6. (c) it has no d atomic orbitals in valence shell
7. (a) 35. (c) CaC2 + N 2 ® (CaCN 2 + C)
8. (b) With the decrease in the electronegativity of central atom Calcium cyanamide
the bond angle decreases
D
9. (a) 36. (d) (NH 4 ) 2Cr2 O 7 ¾¾
® Cr2 O3 + 4H 2 O + N 2
10. (a) Dipole moment decreases with decrease in 37. (d) The cause of inert nature of N2 is the presence of triple
electronegativity of central metal atom .. ..
11. (a) bond N º N
12. (c) The lesser the electronegativity of halogen in NX3 the 38. (b) NH3 is not used as anaesthetic
more is the basic character. N can donate more electrons 39. (c) Ammonia is dried over CaO. Others will react with it
in that case. 40. (b) Ammonia has pyramidal shape with sp3 hybridisation.
13. (a) Nitrogen does not contain vacant d orbitals in valence NH
41. (c) P4 O10 + 2 H 2 O ¾
¾® 4HPO 3 ¾¾¾
3 ® NH PO
4 3
shell hence cannot extend its coordination number more basic
Acidic meta phosphoric meta ammonium
than 3 acid phosphate
The p-Block Elements - Nitrogen Family 683

42. (b) 3CuO + 2 NH 3 ® 3Cu + 3H 2 O + N 2 , -3


57. (a) Formal charge on each oxygen = = -0.75 and
4
O.S. of N in NH3 is –3 and in N2 is zero. Hence loss of 3
electrons 5
P–O bond order is = = 1.25
43. (d) Nitric acid decomposes to give NO2 which is brown 4
(Here 5 is the number of P–O bonds and 4 is the number
4HNO 3 ® 4 NO 2 + 2H 2 O + O 2
of resonating forms)
When air is passed through acid the reaction proceeds 58. (c) The nitroprusside ion is [Fe(CN)6NO+]2–. The magnetic
towards left hand side and brown colour diminishes. moment measurements reveal the presence of 4 unpaired
44. (d) In HNO2 the O.S. of N is +3 which can be increased or electrons in Fe which must be then in Fe++ (3d6) and not
decreased by gain or loss of electrons (O.S. of N varies Fe+++(3d5)
from –3 to +5)
59. (c) NO 3- (7+3×8+1) = 32 electrons
45. (b) White phosphorous is most reactive
46. (b) White phosphorous is P4 and tetrahedral 60. (d) In HNO3 the O.S. of N is +5 and in N2O it is +1. Hence
change in O.S. is +4
47. (c) Ca 3 P2 + 6 H 2 O ® 3Ca (OH) 2 + 2PH 3
1 mole 2 moles 1
61. (a) R2NH hybridisation = (5 + 3 + 0 - 0) = 4 sp3
2
48. (d) PH3 is covalent hydride
1
OH 62. (b) Hybridisation in NO 2+ = (5 + 0 + 0 - 1) = 2 (sp)
| 2
49. (b) HO - P - H it can form two series of salts by
|| 1 1
O NO 3- = (5 + 0 + 1 - 0) = 3 ( sp 2 ) ; NH 4+ = (5 + 4 + 0 - 1) = 4(sp 3 )
2 2
replacement of H attached to oxygen 63. (a) N3– ion has 7 protons and 10 electrons
50. (d) NaOH + H 3PO 4 ® NaH 2 PO 4 + H 2O . Since there is D
64. (b) NH 4 NO 3 ¾¾® N 2 O ­ +2 H 2 O ­
no change in O.N. of P hence its Mol. Wt. which is 98
1
will be equivalent weight also 65. (b) In NO +2 hybridisation is, (5 + 0 + 0 - 1) = 2, sp hence
2
51. (c) 2PCl5 + 8H 2 O ® 2H 3 PO 4 + 10HCl , the product linear
formed is phosphoric acid D 6H O
52. (d) NH4Cl sublimes 66. (b ) 3Mg + N 2 ¾¾® Mg 3 N 2 ¾¾2¾® 3Mg (OH ) 2 + 2 NH 3 ;
53. (a) In BCl 3 there is sp2 hybridisation (planar) in NCl 3 CuSO 4 + 4 NH 3 ® [Cu ( NH 3 ) 4 ]SO 4
hybridisation is sp3 hence shape is pyramidal deep blue colour

54. (c) The valence shell electronic configuration is 4s24p3 67. (c) PF5 has trigonal pyramidal shape in which axial and
similar to nitrogen 2s2p3 equitorial bonds have different lengths
55. (b) For this calculate the formal charge distributed on each 68. (b) Bond angle in NH +4 is 109º28' due to sp3 hybridisation
P–H bond which is given by = Total charge . and symmetrical structure
No. of P - H bonds 69. (d) N2O4 is used as an oxidant with liquid propellants
The more the formal charge the lesser is the covalent 70. (c) FeSO 4 + NO ® FeSO 4 .NO
character. 71. (d) NH3 > NF3 > BF3. NH3 and NF3 have pyramidal shape.
In BF3 is planar.

+2 +1 +1 d–
P2 H 26 + = = +0.33 ; P2 H 5+ = = +0.2 ; PH 4+ = = +0.25 .. F
6 5 4 N N d+
B F
Hence lowest covalent character of P–H bond is d+H H H d– F
d– F F
F m=0
d–
in P2 H 62+
72. (a) It is calgon (Sodium hexametaphosphate)
56. (c) In NF3 and H3O+ the hybridisation is sp3 and both are
73. (b) H 2 AsO -4 2(1) + x +4(–2)= –1 , \ x = +5
pyramidal. In NO 3- and BF3 the hybridisation is sp2 and
74. (b) NCl3 is explosive in nature.
both are planar 75. (b) Inert pair effect (follow text)
684 Chemistry
.. 12. (c) Urea (46.6%N). % of N in other compound are :
– O
..
( NH 4 ) 2 SO 4 = 21.2%;
..
76. (d) O
.. N .. , Bond pairs = 4, lone pair = 0
O..
CaCN 2 = 35.0% and NH 4 NO 3 = 35.0%
..

77. (b) Phosphorous can achieve coordination number 5 due to 13. (a) As the O.N of the central atom of the compounds
vacant d atomic orbitals in valence shell which is not increases acidic strength of that compound also
possible in nitrogen increases and on moving from top to bottom in groups
acidic strength of oxides also decrease due to decreasing
EXERCISE 3 electronegativity in groups.
1. (c) N 2 O 5 + H 2 O ¾¾® 2 HNO 3 +5 +3 +5 +3
2. (a) Magnesium and manganese are the metals that produce P4O10 > P4O6 > As 4O10 > As 4O6
hydrogen with dilute nitric acid 14. (b)
Mg + 2HNO3 ¾ ¾® Mg (NO3)2 + H2 OH OH
3. (a) Both white and red phosphorus are not soluble in CS2
(a) H4P2O7 ÞO P O P O
only white P is soluble.
OH OH
600°C
4. (d) 2H 3 PO 4 ¾¾¾¾ ® 2HPO 3 Pyrophosphoric acid
ortho phosphoric -2H 2 O meta phosphoric
acid acid O
5. (b) In case of nitrogen, d-orbitals are not available. P
(b) H3PO2 Þ OH H
6. (d) P2O5 have great affinity for water. The final product is H
orthophosphoric acid. Hypophosphorous acid

2H2O O
P4O10 4HPO3
Metaphosphoric (c) H3PO3 Þ HO – P – OH
acid H
2H2O
Phosphorous acid
2H 2O
4H3PO4 2H4P2O7 O
orthophosphoric Pyrophosphoric
acid acid (d) H3PO4 Þ HO – P – OH
OH
7. (b) PCl3 + 3H 2 O ® H 3PO 3 + 3HCl orthophosphoric acid

8. (a) PH 4 I + NaOH ® NaI + PH 3 + H 2 O 15. (c) Nitrogen form N2 (i.e. N º N) but phosphorus form P4,
because in P2, pp — pp bonding is present which is a
9. (d) Pure nitrogen in the lab can be obtained by heating
weaker bonding.
ammonium nitrate. Ammonium nitrate is not a stable
16. (d)
compound it dissociate to give nitrogen.
17. (c) Compound Oxidation number of nitrogen
Heat
NH 4 Cl + NaNO 2 ¾¾¾® NH 4 NO 2 N2H4 = –2
NH3 = –3
Heat
¾¾¾® N 2 + 2 H 2 O. N3H = –1/3
10. (b) Cane sugar is oxidised to oxalic acid NH2OH = –1

(2HNO 3 ¾¾
® H 2 O + 2NO 2 + O)18
O
C12 H 22 O11 + 18[O] ¾
¾® 6 (COOH) 2
Cane sugar From HNO3 Oxalic acid 18. (b) H P H Hypophosphorous acid (H3PO2) is a

+ 5H 2O. O
H
C12H22O11 + 36HNO3 ¾¾®
6(COOH)2 + 36NO2 + 23H2O monobasic acid. i.e., it has only one ionisable hydrogen
atom or one OH is present.
11. (c) H3 PO2 is named as hypophosphorous acid. As it 19. (a) 7N = 1s2 2s2 2p3; 2 2 6
15P = 1s 2s 2p 3s 3p
2 3

contains only one P – OH group, its basicity is one. In phosphorous the 3 d- orbitals are available.
The p-Block Elements - Nitrogen Family 685
EXERCISE 4
20. (d) O
||
P 1. (b) PH 3 + HBr ® PH 4 Br Phosphorium bromide
O O
D
O P
O
P O 2. (b) NH 4 CNO ¾¾
® NH 2 CONH 2 . Urea is an organic
||

||
O
O P compound
|| O
O 3. (c) N2O has linear structure ( see structure)
4. (b) By Haber’s Process atmospheric nitrogen is converted
21. (a) PH 3 + 3Cl 2 ¾
¾® PCl 3 + 3HCl into NH3.
.. 5. (b) NH 4 NO3 ® N 2O + 2H 2O.
22. (b) Among the given compounds, the NH 3 is most basic.
N2O is non combustible an d non supporter of
Hence has highest proton affinity
combustion. Other nitrates give O2 which supports
H combustion.
|
23. (c) Hypophosphorous acid H - O - P ® O 6. (b) Liquid ammonia has high vapour pressure which is
| lowered down by cooling, otherwise the liquid will bump.
H
7. (c) By Haber’s process
Two H-atoms are attached to P atom. 8. (d) The oxides of nitrogen being acidic in nature are soluble
24 (b) The bond angle decreases on moving down the group in KOH which is an alkali.
due to decrease in bond pair-bond pair repulsion. 9. (b) (NH 4 ) 2 Cr2 O 7 ® N 2 + Cr2O 3 + 4H 2O
NH3 PH3 AsH3 SbH3 BiH3 green
107º 94º 92º 91º 90º
10. (b) Red P does not react with NaOH to give PH3.
This can also be explained by the fact that as the size of
11. (d) PH3 is not obtained when metaphosphoric acid is heated.
central atom increases sp3 hybrid orbital becomes more
12. (a) The combustibility of PH3 is due to presence of P2H4.
distinct with increasing size of central atom i.e. pure
The pure PH3 is not combustible.
p-orbitals are utilized in M–H bonding.
25. (a) (NH4)2SO4 + 2H2O¾¾ ® 2H2SO4 + NH4OH 13. (c) BiCl 3 + H 2O ® BiOCl + 2HCl . BiOCl gives white ppt.
H2SO4 being strong acid increases the acidity of soil. In which is used as white pigment, under the name of pearl
case of potassium nitrate and superphosphate of lime on white.
hydrolysis strong base also form along with strong acid 14. (a) CaCN 2 + 3H 2O ® CaCO3 + 2NH3
on hydrolysis results into neutralisation.
15. (c) NO2 is paramagnetic and readily dimerises to N2O4 which
In presence of N 2
26. (b) P4 + 3O 2 ¾¾¾¾¾¾¾® P4 O 6 is diamagnetic.
16. (b) PI 3 + 3H 2 O ® H 3 PO 3 + 3HI
Here N2 acts as a diluent and thus retards further
Dibasic acid Monobasic acid
oxidation. Reaction of P4 under other three conditions.
2O
17. (b) 20% nitrogen means if 20 g N then M.wt = 100
(a) P4 + 3O 2 ¾¾
® P4 O 6 ¾¾¾
2® P O
4 10

(c) In moist air, P4O6 is hydrolysed to form H3PO3 if 14 g N then M.wt = 100 ´ 14 = 70
20
P4 O 6 + 6H 2 O ¾¾
® 4H 3 PO 3
At least one N atom must be present in the molecule.
(d) In presence of NaOH,
18. (b) TNT when mixed with NH4NO3 , gives blasting mixture.
P4 + 3OH - + 3H 2O ¾¾
® PH 3 + 3H 2 PO 2- 19. (c) 2NH3 + OCl– ® NH2.NH2 + H2O + Cl–
27. (d) Very pure N2 can be obtainted by thermal decomposition Pt . gauge
20. (a) 4NH 3 + 5O 2 ¾¾¾¾® 4NO + 6H 2 O
of sodium or barium azide. Ba(N 3 ) 2 ¾¾
® Ba + 3N 2
28. (b) The slow decomposition of HNO3 is represented by the 21. (b) CuSO 4 + 4NH 3 ® [Cu( NH 3 ) 4 ] SO 4
eqn.
4HNO3 ® 4NO2 + 2H2O + O2 Blue complex due to Cu(NH3 )2 +
4
(yellow-brown)
22. (b) 4NH 3 + 5O 2 ® 4NO + 6H 2O
686 Chemistry
24. (a) Except (a) all other properties are shown by white
23. (d) NH 3 N d- phosphorous.
25. (b) The conversion of atmospheric nitrogen to nitrogen
d+
H H d+ compounds is known as fixation of nitrogen.
Hd+
26. (c) P2O 5 + 2HNO 3 ® N 2O 5 + 2HPO 3

NF3 : 27. (b) Ca 3 (PO4 )2 + 2H 2SO4 + 5H 2 O ®


N d+
Ca (H 2 PO 4 ) 2 . H 2 O + 2CaSO 4 .2H 2 O
d-
F Fd - 28. (b) 2H 3PO 4 ® 2HPO 3 + 2H 2 O
Fd -
meta phosphoric acid
21B
The p-Block Elements-
Oxygen Family
GENERAL CHARACTERISTICS : 6. Oxidation state
The elements oxygen, sulphur, selenium, tellurium and polonium O S Se Te Po
belong to group VIA or 16 group of periodic table. –1,–2 –2 to +6 –2 to +6 –2 to +6 –2 to +6
These elements are known as chalcogens i.e. ore forming elements. In OF2 the oxidation state of oxygen is +2
1. Electronic configuration 7. Ionisation energy - They possess a large amount of ionisation
Elements At.No. Electronic Confg. Valence shell energy which decrease gradually from O to Po due to increase
electronic confg. in size of atoms and increase in screening effect.
8. Electron affinity - They have high electron affinity which
Oxygen 8 [He]2s2 p 4 2s 2 2p 4
decrease from O to Po. As the size of the atom increases the
Sulphur 16 [Ne]3s 2 3p4 3s 2 3p 4 extra added electron feels lesser attraction by nucleus and
electron affinity decreases.
Selenium 34 [Ar]4s2 4p 4 4s 2 4p 4 9. Electronegativity - It decreases down the group due to
decrease in the effective nuclear charge down the group.
Tellurium 52 [Kr]5s 2 5p 4 5s 2 5p 4 10. Catenation - The tendency to form chains of identical atoms
is known as catenation. It follows the order
Polonium 84 [Xe]6s 2 6p 4 6s 2 6p 4
S-S > Se - Se > O- O > Te - Te
The oxygen differs from the rest of the elements due to its (i)
small size (ii) higher electronegativity (iii) absence of d atomic 226 172 142 126 kJ/mole
orbitals in valence shell (iv) tendency to form multiple The higher the bond strength, the higher is the catenation.
bonding. 11. Atomicity - Oxygen is diatomic, sulphur and selenium octa
2. Metallic and non metallic character atomic with puckered ring structure
O S Se Te Po S
144244
3 1442443 S S S S S
Non metals Metalloids S S , S
S S S
S S
Metal (Radio active) t1/2 138.4 days
3. Abundance O > S > Se > Te > Po Ring S6
Oxygen is the most abundant element. It constitutes 46.6% 12. Allotropy - All the elements exhibit allotropy
of earth’s crust, 21% of air and 89.1% of ocean by weight. · Oxygen - O2 dioxygen and O3 ozone
Sulphur forms about 0.052% of earth’s crust. · Sulphur - Rhombic (or a ) sulphur S8
4. Density - Increases down the group regularly.
Monoclinic (or b ) sulphur S8
5. Melting point and boiling point - Both show a regular increase
down the group due to increase in molecular weight and Van (most stable)
der Waal’s forces of attraction. Plastic (or l ) sulphur open chain
Colloidal (or d ) sulphur
688 Chemistry
The SR changes to SM above 95.4ºC. lewis acid character
· Selenium Rhombic Se8 SF4 + F2 ® SF6
Monoclinic Se8 They have trigonal bipyramidal shape with sp 3d
Grey hybridiation.
Grey is the most stable consisting of regularly arranged Cl Cl
spirals of Se atoms.
· Tellurium Non metallic
Te
Metallic (more stable)
· Polonium a and b both metallic
Cl Cl
13. Atomic radii - Increases regularly from O to Po.
(c) Dihalides - The dihalides eg SCl2, OF2, TeBr 2 are sp3,
14. Ionic radii - Increases regularly from O to Po hybridised and have distorted bond angles due to
15. Atomic volume - Increase regularly from O to Po. electron pair repulsions
16. Multiple bond formation - The tendency of these elements SCl2 OF2 TeBr2
to form multiple bonds to C and N decreases down the group 104º 101.5º 98º
eg S = C = S is moderately stable.
Se = C = Se decomposes readily and
Te = C = Te not known
COMPOUNDS OF SIX GROUP ELEMENTS : S
Cl
(i) Hydrides - All these elements form stable hydrides of the
type H2M either by directly combining with hydrogen or by
the action of acids on metal sulphides, Selenides and Cl
tellurides (d) Dimeric monohalides - The dimeric monohalides are
given by sulphur and selenium eg S2F2, S2Cl2, Se2Cl2
2H 2 + O 2 2H 2 O S2Br2, Se2Br2. These are slowly hydrolysed and undergo
FeS + H 2SO 4 ® H 2S + FeSO 4 disproportionation.
2S2Cl 2 + 2H 2O ® 4HCl + SO 2 + 3S
Na 2Se + H 2SO 4 ® H 2Se + Na 2SO 4
H2O is a liquid due to hydrogen bonding. Others are 2Se 2 Cl 2 ® SeCl 4 + 3Se
colourless gases with unpleasant smell. Cl
H2O > H2S > H2Se > H2Te
104.5º 92.5º 91º 90º S
(all sp3 hybridised) 104º
¬ Stability, volatile character (from H2S to H2Te) S
® Poisonous nature, acidic character, raducing character..
The weakening of M – H bond with the increase in the size of Cl
M (not the electronegativity) explains the acid character of The monohalides have structure similar to H2O2 with
hydrides. distorted bond angle of sp3 hybridisation
(ii) Halides - All these elements form a number of halides. The (iii) Oxides - Ozone is considered as oxide of oxygen O. Oxides
halides of oxygen are not very stable eg OF2, Cl2O7, I2O5 etc of other elements are as follows
(a) Hexahalides - These are formed by fluorine only (not Element Mono Oxide Dioxide Tri Oxide
by Cl, Br, I) where elements exhibit maximum valency of S SO SO2 SO3
+6. SF6, SeF6, TeF6 are colourless gases with sp3d2 Se - SeO2 SeO3
hybridisation and octahedral structure. These are covalent Te TeO TeO2 TeO3
in nature. Due to bigger size of Cl, Br and I the coordination
number of 6 is not achieved. Po PoO PoO2 -
F SO2 is a gas having sp2 hybridisation and V-shape.
F F S S S

S O 119.5 O O O O O
2
SO3. is a gas, sp hybridised and planar in nature.
F F
O O O
F
(b) Tetrahalides - With the exception of SBr4, SI4 and SeI4 S S S
all tetrahalides are known. SF4 is gaseous, SeF4 is liquid
and TeF4 is solid. SCl4 is unstable liquid. These have O O O O O O
The p-Block Elements - Oxygen Family 689
In solid state it exist as a cyclic trimer (SO3)3 a-form or as a 2. Sulphuric acid series
linear chain cross linked sheets
O O O
S O O O
O O
S S S (i) Sulphuric acid H2SO4 HO – S – OH
O S S O
O O O O O O
O
O O O
(ii) Thiosulphuric acid H2S2O3
a-form b- and g- form
SeO2 volatile solid consists of non planar infinite chains

:
O
: :

:
98º O
: :
O O
O O O O

: :

: :

: :

: :
Se 125º Se Se Se H – O – S – O– H or H – O – S – S– H
90º
O O
S
: : O
: :
SeO3 has tetrameric cyclic structure

:
:
O O O O
Se
O O O O (iii)Pyrosulphuric acid H2S2O7 HO – S – O – S – OH
Se Se
O O O O O O
3. Thionic acid series
Se

:
:
O O
: :: :
O O
Oxy acids - Sulphur forms four series of oxy acids

: :

: :
(i) Dithionic acid H2S2O6 HO – S – S – OH
Name Formula Structure
1. Sulphurous acid series O O
: :: :
(ii) Polythionic acid H2SnO6

:
:
(i) Sulpurous acid H2SO3
O O

S HO – S – S n – S – OH
: :
: :

HO OH
O O
:O : (n=3, 4, 5, 6)
(ii) Thiosulphurous acid 4. Peroxy acid series
(i) Peroxomonosulphuric acid
S H2SO5
H2S2O2 HO OH
S O

O HO – S – O – OH

O
(iii)Hyposulphurous acid H2S2O4 HO – S – S – OH
(Caro’s Acid)
O (ii) Peroxodisulphuric acid H2S2O8
O
O O
(iv) Pyrosulphurous acid H2S2O5 HO – S – S – OH HO – S – O – O – S – OH

O O O O
(Marshall’s acid)
Oxy acids of selenium - Selenous acid H2SeO3, Selenic acid
H2SeO4
690 Chemistry
Oxy acids of Tellurium Tellurous acid H2TeO3; Telluric acid (4) Manufacture of KMnO4 and artificial silk.
H2TeO4 Tests:-
OZONE (O3) : (1) Turns starch iodine paper blue.
Discovered by Van Marum by passing electric discharge through (2) Tailing of Hg. Mercury loses its meniscus in contact with O3
air, named by Schonbien (azo - I smell) and Sorret established the and sticks to the surface of glass due to formation of Hg2O
formula O3 as allotrope of oxygen (3) Clean silver foil blackened by O3
It is formed in atmosphere by action of UV rays on O2. It is also (4) It turns benzidene paper brown and tetramethyl base violet
+
formed by (i) slow oxidation of phosphorous in air (ii) Reaction of +
O
O
fluorine with water at low temperature (iii) Electrolysis of water Structure – –
(iv) SO2 reacts with H2O2 O O O O
Preparation lab method : By passing silent electric discharge Oxidation state of O is +1 and –1
through cold, dry oxygen in ozoniser Oxides: Oxides are the binary compounds of oxygen with metals
3O2 ƒ 2O3 + 284.5 kJ and non metals. Based on their oxygen content they have been
classified as
The ozonisers used are
(1) Normal oxides - Oxides containing oxygen according to
1. Siemen’s ozoniser 2. Brodie’s ozoniser normal oxidation number of M eg. H2O, MgO, Al2O3.
Manufacture. For the manufacture of ozonised air Siemen and (2) Polyoxides - These contain more oxygen than normal
Halske’s ozoniser is employed. oxidation number of M and M – O and O – O bonds. They
Electrolytic method. When acidified water, using high current are further classified as
density and Pt. anode, is electrolysed 95% O3 is obtained at anode
rest being O2. (i) Peroxides - They contain O22- ion, produce hydrogen
Properties - Pale blue gas, dark blue liquid and violet black solid peroxide with dil. acids and O2 with concentrated acids
with characteristic strong smell, slightly soluble in water but more eg BaO2, Na2O2
soluble in turpentine oil, glacial acetic acid and carbon tetrachloride. ( dil.)
Decomposition BaO 2 + H 2SO 4 ¾¾¾® BaSO 4 + H 2 O 2
573K (Conc )
2O3 ¾¾¾ ® 3O 2 DH = 284 kJ / mole 2 BaO 2 + 2H 2SO 4 ¾¾ ¾
¾® 2BaSO 4 + 2H 2 O + O 2
Oxidising action (ii) Super oxides - They contain O 2– ion. With water they
O 3 ® O 2 + O (nascent oxygen hence powerfully oxidising) give hydrogen peroxide and oxygen
PbS + 4O ® PbSO 4 2KO 2 + 2H 2 O ® 2KOH + H 2O 2 + O 2
H 2S + O ® H 2 O + S (iii)Dioxides - They give chlorine with conc HCl and oxygen
with Conc H2SO4 eg MnO2, PbO2 etc.
2FeSO 4 + H 2SO 4 + O ® Fe 2 (SO 4 ) 3 + H 2O
2MnO 2 + 4HCl(Conc) ® MnCl 2 + 2H 2 O + Cl 2
2K 4 [Fe(CN) 6 ] + H 2 O + O ® 2K 3[Fe(CN) 6 ] + 2KOH
2MnO 2 + 2H 2SO 4 (Conc) ® 2MnSO 4 + 2H 2 O + O 2
2I 2 + 9O ® I 4O9 [I2 , P,S, As] + O3 ® ic acid
(iv) Suboxides - They contain lower percentage of oxygen
Reducing action eg N2O, C3O2
H 2 O 2 + O 3 ® H 2O + 2O 2 BaO 2 + O 3 ® BaO + 2O 2 They have M – M and M – O bonds. For example O
= C = C = C = O (carbon suboxide)
NaOCl + O 3 ® NaCl + 2O 2
(v) Mixed oxides - Formed by the combination of two simple
Alkaline oxides eg Red lead, Pb3O 4 (PbO 2.2PbO), Fe 3O 4
K 3 [ Fe( CN ) 6 ] ¾¾ ¾¾® K 4 [ Fe( CN ) 6 ]
(FeO+Fe2O3)
O Classification on the basis of chemicals behaviour
Addition reactions CH 2 = CH 2 + O3 ® H 2C CH2 (i) Acidic oxides - Oxides of non metals which give acids
when dissolved in water are called acidic oxides eg.
O O CO2, NO2, P2O5, SO2, SO3, Cl2O7 etc.
Ozonide
CO 2 + H 2 O ® H 2CO 3 carbonic acid
Uses:- The metallic oxides of high oxidation state eg Mn2O7,
(1) Bleaching ivory, oils, flour etc. V2O5 and CrO3
(2) As germicide and disinfectant, for sterilising water (ii) Basic oxides
(3) For improving atmosphere in crowded places (a) Ionic in nature. Oxides of alkali and alkaline earth
The p-Block Elements - Oxygen Family 691
metals eg Na2O, CaO, BaO. In water they give basic 2K 2Cr2O7 + 8H 2SO4 (conc) ® 2
solutions
2K 2SO4 + 2Cr2 (SO4 )3 + 8H 2O + 3O2
Na 2 O + H 2 O ® 2 NaOH
(3) By electrolysis of water either acidified with H2SO4 using
(b) Covalent oxides - Oxides of transition metals are platinum electrodes or by making it alkaline with NaOH
covalent in nature eg CuO, FeO. Insoluble in water. or Ba(OH)2 using nickel electrodes.
(iii) Amphoteric oxides - The oxides which react with both -2e 1
2H 2 O ƒ 2H + 2OH - ¾¾¾ ® 2OH ƒ H 2 O + O2
acids and alkalies are known as amphoteric oxides eg 2
ZnO, Al2O3, SnO etc. (4) By decomposition of steam by chlorine.
ZnO + 2 NaOH ® Na 2 ZnO2 + H 2 O 2H 2O + 2Cl 2 ® 4HCl + O 2
ZnO + 2HCl ® ZnCl 2 + H 2O Manufacture - By fractional distillation of liquid air
Properties - It is colourless, odourless, tasteless slightly heavier
(iv) Neutral oxides - Such oxides do not combine with an than air, sparingly soluble in water, soluble in pyrogallol.
acid or a base eg NO, N2O, CO, H2O etc. Chemical properties - On heating it combines directly with metals
OXYGEN : and non metals.
Chinese observed the presence of oxygen in air. Priestly & C + O 2 ® CO 2
Scheely prepared oxygen by heating suitable oxygen compounds.
P4 + 5O 2 ® 2P2O 5
1 S + O 2 ® SO 2
Occurrence - About th of the atmosphere is free of elemental
5 3000º C
N 2 + O 2 ¾¾ ¾
¾® 2 NO
oxygen. In the combined state it is present in water 89% by weight,
earth’s crust about 50% and plants and animal tissues 50–70%. 2Mg + O 2 ® 2MgO 3Fe + 2O 2 ® Fe 3O 4
Preparation - 4 Na + O 2 ® 2 Na 2O ® Na 2 O 2
(1) By action of heat on oxygen rich compounds
(i) From oxides Pb + O 2 ® PbO 2
D
Combination with O2 is accelerated by using catalyst. Platinum is
2HgO ¾¾® 2Hg + O 2 ­ particularly active.
2Ag 2 O ® 4 Ag + O 2 ­ Pt
2H 2 + O 2 ¾¾® 2 H 2 O
(ii) From peroxides and other oxides
Pt
MnO 4NH 3 + 5O2 ¾¾ ® 4NO + 6H 2 O (Ostwald's process)
2H 2 O 2 ¾¾ ¾
¾2 ® 2H O + O
2 2
V O (Contact process)
D 2SO 2 + O 2 ¾¾2 ¾
5 ® 2SO
3
2BaO 2 ¾¾® 2 BaO + O 2
2CH 3OH + O 2 ® 2HCHO + 2H 2 O
D
2PbO 2 ¾¾® 2PbO + O 2
CuCl
4HCl + O 2 ¾¾ ¾
¾2 ® 2Cl + 2 H O (Deacon' s process)
2 2
D
2Pb 3 O 4 ¾¾® 6PbO + O 2 Uses
D (1) For breathing
3MnO 4 ¾¾® Mn 3 O 4 + O 2
(2) In welding and cutting – oxy-hydrogen or oxy - acetylene
(iii)From certain compounds torch is used.
D (3) In iron and steel industry – to increase the content of blast in
2KClO3 ¾¾® 2KCl + 3O2
MnO2 the Bessemer and open hearth process.
(4) As a fuel in rockets.
2KNO3 ® 2KNO2 + O2
Tests
2CaOCl 2 ® 2CaCl 2 + O 2 (1) With NO it gives reddish brown fumes of NO2.
2KMnO4 ® K 2 MnO4 + MnO2 + O 2 (2) Absorbed by alkaline pyrogallol.
(3) A smouldering wood splinter bursts into flames in a jar of O2.
D
4K 2 Cr2 O 7 ¾¾® 4K 2 CrO 4 + 2Cr2 O 3 + 3O 2
(2) By the action of some chemical reagent on compound Structure
rich in O2. It is paramagnetic with following electronic configuration
2 Na 2O 2 + 2H 2 O ® 2NaOH + O 2 s1s2 s*1s2 s2s 2 s* 2s2 s2p 2 p2p 2x = p2p 2y p* 2p1x = p* 2p1y
692 Chemistry
discharge tube As2O3 is precipitated as As2S3 by passing H2S
Atomic oxygen : O2 ¾¾¾¾¾¾ ® O + O DH = 489.6kJ
1 mm pressure As 2 O 3 + 3H 2S ® As 2S3 + 3H 2 O
Isotopes O16 O17 O18 PbSO4 is insoluble in water and settles down on
10000: 1: 8 dilution and filtered off.
NO and NO 2 are removed by heating with
SULPHUR :
Occurence - It occurs free in volcanic regions. In combined state ( NH 4 ) 2 SO 4
it occurs as ( NH 4 ) 2 SO 4 + NO + NO 2 ® 3H 2 O + H 2SO 4 + 2 N 2
(i) Gypsum CaSO4.2H2O
(f) Concentration - The sulphuric acid is concentrated
(ii) S-Celesite SrSO4 by evaporation
(iii) Galena PbS (2) Contact process - The steps involved are
(iv) Zinc blende ZnS (i) Production of SO2 - It is produced by burning sulphur
(v) Copper pyrites Cu2SFe2S3 and iron pyrites and purified by treating with steam to
(vi) Iron pyrites FeS2 remove dust particles. The arsenic is removed by ferric
Properties - Ordinary sulphur is pale yellow, insoluble in water hydroxide, water vapour removed by conc H2SO4. The
but dissolves in CS2, C6H6 and turpentine gases are filtered through coke filters and purity is tested
Allotrophic forms - Its important allotrophic forms are by Tyndal box.
Crystalline - Rhombic, monoclinic (ii) Conversion of SO2 to SO3 - It is done in contact or
Amorphous - Plastic, milk of sulphur, colloidal sulphur. catalyst chamber after being preheated to 450ºC.
COMPOUNDS OF SULPHUR : 2SO 2 + O 2 == 2SO 3 + x cal
1. Sulphuric acid - It is also known as oil of vitriol and king of 450° - 500ºC 1.6 to 1.7 atm Pressure
chemicals. Catalyst - Formerly platinised asbestose was used which
Manufactures of sulphuric acid is costly and easily poisoned. Now a days V2O5 is used.
(1) Lead chamber process - The various steps involved are (iii)SO3 is absorbed by conc H2SO4 and then water is added
(a) Production of SO2 : By burning S or iron pyrites. to produce the acid of desired concentration
D
S + O 2 ¾¾® SO 2 H 2SO 4 + SO 3 ® H 2S 2O 7
(oleium or pyrosulphuric acid)
4FeS2 + 11O2 ® 2Fe2O3 + 8SO2
H 2S 2O 7 + H 2O ® 2H 2SO 4
(b) Production of catalyst - Oxides of nitrogen
If SO3 is absorbed by water it liberates large amount of
2 NaNO3 + H 2SO 4 ® Na 2SO 4 + 2HNO 3
heat producing acid fog.
2HNO 3 ® H 2O + NO + NO 2 + O 2 Properties - Its specific gravity is 1.8 and it is 98% by weight
(c) Reaction in lead chamber normality is 36N.
Chemical properties
SO 2 + NO 2 ® SO 3 + NO
(i) Action of heat H 2SO 4 ® H 2O + SO 2 + O
SO 3 + H 2 O ® H 2SO 4
Hence it acts as oxidising agent
2NO + O 2 ® 2 NO 2 (used again) Non metals are oxidised to their oxides
Gay-Lussac Tower : The residual gases (mainly air + oxides C + 2O ® CO 2 S + 2O ® SO 2
of nitrogen) from lead chambers + conc. H2SO4 from glover
tower give nitrated acid. 2P + 5O ® P2O 5
2H 2SO 4 + NO + O 2 ® 2 NO.HSO 4 + H 2 O 2KI + 2H 2SO 4 ® K 2SO 4 + SO 2 + I 2 + 2H 2 O
Glover Tower : The nitrated acid from Gay-Lussac tower is 2KBr + 2H 2SO 4 ® K 2SO 4 + SO 2 + Br2 + 2H 2 O
denitrated in this tower.
(ii) Displacement reactions - It displaces volatile acids from
2 NO.HSO 4 + H 2 O ® 2H 2SO 4 + NO + NO 2 their salts.
Nitroso Na 2S + H 2SO 4 ® H 2S + Na 2SO 4
sulphuric acid

(d) Conditions - Temperature 50ºC, excess of steam, lead NaCl + H 2SO 4 ® HCl + NaHSO 4
chamber since lead is not attacked H2SO4. NaNO3 + H 2SO 4 ® HNO 3 + NaHSO 4
(e) Purification - The acid obtained contains the
impurities of PbSO4, AS2O3, NO and NO2 which are Ca 3 (PO 4 ) 2 + 3H 2SO 4 ® 2H 3PO 4 + 3CaSO 4
removed as follows
The p-Block Elements - Oxygen Family 693
Metals above hydrogen in ECS react with dil. H2SO4 (ii) Roasting iron pyrites in excess of air
with evolution of H2.
4FeS2 + 11O 2 ® 2Fe 2O 3 + 8SO 2
M(Fe, Sn, Zn, Al, Mn, Mg) + H2SO4 ® MSO4 + H2
Conc. H2SO4 reacts with all metals except Au and Pt with (iii)Lab method : By heating Cu with conc H2SO4
evolution of SO2. Cu + 2H 2SO 4 ® CuSO 4 + SO2 + 2H 2O
Metal + Conc. H2SO4 ® MSO4 + SO2 + H2O
Properties -
(iii)Dehydrating agent - It is strong dehydrating in nature
(i) As reducing agent
C12 H 22O11 ® 12C + H 2SO 4 .11H 2 O
SO 2 + Cl 2 + 2H 2O ® H 2SO 4 + 2HCl
C 2 H 5OH ® C 2 H 4 + H 2SO 4 .H 2 O
SO 2 + 2FeCl 3 + 2H 2 O ® H 2SO 4 + 2FeCl 2 + 2HCl
HCOOH ® CO + H 2SO 4 .H 2 O
2KIO3 + 5SO 2 + 4H 2O ® 2KHSO 4 + 3H 2SO 4 + I 2
(COOH) 2 ® CO + CO 2 + H 2SO 4 .H 2 O
(iv) Reaction with PCl5 2KMnO4 + 5SO 2 + 2H 2O ® K 2SO 4 + 2MnSO 4 + 2H 2SO 4
HO.SO2.OH + PCl5 ® Cl.SO2 .OH + POCl3 + HCl It decolorises the solution of KMnO4
Chlorosulphonic K 2Cr2 O 7 + H 2SO 4 + 3SO 2 ® K 2SO 4 + Cr2 (SO 4 ) 3 + H 2 O
acid green

HO.SO 2 .OH + 2PCl5 ® Cl.SO 2 .Cl + 2POCl 3 + 2HCl (ii) As oxidising agent
Sulphuryl 2H 2S + SO 2 ® 2H 2 O + 3S ¯
Chloride
(v) Sulphonation of aromatic compounds SO 2 + 2Mg ( burning ) ® 2MgO + S ¯
C6 H6 + H 2SO4 ® C6 H5 .SO2 OH + H2O 3Fe + SO 2 ® 2FeO + FeS
Benzene
sulphonic acid PbO 2 + SO 2 ® PbSO 4
(vi) Precipitation - Barium and lead are precipitated as (iii)Burning of potassium - Potassium burns in SO2 giving
sulphates sulphate and thiosulphate
BaCl 2 + H 2SO 4 ® BaSO 4 + 2HCl 4K + 3SO 2 ® K 2SO3 + K 2S2O3
Pb( NO 3 ) 2 + H 2SO 4 ® PbSO 4 + 2HNO 3 (iv) Bleaching action - Its bleaching action is due to
Hence these are gravimetrically estimated reduction
Fuming sulphuric acid [H2SO4.SO3] SO 2 + 2H 2O ® H 2SO 4 + 2H
With ammonia it forms (NH4)2 SO4 hence NH3 can not
Structure : It is sp2 hybridised and has V- shape.
be dried over H2SO4
Uses - It is used as oxidising, dehydrating agent and for the
S S S
preparation of dyes, drugs, explosives, volatile acids etc.
Structure - It has tetrahedral structure O 120º O O O or O O
120º
O O S- in excited state

S S
HO OH or HO OH

O O sp2
hybridisation
S- in doubly excited state s bond from sp2 -p overlap
one p bond from pp-pp bond overlap
other p bond from pp-dp overlap
SULPHUR TRIOXIDE (SO3) :
sp3 hybridisation These orbitals form two Preparation
p-bonds with two oxygen (i) By heating conc H2SO4 with P2O5
SULPHUR DIOXIDE (SO2) H 2SO 4 + P2 O 5 ® SO 3 + 2HPO 3
Preparation (ii) Manufacture : It is done by contact process
D
(i) By heating sulphur in air S + O 2 ¾¾® SO 2 2SO 2 + O 2 == 2SO 3
694 Chemistry
Properties - It dissolves in water with loud hissing sound and (ii) Reducing nature
evolution of heat 2KMnO 4 + 3H 2SO 4 + 5H 2S ® K
Chemical properties
K 2SO 4 + 2MnSO 4 + 8H 2 O + 5S
1. Action of heat 2SO 3 == 2SO 2 + O 2
K 2Cr2 O 7 + 4H 2SO 4 + 3H 2S ®
2. Water SO 3 + H 2 O == H 2SO 4 K 2 Cr2O7 + 4H 2SO4 + 3H 2S ® K 2SO4 + Cr2(SO 4 )3 + 7H 2O + 3S

3. With H 2SO 4 (Conc) : H 2SO 4 + SO 3 ® H 2S 2O 7 H 2S + Cl 2 ® 2HCl + S , H 2 O 2 + H 2S ® 2H 2O + S


(oleum) Uses - In qualitative analysis

4. Acidic nature: CaO + SO 3 ® CaSO 4 SODIUM THIOSULPHATE - Na2S2O3 .5H2O :


Preparation
5. With HCl : SO 3 + HCl ® Cl.SO 2 OH
(Chloro Sulphonic acid) (i) By boiling Na 2SO 3 with flowers of sulphur in absence
6. Oxidising agent - It is powerful oxidising agent of air
SO 3 + 2HBr ® H 2 O + Br2 + SO 2 Na 2SO 3 + S ® Na 2S2 O 3
Structure - It has planar triagonal structure due to sp2 hybridisation (ii) By heating sodium hydrogen sulphide and sodium
O
O
hydrogen sulphite together
O O
or 2NaHS + 4 NaHSO 3 ® 3Na 2S 2O 3 + 3H 2O
S S S S
Properties - It is also known as hypo
O O O O O O O O
1. Complexing - It dissolves silver halides forming complex
S- in excited state
AgX + 2 Na 2S2O3 ® Na 3[Ag(S2O 3 ) 2 ] + NaX
Sodium argento thiosulphate (soluble complex)
or
sp 2 2. It absorbs halogens -
hybridisation
2 Cl 2 + H 2O + Na 2S2O3 ® Na 2SO 4 + 2HCl + S
s bond from sp -p overlap
p bonds from pp-pp and pp-dp overlap Hence used to remove last traces of chlorine
or doubly excited state I 2 + 2Na 2S2O 3 ® 2 NaI + Na 2S 4O 6
used for estimation of iodine
sp 2 3. Reaction with AgNO3
hybridisation
2AgNO3 + Na 2S2 O3 ® Ag 2S2 O3 + 2NaNO3
O
Silver thiosulphate
S (white precipitate)
Structure :
O O Ag 2S2 O3 + H 2 O ® Ag 2S + H 2SO 4

Allotropic forms - It has three allotropic forms a , b and g Blue precipitate


Uses : In the manufacture of H2SO4 and drying agent. 4. Action of dil. H2SO4

HYDROGEN SULPHIDE (H2S) Na 2S 2 O 3 + dil. H 2SO 4 ® Na 2SO 4 + SO 2 + S + H 2 O


yellow
Preparation : Lab Method - precipitat e
By action of dil. H2SO4 or HCl on iron sulphide
Uses -
FeS + 2H 2SO 4 ® FeSO 4 + H 2S
(i) It is used as an antichlor compound.
Properties - Colourless poisonous gas with odour of rotten eggs
fairly soluble in water (ii) Fixer in photography to remove AgBr left.
(i) Combustibility - 2H 2S + 3O 2 ® 2H 2 O + 2SO 2 (iii) In the metallurgy for extraction of Ag and Au.
The p-Block Elements - Oxygen Family 695

Very Short/Short Answer Questions 18. Bleaching action of SO2 is due to


(a) reduction (b) oxidation
1. Why does the tendency to show –2 oxidation state
(c) hydrolysis (d) its acidic nature
diminishes from S to Po?
19. Difference between S and S2– as S2– has
2. Why does oxygen not show an oxidation state of + 4 and + (a) larger radii and large size
6 like sulphur? (b) smaller radii and large size
3. Sulphur forms SF6 while oxygen does not form OF6. Why? (c) larger radii and small size
4. Name allotropes of oxygen. (d) smaller radii and small size
5. H2S is acidic while H2O is neutral. Why? 20. Polyanion formation is maximum in
(a) nitrogen (b) oxygen (c) sulphur (d) boron
6. Compare the structure of SO3 and SeO3
21. The acid which has a peroxy linkage is
7. H2S has a stronger reducing behaviour in comparison to
(a) sulphurous acid (b) pyrosulphuric acid
that of water. Explain.
(c) dithionic acid (d) caro’s acid
8. Oxygen shows covalency of two whereas sulphur shows
22. Which compound is used in photography?
covalency upto six. Why?
(a) Na2SO5 (b) Na2S2O8
9. Why do hydrides of oxygen and sulphur differ in physical
state? (c) Na2S2O6 (d) Na2S2O3
10. Why do sulphurous acid a reducing agent? 23. By passing H 2S gas in acidified KMnO4 solution, we get
11. Among following, which has highest bond angle: (a) S (b) K2S
H2O, H2S, H2Se or H2 Te (c) MnO2 (d) K2SO3
12. S in vapour state exhibits paramagnetism. Why? 24. A gas that cannot be collected over water is :
13. Thermal stability of water is much higher than that of H2S. (a) N2 (b) O2
Explain. (c) SO2 (d) PH3
14. Why are group 16 elements called chalcogens? 25. On electrolysis of dilute sulphuric acid using platinum
electrodes, the product obtained at the anode will be
Long Answer Questions
(a) hydrogen (b) oxygen
15. What is catenation illustrate it for oxygen group.
(c) hydrogen sulphide (d) sulphur dioxide.
16. What is inert pair effect and what is its reason.
26. It is possible to obtain oxygen from air by fractional distillation
Multiple Choice Questions because
17. Which one of the following property is not correct for (a) oxygen is in a different group of the periodic table from
ozone? nitrogen
(a) It oxidises lead sulphide (b) oxygen is more reactive than nitrogen
(b) It oxidises potassium iodide (c) oxygen has higher b.p. than nitrogen
(d) oxygen has a lower density than nitrogen.
(c) It oxidises mercury
(d) It cannot act as bleaching agent.

1. Electron affinity of sulphur is 2. All the elements of oxygen family are


(a) more than O and Se (a) non metals (b) metalloids
(b) more than O but less than Se (c) radioactive (d) polymorphic
(c) less than O but more than Se 3. Which shows maximum catenation property ?
(d) equal to O and Se (a) S (b) Se
(c) Te (d) O
696 Chemistry
4. A group 16 element exists in monoatomic state in the metallic 19. There is S – S bond in
lattice. It also exists in two crystalline forms. The metals is (a) H2S2O7 (b) H2S2O8
(a) Po (b) S (c) H2S2O6 (d) H2SO5
(c) Se (d) Te 20. Oleum is chemically known as
5. Atomicity of sulphur in rhombic sulphur is (a) H2SO3 (b) H2SO5
(a) 1 (b) 2 (c) H2S2O7 (d) H2S2O8
(c) 8 (d) 6 21. S - S bond is present in
6. Which of the following hydrides has the lowest boiling point
? (a) S2 O 27 - (b) S3O9
(a) H2O (b) H2S (c) S2 O 24 - (d) S2 O32 -
(c) H2Se (d) H2Te
7. Which of the following hydrides is most acidic ? 22. Which of the following has pp- dp bonding ?
(a) H2Te (b) H2Se
(a) NO 3- (b) SO 32 -
(c) H2O (d) H2S
8. Which of the following hydrides shows the highest boiling (c) BO 33- (d) CO 32-
point ?
23. Caro’s acid is
(a) H2O (b) H2S
(c) H2Se (d) H2Te (a) H2SO3 (b) H3S2O5
9. Electronegativity of oxygen is more than sulphur yet H2S is (c) H2SO5 (d) H2S2O8
acidic while water is neutral. This is because 24. H2S reacts with O2 to form
(a) water is highly associated compound (a) H2O + S (b) H2O + SO2
(b) molecular mass of H2S is more than H2O (c) H2O + SO3 (d) H2SO4 + S
(c) H2S is gas while H2O is a liquid 25. Which is the best oxidising agent among the following ?
(d) H–S bond is weaker than H–O bond (a) S (b) O
10. What is the hybridization of S in SF4 ? (c) Se (d) Te
(a) sp 3d 2 (b) sp 3 d 26. On heating KClO3 we get
(c) sp d3 3 (d) sp 3 (a) KClO2 + O2 (b) KCl + O2
11. The shape of the molecule SF3Cl3 is (c) KCl + O3 (d) KCl + O2 + O3
(a) trigonal bipyramidal (b) cubic 27. Oxygen molecule is
(c) octahedral (d) tetrahedral (a) diamagnetic with no-unpaired electron(s)
12. States of hybridization of P in PF5 and S in SF6 are (b) diamagnetic with two unpaired electrons
respectively? (c) paramagnetic with two unpaired electrons
(a) sp 3d2,sp 3d (b) sp3d,sp 3d 2 (d) paramagnetic with no unpaired electron(s)
3
(c) sp ,sp d 3 (d) sp2,d 3 28. The number of electrons that are paired in oxygen molecule
are
13. Which of the following is not tetrahedral ?
(a) 16 (b) 12
(a) SCl4 (b) SO 24 - (c) 14 (d) 7
29. Which of the following is incorrect ?
(c) Ni(CO)4 (d) NiCl 24 - (a) O2 is weaker oxidant than O3
14. In OF2 molecule, the total number of bond pairs and lone pairs (b) O2 has larger bond length than O3
of electrons present respectively are (c) Both O2 and O3 are paramagnetic
(a) 2, 6 (b) 2, 8 (d) O2 is linear and O3 is angular in shape
(c) 2, 10 (d) 2, 9 30. Which of the following species possesses the highest bond
15. The type of hybridization in water molecule is order ?
(a) sp (b) sp 2 (a) O2 (b) O +2
(c) sp 3 (d) None of these
16. Hybridization of S in SO3 is (c) O 2– (d) O 22 –
(a) sp 2 (b) sp 3
(c) sp d2 (d) sp 3d 2 31. In the species O2, O +2 , O 2– and O 22 – , the correct decreasing
17. Permonosulphuric acid is known as order of bond strength is
(a) Marshall’s acid (b) Caro’s acid (a) O 2 > O 2+ > O 2– > O 22 -
(c) Sulphuric acid (d) None of these
18. S – S bond is not present in (b) O 2+ > O 2 > O 2– > O 22 -
(a) S2 O 24 - (b) S2 O52 - (c) O 22- > O 2- > O 2+ > O 2
(c) S2 O32 - (d) S2 O 27 - (d) O -2 > O 22- > O 2 > O +2
The p-Block Elements - Oxygen Family 697
32. Which is most acidic in nature ? 47. Bleaching action of SO2 is due to its
(a) Na2O (b) MgO (a) oxidising property (b) acidic property
(c) Al2O3 (d) CaO (c) reducing property (d) basic property
33. Which of the following oxides reacts with both HCl and NaOH 48. H2S does not produce metallic sulphide with
? (a) CdCl2 (b) ZnCl2
(a) CaO (b) ZnO (c) COCl2 (d) CuCl2
(c) N2O5 (d) CO2 49. By passing H2S gas in acidified KMnO4 solution, we get
34. Which of the following is formed by the action of water on (a) K2S (b) S
sodium peroxide ? (c) K2SO3 (d) MnO2
(a) H2 (b) N2 50. Na2S2O3 is prepared by
(c) O2 (d) CO2 (a) reacting H2S2O3 with NaOH
35. Sulphuric acid reacts with PCl5 to give (b) reducing Na2SO4 with S in alkaline medium
(a) thionyl chloride (b) sulphur monochloride (c) heating NaOH and S
(c) sulphuryl chloride (d) sulphur tetrachloride (d) reducing Na2SO4 with S in acidic medium
36. Which one of the following reacts with conc. H2SO4? 51. What are the products formed when chlorine is passed
(a) Au (b) Ag through aqueous hypo solution?
(c) Pt (d) Pb (a) Na2SO3 + HCl + S (b) Na2SO3 + SO3 + HCl
37. HCOOH reacts with conc. H2SO4 to produce (c) Na2SO4 + HCl + S (d) Na2SO4 + HCl + SO2
(a) CO (b) CO2 52. What is the oxidation number of sulphur in Na2S4O6?
(c) NO (d) NO2
38. The number of dative bonds in sulphuric acid molecule is 2 3
(a) (b)
(a) 0 (b) 1 3 2
(c) 2 (d) 4 3 5
39. What is the number of sigma ( s ) and pi ( p ) bonds present (c) (d)
5 2
in sulphuric acid molecule ?
(a) 6 s , 2 p (b) 6 s , 0 p 53. The metal with highest electrical resistance at room
temperature is
(c) 2 s , 4 p (d) 2 s , 2 p
(a) Pb (b) Te
40. The oxidation number of sulphur is –1 in
(c) Po (d) Fe
(a) FeS (b) FeS2
54. The compound containing coordinate bond is
O O O (a) O3 (b) SO3
|| || || (c) H2SO4 (d) All of these
(c) NaO - S = O (d) S - S - ONa
|| || 55. Which of the following is not a reducing agent ?
ONa ONa (a) SO2 (b) H2O2
(c) CO2 (d) NO2
41. Bromine water reacts with SO2 to form ?
56. When potassium ferrocyanide crystals are heated with conc.
(a) H2O and HBr (b) H2SO4 + HBr
H2SO4, the gas evolved is
(c) HBr and S (d) S and H2O (a) SO2 (b) NH3
42. Which of the following behaves as both oxidising as well as (c) CO2 (d) CO
reducing agent ? 57. Correct order of O – O bond length (increasing) in O2, H2O2
(a) H2SO4 (b) SO2
and O3 is
(c) H2S (d) HNO3
(a) H2O2 < O3 < O2 (b) O2 < O3 < H2O2
43. By which of the following SO2 is formed ?
(c) O3 < O2 < H2O2 (d) O3 < H2O2 < O2
(a) Reaction of dil. H2SO4 with O2
58. Which of the following does not exhibit sp3 hybridisation?
(b) Hydrolysis of dil. H2SO4
(a) SO2 (b) CH4
(c) Reaction of conc. H2SO4 with Cu
(d) None of these (c) NH3 (d) SO 24 -
44. The compound of sulphur that can be used as refrigerant is
59. Which of the following species is basic & reducing?
(a) SO2 (b) SO3
(c) S2Cl2 (d) H2SO4 (a) SO 32 - (b) SO 24 -
45. Which of the following causes damage to the building
containing calcium and responsible for cough and choking (c) S2 O 24 - (d) HSO -4
in human? 60. Iron sulphide is heated in air to form A, an oxide of sulphur.
(a) Sulphur (b) Carbon A is dissolved in water to give an acid. The basicity of this
(c) Nitrogen dioxide (d) Sulphur dioxide acid is
46. Number of bonds in SO2 are (a) 2 (b) 3
(a) two s and two p (b) two s and one p (c) 1 (d) zero
(c) two s and three p (d) None of these
698 Chemistry

1. Hypo is used in photography to [CBSE-PMT1988] 11. By passing H 2S gas in acidified KMnO4 solution, we get
(a) Reduce AgBr grains to metallic silver [CBSE-PMT 1995]
(b) Convert metallic silver to silver salt (a) S (b) K2S
(c) Remove undecomposed silver bromide as a soluble (c) MnO2 (d) K2SO3
complex 12. Oxidation of thiosulphate by iodine gives
(d) Remove reduced silver [CBSE-PMT 1996]
2. Oxygen will directly react with each of the following elements (a) tetrathionate ion (b) sulphide ion
except [CBSE-PMT 1989] (c) sulphate ion (d) sulphite ion
13. During its reactions, ozone [CBSE-PMT 1999]
(a) P (b) Cl (c) Na (d) S.
(a) can only combine with hydrogen atoms
3. The gases respectively absorbed by alkaline pyrogallol and (b) accepts electrons
oil of cinnamon are [CBSE-PMT 1989] (c) loses electrons
(a) O3, CH4 (b) O2, O3 (c) SO2,CH4 (d) N2O, O3. (d) shows the role of electrons to be irrelevant
4. It is possible to obtain oxygen from air by fractional distillation 14. Which of the following is the most basic oxide?
because [CBSE-PMT 1989] (a) Sb2O3 (b) Bi2O3 [CBSE-PMT 2006]
(a) oxygen is in a different group of the periodic table from (c) SeO2 (d) Al2O3
nitrogen 15. Which one of the following compounds is a peroxide ?
(b) oxygen is more reactive than nitrogen [CBSE-PMT 2010]
(c) oxygen has higher b.p. than nitrogen (a) KO 2 (b) BaO2
(d) oxygen has a lower density than nitrogen.
5. Which would quickly absorb oxygen ? (c) MnO 2 (d) NO 2
[CBSE-PMT1991, 92] 16. Match List - I (substances) with List - II (processes) employed
(a) Alkaline solution of pyrogallol in the manufacture of the substances and select the correct
(b) Conc. H2SO4 option. [CBSE-PMT 2010]
(c) Lime water List - I List - II
(d) Alkaline solution of CuSO4 Substances Processes
6. Oleum is [CBSE-PMT 1991] (A) Sulphuric acid (i) Haber’s process
(a) Castor Oil (b) Oil of vitriol (B) Steel (ii) Bessemer’s
(c) Fuming H2SO4 (d) None of them process
7. A certain compound (X) when treated with copper sulphate
(C) Sodium hydroxide (iii) Leblanc process
solution yields a brown precipitate. On adding hypo solution,
(D) Ammonia (iv) Contact process
the precipitate turns white. The compound is
(a) K2CO3 (b) KI [CBSE-PMT 1994] Options:
(c) KBr (d) K3PO4 (A) (B) (C) (D)
8. Polyanion formation is maximum in [CBSE-PMT 1994] (a) (iv) (ii) (iii) (i)
(a) Nitrogen (b) Oxygen (b) (i) (iv) (ii) (iii)
(c) Sulphur (d) Boron (c) (i) (ii) (iii) (iv)
9. The acid which has a peroxy linkage is [CBSE-PMT 1994] (d) (iv) (iii) (ii) (i)
(a) Sulphurous acid (b) Pyrosulphuric acid 17. Which of the following oxide is amphoteric ?
(c) Dithionic acid (d) Caro’s acid [CBSE-PMT 2011 M]
10. About 20 km above the earth, there is an ozone layer. Which (a) SnO2 (b) CaO (c) SiO2 (d) CO2
one of the following statements about ozone and ozone layer
18. Sulphur trioxide can be obtained by which of the following
is true? [CBSE-PMT 1995]
reaction : [CBSE-PMT 2012 S]
(a) ozone has a triatomic linear molecule
(b) Fe 2 ( SO4 )3 ¾¾
Δ Δ
(b) it is harmful as it stops useful radiation (a) CaSO 4 + C ¾¾ ® ®
(c) it is beneficial to us as it stops U.V radiation
Δ Δ
(d) conversion of O3 to O2 is an endothermic reaction (c) S + H 2SO 4 ¾¾ ® (d) H 2SO 4 + PCl5 ¾¾ ®
The p-Block Elements - Oxygen Family 699
19. Roasting of sulphides gives the gas X as a by product. This 23. Excess of KI reacts with CuSO4 solution and then Na2S2O3
is colorless gas with choking smell of burnt sulphur and solution is added to it. Which of the statements is incorrect
caused great damage to respiratory organs as a result of for this reaction ? [AIEEE 2004]
acid rain. Its aqueous solution is acidic, acts as a reducing (a) Na2S2O3 is oxidised
agent and its acid has never been isolated. The gas X is : (b) CuI2 is formed
(a) SO2 (b) CO2 [NEET 2013] (c) Cu2I2 is formed
(c) SO3 (d) H2S (d) Evolved I2 is reduced
20. Which of the following does not give oxygen on heating? 24. Which of the following statements regarding sulphur is
(a) Zn(ClO3)2 (b) K2Cr2O7 [NEET 2013] incorrect? [AIEEE 2011]
(c) (NH4)2Cr2O7 (d) KClO3 (a) S2 molecule is paramagnetic.
21. Which one of the following is an amphoteric
(b) The vapours at 200°C consists mostly of S8 rings.
oxide ? [AIEEE 2003]
(c) At 600°C the gas mainly consists of S2 molecules.
(a) Na2O (b) SO2
(d) The oxidation state of sulphur is never less than +4 in its
(c) B2O3 (d) ZnO
compounds.
22. KO2 (potassium super oxide) is used in oxygen cylinders in
25. Aqueous solution of Na2S2O3 on reaction with Cl2 gives –
space and submarines because it [AIEEE 2002]
[IIT-JEE 2008]
(a) absorbs CO2 and increases O2 content
(a) Na2S4O6 (b) NaHSO4 (c) NaCl (d) NaOH
(b) eliminates moisture
(c) absorbs CO2
(d) produces ozone.

1. When SO2 gas is passed through an acidified solution of 7. Which reaction represents the oxidizing behaviour of H2SO4
K2Cr2O7 (a) 2PCl 5 + H 2SO 4 ® 2POCl 3 + 2HCl + SO 2Cl 2
(a) the solution becomes blue
(b) 2NaOH + H 2SO 4 ® Na 2SO 4 + 2H 2O
(b) the solution becomes colourless
(c) SO2 is reduced (c) NaCl + H 2SO 4 ® NaHSO 4 + HCl
(d) green Cr2(SO4)3 is formed
(d) 2HI + H 2SO 4 ® I 2 + SO 2 + 2H 2 O
2. FeCl3 solution on reaction with SO2 changes to
(a) FeCl2 (b) Fe2(SO4)3 8. Which characteristic is not correct about H2SO4 ?
(c) Fe2(SO3)3 (d) FeSO4 (a) Reducing agent (b) Oxidising agent
3. A considerable part of the harmful ultraviolet radiation of the (c) Sulphonating agent (d) Highly viscous
sun does not reach the surface of the earth . This is because 9. Which of the following is a saline oxide ?
high above the earth’s atmosphere there is a layer of (a) Na2O2 (b) BaO2
(a) CO2 (b) hydrogen (c) Na2O (d) Fe2O3
(c) ozone (d) ammonia 10. In the manufacture of H2SO4 the nitrated acid from the Gay -
4. On passing H2S through acidified FeCl3 solution FeCl3 is Lussac’s tower is chemically
converted into (a) H2SO4.NO2 (b) H2SO4.NO
(a) FeCl2 (b) Fe2( SO4)3 (c) H2SO4. 2NO (d) HSO4. NO
(c) FeS (d) FeSO4 11. Potassium chlorate on heating with conc H2SO4 gives
5. H2SO4 is added while preparing a standard solution of (a) chlorine dioxide (b) HClO4
Mohr’s salt to prevent (c) KHSO4 (d) All of these
(a) hydration (b) reduction 12. Sulphur forms the chlorides S2Cl2 and SCl2. The equivalent
(c) hydrolysis (d) complex formation mass of sulphur in SCl2 is 16g /mol . Therefore the equivalent
6. Concentrated H2SO4 is not used to prepare HBr from KBr mass of sulphur in S2Cl2 is
because it (a) 32 g / mol (b) 16 g /mol
(a) oxidizes HBr (c) 64 g/mol (d) 8 g/ mol
(b) reduces HBr 13. Reagent used to distinguish H2O2 and O3 is
(c) causes disproportionation of HBr (a) PbS (b) starch and iodine
(d) reacts too slowly with KBr (c) KMnO4 (d) bleaching powder
700 Chemistry
14. Which of the following does not react with AgCl 22. Which of the following statements is correct :
(a) Na2S2O3 (b) NH4OH (a) Ozone is a resonance hybrid of oxygen
(c) NaNO3 (d) Na2CO3 (b) Ozone is an isomer of oxygen
15. When KBr is treated with conc. H2SO4 reddish brown gas is
(c) Ozone has no relationship with oxygen
evolved . The gas is
(a) Br2 (b) Br2 + HBr (d) Ozone is an allotropic modification of oxygen
(c) NO2 (d) H2O2 23. The crystals of ferrous sulphate on heating give :
16. One gas bleaches the colour of the flowers by reduction (a) FeO + SO 2 + H 2 O
while the other by oxidation. The gases are
(a) CO and CO2 (b) H2S and Br2 (b) FeO + SO 3 + H 2SO 4 + H 2O
(c) SO2 and Cl2 (d) NH3 and SO3
17. Which oxide is of different type than others (c) Fe 2 O 3 + SO 2 + H 2SO 4 + H 2O
(a) MnO2 (b) PbO2
(d) Fe 2O 3 + H 2SO 4 + H 2 O
(c) TiO2 (d) Na2O2
18. In which reaction does SO2 act as oxidising agent 24. In the upper layers of atmosphere ozone is formed by the
(a) Acidified KMnO4 (b) Acidified K2Cr2O7 (a) action of ultraviolet rays on oxygen
(c) Acidified C2H5OH (d) H2S (b) action of electric discharge on oxygen molecules
19. Hydrogen sulphide reacts with lead acetate forming a black
(c) combination of oxygen molecules
compound which reacts with H2 O2 to form another
compound . The colour of the compound is (d) None of these
(a) black (b) yellow 25. When sugar is treated with conc. sulphuric acid, the sugar is
(c) white (d) pink charred. In this process, sugar is
20. Hydrolysis of one mole of peroxodisulphuric acid produces (a) reduced (b) oxidized
(a) two moles of sulphuric acid (c) sulphonated (d) dehydrated
(b) two moles of peroxomonosulphuric acid
(c) one mole of sulphuric acid and one mole of 26. H 2SO 4 and H 2SO 3 can be distinguished by the addition
peroxomonosulphuric acid of :
(d) one mole of sulphuric acid, on e mole of
peroxomonosulphuric acid and one mole of hydrogen (a) magnesium powder (b) NaHSO 4 solution
peroxide.
(c) FeCl3 solution (d) litmus solution
21. Oxygen gas can be prepared from solid KMnO4 by :
27. Yellow ammonium sulphide is
(a) treating the solid with H 2 gas
(a) ( NH 4 ) 2 S8 (b) ( NH 4 ) 2 S
(b) strongly heating the solid
(c) dissolving the solid in dil. H 2SO 4 (c) ( NH 4 ) 2 S x (d) ( NH 4 ) 2 S 4
(d) dissolving solid in dil. HCl
The p-Block Elements - Oxygen Family 701

EXERCISE 1 14. (b) F – O – F Number of bond pairs = 2, Number of lone


2. Because oxygen do not have vacant d-orbitals. pairs = 8.
4. Dioxygen (O2) and ozone (O3). 15. (c) H2O is sp3 hybridised
7. H2S is a stronger reducing agent as compared to water. This 16. (a) In SO3, sp2 hybridisation
is because S – H bond is weaker than O – H bond. 17. (b) It is Caro’s acid
11. Bond angle is highest in H2O (104.5) and minimum in H2Te 18. (d) See oxy acids of sulphur
(90°) 19. (c) See oxy acids of sulphur
13. This is because of O – H bond energy being higher than 20. (c) O. N. of S in H2S2O8 is + 6 (see structure and calculate)
that of S – H bond energy. 21. (c) See structures of oxy acids of sulphur
14. The elements of group 16 are called chalcogens because
many metals are found as oxides and sulphides and a few as 22. (b) p p –d p bonding is present in SO 32 - , N, B, C have no
selenides and tellurides. vacant d atomic arbitals.
17. (d) 18. (a) 19. (a) 20. (c) 21. (d) 22. (d)
23. (a) 24. (c) 25. (b) 26. (c)
EXERCISE 2
1. (a) Electron affinity increases from left to right in period and
decreases from top to bottom in a group but electron
affinity of O is less than S due to small size.
2. (d) All exhibit polymorphism
3. (a) Sulphur (see text) O S O– pyramidal
4. (a) Po is monoatomic and has a and b metallic forms O–
5. (c) It is 8 (see general discussion). pyramidal
6. (b) H2S (see text) 23. (c) It is H2SO5 (see text)
7. (a) H2Te (see text)
24. (a) 2H 2S + O 2 ® 2H 2 O + 2S
8. (a) H2O (due to intermolecular H - bonding)
9. (d) SH–bond is weaker than, O–H bond. Hence H2S will 25. (b) Oxygen being more electronegative
furnish more H+ ions D
26. (b) 2KClO 3 ¾¾® 2 KCl + 3O 2
1 27. (c) See text it is paramagnetic with two unpaired electrons
10. (b) For hybridisation = (Number of valence electrons of
2 28. (c) Total number of electrons in O2 is 16. It has 2 unpaired
central metal atom + Monovalent atoms + Negative charge electrons, the rest 14 are paired.
29. (c) O2 is paramagnetic and O3 is diamagnetic
1
– Positive charge) = (6 + 4 + 0 – 0) = 5 (hence sp3d.)
2 1
30. (b) Bond order = (Number of bonding electrons –
11. (c) Hybridisation is sp3d2 , hence octahedral 2
12. (b) antibonding electrons)
10 - 6 10 - 5 10 - 7
= 2; O+ =
1
13. (a) Hybridisation in SCl4 = (6 + 4 + 0 – 0 ) = 5 sp3d, similarly O2 = = 2.5;O2– = = 1.5;
2 2 2 2 2
-- 10 - 8
SO 4 is sp3 hybridised Ni(CO)4 O.S. of Ni is 0. Electronic O2 - = =1
2 2
configuration [Ar]3d84s 2 CO strong ligand hence
[Ar]3d10 \ Hybridisation sp3 and Ni (Cl ) 24 - is sp3. Highest bond order 2.5 O +2
702 Chemistry
31. (b)
56. (d) K 4 [Fe(CN)6 ] + 6H 2SO4 + 6H 2O ®
32. (c) Oxides of alkali and alkaline earth metals are basic Al2O3
is amphoteric hence acidic in comparision to others. 2K 2SO 4 + FeSO 4 + 6CO + 3 ( NH 4 ) SO4
2
33. (b) ZnO is amphoteric (see oxides).
57. (b) H2O2 > O3 > O2
34. (c) 2Na 2 O 2 + 2H 2O ® 4 NaOH + O 2 B. O. is 1, 1.5 and 2 respectively.

35. (c) HO.SO 2OH + 2PCl5 ® ClSO2Cl + 2POCl3 + 2HCl O


Sulphuryl chloride
H – O – O – H; O O; O = O
36. (b) 2Ag + 2H 2SO 4 ® 2H 2 O + SO 2 + Ag 2SO 4 . 58. (a) SO2 is sp2
hybridised whereas others are sp3 hybridised.
Au, Pt does not react. Pb forms insoluble PbSO4 59. (a) A base can donote electrons hence all are bases but
37. (a) HCOOH ® H 2O + CO (Dehydration) SO 32 - can be oxidised to SO 24 - hence it is reducing also.
38. (c) See structure of acids 60. (a) 4FeS + 7O 2 ® 2Fe 2 O 3 + 4SO 2
O
||
39. (a) H - O - S- O - H ; 6s & 2p
|| It is dibasic
O
EXERCISE 3
2+ 2- 1. (c) Undecomposed AgBr forms a soluble complex with hypo
40. (b) In Fe | S2 O.N of S is –1 AgBr + 2 Na 2S2O 3 ® Na 3[Ag(S 2O 3 ) 2 ] + NaBr
soluble complex
41. (b) Br2 + SO 2 + 2H 2O ® 2HBr + H 2SO 4 It is washed with water and the image is fixed
42. (b) In SO2, S is in + 4 O.S. which can be increased or decreased 2. (b) S + O 2 ¾¾
® SO 2 (burns with blue light)
by gain or loss of electrons. Hence SO2 can act as both 4Na + O 2 ¾¾
® 2NaO
oxidising and reducing agent. (burns with yellow light)
43. (c) Cu + 2H 2SO4 (conc) ® CuSO4 + SO 2 + 2H 2O P4 + 3O 2 ¾¾
® P4 O 6
44. (a) Due to large enthalpy of vaporisation SO2 can be used as P4 + 5O 2 ¾¾
® P4 O10
refrigerant Cl + O 2 ¾¾
® No reaction
45. (d) 2SO 2 + 2H 2 O + O 2 ® 2H 2SO 4 . It reacts with marble Chlorine does not react directly with oxygen.
3. (b) Alkaline pyrogallol absorbs O2 and oil of cinnamon
(CaCO3) causing damage. absorbs O3.
46. (b) 2s, one p see structure 4. (c) Air is liquified by making use of the joule-Thomson effect
(cooling by expansion of the gas). Water vapour and
47. (c) SO 2 + 2H 2 O ® H 2SO 4 + 2H . Bleaching action is due CO2 are removed by solidification. The remaining
to reduction. constituents of liquid air i.e., liquid oxygen and liquid
48. (c) COCl2 does not contain metal others give ZnS, CdS CuS nitrogen are separated by means of fractional distillation
(b.p. of O2 = –183°C : b. P. of N2 = – 195.8°C).
49. (b) 2KMnO 4 + 3H 2SO 4 + 5H 2S ® 5. (a) Alkaline solution of pyrogallol absorbs oxygen quickly.
® K 2SO 4 + 2MnSO 4 + 8H 2O + 5S 6. (c) Oleum is H 2S2 O 7 ( H 2SO 4 + SO 3 ) which is obtained
50. (c) See preparation by dissolving SO 3 in H2 SO4 and is called fuming
6 NaOH + 4S ® Na 2S2 O3 + 2 Na 2S + 3H 2 O sulphuric acid.
7. (b) KI reacts with CuSO4 solution to produce cuprous iodide
51. (c) Na 2S 2O 3 + Cl 2 + H 2O ® 2HCl + S + Na 2SO 4 (white precipitate) and I2 (which gives brown colour)
52. (d) Na2S4O6 Iodine reacts with hypo (Na 2 S2 O35H2 O) solution.
Let O.N. of S be x; 2 × (+1) + 4 × (x) + 6 × (–2) = 0 Decolourisaiton of solution shows the appearance of
white precipitate.
5 2CuSO4 + 4KI ® 2K 2SO4 + 2CuI + I2
\x = Cuprous iodide (Brown colour
2 (White ppt.) in solution)
53. (b) Ans is (Te)
54. (d) All (see text) 2Na 2S2 O3 + I2 ¾¾
® Na 2S4 O6 + 2NaI
Sod. tetra
55. (c) CO2 in which C cannot increase its O.N. thionate
(colourless)
The p-Block Elements - Oxygen Family 703
8. (c) Due to greater tendency for catenation, sulphur shows D
property of polyanion formation to a greater extent. For 2 KClO3 ¾¾® 2KCl + 3O2
D
example, in polysulphides such as S32 - , S24 - , S52 - 4K2Cr2O7 ¾¾® 4K2CrO4 + 2Cr2O3 + 3O2
9. (d) Caro’s acid is H 2SO 5 which contains one S – O – O 21. (d) Na2O (basic), SO2 and B2O3 (acidic) and ZnO is
– H peroxy linkage. It is also known as permonosulphuric amphoteric
acids. 22. (a) 2KO2 + 2H2O ® 2KOH + H2O2 + O2 eliminates moisture.
-1 0
O 23. (b) 4KI+ 2CuSO4 ® I2 + Cu 2I2 + 2K 2SO4
||
H – O – O – S – OH 0 2+ -2.5 -1
|| I2 + 2Na 2S2O3 ® Na 2S4O6 + 2NaI
O In this CuI2 is not formed.
Caro's acid
24. (d) Oxidation state of sulphur varies from – 2 to + 6 in its
10. (c) Ozone layer is beneficial to us, because it stops harmful various compounds.
ultraviolet radiations to reach the earth. 25. (b) The following reaction occurs
Na2S2O3 + 4Cl2 + 5H2O ¾¾ ® 2 NaHSO4 + 8HCl.
11. (a) 2 KMnO 4 + 5 H 2 S + 3H 2 SO 4 ¾¾®
K 2 SO 4 + 2MnSO 4 + 5S + 8H 2 O. EXERCISE 4
Thus in this reaction sulphur (S) is produced. 1. (d) K 2 Cr2O 7 + H 2SO 4 + 3SO 2 ® K 2SO 4 + Cr2 (SO 4 ) 3 + H 2O
Green
12. (a) 2 S2 O 3-2 + I 2 ® S 4O 6-2 + 2I -
Tetrathion ate 2. (a) 2FeCl3 + SO 2 + 2H 2 O ® 2FeCl 2 + H 2SO4 + 2HCl
13. (a) Since ozone can easily lose oxygen atom (nascent oxygen), 3. (c) U.V rays are absorbed by O3
it acts as a powerful oxidising agent, and hence reacts with
hydrogen atoms. 4. (a) 2FeCl 3 + H 2S ® 2FeCl 2 + 2HCl + S
14. (b) More the oxidation state of the central atom (metal) more 5. (c) FeSO 4 is hydrolysed to Fe (OH)2 which is converted
is its acidity. Hence SeO2 (O. S. of Se = +4) is acidic.
back to FeSO4
Further for a given O.S., the basic character of the oxides
increases with the increasing size of the central atom.
FeSO 4 + 2H 2O ® Fe(OH) 2 + H 2SO 4
Thus Al2O3 and Sb2O3 are amphoteric and Bi2O3 is
basic. 6. (a) 2HBr + H 2SO 4 ® Br2 + 2H 2O + SO 2 HBr is oxidised
to Br2
O
7. (d) 2HI + H 2SO 4 ® I 2 + SO 2 + 2H 2 O (HI is oxidised
15. (b) Ba to I2 ) ( H 2SO 4 is reduced to SO2)
8. (a) In H 2SO 4 , the S atom is present in its highest oxidation
O
state of +6. Hence H 2SO 4 can act an oxidant only by
16. (a)
gain of electrons
(A) Sulphuric acid (iv) Contact process
9. (c) Oxides which are more ionic in nature (salt - like) are
(B) Steel (ii) Bessemer’s
known as saline oxides e.g. oxides of alkali metals
process
(C) Sodium hydroxide (iii) Leblanc process 10. (d) 2H 2SO 4 + NO + O 2 ® 2 NO.HSO 4 + H 2 O
Nitrated acid
(D) Ammonia (i) Haber’s process
17. (a) SnO2 is an amphoteric oxide because it reacts with 11. (d) 3KClO 3 + 3H 2SO 4 ® 3KHSO 4 + HClO 4 + 2ClO 2 + H 2 O
acids as well as with bases to form corresponding salts.
SnO2 + 2H2SO4(conc) ¾¾ ® Sn(SO4)2 + 2H2O 12. (a) Apply the formula WA = WB ; 64 = 71 ,\ E = 32
SnO2 + 2NaOH ¾¾ Na SnO EA E B E 35.5
® 2 3 + H2O
18. (b) Fe 2 (SO4 )3 ¾¾
® Fe2O3 + 3SO3 13. (c) The pink colour of KMnO4 is decolorised by H2O2 and
Δ not by O3
19. (a) Based on the features given above the gas must be 2KMnO 4 + 3H 2SO 4 ® K 2SO 4 + 2MnSO 4 + 3H 2 O + 5O
SO2 .
H2O2 + O ® H2O + O2
D
20. (c) (NH4)2Cr2O7 ¾¾® N2 + Cr 2O3 + 4H2O 14. (c) There is no reaction with NaNO3 and AgCl while all
D other react to give Ag2S2O3, [Ag(NH3)2]Cl and Ag
Zn(ClO3)2 ¾¾® ZnCl2 + 3O2 respectively
704 Chemistry
15. (a) 2KBr + 3H 2SO 4 ® 2KHSO 4 + SO 2 + Br2 + 2H 2 O D
21. (b) 2KMnO 4 ¾¾® K 2 MnO4 + 4MnO 2 + O 2
16. (c) SO2 bleaches by reduction and Cl 2 by oxidation
17. (d) N2O2 is peroxide while others are dioxides 22. (d) Ozone is an allotrope of oxygen.
18. (d) SO 2 + 2H 2S ® 2H 2O + 3S. 23. (c) FeSO 4 . 7H 2 O ® FeSO 4 + 7H 2 O
The O.S of S changes from – 2 to 0 by loss of electrons.
D
Hence SO2 is oxidising 2FeSO 4 ¾¾® Fe 2O 3 + SO 2 + SO 3
19. (c) (CH 3COO) 2 Pb + H 2S ® PbS + 2CH 3COOH UV
24. (a) 3O 2 ¾¾
¾® 2O 3
PbS + 4H 2O 2 ® PbSO 4 + 4H 2O
White
25. (d) C12 H 22O11 ¾H SO 4
¾2 ¾¾ ® 12C + 11H 2 O
D black
O O

20. (c) HO S O O S OH + H2O 26. (c) FeCl 3 acts as oxidant and H 2SO 3 as reductant.

O 27. (c) Yellow ammonium sulphide is ( NH 4 ) 2 S x


O

peroxodisulphuric acid

O O O

O S OH + H O HO S O OH + HO S OH

O O O

peroxomonosulphuric acid sulphuric acid


21C
The p-Block Elements -
Halogens
GENERAL CHARACTERISTICS : 7. Oxidation State -
The group 17 of periodic table consists of five elements fluorine, F2 Cl2 Br2 I2 At
chlorine, bromine, iodine and astatine and are known as -1 -1 to + 6 -1 to +6 -1 to +7 -1,+1,+5
halogens (sea salt forming elements). Astatine is artificially 8. Bond energy and bond length - The bond length increases
prepared by radioactive element. from fluorine to iodine
1. Electronic configuration - F-F Cl-Cl Br-Br I-I
Element Symbol At No. Valence shell Bond length (Å) 1.42 1.99 2.28 2.67
electronic configration Bond energy (kJ mol-1) 158.8 242.6 192.8 151.1
Due to small size the interelectronic repulsions between non
Fluorine F 9 [ He] 2s 2 2 p5 bonding electrons are high in case of fluorine which results
in weakening of F-F bond.
Chlorine Cl 17 [ Ne] 3s 2 3 p5 9. Density - It increases down the group in a regular fashion
and follows the order F > Cl > Br > I
Bromine Br 35 [ Ar ] 3d 10 , 4s 2 4 p 5
(i) Atomic radii (ii) Ionic radii
Iodine I 53 [ Kr ] 4d10 , 5s 2 5 p5 (iii) atomic volume (iv) density
(v) Electronegativity (vi) Oxidising power
Astatine At 85 [ Xe ] 4 f 14 , 3d10 , 6s 2 6 p5 (vii) Reactivity (viii) Affinity for hydrogen
(ix) Reduction potential
2. Physical State - Intermolecular forces in halogens are weak
(x) Solubility, all follow the above given order.
and increase down the group. Thus F2, Cl2 are gases, Br2
10. Ionisation potential - The ionisation potential of halogens
volatite liquid and I2 volatile solid.
is very high and value decreases down the group. Thus
3. Atomicity - All halogens are diatomic in nature. iodine also forms I+ and I3+ and forms compounds like ICl,
4. Abundance - Being very reactive in nature they are not ICN, IPO4. In molten state these compounds conduct
found free in nature. Their presence in earth’s crust follows electricity and show ionic character.
the order. 11. Electron affinity - The halogens have the high values for
F2 > Cl2 > Br2 > I2 > At electron affinity following the order.
5. Colour - They absorb light in the visible range forming excited Cl > F > Br > I
states and thus they are coloured in nature. Due to small size of fluorine (hence high electron density)
the extra electron to be added feels more electron - electron
F2 Cl2 Br2 I2
repulsion. Therefore fluorine has less value for electron
Pale yellow yellowish green reddish brown deep violet affinity than chlorine.
6. Metallic Character - All the elements are non metals and 12. Solubility - Halogens are soluble in water which follow the
metallic character increases down the group. order
706 Chemistry
F2 > Cl2 > Br2 > I2 HF and HCl can be obtained by heating fluorides and
The solubility of iodine in water is enhanced in presence of chlorides with conc. H2SO4
KI CaF2 + H2SO4 ® CaSO4 + 2HF
2 NaCl + H2SO4 ® Na2SO4 + 2HCl
KI + I2 KI3 K+ + I3- HBr and HI are strong reducing agents in nature and reduce
In organic solvents like CS2, CHCl3 and CCl4 the Cl2, Br 2 conc. H2SO4 and cannot be prepared by this method.
and I2 are more soluble and give coloured solutions. Thus 2HBr (or 2HI) + H2SO4 ® Br2 (or I2) + 2H2O + SO2
Cl 2 gives yellow, Br 2 gives brown and I 2 gives violet Hence they are prepared by hydrolysis of phosphorous
solution. halides
13. Properties of halide ions (X-) - Some properties of halide PX3 + 3H2O ® H3PO3 + 3HX (X = Br, I)
ions follow the following order Properties - HF is a low boiling liquid due to intermolecular
F– > Cl– > Br– > I– hydrogen bonding, while HCl, HBr, HI are gases. The bpt follows
¬ Basic character the trend HF > HI > HBr > HCl
¬ Reducing character Some other properties observe the following trend :
¬ Heat of hydration HI > HBr > HCl > HF
14. Nature of bonds with other elements - Halogens with metals ¬ Acid Strength
of low ionisation potentials form ionic bonds. The ionic ¬ Reducing character
character follows the order ¬ Bond length
M - F > M - Cl > M - Br > M - I
Trend for following properties : HI < HBr < HCl < HF
The more the ionic character, the more is the mp and bp of
Thermal stability ®
halides .
Halogens with metals of high ionisation potentials Dipole moment ®
(transition metals) and non metals form covalent bond. Bond strength ®
Stability ®
COMPOUNDS OF HALOGENS :
Itching of glass : Glass contains silica which reacts with HF.
1. Hydracids (HX) - All halogens directly combine with
hydrogen to form HX known as hydracid. SiO 2 + 4HF ® SiF4 + 2H 2 O + 2HF ® H 2SiF6
H2 + X2 ® 2HX Hydro fluorosilicic acid

2. Oxy acids - All form oxy acids of the type HOX, HXO2, HXO3 and HXO4 as shown below
Name O. N. of X Fluorine Chlorine Bromine Iodine Salt’s name
Hypohalous acid, HXO +1 HOF HOCl HOBr HOI Hypohalite
Halous acid, HXO2 +3 - HClO2 - - Halite
Halic acid, HXO3 +5 - HClO3 HBrO3 HIO3 Halate
Perhalic acid, HXO4 +7 - HClO4 HBrO4 HIO4 Perhalate
Trend for following properties
HXO4 > HXO3 > HXO2 > HXO ˆˆ† 2X – + XO3–
3 – OX ‡ˆˆ
¬ Thermal stability Rate of disproportionation
¬ Acid strength
IO - > BrO - > ClO -
Oxidising nature ®
The conjugate bases obtained from above acids are The structure of HClO and HFO are
O O
XO – , XO2– , XO3– and XO4– . Their stability increases with
H F H Cl
the increase of oxygen atoms due to greater chances of 97° 103°
dispersal of negative charge. The more is the stability of Bleaching powder CaOCl2 is a mixed salt of HOCl and
conjugate base, the more is the acid character. HCl
(ii) Halic acids - HClO3 and HBrO3 exist in aqueous solution
XO4– > XO3– > XO2– > XO –
and HIO3 is a white solid. The stability follows the order
(i) Hypohalous acids (HOX) - They are formed in aqueous
HIO3 > HBrO3 > HClO3 ® Acid strength
solution by disproportionation of halogens
They are strong oxidising in nature
X2 + H2O HOX + HX
Acid character and thermal stability follows the order XO3– + 5X – + 6H + ® 3X 2 + 3H 2O
HClO > HBrO > HIO Their salts, NaClO3 is a powerful weed killer and KClO3
Hypohalites disproportionate in aqs. solution to halides (Berthelot’s salt) is used in fire works and matches as
and halates. oxidising agent.
The p-Block Elements - Halogens 707
(iii)Perhalic acids - Perhalates are obtained by electrolytic Examples IBr, BrCl,ClF3, BrF3, ICl3BrF5, IF5, IF7 ClF, BrF,
oxidation of halates ICl
4ClO 3– ® Cl – + 3ClO 4– , 4IO3– ® I – + 3IO 4- Properties - They are covalent, more reactive, strong
oxidising and diamagnetic in nature.
Perborate is obtained by oxidation with F2 in basic
Preparation - The are prepared by direct combination of
solution
halogens
BrO3– + F2 + 2OH – ® BrO4- + 2F – + H 2O
500K
Cl 2 + F2 ¾¾¾® 2ClF
They are strong oxidising agents following the order
By the action of halogens on lower interhalogen
ClO - < BrO - > IO -
4 4 4 ClF3 + F2 ® ClF5
Structure of oxyacids
Polyhalide ions - The ions containing more than two halogen
(a) Hypochlorous acid (b) Chlorous acid atoms are known as polyhalide ions eg.
(HOCl) (HClO2)
_
H H KI + I 2 ® KI3 K+ + I 3
O O
Other examples are
Cl Cl _
Br3– , C l3 , IC12– , IBr2– , ICl4– , BrF4– , I 5– , IF6–
.. ..
..

..

..

O Iodine Cation - Iodine also forms I+ and I3+ cations due to less
(c) Chloric acid (d) Perchloric acid ionisation energy as (CH3COO)3I, I (ClO4)3 and IPO4 have been
(HClO3) (HClO4) isolated.
H H Pseudohalogens and pseudohalides - The substances behaving
O O like halogens are known as pseudohalides.
Some examples are
Cl Cl Pseudo halogen Pseudohalide ion
..

O O O (CN)2 Cyanogen CN – Cyanide


O O
3. Oxides - All form oxides of different types. Halogens do (OCN)2 Oxycyanogen OCN – Cyanate
not combine directly with oxygen hence they are prepared
by indirect methods. They are (SCN)2 Thiocyanogen SCN – Thiocyanate
OF2 Cl2O Br2O I2O4
O2F2 Cl2O3 BrO2 I2O5 (SeCN)2 Selenocynogen SeCN – Selenocyanide
ClO2 BrO3
NCN – Cyanamide
Cl 2O 6
Cl2O7 N 3- Azide
Structure of some oxides
FLUORINE (SUPER HALOGEN) :
O Cl O O O
115º Discovered by Moisson 1886
Cl Cl O O Cl Cl O Occurrence - In combined state it occurs as
112° 118° O O 119º O
1. Fluorspar CaF2
All oxides are powerful oxidising agents and decompose
with explosion when heated or hit. 2. Cryolite Na3AlF6 and
4. Interhalogen compounds - Halogens combine among 3. Fluorapatite CaF2 .3Ca 3 (PO 4 ) 2 .
themselves to form compounds known as interhalogens or
interhalogen compounds of the type AX, AX3, AX5 and It is present in small amounts in soil, river water, bones and teeth
AX7. Their structure and hybridisation is as follows: of animals.
AX type sp3 AX3 sp3d AX5 sp3d2 AX7 sp3d3 Preparation
Linear T-shaped Square Pentagonal (1) Dennis method - By electrolysis of fused sodium or
pyramidal bipyramidal potassium hydrogen fluoride (dry) between graphite
X electrodes . (KHF2 is known as Fremy salt)
X
X
X
X KHF2 ® KF + HF HF ® H + + F-
X X
2H + + 2e – ® H 2 (At cathode)
A
A X A X
A-X X

X
X X X 2F – - 2e – ® F2 (At anode)
X
708 Chemistry
(2) Whytlaw Gray’s method - Electrolysis of fused KHF2 is (7) With NH 3 - 2 NH 3 + 3F2 ® N 2 + 6HF and some NF3
carried out in Cu cell (electrically heated) which serves
( not explosive)
the purpose of cathode, Anode is of graphite
(3) Modern method - Electrolysis of fused mixture of KF (8) With H2S - H 2S + 4F2 ® SF6 + 2HF
and HF is carried out in steel vessel (cathode). Anode is (9) With hydrocarbons - It reacts with hydrocarbons
of graphite violently eg CH4
Properties - It is pale greenish yellow gas can be condensed to CH 4 + 4F2 ® 4HF + CF4
pale yellow liquid and then pale yellow solid. It is very reactive.
Hence fluorination is carried out in presence of N2 (it
(1) Most active - It directly combines with metals and non dilutes F2) and calalyst copper gauge.
metals eg Al, Mg, C, P, S, As, Sb, Br2, I2 etc to from
Uses
fluorides, Cu and Hg form a protective coating of
(1) Fluoro derivatives are solvents, lubricants, refrigerants,
fluoride.
fire extinguishers, fungicides, germicides, dyes and
(2) Reaction with Xe - With Xenon it forms three definite plastics etc.
halides
(2) For separation of U235 by forming UF6 from natural
XeF2, XeF4 and XeF6 having the following structure uranium
F (3) Preparation of Teflon (C2F4)n
FREONS :
:

: Xe
:

XeF2 hybridisation sp3d Chlorofluoro compounds of methane and ethane are known as
freons. They are extremely, unreactive, non corrosive, easily
F
liquefiable compounds. Freon-12 (CCl2F2) is most common and
Linear symmetrical prepared as
D
3CCl 4 + 2SbF3 ¾¾® 3CCl 2 F2 + 2SbCl 3
F F They are used as refrigerants and propellants .
Magic acid - FSO3H SbF5 is strongest acid and known as magic
XeF4 hybridisation sp3d2 Xe acid.
Chlorine :
F F It was discovered by Scheele by heating HCl acid (muriatic acid)
with MnO2 and named it oxymuriatic gas. Davy established its
nature and called it chlorine.
Square planar Occurrence - Being very reactive does not occur free in nature. It
is widely distributed as chlorides. Common salt NaCl is most
F important chloride present in sea water and as rock salt.
F F Preparation - By oxidation of Conc HCl
XeF6 sp3d3 hybridisation Xe MnO2 + 4HCl ® MnCl 2 + 2H 2 O + Cl 2

F F 2KMnO 4 + 16HCl ® 2KCl + 2MnCl 2 + 8H 2 O + 5Cl 2

F K 2Cr2O7 + 14HCl ® 2KCl + 2CrCl3 + 2H 2O + 3Cl2

Distorted octahedral CaOCl 2 + 2HCl ® CaCl 2 + H 2 O + Cl 2


(3) With hydrogen even in dark - NaOCl + 2HCl ® NaCl + H 2 O + Cl 2
H 2 + F2 ® 2HF DH = -536 kJ PbO 2 + 4HCl ® PbCl 2 + 2H 2O + Cl 2
(4) With water - 2F2 + 2H 2O ® 4HF + O 2 Pb 3O 4 + 8HCl ® 3PbCl 2 + 4H 2 O + Cl 2
3F2 + 3H 2O ® 6HF + O 3 In place of HCl mixture of (NaCl + Conc.H2SO4) can be used
(5) Oxidising nature - 2NaCl + MnO 2 + 3H 2SO 4 ®
KClO3 + F2 + H 2O ® KClO 4 + H 2 F2 ® 2NaHSO 4 + MnSO 4 + 2H 2O + Cl 2
2KCl + F2 ® 2KF + Cl 2 Manufacture
(i) Weldon’s process - By heating pyrolusite with Conc
(6) With alkali - dil 2NaOH + 2F2 ® F2O + 2 NaF + H 2O HCl
Conc. 4 NaOH + 2F2 ® O 2 + 4NaF + 2H 2 O MnO2 + 4HCl ® MnCl 2 + 2H 2 O + Cl 2
The p-Block Elements - Halogens 709
(ii) Deacon’s process - In this process HCl is oxidised by Uses - It is used as a (i) bleaching agent (ii) disinfectant (iii) in the
O2 in presence of CuCl2 as catalyst at 400° C manufacture of CHCl 3, CCl 4, DDT anti knock compounds,
4HCl + O 2 ® 2Cl 2 + 2H 2 O bleaching powder, poisonous gas phosgene (COCl 2), tear gas
CCl3NO2 and mustard gas ClC2H4SC2H4Cl.
(iii)Electrolytic process - By the electrolysis of brine
Euchlorine - It is a mixture of chlorine and chlorine dioxide and
solution in Nelson cell
obtained by heating KClO3 with conc. HCl
NaCl Na+ + Cl–
2KClO3 + 4HCl ® 2KCl + Cl 2 + 2ClO 2 + 2H 2O
2Na + + 2e ® 2Na + 2H2O ® 2NaOH + H2 (at cathode) BROMINE :
It was discovered by Balard (1826)
2Cl – - 2e – ® 2Cl ® Cl 2 (at anode)
Occurrence - In nature it occurs in combined state only as
Pure Chlorine - By heating AuCl3 or PtCl4 in hard glass tube bromides of Na, K and Mg in sea water. Carnallite KCl.MgCl2.6H2O
448K 463K
2AuCl3 ¾¾¾® ëé 2AuCl + 2Cl2 ûù ¾¾¾® 2Au + 3Cl2 contains some bromo carnallite KBr.MgBr 2.6H2O in Germany
(Stass furt). In Chile and Mexico in silver mines as AgBr.
873K
PtCl 4 ¾¾¾® Pt + 2Cl 2 Preparation - Lab method -
(i) By heating mixture of potassium bromide maganese
Properties - It is yellowish green gas. Collected by upward
dioxide and conc H2SO4
displacement of air, poisonous in nature, soluble in water. It’s
aqueous solution is known as chlorine water which on careful 2KBr + 3H 2SO 4 + MnO 2 ®
cooling gives chlorine hydrate Cl 2.8H2O 2KHSO 4 + MnSO4 + 2H 2O + Br2
(1) Action of water - Cl 2 + H 2O ® HOCl + HCl (ii) By passing chlorine through solution of a bromide.
HOCl ® HCl + O 2KBr + Cl 2 ® 2KCl + Br2
Coloured matter + nascent O ® colourless matter Manufacture -
The bleaching action of chlorine is due to oxidation by (1) From carnallite - The mother liquor left after the
nascent [O]. crystallisation of chlorides from carnallite
U.V light (KCl.MgCl2.6H2O) contains bromides of Na, K and Mg
(2) Action of Hydrogen - H 2 + Cl 2 ¾¾¾ ¾¾® 2HCl and is known as bittern.
Charcoal catalyst
They combine explosively but in presence of charcoal 2KBr + Cl 2 ® KCl + Br2
catalyst combination is smooth at room temperature MgBr2 + Cl 2 ® MgCl 2 + Br2
(3) Displacement reactions -
Hot mother liquor
2KBr + Cl 2 ® 2KCl + Br2
2KI + Cl 2 ® 2KCl + I 2
Br2
(4) Action of NaOH Cold -
2 NaOH + Cl 2 ® NaCl + NaOCl + H 2O
Cl2 gas
Aqeous solution of NaOCl is called Javelle water. It is
used as bleaching agent.hot and Conc. Uncondensed vapour passed through tower packed
with moist iron fillings when these are absorbed and
6 NaOH + 3Cl 2 ® 5NaCl + NaClO3 + 3H 2O
yield ferrosoferric bromide.
(5) Action of H2S - H 2S + Cl 2 ® 2HCl + S 3Fe + 4Br2 ® Fe 3Br8
(6) Action of dry SO2 - SO 2 + Cl 2 ® SO 2 Cl 2 Alternative method - By electrolysis of above mother
liquor
(7) Action of CO - CO + Cl 2 ® COCl 2
(8) Oxidising properties - In aqueous solution Cl2 acts as MgBr2 ® Mg ++ + ( 2Br ) ® Br2 + 2e –
oxidising agent Even if some MgCl2 is decomposed, the Cl2 evolved
SO 2 + Cl 2 + 2H 2O ® H 2SO 4 + 2HCl reacts with MgBr2 to liberate Br2
(2) From sea water - The sea water is concentrated when
2FeSO 4 + Cl 2 + H 2SO 4 ® Fe 2 (SO 4 )3 + 2HCl salts separate as crystals. The mother liquor contains
Na 2S 2O 3 + Cl 2 + H 2 O ® Na 2SO 4 + 2HCl + S MgBr2 and is treated as above to get bromine.
In America sea water is acidified with 0.1%H2SO4 and
(9) Reaction with ammonia -
chlorine is passed through it. The vapours liberated is
(i) When ammonia is in excess
passed in Na2CO3 solution.
8NH 3 + 3Cl 2 ® N 2 + 6 NH 4Cl MgBr2 + Cl2 ® MgCl2 + Br2
(ii) When chlorine is in excess 3Br2 + 3Na 2 CO 3 ® 5 NaBr + NaBrO 3 + 3CO 2
NH 3 + 3Cl 2 ® NCl3 + 3HCl
5 NaBr + NaBrO3 + 6HCl ® 6 NaCl + 3H 2 O + 3Br2
710 Chemistry
Properties - Bromine is a heavy dark brown liquid gives irritating Manufacture -
vapour. Density 3.2g/ml bpt 58.5ºC and fpt- 7ºC. It is soluble in (i) From sea weeds
water and gives bromine water about 3.6% at 20º C. D
Saturated solution of bromine on cooling gives bromine hydrate 2 NaI + MnO 2 + 3H 2SO 4 ¾¾®
Br2.8H2O.
Chemical properties - ® 2 NaHSO 4 + MnSO 4 + 2H 2 O + I 2
(i) Combination with hydrogen - (ii) From Caliche -
Pt at room temp . 2 NaIO3 + 5NaHSO 3 ®
H 2 + Br2 ¾¾¾ ¾¾¾® 2HBr
or at 200º C
® 2 Na 2SO 4 + 3NaHSO 4 + H 2O + I 2
(ii) Oxidising nature - Under ordinary conditions it does Exact amount of NaHSO3is to be added since it reacts
not react with water but in presence of an oxidisable with I2, if present in excess.
substance it gives HBr.
NaHSO 3 + I 2 + H 2O ® NaHSO 4 + 2HI
Br2 + H 2O ® 2HBr + O
Purification - It contains the impurities of Cl2, Br 2 and H2O.
Thus it oxidises SO2 to H2SO4
Cl2 and Br 2 are removed by distillation with KI.
SO 2 + H 2 + O ® H 2SO 4 2KI + Cl 2 ® 2KCl + I 2
Sodium sulphite to sulphate
2KI + Br2 ® 2KBr + I 2
Na 2SO 3 + O ® Na 2SO 4
Water is removed by distillation over concentrated H2SO4. Further
Thiosulphite to sulphate
it is purified by sublimation.
Na 2S2 O 3 + O ® Na 2SO 4 + S Properties - It is black (dark violet) shining solid with metallic
Arsenite to arsenate lustre. It sublimes on heating (mpt 114ºC). It is metallic in character
Na 3AsO3 + O ® Na 3AsO 4 and forms I+ and I3+ ions.
Hydrogen sulphide to sulphur Chemical properties -
(i) Solubility - It is slightly soluble in water but dissolves
H 2S + O ® H 2 O + S
in presence of KI
(iii)Reaction with alkali
KI + I2 KI3
(a) In cold 2NaOH + Br2 ® NaBr + NaBrO + H 2O
(ii) Combination with elements -
(b) In hot 6 NaOH + 3Br2 ® 5NaBr + NaBrO3 + 3H 2O Pt
H 2 + I 2 ¾¾® 2 HI
(iv) Displacement reaction - 2KI + Br2 ® 2KBr + I 2 D

(v) Action of ammonia - 8NH 3 + 3Br2 ® 6 NH 4 Br + N 2 D


2P + 3I 2 ¾¾® 2PI 3
(vi) Action of organic Compounds -
(iii)Displacement reactions -
C 2 H 4 + Br2 ® C 2 H 4 Br2
2KClO3 + I 2 ® 2KIO3 + Cl 2
C 2 H 6 + Br2 ® C 2 H 5 Br + HBr
2KBrO3 + I 2 ® 2KIO3 + Br2
(vii) Br2 + moist starch paper = brown colour or Br2 + moist (iv) Reaction with alkalies
starch ® KI paper = violet (a) dilute and cold NaOH
Uses- 2 NaOH + I 2 ® NaI + NaOI + H 2 O
(i) In the manufacture of tetraethyl lead an important
antiknock compound (b) Concentrated and hot NaOH
4C2 H 5Br + 4Na.Pb ® (C 2 H5 ) 4 Pb + 4NaBr + Pb 6 NaOH + I 2 ® 5NaI + NaIO3 + 3H 2O
sodium-lead alloy
Hypoiodite decomposes even at room temperature.
(ii) NaBr and KBr are used as sedatives
(iii) AgBr is used in photography 3NaOI ® 2 NaI + NaIO3
(iv) Ethylene bromide increases efficiency of TEL. (v) Action of ammonia - 2NH 3 + 3I 2 ® NH 3 .NI3 + 3HI
(v) NaBr and KBr are used as sedatives.
Explosive compound
IODINE :
8NI3 .NH 3 ® 5N 2 + 9I 2 + 6 NH 4 I
Iodine was discovered by Courtios.
Occurrence - It is not found in free state. It occurs in sea weeds (vi) Reaction with hypo -
as NaI. In Caliche (Chile saltpeter) as NaIO3. 2Na 2S2O3 + I2 ® Na 2S4O6 + 2NaI
Preparation - Lab method : By heating of a mixture of MnO2, (vii) Action of strong oxidising agents, eg conc HNO3 , O3
H2SO4 and KI, and Cl2 They produce iodic acid.
2KI + 3H 2 SO 4 + MnO 2 ® 2KHSO 4 + MnSO 4 + 2H 2 O + I 2 I 2 + 10HNO 3 ® 2HIO 3 + 10NO 2 + 4H 2O
The p-Block Elements - Halogens 711
Uses Structure
(i) In medicines eg as tincture of iodine, iodex, iodoform
(ii) Solution of KI and I2 used in the treatment of goitre. Cl
Tincture of iodine - It is mixture of (i) Due to Odling Ca
1 1
O z . Iodine + O z . KI + 1 pint rectified spirit. OCl
2 4
BLEACHING POWDER - CaOCl2.H2O : (ii) According to Bunn, Clark and Clifford, it is a mixture of
The composition of bleaching powder is calcium hypochlorite Ca (OCl )2 . 4H 2 O and basic calcium
Ca(OCl)2 .CaCl2 .Ca(OH) 2 .2H 2 O
chloride CaCl 2Ca (OH )2 .2H 2O .
Manufacture - It is manufactured by the action of chlorine on
slaked lime. Ca(OH)2 ABNORMAL PROPERTIES OF HYDROFLUORIC
Ca (OH) 2 + Cl 2 ® CaOCl 2 + H 2O ACID:
The manufacture is carried out in Hosenclever plant or 1. It is highly poisonous and has corrosive action on skin
Bachmann’s plant - (Modern process) 2. It is exist as H2F2 even in gaseous state and forms two
Properties - It is yellowish powder with strong smell of chlorine. series of salts KHF2 (FREMY’S SALT) and K2F2.
1. By the action of dilute acids or carbon dioxide it loses its
3. It is not oxidised even by strong oxidising agents.
chlorine
4. It reacts with silica and glass
CaOCl 2 + CO 2 ® CaCO3 + Cl 2
SiO 2 + 2H 2 F2 ® SiF4 + 2H 2O
CaOCl 2 + H 2SO 4 ® CaSO 4 + H 2O + Cl 2
The chlorine thus obtained as known as “available SiF4 + H 2 F2 ® H 2 SiF6
chlorine” A good sample of bleaching powder contains 35- Hydrofluorosili cic acid
40% available chlorine the value of which goes on
decreasing on keeping the powder due to the following Na 2SiO 3 + 3H 2 F2 ® Na 2SiF6 + 3H 2 O
change
CaSiO3 + 3H 2 F2 ® CaSiF6 + 3H 2O
6CaOCl 2 ® Ca (ClO 3 )2 + 5CaCl 2
Hence it is used for etching glass
2. It give O2 in presence of cobalt chloride solution
5. On heating with MnO2 and H2SO4 it does not give F2
CoCl
2CaOCl 2 ¾¾ ¾
¾2 ® 2CaCl + O
2 2 while other hydrogen halides (HX) give X2
3. In presence of slight amount of dilute acid it loses oxygen.
MnO2 + H 2SO 4 ® MnSO 4 + H 2O + O
2CaOCl 2 + H 2SO 4 ® CaCl 2 + CaSO 4 + 2HClO
2HX + O ® H 2 O + X 2
HClO ® HCl + O
The evolution of nascent oxygen makes it oxidising and 6. AgF and PbF2 are soluble in water while chlorides, bromides
bleaching agent. and iodides of silver (Ag) and lead (Pb) are insoluble in
4. It reacts with ethyl alcohol or acetone to form chloroform water.
CaOCl 7. CaF2 and SrF2 are insoluble in water while chlorides,
CH 3 CH 2 OH or CH 3COCH 3 ¾¾ ¾¾
2 ® CHCl
3
distill bromides and iodides af Ca and Sr are soluble.
5. It does not form clear solution with water and aqueous 8. Azeotropes of hydracids

2+
_ _ Hydrogen halide Composition Boiling
solution contains Ca , C l and OC l ions. point ºC
Uses
H2F2 36% 120
(i) It is used as disinfectant and germicide
(ii) For the manufacture chloroform HCl 20.4% 110
(iii) For the sterlisation of drinking water HBr 47.0% 126
(iv) For making unshrinkable wool
HI 57.0% 127
(v) For bleaching cotton, wood pulp.
712 Chemistry

Very Short/Short Answer Questions 16. The order of reactivity of halogens is


(a) F > Cl > Br > I (b) F < Cl < Br < I
1. Why is HF the weakest acids among hydrohaloacids inspite (c) F < Cl > Br < I (d) F < Cl < Br > I
of the fact that F is most electronegative?
17. Which of the following statements is correct ?
2. Why are the interhalogens compounds more reactive than
(a) only iodine forms oxy acid
the halogens (except F2)?
(b) only chlorine and bromine form oxy acid
3. Show that Cl2 gas can be obtained from bleaching powder.
(c) All the halogens form oxy acid
4. Why is fluorine most reactive among halogens? (d) All the halogens form oxy acids except fluorine
5. Why is HF stored in wax-coated glass bottles? 18. Of the four elements, the one having maximum electron
6. Name a halogen which does not exhibit positive oxidation affinity is :
state. (a) Fluorine (b) Chlorine
7. Arrange the following in the decreasing order of property (c) Bromine (d) Iodine
indicated. 19. Which one of the following orders is not in accordance
(i) F2, Cl2, Br, I2 – Bond energy with the property stated against is ?
(ii) HF, HCl, HBr, HI – Acid strength (in water) (a) HI > HBr > HCl > HF : Acidic property in water
(iii) M – F, M – Cl, M – Br, M – I – Ionic character of the (b) F2 > Cl2 > Br2 > I2 : Electronegativity
bond. (c) F2 > Cl2 > Br2 > I2 : Bond dissociation energy
(iv) ClO4–, BrO4–, IO4– – Oxidizing power (d) F2 > Cl2 > Br2 > I2 : Oxidising power
8. Compare the acidic strengths of HF and HCl & explain the 20. Which of the following species has four lone pairs of
difference. electrons?
9. Which of the following is regarded as the weakest acid and (a) I (b) O -
why? (c) Cl - (d) He
HF, HCl, HBr or HI. 21. Which one is the correct order of the size of iodine species?
10. HF has higher boiling point than HCl. Why? (a) I > I+ > I– (b) I > I– > I+
11. Iodine forms I3– but F2 does not form F3- ions. Why? (c) I+ > I– > I (d) I– > I > I+
22. Cl2 reacts with hot and conc. NaOH to give –
12. Why is perchloric acid, HClO4 a stronger acid than sulphuric
acid? (a) NaClO (b) NaClO3
(c) NaClO2 (d) NaClO4
13. Fluorine does not undergo disproportionation reactions but
other halogens do. Explain why? 23. Which one is most stable to heat –
(a) HClO (b) HClO2
Long Answer Questions (c) HClO3 (d) HClO4
14. Arrange the following in order of property indicated for each 24. Conc. HNO3 reacts with I2 to form :
set: (a) HI (b) HOI
(i) F2, Cl2, Br2, I2 – increasing bond dissociation enthalpy. (c) HIO2 (d) HIO3
(ii) NH3 , PH3, AsH3, SbH3, BiH3 – increasing base strength 25. Which of the following has maximum bond energy?
(iii) HClO4, HClO3, HClO2, HClO – increasing acidic strength (a) Cl2 (b) F2
(iv) HOCl, HOBr, HOI – increasing acidic strength. (c) Br2 (d) I2
26. Iodine is a :
Multiple Choice Questions (a) electrovalent solid (b) atomic solid
15. The correct order of acidic strength is (c) molecular solid (d) covalent solid
(a) HF < HCl < HBr < HI 27. Fluorine exhibits an oxidation state of only –1 because
(a) it can readily accept an electron
(b) HBr < HCl < HI < HF
(b) it is very strongly electronegative
(c) HCl < HBr < HF < HI
(c) it is a non-metal
(d) HI < HBr < HCl < HF
(d) it belongs to halogen family
The p-Block Elements - Halogens 713

1. Among F, Cl, Br and I the lowest ionization potential will be 15. Hydrochloric acid at 25ºC is
of (a) ionic and liquid (b) covalent and liquid
(a) fluorine (b) chlorine (c) ionic and gas (d) None of these
(c) bromine (d) iodine 16. Which of the following is most volatile ?
2. The electron affinity of the halogens follows the order (a) HI (b) HBr
(a) F > Cl > Br > I (b) F < Cl < Br < I (c) HCl (d) HF
(c) F < Cl > Br > I (d) F < Cl < Br > I 17. Which of the following sets gives the correct arrangement of
3. The electronegativity follows the order the compounds involved based on their bond strengths ?
(a) F > O > Cl > Br (b) F > Cl > Br > O (a) HF > HCl > HBr > HI (b) HI > HBr > HCl > HF
(c) O > F > Cl > Br (d) Cl > F > O > Br (c) HF > HBr > HCl > HI (d) HCl > HF > HBr > HI
4. The bond energies of F2 , Cl2, Br2 and I2 are 155, 244, 193 and
18. At room temperature, HCl is a gas while HF is a low boiling
151 kJ mol–1 respectively. The weakest bond will be in
liquid. This is because
(a) Br2 (b) Cl2
(a) H- F bond is covalent (b) H- F bond is ionic
(c) F2 (d) I2
(c) HF has metallic bond (d) HF has hydrogen bond
5. The correct order of reactivity of halogens with alkalies is
(a) F > Cl > Br > I (b) F < Cl > Br < I 19. Which of the following has the highest bond strength?
(c) F < Cl < Br < I (d) F < Cl < Br > I (a) HI (b) HCl
6. The correct order of increasing oxidising power is (c) HF (d) HBr
(a) F2 > Br2 > Cl2 > I2 20. Which of the following is the strongest acid ?
(b) F2 < Cl2 < Br2 < I2 (a) HOCl (b) HOClO2
(c) Cl2> Br2 > F2 > I2 (c) HOClO3 (d) HOClO
(d) I2 < Br2 < Cl2 < F2 21. The geometry of ClO3- according to valence shell electron
7. Which of the following halogen does not exhibit positive
oxidation state in its compounds? pair repulsion (VSEPR) theory will be
(a) Cl (b) Br (a) planar triangle (b) pyramidal
(c) I (d) F (c) tetrahedral (d) square planar
8. The halogen that is most easily reduced is 22. Oxidation state of chlorine in hypochlorous acid is
(a) F2 (b) Cl2 (a) + 1 (b) + 2
(c) Br2 (d) I2 (c) – 1 (d) – 2
9. The bond energies of F2 , Cl2, Br2 and I2 are 37, 58, 46 and 36 23. The correct decreasing order of acidic character is
kcal mol–1 respectively. The strongest bond is present in (a) HClO > HBrO > HIO (b) HIO > HBrO > HClO
(a) Br2 (b) I2 (c) HBrO > HIO > HClO (d) HClO > HIO > HBrO
(c) Cl2 (d) F2 24. The element which never acts as reducing agent in a
10. Which one of the following order is correct for the bond chemical reaction is
energies of halogen molecules ? (a) O (b) Li
(a) I2 > Cl2 > Br2 (b) Br2 > Cl2 > I2 (c) F (d) C
(c) I2 > Br2 > Cl2 (d) Cl2 > Br2 > I2 25. Unlike other halogens, fluorine does not show higher
11. Which one of the following elements shows different oxidation states because
oxidation states ? (a) it is highly electronegative
(a) Sodium (b) Fluorine (b) it has no d- orbitals
(c) Chlorine (d) Potassium
(c) its atomic radius is very small
12. Which of the following halogens exhibit only one oxidation
(d) the F– ion is stable and isoelectronic with neon
state in its compounds ?
26. An element M has an atomic mass 19 and atomic number 9.
(a) Bromine (b) Chlorine
Its ion is represented by
(c) Fluorine (d) Iodine
13. Which has the highest bond energy ? (a) M+ (b) M2 +
(c) M – (d) M2 –
(a) F2 (b) Cl2
(c) Br2 (d) I2 27. Which one of the following halogen liberates oxygen when
14. Which is the weakest out of HF, HCl, HBr and HI? passed through hot concentrated KOH solution ?
(a) HF (b) HCl (a) I2 (b) Cl2
(c) HBr (d) HI (c) Br2 (d) F2
714 Chemistry
28. One mole of fluorine is reacted with two moles of hot and 40. The reaction of KMnO4 and HCl results in
concentrated KOH. The products formed are KF, H2O and (a) oxidation of Mn in KMnO4 and production of Cl2
O2. The molar ratio of KF, H2O and O2 respectively is (b) reduction of Mn in KMnO4 and production of H2
(a) 1 : 1 : 2 (b) 2 : 1 : 0.5 (c) oxidation of Mn in KMnO4 and production of H2
(c) 1 : 2 : 1 (d) 2 : 1 : 2 (d) reduction of Mn in KMnO4 and production of Cl2
41. Chlorine acts as a bleaching agent only in
29. Fluorine oxidizes HSO -4 to (a) dry air (b) sunlight
(c) moisture (d) oxygen
(a) S2 O32 - (b) S2 O82 -
42. In the reaction
(c) S4 O 26 - (d) SO 2 - -
3Br2 + 6CO 32 - + 3H 2 O ¾
¾® 5Br + BrO 3 + 6HCO 3
30. Chlorine cannot displace (a) bromine is oxidised and carbonate is reduced
(a) Fluorine from NaF (b) bromine is both oxidised and reduced
(b) Iodine from NaI (c) bromine is reduced and water is oxidised
(c) Bromine from NaBr (d) bromine is neither oxidised nor reduced
(d) None of these 43. The solubility of I2 increases in water in presence of
31. Cl2 gas is dried over (a) KI (b) H2SO4
(a) CaO (b) NaOH (c) KMnO4 (d) H2S
(c) KOH (d) conc. H2SO4 44. Oxidation of thiosulphate with iodine gives
32. The outer electronic structure of 3s2 3p5 is possessed by (a) sulphate ion (b) sulphite ion
(a) O (b) Cl (c) tetrathionate ion (d) sulphide ion
(c) Br (d) Ar 45. Which of the following cann ot work as oxidizing
agent ?
33. Chlorine acts as a bleaching agent only in the presence of
(a) O2 (b) KMnO4
(a) dry air (b) moisture
(c) H2O2 (d) I–
(c) sunlight (d) None of these 46. On boiling an aqueous solution of KClO3 with I2 the products
34. When chlorine reacts with cold and dilute solution of sodium obtained are
hydroxide, the products obtained are (a) KIO3+Cl2 (b) KCl+I2O5
(a) Cl– + ClO– (b) Cl– + ClO -2 (c) KIO4+Cl2 (d) No reaction takes place
47. When KI and CuSO4 solutions are mixed, it gives
(c) Cl– + ClO3- (d) Cl– + ClO -4 (a) K2SO4 + I2 (b) Cu2I2 + K2SO4
(c) CuI2 + K2SO4 (d) Na2SO4 + I2
35. In the reaction
48. When I2 is passed through KCl, KF and KBr solutions
2Br + X2 ® Br2 ,2X- ,X2 is (a) Cl2 and Br 2 are evolved
(a) Cl2 (b) Br2 (b) Cl2 is evolved
(c) I2 (d) N2 (c) Cl2, Br 2 and F2 are evolved
36. Chlorine is liberated when we heat (d) None of these
(a) KMnO4 + NaCl (b) K2Cr2O7 + MnO2 49. Sea weeds are an important source of
(c) Pb(NO3)2 + MnO2 (d) K2Cr2O7 + HCl (a) iodine (b) bromine
37. Which of the following is correct about the reaction? (c) chlorine (d) Both (a) and (b)
50. Which of the following elements exhibits the most basic
heat
3NaClO ¾¾¾ ® NaClO3 + 2NaCl properties ?
(a) It is disproportionation reaction (a) F (b) Cl
(b) Oxidation number of Cl decreases as well as increases in (c) Br (d) I
this reaction 51. Which of the following is used as an antiseptic?
(c) This reaction is used for the manufacture of halates (a) I (b) Br
(d) All of these (c) Cl (d) F
52. In the presence of cobalt chloride (CoCl2), bleaching powder
38. Which of the following is used in the preparation of chlorine ?
decomposes to form
(a) Only MnO2 (b) Only KMnO4
(a) CaCO3 and O3 (b) ClO2 and CaO
(c) Both MnO2 and KMnO4 (d) Either MnO2 or KMnO4 (c) Cl2O and CaO (d) CaCl2 and O2
39. A greenish yellow gas reacts with an alkali metal hydroxide 53. In the reaction
to form a halate which can be used in fire works safety
matches. The gas and halate respectively are ® H 2+ NO 3 + F - base is
HNO 3 + HF ¾¾
(a) Br2 KBrO3 (b) Cl2, KClO3 (a) HF (b) HNO3
(c) I2, NaIO3 (d) Cl2, NaClO3 (c) HF and HNO3 (d) None of these
The p-Block Elements - Halogens 715
54. The aqueous solution of which of the following has 58. The hybridization in ICl7 is
maximum pH ? (a) sp3d 3 (b) d2sp 3
(a) NaClO (b) NaClO2 (c) sp3d (d) sp3
(c) NaClO3 (d) NaClO4 59. Which of the following bond is strongest ?
55. Which of the following is not the characteristic of
(a) F – B (b) F – Cl
interhalogen compounds ?
(a) They are more reactive than halogens (c) F – Br (d) Cl –Br
(b) They are quite unstable but none of them is explosive 60. “Fluorosis” disease is caused due to the reaction of .............
(c) They are covalent in nature with excess of fluoride in the body.
(d) They have low boiling points and are highly volatile. (a) Ca (b) Mg
56. Which bond is most polar ? (c) Fe (d) K
(a) Cl - F (b) Br - F 61. Hydrogen bond is strongest in
(c) I - F (d) F - F (a) F – H...........O
57. On heating KClO3 we get (b) F – H...........N
(a) KClO2 + O2 (b) KCl + O2 (c) F – H...........F
(c) KCl + O3 (d) KCl + O2 + O3
(d) All are equally strong

1. Which among the following is paramagnetic ? 7. Which is the best description of the behaviour of bromine in
(a) Cl 2 O (b) ClO2 [CBSE-PMT 1994] the reaction given below ? [ CBSE-PMT2004]
(c) Cl 2O 7 (d) Cl 2O 6 H 2 O + Br2 ® HOBr + HBr
2. Which one of the following oxides of chlorine is obtained by (a) Proton acceptor only
passing dry chlorine over silver chlorate at 90°C ? (b) Both oxidized and reduced
[CBSE-PMT 1994] (c) Oxidized only
(d) Reduced only
(a) Cl2 O (b) ClO3
8. Which one of the following orders correctly represents the
(c) ClO2 (d) ClO4 increasing acid strengths of the given acids?
3. The formula for calcium chlorite is [CBSE-PMT 1994] [CBSE-PMT 2007]
(a) Ca(ClO )
4 2 (b) Ca(ClO )
3 2
(a) HOClO < HOCl < HOClO3 < HOClO2
(b) HOClO2 < HOClO3 < HOClO < HOCl
(c) CaClO 2 (d) Ca(ClO 2 ) 2
(c) HOClO3 < HOClO2 < HOClO < HOCl
4. Regarding F– and Cl– which of the following statements is/
(d) HOCl < HOClO < HOClO2 < HOClO3
are correct? [ CBSE-PMT1996]
(i) Cl– can give up an electron more easily than F– 9. Which one of the following arrangements does not give the
(ii) Cl– is a better reducing agent than F– correct picture of the trends indicated against it ?
(iii) Cl– is smaller in size than F– [CBSE-PMT 2008]
(iv) F– can be oxidized more readily than Cl– (a) F2 > Cl2 > Br2 > I2 : Oxidizing power
(a) (i) and (ii) (b) (i), (ii) and (iv) (b) F2 > Cl2 > Br2 > I2 : Electron gain enthalpy
(c) (iii) and (iv) (d) only (i) (c) F2 > Cl2 > Br2 > I2 : Bond dissociation energy
5. Which one is the correct order of the size of iodine species? (d) F2 > Cl2 > Br2 > I2 : Electronegativity.
[CBSE-PMT 1997] 10. In the case of alkali metals, the covalent character decreases
(a) I > I+ > I– (b) I > I– > I+ in the order: [CBSE-PMT 2009]
(c) I+ > I– > I (d) I– > I > I+
(a) MF > MCl > MBr > MI
6. Which of the following statements is not true ?
[CBSE-PMT 2003] (b) MF > MCl > MI > MBr
(a) HF is a stronger acid than HCl (c) MI > MBr > MCl > MF
(b) Among halide ions, iodide is the most powerful reducing (d) MCl > MI > MBr > MF
agent 11. Among the following which is the strongest oxidising agent?
(c) Fluorine is the only halogen that does not show a variable [CBSE-PMT 2009]
oxidation state (a) Br2 (b) I2
(d) HOCl is a stronger acid than HOBr (c) Cl2 (d) F2
716 Chemistry
12. The correct order of increasing bond angles in the following 16. Which among the following factors is the most important in
species are : [CBSE-PMT 2010] making fluorine the strongest oxidizing halogen ?
(a) Hydration enthalpy [AIEEE 2004]
(a) Cl 2 O < ClO 2 < ClO 2– (b) ClO2 < Cl 2O < ClO 2–
(b) Ionization enthalpy
(c) Cl 2O < ClO 2– < ClO2 (d) ClO 2– < Cl2 O < ClO2 (c) Electron affinity
13. Which is the strongest acid in the following : [NEET 2013] (d) Bond dissociation energy
(a) HClO3 (b) HClO4 17. Among the properties (a) reducing (b) oxidising (c)
(c) H2SO3 (d) H2SO4 complexing, the set of properties shown by CN– ion towards
14. Which of the following is a polar molecule ? [NEET 2013] metal species is [AIEEE 2004]
(a) SF4 (b) SiF4 (a) c, a (b) b, c
(c) XeF4 (d) BF3 (c) a, b (d) a, b, c
15. Concentrated hydrochloric acid when kept in open air 18. The correct order of the thermal stability of hydrogen halides
sometimes produces a cloud of white fumes. The explanation (H–X) is [AIEEE 2005]
for it is that [AIEEE 2003] (a) HI > HCl < HF > HBr
(a) oxygen in air reacts with the emitted HCl gas to form a (b) HCl< HF > HBr < HI
cloud of chlorine gas (c) HF > HCl < HBr > HI
(b) strong affinity of HCl gas for moisture in air results in (d) HI < HBr > HCl < HF
forming of droplets of liquid solution which appears like a 19. Identify the incorrect statement among the following.
cloudy smoke. [AIEEE 2007]
(c) due to strong affinity for water, concentrated (a) Br2 reacts with hot and strong NaOH solution to give
hydrochloric acid pulls moisture of air towards itself. NaBr and H2O.
This moisture forms droplets of water and hence the
(b) Ozone reacts with SO2 to give SO3.
cloud.
(c) Silicon reacts with NaOH(aq) in the presence of air to
(d) concentrated hydrochloric acid emits strongly smelling
give Na2SiO3 and H2O.
HCl gas all the time.
(d) Cl2 reacts with excess of NH3 to give N2 and HCl.

1. Which of the following species is not a pseudo halide 5. Antichlor is a compound


(a) CNO– (b) RCOO– (a) which absorbs chlorine
(b) which removes Cl2 from a material
(c) OCN– (d) NNN–
(c) which liberates Cl2 from bleaching powder
2. HBr and HI reduce sulphuric acid ; HCl can reduce KMnO4
(d) which acts as a catalyst in the manufacture of Cl2
and HF reduces 6. Which reaction cannot be used for the preparation of the
(a) H 2SO 4 (b) KMnO4 halogen acid
(c) K 2Cr2 O 7 (c) None of these (a) 2KBr + H 2SO 4 (conc.) ® K 2SO 4 + 2HBr
3. Gaseous HCl is a poor conductor of electricity while its (b) NaCl + H 2SO 4 (conc.) ® NaHSO 4 + HCl
aqueous solution is a good conductor this is because (c) NaHSO 4 + NaCl ® Na 2SO 4 + HCl
(a) H 2 O is a good conductor of electricity (d) CaF2 + H 2SO 4 (conc.) ® CaSO 4 + 2HF
(b) a gas cannot conduct electricity but a liquid can 7. When an aqueous solution of hypochlorites is heated
(c) HCl gas does not obey Ohm’s law, whereas the solution (a) chlorine is evolved
does (b) chlorine is formed
(d) HCl ionises in aqueous solution (c) chlorate is formed
4. When NaCl or KCl is heated with conc. H2SO4 and s o l i d (d) chlorine peroxide is formed
K2Cr2O7 ,we get 8. Which of the following oxyacids of chlorine is formed on
(a) chromic chloride shaking chlorine water with freshly precipitated yellow oxide
(b) chromous chloride of mercury
(c) chromyl chloride CrO 2 Cl 2 (a) HClO 3 (b) HClO 2
(d) chromic sulphate (c) HClO (d) HClO 4
The p-Block Elements - Halogens 717
9. By the action of concentrated HCl on potassium chlorate we 20. When chlorine water is exposed to sunlight, O2 is liberated.
get this mixture of gases Hence
(a) CO 2 + Cl 2 (b) O 2 + ClO 2 (a) hydrogen has little affinity to O2
(b) hydrogen has more affinity to O2
(c) Cl 2 + ClO 2 (d) O 2 + Cl 2 + ClO 2 (c) hydrogen has more affinity to Cl2
10. A gas reacts with CaO, but not with NaHCO3 The gas is (d) hydrogen has little affinity to Cl2
(a) CO2 (b) Cl2 21. The correct order of acidic strength is
(d) N2 (d) O2 (a) Cl2O7 > SO2 > P4O10
11. Bleaching powder slowly loses its activity when it stands in (b) CO2 > N2O5 > SO3
air .This is due to (c) Na2O > MgO > Al2O3
(a) reaction with moisture to liberate O2 (d) K2O > CaO > MgO
22. Bleaching powder on standing forms mixture of :
(b) auto-oxidation
(c) loss of CaCl2 (a) CaO + Cl 2 (b) CaO + CaCl 2
(d) formation of Ca(OH)2 (c) HOCl + Cl 2 (d) CaCl 2 + Ca (ClO3 ) 2
12. The greater reactivity of fluorine is due to
23. When Cl 2 water is added to an aqueous solution of
(a) low energy of the F - F bond
(b) small size potassium halide in presence of CCl4 a violet colour is
(c) high heat of Hydration obtained. On adding more of Cl 2 water, the violet colour
(d) All of these disappears and a colourless solution is obtained. This test
13. SO2 reacts with chlorine to form confirms presence of which of the following in aqueous
solution :
(a) sulphur monochloride (b) sulphur dichloride
(a) Bromide (b) Chloride
(c) sulphuryl chloride (d) sulphur trichloride
(c) Iodide and bromide (d) Iodide
14. HF present as impurity in gaseous F2 can be removed by
24. In a given sample of bleaching powder the percentage of
passing over
available chlorine is 49. The volume of chlorine obtained if
(a) P2O5 (b) NaF 10 g of the sample is treated with HCl at NTP is :
(c) H2SO4 (d) CaCl2 (a) 15 litre (b) 1.5 litre
15. Which of the following statement is incorrect ? (c) 3 litre (d) 150 litre
(a) Chlorine can bleach a wet piece of cloth 25. Most of the elementary gases are obtained by chemical
(b) Iodine stain can be removed by hypo solution reaction of their compounds. For example, chlorine is obtained
(c) Bromine can be prepared from carnallite by allowing KMnO4 to react with HCl. Fluorine, however,,
(d) Bromine is liberated when iodine is passed through an can be obtained only by the electrolysis of a fluorides. This
acidified KBr solution is because :
16. The ion that cannot undergo disproportionation is (a) it is easy to electrolyse a fluoride
(a) ClO4– (b) ClO3– (b) fluorine is highly poisonous
(c) ClO2– (d) ClO– (c) fluorine is strongest chemical oxidising agent
17. Which acid can combine with its own salt again (d) fluorine is a highly reactive gas
26. Which reaction yields greatest quantity of chlorine from
(a) HF (b) HBr
given quantity of HCl :
(c) HCl (d) HI
18. If Cl2 gas is passed into aqueous solution of KI containing (a) Warming conc. HCl with MnO2
some CCl4 and the mixture is shaken then (b) Warming conc. HCl with PbO 2
(a) upper layer becomes violet
(c) Mixing conc. HCl with KMnO4
(b) lower layer becomes violet
(c) homogenous violet layer is formed (d) Treating bleaching powder with HCl
(d) None of these 27. Fluorine is a stronger oxidising agent than chlorine in
aqueous solution. This is attributed to many factors except
19. Interhalogen compounds are more reactive than the
(a) heat of dissociation (b) ionisation potential
individual halogen because
(c) heat of hydration (d) electron affinity
(a) two halogens are present in place of one
(b) they are more ionic 28. Which pair gives Cl 2 at room temperature :
(c) their bond energy is less than the bond energy of the (a) NaCl + Conc. H 2SO 4 (b) Conc. HCl + KMnO4
halogen molecule
(c) NaCl + Conc. HNO 3 (d) NaCl + MnO2
(d) they carry more energy
718 Chemistry

EXERCISE 1 21. (b) Hybridisation is sp3 and shape pyramidal


.. – .. –
1. Due to widespread H-bonding in HF, H+ is not easily formed. Cl Cl
Due to small bond length of H – F, bond dissociation energy :
:O: :O: or :O: O
:
is high. :O: :O:

:
:

:
2. This is because A – X bond in interhalogens in weaker than

:
the X – X bond in halogens. 22. (a) In HClO the O.S of Cl is + ve (+1)
3. CaOCl2 + H2SO4 ¾¾ ® CaSO4 + H2O + Cl2. 23. (a) Since the electronegativity of halogens follow the order
4. It is most electronegative and is smallest in its atomic size, Cl > Br > I the strength of acids follow the order HClO >
and has relatively less bond energy. HBr > HIO
5. HF reacts with silica (SiO2) present in glass and forms water 24. (c) Fluorine has highest reduction potential hence it is
soluble acid. strongest oxidising agent in nature
SiO2 + 4HF ¾¾ ® SiF + 2H O
4 2
25. (b) Due to absence of d atomic orbitals the fluorine does
not exhibit higher O.S.
SiF4 + 2HF ¾¾
® H SiF
2 4 26. (c) The element is F and ion is represented by M–
(Water soluble)
27. (d) F2 reacts with KOH to give O2 and O3 (see text)
6. F being the most electronegative element does not show
28. (b) 2F2 + 4KOH ® 4KF + O 2 + 2H 2 O for 1 mole of F2 the molar
positive oxidation states.
7. (i) Cl – Cl > Br – Br > F – F > I – I ratio.
(ii) HI > HBr > HCl > HF F2 KOH KF O2 H2O
(iii) M – F > M– Cl > M – Br > M – I 1
(iv) BrO4– > ClO4– > IO4– 1 2 2 1
2
15. (a) 16. (a) 17. (d) 18. (b) 19. (c) 20. (c)
21. (d) 22. (b) 23. (d) 24. (d) 25. (a) 26. (c) 29. (b) HSO -4 - e ® HSO 4 ® H 2S 2 O8 ® S2 O8 2 -
27. (b) 30. (a) Chlorine cannot displace F from NaF. The reactivity
EXERCISE 2 follows the order F > Cl > Br > I
31. (d) Cl2 will react with CaO, NaOH and KOH (See text). It will
1. (d) Ionisation potential decreases down the group. not react with conc. H2SO4 hence can be dried over it.
2. (c) Since fluorine has small size the electron affinity of Cl is 32. (b) 3s2 3p5 is electronic configuration of Cl
more than F 33. (b) In presence of moisture only, chlorine will give O;
3. (a) Follow text
Cl 2 + H 2O ® HCl + HOCl
4. (d) The lesser the bond energy, the weaker is the bond
5. (a) (See text) Reactivity follows the order F > Cl > Br > I 34. (a) 2NaOH + Cl2 ® NaCl + NaOCl + H 2 O
6. (d) See text
7. (d) Fluorine exhibit -ve oxidation state hence Cl- and OCl-
8. (a) Since F2 is most oxidising, it is easily reduced _ _
35. (a) 2Br + Cl 2 ® Br2 + 2Cl Chlorine is more oxidising is
9. (c) The more the bond energy the stronger is the bond
10. (d) See text nature
11. (c) Chlorine shows O.S. from –1,+1 to +7, whereas others 36. (d) K 2 Cr2 O 7 + Conc HCl ® Cl 2 See text
show O.S. as Na® +1 ; K® +1 ; F® –1
+1 +5 -1
12. (c) Fluorine always –1 37. (d) NaClO ® NaClO3 + 2NaCl All statements are correct
13. (b) See text
as evident from the reaction
14. (a) HF, due to intermolecular H-bonding is weakest among
38. (d) MnO2 or KMnO4 with conc HCl give Cl2 (see text)
HX acids
15. (d) HCl acid at 25º C is a gas and polar in nature 39. (b) 3Cl 2 + 6KOH ® KClO3 + 5KCl + 3H 2O
16. (c) Volatile character HCl > HBr >HI > HF KClO3 is used in fire works and safety matches and Cl2
17. (a) See text is greenish yellow gas
18. (d) Due to hydrogen bonding HF is a liquid +7 +2
19. (c) Follow text 40. (d) 2 KMnO 4 + 16HCl ® 2 MnCl 2 + 2KCl + 8H 2 O + 5Cl 2
20. (c) Oxy acids having higher + O.S for halogen are
O.S of Mn changes from +7 to +2 hence reduction occurs
more stronger in nature . Hence the order
and Cl2 is formed.
HClO4 > HClO3 > HClO2 > HClO
The p-Block Elements - Halogens 719
41. (c) Cl 2 + H 2O ® HCl + HOCl 4. (d) The halide ions act as reducing agents . F– ion does not
_ show any reducing nature but Cl–, Br– & I– ion act as
42. (b) 3Br2 + 6CO32 - + 3H 2 O ® 5Br + BrO3- + 6HCO 3- reducing agents and their reducing nature is in increasing
O.S. of Br2 changes from 0 to –1 and +5 which is reduction order
as well as oxidation. Cl – Br – I –
¾¾¾¾¾¾¾¾¾¾¾¾ ®
43. (a) KI + I 2 ® KI3 (Soluble complex) Re ducing nature increases
5. (d) We know that positive ion is always smaller and negative
44. (c) 2 Na 2 S2 O3 + I 2 ® Na 2 S4 O 6 + 2 NaI ion is always larger than the corresponding atom.
Sodium
tetrathion ate Therefore the correct order of the size is I - > I > I +
6. (a) F is more electronegative than Cl therefore HF bond is
45. (d) I- can act as reducing agent only and not oxidising stronger than HCl and hence proton is not given off easily
since its O.S. can change from 0 to +7 only and hence HF is a weakest acid.
46. (a) 2KClO3 + I 2 ® 2KIO3 + Cl 2 0 +1 -1
+5 O.S. of iodine is more stable than chlorine 7. (b) H 2 O + B r2 ¾¾
® HOBr+ HBr
47. (b) 2KI + CuSO 4 ® K 2SO 4 + CuI 2 Thus here oxidation number of Br increases from 0 to +1
and also decreases from 0 to –1. Thus it is oxidised as
2CuI 2 ® Cu 2 I 2 + I 2 well as reduced.
48. (d) Reactivity follows the order F > Cl > Br > I 8. (d) HO Cl < HO Cl O < HO Cl O 2 < HO Cl O3
49. (a) See text +1 +3 +5 +7

50. (d) Basic character increases down the group In case of oxyacids of similar element as the oxidation
number of the central atom increases, strength of acid
51. (a) Iodine is used as an antiseptic
also increases.
52. (d) 2CaOCl 2 ¾CoCl
¾¾ ¾ 2 ® 2CaCl + O (see text )
2 2 9. (b, c) From the given options we find option (a) is correct.
The oxidising power of halogens follow the order
+
F2 > Cl2 > Br2 > I2. Option (b) is incorrect because it in
53. (b) In HNO3 + HF ® H 2 NO3 + F- HNO3 accepts H+ hence
not the correct order of electron gain enthalpy of
it is a base halogens.
54. (a) NaClO is a salt of strong base and weak acid hence on The correct order is Cl2 > F2 > Br 2 > I2. The low value of
hydrolysis the solution will be most basic, therefore F2 than Cl2 is due to its small size.
maximum pH. Option (c) is incorrect. The correct order of bond
55. (d) Interhalogen compounds are not highly volatile dissociation energies of halogens is
56. (c) I – F is most polar. The more the electronegativity Cl2 > Br2 > F2 > I2.
difference, the more is polar character of bond Option (d) is correct. It is the correct order of
57. (b) 2KClO3 ® 2KCl + 3O 2 electronegativity values of halogens. Thus option (b)
and (c) are incorrect.
1
58. (a) ICl7. The hybridisation is (7 + 7 + 0 - 0) = 7 (sp3d3 ) 10. (c) MI > MBr > MCl > MF. As the size of the anion decreases
2 covalent character also decreases.
59. (a) When atomic orbitals present in the same principle energy 11. (d) Since all the halogens have a strong tendency to accept
level overlap, a strong bond is formed. In case of F and electrons. Therefore halogens act as strong oxidising
B, atomic orbitals are from second principle energy levels. agents and their oxidising power decreases from fluorine
60. (a) Fluorosis is caused due to reaction of Ca with excess of to iodine.
fluoride in the body 12. (c) The correct order of increasing bond angle is
61. (c) The strength of H- bonding follows the order Cl2 O < ClO 2- < ClO 2
F>O>N
Cl2O2 ClO2 2
ClO2 2

EXERCISE 3 ..
O . Cl . .
..

..
Cl
1. (b) ClO2 contains 7 + 12 i.e. 19 electrons (valence) which is
an odd number, i.e. there is (are) free electron(s). Hence it . . 110° . . . . 118°
Cl Cl .O. O O O
..

..
..

..
..
..

..
..

.. ..
..

is paramagnetic in nature.
..

* In ClO2 there are 2 lone pairs of electrons present on
2. (c) Pure ClO2 is obtained by passing dry Cl2 over AgClO3 at
90°C. the central chlorine atom. Therefore the bond angle in
90°C ClO2– is less than 118° which is the bond angle in ClO2
2AgClO 3 + Cl 2 (dry ) ¾¾¾® 2AgCl + 2ClO 2 + O 2
which has less number of electrons on central chlorine
3. (d) Calcium chlorite is Ca (ClO2 ) 2 atom.
720 Chemistry
13. (b) HClO4 is the strongest acid amongst all because the
11. (b) 6CaOCl 2 ® Ca (ClO3 ) 2 + 5CaCl 2
oxidation state or Cl is maximum (+7).
14. (a) SF4 has 4 bond pairs and 1 lone pair of electrons, sp3d (disproportionation or auto oxidation)
12. (d) All the factors are responsible for the greater reactivity
F of F2
| F
hybridisation leads to irregular shape S and
| 13. (c) SO 2 + Cl 2 ® SO 2 Cl 2
F Sulphuryl chloride
F
resultant m ¹ 0. 14. (b) NaF + HF ® NaHF2
15. (a) 4HCl + O 2 ® 2Cl 2 + 2H 2 O 15. (d) The reactivity of halogens follows the order
cloud of white fumes F2 > Cl 2 > Br2 > I 2 Hence I2 cannot replace Br 2 from
16. (d) The fluorine has low dissociation energy of F - F bond KBr.
and reaction of atomic fluorine is exothermic in nature
16. (a) In ClO -4 the Cl is present in its highest O.S of +7 hence
17. (a) CN - ion acts good complexing as well as reducing it can undergo reduction and not oxidation
agent.
17. (a) KF + HF ® KHF2
18. (c) The H–X bond strength decreases from HF to HI. i.e.
HF > HCl > HBr > HI. Thus HF is most stable while HI is 18. (b) 2KI + Cl 2 ® 2KCl + I 2
least stable. This is evident from their decomposition
I 2 + CCl 4 ® Violet Colour
reactions. The decreasing stability of the hydrogen halide
is also reflected in the values of dissociation energy of Note: The excess of Cl2 should be avoided. The layer
the H–X bond may become colourless due to conversion of I 2 to HIO 3

H- F H - Cl H - Br H- I I 2 + 5Cl 2 + 6H 2O ® 2HIO3 + 10HCl


135kcal mol-1 103kcal mol-1 87 kcal mol-1 71kcal mol-1 In case of Br 2 : Br2 + 2H 2 O + Cl 2 ® 2HBrO + 2HCl
19. (d) Chlorine reacts with excess of ammonia to produce The layer test is based upon distribution law
ammonium chloride and nitrogen. 19. (c) The bond energy of interhalogen compounds is less than
3Cl2 + 8NH3 (excess)¾¾® 6NH4Cl + N2 the bond energy of halogens
EXERCISE 4 1
20. (c) Cl 2 + H 2 O ® 2HCl + O2
1. (b) RCOO– is not pseudo halide . See list of pseudo halides 2
2. (d) HF does not act as reducing agent Hydrogen has more affinity for chlorine.
3. (d) In gaseous state the HCl is covalent in nature while in 21. (a) Non-metallic oxides are acidic and acidic character
decreases with decreasing non-metallic character.
aqueous solution it ionises to give H+ and C l ions
22. (d) 6CaOCl 2 ® Ca (ClO3 ) 2 + 5CaCl 2
4. (c) 2 NaCl + K 2 Cr2 O 7 + 4 H 2SO 4 ® It is auto oxidation.
23. (d) 2KI + Cl 2 ® I 2 + 2KCl
® Na 2SO4 + 2KHSO 4 + CrO 2 Cl 2 + H2O
Chromyl Chloride I 2 + CCl 4 ® violet
5. (b) Antichlor is a compound which removes unreacted I 2 + 5Cl 2 (Excess) + 6H 2O ® 2HIO 3 + 10HCl
chlorine from a material. For example hypo violet colour disappear
6. (a) HBr is oxidized to Br2 24. (b) 10 g of CaOCl 2 will give
H 2SO 4 + 2HBr ® SO 2 + 2H 2 O + Br2
4.9 ´ 22.4
4.9 g of Cl 2 = = 1.5 litre
7. (c) 3NaOCl ® NaClO 3 + 2 NaCl (disproport ionation ) 71
Hypochlori te Chlorate Chloride 25. (c) Statement (c) is correct.
8. (c) HgO + 2Cl 2 + H 2 O ® HgCl 2 + 2HOCl 26. (d) Bleaching powder + HCl ¾
¾® Cl2 (maximum)

9. (c) 2KClO3 + 4HCl ® 2KCl + Cl 2 + 2ClO 2 + 2H 2O 27. (b) Except ionisation potential other factors are true to explain
The mixture of (Cl 2 + ClO 2 ) is known as ‘euchlorine”. the oxidising (strong) behaviour of F2 .

10 (b) 2CaO + 2Cl 2 ® CaCl 2 + Ca (ClO) 2 28. (b) 2KMnO 4 + 16HCl ® 2KCl + 2MnCl 2 + 8H 2 O + 5Cl 2
21D
The p-Block Elements-
Noble Gases
INTRODUCTION : 6. Melting and boiling points- Due to the increase in magnitude
The group 18 of the periodic table consists of colourless, odourless of Van der waals forces, the mpt and bpt increases from He
gases at room temperature, isolated by William Ramsay and lord to Rn.
Rayleigh in 1898 from air. They realized that a sample of nitrogen 7. Atomic radii- The atomic radii increases from He to Rn and
formed synthetically had a different density than obtained from air it corresponds to the vander waals radii.
after removing oxygen, water vapour and carbon dioxide. The 8. Critical temperature/critical pressure- The critical
sample from air was heavier and it was found to contain 1.0% temperature (Tc) and Critical pressure (Pc) of noble gases
mixture of inert gases by volume (except Rn). Radon was obtained increase down the group
by radioactive disintegration of radium.
Element He Ne Ar Kr Ye Rn
Helium is second most abundant element in the universe.
Argon is the most abundant of all the inert gases in the Tc(K) 5.1 44.3 150.6 211.0 256.4 373.5
atmosphere. Pc(Atm) 2.26 26.86 47.99 54.3 58.2 62.4
GENERAL CHARACTERISTICS : 9. Density- The density of noble gases increases down the
1. Electronic configuration : group.
Element Sym bol At No. Valence shell electronic confg. 10. Heat of vaporisation - They have very low values of heat of
vaporisation due to weak van der waals forces of attraction.
Helium He 2 1s 2
The value increases down the group.
Neon Ne 10 [He] 2s 22p 6
11. Solubility in water- They are slightly soluble in water and
Argon Ar 18 [Ne] 3s 23p 6
solubility increases from He te Rn.
Kr yp to n Kr 36 [Ar] 3d104s 24p 6
12. Liquefication - It is extremely difficult to liquefy inert gases
Xenon Xe 54 [Kr] 4d10, 5s 25p 6 due to weak van der waals forces of attraction among their
Radon Rn 86 [Xe] 4f14, 3d10, 6s26p 6 molecules. Hence they possess low value of critical
temperature also.
2. Physical state - They are all gases under ordinary conditions
of temperature and pressure. 13. Ionisation energy- All noble gases possess very stable (ns2
3. Presence - Except Radon all occur in atmosphere in dry air and ns2p6) electronic configuration. Therefore, ionisation
10% by volume. Main commercial source of helium is natural energy of noble gases is very high and decreases down the
gas . group.
4. Abundance- In 1.0% air the abundance follows the order 14. Electron affinity- Due to the presence of stable electronic
configuration they have no tendency to accept additional
Ar> Ne>He>Kr> Xe
electron. Therefore electron affinity is almost zero.
5. Atomicity- The Cp/Cv =1.67 shows their monoatomic nature.
722 Chemistry
15. Polarizability- The polarizability increases with the size and SEPARATION OF RARE GASES BY DEWAR’S
hence follows the order. METHOD:
He< Ne< Ar< Kr< Xe It is as follows
16. Adsorption by charcoal- Except helium all are adsorbed by [He, Ne, Ar, Kr, and Xe] + Coconut charcoal at 171K
coconut charcoal at low temperature. The extent of
adsorption increases down the group.
DISCOVERY : He, Ne unadsorbed Ar, Kr, Xe Absorbed
+ charcoal at 93K +Coconut charcoal
(i) Argon- It was isolated by Rayleigh and confirmed by at liquid air temperature
Ramsay, from air (free from O2, moisture and CO2). Due to its
Ne He Ar Kr, Xe
inertness the gas was named as argon (meaning lazy). diffused Remain in first
Adsorbed unadsorbled
(ii) Helium- It was observed in the spectrum of the sun hence in another Charcoal
Charcoal Temperature
name helium, from helios, which means sun, by Lockyer raised to 183K
and Frankland. Ramsay obtained it from gases occluded in
Kr Xe
uranium minerals. evolved remains
adsorbed
(iii) Neon, Krypton and Xenon- These were obtained by
fractional distillation of liquid air under reduced pressure.
These were named as neon means “new” krypton means From liquid air - The difference in the boiling points of
“hidden” and Xenon means “stranger or foreigner”. various constituents of liquid air make possible their
separation by fractional distillation.
(iv) Radon- Spectroscopically was noticed by Dorn and isolated
by disintegration of radium by Rutherford and Soddy. Element He Ne N2 Ar O2 Kr Xe
B.pt (K) 4 27 77 87 90 121 124
Ra 226 ¾
¾® Rn 222 + He 4
88 86 2 HELIUM FROM OTHER SOURCES:
ISOLATION OF RARE GASES : He is also obtained by heating mineral clavite to 1273K . The gas
The following methods are employed for the isolation of Rare evolved is collected over potash solution to free from any CO2.
gas mixture. It may also be obtained from mineral by heating with dil.H2SO4
or KHSO4.
(i) Ramsay and Rayleigh’s first method-
Properties- The noble gases are inert in nature because of their
Air (dry and free from CO2)
completely filled s and p subshells. In 1962 the first compound of
D D noble gases was prepared. It is hexa fluoroplatinate prepared by
¾¾® 2Cu + O 2 ¾
¾® CuO ¾¾®
Cu Mg Bartlett.

2Mg + O 2 ® 2MgO and 3Mg + N 2 ® Mg 3 N 2 Xe + PtF6 ® Xe[PtF6 ]

Residual gas - mixture of inert gases. Now many compounds of Xe and Kr are known with fluorine and
(ii) Ramsay and Rayleigh’s second method oxygen.
No compounds of He, Ne or Ar exist, except under very special
discharge NaOH conditions (very unstable, not neutral compounds)
Air + O 2 ¾¾¾¾® N 2 + O 2 ® 2NO ¾¾¾® 2NO 2
6000 -8000V Rn is known to react with fluorine but its radio activity makes the
2NO + O 2 ® 2NO 2 study of its compounds difficult and dangerous. Kr forms one
(CO2 and NO2 are absorbed by NaOH and O2 if any is stable neutral molecule, KrF2. A, Kr- N bond has also been
removed by alkaline pyrogallol) reported, stable only below –50º C.
(iii) Fischer and Ringe’s method- Xe forms several compounds with fluorine and oxygen and Xe-N
and Xe-C bonds have been reported.
90%CaC2 D
R Air 10%CaCl ¾¾® CaC2 + N 2 ® [CaCN 2 + C ] 673K
Xe + F2 ¾¾¾® XeF2 673K
Xe + 2F2 ¾¾¾® XeF4
2 373K Nitrolium (C is graphite)
5- 6Atm
2 :1
C + O 2 ® CO 2 1: 5

2CaC2 + 3CO 2 ® 2CaCO3 + 5C


573K
Xe + 3F2 ¾¾ ¾¾¾® XeF6
CO2 is absorbed in KOH and residual gas is dried over 50- 60 Atm
Conc H2 SO4 and P2 O5 1 : 20
The p-Block Elements -Noble Gases 723
STRUCTURES OF XENON FLUORIDES : Uses
(1) Helium - It is non-inflamable. It has low density and its
F lifting power is 92% that of hydrogen and used to lift weather
balloons and airships. It is used as breathing mixture (or

..
XeF2 hybridization sp3d Xe Linear

..
oxygen dilutant) for divers. Mixture of O2 and He is used in
the treatment of asthma. It is also used for inflating the tyres

..
F of aeroplanes.
.. (2) Neon - When an electric current is passed through a sample
F F
of the gas, it has a characteristic orange - red glow. Neon
XeF4 hybridization sp3d2 Xe square planar lighting is used for advertising. The noble gases argon
F .. F (purple), Xenon (blue green) and Krypton (pale violet) are
also used in “neon” lighting.
F
F .. F (3) Argon - It is used primarily to create an inert atmosphere in
XeF6 Hybribisation sp3d3 Xe light bulbs, welding and fluorescent bulbs. It is also used in
geiger counters, as it becomes ionised in the presence of
F F radiation. The ratio of 40K to 40Ar can be used to date the
F
Pentagonal pyramidal or distorted octahedral age of rocks since argon is obtained by radioactive decay
XeO3 is the anhydride of Xenic acid H2XeO4 - The structures of of an isotope of potassium.
oxyfluorides and oxides of Xenon are as follows. (4) Krypton - The light emitted by Krypton in an electric
O discharge tube is used for runway and approach lights in
..
F F airports.
Xe Xe (5) Xenon - It is used in electrical flash bulbs for high speed
.. O O photography.
F F
O (6) Radon - In radiotherapy of cancer.
XeOF4 XeO3
CLATHRATES :
Square pyramidal trigonal pyramidal
A number of organic and inorganic compounds having noble
(sp3d2) (sp3)
gases trapped into the cavities of crystal lattices are called
enclosure or clathrate compounds. They are known as cage
F
compounds also.
Xe = O The substance having cavities in crystal lattices is called the
host and atom of noble gases entraped in it is called the guest
F which are held by Van der waals forces of attraction. The clathrates
XeOF2 are non stoichiometric compounds. When clathrates are heated
T – shaped or dissolved the guest atom escapes from the host.
(sp3d)
He and Ne do not form clathrates due to their small size.
F Types of Clathrates - They are of two types.
(i) Gas hydrates - Solid water having entraped Ar, Kr, or Xe
Xe Xe
(ii) Quinol Clathrates - Quinol having entraped Ar, Kr, and Xe
F
Uses of Clathrates
XeO2F2 XeO4
See-Saw (i) Separation of noble gases - Since Ne does not form a
Tetrahedral
(sp3d) clathrate with Quinol it is separated from Ar, Kr and Xe. The
(sp3)
latter form a clathrate with quinol.
(ii) Xe- 133 clathrate is a source of g- radiations
F F
Xe F (iii) Kr - 85 clathrate is a source of b - radiations
F F (iv) As an anaesthetic - Xe clathrate is used for this
– (v) For transporting isotopes of noble gases.
XeF5
Pentagonal planar
3 3
(sp d )
724 Chemistry

Very Short/Short Answer Questions 14. In XeF2, XeF4, XeF6 the number of lone pairs on Xe are
1. Why do noble gases form compounds with fluorine and respectively
oxygen only? (a) 2, 3, 1 (b) 1, 2, 3 (c) 4, 1, 2 (d) 3, 2, 1.
2. Which compound led to the discovery of noble gas 15. Total number of lone pair of electrons in XeOF4 is
compounds. (a) 0 (b) 1 (c) 2 (d) 3
3. Among the noble gases, Xe forms maximum noble gas 16. Which of the following is least polarisable ?
compounds with fluorine. Why is it so? (a) Ne (b) He (c) Xe (d) Kr
4. Why do boiling points of noble gases increases from helium 17. End-product of the hydrolysis of XeF6 is
to radon? (a) XeF4O (b) XeF2O2 (c) XeO3 (d) XeO3 –
5. Neon is generally used for warning signals. Why? 18. For advertisement, the coloured discharge tubes contain :
6. Give equations for the following : (a) He (b) Ne (c) Ar (d) Kr
(i) XeF2 + H2O ¾¾ ® 19. Which one of the following noble gases is not found in the
(ii) XeF6 + H2O ¾¾ ® atmosphere?
(iii) XeF6 + PF5 ¾¾ ® (a) Rn (b) Kr (c) Ne (d) Ar
(iv) XeF6 + NaF ¾¾ ® 20. Noble gases are group of elements which exhibit
7. How is XeO3 prepared? Write chemical equation. (a) high chemical activity
8. Solubility of noble gases in water increases as we more down (b) low chemical activity
the group. Explain. (c) minimum electronegativity
9. Why noble gases have low boiling point? (d) paramagnetic properties
10. Xenon does not form fluorides, such as XeF3 and XeF5. 21. Which one of the following statements regarding helium is
Why? incorrect ?
(a) It is used to produce an d sustain powerful
Multiple Choice Questions
superconducting magnets.
11. Which of the following fluorides of xenon is impossible?
(b) It is used as a cryogenic agent for carrying out experiments
(a) XeF2 (b) XeF3 (c) XeF4 (d) XeF6
at low temperatures.
12. The structure of XeF6 is (c) It is used to fill gas balloons instead of hydrogen because
(a) distorted octahedral (b) pyramidal it is lighter and non-inflammable.
(c) tetrahedral (d) None of these (d) It is used in gas-cooled nuclear reactors.
13. Which of the following statements is false ? 22. Which inert gas show abnormal behaviour on liquefaction
(a) Radon is obtained from the decay of radium (a) Xe (b) He (c) Ar (d) Kr
(b) Helium is inert gas 23. The ease of liquefaction of noble gases increases in the order
(c) Xenon is the most reactive among the rare gases (a) He < Ne < Ar < Kr < Xe (b) Xe < Kr < Ne < Ar < He
(d) The most abundant rare gas found in the atmosphere is (c) Kr < Xe < He < Ne < Ar (d) Ar < Kr < Xe < Ne < He
helium

1. The last member of the family of inert gases is (a) Neon (b) Radon
(a) argon (b) radon (c) Argon (d) Helium
(c) xenon (d) neon 4. Number of unpaired electrons in inert gas is
2. Which of the following is the correct sequence of the noble (a) zero (b) 8
gases in their group in the periodic table? (c) 4 (d) 18
(a) Ar, He, Kr, Ne, Rn, Xe (b) He, Ar, Ne, Kr, Xe, Rn 5. In the following four elements, the ionisation potential of
(c) He, Ne, Kr, Ar, Xe, Rn (d) He, Ne, Ar, Kr, Xe, Rn which one is the highest ?
3. Which of the following noble gases do not have an octet of (a) Oxygen (b) Argon
electrons in its outermost shell? (c) Barium (d) Cesium
The p-Block Elements -Noble Gases 725
6. The elements which occupy the peaks of ionisation energy 21. What are the products formed in the reaction of xenon
curve are hexafluoride with silicon dioxide ?
(a) Na, K, Rb, Cs (b) Na, Mg, Cl, I (a) XeSiO4 + HF (b) XeF2 + SiF4
(c) Cl, Br, I, F (d) He, Ne, Ar, Kr (c) XeOF4 + SiF4 (d) XeO3 + SiF2
7. Gradual addition of electronic shells in the noble gases 22. XeF6 on complete hydrolysis gives
causes a decrease in their (a) Xe (b) XeO2
(a) ionisation energy (b) atomic radius (c) XeO3 (d) XeO4
(c) boiling point (d) density 23. XeF4 involves which hybridization
8. Which of the following noble gas is least polarisable? (a) sp (b) sp 2
2
(c) sp d (d) sp 3d 2
(a) He (b) Xe
(c) Ar (d) Ne 24. Shape of XeOF4 is
9. In which of the following groups, when He is placed, its all (a) octahedral (b) square pyramidal
the properties are satisfied (c) pyramidal (d) T-shaped
(a) with alkali metals (b) with halogens 25. The hybridization of Xe in XeF2 is
(c) with inert gases (d) None of these (a) sp 3 (b) sp 2
3
(c) sp d (d) sp 2 d
10. The most abundant inert gas in the atmosphere is
(a) He (b) Ne 26. Which is a planar molecule ?
(c) Ar (d) Kr (a) XeO4 (b) XeF4
11. The lowest boiling point of helium is due to its (c) XeOF4 (d) XeO2F2
(a) inertness 27. Which of the following has sp3 hybridization ?
(b) gaseous nature (a) XeO3 (b) BCl3
(c) high polarisability (c) XeF4 (d) BBr3
28. Which of the following two are isostructural ?
(d) weak van der Waals forces between atoms
12. Which one of the following elements is most reactive ? (a) XeF2, IF2- (b) NH3, BF3
(a) He (b) Ne
(c) Ar (d) Xe (c) CO32 - ,SO32 - (d) PCl5, ICl5
13. Which of the noble gas has highest polarisability? 29. The number of lone pair of electrons present on Xe in XeF2
(a) He (b) Ar is
(c) Kr (d) Xe (a) 3 (b) 4
14. What is the atomic number (Z) of the noble gas that reacts (c) 2 (d) 1
with fluorine ? 30. Match List I with List II and select the answer using the
(a) 54 (b) 10 codes given below :
(c) 18 (d) 2 Code List I Code List II
15. Noble gases are group of elements which exhibit very A XeF4 1 Distorted octahedral
(a) high chemical activity
B XeF6 2 Tetrahedral
(b) low chemical activity
C XeO3 3 Square planar
(c) minimum electronegativity
(d) much paramagnetic properties D XeO4 4 Trigonal pyramidal
16. The correct order of solubility in water for He, Ne, Ar, Kr, Xe (a) A-4, B-1, C-3, D-2 (b) A-2, B-3, C-1, D-4
is (c) A-1, B-4, C-2, D-3 (d) A-3, B-1, C-4, D-2
(a) He > Ne > Ar > Kr > Xe (b) Ne > Ar > Kr > He > Xe 31. Hybridization and structure of XeF4 is
(c) Xe > Kr > Ar > Ne > He (d) Ar > Ne > He > Kr > Xe (a) sp3d, trigonal bipyramidal(b) sp3, tetrahedral
17. The noble gas which was discovered first in the sun and (c) sp3d2, square planar (d) sp3d2, hexagonal
then on the earth 32. Number of lon e pairs of electrons on Xe atoms
(a) argon (b) xenon XeF2, XeF4 and XeF6 molecules are respectively
(c) neon (d) helium (a) 3, 2 and 1 (b) 4, 3 and 2
18. XeF4 on partial hydrolysis produces (c) 2, 3 and 1 (d) 3, 2 and 0
(a) XeF4 (b) XeOF2 33. Which one of the following is correct pair with respect to
(c) XeOF4 (d) XeO3 molecular formula of Xenon compound and hybridization
19. Which element out of He, Ar, Kr and Xe forms least number state of Xenon in it?
of compounds ? (a) XeF4, sp3 (b) XeF2, sp
(a) He (b) Ar (c) XeF2, sp3d (d) XeF4, sp2
(c) Kr (d) Xe 34. The coloured discharge tubes for advertisement mainly
20. The element which has not yet been reacted with F2 is contain
(a) Ar (b) Xe (a) xenon (b) helium
(c) Kr (d) Rn (c) neon (d) argon
726 Chemistry
35. Sea divers go deep in the sea water with a mixture of which 37. In the clathrates of xenon with water, the nature of bonding
of the following gases between xenon and water molecule is
(a) O2 and He (b) O2 and Ar (a) covalent
(c) O2 and CO2 (d) CO2 and Ar (b) hydrogen bonding
36. Which of the following is the life saving mixture for an (c) co-ordinate
asthmas patient ? (d) dipole-induced dipole interaction
(a) Mixture of helium and oxygen 38. Which of the following cannot be formed ?
(b) Mixture of neon and oxygen (a) He2+ (b) He+
(c) Mixture of xenon and nitrogen (c) He (d) He2
(d) Mixture of argon and oxygen

1. Noble gases do not react with other elements because (c) It is used to fill gas balloons instead of hydrogen because
(a) they are mono atomic [CBSE PMT 1994] it is lighter and non-inflammable
(b) they are found in abundance (d) It is used in gas-cooled nuclear reactors
(c) the size of their atoms is very small 5. Which one of the following reactions of xenon compounds
(d) they are completely paired up and stable electron shells is not feasible? [AIEEE 2009]
2. Which of the following statements is false ? (a) 3XeF4 + 6H 2 O ¾¾ ® 2Xe + XeO 3 +12HF +1.5O 2
[CBSE PMT 1994] (b) 2XeF2 + 2H 2 O ¾¾ ® 2Xe + 4HF + O 2
(a) Radon is obtained from the decay of radium
(c) XeF6 + RbF ¾¾ ® Rb[XeF7 ]
(b) Helium is inert gas
(c) Xenon is the most reactive among the rare gases (d) XeO3 + 6HF ¾¾ ® XeF6 + 3H 2 O
(d) The most abundant rare gas found in the atmosphere is 6. Which of the following has maximum number of lone pairs
helium associated with Xe ? [AIEEE 2011 RS]
3. XeF2 is isostructural with : [NEET 2013] (a) XeF4 (b) XeF6
(a) ICl2– (b) SbCl3 (c) XeF2 (d) XeO3
(c) BaCl2 (d) TeF2 7. Total number of lone pair of electrons in XeOF4 is
4. Which one of the following statement regarding helium is (a) 0 (b) 1 [IIT-JEE 2004]
incorrect ? [AIEEE 2004] (c) 2 (d) 3
(a) It is used to produce and sustain powerful 8. The shape of XeO2F2 molecule is [IIT-JEE 2012]
superconducting magnets (a) trigonal bipyramidal (b) square planar
(b) It is used as a cryogenic agent for carrying out (c) tetrahedral (d) see-saw
experiments at low temperatures

1. Density of nitrogen gas prepared from air is slightly greater 4. If two liters of air is passed repeatedly over heated copper
than that of nitrogen prepared by chemical reaction from a and heated Mg till no further reduction in volume takes place
compound of nitrogen due to the presence of the volume finally obtained will be approximately
(a) argon (a) 200 mL (b) 20 mL
(b) carbon dioxide (c) zero (d) 10 mL
(c) some N3 molecules analogous to O3 5. The atomic weight of noble gases is obtained by using the
(d) greater amount of N2 molecules derived from N-15 relationship
isotope (a) Atomic weight = equivalent weight × valency
2. A radioactive element X decays to give two inert gases X is (b) Atomic weight = equivalent weight/ valency
(c) At. weight = valency/ equivalent weight
(a) 238 (b) 226
92 U 88 Ra (d) 2 × V.D. = molecular weight = atomic weight
(c) Both (a) and (b) (d) Neither (a) nor (b) 6. Compounds formed when the noble gases get entrapped in
3. Coconut charcoal at 100°C adsorbs a mixture of the cavities of crystal lattices of certain organic and inorganic
(a) He and Kr (b) Ar, Kr and Xe compounds are known as
(c) Kr and Xe (d) He and Ne (a) Interstitial compounds (b) Clathrates
(c) Hydrates (d) Picrates
The p-Block Elements -Noble Gases 727
7. The noble gas which behaves abnormally in liquid state is 11. Which is called lazy gas?
(a) Xe (b) Ne (a) Kr (b) Ar
(c) He (d) Ar (c) He (d) Ne
8. Which is called stranger gas 12. Which statement about noble gases is not correct?
(a) Kr (b) Xe (a) Xe forms XeF6
(c) He (d) Ne (b) Ar is used in electric bulbs
9. In order to prevent the hot metal filament from getting burnt, (c) Kr is obtained during radioactive disintegration
when the electric current is switched on, the bulb is filled (d) He has the lowest b.pt among all the noble gases
with 13. Which has the same electronic configuration as of inert gas?
(a) CH4 (b) An inert gas (a) Ag3+ (b) Cu2+
(c) CO2 (d) Cl2 4+
(c) Pb (d) Ti4+
10. In Kroll and IMI process of the production of titanium, the
14. Which noble gas does not form clathrates?
inert gas used is
(a) Xe (b) Kr
(a) Ne (b) Ar
(c) He (d) Ar
(c) Kr (d) Xe

EXERCISE 1 11. (d) Due to weak van der Waal’s forces, He has lowest boiling
point
2. Formation of O2+ [PtF6]–
led to the discovery of first noble
12. (d) Xe forms maximum compounds hence it is most reactive
gas compound Xe[PtF6].
13. (d) The larger the size the more is the polarisiability
5. Neon lights are visible from distance even in fog and mist
14. (a) Atomic number Xe is 54. Xe reacts with F2 and forms
and hence neon is generally used for warning signals. many compounds
6. (i) 2XeF2 + 2H2O ¾¾ ® 2Xe + 4HF + O2 15. (b) Noble gases exhibit low chemical activity
(ii) XeF6 + 3H2O ¾¾ ® XeO3 + 6HF 16. (c) Solubility increases from He to Rn
(iii) XeF6 + PF5 ¾¾ ® [XeF5]+ [PF6]– 17. (d) He was observed in the spectrum of the sun
(iv) XeF6 + NaF ¾¾ ® Na+ [XeF7]–
18. (b) XeF4 + H 2O ® 2HF + XeOF2
7. XeO3 is prepared by hydrolysis of XeF6 or XeF4.
Hydrolysis 19. (a) No compound of He as yet been reported
XeF6 + 3H2O ¾¾¾¾ ¾ ® XeO3 + 6HF
20. (a) No compound of Ar as yet been reported with F2
6XeF4 + 12H2O ® 4Xe + 2XeO3 + 24HF + 3O2
11. (b) 12. (a) 13. (d) 14. (d) 15. (b) 16. (b) 21. (c) 2XeF6 + SiO 2 ® SiF4 + 2XeOF4
17. (c) 18. (b) 19. (a) 20. (b) 21. (c) 22. (b) 22. (c) XeF6 + 3H 2 O ® 6HF + XeO 3
23. (a)
1
23. (d) Hybridisation in XeF4 = (8 + 4 + 0 - 0) = 6 sp3d 2
EXERCISE 2 2
1. (b) Radon is the last member of family 24. (b) XeOF4 square pyramidal
2. (d) He, Ne, Ar, Kr, Xe, Rn 25. (c) Hybridisation of XeF2 is sp3d
3. (d) Electronic configuration of He is 1s2 26. (b) XeF4 is planar (see text)
4. (a) Inert gases do not contain unpaired electrons 27 (a) In XeO3 the hybridisation is sp3
5. (b) Ionization potential of inert gases is highest in periodic 28. (a) XeF2 and IF2- both are linear and have hybridisation
table due to stable electronic configuration.
sp 3 d
6. (d)
7. (a) Ionisation energy decreases as we move away from F
nucleus due to less electrostatic attraction between
electrons and nucleus 29. (a) XeF2 has Xe structure hence number of lone pair
8. (a) The smaller the size the least is the polarisability
F
9. (c) The differentiating electron enter in s subshell in case of
of electrons 3
He, hence it is s- block element. Its electronic
configuration l s2 makes it inert in nature hence it is 30. (d) XeF4 is square planar (A-3) ; XeF6 is distorted octahedral
placed with inert gases. (B-1); XeO3 trigonal pyramidal (C-4) and XeO4 is
tetrahedral (D-2).
10. (c) Ar is the most abundant in atmosphere
728 Chemistry
31. (c) Hybridisation of XeF4 is sp3d2 and structure is square F
planar F
32. (a) XeF2 XeF4 XeF6
Valence electrons of Xe 8 8 8
XeF6 : F Xe F
Electrons involved 2 4 6
in bond formation
Lone pairs left 3 2 1 F F
33. (c) Hybridisation in each case is XeF4sp3d2, XeF2sp3d,
34. (c) Coloured discharge tubes mainly contain Neon
35. (a) Breathing mixture is (O2 + He) Xe
36. (a) Mixture of (He + O2) is used for asthma patient
37. (d) In clathrates the forces are dipole - induced dipole XeO3 : O O
interaction O
1 1p
38. (d) He2 Bond order = (2 - 2) = 0 Hence cannot be formed.
2 Hence XeF2 has maximum no. of lone pairs of electrons.
EXERCISE 3 7. (b) In XeOF4, Xenon is sp3d2 hybridised and has one lone
1. (d) On account of highly stable ns2np6 configuration in the pair.
valence shell. These elements have no tendency either 8. (d) XeO2F2 has trigonal bipyramidal geometry, but due to
to lose gain or share electrons with atoms of other presence of lone pair of electrons on equitorial position,
elements i.e., their combining capacity or valency is zero. its actual shape is see-saw.
Further all the orbitals in the atoms of these elements are F
doubly occupied i.e electrons are not available for O
sharing. Xe
2. (d) The most abundant rare gas found in the atmosphere is
argon and not helium. O
F
3. (a) F ¾ Xe ¾ F sp3d and Linear EXERCISE 4
1. (a) Air contains about 1% inert gases, mainly Ar. (At.wt
– sp3d and Linear
Cl ¾ I ¾ Cl 40) The atomic wt of N2 is 28
4. (c) Helium is heavier than hydrogen although it is non- 226
inflammable
2. (b) 88 Ra ®86 Rn 222 + 2 He 4 . Both are inert gases
5. (d) The products of the concerned reaction react each 3. (b) Coconut charcoal at 100°C absorbs Ar, Kr and Xe
other forming back the reactants. air
4. (b) Cu and Mg ¾¾ ¾ ¾ ¾¾® CuO, MgO and Mg3N2
® XeO3 + 6HF .
XeF6 + 3H 2 O ¾¾ mainly N 2 and O 2

F Air contains 1% inert gases which are left unreacted


1% of two liters is 20 ml
5. (d) 2 × VD = M.wt = Atomic wt of inert gas since inert
gases are monoatomic in nature
Xe 6. (b) The statement stands for the definition of Clathrates
6. (c) XeF2 :
7. (c) He behaves abnormally in liquid state
8. (b) Xe is also known as stranger gas
9. (b) Inert gases do not support combustion
F 10. (b) Argon is used in Kroll and IMI process for titanium to
3p provide an inert atmosphere
11. (b) Ar is called lazy gas
F F 12. (c) He is obtained during radioactive decay

Xe 13. (d) 22 Ti = 1s 2 , 2s 2 p 6 , 3s 2 p 6 d 2 , 4s 2 ; Ti 4+ = 1s 2 , 2s 2 p 6 , 3s 2 p 6
14. (c) He does not form clathrates due to small size and low
XeF4 : F F molecular weight. Ne also does not form clathrates for
the same reason
2 p
22
The d & f Block Elements

GENERAL CHARACTERISTICS : Zn ,Cd, Hg ,the end members of first three series have their
1. Transition elements :- Elements where the last orbitals filled general electronic configuration (n - 1)d 10 ns 2 . These do not
are the d orbitals known as transition elements. They have
been placed in the middle of the periodic table between show properties of transition elements to any appreciable
electropositive s-block and electronegative p-block elements. extent and are called non-typical transition elements.
3. Classification - Transition elements consist of the following
2. General Electronic Configuration :- Transition metals have four series
the electronic configuration (n - 1)d1-10 ns0 - 2 . When
3d series 21 Sc to 30 Zn 3d1-10 4s1- 2
electrons fill orbitals, ns-orbital is filled first than (n - 1) d- 4d series 39 Y to 48Cd 4d 1-105s0 - 2
orbital.When losing during oxidation ,ns electrons are lost 5d series 57 La 72 Hf to 80 Hg 5d1-10 6s1- 2
first than (n-1)d electrons. 6d series 89 Ac 104 Rf to 111Rg

4. Electronic configurtion of transition elements-


3d-Series 4d-Series 5d-Series
Atomic Element Electronic Atomic Element Electronic Atomic Element Electronic
number configuration number configuration number configuration
21 Sc [Ar]3d 1 4s 2 39 Y [Kr ]4d 15s 2 57 La [Xe]5d1 6s 2
22 Ti [Ar ]3d 2 4s 2 40 Zr [Kr ]4d 2 5s 2 72 Hf [Xe]4f 14 5d 2 6s 2
23 V [Ar ]3d 3 4 s 2 41 Nb [Kr ]4d 4 5s1 73 Ta [Xe]4f 14 5d 3 6s 2
24 Cr [Ar]3d 5 4s1 42 Mo [Kr ]4d 5 5s1 74 W [Xe]4f 14 5d 4 6s 2
25 Mn [Ar ]3d 5 4s 2 43 Tc [Kr ]4d 5 5s 2 75 Re [Xe]4f 14 5d 5 6s 2
26 Fe [Ar ]3d 6 4s 2 44 Ru [Kr ]4d 7 5s1 76 Os [Xe]4f 14 5d 6 6s 2
27 Co [Ar ]3d 7 4s 2 45 Rh [Kr ]4d 8 5s1 77 Ir [Xe]4f 14 5d 7 6s 2
28 Ni [Ar ]3d 8 4 s 2 46 Pd [Kr ]4d 10 5s0 78 Pt [Xe]4f 14 5d 9 6s1
29 Cu [Ar]3d 10 4s1 47 Ag [Kr ]4d 10 5s1 79 Au [Xe]4f 14 5d10 6s1
30 Zn [Ar]3d 10 4s 2 48 Cd [Kr ]4d 10 5s 2 80 Hg [Xe]4f 14 5d10 6s 2
730 Chemistry
5. Physical properties of transition elements-
(i) Metallic character- Transition metals can lose valence
9 Ir Au
electrons and form cations M ® M+n + ne - They have 5d
simple hcp, ccp and bcc lattices characteristic of true metals. Pt
Except Hg they are solids at room temperature and are Os
dense ( d = 5 g / cm 3 in general, in case of osmium 22.6g/

–1
Pd

kJ mol
8
cm3 ), lustrous, malleable, ductile thermal and electrical Hf W Fe Co
Ta Re Cu 3d
conductors.

–2
There is gradual decrease in electropositive character from Ni

1E1 × 10
left to right Mn Ag
Rh 4d
(ii) Melting and boiling point - Due to strong metallic 7 Mo Ru
Tc
bond , they have high mpts and bpts. The mpts of these Zr Nb
elements rise to a maximum and then fall with the increase Ti
in atomic number the manganese and technitium show Cr
V
abnormal values as shown by graph) Sc
6
W
Transition elements
3500
Re First ionization energies of 3d-4d- and 5d-
Ta transition series
3000
Mo Os
Tc
Ir
2500 Nb (iv) Electrode potential ( E 0 ) ( E 0 M n+ M ) is governed by
Ru
three factors
Melting Point (K)

Zr Cr Rh
2000 Ti
V Pt (a) Heat of sublimation
Hf Fe
Y Pd (b) Heat of ionisation
(c) Heat of hydration
)

Co Ni
ies eries

1500
s)

Sc Cu
serie
s

Mn
For the 3d transition metals the E 0 M 2 + / M values are
(4d

Au
Ag
(5d
ser

1000
La V Cr Mn Fe Co Ni Cu
(3d

Zn
-1.18 -0.91 -1.18 -0.44 -0.28 -0.25 0.35
500 (Volts)
Cd The irregular trend is due to variation in ionization energies
and sublimation energies. Except copper
Hg 3d elements are good reducing agents but weaker than s-
1 2 3 4 5 6 7 8 9 10 11 12
block elements.
(v) Oxidation states -In different types of compounds ,
Periodic Groups
Melting point of Transition metals transition metals exhibit different oxidation states. The
highest oxidation state is exhibited in fluorides and oxides.
(iii) Ionisation energy- The ionisation energy increases with
In lower oxidation state the compounds formed are ionic
the increase in the atomic number but not in regular manner.
and in higher oxidation state they are covalent in nature.
The ionisation energies of 5d elements are higher than
Osmium exhibit +8 O.S. (highest)often but Ru exhibit +8
those of 4 d and 3d elements due to greater effective nuclear
oxidation state rarely. Transition metals also show oxidation
charge which in turn is due to poor shielding of nucleus by
states +1 and zero.
4f electrons.
Fe3+ is more stable than Fe2+. Hence Fe2+ act as reducing
Formation of Ni ++ requries 2.49 kJ mol-1 and formation agent Cr3+ is more stabe than Cr2+. Hence Cr2+ act as
Pt ++ requries 2.66 kJ mol -1 . Hence Ni (II) compounds reducing. Mn2+ is more stable than Mn 3+ Hence Mn3+ act
are thermodynamically more stable than Pt (II) Compounds. as oxidising agent
(vi) Atomic and Ionic radii - The values for atomic radii and
Formation of Ni 4 + requires 11.29 kJ mol-1 and ionic radii are in between the values for s and p-block
elements. In 3d transition series the ionic radii for
formation of Pt 4 + requires 9.36 kJ mol-1 . Hence Pt
M 2 + ion decreases upto the middle of the period then
(IV) compounds are relatively more stable than nickel (IV)
becomes almost constant (see table)
compounds. Thus K 2 PtCl6 is well known where as the Due to lanthanide contraction the second and third member
corresponding nickel compound is not known. (Ionisation of each group have atomic redii close to each other (Zr.160
energy graph is sketched here for ready reference) pm ,Hf 159pm)
The d & f-Block Elements 731
TABLE

Element Sc Ti V Cr Mn Fe Co Ni Cu Zn
Atomic radii (pm) 144 132 122 117 117 117 116 115 117 125
I. E. kJ mol–1 631 656 650 652 717 762 758 736 745 906
Oxidation State +3 (+2) +2 +2 +2 +2 +2 +2 +1 +2

+3 +3 +3 (+3) +3 +3 (+3) +2

+4 +4 (+4) +4 (+4) (+6) (+4)

+5 (+5) (+6)

+6 +7

Element Y Zr Nb Mo Tc Ru Ph Pd Ag Cd
Atomic radii 180 160 160 146 136 134 134 137 144 154
(pm)
I.E. (kJ mol–1) 616 674 664 685 703 711 720 804 731 876
Oxidation State +3 (+3) (+2) (+2) +4 +2 +3 +2 +1 +2
+4 (+3) +3 (+5) +3 +4 (+3) (+2)
(+4) +4 +7 +4 (+6) +4 (+3)
+5 +5 (+5)
+6 (+6)
(+7)
(+8)

Element La Hf Ta W Re Os Ir Pt Au Hg
Atomic radii (pm) 187 159 146 139 137 135 136 138 144 157
I.E. (kJ mol–1) 541 760 760 770 759 840 900 870 889 1007
Oxidation State +3 (+3) (+2) +2 +3 (+2) (+2) +2 +1 +1
+4 (+3) (+3) +4 (+3) +3 (+3) +3 +2
(+4) +4 +5 +4 +4 +4
+5 +5 +6 +6 (+6) (+5)
+6 +7 +8 (+6)
(vii)Density - d-block elements have high density because of their small atomic sizes and strong metallic bonding.
Density Sc Ti V Cr Mn Fe Co Ni Cu Zn
g/ml 3.0 4.54 6.10 7.19 7.40 7.87 8.70 8.90 8.92 7.13

(viii)Atomic volume - Atomic volume decreases along the Colour of a complex depends on the metal, its oxidation
period due to decrease in size. state and its ligands. e.g.
(ix) Reactivily -d-block elements are less reactive due to high 2+
ionisation energies. Some are almost inert and known as é Cu ( H 2O ) ù is Pale blue
ë 4û
noble metals, e.g. Au, Pt, Ru, Rh, Os, Ir etc.
2+
(x) Complex formation - They are well known to form a large éCu ( NH3 ) ù is Dark blue
ë 4û
number of complex compounds mainly due to
(i) Small atomic size and higher nuclear charge. CuSO 4 .5H 2 O is blue in colour and anhydrous CuSO4 is
(ii) Presence of partly filled or vacant orbitals colourless. In absence of ligands all d orbitals are degenerate
eg. K 4 [Fe (CN )6 ] (same energy) and the possbility of d-d excitation is no
(xi) Coloured ions - The colour exhibited by transition metal more. In presence of ligand d and d have higher
x 2 -y2 z2
ions is due to the presence of unpaired electrons in d- energy, d-d transition take place by absorption of light,
orbitals which permits the d-d excitation of electrons. hence the colour.
732 Chemistry
(xii) Magnetic properties -They are 2. Extraction - It is extracted from haematite Fe2 O3 in a blast
(i) Paramagnetic- This is due to the presence of unpaired
furnace by reduction with carbon and carbon monoxide. The
electrons in d-orbitals. Paramagnetic character increases
steps inolved are-
with the number of unpaired electrons.
(ii) Diamagnetic - Diamagnetic substances are repelled (a) Concentration - The crushed ore is agitated with water
and then concentrated by electromagnetic method.
by an applied magnetic field.
(iii) Ferromagnetism - In this case permanent magnetic (b) Roasting or Calcination- To remove volatile substances
moment is acquired by substance e.g. Fe. Magnetic and organic matter. FeO changes to Fe2 O3
moment is given by
S + O 2 ® SO2
= n(n + 2 ) B.M. where n = number of unpaired 4As + 3O2 ® 2As2 O3
electrons and B.M. = Bohr magneton (unit of magnetic
moment) FeCO 3 ® FeO + CO 2
(xiii)Catalytic properties - The transition metals and their 4FeS 2 + 11O 2 ® 2Fe 2 O 3 + 8SO 2
compounds behave as catalyst due to 4FeO + O 2 ® 2Fe 2 O 3
(a) The presenc of partly filled d-orbitals and exhibiting (c) Smelting - Roasted or calcinated ore is mixed with lime
various oxidation states. stone and coke and fed into blast furnace. Reactions taking
(b) Their formation of intermediate complex with reactants place in the blast furnace.
and thus lowering the energy of activation (i) Lower region C + O 2 ® CO 2 + 97, 000 cal.
(c) Their rough surface area provides active sites for
adsorption of reactant molecules. eg. (ii) Middle region
Iron in the preparation of NH 3 (Habers process) CO2 + C ® 2CO - 39000 cal.
Finely divided nickel for hydrogenation CaCO3 ® CaO + CO2 - heat
Pt or V 2 O5 in the preparation of H 2 SO4 CaO + SiO 2 ® CaSiO3 (Slag )
(Contact process)
Fe 2 O 3 + 3CO ® 2Fe + 3CO 2 ­
Pt in the preparation of nitric acid (Ostwald’s process)
Fe 2 O 3 + CO ® 2FeO + CO 2
(xiv)Formation of alloys -d block elements have a strong (iii) Upper region Fe3O 4 + CO ® 3FeO + CO 2
tendency to form alloys since their atomic sizes are very FeO + C ® Fe + CO
similar and in the crystal lattice one metal can be readily The gases leaving the furnace contain CO and used to heat
replaced by another. Alloys so formed are hard, have high incoming air blast.The two layers in the blast furnace are-
m.pts. The metals Mo, W, Cr, Ni, and V are used for the
Upper layer - Molten Iron - It is poured out in moulds and
production of stainless steel and alloy steel.
known as PIG IRON or CAST IRON.It contains 3-5% carbon
Amalgam is an alloy formed by mercury with other metals.
and varying amounts of Mn, Si, P and S which make the
Iron and platinum do not form any alloy with mercury. iron hard and brittle.
(xv) Interstitial compounds - The empty space persent in a
crystal lattice is known as interstitial place. The non metal Lower layer - Molten CaSiO 3 ( slag )
atoms due to their small size (eg H, B, N, C etc.) when 3. Wrought -Iron -It is obtained by heating cast iron with haematite.
occupy such place the resulting compound is known The impurities are oxidised.
interstitial compound. Such compounds are hard and rigid Fe 2 O 3 + 3C ® 2Fe + 3CO ­
e.g. cast iron and steel.
(xvi)Non stoichiometric compounds -The compounds not 2Fe 2 O 3 + 3Si ® 4Fe + 3SiO 2 ­
having the elements in the exact ratio as shown by the
Fe 2O3 + 3Mn ® 2FeO + 3MnO
moleculer formula are known as non stoichiometric
compouds e.g., FeO, CuO etc. In FeO the Fe : O is approx. MnO + SiO 2 ® MnSiO 3 ( slag )
0.94 : 1 and not exactly 1:1. It containes carbon 0.2-0.5% and traces of P and Si. It is pure
form of Iron and soft , malleable, ductile. It is used to make
IRON [Fe, At.No.26, [Ar]3d 4s ]
6 1
magnets in electric cranes and dynamos, railway carriage
Being reactive in nature it does not occur in free state. couplings being corrosion resistant.
1. Ores of Iron- 4. Steel - It contains carbon 0.1-1.5%and manufactured by
Haematite Fe2 O3 following methods.
Magnetite Fe3 O4 (a) Bessemer process - Molten pig iron is heated in large pear
shaped furnace lined with silica bricks at 1873K when
Limonite or hydrated ferric oxide Fe 2 O3 .3H 2 O
impurities such as Mn, Si, C burn off. When all carbon is
Iron pyrites FeS 2 completely burn off the requisite amount of carbon is added.
Siderite FeCO3 Bessemer converter is lined with lime (CaO) or magnesia
(MgO) when pig iron contains high percentage of
copper pyrites CuFeS2
phosphorous.
The d & f-Block Elements 733
(5) Types of steel -
P4 + 5O2 ® P4 O10 (i) Soft steel- contains carbon 0.25%
P4O10 + 6CaO ® 2Ca 3 ( PO 4 )2 (ii) Mild steel - contains carbon 0.25-0.5%
(iii) Hard steel- contains carbon 0.5-1.5 %
Thomas slag (iv) Alloy steel- contains varying percentage of Ni, Cr, Mn,
Co,W, V e.g. stainless steel is an alloy of Fe, Cr and Ni.
(b) Open hearth process- The cast iron, scrap iron , haematite (6) Heat treatment of steel - The hardness of steel depends on its
ore and lime are mixed together and melted in open hearth carbon content and heat treatment.
(i) Quenching- It involves the heating of steel to red hot
furnace lined with SiO 2 or calcined dolomite (MgO. CaO) (1123K) and cooling it by plunging into cold water or oil. It
depending upon the nature of impurities. makes the steel hard and brittle.
(ii) Annealing- The steel is heated well below red heat and
2Fe 2 O3 + 3Si ® 4Fe + 3SiO 2 then cooled slowly. The steel becomes soft.
Fe 2 O 3 + 3Mn ® 2 Fe + 3MnO (iii) Tempering - In this process the quenched steel is reheated
MnO + SiO 2 ® MnSiO 3 (slag ) to 504 to 574K and allowed to cool slowly. The brittleness
P4 + 5O 2 ® P4 O10 disappears and hardness is retained.
P4 O10 + 6CaO ® 2Ca 3 (PO 4 )2 (slag ) (iv) Nitriding- It involves the heating of steel in an atmosphere
of ammonia when surface is coated with iron nitride. The
(c) Electric furnace process - It is combination of Bessemer steel becomes hard.
(v) Case hardening-The steel is heated in charcoal and then
and open hearth process. quenched.The steel becomes hard.
(7) Alloys of steel-The important alloy steels are
Name Composition Uses and Properties
(i) Tungsten steel Fe 94%, W 5%, C 1% It is very hard, resistant to water and used for making
Rock drills and Safetys
(ii) Stainless steel Fe 73% Cr 18%, Ni 8%, C 1% It is resistant to corrosion. Used for making cutlery
(iii) Manganese steel Fe 86%, Mn 13%, C 1% It is hard, used for manufacturing high speed cutting tools
(iv) Invar Fe 64%, Ni 36% It has small coefficent of expansion, used in watches,
meter scales and pendulum rods
(v) Permalloy Fe 21%, Ni 78%, C 1% It is strongly magnetised by electric current and lose
magnetism when current is let off, used for manufacturing
electromagnets and ocean cable.
(vi) Nickel steel Fe96-98%, Ni 2-4% Resistant to corrosion, hard and elastic wire, used for
making cables, gears and drive shafts.

(8) Some chemical properties of Iron- (9) Passivity - The inertness exhibited by metals under conditions
(i) Red hot iron burns in O2 giving sparks when chemical activity is to be expected is called passivity.Iron
becomes passive with conc. HNO3 , Chromic acid ,
3Fe + 2O 2 ® Fe3O 4
(ii) When steam is passed over red hot iron, hydrogen is conc. H2SO4 and KMnO4 etc. It is due to the formation of a
liberated and megnetic oxide of iron (ferroso ferric oxide) is thin layer of oxide at the surface of iron.
formed- (10) Compounds of Iron :
3Fe + 4H 2 O ® Fe3O 4 + 4H 2 (1) Ferric chloride -
(i) It is FeCl3 .6H 2 O
(iii) Action of dil. H 2SO4 -
(ii) Preparation - (i) By passing dry chlorine over heated iron,
Fe + H 2SO 4 ® FeSO 4 + H 2 anhydrous ferric chloride is obtained
(iv) Hot and conc. H 2SO 4 - 2Fe + 3Cl2 ® 2FeCl3 ® Fe 2 Cl6
Fe + 2H 2SO 4 ® FeSO 4 + SO 2 + H 2 O (iii) By the action of hydrochloric acid on ferric hydroxide or
ferric oxide
(v) Action of dil. HNO3 -
Fe ( OH )3 + 3HCl ® 2FeCl 3 + 3H 2 O
4Fe + 10HNO 3 ® 4Fe ( NO3 )2 + NH 4 NO3 + 3H 2O Fe 2 O 3 + 6HCl ® 2FeCl 3 + 3H 2 O
734 Chemistry

It is soluble in water , alcohol and ether. FeCl3 .6H 2 O is yellow.. · Reducing nature - It is strong reducing is nature.
Its aqueous solution is acidic. Sublimes at 300ºC, covalent and 2 KMnO 4 + 3H 2 SO 4 ® K 2 SO 4 + 2MnSO 4 + 3H 2 O + 5O
dissociates above 973K first into FeCl3 then FeCl 2 and Cl2 2 FeSO 4 + H 2 SO 4 + O ® Fe 2 (SO 4 )3 + H 2 O
e.g. · Addition compound with NO which is dark brown
973K 973K
Fe2Cl6 2 FeCl3 2 FeCl2 + Cl2 D
above FeSO 4 + NO ® FeSO 4 .NO ¾¾® FeSO 4 + NO

Structure Cl Cl Cl · FeSO4 + 6KCN ® K4 éë Fe (CN )6 ùû + K 2SO 4


Fe Fe Uses - As mordant in dyeing, insecticide and in the preparation
of Mohr’s salt.
Cl Cl Cl 3. Ferrous ammonium sulphate (Mohr’ s salt) -

· Oxidising nature of FeCl3 is shown by following reactions FeSO 4 ( NH 4 )2 SO 4 .6H 2O


Preparation - By mixing saturated solutions of
2FeCl 3 + H 2S ¾
¾® 2 FeCl 2 + 2 HCl + S
FeSO 4 and ( NH 4 ) 2 SO4 at about 50o C and cooling.
2 FeCl 3 + SO 2 + 2H 2 O ¾
¾® 2FeCl 2 + H 2SO 4 + 2 HCl
Properties - It is light green crystalline compound and does
· ¾® Fe(OH )3 ¯ +3NH 4Cl (brown ppt )
FeCl3 + 3NH 4 OH ¾ not effloresce.
4. Ferric oxide Fe2 O3 - In nature it occurs as haematite.
· FeCl 3 + 3NH 4CNS ¾
¾® Fe Used in Bosch process as calalyst and polishing powder by
jewellers and as red pigment.
® Fe(SCN)3 + 3NH 4Cl (blood red colour)
5. Iron sulphide FeS - By heating iron fillings with dil. H 2SO4 ,
· 4FeCl 3 + 3K 4 [Fe(CN )6 ] ¾
¾® FeS + H 2SO 4 ® FeSO 4 + H 2S
The reaction is carried out in Kipp’s apparatus. FeSO4 is
® Fe 4 [Fe(CN )6 ]3 + 12KCl (prussion blue )
obtained as by product. H 2 S finds an extensive
Ferric cyanide
application in analytical chemistry. It has smell of rotten eggs.
· Action of heat -
2FeCl 3 .6H 2 O ¾¾® Fe 2 O 3 + 6HCl + H 2 O COPPER - 29 Cu [ Ar ]3d 10 4 s1
Uses - Used in medicine as ASTRINGENT and ANTISEPTIC. Its It occurs in nature in large quantities in Michegan (USA).
concentrated solution is used for etching copper and silver. Important ores are

2Fe 3+ + Cu ® 2 Fe 2+ + Cu 2 + (i) Copper glance Cu 2S

Fe 3+ + Ag ® Fe 2 + + Ag + (ii) Copper pyrites CuFeS2

2. Ferrous Sulphate FeSO4 .7 H 2O (Green vitriol) : (iii) Malachite Cu ( OH ) 2 CuCO3


Prepartion (iv) Cuprite Cu 2O (Ruby copper) (v) Azurite
(i) Fe + dil H 2SO 4 ® FeSO4 + H 2 Cu ( OH )2 .2CuCO 3
(ii) Manufacture - From iron pyrites by oxidation by air Extraction - It is mainly extracted from copper pyrites.
2FeS2 + 2H 2 O + 7O 2 ® 2FeSO 4 + 2H 2SO 4 Concentration - The ore is concentrated by froth floatation
process.
Properties - Hydrated FeSO 4 .7H 2O is green and
Roasting - The concentrated ore is strongly heated by hot
anhydrous FeSO4 is colourless. blast of air on the hearth of reverberatory furnace. The following
· Action of heat - changes take place
300 0 C strong
® FeSO 4 + 7H 2 O ¾¾ ¾® Fe 2 O 3 + SO 2 + SOS3 + O 2 ® SO 2 ­
FeSO 4 .7H 2 O ¾¾ ¾¾
300 C
7H 2 O ¾¾¾¾
strong
® FeSO 4 + 7H 2 O ¾¾ ¾® Fe 2 O 3 + SO 2 + SO 3 4As + 3O 2 ® 2As 2 O 3 ­

· Its aqueous solution is acidic due to cationic hydrolysis 4Sb + 3O 2 ® 2Sb 2 O 3 ­


2CuFeS2 + O 2 ® Cu 2S + 2FeS + SO 2
Fe + + + 2 H 2O Fe (OH )2 + 2 H +
2FeS + 3O 2 ® 2FeO + 2SO 2
The d & f-Block Elements 735
Smelting- The roasted ore is mixed with sand and heated in The impurities like Zn, Fe, Ni,Co remain in solution being more
blast furnace. electropositive in nature and Ag, Au, Pt, (less electropositive)
2FeS + 3O 2 ® 2FeO + 2SO 2 collect below the anode in the form of anode mud or slime
FeO + SiO 2 ® FeSiO 3 ( slag ) 99.99% pure copper is obtained. Anode mud provides about
25% of U.S. Silver production and 13% of U.S. gold production.
Cu 2 O + FeS ® FeO + Cu 2S Properties of copper - In aqueous solution it has two oxidation
The mixture of copper and iron sulphides melt together to form states +1 (cuprous) and +2 (cupric). Cu (I) salts tend to be white
“matte”. and insoluble in water while many salts of Cu (II) are water
Bessemerisation - The molten matte mixed with little sand is solubale however Cu(OH)2 is insoluble. CuS is one of the least
poured into Bessemer converter. The following changes take soluble compounds.
place · Cu (I) disproportionates easily in aqueous solution.
2FeS + 3O 2 ® 2FeO + 2SO 2
2Cu + ® Cu 2+ + Cu
FeO + SiO 2 ® FeSiO3 ( slag )
· Action of O 2 - It is attacked by O 2 first forming copper (I)
2Cu 2S + 3O 2 ® 2Cu 2 O + 2SO 2
2Cu 2 O + Cu 2S ® 6Cu + SO 2 oxide Cu 2O (Red) and then copper (II) oxide CuO (Black).
The copper thus produced is called ‘blister copper’ and contains · It forms a green layer of basic carbonate in presence of
2.0% impurities of Ag, Au, Ni, Zn, Pb, Sn, As, S etc. CO2 and moisture.
Refining - It is carried out by either of the following methods
(i) By polling - The melt is stirred vigrously with green poles of 2Cu + H 2 O + CO2 + O2 ® Cu ( OH )2 .CuCO3
wood and oxides are reduced by hydrocarbons emanating
from wood. · Not attacked by dilute acids e.g. HCl and H 2SO4 but
(ii) Electrolytic refining of copper- Slabs of impure copper are dissolves in these acids in presence of air.
made anode and thin sheets of pure copper as the cathode.
2Cu + 2H 2SO 4 ( dil.) + O 2 ® 2CuSO 4 + 2H 2O
Acidic copper sulphate is used as electrolyte.
CuSO 4 Cu + + + SO4- - Cu + 2H 2SO4 ( Conc. ) ® CuSO 4 + SO 2 + 2H 2O
3Cu + 8HNO3 ( dil.) ® 3Cu ( NO3 )2 + 2NO + 4H 2O
At Cathode Cu 2+ + 2e - ® Cu
Cu + 4HNO3 ( Conc.) ® Cu ( NO3 )2 + 2NO 2 + 2H 2O
At anode Cu ® Cu 2+ + 2e-
Alloys of Copper :
Name Composition Properties Uses
1 Bronze Cu-75-90% Brittle, hard Coins, statues Blades
Sn 10-25% resistant to corrosion
2 Bell metal Cu 80% Hard brittle Bells
Sn 20%
3 Gun metal Cu 88% Very hard
Sn 10% resistant to Corrosion For making guns,
Zn 2% canons machine parts
4 Brass Cu 60-80% Malleable and Utensils, casting,
Zn 20-40% ductile, hard cartriges
5 Aluminum bronze Cu-90% light, strong Jewellary, coins,
Al-10% resistant to corrosion frames
6 Monel metal Cu-30% very resistant to Automobile,
Ni-67% chemical action engine parts
Fe+Mn-3% high tensile strength
7 German silver or Cu-50% White shining Utensils
nickel silver Zn-30% malleable, ductile Ornaments
Ni-20% resistant to corrosion
8 Silica bronze Cu-97% very hard Telephone and
Sn-2% non corrosive telegraph wires
Si-1% good conductor
9 Phosphorous bronze Cu-85% very hard Suspension wire
Sn-13% tough non instrument, gears
P-2% corrosive ships, propellors
736 Chemistry
Compounds of copper : Cuprous chloride - Copper (I) chloride ( CuCl or Cu 2 Cl 2 )
Copper Sulphate CuSO 4 .5H2O (Blue vitriol or nila thotha) - Preparation -
Preparation - (i). By dissolving Cu (II) oxide or carbonate in (i) It can be prepared from Cu alone or in combination
dilute H 2SO4 . with CuO,CuCl2 , CuSO4 by action of concentrated
CuO + H 2SO 4 ® CuSO 4 + H 2 O hydrochloric acid.
CuCO3 + H 2 SO4 ® CuSO 4 + CO 2 + H 2 O 2Cu + 2HCl + O ® Cu 2 Cl2 + H 2O
Manufacture -Form scrap copper CuO + Cu + 2HCl ® Cu 2 Cl 2 + H 2 O
2Cu + 2H 2SO4 ( dil, hot ) + O2 ( air ) ® 2CuSO 4 + 2H 2 O CuCl2 + Cu ® Cu 2 Cl2

Properties - Blue crystalline compound. CuSO 4 + Cu + 2HCl ® Cu 2Cl 2 + H 2SO 4


(i) Action of heat - (ii) By passing SO2 in a solution of CuSO4 and NaCl
373 K
CuSO 4 .5H 2 O ¾¾¾® CuSO 4 .H 2O ¾¾¾®
423 K 2CuSO4 + 2NaCl + SO 2 + 2H 2O ® Cu 2Cl 2 + Na 2SO4 + 2H 2S
Blue Bluish white 2CuSO4 + 2NaCl + SO2 + 2H 2O ® Cu 2Cl2 + Na 2SO4 + 2H 2SO4
Properties - It is a white solid, almost insoluble in water.
strong
CuSO 4 ¾¾¾ ® CuO + SO3 · Action of conc. HCl - It dissolves forming soluble complex.
White Cu 2Cl 2 + 2HCl ® 2H [CuCl 2 ]

(ii) Action of NH 4OH - or Cu 2Cl 2 + 6HCl ® 2H3CuCl4


On dilution white precipitate again appears.
CuSO 4 + 2 NH 4 OH ® Cu (OH )2 + (NH 4 )2 SO 4
· Action of ammonia - It dissolves forming soluble complex
Cu (OH )2 + 2 NH 4 OH + (NH 4 )2 SO 4 Cu 2 Cl 2 + 4NH3 ® 2Cu ( NH3 )2 Cl
· Action of acetyline - Red precipitate of cuprous acetylide is
® éëCu ( NH3 ) 4 ùû SO 4 + 4H 2 O obtained.
Cu 2Cl 2 + C2 H 2 + 2NH 4 OH ® CuC º CCu + NH 4 Cl + 2H 2 O
Tetrammine copper sulphate is known as SCHWITZER’S
REAGENT. It is used to dissolve cellulose in theCu 2 Cl 2 + C2 H 2 + 2NH 4OH ® CuC º CCu + NH 4Cl + 2H 2O
manufacture of artificial silk. · With carbon monoxide it forms addition product.
(iii) Action of KI - Cu 2Cl 2 + 2CO ® 2CuCl.CO
· With air - In air it is slowly oxidised to green basic cupric
2CuSO 4 + 4KI ® Cu 2 I2 + 2K 2SO4 + I 2
chloride.
(It does not react with KCl, KBr or KF) 3CuO.CuCl2 .3H 2 O
(iv) Action of potassium ferrocyanide -
· With NaOH -
2CuSO4 + K 4 é Fe ( CN )6 ù ® Cu 2 é Fe ( CN )6 ù + 2K 2SO 4 Cu 2 Cl 2 + 2NaOH ® Cu 2 O + 2NaCl + H 2 O
ë û ë û
Chocolate coloured pricipitate Yellow ppt. changing to red
(v) Action of KCN -
· With H 2S - 2CuCl + H 2S ® Cu 2S + 2HCl
2CuSO4 + 10KCN ® 2K3 éë Cu ( CN )4 ùû + 2K 2SO4 + ( CN )2 black ppt.

(vi) Structure of CuSO 4 .5H 2 O - Uses - In gas analysis for absorbing C 2 H 2 and CO. In

H 2O OH 2 H O O combination with NH 4 Cl as catalyst for synthetic rubber..


Cupric Chloride Copper (II) Chloride CuCl2 .2H 2O -
Cu O S
Preparation -
OH 2 O (i) Form copper , cupric oxide or copper carbonate by the
H 2O H O
action of conc. HCl.
Four H 2 O with cation and fifth with anion. 2Cu + 4HCl + O 2 ® 2CuCl + 2H 2 O
Uses - In electroplating, as mordant in dyeing. CuO + 2HCl ® CuCl 2 + H 2 O
CuCO 3 + 2HCl ® CuCl2 + CO 2 + H 2 O
· Bordeaux mixture -(Mixture of CuSO4 .5H 2O + Lime) is
(ii) Anhydrous cupric chloride is prepared by burning copper
known as Bordeaux mixture.It is used as fungicide. In the in current of chlorine
preparation of Fehling solution and electric battries.
Cu + Cl2 ® CuCl2
The d & f-Block Elements 737

Properties - Hydrated CuCl2 .2H 2O ® greenish SILVER (Ag)


It is found in nature and in combined state.
Anhydrous CuCl 2 ® Brown powder Principal ores are -
· Aqueous dilute solution is blue due to complex (i) Argentite (silver glance) Ag 2S
(ii) Horn silver AgCl,
[Cu.(H2O)4 ]2+ (iii) Pyrargyrite (ruby silver) Ag 2S.Sb2S3.
In small quantities in lead ,copper and zinc ores.
· Concentrated solution is green due to complex [ CuCl4 ]2-
1. Extraction- Mac Arther Forest’s cyanide process -
· With Ammonia - First a precipitate which dissolves in (i) Concentraction- Ore is concentrated by froth floatation
process.
excess of NH 4 OH . (ii) Treatment with NaCN - The powdered ore is treated with
CuCl 2 + NH 4 OH ® Cu ( OH ) 2 ¯ + NH 4 Cl ® Cu NH3 blueNaCN solution (0.7%) and air is bubbled through the mixture.
2+ (a) 8NaCN + 4Ag + 2H 2 O + O2
Cl 2 + NH 4OH ® Cu OH ¯ + NH 4Cl ® éëCu ( NH3 )4 ùû blue
® 4Na éë Ag ( CN ) 2 ùû + 4NaOH
· Action of heat - 2CuCl2 ® Cu 2 Cl2 + Cl2
(b) AgCl + NaCN ® AgCN + NaCl
anhydrous
AgCN + NaCN ® Na éë Ag ( CN )2 ùû
· Hydrated salt on heating gives
3CuCl2 .2H 2 O ® CuO + Cu 2Cl 2 + 2HCl + Cl 2 + 5H 2 O (c) Ag 2 S + 2 NaCN Na 2 S + 2 AgCN
AgCN + NaCN ® Na [Ag (CN ) 2]
Structure -
Reversible reaction is prevented by oxidation of Na 2S
Cl Cl Cl by air.
Cu Cu Cu 4Na 2S + 2H 2 O + 5O 2 ® 2Na 2SO 4 + 2S + 4NaOH
(iii) Precipitation of silver - It is done with zinc.
Cl Cl Cl
Zn + 2Na éë Ag ( CN ) 2 ùû + 4NaOH ®

Cl Na 2 ZnO2 + 4NaCN + 2Ag + 2H 2O


Cl
(iv) Refining - Electrolytic method
Cu Cu Cu
Anode - impure silver (Ag ® Ag + + e - )
Cl Cl
Cathode - pure silver (Ag + + e - ® Ag)
Electrolyte - AgNO3 solution + 10% HNO3
Cuprous oxide - Cu 2O (Red oxide of copper) 2. Silver from argentiferous lead (Desilverisation of lead). Lead,
Preparation - When Fehling solution is reduced by glucose extracted form galena (PbS) contains small amount of silver and
is called argentiferous lead. Silver is recovered from it by
or aldehyde.
(i) Parke’s process - Molten argentiferous lead is shaken with
CuSO 4 + 2NaOH ® Na 2 SO 4 + Cu ( OH )2 ® CuO + H 2 O zinc when whole of silver passes into zinc. On cooling Ag-
Zn alloy solidifies and being lighter floats over molten lead.It
2CuO + HCHO ® Cu 2 O + HCOOH
is separated, melted and distilled.Zinc distills over and silver
Properties - Red colour ,insoluble in water.It forms stable is left behind. Success of the method depends upon the
complexes. fact that
(a) Silver is more soluble in molten zinc and
Uses- In making ruby red glass and enamel.In manufacturing
(b) Molten Zn and lead are immiscible
anti rust paints. (ii) Pattison’s process - (When silver is less than 1.0%).The
Cupric oxide (black oxide of copper) - lead-silver mixture containing 2.6%. Silver melts at lower
Preparation - By heating malachite which is native copper temperature than pure lead. When molten argentiferous
carbonate. CuCO 3 .Cu ( OH ) 2 ® 2CuO + H 2 O + CO 2 lead is allowed to cool pure lead solidifies first and removed.
The silver content of the mixture is allowed to raise to
Properties - Black powder reduced to metallic copper 2.6%.The silver is then recovered by cupellation.
Properties - It is a noble metal not attacked by atmospheric oxygen.
by H2 or CO .
The surface is tarnished due to fromation of Ag 2S due to H 2S
Uses - In the manufacture of glass.It gives green colour to glazes present in air.
and glass.
4Ag + 2H 2S + O2 ® 2Ag 2S + 2H 2O
738 Chemistry
· Dissolves in dilute and concentrated nitric acid GOLD
3Ag + 4HNO 3 ( dil.) ® 3AgNO 3 + 2H 2 O + NO ­ Occurrence - It occurs free as Reef gold, Vein gold or auriferous
quartz. Some improtant ores are
Ag + 2HNO 3 ( conc. ) ® AgNO 3 + H 2 O + NO 2 ­
(i) Claverite AuI2 ,
· Dissolves in alkalicyanide
(ii) Sylvanite ( Ag.Cu ) Te 2
4Ag + 8KCN + 2H 2 O + O 2 ® 4K éë Ag ( CN )2 ùû + 4KOH
(iii) Auriferous pyrites. These are sulphide ores of Cu, Ag, lead
· Dissolves in conc. sulphuric acid (not in dil. sulphuric acid) which contain gold.
2Ag + 2 H 2SO 4 ® Ag 2SO 4 + 2H 2 O + SO 2 ­ Extraction - By cyanide or Mac Arther Forest cyanide process
Uses- For making ornaments (80%Ag+20%Cu), electroplating, -
preparation of mirrors. (i) Concentration - Sulphides and tellurides are concentrated
Fineness- It is the amount of silver present in 1000 parts of silver by froth floatation process.
alloy. 925 fine silver means an alloy of 92.5% silver and 7.5%copper. (ii) Roasting - The concentrated ore is roasted to remove
Compounds of silver - oxidisable impurities of Te, As and S.
Silver nitrate or Lunar caustic AgNO3 : (iii) Formation of complex- NaCN solution is sprayed over
the crushed ore and the gold with air, forming complex ion
Preparation - By the action of dilute nitric acid on silver.
in solution.
3Ag + 4HNO3 ® 3AgNO3 + NO + 2H 2O
4Au + 8NaCN + 2H 2 O + O 2 ®
Properties - It is colourless crystalline solid, soluble in water. It leaves
black deposit when rubbed on the skin due to formation of finely
4Na éë Au ( CN )2 ùû + 4NaOH
divided silver.
· Action of heat - solution
2AgNO3 ® 2AgNO2 + O2 ­ The gold is then recovered as a solid by reduction.
D 2Na éë Au ( CN )2 ùû + Zn ® Na 2 éë Zn ( CN ) 4 ùû + 2Au
2AgNO3
¾¾ ®2Ag + 2NO2 + O2 ­
Strong
(iv) Parting - Removal of impurities of Ag and Cu from gold is
· Tollen’s reagent - The ammonical silver nitrate solution is
known as parting.Impure gold is boiled with conc. H 2SO4
known as Tollen’s reagent.
when Ag and Cu dissolve and Au remains unaffected.
2AgNO 3 + 2 NH 4 OH ® Ag 2 O ¯ + 2 NH 4 N O 3 + H 2 O
(v) Purification - By electrolytic method using gold chloride
Ag 2 O + 4 NH 4 OH ® 2 é Ag ( NH 3 )2 ù OH + 3H 2 O 2.5-6.0% and conc. HCl.
ë û
· Reaction with aqueous solution of certain compounds. · Plattner chlorine extraction process - From auriferous pyrites.
The moistened auriferous pyrites is saturated with chlorine,
NaCl leached with water then treated with FeSO4 or H 2S which
¾¾¾® AgCl(white precipitate)
precipitates gold. Auriferous pyrites (moistened)
¾¾¾® AgBr(pale yellow)
NaBr
+Cl2 ® AuCl3
¾® AgI (yellow)
NaI
¾¾
AuCl3 + 3FeSO4 ® Au + FeCl3 + Fe 2(SO 4 )3
® Ag2CrO4 (red )
K CrO
¾¾2¾¾
4
2AuCl3 + 3H 2S ® 2Au + 6HCl + 3S
¾¾¾¾® AgCNS (white)
NaCNS
Impurities of Ag and Cu are removed by parting (as above).
¾¾¾¾® Ag3 PO 4 (yellow)
Na3 PO 4
Properties - Pure gold is soft, hardened by Ag or Cu.
AgNO3 Fineness of gold - It is expressed in terms of carats. Pure gold is
® Ag2 S2 O3 ¾¾2¾® Ag2 S (black)
Na S O H O
¾¾2¾2¾
¾3
24 carats. 22 carats mean it contains 22 parts by
¾¾¾® AgCN ¾¾¾® K [Ag(CN )2 ]
KCN KCN
weight of gold and 2 parts by weight of other metals generally
Na S copper.
¾¾2¾® Ag2 S
It is very inert and not attacked by oxygen ,water and acids.
Cu
¾¾® Ag deposited · It is attacked by aqua regia (3 conc. HCl + 1 conc. HNO3 )
I 2 (excess )
¾¾¾ ¾
¾® AgI + HIO3 + HNO3
3HCl + HNO 3 ® NOCl + 2H 2 O + 2Cl
Uses - For silvering mirror ,electroplating, in medicines, for the Au + 3Cl ® AuCl3
preparation of silver halides used in photography. Particularly
· It is attacked by chlorine also.
AgBr which is most sensitive to light.
2Au + 3Cl2 ® 2AuCl3
2AgBr ® 2Ag + Br2
The d & f-Block Elements 739

· AuCl3 auric chloride forms red crystals. Soluble in water and · Chemical properties -
(i) Action with acids
decomposed on heating. Zn + H 2SO4 ( dilute ) ® ZnSO4 + H 2 ­
AuCl3 ® AuCl + Cl 2 4Zn + 5H 2SO 4 ( 20%H 2SO4 ) ® 4ZnSO 4 + H 2S + 4H 2 O
light yellow powder 3Zn + 4H 2SO4 ( 20%H 2SO 4 ) ® 3ZnSO 4 + S + 4H 2 O
ZINC (Zn) : Zn + 2H 2SO4 ( hot and conc.) ® ZnSO 4 + SO2 + 2H 2 O
Zinc - It is not found free in nature.The principal ores are - 4Zn + 10HNO3 ( cold and dilute ) ® 4Zn ( NO3 )2 + NH 4 NO3 + 3H 2 O
(i) Zinc blende (sphalerite) ZnS
3Zn + 8HNO 3 (mode. conc. ) ¾
¾® 3Zn (NO )3 + 2 NO + 4H 2 O
(ii) Zincite or Red zinc oxide ZnO
Zn + 4HNO 3 (conc. hot ) ¾
¾® ZnNO 3 + 2 NO 2 + 2 H 2 O
(iii) Franklinite ZnO.Fe 2O3
Zn + 2 NaOH ¾ ¾® Na 2 ZnO 2 + H 2
(iv) Calamine or Zinc spar ZnCO3 · Displacement reactions
(v) Willemite 2ZnO.SiO2 Zn + CuSO 4 ® ZnSO4 + Cu ¯
Extraction - It is extracted by reduction process from ZnS (Zinc Zn + Pb ( NO3 )2 ® Zn ( NO3 ) 2 + Pb
blende).
Concentration - The ore is concentrated by froth flotation Zn + 2Na éë Ag ( CN )2 ùû ® Na 2 éë Zn ( CN ) 4 ùû + 2Ag
process.
Roasting - The concentrated Zinc blende is roasted in a current Zn + 2Na éë Au ( CN )2 ùû ® Na 2 éë Zn ( CN ) 4 ùû + 2Au
of air.
· With non metals
2ZnS + 3O2 ® 2ZnO + 2SO2
Zn + Cl 2 ® ZnCl 2
( SO2 is utilised for the manufacturing of H 2SO4 ) Zn + S ® ZnS
If calamine ore is used, it is calcined. · It is powerful reducing in nature.
ZnCO3 ® ZnO + CO2 Fe 2 ( SO 4 )3 + Zn ® 2FeSO 4 + ZnSO 4

Reduction - The ZnO is reduced by mixing with carbon and Uses- Galvanising, sherardizing, in Parke’s process for
heating in fire clay retort. desilverisation of lead , for extraction of Ag and Au
(Cyanide proces). Zinc compounds are used in paints, filling
ZnO + C ® Zn + CO rubber etc.
Purification - Zinc so obtained contains the impurities of Fe, Compounds of Zn
Pb, Cd, As or Sb. It is purified by ZnO - Zinc oxide, Philospher’s wool, zinc white or chinese white.
(a) Distillation or buring
(b) Electrolytic method Preparation - (i) Zn + air ¾¾¾¾ ® ZnO
D
Anode impure ( Zn ) Zn - 2e- ® Zn ++ (ii) ZnCO3 ¾¾ ® ZnO + CO2
D
Cathode pure ( Zn ) Zn ++ + 2e ® Zn (iii) Zn ( NO3 )2 ¾¾ ® ZnO + NO 2 + O 2
Properties - It is white powder becomes yellow on heating but
Electrolyte - Acidic solution of ZnSO4 .
again white on cooling. It sublimes at 673K.
(c) Liquation - Molten Zn is allowed to flow down on sloping · With alkali - It forms zincate.
hearth when non fusible impurities are left behind.
ZnO + 2NaOH + H 2O ® Na 2 éë Zn (OH )4 ùû
Electrolytic method - Pure ZnSO4 is electrolysed when Zn is
deposited on aluminium cathode. It is scrapped off and melted Sodium Zincate
to obtain 99.95% pure metal. · Reduction ZnO + H 2 ® Zn + H 2 O
· Zinc dust -It is prepared by atomising molten zinc with
ZnO + C ® Zn + CO
blast of air.
· Dissolves in acids to form corresponding salts.
· Granulated Zinc - It is prepared by pouring molten Zinc
into cold water. ZnO + H 2SO4 ® ZnSO 4 + H 2 O
Properities- It is bulish white metal, stable in air. Uses - As a white paint , in medicines, glaze in ceramics and filler
· In moist air a protective covering of basic zinc carbonate is in rubber industry.
ZnSO 4 .7H 2O (White vitriol) -
formed at its surface 2ZnCO3 .3Zn ( OH ) 2 .H 2O
· Action of heat -When heated to 500ºC it catches fire with Preparation - By the action of dil. H 2SO4 on Zinc metal, ZnO
bluish white flames forming ZnO which is very light and or ZnCO3
called philospher’s wool.
( Zn, ZnO or ZnCO 3 ) + dil. H 2 SO 4 ® ZnSO 4 .7H 2 O
740 Chemistry
Properties - Colourless, crystalline compound, highly soluble
· It forms mercuric oxide at 300o C .
in water.
Action of heat - 2Hg + O 2 ® 2HgO
373K 723K
ZnSO4 .7H 2 O ¾¾¾® ZnSO4 .H 2 O + 6H 2 O ¾¾¾® ZnSO 4 anhydrous
· Dilute acids have no +action
ZnO SO 2 +onOmercury
2 except dil. HNO3 .
6 Hg + 8HNO 3 ® 3Hg 2 (NO 3 )2 + 2 NO + 4H 2 O
H 2 O + 6H 2O ¾¾¾® ZnSO 4 ( anhydrous ) Strong
¾¾¾
D
® ZnO + SO 2 + O 2
· With concentrated acids

Uses - It is used to prepare lithophone (BaSO4 + ZnS) white Hg + 4HNO 3 ® Hg ( NO 3 ) 2 + 2NO 2 + 2H 2 O

pigment , galvanising iron and steel, as mordant in Hg + 2H 2 SO 4 ® HgSO 4 + 2SO 2 + 2H 2 O


calico printing , in medicine as eye lotion. · Deadening of Hg - On Shaking vigrously alone or with fats
or sugar it changes to grey powder. This is called deadening
Zn (II) Chloride ZnCl 2 .2H 2O of mercury.
Preparation - It is prepared by the action of dilute HCl on Zn, · Tailing of mercury- In presence of ozone it loses its
meniscus which is known as tailing of mercury.
ZnO or ZnCO3
· Amalgams- The alloys of mercury with metals excepts (Fe
Zn + 2HCl ® ZnCl 2 + H 2 ­ and Pt) are commonly known as amalgams.
ZnO + 2HCl ® ZnCl 2 + H 2 O · Ammonium amalgam - Sodium amalgam when placed in
ZnCO 3 + 2HCl ® ZnCl2 + CO 2 + H 2 O conc. solution of NH 4 Cl , there is swelling and butter like
mass is formed which is ammonium amalgam.
Properties - It is very deliquescent, soluble in water and organic
· Mercury tree - When small amount of Hg is poured into
solvents.
AgNO3 solution. (Ag-Hg) is formed which grows like a
Anhydrous zinc chloride
tree and called mercury tree.
D
ZnCl 2 .2H 2 O ¾¾® ZnCl 2 + 2H 2 O Uses- In thermometers , barometers, electric cells etc.
Uses - As timber preservative, flux in soldering, preparation of Compounds of mercury -
vulcanised paper and fibre. 1. Mercuric oxides HgO
MERCURY (Hg) Preparation - (i) By heating mercury in air or O2
Occurrence - It occurs in free state as small quantities. Its chief
ore is - D HgO (Red )
(i) Cinnabar HgS
Hg + excess air ¾¾®
623K
(ii) Tiemannite
(ii) By heating mercuric nitrate
(iii) Calomel Hg2Cl2.
Extraction - 2Hg ( NO 3 )2 ® 2HgO + 4NO 2 + O 2
(i) Concentration - Cinnabar ore is concentrated by froth
(iii) From mercuric chloride by the action of NaOH
floatation process.
(ii) Roasting- Roasting is carried out in a shaft furnace when HgCl 2 + 2NaOH ® HgO + 2NaCl + H 2 O
mercury is obatined by auto reduction. yellow
2HgS + 3O 2 ® 2HgO + 2SO 2 Red HgO and yellow HgO differ in their particle size. On heating
2HgO ® 2Hg + O 2 yellow form changes to red.
Roasting may be carried out with iron scrap or quicklime. Uses - It is used as pigment in oil paints and as mild antiseptic in
ointments.
HgS + Fe ® FeS + Hg
2. Mercurous chloride or calomel Hg 2 Cl2
4HgS + 4CaO ® 4Hg + 3CaS + CaSO 4
Preparation - (i) From mercurous nitrate
(iii) Purification- It contains the impurities of Zn, Pb, Sn or Bi.
Some of these impurities get oxidised in air and form a black Hg 2 ( NO 3 )2 + 2NaCl ® Hg 2 Cl 2 + 2 NaNO 3
scum on the surface. Finally it is purified by distillation in (ii) From mercuric chloride by action of mercury
vacuum.
HgCl 2 + Hg ® Hg 2Cl 2
Properties - It is silvery white liquid also known as Quick
silver or live silver. It is the heaviest liquid known. Sp. gr. Properties - It is insoluble in water purified by sublimation.
· Action of NH 3 - It becomes black with ammonia.
13.59 at 0 o C .
· Action of air - It is not attacked by air either dry or moist at Hg 2 Cl 2 + 2 NH 3 ® [ Hg + Hg ( NH 2 ) Cl] + NH 4 Cl
ordinary temperature. Black
The d & f-Block Elements 741
· Action of heat - It is dicomposed. Potassium permanganate KMnO 4 ;
Hg 2 Cl2 ® HgCl2 + Hg Preparation - On large scale it is prepared from
Uses - In ceramics for golden colour and as calomel electrode. pyrolusite (MnO2 ) . The steps involved are as follows
3. Mercuric chloride or corrosive sublimate HgCl 2 - (i) Preparation of potassium manganate K 2 MnO 4 - By
Preparation -
fusing manganese dioxides with KOH or K 2CO3 .
(i) By heating mercury in a current of chlorine.
Fusion
Hg + Cl 2 ® HgCl 2 2MnO 2 + 4KOH + O 2 ¾¾¾¾
® 2K 2 MnO 4 + 2H 2O
(ii) Manufacture - By heating mercuric sulphate with equal Fusion
quantity of sodium chloride. 2MnO 2 + 2K 2 CO3 + O 2 ¾¾¾¾
® 2K 2 MnO 4 + 2CO 2
green mass
HgSO 4 + 2NaCl ® HgCl 2 + Na 2SO4
Properties - Colourless, crystalline substance, covalent in nature (ii) Oxidation of K 2 MnO 4 to MnO 4
and gives 5-8% solution in water.
To K2 MnO4 solution is either added
(i) With SnCl2 it is reduced to mercury (i) H2SO4 or
2HgCl2 + SnCl2 ® Hg 2 Cl 2 + SnCl4 (ii) passed CO2 or
Hg 2 Cl2 + SnCl 2 ® 2Hg + SnCl4 (iii) passed O3 or
(ii) With KI -It gives scarlet precipitate soluble in excess of KI. (iv) passed Cl2
HgCl2 + 2KI ® HgI2 + 2KCl 3K 2 MnO 4 + 2H 2SO 4 ¾
¾®
Scarlet ppt. 2K 2SO 4 + 2KMnO 4 + MnO 2 ¯ +2 H 2 O
HgI2 + 2KI ® K 2 HgI4 3K 2 MnO 4 + 4CO 2 + 2H 2 O ¾
¾®
Clear solution
2KMnO 4 + MnO 2 ¯ +4KHCO 3
(Nessler 's solution)
¾® 2KMnO 4 + 2KOH + O 2
2K 2 MnO 4 + O 3 + H 2 O ¾
(iii) With NaOH - It gives HgO.
HgCl 2 + 2 NaOH 2 NaCl + HgO + H 2O ¾® 2KMnO 4 + 2KCl
2K 2 MnO 4 + Cl 2 ¾
Nessler’s reagent - An alkaline solution of K 2 Hg I4 is known (The last oxidation is known as STADELER ‘s process)
(iii) Electrolytic oxidation - Now a days elecrtrolytic oxidation
as Nessler’s reagent. It is used for the identification of ammonia
is prefered. The manganate solution is electrolysed between
and ammonia salts.
iron electrodes separated by diaphragm.
2K 2 HgI4 + 3KOH + NH3 ®
MnO24- ® MnO -4 + e -
I O NH 2 At anode green purple
Hg Hg 3KI 2 H 2O
Brown (iodide of Millon’s base)
At cathode 2H + + 2e - ® H 2
Mercuric iodide ( HgI 2 ) - Properties - It is dark purple solid, soluble in water giving purple
Preparation - (i) By adding KI solution to any mercuric salt solution. Its melting point is 523 K.
solution. Chemical properties -
HgCl2 + 2KI ® HgI 2 + 2KCl (i) Action of heat - 2KMnO4 ® K 2MnO4 + MnO2 + O2
scarlet (ii) Oxidising nature - It is strong oxidising agent , both in
Above 400K alkali as well as in acidic medium and also in neurtral. In
Scarlet HgI 2 HgI 2 Yellow
below acidic medium -
It is soluble in excess of KI forming complex ion [ HgI 4 ]2 - .Its 2 KMnO 4 + 3H 2SO 4 ¾ ¾® K 2 SO 4 + 2MnSO 4 + 3H 2 O + 5O
or MnO -4 + 8H + + 5e - ¾
¾® Mn 2 + + 4H 2 O
alkaline solution is Nessler’s reagent as shown above.
Mercuric sulphate HgSO4 - S 2- ¾¾® S + 2e -
Preparation - (i) By treating Hg with conc. H 2SO4 SO 2 + 2H 2 O ¾ ¾® SO 24 - + 4H + + 2e -
2-
SO 3 + H 2 O ¾ ¾® SO 24- + 2H + + 2e -
Hg + 2H 2SO 4 ® HgSO 4 + SO 2 + 2H 2 O
NO 2- + H 2 O ¾ ¾® NO3- + 2H + + 2e -
Properties - It is white opaque mass, decomposes on heating 2+
Fe ¾ ¾® Fe 3+ + e -
to gives Hg (I) Sulphate.
C 2 O 4 2- ¾
¾® 2CO 2 + 2e -
3HgSO4 ® Hg2SO4 + Hg + 2SO2 + O 2
H 2O2 ¾ ¾® O 2 + 2 H + + 2e -
742 Chemistry
In alkaline medium -
Hot concentrated solution of ( Na 2Cr2 O7 ) Potassium
2KMnO 4 + 2KOH ® 2K 2 MnO 4 + H 2 O + O dichromate is less soluble and seperates out on
or MnO -4 + e - ® MnO24 - crystallisation.
In neutral medium - Properties - It is garnet red prismatic (orange) crystalline
compound having melting point 398°C. Soluble in water.
2KMnO4 + H 2 O ® 2KOH + 2MnO2 + 3O Chemical properties -
or MnO -4 + 2H 2 O + 3e - ® MnO2 + 4OH - (i) Action of heat -
In alkaline medium it oxides potassium iodide to potassium iodate 4K 2 Cr2 O 7 ® 4K 2 CrO 4 + 2Cr2 O3 + 3O 2
and nitro toluene to nitro benzoic acid. Chromic oxide

I - + 6OH - ® IO3- + 3H 2 O + 6e - (ii) Action of cold H 2SO4 -


K 2 CrO7 + 2H 2SO4 ® 2KHSO4 + 2CrO3 + H 2 O
C6 H 4 <CH 2 +7OH - ® C6H 4 < +5H 2O + 6e -
NO NO2
-
3 COO
( Red crystals of chromic anhydride )
(iii) Action of hydrogen - It burns on heating in a current of H 2 . (iii) Action of alkali -
2KMnO4 + 5H 2 ® 2KOH + 2MnO + 4H 2O K 2 Cr2 O7 + 2KOH ® 2K 2 CrO 4 + H 2 O
(iv) Equivalent weight of KMnO4 in different medium - (iv) Oxidising nature - It is powerful oxidising in nature.

Molecular weight K 2 Cr2 O 7 + 4H 2SO 4 ¾


¾®
(a) Equivalent weight in acid medium =
5
(b) Equivalent weight in alkaline medium ® K 2SO 4 + Cr2 (SO 4 )3 + 4H 2 O + 3O

Molecular weight or Cr2 O 7 2 - + 14 H + + 6e - ¾


¾® 2Cr 3+ + 7 H 2 O
=
1
thus ¾® SO 24 - + 2 H + + 2e -
SO 2 + 2 H 2 O ¾
(c) Equivalent weight in neutral medium
Fe 2+ ¾
¾® Fe 3+ + e -
Molecular weight
= C 2 O 4 2- ¾
¾® 2 CO 2 + 2e -
3
(See ionic equations above) ¾® O 2 + 2 H + + 2 e -
H 2O2 ¾
Uses - As oxidising agent , disinfectant, 1% alkaline solution of (v) Formation of chromyl chloride - When a chloride is heated
KMnO4 is used to test unsaturation in organic compounds with potassium dichromate and conc. H 2SO4 orange red
under the name of Baeyer’s reagent. It is used for the volumetric vapour of chromyl chloride are formed.
estimation of Fe + + salts,oxalic acid etc. K 2 Cr2O7 + 4NaCl + 6H 2SO4 ®
Potassium dichromate ( K 2Cr2 O7 ) 2CrO 2Cl2 + 2KHSO4 + 4NaHSO4 + 3H 2 O
(vi) With lead salts it gives insoluble chromate salt.
It is manufactured from chromite ore ( FeCr2 O 4 ) . The steps
2Pb ( NO3 )2 + K 2Cr2O7 + H 2O ®
involved are -
(i) Preparation of sodium dichromate - Finely powdered 2PbCrO4 + 2KNO3 + 2HNO3
chromite is mixed with soda ash and quick lime and roasted Uses - (i) In chrome tanning, (ii) In dyeing-calico
in reverberatory furnace or rotatory furnace in exess of air. printing, (iii) In photography (iv) chromic acid
4Fe ( CrO2 ) 2 + 8Na 2CO3 + 7O2 ® ( mixture of K 2 Cr2 O 7 + H 2 SO 4 ) used as cleaning
agent, (v) In preparation of compounds such as
8Na 2 CrO 4 + 2Fe2O3 + 8CO2
K 2 SO 4 .Cr2 ( SO 4 )2 .24H 2 O, CrO 2 Cl 2 etc.
Chromite can be fused with molten alkali in presence of air.
Structure - It consists of two tetrahedra with common oxygen
4Fe ( CrO2 )2 + 16NaOH + 7O2 ® atom
O
8Na 2CrO4 + 2Fe2O3 + 8H 2O O
The solution is filtered and acidified with dil. H2SO4 when
sodium dichromate is obtained. Cr
Cr O O
2Na 2CrO4 + H 2SO 4 ® Na 2Cr2 O7 + Na 2SO 4 + H 2O
O O– O
(ii) Conversion of sodium dichromate to potassium dichromate. –
Na 2CrO7 + 2KCl ® 2NaCl + K 2 Cr2 O7 Dichromate ion Cr2 O 72 -
The d & f-Block Elements 743
Structure of chromate ion : It has tetrahedral structure (iii) Magnetic properties - Magnetic properties have spin and
O– orbit contributions (Contrast “spin only”of transition
metals). Hence magnetic momentums are given by the
Cr
O formula.
O
O– = 4 S (S + 1) + L(L + 1)
2–
Chromate ion CrO4
where L= Orbital quantum number, S = Spin quantum number
At pH about 4 dichromate ion ( Cr2 O 72 - ) and chromate ion All lanthanide ions with the exception
3+ 3+ 4+
of La , Lu and Ce are paramagnetic in nature. The
( CrO 24- ) exist in equilibrium. These are interconvertible.
trend in magnetic moment is shown by graph.
2– Acid – Acid 2– 10 Ho
Er
2CrO4 + 2H 2HCrO4 Cr2 O7 + H2O Dy
(yellow) Alkali Alkali (orange) 8 Tb
Tm
INNER TRANSITION ELEMENTS
6 Gd
The elements in which the filling of atomic orbitals by electrons Yb
take place in f subshells, two levels inside the outer subshell,
4
are known as inner transition elements. Thus these elements Pr
Nd
form a series within the transition series. They are also known
2 Ce
as f- block elements since the differentiating electron enters the Eu
Sm
f -subshell.
Classification of f-block elements - They have been classified 0 La
58 60 62 64 66 68 70 Lu
into two series. Atomic number ®
(i) 4 f-series (first inner transition series) - The differentiating
(iv) Lanthanide contraction - There is a steady decrease in the
electron enters in 4 f orbitals. The elements belonging to
radii as the atomic number of the lanthanide
this series are also known as Lanthanides or Lanthanones.
elements increases. For every additional proton added in
(ii) 5 f -series (second inner transition series) -The
differentiating electron enters in 5 f orbitals. The elements nucleus the corresponding electron goes to 4 f subshell.
belonging to this series are also known as Actinides or The shape of f -orbitals is very much diffused and they
actinones. have poor shielding effect. The effective nuclear charge
For the sake of symmetry of the periodic table they have been increases which causes the contraction in the size of electron
placed outside the periodic table. charge cloud. This contraction in size is quite regular and
Lanthanides - The fifteen elements from lanthanum (At. no. 57) known as Lanthanide contraction.
to lutetium (At. no. 71) are known as lanthanides or rare earths
(because they were obtained as earths (oxides) from relatively
rare minerals).
The properties of inner transition elements are : following
(i) Electronic Configuration - The general electronic
configuration of these elements is
0 -1
[ Xe ] 4f 0 -14 5d 6s 2
1
The lanthanum electronic configuration [ Xe ] 4f 0 5d 6s 2 and
lutetium electronic configuration [ Xe ] 4f 14 5d1 6s 2 , have no
partially filled 4 f orbital in their ground state, are considered as
lanthanides due to their properties close to these elements.
(ii) Oxidation state - The common oxidation state of lanthanides
(v) Consequences of lanthanide contraction -
is +3 but some elements also exhibit +2 and +4
(a) Covalent character of cations increase.
oxidation states in which they leave behind stable ions eg.
(b) Electronegatively - The electronegativity of trivalent
Eu 2+ [ Xe ] 4f 7 , Yb 2+ [ Xe ] 4f 14 ions increase slightly.
Ce 4+ [ Xe ] 4f 0 , Tb [ Xe ] 4f 7 (c) Basicity - There is decrease in basic strength of oxides
and hydroxides.
An aqueous solution of Ce 4+ is a good oxidising agent. (d) Eovalue - There is small increase in standard electrode
The Eu 2 + and Yb2+ can exist in aqueous solution and are potential values.
(v) Colour - The species containing unpaired electrons are
good reducting agents. But there are exceptions also e.g.
coloured and so is the case with lanthanide ions. The f-
Sm 2 + [ Xe ] 4f 6 Tm 2+ [ Xe ] 4f 13 f transitions are possible due to absorption of light from
Pr 4 + [ Xe] 4f 1 Nb [ Xe ] 4f 2 the visible region.
744 Chemistry
(vii) Melting and boiling point - Lanthanides have high (x) Ionisation energies - The ionisation energy values of
melting and boiling points but there is no regular trend. lanthanides are not very high due to their large size
(viii) Density - Lanthanides have densities varying and are comparable with those of alkaline earth metals.
(xi) Complex compound - Due to having large ionic size
from 6.77 to 9.74 g cm -3 . But there is no definite trend they have little tendency to form complexes.
for these values. (xii) Reactivity - Due to their low values of ionisation
energies the lanthanides are very reactive.
(ix) TheElement electronegatively values of lanthanides
(xiii) Alloys - They form alloy especially with iron e.g. MISCH
are almost same as that of s-block elements.
METAL rare earths 94 - 95% iron 5% and
Lanthanides form ionic compounds.
S , C , Ca and Al in traces .
Atomic Name Symbol Electronic Oxidation Density Melting Boiling M 3+ radii
number configuration states (g/ml) point (K) point (K) (pm)
57 Lanthanum La [Xe]4 f 5d 6s
0 1 2 +3 6.17 1193 3742 106

58 Cerium Ce [Xe]4 f 15d 1 6s 2 +3,+4 6.77 1068 3741 103


59 Praseodymium Pr [Xe]4 f 3 6 s 2 +3,+4 6.78 908 3400 101

60 Neodymium Nd [Xe]4 f 4 6s 2 +2,+3 7.00 1297 3300 100

61 Promethium Pm [Xe]4 f 5 6 s 2 +3 7.20 ------ ----- 98

62 Samarium Sm [Xe]4 f 6 6 s 2 +2,+3 6.93 1345 2173 96

63 Europium Eu [Xe]4 f 7 6s 2 +2,+3 5.24 1099 1712 95

64 Gadolinium Gd [Xe]4 f 7 5d 16s 2 +3 7.95 1585 3273 94

65 Terbium Tb [Xe]4 f 9 6s 2 +3,+4 8.33 1629 3073 92

66 Dysprosium Dy [Xe]4 f 10 6s 2 +3,+4 8.56 1680 2873 91

67 Holmium Ho [Xe]4 f 11 6s 2 +3 8.76 1734 2873 89

68 Erbium Er [Xe]4 f 12 6s 2 +3 9.16 1770 3173 88

69 Thulium Tm [Xe]4 f 13 6s 2 +2,+3 9.35 1818 2000 87

70 Ytterbium Yb [Xe]4 f 14 6s 2 +2,+3 7.01 1097 1700 86

71 Lutetium Lu [Xe]4 f 14 5d 1 6s 2 +3 9.74 1925 3600 85

ACTINIDES (iii) Magnetic properties - The magnetic moments


of actinide ions are smaller than theoretical values.
Actinides - The fifteen elements from actinium (At. no. 89) to It is hard to interpret due to large spin orbit coupling.
lawrencium (At. no. 103) are known as actinides and constitute (iv) Actinide contraction - It is similar to lanthanide
the 5f. Series. From neptunium to onwards the elements are contraction due to poor shielding of 5 f electrons.
man made (artificially prepared) and also known as trans
(v) Melting and boiling points - They have high values for
uranium elements.
melting and boiling points but there is no regular trend.
(i) Electronic configuration - The differentiating electron (vi) Density - The value of density vary from 7.0 gcm–3 to
enters the 5 f atomic orbital. Their general electronic 19.84 gcm–3. Again there is no regular trend.
configuration is (vii)Reducing character -They are strong reducing agents
[ Rn]5 f 0-14 6d 0- 2 7 s 2 as they have high E 0 values approximately 2.0 volts.
Since there is not much difference between 5 f and 6d, it (viii)Reactivity - Actinides are very reactive in nature and
is difficult to predict whether the electron has entered 5 combine with oxygen and halogens like lanthanides.
f or 6d. (ix) Coloured ions - Actinide ions are coloured due to the
(ii) Oxidation state - The common oxidation state is +3 but presence of unpaired electrons and f - f transitions.
other oxidation states are also exhibited by actinides the (x) Complex formation - They have higher tendency to form
maximum being +7. complex compounds.
The d & f-Block Elements 745

Very Short/Short Answer Questions 16. Complete the following chemical equations:
1. Why is HCl not used to acidify a permanganate solution in (i) MnO-4 ( aq ) + S2O32 - ( aq ) + H 2 O ( l ) ®
volumetric estimations of ferrous ion (Fe2+) or oxalate ion
(C2O42–)? (ii) Cr2 O72- ( aq ) + Fe 2+ ( aq ) + H + ( aq ) ®
2. Why does V2O5 act as catalyst?
3. What is the oxidation state of chromium in (a) potassium 17. State reason for the following :
dichromate (b) potassium chromate? (i) Cu (I) ion is not stable in an aqueous solution.
4. The d-electronic configuration of Co2+ and Cu2+ is d7 and d9 (ii) Unlike Cr3+, Mn2+, Fe3+ and the subsequent other M2+
respectively. Which one of these ions will be more ions of the 3d series of elements, the 4d and the 5d
paramagnetic? series metals generally do not form stable cationic
5. Lanthanum (Z = 57), gadolinium (Z = 64) and lutetium (Z = species.
71) are extra-ordinarily stable in + 3 oxidation state. Explain. 18. How would you account for the following:
6. Why KMnO4 is used in clearing surgical instruments in (i) Among lanthan oids, Ln (III) compounds are
hospitals? predominant, However, occasionally in solutions or in
7. Write chemical equation for the conversion of: solid compounds, + 2 and + 4 ions are also obtained.
(a) Chromite ore to sodium chromate
(ii) The E° for copper is positive (0.34 V). Copper
(b) Pyrolusite to potassium manganate M 2 + /M
(c) Potassium permanganate to manganese dioxide is the only metal in the first series of transition elements
8. There is only a very small increase in IE of an element in a showing this behaviour.
transition series (or d-block elements) when we move along (iii) The metallic radii of the third (5d) series of transition
the period as compared to s and p-block elements. Explain, metals are nearly the same as those of the correspond-
why. ing members of the second series.
9. Among lanthanides, why are ions like Sm2 +, Eu2+ and Yb2+ 19. (a) Which metal in the first transition series (3d series)
in aqueous solutions good reducing agents but an aqueous exhibits + 1 oxidation state most frequently and why?
solution of Ce4+ is a good oxidising agent? Explain. (b) Which of the following cations are coloured in aque-
10. (a) Which oxidation state of Mn is most stable and why? ous solutions and why?
(b) Which transition element show highest oxidation state
Sc3+, V3+, Ti4+, Mn2+
and in which compound?
(At. nos. Sc = 21, V = 23, Ti = 22, Mn = 25)
(c) Which is the densest transition element?
11. Which of the following oxides is basic, amphoteric and acidic Long Answer Questions
oxide: Sc2O3, MnO, FeO, Cu2O, MnO2, CuO, TiO2, Mn2O7,
CrO3. Cr2O3 20. The sums of first and second ionisation energies and those
12. A substance is found to have a magnetic moment of 4×89 of third and fourth ionisation energies of nickel and platinum
BM. How many unpaired electrons does it contain? are given below:
Element IE1 + IE2 (kJ mol–1) IE3 + IE4 (kJ mol–1)
13. Write the IUPAC name of éëCo ( NH3 )5 Cl ùû Cl 2 . [Atomic no. Ni 2×49 8×80
of Co = 27] Pt 2×66 6×70
14. Compare the chemistry of the actinoids with that of Taking these values in account, write
lanthanoids with reference to (a) the most common oxidation states of Ni and Pt and its
(i) electronic configuration reason.
(ii) oxidation states (b) the name of the metal (Ni or Pt) which can form
(iii) chemical reactivity compounds is + 4 oxidation state more easily and why?
15. Write the structures and names of all the stereoisomers of 21. Explain the following:
the following compounds: (a) Transition elements show similarity in properties along
the period as well as down the group (i.e., horizontal as
(i) é Co ( en ) ù Cl3 well as vertical similarities).
ë 3û
(b) In transition series, it is observed that melting point
(ii) é Pt ( NH3 ) Cl2 ù first increases and then starts decreasing.
ë 2 û
(c) Copper is a transition element but not sodium.
(iii) éë Fe ( NH3 ) 4 Cl 2 ùû Cl (d) Zinc atom is bigger than copper atom.
(e) Transition metals have low reactivity.
746 Chemistry
22. (a) Describe with the help of chemical equations the 26. Cuprous compounds such as CuCl, CuCN and CuSCN are
preparation of potassium dichromate from chromite ore. the only salts stable in water due to
(b) Give balanced chemical equations for what happens (a) high hydration energy of Cu+ ions
when acidified potassium dichromate solution reacts
(b) their inherent tendency not do disproportionate
with
(i) ferrous sulphate solution. (c) diamagnetic nature
(ii) hydrogen sulphide gas. (d) insolubility in water
(c) Draw the structure of Cr2O72–. 27. The addition of excess of aqueous HNO3 to a solution
containing [Cu(NH3)4]2+ produces
Multiple Choice Questions (a) Cu+
23. Which element belongs to d-block ? (b) [Cu(H2O)4]2+
(a) Na (b) Ca (c) Cu(OH)2
(c) Cu (d) Ar (d) Cu(NO3)2
24. (n - 1)d 10ns 2 is the general electronic configuration of 28. An extremely hot copper wire reacts with steam to give
(a) Fe, Co, Ni (b) Cu, Ag, Au (a) CuO (b) Cu 2O
(c) Zn, Cd, Hg (d) Se, Y, La (c) Cu 2 O 2 (d) CuO2 .
25. Which of the following is not property of the transition 29. Which is not amphoteric ?
elements?
(a) Al 3+ (b) Cr 3 +
(a) Colour (b) Paramagnetism
(c) Fixed valency (d) None of these (c) Fe 3+ (d) Zn 2+

1. How many ‘d’ electrons are present in Cr 2 + ion? (b) All three are colourless
(a) 4 (b) 5 (c) All three are coloured
(c) 6 (d) 3 (d) Only Ni 2 + is coloured an d Zn 2+ and Cr 3+ are
2. Which of the following ions having following electronic colourless
structure would have maximum magnetic moment?
7. (n - 1)d 10ns 2 is the general electronic configuration of
(a) 1s 2 2 s 2 2 p 6 3s 2 3 p 6 3d 3 (b) 1s 2 2s 2 2 p 6 3s 2 3 p 6 3d 5
(a) Fe, Co, Ni (b) Cu, Ag, Au
2 2 6 2 6 7 2
(c) 1s 2s 2 p 3s 3 p 3d (d) 1s 2s 2 p 3s 3 p 3d 2 6 2 6 9 (c) Zn, Cd, Hg (d) Se, Y, La
8. Common oxidation state of scandium, a transition element
3. Which of the following element does not belong to first
transition series? is/are [Ar.No.of Sc = 21]
(a) Fe (b) V (a) +4 (b) +1
(c) Ag (d) Cu (c) +2 and +3 (d) +4 and +1
4. The transition metals have a less tendency to form ions 9. Which of the following is not correct about transition metals?
due to (a) Their melting and boiling points are high
(a) high ionisation energy (b) Their compounds are generally coloured
(b) low heat of hydration of ions (c) They can form ionic or covalent compounds
(c) high heat of sublimation (d) They do not exhibit varialbe valency
(d) All of these 10. Which of the following has maximum unpaired
5. Which of the following oxides of manganese is amphoteric? electrons?
(a) Scandium (b) Chromium
(a) MnO 2 (b) Mn 2O3
(c) Manganese (d) Iron
(c) Mn 2O7 (d) MnO 11. The last electron in d-block elements goes to
(a) (n-1) d (b) nd
6. Of the ions Zn 2+ , Ni 2+ and Cr 3+ (c) np (d) (n-1) s
( At.Nos Zn = 30, Ni = 28,Cr = 24) 12. Which of the following ions is coloured?
(a) Cu + (b) Cu 2+
(a) Only Zn 2+ is colourless and Ni 2 + and Cr 3+ are
coloured (c) Ti4+ (d) V5+
The d & f-Block Elements 747
13. What is wrong about transition metals? 26. Elements which generally exhibit multiple oxidation states
(a) Diamagnetic and whose ions are usally coloured are
(b) Paramagnetic (a) metalloids (b) non-metals
(c) Form complexes (c) transition elements (d) gases
(d) Shows variable oxidation state 27. Transition metals mostly are
14. In 3d-series atomic number (Z) varies from (a) diamagnetic
(a) Z = 21- 30 (b) Z = 22 - 30 (b) paramagnetic
(c) Z = 20 - 30 (d) Z = 31 - 40 (c) neither diamagnetic nor paramagnetic
15. The first element in the 3d-transition series is (d) both diamagnetic and paramagnetic
(a) Sc (b) Ti 28. Transition metals usually exhibit highest oxidation states in
(c) V (d) Ca their
16. Which one of the following ionic species will impart colour (a) chlorides (b) fluorides
to an aqueous solution? (c) bromides (d) iodides
(a) Ti 4+ (b) Cu + 29. Which one of the following ions exhibit highest
magnetic moment?
(c) Zn 2 + (d) Cr 3+ (a) Cu 2+ (b) Ti3+
17. Formation of interstitial compound makes the
transition metal (c) Ni 2 + (d) Mn2+
(a) more soft (b) more ductile 30. Which of the following is colourless in water?
(c) more metallic (d) more hard
18. The elements which exhibit both vertical and horizontal (a) Ti3+ (b) V 3+
similarites are (c) Cu 3+ (d) Sc3+
(a) inert gas elements (b) representative elements
31. The valence shell of transition elements consists of
(c) rare elements (d) transition elements
(a) nd orbitals (b) (n-1) d orbitals
19. Oxidation state of osmium (Os) in OsO4 is (c) ns np nd orbitals (d) (n-1) d ns orbitals
(a) +4 (b) +6 32. Which one of the following is diamagnetic ion?
(c) +7 (d) +8 (a) Co 2+ (b) Cu 2+
20. Which of the following transition element shows the
highest oxidation state ? (c) Mn2+ (d) Sc3+
(a) Mn (b) Fe 33. Which group contains coloured ions out of
(c) V (d) Cr
21. Which of the following has more unpaired d-electrons? 1. Cu 2+ 2. Ti 4+
(a) Zn + (b) Fe 2 + 3. Co 2+ 4. Fe 2 +
(c) Ni + (d) Cu + (a) 1, 2, 3, 4 (b) 1, 3, 4
(c) 2, 3 (d) 1, 2
22. An atom has electronic configuration
34. If a non metal is added to the interstital sites of a metal, then
1s 2 2s 2 2 p 6 3s 2 3 p 6 3d 3 4s 2 in which group would it be the metal becomes
placed? (a) softer (b) less tensile
(a) Fifth (b) Fifteenth (c) less malleable (d) more ductile
(c) Second (d) Third 35. Transition metal with low oxidation number will act as
23. Iron loses its magnetic property at (a) a base (b) an acid
(a) Curie point (b) Boiling point (c) an oxidising agent (d) None of these
(c) Melting point (d) 1000 K 36. Which of the following elements does not show
24. The number of unpaired electrons in a nickel atom in ground variable oxidation states?
state are (At. No. of Ni = 28) (a) Copper (b) Iron
(a) 2 (b) 5 (c) Zinc (d) Titanium
(c) 3 (d) 7 37. Which one of the following is an example of non-typical
25. Zinc and mercury do not show variable valency like d-block transition elements ?
element because (a) Li, K, Na (b) Be, Al, Pb
(a) they are soft (c) Zn, Cd, Hg (d) Ba, Ga, Sr.
(b) their d-shells are complete 38. Super conductors are derived from compounds of
(c) they have only two electrons in the outermost shell (a) p-Block elements (b) lanthanides
(d) their d-shells are incomplete (c) actinides (d) transition elements
748 Chemistry
39. Corrosion of iron can be prevented by coating the surface 52. The oxidation number of Fe in Fe3O 4 is
with
(a) +2 (b) +3
(a) Zn (b) Sn
(c) 8/3 (d) 2/3
(c) Ni (d) Any of these
53. The iron obtained from blast funace is
40. Which of the following compound is called Turnbull’s blue?
(a) pig iron (b) silver
(a) Ferricyanide (b) Ferrous ferricyanide
(c) soft iron (d) steel
(c) Ferrous cyanide (d) Ferri-ferrocyanide
54. Which of the following gives blood red colour with
41. In an octahedral crystal field splitting the number of orbitals
KCNS?
pushed down in energy is
(a) 3 (b) 2 (a) Cu 2+ (b) Fe3+
(c) 5 (d) zero
(c) Al3+ (d) Zn 2 +
42. Which one of the following pairs of ions have the same
55. The protection of steel by chrome plating is due to
electronic configuration?
(a) cathodic protection
(a) Cr3+ ,Fe3+ (b) Fe3+ , Mn 2+ (b) anodic protection
(c) covering of steel surface
(c) Fe3+ , Co3+ (d) Sc3+ ,Cr3+ (d) formation of alloy with iron
43. Bonding in ferric chloride is 56. The nature of an aqueous solution of FeCl 2 is
(a) covalent (b) ionic (a) acidic (b) basic
(c) co-ordinate (d) None of these (c) amphoteric (d) neutral
44. Which one of the following elements constitutes a major 57. Which one of the following is the correct configuration of
impurity of pig iron?
Fe3+ ( Z = 26 ) ?
(a) Silicon (b) Oxygen
(c) Sulphur (d) Graphite (a) [Ar ]4s 2 3d 6 (b) [Ar ]4s 2 3d 3
45. A compound of a metal ion M x + ( Z = 24 ) has a spin (c) [Ar ]3d 5 (d) [Ar ]4s 2 3d 4
only magnetic moment of 15 Bohr Magnetons. The number 58. The number of ions formed on dissolving one molecule of
of unpaired electrons in the compound are FeSO 4 ( NH 4 )2 SO 4 .6H 2O
(a) 2 (b) 4
(Mohr’s salt) is/are
(c) 5 (d) 3
(a) 6 (b) 3
46. Which of the following configuration is correct for iron?
(c) 5 (d) 4
(a) 1s 2 2s 2 2 p 6 3s 2 3 p 6 3d 5 59. If steel is heated to a temperature well below red heat and is
then cooled slowly the process is called
(b) 1s 2 2s 2 2 p 6 3s 2 3 p 6 4s 2 3d 5 (a) tempering (b) hardening
(c) softening (d) annealing
(c) 1s 2 2s 2 2 p 6 3s 2 3 p 6 4s 2 3d 7
60. Which of the following is obtained when Fe reacts with
(d) 1s 2 2s 2 2 p 6 3s 2 3 p 6 4s 2 3d 6 dil. HNO3 ?
47. Stainless steel contains (a) N 2O (b) NO
(a) Fe + Cr + Cu (b) Fe + C + Ni
(c) NO2 (d) None of these
(c) Fe + Cr + Ni + C (d) Fe + Ni + Cu 61. Wrought iron, pig iron and steel differ in properties due to
48. Which of the following is not an element? (a) carbon content (b) malleability
(a) Graphite (b) Diamond (c) conductivity (d) softness
(c) 22-Carat gold (d) Rhombic sulphur 62. Green vitriol is
49. Which of the following elements does not exist in the native
(a) FeSO 4 .7H 2O (b) ZnSO 4 .7H 2O
form?
(a) Au (b) Pt (c) CaSO 4 .2H 2O (d) CuSO 4 .5H2O
(c) Fe (d) S 63. Which of the following is not an ore of iron?
50. In the extraction of iron, slag is produced which is (a) Limonite (b) Magnetite
(a) CO (b) FeSiO3 (c) Casiterite (d) All of these
64. Corrosion is basically a
(c) MgSiO3 (d) CaSiO3
(a) altered reaction in presence of H2O
51. The nature of an aqueous solutionof FeCl3 is (b) electrochemical phenomenon
(a) acidic (b) basic (c) interaction
(c) amphoteric (d) neutral (d) union between light metal and heavy metal
The d & f-Block Elements 749
65. Which of the following are the constituents of gun metal?
76. A metal which does not liberate H2 ( g ) from acids is
(a) Cu, Sn (b) Cu, Sn, Zn
(c) Zn, Sb, Pb (d) Cu, Ca, Pb (a) Cu (b) Fe
(c) Mn (d) Zn
66. When Cu reacts with AgNO3 solution, the reaction takes
77. Cuprous ion is colourless while cupric ion is coloured
place is because
(a) oxidation of Cu (b) reduction of Cu (a) both have half filled p-and d-orbitals
(c) oxidation of Ag + (d) reduction of NO3- (b) cuprous ion has incomplete d-orbital and cupric ion has
a complete d-orbital
67. Bordeaux used as fungicide is a mixture of
(c) both have unpaired electrons in the d-orbitals
(a) CuSO 4 + Ca (OH )2 (b) CaSO 4 + Cu (OH )2 (d) cuprous ion has complete d-orbital and cupric ion has
an imcomplete d-orbital.
(c) CuCO3 + Cu (OH )2 (d) CuO + CaO
78. In Cu ( Z = 29)
68. Which of the following ions will liberate iodine when
treated with KI? (a) 13 electrons have spin in one direction and 16 electrons
in other direction
(a) Cu 2+ (b) Fe 2 + (b) 14 electrons have spin in one direction and 15 electrons
(c) Pb 2 + (d) Sn 2+ in other direction
(c) All the electrons have spin in one direction
69. Which one of the following pairs of substances on
(d) None of these
reaction will not evolve H2 gas? 79. Cuprous ore among the following is
(a) Iron and H 2SO 4 (aq.) (a) chalcopyrites (b) azurite
(c) cuprite (d) malachite
(b) Iron and steam
80. The white anhydrous copper sulphate decomposes to give
(c) Copper and HCl ( g ) (a) CuSO 4 .5H2O (b) CuSO4.H 2O
(d) Sodium and ethyl alchohol
(c) CuO + SO3 (d) Cu
70. Which of the following metals is sometimes found in native
state in nature? 81. An important ore of copper is
(a) Al (b) Cu (a) bauxite (b) galena
(c) Fe (d) Mg (c) siderite (d) chalcopyrites
71. Which of the following is ionic in nature? 82. When the sample of copper with zinc impurity is to be purified
by electolysis, the appropriate electordes are
(a) CuF2 (b) CuCl2 Cathode Anode
(c) CuBr2 (d) None of these (a) Pure zinc Pure copper
(b) Impure sample Pure copper
72. CuSO4 reacts with KCN solution and forms
(c) Impure zinc Impure sample
(a) Cu (CN ) (b) Cu (CN )2 (d) Pure copper Impure sample
83. Which of the following compounds gives red
(c) K 3 éëCu (CN )4 ùû (d) K 4 [Cu (CN )6 precipitate with AgNO3 ?
73. When KI and CuSO4 solutions are mixed, it gives (a) KI (b) K 2CrO 4
(a) K 2SO 4 + I2 (b) Cu 2 I2 + K 2SO 4 (c) NaBr (d) NaNO3
(c) CuI2 + K 2SO4 (d) Na 2SO4 + I2 84. When silver nitrate is heated to red hot, what is formed?
74. The final step in the metallurgical extraction of Cu metal from (a) Ag (b) Ag 2O
Cu pyrites takes place in a Bessemer converter. The reaction
taking place is (c) Ag 2O3 (d) AgO 2
85. When horn silver ore is dissolved in excess of sodium
(a) Cu 2S + O 2 ® 2Cu + SO2
cyanide solution compound formed is
(b) 4Cu 2O + FeS ® 8Cu + FeSO4
(a) AgCN (b) Na [ AgCN)2 ]
(c) 2Cu 2O + Cu 2S ® 6Cu + SO 2
(c) Na 2 éë Ag (CN )3 ùû (d) Na 4 éë Ag (CN )5 ùû
(d) Cu 2S + 2FeO ® 2CuO + 2Fe + SO2
75. Matte contains mainly 86. In the chemical reaction
(a) Cu 2S and FeS (b) CuS and Fe 2S3 Ag 2O + H2O + 2e- ® 2Ag + 2OH-
(a) water is oxidised (b) water is reduced
(c) Fe (d) Cu 2S (c) silver is oxidised (d) silver is reduced
750 Chemistry
87. AgCl is soluble in NH4OH solution. The solubility is due to 98. Acidified solution of chromic acid on treatment with
the formation of H 2O2 gives blue colour which is due to
(a) AgOH (b) Ag 2O (a) CrO3 + H 2O + O2 (b) CrO5 + H 2O
+ (c) H 2Cr2O7 + H 2O + O2 (d) None of these
(c) éë Ag ( NH3 )2 ùû (d) NH 4Cl
99. In the extraction of zinc, the formation of blue flame is due
88. The compound that is soluble in water is the burning of
(a) AgF (b) AgCl (a) ZnO (b) C
(c) AgBr (d) AgI (c) Zn (d) CO
89. In the metallurgy of which of the following cupellation 100. Which of the following compounds turns black on
process is used? addition of NH 4OH ?
(a) Copper (b) Silver
(a) PbCl2 (b) Hg2Cl2
(c) Iron (d) Aluminium
90. When potassium ferrocyanide crystals are heated with (c) HgCl 2 (d) AgCl
conc. H 2SO 4 the gas evolved is 101. Mercury is the only metal which is liquid at 0°C. This is due
to its
(a) SO2 (b) NH3 (a) very high ionisation energy and weak metallic bond
(c) CO2 (d) CO (b) low ionisation potential
91. In the extraction of silver, Ag 2S is dissolved in (c) high atomic weight
(d) high vapour pressure
(a) HCl (b) HNO3
102. Which one of the following is false for Hg?
(c) KCN (d) H2SO4 (a) It can evolve hydrogen from H2S
92. Parke’s process is used (b) It is metal
(a) to extract Ag using NaCN (c) It has high specific heat
(b) to extract Cu using CuFeS2 (d) It is less reactive than hydrogen.
(c) to extract Ag from argentiferrous lead 103. Mercury sticks to the surface of the glass when it comes in
(d) to extract Ag by forming amalgam contact with
93. In the equation (a) Hg2O (b) HNO3
(c) O3 (d) grease
-
4M + 8CN - + 2H 2O + O2 ® 4 éë M (CN )2 ùû + 4OH - 104. Which compound is volatile on heating?
(a) MgCl2 (b) Hg Cl2
Identify the metal M. It is (c) ZnCl2 (d) None of these
(a) copper (b) iron 105. Which ion is detected by Nessler’s reagent?
(c) gold (c) zinc
(a) NH +4 (b) MnO-4
94. A metal which is not affected by conc. H 2SO 4 , HNO3
or alkalies forms an compound X This compound X can be (c) PO34- (d) CrO24 -
used to give a complex which finds its application for tonning 106. Mercurous ion is represented as
in photography. The metal is (a) Hg 22 + (b) Hg 2 +
(a) Au (b) Ag
(c) Hg (d) Cu (c) Hg + Hg 2 + (d) 3Hg3+
95. Which of the following is not the characteristic of zinc? 107. Nessler’s reagent is
(a) It is a volatile metal (a) potassium iodide in mercuric iodide
(b) It dissolves in alkali forming sodium zincate (b) TiCl 4
(c) It is brittle at very high temperatures
(d) Zinc dust is used as reducing agent (c) anhydrous AlCl3
96. Galvanization of iron denotes coating with (d) Al 2O3 / Cr2O3
(a) Al (b) Sn 108. The equivalent weight of KMnO4 (formula weight = M )
(c) Cd (d) Zn when it is used as an oxidant in neutral medium is
97. When Zn is treated with excess of NaOH, the product
M
obtained is (a) M (b)
2
(a) Zn (OH )2 (b) ZnOH
M M
(c) (d)
(c) Na 2 ZnO2 (d) None of these 3 5
The d & f-Block Elements 751
109. The starting material for the manufacture of KMnO4 is 120. The colour of CuCr2O7 solution in water is green because?
(a) pyrolusite (b) manganite (a) Cr2 O72 - ions are green
(c) magnatite (d) haematite
110. An explosion take place when conc. H2SO4 is added to (b) Cu 2+ ions are green
KMnO4. Which of the following is formed? (c) Both Cr2O2– 2+
7 and Cu ions are green
(a) Mn 2O7 (b) MnO 2 (d) Cu2+ ions are blue and Cr2 O72 - ions are yellow
(c) MnSO4 (d) M 2O3 121. The most common lanthanide is
(a) lanthanum (b) cerium
111. If KMnO4 is reduced by oxalic acid in an acidic
(c) samarium (d) plutonium
medium then oxidation number of Mn changes from
122. Total number of inner transition elements in the periodic table
(a) 4 to 2 (b) 6 to 4 is
(c) +7 to +2 (d) 7 to 4 (a) 10 (b) 14
112. The correct formula of permanganic acid is (c) 28 (d) 30
(a) HMnO4 (b) HMnO5 123. The electronic configuration of gadolinium (At. No. 64) is
(c) H 2 MnO4 (d) H 2 MnO3 (a) [ Xe]4 f 8 5d1 6s 2 (b) [ Xe]4 f 7 5d1 6s 2
113. In the preparation of KMnO4 pyrolusite (MnO2 ) is first (c) [ Xe]4 f 3 5d 5 6s 2 (d) [ Xe]4 f 6 5d 2 6 s 2
converted to potassium manganate ( K 2MnO4 ). In this 124. Which of the following element is responsible for oxidation
of water to O2 in biological process?
conversion, the oxidation state of manganese changes from
(a) Fe (b) Mn
(a) +1 to +3 (b) +2 to +4 (c) Cu (d) Mo
(c) +3 to +5 (d) +4 to +6 125. Which of the following compounds is not coloured?
114. Which of the following oxides of Cr is amphoteric
(a) Na 2CuCl4 (b) Na 2CdCl4
(a) CrO2 (b) Cr2O3
(c) CrO5 (d) CrO3 (c) K 4 Fe(CN)6 (d) K3Fe(CN)6
115. In potassium manganate the oxidation state of 126. The +3 ion of which one of the following has half filled 4f
manganese is subshell?
(a) +5 (b) +6 (a) La (b) Lu
(c) +7 (d) +8 (c) Gd (d) Ac
127. Which of the following metal is found in gun metal, monel
116. Number of electrons transfered in each case when KMnO4 metal and constant alloys?
acts as an oxidising agent to give MnO2, Mn2+, Mn(OH)3 (a) Zn (b) Hg
(c) Fe (d) Cu
and MnO 24- are respectively
128. Non-lanthanide atom is
(a) 3,5,4 and 1 (b) 4,3,1 and 5 (a) La (b) Lu
(c) 1,3,4 and 5 (d) 5,4,3 and 1 (c) Pr (d) Pm
117. In an alkaline condition KMnO4 reacts as follows: 129. In which of the following lanthanides oxidation state +2 is
most stable?
2KMnO 4 + 2KOH ® 2K 2 MnO4 + H 2O + O (a) Ce (b) Eu
Its equivalent weight is (c) Tb (d) Dy
(a) 31.6 (b) 52.7 130. Which of the following elements shows maximum number of
different oxidation states in its compounds?
(c) 49 (d) 158
(a) Eu (b) Ld
118. In which of the following compounds manganese has
(c) Gd (d) Am
oxidation number equal to that of iodine in KIO4 ?
131. The alloy best suited for making meter scales is
(a) Potassium manganate
(a) stainless steel (b) invar
(b) Potassium permanganate (c) alnico (d) tungsten steel
(c) Manganous chloride 132. Which metal is present in brass, bronze and German silver?
(d) Manganese chloride (a) Zn (b) Mg
119. Which of the following compounds has colour but no (c) Cu (d) Al
unpaired electrons? 133. Brass is an alloy of
(a) KMnO4 (b) K2MnO4 (a) Al and Zn (b) Cu and Al
(c) MnSO4 (d) MnCl2 (c) Ni and Zn (d) Cu and Zn
752 Chemistry

1. Which one of the following ions is the most stable in aqueous 11. Acidified K2Cr2O7 solution turns green when Na2SO3 is
solution? [CBSE-PMT 2007] added to it. This is due to the formation of :
(a) V3+ (b) Ti3+ (c) Mn 3+ (d) Cr3+ [CBSE-PMT 2011]
(At.No. Ti = 22, V = 23, Cr = 24, Mn = 25) (a) Cr2(SO4)3 (b) CrO42– (c) Cr2(SO3)3 (d) CrSO4
2. Identify the incorrect statement among the following : 12. Which of the statements is not true? [CBSE-PMT 2012]
[CBSE-PMT 2007] (a) On passing H2S through acidified K2Cr2O7 solution, a
(a) Lanthanoid contraction is the accumulation of successive milky colour is observed.
shrinkages. (b) Na 2Cr 2 O7 is preferred over K2 Cr 2O7 in volumetric
(b) As a result of lanthanoid contraction, the properties of analysis.
4d series of the transition elements have no similarities (c) K2Cr2O7 solution in acidic medium is orange.
with the 5d series of elements. (d) K2Cr2O7 solution becomes yellow on increasing the pH
(c) Shielding power of 4f electrons is quite weak. beyond 7.
(d) There is a decrease in the radii of the atoms or ions as 13. Which one of the following does not correctly represent the
one proceeds from La to Lu. correct order of the property indicated against it?
3. The correct order of decreasing second ionisation enthalpy [CBSE-PMT 2012]
of Ti (22), V(23), Cr(24) and Mn (25) is : [CBSE-PMT 2008] (a) Ti < V < Cr < Mn : increasing number of oxidation states
(a) Cr > Mn > V > Ti (b) V > Mn > Cr > Ti (b) Ti3+ < V3+ < Cr3+ < Mn3+ : increasing magnetic moment
(c) Mn > Cr > Ti > V (d) Ti > V > Cr > Mn (c) Ti < V < Cr < Mn : increasing melting points
4. Which one of the elements with the following outer orbital (d) Ti < V < Mn < Cr : increasing 2nd ionization enthalpy
configurations may exhibit the largest number of oxidation 14. Four successive members of the first series of the transition
states? [CBSE-PMT 2009] metals are listed below. For which one of them the standard
(a) 3d 54s1 (b) 3d 54s2 (c) 3d 24s2 (d) 3d 34s2
5. Out of TiF62– , CoF63– , Cu2Cl2 and NiCl2– 4 (Z of Ti = 22,
( º
)
potential E M 2 + / M value has a positive sign?

Co = 27, Cu = 29, Ni = 28), the colourless species are: [CBSE-PMT 2012]


[CBSE-PMT 2009] (a) Co (Z = 27) (b) Ni (Z = 28)
(c) Cu (Z = 29) (d) Fe (Z = 26)
(a) Cu2Cl2 and NiCl 2– (b) TiF62– ,and Cu2Cl2
4 15. The catalytic activity of transition metals and their
(c) CoF63– ,and NiCl2– (d) TiF62– ,and CoF63– , compounds is mainly due to : [CBSE-PMT 2012]
4
(a) their magnetic behaviour
6. Which of the following ions will exhibit colour in aqueous (b) their unfilled d-orbitals
solutions? [CBSE-PMT 2010]
(c) their ability to adopt variable oxidation state
(a) La 3+ (Z = 57) (b) Ti3+ (Z = 22) (d) their chemical reactivity
16. Which of the following exhibit only + 3 oxidation state ?
(c) Lu 3+ (Z = 71) (d) Sc3+ (Z = 21) [CBSE-PMT 2012]
7. Which one of the following ions has electronic configuration (a) U (b) Th (c) Ac (d) Pa
[Ar] 3d 6 ? [CBSE-PMT 2010] 17. A magnetic moment of 1.73 BM will be shown by one among
(a) Ni 3+ (b) Mn 3+ (c) Fe3+ (d) Co3+ the following : [NEET 2013]
(At. Nos. Mn = 25, Fe = 26, Co = 27, Ni = 28) (a) [Ni(CN)4]2- (b) TiCl4
8. Which of the following pairs has the same size? (c) [CoCl6]4- (d) [Cu(NH3)4]2+
[CBSE-PMT 2010] 18. Which of the following lanthanoid ions is diamagnetic ?
(At nos. Ce = 58, Sm = 62, Eu = 63, Yb = 70) [NEET 2013]
(a) Fe 2+ , Ni 2+ (b) Zr 4+ , Ti 4+ (a) Sm2+ (b) Eu2+ (c) Yb2+ (d) Ce2+
(c) Zr 4+ , Hf 4+ (d) Zn 2+ , Hf 4+ 19. KMnO4 can be prepared from K2MnO4 as per the reaction:
9. Which of the following oxidation states is the most common
3MnO 24 - + 2H 2O 2MnO24 - + MnO2 + 4OH -
among the lanthanoids? [CBSE-PMT 2010]
(a) 3 (b) 4 (c) 2 (d) 5 The reaction can go to completion by removing OH– ions by
10. For the four successive transition elements (Cr, Mn, Fe and adding. [NEET 2013]
Co), the stability of +2 oxidation state will be there in which of (a) KOH (b) CO2 (c) SO2 (d) HCl
the following order? [CBSE-PMT 2011] 20. Most common oxidation states of Ce (cerium) are
(a) Mn > Fe > Cr > Co (b) Fe > Mn > Co > Cr [AIEEE 2002]
(c) Co > Mn > Fe > Cr (d) Cr > Mn > Co > Fe (a) +2, +3 (b) +2, +4 (c) +3, +4 (d) +3, +5.
The d & f-Block Elements 753
21. Arrange Ce+3, La+3, Pm+3 and Yb+3 in increasing order of their 32. Which of the following factors may be regarded as the main
ionic radii. [AIEEE 2002] cause of lanthanide contraction? [AIEEE 2005]
(a) Yb+3 < Pm+3 < Ce+3 < La+3 (b) Ce+3 < Yb+3 < Pm+3 < La+3 (a) Greater shielding of 5d electrons by 4f electrons
(c) Yb+3 < Pm+3 < La+3 < Ce+3 (d) Pm+3 < La+3 < Ce+3 < Yb+3. (b) Poorer shielding of 5d electrons by 4f electrons
22. Which of the following ions has the maximum magnetic (c) Effective shielding of one of the 4f electron by another in
moment? [AIEEE 2002] the subshell
(a) Mn +2 (b) Fe+2 (c) Ti+2 (d) Cr+2. (d) Poor shielding of one of the 4f electron by another in the
23. Which is the correct order of ionic sizes? [AIEEE 2002] subshell
(a) Ce > Sn > Yb > Lu (b) Sn > Ce > Lu > Yb
33. Nickel (Z = 28) combines with a uninegative monodentate
(c) Lu > Yb > Sn > Ce (d) Sn > Yb > Ce > Lu.
ligand X– to form a paramagnetic complex [NiX4]2–. The
(At.Nos. : Ce = 58, Sn = 50, Yb = 70 & Lu = 71)
number of unpaired electron(s) in the nickel and geometry of
24. When KMnO4 acts as an oxidising agent and ultimately forms
this complex ion are, respectively: [AIEEE 2006]
[MnO4]–1, MnO2, Mn2O3, Mn+2 then the number of electrons
transferred in each case respectively is [AIEEE 2002] (a) one, square planar (b) two, square planar
(a) 4, 3, 1, 5 (b) 1, 5, 3, 7 (c) 1, 3, 4, 5 (d) 3, 5, 7, 1. (c) one, tetrahedral (d) two, tetrahedral
25. The radius of La3+ (Atomic number of La = 57) is 1.06Å. Which 34. Lanthanoid contraction is caused due to [AIEEE 2006]
one of the following given values will be closest to the radius (a) the same effective nuclear charge from Ce to Lu
of Lu3+ (Atomic number of Lu = 71) ? [AIEEE 2003] (b) the imperfect shielding on outer electrons by 4f electrons
(a) 1.40 Å (b) 1.06 Å (c) 0.85 Å (d) 1.60 Å from the nuclear charge
26. What would happen when a solution of potassium chromate (c) the appreciable shielding on outer electrons by 4f
is treated with an excess of dilute nitric acid? [AIEEE 2003] electrons from the nuclear charge
(a) Cr2 O 72- and H2O are formed (d) the appreciable shielding on outer electrons by 5d
electrons from the nuclear charge
(b) CrO 24- is reduced to +3 state of Cr 35. The "spin-only" magnetic moment [in units of Bohr magneton,
(µB)] of Ni2+ in aqueous solution would be (At. No. Ni = 28)
(c) CrO 24- is oxidized to +7 state of Cr [AIEEE 2006]
(d) Cr3+ and Cr2 O 72- are formed (a) 6 (b) 1.73 (c) 2.84 (d) 4.90
36. Identify the incorrect statement among the following:
27. Which one of the following nitrates will leave behind a metal
[AIEEE 2007]
on strong heating ? [AIEEE 2003]
(a) 4f and 5f orbitals are equally shielded.
(a) Copper nitrate (b) Manganese nitrate
(c) Silver nitrate (d) Ferric nitrate (b) d-Block elements show irregular and erratic chemical
28. The atomic numbers of vanadium (V), Chromium (Cr), properties among themselves.
manganese (Mn) and iron (Fe) are respectively 23, 24, 25, and (c) La and Lu have partially filled d-orbitals and no other
26. Which one of these may be expected to have the highest partially filled orbitals.
second ionization enthalpy ? [AIEEE 2003] (d) The chemistry of various lanthanoids is very similar.
(a) Cr (b) Mn (c) Fe (d) V
29. Cerium (Z = 58) is an important member of the lanthanoids. Which 37. The actinoids exhibit more number of oxidation
of the following statements about cerium is incorrect? states in general than the lanthanoids. This is because
[AIEEE 2004] [AIEEE 2007]
(a) The +4 oxidation state of cerium is not known in solutions (a) the 5f orbitals extend further from the nucleus than the 4f
(b) The +3 oxidation state of cerium is more stable than the orbitals
+4 oxidation state (b) the 5f orbitals are more buried than the 4f orbitals
(c) The common oxidation states of cerium are +3 (c) there is a similarity between 4f and 5f orbitals in their
and +4 angular part of the wave function
(d) Cerium (IV) acts as an oxidizing agent (d) the actinoids are more reactive than the lanthanoids.
30. Calomel (Hg 2Cl 2 ) on reaction with ammonium hydroxide
gives [AIEEE 2005] 38. Larger number of oxidation states are exhibited by the actinoids
(a) HgO (b) Hg 2O than those by the lanthanoids, the main reason being
[AIEEE 2008]
(c) NH 2 – Hg – Hg – Cl (d) Hg NH 2 Cl (a) 4f orbitals more diffused than the 5f orbitals
31. The lanthanide contraction is responsible for the fact that (b) lesser energy difference between 5f and 6d than between
[AIEEE 2005] 4f and 5d orbitals
(a) Zr and Zn have the same oxidation state (c) more energy difference between 5f and 6d than between
(b) Zr and Hf have about the same radius 4f and 5d orbitals
(c) Zr and Nb have similar oxidation state
(d) more reactive nature of the actionids than the lanthanoids
(d) Zr and Y have about the same radius
754 Chemistry
39. Amount of oxalic acid present in a solution can be determined 43. Iron exhibits +2 and + 3 oxidation states. Which of the
by its titration with KMnO4 solution in the presence of following statements about iron is incorrect ? [AIEEE 2012]
H2SO4.The titration gives unsatisfactory result when carried (a) Ferrous oxide is more basic in nature than the ferric oxide.
out in the presence of HCl, because HCl [AIEEE 2008]
(b) Ferrous compounds are relatively more ionic than the
(a) gets oxidised by oxalic acid to chlorine
corresponding ferric compounds.
(b) furnishes H+ ions in addition to those from oxalic acid
(c) Ferrous compounds are less volatile than the
(c) reduces permanganate to Mn 2+
corresponding ferric compounds.
(d) oxidises oxalic acid to carbon doxide and water
(d) Ferrous compounds are more easily hydrolysed than the
40. Knowing that the chemistry of lanthanoids(Ln) is dominated
corresponding ferric compounds.
by its + 3 oxidation state, which of the following statements is
incorrect? [AIEEE 2009] 44. Which of the following arrangements does not represent the
(a) The ionic size of Ln (III) decrease in general with correct order of the property stated against it ?
increasing atomic number [JEE M 2013]
(b) Ln (III) compounds are generally colourless. (a) V2+ < Cr2+ < Mn2+ < Fe2+ : paramagnetic behaviour
(c) Ln (III) hydroxide are mainly basic in character. (b) Ni2+ < Co2+ < Fe2+ < Mn2+ : ionic size
(d) Because of the large size of the Ln (III) ions the bonding (c) Co3+ < Fe3+ < Cr3+ < Sc3+ : stability in aqueous solution
in its compounds is predominantly ionic in character. (d) Sc < Ti < Cr < Mn : number of oxidation states
41. In context of the lanthanoids, which of the following 45. Four successive members of the first row transition elements
statements is not correct? [AIEEE 2011] are listed below with atomic numbers. Which one of them is
(a) There is a gradual decrease in the radii of the members expected to have the highest E° 3+ 2+ value ?
with increasing atomic number in the series. M /M
[JEE M 2013]
(b) All the members exhibit +3 oxidation state.
(c) Because of similar properties the separation of (a) Cr(Z = 24) (b) Mn(Z = 25)
lanthanoids is not easy. (c) Fe(Z = 26) (d) Co(Z = 27)
(d) Availability of 4f electrons results in the formation of 46. Oxidation states of the metal in the minerals haematite and
compounds in +4 state for all the members of the series. magnetite, respectively, are [IIT-JEE 2011]
42. The outer electron configuration of Gd (Atomic No. : 64) is : (a) II, III in haematite and III in magnetite
[AIEEE 2011] (b) II, III in haematite and II in magnetite
(a) 4f 3 5d5 6s2 (b) 4f 8 5d 0 6s2 (c) II in haematite and II, III in magnetite
(c) 4f 4 5d 4 6s2 (d) 4f 7 5d 1 6s2 (d) III in haematite and II, III in magnetite

1. In nitroprusside ion the iron and NO exist as FeII and NO+ 4. Titanium shows magnetic moment of 1.73 B.M. in its
rather than FeIII and NO. These forms can be differentiated compound. What is the oxidation number of Ti in the
by compound?
(a) estimating the concentration of iron (a) +1 (b) +4
(b) measuring the concentration of CN (c) +3 (d) +2
(c) measuring the solid state magnetic moment 5. Which of the following is paramagnetic?
(d) thermally decomposing the compound. 4-
2. Anhydrous ferric chloride is prepared by (a) é Fe (CN ) ù
ë 6û
(b) éë Ni (CO )4 ùû
(a) heating hydrated ferric chloride at a high temperature in
2-
a stream of air (c) é Ni (CO ) ù
ë 4û
(d) [CoF6 ]3-
(b) heating metallic iron in a stream of dry chlorine gas
6. If an aqueous solution of KCN is added to a solution of
(c) reaction of metallic iron with hydrochloric acid
ferrous salt then the complex formed is represented by
(d) reaction of metallic iron with NaCl solution
4- 3-
3. When MnO2 is fused with KOH, a coloured compound is (a) é Fe (CN ) ù (b) éë Fe (CN )6 ùû
ë 6û
formed, the product and its colour is:
(a) K2MnO4, purple green (b) KMnO4, purple 3+ 2+
(c) é Fe (H 2 O) ù (d) éë Fe (H 2 O)6 ùû
(c) Mn2O3, brown (d) Mn3O4, black ë 6û
The d & f-Block Elements 755
7. The temperature of blast furnace to produce iron from its 12. In the extraction of copper, the reaction which takes place in
Bessemer converter is
ore Fe2 O3 varies from 500o C at the top of the furnace to
(a) 2CuFeS2 + O 2 ® Cu 2S + 2FeS + SO 2
about 1900o C at the bottom of the furnace. The reaction
(b) 2Cu 2O + Cu 2S ® 6Cu + SO 2
between the ore Fe 2O3 and CO at the lowest temperature
(c) 2Cu 2S + 3O2 ® 2Cu2O + 2SO2

( ~ 500 C ) is
o
(d) 2FeS + 3O 2 ® 2FeO + 2SO2
13. Copper is extracted from copper pyrites ore by
(a) 3Fe 2O3 + CO ® 2Fe3O4 + CO 2 heating in a blast furnace. The method is based on the
principle that
(b) Fe 2O3 + CO ® 2FeO + CO 2
(a) copper has more affinity for oxygen than sulphur at
(c) Fe2O3 + 3CO ® 2Fe + 3CO2 high temperature
(b) iron has less affinity for oxygen than sulphur at high
(d) Fe 2O3 + CO + CaCO3 ® Fe2O3 + CO + CO2 + CaO temperature
(c) sulphur has less affinity for oxygen at high temperature
8. The blue complex formed on addition of conc.
(d) copper has less affinity for oxygen than sulphur at high
NH 4OH solution to a Cu 2+ salt solution has the temprature
structure? 14. Colourless solutions of the following four salts are placed
separately in four different test tubes and a strip of copper is
2+ 2+ dipped in each one of these. Which solution will turn blue?
(a) éCu ( NH 4 ) ù
ë 4û
(b) éëCu ( NH3 )2 ùû
(a) KNO3 (b) AgNO3
2+ 2+
(c) éCu ( NH3 ) ù (d) éëCu ( NH 4 )2 ùû (c) Zn(NO3)2 (d) ZnSO4
ë 4û
15. Which of the following does not react with AgCl?
9. When CuSO4 solution is added to K 4 éë Fe (CN 6 )ùû the (a) Na 2S2O3 (b) NH 4OH
formula of the product formed is? (c) NaNO3 (d) Na 2CO3
Cu 2 Fe (CN )6 16. Which of the following reactions represents “developing”
(a) (b) KCN
in photography
(c) Cu (CN )3 (d) Cu (CN )2 (a) AgNO3 + NaBr ® AgBr + NaNO3

10. In order to refine “blister copper” it is melted in (b) AgBr + 2Na 2S2O3 ® Na 3 éë Ag (S2O3 )2 ùû + NaBr
furnace and is strirred with green logs of wood. The
purpose is (c) AgBr + hv ® AgBr*
(a) to expel the dissolved gases in the blister copper
(d) C6 H 4( OH ) + +2AgBr* ® C6 H 4O 2 + 2HBr + 2Ag
(b) to bring the impurities to surface and oxidise them 2

(c) to increase the carbon content of copper 17. In the extraction of silver from argentite ore the ore i s
treated with dilute solution of NaCN in water in the presence
(d) to reduce the metallic oxide impurities with hydrocarbon of Y, whereby the following reaction takes place
gases liberated from the wood.
11. Identify the incorrect statement Ag 2 X + 4NaCN + 2Y ® 2Na éëAg (CN )2 ùû + Na 2XO 4

(a) CuSO4 reacts with KCl in aqueous solution to give X and Y in this reaction are represented by
(a) Cl and S (b) S and O2
Cu 2Cl2 (c) O and O2 (d) O and S
(b) CuSO4 r eacts with KI in aqueous solution to 18. The black compound formed during the reaction
between sodium thiosulphate and silver nitrate is
give Cu 2 I2
(a) silver thiosulphate ( Ag2S2O3 )
(c) CuSO4 react with NaOH and glucose in aqueous
(b) silver sulphide ( Ag 2S)
medium to give Cu 2O
(c) silver sulphate ( Ag 2SO 4 )
(d) CuSO4 on strong heating gives CuO
(d) silver sulphite ( Ag 2SO3 )
756 Chemistry
19. Zn gives H2 gas with H2SO4 and HCl but not with HNO3 (b) 10 ml of 1M KMnO 4 solution oxidises
because
10 ml of 5N FeSO 4 solution
(a) Zn acts as an oxidising agent when react with HNO3
(c) 10 ml of 1M KMnO 4 solution oxidises
(b) HNO3 is weaker acid than H 2SO4 and HCl
10 ml of 1M FeSO4 solution
(c) In electrochemical series Zn is above hydrogen
- (d) 10 ml of 1N KMnO 4 solution oxidises
(d) NO3 ion is reduced in preference to hydronium ion
20. Which of the following is used in the preparation of 10 ml of 0.1M FeSO4 solution
chlorine? 22. Number of moles of K 2Cr2O7 reduced by one mole of Sn 2+
(a) Only MnO 2 ions is
(b) Only KMnO4 1
(a) (b) 3
(c) Both MnO 2 and KMnO4 3
1
(d) Either MnO 2 or KMnO4 (c) (d) 6
6
21. In acidic medium KMnO4 oxidises FeSO4 solution. Which
23. CrO3 dissolves in aqueous NaOH go give
of the following statements is correct?
(a) 10 ml of 1N KMnO 4 solution oxidises (a) CrO24 - (b) Cr (OH )3

10 ml of 5N FeSO 4 solution (c) Cr2 O72 - (d) Cr (OH )2


The d & f-Block Elements 757

EXERCISE 1 14. (a) 3d series starts from Sc(Z - 21) and ends with
3. In both K2Cr2O7 and K2CrO4, Cr is in + 6 oxidation state. Zn(Z –30).
4. Co2+ contains three unpaired electrons \ more paramagnetic 15. (a)
5. The extra-ordinary stability of La, Gd and Zu is due to the
16. (d) Only Cr 3+ [ Ar ]3d 3 contains unpaired electrons hence
empty, half-filled and fulley filled 4f-orbitals respectively.
6. This is because KMnO4 has a germicidal action. will give coloured solution
10. (a) + 2 oxidation state (exactly half-filled d-orbitals) 17. (d) A covalent bond is formed between small interstial
(b) Osmium, (Os) in osmium tetraoxide (OsO4). non-metal and transition metal which make it hard
(c) Osmium (Os) is the densest transition element. 18. (d) Transition element due to similar (almost) sizes exhibit
11. Acidic: Mn2O7 Cr O3 (oxides in higher oxidation states). both vertical and horizontal similarities.
Amphoteric: MnO2, CuO, TiO2 (oxides in moderate oxidation 19. (d) O.S. of Os in OsO4 is + 8
states). 20. (a) Highest O.S. by Mn (+7)
Basic: MnO, Cr2O3 (oxides in higher oxidation states).
12. n = 4. 21. (b) Zn + [ Ar ] 3d10 4s1 , Fe 2+ [ Ar ] 3d 6 4s0 , Ni + [ Ar ] 3d 8 4s1,
20. (b) Pt can form compounds of + 4 oxidation state more easily
21. (e) This is because of Cu + [ Ar ] 3d10 4s0 ;
(i) high heat of atomisation, and
(ii) high ionisation energies Fe 2 + contain maximum number of unpaired electrons.
23. (c) 24. (c) 25. (c) 26. (a) 27. (b) 22. (a) Group number is given by [ns + (n - 1)d ] electrons.
28. (b) 29. (c)
\ [2 + 3] = 5
EXERCISE 2 23. (d) Iron loses magnetic properties at 1000K.
24. (a) Ni ( 28) Ni [ Ar ]3d8 4s 2 contain 2 unpaired electrons.
1. (a) Cr 2 + electronic configuration is [ Ar ]3d , hence number
4
25. (b) Zn and Hg have d-shells completely filled hence they do
of d electrons is 4.
not exhibit variable valency.
2. (b) The more the number of unpaired electrons, the more is
26. (c) The statement is true for transition elements.
magnetic moment.Therefore the answer is (b).
27. (b) Transition metals are generally paramagnetic since they
3. (c) Ag belongs to second transition series.
contain unpaired electrons.
4. (d) All statements are correct.
28. (b) Since reduction potential of fluorine is highest transition
5. (a) The more the oxidation state, the more is the acid
metals exhibit highest oxidation state with fluorine.
character, MnO, Mn 2O3 are basic, MnO 2 is amphoteric 29. (d) Since Mn2+ contains maximum number of unparied
and Mn 2O7 is acidic. electrons hence it has maximum magnetic moment
30. (d) Since Sc3+ does not contain any unpaired electron it is
6. (a) Electronic configuration of
colourless in water.
Zn 2+ [ Ar ] 3d10 4s 0 ; Ni 2 + [ Ar ] 3d8 , 4s 0 ; 31. (d) Since transition metals can lose electrons from
Cr3+[Ar]3d3, 4s0Zn2+ colourless (no unpaired electrons) (n - 1)d ns orbitals hence they are valence orbitals.
Ni2+ and Cr 3+ coloured due to unpaired electrons 32. (d) Sc3+ is diamagnetic as it does not contain any unpaired
7. (c) electron while others contain.
8. (c)
9. (d) Transition metals exhibit variable valency 33. (b) Cu 2+ [ Ar ] 3d 9 Ti 4+ [ Ar ] 3d 0 Co 2+ [ Ar ]3d 7 Fe2+ [Ar ]3d 6
10. (b) Cr has maximum unparied electrons .The number is 6. 1,3,4 are coloured ions hence the answer is b.
34. (c) If non metal is added to the interstital site the metal
24 Cr : [Ar]3d 5 4s1 becomes less malleable due to formation of covalent bond
11. (a) between metal and non metal
35. (b) Transition metal in lower oxidation state will exist as cation
12. (b) Cu + [ Ar ] 3d10 , Cu 2+ [ Ar ] 3d9 , Ti 4+ [ Ar ] 3d 0 , V5+ [ Ar ] 3d 0 .
which are lewis acids.
only Cu2+ contains unpaired electron hence it is coloured 36. (c) Zinc does not show variable oxidation state due to
13. (a) They may or may not be diamagnetic completely filled d-orbitals
758 Chemistry
37. (c) Zn, Cd, Hg do not show properties of transition elements 62. (a) Green vitrol is FeSO 4 .7H 2O
hence they are known as non typical transition elements
63. (c) Cassiterite is not an ore of iron. Liomite and magnetite
38. (d) Super conductors are derived from compounds of
are the ores of iron. Cassiterite is an ore of Sn.
transition metals
64. (b) Corrosion is electrochemical phenomenon (See chapter
39. (d) Corrosion can be prevented by Zn (galvanizing or
on electrochemistry)
sherardizing) with Sn (known as tinning) and Ni by 65. (b) Gun metal is alloy of Cu, Zn, Sn.
electroplating
2+
40. (b) Ferrous ferricyanide is known as Turbull’s blue 66. (a) Cu 0 + 2AgNO3 ® Cu ( NO3 ) + 2Ag .
41. (a) In octahedral crystal field the d - orbitals split in to t2g 2
and eg orbitals. eg (dxy ,dyz and dzx ) occupy lower level
(
It is oxidation of copper Cu 0 ® Cu 2 + )
( )
and eg d x 2 - y2 , d z2 occupy higher energy level.
67. (a) CuSO 4 + Ca(OH) 2 is Bordeaux mixture
42. (b) 24 Cr , 26 Fe , 27 Co , 25 Mn , 21Sc 68. (a) 2Cu 2+ + 4KI ® Cu 2 I 2 + I 2 + 4K +
Electonic configuration
69. (c) Cu + HCl ® No reaction (others will give H2) See
Cr : [ Ar ] 3d5 4s1 , Fe :[ Ar ] 3d 6 4s2 , Co : [ Ar ] 3d 7 4s 2 , applications of ECS (electrochemistry).
70. (b) Cu is found in native state
Mn : [ Ar ] 3d5 4s 2 , Sc :[Ar ] 3d1 4s 2 71. (a) CuF2 is ionic in nature (Fajan’s rule ), for fixed cation the
Configuration of ions smaller the anion, the more is ionic character.

Cr3+ Fe3+ Co3 + Mn 2 + Sc3+ 72. (c) 2CuSO 4 + 10KCN ® 2K 3 éëCu (CN )4 ùû + 2K 2SO 4 + (CN )2
[Ar] 3d3 [Ar] 3d5 [ Ar] 3d6 [Ar ] 3d5 [Ar] 3d0 73. (b) Correct (see text)
74. (c)
Fe3+ , Mn 2+ have same electronic configuration 75. (a) Matte contains Cu2S and FeS
43. (a) Due to small size of Fe3+ the bonding in FeCl3 is covalent. 76. (a) Cu has more reduction potential than hydrogen. Cu does
44. (d) The impurity in pig iron is due to carbon hence graphite not evolve H2 with acids (See application of ECS in
chapter of elctrochemistry)
45. (d) Magetic moment = n (n + 2 ) where n = number of
77. (d) In Cu [ Ar ] 3d
+ 10
there is no unpaired electron,
unpaired electrons 15 = n(n + 2 ) \ n = 3
46. (d) Cu 2 + [ Ar ] 3d 9 contains one unpaired electron hence
47. (c) Stainless steel contains Fe, Cr, Ni and C. coloured
48. (c) 22 carat gold is an alloy (see text ) 78. (b) Out of 29 electrons, 14 electrons have spin in one
49. (c) Iron does not exist in native form direction and 15 electrons in other direction.
50. (d) Slag is CaSiO3 79. (c) Cuprite Cu 2O
51. (a) Acidic due to cationic hydrolysis
D
80. (c) CuSO 4 ¾¾ ® CuO + SO3
ˆˆ† Fe(OH) + 3HCl
FeCl3 + 3H2O ‡ˆˆ
81. (d) Chalcopyrites CuFeS2 , Bauxite ( Al2O3.2H 2O) ,Galena
3

ˆˆ† Fe(OH)3 + 3H+


Fe+++ + 3H2O ‡ˆˆ
(PbS), Siderite ( FeCO3 )
52. (c) Let the O.N. be x \ 3 x + 4(- 2) = 0 \ x = 8 3 82. (d) In electrolytic purification anode is always of impure
53. (a) Iron obtained from blast furnace is known as pig Iron sample and cathode of pure metal
54. (b) Fe3+ + 3KCNS ® Fe(CNS)3 + 3K+ 83. (a) 2AgNO3 + K 2 CrO 4 ® Ag 2 CrO 4 + 2KNO 3
Ferric sulphocyanide is blood red Red precpitate
55. (c) It is the covering of steel by more reactive metal
56. (a) Acidic due to cationic hydrolysis 84. (a) 2AgNO3 ® 2Ag + 2NO2 + O2

ˆˆ† Fe(OH)2 + 2H+


Fe2+ + 2H2O ‡ˆˆ 85. (b) AgCl + 2NaCN ® Na éë Ag (CN )2 ùû + NaCl
57. (c) Ans(c) correct - -
86. (d) Ag 2 O + H 2 O + 2e ® 2Ag + 2OH silver is reduced
58. (c) FeSO4 . ( NH 4 )2SO4 .6H 2 O in solution will give
(Ag + ® Ag 0 ) gain of electron which is reduction
Fe 2+
,2 (
SO 24 - ), 2 ( NH +4 ) hence total number of ions is 5 87. (c) AgCl ( s ) + 2NH 4 OH ( aq ) ® éë Ag ( NH 3 )2 ùû Cl ( aq ) + 2H 2O ( l )
59. (d) It is annealing
88. (a) AgF being ionic in nature is highly soluble in water others
60. (d) With dil. HNO3, Fe gives NH 4 NO3 are insoluble.
61. (a) They contain different percentage of carbon 89. (b) Cupellation process is used in the metallurgy of silver
The d & f-Block Elements 759

90. (d) K 4 é Fe ( CN ) ù + 6H 2SO4 + 6H 2O ® 158


ë 6û 117. (d) Change in O.S. is by 1. Hence eq wt is = 158
1
2K 2SO4 + FeSO4 + 3 ( NH 4 ) 2SO4 + 6CO 118. (b) In KIO4 O.S of I is +7 and in KMnO4 O.S. of Mn is +7
This is one of the methods for the preparation of carbon
monoxide 119. (a) Electronic configuration of Mn is [ Ar ]3d 5 4 s 2 . Being
transition metal it has 7 valence electrons and all are
91. (c) Ag2S is dissolved in KCN to give K éë Ag (CN )2 ùû
involved in bond formation in MnO-4 . Hence it has no
92. (c) Parke’s process (See text) It is for extraction of silver
unpaired electron
93. (c) This is cyanide process for gold ( M = Au )
120. (d) Green colour of CuCr2O7 is due to blue colour of
94. (a) Cu, Hg and Ag are attacked by conc acids but gold is not
attacked. AuCl3 forms H [AuCl 4 ] a complex with HCl Cu 2+ ions and yellow colour of Cr2 O7 2- ions
which is used for tonning in photography. 121. (b) Cerium is the most common lanthanide
95. (c) At ordinary temperature Zn metal is brittle not at high 122. (c) The number is 28(14 lanthanide +14 Actinides)
temperature 123. (b) Electronic configuration of gadolinium is
96. (d) Galvanisation is coating with zinc
97. (c) Zn + 2 NaOH ® Na 2 ZnO 2 + H 2
[ Xe ] 4f 7 5d1 6s 2
124. (b) Oxidation of water takes place in presence of Mn in
98. (b) H 2 Cr2 O 7 + 4O ® 2CrO5 + H 2 O
biological process
Blue peroxide of
125. (b) Na 2 CdCl4 .Cd 2 + does not contain any unpaired electron
chromium
99. (d) CO burns with blue flame
hence colourless
100. (b) Hg 2Cl 2 with NH4OH form black éë Hg + Hg ( NH 2 ) Cl ùû 126. (c)
101. (a) Metallic bonding in mercury is weak 127. (d) Copper (see alloys)
102. (a) Mercury being less reactive cannot evolve H2 from H2S. 128. (a) La ( lanthanum ) is non lanthanide atom
103. (c) It is tailing of mercury due to formation of
129. (b) Eu 2 + has electronic configuration [ Xe] 4f 7 hence stable
Hg 2O (2Hg + O3 ® Hg 2 O + O 2 )
due to half filled atomic orbitals.
104. (b) HgCl 2 sublimes on heating 130.(d)
105. (a) NH+4 ions are detected by Nessler's reagent (see text) Eu La Gd Am
+1 +1
O.S = +2 ,+3 +3 +3,+4,+5,+6
106. (a) Mercurous ion Hg 22 + or Hg - Hg 131. (b) Invar due to small coefficient of thermal expansion
107. (a) Nessler's reagent is K2[HgI4] dissolved in KOH 132. (c) Copper is present in brass, bronze and german silver (see
list - of alloys )
108. (c) MnO -4 + 3e - ® Mn 4+ . In neutral medium
133. (d) Brass is an alloy of Cu and Zn
\ Eq.wt . = M 3
EXERCISE 3
109. (a) Pyrolusite (It is MnO 2 )
1. (d) For chromium ion + 3 oxidation state is most stable.
110. (a) 2KMnO 4 + H 2SO 4 (Conc ) ® K 2SO 4 + Mn 2 O 7 + H 2O 2. (b) As a result of lanthanoid contraction change in ionic
Explosive radii on going from elements of 4d to 5d transition series
111.(c) In acid medium MnO-4 + 8H + + 5e - ® Mn 2+ + 4H 2O is very small. Thus chemical properties of 4d and 5d series
of transition elements are similar.
(O.S. of Mn changes form +7 to +2)
112. (a) Permanganic acid is HMnO4 3. (a) Ti ; Z (22) is 1s22s22p63s23p63d 24s2
V ; Z (23) is 1s22s22p63s23p63d 34s2
113. (d) In MnO2 (O.S. of Mn is + 4) in K 2MnO4 (O.S.of Mn
Cr ; Z (24) is 1s22s22p63s23p63d 54s1
is +6). Hence O.S. changes by 2
114. (a) CrO2 is amphoteric in nature Mn ; Z (25) is 1s22s22p63s23d 54s2
115. (b) K2MnO4 is potassium manganate O.S. of Mn is +6 The second electron in all the cases (except Cr) has to be
116. (a) removed from 4s-orbital and for Cr it has to be removed
from completely half filled 3d-orbital which have extra
KMnO4 MnO2 Mn 2 + Mn (OH )3 MnO4 2 - stability thus I.E2 is highest for Cr. Now among rest
O.S of Mn +7 +4 +2 +3 +6 elements more will be the nuclear charge more will be the
value of I.E2. As nuclear charge depends upon atomic
electrons 0 3 5 4 1 number. Therefore correct order of I.E2 is Mn > V > Ti.
needed
i.e. Cr > Mn > V > Ti. So correct answer is (a).
760 Chemistry

4. (b) Mn - 3d 5 4s2 17. (d) [Cu(NH3)4]2+ hybridisation dsp2


Cu+2 – 3d9 has one unpaired e-
The no. of various oxidation states possible are + 2, + 3,
+ 4, + 5, + 6 and + 7.
5. (b) The colour exhibited by transition metal ions is due to So magnetic moment
the presence of unpaired electrons in d-orbitals which
permits the d - d excitation of electrons. m= n ( n + 2 ) = 1(1 + 2 )
In TiF62– ,– Ti is in + 4 O.S. ; 3d 0 = colourless
= 3 = 1.73
In CoF63– –, Co is in + 3 O.S ; 3d 5 = coloured 18. (c) Sm2+(Z = 62) [Xe]4f6 6s2 – 6 unpaired e–
2+
Eu (Z = 63) [Xe]4f7 6s2 – 7 unpaired e–
In Cu2Cl2– Cu is in +1 O.S. ; 3d 10 – colourless 2+
Yb (Z = 70) [Xe]4f14 6s2 – 0 unpaired e–
2+ [Xe]4f1 5d1 6s2 – 2 unpaired e–
In NiCl2– 8
4 – Ni is in + 2 O.S ; 3d – coloured
Ce (Z = 58)
2+
Only Yb is diamagnetic.
3+ 19. (b) HCl and SO2 are reducing agents and can reduce MnO4–
6. (b) La : 54 e– = [Xe]
. CO2 which is neither oxidising and nor reducing will
Ti3+ : 19 e– = [Ar] 3 d 1 (Coloured) provide only acidic medium. It can shift reaction in forward
Lu 3+ : 68 e– = [Xe] 4 f 14 direction and reaction can go to completion.
20. (c) Common oxidation states of Ce(Cerium) are
Sc3+ : 18 e– = [Ar] + 3 and + 4
21. (a) According to their positions in the periods, these values
7. (d) Ni 3+ : [Ar] 3d 7 are in the order:
Mn3+ : [Ar] 3d 4 Yb+3 < Pm+3 < Ce+3 < La+3
At. Nos. 70 61 58 57
Fe3+ : [Ar] 3d 5 This is due to lanthanide contraction.
Co3+ : [Ar] 3d 6 22. (a) Mn2+ has the maximum number of unpaired electrons (5)
8. (c) Due to lanthanide contraction, the size of Zr and Hf and therefore has maximum magnetic moment.
(atom and ions) become nearly similar 23. (a)
9. (a) +3 - +7 -
- 4e -e
10. (a) 24. (a) Mn2O3 ¬¾ ¾¾ [KMnO 4 ] ¾¾¾® [MnO4 ]–1
11. (a) The green colour appears due to the formation of
-
Cr+++ion -5e -3e-
+4
Cr2 O72– + 3SO32– + 8H + ¾¾
® 3SO 2– 3+ Mn2+ MnO2
4 + 2Cr + 4H 2 O
12. (b) Na2Cr2O7 is hygroscopic.
1
13. (c) The melting points of the transition element first rise to 25. (c) W µ
a maximum and then fall as the atomic number increases z
manganese have abnormally low melting point. W1 z 2
= Þ 1.06 = 71 Þ W2 = 0.85 Å
14. (c) E o +2 = 0.34 V W 2 z1 W2 57
Cu / Cu

other has – ve E oR.P. ¾® 2Cr O 24 - + H+


26. (a) Cr2O 27 - + OH– ¾
The above equilibrium shifts to L.H.S. on addition of
Eo
Co + + / Co = – 0.28 V acid.
Eo = – 0.25V 1
Ni + + / Ni 27. (c) AgNO 3 ® Ag + NO 2 + O2
2
Eo = – 0.44V 28. (a) Electronic configuration of Cr is
Fe ++ / Fe
15. (c) The transition metals and their compounds are used as 3d 4s
catalysts. Because of the variable oxidation states they
may form intermediate compound with one of the
readtants. These intermediate provides a new path with So due to half filled orbital I.P. is high of Cr.
lowe activation energy. V2O5 + SO2 ® V2O4 + SO3 29. (a) The +4 oxidation state of cerium is also known in solution
2V2O4+ O2 ® 2V2O5
30. (d) Hg 2 Cl 2 + 2NH 4 OH ¾¾
® HgNH 2Cl +NH 4Cl + 2H 2O
16. (c) Ac (89) = [Rn] [6d1] [7s2]
The d & f-Block Elements 761
31. (b) In vertice columns of transition elements, there is an 40. (b) Most of the Ln 3+ compounds except La3+ and Lu3+ are
increase in size from first member to second member as coloured due to the presence of f-electrons.
expected but from second member to third member, there
is very small change in size and some times sizes 41. (d)
are same. This is due to lanthanide contraction. This is 42. (d) The configuration of Gd is [xe] 4f 7 5d1 6s2.
the reason for Zr and Hg to have same radius. 43. (d) Fe3+ is easily hydrolysed than Fe2+ due to more positive
32. (d) In lanthanides, there is poorer shielding of 5d electrons
charge.
by 4f electrons resulting in greater attraction of the
nucleus over 5d electrons and contraction of the atomic 44. (a)
radii. (a) V = 3d 3 4s 2 ; V2+ = 3d 3 = 3 unpaired electrons
Cr = 3d 5 4s 1 ; Cr2+ = 3d 4 = 4 unpaired electrons
33. (d) [Ni X4]2–, the electronic configuration of Ni 2+ is Mn = 3d 5 4s2 ; Mn2+ = 3d 5 = 5 unpaired electrons
Fe = 3d 6 4s 2 ; Fe2+ = 3d 6 = 4 unpaired electrons
Hence the correct order of paramagnetic behaviour
V2+ < Cr 2+ = Fe2+ < Mn2+
(b) For the same oxidation state, the ionic radii generally
3d 4s 4p decreases as the atomic number increases in a particular
transition series. hence the order is
It contains two unpaired electrons and the hybridisation Mn++ > Fe++ > Co++ > Ni++
is sp3 (tetrahedral). (c) In solution, the stability of the compound depends upon
34. (b) The configuration of Lanthanides show that the additional electrode potentials, SEP of the transitions metal ions are
electron enters the 4f subshell. The shielding of one 4f given as
electron by another is very little or imperfect. The Co3+ / Co = + 1.97, Fe3+ / Fe = + 0.77 ;
imperfect shielding of f electrons is due to the shape of f Cr3+ / Cr2+ = – 0.41, Sc 3+ is highly stable as it does not
orbitals which is very much diffused. Thus as the atomic show + 2 O. S.
number increases, the nuclear charge increases by unity (d) Sc – (+ 2), (+ 3)
at each step while no comparable increase in the mutual Ti – (+ 2), (+ 3), (+ 4)
shielding effect of 4f occurs. This causes a contraction Cr – (+ 1), (+ 2), (+ 3), (+ 4), (+ 5), (+ 6)
in the size of the 4f subshell. as a result atomic and ionic Mn – (+ 2), (+ 3), (+ 4), (+ 5), (+ 6), (+ 7)
radii decreases gradually from La to Lu. i.e. Sc < Ti < Cr = Mn
35. (c) The number of unpaired electrons in Ni2+(aq) = 2
Water is weak ligand hence no pairing will take place 45. (d) E°Cr3+ / Cr 2 + = – 0.41 V E°
Fe3+ / Fe 2 + = + 0.77 V
spin magnetic moment
E° = + 1.57 V,, E°Co3+ / Co 2 + = + 1.97 V
Mn3+ / Mn 2 +
= n ( n + 2) = 2( 2 + 2) = 8 = 2.82
46. (d) (i) Haematite is Fe2O3 in which Fe is present in III
36. (a) 4f orbital is nearer to nucleus as compared to 5f orbital oxidation state.
therefore, shielding of 4 f is more than 5 f. (ii) Magnetite (Fe3O4) is an equimolar mixture of FeO
37. (a) More the distance between nucleus and outer orbitals, and Fe2O3.
lesser will be force of attraction on them. Distance
between nucleus and 5f orbitals is more as compared to Oxidation state of Fe in FeO is II.
distance between 4f orbital and nucleus. So actinoids Oxidation state of Fe in Fe2O3 is III.
exhibit more number of oxidation states in general than
the lanthanoids. EXERCISE 4
38. (b) The main reason for exhibiting larger number of oxidation
1. (c) The magnetic moment (m ) of a species is related to its
states by actinoids as compared to lanthanoids is lesser number of unpaired electrons (n) in form of following
energy difference between 5 f and 6d orbitals as compared expressions.
to that between 4f and 5d orbitals.
m= n ( n + 2) B.M
In case of actinoids we can remove electrons from 5f as
well as from d and due to this actinoids exhibit larger The number of unpaired electrons in the given pairs are
as follows:
number of oxidation state than lanthanoids. Thus the
correct answer is option (b) Fe 2 + = 1s 2 ,2s 2 2p 6 ,3s 2 3p 6 3d 6
39. (c) The titration of oxalic acid with KMnO4 in presence of 3d
HCl gives unsatisfactory result because of the fact that
or Thus here n = 4.
KMnO4 can also oxidise HCl along with oxalic acid. HCl
on oxidation gives Cl2 and HCl reduces KMnO4 to Mn2+
762 Chemistry
10. (d) The statement (d) is correct.
Fe 3+ = 1s 2 ,2s 2 2p 6 ,3s 2 3p 6 3d 5
11. (a) 2CuSO4 + 4KI ® Cu 2I2 + 2K 2SO4 + I2 (not given by
3d
KCl).
or Thus here n = 5 Fehling solution gives Cu2O with reducing substances
strong
CuSO 4 ¾¾ ¾® CuO + SO 3
+ + D
NO or N = O : n =0
xx 12. (b) 2Cu 2O + Cu 2S ® 6Cu + SO 2 takes place in Bessemer
´ .. convertor
NO or ´´ N —
—O: n =1
13. (a) (see extraction of copper)
The given combinations differ in the number of unpaired
electrons. Hence these can be differentiated by the 14. (b) Cu + 2AgNO 3 ® Cu (NO 3 )2 + 2 Ag. Cu 2 + will give
measurement on the solid state magnetic moment of blue solution. Cu is below in ECS than K and Zn hence
nitroprusside ion. no reaction with them
2. (b) 2Fe + 3Cl 2 ® 2FeCl 3 15. (c) There is no reaction between NaNO3 and AgCl
3. (a) Stable oxidation state of Mn in alkaline AgCl + 2Na 2S2 O3 ® Na 3 éë Ag (S2O3 )2 ùû + NaCl
medium is +6. So, MnO2 is oxidised to K2MnO 4
by atmospheric oxygen in KOH medium.
AgCl + 2NH 4 OH ® éë Ag ( NH 3 ) 2 ùû Cl + 2H 2 O
2MnO 2 + 4KOH + O 2 ® 2K 2 MnO4 + 2H 2 O
4AgCl + 2Na 2 CO 3 ® 4NaCl + 4Ag + 2CO 2 + O 2
4. (c) Magnetic moment = n (n + 2 )BM
16. (d) Developing in photography involves decomposition
of AgBr
1.73 = n ( n + 2 ) n = 1 , it has one unpaired electron
C6 H 4 (OH )2 + 2AgBr ® C6 H 4O2 + 2HBr + 2Ag
hence electronic configuration is [ Ar ]3d and electronic
1
17. (b) Ag 2 S + 4 NaCN + 2O 2 ® 2 Na [Ag (CN )2 ] + Na 2SO 4
configuration for Z = 22 is [ Ar ] 3d 2 4s2 . Hence charge
X is S and Y is O2
on Ti is +3
5. (d) Fe2+ has electronic configuration 18. (a) 2AgNO3 + Na 2S2O3 ® Ag 2S2O3 + 2NaNO3
3d 4s 19. (d) NO3- ions are reduced by nascent hydrogen
due to strong ligand
Metal + HNO3 ® Metal nitrate + H
Ni has electronic configuration HNO3 + 8H ® N 2 O + 5H 2O
3d 4s
due to strong ligand 20. (c) Both MnO2 and KMnO4 used for the preparation of
chlorine by the action of Conc. HCl
Ni2+ has electronic configuration
MnO2 + 4HCl ® MnCl2 + 2H 2O + Cl2
3d 4s
due to strong ligand 2KMnO4 + 16HCl ® 2KCl + 2MnCl2 + 8H 2O + 5Cl 2
Chlorine is not obtained by dil HCl
Co 3+ has electronic configuration Ag 2S2 O3 + H 2 O ® Ag 2S + H 2SO 4
3d 4s Black ppt
No pairing of electrons 21. (b) 1M KMnO4 º 5N KMnO4 º 5N FeSO4
\ 10×1M º 10×5N º 10 × 5N
3- KMnO4 KMnO4 FeSO4
due to weak ligand hence ëéCoF6 ûù is paramagnetic.
4- 22. (a) Cr2 O27 - + 6e - + 14H + ® 2Cr 3+ + 7H 2O
6. (a) Fe ++ + 6KCN ® éë Fe (CN )6 ùû + 6K +
Sn 2 + ® Sn 4+ + 2e-
7. (c) The reaction at 500 C is Fe2O3 + 3CO ® 2Fe + 3CO2
o
one mol of Sn 2+ provide 2 mol of e - which will reduce
++ 2+
8. (c) Cu + 4NH 4OH ® [Cu(NH3 ) 4 ] + 4H 2O 1 3Cr2 O72-
deep blue comp
9. (a) Ans (a) See text 23. (a) CrO3 + 2NaOH ® Na 2 CrO 4 + H 2O
23
Coordination Compounds

DOUBLE SALTS : (iv) Coordination number - Number of ligand donor atoms


(not number of ligands) in a coordination compound (or
The addition compounds which retain their identity in solid form
complex) or number of electron pairs arising from ligand
only and not in solution are known as double salts eg carnallite. donor atoms to which the metal is directly bonded.
® K + + Cl - + Mg 2 + (aqs)
KCl.MgCl2 .6H 2 O ¾¾ Coordination number range from 1 to 12 (> 12 for some f-
block element.
In aqueous solution carnallite shows the properties of K+, Mg2+ (v) Types of ligands
and Cl– ions (a) Unidentate - Which binds to a metal through a single
. .
COORDINATION COMPOUNDS : point of attachment eg N H 3 , Br etc.
The addition compounds which retain their identity in solid as .. .. ..
well as in solution. Such compounds contain a complex ion formed N H3 H2 O
.. P H3 NO CO CS
by the combination of a metal atom and other species having .. ..

lewis base character eg.


C 5 H 5 N , ( C 6 H 5 ) 3 P , X – (Cl,Br,I) : OH – : CN – O 2- ,
O 22 - , CO 32 - , NO 2 – , SO 24 - , SCN – , CH 3COO – ,
K 4 [Fe ( CN ) 6 ](s ) 4K + + [Fe (CN ) 6 ]4 -
S2 - , S2 O32 - , NO3– , SO 32 - , NH 2 – , NH 2–
Since the complex ion contains a number of coordinate bonds
they are also known as coordination compounds. (b) Bidentate - Which binds to a metal through two points
eg
SOME DEFINITIONS : .. ..
(i) Complex - A central metal atom /ion surrounded by a set of H 2 N - CH 2 - CH 2 N H 2 (binds through N)
..
ligands H 3C - C = N OH
C 2 O 24 - | ..
(ii) Ligand - An ion or a molecule that can have an independent ,
H 3C - C = N OH
existence and can donate a pair of electrons. It can be
(c) Polydentate - Several donor atoms are present in one
negative ion, neutral molecule or positive cation (though molecule
rare in nature) (Dentate- derived from teeth)
(iii) Coordination compound - A neutral complex or ionic ..
_
compound in which atleast one of the ions is a complex CH2COO_
C H 2 - N CH COO
| .. 2 _ (EDTA)
A)
formed between a lewis acid ( e -acceptor) and lewis base
CH 2 - N CH2COO_
CH2COO
(e - pair doner ) eg
(vi) Chelate complex - It is formed when a bi or polydentate
Ni(ClO 4 )2 + 6 NH 3 ® [Ni ( NH 3 ) 6 ](ClO 4 ) 2 ligand uses two or more donor atoms to bind to one metal
atom (“chelate” derived from claw)
Ni = lewis acid centre, NH3 = Lewis base
Most common elements to act as donor atoms are N, P, O, S
Complex ion is always written in square brackets halides and C ( in organic metallic compounds)
764 Chemistry
(vii) Bridging ligands - Such ligands can bind to more than one NOMENCLATURE OF COORDINATION
metal atom COMPOUNDS :
(viii)Homoleptic ligand - Metal bound to only one type of donor Nomenclature of coordination compounds follows different rules
group which are as follows.
(ix) Heteroleptic ligand - Metal bound to more than one type 1. Name the cation, then anion
donor group 2. Non ionic compounds are given one-word name
(x) Co-ordination sphere - The combination of central metal 3. Name ligands
atom and ligands written in “square brackets” is called the (a) Ligands are named first and central atom last
coordination sphere (b) Ligands are named in alphabetical order
(xi) Ionisation sphere - The portion present outside the square (c) Neutral ligands are named the same as the molecule
(except aqua and ammine)
bracket is called ionisation sphere
(d) Anionic ligands are named by adding - O to the stem of
Species present in the coordination sphere are non ionisable
the name (chloride becomes chloro)
and species present in the ionisation sphere are ionisable.
(e) The ligand name is preceeded by a numerical prefix to
indicate how many are present
K4 [ Fe(CN)6 ] di, tri, tetra, penta, hexa
Ionisation Coordination 4. In a neutral or cationic complex, the name of the central
sphere Central Ligand sphere
metal atom is followed by its oxidation number in Roman
metal
atom numerals in parentheses
(xii) Oxidation number - Charge carried by the central metal 5. In anionic complex, the suffix- ate is added to the name of
atom central metal, followed by its oxidation number in Roman
numerals in parentheses
(xiii)Effective atomic number (EAN) - It can be obtained from
6. In case of bridging ligand the word m (mu) is written before
the following simple expression the name of ligand
EAN = Z – O.N + 2 (CN) Formula and names of some ligands
Where Z = Atomic number of central metal atom
H2O aqua OH – hydroxo - NC– Isocyano
O.N. = Oxidation number of central metal atom
CN = Coordination number of central metal atom. CO carbonyl F– fluoro -SCN –
thiocyanato
It is equal to the number of monodentate ligands, twice
the number of bidentate ligands and so on. NH 3 ammine Br – bromo NO3– nitrato
WERNER’S THEORY :
NO nitrosyl CN – cyano -ONO – nitrito
According to Werner’s theory metals have two types of linkages
(valencies) C6 H 5 Phenyl - NCS– Isothiocyanato NH2- imido

1. Primary linkages - Which are satisfied by the negative ions, C5 H 5 N Pyridine SO 24 Sulphato O22- peroxo
ionisable and their number is equal to the O . N of central
metal atom. PH 3 Phosphine NO -2 nitro NH -2 Amido
They are always represented by dotted lines
2. Secondary linkages - Which are satisfied by the negative,
P(C 6 H 5 ) 3 CO 32- Carbonato N 3- nitrido

neutral or a positive species (ligands) and their number is T riphenylphosphine


equal to the coordination number of the central metal atom. H2 N. CSNH 2 O 2- oxo S2 - sulphido
These are non ionisable and represented by complete lines.
Thiourea
Every metal atom has a tendency to satisfy both the
valencies. On the basis of Werner’s theory the structure of H2N. CH2. CH2. NH2 Cl – chloro C 2 O 24 - oxalato
CoCl3. 6NH3 can be represented as follows : ethylene diammine

I– iodo CH 3COO – acetato


H3N NH3 NH3
ligands carrying positive charge have ending of –ium
Cl
+
Cl Co NO 2 nitronium

Cl
[Co( NH 3 ) 6 ]3+ + 3Cl - +
NO nitrosonium
H3N (NH2NH3)+ hydrazinium
NH3 NH3
Coordination Compounds 765
Name of some complex compounds - Following the above rules (iii) Hydration Isomers-Exchange water as ligand and hydrate
here are names of some complex compounds. [Cr(H2O)6 ]Cl3, [Cr(H 2O)5 Cl]Cl2.H 2O and Cr(H 2O) 4 Cl2 C
[Cr(H2O)5 Cl]SO4 Pentaaquachlorochromium
Cr(H 2O)6 Cl3 Cr(H 2O)5 Cl Cl 2H 2O and [Cr(H 2O) 4 Cl 2 ]Cl 2.H 2O
(III) Sulphate
[Co(NH3 )5 H2O]Cl3 and [Co(NH3 )5 Cl]Cl3.H 2O
[Cr(H2O)4 Cl2 ]Cl Tetraaquadichlorochromium (iv) Linkage Isomers - ligands that can bond at more than one
atomic site
(III) Chloride
(ambidentate) : CN– : NC–
K 2 [PtCl 4 ] Potassiumtetrachloroplatinate
[Cr(H 2O)5 CN]2+ and [Cr(H 2 O)5 NC]2 +
(II)
: NCS– and : SCN –
[Co(en) 2 Cl2 ]Cl Dichlorobis (ethylene -
[Co( NH 3 )5 NCS]2+ and [Co( NH 3 )5 SCN ]2 +
diammine) cobalt (III) Chloride
.. .. ..
IV II : NO-2 and : O
.. — N = O
[Pt ( NH3 )4Cl 2 ] [ Pt Cl 4 ] Tetraamminedichloro ..
platinum (IV) [Pt ( NH 3 )3 NO 2 ]+
and [Pt ( NH 3 ) 3 ONO]+
tetrachloroplatinate (II) (v) Coordination Isomerism - Occurs when cationic and anionic
3+ complexes of different metal ions are present in a salt.
NH Interchange of ligand between the complexes give isomers eg
(en)2Co Co(en)2 Bisethylenediaminecobalt (III)
OH [Co( NH3 ) 6 ][Cr (CN) 6 ] is an isomer of [
-m-amido -mhydroxobis
ethylenediaminecobalt (III) ion [Co(CN) 6 ] [Cr ( NH3 ) 6 ]
(vi) Ligand isomerism-Occurs when more than one isomer of
[Fe(H 2 O) 4 (C 2 O 4 )]2 SO 4 Tetraaquaoxalato iron (III) the ligand is possible eg 1, 2 diamino propane and 1,3-diamino
sulphate propane.
(vii) Polymerisation Isomerism - The isomers have the same
[ Ag( NH 3 ) 2 ]Cl Diamminesilver (I) chloride
empirical formula but different molecular weights eg.
[Cu(NH3 ) 4 ]SO4 Tetramminecopper (II) sulphate [Co(NO 2 )3 (NH 3 )3 ] and [Co(NH 3 ) 6 ][Co(NO 2 ) 6 ] .
Triamminetrinitrocobalt (III) Hexamminec obalt (III)
[ Ni(CN) 4 ]2 - Tetracyanonickelate (II) ion Hexanitroc obalt (III)
The molecular weight of the second is twice as that of the
K 2 HgI 4 Potassiumtetraiodomercurate (II) first.
[Cr( NH3 ) 6 ] [Co(C 2O 4 ) 3 ] Hexammine chromiun (III) (viii) Valence Isomerism - In this the same coordinating group is
held by different types of valence bonds. The valence state
trioxalato cobaltate (III)
of the central metal atom then differ in the two isomers. eg.
NH2 [Co( NO)(NH 3 ) 5 ]2 + and [Co( NH 3 ) 5 ( NO)]2 + .
(en)2Co Co(en)2 SO Bis (ethylenediammine)
4 In the first compound the NO group is a negative group and
OH
cobalt (III) m - amido m - oxidation state of Co is +3. In the second compound the NO
hydroroxo bis (ethylene group is neutral and oxidation state of Co is +2.
(ix) Coordination position isomerism - In this coordinating
diammine) cobalt (III)
groups occupy different positions and the isomerism occurs
sulphate
generally in bridged complex. eg.
ISOMERISM IN COORDINATION COMPOUNDS : 2+
NH2
Isomers - two or more forms of a compound having the same (NH3)4 Co Co (Cl)2 (NH3)2 and
composition O2
(i) Structural Isomers - (have different bonding) . They are of
2+
the following types NH2
(ii) Ionization Isomers - Exchange ion between ligand and anion (NH3)3 (Cl) Co Co (Cl) (NH3)3
eg. O2
(x) Stereo isomerism - It is due to different spatial arrangement
[Co(NH3 )4 ClBr ]Cl and [Co(NH3 ) 4 Cl2 ] Br of atoms and groups in a molecule. It is of two types.
(a) Geometrical - It is due to different geometrical
[ Pt(NH3 )3 Cl] NO2 and [Pt(NH3 )3 NO2 ]Cl arrangements of ligands around central metal atom and
is shown by
766 Chemistry
(I) Square planar complexes of the type X X
MA2X2 ; MABX2 ; MABXY A X A A
M M
Type MA2 X-
2 A X A X
[Co(NH3 )2 Cl2 ], [Pt(NH3 )2 Cl2 ] A X
2 H3N Cl Fac (facial) Mer (meridranal)
H3N 1 Cl
The geometrical isomerism is not possible in square planar
Pt Pt MA4, MAB3 and tetrahedral MA4, MA2B2 and MABCD
H3N 4 3 Cl Cl NH3 and Octahedral MA6, MA5B.
Pale yellow Dark yellow Facial (fac) meridional (mer) isomerism - The octahedral
Cis isomer trans isomer coor dination compounds of the type MA 3 B 3 eg.
Positions 1,2 and 1,4 are cis while 1,3 and 2,4 are trans. Co(NO2)3(NH3)3 exhibit this type of geomerical isomerism
Type MABX2 - [Co ( NH 3 ) 2 Cl Br ] of each trio of donor atoms occupy adjacent positions at
the corners of an octahedral face the isomer is known as
H3N Br Br NH3 facial (fac) isomerism. When the positions are around the
Co Co meridian of the octahedron we have meridional (mer) isomer.
H3N Cl H3N Cl NH3 NH3
Cis Trans O2N NH3 H3N NO2
Type MABXY - Co Co

Example is éëPt(NH 3 )(NH 2OH) (NO2 )(py) ùû NO 2 . Its three O2N NH3 O2N NO2
NO2 NH3
isomers are possible.
Fac. Mer.
Type M(AA1)2 - Where AA1 is unsymmetrical bidentate eg
(b) Optical isomerism - Non superimposable mirror images
.. are called optical isomers and may be described as
[Pt(Gly)2], here gly = H 2 N CH 2COO
“chiral’. They are also called enantiomers and they rotate
H2C H2N NH2 CH2 plane polarised light in opposite directions.
Pt
OC O O CO N N N
Cis - isomer N
N Co Cl Cl Co N
H2C H2N O CO
N N
Pt
Cl Cl
OC NH2
O CH2
Trans - isomer Optical isomerism - It is given by octahedral complexes of
(II) Octahedral of the type : MA4XY, MA4X2 MA3X3 the type
MA2X2Y2. M(AA)2X2 and M(ABCDEF). In the last M(AA')3 (Cis or trans) M(AA)3 ; M(AA)2 B2 (Cis not
type 15 geometrical isomers are possible.
trans) ; M(AA)2 BC (Cis form) ; M(AA)B2C2 , MA2B2C2
AA= symmetrical bidentate eg
(Cisform), MA2B2CD, MA2BCDE, MABCDEF.
2+
Type MA4X2 - [Pt ( NH 3 ) 4 Cl 2 ] , [Co ( NH 3 ) 4 Cl 2 ]+ , All exist in three forms two optically active and one
optically inactive. Examples are
[Pt ( NH 3 ) 2 Cl 4 ]
3+ + + +
X1 X ëé Co(en)3 ûù , [ Co(en) 2 Cl2 ] , [Rh(en) 2 Cl2 ] ,, Co(en) 2 CIBr ,
5
A X2 A A
+ + +
M ëé Co(en)
M 3 ûù ; Co(en) 2 Cl2 Rh(en) 2 Cl2 [Co(en)2 ClBr ]+ , ëéCo(en)2 (NH3 )2 Cl2 ûù +
A A3 A A
4 Tetrahedal Complexes - Tetrahedral complexes of the type
A6 X (M(AA 1 ) 2 show optical activity. Examples are
Cis Trans Bis(benzoylacetonato) Be (ii) and Bis (glycinato) Ni (ii)
Note : The positions 1,6 and 2,4 and 3,5 are trans. Tetrahedral complexes can have optical isomers if all four
ligands are different eg MABCD
Type MA3X3 - [Pt ( NH 3 )3 Cl3 ] +, [Co ( NH 3 ) 3 Cl 3 ]
Coordination Compounds 767
VALENCE BOND THEORY : Outer electronic configuration of Fe(At. No 26)
Features of this theory are
1. Uses hybrid orbitals to hold the donated electron pairs for 3d 4s 4p
the formation of the coordinate bonds. Outer electronic configuration of Fe2+
2. Can explain the structure and magnetic properties eg
consider the ions
3- 3+ 3d 4s 4p 4d
ëé Co(CN)6 ûù or ëé Co(NH3 ) 6 ûù 3 2
sp d
Outer electronic configuration of Co At . No. 27 The coordination number is six. We need six empty valence
atomic orbitals.
3d 4s 4p H2O is a weak ligand, pairing of electrons is not possible
Outer electronic configuration of Co3+
hence hybridisation sp 3 d 2 . Hen ce [Fe(H 2 O) 6 ]2 + is
octahedral, paramagnetic in nature, outer complex and high
spin complex.
(CN - and NH3 are strong ligands and do pairing of Third example - Structure of [CuX 4 ]2-
electrons).
The coordination number is six. We need six empty atomic Cu2+ has electronic configuration
orbitals to accomodate electrons donated by CN or NH3.

Since the coordination number is 4, we need


four hybrid atomic orbitals. Hybridisation should be sp 3
3d 4s 4p but X-ray analysis reveals the presence of four ligands in
2 3
d sp hybridisation the same plane hence hybridisation should be square planar.
d2sp3 hybridisation is octahedral. As shown above there is For this Cu2+ should have the configuration.
no unpaired electron hence the complex ions are diamagnetic
in nature. It is inner complex since d atomic orbitals come
from inside and low spin complex 3
dsp
Second example - Structure of [Fe ( H 2 O) 6 ] 2+

Hence [CuX 4 ]2- is square planar, paramagnetic and inner


complex.

Geometry (shape) and magnetic nature of some of the complexes


(Application of valence bond theory)
Atom/Ion/ Configuration Oxidation Type of Geometry No. of Magnetic
Complex state of Hybridization shape unpaired property
metal electrons
3d 4s 4p
Cr3+(d 3) +3 3 Paramagnetic

[Cr(NH3)6]3+ : : : : : : +3 d2sp3(Inner) Octahedral 3 Paramagnetic


2
d sp3
4d
[Cr(H2O)6]3+ : : : : : : +3 sp3d2 (Outer) Octahedral 3 Paramagnetic
3 2
sp d
3d 4s 4p
Ni2+(d 8) +2 2 Paramagnetic
[NiCl4]2- : : : : +2 sp3 Tetrahedral 2 Paramagnetic

sp3
[Ni(CN)4]2- : : : : : +2 dsp2 Square planar 0 Diamagnetic
2
Rearrangement dsp
768 Chemistry

Ni(d 8s2) 0 2 Paramagnetic


[Ni(CO)4] : : : : 0 sp3 Tetrahedral 0 Diamagnetic
3
Rearrangement sp
Cu2+(d9) 1 Paramagnetic

[CuCl4]2- : : : : +2 dsp2 Square planner 1 Paramagnetic


2
dsp

[Cu(NH3)4]2+ : : : : +2 dsp2 Square planar 1 Paramagnetic


2
dsp
One electron is shifted
from 3d to 4p-obrital
3d 4s 4p
Mn2+(d 5) +2 5 Paramagnetic

[Mn(CN)6]4- : : : : : : +2 d2sp3(Inner) Octahedral 1 Paramagnetic


2 3
Rearrangement d sp

[MnCl4]2- : : : : +2 sp3 Tetrahedral 5 Paramagnetic


3
sp
Fe2+(d 6) +2 4 Paramagnetic

[Fe(CN)6]4- : : : : : : +2 d2sp3 (Inner) Octahedral 0 Diamagnetic


2 3
Rearrangement d sp

[Fe(H2O)6]2+ : : : : : : +2 sp3d 2 (Outer) Octahedral 4 Paramagnetic


3 2
sp d

[Fe(NH3)6]2+ : : : : : : +2 sp3d 2 (Outer) Octahedral 4 Paramagnetic


3 2
sp d
Fe3+(d 5) +3 5 Paramagnetic
[Fe(CN)6 ]3- : : : : : : +3 d2sp3 (Inner) Octahedral 1 Paramagnetic
2 3
d sp

Fe 0 Trigonal 4 Paramagnetic

[Fe(CO)5] : : : : : 0 dsp3 (Inner) Bipyramidal 0 Diamagnetic


3
dsp
Co3+(d 6) +3 4 Paramagnetic
[CoF6 ]3- : : : : : : +3 sp3d 2 (Outer) Octahedral 4 Paramagnetic
3 2
sp d

[Co(NH3)6]3+ : : : : : : +3 d2sp3 (Inner) Octahedral 0 Diamagnetic


2 3
Rearrangement d sp
Co2+(d 7) +2 3 Paramagnetic
[Co(H2O)6]2+ : : : : : : +2 sp3d 2 (Outer) Octahedral 3 Paramagnetic
3 2
sp d
Limitations of valence bond theory are following 2. It can not explain colour and spectra
1. It does not explain the relative stability of complexes. 3. The relative stability of structural isomers.
Coordination Compounds 769
CRYSTAL FIELD THEORY : In [NiCl4]2– , Cl– is a weak field ligand which do not forces elec-
Crystal field theory (CFT) was proposed by Bethe and Ven Vleck. trons of Ni2+ for pairing
It gives satisfactory explanation for the proporties and bonding
in co-ordination compounds. The main points of this theory are [ Ni Cl4]2– \ [Ar]
following :- 3d 4s 4p
(i) The attraction between the central metal and ligands in the
complexes is considered to be purely electrostatic

®
®

®
®
®

®
®
Thus bonding in the complex may be ion-ion attraction or
ion dipole attraction. Hybridisation sp3 \ tetrahedral
(ii) Ligands are treated as point of negative charges COLOURS OF METAL COMPLEXES :
(iii) There is no interaction between metal orbitals and ligand It is due to electronic transitions between t2g and eg energy levels.
orbitals The energy of an electron is increased by absorbing light energy
(iv) The d-orbitals present in metal have the same energy in the
and it moves to a higher energy level.
free state. This is called degenerate state of d-orbital. But,
Energy of a photon = Energy difference between the ground
when a complex is formed the ligands destroy the degen-
eracy of these orbitals. This effect is Known as Crystal field state and an excited state
splitting of d- orbitals. c
It accounts for both the colour and the magnetic properties of E = hn = h
l
complexes. It is based on d- orbital energy level splitting
h = Planck’s constant (6.63x10-34 J.sec.) u = frequency of light. E
2 2 2
eg = energy of photon (measured with UV. or visible spectroscopy)
dz dx – y
MAGNETIC PROPERTIES OF METAL COMPLEXES :
D = crystal field
splitting energy • Paramagnetic - unpaired electrons
energy
dxy dxz dyz
t2g • Diamagnetic - no unpaired electrons
• Determined from crystal field splitting diagrams
d orbitals of the free splitting of d-orbitals of the metal ion
metal ion in an octahedral field of ligands STABILITY OF COORDINATION COMPOUNDS IN
SOLUTION :
The size of D depends on -
(i) The nature of the ligand “Spectro chemical series” D Consider the following equilibrium between undissociated
complex ion and dissociated ion.
decreases as shown below
[MLn]b+ Ma+ +nLx-
I – < Br – < Cl – < OH – < F- < C2O 42- < H 2O < CNS– <
[M a + ][Lx - ]n
weak ligands The equilribrium constant Kc =
[(MLn ) b + ]
< NH 3 < en < NO2 – < CN – < CO
strong ligands The smaller the value of Kc, the greater is the stability of complex
(ii) The oxidation state of the metal D is greater for M3+ than ion and vice versa. The reciprocal of equilibrium constant is called
stability constant.
for M2+
(iii) The row of the metal in the periodic table. For a given ligand 1 [MLn]b +
and oxidation state of the metal, D increases going down in Ks = =
K c [M a + ][Lx - ]n
a group eg.
D is greater in [Ru (NH3)6]3+ than in [Fe(NH3)6]3+ The higher the value of Ks, the more is the stability of complex
Effect of strong field ligands and weak field ligands– Strong field ion. The value of Ks depends on.
ligands forces the electrons of central metal for pairing and the 1. Nature of central metal atom - The more the polarizing power
complex formed is known as low spin complex. While weak field of the central metal ion the more is the stability of complex
ligands do not forces the electron of central atom for pairing and ion.
the compelx formed is known as high spin complex. For example– charge
In [Ni (CN)4], CN is a strong field ligand which forces electrons of The polarising power f =
radius
Ni 2+ for pairing Thus complex of Fe3+ is more stable then Fe2+
2. Nature of ligand - Since ligand is a Lewis base the more the
Ni 2+; [Ar] 3d8
®

®
®
®

basic character of ligand the more is the stability of complex


[Ni(CN)4]2– ion. Thus complex ion of F–is more stable than that of
3d 4s 4p Cl or Br
\ [Ar] Chelating ligands give much larger values of stability
®

®
®

®
®

®
®
®

constant.
[Ni(H 2 O)6 ]2+ ¾¾¾ ®[Ni(NH3 )6 ]2 + , K f = 4x108
36NH

electrons provided by 4CN
Hybridisation: dsp 2 (square planar)
[Ni(H 2 O)6 ]2+ ¾¾
3en
¾®[Ni(en)3 ]2+ , K f = 2x1018
770 Chemistry
Perfect or penetrating complexes : They are fairly stable and ORGANOMETALLIC COMPOUNDS :
dissociate negligibly or not at all e.g.
The organic compounds having metal atom directly attached to
the carbon are known as organometallic compounds. They can
Imperfect or normal complexes : The complex ion is reversibly be
dissociated
1. Sigma bonded -
• R - Mg - X Alkyl magnesium halide commonly known as
IMPORTANCE OF COORDINATION COMPOUNDS : Grignard’s reagent
1. Biological processes
Haemoglobin - Oxygen carrier is a complex of Iron (II) 2. -bonded
Chlrorophyll - Green colouring matter of plants is a complex (CH3)4 Sn (Tetramethyl tin), (C2H5)2 Zn (Diethyl Zinc), n–
of Mg C4H9Li (n–butyl lithium)
Vitamin B 12 - It is a complex of cobalt K [PtCl3–h2 –(C2H4)] (Zeise’s salt)
2. Analytical chemistry - Many metal ions are quantitatively
estimated by complex formation eg Cu++, Ni2+, Fe3+, Al3+ CH2
OH OH Cl
CH2 +
Pt K
CH3 C= N N=C CH3
Ni Cl Cl
CH3 C= N N= C CH3

OH OH
Red precipitate of nickel with dimethyl glyoxime
Fe[h5– C5H5]2 Ferrocene Fe
• Separation of Ag+ and Hg 22 + , Ag+ form soluble
complex
AgCl + 2NH4OH ® [Ag (NH3)2]Cl + 2H2O
Soluble complex
Hg 2 Cl 2 + NH 4OH ® Hg ( NH 2 )Cl + Hg + HCl + H 2 O
142
4 43 4 Cr
Black inso lub le Cr[h6– C6H6]2 Dibenzene chromium
3. Metallurgical processes
(a) Bauxite ore of aluminium is purified by soluble complex SYNTHESIS OF ORGANOMETALLIC COMPOUNDS:
formation
1. Synthesis of Grignard’s reagent - By reaction between an
Al2O3 + 3H2 O + 2OH – ® 2 Al(OH) 4 alkyl halide and Mg in presence of ether
Impurites of Fe2O3 are left behind in solution.
ether
(b) Extraction of silver and gold by cyanide process involves R - X + Mg ¾¾¾® R - Mg - X
complex formation
Other metals like Li, Na, Zn, Cd can also be used.
Ag + (aq) + 2NaCN(aq) ® Na[Ag(CN) 2 ](aq) + Na + 2. Synthesis of other organometallic compounds using
+ + Grignand’s reagent
Au (aq) + 2NaCN(aq) ® Na[Au(CN) 2 ](aq) + Na
(c) Nickel is purified by Mond’s process forming volatile ether
PbCl 4 + 4C 2 H 5 Mg - Br ¾¾¾®(C 2 H 5 ) 4 Pb + 4MgBrCl
nickel carbonyl
ether
D
Ni + 4CO ® Ni (CO ) 4 ¾¾® Ni + 4CO PCl 3 + 3C 6 H 5 MgCl ¾¾¾® P( C 6 H 5 ) 3 + 3MgCl 2

4. Photography - Excess of AgBr is removed by complex 3. Preparation of complexes


formation Zeise’s salt :-
AgBr(s) + 3 Na2S2O3 (aq) ® Na3 [Ag(S2O3)2] (aq)+ NaBr (aq)
5. Miscellaneous uses H 2 C = CH 2 + K 2 PtCl4 ® [PtCl3 (C 2 H 4 )]K + + KCl
• K [Ag(CN)2] complex of silver is used in silver plating Dibenzene chromium :-
• EDTA (ethylene diammine tetra acetate) is used for the
2C6 H 6 + Cr(vapour) ® [(C6 H 6 ) 2 Cr]
estimation of Mg2+ and Ca2+ ions and for removal of
hardness of water Ferrocene :-
• [Pt(NH3)2Cl2] known as cisplatin is used in the treatment
2C5H 5MgBr + FeCl2 ® [(C5 H5 ) 2 Fe] + 2MgBrCl
of cancer
Coordination Compounds 771
Metal carbonyls : Preparation of metal carbonyls - By passing CO over heated
The compounds of carbon monoxide with certain transition metal eg
metals are known as metal carbonyls 320 -340 K
Ni + 4CO ¾¾ ¾ ¾¾® Ni( CO ) 4
CO CO CO
CO OC CO Fe + 5CO ¾
¾® Fe( CO ) 5
Ni OC Fe M Uses of organometallic compounds.
OC
CO
CO
CO CO OC CO CO 1. Grignard’s reagent is employed for the synthesis of number
of organic compounds eg alcohols, aldehydes, ketones,
M= Cr, Mo, W
esters etc.
Polynuclear metal carbonyls are also known e.g. Fe3(CO)12
2. Nickel is purified by Monds, process forming Ni(CO)4.
Mn2(CO)10
3. Zeigler Natta Catalyst which is mixture of triethyl aluminium
The metal carbon bond in carbonyls may be
and titanium Chloride (Al (C2H5)3 + TiCl3) is used for
represented as
polymerisation of ethene.
M¬ Cº O. Due to some back bonding by sidewise overlapping
4. Tetraethyl lead is used as antiknock compound Pb(C2H5)4 .
between d orbitals of metal and empty p-orbitals of carbons, the
5. Wilkinson’s catalyst (Ph 3 P) 3 RhCl. for selective
M–C bond length is somewhat shorter and C–O bond is longer
hydrogenation .
than triple bond.
6. C2H5HgCl (ethyl mercury chloride) as fungicide.
M¬ C º O
M C=O
M CºO

s Bond
772 Chemistry

Very Short/Short Answer Questions 11. Arrange the following complexes in order of increasing
electrical conductivity:
1. What is the oxidation state of Co in complex [Co (NH3)2
[Co (NH3)3 Cl3], [Co (NH3)5 Cl] Cl2 , [Co (NH3)6] Cl3, [Co
(NO2) Cl] [Au (CN)2]?
(NH3)4 Cl2] Cl.
2. Write IUPAC name of complex,
12. Write the correct formula for the following compounds :
NO2 3+ (a) CrCl 3×6H2 O (violet, with three chloride ions/unit
(NH 3) Co Co (NH3) . formula)
NH (b) CoCl3×6H2O (light green colour with two chloride ions/
3. When copper sulphate solution is mixed with liquid ammonia unit formula)
(in molar ratio of 1 : 4), the resultant solution does not give (c) CrCl3×6H2O (dark green colour with one chloride ion
the test of Cu2+ ion. Why? (unit formula)
4. How many moles of Cl– ions are formed by one mole of each 13. Write the name, the state of hybridization, the shape and
of the following in aqueous solution : the magnetic behaviour of the following complexes:
(a) K2 [PtCl6]
2- -
(b) [Pt (NH3)3 Cl3]Cl [CoCl4 ]2- , éë Ni ( CN )4 ùû , éëCr ( H 2 O )2 ( C 2 O 4 ) 2 ùû
(c) [Pt (NH3)4 Cl2]Cl2 (At. No.: Co = 27, Ni = 28, Cr = 24)
5. How is tetrabutyl lead is prepared? 14. Write the name, stereochemistry and magnetic behaviour
6. Among Ag(NH3)2Cl, [Ni (CN)4]2– and [CuCl4]2–, which of the following:
(a) has square planar geometry? (At nos. Mn = 25, Co = 27, Ni = 28)
(b) remains colourless in aqueous solution and why? (i) K 4 éë Mn ( CN )6 ùû
[Ag (Z = 47), Ni (Z = 28), Cu (Z = 29)].
7. Assign the formula to complex formed by Fe2+ with following (ii) éë Co ( NH 3 )5 Cl ùû Cl2
ligands:
(a) three cyanide ion and three ammonia molecules. (iii) K 2 éë Ni ( CN ) 4 ùû
(b) one hydroxy ion and two ammonia molecules and three 15. Write the name, the structure and the magnetic behaviour
chloride ions. of each one of the following complexes:
8. Answer the following questions referring to [Al (H2O)6 ]3+ :
(i) éë Pt ( NH 3 ) Cl ( NO 2 ) ùû
(a) Coordination number of Al
(b) Oxidation state of Al (ii) éë Co ( NH 3 ) 4 Cl2 ùû Cl
(c) Hybridisation involved in complex formation
(iii) Ni(CO)4
(d) Shape of complex ion
[Al nos, Co = 27, Ni = 28, Pt = 78)
(e) Paramagnetic or diamagnetic
16. Write the IUPAC names of the following coordination
9. Give (a) linkage isomer of [Cr (CN) (H2O)5]2+ compounds:
(b) ionisation isomer of [Pt Cl2 (NH3)4] Br2
(i) éë Cr ( NH 3 )3 Cl3 ùû
(c) coordination isomer of [Pt (NH3)4] [CuCl4]
10. Mention the application of organometallic compounds in
(ii) K3 éë Fe ( CN )6 ùû
the following areas:
(a) Homogeneous catalysis +
(b) Heterogeneous catalysis (iii) é CoBr2 ( en ) ù , (en = ethylenediamine)
ë 2û
(c) Organic synthesis.
Coordination Compounds 773
Multiple Choice Questions (a) [MA5B] (b) [MA2B4]
17. An example of double salt is (c) [MA3B3] (d) [MA4B2]
(a) Bleaching powder (b) K 4 [Fe(CN) 6 ] 23. Considering H 2 O as a weak field ligand, the number of
(c) Hypo (d) Potash alum unpaired electrons in [Mn (H 2 O) 6 ]2 + will be (At. no. of
18. The number of unpaired electrons in the complex ion
[CoF6]3– is (Atomic no.: Co = 27) Mn = 25)
(a) zero (b) 2 (a) three (b) five
(c) 3 (d) 4 (c) two (d) four
19. Hybridization of Ag in the linear complex [Ag(NH3 )2 ]+ is 24. The complex, [Pt(Py)(NH3)BrCl] will have how many
geometrical isomers ?
:
(a) 3 (b) 4
(a) dsp 2 (b) sp (c) 0 (d) 2

(c) sp
2
(d) sp
3 25. On mixing conc. NH 4OH to a Cu 2 + salt, the following
20. Ammonia will not form complex ions with blue complex is formed :

(a) Ag + (b) Cd 2+ (a) [Cu ( NH 4 ) 4 ]2 + (b) [Cu( NH 3 ) 2 ]2 +


2+
(c) Cu 2 + (d) Pb
(c) [Cu( NH 3 ) 4 ]2 + (d) [Cu ( NH 4 ) 2 ]2 +
21. Which statement is incorrect?
(a) Ni(CO)4 – Tetrahedral, paramagnetic 26. A square planar complex is formed by hybridisation of which
(b) [Ni(CN)4]2– – Square planar, diamagnetic atomic orbitals?
(c) Ni(CO)4 – Tetrahedral, diamagnetic
(d) [NiCl4]2– – Tetrahedral, paramagnetic (a) s, px , py , dyz (b) s, p x , p y , d
x 2 - y2
22. Which one of the following octahedral complexes will not
show geometric isomerism? (c) s, p x , p y , d (d) s, py , pz , dxy
z2
( A and B are monodentate ligands)

1. Which of the following is odd one out ? 6. In the compound lithium tetrahydridoalumninate, the ligand
(a) potassium ferricyanide is
(b) ferrous ammoniumsulphate (a) Al+ (b) H
(c) H – (d) None of these
(c) potassium ferrocyanide
(d) tetraamminecopper (II) sulphate 7. In which of the following coordinate compounds the central
metal atom obeys the EAN rule.
2. CuSO4 dissolves in NH3 due to formation of
(a) K3[Fe(CN)6] (b) K4[Fe(CN)6]
(a) Cu (OH)2 (b) [Cu(NH3)4]SO4
(c) [Cu(NH3)4]SO4 (d) All of these
(c) [Cu(NH3)4(OH)2] (d) CuO 8. The EAN of iron in [Fe(CN)6]3– is
3. K4[Fe(CN)6] is a (a) 34 (b) 36
(a) double salt (b) complex compound (c) 37 (d) 35
(c) neutral molecule (d) None of these 9. Coordination number of Ni in [Ni(C2O4)3]4– is
4. AgCl is soluble in NH4OH solution. The solubility is due to (a) 3 (b) 6
formation of (c) 4 (d) 5
(a) AgOH (b) Ag2O 10. The coordination number of Fe(II) in oxyhaemoglobin is
(c) [Ag(NH3)2]+ (d) NH4Cl (a) 6 (b) 4
5. Some salts although containing two different metallic (c) 8 (d) 10
elements give test for only one of them in solution. Such 11. Correct formula of the complex formed in the brown ring test
salts are for nitrates is
(a) complex (b) double salts (a) FeSO4.NO (b) [Fe(H2O)5NO]2+
(c) [Fe(H2O)5NO] + (d) [Fe(H2O)5NO]3+
(c) normal salts (d) None of these
774 Chemistry
12. According to Lewis, the ligands are 22. The hypothetical complex chloro-diaquatriamminecobalt (III)
(a) acidic in nature chloride can be represented as
(b) basic in nature (a) [CoCl(NH3)3(H2O)2 ]Cl2
(c) some are acidic and others are basic (b) [Co(NH3)3(H2O)Cl3]
(d) neither acidic nor basic (c) [Co(NH3)3(H2O)2Cl]
13. According to the postulates of Werner for coordination
(d) [Co(NH3)3(H2O)3]Cl3
compounds
23. The oxidation state of Fe in K4[Fe(CN)6] is
(a) primary valency is ionizable
(b) secondary valency is ionizable (a) +2 (b) +6
(c) primary and secondary valencies are non-ionizable (c) +3 (d) +4
(d) only primary valency is non-ionizable. 24. Which of the following complex will show geometrical as
14. In [Co(NH3)6]Cl3, the number of covalent bonds is well as optical isomerism (en=ethylenediammine)
(a) 3 (b) 6 (a) Pt(NH3)2Cl2 (b) [Pt(NH3)Cl4]
(c) 9 (d) 18 (c) [Pt(en)3] 4+ (d) [Pt(en)2Cl2]
15. When AgNO3 is added to a solution of Co(NH3)5Cl3, the 25. The number of geometrical isomers from [Co(NH3)3(NO2)3]
precipitate of AgCl shows two ionisable chloride ions. This is
means : (a) 2 (b) 3
(a) Two chlorine atoms satisfy primary valency and one
(c) 4 (d) 0
secondary valency
26. The total number of possible isomers of the complex
(b) One chlorine atom satisfies primary as well as secondary
valency compound [CuII(NH3)4][PtIICl4] is
(c) Three chlorine atoms satisfy primary valency (a) 3 (b) 6
(d) Three chlorine atoms satisfy secondary valency (c) 5 (d) 4
16. The formula dichlorobis (urea) copper (II) is 27. A coordination complex compound of cobalt has molecular
(a) [Cu{O = C(NH2)2}2]Cl2 (b) [CuCl2 {O = C(NH2)2}2] formula containing five ammonia molecules, one nitro group
(c) [Cu{O=C(NH2)2}Cl]Cl (d) [CuCl2{O=C(NH2)2}H2] and two chlorine atoms for one cobalt atom. One mole of
17. Chemical formula for iron (III) hexacyanoferrate (II) is this compound produces three mole ions in an aqueous
(a) Fe[Fe(CN)6] (b) Fe3[Fe(CN)6] solution. On reacting this solution, with excess of AgNO3
(c) Fe3[Fe(CN)6]4 (d) Fe4[Fe(CN)6]3 solution two moles of AgCl get precipitated. The ionic
18. IUPAC name of Na3[Co(NO2)6] is formula of this complex would be
(a) sodium cobaltinitrite (a) [Co(NH3)4NO2Cl] [(NH3)Cl]
(b) sodium hexanitritocobaltate (III) (b) [Co(NH3)5Cl] [Cl(NO)2]
(c) sodium hexanitrocobalt (III) (c) [Co(NH3)5(NO2)]Cl2
(d) sodium hexanitrocobaltate (III) (d) [Co(NH3)5][(NO2)2Cl2]
19. Ligand in a complex salt are 28. The number of isomers exhibited by [Cr(NH3)3Cl3] is
(a) anions linked by coordinate bonds to a central metal (a) 2 (b) 3
atom or ion
(c) 4 (d) 5
(b) cations linked by coordinate bonds to a central metal or
ion 29. For the square planar complex [M (a) (b) (c) (d)] (where M =
(c) molecules linked by coordinate bonds to a central metal central metal and a, b, c and d are monodentate ligands), the
or ion number of possible geometrical isomers are
(d) ions or molecules linked by coordinate bonds to a central (a) 1 (b) 2
atom or ion (c) 3 (d) 4
20. The IUPAC name of K3[Ir(C2O4)3] is 30. Which of the following will exhibit optical isomerism ?
(a) potassium trioxalatoiridium (III) (a) [Cr(en) (H2O)4]3+
(b) potassium trioxalatoiridate (III) (b) [Cr(en)3]3+
(c) potassium tris (oxalato) iridium (III) (c) trans-[Cr(en)(Cl2)(NH3)2]+
(d) potassium tris (oxalato) iridate (III) (d) [Cr(NH3)6]3+
21. In Zeigler- Natta polymerisation of ethylene, the active 31. Which of the following will give maximum number of isomers?
species is
(a) [Co(NH3)4Cl2] (b) [Ni(en)(NH3)4]2+
(a) AlCl3 (b) Et3Al
(c) [Ni(C2O4)(en)2] (d) [Cr(SCN)2(NH3)4]2+
(c) CH2CH2 (d) TiIII
Coordination Compounds 775
32. The compounds [PtCl2(NH3)4]Br 2 and [PtBr 2(NH3)4]Cl 2 45. The valency of Cr in the complex [Cr(H2O)4Cl2]+ is
constitutes a pair of
(a) 1 (b) 3
(a) coordination isomers (b) linkage isomers
(c) 5 (d) 6
(c) ionization isomers (d) optical isomers
33. Which of the following compounds exhibits linkage 46. Which of the following is expected to be a paramagnetic
isomerism ? complex ?
(a) [Co(en)2]Cl3 (b) [Co(NH3)6][Cr(en)3] (a) [Ni(H2O)6]2+ (b) [Ni(CO)4]
(c) [Zn(NH3)4] 2+ (d) [Co(NH3)6]3+
(c) [Co(en)2NO2Cl]Br (d) [Co(NH3)5Cl]Br2
34. Both geometrical and optical isomerisms are shown by 47. Which statement is incorrect ?
(a) [Co(en)2Cl2]+ (b) [Co(NH3)5Cl]2+ (a) [Ni(CO) 4 ] – Tetrahedral, paramagnetic
(c) [Co(NH3)4Cl2] + (d) [Cr(ox)3]3–
35. Which one of the following will not show geometrical (b) [ Ni(CN) 4 ]2 - – Square planar, diamagnetic
isomerism ?
(c) [Ni(CO) 4 ] – Tetrahedral, diamagnetic
(a) [Cr(NH3)4Cl2]Cl (b) [Co(en)2Cl2]Cl
(c) [Co(NH3)5NO2]Cl2 (d) [Pt(NH3)2Cl2 ] (d) [ NiCl4 ]2 - – Tetrahedral, paramagnetic
36. A similarity between optical and geometrical isomerism is
that 48. The geometry of the compound [Pt(NH3)2Cl2] is
(a) each gives equal number of isomers for a given (a) square planar (b) pyramidal
compound (c) tetrahedral (d) octahedral
(b) if in a compound one is present then so is the other 49. The shape of cuprammonium ion is
(c) both are included in stereoisomerism (a) octahedral (b) tetrahedral
(d) they have no similarity
(c) trigonal (d) square planar
37. The type of isomerism present in nitr openta-
50. Which of the following species represent the example of
amminechromium (III) chloride is
dsp2 - hybridisation ?
(a) optical (b) linkage
(c) ionization (d) polymerization (a) [Fe(CN)6]3– (b) [Ni(CN)4]2–
38. Which of the following has square planar structure? (c) [Ag(CN)2]– (d) [Co(CN)6]3–
51. Consider the following complex [Co(NH3)5CO3]ClO4. The
(a) [Ni(CO)4] (b) [NiCl4]2–
coordination number, oxidation number, number of d-
2+
(c) [Ni(CN)4]2– (d) é Ni ( H 2O) ù electrons and number of unpaired d-electrons on the metal
ë 6û are respectively
39. The unpaired electrons in Ni(CO)4 are
(a) 6, 3, 6, 0 (b) 7, 2, 7, 1
(a) zero (b) one
(c) 7, 1, 6, 4 (d) 6, 2, 7, 3
(c) three (d) four
40. Which of the following species represent the example of 52. Atomic numbers of Cr and Fe are respectively 24 and 26.
dsp2 – hybridisation ? Which of the following is paramagnetic with the spin of the
(a) [Fe(CN)6]3– (b) [Ni(CN)4]2– electron ?
(c) [Zn(NH3)4] 2+ (d) [FeF6]3– (a) [Cr(CO)6] (b) [Fe(CO)5]
41. The correct structure of Fe(CO)5 is (Z=26 for Fe) (c) [Fe(CN)6]4– (d) [Cr(NH3)6]3+
(a) octahedral (b) tetrahedral 53. Which of the following will exhibit maximum ionic
(c) square pyramidal (d) trigonal pyramidal conductivity ?
42. The shape of [Cu(NH3)4] is2+
(a) K4[Fe(CN)6] (b) [Co(NH3)6]Cl3
(a) tetrahedral (b) square planar (c) [Cu(NH3)4Cl2] (d) [Ni(CO)4]
(c) pyramidal (d) octahedral
54. Which of the following is organo-metallic compound ?
43. Which one of the following is an example of octahedral (a) Ti(C2H4)4 (b) Ti(OC2H5)4
complex ? (c) Ti(OCOCH3)4 (d) Ti(OC6H5)4
(a) [FeF6]3– (b) [Zn(NH3)4]2+
55. Which is not p - bonded complex ?
(c) [Ni(CN)4]2– (d) [Cu(NH3)4]2+
(a) Zeise’s salt (b) Ferrocene
44. Which of the following is paramagnetic ?
(c) Dibenzene chromiun (d) Tetraethyl lead
(a) [Fe(CN)6]4– (b) [Ni(CO)4]
(c) [Ni(CN)4]2– (d) [CoF6]3–
776 Chemistry

1. Which of the following will give a pair of enantiomorphs? 9. The existence of two different coloured complexes with the
[CBSE-PMT 2007]
composition of [Co(NH3 )4 Cl2 ]+ is due to :
(a) [Cr(NH3)6][Co(CN)6]
(b) [Co(en)2Cl2]Cl [CBSE-PMT 2010]
(c) [Pt(NH3)4] [PtCl6] (a) linkage isomerism (b) geometrical isomerism
(d) [Co(NH3)4Cl2]NO2. (en =NH2CH2CH2NH2) (c) coordination isomerism (d) ionization isomerism
2. The d electron configurations of Cr 2+, Mn2+, Fe2+ and Ni2+ 10. Which one of the following complexes is not expected to
are 3d 4, 3d 5, 3d 6 and 3d 8 respectively. Which one of the exhibit isomerism? [CBSE-PMT 2010]
following aqua complexes will exhibit the minimum
2+
paramagnetic behaviour?
2+
[CBSE-PMT 2007] (a) [ Ni(en)3 ]2+ (b) éë Ni ( NH3 )4 ( H 2O ) 2 ùû
(a) [Fe(H2O)6] (b) [Ni(H2O)6]2+
(c) éë Pt ( NH3 )2 Cl 2 ùû (d) éë Ni ( NH3 ) 2 Cl 2 ùû
(c) [Cr(H2O)6] 2+ (d) [Mn(H2O)6]2+
(At. No. Cr = 24, Mn = 25, Fe = 26, Ni = 28)
11. Of the following complex ions, which is diamagnetic in nature?
3. Which of the following complexes exhibits the highest
[CBSE-PMT 2011]
paramagnetic behaviour ? [CBSE-PMT 2008]
(a) [V(gly)2(OH)2(NH3)2]+ (b) [Fe(en)(bpy)(NH3)2]2+ (a) [NiCl4]2– (b) [Ni(CN)4]2–
(c) [Co(ox)2(OH)2]2– (d) [Ti(NH3)6]3+ (c) [CuCl4]2– (d) [CoF6]3–
where gly = glycine, en = ethylenediamine and bpy = bipyridyl 12. The complexes [Co(NH3 )6] [Cr(CN)6 ] and [Cr(NH3 )6 ]
moities) [Co(CN)6] are the examples of which type of isomerism?
(At.nosTi = 22, V = 23, Fe = 26, Co = 27) [CBSE-PMT 2011]
4. In which of the following coordination entities the magnitude (a) Linkage isomerism (b) Ionization isomerism
D0 (CFSE in octahedral field) will be maximum? (c) Coordination isomerism (d) Geometrical isomerism
[CBSE-PMT 2008] 13. The complex, [Pt(py)(NH3)BrCl] will have how many
(a) [Co(H2O)6] 3+ (b) [Co(NH3)6]3+ geometrical isomers ? [CBSE-PMT 2011]
(c) [Co(CN)6] 3– (d) [Co (C2O4)3]3– (a) 3 (b) 4 (c) 0 (d) 2
(At. No. Co = 27)
14. The d-electron configurations of Cr , Mn , Fe2+ and Co2+
2+ 2+
5. Which of the following does not show optical isomerism?
are d 4 , d 5 , d 6 and d 7, respectively. Which one of the
[CBSE-PMT 2009]
0
following will exhibit minimum paramagnetic behaviour?
(a) [Co(NH3)3Cl3] (b) [Co (en) Cl2 (NH3)2]+
3+
[CBSE-PMT 2011]
(c) [Co (en)3] (d) [Co (en)2Cl2]+ 2+
(a) [Mn(H2O)6] (b) [Fe(H2O)6]2+
(en = ethylenediamine)
(c) [Co(H2O)6]2+ (d) [Cr(H2O)6]2+
6. Which of the following complex ions is expected to absorb
visible light? [CBSE-PMT 2009] (At, nos. Cr = 24, Mn = 25, Fe = 26, Co = 27)
(a) [Ti (en)2(NH3)2]4 + (b) [Cr (NH3)6]3 + 15. Which of the following carbonyls will have the strongest
(c) [Zn (NH3)6]2 + (d) [Sc (H2O)3 (NH3)3]3+ C – O bond ? [CBSE-PMT 2011 M]
(a) [Mn (CO)6] + (b) [Cr (CO)6]
(At. no. Zn = 30, Sc = 21, Ti = 22, Cr = 24)
7. Which of the following complex ion is not expected to absorb (c) [V (CO)6] – (d) [Fe (CO)5]
visible light ? [CBSE-PMT 2010] 16. Which of the following complex compounds will exhibit
highest paramagnetic behaviour? [CBSE-PMT 2011M]
(a) [ Ni(CN)4 ]2- (b) [Cr(NH3 )6 ]3+ (At. No. : Ti = 22, Cr = 24, Co = 27, Zn = 30)
(c) [ Fe(H2 O)6 ]2+ (d) [ Ni(H 2O)6 ]2+ (a) [Ti (NH3)6]3+ (b) [Cr (NH3)6]3+
(c) [Co (NH3)6]3+ (d) [Zn (NH3)6]2+
8. Crystal field stabilization energy for high spin d 4 octahedral
17. Which one of the following is an outer orbital complex and
complex is: [CBSE-PMT 2010]
exhibits paramagnetic behaviour ? [CBSE-PMT 2012 S]
(a) – 1.8 D 0 (b) – 1.6 D 0 + P (a) [Ni(NH3)6]2+ (b) [Zn(NH3)6)]2+
(c) [Cr(NH3)6] 3+ (d) [Co(NH3)6]3+
(c) – 1.2 D 0 (d) – 0.6 D 0
Coordination Compounds 777
18. Red precipitate is obtained when ethanol solution of (b) the number of ligands around a metal ion bonded by pi-
dimethylglyoxime is added to ammoniacal Ni(II). Which of bonds
the following statements is not true ? [CBSE-PMT 2012 M] (c) the number of ligands around a metal ion bonded by
(a) Red complex has a square planar geometry. sigma bonds
(b) Complex has symmetrical H-bonding (d) the number of only anionic ligands bonded to the metal
ion.
(c) Red complex has a tetrahedral geometry.
27. Which one of the following complexes is an outer orbital
(d) Dimethylglyoxime functions as bidentate ligand.
complex ? [AIEEE 2004]
(a) [Co(NH3)6]3+ (b) [Mn(CN)6]4–
OH
H3C C N (c) [Fe(CN)6]4– (d) [Ni(NH3)6]2+
dimethylglyoxime = (Atomic nos. : Mn = 25; Fe = 26; Co = 27, Ni = 28)
H3C C N
OH 28. Coordination compounds have great importance in biological
systems. In this context which of the following statements is
19. Low spin complex of d 6 -cation in an octahedral field will incorrect ? [AIEEE 2004]
have the following energy : [CBSE-PMT 2012 M] (a) Cyanocobalamin is B12 and contains cobalt
(b) Haemoglobin is the red pigment of blood and contains
-12 -12
(a) D0 + P (b) D 0 + 3P iron
5 5
(c) Chlorophylls are green pigments in plants and contain
-2 -2 calcium
(c) D 0 + 2P (d) D0 + P (d) Carboxypeptidase - A is an enzyme and contains zinc.
5 5
29. Which one of the following has largest number of
(D0= Crystal Field Splitting Energy in an octahedral field,
isomers ? [AIEEE 2004]
P = Electron pairing energy)
20. An excess of AgNO3 is added to 100 mL of a 0.01 M solution (a) [Ir(PR 3 ) 2 H(CO)]2 + (b) [Co( NH 3 )5 Cl]2+
of dichlorotetraaquachromium (iii) chloride. The number of
moles of AgCl precipitated would be : [NEET 2013] (c) [Ru ( NH 3 ) 4 Cl 2 ]+ (d) [Co(en ) 2 Cl 2 ]+
(a) 0.002 (b) 0.003 (R = alkyl group, en = ethylenediamine)
(c) 0.01 (d) 0.001 30. The correct order of magnetic moments (spin only values in
21. A square planar complex is formed by hybridisation of which B.M.) is [AIEEE 2004]
atomic orbitals? [AIEEE 2002]
(a) [Fe(CN ) 6 ]4 - > [MnCl 4 ]2 - > [CoCl 4 ]2 -
(a) s, px , py , dyz (b) s, px , p y , d x2 - y 2
(b) [MnCl 4 ]2- > [Fe(CN) 6 ]4 - > [CoCl 4 ]2 -
(c) s, px , p y , d (d) s, py , pz, dxy
z2
(c) [MnCl 4 ]2- > [CoCl 4 ]2 - > [Fe(CN) 6 ]4 -
22. CH3 – Mg – Br is an organometallic compound due to
[AIEEE 2002] (d) [Fe(CN ) 6 ]4 - > [CoCl 4 ]2- > [MnCl 4 ]2 -
(a) Mg – Br bond (b) C – Mg bond
(Atomic nos. : Mn = 25, Fe = 26, Co = 27)
(c) C – Br bond (d) C – H bond.
23. The most stable ion is [AIEEE 2002] 31. The oxidation state Cr in [Cr ( NH 3 ) 4 Cl 2 ]+ is
(a) [Fe(OH)3]3- (b) [Fe(Cl)6]3- [AIEEE 2005]
(c) [Fe(CN)6]3- (d) [Fe(H2O)6]3+.
(a) 0 (b) + 1
24. One mole of the complex compound Co(NH3)5Cl3, gives 3
moles of ions on dissolution in water. One mole of the same (c) + 2 (d) + 3
complex reacts with two moles of AgNO3 solution to yield 32. The IUPAC name of the coordination compound
two moles of AgCl (s). The structure of the complex is K 3[Fe(CN ) 6 ] is [AIEEE 2005]
[AIEEE 2003] (a) Tripotassium hexacyanoiron (II)
(a) [Co(NH3)3Cl3]. 2 NH3 (b) [Co(NH3)4Cl2] Cl . NH3
(b) Potassium hexacyanoiron (II)
(c) [Co(NH3)4Cl] Cl2. NH3 (d) [Co(NH3)5Cl] Cl2
25. In the coordination compound, K4[Ni(CN)4], the oxidation (c) Potassium hexacyanoferrate (III)
state of nickel is [AIEEE 2003] (d) Potassium hexacyanoferrate (II)
(a) 0 (b) +1 33. Which of the following compounds shows optical isomerism?
(c) +2 (d) –1 [AIEEE 2005]
26. The coordination number of a central metal atom in a complex
(a) [Co(CN) 6 ]3 - (b) [Cr(C 2 O 4 )3 ]3 -
is determined by [AIEEE 2004]
(a) the number of ligands around a metal ion bonded by
(c) [ ZnCl4 ]2 - (d) [Cu ( NH 3 ) 4 ]2 +
sigma and pi-bonds both
778 Chemistry
34. Which one of the following cyano complexes would exhibit 44. A solution containing 2.675 g of CoCl3. 6 NH3 (molar mass =
the lowest value of paramagnetic behaviour ? 267.5 g mol–1) is passed through a cation exchanger. The
[AIEEE 2005] chloride ions obtained in solution were treated with excess of
AgNO3 to give 4.78 g of AgCl (molar mass = 143.5 g mol–1).
(a) [Co(CN) 6 ]3 - (b) [Fe(CN) 6 ]3 -
The formula of the complex is [AIEEE 2010]
(c) [Mn (CN) 6 ]3 - (d) [Cr (CN) 6 ]3 - (At. mass of Ag = 108 u)
(At. Nos : Cr = 24, Mn = 25, Fe = 26, Co = 27) (a) [Co(NH3 )6 ]Cl3 (b) [CoCl2 (NH3 )4 ]Cl
35. The value of the ‘spin only’ magnetic moment for one of the
(c) [CoCl3 (NH3 )3 ] (d) [CoCl(NH3 )5 ]Cl2
following configurations is 2.84 B.M. The correct one is
[AIEEE 2005] 45. Which one of the following has an optical isomer?
(a) d5 (in strong ligand field) [AIEEE 2010]
(a) [Zn(en) (NH3)2]2+ (b) [Co(en)3]3+
(b) d3 (in weak as well as in strong fields)
(c) [Co(H2O)4(en)]3+ (d) [Zn(en)2]2+
(c) d4 (in weak ligand fields)
(en = ethylenediamine)
(d) d4 (in strong ligand fields)
46. Which one of the following complex ions has geometrical
36. The IUPAC name for the complex [Co(NO2)(NH3)5]Cl2 is : isomers ? [AIEEE 2011]
[AIEEE 2006] (a) [Ni(NH3)5Br]+ (b) [Co(NH3)2(en)2]3+
(a) pentaammine nitrito-N-cobalt(II) chloride (c) [Cr(NH3)4(en)2]3+ (d) [Co(en)3]3+
(b) pentaammine nitrito-N-cobalt(III) chloride (en = ethylenediamine)
(c) nitrito-N-pentaamminecobalt(III) chloride 47. Which of the following facts about the complex [Cr(NH3)6]Cl3
(d) nitrito-N-pentaamminecobalt(II) chloride is wrong? [AIEEE 2011]
37. In Fe(CO)5, the Fe – C bond possesses [AIEEE 2006] (a) The complex involves d 2 sp 3 hybridisation and is
(a) ionic character (b) s-character only octahedral in shape.
(b) The complex is paramagnetic.
(c) p-character (d) both s and p characters
(c) The complex is an outer orbital complex
38. How many EDTA (ethylenediaminetetraacetic acid)
(d) The complex gives white precipitate with silver nitrate
molecules are required to make an octahedral complex with
solution.
a Ca2+ ion? [AIEEE 2006]
48. Among the ligands NH3, en, CN– and CO the correct order of
(a) One (b) Two their increasing field strength, is : [AIEEE 2011]
(c) Six (d) Three
(a) NH3 < en < CN - < CO
39. Which of the following has a square planar geometry?
[AIEEE 2007] (b) CN - < NH3 < CO < en
(a) [PtCl4]2– (b) [CoCl4]2– (c) [FeCl4]2– (d) [NiCl4]2–
-
(At. nos.: Fe = 26, Co = 27, Ni = 28, Pt = 78) (c) en < CN < NH3 < CO
40. The coordination number and the oxidation state of the
(d) CO < NH3 < en < CN -
element ‘E’ in the complex
[E (en)2 (C2O4)]NO2 (where (en) is ethylene diamine) are, 49. Which among the following will be named as dibromidobis
respectively, [AIEEE 2008] (ethylene diamine) chromium (III) bromide? [AIEEE 2012]
(a) 6 and 2 (b) 4 and 2 (c) 4 and 3 (d) 6 and 3 (a) [Cr (en)3]Br3 (b) [Cr(en)2Br2]Br
41. In which of the following complexes of the Co (at. no. 27), will (c) [Cr(en)Br4]– (d) [Cr(en)Br2]Br
the magnitude of Do be the hightest? [AIEEE 2008] 50. Which of the following complex species is not expected to
(a) [Co(CN)6] 3– (b) [Co(C2O4)3]3– exhibit optical isomerism ? [JEE M 2013]
(c) [Co(H2O)6] 3+ (d) [Co(NH3)6]3+ (a) [Co(en)3]3+ (b) [Co(en)2 Cl2]+
42. Which of the following has an optical isomer [AIEEE 2009] (c) [Co(NH3)3 Cl3] (d) [Co(en) (NH3)2 Cl2]+
(a) [Co(en) (NH3)2]2+ (b) [Co(H2O)4(en)]3+ 51. Among the following metal carbonyls, the C–O bond order is
(c) [Co(en)2 (NH3)2] 3+ (d) [Co(NH3)3Cl] + lowest in [IIT-JEE 2007]
(a) [Mn(CO)6] + (b) [Fe(CO)5]
43. Which of the following pairs represent linkage isomers?
[AIEEE 2009] (c) [Cr(CO)6] (d) [V(CO)6]–
(a) [Pd(PPh3)2(NCS)2] and [Pd ( PPh3)2 (SCN)2] 52. The IUPAC name of [Ni (NH3)4] [NiCl4] is [IIT-JEE 2008]
(b) [Co(NH3)5NO3] SO4 and [Co(NH3)5SO4]NO3 (a) Tetrachloronickel (II) - tetraamminenickel (II)
(c) [PtCl2(NH3)4] Br2 and [Pt Br 2 (NH3)4] Cl2 (b) Tetraamminenickel (II) - tetrachloronickel (II)
(d) [Cu(NH3)4][Pt Cl4] and [Pt(NH3)4][CuCl4] (c) Tetraamminenickel (II) - tetrachloronickelate (II)
(d) Tetrachloronickel (II) - tetrachloronickelate (0)
Coordination Compounds 779

53. Both [Ni(CO)4] and [Ni(CN)4]2– are diamagnetic. The 57. The complex showing a spin-only magnetic moment of
hybridisations of nickel in these complexes, respectively, are 2.82 B.M. is : [IIT-JEE 2010]
[IIT-JEE 2008] (a) Ni(CO)4 (b) [NiCl4]2–
(a) sp3, sp3 (b) sp3, dsp2 (c) dsp2, sp3 (d) dsp2, sp2 (c) Ni(PPh3)4 (d) [Ni(CN)4]2–
54. The spin only magnetic moment value (in Bohr magneton 58. Among the following complexes (K-P)
units) of Cr(CO)6 is [IIT-JEE 2009] K3[Fe(CN)6] (K), [Co(NH3)6]Cl3 (L), Na3[Co(oxalate)3] (M),
(a) 0 (b) 2.84 (c) 4.90 (d) 5.92 [Ni(H2O)6]Cl2 (N), K2[Pt(CN)4] (O) and [Zn(H2O)6](NO3)2 (P)
55. The ionisation isomer of [Cr(H2O)4Cl(NO2)]Cl is the diamagnetic complexes are [IIT-JEE 2011]
[IIT-JEE 2010] (a) K, L, M, N (b) K, M, O, P
(a) [Cr(H2O)4(O2N)]Cl2 (c) L, M, O, P (d) L, M, N, O
(b) [Cr(H2O)4Cl2](NO2) 59. As per IUPAC nomenclature, the name of the complex
(c) [Cr(H2O)4Cl(ONO)]Cl [Co(H2O)4(NH3)2]Cl3 is : [IIT-JEE 2012]
(d) [Cr(H2O)4Cl2(NO2)].H2O (a) Tetraaquadiaminecobalt (III) chloride
56. The correct structure of ethylenediaminetetraacetic acid (b) Tetraaquadiamminecobalt (III) chloride
(EDTA) is [IIT-JEE 2010] (c) Diaminetetraaquacoblat (II) chloride
(d) Diamminetetraaquacobalt (III) chloride
HOOC – H2C CH2– COOH
(a) N – CH = CH – N 60. [NiCl2 {P(C2H5)2(C6H5)}2] exhibits temperature dependent
HOOC – H2C CH2– COOH magnetic behaviour (paramagnetic/diamagnetic). The
HOOC COOH coordination geometries of Ni2+ in the paramagnetic and
(b) N – CH2 – CH2 – N diamagnetic states are respectively [IIT-JEE 2012]
HOOC COOH
(a) tetrahedral and tetrahedral
HOOC–H2C CH2–COOH (b) square planar and square planar
(c) N – CH2 – CH2 – N
HOOC–H2C CH2–COOH (c) tetrahedral and square planar
(d) square planar and tetrahedral
COOH
CH2 61. Consider the following complex ions, P, Q and R.
(d) HOOC–H2C H P = [FeF6]3–, Q = [V(H2O)6]2+ and R = [Fe(H2O)6]2+
N–CH–CH–N
H CH2–COOH The correct order of the complex ions, according to their
CH2 spin-only magnetic moment values (in B.M.) is
HOOC [JEE Advanced 2013]
(a) R < Q < P (b) Q < R < P
(c) R < P < Q (d) Q < P < R

1. Which one is the most likely structure of CrCl3. 6H2O if 1/3 3. Pick a poor electrolytic conductor complex in solution
of total chlorine of the compound is precipitated by adding (a) K 2 [PtCl 6 ] (b) [Co ( NH 3 ) 3 ]( NO 2 ) 3
AgNO3
(c) K 4 [ Fe (CN ) 6 ] (d) [Co ( NH 3 ) 4 ]SO 4
(a) CrCl3. 6H2O
4. Which is correct in the case of [Fe(CN) 6 ]4- complex ?
(b) [ Cr (H2O)3 Cl3]. (H2O)3
(c) [ CrCl2 (H2O)4 ] Cl . 2H2O (a) Diamagnetic (b) Octahedral
(c) d2sp3 - hybridisation (a) All are correct
(d) [ CrCl (H2O)5 ] Cl2 . H2O
5. IUPAC name of Na 3[Co(ONO) 6 ] is
2. Which will give a white precipitate with AgNO3 in aqueous
solution (a) Sodium cobaltinitrite
(b) Sodium hexanitrito cobaltate (III)
(a) [Co(NH3 )5 Cl](NO2 )2 (b) [Pt ( NH 3 ) 6 ]Cl 4
(c) Sodium hexanitrocobalt (III)
(c) [Pt (en )Cl 2 ] (d) [Cu ( NH 3 ) 4 ]SO 4 (d) Sodium hexanitritocobaltate (II)
780 Chemistry

6. Which has highest paramagnetism 12. Which of the following is paramagnetic ?


(a) [Co(NH3)6]3+ (b) [Ni(CO)4]
(a) [Cr(H 2 O) 6 ]3+ (b) [Fe(H 2 O) 6 ]2+
(c) [Ni(Cl)4]2– (d) [Ni(CN)4]2–
(c) [Cu(H 2 O) 6 ]2 + (d) [ Zn(H 2 O) 6 ]2 + 13. Which of the following compounds is not coloured?

7. Which is not true ligands metal complex ? (a) Na2[CuCl6] (b) Na2[CdCl4]
(a) Larger the ligand, the more stable is the metal - ligand (c) K4[Fe(CN)6] (d) K3[Fe(CN)6]
complex 14. Which is paramagnetic ?
(b) Highly charged ligand forms stronger bonds (a) [Ni(H2O)6]2+ (b) [Fe(CN)6]4–
(c) Larger the permanent dipole moment of ligand , the more (c) [Ni(CO)4] (d) [Ni(CN)4]2–
stable is the bond. 15. In the complexes [Fe(H2O)6]3+ , [Fe(CN)6]3–, [Fe(C2O4)3]3–
(d) Greater the ionisation potential of central metal, the and [FeCl6]3–, more stability is shown by
stronger the bond (a) [Fe(H2O)6]3+ (b) [Fe(CN)6]3-
8. In solid CuSO 4 .5H 2O copper is coordinated to (c) [Fe(C2O4)6]3- (d) [FeCl6]3-
(a) 4 water molecules (b) 5 water molecules 16. The most stable complex among the following is
(c) one sulphate molecule (d) one water molecule (a) K3[Al(C2O4)3] (b) [Pt(en)2]Cl
9. The catalyst used for the polymerization of olefins is (c) [Ag(NH3)2]Cl (d) K2[Ni(EDTA)]
(a) Ziegler - Natta catalyst (b) Wilkinson’s catalyst 17. In the silver plating of copper, K[Ag(CN)2] is used instead
(c) Pd- catalyst (d) Zeise’s salt catalyst of AgNO3. The reason is
10. Which of the following hydrate is diamagnetic? (a) a thin layer of Ag is formed on Cu
(a) [Mn(H2O)6]2+ (b) [Cu(H2O)6]3+ (b) more voltage is required
(c) [Co(NH3)6]3+ (d) [Co(H2O)6]2+ (c) Ag+ ions are completely removed from solution
11. The geometry of Ni(CO)4 and Ni(PPh3)2Cl2 are (d) less availability of Ag+ ions, as Cu cannot displace Ag
from [Ag(CN)2]– ion.
(a) both square planar
18. Which of the following organometallic compound is s and
(b) tetrahedral and square planar
p -bonded ?
(c) both tetrahedral
(d) None of these (a) [Fe(h5 - C5 H 5 ) 2 ] (b) [PtCl3 (h2 - C 2 H 4 )]

(c) [Co (CO ) 5 NH 3 ]2 + (d) Al(CH 3 )3


Coordination Compounds 781

EXERCISE 1 6. (c) Lithium tetrahydridoaluminate is Li[AlH4] and the ligand


is H–
1. Co has + 3 oxidation state.
7. (c) K4[Fe(CN)6] obeys the EAN rule
2. Tetraamminecobalt (III) – m-imido-m-nitro tetrammine cobalt 8. (d) EAN is given by : Atomic number of central metal atom +
(III) ion. 2 x C.N. – O.S.
3. A complex is formed, [Cu (NH3)4]2+ which does not dissociate \ EAN of Fe in [Fe(CN)6]3– is 26+12–3 =35
to form Cu2+ ion. 9. (b) Coordinate number is = 2 (number of bindentate ligands
4. (a) Zero. (b) One mole of Cl–. (c) 2 moles of Cl–. \ C.N. of Ni = (2 × 3) = 6
6. (a) [Ni (CN)4]2– 10. (a) The C. N. of Fe(II) in oxyhaemoglobin is 6
(b) [Ag (NH3 )2 ] + Cl- remains colourless in a solution. 11. (b) In the ring test the complex formed is [Fe(H2O)5NO]2+
7. (a) [Fe (CN)3 (NH3)3]– 12. (b) In the complex formation the ligands whether negative,
(b) [Fe (OH) (NH3)2 Cl3]2– neutral or positive always donate electrons to the central
8. (a) Six. (b) + 3. metal atom hence they act as Lewis bases.
13. (a) The primary valencies are ionisable and represented by
(c) sp3d2 hybridisation. (d) Octahedral.
dotted line.
(e) Diamagnetic.
14. (d) The number of ammonia molecules is 6. Each ammonia
9. (a) [Cr (NC) (H2O)5]2+ molecule contains 3 covalent bonds between N and H.
Therefore the number of covalent bonds is 18.
(b) [Pt Br2 (NH3)4] Cl2 15. (a) Since the precipitate of AgCl shows two ionisable
chloride ion the complex must have the structure.
(c) [Cu (NH3)4] [Pt Cl4]
[Co( NH 3 ) 5 Cl]Cl 2 + 2AgNO 3 ®
12. (a) [Cr (H2O)6] Cl3
[Co( NH 3 )5 Cl]( NO 3 ) 2 + 2AgCl
(b) [Cr (H2O)5 Cl] Cl2×H2O
Hence two chlorine atoms satisfy the primary valency
(c) [Cr (H2O)4 Cl2] Cl×2H2O and one, secondary valency
16. (b) [CuCl2{O=C(NH2)2}2] this is dichlorobis(urea) copper
17. (d) 18. (d) 19. (b)
(II) (see rules for naming a complex compound)
20. (d) 21. (a) 22. (a) 17. (d) Chemical formula of iron (III) hexacynaoferrate (II) is
23. (b) 24. (a) 25. (c) Fe4[Fe(CN)6]3
18. (d) IUPAC name of Na3[Co(NO2)6] is sodium hexanitro
26. (b) cobaltate (III)
EXERCISE 2 19. (c) K[Co(CN)4] let the O. N. of Co be x then 1×(+1)+1(+x)+4(–
1) = 0 Þ x = + 3
1. (b) Ferrous ammonium sulphate 20. (b) IUPAC name of K3[Ir(C2O4)3] is
[FeSO4](NH4)2 SO46H2O is a double salt. While all others potassium trioxalato iridate (III)
are complex compounds
21. (d) In Ziegler Natta catalyst the active species the TiIII
2. (b) CuSO 4 + 4 NH 3 ® [Cu ( NH 3 ) 4 ]SO 4 22. (a) The complex chlorodiaquatriammine cobalt (III) chloride
3. (b) K4[Fe(CN)6] is a complex compound can have the structure [CoCl(NH3)3(H2O)2]Cl2
4. (c) AgCl form soluble complex with NH4OH which is 23. (a) O. S of Fe in K4[Fe(CN)6]. Let O. S. of Fe be x then
[Ag(NH3)2]+ 4×(+1)+1×(+x)+6×(–1) = 0 \x = +2
5. (a) Salt may be complex salt. Metal atom present in the 24. (d) [Pt(en)2Cl2] is a complex of the type M(AA)2 B2 which is
coordination sphere appears in the form of complex ion octahedral Such compounds exhibit optical and
and not as simple cation geometrical isomerism both
782 Chemistry
25. (a) [Co(NH3)3(NO2)3] is of the type MA3B3. They give two 42. (b) Shape of [Cu(NH3)4]2+ is square planar.
geometrical Isomers 43. (a) [FeF6]3– the C. N. is 6 hence it is octahedral
26. (d) For [CuII(NH3)4][PtIICl4] four isomers are possible which 44. (d) Fe2+ electronic configuration is [Ar] 3d6. Since CN is
ar e [Cu(NH 3) 4 ] [PtCl 4], [C uCl 4 ][Pt(NH 3 ) 4 ], strong field ligand d electrons are paired . In Ni(CO)4
[PtCl 3(NH 3 )][Cu(NH 3 ) 3 Cl] and [Pt(NH 3 ) 3 Cl]
O. S. of Ni is zero electronic configuration is [Ar}3d8
[Cu(NH3)Cl3]
27. (c) Since the complex with AgNO3 solution gives two moles 4s2. In presence of CO it is [Ar] 3d10 4s0, electrons are
of AgCl it must have two ionisable Cl atoms. Hence the paired. Electronic configuration of Ni2+ [Ar]3d8 4s0, due
complex is [Co(NH3)5NO2]Cl2. On ionisation it will give to CN– ligand all electrons are paired. Co3+ is [Ar] 3d6
three moles of ion viz [Co(NH3)5NO2]2+ +2Cl– since F is weak ligand hence paramagnetic.
28. (d) [Cr(NH3)3Cl3] is of the type MA3B3 and exists in two 45. (b) The valency is given by O. S. In [Cr(H2O)4Cl2]+. O. S. of
isomeric forms. Cr is +3 hence valency of Cr is +3
29. (d) Square planar complex of the formula Mabcd give three
geometrical isomers 46.(a) A,Ni2+ B,Ni C,Zn2+ D,Co3+
30. (b) E.C.[Ar]3d8 [Ar]3d8, [Ar]3d10 [Ar]3d6
31. (d) The complex ion [Cr(SCN) 2(NH3)4]2+ can exhibit 4s0 4s2 4s0 4s0
geometrical and linkage isomerism
H2O weak CO NH3 strong NH3 strong
32. (c) [PtCl2(NH3)4]Br2 and [PtBr2(NH3)4]Cl2 are ionisation
ligand strong ligand
isomers
No pairing Pairing Pairing of Pairing of
33. (c) The complex compound [Co(en)2NO2Cl] Br can have
of electrons of electrons electrons electrons
NO2 group differently linked to central metal atom –
Paramagnetic Diamagnetic Diamagnetic Diamagnetic
O
or O – N = O 47. (a) Ni(CO)4 is tetrahedral but not paramagnetic. It is
N
O diamagnetic
34. (a) The compounds of the type M(AA)2B2 exhibit both 48. (a) [Pt(NH3)2Cl2] O.S. of Pt +2; electronic configuration of
geometrical and optical isomerism Pt+2 = [Xe]4f14 5d8. Ligand NH3 results in pairing of d
35. (c) Octahedral complex of the type MA5B do not show electrons leaving one d orbital empty therefore
geometrical isomerism hybridisation is dsp2 square planar
36. (c) Similarity between optical and geometrical isomerism is 49. (d) Shape of cuprammonium ion [Cu(NH3)4]2+ is square
that both are included in stereo isomerism planar dsp2 hybridisation
37. (b) The compound is [Cr(NH3)5NO2]Cl2 and can exhibit 50. (b) [Ni(CN)4]2– dsp2,
O 51. (a) [Co(NH3)5CO3]ClO 4. Six monodentate ligands are
linkage isomerism due to NO2 group (– N or attached to Co hence C. N. of Co = 6;
O O. N. = x + 5 × (0) + 1 × (–2) + 1× (–1) = 0 \ x = + 3 ;
O – N= O) electronic configuration of Co3+[Ar] 3d64s0 hence number
38. (c) Electronic configuration of Ni2+ is [Ar] 3d8 4s0 4p0. CN– of d electrons is 6. All d electrons are paired due to
is strong ligand and will do pairing of electrons so will strong ligand hence unpaired electron is zero.
have one d orbital left empty. C. N. is 4 so dsp 2 52. (d) [Cr(CO)6] Fe(CO)5 [Fe(CN)6]4– [Cr(NH3)6]3+
hybridisation will take place which is square planar, O.S. 0 0 +2 +3
Ni(CO)4 and [Ni(CN)4]2– are tetrahedral. of M
39. (a) Ni(CO)4. The O. S. of Ni is Zero. Electronic configuration
E.C. [Ar] 3d5 4s1 [Ar] 3d64s2 [Ar] 3d6 [Ar] 3d3
is [Ar] 3d8 4s2 4p0 . In presence of strong ligand CO the
Pairing Pairing Pairing Pairing Pairing
parin g of electrons take place and electronic of e– of e– of e– of e– of e–
configuration will be [Ar] 3d10 4s0 4p0. Hence unpaired
Diamagnetic Paramagnetic Diamagnetic Diamagnetic
electrons is zero .
53. (a) K4 [Fe(CN)6] in solution furnishes 4K++ [Fe(CN)6]4– five
40. (b)
41. (d) Fe(CO) 5 (Z=26) O. S. of Fe is zero . Electronic ions and exhibits maximum ionic conductivity.
configuaration is [Ar]] 3d6, 4s24p0. After pairing of 54. (a) Ti(C2H4)4 is an organometallic compound due to Ti
electrons of d and s orbitals, we have one d atomic orbital directly attached to C- atom
empty. C. N. is 5 so hybridisation is dsp3 which is trigonal 55. (d) Tetraethyl lead Pb(C2H5)4 is not p bonded complex. It is
bipyramidal. s bonded organometallic compound
Coordination Compounds 783
EXERCISE 3 Ti = [Ar]3d 2 4 s2 ; Ti4 + = 3d 0
Cr = [Ar] 3d 5 4s1; Cr3+ = 3d 3
1. (b) Non superimposable mirror images are called optical Zn= [Ar] 3d 10 4s2; Zn2+= 3d 10
isomers and may be described as “chiral’. They are Sc = [Ar] 3d 1 4s2; Sc3+ = 3d 0
also called enantiomers and rotate plane polarised light
in opposite directions. 7. (a) [ Ni(CN)4 ]2- : Number of unpaired electrons = 0
Cl Cl [Cr(NH3 )6 ]3+ : Number of unpaired electrons = 3
Cl Cl
[ Fe(H 2 O)6 ]2+ : Number of unpaired electrons = 4
en Co Co en [ Ni(H 2O)6 ]2+ : Number of unpaired electrons = 2
8. (d) d 4 in high spin octahedral complex

en en eg —

2. (b) Lesser is the number of unpaired electrons smaller will t 2g


be the paramagnetic behaviour. As Cr ++, Mn++, Fe++ CFSE = (–0.4x + 0.6y)D0
and Ni++ contains. Where, x ® electrons in t2g orbital
y ® electrons in eg orbital
Cr++ (3d 4) = CFSE = [0.6 × 1] + [–0.4 × 3] = – 0.6 D 0
= 4 unpaired e–. 9. (b)

Mn++ (3d 5)=

= 5 unpaired e–.

Fe++ (3d 6) =

= 4 unpaired e–.
trans (green)
Ni++ (3d 8) =
Cl +
= 2 unpaired e–. HN
3 Cl
As Ni++ has minimum no. of unpaired e– thus this is
least paramagnetic. Co
3. (c)
4. (c) In octahedral field the crystal field splitting of d- orbitals
HN
3 NH3
of a metal ion depends upon the field produced by the NH3
ligands. In general ligands can be arranged in a series cis (violet)
in the order of increasing fields and splittings which
they produce around a central metal ion. A portion of 10. (d) In éë Ni ( NH3 ) 2 Cl 2 ùû , Ni 2 + is in sp3 hybridisation,
the series is given below. thus tetrahedral in shape. Hence the four ligands are
cyanide > ethylene - diamine > ammonia > pyridine > not different to exhibit optical isomerism. In tetrahedral
thiocyanate > water > oxalate > hydroxide > fluoride > geometry all the positions are adjacent to one another
chloride > bromide > iodide. \ geometrical isomerism is not possible.
Out of the given ligands water, ammonia, cyanide and 11. (b) Ni++ = 3d 8 4s0
oxalate, we can find from the above series of ligands that
the maximum splitting will occur in case of cyanide (CN–
) i.e. the magnitude of D0 will be maximum in case of Since, the coordination number of Ni in this complex is
[Co(CN)6]3+. 4, the configuration of Ni ++ at first sight shows that
5. (a) The octahedral coordination compounds of the type the complex is paramagnetic with two unpaired
MA3B3 exhibit fac-mer isomerism. electron. However, experiments show that the complex
6. (b) Since Cr3+ in the complex has unpaired electrons in the is diamagnetic. This is possible when the 3d electrons
d orbital, hence it will absorb visible light and will be rearrange against the Hund’s rule as shown below. This
coloured is in accordance with the fact that the ligand involved
here is strong i.e., CN– ion.
784 Chemistry
Ni++ (after rearrangement) 15. (a) As positive charge on the central metal atom increases,
4s 4p the less readily the metal can donate electron density
into the p* orbitals of CO ligand (donation of electron
density into p* orbitals of CO result in weakening of
C – O bond). Hence, the C – O bond would be strongest
in [Mn(CO)6]+.
Hence, now dsp2 hybridization involving one 3d, one
4s and two 4p orbitals, takes place leading to four dsp2 16. (b)
hybrid orbitals, each of which accepts four electron (a) [Ti(NH3)6]3+ : 3d1 configuration and thus has one
pairs from CN– ion forming [Ni (CN)4]2– ion. unpaired electron.
[Ni (CN)4]2– (b) [Cr(NH3)6]3+ : In this complex Cr is in +3 oxidation
state.
×× ×× ×× ×× ××
Cr3+ : 3d 3 4s0

four dsp2 hybrid bonds


Thus, the complex is diamagnetic as it has no unpaired It has 3 unpaired electrons thus complex is
electron. paramagnetic
12. (c) Coordination isomerism occurs when cationic and
anionic complexes of different metal ions are present in NH3 NH3 NH3 NH 3 NH3 NH3
a salt. The two isomers differ in the distribution of
ligands in cation and anion e.g.,
[Co (NH3)6] [Cr (CN)6] is an isomer of [Co (CN)6] (c) [Co(NH3)6]3+ : In this complex cobalt ion is in + 3
[Cr (NH3)6] oxidation state with 3d 6 configuration.
13. (a) Complexes of the type MABCD may exist in three Co3+,[Ar]3d 6
isomeric forms. 4s 4p
A B A C

[Co(NH3)6]3+
M M

NH3 NH3 NH3 NH3 NH3 NH3


D C D B
(I) (II)
(inner orbital or d 2sp3 hybrid orbital, low spin complex)
A C A C
\ diamagnetic
(d) In this complex Zn exists as
M M
Zn++ ion
Zn++ ion : 3d 10 4s0
D B B D
(III)
Similarly, [Pt (py) (NH3) BrCl] may exist in three isomeric
form in which Zn++ ion in [Zn(NH3)4]2+
M = Pt, A = Py, B = NH3, C = Br, D = Cl. NH 3 NH 3 NH3 NH 3

14. (c) Cr2+ d4 4


Due to presence of paired electrons complex is
Mn2+ d5 5 diamagnetic in nature.
17. (a) [Ni(NH3)6]2+
Fe2+ d6 4
Ni2+ = 3d 8, according to CFT = t 62g eg2 therefore,
Co2+ d7 3 hybridisation is sp3d 2 and complex is paramagnetic.
Minimum paramagnetic behaviour = [Co (H2O)6]2+
Coordination Compounds 785
18. (c) Nickel ions are frequently detected by the formation of Hence [ Ni( NH 3 ) 6 ]2 + is outer orbital complex
red precipitate of the complex of nickel
dimethylglyoxime, when heated with dimethylglyoxime. 28. (c) Chlorophyll contains Mg and not Ca
29. (d) isomers
CH3 C NOH
+ Ni++ [Ru(NH3 ) 4 Cl2 ]+ ,[Co(NH3 )5 Cl]2 + ,
CH3 C NOH cis and trans none
Dimethylglyoxime
[Ir(PR 3 ) 2 H(CO)]2+ , [Co(en ) 2 Cl 2 ]+
cis and trans cis and trans and
OH O optical isomers

CH3 C N N C CH3 30. (c) Number of unpaired electrons


Ni
CH3 C N N C CH3 [Fe(CN ) 6 ]4- , [CoCl]2- , [MnCl 4 ]2-
zero three five
O OH The greater the number of unpaired electrons, the higher
the value of magnetic moment
Nickel dimethylglyoxime
31. (d) The oxidation state of Cr in [Cr ( NH 3 ) 4 Cl 2 ]+ is + 3.
19. (b) d 6 – t2g 2, 2, 2 e 0,0 (in low spin) 32. (c) K 3[Fe(CN ) 6 ] is potassium hexacyano ferrate (III).
g

C.F.S.E = – 0.4 × 6D0 + 3P C2O4 C2O4

12
= - D + 3P 33. (b) Cr
5 0 Cr C2O4 C2O4

® AgCl +
20. (d) [Cr(H2O)4Cl2] Cl + AgNO3 ¾¾ C2O4
C2O4
[Cr(H2O)4Cl2]NO3
Non-superimposable mirror images, hence optical
wt 1000 isomers.
Molarity = ´ 34. (a) No. of unpaired electron
mol. mass vol.
a) Co3+ 4
b) Fe3+ 1
wt vol. 0.01 ´100
= molarity ´ = c) Mn 3+ 4
mol. mass 1000 1000 d) Cr3+ 3
= 0.001 Lower the number of unpaired electrons in a substance,
the lower is its magnetic moment in Bohr – Magneton
21. (b) A square planer complex is formed by hybridisation of s,
and lower shall be its paramagnetism
px, py and d 2 2 atomic orbitals 35. (d) d5 –––– strong ligand field
x -y
22. (b) Bond between C of organic molecule and metal atom.
23. (b) A more basic ligand forms stable bond with metal ion, Cl- t2g eg
is most basic amongst all.
m = n(n + 2) = 3 = 1.73BM
24. (d) Co (NH3)5 Cl3 ƒ [Co(NH3)5Cl]+2 + 2Cl- d3–– in weak as well as in strong field
\ Structure is [Co (NH3)5 Cl] Cl2.
25. (a) 4 (+ 1) + x + (–1) × 4 = 0
Þ 4+x-4=0 t 2g eg
x= 0
m = 3(5) = 15 = 3.87 B.M.
26. (c) The coordination number of central metal atom in a
d4– in weak ligend field
complex is equal to number of monovalent ligands, twice
the number of bidentate ligands and so on, around the
metal ion bonded by coordinate bonds t2g eg
27. (d) Hybridisation
m = 4(6) = 24 = 4.89
[Fe(CN)6 ]4 -, [Mn(CN)6 ]4- , d4– in strong ligand field
d 2sp3 d 2sp3

[Co(NH3 )3+ ,[Ni(NH3 )6 ]2+ t 2g eg


d 2sp3 sp3d 2
786 Chemistry
\ 3Cl– ions are precipitable
m = 2(4) = 8 = 2.82 Hence the formula of the complex is [Co(NH3)6]Cl3
36. (b) [Co(NO)2(NH3)5]Cl2 45. (b) Option (b) shows optical isomerism [Co(en)3]3+
pentaamminenitro-N-cobalt (III) chloride
37. (d) Due to some backbonding by sidewise overlapping en 3+ en 3+
between d-orbitals of metal and p-orbital of carbon,
the F–C bond has s and p character. en Co Co en
38. (a) EDTA is hexadentate (four donor O atoms and 2 donor
N atoms and for the formation of octahedral complex en
en
one molecule is required
d–form Mirror –form
Ca++ +[H2EDTA–]–––® [Ca(EDTA)]2–+ 2H+
39. (a) Complexes with dsp2 hybridisation are square planar. Complexes of Zn++ cannot show optical isomerism as
So, [PtCl4]2– is square planar in shape. they are tetrahedral complexes with plane of symmetry.
40. (d) In the given complex we have two bidentate ligands
(i.e en and C2O4), so coordination number of E is 6
[Co(H2 O)4 (en)]3+ have two planes of symmetry
hence it is also optically inactive.
(2 × 2 + 1 × 2 = 6)
[Zn(en)2]2+ cannot show optical isomerism
Let the oxidation state of E in complex be x, then
[x + (–2) = 1] or x – 2 = 1
en 3+
or x = + 3, so its oxidation state is + 3 46. (b)
NH3
Thus option (d) is correct.
41. (a) In octahedral complex the magnitude of Do will be
highest in a complex having strongest ligand. Out of Co
the given ligands CN– is strongest. So, Do will be
highest for [Co(CN)6]3–. Thus option (a) is correct. NH3
42. (c) en
NH3 NH3
NH3 NH3 cis

en Co Co en
3+
NH 3

en en
en Co en
Enantiomers of cis- éë Co(en) 2 (NH 3 ) 2 ùû 3+
43. (a) The SCN– ion can coordinate through S or N atom
giving rise to linkage isomerism
M ¬ SCN thiocyanato NH 3
M ¬ NCS isothiocyanato. trans-
44. (a) ® x Cl-
CoCl3 .6NH3 ¾¾
2.675g
47. (c) [Cr (NH3)6]Cl3 is an inner orbital complex, because in
this complex inner d-orbital is used for hybridisation
x Cl- + AgNO3 ¾¾
® x AgCl ¯ (d2 sp3 )
4.78g 48. (a) Ligands can be arranged in a series in the orders of
Number of moles of the complex increasing field strength as given below :
2.675 Weak field ligands :
= = 0.01 moles
267.5 I- < Br - < S2 - < SCN < Cl - < N3- , F -
Number of moles of AgCl obtained < Urea, OH– < oxalate
4.78 Strong field ligands
= = 0.03 moles
143.5 O -- < H 2O < NCS- < EDTA < Py, NH3 <
\ No. of moles of AgCl obtained
= 3 × No. of moles of complex en = SO 3 – < bipy, Phen < NO2- < CH3-
0.03 < C6 H 5- < CN - < CO
\n= =3
0.01 Such a series is termed as spectrochemical series. It is
Coordination Compounds 787
an experimentally determined series based on the 3d 4s 4p
absorption of light by complexes with different ligands.
49. (b) [Cr(en)2Br2]Br
dibromidobis (ethylenediamine) chromium (III) bromide.
50. (c) Octahedral coordination entities of the type Ma3b3
exhibit geometrical isomerism. The compound exists
3d 4s 4p
both as facial and meridional isomers, both contain
plane of symmetry
NH3 NH3
Cl NH3 NH3 Cl sp3

Co Co No. of unpaired electrons = 2


NH3 Cl Cl
Cl Magnetic moment, m = 2.82 BM.
Cl NH3
fac- mer 58. (c) Complex No. of electrons No. of unpaired
51. (d) An anionic carbonyl complex can delocalise more in outer d electron (s)
electron density to antibonding pi-orbital of CO and orbital
hence, lowers the bond order. 3– 5 –
52. (c) The correct IUPAC name of the given compound is [Fe(CN)6] 3d 1 (CN causes
tetramminenickel (II) - tetrachloronickelate (II) thus (c) pairing of
is the correct answer. electrons)
53. (b) In carbonyls O.S. of metal is zero 3+ 6 0
In [Ni(CO)4], the oxidation state of nickel is zero. Its [Co(NH3 )6] 3d
configuration in Ni(CO)4 is [Co(oxal.)3]
3–
3d
6 0
3d 4s 4p [Ni(H2 O)6 ]
2+
3d
8 2
[Ni(CO)4] ; 2– 8 –
[Pt(CN)4 ] 5d 0 (CN causes
3
sp hybridisation pairing of
electrons)
In [Ni(CN)4 ]2– the oxidation state of Ni is 2+ and its
configuration is [Zn(H2 O)6]
2+
3d
10 0
3d 4s 4p
[Ni(CN)4]2 Thus L, M, O and P are diamagnetic.
59. (d) [Co(H2O)4(NH3)2]Cl3
dsp2 hybridisation = Diamminetetraaquacobalt (III) chloride.
Thus the hybridisations of nickel in these compounds 60. (c) In both states (paramagnetic and diamagnetic) of the
are sp3 and dsp2 respectively. given complex, Ni exists as Ni2+ whose electronic
Hence (b) is the correct answer. configuration is [Ar] 3d 84s0.
54. (a) Chromium in Cr(CO)6 is in zero oxidation state and has
[Ar] 18 3d 5 4s 1 as the electronic configuration. 3d 4s 4p
However, CO is a strong ligand, hence pairing up of Ni2+ :
electrons takes place leading to following configuration
in Cr(CO)6. sp3
3d 4s 4p
In the above paramagnetic state the geometry of the
­¯ ­¯ ­¯ ´ ´ ´ ´ ´´ ´´ ´´ ´´
1444444 424444444 3 complex is sp3 giving tetrahedral geometry.
d 2 sp3 Hybridisation The diamagnetic state is achieved by pairing of
Since the complex has no unpaired electron, its electrons in 3d orbital.
magnetic moment is zero.
55. (b) Ionisation isomer of [Cr(H2 O) 4 Cl(NO 2 )]Cl is 3d 4s 4p
[Cr(H2O)4Cl2]NO2.
56. (c) The correct structure of EDTA is
HOOC–H2C CH2–COOH
N – CH2 – CH2 – N dsp2
HOOC–H2C CH2–COOH
Thus the geometry of the complex will be dsp2 giving
57. (b) [NiCl4]2–, O.S. of Ni = +2
square planar geometry.
Ni(28) = 3d 8 4s2
788 Chemistry
61. (b) The electronic configuration of central metal ion in 7. (a) The larger the ligand, the less is the stablilty of metal
complex ions P, Q and R are ligand complex.
P = [FeF6]3–; Fe3+ : 8. (a) Four water molecules are coordinated to copper.
3d H2O OH2 H--O O
Cu O S
Q = [V(H2O)6]2+; V2+ H2O OH2 H--O O
3d
9. (a) For polymerisation of alkenes Ziegler Natta catalyst
R = [Fe(H2O)6 ]2+; Fe2+ (Al(C2H5)3 +TiCl3) is used.
3d
Higher the no. of unpaired electron(s), higher will be 10. (c) Mn2+ Cu3+ Co3+ Co2+
magnetic moment. [Ar]3d5 [Ar]3d8 [Ar]3d6 [Ar]3d7
Thus the correct order of spin only magnetic moment
is No pairing No pairing Pairing No pairing
Q<R<P
EXERCISE 4 of e - of e - of e - of e -
Paramagnetic Paramagnetic Diamagnetic Paramagnetic
1. (c) The ions present in the ionisation sphere are precipited
11. (c) Ni(CO)4 Ni(PPh3)2 Cl2
Hence [CrCl 2 (H 2O) 4 ]Cl.2H 2 O contains 1/3 Cl in
O.S. Ni0 Ni2+
ionisation sphere to be precipited by AgNO3 as AgCl
E.C. [Ar]3d84s2 [Ar]3d84s0
4+
2. (b) é Pt ( NH3 ) ù Cl 4 ƒ é Pt ( NH3 ) ù + 4Cl- ; Pairing of e– No pairing of e–
ë 6û ë 6û
Hybridization sp3 (tetrahedral) sp3 (tetrahedral)
-
Ag + + C l ® AgCl (ppt) 12. (c) [NiCl4]2–, Ni2+ [Ar]3d8, Cl is weak ligand hence no pairing

3. (d) The complex furnishing least number of ions in solution of of e - in d atomic orbitals therefore it is paramagnetic.
will be poor electrolytic conductor. 13. (b) Na2[CdCl4] does not contain any unpaired electron hence
4. (d) [Fe(CN)6 ] 4- complex ion contains 2+ E.C is colourless
26 Fe
14. (a) [Ni (H2O)6]2+ is paramagnetic.
= 1s 2 , 2s 2 p6 ,3s 2 p6 d6 15. (c) [Fe(C2O4)3]3–. The iron is present in the highest oxidation
3d 4s 4p
state Fe3+ and C2 O 24 - is a chelating ligand. Chelates are
( due to strong CN - ligand, the electrons in 3d shell are always form more stable complexes
paired ) 16. (d) K2[Ni(EDTA)]. Since EDTA is hexadentate and chelating
C.N is 6 , hybridisation d sp 2 3 , octahedral, diamagnetic and coordinates from six sides forming more stable
complex
since no unpaired electron is present.
17. (d) In silver plating K[Ag(CN)2] is used which provides
5. (b) Na 3[Co(ONO) 6 ] IUPAC name is constant and required supply of Ag+ ions as Ag(CN)2–
sodium hexanitritocobaltate (III) is very stable. But if AgNO3 is used concentration of
6. (b) [Ag+] in solution will be very large. In that case Ag will
be deposited at faster rate without any uniformity.
[Cr (H 2 O) 6 ]3+ [Fe(H 2 O) 6 ]2+ [Cu (H 2 O) 6 ]2 + [Zn (H 2 O) 6 ]2+
18. (c) In [Co(CO)5NH3]2+ CO is bonded to cobalt through s
EC 3d3 3d 6 3d 9 3d1 0
and p bonding
Unpaired 3 4 1 0
electrons

More the number of unpaired electrons, more is the


paramagnetic character.
24
Haloalkanes and
Haloarenes
Halogen derivatives of alkanes, alkenes, alkynes and arenes are (a) Order of reactivity among HX : HI > HBr > HCl >> HF
known as alkyl halides (haloalkenes), alkenyl halides This is in accordance with the bond length, longer the
H – X bond, weaker it will be.
(haloalkenes), alkynyl halides (haloalkynes) and aryl halides
(b) Order of reactivity among ROH : 3° > 2° > 1° > CH3OH
(halobenzenes), respectively. On the basis of the number (c) tert-Alcohols react with HX by SN1 pathway, while
of halogen atom, they are further classified as mono-, primary alcohols react via SN2, secondary may follow
di-, tri- poly- and per-haloh ydrocarbons. Th e word either or both of the paths (SN1 and SN2 are discussed
perhalohydrocarbon means all the hydrogen atoms of the later).
compound are replaced by corresponding number of halogen (d) Mixture of conc. HCl and anhydrous ZnCl2 is used for
differentiating three types of alcohols (3° > 2° > 1°) under
atoms.
the name of Lucas reagent.
Besides the nature and number of halogen atoms, alkyl halides (e) Primary and secondary alcohols can best be converted to
may be classified into primary (1°), secondary (2°) and tertiary the corresponding chlorides (by SOCl2), bromides (by PBr3)
(3°) according to the nature of the carbon atom bearing halogen. and iodides, by PI3 (P + I2).
(f) SOBr2 is less stable and SOI2 does not exist, PBr5 and
CH3 CH3 PI5 are highly unstable hence not used.
CHCH2Cl CHCl (ii) (a) Alkyl halides are generally not prepared by direct
CH3 CH3 halogenation of alkanes because the reaction generally
Isobutyl chloride (1°) Isopropyl chloride (2°) gives a mixture of several compounds. However,
(–Cl present on 1°C) (–Cl on 2°C) compounds containing only one type of hydrogen atom
can be converted into monohalogeno products in good
CH3 yield by taking excess of the concerned hydrocarbon;
CHCl examples of such compounds are CH4, CH3CH3, (CH3)4C,
C2H5 (CH3)3CCl
C6H5CH3 etc.
sec. Butyl chloride (2°) tert. Butyl chloride (3°) (b) Since allylic hydrogen atoms are much more reactive
(–Cl on 2°C) (–Cl on 3°C) than the vinylic hydrogen, former are easily replaced by
halogen.
Alkyl halides may show chain, position and optical isomerism;
Cl , light
alkenyl halides may show geometrical; while aryl halides may show CH 3CH = CH 2 ¾¾2¾ ¾
¾® ClCH 2 CH = CH 2
position isomerism. Pr opene Allyl chloride

PREPARATION : Cl2, light

(i) From alochols with HX, PX 3 (X = I, Br, Cl) or SOCl2 But-2-ene


Cl , light CH2Cl
R - OH + HX ¾
¾® RX
But-2-ene 1-Chlorobut-2-ene
790 Chemistry

(c) Benzylic hydrogens (hydrogen present on C attached Alkyl fluorides have lowest boiling point of all the alkyl
directly to benzene) are more reactive, hence easily halides, hence can be removed by distillation.
Swarts reaction
replaced than 1°, 2° or 3° hydrogen.
Alkyl chloride/ bromide is heated in presence of a metallic
fluoride such as AgF, Hg2F2, CoF2 or SbF3
light
CH2CH2CH3 + Br2 CH3–Br + AgF ® CH3F + AgBr
For alkyl iodides : (Finkelstein reaction)

n-Propylbenzene acetone
CH 3CHXCH 3 + NaI ¾¾¾
¾®
(X =Cl, Br) Soluble in acetone
light
CHBrCH2CH3
® CH 3CHICH 3 + NaX ¯
Insoluble in acetone
1-Bromo-1-phenylpropane
(v) Hunsdiecker method (suitable for bromides) :
(d) Halogenation of hydrocarbons in presence of light, heat
CH 3 COOAg + Br 2 ¾¾® CH 3 Br + CO 2 + AgBr
and absence of halogen carrier takes place through free-
radical mechanism. (i) Reaction involves free radical mechanism.
(e) More reactivity of benzylic and allylic hydrogens is due (ii) Reaction gives an alkyl halide having one carbon atom
to stability of the corresponding free radical due to less.
resonance.
(iii) Yield of alkyl bromides follows the order : 1° > 2° > 3°.
(f) Bromination at allylic and benzylic positions
may best be carried by N-bromosuccinimide, NBS (iv) Yield is very low in case of chlorides, while iodine in
(Wohl-Ziegler Reaction). such cases react differently.
(iii) Addition of hydrogen halides to alkenes is an example of
electrophilic addition involving carbocations as intermediates 2 RCOOAg + I 2 ¾
¾® RCOOR + CO 2 + 2 AgI
(ionic mechanism). PHYSICAL PROPERTIES :
Markownikoff's rule (i) Alkyl halides, although polar, are insoluble in water due to
(i) When an unsymmetrical alkene or alkyne reacts with inability to form hydrogen bonds with water molecules.
unsymmetrical reagent, then negative part of reagent
(ii) Density :
attach with that carbon atom which contains lesser
number of hydrogen atom during the addition. RI > RBr > RCl (For the same R– or Ar– group)
For example: CH3I > C2H5I > C3H7I (For the same halogen)
(iii) Methyl iodide is densest halide because contribution of alkyl
Br
| part is minimum.
CH3 - CH = CH 2 + HBr ® CH3 - CH - CH3 (iv) Boiling points :
Pr opene 2 - bromopropane
(a) RI > RBr > RCl > RF (When R is same)
CH 2 = CH 2 + HBr ¾
¾® CH 3 - CH 2 Br (b) CH3CH2CH2Cl > CH3CH2Cl > CH3Cl
(For the same halide)
CH 3CH = CH 2 + HBr ¾
¾® CH 3 .CHBr.CH 3 (c) CH3CH2CH2CH2Cl > CH3CH2CHClCH3
Propene > (CH3)2 CHCH2Cl > (CH3)3 CCl
(unsymmetr ical alkene)
(For isomeric halides, b.p. decreases with the increase
(Markownikoff's addition)
in branching)
(ii) Addition of HBr (not HCl, HI and HF) on alkenes in
presence of peroxides takes place in (v) Dipole moment : Except fluoride, dipole moment decreases
anti-Markownikoff's way (Peroxide effect). with the decrease in electronegativity from Cl to I. Fluorides,
Here addition takes place via free-radical mechanism. although having highest electronegativity have lower dipole
peroxide moment than chloride due to the very small size of F and
CH3CH = CH 2 + HBr ¾¾¾¾® CH3CH 2CH 2Br
hence very small C–F bond length which outweighs the effect
(iv) Halide exchange method (anti-Markownikoff's addition)
of electronegativity (recall that m = d × e). Thus the order for
is considered to be best for alkyl fluorides and alkyl iodides.
For alkyl fluorides : dipole moment is CH3Cl > CH3F > CH3Br > CH3I.
ethylene glycol
(vi) Stability order : R – F > R – Cl > R – Br > R – I
RCH 2 X + KF ¾¾¾ ¾¾¾® RCH 2 F + KX (Similar to C – X bond length)
( X =Cl, Br, I) 120°C
Haloalkanes and Haloarenes 791

CHEMICAL PROPERTIES :
1. Nucleophilic substitution :
(i) Alkyl halides undergo nucleophilic substitution very easily.
+ -
¾® R - Z+ : X -
R - X + : Z (nucleophi le) ¾
Source of :Z Product R–Z and its Name
Na+OH– R – OH Alcohols
H2O (water) R – OH Alcohols
Na+OR' R – OR' (Williamson Ethers synthesis)
I– R–I Alkyl iodides
Source of :Z Product R–Z and its Name
RCOO– R – OOCR' Esters
K+CN– R – CN Nitriles
AgCN (covalent) R – CN + R – NC Nitriles and Isonitriles

K + NO -2 R–O–N=O Alkyl nitrites


AgNO2 (covalent) RONO + R – NO2 Nitrites and Nitroalkanes
K+SH– R – SH Thiols (Mercaptans)
K+SR' – R – SR' Thioethers (Sulphides)

N 3- R – N = N+ = N– Azides
NH3 R – NH2 Primary amines
R' NH2 R – NHR' Secondary amines

PPh3 [R - PPh 3 ]+ X – Phosphonium salts

R' C º C- R' C º C - R Alkynes

Ar - H + AlCl3 Ar – R Alkylbenzenes (Friedel-Craft reaction)


- R - CH (COOC 2 H 5 ) 2 Malonic esters
CH (COOC 2 H 5 ) 2
d-
R'-M d+ R– R Alkanes (Coupling)

H :- (from LiAlH4) R– H Alkanes (Reduction)


In case of AgCN and AgNO2, two isomeric products are formed.
®
R - X + AgCN ¾¾
®R -C º N + RN = C -X - :Z
Alkyl nitrile Alkyl isonitrile (major) R - X ¾¾¾® R + ¾¾® R - Z
(slow ) (fast )
O (a) In S N1 mechanism, carbocations are formed
¾® R - N =
R - X + AgNO2 ¾ + R -O- N = O
O Alkyl nitrite (minor) as intermediate, hence more the stability of the
Nitroalkane
Nucleophilic substitution reactions occur either by SN1 intermediate carbocation, greater are chances for
or SN2 mechanism. their formation and hence more reactive will be the
(ii) SN1 (Unimolecular nucleophilic substitution) : parent alkyl halide for SN1 reaction. Hence the order
Although it is a two step process, the rate of reaction of reactivity of alkyl halides toward SN1 reaction
depends only upon the first (slow) step which involves follows the order : 3° > 2° > 1°
ionization of the alkyl halide to form carbocation. Hence (b) When the intermediate carbocation is capable of
rate of reaction depends only upon the concentration of undergoing rearrangement, lesser stable carbocation
the alkyl halides, r = k[RX] and is independent of the (1° < 2° < 3°) rearranges to the more stable
concentration of the nucleophile which adds on the carbocation and hence under such conditions
carbocation in the second (fast) step. unexpected product is formed.
792 Chemistry

CH3 CH3 CH3


| - | Å |
- Br
CH 3 - C - CH 2 Br ¾¾¾® CH 3 - C - C H 2 1,2 -alkyl
¾¾ ¾¾® CH 3 - C - CH 2 CH 3
| | shift Å
CH3 CH 3 3° Carbocation ( more stable)

1° Carbocation ( less stable)

– –
OH OH

CH3 CH3
| |
CH 3 - C - CH 2 OH CH 3 - C - CH 2 CH 3
| |
CH3 OH
Expected (but not formed) Unexpected (but formed)

(c) If the alkyl halide is optically active, the product formed in S reaction is always a racemic mixture. This is due to the
N1
formation of carbocations as intermediates which, being planar (sp2 hybridised) can be equally attacked by the nucleophile
on either side of the face forming two enantiomers.

R2
– |
+X :+X : R1 - C - Z
R2 R R2 |
| R1 + 1 R+2 R3
R1 - C - X C C
|
R2
|
R3 R3 Z - C - R1
R3
|
R3
Alkyl halide Carbocation Enantiomers in
equal amounts
(sp 3 hybridised C) (sp2 hybridised)

However, remember that the departing halide shields to some extent, the frontal attack of the carbocation, recemization is
only partial and the inverted configuration predominates.
(iii) SN2 (Bimolecular Nucleophilic Substitution) : The rate of SN2 reactions depends on the concentration of alkyl halide
as well as nucleophile, i.e. r = k [RX ][ Nu ] . This implies that both the reactants are involved in the rate-determining
step, i.e. the reaction occurs in one step only or it is a concerted reaction. Concerted reactions occur through a transition
state (an imaginary state in which both the reactant molecules are partially linked to each other).
H H
é HH HH ù ù
H ê d - d-d-úú -X :
H
Nu : + C–X ê N u - - - - C| -------XXú ú ¾¾¾® Nu – C
H ê úú H
ë H ûû
Transition state Inverted product

Remember that the nucleophile attacks from the back side of the halide ion, bulkier the alkyl group present on the carbon bearing
halogen lesser will be its tendency to undergo SN2 reaction. Thus the reactivity of alkyl halides towards SN2 mechanism is
CH3
n- C4 H9CH2 X > CHCH2X > (CH3)3C.CH2X > (CH 3)2CHX > (CH3)3CX

2 5

1° Alkyl halides 2° 3°
Since the nucleophile attacks from the back side and the halide ion leaves from the front side, the product obtained will
have an inverted configuration [Walden inversion]. This implies that if the alkyl halide is optically active, the product
will also be optically active, although the sign of rotation may be same or different. All primary alkyl halides undergo
substitution via SN2 pathway.
Haloalkanes and Haloarenes 793
Remember : 3. Reaction with metals :
(a) Polar solvents favour SN1 reactions while non-polar
(i) dry ether
solvents favour SN2 reactions. R – X + 2Li ¾¾ ¾¾® R - Li + LiX
(b) Low concentration of nucleophile favours SN1 reactions,
while high concentration favours SN2. (ii) R - X + Mg ¾dry ether
¾ ¾¾® R - MgX
(c) Rate of reaction in SN1 mechanism is independent of the (Grignard reagent)
nature of the attacking nucleophile because it is not
(iii) 4CH 3CH 2Cl + 4 Pb / Na ¾dry ether
¾ ¾¾® (C 2 H 5 ) 4 Pb + 4 NaCl + 3Pb
involved in rate-determining step; while rate of SN2
TEL
reactions depends upon the strength of the nucleophile.
Strength of some common nucleophiles is (iv) dry ether
CH 3Cl + 2 Na ¾¾¾¾® [CH 3 Na ] ¾¾¾
3¾® CH CH CH Cl
3 3
– – – – Very reactive dry ether
: CN > : OR > : OH > CH 3COO > H 2 O
(d) Primary alkyl halides undergo SN2 reactions, 3° halides (Wurtz reaction)
SN1, while 2° halides may undergo SN2 and/or SN1 Tetraethyl lead, commonly known as TEL, is used as an
mechanism. anti-knocking agent in petrol.
2. Elimination reactions : (a) Solvent used must be perfectly anhydrous because
(i) Alkyl halides lose a molecule of hydrogen halide even a trace of water or alcohol reacts with metals to
(dehydrohalogenation) when heated with alcoholic form insoluble hydroxide or alkoxide that coat the
potash. Dehydrohalogenation is a b-elimination reaction surface of the metal. Moreover, water or alcohol may
in which halogen and hydrogen atoms are lost from the react with the product (organometallic compound)
two adjacent carbon atoms. to form hydrocarbon.
CH 3CH 2CH 2 Br or CH 3CHBrCH 3 ¾ (b) Alkyl lithiums react with copper halides to form
1- Bromopropane 2 - Bromopropane higher alkanes (Corey-House synthesis)
CuI R 'X
alc. KOH 2RLi ¾¾
¾® R 2 CuLi ¾¾
¾® R - R '
¾¾¾¾® CH 3CH = CH 2 + HBr ( - LiI) (1° halide )
heat Propene
(c) Among alkyl halides, order of reactivity is
(ii) Dehydrohalogenation is governed by Saytzeff rule
according to which more highly substituted alkene is I > Br > Cl > F.
the major product, i.e. hydrogen atom is lost from the (d) The reactivity of a metal toward an alkyl halide depends
carbon atom carrying minimum number of hydrogen upon its reduction potential; more easily a metal is
atoms (poor becomes poorer). reduced, more reactive it is, e.g. Mg > Zn.
CH 3CH 2 CHBrCH 3 ¾¾¾ ¾®
alc. KOH 4. Reduction
2 - Bromobutane heat
(a) R - X + Zn + HX ¾
¾® R - H + ZnX 2
® CH 3CH 2CH = CH 2 + CH 3CH = CHCH 3 In this reaction, zinc atoms transfer electrons to the carbon
1- Butene (minor) 2- Butene (major)
atom of the alkyl halide. Zinc is a good reducing agent
(iii) Ease of dehydrohalogenation among halides is : because it has two electrons in an orbital far from the
3° > 2° > 1° nucleus, which are readily donated to an electron
(iv) Elimination reactions dominate over substitution when acceptor.
strong Bronsted base [e.g. NH 2- , Me 3CO - , OC 2 H 5 LiAlH
(b) R - X ¾¾¾¾
4®R - H
etc.) is used and alkyl halide is 3° or 2°.
Remember: Ni
(c) R - X + H 2 ¾¾® R - H + HX
(a) 1° Halides undergo SN2 reactions except when a
Mg H O
hindered strong base like Me 3 CO - is used. (d) R - X ¾¾
¾® R - MgX ¾¾2¾® RH (or RD)
or D 2O
(b) 2° Halides undergo SN2 reactions with weak base
like I–, CN–, RCOO– etc., and elimination reaction
with strong base like RO–. (e) R - I + HI ¾red P
¾¾® R - H + I 2
423 K
(c) 3° Halides undergo SN1 reaction in absence of a
strong base and only solvent acts as a base/ 5. Halogenation :
nucleophile (solvolysis), however in presence of Cl , hv Cl , hv Cl , hv
CH 3 Cl ¾¾2¾
¾® CH 2 Cl 2 ¾¾2¾
¾® CHCl 3 ¾¾2¾¾® CCl 4
strong base (–OR) elimination reaction predominates.
794 Chemistry
6. Isomerisation : Cl
573 K
CH 3CH 2CH 2 Br ¾¾¾® CH 3CHBrCH 3 FeCl
1- Bromopropane anhy. AlCl3 2 - Bromopropane + Cl 2 ¾ ¾ ¾3 ®

ARYL AND ARYL SUBSTITUTED


ALKYL HALIDES
Cl Cl
1. Aryl halides are the compounds having halogen atom directly
Cl
attached to the aromatic ring. They are represented by the
FeCl / Cl
general formula Ar–X where Ar may be phenyl, substituted ¾¾ ¾3 ¾¾2® +
phenyl or other aromatic system, e.g. naphthyl.
All compounds containing aromatic ring and a halogen atom Cl
should not be considered as aryl halides, e.g. benzyl chloride, o-dichloro p-dichloro
C6H5CH2Cl because chlorine is not directly attached to the benzene benzene
ring. Such compounds resemble alkyl halides in structure as (i) Fluorination is difficult to control, while iodination
well as in properties, hence grouped as aryl substituted alkyl is too slow to be useful. Moreover, iodination being
halides. reversible (because of reducing character of HI), the
reaction requires the use of an oxidising agent like
2. The carbon-halogen bonds of aryl halides are both shorter
HNO3, HIO3, HgO etc.
and stronger (due to possibility of resonance) than the
carbon-halogen bonds of R–X and in this respect as well as 5 HI + HIO 3 ¾
¾® 3I 2 + 3H 2 O
in their chemical behaviour, they resemble vinyl halides (CH2
(ii) Introduction of second –Cl is difficult as compared
= CHX) more than alkyl halides.
to first, because of electron withdrawing character
of chlorine (deactivating nature of halogens).
X
(iii) Nuclear halogenation is an electrophilic
substitution reaction.
CH2= CH–X (b) Side chain halogenation :
(i) Halogenation of higher arenes, i.e. other than
Aryl halides Vinyl halides benzene, in presence of light or heat and in the
Halogen attached to C is sp2 hybridised C, absence of a halogen carrier introduces halogen in
C–X bond is shorter and stronger because of
partial double bond character due to the side chain.
delocalisation of electrons on halogens CH3 CH2Cl

CH2– X Cl2, 383 K


Sunlight

H = CH–X R–CH 2–X CH2 = CH–CH2–X Toluene Benzyl Chloride

inyl halides Alkyl halides Allyl halides Benzyl halides CH Cl CHCl2 CCl3
dised C, C–X
r because of Cl , 383 K Cl2, 383 K Cl 2, 383 K
Sunlight Sunlight

3. The strength of the C–X bonds causes aryl halides to react Benzyl Chloride Benzal chloride Benzo trichloride
very slowly in reactions in which cleavage of C–X bond is Remember halogenation of toluene at low
rate determining, i.e. nucleophilic substitution. temperature, in the absence of light and in presence
PREPARATION : of catalyst, gives nuclear substituted products.
By direct halogenation of aromatic hydrocarbons. CH3 CH3 CH3
(a) Nuclear halogenation (suitable for aryl chlorides and Cl
Cl2, FeCl3
bromides) +

Cl
Toluene o-Chlorotoluene p-Chlorotoluene
Haloalkanes and Haloarenes 795
Generally, p-isomer predominates due to steric 3. Hunsdiecker method :
hindrance in the o-position.
C 6 H 5 COOAg + Cl 2 (or Br2 ) ¾
¾® C 6 H 5 Cl + CO 2 + AgCl
(ii) Side chain halogenation occurs by free radical
4. Industrial method for chlorobenzen (Raschig process)
mechanism.
CuCl
(iii) In case, the side chain is larger than methyl group, 2C 6 H 6 + 2HCl + O 2 ¾¾ ¾
¾2 ® 2C H Cl + 2 H O
6 5 2
500 K
side chain halogenation mainly occurs at the benzylic
carbon (carbon directly attached to benzene Physical Properties :
nucleus). This is due to stability of benzylic free (i) Like alkyl halides, aryl halides are insoluble in water due
radical due to resonance. to their incapability of forming H-bonds.
(ii) Aryl halides are less polar than alkyl halides because in

CH2CH3 CH2 CH2 aryl halides, halogen is present on sp2 hybridised carbon
which is more electronegative than the sp3 hybridised
Br2, heat carbon of alkyl halides or halocyclohexanes.
Sunlight Consequently, the electroneg ativity difference between
C and Cl is low in aryl halides than in alkyl halides.
2-phenylethyl radical
(less stable, 1° free radical, (iii) Possibility of resonance in chlorobenzene makes the C–
2º benzylic radical)
Cl bond shorter and hence, its dipole moment is less
• than that of cyclohexyl chloride.
2 2 CHCH3 CHBrCH3

or Br 2
Cl Cl

2-phenylethyl radical Benzylic free 1-Bromo-1-phenylethane


(less stable, 1° free radical, radical (more stable) (Major product) Chlorobenzene Chlorocyclohexane
(C–Cl bond is less polar (C–Cl bond is more polar
and shorter in length), m = 1.7 D and longer in length), m = 2.2 D
(iv) Side chain halogenation can also be carried out with
SO2Cl2 in presence of light and a trace of peroxides (iv) Although, the three isomeric dihalobenzenes have nearly
or with NBS (N-bromosuccinimide) in presence of same boiling points, the p-isomer has higher melting
light and trace of peroxide like benzoyl peroxide. point than the o- and m-isomers. This is due to
2. By the decomposition of diazonium salts : symmetrical nature of the para isomer due to which it is
better packed in the crystal lattice. This explains why
only the para isomer crystallises on cooling a solution
containing ortho and para isomers.
(v) Due to strong intercrystalline forces, the higher melting
point of para isomer is less soluble in a given solvent
than the ortho isomer.
Chemical Properties
1. Low reactivity towards nucleophilic substitution :
(i) When CuCl/HCl or CuBr/HBr is used, the reaction is (i) Aryl halides (like vinyl halides) are less reactive towards
called Sandmeyer reaction. It is the halogen attached to nucleophilic substitutions under ordinary conditions
cuprous halide which enters the ring. (difference from alkyl halides). This low reactivity is due
(ii) When Cu/HCl or Cu/HBr is used, the reaction is called to (a) resonance effect, (b) sp2 hybridisation of carbon
Gattermann reaction. atom holding the halogen atom and (c) less polarity of
(iii) Thermal decomposition of benzenediazonium the C–X bond.
tetrafluoroborate to give fluorobenzene is called Balz-
Schiemann reaction. Cl Cl Cl
(iv) This method for preparing aryl halides is more important
than direct halogenation of arenes in two respects.
(a) Fluorides and iodides can be easily prepared.
(b) Halogenation gives a mixture of o- and p- isomers
I II III
which are difficult to separate.
796 Chemistry
(a) Those in which halogen is present on sp3-hybridised
Cl Cl carbon atom, such halogens and corresponding
halides are highly reactive. For example,

R–CH2–X CH2 =CH–CH2–X

Alkyl halides Allyl halides


III IV V

Resonating structures of chlorobenzene


Possibility of resonance in aryl halides produces two results CH2–X
(a) Stabilisation of the molecule by delocalisation of
electrons. Allyl halides Benzyl halides

(b) The C–Cl bond acquires some double bond (b) Those in which halogen is present on sp2-hybridised
character (structures III, IV and V) and thus becomes carbon atom, such halogens are relatively inert. For
shorter and stronger than the C–Cl bond in alkyl example,
halides.
On the same ground, low reactivity of vinyl chloride X CH3
is explained
X

CH 2 = CH - Cl ¬¾® CH 2 - CH = Cl CH2 = CH–X

(ii) Conversely, benzyl halides do not show delocalisation


Aryl halides Vinyl halides
of electrons of the halogen atom, hence its halogen is
quite reactive towards nucleophilic substitutions (iv) However, aryl halides having electron-withdrawing
(similarity with alkyl halides). Moreover, benzyl halides groups (like –NO2, –CN, –COOH, –SO3H etc.) in ortho
are even more reactive than alkyl halides, because the and para positions undergo nucleophilic substitution
carbocation (benzyl) formed after the removal of halide very easily. Further, greater the number of such groups
ion is stabilised by resonance (remember higher is the in o- and p- positions, more rapid is the reaction and
stability of a species, easier will be its formation). hence less vigorous conditions are required. Thus,
reactivity of chlorine in the following compounds
CH2–Cl CH2 CH2 towards nucleophilic substitution follows the order.

+ Cl Cl
(–Cl –)

O2N NO2 NO2

>
CH2 CH2 NO2 NO2
+ 2,4,6-Trinitrochlorobenzene 2,4-Diinitrochlorobenzene

Cl Cl Cl
+
Similar is the case of allyl carbocation formed from allyl NO
halides.
> >
+
CH 2 = CHCH 2 Br ¾¾ ¾
¾-
® CH 2 = CH - C H 2 ¬¾® NO2
Allyl bromide ( - Br )
2,4,6-Trinitrochlorobenzene p-Nitrochlorobenzene Chlorobenzene
+
C H 2 - CH = CH 2 Remember that the electron-withdrawing groups activate
(iii) Thus halogen derivatives can be categorised into two chlorine of chlorobenzene towards nucleophilic
main groups on the basis of reactivity of the halogen substitution, they deactivate the benzene nucleus
towards electrophilic substitution.
atom.
(v) Since electron-withdrawing groups activate halogen
toward nucleophilic substitution, electron-releasing
Haloalkanes and Haloarenes 797
groups (like –OH, –NH2, –OCH3, –R etc.) deactivate
Cl NH2
toward nucleophilic substitution.
(vi) Aromatic nucleophilic substitution proceeds through (ii)
NH 3, Cu2O
(Dow’s process)
mechanisms different from the SN2 and SN1 mechanism 573K, presure
of alkyl halides.
(a) Bimolecular mechanism is applicable to reactions of
Cl NH2
aryl halides having electron-withdrawing groups
with the usual nucleophiles like NaOH, CH3ONa etc. NO2 NO2
NH , 543K
Here carbanion is formed as an intermediate.
note mild conditions
(b) Benzyne mechanism is applicable when aryl halides
are treated with a strong base like NH 2- or with the NO2 NO2
usual nucleophiles like NaOH, NH3, CH3ONa etc.
under drastic conditions.
Br CN
(vii) Relative reactivity of halogens towards nucleophilic
substitution. The order of reactivity of the halogen atom CuCN, 475K, pyridine
in aryl halides is opposite to that observed in alkyl halides (iii)
(R – I > R – Br > R – Cl > R – F). In aromatic nucleophilic
substitution, fluoride is the most reactive and iodide the Benzonitrile

least. Ar – F > Ar – Cl > Ar – Br > Ar – I


Benzyl chloride undergoes all the above reactions under
The unusually high reactivity of aryl fluorides (although
mild conditions
having a strong C – F bond) is due to very high
electronegativity of fluorine due to which it stabilises aq. NH , aq. KOH
C 6 H 5CH 2Cl ¾¾¾ ¾
3 ¾ ¾ ¾® C H - CH - Y
6 5 2
the carbanion, formed as intermediate during the reaction, aq. KCN
most easily.
2. Important nucleophilic substitution reactions : (where –Y = –NH2, –OH, –CN)

Cl (iv) However, simple aryl halides undergo nucleophilic


ONa
substitution very easily by strong basic nucleophiles,
7% NaOH, 623K, 300 atm. like amide ion, via benzyne mechanism.
(i)

* Cl *
OH KNH2

dil. HCl
(Dow’s process)
Chlorobenzene Benzyne

Note that introduction of –NO2 group in p– and o–


positions make the substitution of –Cl by –OH easier as * NH2
(i) NH 2– *
it is evident by increased mild conditions. (ii) NH 3 (Solvent)
+

Cl NH2
Cl
Aniline having C–NH 2 Aniline having C–NH 2
NO2
where * indicates isotopically labelled carbon, i.e. C-14.
Benzyl chloride does not form benzyne intermediate.
NO2 NO2 (a) Remember triple bond of benzyne is different from
Requires 15% NaOH Requires weak base the triple bond of alkynes, in benzyne one of the p
and 433K Na2CO3 and 403K
bonds is formed by p–p overlap while the other by
Cl
sp2–sp2 overlap.
NO2 3. Electrophilic substitution : Remember that halogens are
O2 N
deactivating but o, p–directing. Thus chlorination, nitration,
sulphonation and Friedel-Craft reaction forms a mixture of o-
NO2 and p-chlorosubstituted derivatives, e.g.
Undergoes hydrolysis even by
warming with H 2O
798 Chemistry
Cl 4. Other reactions :
(i) Formation of Grignard reagent : Since chlorobenzene
anhy. AlCl 3 is less reactive than methyl chloride, THF (a higher
+ CH3Cl
boiling solvent) is used.

Cl Cl Tetrahydrofuran ( THF)
C 6 H 5Cl + Mg ¾¾ ¾ ¾ ¾ ¾ ¾ ¾
¾® C 6 H 5 MgCl
CH3 Chlorobenzene

+ (ii) Fittig reaction :

dry ether
CH3 C 6 H 5Cl + 2Na + ClC6 H 5 ¾¾ ¾¾® C 6 H 5C 6 H 5
Diphenyl
o-Chlorotoluene p-Chlorotoluene
Cl (iii) Wurtz-Fittig reaction :
dry ether
anhy. AlCl 3 C 6 H 5 Cl + 2 Na + ClCH 3 ¾¾ ¾¾® C 6 H 5 CH 3
+ CH 3COCl Toluene

Cl Cl (iv) Ullmann reaction :


COCH3 heat
C 6 H 5 I + 2Cu + IC6 H 5 ¾¾
¾® C 6 H 5C 6 H 5
+ Diphenyl

COCH3 Zn / Cu
(v) Reduction : C 6 H 5 Cl + 2[H ] ¾¾¾ ¾¾
¾® C 6 H 6
o-Chloroacetophenone p-Chloroacetophenone or Ni - Al / NaOH Benzene

However, unlike CH3Cl, chlorobenzene does not undergo Benzyl chloride undergoes all the above reactions forming
Friedel-Craft reaction with benzene. This is because of C 6 H 5CH 2 MgCl, C 6 H 5CH 2 CH 2 C6 H 5 (dibenzyl),
unstability of C6H5+ cation.
Benzyl chloride also undergoes electrophilic substitution C 6 H 5CH 2 CH 3 , C 6 H 5CH 2 CH 2 C 6 H 5 and
easily and mainly in the o- and p-positions. However, here
percentage of m- too is significant (10-15%) as compared to C 6 H 5CH 3 respectively..
that in toulene. Formation of m-isomer is due to the presence (vi) Condensation with chloral to form DDT, 2,2-bis
of Cl as -CH 2 Cl which decreases the electron- (4-chlorophenyl) –1, 1, 1-trichloroethane.
releasing power of the parent methyl group due to its electron
withdrawing nature. Thus we can explain the increased
Cl3C–C=O + conc. H2SO4
percentage of the m-isomer with the increase in number of Cl Cl2C–CH
atoms in the –CH3 group. H Cl
Cl DDT
CH3 CH2—Cl H–C—Cl

on standing
Cl2C=C
Toluene Benzyl chloride Benzal chloride (–HCl)
% of m-isomer (2-5%) (10-15%) (30-35%)

DDT
Cl Cl
CH Cl H–C—Cl Cl—C—Cl DDT is non-biodegradable and slowly loses a molecule
of HCl to form another compound p, p'–dichloro-
diphenyldichloroethene (DDE) which hinders with the
egg shell formation of birds with the result eggs break
off before hatching.
Toluene Benzyl chloride Benzal chloride Benzo trichloride
(30-35%) (60-65%)
Haloalkanes and Haloarenes 799
(vii) Oxidation of the side chain : 3. Properties :
(i) Dehydrohalogenation : Both give alkynes.
CH3 COOH
alc. KOH
Cl Cl ClCH2 - CH 2Cl or CH 3CHCl 2 ¾¾¾ ¾®
HNO3 Ethylene chloride Ethylidene chloride

alc. KOH
o-Chlorotoluene o-Chlorobenzoic acid (–Cl is present)
CH 2 = CHCl or ¾¾ ¾¾ ¾¾
¾® HC º CH
Vinyl chloride preferably NaNH2 Acetylene

(ii) Dehalogenation : Both give alkenes.


CHO CH2Cl Zn / CH OH

S-ar putea să vă placă și